Está en la página 1de 349

DERMATOLOGA

Preparacin Examen de Seleccin 05/06 1 Vuelta


1.

Si en una biopsia cutnea se aprecia una epidermis con


clulas queratinizadas en estratos inferiores, acompaada
de acantlisis, usted sospecha:
1)
2)
3)
4)
5)

2.

1)
2)
3)
4)
5)
3.

Preguntas TEST

4)
5)
7.

Insensibilidad de las lesiones frente al calor.


Aparicin de fenmenos congestivos generalizados.
Picor y urticacin de los elementos a causa del frotamiento.
Aumento de la sensibilidad de las lesiones tras el rascado.
Desaparicin temporal de los elementos explorados.

2)
3)
4)
5)

8.

Probablemente existan personas convivientes tambin


afectadas.
El diagnstico se confirma con el raspado de las lesiones
lineales.
Las lesiones nodulares son las que ms tardan en resolverse.
El tratamiento con permetrina 5% es muy til.
En las embarazadas, el tratamiento de eleccin es el
lindano.

Mujer de 35 aos, propietaria de una peluquera canina,


consulta por la presencia de una ppula eritematosa que se
sigui de un eritema de crecimiento perifrico y migratorio.
M exico A rgentina
C hile U ruguay

2)
3)
4)
5)

Probablemente encontremos lesiones cutneas palmoplantares.


Se puede observar alteracin de las pruebas funcionales
hepticas.
La lesin costrosa negruzca aparece en todos los pacientes.
El diagnstico se establece mediante serologas.
El tratamiento de eleccin son las tetraciclinas.

Varn de 7 aos de edad, con placa de alopecia en cuero


cabelludo, de 3 cm, eritema, descamacin blanquecina y
pelos rotos a distintos niveles, leve prurito, de 15 das de
evolucin. La causa ms probable ser:
1)
2)
3)
4)
5)

10.

Pioderma gangrenoso.
ntrax.
Imptigo contagioso.
Botn de oriente.
Lupus vulgar.

Varn de 17 aos de edad acude a la urgencia por presentar


fiebre de 38,5 C y cefalea intensa desde hace 4 das. Refiere
presentar una lesin costrosa negruzca, rodeada de un halo
rojizo en un tobillo desde hace 8 das, en que se fue de
acampada con unos amigos. A la exploracin, usted aprecia
lesiones maculosas eritematosas de pequeo tamao, salpicadas por tronco y extremidades en escasa cantidad. Seale
la FALSA:
1)

9.

Est ocasionado por una espiroqueta.


En el 10% de los pacientes aparece una artritis crnica de
aparicin tarda.
Sin tratamiento, las lesiones cutneas desaparecen, aunque pueden presentar recadas.
El diagnstico se realiza mediante serologas.
La aparicin de neuropatas perifricas son excepcionales
y tardas.

Acude a su consulta un nio de 7 aos de edad con una lesin


en la frente papulonodular, firme, infiltrada, de 1 cm de
dimetro y 5 semanas de evolucin, cubierta de una costra
superficial, asintomtica, que al ser retirada deja ver una
lcera con bordes sobreelevados e indurados. En el estudio
histolgico se aprecia una lcera epidrmica con un infiltrado inflamatorio compuesto de linfocitos, histiocitos y neutrfilos, junto con presencia de grupos de 2-4 partculas azlgrisceas en el citoplasma de los macrfagos drmicos. Cul
es el diagnstico ms probable?:
1)
2)
3)
4)
5)

Teratogenia.
Xerosis.
Hipercolesterolemia.
Calcificaciones osteoligamentosas.
Hepatotoxicidad.

Varn de 30 aos consulta por presentar desde hace 3


semanas una erupcin en tronco y extremidades de forma
progresiva muy pruriginosa, ms intensamente por las noches. A la exploracin presenta lesiones eritematosas levemente descamativas, de pequeo tamao, con excoriaciones
en tronco y races de extremidades. En las caras laterales de
los dedos se aprecian lesiones vesiculosas sueltas, de pequeo tamao y contenido claro, y alguna lesin lineal de 2-3 mm
de longitud. Tambin se aprecian ndulos rojizos en escroto
y pene. Seale la FALSA:
1)

6.

3)

Pasta al agua.
Crema.
Pomada.
Ungento.
Solucin.

Un varn de 16 aos presenta un intenso acn conglobata


desde hace 2 aos. Su dermatlogo le ha recetado isotretinona oral en dosis de 0,5 mg/kg/da durante 6 meses. Seale el
efecto secundario ms grave de este tratamiento:
1)
2)
3)
4)
5)

5.

1)
2)

El signo de Darier en una mastocitosis consiste en:


1)
2)
3)
4)
5)

4.

Con respecto a su enfermedad, seale la FALSA:

Pnfigo.
Liquen plano.
Eczema numular.
Enfermedad de Darier.
Psoriasis.

Qu vehculo empleara para el tratamiento de lesiones de


tipo descamativo en el cuero cabelludo?:

Seguimiento a distancia

Liquen plano pilar.


Lupus discoide.
Tia tonsurante.
Alopecia areata.
Tricotilomana.

Ante un paciente que acude a consulta por presentar, desde


hace una semana, una placa en la pierna derecha, de crecimiento ascendente progresivo, caliente, brillante, rojiza,
edematosa, indurada, y con un borde elevado, bien diferenciada de la piel adyacente, dolorosa y acompaada de fiebre
y leucocitosis, seale la FALSA:

CTO Medicina C/ Nez de Balboa, 115 28006 MADRID (Espaa) Tfno.: (91) 782 43 32 / Fax: (91) 782 43 27
E-mail: secretaria@ctomedicina.com; iberocto@ctomedicina.com WEB: www.ctomedicina.com; www.iberocto.com

DM Pg. 1

1)
2)
3)
4)
5)
11.

3)
4)
5)
12.

5)

14.

Pg. 2 DM

2)
3)
4)
5)
16.

Sarcoidosis.
Lupus vulgar.
M exico A rgentina
C hile U ruguay

Extirpacin quirrgica de la lesin.


Realizacin de TC abdominal.
Radioterapia.
Infiltracin con interfern alfa.
Gluconato de antimonio intralesional.

Respecto al carcinoma basocelular, sealar la FALSA:


1)
2)
3)
4)
5)

19.

Psoriasis en placas.
Enfermedad de Bowen.
Enfermedad de Paget.
Herpes circinado.
Queratosis seborreica.

Mujer de 77 aos, con hiperglucemias mantenidas desde


hace 2 meses, presenta una placa blanco-amarillenta, esclerosa, con bordes papulosos de brillo blanquecino, localizada
en la mejilla derecha. La paciente refiere aparicin frecuente
de costras sobre la lesin. Seale la actitud correcta:
1)
2)
3)
4)
5)

18.

Existen datos epidemiolgicos que hablan a favor de un


vector infeccioso como etiologa.
La piel suele ser la localizacin de presentacin.
El sarcoma de Kaposi no suele ser causa de muerte.
Una mnima parte de los pacientes llega a tener afectacin
gastrointestinal.
El sarcoma de Kaposi en el VIH tiene el mismo patrn
histolgico que el africano o el de los trasplantados.

Mujer de 74 aos consulta por presentar desde hace 8 aos


una placa bien delimitada en abdomen, de crecimiento
excntrico lentamente progresivo, que mide 4 cm de dimetro, y en cuyo interior se aprecia descamacin, eritema y
lesiones costrosas sin infiltracin. La biopsia demuestra
prdida de polaridad atpica y mitosis en los queratinocitos,
ocupando toda la epidermis, pero respetando la integridad
de la membrana basal. Cul es su diagnstico?:
1)
2)
3)
4)
5)

17.

Granuloma anular.
Lepra tuberculoide.
Morfea en placas.

Sobre el sarcoma de Kaposi en el VIH, seale la respuesta


FALSA, de entre las siguientes:
1)

Acrodermatitis papulosa infantil.


Dermatitis herpetiforme.
Picaduras mltiples de insecto.
Varicela.
Enfermedad pie-mano-boca.

Una mujer de 35 aos presenta una placa nica seca, de


textura irregular, hipopigmentada, con disminucin de la
sensibilidad, de 3 x 3 cm, en la espalda. Biopsia: granulomas
de clulas epitelioides y clulas gigantes tipo Langhans, que
tambin afectan a estructuras nerviosas. El diagnstico ser:
1)
2)

15.

No tratar.
Dar corticoides orales en dosis bajas.
Administrar antibiticos orales.
Tratamiento tpico astringente, antihistamnicos orales y
paracetamol.
Antivricos orales y antitrmicos.

Nia de 6 aos de edad, con un estado general bueno,


presenta lesiones papulosas, eritematosas, algunas cubiertas
por una vescula de contenido claro, suelto y distribuidas en
brazos, piernas, glteos y mejillas, de 8 das de evolucin. Una
analtica demuestra elevacin de las enzimas hepticas. Cul
es el diagnstico ms probable?:
1)
2)
3)
4)
5)

3)
4)
5)

Rifampicina va oral, durante 3 meses.


Antispticos orales potentes, asociados a esteroides intralesionales en las lesiones.
Ciclo de esteroides orales, durante 2 meses.
Cefuroxima oral en cada brote con drenaje quirrgico, si
as lo precisara.
Isotretinona oral, durante 6 meses.

Un varn de 23 aos presenta fiebre y un rash compuesto por


vesculas, pstulas, costras y ppulas eritematosas, de 6 das
de evolucin, diseminadas por la piel y muy pruriginosas, sin
otra sintomatologa. Cul sera su actitud?:
1)
2)
3)
4)

13.

El diagnstico se realiza siempre mediante cultivo de una


biopsia profunda.
Esta misma lesin, si aparece en la cara, debe diferenciarse
en ocasiones de un herpes zoster.
Para diferenciarlo de una celulitis aguda, es til el borde
mal definido que deja reas adyacentes sanas en esta
ltima.
Ocasionalmente puede confundirse en la cara tambin
con un edema angioneurtico.
De no tratarse, pueden aparecer complicaciones no
supurativas posteriormente.

Mujer de 27 aos, que consulta por presentar desde hace ms


de 2 aos brotes repetidos de lesiones nodulares, dolorosas
y abscesificadas, que terminan drenando material purulento
en ambas ingles. Ha sido tratada con diversos ciclos de
antibiticos va oral y drenajes quirrgicos, pero slo ha
conseguido mejoras parciales, sin verse completamente
libre de lesiones. Qu tratamiento cree que podra ofrecerle
mejores expectativas a esta paciente?:
1)
2)

DERMATOLOGA

Preparacin Examen de Seleccin 05/06 1 Vuelta

Es el tumor maligno cutneo ms frecuente.


Cuando aparecen en labio, suelen asentarse sobre una
leucoplasia.
Est constituido por queratinocitos de la capa germinativa
dispuestos en empalizada.
En su etiologa se ha implicado la exposicin solar prolongada.
Los pacientes que han padecido un epitelioma basocelular tienen ms riesgo de presentar otros en distintas localizaciones.

Nia de 2 aos de edad que presenta desde el nacimiento


placa amarillenta y alopcica en el cuero cabelludo. Con
respecto a esta enfermedad, seale la correcta:
1)
2)

Se denomina nevus sebceo de Pringle.


La actitud correcta es la observacin de la lesin a lo largo
de la vida.

CTO Medicina C/ Nez de Balboa, 115 28006 MADRID (Espaa) Tfno.: (91) 782 43 32 / Fax: (91) 782 43 27
E-mail: secretaria@ctomedicina.com; iberocto@ctomedicina.com WEB: www.ctomedicina.com; www.iberocto.com

Preguntas TEST

Seguimiento a distancia

DERMATOLOGA

Preparacin Examen de Seleccin 05/06 1 Vuelta


3)
4)
5)
20.

Qu tumor desarrollar con ms frecuencia un paciente con


mltiples lesiones de morfologa similar a las verrugas planas?:
1)
2)
3)
4)
5)

21.

Preguntas TEST

4)
5)

25.

26.

Nevus adquirido.
Epitelioma basocelular pigmentado.
Mancha caf con leche.
Melanoma.
Queratosis seborreica.

Presentar disqueratosis en palmas y plantas.


El hipertelorismo es llamativo.
La afectacin ocular en forma de fotofobia es casi constante.
Se hereda de manera autosmico dominante.
Es frecuente la presencia de quistes seos mandibulares en
este cuadro.

5)

Liquen plano oral.


Queilitis actnica.
Aftas orales.
Candidiasis oral.
Exantema fijo.

2)
3)
4)
5)

Seale la actitud correcta ante una nia de 6 aos de edad,


con una lesin melanoctica presente desde el nacimiento,
que ocupa la cara externa del muslo izquierdo:
M exico A rgentina
C hile U ruguay

En los melanomas de ms de 4 mm con enfermedad


localizada cutnea.
En los melanomas nodulares.
En melanomas con metstasis pulmonares solitarias.
En los melanomas en nivel III de Clark y superiores.
En los melanomas de las extremidades.

Con respecto a la micosis fungoide, NO es cierto que:


1)
2)
3)
4)
5)

29.

Estas lesiones suelen ser inicialmente planas.


El color suele ser irregular.
La fase radial puede durar hasta dcadas.
En ocasiones se aprecia un infiltrado inflamatorio perilesional.
La extirpacin precoz de la lesin es el mejor tratamiento.

Seale cundo est indicado el tratamiento con interfern


sistmico en un melanoma:
1)

28.

Fototipo cutneo.
Sexo.
Localizacin (reas BANS).
Invasin en profundidad (grosor en mm).
Tipo clnico.

Mujer de 32 aos de edad, que consulta a su mdico por


presentar lesin pigmentada asintomtica en la pierna derecha, cara posterior. Segn la paciente, la lesin ha crecido
progresivamente en los ltimos 3 aos. En la histologa se
aprecian melanocitos atpicos en dermis con grandes ncleos y mitosis. Seale la FALSA, con respecto a esta enfermedad:
1)
2)
3)
4)

27.

Extirpacin quirrgica, en la medida de lo posible.


Observacin a partir de la pubertad.
Tratamiento con lser de la lesin, previa toma histolgica
que confirme su benignidad.
Biopsias semestrales de la lesin.
Electrocoagulacin.

En el melanoma maligno en estadio I, sealar cul es el factor


pronstico ms importante:
1)
2)
3)
4)
5)

Varn de 65 aos, agricultor y fumador habitual, presenta


desde hace 2-3 aos lesiones maculosas blanquecinas que
ocupan prcticamente la totalidad de la semimucosa del labio
inferior, con atrofia del epitelio. No le produce ninguna
molestia significativa, salvo sensacin de tirantez. Qu posibilidad diagnstica le parece la ms adecuada?:
1)
2)
3)
4)
5)

24.

4)
5)

Varn de 27 aos, minero de profesin, presenta desde hace


aos mltiples lesiones papulosas, perladas, alguna erosionada y otras pigmentadas, localizadas en cara, tronco y
extremidades. Seale la respuesta FALSA:
1)
2)
3)

23.

Se trata de una epidermodisplasia verruciforme y desarrollar epiteliomas basocelulares.


Desarrollar carcinomas epidermoides.
Se trata de la enfermedad de Lewandowsky, y desarrollar
queratosis seborreica.
Presentar carcinoma de crvix dorsal, asociado con la
infeccin por VPH.
Angiofibromas.

1)
2)
3)

Varn de 50 aos, que consult por una lesin localizada en


la espalda de 3 aos de evolucin. La lesin era de aspecto
verrucoso-papilomatoso, de color marrn oscuro. A veces
produca prurito, y haba crecido desde su aparicin. La
biopsia mostr una proliferacin exoftica de clulas epidrmicas de aspecto basalioide y globos crneos. Qu diagnstico sospechara?:
1)
2)
3)
4)
5)

22.

Son ms frecuentes en el tronco, aunque le sigue en


frecuencia el cuero cabelludo.
Se deben extirpar antes de la pubertad.
Es un cmulo de histiocitos con alto riesgo de degeneracin maligna.

Seguimiento a distancia

Es un proceso linfoproliferativo maligno T inductor/colaborador.


Se manifiesta inicialmente en la piel.
Las lesiones en la fase premictica pueden ser confundidas con mltiples dermatosis.
En ocasiones puede comenzar en clnica como una
eritrodermia.
Las manifestaciones ganglionares y viscerales suelen aparecer al poco tiempo de evolucin.

Qu tratamiento instaurara en una micosis fungoide con


lesiones eccematosas por el tronco?:
1)
2)
3)
4)
5)

Corticoides tpicos.
UVB.
Radioterapia.
Acitretina.
Interfern alfa.

CTO Medicina C/ Nez de Balboa, 115 28006 MADRID (Espaa) Tfno.: (91) 782 43 32 / Fax: (91) 782 43 27
E-mail: secretaria@ctomedicina.com; iberocto@ctomedicina.com WEB: www.ctomedicina.com; www.iberocto.com

DM Pg. 3

30.

Varn de 58 aos, con astenia y anorexia de 3 meses de


evolucin. Desde hace 2 meses presenta eritrodermia, as
como adenopatas generalizadas. En sangre perifrica se
detectan un 20% de clulas linfoides atpicas. En relacin con
este cuadro, es cierto que:
1)
2)
3)
4)
5)

31.

32.

2)
3)
4)
5)

Las tetraciclinas orales son muy efectivas.


En los casos rebeldes, pueden usarse retinoides orales.
Es frecuente la afectacin de la mucosa conjuntival.
Puede desencadenarse con el caf.
La afectacin renal puede verse en periodos tardos.

En este cuadro, las clulas de Paget se hallan desprovistas


de desmosomas.
Las clulas de Paget contienen mucina y antgeno carcinoembrionario.
La lesin termina siendo bilateral.
A menudo se palpan ndulos en la mama afecta.
Suele aparecer en mujeres de ms de 40 aos.

2)
3)
4)
5)
38.

Ictiosis vulgar.
Hiperqueratosis epidermoltica.
Ictiosis lamelar.
Ictiosis adquirida.
Ictiosis X.

Probablemente tenga afectacin articular.


Las lesiones en mucosas son frecuentes.
No suelen existir factores desencadenantes.
Tiene muy buen pronstico.
Debe tratarse con teraputica oral.

La presencia en la psoriasis de pstulas subcrneas se denomina:


1)
2)

Pg. 4 DM

Microabscesos de Pautrier.
Pstulas de Auspitz.
M exico A rgentina
C hile U ruguay

Psoriasis.
Liquen plano.
Pnfigo.
Pitiriasis rubra pilaris.
Eczema seborreico.

Mujer de 30 aos de edad consulta por presentar desde hace


10 das una erupcin eritematoescamosa, fundamentalmente en tronco, asintomtica. A la exploracin, existe en la
espalda una distribucin de las lesiones en rbol de navidad,
siendo una de ellas de mayor tamao que el resto. Seale la
FALSA:
1)
2)
3)
4)
5)

40.

El acitretino no debe emplearse sin acompaarse simultneamente de anticoncepcin, debido a su prolongada


teratogenicidad.
En estos casos, la isotretinona oral es igual de efectiva que
el acitretino.
El metotrexate puede ser una buena opcin teraputica,
si su funcin heptica es normal.
La ciclosporina ser el frmaco que ms rpidamente
consiga el control del brote.
El metotrexate obliga a mantener los anticonceptivos hasta
12 semanas despus de terminar el tratamiento.

Cul de las siguientes patologas tiene una mala respuesta a


los corticoides?:
1)
2)
3)
4)
5)

39.

Metotrexate.
Corticoides orales.
Acitretina.
Ciclosporina.
Calcipotriol.

Mujer de 31 aos, con psoriasis en placas extensas, afectando


a ms de un 40% de la superficie corporal total. Ni el
tratamiento con medicacin tpica ni la PUVA han conseguido controlar la enfermedad. Seale la FALSA:
1)

Ante una nia con mltiples placas queratsicas, de pequeo


tamao y de reciente aparicin, que presentan una descamacin nacarada con el rascado, seale la respuesta correcta:
1)
2)
3)
4)
5)

35.

37.

Microabscesos de Munro-Sabouraud.
Pstulas de Von Zumbusch.
Pstulas de Kogoj-Lapiere.

Varn de 46 aos, con hepatopata enlica conocida e


infeccin crnica por VHC, presenta una psoriasis generalizada, intensa, que no responde a la medicacin tpica
habitual ni a la fototerapia. Cul sera su siguiente actitud
teraputica?:
1)
2)
3)
4)
5)

Las opacidades corneales puntiformes son caractersticas de


una de las siguientes ictiosis:
1)
2)
3)
4)
5)

34.

36.

Mujer de 50 aos de edad, con anorexia de 2 meses de


evolucin. Desde hace 3 semanas, refiere lesin pruriginosa
en una de las areolas mamarias. En la exploracin se observa
una placa eccematosa bien delimitada. Seale la FALSA:
1)

33.

3)
4)
5)

El prurito no suele ser un sntoma.


Es de larga evolucin.
La hepatoesplenomegalia suele ser evidente.
El uso de retinoides con interfern se ha empleado como
tratamiento de las fases premicticas de la enfermedad.
En esta fase de la enfermedad no se objetivan linfocitos en
dermis.

Mujer de 50 aos de edad, que consulta por presentar


lesiones papulopustulosas en regin facial, as como un
eritema persistente en ambas mejillas. Al parecer, empeora en
verano. Seale la respuesta FALSA en relacin con esta
enfermedad:
1)
2)
3)
4)
5)

DERMATOLOGA

Preparacin Examen de Seleccin 05/06 1 Vuelta

Las recurrencias de este cuadro son raras.


Se cree que est producido por un virus.
El tratamiento de eleccin es la prednisona oral.
Las lesiones pueden salir durante 6-8 semanas.
Las lesiones curarn sin dejar cicatriz ni alteraciones de la
pigmentacin.

Varn de 14 aos de edad, con lesiones papulosas, hiperqueratsicas, de localizacin predominantemente folicular, de
varios meses de evolucin, distribuidas de forma simtrica
por cara anterior de tronco y cuello, con lesiones en V en el
borde libre ungueal. Empeora mucho en verano. Su padre
padece la misma enfermedad. Qu entidad sospecha usted?:

CTO Medicina C/ Nez de Balboa, 115 28006 MADRID (Espaa) Tfno.: (91) 782 43 32 / Fax: (91) 782 43 27
E-mail: secretaria@ctomedicina.com; iberocto@ctomedicina.com WEB: www.ctomedicina.com; www.iberocto.com

Preguntas TEST

Seguimiento a distancia

DERMATOLOGA

Preparacin Examen de Seleccin 05/06 1 Vuelta


1)
2)
3)
4)
5)
41.

3)
4)
5)

Preguntas TEST

46.

2)
3)
4)
5)
47.

3)
4)
5)

Mujer de 26 aos en su primera gestacin, de 24 semanas.


Comienza con placas urticarianas muy pruriginosas, sobre
las que brotan lesiones vesiculosas de contenido claro.
Comenzaron en el abdomen y se extendieron al resto del
tronco. Cmo tratara a esta paciente?:

2)
3)

1)
2)
3)
4)
5)

Dieta sin gluten.


Sulfona.
Prednisona en dosis bajas.
No requiere tratamiento.
Azatioprina.
M exico A rgentina
C hile U ruguay

4)
5)

Se produce por un depsito de inmunocomplejos precapilar.


Las lesiones pueden llegar a hacerse necrticas.
El tratamiento de eleccin de este cuadro se hace con
ciclosporina.
La imagen histolgica caracterstica es la linfocitosis perivascular.
Es excepcional la etiologa vrica de este cuadro.

Con respecto a la pregunta anterior, cul de los siguientes


tratamientos NO empleara nunca?:
1)
2)
3)
4)
5)

49.

Biopsia de una lesin nodular, retirar anticonceptivos y


reposo.
Reposo y esteroides orales.
Antiinflamatorios no esteroideos.
Retirada de anticonceptivos y dar esteroides orales.
Administrar yoduro potsico y reposo.

Mujer de 67 aos, hipertensa y con bronquitis crnica.


Presenta desde hace 4 das lesiones asintomticas, bilaterales, en ambos miembros inferiores. A la exploracin se
aprecian lesiones maculo-papulosas, eritematosas, que no
desaparecen con la digitopresin. Seale la correcta respecto a esta enfermedad:
1)

48.

Sndrome de Weber-Christian.
Adenocarcinoma de pncreas.
Tuberculosis.
Lupus eritematoso sistmico.
Forunculosis miliar.

Mujer de 19 aos, sin antecedentes personales de inters,


que toma anticonceptivos desde hace 2 aos. Consulta por
presentar fiebre, artralgias en rodillas y tobillos de una
semana de evolucin, junto con edema de tobillos y lesiones
nodulares dolorosas en la cara anterior de ambas piernas.
Cul sera la actitud ms adecuada?:
1)

Hacer una IFI en suero.


La respuesta positiva a tratamiento esteroideo es diagnstica.
Realizar una IFD en piel perilesional.
Realizar un TC para descartar neoplasia subyacente.
La respuesta positiva a tratamiento con sulfonas es diagnstica.

Responde a sulfonas.
Debe establecerse el diagnstico diferencial con la enfermedad de Duhring-Brocq.
Los brotes aparecen de manera brusca.
En el 90% de los casos presentan cambios histolgicos
similares a la enfermedad celaca.
Para el diagnstico es esencial la inmunofluorescencia
directa.

Paciente con lesiones nodulares, subcutneas, dolorosas,


presentando alguna de ellas licuefaccin con eliminacin de
material, distribuidas por tronco y extremidades, y que
muestran en el estudio histolgico clulas fantasmas en el
tejido adiposo. Usted sospechar que padece:
1)
2)
3)
4)
5)

Varn de 5 aos de edad, que presenta desde hace meses


lesiones papulosas y ampollosas con intenso prurito en
tronco y extremidades. Se le realiza una biopsia que muestra
un depsito de IgA lineal a lo largo de la membrana basal.
Seale la opcin INCORRECTA:
1)
2)

44.

Acitretina.
Azatioprina.
Corticoides tpicos.
Fluconazol.
Sueroterapia y antibioterapia profilctica de amplio espectro.

Varn de 74 aos, que consulta por presentar en los ltimos


2 meses lesiones eritematoedematosas, con aparicin de
ampollas tensas en tronco y extremidades muy pruriginosas,
que no han desaparecido tras tratamiento con esteroides
tpicos y antihistamnicos orales. La biopsia de una lesin
demuestra una ampolla subepidrmica, con infiltrado inflamatorio donde existen numerosos eosinfilos. Cul sera lo
ms indicado para confirmar su diagnstico?:
1)
2)

43.

45.

Varn de 52 aos, con fiebre, mal estado general y lesiones


ampollosas, flccidas, que se distribuyen por toda la superficie corporal, dando lugar a grandes erosiones. Adems se
aprecian importantes erosiones en mucosa oral, genital y
conjuntival. El tratamiento ms indicado ser:
1)
2)
3)
4)
5)

42.

Pitiriasis rubra pilar.


Dermatitis seborreica.
Enfermedad de Hailey-Hailey.
Enfermedad de Darier.
Enfermedad de Grover.

Seguimiento a distancia

Tratamiento del factor desencadenante en caso de identificarse.


AINES.
Corticoides orales.
Inmunosupresores.
PUVA.

Nio de 8 aos de edad, con dolor articular y abdominal,


lesiones purpricas en extremidades inferiores, hematuria y
proteinuria. Seale la correcta con respecto a la enfermedad
que sospecha:
1)
2)
3)
4)
5)

Es una vasculitis linfocitaria.


Cursa con depsito de IgA en vasos drmicos.
Es ms frecuente en mujeres.
En el 90%, se asocia a alteraciones neurolgicas.
Suele comenzar en las primeras 48 horas desde el inicio
de una infeccin estreptoccica de vas respiratorias altas.

CTO Medicina C/ Nez de Balboa, 115 28006 MADRID (Espaa) Tfno.: (91) 782 43 32 / Fax: (91) 782 43 27
E-mail: secretaria@ctomedicina.com; iberocto@ctomedicina.com WEB: www.ctomedicina.com; www.iberocto.com

DM Pg. 5

50.

Mujer de 27 aos, que presenta en 1/3 posterior de mucosa


geniana un fino reticulado blanquecino, que le ocasiona
molestias espordicas. Adems, presenta alguna ppula violcea, pruriginosa en superficies de flexin de extremidades.
Seale la FALSA en relacin a su patologa:
1)
2)
3)
4)
5)

51.

3)
4)
5)

5)

Isotretinona oral + anticonceptivos orales.


Minociclina oral.
Adapaleno + eritromicina tpicos.
Acetato de ciproterona + anticonceptivos orales.
Doxiciclina oral + anticonceptivos orales.

Se emplea en dosis de 1 mg/da.


Los efectos secundarios ms frecuentes se refieren a la
esfera sexual.
Acta inhibiendo la 5-alfa-reductasa a nivel folicular.
Se logra un recrecimiento capilar evidente hasta en el 90%
de los casos.
Una vez suspendido el tratamiento, se pierde el beneficio
transcurridos unos meses.

3)
4)
5)
57.

Este caso debe tratarse con corticoides orales.


El curso es progresivo e invariable.
Se asocia a la presencia de dermatitis seborreica.
Existe atrofia cutnea en los mrgenes de la placa, lo que
origina la presencia de pelos peldicos.
La produccin de melanina a nivel del pelo afectado no
se altera.

2)
3)
4)
5)

Es frecuente que venga precedida de una infeccin del


tracto respiratorio alto.
En ocasiones se asocia a malignidad, fundamentalmente
carcinomas pulmonares.
Los antibiticos no alteran el curso de la enfermedad.
El tratamiento de eleccin son los corticoides orales.
Histolgicamente aparece edema en dermis papilar y un
intenso infiltrado en dermis de neutrfilos, sin vasculitis.

Varn de 53 aos, diagnosticado de enfermedad de Crohn


desde hace 10 aos. Presenta desde hace 2 meses una lesin

Pg. 6 DM

M exico A rgentina
C hile U ruguay

Puede encontrarse melanosis e hipertricosis.


Est ocasionada por un dficit de uro III descarboxilasa.
Su orina ser de color marrn.
Es frecuente la afectacin heptica.
Debe tratarse retirando el alcohol y administrando corticoides.

Un nio de 15 meses que presenta placas erosivas eritematosas, periorificiales y acras, diarrea, fragilidad ungueal y
alopecia, debe tratarse con:
1)
2)
3)
4)
5)

60.

Dficit de vitamina C.
Dficit de cido nicotnico.
Dficit de zinc.
Glucagonoma.
Dficit de vitamina B6.

Varn de 48 aos, alcohlico crnico, que presenta en dorso


de manos, cara y orejas lesiones ampollosas, tensas, de
contenido seroso. En la inmunofluorescencia no se detecta
depsito de anticuerpo alguno. Seala la FALSA respecto a la
enfermedad que padece este paciente:
1)
2)
3)
4)
5)

59.

Probablemente tenga una hiperfuncin tiroidea.


La correccin del trastorno de base no influye en la
evolucin de las lesiones cutneas.
El tratamiento con corticoides potentes es bastante satisfactorio.
Las lesiones pueden adoptar aspecto elefantisico en
ocasiones.
Se debe descartar tambin afectacin ocular.

Mujer de 53 aos, bebedora en exceso, consulta por enrojecimiento de cara y cuello en el ltimo mes, cada vez con
mayor intensidad. En las dos ltimas semanas han brotado
ampollas de contenido claro en dorso de antebrazos y
manos. En la ltima semana se ha aadido un incremento en
el nmero de deposiciones, pero sin productos patolgicos.
Cul es su primera sospecha diagnstica?:
1)
2)
3)
4)
5)

58.

Ceftacidina.
Cloroquina.
Desbridamiento quirrgico.
Corticoides.
Acitretina.

Varn de 48 aos, con presencia de placas rosadas, infiltradas con dilatacin de orificios foliculares en la cara anterior
de ambas piernas. La biopsia de una de ellas demuestra el
depsito de mucina en grandes cantidades en la dermis.
Seale la FALSA:
1)
2)

Varn de 43 aos de edad, que acude a consulta por fiebre,


artralgias y ppulas y placas eritematosas, dolorosas induradas distribuidas por el tronco. En el hemograma destaca
leucocitosis con neutrofilia. Seale la FALSA, con respecto a
esta enfermedad:
1)

55.

56.

Varn de 27 aos, opositor, que desde hace 30 das presenta


5 placas alopcicas en cuero cabelludo, sin otras alteraciones
cutneas. Seale la correcta, respecto a esta enfermedad:
1)
2)
3)
4)

54.

1)
2)
3)
4)
5)

Seale la FALSA sobre el tratamiento con finasteride de la


alopecia andrognica masculina:
1)
2)

53.

La afectacin mucosa es frecuente.


Pueden afectarse en un 10% de los casos las uas.
Se trata con corticoides.
Los retinoides no se muestran eficaces en este proceso.
Debe solicitarse una serologa heptica.

ulcerada, con crecimiento perifrico y localizada en pierna


derecha. Seale el tratamiento correcto de su enfermedad
cutnea:

Qu tratamiento empleara usted ante una mujer de 18 aos


con acn papulopustuloso moderado, leve alopecia difusa e
hirsutismo facial moderado?:
1)
2)
3)
4)
5)

52.

DERMATOLOGA

Preparacin Examen de Seleccin 05/06 1 Vuelta

Vitamina C.
Acido nicotnico.
Sulfato de zinc.
Esteroides orales.
Dieta exenta en gluten.

Sealar la correcta sobre la necrobiosis lipodica:

CTO Medicina C/ Nez de Balboa, 115 28006 MADRID (Espaa) Tfno.: (91) 782 43 32 / Fax: (91) 782 43 27
E-mail: secretaria@ctomedicina.com; iberocto@ctomedicina.com WEB: www.ctomedicina.com; www.iberocto.com

Preguntas TEST

Seguimiento a distancia

DERMATOLOGA

Preparacin Examen de Seleccin 05/06 1 Vuelta


1)
2)
3)
4)
5)
61.

Mujer de 68 aos, con lesiones papulosas y eritematosas de


localizacin periorificial y en superficies flexoras, que descaman y van creciendo por la periferia, de varias semanas de
evolucin, acompaadas de diarrea, anemia, malabsorcin,
prdida de peso e hiperglucemia. Cul es su diagnstico?:
1)
2)
3)
4)
5)

62.

3)
4)
5)

Preguntas TEST

5)

69.

Fomentos con sulfato de zinc.


Corticoides tpicos.
Batera estndar de pruebas epicutneas.
Seguramente deber evitar contactos con sustancias que
contengan cromo.
Los antihistamnicos tpicos son poco tiles para tratar el
cuadro.

Cara.
Porcin extensora de extremidades.
Tronco.
Palmas y plantas.
Flexuras de codos y rodillas.

Cul de las siguientes respuestas es un dato caracterstico


del eccema atpico del lactante?:
M exico A rgentina
C hile U ruguay

Cloxacilina.
Ciclosporina.
Hidrocortisona.
Budesonida en fomento.
Acitretina.

Varn de 47 aos de edad, que consulta por presentar desde


hace 5 das una placa violcea, de bordes bien definidos, no
infiltrada, pruriginosa, centrada por una vescula en la cara.
Refiere la ingesta de un medicamento que no recuerda el da
de la aparicin de la lesin. Haba presentado algn episodio
previo. Seale la FALSA con respecto a esta patologa:
1)
2)
3)
4)
5)

71.

Suele comenzar entre los 20 y 30 aos.


Cursa con habones de gran tamao.
Suele respetar palmas y plantas.
Puede cursar con hipotensin y dolor abdominal.
El desencadenante puede ser un aumento de la temperatura corporal por la sudacin.

Paciente de 63 aos, cardipata e hipertenso, en tratamiento


desde hace 3 semanas con captopril. Desde hace 7 das
presenta un exantema pruriginoso por toda la superficie
corporal, que en las ltimas 48 horas ha evolucionado en
forma de grandes erosiones y despegamientos de mucosas.
Usted considera ms adecuado el tratamiento con:
1)
2)
3)
4)
5)

70.

Loratadina.
Hidroxicina.
Corticoides orales.
Dexclorfeniramina.
Anti-H1 + anti-H2.

Una de las siguientes caractersticas de la urticaria colinrgica es FALSA:


1)
2)
3)
4)
5)

La forma infantil de la dermatitis atpica suele localizarse


tpicamente en:
1)
2)
3)
4)
5)

66.

68.

Predominio de lesiones de prrigo.


Aparecer inmediatamente tras el nacimiento.
Ausencia de prurito.
Afectacin flexural.
Respetar el tringulo nasogeniano.

Un varn de 27 aos consulta por la aparicin de hinchazn


y dolor en dorso de pies, que aparece entre 3-6 horas despus
de caminar. El dolor y la hinchazn se mantienen varias horas,
incluso en alguna ocasin hasta 1 da. Cmo lo tratara?:
1)
2)
3)
4)
5)

Las manchas caf con leche son el signo ms precoz.


La presencia de "pecas" axilares puede considerarse casi
patognomnico.
El tumor intracraneal solitario ms frecuente es un glioma
del nervio ptico.
Se ha demostrado que tiene una herencia autosmica
recesiva.
Los neurofibromas pueden experimentar una transformacin sarcomatosa.

Mujer de 35 aos consulta por lesiones eritematosas, vesiculosas, exudativas, muy pruriginosas en los lbulos de los
pabellones auriculares, de varios meses de evolucin. Cul
de las siguientes actitudes NO considera adecuada?:
1)
2)
3)
4)

65.

67.

Angiofibromas faciales.
Adenomas sebceos.
Tumores de Koenen.
Ndulos de Lisch.
Mancha hipocrmica en hoja de fresno.

NO es cierto que, en la neurofibromatosis:


1)
2)

64.

Acantosis nigricans.
Sndrome de Bazex.
Eritema necroltico migratorio.
Acrodermatitis enteroptica.
Sndrome carcinoide.

1)
2)
3)
4)
5)

La lesin ms precoz de la esclerosis tuberosa es:


1)
2)
3)
4)
5)

63.

Las zonas de aparicin ms frecuentes son los brazos.


Son placas atrficas, eritematosas y amarillentas con telangiectasias.
No dejan lesiones residuales cuando curan.
Responden muy bien a los corticoides tpicos.
Siguen un curso paralelo a la evolucin de la glucemia.

Seguimiento a distancia

Frecuentemente se afectan las mucosas.


Al curar, no deja alteraciones de la pigmentacin.
Pueden existir varias lesiones a la vez.
Al tomar de nuevo el frmaco responsable, la lesin
reaparece en la misma localizacin.
Sulfamidas, barbitricos y AINEs son frmacos habitualmente responsables.

En el vitligo, es cierto que:


1)
2)
3)
4)
5)

No existe predisposicin gentica.


Casi siempre se encuentran factores desencadenantes.
Las lesiones del vitligo generalizado tienden a la simetra.
Histolgicamente existe disminucin de melanosomas.
El tratamiento habitual es con corticoides orales.

CTO Medicina C/ Nez de Balboa, 115 28006 MADRID (Espaa) Tfno.: (91) 782 43 32 / Fax: (91) 782 43 27
E-mail: secretaria@ctomedicina.com; iberocto@ctomedicina.com WEB: www.ctomedicina.com; www.iberocto.com

DM Pg. 7

Seguimiento a distancia

Varn de 62 aos de edad, consulta por aparicin de lesiones


en tronco y extremidades que han brotado en los ltimos 2
meses, pigmentadas, sobreelevadas, de aspecto aterciopelado, con comedones en superficie y asintomticas. Usted
sospecha:
1)
2)
3)
4)
5)

73.

Con respecto a las siguientes dermatosis paraneoplsicas, es


cierto que:
1)
2)
3)
4)
5)

74.

El sndrome carcinoide puede cursar con lesiones semejantes al escorbuto.


El eritema necroltico migratorio es un cuadro que se
asocia a glucagonoma.
La acantosis nigricans maligna respeta casi siempre las
mucosas.
La ictiosis adquirida es tpica de los tumores slidos.
La alopecia mucinosa del adulto se asocia a cncer de
estmago.

Sealar la afirmacin FALSA respecto a la telangiectasia


macular eruptiva perstans:
1)
2)
3)
4)
5)

75.

Acantosis nigricans maligna.


Signo de Darier.
Signo de Leser-Trlat.
Sndrome de Gryzybowsky.
Sndrome de Gorlin.

Son lesiones maculosas telangiectsicas.


Un 25% tienen afectacin sistmica.
Evoluciona en brotes.
Se presenta en adultos.
Presentan signo de Darier.

Se conoce como sndrome de Sturge-Weber:


1)
2)
3)
4)
5)

Pg. 8 DM

Hipertrofia hemiangiectsica de un miembro.


Aparicin de mltiples verrugas seborreicas.
Angiomatosis encefalotrigeminal.
Malignizacin de un nevus organoide.
Angiocondromatosis.

M exico A rgentina
C hile U ruguay

CTO Medicina C/ Nez de Balboa, 115 28006 MADRID (Espaa) Tfno.: (91) 782 43 32 / Fax: (91) 782 43 27
E-mail: secretaria@ctomedicina.com; iberocto@ctomedicina.com WEB: www.ctomedicina.com; www.iberocto.com

Preguntas TEST

72.

DERMATOLOGA

Preparacin Examen de Seleccin 05/06 1 Vuelta

DERMATOLOGA

Preparacin Examen de Seleccin 05/06 1 Vuelta


Pregunta 1.- R: 4
La queratinizacin precoz y anmala que ocurre en capas profundas de la epidermis puede ocurrir en procesos benignos como la
enfermedad de Darier o malignos como la enfermedad de Bowen o el
carcinoma espinocelular. En las primeras fases de la enfermedad de
Darier se observa prdida de adhesin de las clulas epidrmicas, con
acantlisis. Adems presentan clulas en el estrato espinoso que se
denominan cuerpos redondos. Los cuerpos redondos son clulas
disqueratsicas que muestran una parcial y prematura queratinizacin y se caracterizan por poseer un ncleo central oscuro y picntico
con citoplasma claro eosinfilo.
En el liquen plano se aprecia un denso infiltrado linfocitario que se
dispone en banda por la dermis superior. En la epidermis se aprecia
una acantosis irregular, en dientes de sierra, y queratinocitos degenerados, disqueratsicos, que se denominan cuerpos slidos o cuerpos
de Civatte.
En el pnfigo se aprecia formacin de una ampolla intraepidrmica por acantlisis. Ver figura a pie de pgina.

Comentarios TEST

Pregunta 2.- R: 5
Al ser lesiones en zonas pilosas ser preferible emplear frmulas
galnicas como una solucin, locin, gel o una espuma. Cuando las
lesiones son queratsicas, descamativas, es preferible emplear vehculos grasos o muy grasos como cremas, pomadas y ungentos. En los
casos de lesiones agudas, con componente exudativo, se deben emplear vehculos con alto contenido en agua como los fomentos, pastas
al agua y soluciones. La difusin de una sustancia a travs de la capa

Seguimiento a distancia

crnea es un proceso pasivo que sigue las leyes de difusin de Fick; es


directamente proporcional al coeficiente de particin de la droga
entre su vehculo y la crnea e inversamente proporcional al espesor
de esta capa. Por tanto la liberacin del principio activo de su vehculo y la difusin en la capa crnea es tanto mayor cuanto menos sea la
afinidad de la droga por el vehculo.

Pregunta 2.

Tratamiento tpico en dermatologa.

Pregunta 1. Alteraciones histolgicas cutneas.


M exico A rgentina
C hile U ruguay

CTO Medicina C/ Nez de Balboa, 115 28006 MADRID (Espaa) Tfno.: (91) 782 43 32 / Fax: (91) 782 43 27
E-mail: secretaria@ctomedicina.com; iberocto@ctomedicina.com WEB: www.ctomedicina.com; www.iberocto.com

DM Pg. 1

DERMATOLOGA

Preparacin Examen de Seleccin 05/06 1 Vuelta

Pregunta 3.- R: 3
El signo de Darier consiste en la aparicin de habones sobre una
lesin, cuando sta es traumatizada por friccin o presin. La presencia de un signo de Darier es patognomnico de mastocitosis. Las
mastocitosis son proliferaciones benignas de mastocitos. Estos pueden
proliferar por todos los rganos pero la mayora de las ocasiones lo
hacen por la piel, en especial en dermis. Ah forman unos acmulos
llamados mastocitomas. Estos mastocitomas le otorgan a la piel una
coloracin griscea justo en la zona donde se encuentran. Cuando
rascamos estas mculas aparece un habn, ya que los mastocitos
subyacentes degranulan la histamina.
De ah que la forma clnica ms frecuente de mastocitosis se denomina urticaria pigmentosa. No se debe confundir un signo de Darier,
con un dermografismo. En el dermografismo los habones aparecen
en cualquier zona de la piel sobre la que aplicamos un traumatismo,
mientras que el signo de Darier slo se presenta cuando esa presin la
ejercemos sobre una zona de piel con lesin, es decir sobre un
mastocitoma.
Pregunta 4.- R: 5
Los retinoides son derivados de la vitamina A y se emplean en
muchas patologas dermatolgicas. Fundamentalmente empleamos
la isotretinona y la acitretina. Los retinoides ejercen una variedad de
efectos biolgicos, como disminuir la capacidad proliferativa celular,
regular la diferenciacin celular tanto a tejidos adultos como embrionarios, son capaces de evitar o retardar la carcinognesis y son inhibidores de la queratinizacin. La isotretinona se emplea sobre todo en
el acn severo y la acitretina o etretinato en las formas graves de psoriasis.
Los efectos secundarios son muy parecidos en ambos frmacos con
alguna diferencia. El efecto ms grave es el que concierne a la

teratogenia, que hace que sea imprescindible la anticoncepcin hasta despus de 1 mes en el caso de la isotretinona, y hasta dos aos
despus en el caso de la acitretina. Otro efecto secundario importante
es la hepatotoxicidad, siendo el ms grave en los varones. La aparicin
de xerosis cutnea y de queilitis son los efectos secundarios ms frecuentes (95% de los casos).
En un 12% de los casos se producen modificaciones seas o
ligamentosas, con presencia de calcificaciones. Por este motivo los
retinoides estn contraindicados en la infancia. Adems no deben
asociarse tratamientos con tetraciclinas por el peligro de pseudotumor cerebral.
Los retinoides tpicos tienen efectos secundarios leves, y por lo
general no requieren cuidados especiales.
Pregunta 5.- R: 5
En la anamnesis destaca que las lesiones son pruriginosas y que el
prurito es de predominio nocturno. Adems, la localizacin de las
mismas es en lugares caractersticos como las caras laterales de los
dedos, y con una morfologa papulosa y en ocasiones lineales. Todo
esto debe hacernos pensar que el paciente presenta una escabiosis.
Las escabiosis se caracterizan por prurito intenso de predominio nocturno y prurito familiar, ya que la sarna es extremadamente contagiosa, cuando existe un contacto personal. El diagnstico de presuncin
nos lo da por tanto la historia clnica y la exploracin fsica, aunque la
certeza slo la tendremos cuando observemos el caro en la piel. Esto
se logra con el raspado de las lesiones por lo que la respuesta 2 es
correcta. Ver figura a pie de pgina.
La aparicin de las lesiones nodulares se deben a una hipersensibilidad del caro. A este tipo de sarna se le denomina sarna nodular.
Para la desaparicin de las lesiones nodulares no slo es necesario

Pregunta 5. Sarna.

Pg. 2 DM

M exico A rgentina
C hile U ruguay

CTO Medicina C/ Nez de Balboa, 115 28006 MADRID (Espaa) Tfno.: (91) 782 43 32 / Fax: (91) 782 43 27
E-mail: secretaria@ctomedicina.com; iberocto@ctomedicina.com WEB: www.ctomedicina.com; www.iberocto.com

Comentarios TEST

Seguimiento a distancia

DERMATOLOGA

Preparacin Examen de Seleccin 05/06 1 Vuelta


matar los caros sino que es necesario que se eliminen fsicamente los
restos de los caros. La sarna nodular no se trata nicamente con
escabicidas sino que precisa el uso de antihistamnicos y de corticoides. El tratamiento de eleccin de la sarna es la permetrina al 5% que
se emplea de forma tpica. Se puede emplear tanto en embarazadas
como en nios pequeos. El lindano es una alternativa de uso tpico,
pero es neurotxico y muy irritante, por lo que no se debe emplear en
embarazadas ni en nios. En las sarnas muy extendidas o en
inmunodeprimidos puede usarse la ivermectina oral.
Pregunta 6.-R: 5
La paciente presenta una lesin eritematosa y migratoria. Adems
entre sus antecedentes destaca que tiene un contacto habitual con los
perros. Todo sto debe llevarnos a pensar que la paciente presenta
una enfermedad de Lyme. Esta enfermedad est causada por una
espiroqueta, Borrelia burgdorferi, y es transmitida por la picadura de
una garrapata del gnero Ixodes. Despus de la picadura, y tras un
perodo de incubacin de entre 7 y 20 das, aparece desde ese lugar
un eritema homogneo, anular, que se va extendiendo. Esta lesin
puede persistir meses o semanas y acompaarse en la mitad de los
casos de fiebre o cefalea. Como dice la respuesta 2 la afectacin
articular es tarda y aparece en una minora de los pacientes. La afectacin sistmica tambin incluye afectacin cardaca en forma de
bloqueo aurculo-ventricular y afectacin neurolgica. Tanto la afectacin cardaca como la neurolgica suelen ser afectaciones precoces y no tardas como se expone en la respuesta 5.
El diagnstico de esta enfermedad se realiza mediante serologas y
el tratamiento debe incluir tetraciclinas cuando no existe afectacin
visceral y slo es cutnea, y s debe incluir en el caso de afectacin
visceral tratamiento con penicilina.

Pregunta 6. Enfermedades cutneas transmitidas por artrpodos.

Comentarios TEST

*DUUDSDWDV)LHEUHERWRQRVDPHGLWHUUiQHD 5LFNHWVVLDFRQRULL 
(QIHUPHGDGGH/\PH %RUUHOLDEXJGRUIHUL 
0RVTXLWRV%RWyQGH2ULHQWH /HLVKPDQLD 

Seguimiento a distancia

cida por la Rickettsia Conorii, y transmitida por una garrapata del


gnero Riphicephalus. La respuesta 1 es correcta ya que el exantema
aparece por todo el cuerpo, incluyendo palmas y plantas. En algunos
casos se puede apreciar una pequea afectacin heptica. La respuesta 3 es falsa ya que la mancha negra slo se aprecia en el 75% de
los casos. La mancha negra no es ni ms ni menos que el lugar de la
picadura de la garrapata. Despus aparece fiebre y cefalea que se
puede prolongar durante das o incluso semanas. Por eso la fiebre
botonosa mediterrnea es una de las causas que hay que buscar en
una fiebre de origen desconocido.
El exantema es lo ltimo en aparecer, y no suele ser pruriginoso. El
diagnstico de una fiebre botonosa mediterrnea se fundamenta en
la realizacin de serologas y en la clnica.
Suele ser una enfermedad que evoluciona de forma satisfactoria
con una respuesta espectacular a las tetraciclinas.
Pregunta 9.- R: 3
La respuesta correcta es la 3 por distintos motivos.
En primer lugar tanto el liquen plano pilar como el lupus discoide
no son patologas tpicas de la infancia. Adems ambas patologas
producen una destruccin del folculo piloso y por tanto dejan como
secuela alopecia cicatricial. En la descripcin clnica no se nos da
ningn dato que nos oriente hacia la existencia de tejido cicatricial.
La tricotilomana consiste en arrancarse el pelo de forma consciente o
inconsciente. Eso podra justificar la presencia de pelos rotos pero no
el eritema, la descamacin ni el prurito. El cuanto a la alopecia areata,
este proceso s es bastante frecuente en la infancia, en especial en
nios que presentan dermatitis atpica. La alopecia areata podra dar
pelos rotos pero nunca veramos la sintomatologa aadida de este
caso como el eritema y la descamacin. En la alopecia areata se
aprecia un infiltrado linfocitario perifolicular pero la piel est intacta.
Por tanto, la nica posibilidad era la respuesta 3. La tia tonsurante
se ve sobre todo antes de la adolescencia y produce una alopecia
reversible. En este caso, una buena alternativa teraputica ser la
griseofulvina, dada la edad del paciente.
Adems el tratamiento ser oral, ya que trataremos de forma oral
aquellas micosis con afectacin de pelo, uas, inflamatorias o presencia de varias placas.

Pregunta 7.-R: 4
En el enunciado llama la atencin de una histologa con histiocitos
que contienen en el citoplasma una serie de microorganismos. Esta
histologa es tpica de la leishmaniasis. Un pioderma gangrenoso no
puede ser ya que no suele aparecer en la cara, sino ms frecuentemente en las extremidades. Adems no es propio de la infancia. El
ntrax y el imptigo contagioso hubiesen evolucionado a mejor o a
peor durante esas 5 semanas, pero no hubiera permanecido con la
misma morfologa durante tanto tiempo. El lupus vulgar es la forma
ms frecuente de tuberculosis cutnea, pero su localizacin suele ser
en la regin preauricular. El paciente de esta pregunta presenta una
leishmaniasis cutnea del viejo mundo o botn de Oriente. Es una
enfermedad producida por protozoos, en nuestro medio por la
Leishmania donovani, variedad infantum y por la Leishmania trpica. El
vector de transmisin de la enfermedad es una mosca Phlebotomus,
siendo el reservorio de la enfermedad los perros. El botn de Oriente
deja inmunidad y debemos pensar en l cuando nos encontremos
ante una lesin papulosa, que tarda en curar, localizada en la cara de
un nio o nia de pocos aos de edad.
Adems de la clnica nos orienta en el diagnstico la histologa y el
diagnstico inmunolgico que se efecta mediante una intradermorreaccin, la prueba de Montenegro, que resulta positiva en el 75%
de los casos.
En este paciente el tratamiento se realizara mediante inyecciones
intralesionales de gluconato de antimonio.
Pregunta 8.-R: 3
Tenemos un paciente con fiebre, cefalea y lesiones cutneas. Entre
sus antecedentes tenemos el hecho de que ha estado en el campo
unos das antes de que se iniciara el cuadro clnico. Adems el paciente tiene un exantema, que junto a la fiebre y sus antecedentes
debe hacernos pensar en fiebre botonosa mediterrnea. Est produM exico A rgentina
C hile U ruguay

Pregunta 9.

Tia tonsurante.

CTO Medicina C/ Nez de Balboa, 115 28006 MADRID (Espaa) Tfno.: (91) 782 43 32 / Fax: (91) 782 43 27
E-mail: secretaria@ctomedicina.com; iberocto@ctomedicina.com WEB: www.ctomedicina.com; www.iberocto.com

DM Pg. 3

Pregunta 10.- R: 1
Clnicamente estamos ante una placa bien definida, caliente, dolorosa, eritematosa, y que se acompaa de fiebre y leucocitosis. Por
tanto estamos ante una erisipela.
La respuesta falsa es la 1, ya que el diagnstico se realiza bien
mediante hemocultivos o por la clnica. La localizacin ms frecuente
de las erisipelas es en los miembros inferiores, en especial en mujeres
con antecedentes de insuficiencia venosa, diabetes mellitus u obesidad. El herpes zoster no siempre da vesculas, con lo que en ocasiones
puede cursar con la presencia de placas eritematosas y dolorosas, que
pueden llevarnos a confundirlo con una erisipela. Esto sucede de
manera frecuente en la cara, en especial con el herpes zoster trigeminal.
La diferencia entre una erisipela y una celulitis es que en esta ltima,
se llega a afectar el tejido celular subcutneo, por tanto tiene una
afectacin ms profunda. Este hecho hace que los lmites de una
celulitis estn mal definidos. Tanto la erisipela como la celulitis, van a
tener afectacin del estado general.
El tratamiento tanto de las celulitis como de la erisipela debe incluir
penicilina administrada de forma sistmica. La evolucin con el tratamiento adecuado suele ser positiva y sin dejar secuelas.
Pregunta 11.- R: 5
En este caso llama la atencin que la paciente presenta lesiones
que se abscesifican, con una evolucin muy larga pues llega a los dos
aos. Adems la localizacin es especfica de ambas regiones inguinales y parece que con los tratamientos intentados se logra mejorar el
proceso, pero acaba recidivando. Un chancro blando produce fistulizacin y abscesificacin inguinal, pero no es bilateral. Adems en los
antecedentes no aparece el chancro. El linfogranuloma venreo tambin da abscesificacin y fistulizacin bilateral. Lo que ocurre es que
hubiera respondido a algunos de los ciclos de antibiticos orales, que
suponemos habran incluido las tetraciclinas. Tampoco figuran antecedentes que nos hagan sospechar una enfermedad de transmisin
sexual. La enfermedad de Crohn puede en algunos casos dar fstulas
perianales pero no cuadra con el enunciado de la pregunta. En este
caso estamos ante una hidrosadenitis crnica supurativa.
La hidrosadenitis supurativa aparece en zonas apocrinas. El tratamiento es con cloxacilina, o con ciclos largos de tetraciclinas.
En algunos casos es necesario recurrir a la exresis quirrgica de la
glndula o al drenaje quirrgico de los abscesos.
En los casos ms recidivantes se pueden emplear los retinoides
orales, la isotretinona, que mejoran sustancialmente el cuadro.
Pregunta 12.- R: 4
La varicela se caracteriza por la presencia de lesiones en diferentes
estados, es decir, en fase de ppula, vescula y costra. Cada vez que
vuelven a aparecer lesiones papulosas significa que existe una nueva
vivencia. Adems la varicela se acompaa de un gran quebrantamiento del estado general, con fiebre alta y cefalea. La varicela no debe ser
tratada salvo que sea una varicela en un inmunodeprimido, neonato,
que exista alguna complicacin o que la afectacin cutnea sea muy
intensa. Por eso en este caso la respuesta correcta es la 4.
La complicacin ms frecuente de la varicela es la sobreinfeccin
bacteriana de las lesiones cutneas. Una de las complicaciones graves
de la varicela es la neumona por el virus varicela zoster. Esta neumona puede cursar con escasas alteraciones en las imgenes radiolgicas, por lo que en ocasiones, en especial si sospechamos que el paciente presenta esta complicacin es recomendable la realizacin de
una gasometra arterial.
Cuando la varicela precisa un tratamiento especfico se realiza con
aciclovir a dosis de 800mg/5h durante 7 das. La varicela no complicada slo precisa un tratamiento sintomtico para el prurito y para la
fiebre.
Pregunta 13.- R: 1
Vemos en esta nia que las lesiones que presenta son ppulas y
vesculas, pero el estado general es bueno, con lo que descartamos
que se trate de una varicela. La dermatitis herpetiforme cursa con
lesiones papulosas, vesiculosas, que se distribuyen por las zonas de
extensin de manera simtrica. La dermatitis herpetiforme no suele
comenzar en la infancia y adems se asocia con mucha frecuencia
Pg. 4 DM

M exico A rgentina
C hile U ruguay

DERMATOLOGA

Preparacin Examen de Seleccin 05/06 1 Vuelta

a la enfermedad celaca pero no a alteraciones hepticas. Por la


clnica podramos pensar en las picaduras de insecto pero es excepcional que se acompaen de alteraciones en las enzimas hepticas.
En cuanto a la enfermedad de pie-mano-boca la distribucin de las
lesiones no se corresponde con la descripcin de la pregunta. La
respuesta correcta es la 1. Esta enfermedad, autolimitada, representa
un patrn de respuesta, propia de la edad infantil frente a la primoinfeccin por el virus de la hepatitis B. La clnica tiene una instauracin brusca, distribuyndose por extremidades, cara, nalgas, palmas
y plantas. La hepatitis suele iniciarse de manera simultnea. No tiene
un tratamiento especfico, pues es un cuadro autoinvolutivo. Cuando se produce por otros virus distintos al VHB hablamos del Sndrome de Gianotti-Crosti en vez de enfermedad de Gianotti-Crosti.
Pregunta 14. R: 4
La lepra tuberculoide afecta a la piel y a los nervios perifricos. En
la piel se aprecian una o pocas mculas, hipocrmicas, asimtricas
y que por afectacin de los anejos son alopcicas y secas. Llama la
atencin que estas mculas son anestsicas o hipoestsicas debido a
la precoz afectacin sensitiva que se produce en la lepra tuberculoide.
La prueba de la lepromina es muy positiva. Slo el 1% de las lepras
tuberculoides acaban siendo lepras lepromatosas. Los bordes de las
lesiones cutneas de la lepra tuberculoide son ligeramente elevados,
muy bien delimitados y en ocasiones con morfologa anular.
Las lesiones cutneas de la lepra lepromatosa son mltiples mculas amplia y simtricamente distribuidas, con bordes muy mal definidos y que con el paso del tiempo se engrosarn formando placas.
Estas placas en la cara dan origen a la facies leonina, con prdida del
pelo de cejas y pestaas (madarosis). En la lepra lepromatosa la afectacin neurolgica es mas tarda que en la tuberculoide.

Pregunta 14. Diagnstico diferencial de LT y LL.

7XEHUFXORLGH

/HSURPDWRVD

0iFXODV

1yGXORVOHSURPDV

OHVLRQHV

0XFKDVOHVLRQHV

$VLPpWULFDV

6LPpWULFDV

(ULWHPDQRGRVR (1 DXVHQWH

(1SUHVHQWHFRQIUHFXHQFLD

(QJURVDPLHQWRDVRFLDGRGH

(QJURVDPLHQWRGLIXVR

QHUYLRVSHULIpULFRV

$IHFWDFLyQWDUGtD

$IHFWDQHUYLRV

$QHVWHVLDOLJHUDRDXVHQWH

$QHVWHVLDLQWHQVD
%LRSVLDSLHO
*UDQXORPDVWXEHUFXORLGHV

,QILOWUDGRGHKLVWLRFLWRVLV

&pOVWLSR/DQJKDQV

PDFUyIDJRVFRQJORELV

$QHVWHVLDLQWHQVD

%DQGDGH8QQD

5HDFFLRQHV
0LWVXGD

0LWVXGD

%DFLORVFRSLD

%DFLORVFRSLD

7UDWDPLHQWR
5LIDPSLFLQDPJPHV

5)3PJPHV

6XOIRQDPJGtD

6)PJGtD
&ORIDFLPLQDPJPHV

7UDWDPLHQWRPHVHV

7UDWDPLHQWRDxRV

6HJXLPLHQWRDxRV

DxRV

/HSURUUHDFFLRQHVWLSR,

7LSR
)HQyPHQRGH/XFLR

Pregunta 15.- R: 4
El sarcoma de Kaposi es una proliferacin vascular multifocal que
puede afectar tanto a la piel como a las mucosas. Existen cuatro variantes, clsico, endmico, asociado a inmunosupresores y asociado a SIDA.
En el Kaposi asociado a SIDA existen datos epidemiolgicos que indican la etiologa vrica del cuadro, especficamente por un virus herpes
8. Las lesiones se inician como pequeas ppulas violceas que a dife-

CTO Medicina C/ Nez de Balboa, 115 28006 MADRID (Espaa) Tfno.: (91) 782 43 32 / Fax: (91) 782 43 27
E-mail: secretaria@ctomedicina.com; iberocto@ctomedicina.com WEB: www.ctomedicina.com; www.iberocto.com

Comentarios TEST

Seguimiento a distancia

DERMATOLOGA

Preparacin Examen de Seleccin 05/06 1 Vuelta


rencia del clsico suelen localizarse ms en la cara y en el tronco. No
suele ser la causa de muerte del paciente ya que cuando se inician las
terapias antirretrovirales el cuadro suele estabilizarse e incluso
involucionar. La afectacin de la mucosa oral acontece en un 25% de
los pacientes. Adems, cuando existe afectacin extracutnea, sta suele localizarse en ganglios linfticos, pulmn y mucosa gastrointestinal.
Las alteraciones histolgicas son idnticas en las cuatro variantes de
Kaposi.
En cuanto al tratamiento del Kaposi asociado al VIH se emplea la
radioterapia, interfern alfa y ltimamente se estn realizando estudios clnicos con derivados de los retinoides.

Comentarios TEST

Pregunta 16.- R: 2
En la histologa de la lesin ya nos habla de la presencia de mitosis y
de mltiples atipias celulares. Esto ya nos debe inducir a pensar en que
la lesin de la paciente se trata de un tumor. Una psoriasis en placas no
puede ser, ya que adems de que no presenta una histologa especfica,
no es lgico pensar que una placa de psoriasis va a crecer de forma
progresiva y se va a localizar en el mismo emplazamiento durante un
perodo tan largo de tiempo. Un herpes circinado, tia corporis, s
puede persistir muchos aos, incluso con crecimiento progresivo, pero
la biopsia tampoco es como nos la exponen. En cuanto a las queratosis
seborreicas, son lesiones benignas, con proliferacin epidrmica, sin
llegar a traspasar la membrana basal. Pero por supuesto una queratosis
seborreica no tendra atipias. La enfermedad de Paget se localizar en la
areola mamaria. En su histologa es muy caracterstico que las clulas no
establezcan puentes intercelulares entre ellas.
La enfermedad de Bowen es un carcinoma epidermoide in situ
que ocurre con preferencia en el tronco. La aparicin de lesiones
mltiples hacen necesario descartar intoxicaciones por arsnico. En
este caso, al ser un carcinoma in situ, en vez de ciruga podramos
realizar una electrocoagulacin o radioterapia.
Pregunta 17.- R: 1
La lesin que presenta la paciente, es una lesin papulosa, brillante, blanquecina. Esto nos indica que se trata de un epitelioma
basocelular. El tratamiento de eleccin de un epitelioma basocelular
es la extirpacin quirrgica. No son necesarios ms que escasos mrgenes ya que no existe el riesgo de metstasis. La presencia de
hiperglucemias mantenidas en una mujer de 77 aos no debe ser una
contraindicacin quirrgica. La respuesta 2, realizacin de un TAC
abdominal, no tiene sentido, ya que el carcinoma basocelular no
metastatiza. La realizacin de radioterapia se reserva para aquellos
epiteliomas basocelulares, de gran tamao, en los que la ciruga sera
una mala solucin teraputica. La radioterapia no est indicada en
los carcinomas basocelulares esclerodermiformes, ya que tiene malos
resultados. En cuanto al interfern y al imiquimod son alternativas
teraputicas en epiteliomas basocelulares en los que tampoco sera
factible la ciruga. Tanto el interfern alfa, como el imiquimod tienen
efectos inmunomoduladores. Tambin se estn empleando ambas
sustancias en el tratamiento de algunas formas clnicas de melanoma.
Pregunta 18.-R: 2
El carcinoma basocelular es el tumor maligno ms frecuente. Lo es
an ms que el de la prstata, mama o pulmn. Su nombre se debe a
estar formado por clulas de la capa basal y de la epidermis y de los
anejos cutneos. Histolgicamente el tumor est formado por clulas
epiteliales de la capa basal, que se distribuyen formando nidos y
cordones, y se disponen en empalizada. En ocasiones los carcinomas
basocelulares pueden contener elevadas cantidades de melanina y
mostrar un aspecto pigmentoso. El carcinoma basocelular tambin se
denomina epitelioma basocelular porque a diferencia de otros
carcinomas slo de forma excepcional produce metstasis. El carcinoma basocelular se distribuye por zonas en las que existen folculos
pilosebceos. Es ms frecuente sobre las lneas de cierre embrionario
y sobre lesiones hamartomatosas anexiales, no coincidiendo en cambio con las zonas de mayor incidencia de radiacin solar. Se piensa
por tanto que existiran factores locales, como la presencia de clulas
pluripotenciales del germen epitelial primario, que participaran en la
etiopatogenia de este tumor.
El carcinoma basocelular no se localiza nunca en las mucosas. Un
M exico A rgentina
C hile U ruguay

Seguimiento a distancia

paciente que ya ha presentado un carcinoma basocelular es posible


que a lo largo de su vida tenga otros. sto se debe a que se supone que
ese paciente ha estado expuesto de forma crnica e intensa a las
radiaciones solares.
Pregunta 19.- R: 4
El nevus sebceo de Jadasshon es una proliferacin localizada de
lbulos de glndulas sebceas. Es muy caracterstico que se localice
en el cuero cabelludo, como una placa congnita, con frecuencia
con ausencia de pelo. La placa es de consistencia elstica, coloracin
amarillenta y superficie ligeramente rugosa. Es frecuente que con los
aos aparezcan sobre el nevus sebceo otras lesiones de naturaleza
tumoral. La lesin tumoral benigna que aparece con mayor frecuencia es el siringocistoadenoma papilfero, y de los malignos el epitelioma basocelular, que aparece en el 30% de los casos. Su imagen clnica y localizacin le hacen inconfundible aunque en ocasiones se
puede confundir con una aplasia cutis o un xantogranuloma juvenil.
El problema de degeneracin maligna acontece a partir de la pubertad. Por eso el tratamiento consiste en la extirpacin quirrgica antes
de la pubertad. En el caso de que no se pudiera extirpar por ser muy
amplia la lesin o por otros factores, entonces se debern seguir revisiones peridicas. La respuesta 1 trataba de confundirnos pues lo que
existe es el adenoma sebceo de Pringle, que en realidad se trata de
angiofibroma que se localiza en la regin centrofacial y es muy especfica de la esclerosis tuberosa.
Pregunta 20.- R: 2
La epidermodisplasia verruciforme o tambin llamada enfermedad de Lewandwosky-Lutz es una enfermedad hereditaria con carcter autosmico recesivo que determina una respuesta anormal frente
a la infeccin por el VIH. La clnica consiste en la aparicin de mltiples lesiones, bien verrugas planas o bien verrugas vulgares. En un 30%
de los pacientes existe transformacin maligna de las lesiones, el tumor al cual degeneran con mayor frecuencia es al carcinoma
epidermoide.
El carcinoma epidermoide o epitelioma espinocelular es el tumor
maligno ms frecuente en mucosas. Es excepcional que aparezca
sobre piel sana ya que tiene tendencia a localizarse sobre lesiones
preexistentes.
Los factores que predisponen a la aparicin de un carcinoma epidermoide son la gentica (personas de piel clara), radiacin (exposicin a la
radiacin ultravioleta), lesiones cutneas inflamatorias y degenerativas
crnicas (lupus vulgar, lupus crnico, cicatrices, radiodermitis, quemaduras, etc.), inmunosupresin, infecciones virales (en especial las ocasionadas por VPH), lesiones precancerosas (queratosis cutneas). Las localizaciones ms comunes del carcinoma espinocelular son aquellas que estn ms expuestas a las radiaciones ultravioletas.
Pregunta 21.- R: 5
Bien, nos hablan de una lesin de 3 aos de evolucin, que le
ocasiona al paciente cierto prurito. La lesin era verrucosa, de coloracin marrn oscura. Como vemos en ningn momento nos hablan
de ppulas perladas, brillo blanquecino o superficie brillante. Por
tanto no podemos pensar que se trate de un carcinoma basocelular.
Por la descripcin clnica podra tratarse de un melanoma o de un
nevus melanoctico. Lo que ocurre es que en la histologa nos estn
hablando de proliferacin de clulas epidrmicas y presencia de globos crneos. Un melanoma o un nevus melanoctico presentaran en
la histologa melanocitos, que en el caso de melanoma seran atpicos.
La queratosis seborreica es un tumor benigno de una incidencia
muy alta en la raza blanca. Su nmero ha aumentado con el paso de
los aos. Clnicamente son lesiones papulosas, blandas, untuosas o
grasientas al tacto. No se ha descrito transformacin maligna. Debe
plantearse el diagnstico diferencial en algunos casos, en especial con
el epitelioma basocelular pigmentoso y con el melanoma. El tratamiento de las queratosis seborreicas es la extirpacin mediante
curetaje, electrocoagulacin o crioterapia.
Pregunta 22.- R: 3
En esta pregunta llama la atencin que el paciente presente unas
lesiones papulosas, perladas, localizadas por toda la superficie corpo-

CTO Medicina C/ Nez de Balboa, 115 28006 MADRID (Espaa) Tfno.: (91) 782 43 32 / Fax: (91) 782 43 27
E-mail: secretaria@ctomedicina.com; iberocto@ctomedicina.com WEB: www.ctomedicina.com; www.iberocto.com

DM Pg. 5

ral. Clnicamente las lesiones se corresponden con la descripcin


morfolgica de carcinomas basocelulares. Ahora bien, no parece lgico encontrar carcinomas basocelulares en una persona joven. Adems el carcinoma basocelular se relaciona con la exposicin solar
crnica, y un minero no est expuesto, en condiciones normales, a
las radiaciones solares. El cuadro que presenta el paciente es un sndrome de Gorlin. El sndrome del Nevus basocelular o sndrome de
Gorlin es una enfermedad gentica que sigue un patrn de herencia
autosmica dominante. Afecta por igual a ambos sexos aunque es
ms frecuente en personas de piel clara. Adems de cursar con la
aparicin de mltiples epiteliomas basocelulares presenta mltiples
lesiones pigmentadas en palmas y plantas, anomalas esquelticas, en
especial queratoquistes odontognicos maxilares y cierto retraso mental. No incluye como se cita en la respuesta 3 afectacin ocular. En el
tratamiento del sndrome de Gorlin se han utilizado los retinoides por
va oral para prevenir el desarrollo de nuevas lesiones e impedir la
progresin de las ya existentes.
Pregunta 23.- R: 2
El que las lesiones expuestas en la pregunta se encuentren en las
zonas del labio inferior, nos debe hacer pensar que el sol tiene un
papel en el desarrollo de estas lesiones. Adems existen antecedentes
de exposicin solar como es el hecho de que se trate de un agricultor
y adems existe un factor carcinognico como es el hbito tabquico.
El liquen plano oral suele aparecer en la mucosa lingual o geniana
pero es raro que aparezca en la mucosa labial. Las aftas orales son
lesiones erosivas, ulceradas, no aparecen en la semimucosa ni se relacionan con la exposicin solar. El exantema fijo medicamentoso tiene
predileccin por aparecer en mucosas, aunque nunca persiste la lesin tanto tiempo, 3 aos, y no se relaciona con el sol. La queilitis
actnica es el equivalente de la queratosis actnica pero en este ltimo
caso localizada en la semimucosa del labio inferior. Su diagnstico y
tratamiento son importantes debido a la alta tasa de metstasis del
carcinoma epidermoide de labio. Siempre que existan dudas debe
realizarse una biopsia para tener un diagnstico de certeza. El tratamiento de las queratosis actnicas se basa en la crioterapia, el lser
CO2 y el uso del 5-fluoracilo.
Pregunta 24.- R: 1
Los dos grupos de nevus melanocticos que parecen asociarse
significativamente con un mayor riesgo de transformacin maligna a
melanoma son los nevus congnitos de gran tamao y los nevus
displsicos. Un nevus congnito grande o gigante, tiene un riesgo de
degeneracin a melanoma que oscila entre el 5% y el 20% segn las
series publicadas. En cualquier caso, el riesgo es muy alto. La paciente de la pregunta presenta un nevus congnito de gran tamao
pues le ocupa la cara externa del muslo izquierdo. Por esa razn la
respuesta mejor sera la de extirparlo quirrgicamente en la medida
de lo posible. Si el nevus congnito fuera inoperable deberamos
entonces observarlo y en el momento que tuviramos dudas acerca
de si existe transformacin maligna de la lesin en algn lugar deberamos extirpar esa zona y someterla a un estudio histolgico. El
hacer biopsias semestrales de la lesin es un absurdo ya que siempre
quedaran zonas sin biopsiar.
En cuanto a la eliminacin con lser de la lesin no sera una
buena alternativa teraputica. El lser lo que hace es quemar. En un
nevus congnito existen melanocitos hasta en la dermis profunda,
con lo que la quemadura tendra que ser muy agresiva. Adems luego
quedara una cicatriz que podra enmascarar la degeneracin maligna de alguno de los melanocitos que hubieran quedado. Adems el
resultado esttico no sera bueno en absoluto.
Pregunta 25.-R: 4
El factor pronstico ms importante en el melanoma en estadio I es
la profundidad medida en milmetros desde la capa granulosa de la
epidermis. A esto se le llama ndice de Breslow. Este factor pronstico
ha sustituido a los niveles de invasin de Clark que relacionan el nivel
de penetracin del tumor con el pronstico. Otros factores histolgicos
que otorgan un factor pronstico desfavorable son un alto ndice
mittico y la existencia de satelitosis microscpica. Existen adems
factores pronsticos clnicos como el hecho de que las mujeres tenPg. 6 DM

M exico A rgentina
C hile U ruguay

DERMATOLOGA

Preparacin Examen de Seleccin 05/06 1 Vuelta

gan un pronstico ms favorable que los varones. Adems las lesiones


en las extremidades tienen mejor pronstico que las localizadas en el
tronco, cabeza y cuello. No sucede lo mismo con las localizadas en
las palmas, plantas y subungueales. En cuanto al riesgo de tener
melanoma, es ms alto cuanto ms bajo el fenotipo cutneo, es decir,
cuanto ms clara es la piel del paciente. El melanoma es ms frecuente en mujeres que en varones con una proporcin de 2 a 1. Es un
tumor muy raro antes de la pubertad, pero cuando lo hace antes de la
pubertad, el 50% de las ocasiones aparece sobre un nevus congnito
gigante.
El melanoma es actualmente el tumor con mayor tasa de incidencia en los adultos jvenes.

Pregunta 25. Clasificacin de Clark.

Pregunta 26.- R: 3
La paciente presenta una lesin de pocos aos de evolucin, localizada en una extremidad. En la histologa de la lesin se aprecian
melanocitos atpicos con muchas mitosis. Por lo tanto debemos pensar
en un melanoma. El melanoma ms frecuente de todos es el melanoma
de extensin superficial. Adems esta forma clnica de melanoma es la
ms frecuente en los adultos y jvenes. El melanoma de extensin superficial se presenta inicialmente como una lesin plana, que paulatinamente se va elevando. El color oscila entre el marrn y el negro.
Cuando al principio vemos que la lesin es plana, se corresponde con
la fase radial del crecimiento. Posteriormente la superficie se va haciendo irregular con zonas papulosas e incluso nodulares. Esto significa que
el melanoma de extensin superficial ha entrado en fase de crecimiento vertical o invasiva. La respuesta 3 es la falsa ya que el crecimiento
radial del melanoma de extensin superficial se prolonga durante un
mximo de 5 aos. El lntigo maligno melanoma, es el melanoma ms
frecuente en sujetos de edad avanzada. Su fase de crecimiento radial s
que puede durar hasta dcadas. La extirpacin precoz de la lesin es el
mejor tratamiento. Debe realizarse una extirpacin con mrgenes, que
se determinan en funcin del ndice de Breslow.
Pregunta 27.- R: 1
El tratamiento del melanoma cutneo primario se realiza mediante
extirpacin quirrgica con mrgenes de seguridad. Actualmente se recomienda en los melanomas con un ndice de Breslow menor de 1 ampliar
mrgenes hasta en 1 cm. Cuando el ndice de Breslow es mayor de 1 mm
se recomienda extirpar 2 cms de margen, hasta llegar a la fascia. La realizacin de la tcnica del ganglio centinela se realiza en aquellos melanomas con una profundidad o ndice de Breslow mayor a 1. Esta tcnica
consiste en estudiar si el ganglio ms cercano al tumor se encuentra
afectado o no. La tcnica del ganglio centinela no parece que mejore el
pronstico del paciente, slo logra diagnosticar mejor la enfermedad. El
uso de interfern alfa en el melanoma se reserva para los casos con un
elevado ndice de Breslow y por tanto con altas posibilidades de desarrollar metstasis. Esto es as porque el interfern alfa parece disminuir o
retrasar el desarrollo de metstasis. Por eso en la pregunta la respuesta
correcta es la 1. Cuando ya existen metstasis desarrolladas, el interfern
alfa no parece jugar ningn papel.
Pregunta 28.- R: 5
La micosis fungoide es un proceso linfoproliferativo maligno T
inductor/colaborador de causa desconocida. Se origina primaria-

CTO Medicina C/ Nez de Balboa, 115 28006 MADRID (Espaa) Tfno.: (91) 782 43 32 / Fax: (91) 782 43 27
E-mail: secretaria@ctomedicina.com; iberocto@ctomedicina.com WEB: www.ctomedicina.com; www.iberocto.com

Comentarios TEST

Seguimiento a distancia

DERMATOLOGA

Preparacin Examen de Seleccin 05/06 1 Vuelta

Seguimiento a distancia

Comentarios TEST

Pregunta 28. Micosis fungoide.

mente en la piel pudiendo permanecer en ella hasta dcadas. A


partir de ah puede afectar ganglios linfticos, rganos internos y
acarrear la muerte del paciente. Las fases premicticas son aquellas
fases previas al diagnstico histolgico de la enfermedad. Las lesiones en la fase premictica pueden ser confundidas con mltiples
dermatosis como los eccemas, psoriasis o tias. En ocasiones la enfermedad puede comenzar como una eritrodermia como el hombre rojo de Hallopeau o con la forma agresiva de linfoma cutneo
que es el Sndrome de Szary. En la tercera fase de la micosis
fungoide, fase tumoral, aparecen lesiones tumorales sobre la piel
normal o sobre las lesiones previas. A partir de esta fase comienzan a
producirse manifestaciones extracutneas, afectndose con mucha
frecuencia los ganglios linfticos, hgado y bazo. La afectacin de la
mdula sea es muy infrecuente.
La respuesta 5 es falsa ya que en la micosis fungoide las manifestaciones ganglionares y viscerales aparecen al cabo de aos de evolucin. Ver dibujo en la pgina siguiente.
Pregunta 29.- R: 1
No existen tratamientos curativos de la micosis fungoide, sino
que los tratamientos que se emplean lo que logran es retrasar la
enfermedad y prolongar la vida de los pacientes. La eleccin del
tratamiento va a depender del estadio evolutivo de la enfermedad.
En la fase de eccema y de placas recurrimos a los tratamientos
tpicos. Los que ms se emplean son los corticoides tpicos y la
mostaza nitrogenada. La mostaza nitrogenada tiene un efecto secundario que es la alta frecuencia con la que induce hipersensibilidad retardada. Este fenmeno ha sido propuesto como uno de
los mecanismos por los que frenara a las clulas neoplsicas. Tambin en las primeras fases de la enfermedad se han empleado la
fototerapia (UVB) y la fotoquimioterapia (PUVA). En la fase tumoral
se emplea el tratamiento con radioterapia, ya que las lesiones cutneas de micosis fungoide son muy sensibles al efecto de las radiaciones ionizantes.
En la fase tumoral tambin se emplean el interfern alfa, la acitretina y la fotofresis extracorprea con la que se han obtenido muy
buenos resultados, con remisiones de hasta el 80%.

M exico A rgentina
C hile U ruguay

Pregunta 29. Tratamiento de la micosis fungoide segn estadios.

Pregunta 30.- R: 3
El sndrome de Szary viene definido por la trada de eritrodermia,
aparicin en sangre perifrica de ms de un 10% de clulas de Szary,
que son clulas linfoides atpicas, y linfadenopatas generalizadas. La
respuesta 1 es falsa ya que el sndrome de Szary ocasiona mucho
prurito, lgico al presentar eritrodermia. El sndrome de Szary, a diferencia de la micosis fungoide, tiene una evolucin corta. Existe discusin an sobre si la eritrodermia del sndrome de Szary es diferente o
no de la que puede presentar una micosis fungoide. Para algunos
autores ambos procesos seran la misma entidad patolgica. La respuesta 3 es correcta ya que al existir linfadenopatas generalizadas es
frecuente que aparezca hepatomegalia o esplenomegalia.
Pregunta 31.- R: 5
Se trata de una mujer con lesiones eritematosas en ambas mejillas, que
claramente presentan historia de fotosensibilidad. Dado el sexo femenino y la edad de la paciente deberamos pensar en dos patologas: roscea
o lupus eritematoso. En la roscea son tpicas la presencia, en algunos

CTO Medicina C/ Nez de Balboa, 115 28006 MADRID (Espaa) Tfno.: (91) 782 43 32 / Fax: (91) 782 43 27
E-mail: secretaria@ctomedicina.com; iberocto@ctomedicina.com WEB: www.ctomedicina.com; www.iberocto.com

DM Pg. 7

DERMATOLOGA

Preparacin Examen de Seleccin 05/06 1 Vuelta

estados, de lesiones papulosas o pustulosas. Este tipo de lesiones no se


observan en el lupus eritematoso. Adems las respuestas de la pregunta
nos orientan ms hacia la roscea. El tratamiento de la roscea es difcil.
Un punto primordial es evitar los estmulos que facilitan la vasodilatacin
como las bebidas calientes, el sol, picantes, cambios bruscos de temperatura, etc. El tratamiento de eleccin son las tetraciclinas orales, que se
emplean a las mismas dosis que en el acn vulgar. Como alternativa se
pueden emplear el metronidazol bien en formulacin tpica o de forma
oral. En los casos resistentes o muy intensos se pueden utilizar los retinoides
(isotretinona) de forma oral. El tratamiento del rinofima requiere correccin quirrgica. La respuesta falsa es la 5 ya que en la roscea no existe
compromiso de rganos internos, cosa que s hubiera sido factible en el
caso de tratarse de un lupus eritematoso.

Clnicamente cursa con la aparicin, casi desde el nacimiento, de


mltiples lesiones de coloracin oscura, grandes, que se distribuyen
por el tronco y extremidades, afectando con frecuencia a las flexuras.
En el 50% de los casos aparecen opacidades a nivel corneal. La enfermedad se transmite de manera autosmica recesiva ligada al cromosoma X. La histopatologa no es especfica y el tratamiento consiste en
el empleo de emolientes y en los casos severos el empleo de retinoides
orales.

Pregunta 33. Tipos de ictiosis.


,FWLRVLVOHYHV

,FWLRVLVYXOJDU

Pregunta 31. Dermatosis agravadas por el sol.

/(6
3RUILULDV
'DULHU
5RViFHD
3pQILJRIROLiFHR

+HUSHV
3UHFDQFHURVLV [S FiQFHU
)RWRDOHUJRIRWRWR[LFRGHUPLDV
3HODJUDFDUFLQRLGH
$OELQLVPRRFXORFXWiQHR

Pregunta 32.- R: 3
La enfermedad de Paget representa un carcinoma epidermotropo
de los conductos glandulares de la mama. Ante cualquier lesin eccematosa en una mama siempre debemos establecer el diagnstico diferencial con la enfermedad de Paget del pezn. Clnicamente se caracteriza por ser lesiones generalmente unilaterales y de lento crecimiento.
Las lesiones suelen tener bordes muy bien delimitados y son unilaterales por lo que la respuesta 3 es falsa. Normalmente afecta a mujeres
mayores de 40 aos, siendo excepcional en varones.
Las clulas de Paget son clulas de gran tamao, con grandes ncleos
y sin desmosomas o tonofibrillas. La mastectoma es el tratamiento ms
adecuado con linfadenectoma regional si precisa. No est recomendada la extirpacin cutnea local o la irradiacin de las lesiones cutneas.
La enfermedad de Paget extramamaria clnica e histolgicamente se parece a la de la mama aunque la mayora de las veces no asocia un
carcinoma subyacente. Se localiza con ms frecuencia en la regin
anogenital y regiones axilares. En cerca del 20% de los casos localizados
en regin anogenital se encuentra un carcinoma primario en otros rganos, principalmente recto, uretra, vejiga y cuello uterino.
Pregunta 33.- R: 5
La ictiosis X afecta a los varones aunque portadores femeninos
pueden mostrar rasgos fenotpicos aislados. La enfermedad se produce debido a que existe una ausencia de la enzima microsomal sulfatasa
esteroidea. La ausencia de la enzima microsomal sulfatasa esteroidea
determina un aumento de colesterol sulfato en el estrato crneo a
cuyo nivel aparece un aumento de la extensin intracorneal caracterstica de la enfermedad.

Pregunta 33. Histologa de la ictiosis.

Pg. 8 DM

M exico A rgentina
C hile U ruguay

,FWLRVLVJUDYHV

,FWLRVLVOLJDGDDO
;

,FWLRVLV

,FWLRVLVODPHODU

HSLGpUPLFD

$'

/LJDGDDO;

$5

$'

5DUR

5DUR

6t

6t

3DWRJHQLD

'HVFRQRFLGD

$XVHQFLDGH
DU\OVXOIDWDVD
&

'HVFRQRFLGD

'HVFRQRFLGD

$3

+LSHUTXHUDWRVLV
PLWRVLV
+LSHUTXHUDWRVLV +LSHUTXHUDWRVLV +LSHUTXHUDWRVLV 9DFXROL]DFLyQ
+LSHUJUDQXORVLV
PLWRVLV
GHFpOXODV
JUDQXORVDV

$VRFLDFLRQHV

$WRSLD

2SDFLGDGHV
FRUQHDOHVVLQ
FHJXHUD

1RWDV

/RPiV
IUHFXHQWH

(VFDPDV
RVFXUDV

7UDWDPLHQWR

(PROLHQWHV\TXHUDWROtWLFRV

+HUHQFLD

3UHVHQWHDO
QDFLPLHQWR

2SDFLGDGHV
FRUQHDOHVFRQ 'HIRUPLGDGHV
FHJXHUD
3UHVHQWDQ
DPSROODV
5HWLQRLGHVRUDOHV

Pregunta 34.- R: 4
Destaca en el enunciado la aparicin de lesiones descamativas,
nacaradas. Una de las enfermedades que cursa con descamacin
nacarada es la psoriasis. Adems el hecho de que las lesiones sean ppulas
y que aparezcan en una paciente de pocos aos nos debe hacer pensar
en una psoriasis en gotas. La psoriasis en gotas cursa con numerosas
lesiones pequeas en gotas que se distribuyen en especial por el tronco.
Es una enfermedad que aparece fundamentalmente en la infancia. Suele aparecer despus de infecciones del tracto respiratorio superior, especialmente despus de infecciones estreptoccicas. La psoriasis en gotas
tiene buen pronstico ya que se resuelve espontneamente despus de
unas semanas tras la infeccin estreptoccica. La psoriasis en gotas no
suele por tanto tener afectacin articular. Respecto a la respuesta 2, la
aparicin de lesiones de psoriasis en mucosas es muy infrecuente. Cuando aparece en mucosas lo hace en menos del 1% de los pacientes. La
respuesta 5 es falsa ya que al tener buen pronstico el tratamiento se
fundamenta en el uso de productos tpicos.
Pregunta 35.- R: 3
La respuesta correcta es la 3. El aspecto histolgico de una biopsia
est relacionado con el estadio evolutivo de la lesin. La primera
alteracin objetivable consiste en afectacin vascular con dilatacin
capilar, edema papilar e infiltrado mononuclear perivascular. En esta
fase puede existir paraqueratosis. Adems las crestas papilares se alargan con exudacin de neutrfilos hacia la epidermis. Estos neutrfilos
emigran hacia las zonas paraqueratsicas constituyendo los microabscesos de Munro. Tambin aparecen neutrfilos en la porcin superior del estrato espinoso, as se constituyen las pstulas espongiformes
de Kogoj. Otros hallazgos caractersticos de la psoriasis son la ausencia
de capa granulosa, acantosis e hiperqueratosis. El fenmeno del roco
hemorrgico de Auspitz se debe a la papilomatosis, al estar elongadas
las papilas drmicas los capilares suben hasta niveles muy altos con lo
que fcilmente se rompen al desprender escamas de una placa de
psoriasis.

CTO Medicina C/ Nez de Balboa, 115 28006 MADRID (Espaa) Tfno.: (91) 782 43 32 / Fax: (91) 782 43 27
E-mail: secretaria@ctomedicina.com; iberocto@ctomedicina.com WEB: www.ctomedicina.com; www.iberocto.com

Comentarios TEST

Seguimiento a distancia

DERMATOLOGA

Preparacin Examen de Seleccin 05/06 1 Vuelta


Pregunta 36.- R: 4
El paciente padece una psoriasis, aparentemente grave porque no
responde a la teraputica tpica y tambin debido a que afecta a una
gran extensin de la superficie corporal.
Dentro de los antecedentes del paciente vemos que es un hepatpata por el alcohol y por la infeccin por el VHC. Si furamos a
instaurarle un tratamiento la respuesta 1 la descartaramos debido a
que el Metotrexate es hepatotxico. En cuanto a la respuesta 2 es una
mala opcin ya que los corticoides orales o sistmicos no deben
usarse en la psoriasis debido a que cuando se suspenden los enfermos
presentan un rebrote en su enfermedad. En la prctica clnica los
corticoides sistmicos en la psoriasis llegamos a usarlos en casos extremos. La respuesta 3, acitretina, sera el tratamiento de eleccin en
cualquier psoriasis grave. El problema es que en el presente caso no
podemos emplearla ya que uno de los efectos secundarios de los
retinoides es la hepatotoxicidad.
Como alternativa a los retinoides en la psoriasis grave, empleamos
la ciclosporina. El principal efecto secundario de la ciclosporina es
que es nefrotxica. En cuanto a la ltima respuesta, el calcipotriol,
es falsa ya que es un producto tpico que se emplea en psoriasis
leves.

Comentarios TEST

Pregunta 37.- R: 2
De nuevo nos encontramos ante una psoriasis grave, con afectacin de mas del 40% de la superficie corporal total. El tratamiento de
eleccin sera la acitretina. El problema que nos encontramos es que
la pacientes es una mujer en edad frtil con lo que tendramos que
mantener anticoncepcin durante al menos 2 aos. Adems los tratamientos con retinoides son tratamientos largos. La respuesta 2 es la
falsa ya que la acitretina no tiene las mismas acciones que la isotretinona. El principal efecto de la isotretinona es la disminucin de la
secrecin sebcea por atrofia de las glndulas, adems disminuye la
extensin de los queratinocitos del infundibular folicular, favoreciendo la eliminacin de comedones.
La alternativa a los retinoides es la ciclosporina, y la alternativa a
esta ltima es el uso de metotrexate. El metotrexate es de eleccin en
la artropata psorisica. La PUVA est indicada principalmente en las
psoriasis en placas que abarcan el 30% de la superficie corporal.
Tambin puede emplearse en las psoriasis graves el RePUVA que es la
combinacin de los retinoides y el PUVA.
Pregunta 38.- R: 4
El tratamiento de eleccin de una psoriasis leve son los corticoides
tpicos. Los tratamientos de la psoriasis varan en funcin de la localizacin, severidad, tratamientos previos, sexo y edad del paciente. En
la psoriasis pustulosa y eritrodrmica el tratamiento de eleccin es la
acitretina que en ocasiones se puede asociar a la PUVA, siendo muy
efectivo. Como alternativa a la acitretina podemos emplear la
ciclosporina. El tratamiento de eleccin del liquen plano son los
corticoides, que se emplearn de forma tpica o sistmica en funcin
de la extensin. En los casos severos de liquen plano podemos emplear la acitretina. En el caso de las enfermedades ampollosas
autoinmunes el tratamiento de eleccin son los corticoides, salvo en
la dermatitis herpetiforme en la que emplearemos sulfonas. Los corticoides se emplean en el pnfigo a dosis altas. La respuesta correcta es
la 4 pues la pitiriasis rubra pilaris se trata con queratolticos y acitretina
ya que los corticoides no se muestran eficaces.
Pregunta 39.- R: 3
Destaca en el enunciado que existe una lesin de mayor tamao,
y otras que siguen una distribucin en rbol de Navidad. Los hallazgos
clnicos son caractersticos constituyendo la clave para el diagnstico.
La lesin de gran tamao suele ser la lesin inicial, placa herldica,
que habitualmente se localiza en el tronco. El resto de las lesiones
tienen una morfologa ovalada, con descamacin perifrica, semejantes a la placa herldica pero con un menor tamao. La distribucin por el tronco, en rbol de Navidad, es muy caracterstica. Las
lesiones son asintomticas, sin producir prurito. Las lesiones persisten
4 - 8 semanas, llegando a regresar espontneamente. Por este motivo
la respuesta falsa es la 3. La pitiriasis rosada de Gibert es de causa
desconocida, aunque por el hecho de la variabilidad estacional y de
M exico A rgentina
C hile U ruguay

Seguimiento a distancia

que sean muy raras las recidivas, se ha postulado que tuviera una
etiologa vrica, probablemente por virus herpes. El tratamiento de la
pitiriasis rosada es con emolientes. La exposicin solar mejora el cuadro. Ver dibujo en la siguiente pgina.
Pregunta 40.- R: 4
La pitiriasis rubra pilar cursa con la aparicin de lesiones papulosas
queratsicas, foliculares, que confluyen formando placas que de forma caracterstica contienen islotes de piel sana respetada. En cuanto a
la dermatitis seborreica es una patologa que no cursa con ppulas y
que adems mejora con la exposicin solar. La enfermedad de HaileyHailey es una enfermedad gentica, autosmica dominante, que tambin se denomina pnfigo benigno familiar. Cursa con lesiones
vesiculosas por axilas, cuello y regiones inguinales. Se han obtenido
buenos resultados con el tratamiento de esta enfermedad con el empleo de antibiticos o antifngicos tpicos. La respuesta correcta es la
4 ya que la enfermedad de Darier tiene unos hallazgos clnicos muy
tpicos, como son la localizacin de las lesiones en zonas seborreicas,
queratosis puntata en palmas y plantas y la presencia de tonalidades
blanquecinas y rojizas con una morfologa en V en el borde libre
ungueal. Los pacientes con enfermedad de Darier deben protegerse
del sol ya que este empeora mucho la enfermedad. En formas extensas se emplean los retinoides ya que los corticoides no mejoran e
incluso pueden exacerbar la enfermedad.

Pregunta 39. Pitiriasis rosada. Recordad que es ms frecuente en mujeres.

Pregunta 41.- R: 2
Nos encontramos ante un varn, adulto, que presenta lesiones
ampollosas y erosivas en la cavidad oral, mucosa genital y lesiones
ampollosas por toda la superficie corporal.
Ante este caso nos podemos plantear que el paciente tiene un
imptigo ampolloso, ya que las ampollas del imptigo por estafilococo suelen ser flccidas. El problema es que el imptigo ampolloso no
afecta mucosas y adems no suele acompaarse de mal estado general, fiebre etc. Otra patologa que nos podemos plantear como responsable del cuadro es el pnfigo vulgar. El pnfigo vulgar aparece en
estados medios de la vida, cursa con ampollas flccidas que aparecen
sobre piel aparentemente sana. La afectacin mucosa en el pnfigo
vulgar es prcticamente constante tanto de la oral como de la genital.
El pnfigo vulgar es una enfermedad grave que puede comenzar lentamente, con un escaso nmero de lesiones y permanecer as largo
tiempo. Aproximadamente en el 50% de los casos, las lesiones se
inician en las mucosas. Al ser una enfermedad grave el tratamiento
debe ser agresivo, empleando para ello inmunosupresores. De eleccin se emplean corticoides a dosis altas y administrados de forma
sistmica. Cuando existe contraindicacin en el uso de corticoides o
falta de respuesta, se emplean otros inmunosupresores como la azatioprina, ciclofosfamida o micofenolato.

CTO Medicina C/ Nez de Balboa, 115 28006 MADRID (Espaa) Tfno.: (91) 782 43 32 / Fax: (91) 782 43 27
E-mail: secretaria@ctomedicina.com; iberocto@ctomedicina.com WEB: www.ctomedicina.com; www.iberocto.com

DM Pg. 9

DERMATOLOGA

Preparacin Examen de Seleccin 05/06 1 Vuelta

Seguimiento a distancia

Pregunta 44. Diagnstico diferencial de las enfermedades ampollosas.

(WLRSDWRJHQLD
&OtQLFD
$IHFWDFLyQ
PXFRVD
$3

3HQILJRLGH

$XWRLQPXQH
)ROLiFHR
5DUR
DPSROODV
(URVLRQHV

9XOJDU
DPSROODV
IOiFFLGDV
6t
$PSROOD
LQWUDHSLGpU
PLFD
VXSUDEDVDO

1LNROVN\

1R

$XWRLQPXQH
$PSROODVWHQVDV
VREUHSLHO
HULWHPDWRVD
3UXULWRLQWHQVR

+HUSHV
*HVWDWLRQLV

'HUPDWLWLV
KHUSHWLIRUPH

$XWRLQPXQH

*OXWHQ

/HVLRQHV
KHUSHWLIRUPHVHQ
DEGRPHQPX\
SUXULJLQRVDV

6t 

1R

$XWRLQPXQH

/HVLRQHVKHUSHWLIRUPHV +LSHUIUDJLOLGDG
PX\SUXULJLQRVDVHQ
FXWiQHD
iUHDVGHH[WHQVLyQ

(QWUHSHQILJRLGH
\'KHUSHWLIRUPH

1R

$PSROOD
$PSROOD
$PSROOD
LQWUDHSLGpU VXEHSLGpUPLFD
VXEHSLGpUPLFD
FRQLQILOWUDGR
PLFDHQ
GpUPLFRHRVLQyILOR FRQHRVLQyILORV
JUDQXORVD

(SLGHPyOLVLV
'HUPDWRVLV,J$
DPSROORVDDGTXLULGD
OLQHDO

$PSROOD
$PSROOD
VXEHSLGpUPLFDFRQ
PLFURDEVFHVRVGHSPQ VXEHSLGpUPLFD
HQODVSDSLODVGpUPLFDV FROiJHQR9,,

,)'

,J*HQWUHORVTXHUDWLQRFLWRV ,J*OLQHDOHQPE ,J*OLQHDOHQPE ,J$JUDQXODUHQYpUWLFH


GHSDSLODV

,J$OLQHDOHQPE

,),

HQHO

HQHO

HQHO

,J$JUDQXODUHQYpUWLFH
GHSDSLODV

,J$OLQHDOHQPE

772

&RUWLFRLGHVVLVWpPLFRV
,QPXQRVXSUHVRUHV

&RUWLFRLGHV
VLVWpPLFRV

&RUWLFRLGHV
VLVWpPLFRV

6XOIRQDGLHWDVLQ
JOXWHQ

1RWDV

(OSpQILJRYXOJDUHVHOPiV
JUDYH

(OSHQILJRLGH
FLFDWUL]DOHVXQD
YDULDQWH

Pregunta 41. Diagnstico diferencial de lesiones en mucosa oral.


/LTXHQSODQRGHPXFRVDRUDO
WDPELpQHQRWUDV
ORFDOL]DFLRQHV 

5HWLFXODGRSDSXODUEODQTXHFLQRHQWHUFLR
SRVWHULRUGHPXFRVD\XJDOGHODUJD
HYROXFLyQ0ROHVWLDVYDULDEOHV

/LTXHQHURVLYRGHPXFRVDV
IRUPDSRFRIUHFXHQWHGH
OLTXHQSODQR 

/HVLRQHVHURVLYDVURGHDGDVGHiUHDV
EODQTXHFLQDVSDSXODUHVPX\GRORURVDV
HQPXFRVD\XJDO$YHFHVTXHLOLWLV
DGKHUHQWH

$IWRVLVUHFXUUHQWH GHVFDUWDU
5HLWHUQRGXHOHQ%HKoHW(76
FHOLDTXtDHQILQIODPDWRULD
LQWHVWLQDO 

OFHUDVRUDOHVGRORURVDVGHGLiPHWUR
YDULDEOHTXHFXUDQ
HQPHQRVGHGtDVVLQUHODFLyQFRQ
RWUDSDWRORJtD

/(6 ODV~OFHUDVVRQXQ
FULWHULRGLDJQyVWLFR 
0X\LQHVSHFtILFDVGHMDQ
FLFDWUL]

OFHUDVRUDOHVFRQHULWHPDFLUFXQGDQWH
JLQJLYLWLVSHWHTXLDVTXHLOLWLVGHVFDPDWLYD
DGKHUHQWHSODFDVUHWLFXODUHVSODWHDGDV

3pQILJR (QHOHVOD
PDQLIHVWDFLyQ 

$PSROODJUDQGHIOiFFLGDVHGHVSUHQGH\
GD~OFHUDVDPSOLDVTXHSURJUHVDQKDVWD
ODELRV(YROUiSLGDPDOHVWDUJHQHUDO

3HQILJRLGH 1RHVOD
PDQLIHVWDFLyQ 
'HVFDUWDUHSLGHUPyOLVLV
DPSROORVD

(URVLRQHVRUDOHVFRQHULWHPDSHULIpULFR
5HVSHWDQHOODELR)UHFTXHLOLWLV
GHVFDPDWLYD7LHQGHQDFXUDU\QR
SURJUHVDQ

6G6WHYHQV-RKQVRQ HULWHPD
PXOWLIRUPHPD\RU 
'HVFDUWDUQHFUyOLVLVHSLGpUPLFD
Wy[LFD SRVWIiUPDFRV 

(ULWHPDHGHPDJUDOL]DGRDPSROODV~OFHUDV
FRQLQIODPDFLyQSHULIpULFD\0(*HQ
MyYHQHVWUDVLQIHFFLyQSRU0\FRSODVPD

*LQJLYRHVWRPDWLWLVSRUKHUSHV
VLPSOH
6LYHVtFXODVHQRURIDULQJH
KHUSDQJLQD &R[VDFNLH$ 

,GHP HULWHPDHGHPD~OFHUDVPDOHVWDU
FRQDGHQRSDWtDV\SDUWLFLSDFLyQGHHQFtDV

Pg. 10 DM

M exico A rgentina
C hile U ruguay

&RUWLFRLGHV
VLVWpPLFRV
,QPXQRVXSUHVRUHV

6XOIRQD

SUHVHQWDQ
LQWROHUDQFLDDOJOXWHQ
SHURVyORHOFRQ
FOtQLFD
Pregunta 42.- R: 3
Nos encontramos ahora ante un paciente con aparente buen estado general, de edad avanzada, con lesiones ampollosas tensas por el
tronco y extremidades. Con ampollas tensas cursan aquellas patologas que tienen despegamiento subepidrmico. El herpes gestationis
cursa con despegamiento subepidrmico pero como es natural, en
un varn, y encima de 74 aos, es "difcil" que llegue a tener un herpes
gestationis.
La dermatitis herpetiforme tambin tiene despegamiento subepidrmico y por tanto ampollas tensas, pero esta opcin la descartamos
ya que la dermatitis herpetiforme no suele aparecer a esas edades tan
avanzadas y menos an sin su distribucin herpetiforme. Por lo tanto
estamos ante un penfigoide ampolloso. Lo ms adecuado para realizar un diagnstico de certeza en cualquier enfermedad ampollosa de
aparente etiologa inmunolgica es la realizacin de una biopsia de la
piel perilesional. Debe ser de la zona perilesional porque si se recoge
del centro de la lesin veremos que existe una ampolla pero no seremos capaces de ver el depsito de anticuerpos. Por eso realizaremos
una biopsia perilesional y tambin mediante tcnicas de
inmunofluorescencia directa determinaremos el tipo de anticuerpo
depositado y dnde lo ha hecho. La inmunofluorescencia indirecta
en el penfigoide no aporta mucha informacin y apenas tiene valor.
En cuanto a la respuesta a los corticoides, el penfigoide suele responder bien a dosis moderadas de los mismos, pero por supuesto eso
no es diagnstico como insina la respuesta 2.
Pregunta 43.- R: 4
Se trata de un paciente de corta edad, con lesiones ampollosas
pruriginosas. En la biopsia existe un depsito de IgA lineal. Una de las
enfermedades que cursa con depsito de IgA en la dermis es la dermatitis
herpetiforme. Pero la dermatitis herpetiforme tiene un depsito granular,
en las papilas drmicas, y no de forma lineal. La respuesta 1 es correcta
pues si bien la sulfona es el tratamiento de eleccin de la dermatitis
herpetiforme, tambin es el tratamiento de la dermatosis IgA lineal de la
infancia. Debe establecerse el diagnstico diferencial con la dermatitis
herpetiforme por su similitud clnica, tanto es as que hasta 1970 no se

CTO Medicina C/ Nez de Balboa, 115 28006 MADRID (Espaa) Tfno.: (91) 782 43 32 / Fax: (91) 782 43 27
E-mail: secretaria@ctomedicina.com; iberocto@ctomedicina.com WEB: www.ctomedicina.com; www.iberocto.com

Comentarios TEST

3pQILJR

DERMATOLOGA

Preparacin Examen de Seleccin 05/06 1 Vuelta


desligaron ambas enfermedades. Las lesiones aparecen de forma brusca,
se acompaan de prurito intenso, en forma de vesculo-ampollas, tensas
de componente claro que despus se convierten en hemorrgico. La
respuesta 4 es falsa pues al contrario que la dermatitis herpetiforme que se
asocia a enfermedad celaca en el 90% de las ocasiones no ocurre lo
mismo con la dermatosis IgA lineal de la infancia. La base diagnstica ms
importante, ya que es la nica que nos permite diferenciarla de otros
procesos ampollosos como dato positivo es la presencia de IgA en la zona
de la membrana basal y de forma lineal.

Comentarios TEST

Pregunta 44.- R: 3
La pregunta es muy sencilla pues ante una mujer embarazada con
lesiones vesiculares periumbilicales debemos pensar inmediatamente
en la penfigoide gestacional o herpes gestationis. Las lesiones pueden
aparecer en cualquier momento del embarazo o del puerperio. Pueden verse tambin en otras situaciones como el coriocarcinoma, menstruacin o incluso con la ingesta de anticonceptivos. La morfologa
de las lesiones en el herpes gestationis vara desde las papulovesculas
a las placas urticariformes. Suelen comenzar alrededor del ombligo
para extenderse despus sobre todo por el tronco. Una vez que aparecen las lesiones tienen tendencia a persistir a lo largo del embarazo.
Suele recurrir en nuevos embarazos y cuando as sucede suele iniciarse de forma ms precoz. Los corticoides tpicos y las cremas hidratantes
pueden eliminar el picor pero son insuficientes para el control de la
enfermedad. En la mayor parte de los casos es necesaria la administracin de corticoides a dosis moderada y de forma oral. La respuesta 3
por tanto es la correcta. El tratamiento se instaura principalmente para
disminuir la incidencia de prematuridad y de bajo peso al nacimiento
del feto. La histologa del herpes gestationis es similar a la del penfigoide
ampolloso pero en la inmunofluorescencia directa se demuestra el
depsito de C3 a lo largo de la membrana basal.
Pregunta 45.- R: 2
El paciente tiene lesiones nodulares, que llegan a eliminar material.
La enfermedad que tiene como lesiones elementales los ndulos subcutneos son las paniculitis. Por eso deberamos pensar en la respuesta 1 o 2 como respuestas correctas. El Sndrome de Weber-Christian
puede ser una paniculitis traumtica o un sndrome idioptico, recurrente y febril. En la pregunta nos estn dando datos muy caractersticos de la paniculitis pancretica como son la histologa y la morfologa de las lesiones. As la imagen histolgica caracterstica es la presencia de focos de necrosis grasa en el panculo adiposo con presencia
de clulas grasas necrotizantes o clulas fantasmas. Las lesiones suelen
acompaarse de elevacin en sangre y orina de la amilasa. La morfologa clnica es la de ndulos de localizacin diferente, que se suelen
ulcerar eliminando un material aceitoso. La paniculitis grasa
pancretica est asociada a un cncer de pncreas en el 40% de los
casos. Por estos motivos la respuesta correcta es la 2. Las lesiones de la
paniculitis pancretica pueden preceder en el tiempo al adenocarcinoma de pncreas.
Pregunta 46.- R: 1
De nuevo un caso sencillo ya que nos aportan todos los datos para
saber de qu enfermedad se trata. Sin embargo la respuesta no es fcil.
Vemos que se trata de una mujer joven con lesiones nodulares en cara
anterior de ambas piernas. Adems presenta fiebre, artralgias y las
lesiones la ocasionan dolor. Se trata pues de una paniculitis septal sin
vasculitis, es decir del eritema nodoso. El eritema nodoso es la paniculitis
ms frecuente, aparece con mayor frecuencia en mujeres, jvenes o
de edad media y de forma caracterstica cursa con lesiones nodulares,
contusiformes, en cara anterior de ambas piernas. La etiologa del
eritema nodoso es desconocida, pensndose en mecanismos
inmunolgicos mltiples. Cuando el eritema nodoso es recidivante la
causa ms frecuente son las infecciones estreptoccicas de vas altas.
En las mujeres es frecuente encontrar la ingesta de anticonceptivos
como responsable del cuadro. Es importante pensar tambin en las
siguientes enfermedades como responsables del cuadro: infecciones
por yersinia, sarcoidosis, tuberculosis y enfermedad inflamatoria intestinal. El tratamiento del eritema nodoso se fundamenta en la retirada de las causas, el reposo, administracin de antiinflamatorios y/o
corticoides.
M exico A rgentina
C hile U ruguay

Seguimiento a distancia

Vemos entonces que todas las respuesta de la pregunta son correctas. Pero la ms correcta es la respuesta 1, que adems de tratamiento
realiza una biopsia, y por lo tanto tenemos un diagnstico de certeza.
Pregunta 47.- R: 2
Estamos ante una paciente, probablemente polimedicada ya que
tiene bronquitis crnica e hipertensin. Desde el punto de vista cutneo apreciamos lesiones de pocos das de evolucin que le aparecen
en miembros inferiores. En la morfologa de las lesiones es llamativo el
hecho de que son rojas, eritematosas, y no blanquean a la presin. Es
la tpica lesin de vasculitis. La forma ms frecuente de vasculitis cutnea es la vasculitis por hipersensibilidad. La etiologa de las vasculitis
cutneas es muy variada, pudiendo estar causada por infecciones,
tumores, medicamentos, enfermedades autoinmunes... etc. El mecanismo de produccin del dao vascular se explica por el depsito de
inmunocomplejos antgeno-anticuerpo. Estos complejos se depositan en la pared de los vasos pequeos, en especial en las vnulas. Por
este motivo la respuesta 1 es falsa. Las vasculitis por hipersensibilidad
suelen localizarse, clnicamente, en las zonas ms declives, debido a
que el depsito de los inmunocomplejos se produce por gravedad. La
forma clnica ms frecuente de presentacin es la prpura palpable.
La vasculitis por hipersensibilidad se caracteriza por la degeneracin y
necrosis fibrinoide de los vasos. Por eso tambin se denomina vasculitis
necrotizante y leucocitoclstica. Por lo tanto la repuesta correcta es la
2. El tratamiento consiste en retirar la causa desencadenante si se
conoce. El uso de los inmunosupresores est indicado cuando hay
afectacin sistmica o progresin de la enfermedad.
Pregunta 48.- R: 5
Ver comentario de la pregunta anterior.
Pregunta 49.- R: 2
Un tipo de vasculitis por hipersensibilidad caracterstica es la prpura de Schnlein-Henoch, que se caracteriza por tener depsito de
IgA en las vasos drmicos. Clnicamente cursa con la aparicin de una
prpura palpable, que puede llegar a ulcerarse. Es muy tpico que se
acompae de un fuerte dolor abdominal existiendo incluso casos de
invaginacin intestinal e infartos con perforacin. La histologa es la
misma que la de las vasculitis por hipersensibilidad, con lo que la
respuesta 1 es falsa. Se produce ms frecuentemente en nios, y normalmente menores de 10 aos. La afectacin del sistema nervioso
central es una complicacin grave pero que aparece en menos del
1% de los pacientes. Por lo tanto la respuesta 4 es falsa. La respuesta 5
tambin es incorrecta ya que si bien es cierto que se ha asociado a
infecciones estreptoccicas, la aparicin de las lesiones cutneas suele comenzar al cabo de 10 das y no a las 48 h. El curso normal de la
prpura Schnlein-Henoch es a la curacin, teniendo en general
buen pronstico. La afectacin renal aparece en un 20% de los casos
en forma de glomerulonefritis. De este porcentaje un 1% llegan a
desarrollar una insuficiencia renal crnica. El tratamiento es el mismo
que el de otras vasculitis, es decir retirar la causa, y el uso de
imnunosupresores en el caso de complicaciones.
Artralgias
(Glomerulonefritis sin IR)

Dolor clico abdominal


Prpura palpable
MMII y glteos

Pregunta 49. Prpura de Schnlein-Henoch.

CTO Medicina C/ Nez de Balboa, 115 28006 MADRID (Espaa) Tfno.: (91) 782 43 32 / Fax: (91) 782 43 27
E-mail: secretaria@ctomedicina.com; iberocto@ctomedicina.com WEB: www.ctomedicina.com; www.iberocto.com

DM Pg. 11

Pregunta 50.- R: 4
El liquen plano es una enfermedad de causa desconocida que
aparece con mayor frecuencia entre los 25 y los 65 aos. Es muy raro
verlo en la infancia o en la vejez. Los hallazgos clnicos caractersticos
consisten en ppulas poligonales, brillantes, violceas y que son muy
pruriginosas. Las ppulas no son de gran tamao y se distribuyen por
la parte distal de las extremidades y en especial por las superficies
flexoras de las muecas, tobillos y antebrazos. Alrededor del 50% de
los pacientes presentan lesiones en mucosas, especialmente en mucosa oral. Incluso puede cursar nicamente con lesiones en mucosas
donde la morfologa de las lesiones es distinta a la que vemos en la
piel. En las mucosas se ven estriaciones blanquecinas aunque en ocasiones se ven formas erosivas que producen grandes molestias a los
enfermos. Adems estas formas, cuando son crnicas pueden acabar
siendo carcinomas. La alteracin ungueal es una caracterstica menor
del liquen plano que se ve en el 10% de los casos. Se ha encontrado
asociacin entre el liquen plano y el VHC, sobre todo las formas con
afectacin mucosa. El tratamiento del liquen plano es muy variado en
funcin de la intensidad del proceso. Los casos leves se tratan con
corticoides tpicos mientras que en los casos muy intensos se usan
corticoides orales o acitretina. Incluso en los casos graves se llegan a
emplear inmunosupresores potentes como la ciclosporina.
Pregunta 51.- R: 4
El tratamiento de un acn leve debe ser en principio tpico. Dentro
de los tratamientos tpicos no existe uno que sea de eleccin. Si las
lesiones son inflamatorias vamos a emplear los antibiticos tpicos.
Dentro de stos los que ms se emplean son la clindamicina y la
eritromicina. Cuando lo que predomina son lesiones comedonianas
preferiremos entonces o el perxido de benzoilo o los retinoides tpicos. El perxido de benzoilo tiene accin queratoltica y como efectos
secundarios puede producir irritacin y descamacin cutnea. Los
retinoides tpicos tienen una potente accin comedoltica y exfoliante.
Se emplea la isotretinona, adapaleno y el tazaroteno. Cuando el acn
es grave o un acn moderado que no responde a tratamiento tpico,
entonces vamos a emplear los tratamientos sistmicos. El de eleccin
sern los antibiticos orales, en especial las tetraciclinas. Una complicacin con el tratamiento prolongado con antibiticos es la foliculitis por
gram -. Si no responde con antibiticos entonces emplearemos los
retinoides, en forma de isotretinona. Existen unas excepciones a sto, y
una de ellas es la que nos exponen en la pregunta. Cuando existen
evidencias de alopecia andrognica, hirsutismo y acn, entonces preferiremos el empleo de hormonas antiandrognicas como el acetato de
ciproterona. Debe emplearse asociado con un estrgeno como el
etinilestradiol para conseguir una accin anticonceptiva y para minimizar los efectos secundarios sobre el ciclo menstrual.
Pregunta 52.- R: 4
La alopecia androgentica es la calvicie comn. En cierta forma es
un proceso fisiolgico en el cual los folculos producen cada vez pelos
ms pequeos y de menos dimetro. Con el paso del tiempo el folculo
se va miniaturizando hasta que se convierte en folculos atrficos. La
etiologa de este proceso es gentica y hormonal. Hormonal debido a
que algunos metabolitos de la testosterona inhiben el metabolismo de
los folculos. Este proceso afecta a la mayora de los varones y a un
porcentaje elevado de mujeres. En la mujer la alopecia androgentica
se trata con acetato de ciproterona, que es un potente antiandrgeno.
En el varn el tratamiento, nunca curativo sino que lo que hace es el
enlentecimiento del proceso, es con finasteride a dosis de 1 mg. Produce un recrecimiento temporal en el 50% de los pacientes. Al suspender
el tratamiento sigue la cada. El finasteride inhibe la 5-alfa-reductasa y
como efecto secundario a destacar, la impotencia sexual que aparece
en el 1% de los pacientes. Otra alternativa al finasteride, aunque menos
eficaz, es el empleo de minoxidil al 2% de forma tpica. El minoxidil es
un vasodilatador y como efectos secundarios destaca la cefalea y probables complicaciones cardacas en pacientes con patologa de base.
Por todo lo dicho la respuesta falsa es la 4.
Pregunta 53.- R: 1
Varias placas alopcicas, con piel normal. Con esta descripcin no
nos debemos plantear que se trate de una alopecia cicatricial. Por tanto
Pg. 12 DM

M exico A rgentina
C hile U ruguay

DERMATOLOGA

Preparacin Examen de Seleccin 05/06 1 Vuelta

descartamos la posibilidad de lupus eritematoso crnico, liquen plano


pilar, neoplasia, esclerodermia... etc. Tampoco puede tratarse de una
tia, pues debera tener descamacin y eritema. El antecedente de ser
opositor es clave para el diagnstico pues una de las alopecias no
cicatriciales que se ha relacionado con la tensin emocional es la alopecia areata. La etiologa de la alopecia areata es desconocida aunque
se piensa que alteraciones inmunolgicas juegan un papel importante.
Es frecuente su asociacin con otros procesos autoinmunes: el vitligo,
tiroiditis, dermatitis atpica etc. En la histologa es tpico el hallazgo de
un infiltrado linfocitario que aparece alrededor de los bulbos pilosos.
La respuesta 2 es falsa ya que el curso es variable, con remisiones espontneas. La respuesta 5 tambin es falsa ya que es frecuente la alteracin
de la sntesis de melanina y que aparezcan pelos blancos. Existen mltiples tratamientos para la alopecia areata. Cuando son pocas placas y
de pequeo tamao se utilizan corticoides tpicos o intralesionales;
cuando son varias y de gran tamao se prefieren corticoides sistmicos.
Alternativas son la puvaterapia, difenciprona y dinitrobenceno.
Pregunta 54.- R: 2
En principio la pregunta parece difcil, pero existen ya datos importantes que deben hacernos pensar en una patologa. Destacan la
fiebre, la leucocitosis con neutrofilia y un cuadro con lesiones cutneas. Se trata de la antiguamente llamada dermatosis neutroflica febril o Sndrome de Sweet. La enfermedad es de causa desconocida
aunque en el 80% de los casos se asocia con un cuadro infeccioso. El
otro 20% se asocia a procesos linfoproliferativos, en especial a
leucemias. Por esta razn la respuesta 2 es incorrecta. Las lesiones
cutneas habitualmente son precedidas de cuadros infecciosos, con
mayor frecuencia respiratorios. Las lesiones suelen ser papulopstulas
que predominan en superficie de extensin. La histologa no es
diagnstica pero s muy sugestiva del cuadro, pues se aprecia un infiltrado neutroflico en dermis. El sndrome de Sweet es una dermatosis
benigna. El problema surge de esa asociacin con procesos linfoproliferativos. Lo ms habitual es que el diagnstico del Sndrome de
Sweet anteceda al diagnostico del tumor. El tratamiento de esta dermatosis neutroflica febril es con corticoides sistmicos en pauta decreciente. Tambin pueden emplearse el yoduro potsico y la
colchicina.
Pregunta 55.- R: 4
Nos encontramos ante un paciente con una lesin ulcerada y como
antecedentes de inters el que padezca una enfermedad inflamatoria
intestinal. El pioderma gangrenoso se caracteriza clnicamente por una
lcera irregular con un borde inflamatorio y una base necrtica. Las
lesiones comienzan como ndulos dolorosos que se abren y ulceran
secretando un exudado purulento o hemorrgico. El borde es irregular,
dentado y tiene una caracterstica coloracin prpura o violcea. Los
lugares donde se localiza con mayor frecuencia el pioderma gangrenoso son las extremidades inferiores, nalgas y abdomen. Hemos pensado
en un pioderma gangrenoso ya que una de las asociaciones frecuentes
del mismo es con la enfermedad inflamatoria intestinal. Es ms frecuente su asociacin con la colitis ulcerosa que con el Crohn. El curso de la
enfermedad intestinal es independiente de la enfermedad cutnea. Otras
enfermedades con las que se ha asociado son con las artritis, gammapata
monoclonal, leucemia mieloide y Enfermedad de Takayasu. El tratamiento del pioderma gangrenoso es con corticoides administrados de
forma sistmica. En los pacientes con enfermedad subyacente el tratamiento no slo se debe dirigir al pioderma gangrenoso sino tambin al
trastorno sistmico. La respuesta 3 sera una opcin mala debido al
fenmeno de patergia que consiste en que un traumatismo trivial provoca nuevas lesiones o agrava las preexistentes. Este fenmeno tambin
puede ser la causa del rechazo de injertos cutneos autlogos o del
desarrollo de nuevas lesiones en los sitios donantes del injerto.
Pregunta 56.- R: 3
Con la descripcin de la lesin nicamente podemos tener una
aproximacin diagnstica. Se nos describen lesiones en cara anterior
de las piernas.
En esa localizacin nos encontramos el eritema nodoso, pioderma
gangrenoso, mixedema pretibial y necrbiosis lipodica. En el mixedema pretibial es caracterstico el depsito importante de mucina a

CTO Medicina C/ Nez de Balboa, 115 28006 MADRID (Espaa) Tfno.: (91) 782 43 32 / Fax: (91) 782 43 27
E-mail: secretaria@ctomedicina.com; iberocto@ctomedicina.com WEB: www.ctomedicina.com; www.iberocto.com

Comentarios TEST

Seguimiento a distancia

DERMATOLOGA

Preparacin Examen de Seleccin 05/06 1 Vuelta


nivel de dermis media y profunda. El mixedema pretibial forma parte
de la enfermedad de Graves, acompaada de la oftalmoplejia e hipertiroidismo. Aunque estas caractersticas pueden presentarse de forma independiente, el mixedema pretibial suele aparecer en presencia de oftalmopata. La correccin de la alteracin hormonal carece
de efectos sobre las lesiones cutneas. De tal forma que el 50% de los
mixedemas pretibiales aparecen despus de que el paciente se ha
vuelto eutiroideo. Esta dermopata infiltrativa no es patognomnica
del hipertiroidismo ya que se han comunicado lesiones similares en
pacientes con hipotiroidismo. Los factores patognicos del mixedema
pretibial no estn claros. El tratamiento del mixedema pretibial no es
muy satisfactorio. Pueden brindar cierto alivio los corticoides sistmicos o intralesionales pero en ningn caso lo que dice la respuesta 3.

Pregunta 56. Hipotiroidismo vs. hipertiroidismo.

+LSRWLURLGLVPR
3,(/

6HFDSiOLGD\IUtD

Comentarios TEST

0L[HGHPD
/HVLyQFXWiQHDPiV
JHQHUDOL]DGR RMRV
FDUDFWHUtVWLFD
PDQRV\SLHV 

+LSHUWLURLGLVPR
+~PHGD
HULWHPDWRVD\
FDOLHQWH
0L[HGHPD
SUHWLELDO

1RGHMDIyYHD

1RGHMDIyYHD

'HSyVLWRiFLGR
KLDOXUyQLFRHQ
GHUPLV

'HSyVLWRiFLGR
KLDOXUyQLFRHQ
GHUPLVSURPRYLGR
SRUIRUPDFLyQGH
FRPSOHMRV$J$F
 HQIHUPHGDGGH
*UDYHV 

5HODFLRQDGRFRQHO
GpILFLWGH+
WLURLGHDV

1RUHODFLRQDGR
FRQHOH[FHVRGH+
WLURLGHDV/D
QRUPDOL]DFLyQQR
UHJUHVDHOSURFHVR

7UDWDPLHQWR
KRUPRQDO
VXVWLWXWLYR

7UDWDPLHQWRFXUD
RFOXVLYD
FRUWLFRLGHD

2WUDV

+LSHUFDURWLQHPLDHQ
FDSDFyUQHD
3~USXUD\
HTXLPRVLV

3UXULWR
GHUPRJUDILVPR
HOHYDGRIOXVKLQJ
WHODQJLHFWDVLDV

3(/2

6HFR\GHVOXVWUDGR
$ORSHFLDH[WGH
ODVFHMDV
 PDGDULRVLV 

6XDYHIULDEOH\GH
FUHFLPLHQWRUiSLGR

8f$6

'HVOXVWUDGDV
FUHFLPLHQWROHQWR
(VWULDV
ORQJLWXGLQDOHV\
WUDQVYHUVDOHV

8xDVGH3OXPPHU
FyQFDYDV\FRQ
RQLFyOLVLVGLVWDO
$FURSDTXLD
WLURLGHD

enteropata pero adems es caracterstica la hiperglucemia, que no


figura en el cuadro clnico de la paciente. La pelagra est producida
por el dficit de niacina, cido nicotnico o vitamina B3. La mayora
de los casos aparecen en alcohlicos crnicos. Tambin pueden
aparecer en el sndrome carcinoide. Clnicamente se caracteriza
por una trada en la que destaca una erupcin eritemato-vesicular,
que dejan una hiperpigmentacin caracterstica. Las lesiones suelen
aparecer en zonas fotoexpuestas. Cuando aparecen en el cuello
dejan una erupcin simtrica que se denomina "collar de Casal". El
tratamiento consiste en administrar nicotinamida, pues la administracin del cido nicotnico produce un cuadro de enrojecimiento
facial.
Pregunta 58.- R: 5
Volvemos a encontrarnos con un paciente alcohlico, y con lesiones cutneas. En este caso las lesiones cutneas se distribuyen por
dorso de manos, cara, es decir por zonas claramente fotoexpuestas.
Pensaremos entonces en dos patologas: porfiria cutnea tarda y pelagra. Esta ltima se acompaa de demencia y diarrea que aparece en el
50% de los casos, por lo que no puede ser la patologa del paciente.
La porfiria hereditaria puede aparecer a cualquier edad pero la forma
adquirida suele aparecer en la edad adulta. La respuesta falsa es la 5
pues el tratamiento se fundamenta en evitar los factores desencadenantes as como la exposicin solar. Las sangras peridicas son el
tratamiento de eleccin de la porfiria cutnea tarda. Un tratamiento
alternativo es el empleo de cloroquina. Existe un cuadro idntico
clnicamente a la porfiria cutnea tarda, que se presenta en pacientes
en hemodilisis y se denomina pseudoporfiria. La nica diferencia es
que no poseen alteraciones bioqumicas.

Pregunta 58. Manifestaciones clnicas de la porfiria cutnea tarda.

Pregunta 57.- R: 2
Ante un paciente con antecedentes de ingesta alcohlica y lesiones cutneas, debemos pensar en el MIR en dos patologas, la porfiria
cutnea tarda y la pelagra. La respuesta 1 es falsa debido a que el
escorbuto cursa con hemorragias gingivales. La respuesta 3 es falsa
pues la acrodermatitis enteroptica suele aparecer al suspenderse la
lactancia. Adems no es esa la distribucin caracterstica de presentacin de las lesiones cutneas. El glucagonoma puede cursar con
M exico A rgentina
C hile U ruguay

Seguimiento a distancia

Pregunta 59.- R: 3
Destacan en el caso actual la escasa edad del paciente y la distribucin de las lesiones cutneas, que es periorificial. Enfermedades con
clara distribucin periorificial tenemos el vitligo, que por supuesto
clnicamente no tiene ninguna relacin con el caso actual. Otra de las
patologas claramente caracterizada por distribucin periorificial es la
lentiginosis de Peutz-Jeghers. Tambin en el eritema necroltico migratorio es posible ver esa distribucin, pero suele aparecer asociado a
un glucagonoma. La acrodermatitis enteroptica cursa con lesiones
en regiones periorificiales, y en zonas acras. En la cara suele aparecer
un cuadro similar a la dermatitis seborreica. En las uas aparecen
lesiones transversales de Beau, debido a interrupciones temporales en
el crecimiento de las mismas. Existe tambin cada de pelo. Adems es
frecuente que esta clnica cutnea se acompae de diarrea. La acrodermatitis enteroptica es un cuadro infrecuente, transmitido de forma autosmica recesiva, y su etiopatogenia es la deficiente absorcin
intestinal del zinc. Por eso la respuesta correcta es la 3 ya que el

CTO Medicina C/ Nez de Balboa, 115 28006 MADRID (Espaa) Tfno.: (91) 782 43 32 / Fax: (91) 782 43 27
E-mail: secretaria@ctomedicina.com; iberocto@ctomedicina.com WEB: www.ctomedicina.com; www.iberocto.com

DM Pg. 13

Seguimiento a distancia

DERMATOLOGA

Preparacin Examen de Seleccin 05/06 1 Vuelta

tratamiento es con sulfato de zinc oral. Adems de esta enfermedad


hereditaria por dficit de zinc tambin se ha descrito un cuadro adquirido en pacientes con malnutricin asociada a cirrosis alcohlica
o pancreatitis alcohlica.
Pregunta 60.- R: 2
Las necrobiosis lipodica es una enfermedad degenerativa del colgeno que clnicamente se caracteriza por placas amarillentas, con
aspecto atrfico, que pueden llegar a ulcerarse. La causa de la necrobiosis lipodica es desconocida. Es un hallazgo tpico en los diabticos
pero tan slo aparece en el 0,3% de los mismos. Cuando se asocia a
diabetes mellitus el curso evolutivo de las lesiones es independiente
del de la diabetes. Aunque en la mayora de los casos aparece despus de establecerse la diabetes es posible incluso que la anteceda. La
localizacin clsica es en la cara anterior de los miembros inferiores.
Este hecho hace que tengamos que establecer diagnstico diferencial
con otras patologas que clsicamente aparecen en la cara anterior de
los miembros inferiores como el mixedema pretibial, eritema nodoso,
pioderma gangrenoso, vasculitis cutnea, etc. Adems un diagnstico
diferencial frecuente es con el granuloma anular, pues ambas patologas tienen una histologa similar y son lesiones cutneas que se asocian a la diabetes. El tratamiento de la necrobiosis lipodica se fundamenta en el uso de corticoides tpicos o intralesionales obtenindose
buenos resultados. Otras lesiones cutneas que se aprecian en los
diabticos son la dermopata diabtica que se aprecia en el 50% de
los mismos, los xantomas eruptivos, el escleredema y la lipodistrofia
generalizada.
Pregunta 61.- R: 3
Lesiones cutneas periorificiales, acompaadas de diarrea y prdida de peso. Esa descripcin del cuadro concuerda con una acrodermatitis enteroptica, aunque con 68 aos es evidente que la lactancia
materna le queda un poco alejada. Adems el dato de la anemia y en
especial de hiperglucemia, junto a que las lesiones cutneas van cambiando de localizacin nos debe hacer pensar en el eritema necroltico
migratorio.

frecuente verlas alrededor de la boca, genitales y pliegues. En la


histologa llama la atencin la necrosis de las capas epidrmicas
superiores y en algunos casos con acantlisis, por lo que puede
parecer un pnfigo pero la inmunofluorescencia directa e indirecta
es negativa. Se han descrito tambin casos de eritema necroltico
migratorio en pacientes con cirrosis y pancreatitis crnica.
Pregunta 62.- R: 5
La esclerosis tuberosa o enfermedad de Pringle Bourneville se
transmite por herencia autosmica dominante. La expresin clnica
de la enfermedad puede ser muy variable, incluso en pacientes de la
misma familia. La trada clsica que caracteriza a la enfermedad slo
se encuentra en el 30% de los pacientes. Esta trada comprende el
adenoma sebceo, retraso mental y epilepsia. La primera lesin cutnea en aparecer es la mcula hipocrmica con aspecto lanceolado o en hoja de fresno. Aparecen en la infancia estando con frecuencia presentes al nacimiento. Son el marcador ms temprano de
la enfermedad pero no son patognomnicas. Los angiofibromas o
adenomas sebceos de Pringle estn presentes en el 80% de los
casos. Aparecen a partir de los 2 aos de edad. Clnicamente son
ppulas rojizas que se distribuyen de forma caracterstica, simtrica,
por las zonas malares, regin perioral y paranasal. Otra de las lesiones cutneas tpicas son los fibromas periungueales o tumores de
Koenen, que aparecen en el 50% de los pacientes, comienzan a
surgir despus de la pubertad. Las eflides no son lesiones de la
esclerosis tuberosa sino de la neurofibromatosis. Otras anomalas
asociadas a la esclerosis tuberosa son los rabdomiomas cardacos,
alteraciones radiolgicas mostrando pseudoquistes en las falanges,
epilepsia en el 80% de los pacientes y hamartomas en la sustancia
gris del cerebro.

Glucagonoma in situ

Lesin en piegue inguinal


caracterstica

Pregunta 61. Glucagonoma y manifestaciones cutneas asociadas.

Esta dermatosis se encuentra dentro del sndrome del glucagonoma caracterizado por prdida de peso, diabetes, anemia y tumor
pancretico de clulas alfa secretoras de glucagn. Esta enfermedad
aparece con ms frecuencia en mujeres postmenopusicas. La enfermedad cutnea es el resultado de la glucagonemia. Las lesiones
cutneas desaparecen despus de extirpar el tumor productor de
glucagn. La clnica de las lesiones cutneas es muy caracterstica
con presencia de eritema de aspecto anular, vesculas y ampollas en
el borde externo. Las lesiones tienen tendencia a diseminarse y es
Pg. 14 DM

M exico A rgentina
C hile U ruguay

Pregunta 62. Esclerosis tuberosa.

Pregunta 63.- R: 4
La neurofibromatosis tiene una herencia autosmica dominante,
con penetrancia variable. El gen responsable de la neurofibromatosis

CTO Medicina C/ Nez de Balboa, 115 28006 MADRID (Espaa) Tfno.: (91) 782 43 32 / Fax: (91) 782 43 27
E-mail: secretaria@ctomedicina.com; iberocto@ctomedicina.com WEB: www.ctomedicina.com; www.iberocto.com

Comentarios TEST

Lesin anular perioral


caracterstica

DERMATOLOGA

Preparacin Examen de Seleccin 05/06 1 Vuelta


tipo I ha sido aislada en el cromosoma 17. Es una de las enfermedades genticas ms frecuente. Se distinguen 2 tipos siendo la clsica la
tipo I o neurofibromatosis perifrica. La tipo II o central se caracteriza por tener neurofibromas que afectan de forma bilateral al acstico. La manifestacin ms temprana de la neurofibromatosis son las
manchas caf con leche que pueden estar presentes en el nacimiento. No son patognomnicas pues pueden verse en otras enfermedades como el Sndrome de McCune-Albright, Sndrome de Bloom,
esclerosis tuberosa y en el 20% de las personas sanas. La presencia
de 6 o ms manchas caf con leche indican alta probabilidad de
tener neurofibromatosis. Sin embargo la lesin ms caracterstica de
la neurofibromatosis son los neurofibromas cutneos que es muy
raro que aparezcan antes de los 6 meses de edad. La presencia de
eflides en los pliegues (signo de Crowe) es un signo patognomnico
de la enfermedad. Otras anomalas y enfermedades asociadas son
alteraciones seas que se observan en el 50% de los pacientes, gliomas
del nervio ptico que se aprecian en el 15% de los pacientes y la
aparicin de tumores como el rabdomiosarcoma o el tumor de
Wilms.

Pregunta 63. Caractersticas de las facomatosis.


)DFRPDWRVLV

$OWHUDFLRQHVFXWiQHDVFDUDFWHUtVWLFDV

0DQFKDVFDIpFRQOHFKH
1HXURILEURPDV
(IpOLGHVD[LODUHV
1yGXORVGH/LVFK
1HXURILEURPDWRVLV,,
(VFDVDVRQXODV
0DQFKDVKLSRFUyPLFDVODQFHRODGDV
$QJLRILEURPDVIDFLDOHV
(VFOHURVLVWXEHURVD
)LEURPDVSHULXQJXHDOHV\VXEXQJXHDOHV
 WXPRUHVGH.RHQHQ 
3ODFDGHSLHOGH&KDJUHQ SLHOGH=DSD 
6tQGURPHGH6WXUJH:HEHU 0DQFKDHQYLQRGHRSRUWR
(QIHUPHGDGGH
3RFRIUHFXHQWHVHLQHVSHFtILFDV
9RQ+LSSHO/LQGDX
6tQGURPHGH
+LSHUSLJPHQWDFLyQ
%ORFK6XO]EHUJHU

Comentarios TEST

1HXURILEURPDWRVLV,

Pregunta 64.- R: 4
La respuesta 1 es correcta ya que al tratarse de una lesin exudativa estar indicada la aplicacin de fomentos. La respuesta 2 tambin
es correcta pues el tratamiento de todos los eccemas se fundamenta
en la administracin de corticoides tpicos u orales en funcin de la
gravedad e intensidad de las lesiones. En cuanto a la respuesta 3, la
realizacin de pruebas epicutneas estar indicada pues si sospechamos un eczema de contacto alrgico ser til saber cul es el o los
alergenos implicados. La respuesta 5 tambin es correcta pues los
antihistamnicos tpicos no son muy eficaces y pueden producir sensibilizaciones con lo que agravaran o enmascararan el proceso. La
respuesta falsa es la 4 pues el cromo no es un alergeno frecuente en los
eccemas de contacto alrgico en mujeres. El alergeno ms frecuentemente implicado a mujeres es el nquel. Su fuente de exposicin ms
frecuente es la bisutera. Hemos pensado que la paciente tena un
eccema de contacto alrgico porque las lesiones son pruriginosas. En
las orejas la causa ms frecuente son el empleo de gotas ticas y de
bisutera. En los hombres la fuente principal de sensibilizacin son los
cementos que contienen sales de cromo en su composicin. En muchas ocasiones se va a acompaar de sensibilizacin concomitante a
sales de cobalto y de nquel.
Pregunta 65.- R: 5
La dermatitis atpica es un proceso muy frecuente que afecta al
10% de la poblacin general. Suele asociarse a otras patologas como
el asma alrgico y la rinitis alrgica. Es una enfermedad de causa
M exico A rgentina
C hile U ruguay

Seguimiento a distancia

desconocida pero probablemente inmunolgica, con alteraciones


tanto de la inmunidad humoral como celular.
La dermatitis atpica es una enfermedad crnica que cursa en
brotes de lesiones muy pruriginosas y se caracteriza por la presencia
de lesiones eccematosas. La atpica del lactante no suele aparecer
hasta despus del segundo mes de vida. En esta fase predominan las
lesiones eccematosas exudativas, distribuyndose por el cuero cabelludo y la cara, aunque respetando el tringulo nasogeniano. La atopia
infantil puede ser el progreso de la del lactante o aparecer por primera
vez. Son lesiones mucho ms secas y con tendencia a distribuirse por
las flexuras. Uno de los fallos ms habituales en el examen MIR es
considerar a la atopia infantil como si fuera una atopia del lactante. La
mayora de las atopias aparecen en la infancia, siendo raras las que
aparecen en edad adulta. Esto sucede en el 10% de los casos. La
atopia del adulto aparece en la cara, el tronco y dorso de las manos.
El sntoma comn en todos los eccemas atpicos es el prurito.
Pregunta 66.- R: 5
Ver comentario de pregunta anterior.
Pregunta 67.- R: 3
Son lesiones eritematosas, dolorosas, que claramente comienzan o
se desencadenan con la presin de los zapatos al caminar. La urticaria
por presin es una urticaria poco frecuente y representa el 2,5% de las
urticarias fsicas. La lesin que produce es similar al angioedema y se
traduce por un edema profundo en el tejido celular subcutneo,
acompaado de linfocitos, neutrfilos y eosinfilos. Las lesiones aparecen entre 3-6 horas despus del estmulo por presin, con una
duracin aproximada de 24-48h. Las lesiones pueden dar prurito
pero lo ms llamativo es que resulten dolorosas. Estas lesiones pueden
aparecer en cualquier localizacin donde exista presin, por eso son
zonas frecuentes los pies, glteos y manos.
El diagnstico viene dado por la historia clnica aunque pueden
emplearse diversas tcnicas para su confirmacin, mediante la aplicacin de un peso sobre la piel. Los tratamientos no suelen ser muy
eficaces. Los antihistamnicos son de nula utilidad. Pueden responder
al uso de los corticoides orales. Por eso la respuesta correcta es la 3.
Pregunta 68.- R: 2
La urticaria colinrgica es una urticaria muy caracterstica que aparece cuando se producen aumentos bruscos de la temperatura corporal y que cursa con la aparicin de lesiones habonosas de pequeo tamao. Por ese motivo la respuesta falsa es la 2. Representa el 30%
de las urticarias fsicas siendo ms frecuente en adolescentes y adultos
jvenes. Es frecuente su asociacin con el dermografismo y la urticaria
por fro. La denominacin de colinrgica le viene por tener una susceptibilidad aumentada a la acetilcolina ya que la inyeccin
intracutnea de acetilcolina o de pilocarpina inducen la aparicin de
lesiones habonosas. El cuadro tpico es el de un paciente que al poco
de tiempo de haber comenzado a hacer ejercicio, que conlleve un
aumento de la temperatura corporal y de la sudoracin, comienza
con lesiones habonosas, de pequeo tamao, redondeadas, que se
distribuyen sobre todo por el tercio superior del tronco. En ocasiones
el brote de lesiones pueden acompaarse de nuseas, cefalea o trastornos gastrointestinales. Otras situaciones que pueden desencadenar
las crisis son los alimentos ricos en especias, baos o duchas con agua
muy caliente y en situaciones de tensin emocional. Responde bien a
la hidroxicina.
Pregunta 69.- R: 2
Estamos ante un paciente que presenta desde hace una semana
lesiones muy pruriginosas distribuidas por toda la superficie corporal.
El antecedente que claramente relacionamos con el proceso, es la
ingesta de captopril 3 semanas antes. Bien, la reaccin cutnea ms
frecuente ocasionada por medicamentos es el exantema, que tiende
a confluir y a ser muy pruriginoso. Los medicamentos ms frecuentemente implicados son los betalactmicos, sulfamidas, captopril, AINES
y las hidantonas.
Por tanto nos encontramos al principio con un exantema medicamentoso por captopril. Lo que sucede es que en las ltimas 48 horas
ha empeorado y entonces comienza con despegamientos cutneos y

CTO Medicina C/ Nez de Balboa, 115 28006 MADRID (Espaa) Tfno.: (91) 782 43 32 / Fax: (91) 782 43 27
E-mail: secretaria@ctomedicina.com; iberocto@ctomedicina.com WEB: www.ctomedicina.com; www.iberocto.com

DM Pg. 15

DERMATOLOGA

Preparacin Examen de Seleccin 05/06 1 Vuelta

mucosos. Es decir el cuadro ha evolucionado hacia una necrlisis


epidrmica txica.
Este proceso es la reaccin cutnea ms grave ocasionada por
medicamentos. Se caracteriza por la aparicin de erosiones y ampollas flccidas, con prdida de la epidermis de extensas zonas de la
superficie corporal. Adems aparecen despegamientos mucosos extensos. Aunque no se conoce exactamente el por qu aparece el
cuadro, existen evidencias indirectas que abogan por un mecanismo
inmunolgico. Por eso el tratamiento se fundamenta en medidas de
soporte y en el uso de inmunodepresores como corticoides, ciclofosfamida, ciclosporina o azatioprina.
Pregunta 70.- R: 2
Nuevamente tenemos como antecedente el que el paciente hubiera ingerido un frmaco 5 das antes. En este caso estamos ante una
lesin nica y que el paciente refiere que tuvo episodios similares
tiempo atrs. El exantema fijo medicamentoso se caracteriza por la
presencia de lesiones en piel y/o mucosas. Suelen ser lesiones maculosas, eritematosas y ocasionalmente ampollosas. Cuando la administracin del medicamento se interrumpe las lesiones pierden actividad
dejando una lesin residual pigmentada. Por eso decimos exantema
fijo medicamentoso, fijo que deja marca. Cuando se vuelve a ingerir el
medicamento las lesiones vuelven a aparecer en los mismos sitios que
la vez anterior, es decir donde quedaron las marcas. Tambin pueden
aparecer en nuevas localizaciones. Para ayudarnos a recordarlo podemos decir entonces que el exantema fijo sale en los mismos sitios.
Normalmente aparecen como lesin nica o en nmero escaso. La
localizacin de las lesiones es muy variada pero son lugares tpicos las
mucosas genitales.
Por eso antes lesiones ampollosas, o erosivas en las mucosas debemos incluir siempre al exantema fijo medicamentoso como diagnstico diferencial. Por todo lo comentado, la respuesta falsa es la 2.
Pregunta 71.- R: 3
El vitligo es una enfermedad bastante frecuente que se caracteriza
por la desaparicin de los melanocitos en algunas zonas o en toda la
piel.
Afecta al 1% de la poblacin y el 25% de los pacientes tienen
antecedentes familiares. La etiologa del vitligo es desconocida aunque existen dos teoras, una basada en que un mediador neuroqumico
destruira los melanocitos (recordad que el melanocito deriva de la
cresta neural) y otra teora implicara el sistema inmune. El comienzo
del vitligo puede ser brusco o gradual, puede existir con factor
desencadenante a agravante como los traumatismos ya que es una
enfermedad con fenmeno de isomorfismo de Koebner. La despigmentacin puede ocurrir en cualquier parte del cuerpo pero hay
reas de despigmentacin caractersticas como las prominencias seas
(codos y rodillas). Tambin es frecuente su distribucin periorificial. A
menudo el vitligo, en especial el generalizado, presenta lesiones simtricas como afirma la respuesta 3. Los pacientes con vitligo es ms
frecuente que tengan otras enfermedades autoinmunes, como las
patologas tiroideas que son las ms frecuentes. Los pacientes con
melanoma, en especial los metastsicos, pueden sufrir vitligos incluso
a distancia de la localizacin del melanoma. El tratamiento del vitligo
no es satisfactorio, emplendose corticoides tpicos y PUVA.

mos aos est en discusin la verdadera existencia de este sndrome


dado que las queratosis seborreicas son muy frecuentes en personas
mayores.
Pregunta 73.- R: 2
La acantosis nigricans cursa con la aparicin de placas hiperqueratsicas de coloracin griscea o azulada, que aparecen por zonas de
flexin, especialmente en regin cervical y axilas. Podemos distinguir
una acantosis nigricans benigna puede aparecer de forma familiar,
autosmica dominante o asociada a patologas como endocrinopatas, obesidad, uso prolongado de corticoides, enfermedad de Wilson,
etc. La acantosis nigricans maligna se denomina as porque en la mayora de los casos se asocia a un carcinoma interno. Las lesiones
cutneas pueden coexistir con el tumor o incluso antecederlo en
aos. En el 60% de los casos se trata de adenocarcinomas gstricos. La
clnica de todas las acantosis nigricans es muy parecida diferencindose nicamente en la intensidad y extensin de las lesiones. Las
lesiones de la acantosis nigricans maligna se desarrollan rpidamente,
siendo ms severas que las benignas. Con mucha frecuencia presentan una queratodermia amarillenta en palmas y plantas y en el 50%
de los casos existe afectacin de mucosas con lesiones hipertrficas
en mucosa oral. La acantosis nigricans maligna regresa al extirpar el
tumor asociado.
Pregunta 74.- R: 2
La telangiectasia macular eruptiva perstans es una rara variante de
mastocitosis que aparece limitada a la piel. Por ese motivo la respuesta
falsa es la 2. Esta variante de mastocitosis es caracterstica de los adultos. Clnicamente se caracteriza por la aparicin de mculas de unos
5 mm, eritematosas, difusas, con mltiples telangiectasias, que a diferencia de las araas vasculares no poseen el vaso central.
Las mastocitosis sistmicas se caracterizan por la proliferacin de
mastocitos a nivel de la mdula sea, bazo e hgado. Casi siempre
aparecen en mastocitosis de aparicin tarda, aunque tambin aquellas mastocitosis infantiles que persisten en la vida adulta presentan
una alta probabilidad de desarrollo de enfermedad sistmica.
Pregunta 75.- R: 3
El Sndrome de Sturge-Weber se caracteriza por la asociacin de
un nevus en mancha de Oporto asociado a una angiomatosis Leptomenngea occipital. El nevus "flammeus" aparece de forma unilateral
en las ramas oftlmicas y maxilar del trigmino, pudiendo extenderse
hasta la lnea media. Aparece asociado a una angiomatosis leptomenngea homolateral que cursa con epilepsia en el 90% de los casos.
Pueden tener tambin afectacin ocular y un cierto retraso mental. La
angiomatosis leptomenngea puede tambin manifestarse como hemiplejias transitorias o permanentes. El diagnstico es sencillo pues se
establece ante la asociacin de Nevus en mancha de Oporto y epilepsia. En los estudios radiolgicos se aprecian calcificaciones corticales.
Otra enfermedad en la que aparece el Nevus Flammeus o en mancha
de Oporto es en el Sndrome de Klippel-Trenaunay. Este sndrome se
caracteriza por la asociacin de nevus flammeus que suele aparecer
en un miembro con hipertrofia de partes blandas, habitualmente miembros inferiores. Puede existir tambin una insuficiencia venosa crnica en el miembro afecto.

Pregunta 72.- R: 3
La queratosis seborreica es un tumor benigno muy frecuente en la
raza blanca. Aparece a partir de los 50 aos y a lo largo de la vida
puede sufrir regresiones espontneas. Aparecen sobre piel previamente
sana, en tronco, cara y cuello. Suelen ser lesiones papilomatosas,
marronceas e incluso negras. Al tacto es caracterstica la sensacin
grasienta o untuosa. No se han descrito transformaciones malignas de
esta patologa. Su histologa se caracteriza por ser proliferaciones
epiteliales con hiperqueratosis y formacin de quistes crneos. El tratamiento es el curetaje, crioterapia o electrocoagulacin de las mismas cuando plantean problemas mecnicos o estticos. Cuando aparecen mltiples queratosis seborreicas, podemos encontrarnos ante el
signo de Leser-Trelat que consiste en la aparicin sbita de queratosis
seborreicas acompaando a una neoplasia. En el 60% de los casos el
tumor maligno encontrado es una neoplasia abdominal. En los ltiPg. 16 DM

M exico A rgentina
C hile U ruguay

CTO Medicina C/ Nez de Balboa, 115 28006 MADRID (Espaa) Tfno.: (91) 782 43 32 / Fax: (91) 782 43 27
E-mail: secretaria@ctomedicina.com; iberocto@ctomedicina.com WEB: www.ctomedicina.com; www.iberocto.com

Comentarios TEST

Seguimiento a distancia

DIGESTIVO

Preparacin Examen de Seleccin 05/06 1 Vuelta


1.

En cul de los siguientes trastornos esperara encontrar una


disfagia orofarngea?:
1)
2)
3)
4)
5)

2.

Preguntas TEST

Disminucin de peso.
Dficit de vitamina B12.
Carcinoma esofgico.
Aspiracin pulmonar.
Esofagitis.

Miotoma de Heller.
Reseccin esofgica.
Toxina botulnica.
Nitritos orales.
Funduplicatura laparoscpica.

Una mujer de 42 aos consult por dolor precordial irradiado a espalda. Aportaba un extenso estudio cardiolgico,
incluyendo una coronariografa que era normal. Un estudio
radiolgico con bario y una esofagogastroscopia fueron
normales. Se realiz una manometra que demostr, tras la
deglucin, contracciones simultneas de gran amplitud en la
parte inferior del esfago; la presin basal del esfnter esofgico inferior estaba ligeramente aumentada, pero se relajaba durante la deglucin. Qu diagnstico inicial hara a
esta paciente?:
1)
2)
3)
4)
5)

6.

8.

2)

Acalasia cricofarngea.
Acalasia vigorosa.
Espasmo esofgico difuso.
Probable amiloidosis del esfago.
Anillo esofgico tipo B.

Si en el estudio de un paciente se encuentra con aperistalsis


en los dos tercios inferiores del esfago con hipotensin del
esfnter esofgico inferior, qu enfermedad explicara mejor el cuadro?:
1)
2)
3)

Acalasia clsica.
Hernia hiatal paraesofgica.
Sndrome de Boerhaave.
M exico A rgentina
C hile U ruguay

3)
4)
5)
9.

2)
3)
4)
5)

Histolgicamente es un adenocarcinoma en menos del


10% de los casos.
La incidencia es similar en todas las regiones del mundo.
Suelen diagnosticarse precozmente por la disfagia.
La presencia de una lcera en el esofagograma descarta el
diagnstico de carcinoma.
Actualmente la supervivencia a los 5 aos es del 50%.

Seale cul es el mejor mtodo para la estadificacin T de los


tumores esofgicos:
1)
2)
3)
4)
5)

11.

Debera hacerse usted endoscopia y biopsias cada 1,5-2


aos.
Dado el cuadro clnico que presenta el paciente, debera
usted tomar omeprazol.
Su enfermedad se debe probablemente al reflujo gastroesofgico.
Tiene usted riesgo de adenocarcinoma de esfago, slo si
existe colonizacin por H. pylori.
El omeprazol probablemente no le revertir el esfago de
Barrett.

Seale la afirmacin correcta en relacin al carcinoma de


esfago:
1)

10.

Ecografa abdominal.
Esofagograma con bario.
Endoscopia.
pHmetra.
Test de Bernstein.

Un paciente de 63 aos ha sido estudiado en un hospital por


dolor torcico, y en una endoscopia digestiva alta con toma
de biopsias le han diagnosticado de esfago de Barrett, sin
observarse displasia. Le consulta a usted para que le aconseje
sobre su enfermedad; cul de las siguientes consideraciones
NO sera correcta en este paciente?:
1)

Mujer de 48 aos con diagnstico de acalasia que, tras


someterse a dos dilataciones neumticas, presenta recurrencia de su disfagia. Cul de los siguientes tratamientos
sera considerado de eleccin?:
1)
2)
3)
4)
5)

5.

Carcinoma epidermoide de esfago.


Adenocarcinoma gstrico.
Carcinoma de pulmn.
Linfoma.
Carcinoma de pncreas.

Sndrome de Sjgren.
Esclerodermia.

Varn de 62 aos, con antecedente de pirosis de larga


evolucin y, ocasionalmente, regurgitacin de alimentos.
Consulta porque, desde hace tres meses, nota dolor retroesternal con la deglucin y disfagia progresiva. Cul de las
siguientes pruebas diagnsticas debera solicitar a continuacin?:
1)
2)
3)
4)
5)

Una de las siguientes NO suele verse como complicacin de


la acalasia. Selela:
1)
2)
3)
4)
5)

4.

7.

Seale cul es el tumor que con ms frecuencia da lugar a una


acalasia secundaria:
1)
2)
3)
4)
5)

3.

Enfermedad de Parkinson.
Esfago de Barrett.
Acalasia.
Anillo de Schatzki.
Hernia hiatal.

4)
5)

Seguimiento a distancia

Endoscopia.
Radiologa con contraste.
Ultrasonografa endoscpica.
TC.
Laparoscopia.

Es cierto que, en la infeccin por H. pylori:


1)
2)
3)
4)
5)

La bacteria se identifica en la lmina propia de la mucosa.


Se asocia siempre a colonizacin de la mucosa duodenal.
El cultivo es la prueba diagnstica ms sensible.
La tasa de portadores sanos es muy elevada.
Se visualiza al microscopio ptico como Gram negativo y
flagelado.

CTO Medicina C/ Nez de Balboa, 115 28006 MADRID (Espaa) Tfno.: (91) 782 43 32 / Fax: (91) 782 43 27
E-mail: secretaria@ctomedicina.com; iberocto@ctomedicina.com WEB: www.ctomedicina.com; www.iberocto.com

DG Pg. 1

12.

13.

El aumento plasmtico de la gastrina-17 en un paciente con


gastritis crnica atrfica suele deberse en primer lugar a:

2)
3)

1)
2)
3)
4)
5)

4)

18.

2)
3)
4)
5)
19.

Realizar una gastroscopia y solicitar B12.


Un estudio baritado gastroduodenal.
Repetir la determinacin de los Ac anticlula parietal.
Tratamiento inmediato con cido flico.
Revisiones peridicas, ya que la anemia se corregir con
el tratamiento para el Addison.

5)

3)
4)
5)

Adenocarcinoma gstrico tipo intestinal.


Gastritis crnica tipo A.
Gastritis eosinoflica.
Mntrier.
Gastritis aguda por H. pylori.

Meloxicam.
Diclofenaco.
Rofecoxib.
Ibuprofeno.
Nabumetona.

Seale la correcta:
1)

Pg. 2 DG

La lcera gstrica se perfora con ms frecuencia que la


duodenal.
M exico A rgentina
C hile U ruguay

Se debe solicitar gastrina basal.


La biopsia de las zonas ms atrficas es til para demostrar
H. pylori.
La erradicacin de H. pylori precisa 14 das de tratamiento .
Las localizadas en fundus suelen necesitar ciruga.
Debe realizarse mantenimiento con un frmaco antisecretor.

Un paciente de 76 aos, con antecedente de ulcus gstrico


sangrante por H. pylori al que se le erradic con xito la
infeccin, consulta porque se le ha prescrito piroxicam por
una artritis traumtica de rodilla. Cul de las siguientes
medidas de gastroproteccin recomendara a continuacin?:
1)
2)
3)
4)
5)

22.

Debe suspenderse previamente la toma de omeprazol.


Es de eleccin el cultivo de la mucosa antral.
El test del aliento es poco sensible.
Slo est indicada ante la sospecha de resistencia bacteriana.
Debe realizarse siempre en las lceras duodenales.

En caso de ulcus gstrico, es correcto afirmar que:


1)
2)

21.

Test del aliento para H. pylori, para confirmar la erradicacin.


Tratar con metronidazol, si hay alergia a amoxicilina.
Tratamiento con bismuto coloidal, si presenta intolerancia a claritromicina.
Cultivo de la mucosa antral para H. pylori, si hay fracasos
teraputicos.
Evitar biopsias de los bordes de la lcera.

Para confirmar la erradicacin de H. pylori, es correcto


afirmar que:
1)
2)
3)
4)

20.

Un 10% de las lceras duodenales recidivan.


Las lceras duodenales suelen ser ms grandes que las
gstricas.
Las lceras duodenales aparecen sobre todo en la primera
porcin del duodeno.
Las lceras del fundus siempre son benignas.

En el tratamiento de un paciente con ulcus duodenal sangrante y test de la ureasa positivo, son correctas las siguientes
decisiones, EXCEPTO:
1)

Cul de los siguientes AINEs tiene menor potencial gastroerosivo?:


1)
2)
3)
4)
5)

17.

Sucralfato.
Misoprostol.
Famotidina.
Lansoprazol.
Ninguna medida farmacolgica.

A un varn de 46 aos se le remite a consultas para realizar


una gastroscopia por dispepsia refractaria a tratamiento con
ranitidina. En la analtica se observa Hto normal y ligera
hipoalbuminemia con VSG normal. El estudio endoscpico
demuestra en fundus gstrico pliegues muy engrosados. Con
estos datos, cul sera el diagnstico ms probable?:
1)
2)
3)
4)
5)

16.

5)

Mujer de 52 aos, recientemente diagnosticada de Addison,


que es remitida a consulta por presentar ttulos muy elevados
de anticuerpos anticlula parietal gstrica. Actualmente se
encuentra asintomtica. La Hb es de 9,6 g/dl y el volumen
corpuscular es de 113 fl. Con estos datos, qu aconsejara
a la paciente?:
1)
2)
3)
4)
5)

15.

Asociacin a carcinoma gstrico.


Gastritis por alcohol.
Tratamiento con sucralfato.
Desarrollo de un tumor carcinoide.
Presencia de hipoclorhidria.

Mujer de 46 aos, sin antecedentes de inters, que consulta


por un episodio mensual de migraa que cede en 24 horas
con 2 dosis de ibuprofeno. No presenta sntomas disppticos
y se decide continuar con el mismo tratamiento analgsico.
En relacin con la profilaxis de la lcera pptica por AINEs,
sera adecuado recomendar:
1)
2)
3)
4)
5)

14.

DIGESTIVO

Preparacin Examen de Seleccin 05/06 1 Vuelta

Acexamato de Zinc.
Omeprazol oral.
No precisa ninguna.
Ranitidina-bismuto al menor sntoma.
Sucralfato, y si hay sntomas, aadir misoprostol.

A un varn de 56 aos, sin antecedentes de inters, excepto


tabaquismo moderado, se le ha prescrito ibuprofeno por
dolor articular. Cul de las siguientes medidas es la ms
adecuada?:
1)
2)
3)
4)
5)

Parches de nicotina, para evitar riesgo de ulcus.


Misoprostol, 3-4 veces al da.
Administrar rofecoxib en vez de ibuprofeno.
Recomendar dieta rica en lcteos.
Ranitidina y magaldrato, si tiene dispepsia.

CTO Medicina C/ Nez de Balboa, 115 28006 MADRID (Espaa) Tfno.: (91) 782 43 32 / Fax: (91) 782 43 27
E-mail: secretaria@ctomedicina.com; iberocto@ctomedicina.com WEB: www.ctomedicina.com; www.iberocto.com

Preguntas TEST

Seguimiento a distancia

DIGESTIVO

Preparacin Examen de Seleccin 05/06 1 Vuelta


23.

Seale cul de los siguientes NO es un factor predisponente


de cncer gstrico:
1)
2)
3)
4)
5)

24.

Preguntas TEST

Linfoma.
Mala realizacin de la dieta.
Esprue colgeno.
Neoplasia esofgica.
Error diagnstico.

32.

Ingesta de hidrxido de magnesio.


Sndrome del intestino irritable.
Administracin de lactulosa.
Sndrome carcinoide.
Diarrea postvagotoma con piloroplastia.

Carcinoma esofgico.
Pericarditis constrictiva.
Enfermedad de Whipple.
Colitis granulomatosa.
Hipertiroidismo.

En el sobrecrecimiento bacteriano, es cierto todo lo siguiente, EXCEPTO:


1)
2)
3)
4)

La esteatorrea se debe a desconjugacin de las sales


biliares.
Los test del aliento son tiles para el diagnstico.
Suele haber recurrencias.
Puede haber atrofia intestinal leve.
M exico A rgentina
C hile U ruguay

Sigmoidoscopia.
Colonoscopia.
Enema opaco.
Gastroduodenoscopia.
Arteriografa.

La presencia de anemia ferropnica y melenas en una mujer


que tiene 23 aos, sugiere fundamentalmente el diagnstico
de:
1)
2)
3)
4)
5)

34.

Nueva colonoscopia hasta ciego.


Arteriografa.
Gastroduodenoscopia.
Estudio baritado gastroduodenal.
Enema opaco.

Cul de las siguientes pruebas realizara en primer lugar tras


una hematoquecia?:
1)
2)
3)
4)
5)

33.

Iletis tuberculosa.
Enfermedad de Whipple.
Esprue refractario.
Giardiasis.
Linfangiectasia intestinal.

Varn de 80 aos, con antecedente de dos episodios de


hemorragia digestiva alta por lcera duodenal hace 30 aos,
y estenosis artica con insuficiencia cardaca grado II. Consulta por presentar, hace diez das, rectorragias que le
duraron dos das. Actualmente contina asintomtico. Una
sigmoidocolonoscopia hasta ngulo esplnico no demuestra
lesiones. Qu medida diagnstica recomendara a continuacin?:
1)
2)
3)
4)
5)

Las siguientes enfermedades pueden cursar con entero-pata


y prdida de protenas, EXCEPTO:
1)
2)
3)
4)
5)

29.

Los anticuerpos IgA antitransglutaminasa.


Asociacin a HLA B51.
Hiperplasia crptica.
Infiltracin linfoplasmocitaria de la lmina propia.
Linfocitos intraepiteliales.

Entre las siguientes, es una causa de diarrea secretora:


1)
2)
3)
4)
5)

28.

31.

Cul le parece la causa ms frecuente de que un paciente con


esprue celaco contine con sntomas tras iniciar tratamiento?:
1)
2)
3)
4)
5)

27.

Sigmoidocolonoscopia.
Leucocitos fecales.
Grasas fecales.
Coprocultivo.
Trnsito intestinal.

Es excepcional en los gastrectomizados.

Un varn de 14 aos consulta por diarrea crnica. Presenta


ascitis y edemas en miembros inferiores, ms intensos sobre
todo en el miembro inferior derecho. En los anlisis aparece
hipoalbuminemia y linfopenia. La serologa de VIH es negativa, no hay proteinuria y el aclaramiento de alfa-1-antitripsina est aumentado. La paracentesis muestra una ascitis
quilosa. Se realiza una biopsia intestinal que confirma el
diagnstico clnico de:
1)
2)
3)
4)
5)

En la enfermedad celaca, NO es un hallazgo caracterstico:


1)
2)
3)
4)
5)

26.

30.

En un enfermo con diarrea y fiebre de 7 das de evolucin,


cul de las siguientes pruebas diagnsticas indicara en
primer lugar?:
1)
2)
3)
4)
5)

25.

Esfago de Barrett.
Infeccin por H. pylori.
Sndrome de Lynch tipo II.
Anemia perniciosa.
Alcoholismo crnico.

5)

Seguimiento a distancia

Ulcus duodenal.
Divertculo de Meckel.
Cncer de colon.
Poliposis rectal.
Adenocarcinoma gstrico.

Varn de 16 aos, con antecedentes de 2 3 episodios


anuales de dolor en fosa ilaca derecha, que duran entre
2 y 5 das. Habitualmente se acompaan de diarrea y se
resuelven de forma espontnea. Consulta porque, en el
ltimo episodio, el dolor es ms intenso, se acompaa de
fiebre y tres deposiciones al da. En la exploracin fsica
se encuentra con buen estado general. En la analtica
destaca leucocitosis. El coprocultivo es negativo. En una
sigmoidocolonoscopia se aprecian lceras aftoides en
ciego y colon derecho. En las biopsias se aprecia un
infiltrado inflamatorio inespecfico. Cul, de entre las

CTO Medicina C/ Nez de Balboa, 115 28006 MADRID (Espaa) Tfno.: (91) 782 43 32 / Fax: (91) 782 43 27
E-mail: secretaria@ctomedicina.com; iberocto@ctomedicina.com WEB: www.ctomedicina.com; www.iberocto.com

DG Pg. 3

DIGESTIVO

Preparacin Examen de Seleccin 05/06 1 Vuelta

siguientes opciones teraputicas, considera la ms apropiada para este paciente?:


1)
2)
3)
4)
5)
35.

4)
5)

2)
3)
4)
5)
39.

Esteroides, como mantenimiento tras brote leve-mo-derado, ya que garantizan una remisin prolongada.
Azatioprina, ante enfermedad crnicamente activa.
En brotes graves, puede indicarse ciclosporina i.v.
Colestiramina puede usarse en casos de afectacin ileal
severa, para control de la diarrea.
Sulfasalazina es eficaz en la colitis activa.

Una mujer de 35 aos, diagnosticada de colitis ulcerosa 4


aos antes, ingresa con fiebre, distensin abdominal dolorosa, diarrea de mltiples deposiciones y signos de deshidratacin. Presenta leucocitosis y anemia. Qu prueba
diagnstica realizara en primer lugar?:
1)

Pg. 4 DG

5)

42.

3)
4)
5)

Colonoscopia.
M exico A rgentina
C hile U ruguay

El tamao.
El tipo histolgico.
El grado de displasia.
La localizacin.
La multiplicidad.

El sntoma ms comn de los plipos colorrectales es:


1)
2)
3)
4)
5)

45.

Suele originarse en el colon derecho.


Es una complicacin poco frecuente de la enfermedad
diverticular del colon.
En la mayora de los casos, la hemorragia recidiva.
Suele presentarse como hematoquecia indolora.
Suele dejar de sangrar espontneamente.

Seale cul de entre las siguientes caractersticas de los


plipos de colon NO se correlaciona con el potencial de
malignizacin:
1)
2)
3)
4)
5)

44.

TC.
Eco abdominal.
Rx lateral de abdomen.
Enema opaco.
Ecografa hidroclica.

En relacin a la hemorragia diverticular, es FALSO:


1)
2)

43.

Se produce por la obstruccin proximal de la arteria


clica izquierda.
El recto est afectado en el 80% de los casos.
La angiografa es la tcnica diagnstica de eleccin.
Las imgenes en huella dactilar la diferencian de la
forma aguda.
Los sntomas remiten de forma habitual en 3 4 semanas
y no recidivan.

Ante la sospecha de una diverticulitis complicada con un


absceso de pared, solicitara como prueba diagnstica ms
rentable:
1)
2)
3)
4)
5)

Dolor abdominal.
Prdida de peso.
Masa abdominal.
Friabilidad al estudio endoscpico.
Fstulas.

En el tratamiento de la enfermedad de Crohn, las siguientes


pautas son adecuadas, EXCEPTO en el caso de:
1)

2)
3)
4)

41.

Ecografa abdominal.
Enema opaco.
Radiografa de abdomen.
TC abdominal.

En la colitis isqumica subaguda, es cierto que:


1)

Mesalazina y metronidazol.
Sulfasalazina sola.
Esteroides y metronidazol.
Mesalazina y esteroides.
Ciprofloxacino y mesalazina.

Uno de los siguientes hallazgos es ms frecuente en la colitis


ulcerosa que en la enfermedad de Crohn. Selelo:
1)
2)
3)
4)
5)

38.

Puede haber ANCA positivos en la colitis ulcerosa.


La colitis ulcerosa es ms frecuente en fumadores.
En el 50% de las personas con colitis ulcerosa aparece una
colangitis esclerosante.
La colitis ulcerosa afecta siempre y de forma uniforme a
todo el colon.
Es caracterstico de la colitis ulcerosa la presencia de
hemorragias sin moco.

Una paciente de 32 aos consulta por presentar, desde 2


semanas antes, diarrea sanguinolenta de 6 deposiciones diarias, dolor abdominal y febrcula. Dos coprocultivos fueron
negativos. Una colonoscopia demostr una mucosa eritematosa y granular con hemorragias puntuales desde recto a
ngulo esplnico del colon. La biopsia fue compatible con el
diagnstico de enfermedad inflamatoria crnica intestinal.
Cul de las siguientes actitudes sera ms correcta?:
1)
2)
3)
4)
5)

37.

40.

Seale la correcta:
1)
2)
3)

36.

Antagonistas de TNF.
Esteroides.
Metronidazol y mesalazina.
Azatioprina.
6-Mercaptopurina.

2)
3)
4)
5)

Intususcepcin.
Prolapso.
Rectorragia.
Estreimiento.
Diarrea.

Un varn de 66 aos, sin antecedentes personales ni familiares de inters, consulta por hematoquecia. En una colonoscopia se le extirpan 3 plipos adenomatosos sin signos de
displasia en sigma y colon izquierdo. En relacin con el
cribado del cncer de colon, sera lo ms apropiado someter
peridicamente al paciente a:
1)
2)

Test de hemorragias ocultas.


Colonoscopia.

CTO Medicina C/ Nez de Balboa, 115 28006 MADRID (Espaa) Tfno.: (91) 782 43 32 / Fax: (91) 782 43 27
E-mail: secretaria@ctomedicina.com; iberocto@ctomedicina.com WEB: www.ctomedicina.com; www.iberocto.com

Preguntas TEST

Seguimiento a distancia

DIGESTIVO

Preparacin Examen de Seleccin 05/06 1 Vuelta


3)
4)
5)
46.

5)

49.

Preguntas TEST

52.

Cul de las siguientes alteraciones NO se asocia con un


aumento aislado de la bilirrubina indirecta srica?:
1)
2)
3)
4)
5)

Ictericia fisiolgica del recin nacido.


Ictericia por lactancia materna.
Ictericia hemoltica.
Sndrome de Gilbert.
Sndrome de Dubin-Johnson.

La presencia de cncer de colon en varios miembros de una


familia, a edades inferiores a los 50 aos y sin evidencia de
poliposis asociada, le sugiere un sndrome de:
1)
2)
3)
4)
5)

1)
2)
3)
4)
5)

Lynch tipo I.
Gardner.
Muir-Torre.
Cowden.
Peutz-Jeghers.

En el seguimiento de un paciente intervenido de carcinoma


de colon estadio C, la elevacin del antgeno CEA significa:

54.

Recidiva tumoral.
Indicacin de test de hemorragias ocultas.
Error inicial en el estadiaje.
Sndrome de Lynch.
Una segunda neoplasia a nivel rectal.

Mujer de 46 aos, con antecedente de enfermedad celaca en


la adolescencia, que consult por presentar diarrea acuosa
de 3 meses de evolucin. En ocasiones se acompaaba de
moco, pero no de sangre ni pus. Un test de grasas fecales fue
de 3 g/da. Los coprocultivos y estudios de parsitos fecales
fueron negativos. Los anticuerpos antigliadina y antiendomisio y los ANCA fueron negativos. Las biopsias de duodeno y
yeyuno mostraron una mucosa normal. En una posterior
colonoscopia se observ una mucosa ligeramente enrojecida en sigma y colon izquierdo. En el estudio histolgico se
demostr un infiltrado predominantemente crnico con
linfocitos y plasmticas, sin microabscesos ni granulomas.
Cul de los siguientes diagnsticos es el ms probable?:
1)
2)
3)
4)
5)

53.

Rectosigmoidoscopia cada 2 aos.


Colonoscopia cada 3 aos.
Enema opaco anual.
Ningn seguimiento.
Proponerle colectoma, aunque no tenga plipos, y endoscopia alta cada 2 aos, posteriormente.

Una paciente de 29 aos consulta por padecer astenia leve.


En la exploracin fsica se aprecia una dudosa ictericia y el
resto es normal. Se realiza una analtica de orina urgente que
demuestra dos cruces de bilirrubina. Cul de los siguientes
trastornos sera MENOS probable?:

1)
2)
3)
4)
5)
50.

El riesgo depende de la extensin de la enfermedad.


El riesgo se relaciona con la duracin de la enfermedad.
El riesgo se relaciona con el grado de displasia.
Se recomienda colectoma profilctica a los 15 aos de
evolucin de la enfermedad.
Existe mayor riesgo cuando la enfermedad se presenta a
edades ms jvenes.

Un paciente de 44 aos consult por rectorragias. Una


colonoscopia demostr un cncer de sigma y mltiples
plipos adenomatosos en todo el colon. A su hermana de 47
aos se le realiz una sigmoidoscopia que fue normal. Qu
tipo de seguimiento habra que hacerle a la hermana?:
1)
2)
3)
4)
5)

Antecedentes familiares.
Sndrome del colon irritable.
Colitis ulcerosa.
Poliposis colorrectal.
Sndrome de los adenomas planos.

En relacin a la asociacin entre el adenocarcinoma de colon


y la colitis ulcerosa, seale la que es INCORRECTA:
1)
2)
3)
4)

48.

51.

En la aparicin de un carcinoma colorrectal, NO se considera


un factor importante la existencia previa de:
1)
2)
3)
4)
5)

47.

Eco hidroclica.
Niveles de antgeno CEA.
Si est asintomtico, solamente revisiones en consulta.

Seguimiento a distancia

Sndrome de intestino irritable.


Colitis ulcerosa.
Celaca colgena.
Hipertiroidismo.
Colitis linfoctica.
M exico A rgentina
C hile U ruguay

Cul es la primera exploracin instrumental que debe realizarse en un paciente con colestasis?:
1)
2)
3)
4)
5)

55.

Colecistografa oral.
Colangiografa intravenosa.
Colangiografa retrgrada endoscpica.
Ecografa.
TC.

Seale cul de las siguientes NO es una caracterstica del


sndrome alcohol-paracetamol:
1)
2)
3)
4)
5)

56.

Coledocolitiasis.
Ampuloma.
Hepatitis.
Colestasis benigna recurrente.
Sndrome de Gilbert.

Puede producir un fallo heptico agudo.


Est descrito incluso con el empleo de dosis teraputicas
de paracetamol.
Tiene un buen pronstico, en relacin al fallo heptico
agudo en general.
Es una de las causas ms frecuentes de fallo heptico
agudo.
Las transaminasas no suelen estar muy elevadas.

El tratamiento de eleccin de una ascitis a tensin es:


1)
2)
3)
4)
5)

Dieta hiposdica y de restriccin hdrica.


Espironolactona 100 mg/da.
Paracentesis evacuadora y reposicin con albmina.
Espironolactona asociado a furosemida.
Colocacin de TIPS.

CTO Medicina C/ Nez de Balboa, 115 28006 MADRID (Espaa) Tfno.: (91) 782 43 32 / Fax: (91) 782 43 27
E-mail: secretaria@ctomedicina.com; iberocto@ctomedicina.com WEB: www.ctomedicina.com; www.iberocto.com

DG Pg. 5

57.

3)
4)
5)

4)
5)

4)
5)

Est producida generalmente por gramnegativos.


Cursa con peritonismo en el 80% de los casos.
En el tratamiento emprico, no es necesario cubrir anaerobios.
Su tratamiento no incluye la ciruga.
El parmetro ms utilizado para el diagnstico de peritonitis bacteriana espontnea es la cifra de polimorfonucleares.

64.

Si asocia antecedentes de alcoholismo.


En caso de hemorragia digestiva alta.
nicamente a partir de haber padecido un segundo
episodio de peritonitis.
Cuando las protenas en lquido asctico son >1 g/dl.
Cuando los neutrfilos en lquido asctico son <100/
mm3.

Proliferacin mesangial.
Trombos de bilirrubina en las asas de Henle.
Infiltracin intersticial aguda de linfocitos.
Trombosis de la vena renal.
Histologa normal.

Paciente de 45 aos, en tratamiento con espironolactona por


ascitis secundaria a cirrosis en estadio C, y con varios episodios de sangrado por varices esofgicas, por lo que se
encuentra a la espera de trasplante. Reingresa por un nuevo

Pg. 6 DG

M exico A rgentina
C hile U ruguay

HBsAg.
ADN-VHB.
Anti-HBc de tipo IgM.
HBeAg.
Actividad ADN-polimerasa.

Seale cul de las siguientes manifestaciones extrahepticas


de la infeccin por el virus de la hepatitis C es ms frecuente:
1)
2)
3)
4)
5)

67.

El ARN-VHC es el mejor marcador para el diagnstico.


Tiene un riesgo elevado de evolucin a la cronicidad.
Es frecuente en drogadictos parenterales.
La mayora tienen historia de transfusin.
Un porcentaje importante de casos no tienen un antecedente epidemiolgico de inters.

Uno de los siguientes marcadores serolgicos es imprescindible para diagnosticar una infeccin aguda por VHB:
1)
2)
3)
4)
5)

66.

Nitritos en dosis bajas.


Propranolol.
Ligadura endoscpica.
nicamente vigilancia peridica.
Losartn.

Seale la respuesta INCORRECTA en relacin a la infeccin


por el VHC:
1)
2)
3)
4)
5)

65.

Quinolonas orales.
Colocacin de TIPS.
Administrar lactulosa en enemas.
Transfundir concentrados de hemates.
Aumentar la dosis del diurtico.

A un varn de 44 aos, asintomtico, se le realiz una biopsia


heptica por hepatitis VHC. El estudio histolgico revel
como hallazgo una cirrosis. Una ecografa abdominal y la
endoscopia oral fueron normales. En relacin con la profilaxis de la hemorragia por varices esofgicas, qu cree lo ms
adecuado recomendar?:
1)
2)
3)
4)
5)

Cul de los siguientes datos histolgicos renales es tpico del


sndrome hepatorrenal?:
1)
2)
3)
4)
5)

62.

63.

Indicara la profilaxis antibitica con quinolonas de la peritonitis bacteriana espontnea en un cirrtico con ascitis:
1)
2)
3)

61.

1)
2)
3)
4)
5)

Carcinomatosis peritoneal, sin metstasis hepticas.


Metstasis masivas hepticas, sin carcinomatosis peritoneal asociada.
Ascitis quilosa de origen tumoral.
Metstasis masivas hepticas, con gran aumento de la
fosfatasa alcalina.
Pacientes con aumento de la alfafetoprotena en lquido
asctico.

Seale la respuesta FALSA en relacin a la peritonitis bacteriana espontnea:


1)
2)
3)

60.

Sndrome de Budd-Chiari.
Cirrosis biliar primaria.
Tuberculosis peritoneal.
Enfermedad venooclusiva heptica.
Trombosis de la porta.

En cul de los siguientes casos de ascitis maligna suele


encontrarse citologa positiva en lquido asctico?:
1)
2)

59.

episodio de hematemesis. En una endoscopia se observan


varices grado IV con puntos sangrantes. Tras 48 horas de
iniciar el tratamiento con somatostatina y esclerosis de las
varices, contina sangrando con descenso notable de la TA
y del hematcrito. Las siguientes medidas podran estar
indicadas, EXCEPTO:

Un paciente de 47 aos consult por molestias abdominales


inespecficas y aumento del permetro abdominal. En la
exploracin fsica se detect matidez en ambos flancos, y la
ecografa confirm la presencia de lquido libre peritoneal.
El estudio del lquido asctico demostr un gradiente albmina srica/albmina en ascitis de 3,1. Cul de estos diagnsticos sera MENOS probable?:
1)
2)
3)
4)
5)

58.

DIGESTIVO

Preparacin Examen de Seleccin 05/06 1 Vuelta

Crioglobulinemia mixta esencial.


Glomerulonefritis membranoproliferativa.
Glomerulonefritis membranosa.
Anemia aplsica.
Polineuropata perifrica.

En cul de las siguientes situaciones existe mayor riesgo de


cronificacin de la infeccin por VHB?:
1)
2)
3)
4)
5)

Adquisicin por va parenteral.


Tras contagio sexual.
Adquisicin en la infancia.
Contagio en la edad adulta.
Tras contagio concomitante con infeccin por VIH.

CTO Medicina C/ Nez de Balboa, 115 28006 MADRID (Espaa) Tfno.: (91) 782 43 32 / Fax: (91) 782 43 27
E-mail: secretaria@ctomedicina.com; iberocto@ctomedicina.com WEB: www.ctomedicina.com; www.iberocto.com

Preguntas TEST

Seguimiento a distancia

DIGESTIVO

Preparacin Examen de Seleccin 05/06 1 Vuelta


68.

Un paciente de 28 aos, diagnosticado de infeccin crnica


por VHB en fase no replicativa, sufre un cuadro de hepatitis
aguda. Los datos de laboratorio confirman el diagnstico
clnico. Los marcadores vricos dan los siguientes resultados:
HBsAg+, HbeAg-, anti-HBc+, ADN VHB-, anti-VHC-, HDAg+, anti-HD IgM-, anti-HD IgG- e IgM anti-VHA-. Qu
diagnstico hara?:
1)
2)
3)
4)
5)

69.

Un paciente de 23 aos, ADVP activo, consulta por un cuadro


clnico y bioqumico compatible con una hepatitis aguda.
Presenta los siguientes marcadores: HBsAg-, anti-HBs-, antiHBc+, anti-HBe+, anti-VHC-, IgM anti-VHA+. Qu diagnstico hara?:
1)
2)
3)
4)
5)

70.

Preguntas TEST

74.

4)
5)
75.

Necrosis heptica con puentes.


Colapso multilobular.
Ndulos de regeneracin.
Necrosis parcelar perifrica.
Infiltrado mononuclear del espacio porta.

Un paciente de 43 aos consulta por molestias abdomina-les


inespecficas. En la exploracin fsica presenta nicamente
un discreto aumento del tamao heptico. Los datos de
laboratorio ofrecen los siguientes resultados: GPT 100 UI/l,
GOT 87 UI/l, HBsAg+, HBeAg-, anti-HBc+ de tipo IgG, antiHBe+, ADN-VHB+. El resto de los marcadores vricos dieron
resultado negativo. Qu diagnstico hara?:
1)
2)
3)
4)
5)

Infeccin crnica por VHB en fase no replicativa.


Infeccin aguda por VHB.
Infeccin crnica por la cepa mutante precore.
Infeccin pasada por VHB.
Probable hepatopata txica o metablica. Se puede
descartar enfermedad vrica.

M exico A rgentina
C hile U ruguay

El interfern gamma es el frmaco ms eficaz.


La respuesta es inferior en los VIH positivos.
La respuesta es superior en las mujeres.
Los niveles elevados de ALT predicen una buena respuesta.
Los pacientes con niveles bajos de ADN-VHB responden
mejor.

Respecto a un paciente de 62 aos, con cirrosis posthe-patitis


por VHC, al que se va a someter a un trasplante heptico, es
correcto afirmar que:
1)
2)
3)
4)
5)

78.

Biopsia heptica.
Interfern.
Solicitar anticuerpos antimitocondriales.
Interfern + ribavirina.
Seguimiento evolutivo.

En relacin con la terapia antiviral de la hepatitis crnica por


VHB, es cierto todo lo siguiente, EXCEPTO:
1)
2)
3)
4)
5)

77.

La eliminacin del virus del organismo.


La seroconversin anti-e.
Que el virus, aunque se replique, no produzca lesin
heptica.
Evitar la aparicin de mutantes precore y de escape.
Mejorar la cirrosis heptica.

A una mujer de 37 aos, ex ADVP, se le descubren anticuerpos


anti-VHC positivos. Durante seis meses de seguimiento, las
transaminasas y el resto de la bioqumica heptica fueron
normales. El RNA VHC fue +. En el manejo de la paciente,
recomendara:
1)
2)
3)
4)
5)

76.

Est contraindicada en VIH positivos.


Est indicado en la mutante Ag e negativo.
Su efecto adverso ms frecuente es el hipotiroidismo.
La va de administracin es intramuscular semanal.
Debe usarse asociado a interfern.

Cul es el objetivo principal del tratamiento de la hepatitis


crnica por VHB?:
1)
2)
3)

Seale el dato histolgico mnimo para poder hacer el diagnstico de hepatitis crnica activa:
1)
2)
3)
4)
5)

72.

Vacunacin completa de nuevo.


Una dosis de Ig y una dosis de recuerdo de vacuna.
Dos dosis de Ig.
Ningn tratamiento.
Una dosis de recuerdo de vacuna.

En relacin con el empleo de lamivudina en el tratamiento de


la hepatitis crnica VHB, es cierto que:
1)
2)
3)
4)
5)

Infeccin aguda por VHB.


Infeccin crnica por VHB.
Infeccin aguda por VHA.
Probable sobreinfeccin delta.
Hepatitis no vrica.

Un paciente vacunado frente al VHB hace 2 aos y con


respuesta desconocida a la vacuna sufre un contacto accidental con el VHB. Su ttulo de anti-HBs es <10 mUI/ml. Qu
le recomendara?:
1)
2)
3)
4)
5)

71.

Reactivacin de la infeccin por VHB.


Seroconversin anti-e.
Sobreinfeccin delta.
Coinfeccin delta.
No hay datos para poder asegurar un diagnstico con
certeza.

73.

Seguimiento a distancia

Es muy probable que el nuevo hgado se infecte por VHC.


En un alto porcentaje, se detecta anti-VHC.
La presencia de crioglobulinas es una indicacin incluso
en estadio A de Child.
No puede ser trasplantado por la edad.
Si el genotipo es Ib, es menor el riesgo de recidiva precoz
postrasplante.

Una paciente de 18 aos, sin antecedentes de inters, consulta


por hipertransaminasemia persistente. En el estudio inmunolgico se detectan ANA y ANCA positivos con marcadores
vricos negativos. Ante esta situacin, es correcto afirmar que:
1)
2)
3)

Presenta una colangitis esclerosante.


Se trata de una hepatitis crnica autoinmune tipo 2.
La biopsia heptica no establecer el diagnstico de
certeza.

CTO Medicina C/ Nez de Balboa, 115 28006 MADRID (Espaa) Tfno.: (91) 782 43 32 / Fax: (91) 782 43 27
E-mail: secretaria@ctomedicina.com; iberocto@ctomedicina.com WEB: www.ctomedicina.com; www.iberocto.com

DG Pg. 7

4)
5)
79.

3)
4)
5)

Interfern.
Penicilamina.
Trientine.
Esteroides.
cido ursodesoxiclico.

Niveles de IgM.
Anticuerpos antimitocondriales.
Hormonas tiroideas.
Niveles de bilirrubina.
Niveles de triglicridos.

Existe asociacin con el HLA-B8.


La mayora de los casos aparecen en pacientes que tienen
colitis ulcerosa.
La colectoma en la colitis ulcerosa protege del desarrollo
de colangitis esclerosante.
El diagnstico se establece basndose en los hallazgos
colangiogrficos.
Es ms frecuente en varones.

El hijo de 26 aos de un paciente afecto de hemocromatosis


presenta un ndice de saturacin de transferrina muy elevado
(65 %) y ferritina srica de 550 (N < 200). A continuacin,
se debe aconsejar:
1)
2)
3)
4)
5)

84.

2)
3)
4)
5)
85.

86.

Biopsia heptica.
Estudio mutaciones gen HFE.
Flebotomas.
D-penicilamina.
TC abdominal.

Seale cul de las siguientes es correcta, en la hemocromatosis primaria:

Pg. 8 DG

M exico A rgentina
C hile U ruguay

Confirmacin del diagnstico de pancreatitis.


La etiologa.
El pronstico.
La evolucin.
Modificaciones del tratamiento.

Cul es la causa ms frecuente de las pancreatitis agudas?:


1)
2)
3)
4)
5)

90.

Se reabsorbe menos agua.


Se pierde colesterol por la diarrea.
Disminuye la reabsorcin de sales biliares.
Aumenta la reabsorcin de colesterol.
El flujo biliar es ms lento.

La ecografa abdominal en una pancreatitis aguda ofrece


particularmente informacin sobre:
1)
2)
3)
4)
5)

89.

Insuficiencia renal crnica.


Estado de portador crnico del VHB.
Enfermedad de Crohn.
Estados hemolticos crnicos.
Obesidad.

Seale la causa por la que la bilis se vuelve ms litognica tras


las resecciones ileales:
1)
2)
3)
4)
5)

88.

Cirrosis alcohlica.
Hepatitis crnica VHB mutante precore.
Obesidad.
Diabetes mellitus.
Tratamiento con corticoides.

En cul de las siguientes enfermedades existe mayor riesgo


de desarrollar clculos pigmentarios en la vescula biliar?:
1)
2)
3)
4)
5)

87.

El hipogonadismo puede ser el cuadro de presentacin en


jvenes.
La intolerancia hidrocarbonada mejora con las flebotomas.
El gen anmalo est en el cromosoma 12.
El hipogonadismo se debe al acmulo de hierro en las
gnadas.
Cursa con herencia autosmica dominante.

Cul de las siguientes entidades NO se asocia de forma


habitual con esteatosis heptica?:
1)
2)
3)
4)
5)

Seale la opcin FALSA en relacin a la colangitis esclerosante:


1)
2)

83.

1)

Cul de estos parmetros tendra MENOS valor a la hora de


evaluar a un paciente con cirrosis biliar primaria?:
1)
2)
3)
4)
5)

82.

Trasplante.
Embolizacin.
Alcoholizacin percutnea.
Ciruga.
Cisplatino y adriamicina.

Una paciente de 38 aos consult por prurito y astenia. En


la exploracin fsica presentaba una hepatomegalia lisa a 2
centmetros del reborde costal. Los datos de laboratorio
fueron: bilirrubina 1,6 mg/dl; fosfatasa alcalina 1.040 U/L
(normal <280 U/L); el resto de la bioqumica heptica era
normal; los anticuerpos antimitocondriales fueron positivos. Una ecografa abdominal fue normal. Cul de los
siguientes frmacos sera ms til para la paciente?:
1)
2)
3)
4)
5)

81.

Hay que descartar una enfermedad de Wilson.


Debe recibir tratamiento con tacrolimus.

A un paciente de 51 aos con cirrosis grado C y frecuentes


episodios de encefalopata y ascitis, se le aprecia una lesin
de 3 cm en lbulo heptico derecho. Una PAAF confirma que
se trata de un hepatocarcinoma. Cul de las siguientes
opciones estara indicada?:
1)
2)
3)
4)
5)

80.

DIGESTIVO

Preparacin Examen de Seleccin 05/06 1 Vuelta

Alcoholismo.
Litiasis biliar.
Microlitiasis.
Esfnter de Oddi hipertensivo.
Fibrosis qustica.

Cul de las siguientes pruebas es la de mayor sensibilidad


para el diagnstico de pancreatitis crnica?:
1)
2)
3)
4)
5)

Ecografa abdominal.
Rx simple de abdomen.
TC.
CPRE.
Test de secretina.

CTO Medicina C/ Nez de Balboa, 115 28006 MADRID (Espaa) Tfno.: (91) 782 43 32 / Fax: (91) 782 43 27
E-mail: secretaria@ctomedicina.com; iberocto@ctomedicina.com WEB: www.ctomedicina.com; www.iberocto.com

Preguntas TEST

Seguimiento a distancia

DIGESTIVO

Preparacin Examen de Seleccin 05/06 1 Vuelta


91.

Todos los siguientes son criterios de Ransom, EXCEPTO uno


de ellos. Selelo:
1)
2)
3)
4)
5)

92.

Preguntas TEST

4)
5)

Papilotoma endoscpica.
Pancreatectoma.
Administracin de antibiticos, ms el tratamiento habitual.
Colecistectoma urgente.
Colecistectoma laparoscpica.

El parmetro ms apropiado para indicar el tratamiento con


enzimas pancreticas como substitucin es:
1)
2)
3)
4)
5)

99.

Test de la secretina.
Quimiotripsina fecal.
Tripsina srica.
Grasas fecales.
Test del pancreolauryl.

En el manejo de la ascitis pancretica, cul de entre las


siguientes medidas diagnsticas es la ms importante?:
1)
2)
3)
4)
5)

100.

Pancreatitis aguda ceflica.


Pancreatitis crnica etlica.
Carcinoma de pncreas.
Pancreatitis crnica obstructiva.
Nesidioblastosis.

CPRE.
TAC.
Ecografa.
Gammagrafa.
Laparoscopia.

Las siguientes medidas pueden ser tiles en el tratamiento del


dolor en la pancreatitis crnica, EXCEPTO:
1)
2)
3)
4)
5)

Pancreatectoma ceflica.
Suplementos de enzimas pancreticos.
AINEs.
Antagonistas del TNF.
Dilatacin y endoprtesis del Wirsung.

Cul es la causa ms frecuente de pancreatitis crnica?:


1)
2)
3)
4)
5)

96.

98.

Dficit de B12.
Ictericia.
Ascitis pancretica.

Paciente alcohlico, de 49 aos, que consulta por dolor


epigstrico postprandial de 3 meses de evolucin. En una
ecografa se observa una masa pancretica que se confirma
en la TC. Una CPRE muestra una estenosis nica en el
Wirsung. Con estos datos, el diagnstico ms probable es:
1)
2)
3)
4)
5)

Necrosectoma pancretica.
Somatostatina i.v.
Imipenem i.v.
Esteroides i.v.
Drenaje percutneo dirigido por TC.

Una paciente de 70 aos ingresa en el hospital por dolor


abdominal y vmitos. Los estudios analticos demuestran una
amilasa muy elevada. Una ecografa demuestra una ligera
dilatacin de la va biliar y colelitiasis. A las 48 horas de ingreso
contina con fuerte dolor abdominal, el Hto ha disminuido
un 15%, el calcio a 7,8 mg/dl, el BUN ha aumentado 15 mg/
dl y la albmina es 3,1 g/dl. Cul de los siguientes tratamientos sera ms adecuado?:
1)
2)
3)

95.

Diferir el inicio de la alimentacin oral.


Iniciar inmediatamente tratamiento antibitico.
Tratar precozmente con glucagn.
Ciruga precoz.
Tratar con esteroides.

Un paciente de 33 aos, alcohlico, ingresa por una pancreatitis aguda sin evidencia de litiasis biliar. A las 24 horas,
presenta fiebre, hipotensin, oliguria e hipoxemia. Un TC con
contraste urgente demuestra un rea de necrosis pancretica del 70%, sin evidencia de absceso. Una puncin aspiracin
con aguja fina de la zona afectada demuestra bacilos gramnegativos. Entre las siguientes, qu medida teraputica
propondra para mejorar la evolucin?:
1)
2)
3)
4)
5)

94.

97.

Para evitar complicaciones en el curso de la pancreatitis


aguda moderada, es necesario:
1)
2)
3)
4)
5)

93.

Calcio < 8 mg/dl.


pO2 < 60 mmHg.
GOT>250 UI.
Leucocitosis >16.000.
PCR > 2 veces superior al lmite normal.

3)
4)
5)

Seguimiento a distancia

Alcoholismo crnico.
Malnutricin calrico-proteca.
Hiperlipemia.
Mucoviscidosis.
Poliovirus.

En el curso de la pancreatitis crnica, es EXCEPCIONAL que


suceda:
1)
2)

Aparicin de calcificaciones pancreticas.


Cetoacidosis diabtica.
M exico A rgentina
C hile U ruguay

CTO Medicina C/ Nez de Balboa, 115 28006 MADRID (Espaa) Tfno.: (91) 782 43 32 / Fax: (91) 782 43 27
E-mail: secretaria@ctomedicina.com; iberocto@ctomedicina.com WEB: www.ctomedicina.com; www.iberocto.com

DG Pg. 9

DIGESTIVO Y CIRUGA GENERAL


Preparacin Examen de Seleccin 05/06 1 Vuelta
Pregunta 1.-R: 1
El trmino disfagia orofarngea incluye a las enfermedades farngeas y a las del esfago superior. Las enfermedades involucradas con
mayor frecuencia producen disfagia por:
1. Debilidad de los msculos farngeos, como en las miopatas.
2. Rigidez muscular, como en el Parkinson que es la respuesta correcta.
3. Alteraciones en la motilidad secundarias a origen neurgeno, bien
de nervio perifrico o de SNC como es el caso de los ictus.
Estas enfermedades suelen comportarse como disfagias motoras y
por tanto tienen sntomas tanto con la ingesta de slidos como de
lquidos. El sntoma que orienta con mayor precisin al origen orofarngeo de una disfagia es la presencia de episodios de aspiracin relacionados con la deglucin (atragantamiento).
La acalasia es tambin una enfermedad motora, por tanto con
disfagia a slidos y lquidos, pero afecta al tercio inferior del esfago
fundamentalmente. El esfago de Barrett causa disfagia, si se asocia a
una esofagitis con estenosis pptica. El anillo de Schatzki, o anillo B, es
una protrusin en la zona inferior del esfago que causa disfagia recurrente e intermitente.
Pregunta 2.-R: 2
El diagnstico de certeza de la acalasia es manomtrico. Sin embargo siempre debe realizarse una endoscopia con biopsias del segmento distal del esfago para excluir neoplasias que simulen una
acalasia, y que se conocen como acalasias secundarias. Estos tumores
cursan con una disfagia semejante a la acalasia primaria, porque infiltran la pared del esfago alterando la motilidad de su zona distal y
esfnter. Los tumores mucosos esofgicos, como el carcinoma epidermoide o el adenocarcinoma, al ser mucosos se comportan como una
obstruccin y, por tanto, inicialmente con disfagia a slidos y en los
estados muy avanzados, a lquidos. Entre los tumores que causan
acalasia secundaria destacan:
1. El ms frecuente de todos: el adenocarcinoma gstrico, que origina acalasia cuando progresa infiltrando el espesor de la pared y
llega a la zona distal del esfago y al esfnter esofgico inferior. Por
eso la respuesta correcta es adenocarcinoma gstrico e incluso, si
se hubiera expuesto, de origen fndico o de la unin gastroesofgica, an hubiera sido ms correcta la respuesta.
2. Los situados en mediastino posterior: extensin de un microctico
de pulmn y los linfomas.

Comentarios TEST

1.
2.

3.
4.

Pregunta 3.-R: 2
Entre las complicaciones de la acalasia cabe citar:
Disminucin de peso por la desnutricin, debida a que por la
disfagia el paciente ingiere menor cantidad de alimento.
La esofagitis que se produce por el contacto repetido del alimento
con la pared esofgica. En ocasiones se debe a una infeccin aadida por Cndida; pero no es posible que sea por reflujo, ya que
en esta enfermedad el esfnter est hipertnico en reposo y no
permite el reflujo gstrico al esfago.
Es de inters destacar que:
Los pacientes tratados con dilatacin endoscpica tienen posteriormente reflujo al quedar el esfnter incompetente.
Si un paciente con clnica de pirosis por reflujo gastroesofgico
presenta de forma simultnea desaparicin de su pirosis (es decir, mejora del reflujo) y a la vez aparicin de disfagia se debe
sospechar que se ha establecido una acalasia.
Tienen ms riesgo de cncer de esfago.
El dficit de vitamina B12, respuesta correcta, no se puede atribuir
a la acalasia, ya que los problemas para su absorcin no se deben
a patologa esofgica y s a gstrica (gastritis autoinmune), insuficiencia exocrina pancretica, sobrecrecimiento bacteriano o alteraciones en la pared del ileon distal.

Pregunta 4.-R: 1
El tratamiento de eleccin de la acalasia es la dilatacin neumtica
endoscpica, por ser menos cruenta que la ciruga; pero cuatro circunstancias obligan a plantear como primera opcin la ciruga:
1. El fracaso de las dilataciones endoscpicas (respuesta correcta la 1).
2. La imposibilidad de someterse a la dilatacin por presentar erosioM exico A rgentina
C hile U ruguay

Seguimiento a distancia

nes o divertculos en el esfago distal, ya que existira riesgo de


hemorragia o perforacin.
3. Cuando deseamos plantear al paciente un tratamiento definitivo.
La miotoma de Heller es ms eficaz: presenta menos fracasos que
las dilataciones. Por eso en adolescentes y jvenes se prefiere de
primera eleccin el tratamiento quirrgico. ste se recomienda
por va laparoscpica y con ciruga antirreflujo asociada.
La toxina botulnica administrada va endoscpica en esfago distal
se reserva para los casos de imposibilidad de someterse al tratamiento
dilatador o la miotoma. La toxina acta paralizando a las nicas
fibras que permanecen funcionantes en el esfnter (las que lo mantienen contrado permanentemente) y, por tanto, como consecuencia
de su accin el esfnter quedar constantemente relajado. Su desventaja es que requiere repetidas administraciones. Los nitritos orales
nicamente alivian el dolor torcico, pero no son resolutivos del problema esencial de la enfermedad.
Pregunta 5.-R: 3
La presencia en una manometra de contracciones simultneas de
gran amplitud y duracin en el esfago distal puede deberse a dos
entidades: el espasmo esofgico difuso, que es la respuesta correcta y
la acalasia vigorosa. Se distinguen porque esta ltima presenta adems
dificultad para relajar el esfnter durante la deglucin. En realidad la
acalasia vigorosa no es una enfermedad o entidad clnica como tal,
sino que se trata generalmente de una acalasia en fases iniciales de su
evolucin y que por la denervacin de la pared esofgica presenta
contracciones de gran amplitud, por lo que adems de disfagia, puede tener episodios de intenso dolor torcico, retroesternal, y en ocasiones odinofagia.
La amiloidosis esofgica tiene un comportamiento clnico y manomtrico muy parecido a la esclerodermia: disfagia motora y reflujo.
El anillo B o de Schatzki causa disfagia intermitente a slidos y
menos frecuentemente dolor.
La acalasia cricofarngea o barra farngea es una enfermedad motora que provoca disfagia orofarngea. Debe diagnosticarse por
manometra y, en ocasiones, se aprecia en la Rx cervical lateral una
protrusin en la regin prevertebral anterior, que corresponde al
msculo cricofarngeo contrado.
Pregunta 6.-R: 5
Los trastornos motores del esfago distal son fundamentalmente el
espasmo esofgico difuso, la acalasia y la esclerodermia. Estos dos
ltimos presentan ondas sugestivas de un peristaltismo reducido e
incluso en ocasiones falta absoluta de movimiento o aperistalsis. Fundamentalmente la esclerodermia se diferencia de la acalasia en la
manometra en que el esfnter esofgico inferior est hipotnico (por
eso la respuesta correcta es esclerodermia) y se relaja bien con la
deglucin (ver tabla en pgina siguiente).
El sndrome de Sjgren puede dar lugar, en ocasiones, a disfagia
alta por la intensa sequedad de la mucosa.
El sndrome de Boerhaave es la perforacin espontnea del esfago, generalmente tras una maniobra intempestiva, como un vmito o
un valsalva. Presenta la clnica de dolor retroesternal, semejante a la
perforacin iatrgena y, adems, derrame pleural con lquido rico en
amilasa.
Pregunta 7.-R: 3
El antecedente de pirosis y regurgitacin es altamente sugestivo de
enfermedad por reflujo gastroesofgico cido. Con estos sntomas no
se precisan pruebas diagnsticas y la actitud adecuada es instaurar
tratamiento con inhibidores de la bomba de protones (no con antiH2) y asumir el diagnstico si desaparece la sintomatologa. La aparicin de dolor o disfagia plantea que ha surgido una complicacin:
bien una estenosis pptica o un carcinoma, por lo que es necesario la
realizacin de una endoscopia (la respuesta correcta). La pH-metra,
prueba de certeza de la enfermedad por reflujo gastroesofgico, tiene
sus indicaciones:
1. Refractariedad al tratamiento mdico.
2. Valoracin pre y postquirrgica.

CTO Medicina C/ Nez de Balboa, 115 28006 MADRID (Espaa) Tfno.: (91) 782 43 32 / Fax: (91) 782 43 27
E-mail: secretaria@ctomedicina.com; iberocto@ctomedicina.com WEB: www.ctomedicina.com; www.iberocto.com

HM Pg. 1

DIGESTIVO Y CIRUGA GENERAL

Seguimiento a distancia

Preparacin Examen de Seleccin 05/06 1 Vuelta

Pregunta 6. Caractersticas clnicas de las principales enfermedades esofgicas que cursan con disfagia.

12131451

6423678967
51

64 14
8 648 528
95 48

217258
1

6

123456728672956597

2 2 2647

123456728672956597

9562 2647

 2647

4

92249

92249



4



4

9

4

4

$"#4

49229

9 9689 392 4!236"4#427


"45#926#68676279# 566

%6 542&62973'26(6 )65429


 5#62*9 9689 39
979#68565

$6 342"6627,85472#44

#689 392"6627,8547 0716*6236 342"6627,8547


"66282547297
2  +4*97(6!2
&29-7392".5562592"974
2/"946
#4428254726#4"6656

3939
392&2 4297
 
#4 2982973.72&2#45672'"567

42 425929184

"4*97(62 29-7392".556

0716*62"4*97(62 342#4
".5562592"974

592"974

2/ 26 4936286
22"97, 25982//326768
22973'257 562"94
228629866#, 2598
22971 392972 468

3. Sntomas predominantes extraesofgicos, por ejemplo. tos nocturna, asma o laringitis.


El test de Bernstein, o de provocacin con una ligera cantidad de
cido, se utiliza para aquellos casos en los que predomina una clnica
con episodios aislados y limitados de intenso dolor retroesternal parecido al angor y con pH-metra no diagnstica.
El esofagograma con bario tiene poca utilidad, nicamente como
prueba complementaria a la endoscopia en el estudio de las estenosis.
Pregunta 8.-R: 4
El esfago de Barrett es una complicacin del reflujo. Se considera a
su vez una forma de peor pronstico si se acompaa de displasia en
alguna de las biopsias. Se considera displasia grave si la displasia aparece en todas las muestras y se comprueba por dos anatomopatlogos y
se asume que el riesgo de un adenocarcinoma injertado en el esfago
de Barret es muy elevado, por tanto se indica la esofaguectoma de la
zona afecta. A diferencia de una esofagitis por reflujo no complicada, el
esfago de Barrett requiere controles endoscpicos para valorar el riesgo de progresin a adenocarcinoma aunque el paciente se encuentre
asintomtico. Si se observaron displasias se recomienda endoscopia
cada seis meses. Si no hay displasia, como en el presente caso, cada ao
y medio o dos aos.
El Barrett est provocado por el reflujo, sin embargo, puede persistir aunque mejore o cure la esofagitis. El paciente debe tomar omeprazol porque es muy probable que en el momento del diagnstico
tenga una esofagitis acompaante, al menos moderada.
La respuesta a sealar es la 4, pues es muy controvertida en el
esfago de Barret la relacin entre adenocarcinoma de esfago y
colonizacin por H.pylori; actualmente se piensa que no guardan
relacin.
Pregunta 9.-R: 1
El carcinoma epidermoide sigue siendo el grupo histolgico ms
frecuente. Desgraciadamente se diagnostica cuando aparece disfagia
y significa que la enfermedad est muy avanzada y, por ello, la supervivencia a los 5 aos oscila entre el 15 al 20%.

Pg. 2 HM

M exico A rgentina
C hile U ruguay

26 4936623"#67
22#4 36##4 97
229"935672592*6
226"8352&2783' 967
222922#49 )6 29 286
22"6392 1942598
2297,16*4

2564623"#626*9 29
2276#6#4#947

2/ 547#4"62&24"76

22/72'7219#9 3929 298


739#42954!
789*4239#42 194
7&289*4227"94

En ocasiones se aprecian ulceraciones visibles en el estudio baritado. La respuesta correcta es la 1, porque el adenocarcinoma supone
aproximadamente un 10% o menos, aunque en los ltimos aos ha
aumentado su incidencia, posiblemente en relacin con el esfago
de Barrett.
Pregunta 10.-R: 3
Para los tumores esofgicos malignos podemos concretar que
para:
El diagnstico es de eleccin la toma de biopsias por endoscopia.
Evaluar metstasis en pulmn, hgado u otras localizaciones: el
TAC y la resonancia magntica son las mejores pruebas.
Conocer si hay adenopatas tumorales, estadiaje N, es de eleccin
el TAC torcico.
Determinar el grado de extensin intramural en el esfago es la
ecografa endoscpica (respuesta correcta).
Pregunta 11.-R: 5
Es un gram negativo y flagelado (respuesta correcta la 5), que afecta
a la mucosa antral y, en ocasiones, a la duodenal prxima al ploro;
por tanto no se asocia siempre a colonizacin duodenal. Suele encontrarse en las capas profundas del moco o entre ste y el epitelio,
pero no invade la lmina propia. Su infeccin siempre produce lesin histolgica, por tanto no hay portadores, slo enfermos, aunque
frecuentemente asintomticos.
En relacin con el diagnstico de la infeccin debemos recordar
que (ver tabla):
1. El cultivo de la mucosa antral es la prueba de certeza, pero es poco
sensible y difcil de realizar.
2. El estudio histolgico de la mucosa con tincin de Giemsa es muy
eficaz para el diagnstico.
3. La prueba de la ureasa sobre histologa o el test del aliento con urea
marcada con istopo C13 son muy rentables. Esta ltima tiene una
sensibilidad y especificidad superior al 95%, siempre que el paciente no est tomando antibitico o inhibidor de la bomba de
protones al realizarse la prueba.

CTO Medicina C/ Nez de Balboa, 115 28006 MADRID (Espaa) Tfno.: (91) 782 43 32 / Fax: (91) 782 43 27
E-mail: secretaria@ctomedicina.com; iberocto@ctomedicina.com WEB: www.ctomedicina.com; www.iberocto.com

Comentarios TEST

9 !

2092989##, 28626 4936


2254 5922792479(622"97, 2598
22//32 46824269 3656

22462"97, 2676825982#9"4
22798929736269 3656

2206 39286259*8#, 26"69#9


22#4 36##4 972783' 967

256462679 #62592#'66226.96

736*9 29 28"#425928448

DIGESTIVO Y CIRUGA GENERAL


Preparacin Examen de Seleccin 05/06 1 Vuelta
Pregunta 11. Fiabilidad de las pruebas diagnsticas del H. pylori.
123456

7487959
9 6 

12345672879
4

626427967

7 
76727 44


22222222
844





2 72
72 2 7
!"68
3866

22222222#8
$4

%

&7
2
2677
67
87
224
8
5
8
4

22222222'787
2 7
222222228
$
 267

(

)68

12&2
784

%

*62464562
766

2876+8

,-272 7+.

%

/42
8
727867

07
28
$2 2 1
28
27278
4
7
7
4
41
328+
24277 
7272&35361

1237644

Comentarios TEST

Pregunta 12.-R: 5
La gastrina aumenta, en general, como consecuencia de la reduccin de la acidez gstrica, por tanto la hipoclorhidria es la causa ms
frecuente (respuesta correcta la 5). Por tanto las situaciones que causan hipoclorhidria provocan aumento de la gastrina, como son las
gastritis atrficas y el uso de anti-H2 o inhibidores de la bomba de
protones. No lo causan el sucralfato, que es un protector de la superficie ulcerada o erosionada pero no un antisecretor.
Es cierto que algunos carcinomas gstricos se asocian a hipoclorhidria y, por tanto, a aumento de gastrina. Esta hipoclorhidria de los
tumores se debe a que coexiste una gastritis atrfica severa, no es
por tanto un efecto tumoral. Otras causas de hipergastrinemia son
los gastrinomas y menos frecuentemente los tumores carcinoides.
Pregunta 13.-R: 5
La profilaxis de la lcera por AINES, o gastroproteccin, se realiza
con omeprazol o misoprostol. No se ha demostrado utilidad de los
anti-H2, sucralfato o anticidos. Aunque el riesgo de padecer una
lesin gstrica, incluso una lcera complicada, es superior por el mero
hecho de tomar AINES, se considera que slo procede administrar
omeprazol o misoprostol cuando coexisten factores de riesgo que
aumentan ste de forma intolerable. Son factores de riesgo: edad
superior a 70 aos, antecedente de lcera, estar anticoagulado, altas
dosis de AINES, combinaciones de los mismos, uso concomitante de
corticoides y una enfermedad grave (no necesariamente estar hospitalizado). En el presente caso la paciente no tiene ningn factor de
riesgo, por lo que la respuesta adecuada es no recomendar ninguna
medida farmacolgica.
Pregunta 14.-R: 1
Los anticuerpos anticlula parietal gstrica se usan para el diagnstico de la gastritis autoinmune (tipo A). Aunque son muy sensibles, son
poco especficos y pueden aparecer ttulos, en general moderados,
en otras enfermedades autoinmunes o en sujetos sanos. Los antifactor
intrnseco son ms especficos; sin embargo para el diagnstico es
obligado una biopsia del fundus gstrico, donde se objetive la atrofia.
La anemia macroctica sugiere dficit de B12, por todo ello la respuesta correcta es la primera.
Un estudio baritado no permite hacer el diagnstico de gastritis, ni
aguda ni crnica.
Pregunta 15.-R: 4
Entre las causas de pliegues gstricos engrosados cabe destacar:
M exico A rgentina
C hile U ruguay

1.
2.
3.
4.
5.
6.

Seguimiento a distancia

Sfilis.
Tuberculosis.
Sarcoidosis.
Linfoma.
Carcinoma gstrico, sobre todo tipo difuso.
La enfermedad de Mntrier. La asociacin de engrosamiento en
fundus, dispepsia e hipoalbuminemia, sin que se observen reactantes bioqumicos de enfermedad aguda, sugiere este diagnstico.
La confirmacin sera histolgica.

Pregunta 16.-R: 3
El mecanismo por el cual los AINES producen gastrotoxicidad es
por la reduccin de las prostaglandinas gstricas como consecuencia
de la inhibicin de la COX-1. A ms poder de inhibicin de la COX1 ms gastroerosivo es el frmaco. En la escala de gastroerosividad son
piroxicam y ketorolaco los de mayor capacidad erosiva e ibuprofeno
el de menor. Entre los nuevos AINES inhibidores selectivos de la COX2 se encuentran los menos gastroerosivos de todos: rofecoxib (respuesta correcta) y celecoxib. stos reducen un 50% el riesgo
gastroerosivo frente a frmacos tradicionales como diclofenaco o naproxeno.
Pregunta 17.-R: 4
La causa ms frecuente de lcera es la infeccin por Helicobacter
pylori. Sus lceras aparecen fundamentalmente en antro gstrico y,
sobre todo, en la primera porcin del duodeno.
Las lceras gstricas son ms grandes que las duodenales, por eso
tienen ms riesgo relativo de sangrado digestivo. Aunque son ms grandes, las gstricas se perforan menos, dado el mayor espesor de la
pared gstrica. Se calcula que entre el 6 y el 8% de las lceras gstricas
pueden ser malignas. Cuando una lcera aparece en fundus, la primera posibilidad es que siga siendo benigna, pero respecto a lo que
sucede en otras regiones gstricas aumentan las probabilidades de
que sea maligna. Las diferencias entre duodenales y gstricas se observan en la tabla.

Pregunta 17. Diferencias entre lcera duodenal y gstrica.

123456789
84 62

56

1234567 536

8
6 27
1234256789
4 6
9
4 68
8 262 2
6
8492
6
266
6
22

89

4 2 6

4
64
682

 865 

!865 

" #

$ % #

&
68 8

&
68 8

' 
829(

)9

' 
829(

2 8)9


2363
4

12 2
6 7
*2

66

967
92

6 7
+
4 ,

-2

668./ #07
92

6 89 

242 2,

6 3

128 
627
2   968.
9
26078 8
46

3   96829
27
.26
6

6 60

%"#82 8/865 

4
242 2


36263
868
72
5
43543 7 38

662
2 36

438!6

CTO Medicina C/ Nez de Balboa, 115 28006 MADRID (Espaa) Tfno.: (91) 782 43 32 / Fax: (91) 782 43 27
E-mail: secretaria@ctomedicina.com; iberocto@ctomedicina.com WEB: www.ctomedicina.com; www.iberocto.com

HM Pg. 3

Pregunta 18.-R: 3
Cuando una lcera gastroduodenal presenta una hemorragia digestiva y se confirma la infeccin por H.pylori, la erradicacin del
microorganismo es una prioridad. Se inicia el tratamiento oral en
cuanto el paciente comienza a tener una ingesta de alimentos. Se
usa la triple terapia con omeprazol, amoxicilina y claritromicina,
cuya eficacia es del 95%; pero en caso de alergia o intolerancia a
alguno de los antibiticos se usa metronidazol y timidazol, no bismuto
coloidal, por eso la respuesta correcta es la 3. ste se utiliza en combinacin con tetraciclina, metronidazol y omeprazol en caso de
que la triple terapia previa no consiga la erradicacin de H.pylori.
Si, an con esta triple terapia, no se consiguiera erradicar, se deberan tomar cultivos de la mucosa para obtener antibiograma y as
seleccionar la terapia.
Despus de una lcera con hemorragia es obligado confirmar la
erradicacin de H.pylori con un test del aliento. En el momento del
sangrado de una lcera se deben evitar las biopsias de los bordes,
pero es que adems en esta pregunta se trata de una lcera duodenal,
por lo que de todas formas es innecesaria la biopsia.
Pregunta 19.-R: 1
En las lceras gstricas debe confirmarse siempre la erradicacin
de H.pylori tras completar tratamiento erradicador siendo altamente
recomendable en las duodenales (por tanto no deben realizarse siempre de forma obligada, aunque sto no es contradictorio con el concepto de altamente recomendable), pasando a ser obligatorio si las
duodenales se complicaran con hemorragia.
La prueba de eleccin es el test del aliento y para evitar falsos
negativos debe realizarse entre 15 das y un mes despus de haber
suspendido omeprazol y antibiticos.
Pregunta 20.-R: 5
Las dos pruebas ms utilizadas para el diagnstico de la infeccin
por H.pylori en ulcus gstrico son el test de la ureasa y el estudio con
tinciones de la mucosa biopsiada. Su sensibilidad depende de la calidad de la muestra, por eso se debe evitar tomar muestras de las zonas
con mayor atrofia o metaplasia, pues en estas localizaciones es muy
difcil demostrar la presencia de la bacteria.
El tratamiento erradicador con la triple terapia requiere entre 7 y 10
das y, en principio, no es necesario llegar hasta 14 das, pues el beneficio adicional es mnimo. Las lceras gstricas una vez concluido el
tratamiento erradicador requieren un mantenimiento durante un perodo no concretado de semanas (oscila entre 4 y 8) con un frmaco
antisecretor, preferentemente un anti-H2 (esta es la respuesta correcta) y
no es necesario en las duodenales, a no ser que debutaran sangrando o
fueran gigantes. La ciruga del ulcus est fundamentalmente motivada
por sus complicaciones, no por su localizacin. En un ulcus no refractario y sin otros sntomas ms que los habituales de la enfermedad
ulcerosa, no es necesario solicitar gastrina.
Pregunta 21.-R: 2
El paciente presenta un factor de riesgo para la gastropata y lcera
complicada por AINES, como es la edad mayor de 70 aos, por lo
que requiere omeprazol oral (la respuesta correcta).
Parece que erradicar H.pylori reducira el riesgo de lcera complicada al tomar un AINE, pero no se ha demostrado que evite tomar el
gastroprotector. En relacin a la ranitidina bismuto coloidal es un
frmaco con actividad frente a H.pylori y buena alternativa al omeprazol en la triple terapia.
Pregunta 22.-R: 5
No tiene factores de riesgo para asociar gastroprotector al consumo
de ibuprofeno, por lo que no necesita misoprostol y no es necesario
sustituir ibuprofeno por rofecoxib.
El tabaquismo tiene un efecto deletreo comprobado bsicamente como elemento que dificulta la cicatrizacin de las lceras. La dieta
no tiene influencia.
Una dispepsia inducida por antiinflamatorios debe tratarse retirando si es posible el antiinflamatorio y aadiendo un anticico o
un anti-H2 durante un tiempo limitado, por eso la respuesta correcta es la 5.
Pg. 4 HM

M exico A rgentina
C hile U ruguay

DIGESTIVO Y CIRUGA GENERAL

Preparacin Examen de Seleccin 05/06 1 Vuelta

Pregunta 23.-R: 5
H.pylori es un reconocido agente carcingeno debido a la atrofia gstrica y metaplasia que induce. La gastritis tipo A, frecuentemente asociada a anemia perniciosa, es tambin un factor de riesgo
para el cncer. El adenocarcinoma en el Barrett puede, a veces,
localizarse dentro de lo que se considera anatmicamente el estmago. El Lynch tipo II, asocia cncer colorrectal hereditario sin
plipos con cncer gstrico. El alcoholismo crnico puede facilitar
una gastritis, pero no hay evidencia de que sea un factor causal de
lcera o cncer gstrico.
Pregunta 24.-R: 2
Habitualmente en una diarrea aguda, de menos de 14 das de evolucin, no se requiere obtener el diagnstico etiolgico, pues en general
son procesos leves y autolimitados. En caso de deshidratacin, sepsis,
estado de inmunosupresin o edad muy avanzada, al igual que en las
duraciones prolongadas del cuadro diarreico de ms de 10 das es necesario obtener el diagnstico. La primera prueba, y por eso es la respuesta
correcta, es el estudio de los leucocitos fecales. Si es positivo sugiere un
origen inflamatorio y hay que esperar al resultado del coprocultivo para
el diagnstico. En caso de que las pruebas bacteriolgicas no consiguieran el diagnstico, se realizara una colonoscopia. Si no hay leucocitos en
las heces, se entiende que es una diarrea acuosa aguda, en general debida a toxinas bacterianas, que suelen ser autolimitadas y precisan diagnsticos microbiolgicos y epidemiolgicos especficos.
Pregunta 25.-R: 2
Para el diagnstico de enfermedad celaca se precisa una biopsia
que demuestre atrofia de las vellosidades, hiperplasia de las criptas e
infiltracin linfoplasmocitaria de la lmina propia. Si, adems, el
estudio inmunolgico es positivo, la actitud diagnstica pasa por
retirar el gluten de la dieta, observar la mejora clnica y la negativizacin de los anticuerpos. Si sto ocurriera no es obligada una segunda biopsia dos aos despus de la primera que confirme la recuperacin histolgica.
Los linfocitos intraepiteliales estn aumentados en nmero en esta
enfermedad y desempean un papel patgeno importante. Se considera celaca latente a la positividad de los anticuerpos y aumento de
linfocitos intraepiteliales en un familiar de un celiaco. La asociacin al
HLA es fundamentalmente al DQ2 y no al B51. En relacin con los
anticuerpos se utilizan los antigliadina IgG e IgA, los antiendomisio IgA
y, recientemente, los antitransglutaminasa IgA, que son muy sencillos
de realizar, sensibles y especficos.
Pregunta 26.-R: 2
Entre las causas de refractariedad al tratamiento sin gluten se encuentran por orden de probabilidad:
1. Incumplimiento diettico, que es la respuesta correcta.
2. Linfoma tipo T intestinal.
3. Esprue colgeno.
4. Otros diagnsticos distintos a la propia enfemedad celiaca, como
el esprue autoinmune o el tropical.
Pregunta 27.-R: 4
En general secretora y acuosa suele ser un trmino equivalente.
Son causas los tumores endocrinos, como el vipoma, pepoma, o tumor carcinoide (respuesta correcta), que liberan sustancias estimuladoras de la liberacin de lquido a la luz intestinal o que alteran la
permeabilidad del epitelio intestinal. Otras causas son los plipos vellosos grandes y mltiples. En los casos agudos, las diarreas por toxinas
bacterianas, como Staph.aureus o E.Coli.
La vagotoma y el intestino irritable son diarreas motoras y el hidrxido de magnesio y la lactulosa osmticas.
Pregunta 28.-R: 5
La enteropata pierdeprotenas no es una enfermedad, sino una
modalidad de expresin clnica en la que predominan la prdida de
protenas por la luz intestinal frente a la maldigestin o malabsorcin
proteica de otros procesos. Son causa la propia enfermedad celaca,
las infiltraciones tumorales de la pared intestinal, la linfangiectasia o
las obstrucciones al drenaje del conducto torcico, como el carcino-

CTO Medicina C/ Nez de Balboa, 115 28006 MADRID (Espaa) Tfno.: (91) 782 43 32 / Fax: (91) 782 43 27
E-mail: secretaria@ctomedicina.com; iberocto@ctomedicina.com WEB: www.ctomedicina.com; www.iberocto.com

Comentarios TEST

Seguimiento a distancia

DIGESTIVO Y CIRUGA GENERAL


Preparacin Examen de Seleccin 05/06 1 Vuelta

Seguimiento a distancia

ma de esfago o la pericarditis constrictiva. La prueba diagnstica


utilizada con ms frecuencia es el aclaramiento entre las cifras fecales
y sricas de alfa1-antitripsina.
El hipertiroidismo provoca una diarrea por hipermotilidad (respuesta correcta) y no causa una enteropata pierdeprotenas.
Pregunta 29.-R: 5
En general siempre existe un factor o antecedente personal asociado que justifica el sobrecrecimiento bacteriano. En los gastrectomizados es muy frecuente (respuesta correcta) sobre todo tipo Billroth II,
tambin en los by-pass intestinales, sndrome de intestino corto,
divertculos yeyunales, fstulas y asas ciegas que son los ejemplos ms
frecuentes.
Produce maldigestin por alteracin de las sales biliares. A veces
hay ligera atrofia intestinal, que no es responsable de los trastornos
nutricionales. Si no se resuelve la enfermedad subyacente, es frecuente la recurrencia. La prueba diagnstica de certeza es el recuento de
aerobios y anaerobios en jugo intestinal aspirado superior a 100.000
UFC; pero la dificultad de esta prueba obliga a utilizar los test del
aliento, siendo el ms rentable el de xilosa con carbono isotpico.
Pregunta 30.-R: 5
Esta pregunta hace referencia al diagnstico diferencial de la ascitis
quilosa:
1. Los traumatismos abdominales con fistulas linftico-peritoneales.
2. Los linfomas.
3. La cirrosis heptica ocasionalmente.
4. La linfangiectasia primaria (respuesta correcta). Se acompaa de
edemas en MMII, linfopenia y aclaramiento de alfa-1-antitripsina
elevado pues es una enteropata pierde protenas.

Comentarios TEST

Pregunta 31.-R: 1
La causa ms frecuente de hemorragia digestiva baja en un anciano es el sangrado por divertculos de colon, sobre todo los situados en
el hemicolon derecho. Sin embargo, suele ser autolimitado y raramente recurrente.
La angiodisplasia aparece como segunda causa, pasando a ser la
primera en caso de sangrado recurrente. Se observa con ms frecuencia si coexiste estenosis artica o insuficiencia renal y para su valoracin la colonoscopia no debe limitarse hasta ngulo esplnico como
en este caso, sino realizarse completa hasta ciego. Esta prueba es la de
mayor rentabilidad diagnstica superior incluso a la propia arteriografa
permitiendo el tratamiento con fotocoagulacin en los casos en que
el sangrado se limite a un nmero reducido de lesiones.

Pregunta 32. Hemorragia digestiva baja.

Pregunta 34.-R: 3
La presencia de aftas en colon sugiere una colitis de Crohn y en
este caso leve-moderada. Su tratamiento incluye:
1. Por afectar el colon se recomienda, aunque no es obligado, metronidazol a dosis no muy elevadas como inmunomodulador.
2. 5-aminosalicilatos, p.e. mesalazina, aunque su eficacia es menor
que en la colitis ulcerosa y frecuentemente se necesitan aadir
dosis bajas de esteroides orales o en enema.
Si hubiera sido severa los corticoides se pautan desde el inicio y a
dosis altas (ver figura).

Pregunta 32.-R: 1
Las causas ms frecuentes de hemorragia digestiva baja varan segn la edad. En los mayores de 60 aos, que es el grupo con mayor
incidencia, el orden por frecuencia es:
1. Divertculos.
2. Angiodisplasia.
3. Plipos.
4. Neoplasia.
Tericamente la prueba de eleccin es una colonoscopia completa,
pero esta prueba requiere una preparacin. Una sigmoidoscopia se
puede realizar casi de forma inmediata por lo que puede ser la primera
prueba a realizar sobretodo si el sangrado es relevante. El algoritmo
diagnstico se observa en la figura. La gastroduodenoscopia no aporta
nada de inters en una HDB y la arteriografa es til slo si el sangrado es
cuantioso.
Pregunta 33.-R: 1
La presencia de melenas es indicativo de una hemorragia digestiva,
en el 80% de los casos alta. La causa ms frecuente de hemorragia
digestiva en un paciente previamente sano es el ulcus duodenal, (respuesta correcta) en segundo lugar el ulcus gstrico, en tercero las
erosiones y gastritis de stress. En este caso coexiste anemia ferropnica
que en una mujer joven es secundaria al propio ulcus o a prdidas
menstruales. Es de inters recordar que en un anciano la anemia
ferropnica obliga a excluir la neoplasia de colon.
M exico A rgentina
C hile U ruguay

Pregunta 34. Tratamiento de un brote de enfermedad inflamatoria intestinal.

Pregunta 35.-R: 1
Los ANCA patrn perinuclear aparecen en el 70% de las colitis
ulcerosas, pero es tambin conveniente que recordemos que en el
MIR aparecen en la poliangetis microscpica, hepatitis autoinmune,
colangitis esclerosante, y el patrn citoplasmtico en la enfermedad
de Wegener.
Actualmente se han descrito en algunos casos de Crohn anticuerpos antisacharomyces (ASCA).
Son caractersticas de la colitis ulcerosa:
1. Es muy infrecuente en fumadores.
2. Afecta siempre al recto y slo en un 20% a todo el colon.

CTO Medicina C/ Nez de Balboa, 115 28006 MADRID (Espaa) Tfno.: (91) 782 43 32 / Fax: (91) 782 43 27
E-mail: secretaria@ctomedicina.com; iberocto@ctomedicina.com WEB: www.ctomedicina.com; www.iberocto.com

HM Pg. 5

DIGESTIVO Y CIRUGA GENERAL

Preparacin Examen de Seleccin 05/06 1 Vuelta

3. El sntoma clnico ms frecuente es la rectorragia, pero casi siempre


con gran emisin de moco, incluso en las remisiones.
4. Aproximadamente entre el 6 y el 10% de los que presentan colitis
ulcerosa, y no el 50%, asocian una colangitis esclerosante.
Pregunta 36.-R: 4
La descripcin endoscpica de una mucosa eritematosa y granular
en colon o recto es muy sugestiva de colitis ulcerosa, ms an si se
describieran lesiones petequiales, erosiones o lceras. No obstante, el
diagnstico definitivo no es endoscpico sino que se debe disponer de
histologa compatible aunque no sea patognomnica. La presencia de
sepsis, deshidratacin severa, o, como en este caso, seis ms deposiciones al da, supone que el episodio es severo y, por tanto, el tratamiento es con 5-aminosalicilatos ms esteroides (respuesta n 4). Los salicilatos
como nico tratamiento se emplean en las formas leves. En la colitis
ulcerosa el metronidazol slo se utiliza si se asocia megacolon txico.
Pregunta 37.-R: 4
En general, tanto la colitis ulcerosa como la enfermedad de Crohn
pueden presentar los mismos sntomas, pero son ms tpicas de colitis
ulcerosa la rectorragia y la friabilidad al estudio endoscpico (respuesta correcta) y ms propio de la enfermedad de Crohn: dolor
abdominal sobre todo en fosa ilaca derecha, prdida de peso, fiebre,
presencia de masa abdominal, fstulas y cuadros obstructivos o
pseudoobstructivos. Es conveniente recordar que aunque la enfermedad de Crohn pueda afectar a capas profundas de la pared intestinal,
es ms frecuente la presencia de perforaciones en la colitis ulcerosa,
posiblemente por su asociacin a megacolon.

Pregunta 38.-R: 1
Los esteroides son tiles para lograr la remisin. Pero tanto en la
colitis ulcerosa como en la enfermedad de Crohn, su uso como mantenimiento no garantiza una remisin prolongada (respuesta correcta
la nmero 1). Cuando se suspenden o se reduce la dosis no se modifica el riesgo de padecer un nuevo brote. Por tanto, en el mantenimiento de la enfermedad de Crohn se opta ante un primer brote leve
o moderado por no tomar ninguna medida, salvo si la afectacin en el
brote fue una colitis que mejor con salicilatos, en cuyo caso se puede optar por probar a continuar con este tratamiento como mantenimiento. En caso de brote inicial muy grave o brotes repetidos, se pasa
directamente al mantenimiento con un inmunosupresor como la
azatioprina.
En los brotes graves refractarios a dosis altas de corticoides puede
emplearse ciclosporina i.v.
En algunos casos los enfermos de Crohn tienen diarrea no por
actividad de la enfermedad sino por sus secuelas:
1. Sobrecrecimiento bacteriano.
2. Intestino corto postquirrgico.
3. Ante extensa afectacin ileal pueden no absorberse bien los cidos
biliares que al pasar a colon provocaran una diarrea secretora
cuyo tratamiento es la colestiramina.
Pregunta 39.-R: 4
La presencia de diarrea con mltiples deposiciones y deshidratacin en una enferma con colitis ulcerosa sugiere un brote severo. La
distensin abdominal no es propia del brote y sugiere un megacolon
txico, que se diagnostica en una Rx simple de abdomen con dimetro

Pregunta 46. Caractersticas ms relevantes de las poliposis familiares.

Pg. 6 HM

M exico A rgentina
C hile U ruguay

CTO Medicina C/ Nez de Balboa, 115 28006 MADRID (Espaa) Tfno.: (91) 782 43 32 / Fax: (91) 782 43 27
E-mail: secretaria@ctomedicina.com; iberocto@ctomedicina.com WEB: www.ctomedicina.com; www.iberocto.com

Comentarios TEST

Seguimiento a distancia

DIGESTIVO Y CIRUGA GENERAL


Preparacin Examen de Seleccin 05/06 1 Vuelta
luminal de colon transverso superior a 6 cm. Esta situacin es muy grave
y obliga al tratamiento propio de las formas severas ms metronidazol. Si
no responde en 48 h o se perfora, est indicada la colectoma urgente.
Pregunta 40.-R: 5
Es una lesin por alteracin de la microvasculatura arterial y, por
tanto, la arteriografa no es diagnstica y s lo es la histologa del colon;
por esto, tanto las respuestas 1 como 3 son falsas. Afecta al colon, sobre
todo al ngulo esplnico, pero respeta al recto. Las imgenes en la Rx en
huella dactilar se observan tambin en las formas agudas.
Los sntomas, generalmente diarrea, suelen remitir de forma habitual en unas pocas semanas y no suele haber recurrencias, por eso la
respuesta correcta es la 5.
Pregunta 41.-R: 1
Para el diagnstico de la enfermedad diverticular del sigma o colon
la prueba de eleccin es el enema opaco. Est contraindicado su
prctica si se sospecha una complicacin como la diverticulitis, por el
riesgo de perforacin y peritonitis clnica. El diagnstico de diverticulitis es clnico y bioqumico: anciano con fiebre, dolor en fosa ilaca
izquierda y leucocitosis. La sospecha de una diverticulitis complicada
con un absceso de pared por persistencia de la fiebre o una palpacin muy sugestiva de peritonismo, se comprueba mediante un TAC.
Esta prueba es la ms rentable para el diagnstico de masas, tumores y
abscesos del espesor de la pared intestinal.
Pregunta 42.-R: 3
La enfermedad diverticular del colon es muy frecuente, generalmente es asintomtica. En caso de producirse clnica es ms frecuente
la diverticulitis que la hemorragia; pero es tan prevalente la enfermedad que la hemorragia diverticular, sobre todo localizada en colon
derecho, es la causa ms frecuente de hemorragia digestiva baja en
mayores de 60 aos. Como no suele ocurrir sobre un divertculo
inflamado: es indolora, autolimitada y no recidiva; por eso la falsa es
la nmero 3.

Comentarios TEST

Pregunta 43.-R: 4
El punto fundamental que da mal pronstico en cuanto al potencial de malignizacin de un plipo es la presencia y grado de displasia en su interior. Lgicamente a ms plipos y de mayor tamao,
ms riesgo de neoplasia. El tipo histolgico velloso de los adenomas
tiene ms riesgo que el tubular. El hiperplsico no es una lesin
premaligna. El rea de intestino grueso que est afectada no se asocia a un mayor riesgo de transformacin neoplsica (respuesta correcta la 4).
Pregunta 44.-R: 3
Los plipos, tanto hiperplsicos como adenomatosos, suelen ser
asintomticos; pero en caso de producir sntomas suelen presentar
rectorragia o hematoquecia. Unicamente producen diarrea cuando
son de tipo velloso y adems muy indiferenciados, mltiples y de gran
tamao. La intususcepcin es menos frecuente y puede provocar
dolor e leo.
Pregunta 45.-R: 2
En poblacin sana con edad superior a los 50 aos y sin factores
de riesgo asociados se recomienda un screening con un test de hemorragias ocultas cada dos aos. Pero siempre que exista un factor de
riesgo personal (antecedente de colitis ulcerosa o como en este caso
plipos adenomatosos previos) o familiar por antecedente de neoplasia de colon, el cribado debe hacerse con colonoscopia cada ao y
medio o dos. Este plan es indefinido, pues es la nica forma de detectar precozmente un cncer de colon.
Pregunta 46.-R: 2
Las caractersticas ms relevantes de las poliposis familiares se
ilustran en la figura. Son factores de riesgo bien reconocidos para el
cncer de colon: la presencia de cualquier poliposis familiar, excepto en el sndrome de Cowden; los antecedentes familiares de
cncer de colon y los antecedentes de enfermedades como la colitis ulcerosa o el Crohn. No as el sndrome del intestino irritable (por
eso es la respuesta correcta). Otro tema bien distinto es que en
M exico A rgentina
C hile U ruguay

Seguimiento a distancia

ocasiones su cuadro clnico puede ser parecido al de un cncer de


colon incipiente y, por eso, cuando la clnica de colon irritable
aparece por primera vez en sujetos de ms de 50 aos se debe
excluir la presencia de un cncer de colon mediante la realizacin
de pruebas diagnsticas como la colonoscopia o el enema opaco.
Pregunta 47.-R: 4
La colitis ulcerosa y la enfermedad de Crohn son enfermedades en
las que aumenta el riesgo de padecer cncer de colon. Este aumento
es ms notorio en la colitis ulcerosa.
El riesgo depende fundamentalmente del grado de actividad de la
enfermedad y de la duracin de la misma. Aumenta a partir de los 10
aos en enfermos con formas crnicamente activas o con multibrotes;
por eso si la enfermedad debuta en edad juvenil hay un mayor potencial de padecer ms brotes a lo largo de la vida y por tanto ms riesgo
de desarrollar un carcinoma.
Otro factor adicional es la extensin de la enfermedad, por ejemplo: no hay ms riesgo de cancer cuando nicamente present
proctitis.
En los casos en que su severidad as lo indique y a partir de los 10
aos de evolucin, se recomiendan colonoscopias cada dos aos
para tomar biopsias y valorar la presencia de neoplasias precoces. Si
se demuestran zonas displsicas se asume tal riesgo de neoplasia y se
recomienda la colectoma; pero sta no se indica por el mero hecho
de llevar 15 aos de evolucin (por eso la respuesta incorrecta es la 4).
Pregunta 48.-R: 1
La causa ms frecuente de cancer de colon hereditario, a edad
inferior a los 40 aos, es el carcinoma colorrectal hereditario no asociado a plipos (respuesta correcta la nmero 1). Esta entidad es cinco
veces ms frecuente que la poliposis adenomatosa familiar del colon.
Pregunta 49.-R: 1
Un paciente con cncer de colon necesita para su seguimiento:
1. Colonoscopias peridicas.
2. Ecografa abdominal para observar posibles lesiones metastsicas
hepticas.
3. La determinacin del antgeno carcinoembrionario (CEA). Su elevacin significa recidiva tumoral (respuesta 1). El test de hemorragias ocultas no est indicado por su baja rentabilidad para el seguimiento de las neoplasias de colon.
Las segundas neoplasias pueden surgir en otra localizacin en
un enfermo con cncer de colon despus de la ciruga. Por ejemplo
en un enfermo con colectoma puede aparecer una segunda neoplasia en recto; pero esta situacin, adems de infrecuente, no tiene
por qu cursar necesariamente con CEA elevado, puesto que este
segundo tumor puede encontrarse en un estado poco avanzado.
Pregunta 50.-R: 5
El que los anticuerpos antigliadina sean negativos hace muy improbable que se trate de una celaca o su variante espre colgeno.
En el sndrome de intestino irritable, tanto la visin colonoscpica
como las biopsias son normales. El hipertiroidismo da una diarrea
motora.
La respuesta correcta es la colitis linfoctica. Esta entidad se asocia
con cierta frecuencia a la celiaca. En general o no se aprecian alteraciones endoscpicas en el colon o son leves e inespecficas. El diagnstico se establece al obtener muestras por biopsia para estudio
histologico (hay que resaltar que en un protocolo de diarrea se toman
biopsias incluso con un aspecto endoscpico normal). Se aprecia un
infiltrado linfoplasmocitario y en ocasiones una banda de colgeno
en cuyo caso recibe el nombre de colitis colgena. El tipo de diarrea
que produce es secretora.
Pregunta 51.-R: 4
En un paciente de edad media la presencia de mltiples plipos
adenomatosos y un cncer en sigma o colon sugiere fuertemente que
se trate de una poliposis adenomatosa familiar del colon. Esta entidad,
dada su herencia autosmica dominante y alto grado de penetrancia,
obliga a los familiares a someterse a un cribado del cncer de colon.
Este debe iniciarse antes de los 20 aos y si se demuestran plipos

CTO Medicina C/ Nez de Balboa, 115 28006 MADRID (Espaa) Tfno.: (91) 782 43 32 / Fax: (91) 782 43 27
E-mail: secretaria@ctomedicina.com; iberocto@ctomedicina.com WEB: www.ctomedicina.com; www.iberocto.com

HM Pg. 7

Seguimiento a distancia

DIGESTIVO Y CIRUGA GENERAL

Preparacin Examen de Seleccin 05/06 1 Vuelta

extirpar el colon. Si con edad superior a 40 aos no se han demostrado plipos, es que ese familiar no va a padecer la enfermedad y por
tanto no tiene sentido someterle a colonoscopias de seguimiento (respuesta correcta es la nmero 4).
Pregunta 52.-R: 5
El aumento aislado de bilirrubina indirecta, o no conjugada, es
generalmente leve o moderado, excepto en el recin nacido con
ictericia fisiolgica que puede llegar a producir Kernicterus. Causas
habituales son las anemias hemolticas y el sndrome de Gilbert. En
ambos casos la bilirrubina no suele exceder de 4-5 mg/dl. Tambin
se observa en una forma o variante del Gilbert, como es el CriglerNajar, donde alcanza cifras ms importantes y puede ser incluso
mortal en nios con el tipo I. Otras causas son el hipotiroidismo, la
lactancia materna o los estados de hipoalbuminemia severa. Todas
estas entidades no presentan bilirrubinuria. El sndrome de DubinJohnson se caracteriza porque la bilirrubina ya conjugada no puede pasar a los canalculos biliares y, por tanto, la bilirrubinemia es
de tipo conjugado o directo, por eso es la respuesta correcta. Muy
parecido a este cuadro es el sndrome de Rotor, por tanto tambin
con elevacin predominante de la bilirrubina conjugada. Estos dos
ltimos diagnsticos s se acompaan de bilirrubinuria.

Ambas opciones son vlidas, pero es ms eficiente, pues ahorra


costes, la paracentesis.
3. Si la ascitis es a tensin, la paracentesis evacuadora con reposicin
de albmina es el tratamiento de eleccin (respuesta correcta).
4. En los casos refractarios, surge la indicacin de transplante heptico. Se ha comprobado que en esta situacin la colocacin de TIPS
resuelve la ascitis refractaria en el 30% de los casos.

Pregunta 53.-R: 5
Siempre que hay colestasis, un Dubin-Johnson o un sndrome de
Rotor, predomina la bilirrubina conjugada y se encuentra bilirrubina
en orina, puesto que sta s puede filtrarse por el rin. El sndrome de
Gilbert (respuesta correcta) causa aumento de bilirrubina indirecta y,
por tanto, no cursa con hiperbilirrubinuria.

Pregunta 56. Manejo de la ascitis.

Pregunta 57.-R: 3
Para determinar la presencia de hipertensin portal se debe constatar:
1. Que las presiones estn aumentadas en vena porta, determinadas
por mtodo cruento, o
2. De forma indirecta, visualizando varices esofgicas o gstricas en la
endoscopia.

Pregunta 55.-R: 5
La intoxicacin por paracetamol produce un fallo heptico agudo por un efecto txico directo por consumir los niveles de glutatin
del hepatocito. Se produce con dosis superiores a los 8 gramos al
da, excepto en alcohlicos, en los que incluso se puede producir
con dosis dentro del rango teraputico habitual. Tiene buen pronstico si se consigue administrar precozmente N-acetilcistena i.v.
Las transaminasas estn muy elevadas y se acompaa de alteraciones en la coagulacin y si hay fracaso heptico establecido, de
encefalopata (por eso la 5 es la respuesta correcta).

Otro tema bien distinto es evaluar si el origen de un lquido asctico es atribuible o no a la hipertensin portal. Para ello se utiliza el
gradiente, o resta, entre la albmina en suero y la albmina en
ascitis. Cuando esta resta da un resultado superior a 1,1 gr/dl se
presume que la ascitis es por hipertensin portal, como sucede en
cualquier tipo de cirrosis, BuddChiari, enfermedad venooclusiva
heptica, esquistosomiasis o trombosis de la porta. Si el gradiente es
menor de 1,1 sugiere otros diagnsticos distintos al de hipertensin
portal, como la tuberculosis peritoneal (es la respuesta correcta, ya
que el gradiente es de 3,1) o la carcinomatosis peritoneal.

Pregunta 56.-R: 3
El tratamiento de la ascitis por descompensacin hidrpica de un
cirrtico es (ver figura de la pgina siguiente):
1. En los casos leves-moderados: restriccin diettica de sal y espironolactona con dosis progresivas segn resultado si no fuera suficiente la de 100 mg/da. En ocasiones puede ser necesario aadir
un segundo diurtico.
2. En los casos severos se puede optar por la combinacin de espironolactona y otro diurtico como furosemida o torasemida, o bien
por la paracentesis evacuadora con reposicin de albmina i.v.

Pregunta 58.-R: 1
La presencia de clulas tumorales malignas en el lquido asctico
sucede porque el peritoneo est infiltrado por un tumor. En general
suele ser metastsico fundamentalmente de colon o estmago (respuesta correcta 1) y es independiente el que se asocie o no a metstasis hepticas.
Esta situacin confiere muy mal pronstico, ya que la sobrevida de
una neoplasia con carcinomatosis peritoneal oscila, como mediana,
en los 6 meses. Si hubiera sido quilosa se nos describira un aumento
notable de los triglicridos en el lquido.

Pg. 8 HM

M exico A rgentina
C hile U ruguay

CTO Medicina C/ Nez de Balboa, 115 28006 MADRID (Espaa) Tfno.: (91) 782 43 32 / Fax: (91) 782 43 27
E-mail: secretaria@ctomedicina.com; iberocto@ctomedicina.com WEB: www.ctomedicina.com; www.iberocto.com

Comentarios TEST

Pregunta 54.-R: 4
En un sndrome colesttico la ecografa (respuesta correcta) es la
primera prueba que debe realizarse. Si se demuestra dilatacin de la
va biliar principal supone probablemente que se trate de una colestasis extraheptica y el siguiente paso diagnstico es un TAC. En caso
de no obtener el diagnstico, se realiza una colangiografa retrgrada
endoscpica. Esta tcnica es cruenta: puede producir un 5% de pancreatitis y un 1% de colangitis, por eso actualmente estn cobrando
importancia otras tcnicas:
1. La ecografa endoscpica de la va biliar, situando el endoscopio
en duodeno se realiza un barrido de pncreas y va biliar. Tiene
una alta rentabilidad para las enfermedades del coldoco, sobre
todo la litiasis.
2. La colangiografa por resonancia magntica, que es incruenta y no
precisa contraste.
3. La colangiografa i.v. y la oral no tienen utilidad, salvo en los casos
previos a la litotricia biliar para evaluar si la vescula es funcionante.

DIGESTIVO Y CIRUGA GENERAL


Preparacin Examen de Seleccin 05/06 1 Vuelta
Pregunta 59.-R: 2
Se produce por la colonizacin y posterior infeccin del peritoneo por microorganismos propios de la flora intestinal del intestino
delgado: por eso se produce en primer lugar por gramnegativos, como
E.coli, y en segundo lugar por cocos gram positivos, incluido el neumococo.
La clnica dolorosa es poco relevante. Es muy infrecuente el peritonismo, por eso la respuesta a sealar es la nmero 2. A diferencia
de las secundarias la presencia de anaerobios es excepcional. El
tratamiento es mdico, con cefalosporinas de tercera generacin.
La prueba diagnstica de certeza es el cultivo del lquido asctico,
pero, dada la gravedad de la enfermedad, se precisa iniciar un tratamiento emprico y, para ello, es suficiente con demostrar un recuento
de neutrfilos superior a 250/mm3 en lquido asctico.
Pregunta 60.-R: 2
Las indicaciones de la profilaxis antibitica con quinolonas orales
en la peritonitis bacteriana espontnea de un cirrtico son:
1. Sin antecedentes de haber padecido antes una peritonitis: si se
presenta una hemorragia digestiva (respuesta correcta) o las protenas en lquido asctico son inferiores a 1 gr/dl.
2. Como profilaxis secundaria despus de un episodio de peritonitis.
Se mantiene la profilaxis con quinolonas de forma indefinida hasta
que el paciente deje de tener ascitis.

Comentarios TEST

Pregunta 61.-R: 5
El sndrome hepatorrenal se produce como una forma de evolucin terminal de una hepatopata crnica avanzada. Pero tambin se
reconocen factores precipitantes del sndrome hepatorrenal: la peritonitis bacteriana espontnea, la hepatitis alcohlica aguda y cada vez
con menor frecuencia el uso intempestivo de diurticos.
Se trata de una vasoconstriccin funcional irreversible, que afecta
bsicamente a las arteriolas de filtracin, pero en escasa cuanta a las
arterias de las cuales se obtiene la perfusin de la nefrona, por lo cual
sta no resulta daada desde el punto de vista morfolgico y as se
entiende que exista un fallo en la filtracin renal, y, por tanto, insuficiencia renal progresiva y severa con un rin histolgicamente normal (respuesta correcta la 5).
Pregunta 62.-R: 5
El tratamiento de eleccin del sangrado por varices esofgicas es la
combinacin de un tratamiento endoscpico como la esclerosis o, si
es posible, la ligadura endoscpica de las varices, ms el farmacolgico con somatostatina o un anlogo de la vasopresina. Si este tratamiento no es eficaz y contina con sangrado, se indica la colocacin
de un TIPS, excepto en el estado A de Child, en donde se indica la
ciruga urgente derivativa. Si esta recidiva es severa y no permite esperar a estos procedimientos, hay que colocar una sonda de Sengstaken
hasta que se pueda realizar el tratamiento intervencionista.
Se deben transfundir hemates si hay sndrome anmico, prescribir
quinolonas orales para la profilaxis de la peritonitis y lactulosa para la
profilaxis de la encefalopata heptica.
Est contraindicado el empleo de diurticos o subir las dosis, puesto que provocaran un mayor deterioro hemodinmico (respuesta
correcta).
Pregunta 63.-R: 4
La profilaxis primaria de la hemorragia digestiva por varices debe
hacerse con propanolol o nadolol. Ha demostrado reducir el nmero de primeros episodios de sangrado y la mortalidad atribuible
al mismo. Se debe iniciar siempre que se observen varices y que stas
sean de riesgo, es decir, grandes (grado III IV). Si no se observan,
como ocurre en este caso, o son pequeas, nicamente se debe
someter a vigilancia endoscpica peridica para ver cmo evolucionan las varices y por tanto no se precisa un tratamiento farmacolgico.
Pregunta 64.-R: 4
La infeccin por el virus de la hepatitis C sucede por va parenteral
y por eso se observa en drogadictos, pero cada vez con mayor frecuencia hay casos en que no se demuestra un antecedente que perM exico A rgentina
C hile U ruguay

Seguimiento a distancia

mita justificar la infeccin, debi ser, pues, inaparente. Por tanto es


una respuesta falsa que la mayora tengan historia de transfusin, esto
s ocurri en los primeros aos de descripcin de la enfermedad, pero
no en la actualidad.
Se ha calculado que el 80% de los casos evolucionan a la cronicidad. El RNA es el mejor marcador para el diagnstico. Inicialmente se
hace un cribado con los anticuerpos anti-VHC y, si son positivos, la
infeccin se confirma solicitando amplificacin genmica viral, que
demuestra si hay positividad para el RNA viral.
Pregunta 65.-R: 3
Para diagnosticar una hepatitis aguda se precisa, tanto en los casos
que tengan clnica como en los asintomticos, demostrar un incremento notable y agudo de las transaminasas. Una vez que tenemos el
diagnstico probable clnico-bioqumico o slo bioqumico es cuando se solicitan los marcadores virales y, en concreto, la serologa IgM
es la que va a ser positiva en los casos agudos (respuesta correcta).
En la infeccin aguda por VHB los 5 marcadores de esta pregunta
son positivos, pero el que es especfico de hepatitis aguda es el anticuerpo IgM.
Pregunta 66.-R: 1
La infeccin por el virus de la hepatitis C afecta tambin a rganos
extrahepticos, sobre todo a los ganglios linfticos, generando riesgo
de linfomas y de enfermedades de naturaleza inmune como la aplasia
medular, eritroblastopenia, glomerulonefritis, liquen plano, sndrome
seco, y, sobre todo la ms frecuente, la crioglobulinemia mixta esencial. Esta entidad es una vasculitis que cursa con crioglobulinas positivas en sangre, polineuropata, afectacin renal y prpura. La etiologa
es, prcticamente siempre, debida a la infeccin por el virus de la
hepatitis C.
Pregunta 67.-R: 3
En general la hepatitis por virus B tiene una tasa de cronificacin
baja que oscila alrededor del 5% en adultos. La va de adquisicin no
influye en la cronicidad, salvo en los nios que lo suelen adquirir en
el canal del parto y tienen un riesgo mucho ms elevado de padecer
una forma crnica, llegar a cirrosis o quedar como portadores. Por
eso la respuesta correcta es la 3.
Pregunta 68.-R: 3
Los pacientes con VHB y hepatitis crnica o portadores crnicos,
cuando sufren un episodio de hepatitis aguda debe pensarse en las
siguientes posibilidades:
1. Que est sucediendo el proceso de seroconversin: la prdida de
la capacidad replicativa del virus. Esta es la causa ms frecuente de
elevacin de las transaminasas; en general es poco intensa y no
suele cursar con complicaciones, excepto ligera ictericia. Se diagnostica porque el DNA viral se negativiza.
2. Sobreinfeccin por un virus de la hepatitis A. Esta situacin es de
alto riesgo para desarrollar un fracaso heptico fulminante. Se recomienda que estos enfermos estn vacunados para el virus A, si no
han padecido previamente la infeccin. Esta situacin se diagnostica porque la IgM anti-VHA es positiva.
3. Sobreinfeccin por el virus Delta. Esta situacin es hoy infrecuente,
slo se observa en hemoflicos y ADVP. Conlleva un riesgo elevado
de hepatitis fulminante y, tambin, favorece que el virus Delta quede
en situacin de cronicidad. Se diagnostica porque se observan, como
en el presente caso, marcadores de infeccin aguda por virus D. El
ms importante y precoz de ellos es el antgeno Delta, que es positivo. En ningn caso puede ser una coinfeccin, ya nos lo aclara el
enunciado, en el que se describe a un paciente diagnosticado de
infeccin crnica por el VHB y la coinfeccin es la situacin de dos
infecciones agudas simultneas B y D (ver figuras).
Pregunta 69.-R: 3
La positividad IgM para el virus A en una hepatitis aguda ofrece el
diagnstico de seguridad de hepatitis aguda por VHA. Por eso la respuesta correcta es la nmero 3. En relacin con la serologa del virus
B, es probable que el enfermo se encuentre en una fase previa a la
aparicin de la respuesta inmune: todava no la tiene porque es anti-

CTO Medicina C/ Nez de Balboa, 115 28006 MADRID (Espaa) Tfno.: (91) 782 43 32 / Fax: (91) 782 43 27
E-mail: secretaria@ctomedicina.com; iberocto@ctomedicina.com WEB: www.ctomedicina.com; www.iberocto.com

HM Pg. 9

Seguimiento a distancia

DIGESTIVO Y CIRUGA GENERAL

Preparacin Examen de Seleccin 05/06 1 Vuelta

S negativo. La asociacin de: anti-E positivo, anti-Core positivo, aunque no explican si es IgG o IgM, ms un antgeno de superficie negativo sugiere que el paciente est atravesando el perodo ventana previo a la resolucin virolgica definitiva.

gativo y anti-e positivo. Esto es lo contrario de lo que sucede habitualmente con la cepa salvaje, que cuando replica es: DNA virus B positivo, pero con antgeno e positivo y anti-e negativo.
Pregunta 73.-R: 2
Lamivudina es un antiviral muy eficaz para el tratamiento del VHB.
Sus indicaciones son:
1. En la cepa mutante precore (por eso la respuesta correcta es la 2).
2. En VIH positivos.
3. En intolerancia o contraindicacin al Interfern.
Tiene como ventajas que la administracin es oral y tiene escasos
efectos adversos frente a Interfern; sin embargo se debe prolongar su
uso alrededor de un ao, a diferencia del Interfern, que oscila alrededor de los cuatro meses.
Lamivudina es una alternativa a Interfern, pero no est indicado
su uso combinado.

Pregunta 68. Coinfeccin y sobreinfeccin por hepatitis D.

Pregunta 70.-R: 2
En general un ttulo de anti S menor de 10 es muy bajo y, aunque
no excluye que el paciente tenga una respuesta inmune porque no es
un inmunodeprimido, lo adecuado es plantearlo como un fallo parcial en la respuesta y administrar inmunoglobulina especfica y una
dosis de recuerdo de la vacuna, siendo innecesaria la vacunacin
completa. Es ms seguro esta pauta que administrar nicamente una
dosis de recuerdo.
Pregunta 71.-R: 4
El colapso multilobular es propio de las situaciones de fracaso
heptico fulminante. La presencia de un infiltrado mononuclear slo
nos permitir establecer el diagnstico de hepatitis persistente y no el
de crnica activa. Es muy importante, porque en las situaciones de
hepatitis persistente no est indicado el tratamiento antiviral, pues se
da un margen a la evolucin espontnea a la curacin. Para establecer el diagnstico de hepatitis crnica activa se necesita, como mnimo, demostrar cierto grado de necrosis (por eso es la respuesta correcta), que puede llegar a establecer puentes de tejido colgeno en las
formas ms avanzadas. La presencia de fibrosis masiva y ndulos de
regeneracin superan el diagnstico de hepatitis crnica: ya se trata
de una cirrosis heptica.
Pregunta 72.-R: 3
Se trata de una infeccin por el VHB, que indudablemente est en
fase replicativa, ya que el DNA, marcador especfico de esta situacin,
es positivo, por lo que la respuesta 1 es falsa. No hay una situacin de
hepatitis aguda y el anti-Core es de tipo IgG nicamente. La respuesta
correcta es la nmero 3: se trata de una infeccin por la mutante
precore. Es fcil detectarla en el enunciado de la pregunta, porque es
un paciente con replicacin, DNA positivo, pero con antgeno e nePg. 10 HM

M exico A rgentina
C hile U ruguay

Pregunta 75.-R: 5
La enferma tiene, sin duda, una infeccin por virus C, puesto que
es anti-VHC positivo y el RNA es tambin positivo. Pero con normalidad de las transaminasas no se indica la biopsia heptica, porque a
pesar de que est documentada la infeccin, no se va a prescribir
tratamiento. Slo se realiza biopsia a los pacientes con hipertransaminasemia prolongada. Slo reciben tratamiento los pacientes en los
que la biopsia demuestre hepatitis crnica activa, no se trata de entrada a los que tienen hepatitis persistente en la histologa. Por eso este
paciente precisa nicamente seguimiento evolutivo, que es la respuesta correcta.
El tratamiento, en caso de necesitarse, es actualmente la combinacin de Interfern y Ribavirina, que consigue eficacia en prcticamente el 60% de los sujetos. Es un factor predictivo de eficacia teraputica comprobar que a las 12 semanas de tratamiento el RNA viral
se ha negativizado. Se considera xito teraputico cuando se cumple
el concepto de respuesta teraputica sostenida: negativizacin persistente del RNA seis meses despus de concluir la terapia.
Pregunta 76.-R: 1
En el tratamiento de la hepatitis crnica por VHB se utiliza Interfern alfa, no gamma (respuesta correcta la 1). Este ltimo se usa cuando
hay enfermedades con deficiencia del sistema inmunolgico, como
la enfermedad granulomatosa crnica. En relacin con el tratamiento
con Interfern en la infeccin crnica por el VHB, se han objetivado
factores de buen pronstico:
1. El de mayor impacto predictivo es el de niveles bajos de DNA.
2. Niveles entre 100 y 300 de transaminasas predicen una mejor
respuesta que cuando oscilan entre 50 y 100.
3. La respuesta es superior en mujeres e inferior en VIH positivos.
Pregunta 77.-R: 1
Actualmente las indicaciones de trasplante heptico han aumentado y la edad slo es un lmite y no absoluto por encima de los 70
aos. Son factores de mal pronstico para la recidiva: el genotipo viral
cuando es I, sobre todo si es b. Es muy probable que el hgado se
reinfecte por el VHC, pero la historia natural de esta infeccin, si no
existe otro factor asociado como el alcohol, suele ser la de una evolucin lenta a la cirrosis en un perodo de tiempo de alrededor de 20
aos y, adems, si fuese necesario, se administra tratamiento antiviral.

CTO Medicina C/ Nez de Balboa, 115 28006 MADRID (Espaa) Tfno.: (91) 782 43 32 / Fax: (91) 782 43 27
E-mail: secretaria@ctomedicina.com; iberocto@ctomedicina.com WEB: www.ctomedicina.com; www.iberocto.com

Comentarios TEST

Pregunta 74.-R: 2
El objetivo primordial es lograr que deje de replicar el virus B, es
decir que se haga DNA negativo. Este proceso tambin se conoce
como seroconversin anti-e, ya que los pacientes pierden el antgeno
e y desarrollan anticuerpos anti-e. El fundamento de este objetivo es
que, una vez pasado a no replicante, la progresin histolgica de la
hepatitis queda detenida y, por tanto, se reduce casi por completo el
riesgo de desarrollar cirrosis heptica. Este objetivo se consigue entre
el 50-60% de los casos, pero slo en un 15% se consigue negativizar el
antgeno de superficie y conseguir la inmunidad, siendo anti-s positivo. La mayora de los pacientes no consiguen eliminar por completo
el virus y, aunque no sean replicantes, quedan como portadores.

DIGESTIVO Y CIRUGA GENERAL


Preparacin Examen de Seleccin 05/06 1 Vuelta

Seguimiento a distancia

Pregunta 82. Diferencias entre CBP y CEP.

Epidemiologa

CIRROSIS BIL IAR PRIMARIA

COL ANGITIS ESCL EROSANTE PRIMARIA

M ujer.

Va rn.

Patogenia

- Alt. inm unida d hum ora l y celula r.


- Ac. a ntim itocondria les M 2.

- Desconocida .
- Asocia cin HLA B8.

Anatoma
Patolgica

- Destruccin y fibrosis de los conductos bilia res intra y extra hep ticos.

I Cola ngitis destructiva .


II Gra nulom a s e infla m a cin periporta l.
III Fibrosis septa l sin gra nulom a s.
IV Cirrosis.

- Lo m s frecuente ASTENIA Y PRURITO.


- Hepa toesplenom ega lia .
- Lesiones por ra sca do.

- Asintom ticos dura nte m uchos a os.


- Luego prurito, a stenia , ictericia , prdida de peso.

- Lo m s frecuente Sd. SECO.


- Escleroderm ia , hipotiroidism o.
- ATR.

- EII sobre todo COLITIS ULCEROSA.


- Fibrosis retroperitonea l o m edia stnica .

L aboratorio

- Aum ento de FOSFATASA ALCALINA y resto de


enzim a s de colesta sis con tra nsa m ina sa s norm a les.
- Aum enta el colesterol e IgM .

- Colesta sis crnica .


- 1/3 hiperga m m a globulinem ia .
- 1/2 a um enta IgM , 65 % p-ANCA+.

Diagnstico

- Sospecha clnica y la bora torio.


- Confirm a cin: BIOPSIA.

- CPRE: DE ELECCIN.
- Estenosis y dila ta cin de los conductos intra y extra hep ticos.
Sintom tico: el m ism o.

Tratamiento

Sintom tico:
- Colestira m ina .
- Vita m ina s liposolubles.
Especfico:
- Ac. ursodesoxiclico.
- Colchicina , m etotrexa te.
- Tra spla nte.

Clnica
Enfermedades
asociadas

Pronstico

- M ejor correla cin con el nivel de


BILIRRUBINA.

Comentarios TEST

Por eso la respuesta correcta es la nmero 1.


La presencia de crioglobulinas en sangre no establece ni excluye
indicaciones de trasplante. nicamente si se asocia a clnica de crioglobulinemia est indicado el tratamiento secuencial con corticoides
y, posteriormente, Interfern y Ribavirina.
Pregunta 78.-R: 3
La presencia de ANA y ANCA en un enfermo con hipertransaminasemia persistente es muy sugestivo de hepatitis autoinmune. Esta situacin obliga a excluir una infeccin viral y a practicar una biopsia
heptica que demuestre una hepatitis, aunque no hay criterios
histolgicos patognomnicos, por eso la respuesta 3 es la verdadera.
Se trata de una hepatitis crnica autoinmnune tipo 1, ya que esta
presenta habitualmente ANA y/o antimsculo liso positivos. La tipo 2
se caracteriza por presentar estos anticuerpos negativos y ser anti-LKM
positivo. Los ANCA, variedad perinuclear, pueden ser positivos en
ambos tipos de hepatitis autoinmune. El tratamiento es con corticoides y casi todos requieren un largo mantenimiento con azatioprina
una vez conseguida la remisin. El tacrolimus empleado en el trasplante heptico se est utilizando en algunos casos de hepatitis autoinmune a nivel experimental.
Pregunta 79.-R: 1
El tratamiento de eleccin de un hepatocarcinoma es la extirpacin
quirrgica siempre que se pueda. Peores resultados ofrece el trasplante
heptico, pero pasa a ser de eleccin como en este caso si el paciente
tiene un grado funcional heptico avanzado, como es el grado C de
Child, ya que en esta situacin no tolerara la ciruga, pues le dejara en
una situacin de insuficiencia heptica grave. En las situaciones de
lesiones grandes y mltiples no se puede realizar ciruga y se intenta
como tratamiento paliativo para mejorar la supervivencia la
alcoholizacin percutnea. Cuando hay metstasis o trombosis de la
porta extraheptica nicamente se utiliza el tratamiento sintomtico.
M exico A rgentina
C hile U ruguay

Especfico:
- M etotrexa te.
- Tra spla nte
- 10 a os de supervivencia tra s dia gnstico.
- Aum enta la incidencia de COLANGIOCARCINOM A.

Pregunta 80.-R: 5
Presenta elevacin predominante de la fosfatasa alcalina, suponemos que de origen heptico, por lo que presenta una colestasis. Como
la ecografa abdominal fue normal, no demostrndose por tanto dilatacin de la va biliar, es que la colestasis es intraheptica. En este
contexto los anticuerpos antimitocondriales positivos sugieren fuertemente el diagnstico de cirrosis biliar primaria. A continuacin estara
indicado una biopsia heptica para confirmarlo.
El tratamiento es con cido ursodesoxiclico, que mejora el pronstico de la enfermedad y apenas tiene efectos adversos por lo que
es muy bien tolerado. Los esteroides no tienen utilidad y, en las fases
avanzadas de la enfermedad, cuando ya se ha desarrollado hipertensin portal est indicado el trasplante heptico.
Pregunta 81.-R: 5
En la valoracin pronstica de la cirrosis biliar primaria es fundamental conocer el nivel de bilirrubina, porque cuanto mayor sea sta,
es decir ms colestasis, se presume mayor territorio biliar afecto. Los
anticuerpos antimitocondriales son tiles, sobre todo, porque sugieren el diagnstico. Igual sucede con el aumento policlonal de IgM.
Las hormonas tiroideas se solicitan porque es muy frecuente la
asociacin a tiroiditis y, sobre todo, el desarrollo de hipotiroidismo en
estos enfermos. Los niveles de colesterol pueden ser tiles, puesto que
estn elevados en los enfermos con colestasis, no as los niveles de
triglicridos, por eso es la respuesta que hay que sealar.
Pregunta 82.-R: 3
Esta enfermedad es infrecuente. Casi siempre suele ser ANCA patrn perinuclear positivo. Con mucha frecuencia existe el antecedente previo de colitis ulcerosa o se asocia a ella a lo largo de su evolucin. Esta asociacin es independiente y por lo tanto el desarrollo de
la colitis ulcerosa no influye en el desarrollo y complicaciones de la
colangitis esclerosante. Es ms frecuente en varones, se asocia al HLA-

CTO Medicina C/ Nez de Balboa, 115 28006 MADRID (Espaa) Tfno.: (91) 782 43 32 / Fax: (91) 782 43 27
E-mail: secretaria@ctomedicina.com; iberocto@ctomedicina.com WEB: www.ctomedicina.com; www.iberocto.com

HM Pg. 11

DIGESTIVO Y CIRUGA GENERAL

Preparacin Examen de Seleccin 05/06 1 Vuelta

Pregunta 91. Actitud diagnstica y teraputica en la pancreatitis aguda.

B8 y su diagnstico se establece mostrando en la colangiografa retrgrada endoscpica un patrn de estenosis irregulares, con dilataciones postestenticas por todo el rbol biliar (ver tabla en pgina siguiente). Adems la biopsia heptica, aunque no ofrece el diagnstico
es de inters, pues permite conocer si se ha establecido ya una cirrosis
biliar secundaria, con lo que tiene de valor pronstico este dato.
Pregunta 83.-R: 2
En el screening de los familiares de un paciente con hemocromatosis se valora el ndice de saturacin de transferrina, que se considera
patolgico cuando es superior al 45% y la ferritina. El primero de ellos
es el ms precoz en alterarse. Si estos marcadores bioqumicos estn
Pg. 12 HM

M exico A rgentina
C hile U ruguay

alterados, el siguiente paso es evaluar las mutaciones del gen HFE y, si


se encuentra homozigoto o heterozigoto se establece el diagnstico
de hemocromatosis sin necesidad de una biopsia heptica y se indican a continuacin las flebotomas peridicas.
La biopsia heptica es til para el diagnstico de aquellos casos
con sospecha clnica o bioqumica y estudio gentico negativo. Adems tambin se realiza en los casos con elevacin persistente de las
transaminasas para evaluar si hay fibrosis o cirrosis heptica.
Pregunta 84.-R: 1
El gen anmalo est en el cromosoma 6. La herencia es autosmica recesiva.

CTO Medicina C/ Nez de Balboa, 115 28006 MADRID (Espaa) Tfno.: (91) 782 43 32 / Fax: (91) 782 43 27
E-mail: secretaria@ctomedicina.com; iberocto@ctomedicina.com WEB: www.ctomedicina.com; www.iberocto.com

Comentarios TEST

Seguimiento a distancia

DIGESTIVO Y CIRUGA GENERAL


Preparacin Examen de Seleccin 05/06 1 Vuelta
Entre el cuadro clnico destacan: cirrosis y sus complicaciones;
miocardiopata; pigmentacin bronceada de la piel; diabetes mellitus que una vez establecida no mejora con las flebotomas; insuficiencia exocrina del pncreas; artropata generalmente con condrocalcinosis que no mejora con el tratamiento e hipogonadismo, que puede
ser el cuadro de presentacin en jvenes por eso es la respuesta correcta y que se debe no a una lesin genital sino hipofisaria.
Pregunta 85.-R: 2
La macroesteatosis heptica es una situacin en la que predomina
el acmulo de triglicridos en el interior del hepatocito, con lo que
stos adquieren un aspecto hinchado y el hgado ecogrficamente
presenta una ecogenicidad brillante. Suele cursar con un ligero
aumento de GOT o sin ninguna alteracin. Suele ser reversible, aunque recientemente se estn describiendo algunos casos de cirrosis
idioptica atribuibles a la evolucin de esta entidad.
Las causas ms frecuentes son la obesidad, diabetes mellitus tipo 2,
el alcoholismo y el uso de corticoides. No debemos confundir esta
entidad con la esteatohepatitis microvesicular, que tambin se puede
producir en el alcohol y que se observa en el sndrome de Reye o en
el consumo de cido valproico. Este cuadro es ms grave y cursa
como una hepatitis aguda severa.
Pregunta 86.-R: 4
Los clculos de bilirrubina se producen porque pasa a la va biliar
una cantidad de bilirrubina no conjugada superior a la tolerable. Esto
sucede cuando se saturan sus mecanismos por:
1) Hiperaporte, como ocurre en los estados hemolticos crnicos,
respuesta correcta, o en el sndrome de Gilbert. Los clculos son
negros.
2) Lesiones de la va biliar, como en las colangitis crnicas o en las
cirrosis, siendo en esta ocasin clculos de bilirrubina marrones.

Comentarios TEST

Los clculos de colesterol se producen porque se reduce la absorcin de sales biliares, como sucede en la enfermedad de Crohn, o
aumenta la excrecin heptica de colesterol, o hay una situacin de
hipomotilidad biliar como sucede en la obesidad y en la gestacin.
Pregunta 87.-R: 3
La bilis se vuelve ms litognica despus de resecar el leon o por
una enfermedad ileal extensa porque no se reabsorben las sales biliares y como consecuencia aumenta la proporcin en la va biliar del
componente de colesterol sobre el de cidos biliares. Esto provoca
que el colesterol cristalice. En una bilis con cristales de colesterol, ante
cualquier situacin de estasis en la vescula, se formarn clculos por
aposicin de calcio sobre los cristales de colesterol. Por eso en la
enfermedad de Crohn los clculos son de colesterol.
Pregunta 88.-R: 2
El diagnstico de pancreatitis aguda se establece en un enfermo
con dolor abdominal agudo por la elevacin de la amilasa o la lipasa
por encima de tres veces el lmite superior de la normalidad. En ocasiones se sospecha el cuadro, pero las enzimas no son diagnsticas y,
entonces, se requiere la realizacin de un TAC, que observando las
alteraciones radiolgicas inflamatorias del pncreas, permite establecer el diagnstico de pancreatitis aguda.
El inters de la ecografa en el manejo de la pancreatitis aguda se
dirige ms a evaluar su etiologa. Si se demuestra litiasis o microlitiasis
biliar se establece en la ecografa que el diagnstico etiolgico de la
pancreatitis aguda es biliar. Si fuese necesario evaluar las complicaciones de una pancreatitis o su pronstico, el TAC ofrece ms informacin que la ecografa.
Pregunta 89.-R: 2
La litiasis biliar es la causa ms frecuente de pancreatitis aguda,
representando alrededor del 60%. El alcoholismo es la segunda causa
oscilando entre el 30 y 40% de los casos dependiendo de las reas
geogrficas. En tercer lugar con menor frecuencia est involucrada la
microlitiasis y por ltimo las causas idiopticas u otras inhabituales
entre las que se incluyen: el esfnter de Oddi hipertensivo, la fibrosis
qustica y frmacos como pentamidina, DDI o azatioprina.
M exico A rgentina
C hile U ruguay

Seguimiento a distancia

Pregunta 90.-R: 4
El diagnstico de pancreatitis crnica se establece, si el paciente
presenta sntomas, solicitando en primer lugar una Rx simple de abdomen para evaluar la presencia de calcificaciones. Si stas se demuestran el diagnstico queda establecido. Si no se observan, entonces se solicita una TAC, que es la prueba ms sensible para observar las
calcificaciones pancreticas. Si el TAC no es diagnstico, entonces se
recurre a la CPRE, que es la prueba ms sensible y especfica, la que
detecta los casos ms precoces, pero que no se utiliza de entrada
porque es cruenta y tiene efectos secundarios. El test de la secretina
diagnostica insuficiencia pancretica exocrina, pero es de escasa utilidad para el diagnstico de pancreatitis crnica.
Pregunta 91.-R: 5
Entre los indicadores de gravedad el ms precoz es la elevacin de
los niveles sricos de la elastasa de los neutrfilos. Menos inters tiene
determinar la PCR (protena C reactiva) o el pptido activador del
tripsingeno urinario. Entre los indicadores radiolgicos de gravedad
destacan los que se obtienen con un TAC con contraste, ya que ms
de un 50% de tejido necrtico confiere mal pronstico.
Clsicamente se han utilizado los criterios de Ransom (ver esquema
en pgina siguiente), entre los que no est la determinacin de PCR .
Pregunta 92.-R: 1
Una pancreatitis aguda leve o moderada requiere como tratamiento:
1. Analgesia, excepto mrficos porque contraen el esfnter de Oddi.
2. Fluidoterapia i.v.
3. Dieta absoluta a slidos y lquidos, al menos 3 das. Es muy importante para evitar complicaciones diferir el inicio de la alimentacin
oral al menos hasta que el paciente est asintomtico, por eso la
respuesta correcta es la nmero 1. Slo est indicado colocar una
sonda nasogstrica de aspiracin si hay vmitos secundarios a un
leo paraltico asociado.
No est indicado administrar antibiticos.
Pregunta 93.-R: 1
En el manejo de una pancreatitis aguda grave es muy importante
evaluar mediante una puncin si presenta infeccin del tejido necrtico, porque en ese caso est indicada la necrosectoma quirrgica. Si
no se demuestra infeccin, nicamente se toman medidas de soporte,
antibiticos i.v. (preferentemente que perfundan bien en tejido pancretico, como imipenem o las quinolonas). La somatostatina no ha
demostrado su utilidad en el manejo de estos enfermos, que, adems,
requieren nutricin parenteral o, si fuera posible, nutricin enteral
yeyunal, porque esta ltima parece reducir las tasas de infeccin de la
necrosis pancretica.
Pregunta 94-R: 1
Se trata de una pancreatitis aguda de origen biliar que cumple
criterios de gravedad. En este caso, adems del manejo general de una
pancreatitis aguda grave, se ha comprobado que la papilotoma
endoscpica realizada en las primeras 72 h. mejora el pronstico de
la enfermedad, por eso es la respuesta correcta. En este caso la colecistectoma estara indicada, pero en un segundo paso cuando se resuelva la pancreatitis aguda.
Pregunta 95.-R: 1
La causa ms frecuente de pancreatitis crnica en nuestro medio
es el alcoholismo crnico, oscilando alrededor del 70% de los casos.
Generalmente se inicia manifestndose como episodios de pancreatitis aguda recurrente y, finalmente, como dolor crnico. Otras causas
como hipertrigliceridemia, hipercalcemia, fibrosis qustica o secundaria a obstruccin de los conductos pancreticos son menos frecuentes.
Pregunta 96.-R: 2
Adems del dolor puede aparecer diabetes mellitus en los estados
finales de la enfermedad. Esta diabetes tiene parecidas complicaciones a las de la idioptica, excepto:
1. Menor tendencia a la cetoacidosis
2. Como tiene poco glucagn, ms hipoglucemias.

CTO Medicina C/ Nez de Balboa, 115 28006 MADRID (Espaa) Tfno.: (91) 782 43 32 / Fax: (91) 782 43 27
E-mail: secretaria@ctomedicina.com; iberocto@ctomedicina.com WEB: www.ctomedicina.com; www.iberocto.com

HM Pg. 13

Seguimiento a distancia

DIGESTIVO Y CIRUGA GENERAL

Preparacin Examen de Seleccin 05/06 1 Vuelta

Adems se desarrolla maldigestin, con esteatorrea, prdida de


peso y malabsorcin de las vitaminas liposolubles, pero tambin dficit de B12, ya que la lipasa es necesaria para la absorcin de esta
vitamina.
En ocasiones cursa con masas pancreticas que al obstruir la va
biliar causan ictericia.
Una complicacin infrecuente es la fuga de lquido de un pseudoquiste o del propio Wirsung al interior del peritoneo, provocando el
cuadro de ascitis pancretica. Se diagnostica por una amilasa muy
elevada en lquido asctico.
Pregunta 97.-R: 3
En una CPRE la presencia de una estenosis nica en un enfermo
con dolor sugiere mucho ms un carcinoma de pncreas que pancreatitis crnica. Las pancreatitis crnicas de cualquier tipo suelen
tener irregularidades mltiples, con estenosis y reas de dilatacin
postestenticas.
Pregunta 98.-R: 4
Una vez conocido que un paciente tiene insuficiencia pancretica exocrina, bien mediante el test ms fiable, como el de la secretina,
o con los de menor rentabilidad como el de pancreolauryl o elastasa
fecal, queda por determinar si hay que administrar enzimas pancreticas y, para ello, es suficiente con conocer el valor de las grasas
fecales (respuesta correcta), porque este es el marcador que establece la indicacin del tratamiento sustitutivo. De igual forma la valoracin del beneficio teraputico se establece: con los cambios en la
clnica y la cuantificacin de las grasas fecales que demuestre desaparicin o reduccin de la esteatorrea.
Pregunta 99.-R: 1
Para el manejo de la ascitis pancretica es fundamental detectar el
punto de fuga del jugo pancretico entre los conductos pancreticos
y el peritoneo. Para ello, es de eleccin la CPRE. Aunque la TAC
pueda mostrar un pseudoquiste es ms difcil que detecte el punto de
fuga, algo crucial para establecer las indicaciones quirrgicas.

Los antagonistas del TNF no tiene utilidad en la pancreatitis crnica, recordemos que para el examen MIR pueden estar indicados en
las fstulas refractarias de la enfermedad de Crohn y en casos de artritis
reumatoide refractarios al tratamiento convencional.

Pg. 14 HM

M exico A rgentina
C hile U ruguay

CTO Medicina C/ Nez de Balboa, 115 28006 MADRID (Espaa) Tfno.: (91) 782 43 32 / Fax: (91) 782 43 27
E-mail: secretaria@ctomedicina.com; iberocto@ctomedicina.com WEB: www.ctomedicina.com; www.iberocto.com

Comentarios TEST

Pregunta 100.-R: 4
El manejo del dolor en la pancreatitis crnica se realiza de forma
progresiva:
1. Analgesia convencional: paracetamol, metamizol, AINES.
2. Suplementos con enzimas pancreticos en ocasiones alivian el dolor.
3. Analgsicos de tercer nivel, excepto agonistas mrficos puros.
4. Dilataciones o colocacin de prtesis por CPRE si hay obstrucciones del Wirsung responsables del dolor.
5. Ciruga derivativa o incluso pancreatectomas parciales.

CIRUGA GENERAL

Preparacin Examen de Seleccin 05/06 1 Vuelta

1.

GENERALIDADES.

1)

El procedimiento consistente en colocar un tubo en el estmago que salga a la piel, con la intencin de utilizarlo para
alimentar a un paciente, se llama:

2)

1)
2)
3)
4)
5)
2.

La fiebre que se produce tras la fuga de una anastomosis


gastrointestinal aparece con mayor frecuencia:
1)
2)
3)
4)
5)

3.

2)
3)
4)
5)

4.

Preguntas TEST

7.

La sepsis es la causa ms comn de muerte en pacientes


con fstulas gastrointestinales.
En las fstulas gastrointestinales est indicada la ciruga tras
6-8 semanas de tratamiento conservador (antibiticos y
reposo digestivo con NPT).
Las anastomosis colorrectales estn ms predispuestas a
la disrupcin que las del estmago e intestino delgado.
Suele ser habitual la existencia de un leo paraltico, de
duracin de alrededor de una semana, tras la ciruga.
La hernia paraestomal es la complicacin ms frecuente,
que requiere intervencin quirrgica tras colocar una
colostoma o ileostoma.

4)
5)
8.

Innecesario.
Ciruga urgente.
Antispticos locales.
Antibioterapia tpica.
Sintomtico.

2)
3)
4)
5)

Los quemados drmicos superficiales superiores al 50%


son crticos.
La traccin del folculo piloso diferencia las quemaduras
de 1 y 2 grado.
La profundidad de la quemadura aumenta con la edad.
No est indicada de forma sistemtica la profilaxis antibitica en los quemados.
En la fase tarda del sndrome de inhalacin la mortalidad
se debe a neumona.

Asociar antibiticos al tratamiento.


Descompresin con enemas.
Colectoma total + ileostoma.
Continuar observacin 48 h.
Intervencin de Hartmann.

Cul de entre las siguientes opciones es FALSA con respecto


al carcinoma de colon asociado a la enfermedad inflamatoria
intestinal?:

INTESTINO.

1)
2)
3)

Respecto de la enfermedad diverticular del intestino grueso,


seale la respuesta INCORRECTA:

4)
5)

M exico A rgentina
C hile U ruguay

Brote agudo de colitis ulcerosa - salazopirina.


Cncer de colon sobreinfectado - ciruga.
Sndrome de Ogilvie - aspiracin gstrica.
Megacolon txico - corticoterapia y antibioterapia.
Diverticulitis perforada - intervencin de Hartmann.

Pasadas 24 h, el enfermo no evoluciona favorablemente;


ahora la actitud ser:
1)
2)
3)
4)
5)

11.

Enema opaco.
Rx simple de abdomen.
Colonoscopia.
TC.
Ecografa abdominal.

El binomio diagnstico - teraputico en este paciente ser:


1)
2)
3)
4)
5)

10.

Es ms frecuente en el hombre.
La colonoscopia es el mejor mtodo diagnstico.
Es el tipo ms frecuente de fstula de la enfermedad
diverticular.
Es caracterstica la fecaluria y la neumaturia.
No es obsttrica.

Paciente varn de 29 aos, diagnosticado hace dos aos de


colitis ulcerosa; acude a Urgencias por presentar fiebre,
diarrea sanguinolenta, y dolor y distensin abdominales, con
signos de hipersensibilidad generalizada. Qu exploracin
nos acercar al diagnstico?:
1)
2)
3)
4)
5)

9.

El diagnstico de la diverticulitis ha de ser fundamentalmente clnico, siendo la TC la mejor prueba de imagen.


En los pacientes con diverticulitis, la intervencin quirrgica se debe realizar en aquellos enfermos que no respondan al tratamiento mdico.
La mayora de los enfermos con diverticulosis son sintomticos y precisan intervencin.
La intervencin de Hartmann es la operacin ms comn
para el tratamiento de la diverticulitis con peritonitis
generalizada.
El signo fsico ms comn en la diverticulitis es la hipersensibilidad a la palpacin en fosa ilaca izquierda.

Seale la opcin FALSA sobre la fstula colovesical:


1)
2)
3)

Una de las siguientes afirmaciones en relacin a las quemaduras NO es correcta:


1)

6.

5)

El tratamiento de las heridas quirrgicas infectadas que dan


fiebre en las primeras 24 horas suele ser:
1)
2)
3)
4)
5)

5.

4)

A partir del 12 da postoperatorio.


Nunca.
Alrededor del primer mes.
A las 48-72 horas de la intervencin.
En la segunda semana del postoperatorio.

En relacin a las complicaciones de la ciruga gastrointestinal,


seale la respuesta INCORRECTA:
1)

3)

Gastrotoma.
Gastrostoma.
Gastrectoma.
Tubulizacin gstrica.
Gastroplastia.

Seguimiento a distancia

El riesgo es mayor en pacientes con colitis ulcerosa.


El riesgo aumenta con el tiempo de evolucin.
Tiene mejor pronstico que en pacientes sin enfermedad
inflamatoria intestinal.
Con mayor frecuencia son mltiples.
Tienen mayor tendencia a surgir en colon ascendente.

CTO Medicina C/ Nez de Balboa, 115 28006 MADRID (Espaa) Tfno.: (91) 782 43 32 / Fax: (91) 782 43 27
E-mail: secretaria@ctomedicina.com; iberocto@ctomedicina.com WEB: www.ctomedicina.com; www.iberocto.com

CG Pg. 1

12.

El antibitico que suele utilizarse habitualmente para el


manejo de la afectacin perianal en la enfermedad de Crohn,
pero que tiene el inconveniente de poder producir neuropata en tratamientos prolongados, es:
1)
2)
3)
4)
5)

13.

15.

5)
16.

18.

19.

Slo radioterapia.
QT con ciclofosfamida y radioterapia opcional.
QT con 5-FU y radioterapia opcional.
Ampliar mrgenes con amputacin abdominoperineal
(Miles).
Braquiterapia.

2)
3)
4)
5)

La reseccin de metstasis hepticas aisladas puede ofrecer un aumento de la supervivencia.


Las lesiones del colon derecho requieren hemicolectoma derecha.
En lesiones obstructivas del lado derecho se realiza reseccin y anastomosis primaria.
En el estadio B2 de la clasificacin de Astler y Coller, estn
afectos los ganglios linfticos.
En el estudio del cncer de recto distal debe realizarse
radiografa de trax.

Paciente diagnosticado de adenocarcinoma de colon. En la


ciruga se descubre una lesin de 2 cm de dimetro en la
superficie anterior del lbulo heptico derecho. Se toma una

Pg. 2 CG

M exico A rgentina
C hile U ruguay

Ileostoma proximal ms reseccin.


Hartmann.
Hemicolectoma derecha con anastomosis ileotransversa.
Colostoma de descarga proximal.
Colectoma total con anastomosis ileoanal.

Mujer que historiamos en la consulta de ciruga por rectorragias repetidas y abundantes, sin repercusin hemodinmica
actual. La exploracin abdominal resulta anodina. Entre las
peticiones del estudio programado, NO incluira:
1)
2)
3)
4)
5)

22.

Cncer obstructivo de antro gstrico.


Cncer obstructivo de colon.
Vlvulo de sigma.
Obstruccin adherencial.
leo biliar.

Es diagnosticado por TC de cncer de colon derecho obstructivo. Ante esta situacin clnica, el tratamiento de eleccin ser:
1)
2)
3)
4)
5)

21.

Estudio radiolgico.
Estudio analtico.
Reposicin hidroelectroltica.
Historia detallada de antecedentes.
Ciruga urgente.

Si entre los antecedentes del paciente slo consigue reflejar


debilidad de 6 meses de evolucin y una anemia microctica
como hallazgo analtico, su primera sospecha diagnstica ser:
1)
2)
3)
4)
5)

20.

Se decide que es un caso inoperable.


Reseccin segmentaria del tumor.
Colostoma de descarga proximal.
Reseccin reglada del colon y exresis completa de la
metstasis.
Reseccin reglada del colon y hepatectoma derecha
reglada.

Un paciente varn, de 60 aos, con astenia de meses de


evolucin, acude a Urgencias por cuadro de ausencia de
deposicin desde hace 7 das, mal estado general, distensin
abdominal y dolor clico infraumbilical. Su actitud inicial NO
incluir:
1)
2)
3)
4)
5)

Respecto del cncer de colon, es FALSO que:


1)

17.

5)

TC abdominal.
RM.
Gammagrafa sea.
Rx simple de abdomen.
TC torcico.

Es operada por reseccin anterior baja. La anatoma patolgica muestra tumor de 4 cm, que alcanza capa muscular,
invadindola totalmente, sin ganglios infiltrados. El tratamiento coadyuvante ser:
1)
2)
3)
4)

1)
2)
3)
4)

Tacto rectal.
Tacto vaginal.
Sistemtica de sangre.
Endoscopia.
Enema opaco.

Es diagnosticada por biopsia de adenocarcinoma de recto, a


10 cm del margen anal. El estudio de extensin lo hace usted
con:
1)
2)
3)
4)
5)

biopsia, y sta es informada intraoperatoriamente como


metstasis de un adenocarcinoma de colon. Ante ello, la
actitud quirrgica ser:

Cefotaxima i.v.
Mupirocina tpica.
Amoxicilina + clavulnico oral.
Gentamicina i.v.
Metronidazol oral.

Paciente mujer, que acude a Urgencias a sus 66 aos, por


deposiciones sanguinolentas desde hace diez das, sin dolor.
Refiere notarse ms estreida de lo que era previamente, con
ocasionales deposiciones lquidas. Se nota cansada y con
menos apetito, y dice que ha perdido tres Kg en dos meses.
La primera prueba que realizara usted sera:
1)
2)
3)
4)
5)

14.

CIRUGA GENERAL

Preparacin Examen de Seleccin 05/06 1 Vuelta

Arteriografa de AMS.
Enema opaco.
Colonoscopia.
TC.
Analtica rutinaria.

En el diagnstico diferencial del cuadro NO estamos incluyendo:


1)
2)
3)
4)
5)

Cncer de colon.
Angiodisplasia de colon.
Hemorroides.
Isquemia mesentrica.
Diverticulosis de sigma.

CTO Medicina C/ Nez de Balboa, 115 28006 MADRID (Espaa) Tfno.: (91) 782 43 32 / Fax: (91) 782 43 27
E-mail: secretaria@ctomedicina.com; iberocto@ctomedicina.com WEB: www.ctomedicina.com; www.iberocto.com

Preguntas TEST

Seguimiento a distancia

CIRUGA GENERAL

Preparacin Examen de Seleccin 05/06 1 Vuelta


23.

Durante el estudio, la paciente acude a Urgencias con dolor


en FII, fiebre, defensa abdominal e irritacin peritoneal.
Solicitaremos:

4)
5)

1)
2)
3)
4)
5)

HEPATOBILIOPNCREAS.

Analtica y colonoscopia.
Colonoscopia y TC.
Enema opaco y analtica.
Analtica, radiografa de trax y abdomen.
Arteriografa y TC.

29.

ESTMAGO.
24.

Respecto a las complicaciones de la lcera pptica, seale la


respuesta correcta:
1)
2)
3)
4)
5)

25.

Preguntas TEST

3)
4)
5)
28.

Gastrectoma subtotal paliativa.


Gastrectoma total + linfadenectoma + omentectoma
asociada.
Gastroenterostoma + pancreatectoma parcial.
Antrectoma + linfadenectoma.
Gastrectoma subtotal + linfadenectoma.

Qu son las lceras de Cushing?:


1)
2)
3)

31.

Las UGD tpicas de politraumatizados con afectacin del


SNC (por hipersecrecin).
lceras por isquemia crnica del tronco celaco.
lceras por isquemia aguda en quemados.
M exico A rgentina
C hile U ruguay

Ciruga urgente.
CPRE + esfinterotoma.
Antibiticos.
Espasmolticos y analgsicos.
Dieta absoluta + SNG + repetir analtica en 4 horas.

Si la paciente anterior fuera sometida a colecistectoma por


clicos repetidos, digestiones pesadas y sensacin de distensin abdominal con meteorismo postprandial, cul sera el
probable diagnstico anatomopatolgico?:
1)
2)
3)
4)
5)

33.

Clico biliar.
Colecistitis aguda.
leo biliar.
UGD.
Pancreatitis.

El tratamiento de la paciente referida ser:


1)
2)
3)
4)
5)

32.

Dilatacin de va biliar extraheptica.


Dilatacin de va biliar extra e intraheptica.
Tumor en cabeza de pncreas.
Colelitiasis simplemente.
Coledocolitiasis.

Qu sospecha diagnstica tiene Vd?:


1)
2)
3)
4)
5)

Sutura de la lcera + vagotoma troncular + piloroplastia.


Antrectoma + anastomosis tipo Billroth I.
Gastrectoma 2/3 + vagotoma + Y de Roux.
Vagotoma supraselectiva.
Sutura de la lcera + epiploplastia.

Un paciente es diagnosticado por endoscopia de carcinoma


gstrico antral de tipo enteroide. Durante la laparotoma
exploradora se objetiva un tumor que afecta al antro y
cuerpo, extenso, con algunos ganglios positivos infrapilricos y en curvadura menor. El tratamiento ser:
1)
2)

30.

lceras por hipersecrecin en los quemados.


lceras tpicas de pacientes psiquitricos.

Paciente mujer, de 52 aos, que acude a Urgencias por dolor


en epigastrio e hipocondrio derecho, que aumenta y disminuye, pero no desaparece, desde hace 2 horas, acompaado
de nuseas y vmitos. El dolor se irradia hacia la espalda. La
paciente est afebril, la exploracin fsica es normal y la
analtica con amilasa y bilirrubina no presenta alteraciones.
Qu esperara encontrar si hace una ecografa?:
1)
2)
3)
4)
5)

Neumotrax.
lcera antral sangrante.
Diseccin artica.
Perforacin de lcera duodenal.
Pancreatitis aguda.

El tratamiento que considera de eleccin sera:


1)
2)
3)
4)
5)

27.

La perforacin afecta a la cara posterior duodenal, produciendo un aumento del nivel de amilasa srica.
La hemorragia de la lcera gstrica es ms frecuente en
cara posterior, afectando a la arteria gastroduodenal.
El tratamiento definitivo de la estenosis es quirrgico, tras
restablecer la correcta nutricin del paciente.
Es tpico de la perforacin los vmitos, acompaados de
"abdomen en tabla".
El tratamiento de entrada de la HDA es quirrgico.

Paciente varn, de 40 aos, que presenta un cuadro sbito


de dolor "en pualada" en epigastrio, con sudoracin y
taquicardia. La Rx de trax en bipedestacin confirmar:
1)
2)
3)
4)
5)

26.

Seguimiento a distancia

Vescula sin alteraciones.


Simplemente colelitiasis.
Clico biliar.
Colecistitis crnica + colelitiasis.
Adenomiosis de vescula.

Paciente varn, de 83 aos, con doble lesin valvular y


fibrilacin auricular crnica, en tratamiento con anticoagulantes orales, que acude a Urgencias por un cuadro de dolor
en epigastrio e hipocondrio derecho, irradiado a la espalda,
de 7 horas de evolucin. Ha presentado nuseas y vmitos,
sin otras alteraciones del trnsito. Temperatura: 37,9C. TA:
140/90 mmHg. Coloracin normal de piel y mucosas. A la
palpacin, el enfermo se queja y defiende cuando se toca en
hipocondrio derecho. En la analtica de sangre hay 15.700
leucocitos (83% neutrfilos, 4% cayados), amilasa: 90 U/l,
bilirrubina total: 1,1 mg/dl, y resto sin alteraciones significativas. Cul sera la secuencia ms correcta de diagnstico y
tratamiento?:
1)
2)

Gammagrafa con HIDA / analgesia + espasmolticos.


CPRE / analgesia + espasmolticos.

CTO Medicina C/ Nez de Balboa, 115 28006 MADRID (Espaa) Tfno.: (91) 782 43 32 / Fax: (91) 782 43 27
E-mail: secretaria@ctomedicina.com; iberocto@ctomedicina.com WEB: www.ctomedicina.com; www.iberocto.com

CG Pg. 3

3)
4)
5)
34.

2)
3)
4)
5)

3)
4)
5)

La coledocolitiasis, tras colecistectoma previa, se trata con


esfinterotoma endoscpica.
La incidencia de la colecistitis calculosa aguda es ms
frecuente en varones.
La colecistitis enfisematosa es ms frecuente en pacientes
con hipertensin.
Est indicada la colecistectoma en pacientes asintomticos.
El carcinoma de clulas escamosas es el tipo ms frecuente
de cncer de vescula.

No control del dolor con AINEs en dosis plenas.


Dilatacin de asas intestinales en mesogastrio en la Rx de
abdomen y distensin abdominal.
Aparicin de dolor, que aumenta con la palpacin y
vmitos repetidos, con aumento de la bilirrubina y aparicin de ictericia.
Imagen de borramiento de la grasa en rea pancretica en
el TC, con posibles zonas de necrosis.
Existencia de necrosis peripancretica en el TC, con
cultivo positivo tras puncin.

Paciente varn, de 50 aos, bebedor habitual, tratado hace


tres semanas por un cuadro de dolor epigstrico irradiado en
cinturn (no aporta informes), que requiri ingreso. Acude
ahora con dolor persistente en hemiabdomen superior,
sensacin de saciedad temprana, nuseas y vmitos; se
evidencia masa palpable en hipocondrio derecho. La analtica es inespecfica. La exploracin que ms datos nos aportar sobre la patologa que refiere el enfermo ser:
1)
2)
3)
4)
5)

37.

38.

Ecografa abdominal.
CPRE.
TC abdominal.
Rx simple abdomen.
Gastroscopia.

2)
3)
4)
5)
39.

2)
3)
4)
5)
40.

3)
4)
5)

Pg. 4 CG

Reseccin pancretica, incluyendo el pseudoquiste.


Observacin y seguimiento mediante la realizacin de
ecografas peridicas.
Puncin percutnea del pseudoquiste.
Intervencin de Whipple.
Cistogastrostoma quirrgica.
M exico A rgentina
C hile U ruguay

Se relaciona con el consumo de esteroides.


Se diagnostica por RM o ecografa.
Suelen ocasionar ictericia por compresin biliar.
Pueden malignizarse.
Su tratamiento es quirrgico por el peligro de complicaciones.

Sobre los tumores hepticos, seale la opcin FALSA:


1)
2)
3)
4)
5)

43.

Malignizacin y colestasis.
Colestasis y aparicin de "lesiones satlite".
Colestasis por compresin y hemorragia.
Hemorragia y malignizacin.
Malignizacin y sndromes paraneoplsicos.

Cul de las siguientes afirmaciones es cierta con respecto al


hemangioma heptico?:
1)
2)
3)
4)
5)

42.

Pancreaticoyeyunostoma latero-lateral (operacin de


Puestow).
Observacin.
Quistoyeyunostoma urgente.
Puncin percutnea bajo control radiolgico.
Intervencin de Whipple.

Los motivos que nos empujan a operar los adenomas hepticos son sus potenciales complicaciones, a saber:
1)
2)
3)
4)
5)

41.

Pseudoquiste pancretico no complicado, de 5 cm de


dimetro.
Colangitis esclerosante, que no ha respondido a corticoides.
Pancreatitis crnica con dilatacin del Wirsung.
Clculo en leon terminal, no resuelto por litotoma endoscpica.
Recidiva del cncer de pncreas.

El paciente anterior acude a Urgencias por presentar dolor


abdominal en mesogastrio, acompaado de fiebre en picos
>38,5C con escalofros, objetivndose en la analtica de urgencias leucocitosis de 22.000 con neutrofilia y desviacin izquierda, sin foco infeccioso aparente. Cul sera su actitud ahora?:
1)

La informacin de dicha prueba es concluyente, demostrando un pseudoquiste pancretico de aproximadamente 7 cm


de dimetro. Su actitud ser?:
1)
2)

Cualquiera que fuera su actitud, el paciente acude a revisin


a las 6 semanas, para control. Tras repetir la prueba, que fue
diagnstica, se decide continuar observacin. El hallazgo
ms probable habr sido:
1)

Cul de las siguientes es indicacin de ciruga urgente en un


paciente de 58 aos, con antecedentes de herniorrafia,
colelitiasis e HTA, ingresado por dolor abdominal en mesogastrio, sin signos de irritacin peritoneal, que no cambia con
posturas, coloracin azulada en los flancos, 12.000 leucocitos/mm3, hipocalcemia, amilasa de 3.940 U/l, bilirrubina
total: 1,9 mg/dl y derrame pleural derecho?:
1)
2)

36.

Ecografa abdominal / ciruga urgente.


Ecografa abdominal / analgesia + antibiticos.
TC abdominal / ciruga urgente.

Respecto de las enfermedades del sistema biliar, seale la


respuesta correcta:
1)

35.

CIRUGA GENERAL

Preparacin Examen de Seleccin 05/06 1 Vuelta

El tratamiento curativo del hepatocarcinoma es la ciruga.


En los hepatocarcinomas irresecables sin enfermedad
extraheptica est indicado el trasplante heptico.
El cncer de coldoco se denomina tumor de Klatskin.
El angiosarcoma heptico se relaciona con exposicin a
cloruro de vinilo.
El carcinoma fibrolamelar es ms frecuente en adultos
jvenes y adolescentes.

Paciente varn de 70 aos, fumador y diabtico. Deprimido


desde hace 2 meses. Estudiado en la consulta de digestivo por
prdida de peso y dolor epigstrico opresivo que se irradia
a la espalda. El enfermo est ictrico y la exploracin abdominal es anodina. Qu prueba diagnstica NO nos ayudar
en este caso?:

CTO Medicina C/ Nez de Balboa, 115 28006 MADRID (Espaa) Tfno.: (91) 782 43 32 / Fax: (91) 782 43 27
E-mail: secretaria@ctomedicina.com; iberocto@ctomedicina.com WEB: www.ctomedicina.com; www.iberocto.com

Preguntas TEST

Seguimiento a distancia

CIRUGA GENERAL

Preparacin Examen de Seleccin 05/06 1 Vuelta


1)
2)
3)
4)
5)
44.

hemiabdomen inferior. Ahora refiere dolor intenso, constante


y generalizado; ha presentado dos episodios de rectorragia. En
la exploracin fsica, impresiona de gravedad, con distensin
abdominal, signos de peritonismo y ausencia de ruidos hidroareos. Rx de abdomen: leo paraltico con distensin de
intestino grueso hasta el ngulo esplnico. La actitud ser:

Antes de obtener los resultados, y en orden de frecuencia,


esperamos encontrar:
1)
2)
3)
4)
5)

45.

TC.
CPRE.
Ecografa abdominal.
Estudio gastroduodenal.
Gammagrafa con Tc99.

Cncer de cabeza de pncreas - adenocarcinoma ductal.


Cncer de cola de pncreas - adenocarcinoma lobulillar.
Cncer de cabeza de pncreas - adenocarcinoma lobulillar.
Colangiocarcinoma distal.
Ampuloma.

1)
2)
3)
4)
5)
50.

Por tanto, el nico tratamiento curativo ser:


1)
2)
3)
4)
5)

Pancreatectoma total.
Intervencin de Whipple.
Pancreatectoma corporocaudal.
Trasplante hepatopancretico.
Quimiorradioterapia.

51.

1)
2)
3)
4)
5)
47.

Preguntas TEST

2)
3)
4)
5)

3)
4)
5)
52.

Si el vmito precede al dolor, debe ponerse en duda el


diagnstico.
La perforacin del apndice suele producir un alivio de
la sintomatologa.
El signo del psoas es bastante especfico.
El dolor se localiza en fosa ilaca derecha desde el inicio
del cuadro.
En nios menores de dos aos, es ms frecuente la
apendicitis que la invaginacin intestinal.

53.

3)
4)
5)

Es el abdomen agudo que ms a menudo necesita ciruga.


Tras la perforacin, comienza a aparecer distensin abdominal por leo.
En pacientes ancianos, se asemeja a una obstruccin
mecnica de intestino delgado.
Es frecuente en lactantes.
La incidencia de perforacin se acerca al 100% antes del
primer ao de vida.

Paciente mujer, de 72 aos, DMID, que present dolor abdominal de inicio sbito, 10 horas antes de su ingreso, en
M exico A rgentina
C hile U ruguay

Antibioterapia i.v. y ver evolucin.


Laparotoma urgente.
Arteriografa mesentrica superior.
Batera de pruebas radiolgicas (eco, Rx, TC).
Fasciotoma urgente.

Ya que su primera sospecha diagnstica es:


1)
2)
3)
4)
5)

54.

Reseccin del segmento clico afecto y anastomosis primaria trmino-terminal (T-T).


Reseccin del segmento de intestino delgado afecto y
anastomosis T-T.
Reseccin del segmento clico afecto y exteriorizacin de
su extremo proximal (Hartmann).
Laparotoma exploradora.
Cierre de la perforacin y colostoma de descarga proximal.

Paciente mujer, de 40 aos, que vive en un pueblo, acude a


Urgencias con cuadro evolucionado de abdomen agudo,
ms focalizado en fosa ilaca derecha, estado sptico en preshock, y que en las ltimas horas se acompaa de escalofros
y fiebre en picos, ictericia y dolor en hipocondrio derecho.
Su actitud ser:
1)
2)
3)
4)
5)

Seale la respuesta INCORRECTA sobre la apendicitis aguda:


1)
2)

49.

2)

En la apendicitis aguda, seale la correcta:


1)

48.

Fiebre tifoidea.
Enteritis neutropnica.
Porfiria aguda intermitente.
Carbunco digestivo.
lcera duodenal.

Isquemia mesentrica aguda.


Carcinoma obstructivo de ngulo esplnico.
Angiodisplasia de ngulo esplnico.
Colitis isqumica gangrenada de ngulo esplnico.
Diverticulitis aguda perforada.

El tratamiento quirrgico empleado ser:


1)

Cul de las siguientes patologas "mdicas" NO cuenta entre


sus complicaciones tpicas con una perforacin digestiva,
con la consiguiente necesidad de ciruga?:

Realizar intervencin quirrgica urgente.


Realizar enema opaco.
Realizar arteriografa de AMI.
Realizar colonoscopia.
Esperar evolucin y repetir estudios radiolgicos.

Al final, la enferma es intervenida quirrgicamente. El diagnstico ms probable ser:


1)
2)
3)
4)
5)

ABDOMEN AGUDO.
46.

Seguimiento a distancia

Gangrena de Fournier.
Absceso tubo-ovrico.
leo biliar.
Pileflebitis.
Cncer de colon perforado con metstasis hepticas.

El tratamiento de un paciente con dolor abdominal generalizado, hipersensibilidad a la palpacin, cuadro obstructivo,
fiebre, 17.000 leucocitos/mm3, signos de hipovolemia, y que
en la radiografa simple de abdomen muestra dilatacin de
colon con "imagen en omega", ser:
1)
2)

Enema opaco teraputico.


Devolvulacin endoscpica.

CTO Medicina C/ Nez de Balboa, 115 28006 MADRID (Espaa) Tfno.: (91) 782 43 32 / Fax: (91) 782 43 27
E-mail: secretaria@ctomedicina.com; iberocto@ctomedicina.com WEB: www.ctomedicina.com; www.iberocto.com

CG Pg. 5

3)
4)
5)
55.

62.

3)
4)
5)
63.

3)
4)
5)
64.

Infarto agudo de miocardio.


Clico nefrtico.
Perforacin de ulcus duodenal.
Diverticulitis perforada.
Apendicitis aguda.

2)
3)
4)
5)

Laparotoma urgente con el diagnstico de perforacin de


lcera duodenal.
Endoscopia digestiva alta.
Estudio con contraste hidrosoluble.
Colocar SNG, introducir aire y repetir Rx trax en bipedestacin.
Lavado peritoneal diagnstico.

1)
2)
3)
4)
5)
Pg. 6 CG

Intervencin quirrgica.
Antibiticos.
Espasmolticos.
Analgsicos y evolucin.
Trombolticos.
M exico A rgentina
C hile U ruguay

66.

Aerobilia.
Dilatacin de asa de intestino delgado.
Clculo en leon terminal.
Niveles hidroareos.
Dilatacin de colon sigmoide.

Su sospecha diagnstica ser:


1)
2)
3)
4)
5)

Con ello, objetivamos la aparicin de una imagen de densidad


area bajo ambas cpulas diafragmticas. La actitud teraputica debera ser ahora:

El tratamiento de la lcera complicada es siempre resectivo.


Es frecuente que una perforacin cause un shock hipovolmico.
La perforacin es una complicacin ms frecuente que la
hemorragia.
Un enfermo puede estar perforado y no existir neumoperitoneo.
La mayora de las complicaciones actuales por lcera se
deben a fracaso del tratamiento farmacolgico.

Mujer de 70 aos, con antecedentes de clicos biliares;


acude a Urgencias con un cuadro de dolor abdominal difuso
clico, de 24 h de evolucin, nuseas, vmitos biliosos y
alimentarios. Actualmente asintomtica y afebril. Se solicitan
analtica sangunea y Rx de abdomen. Se pautan espasmolticos etiquetando el cuadro de clico biliar. Con la evolucin,
la enferma empeora, predominando clnica y semiologa de
obstruccin intestinal. En la Rx de abdomen NO esperaremos
encontrar:
1)
2)
3)
4)
5)

65.

Habr que cerrar la perforacin.


Probablemente habr que dar tratamiento erradicador
del H. pylori.
El cierre simple puede ser una opcin correcta.
El tratamiento debe incluir reseccin de la lcera.
Si el test de ureasa intraoperatorio es negativo, debe
realizarse vagotoma troncular + piloroplastia.

De todo lo anterior y de sus amplios conocimientos sobre la


patologa ulcerosa, Vd. deduce que es cierto que:
1)
2)

Infarto agudo de miocardio.


Clico nefrtico.
Perforacin de ulcus duodenal.
Diverticulitis perforada.
Apendicitis aguda.

Apendicitis perforada.
lcera duodenal perforada encubierta con epipln.
Diverticulitis perforada.
Isquemia mesentrica.
Diverticulitis de Meckel.

Con respecto a su tratamiento, cul de estas opciones le


parece FALSA?:
1)
2)

Por ello, el paso siguiente sera:


1)

60.

TC abdominal.
ECG.
Rx de abdomen.
Rx de trax en bipedestacin.
Sedimento de orina.

La radiografa en bipedestacin no demostr neumoperitoneo. La clnica del paciente se exacerb. Ante ello, la sospecha
diagnstica sera ahora:
1)
2)
3)
4)
5)

59.

Laparotoma exploradora.
Adhesilisis.
Descompresin endoscpica.
Reposicin hidroelectroltica y aspiracin gstrica.
Analgesia.

En el caso de que el enfermo fuera intervenido quirrgicamente, los hallazgos quirrgicos que encontrara ms probablemente seran:
1)
2)
3)
4)
5)

La primera sospecha diagnstica ser:


1)
2)
3)
4)
5)

58.

61.

Varn de 38 aos, que presenta dolor sbito epigstrico, de


1 hora de evolucin, con nuseas, sudoracin y taquicardia.
En la exploracin fsica presenta defensa abdominal involuntaria, con abdomen "en tabla". La primera prueba que usted
debe emplear en la orientacin diagnstica sera:
1)
2)
3)
4)
5)

57.

Reseccin del vlvulo de sigma + anastomosis T-T.


Solucin evacuante.
Intervencin de Hartmann con reseccin del segmento
afecto.

El tratamiento inicial de la obstruccin intestinal de cualquier


causa es:
1)
2)
3)
4)
5)

56.

CIRUGA GENERAL

Preparacin Examen de Seleccin 05/06 1 Vuelta

Colecistitis.
Coledocolitiasis.
leo biliar.
Pancreatitis litisica.
Cncer obstructivo de intestino delgado.

El tratamiento de urgencias NO incluir:


1)
2)
3)

Reposo digestivo.
Aspiracin digestiva.
Fluidoterapia intravenosa.

CTO Medicina C/ Nez de Balboa, 115 28006 MADRID (Espaa) Tfno.: (91) 782 43 32 / Fax: (91) 782 43 27
E-mail: secretaria@ctomedicina.com; iberocto@ctomedicina.com WEB: www.ctomedicina.com; www.iberocto.com

Preguntas TEST

Seguimiento a distancia

CIRUGA GENERAL

Preparacin Examen de Seleccin 05/06 1 Vuelta


4)
5)
67.

Preguntas TEST

2)
3)
4)
5)
72.

Antagonistas del calcio.


Reseccin del mun cstico.
Esfinterotoma transduodenal.
CPRE + esfinterotoma endoscpica.
Trasplante heptico.

Cules son las tres exploraciones fundamentales que deben


buscarse al valorar inicialmente a un paciente de 50 aos con
un cuadro clnico sugerente de obstruccin intestinal?:
1)

1)
2)
3)
4)
5)

Bsqueda de hernias, existencia de laparotomas previas


y tacto rectal.
Percusin abdominal, hemograma y Rx abdomen.
Palpacin abdominal, ionograma y Rx abdomen.
Inspeccin abdominal, ionograma y tacto rectal.
Bsqueda de hernias, hemograma y Rx abdomen.

Paciente de 29 aos, operada por endometriosis (anexectoma izquierda) hace 2 aos. Acude a urgencias presentando
un cuadro de dolor abdominal que se ha ido localizando en
fosa ilaca derecha, nuseas, 2 vmitos y febrcula. No refiere
diarrea. Est en la mitad del ciclo. A la exploracin, parece
existir ms dolor en fosa ilaca derecha, sin contractura local,
pero con signo de Blumberg positivo. En el tacto vaginal,
parece que hay ms dolor al palpar el anejo derecho. AnalM exico A rgentina
C hile U ruguay

Pedir valoracin ginecolgica.


Realizar ecografa abdominal.
Realizar ecografa vaginal.
Dejar en observacin 12 horas y hacer nueva valoracin
clnica y analtica.
Intervenir quirrgicamente.

TRAUMATISMOS ABDOMINALES.
73.

Paciente varn de 35 aos, que presenta, tras accidente de


trfico, traumatismo abdominal contuso con shock hipovolmico, dolor y distensin abdominales. En el lavado peritoneal aparece hemoperitoneo. El diagnstico ms probable
ser:
1)
2)
3)
4)
5)

74.

Ciruga.
Antibioterapia.
Hepatoprotectores.
Analgsicos.
Seriar bilirrubina y ver evolucin.

El tratamiento diferido ser:


1)
2)
3)
4)
5)

71.

Coledocolitiasis y colangitis.
Coledocolitiasis y pancreatitis.
Coledocolitiasis y hepatitis.
Mun cstico calculoso.
Cncer de pncreas.

La actitud de urgencia debe ser:


1)
2)
3)
4)
5)

70.

Coledocolitiasis.
Reflujo gastroesofgico.
Dispepsia.
Sndrome postcolecistectoma.
Sndrome depresivo.

Mientras espera los estudios, acude a urgencias por dolor en


hipocondrio derecho y fiebre en picos. En el hemograma
destaca 17.000 leucocitos/mm3 con desviacin izquierda y
6,7 mg% de bilirrubina total; fosfatasa alcalina elevada.
Probablemente la enferma padece:
1)
2)
3)
4)
5)

69.

ticamente presenta 11.000 leucocitos (79 % neutrfilos). La


Rx de abdomen es normal. Cul de las siguientes actitudes
le parece MENOS adecuada?:

Mujer de 58 aos, que fue colecistectomizada de forma


programada por colelitiasis sintomtica. Tras la ciruga, la
paciente present episodios de dolor en epigastrio e hipocondrio derecho, clico, con nuseas y vmitos y sin alteraciones analticas. Este cuadro lo podemos etiquetar como:
1)
2)
3)
4)
5)

68.

CPRE.
Enterolitotoma.

Seguimiento a distancia

Ante un paciente que presenta una herida por arma blanca


penetrante en abdomen, con evisceracin de varias asas de
intestino delgado, qu actitud tomara?:
1)
2)
3)
4)
5)

75.

Rotura esplnica.
Rotura heptica.
Laceracin heptica.
Rotura diafragmtica.
Rotura de vena cava.

Observacin durante 24-48 h.


Laparotoma.
Realizar TC abdominal.
Laparotoma, slo si el enfermo presenta shock.
Lavado peritoneal diagnstico.

Un paciente varn es trado a urgencias por la UVI del 061,


tras sufrir un accidente de trfico. El mdico os cuenta que
el paciente estaba con 90/60 mmHg de tensin cuando lo
recogi, hace 15 minutos, y que le ha administrado 700 cc de
lquidos i.v., 10 mg de midazolam y bloqueante muscular para
la intubacin. El paciente, a su llegada, tiene Glasgow 6/15, TA
120/70 mmHg, 98 lpm, una herida incisa en la frente, mltiples escoriaciones y contusiones, y pupilas algo dilatadas y
reactivas. La placa de trax porttil es normal. La palpacin
abdominal parece dolorosa, aunque no hay marcada distensin del abdomen. La hemoglobina es de 11,8 g/dl, 12.800
leucocitos/mm3 (79% neutrfilos) y saturacin 96%. Cul
sera su actitud en esta situacin?:
1)
2)
3)
4)
5)

Laparotoma urgente.
Realizar TC craneal y abdominal.
Realizar puncin lavado peritoneal.
Realizar ecografa abdominal.
Realizar Rx simple de abdomen.

CTO Medicina C/ Nez de Balboa, 115 28006 MADRID (Espaa) Tfno.: (91) 782 43 32 / Fax: (91) 782 43 27
E-mail: secretaria@ctomedicina.com; iberocto@ctomedicina.com WEB: www.ctomedicina.com; www.iberocto.com

CG Pg. 7

CIRUGA GENERAL

Preparacin Examen de Seleccin 05/06 1 Vuelta


Pregunta 1.- R: 2
El sufijo "-toma" significa cortar. En ciruga se refiere al hecho de
abrir un rgano hueco para mirar en su interior. Por ejemplo, la
gastrotoma, en la que se abre el estmago para controlar un punto
sangrante o extraer un cuerpo extrao; la colotoma, en la que se
abre el colon para extraer un plipo (polipectoma); la coledocotoma, en la que se abre el coldoco para extraer un clculo biliar
retenido en l. El cierre de una "-toma" suele ser una "-rrafia"
(gastrorrafia, colonorrafia, coledocorrafia).
El sufijo "-ectoma" significa cortar y extirpar. Por ejemplo, gastrectoma, colectoma, mastectoma o histerectoma.
El sufijo "-stoma" significa hacer una apertura o comunicacin.
Cuando va precedido de dos rganos se refiere a comunicarlos
entre s (por ejemplo, gastroyeyunostoma que equivale a anastomosis gastroyeyunal), mientras que si va precedido de un solo rgano se entiende que es comunicar ese rgano con la piel, directamente o mediante un tubo, como es el caso que nos ocupa (por
ejemplo, gastrostoma, colostoma, ileostoma o colecistostoma),
estableciendo un estoma.
El sufijo "-pexia" significa pegar o fijar. Por ejemplo, la mastopexia
(donde se fijan las mamas excesivamente colgantes), la cecopexia
(donde se fija un ciego redundante para evitar que se volvue) o la
gastropexia posterior (donde se fija el estmago a los planos prevertebrales para evitar su deslizamiento hacia el trax, tcnica que se
puede utilizar en las hernias de hiato por deslizamiento).
El sufijo "-plastia" significa reconstruir o modificar la forma. Por
ejemplo, la gastroplastia tubular, en la que se construye un tubo
con el estmago de modo que pueda sustituir a un esfago que ha
sido extirpado; o la mamoplastia (de aumento o de reduccin del
tamao mamario); o la gran variedad de gastroplastias empleadas
en la ciruga de la obesidad (ciruga baritrica).

Comentarios TEST

Pregunta 2.- R: 5
Como se expone en el Manual, el diagnstico de la fiebre postoperatoria se enfoca en funcin del momento en el que aparece. Las
causas de fiebre postoperatoria que tienen una relacin ms clara
con la ciruga suelen manifestarse a partir del 3er da postoperatorio: la
infeccin de la herida quirrgica a partir del 5 da y la fiebre en
relacin con fstulas, fugas y abscesos perianastomticos (que constituyen en realidad un mismo proceso con distintos nombres), suele
aparecer al inicio de la segunda semana postoperatoria.
Pregunta 3.- R: 4
Las zonas del tubo digestivo ms predispuestas a complicaciones
anastomticas son sus extremos: esfago y recto; ello explica la mayor
morbilidad con que estn gravadas estas cirugas. Cuando aparece
una fstula anastomtica, el tratamiento es conservador durante 4-8
semanas y consiste en administrar antibiticos para controlar la infeccin y dejar en reposo digestivo para minimizar el dbito a travs de la
fstula y facilitar su cierre espontneo (ello suele implicar la necesidad
de nutricin parenteral, dado que un buen estado nutricional es necesario para una correcta cicatrizacin). Para que la infeccin se mantenga controlada, es importante evitar que se formen grandes abscesos
intraabdominales, para lo cual es fundamental que las colecciones
tengan un camino de salida al exterior (un drenaje que se pudo dejar
en el momento de la ciruga o que se coloca posteriormente con
control radiolgico para dar salida a una coleccin). Si la infeccin
no se controla de esta manera, sobreviene una sepsis, que es la causa
ms frecuente de muerte en estas circunstancias.
El leo paraltico es una respuesta fisiolgica a la agresin quirrgica
(es un leo reflejo) y dura 24-48 horas en la ciruga gastrointestinal y
hasta 5 das en la ciruga del colon. Un leo que se prolonga ms all
de lo esperado puede alertarnos de una complicacin anastomtica,
mxime si se asocia a la aparicin de fiebre.
Pregunta 4.- R: 2
Como veamos previamente, la fiebre en relacin con la infeccin
de herida suele aparecer a partir del 5 da, precedida de dolor local
y eritema. Sin embargo, existen dos infecciones severas de partes blandas que pueden aparecer en el contexto de una ciruga y que originan un cuadro sptico muy grave en las primeras horas postoperatoM exico A rgentina
C hile U ruguay

Seguimiento a distancia

rias: la gangrena gaseosa (causada por Clostridium perfringens) y la


fascitis necrotizante (causada por algunas cepas de estreptococos del
grupo A). En estos casos la cobertura antibitica sistmica es necesaria
pero la piedra angular del tratamiento es el desbridamiento quirrgico urgente de todo el tejido afectado, con la mxima clsica de la
gangrena: "cortar por lo sano".
Pregunta 5.- R: 2
La traccin del folculo piloso permite distinguir si est afectada o
no la raz del pelo, esto es, si hay quemadura en dermis profunda
(distingue el 2 grado superficial del profundo).
Los criterios de gravedad en funcin de la superficie y de la profundidad se exponen claramente en el Manual. Ante un mismo estmulo
gneo, la profundidad de las quemaduras aumenta con la edad. Ello
se debe a que los procesos de envejecimiento conllevan una progresiva deshidratacin tisular, por lo que en tejidos con menos agua la
quemadura es mayor.
La antibioterapia sistmica profilctica no est indicada en los quemados, salvo en 3 situaciones bien concretas:
Cuando existe inhalacin de humos, dado que se asocia a una neumona severa con frecuencia y conlleva importante mortalidad.
Como profilaxis periquirrgica.
En quemaduras por alto voltaje, que conllevan grandes zonas de
necrosis tisular.
Pregunta 6.- R: 3
La diverticulosis del colon (localizada con mayor frecuencia en
sigma) es una condicin de muy alta prevalencia en la poblacin
occidental y no asocia ningn tipo de sintomatologa, por lo que no
precisa tratamiento alguno. Suele ser un hallazgo casual en enema
opaco o colonoscopia.
Cuando se complica es cuando da clnica, hablndose entonces de
enfermedad diverticular del colon: sus dos formas son la diverticulitis
(la complicacin ms frecuente de la diverticulosis) y la hemorragia
diverticular. En estas situaciones s que es preciso un tratamiento, que en
la mayora de los casos de diverticulitis es conservador, con antibiticos.
El cuadro de la diverticulitis es muy similar al de la apendicitis, pero
localizado en fosa iliaca izquierda (generalmente en sigma) y puede ir
desde un dolor suave hasta un cuadro de peritonitis. La mejor prueba
de imagen en su diagnstico es la TC abdominal, estando contraindicados el enema opaco y la colonoscopia. Slo en caso de formas asociadas a peritonitis o con mala evolucin existe una indicacin quirrgica
urgente. En estos casos se suele realizar una intervencin de Hartmann,
dejando la reconstruccin del trnsito para un segundo tiempo.
Pregunta 7.- R: 2
La fstula colovesical tiene un origen esencialmente inflamatorio:
bien sea por diverticulitis (donde un divertculo inflamado se pone en
contacto con la vejiga), bien sea por enfermedad inflamatoria intestinal. La fstula vesicovaginal, en cambio, suele tener un origen obsttrico, mientras que la rectovaginal puede deber su origen a patologa
perianal o ser un complicacin de ciruga oncolgica rectal o de
radioterapia.
Clnicamente, la fstula colovesical se manifiesta como fecaluria,
neumaturia e infecciones urinarias de repeticin.
Para el diagnstico de una fstula colovesical, el mejor abordaje diagnstico es la va urinaria, bien sea por cistoscopia, bien por cistografa.
Pregunta 8.- R: 2
En este caso se nos presenta un paciente con una colitis ulcerosa
que viene con un cuadro agudo que parece una reagudizacin (brote
severo), en el que llama la atencin la existencia de distensin abdominal con hipersensibilidad generalizada. Parece imperativo hacerse
una idea de la distensin colnica subyacente, porque si es superior
a 9 cm el riesgo de perforacin es importante, y si es superior a 12 cm
hay una indicacin absoluta de ciruga urgente; para ello lo ms til
ser la radiologa simple de abdomen.
Pregunta 9.- R: 4
Con la clnica antes mencionada, parece que este brote severo est
presentndose como un megacolon txico, por lo que habr que

CTO Medicina C/ Nez de Balboa, 115 28006 MADRID (Espaa) Tfno.: (91) 782 43 32 / Fax: (91) 782 43 27
E-mail: secretaria@ctomedicina.com; iberocto@ctomedicina.com WEB: www.ctomedicina.com; www.iberocto.com

EP Pg. CG

CIRUGA GENERAL

Preparacin Examen de Seleccin 05/06 1 Vuelta

instaurar un tratamiento inicialmente conservador (pero muy vigilante) del mismo: dieta absoluta, fluidoterapia, antibiticos y corticoides
parenterales.

pia coadyuvante al tratamiento quirrgico (esta quimioterapia suele


ser a base de 5-FU y leucovorn, aunque ahora hay nuevos
quimioterpicos en juego como el oxaliplatino y el irinotecn).

Pregunta 10.- R: 3
Dada su evolucin insatisfactoria a las 24 h, hay una indicacin
quirrgica evidente, de urgencia. Dado que se trata de un colon no
preparado, severamente dilatado y con una enfermedad de base que
afecta a toda su extensin, lo que habr que realizar es una colectoma
total sin anastomosis (haciendo una ileostoma terminal temporal y
difiriendo a un segundo tiempo la realizacin de una anastomosis y la
extirpacin del recto). Todas las otras opciones ofrecidas en esta pregunta son insuficientes y algunas (la 2 y la 4) entraan un riesgo inaceptable.

Pregunta 16.- R: 4
La afectacin linftica define al estadio C en la clasificacin de
Dukes y en todas sus modificaciones posteriores.
Las lesiones del colon derecho se manejan con hemicolectoma
derecha (ampliada a parte del transverso si la lesin est en el ngulo
heptico) y en la mayora de los casos se puede realizar anastomosis
primaria, incluso si son lesiones obstructivas (salvo en casos de perforacin con gran contaminacin local que pondra en riesgo la anastomosis).
Por el peculiar drenaje venoso del recto distal (a vasos ilacos y no
al sistema portal), es posible en esta localizacin la diseminacin al
pulmn sin existir enfermedad heptica. Por eso en esta situacin es
importante la Rx de trax.
La reseccin de metstasis hepticas aisladas de cncer colorrectal
ofrece una supervivencia de hasta el 45 % a los 3 aos.

Pregunta 11.- R: 3
Los pacientes con EII tienen riesgo aumentado de cncer de colon
con respecto a la poblacin general (sobre todo los afectados por
colitis ulcerosa). Este riesgo aumenta con el tiempo de evolucin y
con la extensin de la enfermedad, y estos tumores presentan como
peculiaridad que los diferencia del cncer en otro contexto, el hecho
de que con cierta frecuencia son tumores multifocales (mltiples) y se
presentan en el colon ascendente con una frecuencia superior a la de
la poblacin general (donde la localizacin ms habitual es recto y
sigma).
Su pronstico y tratamiento son iguales que los de un cncer de
iguales caractersticas que aparece fuera del contexto de la EII.
Pregunta 12.- R: 5
El Metronidazol oral es el antibitico que se suele utilizar en tratamiento prolongado para los pacientes con enfermedad de Crohn
perianal que presentan fstulas, con la intencin de que no se formen
abscesos. El Metronidazol tiene el inconveniente de producir neuropata perifrica hasta en un 50% de los pacientes cuando el tratamiento dura 6 meses o ms.
Como alternativas en el tratamiento de la enfermedad perianal (de
menos a ms agresiva en funcin de los fracasos teraputicos previos),
se puede intentar la fistulectoma y el cierre de fstulas con colgajos de
avance, la colostoma de derivacin para evitar que las heces sean
fuente de infeccin para las fstulas y, hasta en el 30% de los pacientes
con afectacin perianal severa, la amputacin abdominoperineal.
Pregunta 13.- R: 1
Ante todo sndrome abdominal, tacto rectal... y vaginal. Este es un
aforismo clsico de la ciruga francesa y debera de ser parte integrante
de nuestra prctica diaria. Estas dos exploraciones pertenecen a la
exploracin fsica completa, que debe preceder a toda prueba complementaria.
En el caso concreto de una paciente que refiere rectorragia, independientemente del contexto clnico de la misma, el tacto rectal es
absolutamente imperativo e inexcusable.
Pregunta 14.- R: 1
Todos los tumores digestivos, a excepcin del esfago superior y
del recto distal-canal anal tienen un drenaje venoso portal, por lo que
su lugar habitual de metstasis es el hgado. Aunque el hgado puede
valorarse convenientemente con la ecografa, sin embargo sta no nos
permite hacernos idea de la masa tumoral en el colon, sus relaciones
con otras estructuras anatmicas, la existencia de adenopatas, etc.
igual de bien que lo hace la TAC, adems de que la ecografa tiene
unas imgenes de difcil interpretacin por quien no la est llevando
a cabo.
Pregunta 15.- R: 3
Dado que no hay adenopatas ni metstasis, podemos descartar los
estadios C y D, respectivamente. El tumor se extiende a TODA la capa
muscular, lo cual implica necesariamente que alcanza la serosa (la
afectacin de serosa es la que marca el 1 el 2, tanto en el B como en
el C). Por ello se trata de un estadio B2, que justamente es el estadio en
el cual est indicado en el cncer colorrectal aadir una quimiotera-

Pg. 2 CG

M exico A rgentina
C hile U ruguay

Pregunta 17.- R: 4
Nos encontramos ante una metstasis sincrnica de un cncer
colorrectal (se descubre a la vez que el tumor primario, y no en el
seguimiento posterior, en cuyo caso se llamara metstasis metacrnica).
Las sincrnicas tienen peor pronstico, pero ello no contraindica su
reseccin si cumplen los criterios actualmente aceptados para ello:
Enfermedad primaria controlada y sin recidiva ni metstasis a otro
nivel.
Enfermo capaz de tolerar una reseccin heptica.
Metstasis en nmero de 3 menos.
Deseable que las metstasis estn anatmicamente cerca, aunque
no imprescindible.
Estos son los factores que se acepta que marcan la indicacin de
resecar las metstasis, con un margen sano de 1 cm (no es necesario
hacer resecciones mayores si se pueden quitar las metstasis con
resecciones menores). Sin embargo, estos criterios en realidad no
son tan estrictos actualmente y se interpretan ms bien como marcadores de mejor pronstico, extendindose cada vez ms las indicaciones para resecar las metstasis que pueden aparecer en el cncer
colorrectal.
Un tema en discusin es si resecar las metstasis sincrnicas en el
mismo acto operatorio o diferir la metastasectoma, sin embargo en
este caso parece una lesin pequea y accesible fcilmente, por lo
que puede hacerse en el mismo acto.
Pregunta 18.- R: 5
Nos encontramos ante un paciente que consulta por un cuadro de
tipo obstructivo, progresivo, con aparente sndrome constitucional de
fondo. Desde luego, el paciente no est clnicamente en situacin de
gravedad que nos obligue a una laparotoma urgente (no nos cuentan
siquiera un cuadro de abdomen agudo). Parece lgico iniciar el estudio con una analtica general y una radiologa simple de abdomen
para valorar dilatacin de asas y nivel de la obstruccin, adems de
dejarle en dieta absoluta, con sonda nasogstrica y fluidoterapia (como
a todo cuadro obstructivo).
Pregunta 19.- R: 2
Ya con los datos de la pregunta anterior, ante un cuadro obstructivo
progresivo asociado a un sndrome constitucional, sospechamos un
cncer de colon. Si adems nos encontramos ahora una anemia asociada, nuestra primera opcin diagnstica ser esa. Un cncer gstrico obstructivo se manifestara de forma ms rpida (no tan larvada) y
con un cuadro predominante de vmitos (como corresponde a una
obstruccin alta) y no de estreimiento. Por otra parte, no hay que
olvidar que el cncer de colon izquierdo es la causa ms frecuente de
obstruccin del colon.
Pregunta 20.- R: 3
Sin embargo, pese a ser ms frecuente la obstruccin por cncer
de colon izquierdo, la existencia de anemia asociada es bastante ca-

CTO Medicina C/ Nez de Balboa, 115 28006 MADRID (Espaa) Tfno.: (91) 782 43 32 / Fax: (91) 782 43 27
E-mail: secretaria@ctomedicina.com; iberocto@ctomedicina.com WEB: www.ctomedicina.com; www.iberocto.com

Comentarios TEST

Seguimiento a distancia

CIRUGA GENERAL

Preparacin Examen de Seleccin 05/06 1 Vuelta


racterstica de la patologa del colon derecho (no olvidemos que el
colon derecho sangra y el colon izquierdo se obstruye) y si el tumor
es voluminoso se presenta igualmente como obstruccin. Dado que
este es el diagnstico de nuestro paciente y que la localizacin derecha es favorable a una anastomosis primaria salvo situaciones catastrficas, el tratamiento de eleccin ser la reseccin segmentaria y
anastomosis.
Pregunta 21.- R: 4
Nos encontramos ante una paciente que consulta de forma ambulatoria (no urgente, ojo!) por un cuadro de rectorragias. En este contexto entran en juego muchas patologas: enfermedad diverticular, EII,
cncer de colon, hemorroides y otra patologa anorrectal benigna,
angiodisplasia intestinal y otras patologas vasculares. Habr que hacer una analtica bsica para ver la repercusin de estas rectorragias y
detectar una posible anemia. Por otra parte, una colonoscopia o un
enema opaco nos darn informacin preciosa sobre la causa del
sangrado y su localizacin. La arteriografa, aunque no se pedira de
entrada, puede ser necesaria ms adelante para evaluar una patologa
vascular intestinal. Sin embargo, la TC no es una prueba de imagen
que nos vaya a dar gran informacin sobre un punto de sangrado en
la luz del tubo digestivo.
Pregunta 22.- R: 4
En el contexto clnico en que nos encontramos (consulta no urgente) y en un cuadro donde predomina la rectorragia esencialmente
(no se nos habla del dolor para nada), podemos descartar de entrada
la isquemia mesentrica (aguda o crnica), dado que su rasgo
prototpico y elemental es el dolor (como en todo cuadro isqumico
a excepcin de los ACVAs).

Pregunta 22. Causas de hemorragia digestiva baja segn la edad.


Menos de 55 aos

Ms de 55 aos

Comentarios TEST

1. Enfermedad anorrectal.
2. Colitis (EII, infecciosas).
3. Diverticulosis.
4. Plipos, cncer (hiperplasia,
hamartomas).
5. Angiodisplasia.

1. Enfermedad anorrectal.
2. Diverticulosis.
3. Angiodisplasia.
4. Plipos, cncer.
5. Enterocolitis (isqumica,
infecciosa, EII, radiacin).

Pregunta 23.- R: 4
La paciente presenta un abdomen agudo (dolor abdominal de
inicio agudo y signos de irritacin peritoneal), focalizado en fosa iliaca
izquierda. Ante esta clnica, nuestra primera sospecha diagnstica es
siempre una diverticulitis.
Inicialmente, se har la valoracin inicial de todo abdomen agudo, es decir una anamnesis y exploracin, con una analtica elemental. En el abdomen agudo es conveniente realizar una RX de
trax, puesto que hay patologa torcica que se presenta como abdomen agudo (tpicamente las neumonas de lbulo inferior) y, adems, podemos detectar un neumoperitoneo en caso de perforacin; tambin la radiologa de abdomen puede mostrar hallazgos
interesantes. Y en esta pregunta nos piden lo que solicitaremos en
primer lugar, y no cul sera la mejor prueba de imagen ante una
sospecha de diverticulitis, que ya hemos dejado claro que es la TAC.
Toda opcin que incluya endoscopia o pruebas con contraste
debe rechazarse por aumentar el riesgo de perforacin (al paciente
obstruido o con abdomen agudo, por arriba slo la SNG y por abajo
slo el dedo: nicas excepciones, el vlvulo de sigma y la colonoscopia descompresiva del Ogilvie).
Pregunta 24.- R: 3
En relacin con las complicaciones de la lcera gastroduodenal,
son por orden de frecuencia: hemorragia, perforacin y estenosis.
Actualmente, la que con ms frecuencia requiere ciruga es la perforaM exico A rgentina
C hile U ruguay

Seguimiento a distancia

cin, dado que la mayora de las hemorragias se manejan con xito


de forma conservadora.
La hemorragia suele suceder en la cara posterior, afectando ramas de la pared, y no a los troncos de la arteria gastroduodenal o de
la pilrica, que no se encuentran incluidas en el espesor de la pared.
La perforacin, por el contrario, suele suceder en cara anterior.
La penetracin es el nombre especial que recibe la perforacin
que sucede en cara posterior, generalmente hacia el pncreas, de
modo que queda contenida y no aparece una comunicacin libre
a cavidad peritoneal, con lo cual no aparece la peritonitis. En lo
tocante a la presentacin de la perforacin ulcerosa, suele iniciarse como un dolor intenso, brusco en su aparicin, bien localizado en epigastrio, a veces irradiado a espalda, hombro o fosa
iliaca (esto ltimo por acumulacin del cido a ese nivel). Raramente aparecen vmitos (el perforado no es vomitador, se afirma
clsicamente). La exploracin muestra un paciente afectado, bastante inmvil, con la forma extrema de la irritacin peritoneal: el
abdomen en tabla (o peritonitis), por el cido libre. Es clsico, pero
de escasa utilidad, el signo de la prdida de la matidez heptica,
causado por el neumoperitoneo. Se considera que la peritonitis
por perforacin ulcerosa es estril hasta que pasan unas 6 horas
del suceso. Ante este cuadro clnico, se debe buscar confirmar el
neumoperitoneo con la realizacin de una radiografa de trax en
bipedestacin (mostrar aire libre bajo las cpulas diafragmticas)
o, si el paciente no tolera la bipedestacin, una radiografa de
abdomen en decbito lateral con rayo horizontal.
La estenosis pilrica es una urgencia diferible en el sentido de
que su correccin no precisa realizarse de forma inmediata y es
conveniente rehidratar al paciente y restaurar una nutricin aceptable y una homeostasis correcta antes de ir a la solucin definitiva,
que es quirrgica (en ocasiones es resectiva y en otras la piloroplastia
basta).
Pregunta 25.- R: 4
Ver comentario anterior y el grfico siguiente.
Rx de trax
Da el diagnstico
S

No

SNG
+
introducir aire
y repetir Rx de trax
da el diagnstico
S

No
TC abdomen

Pregunta 25. Diagnstico de la lcera gastroduodenal perforada.

Pregunta 26.- R: 5
Hasta hace unos pocos aos se habra dado como opcin correcta
en esta pregunta la 1. Sin embargo, las cosas han cambiado mucho en
la ciruga de la lcera con la llegada del Helicobacter pylori y los
inhibidores de la bomba de protones.
Actualmente se recomienda en las complicaciones quirrgicas
de la lcera (hemorragia y perforacin), limitarse a solucionar la
complicacion aguda (es decir, hacer hemostasia en el caso de la
hemorragia y cerrar la perforacin en caso de que exista) SIN asociar
ningn gesto definitivo de los que se preconizaban clsicamente
(que eran la vagotoma troncular + piloroplastia en la mayora de
los casos). En los casos de lceras gstricas de muy gran tamao y
con intenso componente fibrtico-inflamatorio est indicada todava la reseccin gstrica, pero esta situacin es poco frecuente en la
actualidad.

CTO Medicina C/ Nez de Balboa, 115 28006 MADRID (Espaa) Tfno.: (91) 782 43 32 / Fax: (91) 782 43 27
E-mail: secretaria@ctomedicina.com; iberocto@ctomedicina.com WEB: www.ctomedicina.com; www.iberocto.com

EP Pg. CG

Seguimiento a distancia

CIRUGA GENERAL

Preparacin Examen de Seleccin 05/06 1 Vuelta

Todava quedara una indicacin (ms terica que prctica) para tcnicas definitivas (vagotomas o antrectomas) en pacientes con lcera
con Helicobacter negativo o en aquellos con muchos factores para
recidivar (recidivas previas tras tratamiento correcto de una afectacin
ulcerosa crnica), pero estas situaciones son realmente excepcionales.

afectacin localmente irresecable, no hay carcinomatosis peritoneal),


aunque la existencia de ganglios ensombrece algo el pronstico.
En estas circunstancias est indicada una intervencin quirrgica
con intencin curativa (oncolgica), que en el cncer gstrico se
basa en la realizacin de una reseccin gstrica (total o subtotal, en
este caso total porque hay afectacin del cuerpo y no slo del antro)
+ linfadenectoma + omentectoma. En ocasiones se asocia a esta
reseccin la esplenectoma o la pancreatectoma distal, bien porque haya afectacin focal del pncreas o, ms frecuentemente, para
poder asegurar una buena linfadenectoma.
En el dibujo de la pgina siguiente puedes ver la tcnica quirrgica
de la gastrectoma y linfadenectoma del adenocarcinoma gstrico.
Pregunta 28.- R: 1
Las lceras de Cushing son lceras gstricas por hipersecrecin en
pacientes con patologa del sistema nervioso central (traumtica o
tumoral). Se deben distinguir de cara al MIR de las lceras de Curling,
que aparecen en situaciones de hipovolemia aguda (sobre todo en
quemados) y cuya patogenia parece ms relacionada con una hipoperfusin de la pared gstrica por la hipovolemia.
Para acordarse de quin es quin puede sernos de utilidad recordar a Harvey Cushing, padre de la neurociruga, que dio su nombre al
sndrome y enfermedad de Cushing por produccin hipofisaria excesiva de ACTH. De esta manera podremos relacionar su nombre con la
neurociruga y, por lo tanto, con patologa del sistema nervioso central.
Pregunta 29.- R: 4
Ver comentario de la pregunta 32.
Pregunta 30.- R: 1
Ver comentario de la pregunta 32.
Pregunta 31.- R: 4
Ver comentario de la pregunta 32.
Pregunta 32.- R: 4
Nos encontramos con un caso clnico en el que nos presentan la
complicacin ms frecuente de la colelitiasis: el clico biliar. Aparece
cuando un clculo intenta salir de la vescula biliar, induciendo una
respuesta vagal refleja: espasmo de musculatura lisa (con el consiguiente dolor clico), nuseas, vmitos, sudor fro, malestar general. El
paciente presenta exclusivamente sntomas (destaca el dolor clico
en hipocondrio derecho), siendo la exploracin anodina (no hay
irritacin peritoneal ni otros signos) y la analtica rigurosamente normal. En este contexto de exploracin y analtica normales, no hay
indicacin de solicitar prueba de imagen urgente.
El manejo de esta situacin consiste en un tratamiento sintomtico
durante la fase aguda (espasmolticos y analgsicos), con lo cual el
episodio cede y el paciente es dado de alta. De forma programada se
realizar una ecografa para comprobar la existencia de colelitiasis y,
dado que ha presentado sntomas, existe indicacin de colecistectoma electiva (preferentemente laparoscpica).
La pieza de colecistectoma que enviemos al patlogo ser informada probablemente como colelitiasis (algo que ya suponamos) y
colecistitis crnica. Este ltimo hallazgo es el correlato anatomopatolgico del clico biliar, de manera que cualquier persona que es
portadora de una colelitiasis (muchas veces aunque sea asintomtica)
desarrolla habitualmente un infiltrado inflamatorio crnico a nivel de
la vescula. Ello explica que en algunos textos aparezca como complicacin ms frecuente de la colelitiasis el clico biliar y en otros la
colecistitis crnica: son las dos caras de la misma moneda, una vista
por el clnico y otra por el patlogo. (Ver tabla).

Pregunta 26. Intervencin de Billroth II.

Pregunta 27.- R: 2
Nos encontramos ante un cncer gstrico que afecta cuerpo y antro
y que no presenta criterios de irresecabilidad (no hay metstasis, no hay

Pg. 4 CG

M exico A rgentina
C hile U ruguay

Pregunta 33.- R: 4
Nos encontramos aqu con otra complicacin de la colelitiasis: la
colecistitis aguda. En este caso el paciente presenta un abdomen agudo (dolor abdominal de inicio agudo acompaado de signos de irritacin peritoneal, como es la defensa). Un abdomen agudo focalizado
en hipocondrio derecho nos sugiere como primer diagnstico una
colecistitis aguda. El dolor irradiado a espalda, la febrcula y el dolor al
palpar en hipocondrio derecho y epigastrio son caractersticos. Aun-

CTO Medicina C/ Nez de Balboa, 115 28006 MADRID (Espaa) Tfno.: (91) 782 43 32 / Fax: (91) 782 43 27
E-mail: secretaria@ctomedicina.com; iberocto@ctomedicina.com WEB: www.ctomedicina.com; www.iberocto.com

Comentarios TEST

Pregunta 26. Intervencin de Billroth I

CIRUGA GENERAL

Comentarios TEST

Preparacin Examen de Seleccin 05/06 1 Vuelta

Seguimiento a distancia

Pregunta 27. Gastrectoma: tcnica quirrgica.Linfadenectoma segn el territorio afectado.

que no se refiera en este caso, el signo de Murphy es tpico de la


colecistitis, y consiste en que la inspiracin del paciente se corta cuando presionamos en hipocondrio derecho sobre la localizacin hipottica de la vescula (el signo de Murphy ecogrfico es lo mismo, pero
hecho por el ecografista, y tiene ms valor diagnstico por el correlato
de la imagen). Analticamente aparecen hallazgos de infeccin
M exico A rgentina
C hile U ruguay

(leucocitosis, neutrofilia, en casos graves desviacin izquierda), sin otros


hallazgos particulares (ntese que NO hay colestasis significativa ni
citlisis, luego la bilirrubina y los enzimas hepticos no estn
significativamente alterados).
El diagnstico de colecistitis aguda se confirma con la ecografa,
que mostrar signos indirectos de colecistitis: distensin, edema peri-

CTO Medicina C/ Nez de Balboa, 115 28006 MADRID (Espaa) Tfno.: (91) 782 43 32 / Fax: (91) 782 43 27
E-mail: secretaria@ctomedicina.com; iberocto@ctomedicina.com WEB: www.ctomedicina.com; www.iberocto.com

EP Pg. CG

CIRUGA GENERAL

Preparacin Examen de Seleccin 05/06 1 Vuelta

vesicular, pared engrosada, colelitiasis en el 90% de los casos (existe la


colecistitis alitisica, sobre todo en pacientes crticos), a veces signo de
la doble pared.
El tratamiento de la colecistitis, como de todas las complicaciones
de la colelitiasis es, a la postre, quirrgico. Sin embargo, se ha discutido mucho sobre el momento ideal para la ciruga. Parece claro que
hay que evitar realizar la colecistectoma entre el da 3 y el 30 del
episodio inflamatorio, por lo que se proponen dos actitudes igualmente vlidas:
Operar en el momento de hacer el diagnstico, sin demorar la intervencin.
Realizar un tratamiento antibitico inicial al hacer el diagnstico y
reevaluar a las 48 h si est siendo eficaz. Si la evolucin es favorable, completar el tratamiento antibitico (enfriar la colecistitis)
para luego hacer ciruga electiva pasados 2 3 meses; si la evolucin es desfavorable, indicar ciruga urgente antes de que entremos
en la fase indeseable que se inicia hacia el da 3.
En este caso, para que no tengamos dudas, nos ponen un paciente
anciano, cardipata y anticoagulado, de manera que la opcin conservadora aparece como la ms adecuada.
Pregunta 34.- R: 1
La coledocolitiasis aparece cuando un clculo sale de la vescula y
obstruye la va biliar principal, provocando una colestasis (y si se localiza
cerca de la papila, tambin una pancreatitis litisica). Muchas veces el
clculo pasa solo al tubo digestivo, pero en ocasiones se hace necesario
extraerlo, siendo para ello de utilidad la CPRE con extraccin del clculo
y esfinterotoma asociada (antes de la aparicin de la CPRE, esta situacin
se solucionaba con ciruga abierta: coledocotoma y extraccin de clculos, opcin que sigue siendo vlida si la CPRE no es resolutiva).
Normalmente, una vez resuelta la fase aguda de colestasis mediante CPRE, se indica colecistectoma electiva. Sin embargo, cuando la
coledocolitiasis aparece en pacientes ya colecistectomizados, la CPRE
se convierte en la solucin definitiva, siendo sta su indicacin ms
clsica y resolutiva.
La colecistitis aguda calculosa (supone el 90% de las colecistitis,
como vimos en la pregunta anterior) aparece con ms frecuencia en
mujeres, lo cual es lgico si tenemos en cuenta que la prevalencia de
colelitiasis es mucho mayor en el sexo femenino. La colecistitis alitisica (acalculosa), sin embargo, es ms frecuente en varones y en
pacientes crticos o con patologa de base grave (quemados, poli-

traumatizados, SIDA, enfermedad inflamatoria intestinal severa, tras


circulacin extracorprea, tras nutricin parenteral prolongada...).
La colecistitis enfisematosa es un tipo de colecistitis aguda que puede ser litisica o alitisica, que aparece tpicamente en diabticos y es
originada por anaerobios (ello explica la presencia de gas en las vas
biliares o aerobilia). Se presenta como un cuadro sptico muy grave y
con alta mortalidad.
Como norma general (las excepciones se indican en el libro), NO
est indicada la colecistectoma en pacientes asintomticos o con
sntomas no atribuibles a su colelitiasis.
El epitelio de la vescula es glandular, por lo que la neoplasia ms
frecuente a este nivel es el adenocarcinoma. Ver dibujos en pginas 6 y 7.

Pregunta 34. Lugares ms frecuentes de impactacin de clculos biliares.

Pregunta 34. Esfinterotoma endoscpica.

Pg. 6 CG

M exico A rgentina
C hile U ruguay

CTO Medicina C/ Nez de Balboa, 115 28006 MADRID (Espaa) Tfno.: (91) 782 43 32 / Fax: (91) 782 43 27
E-mail: secretaria@ctomedicina.com; iberocto@ctomedicina.com WEB: www.ctomedicina.com; www.iberocto.com

Comentarios TEST

Seguimiento a distancia

CIRUGA GENERAL

Preparacin Examen de Seleccin 05/06 1 Vuelta

Seguimiento a distancia

Pregunta 37.- R: 2
La actitud inicial tras diagnosticar un pseudoquiste es realizar una
vigilancia peridica con ecografas seriadas, durante un perodo aproximado de 6 semanas. Si durante este tiempo disminuye de tamao por
debajo de los 6 cm, la actitud ser conservadora (ello sucede ms frecuentemente con los pseudoquistes de la pancreatitis aguda); si persisten en su tamao por encima de 6 cm (lo cual es ms frecuente en la
pancreatitis crnica), el riesgo de complicaciones es mayor (rotura, hemorragia) y por lo tanto se indica drenaje quirrgico a un asa de intestino delgado (cistoyeyunostoma en Y) o al estmago (cistogastrostoma).
Pregunta 38.- R: 1
Vase comentario anterior.
Pregunta 39.- R: 4
El paciente presenta un cuadro de dolor abdominal y sepsis. La
existencia de un nivel hidroareo en el pseudoquiste confirmar la
sobreinfeccin del mismo, por lo que nos hallamos ante una grave
complicacin del quiste: la sepsis pancretica.
En condiciones normales, como indicamos en la pregunta 37, el
tratamiento de los pseudoquistes es el drenaje interno, evitndose las
punciones percutneas y derivaciones externas, ya que generan fstulas
de muy difcil manejo. Sin embargo, en condiciones crticas (como lo
es la sepsis pancretica), el riesgo vital del enfermo es grave y el peligro
de la fstula pancretica se convierte en un mal menor, de manera
que se trata de drenar el foco sptico lo antes posible de la forma menos
agresiva. Por ello elegimos en esta situacin el drenaje percutneo.
Por otra parte, el hecho de que la lesin se haya abscesificado nos
habla a favor de su independencia fsica (ser una coleccin cerrada,
probablemente sin comunicacin con el Wirsung), lo cual nos tranquiliza algo en relacin con el riesgo de fstula pancretica.

Comentarios TEST

Pregunta 34. Colangitis por coledocolitiasis.

Pregunta 35.- R: 5
Se nos presenta un cuadro clnico caracterstico de una pancreatitis,
con algunos signos de especial gravedad, como es el hecho de que
presente coloracin azulada en flancos (sugiere pancreatitis necrohemorrgica), hipocalcemia y derrame pleural. Sin embargo, por mucha
que sea la gravedad, los pacientes con pancreatitis aguda slo parecen
beneficiarse de la ciruga (segn la evidencia actual) cuando existe una
necrosis que se demuestra infectada mediante cultivo de material obtenido por puncin, situacin en la cual se indica realizar una
necrosectoma. Fuera de este supuesto, la ciruga slo aade morbilidad
y dispara la mortalidad de estos enfermos, que ya de por s es elevada.

Pregunta 40.- R: 4
Los adenomas son neoplasias benignas del hgado. Clsicamente
se relaciona su aparicin con el uso de esteroides (anticonceptivos
en mujeres, que constituyen el grupo ms numeroso, o testosterona
en varones), disminuyendo o a veces incluso desapareciendo cuando se abandona el consumo de estos frmacos.

Pregunta 40. Tumores hepticos benignos.


Adenoma
hepatocelular

Hiperplasia
nodular focal

Hemangioma (es
el ms frecuente)

Paciente tipo

M u je r +
a nticonceptivos

M u je r

M u je r

Clnica

Dolor a bdom ina l


inespecfico

Ha lla zgo

M a sa pa lpa ble
Hem orra gia
Shock
Complicaciones
hipovolm ico
M a ligniza cin (si
m s de 6 cm )

Pregunta 35. Criterios de Ransom a las 48 horas.

Pregunta 36.- R: 3
El cuadro que se refiere que present el paciente en su anterior
ingreso es, evidentemente, una pancreatitis aguda. Ahora regresa con
clnica, lo cual nos debe hacer sospechar una complicacin de aquel
episodio reciente. En concreto, presenta masa palpable en mesogastrio junto con clnica de efecto masa (saciedad, vmitos). Independientemente de lo que sea el cuadro en cuestin, su origen ser probablemente pancretico: dado que sabemos la localizacin, la mejor
prueba de imagen en el pncreas es el TC abdominal.
M exico A rgentina
C hile U ruguay

Tratamiento

Ciruga

No (si ha sido
correcta m ente
dia gnostica do)

No (slo si es
gra nde y/o da n
s n t o m a s )

Tambin aparecen en el contexto de las enfermedades por depsito de glucgeno, situacin mucho ms rara. Hasta el 80% de los
pacientes presentan algn tipo de molestia abdominal, si bien son
sntomas muy inespecficos. Sus dos potenciales complicaciones son
la hemorragia en el seno del tumor, que puede llegar a romperse a la
superficie heptica y causar un hemoperitoneo grave (esto sucede

CTO Medicina C/ Nez de Balboa, 115 28006 MADRID (Espaa) Tfno.: (91) 782 43 32 / Fax: (91) 782 43 27
E-mail: secretaria@ctomedicina.com; iberocto@ctomedicina.com WEB: www.ctomedicina.com; www.iberocto.com

EP Pg. CG

aproximadamente en un 30% de los adenomas), y la posibilidad de


malignizacin (hasta un 10% de los adenomas extirpados contienen
un hepatocarcinoma). Por ello, se recomienda una actitud quirrgica ante adenomas que no regresan al interrumpir el tratamiento
hormonal, as como en los de mayor tamao.
Pregunta 41.- R: 2
Los hemangiomas son el tumor benigno ms frecuente del hgado, siendo francamente prevalentes (aparecen entre el 2% y el 7%
de las autopsias). Su aparicin no se relaciona especialmente con
ningn consumo farmacolgico. Habitualmente son asintomticos,
no se complican y no causan ningn tipo de problemas, salvo que
sean de gran tamao (se llaman gigantes a partir de los 4 cm). El
diagnstico puede establecerse a menudo por su apariencia ecogrfica y con ayuda del Doppler, si bien la RM es de utilidad para
caracterizarlos en caso de duda por su alta sensibilidad y especificidad. Dada su evolucin benigna, no est indicado ningn tipo de
tratamiento en estos tumores.
Sin embargo, en algunos casos en los que el tumor es voluminoso y
aparece clnica de ocupacin abdominal o molestia, est indicada la
reseccin del tumor como tratamiento sintomtico. Ms raramente
aparecen en el contexto de cuadros angiomatosos sistmicos (sndrome
de Kasabach-Merritt), asociados a coagulopata de consumo y trombocitopenia, siendo incluso necesario el trasplante de forma excepcional.
Pregunta 42.- R: 3
El trmino tumor de Klatskin se refiere exclusivamente al colangiocarcinoma que aparece en la unin de ambos conductos hepticos,
en la vecindad de la placa hiliar.
El trasplante es, sin duda, el tratamiento que ofrece mejores resultados al hepatocarcinoma. Sin embargo, dada la escasez de rganos y
la altsima probabilidad de recidiva en los casos que presentan muchos factores de mal pronstico, actualmente se restringen las indicaciones a pacientes con enfermedad de buen pronstico (no metstasis, un tumor nico menor de 5 cm o hasta tres tumores de menos de
3 cm) y cuya situacin funcional heptica impide la reseccin (estadio B C de Child). En los estadios A de Child en los que la reseccin
es factible, habitualmente se recurre a ella.
El angiosarcoma es una neoplasia de muy mal pronstico, cuya
relacin con la exposicin al cloruro de vinilo es bien conocida.
El carcinoma fibrolamelar es una neoplasia heptica de mejor
pronstico que el carcinoma hepatocelular. Sin embargo, hay varios trabajos recientes que afirman que el mejor pronstico obedece exclusivamente al hecho de que aparece en pacientes jvenes y
sobre un hgado sano, sin que histopatolgicamente pueda distinguirse del carcinoma hepatocelular del que en definitiva es parte.
Pregunta 43.- R: 5
Ver pregunta 45.
Pregunta 44.- R: 1
Ver pregunta 45.
Pregunta 45.- R: 2
El caso es el de un paciente de 70 aos que presenta dolor abdominal
y prdida de peso, as como ictericia. En este contexto hay que investigar
una neoplasia abdominal que afecte a la va biliar (bien sea primaria del
rea hepatobiliopancretica, o bien sea primaria de otro rgano y ha
afectado en su crecimiento a la va biliar, como podra ser un cncer
gstrico). El hecho de que el paciente no presente otra clnica digestiva
hace poco probable el cncer gstrico, que raramente debuta con ictericia. En este contexto, lo ms probable es un tumor periampular
(adenocarcinoma de cabeza pancretica, ampuloma, colangiocarcinoma
distal o adenocarcinoma duodenal), de los cuales el ms frecuente es el
cncer de la cabeza del pncreas (adenocarcinoma ductal).
La ecografa abdominal constituye el estudio inicial para cualquier
ictericia obstructiva. La CPRE tambin puede ser de utilidad en este
contexto, para diagnstico diferencial con una coledocolitiasis, para
tomar muestras para estudio citolgico y para colocar una prtesis
paliativa biliar en caso de enfermedad irresecable. La TC es imprescindible como estudio de extensin (local y a distancia). Finalmente,
Pg. 8 CG

M exico A rgentina
C hile U ruguay

CIRUGA GENERAL

Preparacin Examen de Seleccin 05/06 1 Vuelta

el estudio gastroduodenal puede ser de utilidad en el adenocarcinoma duodenal o en el ampuloma, permitiendo ver la masa. Sin embargo, las pruebas de medicina nuclear no aportan nada en el diagnstico de los tumores periampulares.
El pronstico de los tumores periampulares es globalmente malo,
sobre todo por la gran preponderancia del cncer de pncreas (el
ampuloma y el colangiocarcinoma tienen mejor pronstico, sobre todo
el primero). El tratamiento con intencin curativa de todos ellos (si el
estudio de extensin no lo contraindica), es la duodenopancreatectoma ceflica (intervencin de Whipple, descrita en 1935 por vez primera), al cual llegan slo un 20 % de los cnceres de pncreas. Tras un
Whipple (ciruga que tiene una mortalidad de hasta el 15 % en algunas
series), el pronstico es an as malo en el caso del pncreas, sobreviviendo a los 5 aos menos del 20% de los enfermos resecados con
intencin curativa.
La quimioterapia y la radioterapia tienen en el cncer de pncreas
un papel esencialmente paliativo, aunque hay ensayos recientes que
apuntan a una indicacin neoadyuvante (es decir, preoperatoria para
reducir el tamao tumoral).
Pregunta 46.- R: 3
El trmino abdomen agudo se refiere al sndrome compuesto por
la aparicin de dolor abdominal de inicio agudo y signos de irritacin
peritoneal (hipersensibilidad, contractura, rebote). Sin embargo, el
hecho de que un paciente presente un abdomen agudo NO indica la
necesidad de ciruga. Hay muchas causas de abdomen agudo que no
son subsidiarias de ciruga (ver tabla).
En esta pregunta se nos presentan patologas mdicas que se perforan (la fiebre tifoidea lo hace en el 4% de los casos, la colitis por CMV
lo hace ms raramente, el carbunco digestivo puede perforarse y tambin lo hacen hasta el 8% de las lceras duodenales). Sin embargo, la
PAI causa una neuropata que manifiesta crisis severas de dolor abdominal, pero sin afectacin del tubo digestivo (se afectan los plexos
nerviosos, pero no las vsceras).

Pregunta 46.Causas no quirrgicas de dolor abdominal.


Cardacas.

IAM.
Pericarditis aguda.

Pulmonares.

Neumona.
Infarto pulmonar.

Gastrointestinales.

Pancreatitis aguda.
Enfermedad de Crohn.
Adenitis mesentrica.
Diverticulitis no complicada.
Gastroenteritis.
Hepatitis aguda.

Endocrinas.

Insuficiencia SR aguda.
Cetoacidosis diabtica.

SNC y perifrico.

Tabes dorsal
Compresin races nerviosas.

Metablicas.

Porfiria aguda.
Fiebre mediterrnea familiar.
Hiperlipemia.

Hematolgicas.

Crisis drepanoctica.

Ginecolgicas.

EIP.
Embarazo ectpico.
Endometriosis.
Quiste ovrico torsionado.

Pregunta 47.- R: 1
Ver comentario de la pregunta siguiente.

CTO Medicina C/ Nez de Balboa, 115 28006 MADRID (Espaa) Tfno.: (91) 782 43 32 / Fax: (91) 782 43 27
E-mail: secretaria@ctomedicina.com; iberocto@ctomedicina.com WEB: www.ctomedicina.com; www.iberocto.com

Comentarios TEST

Seguimiento a distancia

CIRUGA GENERAL

Preparacin Examen de Seleccin 05/06 1 Vuelta

Comentarios TEST

Pregunta 48.- R: 4
La apendicitis aguda es la causa ms comn de abdomen agudo,
cifrndose en un 7% el riesgo de padecerla a lo largo de la vida. Es ms
rara en los extremos de la vida, siendo excepcional en nios menores
de 2 aos.
Clnicamente aparece un dolor periumbilical o epigstrico con
anorexia, que en las horas siguientes se sita en fosa iliaca derecha,
apareciendo entonces los vmitos y la febrcula o fiebre (raramente
mayor de 38 C si no est complicada). Por ello, cuando los vmitos
preceden al dolor se deben de considerar otros diagnsticos (especialmente una gastroenteritis aguda).
Como en todo abdomen agudo, son imprescindibles los signos
de irritacin peritoneal, que en este caso son variopintos. Sin embargo, el que tiene mayor valor de todos ellos de cara al diagnstico es
el signo de Blumberg (rebote positivo en fosa ilaca derecha). El diagnstico es clnico en la mayora de los casos (la experiencia del
cirujano es fundamental), pero hasta en un 25% existen dudas diagnsticas y se puede recurrir a la ecografa o la TC. An as, el 20% de
las laparotomas por apendicitis NO presentan apendicitis aguda.
En los lactantes y en los ancianos, el cuadro puede ser menos
manifiesto, y sin embargo evoluciona ms rpidamente a la perforacin. En el caso de los ancianos es caracterstico que el cuadro se
presente con caractersticas obstructivas, debido al leo local o al
atrapamiento de algn asa en el plastrn apendicular. Tanto en unos
como en otros puede existir ausencia de leucocitosis en la analtica.
La perforacin no debera de llegar al 10% en el caso de los
adultos, y suele acompaarse de un empeoramiento del dolor, que
se hace ms difuso, aparece mayor irritacin peritoneal, mal estado
general y fiebre ms alta. Hay un mnimo porcentaje de casos en los
que se alivia la sintomatologa transitoriamente al perforarse.
Pregunta 49.- R: 1
Cuando nos encontramos con un dolor abdominal (tanto en la
clnica como en el MIR), debemos de proceder siempre por orden y
hacernos estas 3 preguntas por este orden:
1) Tiene el/la paciente un abdomen agudo?.
2) Hay datos objetivos de gravedad? (de sepsis o peritonitis generalizada).
3) Tengo un diagnstico claro?.
Las respuestas, aunque en apariencia no aporten mucho, nos van
a determinar fcilmente la actitud a seguir en cada caso.
En la paciente que nos ocupa, s tiene un abdomen agudo (dolor
abdominal agudo + irritacin peritoneal) y s que hay ahora datos
objetivos de gravedad (de hecho, en este caso nos lo dicen explcitamente). Por otra parte, no tenemos un diagnstico claro que contraindique la ciruga. En un abdomen agudo con datos de gravedad y
causa desconocida es mandatoria la laparotoma exploradora.
Pregunta 50.- R: 4
La paciente es una diabtica que present un dolor de inicio sbito (tpico de cuadros isqumicos o perforativos), en hemiabdomen
inferior (no ms localizado, como suele suceder en las perforaciones,
pero tambin es una anciana y diabtica, que siempre localizan peor
el dolor). Ha presentado rectorragia y tiene un abdomen con distensin y pocos ruidos (probablemente es un leo reflejo, dado que el
dolor ha precedido a la distensin, a diferencia de lo que suele ocurrir en la obstruccin mecnica). La Rx de abdomen nos muestra un
leo con fin abrupto en ngulo esplnico, lo cual sugiere un proceso
a ese nivel. El ngulo esplnico es el asiento tpico de cnceres de
colon izquierdo (pero la clnica es excesivamente rpida y florida
para una obstruccin por cncer, que suele ser subaguda y ms larvada)
y de colitis isqumica (que es lo que tenemos que pensar en este caso
ante el antecedente de diabetes, el inicio sbito, la rectorragia acompaante y la radiologa).
Pregunta 51.- R: 3
Dado que tenemos un segmento gangrenado del colon, su reseccin es imperativa. Una vez realizada, teniendo en cuenta que se trata
de una enfermedad vascular y de una paciente de riesgo (anciana y
diabtica) a la que hemos operado en un contexto de gravedad, la
anastomosis primaria queda totalmente descartada, siendo de eleccin un Hartmann (colostoma proximal tras la reseccin).
M exico A rgentina
C hile U ruguay

Seguimiento a distancia

Pregunta 52.- R: 2
Otra vez nos encontramos ante un abdomen agudo, localizado
inicialmente en fosa ilaca derecha (la causa ms frecuente en esta
localizacin es la apendicitis aguda), en un contexto clnico de gravedad. Por ello, la decisin debe de ser quirrgica y urgente en cualquier caso por lo previamente expuesto en la pregunta 49.
Pregunta 53.- R: 4
La pileflebitis es una complicacin infrecuente de la apendicitis aguda, consistente en la suelta de mbolos spticos al sistema venoso portal,
con lo que se provocan abscesos hepticos por siembra hematgena.
En esta situacin est indicada la intervencin quirrgica urgente y
la antibioterapia intravenosa. Con ello se suelen resolver los abscesos
hepticos sin necesidad de drenaje percutneo.
Pregunta 54.- R: 5
Nuevamente se nos presenta un caso de abdomen agudo (dolor
agudo + irritacin peritoneal) con signos de gravedad (hipovolemia).
En este contexto la ciruga es imperativa, salvo que tengamos un diagnstico que nos descarte la ciruga (si se tratara de una neumona,
cetoacidosis, IAM, Crohn, etc... lo cual no es el caso). Antes bien, la
existencia de una imagen en omega nos indica que se trata de un
vlvulo de sigma. Dados los datos de gravedad, el intento de tratamiento conservador mediante devolvulacin est contraindicado, ya
que probablemente nos encontramos ya con un severo compromiso
vascular (irreversible) si no es ya una perforacin por gangrena del
sigma. En este contexto el Hartmann es de eleccin.
Pregunta 55.- R: 4
Cualquier cuadro de detencin del trnsito digestivo (que reciben el nombre genrico de leo independientemente de su causa,
obstructiva o paraltica) se maneja inicialmente de forma comn:
dieta absoluta dado que est paralizado el trnsito y existen probablemente vmitos, sonda nasogstrica para aliviar los vmitos y
extraer todo el lquido retenido en el tubo digestivo y sueroterapia
intravenosa (porque el paciente est en dieta absoluta y porque el
secuestro de lquidos en el tubo digestivo facilita su deshidratacin,
amn de que los trastornos hidroelectrolticos estn en la base de
muchos leos paralticos). La analgesia tambin est indicada, aunque en el contexto de un abdomen agudo no diagnosticado puede
enmascarar la clnica. Luego, en funcin de la evolucin o de la
causa (si es conocida y mecnica) se indica el tratamiento etiolgico (como sera la ciruga en caso de obstruccin por cncer) o
paliativo adecuado (como sera una colonoscopia descompresiva
en un sndrome de Ogilvie o pseudoobstruccin aguda del colon).
Pregunta 56.- R: 4
Vase siguiente comentario.
Pregunta 57.- R: 3
Se nos cuenta un abdomen agudo de aparicin sbita (recordemos que en este contexto hay que considerar sobre todo cuadros
isqumicos y perforativos), en un paciente joven (ya podemos ir descartando los isqumicos), muy bien localizado en epigastrio (la buena
localizacin es tpica de los cuadros perforativos) y seguido de la aparicin de un abdomen en tabla (que suele ser reflejo de peritonitis
difusa por existencia de un lquido libre irritante en la cavidad
peritoneal: bilis, sangre, cido, pus...). La sospecha evidente es de
perforacin ulcerosa y para ver el neumoperitoneo solicitaremos una
radiografa de trax en bipedestacin. Si el dolor impide la bipedestacin al enfermo, se realizar una radiografa de abdomen en decbito
lateral y con rayo horizontal.
Pregunta 58.- R: 3
Ver pregunta 60.
Pregunta 59.- R: 4
Ver pregunta 60.
Pregunta 60.- R: 1
Hasta un 25% de las perforaciones ulcerosas no presentan neumoperitoneo en la radiologa simple. Por ello, la ausencia de neumope-

CTO Medicina C/ Nez de Balboa, 115 28006 MADRID (Espaa) Tfno.: (91) 782 43 32 / Fax: (91) 782 43 27
E-mail: secretaria@ctomedicina.com; iberocto@ctomedicina.com WEB: www.ctomedicina.com; www.iberocto.com

EP Pg. CG

ritoneo no debe de modificar nuestra sospecha clnica, sino hacernos


buscar otro medio para confirmarla, que segn el algoritmo que vimos en la pregunta 25, ser la colocacin de SNG e introduccin de
aire repitiendo a continuacin la radiologa simple. Si an as seguimos sin encontrar neumoperitoneo, la TC es el medio ms sensible
para la deteccin del mismo.
Pregunta 61.- R: 2
La ausencia de neumoperitoneo puede obedecer a tres causas:
La lcera se ha tapado con epipln, evitando la salida de ms aire.
La perforacin ha sucedido en cara posterior (penetracin), con lo
que no existe salida para el aire a cavidad libre.
La cantidad de aire que ha salido es muy pequea y la radiologa
simple no la detecta.
Pregunta 62.- R: 4
El cierre de la perforacin es imperativo y slo puede ser quirrgico (abierto o laparoscpico). Hasta la llegada de la era del Helicobacter,
se asociaba una medida definitiva que era la vagotoma troncular +
piloroplastia en casos como este. Sin embargo, actualmente est establecido que no se precisa asociar ningn gesto definitivo al cierre
simple, ya que lo que ms disminuye el riesgo de recidiva ulcerosa es
la erradicacin del Helicobacter pylori. Sin embargo, aunque la mayora de las lceras duodenales estn relacionadas con Helicobacter
pylori, si se descarta su implicacin est indicado realizar una tcnica
definitiva (vagotoma troncular y piloroplastia) para prevenir la recidiva, aunque esta situacin es excepcional.
Las lceras slo se resecan en caso de sospecha de malignidad, lo
cual sucede en lceras gstricas (en este paciente hablamos esencialmente de una duodenal) de evolucin trpida.
Pregunta 63.- R: 4
Ya hemos comentado que la cuarta parte de los perforados no
presentan neumoperitoneo en la radiologa simple.
Las complicaciones de la lcera pptica son, por este orden, hemorragia, perforacin y estenosis pilrica. Suelen aparecer aisladamente, de modo que es poco probable que una perforacin sangre y
excepcional que cause un shock hipovolmico.
La ltima opcin es falsa porque el tratamiento mdico actual para
la lcera pptica (inhibidores de la bomba y terapia erradicadora) es
altamente efectivo, por lo que la ciruga programada de la lcera (por
lcera refractaria a tratamiento) se ha convertido en una situacin
prcticamente inexistente. La casi totalidad de las cirugas por lcera
que se realizan desde los aos 90 son por complicaciones agudas
(sobre todo perforaciones, dado que las hemorragias, aunque ms
frecuentes, se suelen controlar bien endoscpicamente).
Pregunta 64.- R: 5
Ver comentario nmero 66.
Pregunta 65.- R: 3
Ver comentario nmero 66.
Pregunta 66.- R: 4
Se nos presenta el caso de una paciente con antecedentes de clicos de repeticin, que presenta lo que inicialmente parece un nuevo
clico. Sin embargo, la enferma no mejora con el tratamiento sintomtico (lo cual debe hacernos sospechar una complicacin), apareciendo clnica de obstruccin intestinal. Esta secuencia de acontecimientos (clico biliar seguido de obstruccin intestinal) es caracterstica del leo biliar.
Normalmente el leo biliar aparece en el contexto de una colecistitis, aunque no siempre es as. Se forma una fstula biliodigestiva (lo
ms frecuente es que sea colecistoduodenal), a travs de la cual pasan
uno o varios clculos al tubo digestivo, obstruyndolo si son de gran
volumen; esta obstruccin sucede ms frecuentemente en la unin
ileocecal, por ser zona de paso ms estrecho.
Radiolgicamente, la aerobilia aparece por pasar aire del tubo
digestivo a las vas biliares, la dilatacin de asas de intestino delgado y
los niveles hidroareos son signos de obstruccin, y el clculo puede
verse en leon terminal si es radioopaco.
Pg. 10 CG

M exico A rgentina
C hile U ruguay

CIRUGA GENERAL

Preparacin Examen de Seleccin 05/06 1 Vuelta

El tratamiento del leo biliar es inicialmente el de cualquier cuadro obstructivo (SNG + dieta + sueros), seguido de la enterolitotoma (extraccin del clculo que obstruye el intestino). En el acto
quirrgico urgente no es necesario actuar sobre la vescula ni sobre
la fstula, lo cual se har slo si las condiciones son muy favorables.
La CPRE carece por completo de sentido porque no hay patologa a
nivel de la va biliar y, desde luego, no tiene ninguna utilidad ver el
rbol biliar.
Pregunta 67.- R: 4
Ver pregunta 70.
Pregunta 68.- R: 1
Ver pregunta 70.
Pregunta 69.- R: 2
Ver pregunta 70.
Pregunta 70.- R: 4
Se nos presenta el caso de una paciente colecistectomizada que
presenta cuadros de clicos biliares tras la ciruga. Ello define lo que se
conoce como sndrome postcolecistectoma.
Este cuadro obedece frecuentemente a que la colecistectoma no
controla la sintomatologa previa por no ser la colelitiasis culpable del
cuadro. Sin embargo, hay otras causas de sndrome postcolecistectoma, morfolgicas o funcionales, que precisan tratamientos especficos.
En concreto, nuestra paciente presenta posteriormente un cuadro
de dolor en hipocondrio derecho, fiebre en picos e ictericia, caracterstica trada de Charcot de la colangitis. En este contexto, la causa ms
probable es una coledocolitiasis residual (aunque a veces se puede
formar una litiasis coledociana de novo) que obstruye la va biliar y
sta se ha sobreinfectado.
El tratamiento de urgencia de la colangitis, en principio es siempre
conservador: antibiticos i.v. y control sintomtico, programando una
CPRE para intentar extraer el clculo obstructivo y hacer una esfinterotoma en previsin de episodios futuros. En el caso del paciente
colecistectomizado, la CPRE encuentra su mejor y ms brillante indicacin, dado que es por s misma totalmente resolutiva del cuadro, sin
precisar ningn gesto adicional (dado que ya no hay vescula que
extirpar).
Pregunta 71.- R: 1
La causa ms frecuente de obstruccin intestinal es la obstruccin
de delgado, sobre todo por bridas o adherencias postquirrgicas.
Cuando se nos especifique que un paciente no ha sido operado
nunca del abdomen, deberemos buscar como causa ms frecuente
de obstruccin una hernia complicada. Por lo tanto, la sola exploracin abdominal rutinaria (inspeccin del abdomen en busca de laparotomas y exploracin de los orificios herniarios) nos puede poner
en la pista de las 2 causas ms frecuentes de obstruccin intestinal.
Por otra parte, la obstruccin ms frecuente del colon es debida a
cncer de colon, especialmente en localizacin recto-sigma. Una proporcin respetable de estos tumores son lo suficientemente bajos como
para ser accesibles al tacto rectal.
Por ello, con estas tres exploraciones baratas y sencillas, podemos
ponernos en la pista de la causa de la obstruccin. Por otra parte, no
debemos olvidar que todas ellas pertenecen al examen fsico, y que
ste siempre debe preceder a las pruebas complementarias, de modo
que en buena praxis las dems opciones deben ser rechazadas.
Pregunta 72.- R: 5
Ya hemos comentado previamente cules deben ser las preguntas
que nos haremos ante un dolor abdominal para decidir la actitud a
seguir (pregunta 49). Vemoslo aplicado a este caso:
1) Tiene el/la paciente un abdomen agudo? Evidentemente s, puesto que hay un dolor abdominal de inicio agudo acompaado de
signos de irritacin peritoneal (Blumberg en este caso).
2) Hay datos objetivos de gravedad? (de sepsis o peritonitis generalizada) NO hay datos de gravedad en este caso que apremien a una
ciruga urgente.

CTO Medicina C/ Nez de Balboa, 115 28006 MADRID (Espaa) Tfno.: (91) 782 43 32 / Fax: (91) 782 43 27
E-mail: secretaria@ctomedicina.com; iberocto@ctomedicina.com WEB: www.ctomedicina.com; www.iberocto.com

Comentarios TEST

Seguimiento a distancia

CIRUGA GENERAL

Preparacin Examen de Seleccin 05/06 1 Vuelta

Seguimiento a distancia

3) Tengo un diagnstico claro? Tengo un dolor en fosa ilaca derecha, pero ningn diagnstico claro. Hay una historia de endometriosis previa, est justo a la mitad del ciclo... Puede ser nuevamente
endometriosis, simple rotura folicular o folculo hemorrgico, apendicitis, EIP, embarazo ectpico....
Dado que tengo una paciente sin un diagnstico claro y sin datos
de gravedad, parece muy precipitado someterla a una ciruga de la
que va a obtener un muy dudoso beneficio, por lo cual la actitud
menos adecuada es sin duda esa. Cualquiera de las otras opciones es
correcta, dado que busca aclarar un poco el cuadro de la paciente
(mediante pruebas complementarias u observacin clnica).

Comentarios TEST

Pregunta 73.- R: 1
La vscera ms frecuentemente lesionada en los traumatismos abdominales cerrados es el bazo. Su rotura origina un hemoperitoneo
que con frecuencia est en el origen de la inestabilidad hemodinmica que pueden presentar estos pacientes. Sin embargo, la inestabilidad hemodinmica en el paciente con traumatismo cerrado no es
indicacin de ciruga: siempre se debe comprobar mediante una
prueba (ecografa o lavado peritoneal) que el foco de sangrado es
intraperitoneal, porque un paciente puede presentarse tras traumatismo cerrado con un shock hipovolmico secundario a una fractura de
pelvis o fmur o a una lesin torcica.
La siguiente vscera lesionada por orden de frecuencia en el trauma cerrado es el hgado.

Pregunta 74. A la izquierda: laparoscopia; a la derecha laparotoma.

Pregunta 75.- R: 2
Como norma general, el accidente de trfico es el prototipo del
traumatismo abdominal cerrado, pero por ser politraumatizado se
debe de seguir el protocolo ABCDE antes de hacer el balance de
lesiones. El paciente estaba algo hipotenso al recogerlo, pero ha quedado estable con los 700 cc del transporte. Por otra parte, la A y la B
estn aseguradas, dado que el paciente viene intubado. De manera
que tenemos un ABC resuelto y nos centramos en D y E. La exploracin neurolgica no es valorable en un paciente que ha sido sedado
y relajado para la intubacin, por lo que no podemos afirmar nada
en relacin con lesiones del SNC. Sin embargo, en el contexto de un
politraumatismo importante, la TC craneal es imperativa. El hecho de
que exista dolor abdominal justifica en s mismo la realizacin de una
prueba de imagen en abdomen, que dada la estabilidad del paciente
ser una TC.

Pregunta 73. Manejo de un traumatismo abdominal.

Pregunta 74.- R: 2
La actitud en la herida por arma blanca abdominal no viene determinada tanto por la estabilidad del enfermo (como sucede en el trauma
cerrado) como por la penetracin en peritoneo. Si hay datos directos o
indirectos de penetracin en peritoneo (y, ciertamente, la inestabilidad
es un dato indirecto de penetracin, aunque no el nico posible),
entonces hay indicacin de exploracin quirrgica. Es el caso que nos
ocupa, la evisceracin nos indica la penetracin de forma inequvoca.
Si hay certeza de no penetracin, lo indicado es la observacin.
Sin embargo, en muchos casos no tenemos evidencia de penetracin, pero no podemos descartarla. En estas situaciones se puede
realizar un lavado peritoneal o una laparoscopia diagnstica para
intentar aclarar la existencia de lesiones intraabdominales. Algunos
autores defienden la observacin clnica estrecha con exploracin
repetida por un mismo cirujano, en busca de signos de irritacin
peritoneal, pero esta postura no ha sido an aceptada de forma universal.

M exico A rgentina
C hile U ruguay

CTO Medicina C/ Nez de Balboa, 115 28006 MADRID (Espaa) Tfno.: (91) 782 43 32 / Fax: (91) 782 43 27
E-mail: secretaria@ctomedicina.com; iberocto@ctomedicina.com WEB: www.ctomedicina.com; www.iberocto.com

EP Pg. CG

ENDOCRINOLOGA

Preparacin Examen de Seleccin 05/06 1 Vuelta


HIPOTLAMO-HIPFISIS.

2)

1.

3)

En todas las siguientes patologas hipotlamo-hipofisarias


existir hiperprolactinemia, EXCEPTO en:
1)
2)
3)
4)
5)

2.

2)
3)
4)
5)

4)
5)

Preguntas TEST

Mujer con amenorrea secundaria y prolactina basal mayor


de 300 microg/L.
Embarazada con prolactinemia de 400 microg/L.
Mujer con amenorrea secundaria, test de embarazo negativo y prolactinemia de 160 microg/L.
Mujer con alteraciones menstruales, prolactina de 75
microg/L y adenoma hipofisario de 23 mm.
Amenorrea primaria, prolactinemia de 200 microg/L y RM
selar sin alteraciones.

No precisa tratamiento.
Ciruga transesfenoidal.
Ciruga combinada con bromocriptina, con o sin radioterapia.
Cabergolina, que es un agonista dopaminrgico.
Antagonistas de DA, como bromocriptina.

Una paciente de 25 aos, diagnosticada de hiperprolactinemia idioptica, sin tratamiento ni sintomatologa en la actualidad, salvo la existencia de menstruaciones irregulares de
forma ocasional, expresa en una visita rutinaria deseo de
embarazo. Cul de las siguientes actitudes teraputicas sera
la ms apropiada?:
1)
2)
3)

4)
5)
5.

5)
6.

5)
7.

No tratar, y cuando se produzca el embarazo, administrar


dosis bajas de bromocriptina, para mantener cifras de
prolactina dentro de la normalidad.
Iniciar tratamiento con dosis bajas de bromocriptina, y
cuando se produzca el embarazo, aumentar la dosis hasta
mantener cifras de prolactina dentro de la normalidad.
Iniciar tratamiento con dosis bajas de bromocriptina, y
cuando se produzca el embarazo, suspender la medicacin, realizando controles analticos y campimtricos
peridicamente.
Iniciar tratamiento con cabergolina, y cuando se produzca el embarazo, mantener dosis mnimas hasta mantener
cifras de prolactina dentro de la normalidad.
Mantener sin ningn tipo de tratamiento, antes y durante
el embarazo.

Referente a la acromegalia, es cierto que:


1)

El estudio anatomopatolgico postquirrgico determina


si se trata de adenoma o carcinoma.
M exico A rgentina
C hile U ruguay

2)
3)
4)
5)

Se habla de curacin cuando las cifras de IGF-1 son


normales para sexo y edad y la GH es inferior a 2 ng/dl tras
SOG por RIA.
Cuando se realiza el diagnstico de la enfermedad, se
debe descartar la existencia de poliposis colnica.
Cuando se alcanzan los criterios de curacin, la morbimortalidad se equipara a la de la poblacin general.
Puede existir elevacin de GH, tras estmulo con TRH.
Aunque la dopamina en condiciones normales produzca
elevacin en las cifras de GH, la bromocriptina puede
resultar til en el tratamiento.

Paciente de 48 aos, que acude a nuestra consulta por disminucin de la libido, impotencia, astenia, nuseas y vmitos,
letargia, piel seca, estreimiento. En la analtica de rutina destaca: anemia (Hb: 11 g/dl), colesterol normal, iones normales. En
estudio de imagen: RM se observa una lesin hipofisaria pequea (<1cm). Cul ser el diagnstico MENOS probable?:
1)
2)
3)
4)
5)

9.

Bromocriptina.
Octretide.
Lisuride.
Reintervencin con hipofisectoma, aunque no se visualice lesin.
Cateterismo de los senos petrosos para determinar el
origen hipofisario y reintervencin.

En relacin a la acromegalia, es FALSO:


1)

8.

Se puede asociar a HTA, diabetes mellitus e hiperprolactinemia.


La prueba de laboratorio diagnstica es la determinacin
basal de GH en suero.
La hipercalcemia y la hipercalciuria, con litiasis renal, son
frecuentes.
La hipercalcemia asociada sugiere un MEN 2a.

En un paciente acromeglico portador de un macroadenoma,


tras ser operado no se produce curacin. La resonancia magntica realizada despus de la intervencin no muestra la
existencia de restos de adenoma; el paciente se niega a la
radioterapia. Usted qu actitud teraputica le recomendara?:
1)
2)
3)
4)

Varn de 40 aos, sin antecedentes clnicos ni farmacolgicos de inters. Acude por disfuncin sexual, sin otra manifestacin clnica. Niveles sricos de prolactina: 330 microg/L. En
la TC craneal se observa un macroadenoma de 1,5 cm,
limitado a la silla turca. La actitud teraputica a seguir es:
1)
2)
3)

4.

4)

Todos los siguientes datos nos deben hacer pensar en el


diagnstico de prolactinoma, EXCEPTO uno:
1)

3.

Sndrome de Nelson.
Hipofisitis linfocitaria.
Craneofaringioma.
Adenoma cromfobo no funcionante, de 3 cm de dimetro.
Sndrome de Sheehan.

Seguimiento a distancia

Microprolactinoma.
Sarcoidosis.
Infarto hipofisario.
Tuberculosis.
Carcinoma metastsico.

Varn de 30 aos, que acude al servicio de urgencias por


cefalea de inicio brusco, muy dolorosa, con oftalmoparesia,
pupilas midriticas, TC normal y LCR sin alteraciones. A usted
le parece correcto:
1)
2)
3)
4)
5)

Podra tratarse de una apopleja hipofisaria y estara


indicada una RM urgente.
Este cuadro es muy frecuente en la clnica de debut de un
adenoma no funcionante de hipfisis.
Probablemente exista otra patologa neurolgica concomitante, ya que la afectacin de oculomotores nunca se produce.
Se denomina tambin sndrome de Sheehan.
En la histologa se demostrara un infiltrado de clulas
mononucleares en la hipfisis, y por ello se encuadra
dentro del sndrome de hipofisitis linfoctica.

CTO Medicina C/ Nez de Balboa, 115 28006 MADRID (Espaa) Tfno.: (91) 782 43 32 / Fax: (91) 782 43 27
E-mail: secretaria@ctomedicina.com; iberocto@ctomedicina.com WEB: www.ctomedicina.com; www.iberocto.com

ED Pg. 1

10.

Cul de las siguientes afirmaciones, en relacin a un paciente


intervenido de un macroadenoma hipofisario no secretor, es
FALSA?:
1)
2)
3)
4)
5)

11.

2)
3)
4)
5)
12.

Sera til realizar una prueba de hipoglucemia insulnica


para valorar posible dficit de GH y ACTH.
Es probable que presente un panhipopituitarismo secundario a un infarto hipofisario postparto.
En la exploracin fsica debe presentar hiperpigmentacin.
Debe de tratarse con levotiroxina y con hidrocortisona.
El dficit de prolactina suele ser la manifestacin inicial tras
el parto.

Un paciente de 68 aos, diagnosticado hace 25 aos de


sndrome de Cushing e intervenido mediante suprarrenalectoma bilateral, se encuentra en tratamiento con hidrocortisona (30 mg/da). En la revisin se objetiva hiperpigmentacin cutnea intensa. Cul sera su sospecha diagnstica
ms probable?:
1)
2)
3)
4)
5)

13.

Si presentara dficit de TSH y ACTH, se debera iniciar


tratamiento sustitutivo, primero con glucocorticoides y
despus con levotiroxina.
Para descartar inmediatamente despus de la ciruga el
dficit de ACTH, la prueba de eleccin sera administrar
ACTH i.v. y medir la respuesta del cortisol plasmtico.
Si persistieran restos tumorales tras la ciruga, se planteara
la posibilidad de administrar radioterapia.
Si existiera dficit de TSH, el paciente no presentara
hipercolesterolemia.
Inmediatamente despus de la ciruga se debe controlar
de forma estricta la diuresis.

Una paciente de 30 aos acude a nuestra consulta por


amenorrea, piel seca, intolerancia al fro, ganancia de peso,
astenia. Como antecedentes destacan: DM tipo 1, incapacidad para la lactancia tras un parto, hace 6 meses. Seale la
respuesta FALSA:
1)

2)
3)
4)
5)

Pg. 2 ED

14.

El diagnstico de diabetes inspida no queda demostrado


con estos datos.
La misma respuesta se obtiene en la polidipsia primaria.
Est indicada la medicin de ADH en plasma u orina.
Podra tratarse con sueroterapia con 2.500 cc de salino
0,9% i.v. al da, dieta absoluta y ADH nativa.
Podra tratarse con desmopresina.
M exico A rgentina
C hile U ruguay

Cul de estas situaciones NO justifica una diabetes inspida


nefrognica?:
1)
2)
3)
4)
5)

15.

16.

Hipotiroidismo.
Tratamiento con litio.
Uropata obstructiva.
Aldosteronismo primario.
Nefritis pierde sal.

Cul de los siguientes datos NO orienta hacia un SIADH en


un paciente con disminucin del nivel de conciencia?:
1)
2)
3)
4)
5)

Agitacin.
Osm (orina): 350.
Edemas maleolares.
Natremia 120.
Ac. rico: 2,5.

Varn diagnosticado de ca. pulmonar microctico, asintomtico. Exploracin, salvo la auscultacin pulmonar, normal.
Natremia: 130. Natriuria: 25. Osm (sangre): 270. Osm (orina): 300. Qu tratamiento inicial est indicado?:
1)
2)
3)
4)
5)

Restriccin hdrica a 500-1.000 cc/da.


Furosemida y reposicin hidroelectroltica con salino al
0,9% y ClK.
Demeclociclina.
No precisa tratamiento.
Desmopresina s.c. o intranasal.

PATOLOGA TIROIDEA.
17.

Respecto a las acciones del yodo sobre el tiroides, la nica


afirmacin correcta es:
1)
2)

No seguimiento del tratamiento con hidrocortisona.


Sndrome de Nelson.
Falta de absorcin de los glucocorticoides.
Porfiria cutnea.
Recidiva del sndrome de Cushing, posiblemente ectpico.

Paciente de 43 aos, sin antecedentes de inters, que


consulta por polidipsia y poliuria (6-7 litros/da) en las seis
semanas previas. El hemograma y una bioqumica bsica
son normales. Osm (sangre): 290; osm(orina): 100. La
osmolaridad urinaria no se modifica significativamente tras
deshidratacin. Se administran 2 microg. de DDAVP, observndose que la osmolaridad en orina aumenta a 370. Seale
la verdadera:
1)

ENDOCRINOLOGA

Preparacin Examen de Seleccin 05/06 1 Vuelta

3)
4)
5)

18.

Inhibe de forma aguda y transitoria la peroxidasa tiroidea


(efecto Jod-Basedow).
Administrado en dosis elevadas de forma aguda, puede
producir en pacientes con bocio simple un hipertiroidismo con gammagrafa hipocaptante.
El bocio inducido por yoduro se acompaa siempre de
hipotiroidismo.
Los pacientes con enfermedad autoinmune son especialmente resistentes al efecto bloqueante del yodo.
La utilizacin preoperatoria de yodo en los bocios hiperfuncionantes est contraindicada, ya que puede aumentar la vascularizacin de la glndula y producir graves
hemorragias intraoperatorias.

Respecto a la fisiologa tiroidea, sealar la respuesta correcta:


1)
2)
3)
4)

5)

La tiroglobulina es la protena que transporta mayoritariamente T3 y T4.


El transporte del yoduro desde el plasma hasta el interior de
las clulas foliculares es un transporte de membrana pasivo.
La oxidacin del yoduro se realiza por la yodotirosina
deshalogenasa tiroidea.
Las sustancias precursoras monoyodotirosina (MYT) y
diyodotirosina (DYT) son liberadas de la tiroglobulina y
sufren una reaccin de acoplamiento por la peroxidasa
dentro del coloide folicular.
El tiroides es la nica fuente de T4 endgena.

CTO Medicina C/ Nez de Balboa, 115 28006 MADRID (Espaa) Tfno.: (91) 782 43 32 / Fax: (91) 782 43 27
E-mail: secretaria@ctomedicina.com; iberocto@ctomedicina.com WEB: www.ctomedicina.com; www.iberocto.com

Preguntas TEST

Seguimiento a distancia

ENDOCRINOLOGA

Preparacin Examen de Seleccin 05/06 1 Vuelta


19.

La conversin perifrica de T4 a T3 NO disminuye en caso de:


1)
2)
3)
4)
5)

20.

4)
5)
21.

2)
3)
4)
5)

Preguntas TEST

5)

4)
5)
26.

Radioyodo.
Antitiroideos.
Tiroidectoma.
Actitud expectante.
Aumento de ingesta de yodo en la dieta.

El hipotiroidismo se asocia siempre a disminucin del


tamao glandular tiroideo.
El hipertiroidismo se asocia siempre a aumento del tamao glandular tiroideo.
El bocio se asocia con aumento, normalidad o disminucin de la secrecin hormonal.
El cncer de tiroides (variante folicular) se asocia con
frecuencia a tirotoxicosis.
Cuando el tiroides lingual es el nico tejido tiroideo
funcionante, su secrecin suele ser suficiente para mantener un estado eutiroideo.

2)
3)

Ecografa tiroidea.
Exploracin cervical.
Radiografa trax.
Determinacin de tiroglobulina, seguida de rastreo corporal total con I-131.
TC cervicotorcico.

4)
5)
27.

Amiodarona.
Litio.
Tiroiditis de Hashimoto.
Yoduro de forma continuada en pacientes susceptibles.
Propranolol.

Una mujer de 34 aos acude a nuestra consulta para revisin


con un informe mdico en el que aparece el diagnstico de
M exico A rgentina
C hile U ruguay

El tratamiento de eleccin es T4 va i.v.


Debe administrarse tambin hidrocortisona.
Si no se dispone de T4-i.v. se puede emplear T3 en SNG.
Tiene buen pronstico.
Se inicia el tratamiento, a pesar de no tener confirmacin
analtica del diagnstico.

En una mujer con tirotoxicosis por el consumo de ciertos


productos para adelgazar que contienen T3, qu NO esperara encontrar?:
1)
2)
3)
4)
5)

29.

Se trata de un hipotiroidismo primario, se debe iniciar


tratamiento con levotiroxina.
Se trata de un hipotiroidismo secundario, se debe iniciar
tratamiento con levotiroxina.
Se trata de un hipotiroidismo subclnico, se debe iniciar
tratamiento con levotiroxina.
Se trata de un hipotiroidismo subclnico, se puede vigilar
sin ningn tratamiento.
No presenta ninguna patologa tiroidea, no precisa ningn
tratamiento.

Sobre el tratamiento del coma mixedematoso, seale la


FALSA:
1)
2)
3)
4)
5)

28.

Habr que solicitar anticuerpos antimicrosomales para


descartar una tiroiditis crnica autoinmune.
Debemos disminuir la dosis de tiroxina.
Se puede mantener el mismo tratamiento y reevaluar cada
6 meses.
Debemos aumentar la dosis sustitutiva hasta que T4 libre
alcance niveles superiores al nivel mximo, para asegurarnos una adecuada respuesta tisular.
Es necesario asociar esteroides, por si se tratara de un
sndrome poliglandular autoinmune.

Una mujer de 54 aos, sin antecedentes de inters, consulta


por ganancia de 5 kg de peso en el ltimo ao. No refiere otros
sntomas. Sus anlisis generales son normales y sus hormonas
tiroideas: T4 libre 1,0 ng/dl, TSH 6,1 mcU/ml. Seale la
correcta:
1)

NO se encuentra entre las causas de bocio:


1)
2)
3)
4)
5)

25.

2)
3)

Son especficos de la enfermedad de Graves.


Son especficos de la enfermedad de Hashimoto.
Pueden existir en el hipotiroidismo primario, en la enfermedad de Graves y en la enfermedad de Hashimoto.
Inhiben la unin de TSH al receptor.
Producen enfermedad de Graves neonatal.

Son pruebas diagnsticas tiles en el bocio simple todas,


MENOS una:
1)
2)
3)
4)

24.

1)

Sealar la respuesta correcta respecto a la patologa tiroidea:


1)

23.

Ayuno y desnutricin.
Gran traumatismo.
Tratamiento con carbimazol.
Tratamiento con propranolol.
Contrastes yodados.

Paciente que acude por aumento de tamao tiroideo, disnea


de esfuerzo y pltora facial con sncope al elevar los brazos.
Presenta hormonas tiroideas y TSH normales. La actitud ms
adecuada es la siguiente:
1)
2)
3)
4)
5)

22.

hipotiroidismo primario idioptico. Est en tratamiento con


100 microg/da de levotiroxina y aporta una analtica reciente
con los siguientes datos hormonales: T4 libre 1,8 ng/dl (N:
0,5-2,4) y TSH 0,01 mcU/ml (N: 0,5-4). Sealar la respuesta
correcta:

Los anticuerpos antimicrosomales y antitiroglobulina:


1)
2)
3)

Seguimiento a distancia

Tiroglobulina normal o disminuida.


Captacin de yodo disminuida.
T3 y T4 elevadas.
Signos clnicos de tirotoxicosis.
Ausencia de bocio.

De las siguientes, seale cul NO produce un hipertiroidismo


con gammagrafa silente:
1)
2)
3)
4)
5)

Tirotoxicosis facticia.
Coriocarcinoma.
Struma ovarii.
Tirotoxicosis por hamburguesas.
Tiroiditis con tirotoxicosis transitoria.

CTO Medicina C/ Nez de Balboa, 115 28006 MADRID (Espaa) Tfno.: (91) 782 43 32 / Fax: (91) 782 43 27
E-mail: secretaria@ctomedicina.com; iberocto@ctomedicina.com WEB: www.ctomedicina.com; www.iberocto.com

ED Pg. 3

30.

Mujer de 28 aos, que acude a su consulta por presentar,


desde hace tres meses, aumento del cuello, sudacin profusa,
prdida de peso, diarrea, hiperfagia, palpitaciones y "ojos
saltones". Usted NO hara:
1)
2)
3)
4)
5)

31.

2)
3)
4)
5)

1)
2)
3)
4)
5)
36.

2)
3)
4)
5)
37.

Los frmacos antitiroideos pueden producir hipotiroidismo fetal.


Los IgG estimuladores del tiroides pueden provocar hipertiroidismo fetal.
El yodo radiactivo est siempre contraindicado.
La ciruga est siempre contraindicada.
El propranolol est contraindicado.

3)
4)
5)

39.

1)
2)
3)
4)
5)

Yodo radiactivo.
Propiltiouracilo.
Yoduro.
Dexametasona.
Ipodato sdico.

1)
2)
3)
4)
5)

Ante un ndulo tiroideo, cul es la primera prueba que se


debe realizar?:
1)
2)
3)
4)
5)

35.

Gammagrafa tiroidea, para clasificar el ndulo en fro o


caliente.
Puncin-aspiracin con aguja fina.
TSH, T3 y T4.
Biopsia.
Captacin de yodo radiactivo.

Ante un ndulo tiroideo fro, lo ms probable es:

Pg. 4 ED

M exico A rgentina
C hile U ruguay

40.

Puede no ser palpable.


Se trata con tiroidectoma total.
Secreta calcitonina y, a veces, otras sustancias.
Puede transmitirse de forma autosmica dominante.
Las metstasis se tratan con yodo-131.

Una paciente de 42 aos presenta desde hace 3 un ndulo


tiroideo derecho fro. La PAAF inicial fue benigna. Desde
entonces realiza tratamiento supresor con levotiroxina. En la
revisin actual, el ndulo ha crecido significativamente. Una
nueva PAAF sigue reflejando benignidad. Cul le parece la
actitud ms correcta?:

NDULO Y CNCER DE TIROIDES.


34.

Tiroidectoma total.
Linfadenectoma cervical radical, aunque no exista afectacin ganglionar macroscpica.
Tratamiento supresor con hormona tiroidea.
Tratamiento con yodo-131, para ablacin de restos tiroideos tras la ciruga.
Tratamiento con yodo-131 de las metstasis funcionantes.

Sobre el carcinoma medular de tiroides, es FALSO:


1)
2)
3)
4)
5)

Mujer de 75 aos, diagnosticada de enfermedad de Graves,


insuficiencia cardaca con disnea de reposo y angor inestable. Qu tratamiento inicial NO est indicado?:

Hormona tiroidea en dosis supresoras y repeticin de la


PAAF a los 3-6 meses.
Biopsia tiroidea.
Tiroidectoma subtotal.
Antitiroideos y repeticin de la PAAF a los 3-6 meses.
Yodo radiactivo en altas dosis.

En el tratamiento del cncer diferenciado de tiroides, cul


de los siguientes NO est indicado?:
1)
2)

38.

Carcinoma papilar.
Carcinoma folicular.
Adenoma o quiste.
Bocio txico multinodular.
Metstasis.

Mujer de 40 aos, sin antecedentes de inters, que presenta


ndulo tiroideo. PAAF: no concluyente. Cul es la actitud
ms indicada?:
1)

Embarazo.
Nio.
Anciano.
Bocio muy voluminoso retroesternal.
Ndulo fro en gammagrafa tiroidea.

Mujer embarazada, diagnosticada de enfermedad de Graves.


Seale la respuesta FALSA:
1)

33.

Pensar como primera opcin en una enfermedad de


Graves y solicitar TSH, T4 libre y TSI.
Tambin pedira una gammagrafa tiroidea como parte
del estudio.
Le comentar a la paciente que, si es una enfermedad de
Graves, existe un tratamiento que se debe mantener unos
2 aos.
Tambin le comentar que, a medida que mejore la
enfermedad del tiroides, tambin lo har el problema
ocular.
Tambin esperara encontrar anticuerpos anti-TG y antiTPO.

NO es contraindicacin al tratamiento de la enfermedad de


Graves con yodo radiactivo:
1)
2)
3)
4)
5)

32.

ENDOCRINOLOGA

Preparacin Examen de Seleccin 05/06 1 Vuelta

Aumentar la dosis de levotiroxina.


Remitir a ciruga para hemitiroidectoma.
Remitir a ciruga para tiroidectoma total.
Suspender la levotiroxina.
Continuar el tratamiento supresor y revisar anualmente
con ecografa.

En un paciente con un ndulo tiroideo, la actitud a seguir


segn el resultado de la PAAF es la siguiente:
1)
2)
3)
4)
5)

Con citologa maligna, intentar tratamiento con levotiroxina para reducir el tamao tumoral antes de la ciruga.
Ciruga siempre, si la citologa es no concluyente.
Iniciar tratamiento con levotiroxina, si la citologa es
benigna, y vigilar.
Repetir la puncin si se objetiva abundante proliferacin
folicular.
Ciruga siempre, aunque sea benigno.

CTO Medicina C/ Nez de Balboa, 115 28006 MADRID (Espaa) Tfno.: (91) 782 43 32 / Fax: (91) 782 43 27
E-mail: secretaria@ctomedicina.com; iberocto@ctomedicina.com WEB: www.ctomedicina.com; www.iberocto.com

Preguntas TEST

Seguimiento a distancia

ENDOCRINOLOGA

Preparacin Examen de Seleccin 05/06 1 Vuelta


41.

Una paciente, diagnosticada de carcinoma papilar de tiroides, fue tratada con tiroidectoma total y una dosis postoperatoria de 50 mCi de I-131 para ablacin de restos ectpicos,
sin evidenciarse metstasis en el rastreo corporal. A los 6
meses, el rastreo de seguridad es negativo. A los 12 meses de
la operacin, sin embargo, se detectan cifras elevadas de
tiroglobulina. La actitud teraputica ser:
1)
2)
3)
4)
5)

42.

Son datos de sospecha de malignidad de un ndulo tiroideo


todos, MENOS uno de los siguientes:
1)
2)
3)
4)
5)

43.

2)
3)
4)
5)

48.

2)
3)
4)
5)

Si se evidencia fibrosis en la histologa, puede asociarse a


hipotiroidismo en un 25% de los casos.
Est indicada la ciruga para descomprimir.
Puede asociar fibrosis mediastnica y retroperitoneal.
Habra que descartar un carcinoma anaplsico de tiroides.
El diagnstico ms probable es una tiroiditis subaguda, si
la VSG est aumentada.

45.

4)
5)

La causa ms frecuente de sndrome de Cushing es:


1)
2)
3)
4)
5)

Microadenoma hipofisario (enfermedad de Cushing).


Adenoma suprarrenal.
Adrenalitis autoinmune.
Paraneoplsico.
Esteroides exgenos.

M exico A rgentina
C hile U ruguay

51.

Test de ACTH.
Cortisol plasmtico.
Cortisol libre urinario (24 horas).
Ritmo de cortisol (8:00-23:00).
ACTH.

Con respecto a la prueba definitiva con dexametasona (DXM)


para el diagnstico de sndrome de Cushing, seale la correcta:
1)
2)
3)

PATOLOGA SUPRARRENAL.

HTA.
Cara de luna llena.
Obesidad troncular.
Hiperpigmentacin.
Hirsutismo.

Cul de las siguientes pruebas diagnsticas puede ser equivalente a la prueba de despistaje con 1 mg de dexametasona
nocturna, en el sndrome de Cushing?:
1)
2)
3)
4)
5)

50.

Secrecin de ACTH por oat-cell.


Hiperplasia nodular suprarrenal.
Microadenoma hipofisario (<1cm).
Macroadenoma hipofisario (>1cm).
Hiperplasia difusa de clulas corticotropas.

Varn de 57 aos que consulta por hemoptisis. En urgencias


se encuentra afebril. TA: 120/60. Hb: 14,5, Hto: 54,5, leucocitos 10.000 (frmula normal), plaquetas: 250.000. La GAB
muestra pH: 7,55, HCO3: 38, pCO2: 40, pO2: 80. Na: 140,
K: 2,8, Glucosa: 170. La radiografa de trax muestra ensanchamiento hiliar e infiltrado en lbulo superior derecho. Qu
otro dato clnico esperara encontrar si sospechase un sndrome de Cushing por secrecin ectpica de ACTH?:
1)
2)
3)
4)
5)

49.

Hbito cushingoide.
Cortisol en sangre y orina disminuido.
Hipopotasemia.
Hiperpigmentacin.
Hiperlipoproteinemia (LDL).

La causa ms frecuente de sndrome de Cushing por exceso


de ACTH es:
1)
2)
3)
4)
5)

Paciente con aumento del tamao tiroideo, indoloro, de


consistencia ptrea, con disnea y disfagia. Seala la FALSA:
1)

Preguntas TEST

47.

Varn.
Disfona.
Desplazamiento del tiroides con la deglucin.
Ausencia de dolor.
Ndulo fro gammagrficamente.

Tiroiditis subaguda: patogenia probablemente viral, fiebre, dolor cervical irradiado a odos, tiroglobulina y VSG
altas, bocio nodular.
Tiroiditis indolora: fase de hipertiroidismo, seguida de fase
de hipotiroidismo, y finalmente normofuncin.
Tiroiditis de Riedel: tratamiento con antitiroideos en la fase
de hipertiroidismo.
Tiroiditis de Hashimoto: tratamiento con levotiroxina.
Tiroiditis aguda: antibiticos y drenaje quirrgico.

En un paciente con sndrome de Cushing por administracin


exgena de esteroides, NO es esperable encontrar:
1)
2)
3)
4)
5)

Seale la INCORRECTA:
1)

44.

Aumentar la dosis supresora de tiroxina.


Esperar 6 meses y repetir determinacin de tiroglobulina.
Realizar nuevo rastreo corporal, para detectar la existencia
de restos tiroideos o extratiroideos hipercaptantes.
Realizar una TC de cuerpo entero, como primera medida
para detectar metstasis.
La elevacin de la tiroglobulina en los pacientes que
siguen tratamiento supresor con L-T4 no indica, en la
mayora de los casos, la existencia de metstasis.

46.

Seguimiento a distancia

No tiene falsos positivos.


DXM, 0,5 mg/6 h, durante 2 das.
Permite el diagnstico diferencial entre microadenoma
hipofisario y neoplasia suprarrenal.
DXM, 8 mg/da, durante 2 das.
Consigue suprimir el cortisol al 50% en caso de microadenoma hipofisario.

Mujer de 35 aos, con obesidad troncular, cara de luna


llena, estras rojo-vinosas en abdomen, hirsutismo, HTA y
diabetes. Presenta cortisoluria de 24 horas alta. El cortisol no
suprime con 1 mg de DXM ni con 0,5mg/6h durante 2 das.
ACTH est alta. Cortisoluria suprime ms de un 90% con 8 mg/
da de DXM. Cul es el diagnstico ms probable y qu
prueba de imagen haras en primer lugar?:

CTO Medicina C/ Nez de Balboa, 115 28006 MADRID (Espaa) Tfno.: (91) 782 43 32 / Fax: (91) 782 43 27
E-mail: secretaria@ctomedicina.com; iberocto@ctomedicina.com WEB: www.ctomedicina.com; www.iberocto.com

ED Pg. 5

1)
2)
3)
4)
5)
52.

53.

5)

59.

Ausencia de hiperpigmentacin mucocutnea.


Natremia: 155 mEq/L.
Obesidad troncular y estras rojo-vinosas.
Glucemia: 50 mg/dl.
Eosinofilia.

5)

60.

Septicemia por meningococo.


Tratamiento con anticoagulantes.
Supresin tras tratamiento prolongado con AINEs.
Episodio de cardiopata isqumica en un paciente addisoniano.
Fractura de fmur en un paciente que ha recibido tratamiento con dosis altas de esteroides en los ltimos seis meses.

62.

Mujer de 20 aos, en tratamiento con calcio por hipocalcemia


desde hace aos, de la que no recuerda la causa. Refiere astenia,
anorexia, debilidad, mareo, malestar general e hiperpigmentacin de varias semanas de evolucin. Acude a Urgencias por
vmitos y dolor abdominal. En qu diagnostico pensara?:

63.

2)
3)
4)
5)

Pg. 6 ED

M exico A rgentina
C hile U ruguay

Alcohol.
Regaliz.
Tabaco.
Herona.
Alquitrn.

NO es causa de exceso de mineralocorticoides:


1)
2)
3)
4)
5)

Sndrome de Cushing: hipopotasemia, alcalosis, poliglobulia y linfopenia.


Addison: hiperkaliemia, acidosis, anemia y linfocitosis.
Hiperaldosteronismo: hipopotasemia, alcalosis y diabetes inspida nefrognica.
Hipoaldosteronismo: hiperpotasemia, acidosis.
Insuficiencia suprarrenal secundaria: hiperpotasemia, acidosis, anemia, linfocitosis y ausencia de hiperpigmentacin.

Alteracin de la 11-OH-esteroide-DH.
Un tipo de aldosteronismo secundario.
Exceso de consumo de regaliz.
Se denomina as a los tipos de hiperplasia adrenal congnita que cursan con hipermineralcorticismo.
Una alteracin gentica por la cual la aldosterona se
sintetiza en la capa fascicular regulada por el promotor de
glucocorticoides.

Qu sustancia se ha relacionado con el sndrome de exceso


aparente de mineralocorticoides?:
1)
2)
3)
4)
5)

61.

Sndrome de Liddle.
Aldosteronoma o sndrome de Conn.
Defecto de 11-beta-OH-deshidrogenasa.
Adenoma productor de DOCA.
Hiperaldosteronismo secundario.

El sndrome de hiperaldosteronismo remediable con corticoides consiste en:


1)
2)
3)
4)

Seala la asociacin INCORRECTA:


1)

57.

Tuberculosis.
Adrenalitis autoinmune.
Asociada al VIH.
Hemorragia adrenal.
Yatrgena.

Embarazo.
Intolerancia al calcio oral.
Sndrome pluriglandular autoinmune tipo 1.
Sndrome pluriglandular autoinmune tipo 2.
Iniciara tratamiento con T4 intravenosa antes de plantear
cualquier diagnstico, puesto que es una urgencia endocrinolgica.

Paciente que acude por astenia, poliuria, polidipsia y cefalea.


Exploracin normal, salvo TA: 165/110. Na:155, K:2,8,
Glu:110. Alcalosis metablica. ARP disminuida basal y tras
restriccin de sodio. Aldosterona aumentada, tras infusin
de suero salino. Diagnstico ms probable:
1)
2)
3)
4)
5)

Pueden ser causa de insuficiencia suprarrenal aguda todas


las siguientes, EXCEPTO una:
1)
2)
3)
4)

56.

Extirpacin del adenoma productor de glucocorticoides.


Suprarrenalectoma bilateral.
Extirpacin de un tumor carcinoide productor de ACTH.
Ciruga transesfenoidal.
Ketoconazol como tratamiento definitivo.

La interrupcin del tratamiento crnico con corticoides es la


causa ms frecuente de insuficiencia suprarrenal aguda.
Qu dato NO esperara encontrar?:
1)
2)
3)
4)
5)

55.

58.

La causa ms frecuente en la actualidad de enfermedad de


Addison es:
1)
2)
3)
4)
5)

54.

1)
2)
3)
4)
5)

Cushing ectpico - gammagrafa con octretide.


Enfermedad de Cushing - resonancia de las glndulas
suprarrenales.
Cushing ectpico - TC toracoabdominal.
Enfermedad de Cushing - resonancia hipofisaria.
Carcinoma suprarrenal - resonancia abdominal.

En el tratamiento del sndrome de Cushing, se incluyen todos,


MENOS uno de los siguientes:
1)
2)
3)
4)
5)

ENDOCRINOLOGA

Preparacin Examen de Seleccin 05/06 1 Vuelta

Sndrome de Conn.
Defecto de 21-hidroxilasa.
Defecto de 11-hidroxilasa.
Defecto de 17-hidroxilasa.
Defecto de 11-beta-OH-deshidrogenasa.

Ante un paciente con HTA, debilidad y calambres musculares,


en cuya analtica encontramos un potasio de 3,1 mEq/l, la
primera posibilidad diagnstica es:
1)
2)
3)
4)
5)

Hiperplasia suprarrenal bilateral.


Adenoma suprarrenal productor de aldosterona.
Sndrome de Cushing ectpico.
Tratamiento antihipertensivo con diurticos.
Ingesta de regaliz.

En el tratamiento del hiperaldosteronismo NO suele emplearse:


1)
2)

Suprarrenalectoma.
Espironolactona.

CTO Medicina C/ Nez de Balboa, 115 28006 MADRID (Espaa) Tfno.: (91) 782 43 32 / Fax: (91) 782 43 27
E-mail: secretaria@ctomedicina.com; iberocto@ctomedicina.com WEB: www.ctomedicina.com; www.iberocto.com

Preguntas TEST

Seguimiento a distancia

ENDOCRINOLOGA

Preparacin Examen de Seleccin 05/06 1 Vuelta


3)
4)
5)
64.

Si, en una mujer con hirsutismo usted detecta niveles muy


elevados de testosterona, lo ms probable es:
1)
2)
3)
4)
5)

65.

Preguntas TEST

4)
5)
72.

Prednisona.
Fenoxibenzamina.
Propranolol.
Prazosn.
Fentolamina.

73.

TAC abdominal.
Resonancia magntica abdominal.
Gammagrafa con MIBG.
Gammagrafa con I-colesterol.
Arteriografa abdominal.

La mayora de los incidentalomas suprarrenales se corresponden con:


1)
2)

Carcinoma suprarrenal.
Adenoma suprarrenal productor de aldosterona.
M exico A rgentina
C hile U ruguay

4)
5)
75.

Paratiroidectoma subtotal.
Continuar con el mismo tratamiento.
Asociar propranolol.
Medir catecolaminas en plasma y orina.
Medir PTH.

Paciente diagnosticado de carcinoma epidermoide de pulmn, que acude al servicio de urgencias por cuadro de
poliuria y deshidratacin leve, presentando en la analtica
hipercalcemia de 12,5 mg/dl. Resto de parmetros bioqumicos dentro de la normalidad. Seale la respuesta FALSA:
1)
2)
3)

PAAF de la masa con control ecogrfico.


Cortisol libre urinario.
Catecolaminas y sus metabolitos en orina.
17-cetosteroides en orina.
TC abdominal.

Ms de 50 aos.
Calcio total en sangre >12 mg/dl.
Densidad sea menor de -2DS.
Litiasis renal recidivante.
Calciuria > 400 mg/d.

Mujer diagnosticada de hiperplasia paratiroidea con calcemia de 12 mg/dl, a pesar del tratamiento. Antecedentes
personales de HTA mal controlada y asma. Antecedentes
familiares de cncer de tiroides. Qu actitud tomara a
continuacin?:
1)
2)
3)
4)
5)

74.

La causa ms frecuente de hipercalcemia en pacientes


ambulatorios es el hiperparatiroidismo primario.
La causa ms frecuente de hiperparatiroidismo primario
es la existencia de hiperplasia paratiroidea.
En la mayora de los casos, el diagnstico de hiperparatiroidismo primario se realiza de forma casual al detectar en
una analtica hipercalcemia con hipofosfatemia en personas asintomticas.
En el diagnstico diferencial se deben descartar frmacos
y neoplasias.
Las neoplasias, son la causa ms frecuente de hipercalcemia en pacientes hospitalizados.

En un paciente con hiperparatiroidismo primario, cul NO


es criterio de ciruga?:
1)
2)
3)
4)
5)

Paciente con antecedentes de HTA (en tratamiento con


captopril y diurticos, manteniendo TA alrededor de 170/
110 mmHg). Casualmente, se encuentra una masa suprarrenal en una ecografa abdominal. Qu actitud NO tomara?:
1)
2)
3)
4)
5)

70.

3)

Cul de las siguientes pruebas de imagen NO es til en el


diagnstico de feocromocitoma?:
1)
2)
3)
4)
5)

69.

2)

Enfermedad de Addison autoinmune.


Sndrome de Cushing.
Carcinoma suprarrenal.
Tumores virilizantes.
Hiperplasia suprarrenal congnita.

Catecolaminas en plasma.
Prueba con glucagn.
Prueba con fentolamina.
Catecolaminas libres y metanefrinas en orina de 24 h.
Gammagrafa con metayodobencilguanidina - I-131.

En relacin con la hipercalcemia, una de las siguientes afirmaciones es FALSA:


1)

Un enfermo diagnosticado de feocromocitoma va a ser


operado. Qu tratamiento preoperatorio indicara usted?:
1)
2)
3)
4)
5)

68.

71.

Para descartar feocromocitoma, solicitara inicialmente:


1)
2)
3)
4)
5)

67.

Hiperplasia suprarrenal congnita (inicio tardo).


Neoplasia ovrica.
Ovario poliqustico.
Neoplasia suprarrenal.
Idioptico.

Feocromocitoma.
Adenoma suprarrenal no funcionante.
Metstasis.

METABOLISMO DEL CALCIO.

Cul es el trastorno suprarrenal ms frecuente en la infancia?:


1)
2)
3)
4)
5)

66.

3)
4)
5)

Dexametasona.
Antihipertensivos.
Furosemida.

Seguimiento a distancia

Debe recibir tratamiento con suero salino isotnico.


Los niveles de vitamina D sern normales o algo bajos.
La eficacia del tratamiento ser menor que en las hipercalcemias secundarias a adenoma paratiroideo.
Las cifras de PTH intacta estarn elevadas.
En el tratamiento pueden ser de utilidad bifosfonatos,
calcitonina y glucocorticoides.

Paciente diagnosticado de mieloma mltiple e insuficiencia


renal crnica, que acude al servicio de urgencias por cuadro
progresivo de astenia, debilidad de la musculatura proximal,
letargia y oligoanuria. En la analtica presenta: creatinina 4

CTO Medicina C/ Nez de Balboa, 115 28006 MADRID (Espaa) Tfno.: (91) 782 43 32 / Fax: (91) 782 43 27
E-mail: secretaria@ctomedicina.com; iberocto@ctomedicina.com WEB: www.ctomedicina.com; www.iberocto.com

ED Pg. 7

ENDOCRINOLOGA

Preparacin Examen de Seleccin 05/06 1 Vuelta

mg/dl, urea 180 mg/dl, sodio 130 mEq/l, potasio 6 mEq/l,


calcio 14,5 mg/dl. En la exploracin fsica presenta edemas
maleolares. Cul de los siguientes sera el tratamiento inicial
de eleccin?:
1)
2)
3)
4)
5)

Hidratacin con suero salino isotnico.


Furosemida i.v.
Hemodilisis.
Calcitonina.
Bifosfonatos.

1)
2)
3)
4)
5)
80.

Seale cul es la FALSA sobre el tratamiento de la descompensacin hiperosmolar:


1)
2)

DIABETES MELLITUS.

3)

76.

4)
5)

Un paciente presenta, en una analtica realizada en ayunas,


una glucemia basal de 120 mg/dl. Utilizando los criterios
diagnsticos de diabetes mellitus de la ADA de 1997, usted le
podra diagnosticar de:
1)
2)
3)
4)
5)

77.

2)
3)
4)
5)

Se hereda de forma autosmica recesiva ligada al cromosoma 6.


Es una enfermedad que se desarrolla siempre sobre una
base exclusivamente gentica.
Existe una activacin de la inmunidad humoral (anticuerpos antiislote, antiinsulina), permaneciendo inactiva la
inmunidad celular.
Existen anticuerpos contra la descarboxilasa del cido
glutmico (GAD) que pueden reaccionar con los islotes
pancreticos.
Su etiopatogenia es equiparable a la de la DM no insulindependiente.

2)
3)
4)
5)
83.

3)
4)
5)

Se hereda de forma autosmica dominante.


Para su diagnstico se exige la presencia de la enfermedad
en al menos dos miembros de la familia.
Se presenta en edades generalmente inferiores a los 25 aos.
Existe una alteracin de la secrecin de insulina y una
resistencia a la accin de la insulina en los tejidos.
El tipo 2 se debe a una alteracin del gen de la glucoquinasa que se encuentra en el cromosoma 7p.

Varn de 20 aos, sin antecedentes conocidos, que acude


por vmitos, dolor abdominal y poliuria. GAB: pH 7,21,
HCO3: 16, pCO2: 20, pO2: 90. Na 120, K 4,5, Cl 90, glucosa
500, cetonuria positiva, creatinina 1,5. Cul de las siguientes
medidas NO est indicada?:

Pg. 8 ED

M exico A rgentina
C hile U ruguay

Hiperglucemia matutina, como consecuencia de hipoglucemia nocturna, con estmulo de hormonas contrainsulares.
Hiperglucemia matutina por hipersecrecin de GH nocturna.
Hipoglucemia matutina, como consecuencia de hiperglucemia nocturna, con secrecin de insulina.
Hipoglucemia matutina por exceso de insulina administrada.
Hiperglucemia a las 3 a.m., que causa hipoglucemia
nocturna.

Un paciente de 50 aos, obeso, hipertenso y fumador,


consulta por el hallazgo casual de una glucemia en ayunas de
256 mg/dl, no acompaada de sntomas cardinales. En consulta presenta una glucemia capilar en ayunas de 240 mg/dl.
Tras iniciar tratamiento con dieta hipocalrica y metformina
en dosis crecientes hasta alcanzar dosis mximas, el paciente
presenta glucemias basales de 210, 200 y 189 mg/dl y no ha
perdido peso. Cmo modificara su tratamiento?:
1)
2)
3)
4)
5)

84.

Glucosuria.
Cetonuria.
Glucemia.
Hemoglobina glicada (HbA1c).
Fructosamina.

Se llama efecto Somogyi a:


1)

Sobre la diabetes mellitus tipo MODY, seale la FALSA:


1)
2)

79.

82.

Es tpico de la DM autoinmune.
Los aportes de potasio son necesarios ms precozmente
que en la cetoacidosis.
Los aportes de bicarbonato son necesarios cuando existe
acidosis lctica.
La insulina permite el control de la hiperglucemia.
La mortalidad es mayor, en general, que en la cetoacidosis.

En el control de un paciente diabtico, es POCO importante


medir:
1)
2)
3)
4)
5)

Sobre la diabetes mellitus insulinodependiente, seale la


correcta:
1)

78.

La cifra es normal. No se debe realizar ninguna medida.


Si la cifra en ayunas est entre 110 y 126 mg/dl, se
etiquetar como alteracin de la glucosa en ayunas.
La cifra indica alteracin de la glucosa en ayunas. Si la
glucemia, 2 horas despus de 75 g de glucosa, es inferior a
140 mg/dl, se diagnosticar de intolerancia hidrocarbonada.
Si se realiza una sobrecarga de glucosa y la glucemia a las 2
horas est entre 140 y 200 mg/dl, se le etiquetar de diabetes.
Si se repite 1 sola vez y la cifra en ayunas es superior o igual
a 126 mg/dl, se le puede etiquetar de diabetes.

81.

Suero salino 0,9%.


Cuando glucemia sea de 300, suero glucosado 5%.
Insulina i.v.
Potasio i.v.
Bicarbonato i.v.

Insistir en la dieta. Aceptable control glucmico, no hay


que modificar el tratamiento.
Insistir en la dieta. Sustituir biguanidas por acarbosa.
Insistir en la dieta. Asociar acarbosa al tratamiento con
metformina.
Insistir en la dieta. Suspender biguanidas y comenzar con
sulfonilureas.
Insistir en la dieta. Asociar insulina nocturna.

Slo una de las siguientes afirmaciones es cierta, en relacin


con el tratamiento farmacolgico de la diabetes mellitus:
1)

Glimepirida - nuevo y potente inhibidor de las alfaglucosidasas intestinales, que reduce la glucemia postprandial.

CTO Medicina C/ Nez de Balboa, 115 28006 MADRID (Espaa) Tfno.: (91) 782 43 32 / Fax: (91) 782 43 27
E-mail: secretaria@ctomedicina.com; iberocto@ctomedicina.com WEB: www.ctomedicina.com; www.iberocto.com

Preguntas TEST

Seguimiento a distancia

ENDOCRINOLOGA

Preparacin Examen de Seleccin 05/06 1 Vuelta


2)
3)
4)
5)
85.

Paciente de 55 aos, con antecedentes personales de HTA


controlada con dieta, obesidad, hipercolesterolemia en tratamiento con estatinas y diabetes mellitus tipo 2, tratada con
metformina. Acude a la consulta presentando cifras de glucemia capilar en ayunas entre 180-250 mg/dL, con cifras
postprandiales aceptables. En la analtica presenta HbA1c de
8%, LDL:120, TGs: 150, y cifras habituales de presin arterial
de 145/80 mmHg. Cul sera la actitud ms correcta en
cuanto a la modificacin del tratamiento?:
1)
2)
3)
4)
5)

86.

3)
4)
5)

Preguntas TEST

Aadir IECAs, suspender metformina e iniciar tratamiento


con insulina en pauta intensiva, dados los factores de
riesgo cardiovascular que presenta el paciente.
Insistir en dieta hiposdica para el control de la presin
arterial, suspender metformina e iniciar tratamiento con
sulfonilureas o repaglinida.
Aadir IECAs, mantener metformina y aadir rosiglitazona
o pioglitazona, valorando la respuesta en visitas sucesivas.
Aadir IECAs, mantener metformina y aadir al tratamiento sulfonilureas y acarbosa, valorando la respuesta al
tratamiento en visitas sucesivas.
Aadir IECAs, iniciar tratamiento con insulina en pauta
convencional, suspendiendo metformina, al igual que la
estatina, ya que las cifras de LDL y TGs son excelentes.

Seale la asociacin INCORRECTA en cuanto al tipo de


insulina:
1)
2)

87.

Miglitol - biguanida de ltima generacin.


Metformina - sulfonilurea, hipoglucemiante oral.
Repaglinida - sulfonilurea de 4 generacin, de accin
rpida.
Rosiglitazona - acta a nivel del receptor nuclear PPAR,
reduciendo la resistencia insulnica.

Lispro - anlogo de insulina de accin ultrarrpida.


Glargina - insulina de accin muy prolongada, ms de 24
horas.
NPL - insulina de accin intermedia con perfil farmacocintico similar a la NPH.
Insulina inhalada - farmacocintica similar a la insulina
regular recombinante humana.
Insulina asprtica - insulina ultratard.

2)
3)
4)
5)
89.

90.

2)
3)
4)
5)

Debut de diabetes mellitus en un paciente de 14 aos con


cuerpos cetnicos en orina.
Diabetes en una paciente de 38 aos que ha debutado
con una cetoacidosis diabtica.
Nio de 10 aos con DM tipo 1.
Diabtico tipo 2 de 53 aos, en mal control crnico con
pauta de insulinoterapia convencional.
Diabtico tipo 2 de 85 aos de edad, con complicaciones
microvasculares evolucionadas y en mal control crnico
con pauta convencional.

HIPOGLUCEMIAS.
Seala la respuesta correcta respecto al diagnstico diferencial de la hipoglucemia:
M exico A rgentina
C hile U ruguay

Los pacientes con insulinoma tienen una concentracin


de pptido C aumentada paralela a los valores de insulina.
La proinsulina se eleva cuando se administra insulina
exgena.
El pptido C se eleva cuando se administra insulina
exgena.
Los antidiabticos orales elevan la concentracin de
insulina, pero no de pptido C.
El cociente insulina/glucosa es normal en los insulinomas.

La causa ms frecuente de hipoglucemia en un paciente


hospitalizado es:
1)
2)
3)
4)
5)

Tumores.
Desnutricin.
Hepatopata.
Frmacos.
Septicemia.

En el tratamiento del insulinoma se emplean todos los siguientes, EXCEPTO:


1)
2)
3)
4)
5)

Pancreatectoma.
Diazxido.
Octretide.
Estreptozocina.
Fenoxibenzamina.

DISLIPEMIA, OBESIDAD Y NUTRICIN.


91.

En un paciente varn de 50 aos, con un infarto agudo de


miocardio hace 2 aos y que presenta cifras elevadas de
colesterol, cul sera su objetivo de control de la hipercolesterolemia?:
1)
2)
3)
4)
5)

92.

NO indicara el tratamiento intensivo con insulina en uno de


los siguientes:
1)

88.

1)

Seguimiento a distancia

Dentro de las indicaciones de tratamiento hipolipemiante en


los diabticos, seale la verdadera:
1)
2)
3)
4)
5)

93.

LDL < 200.


LDL < 160.
LDL < 130.
LDL < 100.
LDL indetectable.

Nunca se deben usar estatinas, pues alteran las cifras de


glucemia.
Es siempre de eleccin el uso de cido nicotnico.
Se recomienda mantener los niveles de LDL por debajo de
100 mg/dl en los pacientes diabticos con cardiopata
isqumica.
El miglitol es el hipolipemiante ms potente que se conoce
hoy en da.
La cerivastatina es un fibrato que disminuye slo los niveles
de triglicridos.

Seale la asociacin frmaco-mecanismo de accin INCORRECTA:


1)
2)

Atorvastatina - inhibicin de la HMG CoA reductasa.


Colestiramina - interrupcin de la circulacin enteroheptica.

CTO Medicina C/ Nez de Balboa, 115 28006 MADRID (Espaa) Tfno.: (91) 782 43 32 / Fax: (91) 782 43 27
E-mail: secretaria@ctomedicina.com; iberocto@ctomedicina.com WEB: www.ctomedicina.com; www.iberocto.com

ED Pg. 9

Seguimiento a distancia
3)
4)
5)

Seale el efecto ADVERSO fundamental de los inhibidores de


la HMG CoA reductasa:
1)
2)
3)
4)
5)

95.

2)
3)
4)
5)

3)
4)
5)

5)

IMC > 40, sin respuesta a dieta, ejercicio y frmacos.


IMC > 35, con diabetes mellitus.
IMC > 35, con sndrome de apnea del sueo.
Paciente con trastorno de la conducta alimentaria de tipo
bulimia.
IMC > 35, con cardiopata isqumica.

Protenas plasmticas de vida media corta-intermedia, como


prealbmina, transferrina y protena ligada a retinol.
Pliegue tricipital.
Recuento de linfocitos.
Niveles de vitaminas y oligoelementos.
Albmina en un paciente en UCI y situacin crtica.

Disfagia secundaria a ACVA.


Paciente joven en coma por traumatismo craneoenceflico.
Anorexia nerviosa con incapacidad para mantener un
peso mnimo mediante alimentacin oral.
Cuadro de obstruccin intestinal.
Fibrosis qustica con requerimientos nutricionales muy
altos.

Hiperglucemia y alteraciones hidroelectrolticas.


Vmitos.
Sepsis por catter.
Neumotrax.
Esteatosis heptica.

Cul de las siguientes es FALSA respecto a la clasificacin de


la poblacin segn su ndice de masa corporal?:
1)
2)
3)
4)
5)

99.

1)
2)
3)
4)

Son posibles complicaciones asociadas a la nutricin parenteral, todas MENOS una:


1)
2)
3)
4)
5)

98.

En cul de los siguientes casos NO indicara la realizacin de


ciruga baritrica?:

Indicara una nutricin enteral por sonda nasogstrica en


todas las situaciones siguientes, EXCEPTO una:
1)
2)

97.

Arritmias auriculares.
Estreimiento.
Diarrea y nuseas.
Miositis y afectacin heptica.
Colelitiasis.

100.

Cul de los siguientes parmetros NO es til en la valoracin


del estado nutricional de un paciente?:
1)

96.

Gemfibrocil - aumento de hidrlisis de triglicridos y


catabolismo de LDL.
cido nicotnico - disminuye sntesis de VLDL y LDL.
Lovastatina - aumenta la sntesis de cidos biliares.

IMC de 18,5 a 24,9 normalidad.


IMC de 25 a 29,9 sobrepeso.
IMC de 30 a 34,9 obesidad grado 1.
IMC de 35 a 39,9 obesidad grado 2.
IMC mayor de 40 superobesidad o extrema.

En un paciente con un IMC de 38 y con una ingesta habitual


de 3.000 Kcal, qu dieta prescribira?:
1)
2)
3)
4)
5)

500-700 Kcal.
1.000-1.500 Kcal.
1.500-2.500 Kcal.
3.000-3.500 Kcal.
No precisa dieta.

Pg. 10 ED

M exico A rgentina
C hile U ruguay

CTO Medicina C/ Nez de Balboa, 115 28006 MADRID (Espaa) Tfno.: (91) 782 43 32 / Fax: (91) 782 43 27
E-mail: secretaria@ctomedicina.com; iberocto@ctomedicina.com WEB: www.ctomedicina.com; www.iberocto.com

Preguntas TEST

94.

ENDOCRINOLOGA

Preparacin Examen de Seleccin 05/06 1 Vuelta

ENDOCRINOLOGA

Preparacin Examen de Seleccin 05/06 1 Vuelta


HIPFISIS.

Seguimiento a distancia

Pregunta 3.- R: 4

Pregunta 1.- R: 5
La hiperprolactinemia se puede producir en muchas situaciones, siendo la ms frecuente la secundaria a frmacos como estrgenos, opiceos,
neurolpticos, metildopa, etc. El s. de Nelson es un adenoma hipofisario
que aparece aos despus de una suprarrenalectoma bilateral realizada a
pacientes con Cushing. Puede producir hiperprolactinemia por compresin del tallo hipofisario. Este es el mismo mecanismo de los macroadenomas
no secretores de hipfisis. El craneofaringioma es el tumor hipotalmico
ms frecuente en nios, y en su crecimiento compromete las funciones
hipotalmicas, incluyendo la secrecin de dopamina, motivo por el que
eleva las cifras de prolactina. Las formas de hipofisitis linfocitarias que
afectan al tallo tambin elevan las cifras de prolactina. Sin embargo, el s. de
Sheehan consiste en una necrosis isqumica postparto hipofisaria que se
manifiesta por incapacidad para la lactancia, al disminuir la prolactina.
Stress
Succin del
pezn

5HT

VIP, TRH
Prolactina

Secrecin
lctea

Pregunta 1.

Neurolpticos
Frmacos Opiceos
Alfa-metildopa
Reserpina
Dopamina

Estrgenos

Regulacin de la prolactina.

Comentarios TEST

Pregunta 2.- R: 4
Existen cifras orientativas para el diagnstico diferencial de la hiperprolactinemia. Aquellas superiores a 300 microgramos/L son caractersticas del prolactinoma. Igualmente, descartado un embarazo, cifras
mayores a 150 tambin lo son. Las cifras entre 50-100 suelen ser
causas 2 como compresin del tallo (respuesta 4), lesiones hipotalmicas, frmacos, etc. Ver figura a pie de pgina.

Pregunta 3.

Manejo de la hiperprolactinemia.

Pregunta 2. Patologas hipofisarias-hipotalmicas.


M exico A rgentina
C hile U ruguay

CTO Medicina C/ Nez de Balboa, 115 28006 MADRID (Espaa) Tfno.: (91) 782 43 32 / Fax: (91) 782 43 27
E-mail: secretaria@ctomedicina.com; iberocto@ctomedicina.com WEB: www.ctomedicina.com; www.iberocto.com

ED Pg. 1

ENDOCRINOLOGA

Seguimiento a distancia

Preparacin Examen de Seleccin 05/06 1 Vuelta

Pregunta 4.- R: 3
La infertilidad es una manifestacin clnica de los prolactinomas.
Aunque los estrgenos aumentan los niveles de prolactina, la mayora
de los prolactinomas no crecen en el embarazo. Por ello la actitud
correcta es la 3. Ninguno de los agonistas dopaminrgicos es teratognico, por lo que ante unos niveles de prolactina mayores de 400 o
alteraciones campimtricas se reinicia el tratamiento.
Pregunta 5.- R: 2
La acromegalia se debe casi siempre a tumores hipofisarios productores de GH y los pacientes tienen un aumento de morbimortalidad por
causas cardiovasculares y aumento de tumores. Los criterios diagnsticos
actuales son niveles de IGF-1 elevados para sexo y edad y GH mayor de
1 microg/L tras SOG. El MEN 1 asocia tumores de hipfisis, paratiroides y
pncreas y se debe sospechar en un acromeglico con hipercalcemia, ya
que sta no se produce en la acromegalia aislada.

Pregunta 5. Criterios de acromegalia.


CRITERIOS DIAGNSTICOS
SOG para GH (120 min.): GH>1 con IRMA (> 2g/l con RIA).
Aumento de IGF-1 para valor normal segn edad y sex o.

gos de somatostatina (octretide), a los que se pueden asociar agonistas


dopaminrgicos, es la ltima lnea. No est claro todava el papel del
tratamiento con anlogos de somatostatina antes de la ciruga.
Pregunta 7.- R: 2
Se habla de curacin bioqumica en la acromegalia cuando se
normalizan los niveles de IGF-1 y se suprime la GH a menos de 1 tras
SOG. Ello evita el exceso de mortalidad de estos pacientes. Las respuestas paradjicas a GnRH y TRH son menos usadas en la actualidad
como criterios pronsticos. Las indicaciones de colonoscopia en la
acromegalia son: AP o familiares de plipos en colon, clnica intestinal, ms de 10 aos de enfermedad, sobre todo si activa y en pacientes con ms de 3-6 acrocordomas en cuello o dorso superior.
Pregunta 8.- R: 1
En el enunciado nos dan la clnica de varios dficit hormonales
asociados: alteracin de libido e impotencia (hipogonadismo), astenia
y nuseas (insuficiencia suprarrenal), piel seca y letargia (hipotiroidismo). Los iones y colesterol normales orientan hacia un origen hipofisario. Por tanto, de las respuestas, la patologa que no produce hipopituitarismo es el microprolactinoma. Recordar que la sensibilidad de la RM
no es del 100% en la deteccin de toda la patologa selar.
Pregunta 9.- R: 1
La apopleja hipofisaria es un cuadro grave con clnica neurolgica en su presentacin, por necrosis de tumores hipofisarios. El diagnstico diferencial con otras entidades tales como la hemorragia
subaracnoidea o rotura de aneurismas cerebrales, etc, es lo primordial. Su tratamiento debe ser urgente, con dosis altas de corticoides y
neurociruga descompresiva urgente.

SOG para GH (120 min.): GH<1 con IRMA (<2 ug/l con RIA).
IGF-1: normales (para edad y sex o).

Pregunta 10.- R: 2
En endocrinologa, los dficits hormonales se diagnostican con
determinaciones basales si no hay solapamiento con valores normales, en cuyo caso se necesitan realizar pruebas de estimulacin. La
mejor de ellas para valorar el dficit de ACTH es la hipoglucemia con
insulina. Ver el resto en la tabla.

Pregunta 6.- R: 2
El tratamiento de 1 eleccin de la acromegalia es en todos los casos la
ciruga transesfenoidal, seguida por la radioterapia asociada a anlogos
de somatostatina hasta que la radio acte en caso de persistencia de
actividad y resto tumoral tras la ciruga. El tratamiento mdico con anlo-

Pregunta 11.- R: 3
El enunciado nos presenta un caso de s. de Sheehan, que consiste
en una necrosis isqumica postparto hipofisaria que se manifiesta por
incapacidad para la lactancia, al disminuir la prolactina. Dado que
puede producir hipopituitarismo, no habr hiperpigmentacin, ya
que los niveles de ACTH estarn disminuidos.

CRITERIOS DE CURACIN

Pregunta 10. Diagnstico y tratamiento del hipopituitarismo.


Hormona

Determinaciones basales

GH

IGF-1 (puede tener falsos negativos en el


diagnstico de dficit de GH)

1. Hipoglucemia insulnica (la de mayor


utilidad) para GH.
2. Otras: arginina, ornitina, clonidina,
GHR H, hexarelin).

ACTH

Cortisol basal <3,5 es diagnstico,


>18 mg/dL excluye.

1. Hipoglucemia insulnica (la de mayor


utilidad) para cortisol.
2. Test de ACTH con 1 ug para cortisol.

PRL

Prolactina basal (a veces no excluye el dficit). Si


alta indica lesin en hipotlamo o tallo.

TSH

LH/FSH

Pg. 2 ED

Pruebas funcionales

Estimulacin con TR H o metoclopramida


para excluir el dficit.

Test de TR H (en desuso porque no


TSH y T4L basales (un 30% de pacientes con TSH
discrimina bien entre lesin hipotalmica e
basal normal)
hipofisaria)

Testosterona varones.
Menstruacin mujeres.
LH y FSH basales si las anteriores alteradas.

M exico A rgentina
C hile U ruguay

1. Test de estimulacin con GnR H.


2. Estimulacin con clomifeno.

Tratamiento
1. Nios: dficit aislado o combinado.
2. Adultos: dficit GH con hipopituitarismo.

Hidrocortisona oral o IV en caso de crisis.


Primer dficit a sustituir.
No se sustituye.
Lactancia artificial.
Levotiroxina oral despus de corticoides si
dficit de ACTH.
1. Esteroides gonadales si no deseo de
fertilidad.
2. LH y FSH si deseo de fertilidad.
3. GnR H en bomba con pulsos en
algunos casos.

CTO Medicina C/ Nez de Balboa, 115 28006 MADRID (Espaa) Tfno.: (91) 782 43 32 / Fax: (91) 782 43 27
E-mail: secretaria@ctomedicina.com; iberocto@ctomedicina.com WEB: www.ctomedicina.com; www.iberocto.com

Comentarios TEST

El tratamiento de primera eleccin en los prolactinomas, independientemente del tamao, son los agonistas dopaminrgicos (bromocriptina, quinagolida y cabergolina). En caso de fracaso teraputico,
la siguiente opcin es la ciruga transesfenoidal. La radioterapia es
poco eficaz en los prolactinomas y se reserva para los macroprolactinomas resistentes a agonistas dopaminrgicos con resto tumoral tras
ciruga. Todos los macroprolactinomas se tratan siempre, mientras
que los micros o la causa idioptica se tratan si dan sntomas.

ENDOCRINOLOGA

Preparacin Examen de Seleccin 05/06 1 Vuelta


Pregunta 12.- R: 2
El enunciado nos presenta un caso de s. de Nelson que es un
adenoma hipofisario que aparece aos tras una suprarrenalectoma
bilateral realizada a pacientes con Cushing. Puede producir hiperprolactinemia por compresin del tallo hipofisario, hiperpigmentacin por secrecin de ACTH y clnica compresiva local como cefalea,
alteraciones visuales, etc.
Pregunta 13.- R: 5
Ante un s. polirico y polidpsico, lo primero es descartar por su
prevalencia una diabetes mellitus. El siguiente paso es determinar
osmolaridad simultnea en plasma y orina. Aquellos con osm-p normal y osm-o disminuida son candidatos a un test de deshidratacin.
En caso de estar deshidratados, inicialmente no se debe realizar esta
prueba y bastara con la inyeccin de vasopresina de entrada.

Pregunta 13. Diagnstico de diabetes inspida.


DI Central

DI nefrognica

PP

Osm. ur. tras


deshidratacin

No cambia

No cambia

Aumenta

Osm. tras ADH

Su b e > 1 0 %

Su b e < 9 %

Su b e < 9 %

La diabetes inspida central se trata con desmopresina (DDAVP) oral


o nasal en pacientes conscientes, y parenteral si estn inconscientes.

Seguimiento a distancia

to de que la yoduria refleja la cantidad de yodo de que dispone una


persona. Sin embargo, el yodo en situaciones y dosis determinadas
puede producir efectos distintos a los fisiolgicos. Un ejemplo es el
fenmeno de Jod-Basedow que ocurre en pacientes con bocio ante
la administracin de dosis elevadas de yodo, y cuyo efecto es la
aparicin de un hipertiroidismo que en el momento del diagnstico
tiene una gammagrafa hipocaptante. Otro ejemplo es el WolffChaikoff, que consiste en la inhibicin de la organificacin ante la
administracin de dosis altas de yodo en pacientes con hipertiroidismo. Otros efectos son la induccin de bocio o hipotiroidismo en
pacientes con enfermedades autoinmunes de tiroides o la fibrosis
preoperatoria, que se consigue con la administracin de lugol.
Pregunta 18.- R: 5
El transporte de yodo al interior de la clula folicular se realiza mediante un proceso activo en el que se acopla la actividad de la Na/K
ATPasa a un simporte I/Na (NIS). El yodo es organificado en el coloide
(por la peroxidasa tiroidea) y se une a los residuos de tirosina de la
tiroglobulina (protena producida por la clula folicular). Una vez formados, los complejos monoyodotirosina y diyodotirosina se acoplan
para formar las hormonas tiroideas, que son liberadas al torrente sanguneo y el yodo no utilizado se recupera por una dehalogenasa del
citosol de la clula folicular. La nica fuente de T4 es el tiroides, que
tambin produce una pequea cantidad de T3. Sin embargo, los tejidos perifricos son capaces de producir T3 a partir de la desyodacin
de la T4.

Pregunta 14.- R: 1
La DI nefrognica tiene un comienzo ms insidioso que la central
(comienzo brusco). Las causas ms frecuentes de DI nefrognica son
las 2, debidas a enfermedades tubulointersticiales renales o por frmacos como el litio o la demeclociclina. La hipocaliemia o hipercalcemia crnicas tambin pueden producirla. El hipotiroidismo, sin
embargo, es causa de hiponatremia, incluso de SIADH.
Pregunta 15.- R: 3
Los criterios diagnsticos de SIADH incluyen la hiponatremia con
hiposmolaridad plasmtica, volumen circulante normal (no edemas ni
hipotensin), osm en orina inapropiadamente elevada (mayor de 100300) y la exclusin de hipotiroidismo e insuficiencia suprarrenal.

Pregunta 15. Criterios diagnsticos de SIADH.

Comentarios TEST

Mayores

Menores

1. Hiponatremia.
2. Hiposmolaridad plasmtica.
3. No edemas.
4. No deplecin de volumen
(T.A. normal).
5. Falta de dilucin mx ima de orina
(osm orina >100 mOsm/kg).
6. Ex clusin de hipotiroidismo e
insuficiencia suprarrenal.

Pregunta 18. Sntesis de hormonas tiroideas.


1. Sobrecarga hdrica
patolgica.
2. Niveles de AVP plasma y
orina elevados.

*Se precisan todos los criterios mayores para el diagnstico. Los menores son
opcionales.

Pregunta 16.- R: 1
El tratamiento del SIADH es urgente en caso de Na menor de 125
o clnica grave del SNC, en cuyo caso se realiza con una infusin de
suero salino hipertnico con o sin furosemida IV. El tratamiento del
SIADH es conservador con Na mayor de 125 y sin clnica. Para ello
comenzamos con restriccin hdrica (500-1.000cc/d), y si no se controla la hipoNa, usaremos frmacos como el litio o la demeclociclina.
TIROIDES.
Pregunta 17.- R: 2
El yodo es necesario para la formacin de las hormonas tiroideas,
y su metabolismo est muy estrechamente controlado, hasta el punM exico A rgentina
C hile U ruguay

Pregunta 19.- R: 3
Como hemos dicho antes, los tejidos perifricos son capaces de
producir T3 a partir de la desyodacin de la T4. La inhibicin de las
desyodasas perifricas se puede producir en muchas situaciones tales
como enfermedades graves, desnutricin o ayuno severos, politraumatismos (sndrome de enfermedad sistmica no tiroidea) o por la
accin de frmacos tales como la amiodarona, dexametasona, propranolol, contrastes yodados y propiltiouracilo (el nico antitiroideo
que posee este efecto).
Pregunta 20.- R: 3
Las enfermedades tiroideas autoinmunes constituyen un grupo en el
que se encuentran la tiroiditis linfocitaria crnica (de Hashimoto y la
variante atrfica), la enfermedad de Graves-Basedow y la tiroiditis indolora o linfocitaria transitoria. En todas ellas pueden aparecer con frecuencia
variable anticuerpos antitiroideos (antitiroglobulina y antiperoxidasa). Hay
que diferenciarlos de los anticuerpos frente al receptor de TSH, que son
tpicos, aunque no exclusivos de la enfermedad de Graves.
Pregunta 21.- R: 3
El bocio simple se define como un aumento del tamao de la
glndula tiroidea con funcin tiroidea normal, es decir, todas la hormonas normales en plasma. El tratamiento, en el caso de que existan

CTO Medicina C/ Nez de Balboa, 115 28006 MADRID (Espaa) Tfno.: (91) 782 43 32 / Fax: (91) 782 43 27
E-mail: secretaria@ctomedicina.com; iberocto@ctomedicina.com WEB: www.ctomedicina.com; www.iberocto.com

ED Pg. 3

ENDOCRINOLOGA

Preparacin Examen de Seleccin 05/06 1 Vuelta

fenmenos compresivos tales como disfagia, disnea, compresin de


vasos con pltora facial y sncope (signo de Pemberton) es la ciruga
mediante tiroidectoma subtotal. El resto de los casos se debe individualizar, aunque el tratamiento ms empleado es la administracin
de dosis supresoras de tiroxina.
Pregunta 22.- R: 3
Bocio significa aumento del tamao de la glndula tiroidea y no
define su funcionalidad. Podemos tener patologas con bocio e hipotiroidismo como la tiroiditis de Hashimoto, bocio con hipertiroidismo
como la enfermedad de Graves o bocio con normofuncin tiroidea
(bocio simple). El cncer diferenciado de tiroides es raramente
funcionante.

Pregunta 29.- R: 2
La gammagrafa es una prueba til para aproximarse al diagnstico
de hipertiroidismo, ya que nos separa las causas producidas por
hiperfuncin autnoma del tiroides (gammagrafa hipercaptante) de
las que tienen otra fisiopatologa (gammagrafa silente). En este sentido
cabe recordar que el coriocarcinoma produce un hipertiroidismo en
el que la gammagrafa es de hipercaptacin difusa, como en la enfermedad de Graves. En el caso del coriocarcinoma es la beta-hCG la
que estimula el receptor de TSH.

Pregunta 29. Clasificacin de los estados de tirotoxicosis


segn la captacin gammagrfica.

Pregunta 23.- R: 4
Como se ha dicho anteriormente, el bocio simple consiste en aumento del tamao de la glndula tiroidea con normofuncin tiroidea. En su diagnstico se pueden realizar pruebas de imagen para
descartar los fenmenos compresivos en otros rganos o para el diagnstico diferencial con otras entidades (bocio multinodular, ndulos
solitarios, etc.). La determinacin de tiroglobulina y la realizacin de
rastreos con I-131 forman parte del seguimiento de los pacientes operados de cncer diferenciado de tiroides y no juegan ningn papel en
el diagnstico del bocio simple.

Captacin aumentada:
- Enfermedad de Graves.
- Tumores productores de TSH y hCG.
- BMNT y adenoma txico.
Captacin disminuida:
- Tiroiditis y fenmeno de Jod-Basedow.
- Tirotoxicosis facticia.
- Struma ovarii.
- Metstasis funcionantes de carcinoma folicular.

Pregunta 24.- R: 5
La causa ms frecuente de bocio, simple o con hipotiroidismo, es
el dficit de yodo. Muchas de las causas de bocio simple pueden
producir con el tiempo o la dosis hipotiroidismo. Bocigenos como la
amiodarona, el litio, etc. son un ejemplo. Igualmente el yodo en pacientes con patologa autoinmune de tiroides puede producir bocio,
incluso hipotiroidismo.

Pregunta 30.- R: 4
La enfermedad de Graves es autoinmune y el hipertiroidismo no
sigue un curso paralelo al de la oftalmopata en muchos casos. El
tratamiento debe individualizarse dependiendo del grupo de edad
(nios y jvenes, adultos o ancianos) y ante la presencia de embarazo
en una mujer. En general, en nuestro medio se recomienda tratamiento con antitiroideos prolongado en nios, jvenes y adultos (entre 12
y 24 meses) para conseguir un efecto inmunomodulador y as evitar
brotes de hipertiroidismo futuros.

Pregunta 25.- R: 2
El tratamiento sustitutivo del hipotiroidismo se realiza en la actualidad con tiroxina (T4) oral. El objetivo del tratamiento es la normalizacin de los niveles de TSH en caso de hipotiroidismo primario y de los
niveles de T4 libre en el caso de hipotiroidismo central. En la pregunta
nos encontramos ante un caso de hipotiroidismo primario en el que
la TSH del plasma est por debajo del lmite normal, lo que indica
que la dosis empleada es elevada para ese paciente, y la actitud es
disminuir la dosis y reevaluar al paciente pasadas 4-6 semanas con
una nueva determinacin de TSH.

Pregunta 31.- R: 3
La administracin de yodo-131 es una herramienta til en el tratamiento de la enfermedad de Graves, tanto por su eficacia como por su
seguridad. Est contraindicado en embarazadas, nios y jvenes por
debajo de los 20 aos y no debe emplearse ante la existencia de
ndulos tiroideos sin antes investigar la naturaleza de los mismos. En
los adultos mayores y ancianos es de eleccin tras la normalizacin
de la funcin tiroidea con antitiroideos.

Pregunta 26.- R: 4
El hipotiroidismo subclnico es la situacin definida por unos niveles
de TSH plasmtica elevados con una T4 libre normal en plasma. Hoy en
da se conoce que no es una situacin verdaderamente subclnica y se
recomienda su tratamiento siempre que la TSH est por encima de 10
mUI/mL. Cuando est entre 5 y 10, se recomienda tratar en la mayora de
las situaciones, excepto en ancianos o pacientes con cardiopata.

Pregunta 32.- R: 4
En el embarazo, la enfermedad de Graves debe controlarse con
antitiroideos (especialmente con propiltiouracilo) en dosis suficientes
para mantener las hormonas maternas en el lmite alto de la normalidad,
y as evitar el hipotiroidismo fetal. En general, la enfermedad de Graves
mejora a lo largo del embarazo, pero en casos refractarios, dado que el
radioyodo est contraindicado, se debe recurrir a la ciruga.

Pregunta 27.- R: 4
El coma mixedematoso es una urgencia rara hoy en da que ocurre
en pacientes con hipotiroidismo de larga duracin o mal tratados
cuando se exponen a un fro intenso, ciruga, sepsis y otros factores
desencadenantes. El pronstico es malo y conlleva una elevada mortalidad. El tratamiento debe iniciarse ante la sospecha y confirmarse
mediante analtica de hormonas urgente. Inicialmente se administran
glucocorticoides intravenosos, y despus hormona tiroidea, bien T4
intravenosa o T3 por sonda nasogstrica.

Pregunta 33.- R: 1
Las urgencias en el hipertiroidismo son la crisis tirotxica o tormenta tiroidea y las crisis cardacas, como la del enunciado. En estos casos
el tratamiento est dirigido a controlar de forma rpida los sntomas y
la disminucin de la cantidad de hormonas tiroideas en el organismo.
Ello es posible de forma rpida, sobre todo mediante la inhibicin de
la conversin perifrica de T4 a T3. El tratamiento definitivo (en este
caso el yodo radiactivo) debe postponerse hasta la normalizacin de
la situacin de tirotoxicosis.

Pregunta 28.- R: 3
La tirotoxicosis facticia consiste en el hipertiroidismo por ingesta de
hormonas tiroideas. En el caso de la toma de preparados de T3, el
perfil hormonal es una TSH suprimida con T3 libre elevada y T4
disminuida, lo que lleva al diagnstico. En el caso de ingesta de T4, el
diagnstico diferencial debe hacerse con otras situaciones de hipertiroidismo con gammagrafa hipocaptante, ya que en este caso el perfil
hormonal es el de una TSH suprimida con niveles tanto de T3 libre
como de T4 libre elevados.

Pregunta 34.- R: 2
La mayora de los ndulos tiroideos solitarios son benignos (el 95%),
incluso el ndulo solitario, que es fro en la gammagrafa, tambin
tiene alta probabilidad de ser benigno (el 80%). Existen una serie de
factores que aumentan el riesgo de que un ndulo sea maligno, tales
como la existencia de adenopatas, la fijacin a estructuras adyacentes, el crecimiento rpido indoloro del ndulo, el tamao mayor de 4
cms, sexo varn, edades extremas de la vida (ancianos o nios), antecedentes familiares de cncer de tiroides o personales de radiacin en
la infancia. La prueba que aporta hoy en da una mayor razn de

Pg. 4 ED

M exico A rgentina
C hile U ruguay

CTO Medicina C/ Nez de Balboa, 115 28006 MADRID (Espaa) Tfno.: (91) 782 43 32 / Fax: (91) 782 43 27
E-mail: secretaria@ctomedicina.com; iberocto@ctomedicina.com WEB: www.ctomedicina.com; www.iberocto.com

Comentarios TEST

Seguimiento a distancia

ENDOCRINOLOGA

Preparacin Examen de Seleccin 05/06 1 Vuelta


probabilidades positiva (sensibilidad/1-especificidad) es la puncin
aspiracin con aguja fina PAAF, y por tanto, la que se recomienda
como prueba inicial para decidir la actitud.
Pregunta 35.- R: 3
La mayora de los ndulos tiroideos solitarios son benignos (el 95%),
incluso el ndulo solitario, que es fro en la gammagrafa, tambin tiene
alta probabilidad de ser benigno (el 80%), siendo histolgicamente quistes
coloidales o adenomas foliculares no funcionantes.
Pregunta 36.- R: 1
Aunque la PAAF es la mejor opcin diagnstica ante el ndulo
tiroideo, tambin tiene falsos negativos y positivos. En caso de PAAF
maligna se debe intervenir al paciente, y en caso de ser benigna, se
realiza un seguimiento bajo tratamiento supresor con tiroxina. En caso
de PAAF folicular o no concluyente es importante tener en cuenta otros
datos de riesgo para cncer de tiroides ya comentados antes.
Ndulo tiroideo

PAAF
Maligno

Folicular

No concluyente
Tiene
factores
de riesgo
para
ca tiroides

Gammagrafa

Benigno

No

Fro

Caliente

Supresin
con L-T4 3 meses
y repetir PAAF
Aumento de
tamao
Tiroidectoma
total

Estudio
hipertiroidismo

Sin cambio
o reduccin
de tamao

Hemitiroidectoma e
istmectoma para estudio A.P
Maligno

Benigno

Seguimiento

Comentarios TEST

Pregunta 36. Actitud ante el ndulo tiroideo solitario.

Pregunta 37.- R: 2

Seguimiento a distancia

metstasis incluso tras este perodo, lo que obliga a un seguimiento de por


vida de estos pacientes. El tratamiento de eleccin hoy en da es la
tiroidectoma total con linfadenectoma en caso de afectacin ganglionar.
Los pacientes reciben tratamiento supresor con tiroxina de por vida, y en
caso de elevacin de tiroglobulina, se someten a un rastreo con yodo
radiactivo para posterior ablacin de los restos o metstasis captantes.
Pregunta 38.- R: 5
El carcinoma medular de tiroides es espordico en la mayora de los
casos, pero puede ser familiar y asociarse a la neoplasia endocrina mltiple
tipo 2 (carcinoma medular, hiperparatiroidismo y feocromocitoma en el
2a y carcinoma medular, feocromocitoma y neuromatosis en el 2b), de
carcter autosmico dominante con penetrancia variable. En estos casos se
ha descrito en la mayora de las familias afectadas, mutaciones en el
protooncogen RET, que debe investigarse en dicha situacin. El carcinoma
medular secreta calcitonina, la que se usa para el diagnstico y el seguimiento de los pacientes. La terapia ms eficaz y la nica que ha demostrado
modificar la supervivencia es la ciruga. Este cncer no capta I-131 y no
responde de forma eficaz a quimioterapia ni radioterapia externa.
Pregunta 39.- R: 2
Los ndulos tiroideos sospechosos de malignidad deben remitirse
a ciruga para biopsia de la pieza, lo que aporta el diagnstico definitivo. Por tanto, los ndulos que crecen con tratamiento supresor con
levotiroxina, ndulos con PAAF no concluyente en pacientes con
factores de riesgo o PAAF folicular con gammagrafa fra, deben remitirse para hemitiroidectoma y biopsia de la pieza, tras cuyo resultado
se proceder o no segn el caso a ampliar la reseccin con tiroidectoma total y exploracin ganglionar.
Pregunta 40.- R: 3
Aunque la PAAF es la mejor opcin diagnstica ante el ndulo
tiroideo, tambin tiene falsos negativos y positivos. En caso de PAAF
maligna se debe intervenir al paciente, y en caso de ser benigna, se
realiza un seguimiento bajo tratamiento supresor con tiroxina. En caso
de PAAF folicular o no concluyente, es importante tener en cuenta
que una serie de factores aumentan el riesgo de que un ndulo sea
maligno, tales como la existencia de adenopatas, la fijacin a estructuras adyacentes, compresin del larngeo recurrente, el crecimiento
rpido indoloro del ndulo, el tamao mayor de 4 cms, sexo varn,
edades extremas de la vida (ancianos o nios), antecedentes familiares
de cncer de tiroides o personales de radiacin en la infancia, as
como el aumento de riesgo de cncer de tiroides en los ndulos fros.
Pregunta 41.- R: 3
Se trata de una paciente con carcinoma diferenciado de tiroides con
tiroidectoma que recibi dosis ablativa de restos tiroideos con radioyodo.
Ante la elevacin de las cifras de tiroglobulina se sospecha recidiva o
metstasis de la enfermedad y procedemos a un rastreo corporal con
yodo-131 para localizar la enfermedad residual, y en caso de captacin
positiva, la administracin de una dosis ablativa del istopo.
Pregunta 42.- R: 3

Pregunta 42. Factores de riesgo de carcinoma de tiroides


ante un ndulo tiroideo.
Antecedentes

Familiares de cncer de tiroides o MEN 2.


Personales de radiacin en cabeza o cuello, sobre
todo en infancia.

Anamnesis

E da d > 4 5 a o s o < 1 6 .
Sexo masculino.
Ndulo palpable reciente, de crecimiento rpido
e indoloro.
R onquera por parlisis del larngeo recurrente.

Exploracin

Tamao > 4 cms.


Adenopatas palpables.
Fijacin a estructuras profundas (no desplazable
con deglucin).

Pregunta 37. Seguimiento del paciente con carcinoma diferenciado de


tiroides.TG: tiroglobulina, RCT: rastreo con I-131.

El carcinoma diferenciado de tiroides (papilar y folicular) tiene una


supervivencia a los 20 aos de casi un 95% en el papilar y 85% en el
folicular, aunque por otro lado, estn descritas las recidivas tumorales y
M exico A rgentina
C hile U ruguay

CTO Medicina C/ Nez de Balboa, 115 28006 MADRID (Espaa) Tfno.: (91) 782 43 32 / Fax: (91) 782 43 27
E-mail: secretaria@ctomedicina.com; iberocto@ctomedicina.com WEB: www.ctomedicina.com; www.iberocto.com

ED Pg. 5

ENDOCRINOLOGA

Seguimiento a distancia

Preparacin Examen de Seleccin 05/06 1 Vuelta


Pregunta 44. Caractersticas de las principales tiroiditis.

ETIOLOGA

CLNICA

DIAGNSTICO

TRATAMIENTO

Aguda

Bacteriana

Dolor, calor, rubor y tum efaccin en cara


anterior del cuello, sntom as sistm icos de
infeccin

Fiebre, leucocitosis con


desviacin izquierda

Antibiticos.
Drenaje quirrgico.

Subaguda

Viral

Linfocitaria transitoria

Autoinm une?

Linfocitaria crnica

Autoinm une

Bocio indoloro, hipotiroidism o.


A veces hipertiroidism o inicial
autolim itado (Hashitoxicosis).

Fibrosante

Desconocida

Bocio m uy duro, sntom as de


com presin cervical
Hipotiroidism o 25%.

Bocio doloroso nodular.


Dolor cervical anterior, febrcula,
hipertiroidism o.

VSG aum entada, captacin


n u l a e n g a m m a g r a f a ,
autoanticuerpos negativos.

Bocio no doloroso, hipertiroidism o.


Posible hipotiroidism o transitorio.

Una serie de factores aumentan el riesgo de que un ndulo sea


maligno, tales como la existencia de adenopatas, la fijacin a estructuras adyacentes, compresin del larngeo recurrente, el crecimiento
rpido indoloro del ndulo, el tamao mayor de 4 cms, sexo varn,
edades extremas de la vida (ancianos o nios), antecedentes familiares
de cncer de tiroides o personales de radiacin en la infancia, as
como el aumento de riesgo de cncer de tiroides en los ndulos fros.

Aspirina.
Corticoides.
Betabloqueantes.

VSG norm al, captacin nula


e n g a m m a g r a f a , t t u l o s b a jo s
de antiperoxidasa.

Betabloqueantes.
Levotiroxina.

Altos ttulos de antiperoxidasa

Tiroxina

Captacin nula en
gam m agrafa, anticuerpos
n e g a t iv o s

Ciruga si produce
com presin

SUPRARRENAL.
Pregunta 45.- R: 5
La causa ms frecuente de sndrome de Cushing es el yatrgeno o
exgeno. Se debe a la administracin de esteroides de forma crnica
y es la causa de s. Cushing ms frecuente en la prctica clnica. Su
diagnstico se realiza con la demostracin de la supresin del cortisol
y ACTH del paciente que acude con clnica compatible e historia de
tratamiento con esteroides.

Pregunta 43.- R: 3
Las tiroiditis son procesos en los que existe una inflamacin de la
glndula. En muchas de ellas el patrn funcional es de una primera
fase de hipertiroidismo por liberacin de hormonas tiroideas preformadas al torrente sanguneo y una segunda fase de hipotiroidismo
que puede ser autolimitada. La tiroiditis fibrosante o de Riedel consiste en una fibrosis del cuello que puede afectar al mediastino y al
retroperitoneo de forma concomitante, en la que la funcin tiroidea
suele ser normal, existiendo hipotiroidismo hasta en el 25% de los
casos, pero el hipertiroidismo no est descrito en esta entidad.

Pregunta 46.- R: 4
Dado que hemos dicho que la ACTH estar suprimida, no puede
haber hiperpigmentacin. El resto son todas compatibles con el exceso de glucocorticoides y pueden verse en el Cushing exgeno.

Pregunta 44.- R: 5
La tiroiditis granulomatosa subaguda (de Quervain) se debe a causas virales que producen una inflamacin del tiroides que produce
dolor e hipertiroidismo con febrcula y sntomas de catarro de vas
altas. El diagnstico se basa en la comprobacin de VSG elevada,
junto con una gammagrafa hipocaptante o silente. El enunciado nos
muestra una tiroiditis fibrosante, que no debe confundirse con la
tiroiditis subaguda. Si bien la VSG puede estar elevada en la fibrosante,
no suele ser del rango de la de Quervain. Ver tabla en pgina siguiente.

Pregunta 48.- R: 4
El enunciado hace referencia a un Cushing ectpico agresivo en el
que predomina la hiperpigmentacin, las alteraciones metablicas del
exceso de cortisol y la clnica del cncer de base. En estos casos puede
no existir hbito Cushingoide en el paciente, ya que el cambio en el
fenotipo por efecto del hipercortisolismo puede llevar su tiempo.

Pregunta 51. Diagnstico diferencial del sndrome de Cushing.

123456789

7 3749 7
756789 

4244782

943 9



M exico A rgentina
C hile U ruguay

35 9

 492 7892

2492 7892

647 9

39

12

12

34

12534

12

34

34

12534

 39 7 7!7

" 9 739 4982


7 3 # 7 7437
$
6 4 7

Pg. 6 ED

 5

CTO Medicina C/ Nez de Balboa, 115 28006 MADRID (Espaa) Tfno.: (91) 782 43 32 / Fax: (91) 782 43 27
E-mail: secretaria@ctomedicina.com; iberocto@ctomedicina.com WEB: www.ctomedicina.com; www.iberocto.com

Comentarios TEST

Pregunta 47.- R: 3
Dentro del Cushing endgeno nos encontramos con: el adenoma hipofisario productor ACTH (70%): Microadenoma (80-90%) y
macroadenoma (10-20%); tumor ectpico productor ACTH (1520%), adenoma suprarrenal (10%) e hiperplasia nodular suprarrenal (<1%).

ENDOCRINOLOGA

Preparacin Examen de Seleccin 05/06 1 Vuelta


Pregunta 48. Clnica del Cushing ectpico.
Tumores agresivos

Tumores no agresivos

Ej: microctico de pulmn.

Ej: carcinoide.

Hiperpigmentacin.
Alteraciones metablicas:
Hiperglucemia.
Alcalosis metablica.
Hipokalemia.

Seguimiento a distancia

Pregunta 54. Diagnstico diferencial de la insuficiencia suprarrenal.

Fenotipo cushingoide (parecido a


la clnica del Cushing hipofisario)

Pregunta 49.- R: 3
El primer paso en el diagnstico de sndrome de Cushing es en
todos los casos la determinacin del hipercortisolismo. Para ello se
usan varias pruebas, pero se suelen emplear las llamadas de screening
por la comodidad en la realizacin y la posibilidad de hacerlo ambulatoriamente. Son prcticamente equivalentes y se usan: determinacin de cortisol en orina de 24h y cortisol en plasma tras la administracin nocturna de 1 mg de dexametasona.
Pregunta 50.- R: 2
Clsicamente se consideraba como prueba definitiva de confirmacin del hipercortisolismo a la supresin dbil con 0,5 mg de dexametasona cada 6 horas durante dos das. Ms recientemente otras
pruebas tales como la determinacin de cortisol en plasma o en saliva
entre las 23 y 24 horas de la noche parecen incluso ms eficaces
aunque su prctica no es generalizada an.

Comentarios TEST

Pregunta 51.- R: 4
Una vez diagnosticado el sndrome de Cushing , el siguiente paso
es el diagnstico de la etiologa del mismo. Para ello, la determinacin de ACTH es el paso siguiente que permite separar el ACTH
independiente (suprarrenal) del dependiente (central o ectpico).
Otras pruebas tiles son la supresin fuerte con dexametasona y la
estimulacin del eje con CRH o metopirona. Ver tabla.
Pregunta 52.- R: 5
El tratamiento del Cushing es etiolgico, siendo la ciruga transesfenoidal de eleccin en los tumores hipofisarios y la extirpacin del
tumor ectpico o la suprarrenalectoma en caso de adenoma o hiperplasia suprarrenal. Cuando no se localiza la lesin se procede a bloqueo qumico de la secrecin de cortisol con ketoconazol, y en caso
de fracaso del mismo o de aparicin de efectos secundarios se realiza
suprarrenalectoma bilateral.
Pregunta 53.- R: 2
La causa ms frecuente en la actualidad de enfermedad de Addison o insuficiencia suprarrenal primaria es la adrenalitis autoinmune,
en la que una inflamacin crnica de la glndula produce su destruccin y atrofia. Pueden aparecer en estos pacientes anticuerpos
antiadrenales y asociarse a otras patologas autoinmunes. Existen dos
asociaciones dentro de sndromes poliglandulares autoinmunes especficos: el tipo 1, en el que existe una mutacin del gen AIRE,
autosmica recesiva en el que se asocia adrenalitis autoinmune con
hipoparatiroidismo autoinmune y candidiasis, y el tipo 2 o s. de
Schmidt, en el que existe una predisposicin gentica asociada a
determinados haplotipos del HLA y asocia, adems de adrenalitis
autoinmune, diabetes mellitus autoinmune y enfermedad autoinmune tiroidea.
Pregunta 54.- R: 2
La insuficiencia suprarrenal aguda o crisis suprarrenal puede deberse a varias causas, siendo la ms frecuente la suspensin brusca de
un tratamiento glucocorticoideo prolongado. En este caso el paciente
tiene clnica de insuficiencia suprarrenal secundaria, en la que existe
hiponatremia por prdida de la accin de los glucocorticoides en la
nefrona, que fisiolgicamente favorecen el aclaramiento de agua libre. Ver tabla en la pgina siguiente.
M exico A rgentina
C hile U ruguay

Pregunta 55.- R: 3
Aparte de lo comentado en la pregunta anterior, otras causas de
crisis suprarrenal son el debut de una enfermedad de Addison o los
pacientes con dicha patologa que ante el estrs fsico no doblan o
triplican la dosis de glucocorticoides. Causas ms raras son la hemorragia suprarrenal, en nios por meningococemia diseminada (s.
Waterhouse-Friderichsen) y en adultos, asociada a anticoagulacin.
La suspensin brusca de AINEs no produce crisis suprarrenal.
Pregunta 56.- R: 5
Conviene distinguir las alteraciones metablicas de distintas patologas suprarrenales, ya que pueden ser confundidas en el examen MIR,
sobre todo si no se tienen los conceptos claros. Para ello recordar que
el exceso de mineralcorticoides produce alcalosis e hipopotasemia, ya
que la aldosterona reabsorbe sodio en el tbulo distal intercambindolo
por potasio e hidrogeniones. No existe hipernatremia en esta situacin
debido al fenmeno de escape que ocurre con el tiempo por la accin
de los pptidos natriurticos. Al revs, en los casos de hipoaldosteronismo
(lo que puede ser aislado o formar parte de un Addison) existe
hiperpotasemia y acidosis metablica. Hay que recordar que, adems
del papel de aclaramiento de agua libre que tienen los glucocorticoides
en la nefrona, su exceso importante puede activar los receptores tipo 1
renales y producir efecto mineralcorticoide, de ah la posibilidad de
alcalosis e hipopotasemia en el Cushing. Dentro de las causas de diabetes inspida nefrognica recordar que la hipopotasemia crnica (ej.: en
el hiperaldosteronismo primario) y la hipercalcemia crnica (ej.: en el
hiperparatiroidismo primario) pueden producirla.
Pregunta 57.- R: 3
Como se dijo anteriormente, la adrenalitis autoinmune puede asociarse a otras patologas autoinmunes, como en los sndromes
poliglandulares autoinmunes: el tipo 1, en el que existe una mutacin
del gen AIRE, autosmica recesiva, en el que se asocia adrenalitis
autoinmune con hipoparatiroidismo autoinmune y candidiasis y el
tipo 2 o s. de Schmidt, en el que existe una predisposicin gentica
asociada a determinados haplotipos del HLA y asocia, adems de
adrenalitis autoinmune, diabetes mellitus autoinmune y enfermedad
autoinmune tiroidea.
Pregunta 58.- R: 2
En el enunciado se describe un paciente con HTA, hipopotasemia y alcalosis y clnica de diabetes inspida nefrognica. Adems

CTO Medicina C/ Nez de Balboa, 115 28006 MADRID (Espaa) Tfno.: (91) 782 43 32 / Fax: (91) 782 43 27
E-mail: secretaria@ctomedicina.com; iberocto@ctomedicina.com WEB: www.ctomedicina.com; www.iberocto.com

ED Pg. 7

ENDOCRINOLOGA

Preparacin Examen de Seleccin 05/06 1 Vuelta

nos indican cifras de aldosterona elevadas que no se suprime (prueba de supresin con infusin de suero salino, administracin de
fludrocortisona o de captopril) y de renina plasmtica (ARP) disminuida que no se estimula (prueba de estimulacin con ortostatismo
o furosemida). Ello es compatible con un hiperaldosteronismo primario, entre cuyas causas estn el aldosteronoma, la hiperplasia
nodular bilateral, el carcinoma suprarrenal (raro) y el hiperaldosteronismo remediable con glucocorticoides (s. de Sutherland). Las respuestas 1, 3 y 4 corresponden a excesos de otros mineralcorticoides
o alteraciones funcionales que aumentan la actividad mineralcorticoidea y en estos casos tanto la aldosterona como la ARP estn
disminuidas. En el hiperaldosteronismo secundario, la ARP est elevada, lo que aumenta la aldosterona.

te, el hiperaldosteronismo primario es responsable de un 1-2% de


todos los casos de HTA. Por ello, la actitud correcta en estos casos es
suspender el diurtico, administrar suplementos de potasio y reevaluar al paciente a los 15 das. Si presenta entonces todava hipopotasemia, es preciso un estudio de aldosterona y ARP.

Pregunta 59.- R: 5
En circunstancias normales, la regulacin de la aldosterona se realiza por el eje renina-angiotensina y los niveles de potasio en plasma.
Aunque la ACTH es un estimulador de la produccin de aldosterona,
fisiolgicamente tiene mucha menos importancia que los dos anteriores. En el s. Sutherland o hiperaldosteronismo remediable con
glucocorticoides, se produce una translocacin gentica en la que la
aldosterona pasa a producirse en las capas internas de la corteza
suprarrenal con el promotor donde se une la ACTH. Ello produce un
hiperaldosteronismo que se trata con glucocorticoides para reducir
los niveles de ACTH plasmticos.

Pregunta 64.- R: 2
Denominamos hirsutismo a la aparicin de vello en la mujer en las
zonas terminales, es decir, aquellas que son de distribucin de vello
tpicas de varn. El hirsutismo se valora clnicamente por la escala de
Ferriman y se considera que existe cuando la puntuacin es superior
a 7. Cuando el hirsutismo es muy rpido en aparicin y se acompaa
de virilizacin, hay que sospechar un cncer productor de andrgenos
(ovrico si existe exceso grave de testosterona o suprarrenal ante exceso severo de DHEA-S). Afortunadamente las causas ms frecuentes son
las idiopticas (hormonas normales) y funcionales (ovario poliqustico
o defectos enzimticos de la suprarrenal).

Pregunta 60.- R: 2
El cortisol se degrada a cortisona en el rin para evitar su unin a
los receptores mineralcorticoideos. La enzima responsable de este
paso es la 11-OH-esteroide deshidrogenasa. Existen sustancias que
inhiben la actividad de esta enzima tales como el regaliz o la carbenoxolona, o bien puede estar alterada de forma gentica. En estos casos
el cortisol ejerce en concentraciones fisiolgicas efecto
mineralcorticoideo y se produce un sndrome denominado de exceso aparente de mineralcorticoides.

Pregunta 65.- R: 5
La hiperplasia suprarrenal congnita agrupa las alteraciones genticas de la esteroidognesis suprarrenal. La ms frecuente es el dficit de
21-hidroxilasa, del que existen dos formas: la forma clsica (neonatal o
de aparicin en la niez) y no clsica o tarda (de aparicin peripuberal
en la mujer en forma de clnica de hiperandrogenismo).

Pregunta 61.- R: 2
Dentro de los tipos de hiperplasia suprarrenal congnita ms frecuentes, los que cursan con HTA son el dficit de 11-hidroxilasa y el de
17-hidroxilasa. Ambos producen un exceso de mineralcorticoides que
es la DOCA (deoxicorticosterona), un precursor de la aldosterona con
efecto mineralcorticoide menos intenso. El s. de Conn hace referencia
al aldosteronoma y la respuesta 5 al exceso aparente de
mineralcorticoides.

Pregunta 63.- R: 5
La furosemida est contraindicada en el hiperaldosteronismo primario ya que agrava la hipopotasemia y la alcalosis metablica. La
ciruga se reserva para los tumores productores de aldosterona, la
espironolactona para el tratamiento preoperatorio de stos o el tratamiento crnico de la hiperplasia nodular, y los glucocorticoides en el
caso del sndrome de Sutherland.

Pregunta 66.- R: 4
El diagnstico de feocromocitoma es hormonal, y para ello realizamos una determinacin conjunta en orina de 24 horas de catecolaminas y metanefrinas. Su elevacin patolgica nos da el diagnstico, y
el paso siguiente sera proceder a la localizacin del tumor, la mayora
de los cuales estn localizados en la suprarrenal (>90%). Para ello
indicamos un TC o RM suprarrenal. La realizacin de una gammagrafa
con meta-yodo-bencil-guanidina (MIBG) se recomienda, ya que es la
tcnica que mejor localiza metstasis de feocromocitoma o los casos
de tumor extrasuprarrenal. La prueba de provocacin con glucagn
no se recomienda y la de supresin con fentolamina suele realizarse
cuando es necesario administrar el frmaco ante crisis HTA de los
pacientes.
Pregunta 67.- R: 2
El tratamiento preoperatorio del feocromocitoma consiste en el
bloqueo progresivo y completo de los receptores alfa adrenrgicos,
para lo que se emplea fenoxibenzamina por va oral unos 15 a 21
das antes de la ciruga. El uso de betabloqueantes puede ser necesario ante la taquicardia refleja de los pacientes o arritmias, sobre todo
en el momento de la induccin anestsica, pero nunca se deben
administrar antes que el alfa ya que se podran inducir crisis
hipertensivas por bloqueo de los receptores beta vasculares que son
vasodilatadores.
Pregunta 68.- R: 4
La gammagrafa con yodo-colesterol es til para la visualizacin de
la patologa de la corteza suprarrenal, pero no tiene ningn papel en
el diagnstico del feocromocitoma. La arteriografa rara vez es necesaria en la actualidad para el diagnstico del feocromocitoma, y debe
realizarse con precaucin.

Pregunta 61. Esteroidognesis suprarrenal.

Pregunta 62.- R: 4
Desde un punto de vista estadstico, ante un paciente con HTA e
hipopotasemia, dado que los diurticos tiacdicos y de asa son muy
empleados para el tratamiento de la HTA, lo ms probable es que la
hipopotasemia sea un efecto secundario de los diurticos. No obstanPg. 8 ED

M exico A rgentina
C hile U ruguay

Pregunta 69.- R: 1
La realizacin de una PAAF en un feocromocitoma est contraindicada; por ello, ante un ndulo o masa suprarrenal, antes de cualquier otra actitud, debe descartarse la existencia de un feocromocitoma, incluso en el paciente asintomtico, ya que hasta el 10% de los
feocromocitomas son diagnosticados como incidentales.

CTO Medicina C/ Nez de Balboa, 115 28006 MADRID (Espaa) Tfno.: (91) 782 43 32 / Fax: (91) 782 43 27
E-mail: secretaria@ctomedicina.com; iberocto@ctomedicina.com WEB: www.ctomedicina.com; www.iberocto.com

Comentarios TEST

Seguimiento a distancia

ENDOCRINOLOGA

Preparacin Examen de Seleccin 05/06 1 Vuelta

Seguimiento a distancia

Pregunta 73.- R: 4
El caso clnico nos muestra la asociacin de hiperplasia de paratiroides con HTA y cncer de tiroides (medular), es decir un MEN 2a. La
actitud correcta en estos casos, al igual que siempre antes de intervenir
un CMT, es la exclusin de feocromocitoma mediante determinacin
de catecolaminas y metanefrinas en orina de 24h.

Pregunta 73. Asociaciones del MEN 2.


Tipo

MEN 2A

Sndrome

Caractersticas

MEN 2A

Carcinoma medular tiroides (90%).


Feocromocitoma (50%).
Hiperplasia/adenomas paratiroides
(30%).

CMF

Carcinoma medular familiar aislado.

Con amilodosis
liquenoide cutnea

Lesin pruriginosa en dorso

Con enfermedad de MEN 2A o CMF con agangliosis


Hirschsprung
colnica.

Pregunta 69. Actitud ante un incidentaloma suprarrenal.

Pregunta 70.- R: 4
Tras el estudio de un incidentaloma suprarrenal, segn indica la
figura de la pregunta anterior, la mayora sern adenomas no funcionantes, en los que la actitud es vigilancia con pruebas de imagen cada
6-12 meses, y en caso de crecimiento, proceder a la reseccin.
PATOLOGA DEL CALCIO.

Comentarios TEST

Pregunta 71.- R: 2
La causa ms frecuente de hipercalcemia en pacientes ambulatorios
es el hiperparatiroidismo primario que produce una hipercalcemia crnica y asintomtica en la mayora de los pacientes. El diagnstico se
realiza demostrando niveles de PTHi elevados ante cifras elevadas de
calcio en plasma. La causa ms frecuente es el adenoma nico de
paratiroides. Por otro lado, la hipercalcemia tumoral es aguda y sintomtica,
y es la causa ms frecuente de hipercalcemia en pacientes ingresados.
Pregunta 72.- R: 1
Dado que el hiperparatiroidismo primario produce una hipercalcemia crnica y asintomtica en la mayora de los pacientes, y aunque la extirpacin quirrgica cura la enfermedad, hay que valorar su
realizacin. Las indicaciones actuales son: todos los menores de 50
aos, y en los mayores de 50 cuando alguna de las siguientes est
presente: complicaciones, calcio plasma mayor de 12, calcio en orina de 24h mayor de 400 mg, prdida de masa sea en densitometra
mayor de 2 DS. La realizacin de pruebas de localizacin
preoperatorias no es necesaria, pues la exploracin quirrgica es lo
ms eficaz para localizar el adenoma o identificar hiperplasia.

Pregunta 72. Indicaciones de ciruga en el


hiperparatiroidismo primario.
Menores de 50 aos.
Mayores de 50 aos con alguna de las siguientes:
- Calcio srico 1,6 mg/dl por encima del nivel mximo del laboratorio, es decir, mayor de 12 mg/dl en plasma.
- Historia de un episodio de hipercalcemia potencialmente letal.
- Reduccin del aclaramiento de creatinina en ms del 30%.
- Existencia de clculos renales en las Rx abdominales o nefrolitiasis
recidivante.
- Calciuria superior a 400 mg/da.
- Reduccin de la masa sea ms de 2 desviaciones tpicas en la
densitometra.

M exico A rgentina
C hile U ruguay

MEN 2B

MEN 2B
Marfanoide

CMT, feocromocitoma,
ganglioneuromatosis intestinal y
mucosa.
Idem con hbito marfanoide.

Pregunta 74.- R: 4
Los mecanismos de hipercalcemia tumoral son variados. En los
casos de ostelisis intensa, como las metstasis del cncer de mama o
el mieloma mltiple, tanto el calcio como el fsforo suelen estar elevados, al igual que otros marcadores de resorcin sea (fosfatasa alcalina,
etc). La produccin ectpica de hormonas por parte del tumor es otra
causa (hipercalcemia paraneoplsica). La produccin en exceso de
vit D ocurre en algunos linfomas y la de PTHrp (protena relacionada
con la PTH) en muchos tumores, especialmente epidermoides. En
este ltimo caso, la PTHi (fisiolgica) est suprimida.
Pregunta 75.- R: 3
El tratamiento de la hipercalcemia aguda debe ser urgente. La medida inicial ms eficaz es forzar la diuresis con suero salino IV al que se
puede aadir furosemida IV. La calcitonina sc o im tambin es eficaz,
pero sufre efecto de taquifilaxia en 24-48h. En caso de insuficiencia
renal grave o riesgo alto de descompensacin cardiovascular por sobrecarga de volumen se debe dializar al paciente. Los bifosfonatos IV
son muy eficaces, aunque su accin ocurre a las 48-72h y estn
contraindicados en la insuficiencia renal. Los corticoides son eficaces
en la ostelisis, el mieloma y el exceso de vit D, pero su comienzo de
accin es a los 3-5 das de su administracin.
DIABETES MELLITUS.
Pregunta 76.- R: 2
Los criterios diagnsticos de DM actuales son los siguientes: 1)
glucemia mayor o igual a 200 en cualquier momento del da, junto
con clnica cardinal, 2) glucemia mayor o igual a 126 en ayunas y 3)
glucemia mayor o igual a 200 a las 2 horas de una sobrecarga oral con
75 g de glucosa. El diagnstico se establece confirmando cualquiera
de estos criterios otro da. Existen dos categoras que aumentan el
riesgo de desarrollar DM con el tiempo y son: 1) glucosa en ayunas
alterada, cuando la glucemia en ayunas est entre 110-125 y 2) intolerancia a la glucosa, cuando a las 2 horas de una sobrecarga oral con
75 g de glucosa los niveles estn entre 140-199. Estas categoras no
requieren confirmacin.
Pregunta 77.- R: 4
La DM tipo 1 es una enfermedad autoinmune en la que existe una
predisposicin gentica a padecerla asociada a determinados

CTO Medicina C/ Nez de Balboa, 115 28006 MADRID (Espaa) Tfno.: (91) 782 43 32 / Fax: (91) 782 43 27
E-mail: secretaria@ctomedicina.com; iberocto@ctomedicina.com WEB: www.ctomedicina.com; www.iberocto.com

ED Pg. 9

ENDOCRINOLOGA

Preparacin Examen de Seleccin 05/06 1 Vuelta

haplotipos del HLA (cromosoma 6). Factores diversos tales como infecciones virales o consumo precoz de leche de vaca, as como otros
no conocidos, inducen el proceso autoinmune. Tanto la inmunidad
celular, que produce un infiltrado de clulas mononucleares en los
islotes pancreticos (insulitis), como la inmunidad humoral
(anticuerpos) se activan. Los anticuerpos producidos son varios, aunque la mayor rentabilidad diagnstica se consigue con la determinacin conjunta de anti-IA2 y GAD.

Pregunta 77. Autoanticuerpos en la diabetes mellitus tipo I.


Nomenclatura

Antgeno

Utilidad

INS-Ab

Molcula intacta de insulina


del paciente

Baja rentabilidad
diagnstica

ICAs

Anticuerpos frente a clulas


de los islotes pancreticos

Sensibilidad moderada en
el diagnstico

GAD

Decarboxilasa del cido


glutmico

IA2

Anticuerpos frente a
fosfatasa de tirosina-kinasa
de clula beta

Conjuntamente ofrecen la
mayor rentabilidad
diagnstica. Los usados
actualmente

Pregunta 78.- R: 4
La DM tipo MODY (Maturity Onset Diabetes of the Young) se debe
a un conjunto de alteraciones enzimticas de la clula beta pancretica que altera la secrecin de insulina. Produce una DM leve de
curso ms benigno que otros tipos de DM, aunque vara segn el tipo
de enzima alterada. Es una enfermedad autosmica dominante, de
comienzo antes de los 25 aos de edad. El tipo ms frecuente en
Espaa es el MODY 2, por alteracin en el gen de la glucoquinasa. El
tratamiento inicial es con dieta y ejercicio, y si no hay buen control,
con sulfonilureas y eventualmente insulina.
Pregunta 79.- R: 5
La cetoacidosis diabtica (CAD) es la descompensacin aguda tpica de la DMID. Se define por la presencia de cetoacidosis (pH<7,30
con cetosis) e hiperglucemia (generalmente >300). Tiene una mortalidad actual del 1-2%. La fisiopatologa consiste en un dficit de insulina, junto a un aumento de hormonas contrarreguladoras, sobre
todo glucagn. Ello aumenta la cetognesis heptica, la hiperglucemia y la acidosis metablica. La clnica suele ser inespecfica: nuseas, dolor abdominal, obnubilacin que se correlaciona con osmolaridad elevada y la clnica cardinal de diabetes previa y de la causa
precipitante suelen estar presentes. La CAD no se puede revertir sin
insulina, siendo el objetivo la saturacin completa y rpida de los
receptores de insulina del organismo, manteniendo dicho tratamiento hasta revertir el cuadro, para lo que usamos insulina rpida IV. La
reposicin de lquidos se realiza comenzando con SS fisiolgico, y
cuando la glucemia sea de 250-300 se aporta suero glucosado al 5%
para evitar hipoglucemia y edema cerebral. El potasio debe aportarse
con cifras bajas o normales de entrada y el bicarbonato slo cuando
el pH es inferior a 7,0.
Pregunta 80.- R: 1
La descompensacin hiperglucmica hiperosmolar es la descompensacin aguda tpica de la DMNID. La mortalidad es todava elevada, alrededor del 15%. Se define como una situacin de hiperglucemia mayor de 600 mg/dL, junto con una osmolaridad superior a 320
mosm/L. Buscar siempre una causa precipitante. La clnica suele ser
inespecfica: estupor, obnubilacin, incluso coma, junto con
hipotensin y signos de deshidratacin. Lo ms importante para revertir la situacin de DHH es la administracin de lquidos intravenosos, ya que los pacientes pueden tener una prdida hdrica de hasta el
10% de su peso. Por lo dems, se usa un protocolo similar al de la
cetoacidosis diabtica.

Pg. 10 ED

M exico A rgentina
C hile U ruguay

Pregunta 80. Diferencias entre cetoacidosis diabtica y


descompensacin hiperglucmica hiperosmolar.
Caractersticas

CAD

DHH

Actua lm ente 1- 2%.

Depende de la ca usa
desenca dena nte. En
gener a l un 15%.

Requerimientos de ins ulina

Siem pr e necesa r ia pa r a
r ever tir el cua dr o.
0,1 UI/K g/h en
per fusin continua .

No necesa r ia pa r a
r ever tir el cua dr o per o
a cor ta el tr a ta m iento.
Se suele indica r a
dosis m enor.

Sueroterapia

Inicia lm ente SS
fisiolgico, cua ndo
glucem ia 250- 300
a por ta r SG 5%.

M s ca ntida d de
volum en (ha sta 10%
de pr dida s), glucosa do
e n f a s e s f in a l e s .

Suplementos
de pot a s io

Cua ndo el pota sio en


pla sm a nor m a l o
dism inuido.
Esper a r 2 h si K >5,5- 6.

M s pr ecozm ente sa lvo


en ca sos de FRA
pr er r ena l.

Bicarbonato

Cua ndo pH<7,0.

Slo si a cidosis l ctica


concom ita nte.

Mortalidad

Pregunta 81.- R: 1
El objetivo en el tratamiento de la diabetes es evitar las complicaciones agudas, y sobre todo evitar las complicaciones crnicas micro
y macrovasculares. Diversos estudios han relacionado claramente
los niveles elevados de glucemia con las complicaciones crnicas,
sobre todo microangiopticas (DCCT, UKPDS, etc.). Por ello monitorizamos a los pacientes con niveles de glucemia y de otros parmetros que valoran el control a medio-largo plazo (HbA1c o glicada
nos da el control de los 2-3 meses previos, fructosamina de las 2-3
sem previas). La glucosuria no es til, ya que el dintel plasmtico
para que aparezca glucosa en orina es de 180-200, y adems transgresiones dietticas aisladas pueden producirla sin efecto en el control crnico.

Pregunta 81. Objetivos de control metablico de la diabetes.


Valores deseables

Valores aceptables

HbA1c

< 6,5%

< 7,5%

Glucemias en ayunas

80-110

110-140

Glucemia postpandrial
a las 2h

< 140

140-180

Pregunta 82.- R: 1
Ante una hiperglucemia matutina en un diabtico en tratamiento
con insulina caben bsicamente dos posibilidades: 1) fenmeno del
alba debido a los picos de secrecin nocturna de GH y 2) efecto Somogyi
por aumento de hormonas contrarreguladoras tras una hipoglucemia
de madrugada. El diagnstico diferencial se realiza con una determinacin glucmica a las 3 a.m. (normal en el alba y disminuida en Somogyi).
El primer caso se trata aumentando la insulina NPH de la cena y el
segundo disminuyendo la insulina de la cena.
Pregunta 83.- R: 3
El tratamiento de la diabetes tipo 2 se basa en una actitud escalonada que comienza por la dieta y el ejercicio. El segundo paso es la
monoterapia con antidiabticos orales (ADO) hasta la dosis mxima del
frmaco. Si no se controlara bien el paciente el tercer escaln es la
asociacin de ADOs. En caso de no controlarse as, llegara la insulinizacin, empezando por monodosis nocturna de insulina NPH y manteniendo ADOs durante el da. Es importante tener en cuenta la existencia o no de obesidad a la hora de elegir los frmacos.

CTO Medicina C/ Nez de Balboa, 115 28006 MADRID (Espaa) Tfno.: (91) 782 43 32 / Fax: (91) 782 43 27
E-mail: secretaria@ctomedicina.com; iberocto@ctomedicina.com WEB: www.ctomedicina.com; www.iberocto.com

Comentarios TEST

Seguimiento a distancia

ENDOCRINOLOGA

Preparacin Examen de Seleccin 05/06 1 Vuelta


1er escaln

Pregunta 84.- R: 5
La glimepirida es una sulfonilurea de vida media larga que se toma
en monodosis diaria. El miglitol, como la acarbosa, es un inhibidor de
las disacaridasas intestinales. La metformina es una biguanida que disminuye la resistencia insulnica. La repaglinida es una meglitinida que
produce secrecin rpida de insulina. Ver tabla en pgina siguiente.

DIETA Y EJERCICIO

Obesidad

Peso normal o delgados

Pregunta 85.- R: 3
Otros factores adems de la hiperglucemia se han demostrado incluso ms importantes en la produccin de las complicaciones macroangiopticas: dislipemia, hipertensin y tabaquismo. Las recomendaciones actuales incluyen mantener la TA inferior a 130/80 mmHg, LDL
inferior a 130 -incluso recientemente se aconseja inferior a 100- y
niveles de triglicridos inferiores a 150 y HDL superior a 45, junto con
la abstinencia de fumar. Los IECAs se consideran la primera lnea de
tratamiento de la HTA en los diabticos por su efecto nefroprotector
(aunque recientemente los ARA-2 tambin lo han demostrado).

ADO MONOTERAPIA

2 escaln

Sulfonilureas repaglinida

Metformina

3er escaln

ASOCIACIN ADOS

SU glinidas glitazonas I.D.I

Pregunta 86.- R: 5

Metformina I.D.I

Pregunta 86. Tipos de insulina.

INSULINIZAR

4 escaln

Inicio con monodosis


nocturna de insulina
manteniendo ADO .
Si no hay control pasar
a pauta de dos dosis
manteniendo M etformina.

Seguimiento a distancia

Idem.
Si no hay control pasar
a dos dosis de insulina.
No necesario mantener SU .

Pregunta 83. Intervencin teraputica en la diabetes mellitus tipo 2. SU: sulfonilureas; IDI: inhibidores de las disacaridasas intestinales.

Tipo

Inicio

Pico

Duracin

Prolongada

1,5-3 h

10-16 h

24-30 h

NP H o
intermedia

1-1,5 horas

4-6 h

10-12 h

R egular

30 minutos

2-3h

5-6 h

Anlogos

10 minutos

45 minutos

2-3 horas

La insulina asprtica, ya disponible en nuestro pas, es un anlogo de


insulina que, al igual que la insulina lispro, no forma hexmeros en el

Pregunta 84. Antidiabticos orales.

Comentarios TEST

Sulfonilure as
1 Generacin:
Clorpropamida.
Tolazamida.
Acetohexamida
2 Generacin:
Antidiab tico oral Glibenclamida.
Glipizida.
Gliburide.
G l iq u ido n a .
3 Generacin:
Glimepiride.

Me glitinidas

R epaglinida.
Nateglinida.

Me canismo de
accin

Estimulan la
secrecin de
insulina (Ins)

Regula la secrecin de Ins


actuando en receptores
diferentes a los de las
sulfonilureas

Efe ctos adve rsos

Hipoglucemia
(severa y duradera)

Hipoglucemia (escasa)

Contraindicado

Uso

Embarazo.
Hepatopata.
Nefropata.

DM2 no obesos

M exico A rgentina
C hile U ruguay

Embarazo.
Hepatopata.
IR C severa.

De eleccin en DM con
hiperglucemia postprandial

Biguanidas

Metformina

Inhibidore s de la
alfaglucosidasa

Acarbosa.
Miglitol.

Tiazolidine dionas

Pioglitazona.
R osiglitazona.

Disminuye la insulinorresistencia central (hgado)

Inhiben las alfaglucosidasas

Disminuye la insulinorresistencia perifrica


(msculo, grasa) actuando
en receptores PPAR -gamma

Gastrointestinal
(es lo ms frecuente).
Acidosis lctica
(es lo ms grave).

Gastrointestinal

Hepatotoxicidad.
R etencin hdrica.

Favorecen acidosis lctica


Alcohol.
Enfermedad aguda Qx.
Embarazadas.
Insuficiencia cardaca.
N i o s .
Insuficiencia respiratoria.
Hepatopata.
Nefropata.
De eleccin en
DM-2 obeso

1. Monoterapia
(poco eficaz).
2. Asociar a otros ADO.

Hepatopata

Asociar a Metformina

CTO Medicina C/ Nez de Balboa, 115 28006 MADRID (Espaa) Tfno.: (91) 782 43 32 / Fax: (91) 782 43 27
E-mail: secretaria@ctomedicina.com; iberocto@ctomedicina.com WEB: www.ctomedicina.com; www.iberocto.com

ED Pg. 11

ENDOCRINOLOGA

Seguimiento a distancia

Preparacin Examen de Seleccin 05/06 1 Vuelta

Pregunta 87.- R: 5
El tratamiento intensivo con insulina ha demostrado ser claramente
superior al convencional para la prevencin de las complicaciones
crnicas de la DM. El lmite al tratamiento intensivo es el problema de
las hipoglucemias. Por ello no se recomienda el tratamiento intensivo
en los casos en los que las hipoglucemias son menos deseables por sus
graves consecuencias que un control menos perfecto.

Pregunta 87. El tratamiento insulnico intensivo no debe ser


recomendado en los siguientes casos:
Nios menores de 8 aos, por el efecto deletreo que la hipoglucemia puede tener sobre el desarrollo cognitivo.
Diabticos con neuropata autonmica severa, por el riesgo de
sufrir hipoglucemias inadvertidas.
Pacientes con trastornos mentales graves, que no pueden responsabilizarse de un tratamiento intensivo.
Ancianos.
Cardipatas o pacientes con antecedentes de accidentes cerebrovasculares, en los que la hipoglucemia puede tener consecuencias serias.

HIPOGLUCEMIAS.
Pregunta 88.- R: 1
Las hipoglucemias se dividen en su estudio en las de ayunas y las
reactivas o postprandiales. Las primeras pueden ser a su vez hiper o
hipoinsulinmicas. Esta pregunta hace referencia a las hipoglucemias
de ayuno hiperinsulinmicas, en las que el cociente insulina/glucemia
est elevado. Para su diagnstico diferencial es til la determinacin
de pptido C (indica exceso de insulina endgena) y de proinsulina
(elevada en el insulinoma).

Pregunta 88. Diagnstico diferencial de las


hipoglucemias de ayuno hiperinsulinmicas.
PEPTIDO C

PROINSULINA

SU EN ORINA

Insulina
ex gena

Sulfonilureas

Normal

Insulinoma

Pregunta 89.- R: 4
La causa ms frecuente de hipoglucemia en pacientes ingresados
son los frmacos, la mayora insulina y ADOs (sulfonilureas). Otros
frmacos posibles son: pentamidina, aspirina, betabloqueantes, etc.
Algunos tumores (mesenquimales, retroperitoneales) pueden producir hipoglucemias de ayuno hipoinsulinmicas por produccin ectpica de sustancias como la IGF-2. En otras ocasiones, las hipoglucemias de ayuno hipoinsulinmicas son por deplecin de glucgeno,
como en las hepatopatas severas, desnutricin, sepsis, enfermedades
del metabolismo (glucogenosis, etc).
Pregunta 90.- R: 5
El insulinoma es, tras el gastrinoma, el tumor neuroendocrino
gastroenteropancretico ms frecuente. La mayora son solitarios y
benignos y se curan con ciruga. En los insulinomas localizados en
cabeza o cuerpo de pncreas, el procedimiento empleado es la enucleacin, mientras que los de la cola de pncreas se resecan con
pancreatectoma caudal. Para el control de las hipoglucemias antes
de la ciruga o en caso de recidiva o malignidad se emplea diazxido

Pg. 12 ED

M exico A rgentina
C hile U ruguay

o anlogos de la somatostatina (octretide) en algunos pacientes. En


caso de insulinoma maligno, la quimioterapia con estreptozocina y
doxorrubicina parece la ms eficaz. En casos de metstasis hepticas
exclusivamente es posible la quimioembolizacin de las mismas.
DISLIPEMIA, OBESIDAD Y NUTRICIN.
Pregunta 91.- R: 4
Los objetivos de tratamiento en una dislipemia dependen de los
factores de riesgo cardiovascular que tenga el paciente, y la prioridad
son los niveles de colesterol LDL. Los factores de riesgo cardiovascular
principales son: edad > 45 aos en varones, climaterio en mujeres,
HTA, tabaco, diabetes mellitus, enfermedad cardiovascular precoz en
familiares de primer grado. El objetivo de LDL es: 1) menor de 100 en
cardiopata isqumica, diabetes y otras formas de aterosclerosis severa, 2) menor de 130 con 2 o ms factores de riesgo cardiovascular y 3)
menor de 160 con 1 o ningn factor de riesgo cardiovascular.
Pregunta 92.- R: 3
La dislipemia diabtica se caracteriza por HDL disminuida con
LDL normal o levemente elevada con predominio de las formas pequeas y densas (ms aterognicas) y triglicridos elevados, sobre todo
en caso de mal control glucmico. En el tratamiento, la prioridad es la
correccin del nivel de LDL (menor de 100). La cerivastatina ha sido
retirada debido a la alta incidencia de rabdomilisis en combinacin
con gemfibrocilo.
Pregunta 93.- R: 5
Las estatinas tienen como mecanismo de accin la inhibicin de
la HMG CoA reductasa y disminuyen las concentraciones de LDL.
Los fibratos actan sobre los receptores PPAR alfanucleares y aumentan la oxidacin de cidos grasos en el hgado y msculo, por lo
que disminuyen la secrecin heptica de lipoprotenas ricas en TGs.
Aumentan la actividad de la lipoproteinlipasa (LPL) muscular, favoreciendo el metabolismo de los triglicridos. Son los frmacos
hipotrigliceridemiantes ms eficaces. Las resinas disminuyen la absorcin de colesterol y reducen la LDL, pero aumentan los niveles
de TGs. El cido nicotnico inhibe la movilizacin de cidos grasos
de los tejidos. Tambin aumenta la HDL de forma importante.

Pregunta 93. Hipolipemiantes.


Estatinas

Principios
activos
Efe cto
principal

Fibratos

Atorvastatina.
Lovastatina.
Fenofibrato.
Simvastatina.
Gemfibrocilo.
Pravastatina.
Fluvastatina.
Colesterol

Triglicridos

sntesis de
lipoproteincolesterol
lipasa (aumenta
Mecanismo
(por inhibicin hidrlisis de
de accin
de la HMG
TGs) sntesis
CoA reductasa) VLDL
Efe ctos
adve rsos

transaminasas
Miopata
Cataratas

Gastrointestinal
L it ia s is b il ia r
Miopata
metabolismo
de vitamina K

Resinas
fijadoras de
cidos biliares

cido
nicotnico

Colestiramina.
Colestipol.

Niacina

Colesterol

Triglicridos

Circulacin
enteroheptica
sntesis de
de ac. biliares:
VLDL y LDL
la absorcin de
colesterol
Gastrointestinal
(nuseas,
vmitos,
estreimiento,
esteatorrea)

Sofocos
Prurito
Got a
Diabetes
mellitus

Pregunta 94.- R: 4
Los efectos adversos ms frecuentes son las molestias digestivas, los
dolores musculares y la hepatitis (sta en < 1%). La miopata, la
neuropata y el exantema son raros. La incidencia de miopata au-

CTO Medicina C/ Nez de Balboa, 115 28006 MADRID (Espaa) Tfno.: (91) 782 43 32 / Fax: (91) 782 43 27
E-mail: secretaria@ctomedicina.com; iberocto@ctomedicina.com WEB: www.ctomedicina.com; www.iberocto.com

Comentarios TEST

tejido subcutneo tras su inyeccin, y por esta razn, se absorbe ms


rpidamente. Tiene las ventajas de producir menos hipoglucemias que la
insulina rpida y se inyecta en el preciso momento de empezar a comer.

ENDOCRINOLOGA

Preparacin Examen de Seleccin 05/06 1 Vuelta


menta de forma importante al asociarse con fibratos. Las estatinas
interactan con muchos frmacos que son metabolizados por el cit P450 (excepto la pravastatina), lo que debe tenerse en cuenta. No se
pueden usar en nios ni embarazo.
Pregunta 95.- R: 5
La desnutricin se clasifica en 3 tipos en los que se alteran determinados parmetros: 1) Calrica o marasmo. Alteracin de variables
antropomtricas y no de protenas. 2) Proteica o Kwashiorkor. Alteracin de protenas viscerales, con variables antropomtricas en lmites
normales. 3) Mixta, la ms frecuente en el hospital en la que se alteran
ambos parmetros. A veces la valoracin nutricional est limitada por
la enfermedad del paciente (el peso no sirve en un asctico, la albmina en un politrauma, etc).

Seguimiento a distancia

nutricionalmente y aceptada por el paciente. El objetivo es la prdida


progresiva, moderada y mantenida del peso de un 10%.
Pregunta 100.- R: 4
Las indicaciones de la ciruga de la obesidad son: obesidad mrbida que ha fracasado en tratamientos no quirrgicos, sobrepeso del
200% mantenido ms de 5 aos, ausencia de trastornos psiquitricos,
riesgo quirrgico y anestsico aceptable. Tambin ante la presencia
de complicaciones graves de la obesidad mrbida en la de grado 2.

Pregunta 95. Valoracin de la desnutricin.


Calrica o marasmo

Proteica o Kwashiorkor

Mixta

Alteraciones
antropomtricas

Alteraciones de proteinas
plasmticas

Ambas

1. Peso:
- % disminucin del
peso previo.
- % disminucin del
peso ideal.
- Velocidad de
prdida de peso.
- IMC <18,5 (severa
si <16).
2. Pliegues cutneos
(tricipital, etc).
3. Circunferencia
media braquial.

1. Compartimento
muscular.
- B alance nitrogenado.
- ndice creatinina-altura.
2. Compartimento visceral.
a. Proteinas vida media
intermedia.
- Albmina (20 das).
- Transferrina (10 d).
b. Proteinas vida media
corta.
- Prealbmina (2 das).
- Proteina ligadora de
retinol (10 h).

1. Combinacin de
las anteriores.
2. Disminucin de
los linfocitos
totales*.
3. Anergia cutnea*.

Comentarios TEST

* Se alteran en cualquier tipo de desnutricin e indican que es de grado severo.


Si linfos <800 y anergia cutnea.

Pregunta 96.- R: 4
Las indicaciones de la nutricin enteral son: alimentacin oral
menor del 50% en ms de 7 das con tracto GI funcionante, nutricin
precoz en estrs grave con tracto GI funcionante, intestino corto
funcionante de 100-200 cms, disfagia alta severa, fstula enterocutnea de bajo dbito. Contraindicada en obstruccin intestinal, reposo
intestinal absoluto, tracto GI no funcionante.
Pregunta 97.- R: 2
Las complicaciones de la nutricin parenteral son las alteraciones
hidroelectrolticas que obligan a la realizacin de monitorizacin
analtica cada 24-48h, la posibilidad de inducir hiperglucemia o esteatosis heptica por el contenido de glucosa de las soluciones, la
sobrealimentacin con alta mortalidad, si comenzamos con exceso
de caloras, y sobre todo el riesgo de infeccin de la va central, que es
lo que suele limitar su uso en la mayora de los casos. Los vmitos slo
aparecen cuando se dan alteraciones hidroelectrolticas graves.
Pregunta 98.- R: 5
La clasificacin actual segn el IMC deja el lmite inferior del peso
normal rebajado de 20 a 18,5 kg/m2, de acuerdo con las recientes
recomendaciones internacionales. La amplia gama del sobrepeso (IMC
= 25-29,9 kg/m2) en el que est incluida una gran parte de la poblacin adulta, se divide en 2 categoras (grado 1 de 25 a 26,9 y grado 2
de 27 a 29,9). Se define obesidad a partir de IMC de 30 en cuatro
categoras: 1) obesidad grado 1 entre 30-34,9, 2) obesidad grado 2
entre 35-39,9, 3) obesidad mrbida o severa entre 40-49,9 y 4) obesidad extrema si ms de 50.
Pregunta 99.- R: 3
La dieta es el primer escaln de tratamiento de la obesidad, junto
con el ejercicio fsico moderado. Debe ser hipocalrica, equilibrada
M exico A rgentina
C hile U ruguay

CTO Medicina C/ Nez de Balboa, 115 28006 MADRID (Espaa) Tfno.: (91) 782 43 32 / Fax: (91) 782 43 27
E-mail: secretaria@ctomedicina.com; iberocto@ctomedicina.com WEB: www.ctomedicina.com; www.iberocto.com

ED Pg. 13

ESTADSTICA

Preparacin Examen de Seleccin 05/06 1 Vuelta


ESTADSTICA.
1.

Seale cul de las siguientes es una variable cuantitativa


discreta:
1)
2)
3)
4)
5)

2.

Preguntas TEST

8.

9.

Recorrido.
Media aritmtica.
Desviacin tpica.
Mediana.
Coeficiente de variacin.

Media aritmtica.
Moda.
Percentil 50.
Rango.
Mediana.

2)
3)
4)
5)

Se emplea para comparar la variabilidad relativa de diferentes distribuciones.


Se aplica partiendo del problema de que las desviaciones
tpicas no son comparables directamente.
Puede decirse que los valores no son homogneos si la
desviacin tpica supone ms de un tercio de la media.
El coeficiente de variacin permite comparar la variabilidad relativa de dos muestras.
No puede utilizarse para comparar dos muestras expresadas en distintas unidades.

Todas las siguientes afirmaciones con respecto a la distribucin normal son ciertas, EXCEPTO:
M exico A rgentina
C hile U ruguay

20%.
25%.
30%.
40%.
45%.

La inferencia estadstica es:


1)
2)
3)
4)
5)

12.

2%.
4%.
10%.
40%.
52%.

En un colegio de 1.000 alumnos, 200 nios padecen asma y


100 padecen dermatitis atpica. De los pacientes asmticos,
150 no padecen dermatitis. Cul es la probabilidad de que,
escogido un individuo al azar, sea asmtico o atpico?:
1)
2)
3)
4)
5)

11.

15%.
80%.
40%.
20%.
25%.

Sabemos que la enfermedad X causa la muerte al 20% de los


afectados en el primer ao. Si tenemos dos pacientes con esa
enfermedad, cul es la probabilidad de que mueran ambos
en el primer ao?:
1)
2)
3)
4)
5)

10.

La media es igual a la moda, y sta, igual a la mediana.


El 50% de las observaciones son mayores que la moda.
Aproximadamente el 68% de las observaciones caen
dentro de 1 desviacin estndar de la media.
El nmero de observaciones entre 0 y 1 desviacin estndar de la media es el mismo que entre 1 y 2 desviaciones
estndar de la media.
El valor que ms se repite es la media.

Se sabe que el 50% de la poblacin fuma, y que el 10% fuma


y es hipertensa. Cul es la probabilidad de que un fumador
sea hipertenso?:
1)
2)
3)
4)
5)

Respecto al coeficiente de variacin, una de las siguientes


afirmaciones es FALSA:
1)

7.

5)

Cul de los siguientes parmetros es de dispersin?:


1)
2)
3)
4)
5)

6.

4)

3,4; 2; 3.
3; 3,4; 2.
3; 3; 2.
2; 3,5; 3.
3,4; 3; 2.

En la siguiente distribucin: 16, 12, 13, 14, 50, qu medida


de centralizacin elegira?:
1)
2)
3)
4)
5)

5.

Cualitativa dicotmica.
Cuantitativa discreta.
Cualitativa ordinal.
Cuantitativa ordinal.
Cuantitativa continua.

En nueve familias estudiadas, el nmero de hijos por familia


era de 4, 6, 2, 2, 4, 3, 2, 1, 7. La media, la mediana y moda de
hijos por familia es:
1)
2)
3)
4)
5)

4.

Glucemia basal.
Sexo.
Nmero de camas.
Color de pelo.
Respuesta a tratamiento analgsico.

Qu tipo de variable es la graduacin del coma de la escala


de Glasgow?:
1)
2)
3)
4)
5)

3.

1)
2)
3)

Seguimiento a distancia

La descripcin de los resultados obtenidos en la muestra.


La descripcin de los resultados obtenidos en la poblacin.
La generalizacin de los resultados obtenidos en la muestra.
La generalizacin de los resultados obtenidos en la poblacin.
La estimacin de la media muestral a partir de la media
poblacional.

Se ha estudiado una muestra de 100 personas, obtenindose


una glucemia basal media de 80 mg/ml, con una desviacin
estndar de 10 mg/ml. La glucemia basal media de la poblacin ser:
1)
2)
3)
4)
5)

80 2 mg/ml.
80 20 mg/ml.
80 2 mg/ml, p <0,05.
80 2 mg/ml, p <0,01.
80 3 mg/ml, p <0,01.

CTO Medicina C/ Nez de Balboa, 115 28006 MADRID (Espaa) Tfno.: (91) 782 43 32 / Fax: (91) 782 43 27
E-mail: secretaria@ctomedicina.com; iberocto@ctomedicina.com WEB: www.ctomedicina.com; www.iberocto.com

ET Pg. 1

13.

Leemos en un artculo publicado que la frecuencia cardaca


media de los varones fumadores es de 70 5 lpm, con p
<0,05. Esto significa:
1)
2)
3)
4)
5)

14.

2)
3)
4)
5)

2)
3)
4)
5)
18.

19.

2)
3)
4)

El 95% de los sujetos disminuyeron entre 1,5 y 2,5 mmol/


l.
El 95% de los sujetos disminuyeron entre 1 y 3 mmol/l.
Se tiene un 95% de confianza de que el verdadero
descenso se site entre 1,5 y 2,5 mmol/l.
Se tiene un 95% de confianza de que el verdadero
descenso se site entre 1 y 3 mmol/l.
El 95 % de los pacientes con hipercolesterolemia conseguirn descensos entre 1 y 3 mmol/l a ese grado de
significacin.

5)
20.

2)
3)
4)
5)

No es posible estudiando la misma variable, pero s


podemos reducir a la mitad el intervalo de confianza con
un tamao muestral ms pequeo.
Reduciendo el tamao muestral a la mitad.
Seleccionando muestras con una desviacin tpica 2
veces superior.
Aumentando el tamao muestral 4 veces.
Reduciendo la probabilidad de error tipo II a la mitad.

Se ha obtenido que en el intervalo (175-225), est incluido


el 68% central de una muestra de diabticos. Si el tamao
muestral es 100 y la media 200, calcule un intervalo en el que
se incluya el verdadero valor medio poblacional con una
probabilidad del 95%:
1)

Pg. 2 ET

197,5-202,5.
M exico A rgentina
C hile U ruguay

No se rechaza la hiptesis alternativa, siendo cierta.


Se rechaza la hiptesis alternativa, siendo cierta.
No se rechaza la hiptesis nula, siendo cierta.
Se rechaza la hiptesis nula, siendo cierta.
Se clasifica a un enfermo como tal.

En un contraste de hiptesis, la probabilidad de aceptar la


hiptesis nula, siendo falsa es:
1)
2)
3)
4)
5)

23.

7 2.
7 1.
7 0,16.
7 0,32.
7.

El error alfa o tipo I es:


1)
2)
3)
4)
5)

22.

La probabilidad de que el colesterol medio obtenido en


la muestra no sea 260 es menor de 0,05.
El 95% de los sujetos muestreados tiene una colesterolemia entre 240 y 280 mg/dl.
La probabilidad de que el colesterol medio de los pacientes obesos mayores de 45 aos no est entre 250 y 270 es
menor del 5%.
La probabilidad de que el verdadero valor medio poblacional est entre 240 y 280 es del 95%.
El 95% de los sujetos muestreados tiene un colesterol entre
250 y 270.

En una muestra de 37 pacientes se realiza determinacin de


la uricemia, obtenindose una media de 7 y una desviacin
estndar de 1. La estimacin de la media poblacional (p
<0,05) ser:
1)
2)
3)
4)
5)

21.

60 - 70.
55 - 75.
64 - 66.
63 - 67.
62 - 68.

En una revista mdica se publica un artculo en el que se afirma


que el colesterol medio de los pacientes obesos mayores de 45
aos es 260 10 mg/dl, p<0,05. Esto significa que:
1)

204.
220.
240.
202.
230.

195-205.
150-250.
199,5-200,5.
175-225.

En una muestra de 100 mujeres, la media de peso fue de 65


kg con una desviacin tpica de 5 kg. Cul es el intervalo de
confianza del 95 % para la media poblacional?:
1)
2)
3)
4)
5)

El error estndar de la media se puede reducir a la mitad:


1)

17.

196
180
160
198
170

En una muestra de 100 pacientes se ha observado que un


tratamiento produce una disminucin media de la colesterolemia de 2 mmol/l, con un error estndar de la media de 0,5
mmol/l. Cul de las siguientes afirmaciones es correcta?:
1)

16.

La probabilidad de que la frecuencia cardaca media


obtenida en la muestra no sea 70 lpm es menor de 0,01.
El 95% de los sujetos muestreados est entre 60 y 80 lpm.
La probabilidad de que la frecuencia cardaca media de
los varones fumadores est entre 65 y 75 lpm es menor de
0,05.
La probabilidad de que la frecuencia cardaca media de
los varones fumadores no est entre 65 y 75 lpm es menor
de 0,05.
Se tiene un 95% de confianza de que el verdadero valor
poblacional est entre 60 y 80 lpm.

En una muestra de 100 nios se encuentra una colesterolemia media de 200 mg/dl. Usted ha ledo que en la poblacin
peditrica la colesterolemia media es 196 - 204, p<0,05.
Seale qu intervalo incluye al 95% de los nios de su
muestra:
1)
2)
3)
4)
5)

15.

ESTADSTICA

Preparacin Examen de Seleccin 05/06 1 Vuelta

Es la decisin correcta.
Es el error tipo II o beta.
Es la potencia del test.
Es el error tipo I o alfa.
Es nula siempre.

La capacidad de un test de encontrar diferencias, cuando en


realidad las hay:
1)

Es imprescindible.

CTO Medicina C/ Nez de Balboa, 115 28006 MADRID (Espaa) Tfno.: (91) 782 43 32 / Fax: (91) 782 43 27
E-mail: secretaria@ctomedicina.com; iberocto@ctomedicina.com WEB: www.ctomedicina.com; www.iberocto.com

Preguntas TEST

Seguimiento a distancia

ESTADSTICA

Preparacin Examen de Seleccin 05/06 1 Vuelta


2)
3)
4)
5)
24.

Al comparar la diferencia entre dos medias, se encuentra un


valor de la p=0,6. La interpretacin correcta del resultado es:
1)
2)
3)
4)
5)

25.

3)
4)
5)
26.

2)

Preguntas TEST

3)
4)
5)
27.

4)
5)

El nuevo procedimiento es eficaz para disminuir las complicaciones postoperatorias.


El nuevo procedimiento es ineficaz para disminuir las
complicaciones postoperatorias.
La muestra est sesgada.
El resultado carece de relevancia clnica.
No existe la suficiente evidencia para decir que ambos
procedimientos son diferentes.

31.

2)
3)
4)
5)

El nuevo tratamiento es, sin ninguna duda, mejor que el


antiguo.
Si p <0,05, el resultado no es significativo.
El nuevo tratamiento es un 95% ms eficaz que el usual.
Si los dos tratamientos fueran iguales, resultados como los
obtenidos o diferencias an mayores ocurriran con una
probabilidad menor del 5%.
No existe la suficiente evidencia como para decir que
ambos tratamientos son diferentes.

2)
3)
4)
5)

Indique cul es el valor mximo admitido de error alfa para


establecer diferencias mediante contraste de hiptesis en
trabajos cientficos:
1)

5%.
M exico A rgentina
C hile U ruguay

En caso de no existir diferencias entre expuestos y no


expuestos al factor de riesgo, la probabilidad de observar
por azar los resultados obtenidos es menor del 5%.
Concluimos que existe asociacin real, ya que se ha
demostrado asociacin estadsticamente significativa.
Hasta el 95% de los expuestos al factor de riesgo desarrollar la enfermedad.
La asociacin no es estadsticamente significativa.
Se confirma relacin de causalidad entre el factor de riesgo
y la enfermedad estudiada.

Una de las siguientes afirmaciones sobre el contraste de


hiptesis es FALSA:
1)
2)
3)
4)
5)

33.

90%.
95%.
97%.
98%.
99%.

Un estudio epidemiolgico longitudinal concluye que existe


una asociacin estadsticamente significativa entre un factor
de riesgo y determinada enfermedad con un nivel de significacin p <0,05. Cul es la conclusin correcta?:
1)

32.

No puede rechazarse la hiptesis nula.


La probabilidad de que el resultado sea explicable por el
azar es mayor del 5%.
Podra haberse conseguido un resultado estadsticamente
significativo con un tamao muestral mayor.
No hay diferencias.
Las diferencias no exceden lo atribuible al azar.

Se quiere comparar un nuevo tratamiento con otro convencional, para lo que se realiza un contraste de hiptesis, fijando un
error alfa del 1% y un error beta del 3%. Cul es la probabilidad
de que, existiendo las diferencias, stas se detecten?:
1)
2)
3)
4)
5)

Al comparar dos frmacos hipoglucemiantes, observamos


que el nuevo es significativamente mejor que el usual (p
<0,05). Indique la correcta:
1)

28.

3)

30.

1%.
95%.
68%.
99%.

El resultado de un contraste de hiptesis no es estadsticamente significativo. Indique la FALSA:


1)
2)

Con azatioprina mejorarn el 60% de tus pacientes.


La azatioprina es en un 95% mejor que la 6-mercaptopurina.
El nivel de significacin es del 1%.
Hay diferencias entre los dos tratamientos, a ese nivel de
significacin.
No se puede concluir nada.

Se efectu un estudio para valorar un nuevo procedimiento


quirrgico planeado para disminuir la frecuencia de complicaciones postoperatorias. La frecuencia de complicaciones
fue del 40% en 25 pacientes sometidos al nuevo procedimiento, y del 60% en 20 pacientes en quienes se practic el
procedimiento antiguo. La diferencia NO es estadsticamente significativa. Cabe deducir:
1)

29.

Se rechaza la hiptesis nula.


La diferencia es estadsticamente significativa.
La diferencia no se debe al azar.
La diferencia es compatible con la hiptesis nula.
La variacin en el muestreo no es una explicacin probable de la diferencia.

Al tratar una artritis psorisica con azatioprina, mejora el


60% de los pacientes; la 6-mercaptopurina mejora a un 55%.
La diferencia es significativa, p<0,05, lo cual se debe interpretar como que:
1)
2)

2)
3)
4)
5)

Es el error tipo I.
Es el error tipo II.
Es el poder o potencia del test.
Es generalmente de 0,05.

Seguimiento a distancia

El valor p es el nivel de significacin estadstica especificado para el estudio.


El valor p corresponde a la probabilidad alfa de cometer
un error tipo I.
La capacidad del test de detectar diferencias cuando estas
existen aumenta con el aumento del tamao muestral.
Un error tipo I consiste en aceptar la hiptesis nula cuando
es falsa.
Los valores de p elegidos habitualmente son 0,05 y 0,01.

Si en un contraste de hiptesis obtenemos una diferencia


estadsticamente significativa con un nivel de significacin p
<0,05, indique el enunciado FALSO:
1)

Demostramos la falsedad de la hiptesis nula, ya que p


<0,05 indica que existe una diferencia estadsticamente
significativa.

CTO Medicina C/ Nez de Balboa, 115 28006 MADRID (Espaa) Tfno.: (91) 782 43 32 / Fax: (91) 782 43 27
E-mail: secretaria@ctomedicina.com; iberocto@ctomedicina.com WEB: www.ctomedicina.com; www.iberocto.com

ET Pg. 3

2)

3)
4)
5)
34.

4)
5)
35.

3)
4)
5)

Fumar durante la gestacin retarda el crecimiento fetal.


Es muy probable que las diferencias en el peso se deban
al azar.
El nmero de individuos estudiado fue insuficiente para
llegar a un resultado concluyente.
Si no existieran diferencias entre los dos grupos, la probabilidad de encontrar nuestros resultados es menor del 5%.
No hay diferencias entre ambos grupos.

5)
38.

39.

3)
4)
5)

Si aumenta el tamao muestral, disminuye el error tipo I.


Cuando estamos ante un error beta del 3%, la potencia del
test es alta.
La potencia del test es alta si son grandes las diferencias
reales.
Si las diferencias entre dos tratamientos son muy grandes,
es preciso un tamao muestral grande.
Un error tipo I del 5% no equivale a una potencia del 95%.

La prueba estadstica adecuada para comparar las medias de


tensin arterial entre un grupo de fumadores, uno de exfumadores, y otro de personas que nunca han fumado es:
1)
2)
3)
4)

Pg. 4 ET

t de Student para datos independientes.


t de Student para datos apareados.
Anlisis de la varianza.
Chi-cuadrado.
M exico A rgentina
C hile U ruguay

Coeficiente de correlacin.
Anlisis de Regresin.
Test exacto de Fisher.
t de Student para muestras independientes.
Test de Friedman.

Un coeficiente de correlacin de Pearson de -1 indica:


1)
2)
3)
4)
5)

43.

Chi-cuadrado.
Regresin lineal.
Test exacto de Fisher.
t de Student.
Test de Wilcoxon.

Se pretende establecer una ecuacin, mediante la cual pueda


predecirse el tiempo de reproduccin, en base al conocimiento del fotoperodo (nmero de horas de luz/da). Qu
tipo de test utilizaremos?:
1)
2)
3)
4)
5)

42.

t de Student.
ANOVA.
Regresin lineal.
Chi-cuadrado.
Coeficiente de correlacin de Spearman.

Para comparar la tensin arterial diastlica en dos muestras


de mujeres embarazadas y no embarazadas, el test ms
apropiado es:
1)
2)
3)
4)
5)

41.

t de Student.
Test de McNemar.
ANOVA.
Chi-cuadrado.
Test de Friedman.

Se comparan tres tratamientos en cuanto a eficacia en la


reduccin del nmero de lesiones de acn en pacientes
adolescentes de 14 a 16 aos. Qu tipo de test estadstico
aplicaremos?:
1)
2)
3)
4)
5)

40.

Coeficiente de correlacin de Pearson.

Qu test estadstico debemos utilizar para comparar la


presencia cualitativa del antgeno de superficie de la hepatitis
B en estudiantes de Medicina y Odontologa?:
1)
2)
3)
4)
5)

Seale la FALSA:
1)
2)

37.

No hay diferencia de eficacia entre aspirina y ticlopidina.


La ticlopidina es un 5% ms eficaz que la aspirina.
La probabilidad de que las diferencias observadas se
deban al azar es menor del 5%.
Es posible que aumentando el tamao de la muestra
obtengamos diferencias significativas para el mismo nivel
de significacin.
Las diferencias entre ticlopidina y aspirina exceden lo
atribuible al azar.

Se ha comparado el peso de dos grupos de recin nacidos,


uno formado por hijos de no fumadoras y otro formado por
hijos de fumadoras, encontrndose un peso menor en el
segundo grupo (p <0,05). Esto significa que:
1)
2)

36.

Si la hiptesis nula es verdadera, diferencias iguales o


mayores que las observadas pueden ocurrir con una
probabilidad demasiado pequea como para que sea
atribuible exclusivamente al azar.
Un tamao muestral menor puede hacernos encontrar un
nivel de significacin p >0,05 tras el anlisis de los
resultados.
La asociacin estadstica significativa no implica asociacin causal.
La significacin estadstica se refiere slo a la existencia de
una diferencia, no a su magnitud.

Se someten a un estudio comparativo la ticlopidina y la


aspirina en cuanto a capacidad para disminuir el riesgo de
sufrir ACVAs; la ticlopidina parece mostrarse ms eficaz que
la aspirina. Sin embargo, la diferencia no resulta estadsticamente significativa en el contraste de hiptesis para p <0,05.
Usted afirma que:
1)
2)
3)

ESTADSTICA

Preparacin Examen de Seleccin 05/06 1 Vuelta

No hay correlacin.
Existe una dbil correlacin.
Existe una fuerte correlacin.
Al aumentar los valores de la variable X, aumentan los
valores de la variable Y.
Al disminuir los valores de la variable X, disminuyen los
valores de la variable Y.

Son datos apareados:


1)
2)
3)

Tensin arterial en mujeres, frente a tensin arterial en


hombres.
Tiroxina en una muestra pequea, frente a tiroxina en una
muestra grande.
Aclaramiento de creatinina en una muestra, frente a
aclaramiento de creatinina en una poblacin.

CTO Medicina C/ Nez de Balboa, 115 28006 MADRID (Espaa) Tfno.: (91) 782 43 32 / Fax: (91) 782 43 27
E-mail: secretaria@ctomedicina.com; iberocto@ctomedicina.com WEB: www.ctomedicina.com; www.iberocto.com

Preguntas TEST

Seguimiento a distancia

ESTADSTICA

Preparacin Examen de Seleccin 05/06 1 Vuelta


4)
5)
44.

Para comparar el descenso medio en la carga viral del VIH que


se observa al emplear indinavir y saquinavir, se utiliza:
1)
2)
3)
4)
5)

45.

5)

Preguntas TEST

Distribucin t de Student.
Anlisis de regresin.
Correccin de Yates.
Test de Mann-Whitney.
Prueba de Spearman.

Son menos potentes que las paramtricas.


Son tiles en muestras pequeas.
Son tiles en el caso de variables ordinales.
Requieren ciertas asunciones sobre la distribucin de las
variables en la poblacin.
Pueden utilizarse para comparar variables de tipo cuantitativo.

ANOVA.
Test de Spearman.
t de Student.
Chi-cuadrado.
Test de Wilcoxon.

t de Student para datos apareados.


t de Student para datos independientes.
Correlacin de Pearson.
Test de la U de Mann-Whitney.
Test de Wilcoxon.

Qu test estadstico utilizara para comparar la relacin


entre la tensin arterial y la colesterolemia?:
1)
2)
3)
4)
5)

49.

1)
2)
3)
4)
5)

Una muestra de 25 pacientes nefrpatas se someten a determinacin cuantitativa de ADN viral (VHB), antes y despus de
ser sometidos a hemodilisis. Qu test empleara para comparar los resultados obtenidos?:
1)
2)
3)
4)
5)

48.

Es una prueba de contraste de hiptesis til para establecer


relacin entre una variable cualitativa y otra cuantitativa:

Qu prueba estadstica usara para comparar el resultado


(curacin/no curacin) de tres tratamientos distintos?:
1)
2)
3)
4)
5)

47.

Chi-cuadrado.
t de Student.
Correlacin de Pearson.
Test de Spearman.
ANOVA.

50.

Cul de las siguientes afirmaciones sobre las pruebas estadsticas no paramtricas es FALSA?:
1)
2)
3)
4)

46.

Glucemia antes de comer, frente a glucemia despus de


comer.
Talla en hombres frente a talla en mujeres.

Seguimiento a distancia

Correlacin de Pearson.
ANOVA.
Chi-cuadrado.
t de Student.
Test de Wilcoxon.

Para comparar el resultado de un IECA en la reduccin de la


TAS con el de un betabloqueante, empleamos el test:
1)
2)
3)
4)
5)

U de Mann-Whitney.
Test de Wilcoxon.
Test de Friedman.
Rho de Spearman.
t de Student.
M exico A rgentina
C hile U ruguay

CTO Medicina C/ Nez de Balboa, 115 28006 MADRID (Espaa) Tfno.: (91) 782 43 32 / Fax: (91) 782 43 27
E-mail: secretaria@ctomedicina.com; iberocto@ctomedicina.com WEB: www.ctomedicina.com; www.iberocto.com

ET Pg. 5

ESTADSTICA

Preparacin Examen de Seleccin 05/06 1 Vuelta


Pregunta 1.- R: 3
Los estudios estadsticos pretenden llegar a conocer a la poblacin
a partir de un anlisis de una muestra de individuos.
Lo que se estudia acerca de cada individuo de la muestra es lo que
se llama carcter o variable; es decir, se trata de una caracterstica
observable en los individuos. El color de los ojos, el estado civil, el
nmero de hijos, las pulsaciones por minuto, la talla o la velocidad de
sedimentacin constituyen ejemplos de variables.
Las variables cuantitativas son aquellas que pueden medirse,
cuantificarse o expresarse de forma numrica; las variables que no
pueden cuantificarse numricamente reciben el nombre de variables cualitativas; estas variables, por lo tanto, toman valores no numricos.
Dentro de las opciones, hay dos variables cuantitativas: la glucemia
y el nmero de camas. Cuando entre dos categoras cualquiera de
una variable, podemos interponer otra (ej. entre 80 y 81 mg/dl de
glucemia, tenemos 80,5) la variable es continua. El nmero de camas
por el contrario es discreta.

Pregunta 1. Tipos de variables.


VARIABLE

SUBTIPO

EJEMPLO

Cualitativa

Nominal dicotmica

Sexo

Comentarios TEST

Cuantitativa

Nominal no dicotmica

R aza

Ordinal

Nivel socioeconmico

Discreta

N episodicos de asma/sem

Continua

Uricemia

Pregunta 2.- R: 3
Si recuerdas la exploracin del coma o del traumatismo craneoenceflico, se haca referencia a la escala de Glasgow; en ella, se consideraban tres parmetros: respuesta motora, ocular y verbal. Segn el
paciente, se asignaban unos puntos a cada parmetro, que iban desde 3, que se considera una situacin muy grave a 15, que sera una
situacin de normalidad.
Sin embargo, en dicha escala el nmero asignado NO tiene valor
matemtico (7 no significa "doble" peor pronstico que 14), por lo
que la variable NO se considera cuantitativa sino cualitativa.
Las categoras de esta variable (cualitativa ordinal) tienen una manera lgica de ordenarse (de 3 a 15), a diferencia de las variables
cualitativas nominales (por ej. el color de pelo).
Pregunta 3.- R: 5
El primer escaln dentro de la estadstica es el descriptivo. En esta
fase se realiza la recogida de la informacin, que debe ser posteriormente ordenada, resumida y presentada de una forma comprensible
que permita pasar posteriormente a la segunda fase, que es el anlisis
de los datos.
Dentro del apartado de estadstica descriptiva nos encontramos
con las medidas de centralizacin que se mencionan en esta pregunta: la media, la mediana y la moda.
La media se obtiene sumando todos los valores numricos observados y dividiendo el resultado por el nmero de observaciones.
Es la medida de centralizacin ms empleada. Recuerda que es
muy sensible a las observaciones atpicas. En este caso: 31, que es la
suma total, entre 9, que es el nmero de observaciones, da 3,4.
La mediana slo emplea el valor central de la serie (es decir, el que
deja a cada lado, la misma proporcin de la muestra); sin embargo, se
afecta menos por observaciones atpicas (por ejemplo, errores de
medida). En este caso, es 3.
La moda es el dato que ms se repite en una serie, puede no ser un
valor nico, a diferencia de los otros dos parmetros. En este caso, es 2.
Pregunta 4.- R: 4
La medida de centralizacin ms utilizada es la media aritmtica, si
bien cuando en una muestra hay valores atpicos (como en este caso,
M exico A rgentina
C hile U ruguay

Seguimiento a distancia

el 50), el clculo de la media aritmtica puede resultar sesgado, por lo


que es preferible utilizar la mediana, la cual es estable ante estas mediciones atpicas o errores de medicin. Resumiendo, es preferible el
uso de la mediana como medida de centralizacin en los siguientes
casos:
Mediciones atpicas.
Errores de medicin.
Muestra no homognea.
Distribucin asimtrica.
Pregunta 5.- R: 4
El concepto de dispersin implica "cunto estn de separados los
datos con respecto al dato central (a la media)", por tanto, se puede
calcular como la suma de las diferencias con la media de los datos.
Hay que tomar los valores absolutos, ya que, puesto que hay unos
datos mayores que la media y otros menores, el sumatorio de las
diferencias dara cero si no se emplease esta argucia matemtica. Posteriormente se divide la suma total entre el nmero de individuos de la
muestra, para que sea comparable con otras dispersiones en otras
series con distinto nmero de individuos. Si te fijas, lo que hemos
hecho es obtener la desviacin media de los datos con respecto a su
media aritmtica.
Avanzando un poco ms, podemos, a partir de la desviacin media, obtener un mejor ndice de dispersin: si en vez de trabajar con
valores absolutos, elevamos al cuadrado las diferencias, tambin nos
deshacemos de los nmeros negativos, y pasamos a trabajar con "reas" en vez de segmentos (las diferencias se elevan al cuadrado); hemos
obtenido la varianza, que es una excelente medida de dispersin.
El problema de la varianza es que sus unidades son las mismas que
las de los datos pero elevadas al cuadrado, lo cual es matemticamente incmodo, por ello se obtiene la raz cuadrada positiva del resultado de la varianza para manejarse ms cmodamente; a este valor se le
llama desviacin tpica, y es el mejor ndice de dispersin.
El rango o recorrido, que tienes definido en el enunciado, es la
forma ms elemental. Su inters es muy relativo, ya que utiliza solamente dos datos de la serie (el primero y el ltimo), que son, precisamente, los menos representativos, puesto que son los valores extremos
o ms raros de la variable.

Pregunta 5. Formas de medida.


MEDIDA DE
CENTRALIZACIN

MEDIDA DE
DISPERSIN

DISTRIBUCIN
HOMOGNEA

Media

Desviacin tpica
o estndar

DISTRIBUCIN
ASIMTRICA

Mediana

R ango intercuartlico
R ango

Pregunta 6.- R: 5
El coeficiente de variacin es adimensional, debido a que representa el porcentaje de la desviacin estndar sobre la media.
Su utilidad est en que sirve para comparar la variabilidad de dos o
ms distribuciones, estn expresadas en las mismas o en diferentes
unidades. Por tanto, la opcin 5 es la falsa.
La forma de comparar dos desviaciones tpicas de muestras diferentes se hace mediante el coeficiente de variacin, que considera
cada desviacin en relacin a la muestra de la que procede, es decir,
en relacin (divisin) a la media de la muestra. Mediante la relacin,
se obtiene una medida adimensional que puede compararse con la
de otra muestra.
Despus de hablar de las medidas de dispersin surge la pregunta
"Cundo se puede decir que una distribucin es concentrada o dispersa?, a partir de qu valor de la desviacin tpica?". En general, para
distribuciones biolgicas de datos, sin valores negativos, se pueden
considerar concentradas aquellas cuya desviacin tpica no excede
1/3 del valor de la media. Desviaciones superiores a la mitad de la
media se traducen en dispersiones excesivas.

CTO Medicina C/ Nez de Balboa, 115 28006 MADRID (Espaa) Tfno.: (91) 782 43 32 / Fax: (91) 782 43 27
E-mail: secretaria@ctomedicina.com; iberocto@ctomedicina.com WEB: www.ctomedicina.com; www.iberocto.com

ET Pg. 1

Seguimiento a distancia

ESTADSTICA

Preparacin Examen de Seleccin 05/06 1 Vuelta

Pregunta 7.- R: 4
La distribucin normal sigue una forma de campana en la que el
valor central es la media y tambin la mediana (puesto que la campana es simtrica) y la moda (puesto que en ese punto la curva tiene su
mxima altura).
Entre 1 desviacin estndar se encuentra el 68% de las observaciones y entre 2 desviaciones estndar estn el 95% de las observaciones. Por tanto, entre 1 y 2 desviaciones estndar se encuentran el
27% (95-68), por lo que la opcin 4 es incorrecta.
El resto de opciones resumen una serie de caractersticas que debes conocer sobre la curva de Gauss.

dad de ser ambas cosas a la vez es P(AB)= (200-150)/1.000 = 0,05


(si 150 asmticos de los 200 no son atpicos, significa que 50 s lo
son). Por tanto, la probabilidad de que un individuo escogido al azar
sea asmtico o atpico es:
B) = 0,2 + 0,1 - 0,05 = 25%
P(AUB) = P(A) + P(B) - P(A
Para calcular la probabilidad de A o B, cuando A y B no son
excluyentes, hay que restar la interseccin de A y B (la probabilidad
de que se den los dos sucesos simultneamente), porque de no hacerlo estaramos contando esos casos (en este ejemplo, los nios con
asma y dermatitis atpica) dos veces.
Pregunta 11.- R: 3
La razn de existir de la estadstica es la imposibilidad de poder tener
acceso a toda la poblacin. Basndose en esta limitacin, los cientficos
eligen una muestra pequea de dicha poblacin y realizan todas las
mediciones sobre ella, una vez obtenidas las conclusiones, que son
ciertas para el 100% de la muestra, se deben generalizar o inferir los
resultados a la poblacin; durante este proceso hay que aceptar que la
muestra, por muy representativa que sea, nunca va a ser idntica a la
poblacin y, por tanto, la generalizacin va a tener que asociar una
probabilidad de que en el intervalo de confianza estimado NO se encuentre la verdadera medida del parmetro estimado (el valor que se
obtendra si se calculase con toda la poblacin).

Distribucin normal.

Pregunta 8.- R: 4
El 50% de la poblacin fuma y el 10% de la poblacin total fuma
y es hipertensa, es decir, hay 50 fumadores por cada 100 habitantes,
de los que 10 son hipertensos y 40 no.
La proporcin de hipertensos entre los fumadores es, por tanto, de
1 entre 5, que es el 20%.
Se trata de una probabilidad condicionada, consideramos que
P(AB) es la probabilidad de fumar y ser hipertenso y P(B) la probabilidad de ser fumador. Nos piden la probabilidad de ser hipertenso una
vez que se es fumador (P (A/B)). La frmula es:
B)/P(B)
P (A/B)= P (A
Pregunta 9.- R: 2
Es un caso de probabilidades independientes (el hecho de que un
individuo muera no condiciona la muerte del otro), por tanto, la
probabilidad de que un sujeto A fallezca es 0,2 y la probabilidad de
que el sujeto B fallezca es tambin 0,2 (no se modifica por el hecho de
que el sujeto A muera o no).
La probabilidad de que ambos fallezcan es:

IC 99%= X +
2,6

DS
n-1

Aplicando la frmula:

IC 99%= 80 +
2,6

10
100

IC 99%= 80 +
2,6
Como vemos, el resultado NO coincide con las dos ltimas opciones, por lo que la correcta es la tercera:

P(A
B)= 0,2 x 0,2= 0,04 = 4%

IC 95%= 80 +
2

10
100

De cara al clculo del intervalo de confianza, en el MIR, si la raz


cuadrada del tamao muestral es un nmero entero (por ejemplo,
raz de 100), se suele redondear y no se resta 1.

Pregunta 9.

Reglas de probabilidad.

Pregunta 10.- R: 2
Es un caso de probabilidad con sucesos no excluyentes (el hecho
de tener asma no protege contra la atopia y viceversa).
La probabilidad de ser asmtico es P(A)= 200/1.000= 0,2 y la
probabilidad de ser atpico es P(B)= 100/1.000= 0,1; la probabili-

Pg. 2 ET

M exico A rgentina
C hile U ruguay

Pregunta 13.- R: 4
La clave de esta pregunta es tener claro qu es un intervalo de
confianza y cmo se expresa.
Un intervalo de confianza es una horquilla de dos valores entre los
que hay una probabilidad dada de que se encuentre el valor
poblacional de determinada medida (una media, una prevalencia).
Los intervalos de confianza se pueden dar desarrollados (por ejemplo, 65-75 lpm, p<0,05) o sintticos (por ejemplo, 70 5 lpm,
p<0,05).

CTO Medicina C/ Nez de Balboa, 115 28006 MADRID (Espaa) Tfno.: (91) 782 43 32 / Fax: (91) 782 43 27
E-mail: secretaria@ctomedicina.com; iberocto@ctomedicina.com WEB: www.ctomedicina.com; www.iberocto.com

Comentarios TEST

Pregunta 7.

Pregunta 12.- R: 3
En esta pregunta nos estn pidiendo un intervalo en el que con una
confianza determinada (95% o 99%) est la media poblacional de
glucemia, por lo que las dos primeras opciones NO pueden ser correctas, ya que al estimar un parmetro en la poblacin (en este caso
una media) hay que aportar la probabilidad de que en ese intervalo
no est el valor poblacional (la "p").
Como dos de las opciones tienen una p<0,01, calcularemos en
primer lugar el intervalo de confianza del 99% y, en caso de que NO
coincida con estas dos opciones, marcaremos la opcin 3.

ESTADSTICA

Preparacin Examen de Seleccin 05/06 1 Vuelta


Lo ms importante es darse cuenta de que en el enunciado de esta
pregunta YA nos dan el intervalo de confianza (en el 5 est comprendido 2 x error estndar de la media), por lo que la probabilidad de
que la media de frecuencia cardaca en la poblacin NO est en l es
menor del 5%, que es lo que dice la opcin 4.
Pregunta 14.- R: 3
Es importante darse cuenta de que en esta pregunta, aunque nos
dan algunos datos poblacionales, lo que se pide es un intervalo que
comprenda al 95 % de una muestra. Este intervalo (que NO se llama
de confianza), si la variable sigue una distribucin normal (en caso de
que este dato no lo aporten, como ocurre casi siempre en el MIR, se
asume que es as) se calcula:
X 2 x DS
Del enunciado, extraemos que la media es 200 mg/dl y el dato que
nos falta es la desviacin tpica.
Para determinar la desviacin tpica, es necesario darse cuenta de
que en el clculo del dato poblacional que nos aporta el enunciado
(el intervalo de confianza del 95 %) est la desviacin tpica.

IC 95%= X +
2

DS
n-1

Aplicando la frmula:

IC 95%= 200 +
2

DS
100

(Date cuenta de que generalmente al tamao muestral NO se le


resta uno, ya que la diferencia del resultado es mnima y el clculo de
la raz cuadrada es mucho ms sencillo).
Sustituimos el IC 95 % por los valores que nos dan en el enunciado:

Comentarios TEST

196 - 204 = 200 +


2

DS
100

Si desarrollamos esta ecuacin (eligiendo el lmite superior o el


inferior del IC), tenemos:

Seguimiento a distancia

Ya que nos dan el error estndar de la media, lo ms fcil es calcular primero el IC 95 % y, en caso de que no coincida con las opciones
que hablan de la poblacin (la 3 y la 4), desarrollar despus el intervalo que incluye el 95 % de la muestra y comprobar con cul de las
opciones que se refieren a la muestra (opciones 1 y 2) coincide.
IC 95%= X 2 x EEM
Aplicacin de la frmula:
IC 95%= 2 2 x 0,5
IC 95%= 2 1
Es decir, que se tiene un 95 % de confianza de que el verdadero
descenso medio se site entre 1 y 3 mmol/l, que es lo que dice la
opcin 4).
Pregunta 16.- R: 4
Esta pregunta tiene importancia, ya que de su comprensin deriva
el entender cmo se aumenta la precisin en una estimacin de
parmetros.
En la frmula del Error Estndar de la Media, la desviacin tpica
est en el numerador y el tamao muestral en el denominador. Para
reducir el EEM, no podemos actuar sobre el numerador, ya que la
desviacin tpica es una caracterstica que depende de la variable que
estamos midiendo. Lo que s podemos hacer es aumentar el denominador, es decir el tamao muestral, de modo que al aumentar el
tamao muestral cuatro veces (como est dentro de una raz cuadrada), conseguimos disminuir el EEM a la mitad.
De modo que si un intervalo de confianza es demasiado amplio y
queremos reducirlo (esto es, ganar en precisin), deberemos aumentar el tamao muestral.
Pregunta 17.- R: 2
En esta pregunta, nos piden un intervalo que incluya con una
probabilidad del 95 % la verdadera media poblacional, es decir, el
Intervalo de Confianza del 95 %:

IC 95%= X +
2

DS
n-1

Aplicacin de la frmula:
IC 95%= 200 +
2

DS
100

IC 95%= 200 2 (DS/10)


Para su clculo nos falta la desviacin estndar, que no nos la dan
directamente, pero que la podemos buscar en un dato de la muestra.
Nos dicen que el 68 % de la muestra est comprendido entre 175
y 225, por lo que:

204 - 200 = 2 (DS/10)


4 = 2 DS / 10
40 = 2 DS; DS = 20
Ahora que ya tenemos la desviacin tpica, podemos calcular el
intervalo de la muestra por el que nos preguntaban:
X 2 x DS
Aplicacin de la frmula:
200 2 x 20 = 200 40 = 160 - 240
Pregunta 15.- R: 4
Con los datos que aporta el enunciado (tamao muestral, media y
error estndar de la media), podemos calcular la desviacin tpica,
por lo que, de todas la opciones, puede ser cierta alguna que hable de
la muestra (cuyos intervalos se calculan con la media y la desviacin
tpica) o de la poblacin (cuyos intervalos de confianza se calculan
con la media y el error estndar de la media).
M exico A rgentina
C hile U ruguay

X DS = 175 - 225
200 DS = 175 - 225
Desarrollando esta ecuacin, obtendremos que la DS es 25.
Ahora slo tenemos que retomar la frmula del IC 95%:
IC 95%= 200 2 (DS/10)
IC 95%= 200 2 (25/10)
IC 95%= 200 2 x 2,5
IC 95%= 200 5
IC 95%= 195 - 205
Pregunta 18.- R: 3
En esta pregunta, nos piden el Intervalo de Confianza del 95 % y
nos aportan todos los datos necesarios para su clculo:

CTO Medicina C/ Nez de Balboa, 115 28006 MADRID (Espaa) Tfno.: (91) 782 43 32 / Fax: (91) 782 43 27
E-mail: secretaria@ctomedicina.com; iberocto@ctomedicina.com WEB: www.ctomedicina.com; www.iberocto.com

ET Pg. 3

ESTADSTICA

Seguimiento a distancia

DS
n-1

Aplicacin de la frmula:

IC 95%= 65 +
2

5
100

IC 95%= 65 1
IC 95%= 64 - 66
Pregunta 19.- R: 3
Para contestar a esta pregunta hay que tener claro que 260 10
con una p<0,05 no puede ser otra cosa que el intervalo de confianza
del 95 % (recuerda que en la muestra no hablamos de confianza y no
se aporta probabilidad de equivocarnos, es decir, la "p") y que en el 10
ya est incluido 2 x EEM.
Si nos ha quedado claro lo anterior, la probabilidad de que en ese
intervalo NO est la verdadera media poblacional de colesterol ser
del 5%, que es lo que dice la opcin correcta (la 3).
Si te das cuenta con los datos que aporta el enunciado (NO nos
dan tamao muestral, por lo que NO podemos calcular la desviacin
tpica a partir del intervalo de confianza) NO podemos calcular intervalos de la muestra, por lo que aquellas opciones que hablen de la
muestra (1,2 y 5) sern falsas.
Pregunta 20.- R: 4
En esta pregunta, nos piden el Intervalo de Confianza del 95 % y
nos aportan todos los datos necesarios para su clculo:

IC 95%= X +
2

DS
n-1

Aplicacin de la frmula:

IC 95%= 7 +
2

1
36

(Date cuenta de cmo en esta pregunta s hemos restado uno al


tamao muestral, ya que en este caso facilita el clculo).
IC 95%= 7 2 (1/6)
IC 95%= 7 0,32
Pregunta 21.- R: 4
Cuando se comparan dos muestras o grupos (por ejemplo, dos grupos
de personas sometidos a dos tratamientos distintos), incluso aunque entre
estos dos grupos no hubiera diferencia, al medir en ellos una variable
resultado (por ejemplo: respuesta o no a un analgsico o disminucin de
la TA con una antihipertensivo) es muy poco probable que el promedio
de la variable resultado de ambos grupos sea exactamente el mismo.
Para entender en qu consiste el Contraste de Hiptesis, deberemos partir del anterior razonamiento: "en caso de que ambos grupos
sean iguales, se pueden obtener pequeas diferencias en la variable
resultado que se deben a la variabilidad del muestreo".
En Contraste de Hiptesis, al supuesto de que ambos grupos o
muestras provienen de una poblacin homognea (es decir, de que
son iguales o de que no hay diferencias entre ellos), lo llamamos
hiptesis nula (Ho) y las diferencias que se deben al muestreo son
diferencias que se pueden explicar por el azar.
Por lo tanto, podramos repetir el anterior argumento con otros
trminos: "en caso de que la Ho sea cierta, se pueden obtener pequeas diferencias en la variable resultado que se deben al azar".
Y en qu consiste el Contraste de Hiptesis?. Pues en establecer si las
diferencias que se han obtenido entre los dos grupos son probablePg. 4 ET

M exico A rgentina
C hile U ruguay

mente debidas al azar (a la variabilidad en el muestreo), y por lo tanto


no podremos refutar la hiptesis nula o si, por el contrario, las diferencias obtenidas son demasiado grandes y la probabilidad de que se
deban al azar es pequea, y por tanto rechazaremos la Ho y diremos
que las diferencias son compatibles con la Hiptesis Alternativa (H1),
que es aquella que sostiene que ambos grupos son distintos.
Lo ms importante de todo es darse cuenta de que, cuando rechazamos la Ho, no estamos completamente seguros de que las diferencias encontradas no se deban al azar. Lo que ocurre es que, como la
probabilidad de que se deban al azar es tan pequea, nos inclinamos
a pensar que la que es cierta es la H1.
Y qu parmetro estadstico es el que determina cul es la probabilidad de que la diferencia obtenida se deba al azar? Pues el grado
de significacin (la "p").
Proponemos un ejemplo para la comprensin de uno de los conceptos ms importantes de la Estadstica y del bloque de Estadstica en
el examen MIR:
Para comparar la TAD de los hombres y las mujeres, se recluta a 10
mujeres y 10 hombres sin HTA y se les toma la TAD. Se calcula la
media aritmtica de ambos grupos, y el resultado es:
X de TAD en hombres: 80
X de TAD en mujeres: 63
Segn estos resultados, estaramos tentados a decir que la TAD es
distinta en hombres y en mujeres, pero ANTES de afirmar dicha diferencia, deberemos preguntarnos: estas diferencias encontradas no se
debern a la variabilidad que deriva de trabajar con muestras en vez
de con toda la poblacin de mujeres y hombres?.
Imaginamos que se calcula el grado de significacin y se obtiene
un 3%, es decir, la probabilidad de que estos resultados (la diferencia
obtenida en nuestro estudio) se deba al azar es muy pequea, por lo
que tendremos suficiente evidencia para rechazar la hiptesis nula y
decir que entre los dos grupos hay diferencias.
Pero, es posible que habiendo dicho que la TAD en hombres es
distinta a la de mujeres nos hayamos equivocado?.
Efectivamente, la probabilidad de que hayamos rechazado la Ho y
fuese cierta (lo que se denomina error tipo I), coincide con la probabilidad de que las diferencias encontradas se deban al azar (es decir,
que NO haya diferencias), esto es, el grado de significacin.
Por lo que, recapitulando: siempre que rechazamos la hiptesis
nula, deberemos aportar la probabilidad de que nos equivoquemos,
es decir, de cometer un error tipo I: la "p".
Pregunta 22.- R: 2
En el caso de que la hiptesis nula fuese falsa (esto es, si en realidad
hubiese diferencias) y el grado de significacin obtenido no fuera tan
pequeo como el nivel mnimo exigido en Estadstica (5%), NO podremos rechazar esa hiptesis nula, aunque sea falsa.
A esto, en Contraste de Hiptesis, se le denomina error tipo II.
Es importante insistir en que cuando no se tiene suficiente evidencia para rechazar la Ho, no por ello se dice que sea cierta.

Pregunta 22. Contraste de hiptesis.


REALIDAD
EXISTE
DIFERENCIA
Ho falsa
HAY
1-b
DIFERENCIAS
Poder estadstico
SIGNIFICATIVAS. o potencia del
Rechazo Ho.
test
RESULTADOS
DEL TEST
NO HAY
DIFERENCIAS
SIGNIFICATIVAS.
No rechazo Ho.

NO EXISTE
DIFERENCIA
Ho cierta
Error tipo I o
error alfa

Error tipo II o
error beta

CTO Medicina C/ Nez de Balboa, 115 28006 MADRID (Espaa) Tfno.: (91) 782 43 32 / Fax: (91) 782 43 27
E-mail: secretaria@ctomedicina.com; iberocto@ctomedicina.com WEB: www.ctomedicina.com; www.iberocto.com

1- a

Comentarios TEST

IC 95%= X +
2

Preparacin Examen de Seleccin 05/06 1 Vuelta

ESTADSTICA

Preparacin Examen de Seleccin 05/06 1 Vuelta


Pregunta 23.- R: 4
El poder o potencia estadstica es la capacidad de demostrar las
diferencias en caso de ser ciertas (esto es, rechazar la hiptesis nula
siendo falsa sta y siendo cierta la alternativa).
En el caso de que la hiptesis nula fuese falsa (esto es, si en realidad
hubiese diferencias), nos podemos encontrar con dos situaciones:
No rechazamos la Ho: error tipo II.
Rechazamos la Ho: poder estadstico.
Como puedes ver, el poder estadstico es el complementario del
error tipo II, es decir, si antes de arrancar un estudio, aceptamos un
error tipo II del 10 %, sabremos que la probabilidad de demostrar
diferencias, en caso de haberlas, ser del 90 %.
Pregunta 24.- R: 4
Al comparar dos medias nos encontramos que la probabilidad de
que las diferencias halladas puedan deberse al azar es del 60 %
(p=0,6), es decir, la probabilidad de que esos resultados (esas diferencias) puedan deberse a variaciones en el muestreo es bastante alta, por
lo que posiblemente NO haya verdaderas diferencias, es decir, los
resultados son compatibles con la hiptesis nula.

Comentarios TEST

Pregunta 25.- R: 4
Incluso si la eficacia de la azatioprina y la 6-mercaptopurina fuese
igual (Ho cierta), es rarsimo que el porcentaje de respuesta sea exactamente igual en dos grupos de personas; lo que tenemos que decidir es si
las diferencias observadas (60 % de mejora con azatioprina y 55% con 6mercaptopurina) se pueden deber al azar; la respuesta a esta pregunta es
el grado de significacin, que en este caso es menos del 5 %, por lo que
podremos decir que hay diferencias entre esos dos tratamientos (rechazaremos la Ho) con una probabilidad de equivocarnos (es decir, de que se
deban al azar) menor del 5%, que es lo que dice la opcin 4.
Pregunta 26.- R: 5
Nos encontramos con dos grupos de pacientes que, sometidos a
distintos procedimientos quirrgicos, presentan un porcentaje de complicaciones distinto, que desde el punto de vista clnico es importante.
Lo que hay que establecer es con qu probabilidad estas diferencias pueden ser explicadas por el azar; en el enunciado no nos lo dan
directamente, pero la probabilidad de que el azar explique las diferencias debe de ser mayor del 5 %, ya que nos dicen que la diferencia
no es estadsticamente significativa.
Es importante recalcar que, cuando no se tiene suficiente evidencia para rechazar la Ho, no por ello se dice que sea cierta, por eso lo
nico que podremos decir es que no existe evidencia para decir que
ambos tratamientos son diferentes (opcin 5 correcta), pero NO podremos aseverar que los dos tratamientos son iguales.
Pregunta 27.- R: 4
Pongamos un ejemplo que podra ser el del enunciado de esta pregunta: en 35 personas tratadas con el hipoglucemiante A se consigue
como media una reduccin de la glucemia de 45 mg/dl y en 35 personas tratadas con el hipoglucemiante B se consigue como media una
reduccin de la glucemia de 60 mg/dl. En el contraste de hiptesis se
calcula cul es la probabilidad de encontrar estos resultados si los dos
tratamientos fuesen iguales y se obtiene un grado de significacin del 2 %.
Trasladado a cmo est formulada la pregunta, si los dos tratamientos fuesen iguales (si la Ho fuese cierta), resultados como los obtenidos
o diferencias an mayores (en nuestro ejemplo, diferencias mayores a
60 - 45), ocurriran con una probabilidad menor del 5 % (en nuestro
caso con una probabilidad del 2%), que es lo que dice la opcin 4.
Pregunta 28.- R: 1
El valor mximo admitido de error alfa para establecer diferencias
mediante contraste de hiptesis en trabajos cientficos es del 5%; el
otro valor habitualmente utilizado (ms estricto) es el 1%.
Como vemos, el mximo error tipo I aceptado en trabajos cientficos es el 5 %.
Pregunta 29.- R: 4
Cuando el grado de significacin obtenido es mayor que el nivel
crtico fijado (habitualmente el 5%), la probabilidad de que las difeM exico A rgentina
C hile U ruguay

Seguimiento a distancia

rencias observadas sean debidas al azar es demasiado alta (fjate que


"demasiado alta" puede ser un 6 %), por lo que no tenemos suficiente
evidencia para rechazar la hiptesis nula.
No rechazar la hiptesis nula NO significa que sea cierta (es ms,
un grado de significacin cercano al 5 % sugiere que hay diferencias,
pero que stas pueden deberse al azar con una probabilidad no tan
baja como la exigida), por lo que NO podremos decir que no haya
diferencias (opcin 4 falsa).
Pregunta 30.- R: 3
La probabilidad de que existiendo diferencias (es decir, de que
siendo falsa la hiptesis nula), se detecten, o lo que es lo mismo, se
obtenga suficiente evidencia para rechazar la hiptesis nula, es la
potencia estadstica.
La probabilidad de que habiendo diferencias NO se demuestren
es el error tipo II, por lo que:
Error tipo II + Poder Estadstico = 1
Como el Error tipo II y el Poder Estadstico son complementarios, si
la probabilidad de cometer un error beta es del 3 %, la probabilidad
de detectar las diferencias ser del 97% (como dice la opcin 3).
Pregunta 31.- R: 1
Pongamos un ejemplo para su mejor comprensin:
Se realiza un estudio longitudinal en el que se selecciona un grupo
de personas expuestas a HTA y otro grupo de personas que no presentan este factor de estudio; se siguen durante 2 aos a todas las personas participantes y se detecta una Incidencia de ACV entre las personas que padecen HTA de 80 por cada 1.000 personas y entre las que
no tienen HTA de 40 por cada 1.000 personas.
Estas diferencias encontradas se pueden deber al azar, a la variabilidad en el muestreo?.
La respuesta hay que buscarla en el grado de significacin, que en
este caso es menor al 5%, por lo que en caso de que no hubiese una
asociacin entre el factor estudiado (la HTA) y la enfermedad (los
ACVs) la probabilidad de haber encontrado estos resultados sera menor
del 5% (opcin 1 correcta).
NO se confirma relacin de causalidad entre el factor de riesgo y la
enfermedad estudiada, ya que para ello se tienen que cumplir una serie
de criterios de causalidad que en esta pregunta NO se mencionan.
Pregunta 32.- R: 4
Un Error Tipo I consiste en rechazar la Hiptesis Nula cuando en
realidad es cierta (opcin 4 falsa).
La probabilidad de que rechacemos la Ho y sea cierta coincide con la
probabilidad de que las diferencias encontradas se deban al azar (es decir
que NO haya diferencias), sto es, el grado de significacin.
En otras palabras, decimos que la probabilidad de que las diferencias observadas se deban al azar es tan baja como para creer que hay
verdaderas diferencias y en realidad, aunque era muy poco probable, las diferencias encontradas son debidas al azar.
Pregunta 33.- R: 1
No demostramos la falsedad de la hiptesis nula, ya que aunque p
<0,05 (lo que indica que existe una diferencia estadsticamente significativa) siempre existe una pequea probabilidad de que la diferencia encontrada se deba al azar (Error Tipo I), con lo que rechazamos
(no demostramos que es falsa) la Hiptesis Nula a ese nivel de significacin (<5%).
Pregunta 34.- R: 4
Aunque la p haya resultado no significativa (para el nivel crtico del 5
%), no podemos decir que no haya diferencia de eficacia entre aspirina
y ticlopidina (no sugerimos que la hiptesis nula sea cierta, simplemente
no tenemos suficiente evidencia para decir que es falsa).
La p no tiene nada que ver con la magnitud de la eficacia por lo
que la opcin que dice "la ticlopidina es un 5% ms eficaz que la
aspirina" es FALSA.
La probabilidad de que las diferencias observadas se deban al azar
NO es menor del 5% ya que la p no ha sido significativa.

CTO Medicina C/ Nez de Balboa, 115 28006 MADRID (Espaa) Tfno.: (91) 782 43 32 / Fax: (91) 782 43 27
E-mail: secretaria@ctomedicina.com; iberocto@ctomedicina.com WEB: www.ctomedicina.com; www.iberocto.com

ET Pg. 5

ESTADSTICA

Seguimiento a distancia

Preparacin Examen de Seleccin 05/06 1 Vuelta

Pregunta 35.- R: 4
En caso de que no existan diferencias entre dos grupos comparados, la probabilidad de encontrar cierta diferencia (por azar o variabilidad en el muestreo) es el grado de significacin.
En esta pregunta, en caso de que entre el peso de hijos de fumadoras y no fumadoras no hubiese diferencias, la probabilidad de encontrar los resultados obtenidos ser menor del 5 %, por lo que la opcin
correcta es la 4.
Pregunta 36.- R: 4
Si las diferencias entre dos tratamientos son muy grandes, NO es
preciso un tamao muestral grande. Si con un antihipertensivo A se
consigue como media un descenso de TAS de 40 mm de Hg y con
otro antihipertensivo B un descenso de TAS de 12 mm de Hg, aunque
tengas un tamao muestral pequeo, es poco probable que esta diferencia se pueda explicar por el azar.
Otra situacin sera que antes de empezar el estudio, se estableciese como diferencia mnima con significacin clnica 30 mm de Hg;
aqu s que para demostrar diferencias de, al menos, esta magnitud
(relativamente grande) es posible que se necesitase un tamao muestral
grande.
Pregunta 37.- R: 3
Para contestar este tipo de preguntas lo ms importante es encontrar las dos variables del estudio que nos propongan.
En este caso, la variable predictora (V1) ser: no haber fumado
nunca, ser ex-fumador o fumador; vemos que se trata de una variable
cualitativa que tiene ms de dos categoras (variable cualitativa no
dicotmica). La variable resultado (V2) ser: la tensin arterial medida
cuantitativamente.
Con una variable predictora cualitativa de ms de dos categoras y una
variable resultado cuantitativa el test de contraste de hiptesis adecuado
es el Anlisis de la Varianza (ANOVA). La opcin correcta es la 3.

Pregunta 37. Test de contraste de hiptesis.


VARIABLE 1

VARIABLE 2

TEST DE HIPTESIS

Dicotmica

Dicotmica

Chi-cuadrado.
Test exacto de Fisher.

Cualitativa
(> 2 categoras)

Cualitativa
(>= 2 categoras)

Chi-cuadrado

Dicotmica

Cuantitativa

t de Student

Cualitativa
(> 2 categoras)

Cuantitativa

Anlisis de la Varianza

Cuantitativa

Cuantitativa

Coeficiente de correlacin
de Pearson.
Regresin.

Pregunta 38.- R: 4
Se pretende determinar si existe asociacin entre el tipo de carrera que se cursa y la presencia o no de antgeno de la superficie del
virus de la hepatitis B, por lo que se aprecian las siguientes variables:
V1: Carrera universitaria que se estudia: variable cualitativa que en
nuestro estudio es dicotmica, ya que slo tiene dos posibilidades
(Medicina u Odontologa).
V2: Presencia o no del antgeno de superficie del VHB: tambin
variable cualitativa dicotmica, ya que slo tiene dos posibilidades
(dar positivo para Ag VHB o dar negativo).

Pg. 6 ET

M exico A rgentina
C hile U ruguay

El test de contraste de hiptesis adecuado para determinar si existe


asociacin o no (es decir, que las diferencias encontradas entre los dos
grupos de estudiantes se puedan deber al azar) entre dos variables cualitativas (independientemente de que sean dicotmicas o no, aunque
en este ejemplo lo sean ambas) es el test de Chi-cuadrado.
La opcin correcta es la 4.
Pregunta 39.- R: 2
Al buscar las dos variables de este estudio, tienes que darte cuenta de
que la edad de las pacientes NO es una variable, simplemente nos estn
definiendo qu personas participan en el estudio, es decir, nos estn
dando los criterios de seleccin. Mediante los criterios de seleccin se
define la poblacin a la que se podrn extrapolar los resultados de
nuestro estudio, que en este caso ser a adolescentes de 14 a 16 aos.
El objetivo de este estudio es determinar si existen diferencias entre
tres tratamientos en cuanto a la reduccin del nmero de lesiones de
acn, por lo que las variables sern:
Variable predictora: tratamiento utilizado: variable cualitativa de
tres categoras.
Variable resultado: el nmero de lesiones que desaparecen: variable cuantitativa; el resultado lo podan haber expresado como
variable dicotmica (mejora o no), pero en el enunciado nos dejan claro que van a cuantificar el resultado ("nmero de lesiones de
acn").
Con una variable predictora cualitativa de ms de dos categoras y
una variable resultado cuantitativa, el test de contraste de hiptesis
adecuado es el Anlisis de la Varianza (ANOVA).
La opcin correcta es la 2.
Pregunta 40.- R: 4
Imaginemos que tenemos un grupo de mujeres embarazadas a las
que se les toma la TAD y se obtiene como resultado una media aritmtica de 72 mm de Hg, y otro grupo de mujeres no gestantes a las que
se les toma la TAD y se obtiene como resultado una media aritmtica
de 74 mm de Hg.
Posiblemente esta diferencia se deba a la variabilidad en el muestreo (al azar), pero para poder hacer esta afirmacin tendremos que
hacer el test de contraste de hiptesis que corresponda con las variables de este estudio:
V1: estar o no embarazada (variable dicotmica).
V2: TAD (variable cuantitativa).
Con una variable predictora cualitativa de dos categoras y una
variable resultado cuantitativa, el test de contraste de hiptesis adecuado es la t de Student.
La opcin correcta es la 4.
Pregunta 41.- R: 2
Aunque el enunciado de esta pregunta nos quede un poco
confuso, debemos ser capaces de distinguir las siguientes variables:
tiempo de reproduccin y fotoperodo. La primera es una variable
cuantitativa (el tiempo) y la segunda tambin (nmero de horas de
luz), por lo que disponemos de dos posibles test para trabajar:
Anlisis de Regresin.
Test de correlacin.
Cul es la clave para elegir uno o el otro?
Cuando lo que nos estn pidiendo es la ecuacin que relaciona a
las dos variables, dicho de otro modo, la frmula que me permite
decir cunto valdr la y en funcin de la x (por ejemplo, cunto ser
el tiempo de reproduccin si el fotoperodo vale 6 horas) hay que
hacer un Anlisis de Regresin.
La opcin correcta es la 2.
Y cundo se utiliza el test de correlacin?
El test de correlacin de Pearson sirve para establecer, en dos variables que entre ellas siguen una relacin lineal (la ecuacin obtenida
con el anlisis de regresin es lineal, del tipo y=A+Bx), si la relacin
lineal es buena (la ecuacin obtenida se ajusta bien a la realidad) o no
(no es una buena ecuacin para predecir y en funcin de x).
Pregunta 42.- R: 3
Un coeficiente de correlacin de Pearson de -1 indica que existe
una fuerte correlacin lineal entre las dos variables y que esta relacin

CTO Medicina C/ Nez de Balboa, 115 28006 MADRID (Espaa) Tfno.: (91) 782 43 32 / Fax: (91) 782 43 27
E-mail: secretaria@ctomedicina.com; iberocto@ctomedicina.com WEB: www.ctomedicina.com; www.iberocto.com

Comentarios TEST

Las diferencias entre ticlopidina y aspirina NO exceden lo atribuible al azar, ya que sto se puede decir cuando la p es menor al 5 %.
De todas las opciones es cierta la que dice que "es posible que aumentando el tamao de la muestra obtengamos diferencias significativas para
el mismo nivel de significacin", es decir, que aumentando el tamao
muestral (factor ms importante de la potencia estadstica) aumentamos
la probabilidad de que se encuentren diferencias si las hay.

ESTADSTICA

Preparacin Examen de Seleccin 05/06 1 Vuelta


es negativa, es decir, que cuando aumenta una variable la otra disminuye y viceversa.
La opcin correcta es la 3.
Recuerda que los valores del coeficiente de correlacin oscilan de
-1 a 1, y que cuanto ms se acerca a -1 a 1 la relacin lineal ser ms
fuerte, mientras que un coeficiente de correlacin de 0 indica ausencia de relacin lineal.

Comentarios TEST

Pregunta 43.- R: 4
Si se pretende estudiar si hay diferencia o no entre los niveles de
glucosa en sangre en personas que no han comido y personas que ya
han comido, el estudio se puede plantear de dos maneras:
a) Cogemos a una muestra de personas y les extraemos una muestra
de sangre antes de comer, y a otro grupo de personas distintas y les
extraemos sangre despus de que hayan comido.
b) Cogemos a una muestra de personas y les extraemos una muestra
de sangre antes de comer, y a ese mismo grupo de personas les
extraemos sangre despus de que hayan comido.
En ambos casos, las variables de estudio seran:
V1: No haber comido o haber comido (variable dicotmica).
V2: Niveles de glucosa en sangre: variable cuantitativa.
La nica diferencia es que en el primer caso las muestras son independientes (dos grupos de personas distintas) y en el segundo hay una
muestra pareada (se mide la V2 en la mismas personas antes y despus
de comer), por lo que utilizaramos un test de t de Student para datos
apareados, mientras que en el primer caso sera para datos independientes.
La opcin correcta es la 4.

Seguimiento a distancia

Vemos que la primera variable (dicotmica) es pareada, ya que la


V2 se va a medir en los mismos pacientes (antes y despus); la segunda
variable es cuantitativa (nivel de ADN viral), por lo que el test paramtrico que le correspondera es el de la t de Student.
Pero como el tamao muestral es menor de 30, no nos valen los
test paramtricos, por los que tendremos que elegir el equivalente a la
t de Student para datos apareados: el test de Wilcoxon.
La opcin correcta es la 5.

Pregunta 47. Test no paramtricos.


VARIABLE 1

VARIABLE 2

TEST DE HIPTESIS

Dicotmica
independiente

Cuantitativa (n <30)
u ordinal

U de Mann-Whitney.
Test de Wilcoxon.

Dicotmica pareada
o dependiente

Cuantitativa (n <30)
u ordinal

Test de Wilcoxon

Cualitativa
(>2 categoras)

Cuantitativa (n <30)
u ordinal

Test de Kruskal-Wallis

Cualitativa pareada
(>2 categoras)

Cuantitativa (n <30)
u ordinal

Test de Friedman

Ordinal

Ordinal

Rho de Spearman.
Tau de Kendall.

Pregunta 44.- R: 2
Se quiere establecer si hay diferencia entre el descenso de carga
viral que se consigue con indinavir frente al obtenido con saquinavir,
por lo que las variables de este estudio sern:
V1: tratamiento con indinavir o saquinavir: variable cualitativa dicotmica.
V2: descenso en la carga viral: variable cuantitativa.
Con una variable predictora cualitativa de dos categoras y una
variable resultado cuantitativa, el test de contraste de hiptesis adecuado es la t de Student.
La opcin correcta es la 2.

Pregunta 48.- R: 1
Si se quiere saber la manera de relacionarse de dos variables cuantitativas (la ecuacin que informa del tipo de relacin), se debe realizar un Anlisis de Regresin y, en caso de que la relacin entre dos
variables cuantitativas sea lineal, el coeficiente de correlacin informa
de cmo de buena es esa relacin lineal.
La formulacin de esta pregunta es ciertamente ambigua (y nos
tenemos que acostumbrar a este tipo de preguntas en el MIR) y no nos
permite saber por cul de los dos test tiles para dos variables cuantitativas nos preguntan: de suerte que entre las opciones slo hay uno,
por lo que marcaremos la opcin 1 como correcta.

Pregunta 45.- R: 4
La mayora de las preguntas de test de contraste de hiptesis en el
MIR se refieren a test paramtricos, si bien debemos conocer cundo
no se pueden utilizar y es necesario elegir un test no paramtrico:
Cuando el tamao muestral es pequeo (n<30).
Cuando V1 y/o V2 sean variables ordinales.
Los test no paramtricos son menos potentes que los paramtricos,
por ello siempre que sea posible se prefiere utilizar pruebas paramtricas.
Las pruebas no paramtricas, a diferencia de la paramtricas, no
requieren que las variables sigan una distribucin normal, pueden
usarse independientemente de cmo sea la distribucin de las variables en la poblacin (normal, binomial, etc.). La opcin falsa es la 4.

Pregunta 49.- R: 5
Si se comparan dos frmacos (como es el caso de esta pregunta), la
primera variable ser cualitativa dicotmica (IECA frente a betabloqueante).
La variable resultado podra ser tambin dicotmica (disminucin
o no de la TAS), pero si no lo especifican y la variable se puede medir,
debemos sobreentender que la variable resultado ser cuantitativa
(media de descenso de TAS en ambos grupos de tratamiento).
Con una variable predictora cualitativa de dos categoras y una
variable resultado cuantitativa, el test de contraste de hiptesis adecuado es la t de Student.
La opcin correcta es la 5.
En caso de que se interpretara la V2 como una variable cualitativa
(respuesta o no al tratamiento), el test que se debe utilizar es el Chicuadrado, que, como ves, no est entre las opciones.

Pregunta 46.- R: 4
Si se estn comparando tres tratamientos distintos, la variable
predictora ser el tipo de tratamiento (variable cualitativa de tres
categoras).
En este estudio, la variable resultado tambin es cualitativa (curacin o no curacin), por lo que el test de contraste de hiptesis que se
debe utilizar ser el de Chi-cuadrado.
La opcin correcta es la 4.
Pregunta 47.- R: 5
Se pretende ver si hay una diferencia entre los niveles de ADN viral
del VHB en pacientes nefrpatas antes de ser sometidos a hemodilisis
y despus de esta intervencin, por lo que las variables de este estudio
son:
V1: no haber recibido todava la hemodilisis o haberla recibido ya.
V2: ttulo de ADN viral del VHB.
M exico A rgentina
C hile U ruguay

Pregunta 50.- R: 4
De todas las opciones citadas, la nica prueba til para establecer
relacin entre una variable cualitativa y otra cuantitativa es el Test de
Mann-Whitney, que se usa cuando la V1 es una variable dicotmica,
la V2 es cuantitativa, las muestras son independientes y el tamao
muestral es menor de 30.
La opcin correcta es la 4.
El test de la t de Student tambin es til para establecer relacin
entre una variable cualitativa y otra cuantitativa, pero si lees con
cuidado las opciones, te dars cuenta de que no est (la opcin 1 es
la distribucin t de Student, no es test de contraste de hiptesis).

CTO Medicina C/ Nez de Balboa, 115 28006 MADRID (Espaa) Tfno.: (91) 782 43 32 / Fax: (91) 782 43 27
E-mail: secretaria@ctomedicina.com; iberocto@ctomedicina.com WEB: www.ctomedicina.com; www.iberocto.com

ET Pg. 7

MEDICINA PREVENTIVA

Preparacin Examen de Seleccin 05/06 1 Vuelta


1.

Seale la opcin correcta:


1)
2)
3)
4)
5)

2.

4)
5)
3.

2)
3)
4)
5)

Preguntas TEST

2)
3)
4)
5)
8.

La forma ms habitual de intoxicacin en nuestro medio


es la profesional.
El saturnismo es la intoxicacin crnica por plomo y es
clnicamente evidente a dosis bajas (contaminacin).
La clnica neuropsiquitrica es inespecfica.
En el sndrome hematolgico debe hacerse diagnstico
diferencial con talasemias y anemia ferropnica.
La determinacin de coproporfirina III en orina no es
exclusiva del saturnismo.

2)
3)
4)
5)
9.

2)
3)
4)
5)

1)
2)
3)
4)
5)

Portador
Portador
Portador
Portador
Portador

paradjico o pseudoportador.
precoz o incubacionario.
convaleciente.
sano.
pasivo o contacto.

2)
3)
4)
5)

M exico A rgentina
C hile U ruguay

12.

Promiscuidad.
Estadio avanzado de la enfermedad.
lceras genitales y relaciones traumticas.
Otras ETS coexistentes.
Algunos espermicidas.

Las siguientes medidas reducen el riesgo de trasmisin vertical del VIH de forma significativa, EXCEPTO:
1)

Aquella persona que elimina el microorganismo antes de que


aparezca la enfermedad que est incubando se denomina:

El mayor nmero de infecciones de SIDA en Espaa se


produjo entre 1985-88.
La prevalencia de SIDA mxima en Espaa se alcanz en
1988.
El nmero de muertes por SIDA ha disminuido en los
ltimos aos.
La mayora de los infectados por VIH no tienen SIDA.
Aunque ha disminuido el nmero de nuevas infecciones
por VIH en los ltimos aos, el nmero total de infectados
ha aumentado.

Todos los siguientes son factores de riesgo en la trasmisin


sexual en la infeccin por VIH, EXCEPTO:
1)
2)
3)
4)
5)

11.

Mesoendemia.
Endoepidemia.
Holoendemia.
Hiperendemia.
Hipoendemia.

La tendencia de casos diagnosticados de SIDA es ascendente en los ltimos aos.


El nmero de nuevos casos producidos en el 2001 ha sido
mayor en mujeres que en hombres.
Aproximadamente un 1% de los casos de SIDA que se
produjeron en Espaa fueron debidos a transfusiones.
La forma de transmisin ms frecuente en mujeres ha sido
la heterosexual.
De los casos producidos de SIDA en 2001, un 10%
corresponde a la trasmisin madre-hijo.

Seale la INCORRECTA sobre la evolucin del SIDA en


Espaa:
1)

10.

Patogenicidad.
Prdromos.
Contagiosidad.
Infectividad.
Virulencia.

Separar de la comunidad a un enfermo cortando las vas


de trasmisin de la enfermedad a la poblacin sana.
Separar de la comunidad a una persona aparentemente
sana que ha estado expuesta al contagio.
Identificar el microorganismo patgeno y aislarlo para
instaurar un tratamiento correcto.
Utilizacin de material desechable.
Separacin indefinida de sanos y enfermos.

Sobre la situacin epidemiolgica del SIDA en Espaa, una es


cierta:
1)

El tabaco es causa de hipertensin arterial.


El tabaco es la principal causa de muerte evitable.
La obesidad se relaciona con los estados de hiperinsulinismo y resistencia perifrica a la insulina.
Dosis moderadas de alcohol se han relacionado con
disminucin del riesgo de cardiopatas.
La edad es un marcador de riesgo para la hipertensin
arterial.

Aquella enfermedad que se presenta con una incidencia


mayor del 75% en una poblacin se denomina:
1)
2)
3)
4)
5)

6.

1)

La capacidad del agente infeccioso de instalarse y multiplicarse en los tejidos, produciendo o no enfermedad, se denomina:
1)
2)
3)
4)
5)

5.

Aislamiento se define como:

Seale la INCORRECTA sobre la intoxicacin por plomo:


1)

4.

Vacunacin frente a meningococo-prevencin secundaria.


Citologa vaginal en pacientes con antecedentes de displasia-prevencin secundaria.
Mamografa en familiares de pacientes con cncer de
mama-prevencin primaria.
Autopalpacin mamaria-prevencin primaria.
Abandono de tabaco en EPOC-prevencin secundaria.

Una de las siguientes afirmaciones sobre factores de riesgo


NO es correcta:
1)
2)
3)

7.

Seguimiento a distancia

Administracin de AZT a la madre desde el principio del


segundo trimestre hasta el final del embarazo.
Administracin AZT al RN durante 6 meses.
Realizacin cesrea.
Supresin lactancia materna.
Acortar el expulsivo.

Sobre la prevalencia de la hepatitis B, una es INCORRECTA:


1)
2)

Existen aproximadamente 300 millones de personas infectadas con el VHB en todo el mundo.
En Espaa se producen ms de 10000 casos nuevos cada
ao.

CTO Medicina C/ Nez de Balboa, 115 28006 MADRID (Espaa) Tfno.: (91) 782 43 32 / Fax: (91) 782 43 27
E-mail: secretaria@ctomedicina.com; iberocto@ctomedicina.com WEB: www.ctomedicina.com; www.iberocto.com

ED Pg. 1

3)
4)
5)
13.

2)
3)
4)
5)

2)
3)
4)
5)

2)
3)
4)
5)

5)
19.

En el hijo de madre VHB debe administrarse IGHB en las


primeras horas asociando la vacunacin completa.
Cuando se sospecha contacto sexual en un sujeto no
vacunado, debe administrarse IGHB y vacuna, en un
plazo menor a dos semanas.
La pauta de vacunacin de referencia es 0-1-6 meses.
No se realiza verificacin de anticuerpos de forma sistemtica en los sujetos vacunados.
Si se interrumpe la pauta de vacunacin, sta debe ser
reiniciada.

3)
4)
5)
20.

Se inactiva por cloracin del agua, ebullicin y luz ultravioleta.


Hay portadores asintomticos que trasmiten la enfermedad.
En pases subdesarrollados es ms frecuente en nios y
jvenes.
La inmunoprofilaxis pasiva se realiza con IgG no especfica.
La inmunoprofilaxis activa se realiza con vacuna inactivada frente VHA.

VHA.
VHB.
VHC
VHD.
VHE.

5)

2)
3)
4)

1)
2)
3)
4)
5)

Pg. 2 ED

Larngea.
Traqueal.
Endobronquial.
Cavitacin extensa.
Miliar.
M exico A rgentina
C hile U ruguay

5)

Inicialmente hacer Mantoux, y si es negativo, dar quimioprofilaxis primaria con isoniacida.


Hacer un Mantoux, y si es positivo, vacunar con BCG.
Hacer un Mantoux, y si es negativo, repetirlo a las 8
semanas.
Hacer placa de trax. Si es negativa, iniciar quimioprofilaxis secundaria.
Dar triple terapia a la nia.

En la situacin anterior, cul sera la pauta de actuacin si


viviera una joven de 23 aos?:
1)

Cul de las formas de TBC es MENOS contagiosa?:

Efecto Booster.
Debera realizarse un nuevo Mantoux.
La primera tuberculina pudo estar expuesta a la luz.
La tcnica de la primera intradermorreaccin pudo ser
incorrecta.
Debera hacerse a continuacin una placa de trax.

En una chabola conviven 10 personas. Un varn de 34 aos


con tos pertinaz, expectoracin escasa y fiebre persistente es
diagnosticado de TBC. En esa chabola vive una nia de 5 aos.
La actuacin ms correcta con la nia sera:
1)

22.

Joven de 20 aos con Mantoux de 16 mm.


Nio de 9 aos, con Mantoux negativo, y cuyo padre
acaba de ser diagnosticado de tuberculosis pulmonar.
Hombre de 45 aos, Mantoux positivo, que va a ser
sometido a trasplante.
Mujer de 60 aos, Mantoux positivo, a la que se le va a
realizar una histerectoma.
Mujer de 35 aos, con Mantoux de 10 mm., que acaba
de ser diagnosticada de LES.

Un paciente de 60 aos acude a nuestra consulta por tos seca


y fiebre vespertina. La enfermera le pincha una tuberculina
que resulta negativa. A la semana se repite la prueba que esta
vez es positiva. Todo lo siguiente es cierto, EXCEPTO:
1)
2)
3)
4)

21.

Pone de manifiesto el contacto con el bacilo (infeccin).


Es una reaccin de inmunidad celular.
Consiste en inyectar 5 U.I. de tuberculina subcutneas.
El dimetro que se mide es el de la induracin y no el del
eritema.
Es til para identificar candidatos para la quimioprofilaxis.

Seale cul de las siguientes NO es indicacin para la quimioprofilaxis de tuberculosis con isoniacida:
1)
2)

El agente ms frecuentemente causal de la hepatitis postransfusional es:


1)
2)
3)
4)
5)

17.

La eficacia de la vacuna es mayor en nios y adultos


jvenes.
La obesidad o el hbito tabquico reducen la eficacia de
la vacuna.
En pacientes inmunocomprometidos, la eficacia de la
vacuna es aproximadamente del 50%.
En los pacientes inmunocompetentes que no desarrollan
anticuerpos est indicada la revacunacin completa.
La duracin de la proteccin de la pauta completa de
vacunacin parece ser menor de 5 aos.

Respecto al MANTOUX, es FALSO:


1)
2)
3)
4)

En cuanto al VHA, podremos afirmar todo lo siguiente,


EXCEPTO:
1)

16.

18.

Sobre la prevencin de la hepatitis B, es FALSO:


1)

15.

En nuestro pas, hasta un 10% de las infecciones tienen un


curso hacia la cronicidad.
Asia y frica son reas de mayor prevalencia.
En las zonas de mayor prevalencia, la frecuencia de
marcador HbsAc es baja.

En cuanto a la vacuna frente al VHB, una NO es correcta:


1)

14.

MEDICINA PREVENTIVA

Preparacin Examen de Seleccin 05/06 1 Vuelta

2)
3)
4)
5)

Inicialmente hacer Mantoux, y si es negativo, dar quimioprofilaxis primaria con isoniacida.


Hacer un Mantoux, y si es negativo, repetirlo a las 8
semanas. Si ste segundo es positivo, iniciar tratamiento.
Si dos Mantoux son negativos y la placa de trax es
negativa, debera iniciarse quimioprofilaxis secundaria.
Si el segundo Mantoux es positivo y la placa negativa,
debera iniciarse quimioprofilaxis secundaria.
Dar triple terapia a la joven.

CTO Medicina C/ Nez de Balboa, 115 28006 MADRID (Espaa) Tfno.: (91) 782 43 32 / Fax: (91) 782 43 27
E-mail: secretaria@ctomedicina.com; iberocto@ctomedicina.com WEB: www.ctomedicina.com; www.iberocto.com

Preguntas TEST

Seguimiento a distancia

MEDICINA PREVENTIVA

Preparacin Examen de Seleccin 05/06 1 Vuelta


23.

En la situacin anterior, cul sera la pauta de actuacin si


viviera un varn de 45 aos?:
1)
2)
3)
4)
5)

24.

En la situacin anterior, cul sera la pauta de actuacin si


viviera un varn de 70 aos?:
1)
2)
3)
4)
5)

25.

Preguntas TEST

3)
4)
5)
30.

3)
4)
5)
31.

2)
3)
4)
5)

Gotas de Pflugge.
Directamente.
Por la piel.
En relacin con los fmites.
A travs del agua o los alimentos.

5)

El B es el ms prevalente en Espaa.
El C est en aumento en Espaa.
El B produce brotes limitados o casos espordicos.
El A es el que tiene mayor capacidad para producir
epidemias.
Los serogrupos A, B y C dan cuenta del 40% de los casos.

La quimioprofilaxis de la infeccin meningoccica en embarazadas se realiza con:


1)
2)
3)
4)
5)

Penicilina 1 dosis v.o.


Ceftriaxona 1 dosis i.m.
Rifampicina 1 dosis i.m.
Rifampicina 600 mg / 12 h. durante 2d.
Sulfamidas 1 dosis v.o.
M exico A rgentina
C hile U ruguay

Un sujeto que recibi 3 dosis hace 8 aos, no debe


administrarse toxoide.
Un sujeto bien inmunizado que se vacun hace 14 aos
debe iniciar la pauta de vacunacin 0-1-6.
Un individuo que recibi 2 dosis hace 4 aos debe completar la pauta de inmunizacin con otra dosis de toxoide.
Un individuo que ha recibido dos dosis hace 7 aos debe
recibir dos dosis con un mes de intervalo.
Un sujeto que recibi 1 dosis hace 6 aos, debe recibir la
pauta de vacunacin completa.

Acude a urgencias un individuo con una herida incisocontusa


de bordes anfractuosos en la pierna producida por un
cortador de csped. Vd. interroga al paciente, quien le
informa de que no est vacunado frente al ttanos, aunque
tuvo dicha enfermedad hace 4 aos. Su actitud ser:
1)
2)
3)
4)
5)

33.

La prevalencia del ttanos en Espaa est en descenso.


La forma ms frecuente de ttanos en los pases en vas de
desarrollo es el ttanos neonatal.
El ttanos neonatal tiene una gran mortalidad.
La proteccin frente al ttanos es mayoritaria en los sujetos
mayores de 60 aos en nuestro medio.
La espora del ttanos es resistente a la ebullicin.

En caso de vacunacin incompleta frente al ttanos, seale la


FALSA:
1)

32.

La inmunizacin conseguida con la vacuna de polisacridos es pequea en nios menores de 4 aos.


La vacuna de polisacridos previene frente a los serogrupos ms frecuentes en nuestro medio.
La vacuna conjugada es ms inmungena en nios menores de 6 aos.
La vacuna conjugada puede administrarse en nios menores de 1 ao (2-4-6 meses).
Es una indicacin de vacunacin la asplenia.

Sobre el ttanos, seale la INCORRECTA:


1)
2)

Respecto a los serogrupos de N. meningitidis, es FALSO:


1)
2)
3)
4)

28.

2)

Gram (-).
Meningococo.
H. influenzae.
Neumococo.
E. coli.

La infeccin meningoccica se transmite por:


1)
2)
3)
4)
5)

27.

Inicialmente hacer Mantoux, y si es negativo, dar quimioprofilaxis primaria con isoniacida.


Hacer un Mantoux, y si es negativo, repetirlo a las 8
semanas. Si el segundo es positivo, iniciar tratamiento.
Si el Mantoux es negativo, se realiza otro a la semana y si
es negativo, a las 8 semanas del primero.
Si el segundo Mantoux es positivo y la placa negativa,
debera iniciarse quimioprofilaxis secundaria.
Si el segundo Mantoux es positivo y la placa negativa, no
estara indicado continuar el estudio.

Con respecto a la vacunacin de la meningitis meningoccica, una NO es correcta:


1)

El agente causal ms frecuente de meningitis bacteriana entre


los 30-65 aos es:
1)
2)
3)
4)
5)

26.

Inicialmente hacer Mantoux, y si es negativo, dar quimioprofilaxis primaria con isoniacida.


Hacer un Mantoux, y si es negativo, repetirlo a las 8
semanas. Si ste segundo es positivo, iniciar tratamiento.
Si dos Mantoux son negativos y la placa de trax es
negativa, debera iniciarse quimioprofilaxis secundaria.
Si el segundo Mantoux es positivo y la placa negativa,
debera iniciarse quimioprofilaxis secundaria.
Si el segundo Mantoux es positivo y la placa negativa, no
estara indicado continuar el estudio.

29.

Seguimiento a distancia

Administrar 250 U.I. de gammaglobulina antitetnica.


Administrar toxoide tetnico.
No hacer nada, ya que el riesgo tetangeno de la herida es
bajo.
Administrar toxoide ms gammaglobulina.
No hacer nada, ya que tiene inmunidad debida a la
enfermedad pasada.

Sobre la gripe, seale la INCORRECTA:


1)
2)
3)
4)

La enfermedad se presenta en brotes anuales durante los


meses fros.
La incidencia en el grupo de nios de corta edad es muy
elevada.
Los contagios se producen habitualmente a partir del
sujeto portador asintomtico o convaleciente.
La vacunacin se realiza de forma sistemtica en grupos de
alto riesgo con las cepas que circularon en la temporada
anterior.

CTO Medicina C/ Nez de Balboa, 115 28006 MADRID (Espaa) Tfno.: (91) 782 43 32 / Fax: (91) 782 43 27
E-mail: secretaria@ctomedicina.com; iberocto@ctomedicina.com WEB: www.ctomedicina.com; www.iberocto.com

ED Pg. 3

5)
34.

2)
3)
4)
5)

2)
3)
4)
5)

5)

Las vacunas vivas suelen precisar de varias administraciones para conseguir buena respuesta inmunitaria.
Las vacunas inactivas rara vez son enterales.
Las vacunas inactivas suelen precisar de adyuvantes.
La presencia de un sujeto inmunodeprimido en un domicilio no contraindica la vacunacin con virus tipo Salk a
los otros sujetos de la familia.
La produccin de IgA es frecuente tras la administracin
de la vacuna Sabin.

2)
3)
4)
5)
40.

3)
4)
5)
41.

2)
3)
4)
5)

2)
3)

Seale la INCORRECTA respecto a la inmunizacin pasiva:


1)
2)
3)
4)
5)

Pg. 4 ED

La inmunoglobulina hiperinmune frente a VHB se utiliza


en la profilaxis postexposicin.
Est indicada la gammaglobulina inespecfica frente VHA
si se va a estar en riesgo o se ha estado en contacto con un
caso en un periodo de tiempo menor de 2 semanas.
Un nio VIH que ha seguido correctamente el calendario
vacunal y que est en contacto con un caso de sarampin,
debe recibir gammaglobulina.
La inmunoglobulina frente VHZ debe administrarse a un
sujeto inmunocomprometido en las primeras 96 horas
tras el contacto con un caso.
En un nio de 6 meses en contacto con un caso de
sarampin hay que dar inmunoglobulina y vacunar a las
4 semanas de recibir la inmunoglobulina.
M exico A rgentina
C hile U ruguay

4)
5)
43.

Deben asegurarse inmunizacin frente a neumococo y


gripe los mayores de 65 aos.
A partir de los 45 aos no se aconseja la inmunizacin de
rubola.
No es necesaria la vacunacin frente al ttanos en mayores
de 65 aos.
El componente de la vacuna del ttanos para adulto
contiene tambin una pequea dosis de toxoide frente a
la difteria.
No se aconseja la vacunacin frente a la polio en los
adultos.

Con respecto a la inmunizacin del nio VIH, una es FALSA:


1)

Hepatitis A.
Hepatitis B.
Varicela Zoster.
Ttanos.
Difteria.

La eficacia desde la primera dosis es muy elevada.


Puede indicarse en menores de 15 meses si la situacin
epidemiolgica lo aconseja, no evitando la dosis a los 1215 meses.
Si debe realizarse el test de Mantoux, debern esperarse
varias semanas tras la vacunacin TV.
En mujeres embarazadas no inmunizadas puede vacunarse para evitar el sarampin neonatal.
Debe evitarse su administracin en alrgicos al huevo.

Con respecto al calendario vacunal del adulto, la opcin


INCORRECTA es:
1)

42.

En los nios inmunocomprometidos debe administrarse


la DTPa en vez de DTP.
A partir de los 7 aos no se incluye componente diftrico
en la vacuna frente al ttanos.
La eficacia de la DTP es superior al 90%.
Cuando se asocia el antgeno Hib a al DTP, es preciso
continuar con la pauta de vacunacin hasta los 12 aos.
Es importante reiniciar la pauta de vacunacin si el nio
ha saltado una de las dosis.

Con respecto a la vacuna triple vrica, es FALSO:


1)
2)

Ndulo cutneo Td.


Adenitis locorregional BCG.
Vesculas varicela.
Exantema generalizado primera semana tras la vacunacin con rubola.
Artralgias tercera semana tras la vacunacin anti-rubola.

Frente a uno de los siguientes microorganismos NO existe un


suero especfico con eficacia probada para la inmunizacin
pasiva:
1)
2)
3)
4)
5)

38.

En sujetos inmunocomprometidos como el SIDA, est


contraindicada.
No debe administrarse durante el embarazo.
Puede administrarse en nios con fibrosis qustica a los 3
meses de edad.
En algunos casos de vacunacin con componentes de H
humana, se han descrito casos de Guillain- Barr.
Puede administrarse aunque exista anafilaxia documentada a la neomicina.

Seale la correcta:
1)

Con respecto a las reacciones adversas de las vacunas, una


es FALSA:
1)
2)
3)
4)

37.

39.

Con respecto a las vacunas, una de las siguientes es INCORRECTA:


1)

36.

Las glicoprotenas de superficie H y N son las que se tienen


en cuenta para la realizacin de las vacunas del virus A.

Con respecto a la vacunacin frente a la gripe, una es correcta:


1)

35.

MEDICINA PREVENTIVA

Preparacin Examen de Seleccin 05/06 1 Vuelta

Debe evitarse la vacunacin con microorganismos vivos si


es posible.
Se aconseja la vacunacin frente a varicela y TV en nios
no muy inmunodeprimidos.
Si un nio se realiza una herida sucia y ha seguido el
calendario vacunal correctamente, no se aconseja administrar gammaglobulina.
Si en una guardera un nio desarrolla sarampin y asiste
un nio VIH oligosintomtico, bien vacunado frente a
TV, debe recibir inmunoglobulina frente al sarampin.
Se aconseja la vacunacin con polio tipo Salk si el nio
VIH + no padece SIDA.

Una de las siguientes aseveraciones es FALSA respecto al


cncer:
1)
2)
3)

El 50% de los cnceres se diagnostican en el mundo


desarrollado.
El cncer ms frecuente en los pases occidentales es el
cncer de pulmn.
El cncer ms frecuente en las mujeres es el cncer de
mama.

CTO Medicina C/ Nez de Balboa, 115 28006 MADRID (Espaa) Tfno.: (91) 782 43 32 / Fax: (91) 782 43 27
E-mail: secretaria@ctomedicina.com; iberocto@ctomedicina.com WEB: www.ctomedicina.com; www.iberocto.com

Preguntas TEST

Seguimiento a distancia

MEDICINA PREVENTIVA

Preparacin Examen de Seleccin 05/06 1 Vuelta


4)
5)
44.

2)
3)
4)
5)

5)
49.

Los andrgenos estn implicados en la gnesis del cncer


de prstata.
Los anticonceptivos orales disminuyen el riesgo de padecer cncer de ovario.
La nuliparidad es un factor protector para el cncer de
ovario.
La terapia sustitutiva sin progestgenos en la menopausia
aumenta el riesgo de cncer de endometrio.
El embarazo que se produce a edades tempranas disminuye el riesgo de padecer cncer de mama.

1)
2)

5)
50.

1)
2)

1)

3)
4)

2)
3)
4)
5)

Existe una clara relacin dosis - respuesta entre el tabaco


y el riesgo de cardiopata isqumica.
El tabaco es el ms importante de los factores de riesgo
modificables.
El riesgo de cardiopata isqumica es menor en mujeres
fumadoras que en varones fumadores.
El riesgo de cardiopata isqumica tarda unos 10 aos en
descender de forma importante tras el abandono del
tabaco.
El tabaquismo no es factor de riesgo para el desarrollo de
hipertensin arterial.

Con respecto a las hiperlipidemias y a la enfermedad cardiovascular, una NO es correcta:

Indique cul de las siguientes NO es correcta:


El riesgo de cncer de pulmn disminuye al dejar de
fumar, regresando al nivel del no fumador a los 10 aos.
La placa de trax realizada a pacientes de alto riesgo
(varones mayores de 50 aos) no ha demostrado disminuir
la mortalidad por cncer de pulmn.
La elevacin del PSA es la prueba ms sensible para el
diagnstico precoz del cncer de prstata.
Ni la fosfatasa cida ni el PSA son suficientemente especficos como para realizar screening poblacional.
La citologa de Papanicolaou es poco sensible en el
diagnstico de adenocarcinoma de endometrio.

El principal marcador de riesgo es el sexo masculino.


El principal predictor para cardiopata isqumica en los
varones de edades medias es el colesterol.
El aumento de la mortalidad por cardiopata isqumica es
ms evidente con niveles sricos de colesterol superiores
a los 240 mg/dl.
Se observa un aumento de los niveles plasmticos de
colesterol y triglicridos con la edad.
La obesidad parece ser un factor determinante en la
elevacin de colesterol y triglicridos con la edad.

Cul de las siguientes es FALSA respecto a la influencia del


tabaco en el desarrollo de enfermedad cardiovascular?:

4)

5)

Preguntas TEST

4)

1)

4)

5)

Son el principal indicador de cardiopata isqumica en el


varn de mediana edad.
Cursan frecuentemente con aterosclerosis del sector aortoilaco.
Las dietas ricas en grasas saturadas elevan el c-LDL.
No es posible frenar la progresin de la placa de ateroma
ya formada.
El ejercicio fsico produce una elevacin del c-HDL.

Una de las siguientes es FALSA respecto a la epidemiologa de


las enfermedades cardiovasculares:
1)
2)
3)
4)
5)

48.

Se considera que la forma ms oncognica es el consumo


de cigarrillos.
El cncer de labio es especialmente frecuente en los
individuos que fuman en pipa.
Est implicado en un 10% de los cnceres que afectan a
la mujer.
El riesgo de cncer es mayor cuanto mayor es el consumo
de tabaco.
Los fumadores pasivos no tienen mayor riesgo de cncer.

3)

3)

47.

3)

Indique cul de las siguientes afirmaciones sobre factores


hormonales y cncer NO es correcta:

2)

46.

1)
2)

Con respecto al tabaco, seale la FALSA:


1)

45.

La primera causa de aos potenciales de vida perdidos en


la mujer es el cncer de mama.
El cncer de crvix es, como el de mama, un cncer que
aparece con mayor frecuencia en los pases desarrollados.

Seguimiento a distancia

Los factores socioculturales parecen ms importantes que


la determinacin gentica en el desarrollo de cardiopata
isqumica.
Las enfermedades cardiovasculares son la principal causa
de muerte en los pases desarrollados.
En Espaa ha disminuido en los ltimos aos la mortalidad
debida a cardiopata isqumica.
La tercera parte de las muertes por cardiopata isqumica
ocurren antes de los 65 aos (son prematuras).
La enfermedad cerebrovascular es causa de un mayor
porcentaje de las muertes que la cardiopata isqumica.

Sobre los factores y/o marcadores de riesgo para las enfermedades cardiovasculares, una es FALSA:
M exico A rgentina
C hile U ruguay

CTO Medicina C/ Nez de Balboa, 115 28006 MADRID (Espaa) Tfno.: (91) 782 43 32 / Fax: (91) 782 43 27
E-mail: secretaria@ctomedicina.com; iberocto@ctomedicina.com WEB: www.ctomedicina.com; www.iberocto.com

ED Pg. 5

MEDICINA PREVENTIVA

Preparacin Examen de Seleccin 05/06 1 Vuelta


Pregunta 1.- R: 2
Los niveles de prevencin en Medicina son tres. La prevencin primaria corresponde a las medidas de prevencin que se realizan antes
de que aparezca la enfermedad; es decir, se llevan a cabo cuando el
sujeto susceptible est en contacto con el factor de riesgo pero todava
no ha aparecido la enfermedad. Un ejemplo de esto seran las opciones 1 y 3. Con respecto a la opcin 3, aunque la mamografa se realice
en familiares de cncer de mama, no se va a evitar la enfermedad, sino
que se va a hacer un diagnstico precoz. Las medidas de prevencin
secundaria se realizan cuando ya ha aparecido la enfermedad, pero se
diagnostica en una fase precoz (habitualmente en fase asintomtica).
Son medidas de prevencin secundaria, por tanto, todas las pruebas de
screening tales como la opcin 2 y 3. La autopalpacin mamaria, si
bien no ha demostrado disminuir la mortalidad por cncer de mama,
puede considerarse prevencin secundaria al permitir diagnosticar
precozmente la enfermedad. La prevencin terciaria va encaminada a
tratar las secuelas de la enfermedad as como las recurrencias, tal y
como se refiere en la opcin 5.

Comentarios TEST

Pregunta 2.- R: 1
El tabaco es el factor de riesgo ms importante de enfermedades
cardiovasculares y de cncer, suponiendo la principal causa de muerte
evitable en los pases desarrollados. Aun cuando las personas ya hayan cumplido los 65 aos, se benefician de la interrupcin del consumo de tabaco. No se ha demostrado sin embargo que sea causa directa de hipertensin arterial (opcin 1).
Otro factor de riesgo importante en los pases desarrollados fundamentalmente es la obesidad. El exceso de grasas saturadas en la dieta
se ha relacionado con el desarrollo de aterosclerosis, as como con
diversos tipos de cncer como el de mama, tero, colon, prstata y
pulmn. Adems, la obesidad per se, sobre todo la de perfil
andrognico, se ha relacionado con los estados de resistencia a la
insulina e hiperinsulinismo.
Tambin son importantes otros factores tales como la ingesta de
alcohol, que si bien en dosis excesivas puede desencadenar miocardiopatas o enfermedades hepticas, en dosis bajas se ha relacionado
con menor riesgo de cardiopata isqumica.
La edad avanzada y el sexo masculino son los principales marcadores de riesgo para el desarrollo de aterosclerosis. Obsrvese que se
define marcador de riesgo cuando no puede evitarse, mientras que
factor de riesgo cuando ste es evitable.
Pregunta 3.- R: 2
La intoxicacin crnica por plomo se denomina saturnismo y tiene que ver fundamentalmente con la ingesta en determinados grupos
profesionales de riesgo tales como empleados de la industria siderrgica. Inicialmente la ingestin de plomo da lugar a una fase asintomtica
que se define como contaminacin (por eso es incorrecta la opcin
2) y posteriormente, a una fase clnicamente evidente que se denomina intoxicacin. Esta fase de intoxicacin puede manifestarse
clnicamente de 4 formas distintas:
Sndrome neuropsiquitrico: se caracteriza por la aparicin de
irritabilidad, convulsiones, neuritis perifricas o hipertensin intracraneal.
Sndrome gastrointestinal: tambin muy inespecfico, puede presentarse como estreimiento, dolor abdominal tipo clico, mal
sabor de boca y por un tpico ribete gingival (de Burton).
Sndrome hematolgico: es muy caracterstica la anemia microctica e hipocroma con punteado basfilo en los hemates.
Sndrome urinario: aparecen aminoaciduria o glucosuria. Puede
detectarse de manera muy tpica coproporfirina III en orina, si bien
no es patognomnico, ya que puede aparecer en la intoxicacin
por talio (matarratas).
Adems de estos cuadros sindrmicos, la intoxicacin por plomo se
manifiesta de forma muy caracterstica como clico saturnino, que es
un dolor abdominal tipo clico que no cede a las medidas habituales,
pero s de forma muy caracterstica a la administracin de una sal clcica. Esta forma de intoxicacin suele aparecer en los adultos. Tambin es
caracterstica del saturnismo la lesin del nervio radial.
M exico A rgentina
C hile U ruguay

Seguimiento a distancia

Pregunta 4.- R: 4
Las enfermedades transmisibles son las producidas por un agente
vivo. La cadena epidemiolgica se compone de reservorio-mecanismo de transmisin-husped susceptible. Desde el punto de vista epidemiolgico, la forma de reservorio de mayor importancia son los
portadores. stos pueden ser agudos si eliminan el germen durante un
periodo inferior a 6 meses; crnicos, cuando lo eliminan durante ms
de 6 meses; y permanentes, cuando se eliminan de por vida. Las
enfermedades transmisibles tienen habitualmente dos periodos clnicos claramente diferenciados. Uno inicial, caracterizado por la aparicin de clnica inespecfica y que se denomina prdromos (respuesta
2), y otro posterior, en el que ya son evidentes los signos y sntomas
propios de la enfermedad. Segn sea esta interrelacin entre el husped y el microorganismo, se definen los trminos de:
Contagiosidad: es la capacidad de propagacin. Se mide segn la
tasa de ataque (que no es una tasa realmente sino la incidencia
acumulada de una enfermedad infecciosa aguda).
Infectividad: es la capacidad del microorganismo de instalarse en
el husped. Puede estacionarse aqu o evolucionar produciendo
enfermedad.
Patogenicidad: es la capacidad de invasin. Por tanto, es la capacidad de producir enfermedad. Cuando esta capacidad es muy grande se dice que la infeccin es muy virulenta.
En el caso de la tuberculosis, el diagnstico de infectividad se realizara mediante la prueba de la tuberculina, mientras que el de enfermedad tuberculosa por el aislamiento del germen.
Pregunta 5.- R: 3
Las enfermedades trasmisibles pueden presentarse de dos maneras
muy distintas. En primer lugar, pueden manifestarse como caso aislado
o espordico, sin repercusin desde el punto de vista epidemiolgico,
o de forma ms o menos constante (prevalente) en la poblacin. Segn
este nivel de prevalencia se habla de hipoendemia si afecta a menos de
un 10% de la poblacin; mesoendemia, si afecta hasta un 50%;
hiperendemia, si hasta un 75%, y holoendemia si ms del 75% (respuesta 3). Adems de estos casos prevalentes pueden aparecer en la
poblacin casos nuevos de enfermedad (casos incidentes) que dan
lugar a epidemias de gran impacto en la poblacin. En Espaa, el
sistema de vigilancia y recogida de datos ms importante es el sistema
EDO o enfermedad de declaracin obligatoria. En este sistema EDO se
calcula el ndice epidmico, que es la razn entre la incidencia de la
semana en curso y la incidencia de la mediana del quinquenio anterior para la misma semana. Si el brote no alcanza el grado de epidemia
pero la prevalencia es mayor de la habitual, se dice que se est ante
una endoepidemia. El trmino pandemia hace referencia no a la prevalencia de la enfermedad sino a su extensin, refirindose a la enfermedad que est presente a nivel mundial.

Pregunta 5. Presentacin de las enfermedades transmisibles.


Tipos de endemia.
1234562347

89
73429763757

4 625647

1232

456

6 625647

789
2

56 6

46625647

2

6 

 625647

722



Pregunta 6.- R: 2
Si se nos pregunta cul es la definicin correcta de portador, lo
definiremos como la persona que, sin presentar signos o sntomas de
la infeccin, es capaz de eliminar microorganismos (MO). Corresponde a la definicin de portador sano. Estos portadores se llaman agudos
cuando eliminan grmenes durante un tiempo inferior a 6 meses,
crnicos si lo eliminan durante ms tiempo y permanentes cuando lo
eliminan de por vida. Sin embargo, hay circunstancias que definen
tipos concretos de portadores. Definimos como portador paradjico

CTO Medicina C/ Nez de Balboa, 115 28006 MADRID (Espaa) Tfno.: (91) 782 43 32 / Fax: (91) 782 43 27
E-mail: secretaria@ctomedicina.com; iberocto@ctomedicina.com WEB: www.ctomedicina.com; www.iberocto.com

ED Pg. 1

o pseudoportador al que elimina MO que no son patgenos, como


por ejemplo, la transmisin que se produce en la infancia de N.
lactamica (que confiere inmunidad cruzada mediada por el complemento frente a N. meningitidis). El portador precoz o incubacionario
corresponde al enunciado de la pregunta. En este caso los MO se
eliminan antes de la fase clnica, como ocurre en el caso de la VHA,
que se elimina en heces antes de que aparezca la fase ictrica de la
infeccin. Por el contrario, un portador convaleciente lo es cuando
elimina el MO despus del acm de la enfermedad y antes de que sta
se haya resuelto completamente. El portador pasivo es un eslabn
entre el verdadero portador que elimina MO y el husped susceptible. Esto ocurre, por ejemplo, cuando al explorar un nio con diarrea
por rotavirus, por descuido no nos lavamos las manos y a continuacin se explora a un nio susceptible de infeccin por este virus.
Pregunta 7.- R: 1
Se define cuarentena como el aislamiento que se produce de una
persona sana que ha estado expuesta al contagio de una enfermedad.
Las enfermedades cuarentenables son el clera, la fiebre amarilla (ambos durante 5 das) y la peste (6 das). El aislamiento consiste en
apartar de la comunidad a un enfermo para que as no transmita la
enfermedad a la poblacin sana. Segn la forma de transmisin de la
enfermedad se habla de un aislamiento estricto (el mayor nivel de
aislamiento), respiratorio, entrico, de piel y mucosas y protector. Todas las formas de aislamiento tienen en comn el lavado de manos
como barrera en la transmisin de la enfermedad. En el aislamiento
respiratorio no es necesario el empleo de guantes, aunque s mascarilla. Son ejemplos de este tipo de aislamiento la meningitis o la tuberculosis. En el aislamiento entrico es especialmente importante el
empleo de guantes, siendo ejemplo de este tipo de aislamiento la
infeccin por rotavirus o el clera. El aislamiento de piel y mucosas se
hace en enfermedades como las estafilococias o el zoster diseminado.
El aislamiento protector hace referencia al aislamiento del sujeto
inmunosuprimido, para evitar que ste se contagie a partir de
microorganismos ubicuos en el medio ambiente.
Pregunta 8.- R: 3
El nmero de casos de SIDA diagnosticados en 2001 ha sido de
2300. Esto supone un descenso en el nmero de casos con respecto
al ao anterior y refleja una tendencia iniciada en 1995. El hecho
determinante de que esto se est produciendo es la aparicin de los
retrovirales. Por tanto, si tenemos en cuenta que el SIDA es la consecuencia ltima de la infeccin del VIH, la reduccin del nmero de
casos de SIDA no implica obligatoriamente la disminucin del nmero de infecciones por VIH. En Espaa, tradicionalmente el grupo de
riesgo ms castigado por el SIDA lo han sido los UDVP. Este grupo sigue
siendo el ms numeroso de forma global seguido del grupo de
heterosexuales. No obstante, los casos de SIDA en mujeres en el grupo
de heterosexuales se acercan mucho al de los UDVP.
Pese a los controles de laboratorio, todava existe riesgo de SIDA a
partir de hemoderivados. Se estima que 1 de cada 400000 concentrados de hemates de un banco de sangre estn infectados por el VIH.
Esta forma de transmisin del SIDA supuso el ao 2001 el 1% de los
casos.
Aunque el riesgo de transmisin madre-hijo ha disminuido con las
nuevas terapias y con el diagnstico prenatal, ste todava existe, suponiendo tambin el 1% de los casos diagnosticados en el 2001.
Pregunta 9.- R: 2
Si tenemos en cuenta que el SIDA es la ltima etapa de la infeccin
por VIH y que sta se produce a los 10 aos aproximadamente de la
infeccin, podremos entender cul ha sido la evolucin del SIDA en
Espaa en los ltimos 15-20 aos. La mayor incidencia de infeccin
por VIH en nuestro pas tuvo lugar 3-4 aos despus de que fueran
descritos los primeros casos en los homosexuales de Estados Unidos.
El pico de incidencia de VIH ocurri en 1985, por lo que el pico de
casos nuevos de SIDA y de mortalidad por SIDA se produjo en 1995.
Desde que la poblacin tiene ms conciencia de la enfermedad y por
la llegada de la nuevas terapias, el nmero de infecciones nuevas ha
descendido, pero no as la prevalencia de VIH, que se ha mantenido

Pg. 2 ED

M exico A rgentina
C hile U ruguay

MEDICINA PREVENTIVA

Preparacin Examen de Seleccin 05/06 1 Vuelta

en constante aumento, alcanzando en 2000 la cifra de 140000 personas vivas con VIH y libres de SIDA en Espaa.
Si tenemos en cuenta que el SIDA es la causa ms frecuente de
muerte en muchas zonas de Espaa entre los 25 y los 35 aos, esta
enfermedad supone una de las causas ms importantes de aos de
vida perdidos, de forma que en estas edades la expectativa de vida no
ha aumentado en los ltimos 25-30 aos.
Pregunta 10.- R: 5
La forma de transmisin sexual es la ms frecuente a nivel mundial
(pases con patrn tipo I de distribucin de la OMS) y se est convirtiendo en una forma cada vez menos frecuente de contagio en los
pases desarrollados debido a la concienciacin de los grupos tradicionales de riesgo (ADVP, homosexuales).
Cualquier forma de relacin sexual en la que estn involucrados el
semen o las secreciones vaginales es contagiosa. El riesgo heterosexual
es mayor del hombre a la mujer que viceversa, lo que tiene que ver
con la cantidad de vehculo infeccioso. El sexo oral es una forma ms
difcil de transmisin porque la ptialina (maltasa) salival tiene la capacidad de inactivar el VIH. Por el contrario, la relacin anal es de ms
fcil contagio debido a que la mucosa del recto no est preparada
para el traumatismo del coito.
Como en cualquier forma de transmisin, sta es ms frecuente en
fases avanzadas de la enfermedad, debido a una mayor viremia. Por
otra parte la presencia de lesiones genitales inflamatorias o lceras
(recuerda que la causa ms frecuente de lcera genital en nuestro
medio es el VHS) aumenta el riesgo de transmisin sexual.
Los mtodos barrera son los nicos que pueden disminuir la transmisin sexual. Adems de los preservativos es aconsejable el empleo
del espermicida nanoxinol 9, que inhibe el VIH. No obstante este
espermicida nunca deber emplearse como mtodo profilctico sin
condn.
Pregunta 11.- R: 2
La transmisin materno-infantil se ha reducido en pases como
Espaa (1% de los casos nuevos de SIDA en 2001), pero supone una
forma muy frecuente de contagio en los pases desarrollados. El mximo riesgo de infeccin corresponde a fases de viremia maternas elevadas como son el embarazo en etapas tardas de la enfermedad o la
primoinfeccin que coincide con la gestacin. Menos de un 20% de
los recin nacidos vivos de madre con VIH se infectan y, con la introduccin del tratamiento antirretroviral, la modificacin de la prctica
obsttrica (parto por cesrea) y la supresin de la lactancia materna
(pases desarrollados), este riesgo ha descendido hasta un 5%. Si bien
la transmisin puede realizarse en cualquier momento del embarazo,
es en el momento del parto cuando tiene lugar el mayor riesgo, por lo
que se recomienda la cesrea cuando sea factible realizarla. La seguridad y eficacia de la mayora de los antirretrovirales es desconocida,
salvo para el AZT. Se recomienda el tratamiento con este frmaco a la
madre desde el segundo trimestre de embarazo y hasta 6 semanas en
el neonato. Sin embargo parece lgico pensar que la asociacin de
frmacos pueda ser ms eficaz. Otros antirretrovirales que parecen
eficaces y seguros son el ddI, el 3TC y la nepiravina.
Pregunta 12.- R: 5
Se estima que hasta un 5% de la poblacin mundial es portadora
del virus de la hepatitis B, manteniendo una endemicidad ms o menos constante segn los distintos pases. Esta elevada prevalencia ocasiona unos 2 millones de muertes anuales causadas por el VHB.
Puede hablarse de 3 niveles de endemicidad. Obsrvese que, cuanto
mayor es la prevalencia de la enfermedad, tambin es mayor la prevalencia de anticuerpos protectores anti-HBs (respuesta 5 incorrecta).
Mientras que en los pases de prevalencia baja (<1%) la presencia de
anticuerpos protectores es aproximadamente del 5%, en los pases de
prevalencia media (aprox. 5%), los niveles serolgicos de proteccin
estn entre un 20-50%. Asia y frica son las reas con mayor prevalencia (10-20%) con unos niveles de anti-HBs en torno a un 70-95%.
En Espaa, la incidencia es de 12000 casos al ao. De ellas slo el
10% evoluciona a la cronicidad y un 2% desarrolla hepatocarcinoma
(recuerda que slo evolucionan a la cronicidad las hepatitis de trans-

CTO Medicina C/ Nez de Balboa, 115 28006 MADRID (Espaa) Tfno.: (91) 782 43 32 / Fax: (91) 782 43 27
E-mail: secretaria@ctomedicina.com; iberocto@ctomedicina.com WEB: www.ctomedicina.com; www.iberocto.com

Comentarios TEST

Seguimiento a distancia

MEDICINA PREVENTIVA

Preparacin Examen de Seleccin 05/06 1 Vuelta


misin no enteral). En Espaa, la evolucin a la cronicidad no es tan
elevada como en Asia o frica debido a la forma de transmisin predominante de la enfermedad. En las reas con mayor prevalencia la
transmisin es fundamentalmente de madre a hijo, lo que se acompaa de desarrollo de cronicidad, hepatopata crnica y cncer. Sin
embargo, en Espaa el grupo de mayor riesgo lo suponen los contagios por va parenteral que suelen producirse por tanto en adultos y se
acompaan habitualmente de menor riesgo de cronicidad.

Comentarios TEST

Pregunta 13.- R: 5
La vacuna que se emplea en la actualidad frente a la hepatitis B es
la de segunda generacin o de ADN recombinante. La pauta habitual de aplicacin consiste en 3 dosis (recuerda que slo suelen
precisar una dosis las vacunas de microorganismos vivos atenuados).
La pauta es 0-1-6 meses con una elevada eficacia. Un 90% de los
adultos sanos y un 95% de nios y jvenes alcanzan niveles protectores (anti-HBs mayor de 10 mUI/ml). La edad mayor de 40 aos, los
fumadores y los obesos presentan una respuesta a la vacuna ms
reducida. Asimismo, en inmunosuprimidos, la proteccin es alcanzada por un 40-60% de los sujetos. No se conoce el hecho de por
qu algunos sujetos sanos no responden a la vacunacin. En stos se
recomienda volver a vacunar con la pauta completa, y si de nuevo
no alcanzan ttulos protectores, no se vuelve a intentar la vacunacin.
La vacuna frente a VHB no slo es una vacuna eficaz, sino que
adems induce la formacin de anticuerpos neutralizantes a largo
plazo. Aun con ttulos bajos, el contacto con el VHB produce una
respuesta inmunolgica eficaz incluso trascurridos ms de 10 aos.
Debes recordar que como la vacuna no es del virus completo sino
de una fraccin (HBs-Ag), el nico anticuerpo que puede aparecer al
hacer la serologa frente a VHB en un vacunado es el anti-HBs. Por otra
parte, tambin es importante que se recuerde que la vacuna es una
profilaxis pre-exposicin, por lo que no est indicada cuando un
sujeto presente ya en plasma cualquier tipo de antgeno o anticuerpo.
Pregunta 14.- R: 5
La profilaxis frente a la hepatitis B puede ser:
1) Profilaxis pre-exposicin: Vacuna.
La pauta de referencia es 0-1-6 meses. En Espaa forma parte del
calendario vacunal a los 0-2-6 meses de vida o las 3 dosis a los 11
aos en aquellos nios que no estuvieran vacunados previamente.
Cuando se precise una inmunizacin rpida se pueden aproximar
las 3 dosis, asegurando la inmunidad mediante una dosis al ao (01-2-12). El hecho de interrumpir el programa de vacunacin no
implica reanudarlo nuevamente sino continuarlo donde se abandon (recuerda que esto es lo habitual para cualquier tipo de
vacuna).
No se recomienda determinacin de ttulos frente a VHB de forma
sistemtica salvo a personas especialmente en riesgo (trabajadores
sanitarios, requerimiento de politrasfusin, en lista de trasplante) o
el inmunosuprimidos (VIH, trasplantados, hemodilisis).
2) Profilaxis post-exposicin: Vacuna + Inmunoglobulina.
Exposicin perinatal: IGHB especfica hiperinmune en las 12 primeras horas y vacunacin (la proteccin para el RN con esta pauta
es superior al 95%).
Exposicin en el lactante que convive con infectado de forma
aguda: IGHB + vacuna lo antes posible.
Contacto sexual: vacuna + IGHB en las 2 primeras semanas.
Contacto percutneo. Si el sujeto expuesto no est vacunado o si
lo est pero desconoce su serologa o es negativa: vacuna + IGHB
lo antes posible. Si presenta ttulos protectores de anticuerpos no es
necesario ni vacuna ni IGHB.
Pregunta 15.- R: 2
El virus VHA es un picornavirus (ARN virus) monocatenario de transmisin fecal-oral. Recuerda que los virus de la hepatitis que se transmiten de esta forma (VHA y VHE) nunca cronifican por lo que no
pueden desarrollar cirrosis y hepatocarcinoma. Si no tienen la capacidad de cronificar esto quiere decir que la opcin 2 es incorrecta: no
hay portadores asintomticos y la infeccin siempre se va a transmitir
M exico A rgentina
C hile U ruguay

Seguimiento a distancia

desde un sujeto enfermo. Es importante recordar que el periodo de


mxima contagiosidad no ocurre en la fase ictrica sino en el periodo
preclnico (el aislamiento por tanto en esta enfermedad no tiene ningn sentido ya que cuando es clnicamente relevante el sujeto ya no
es contagioso).
Al transmitirse fecal-oral, es muy importante el agua como vehculo
contaminante por lo que hay que recordar que mediante la ebullicin, la cloracin y con la radiacin U.V. el virus se inactiva.
Los grupos de riesgo son distintos segn la prevalencia de la enfermedad. En los pases donde la hepatitis A es endmica el grupo fundamental de riesgo lo suponen los nios que frecuentemente presentan
formas subclnicas de la enfermedad, mientras que en los pases desarrollados en los que la prevalencia es baja, el grupo fundamental de
riesgo lo suponen los individuos que emigran a un pas de elevada
prevalencia.
Pregunta 16.- R: 3
El virus de la hepatitis que es causa ms frecuentemente de hepatitis postrasfusional es el virus VHC. El riesgo de transmisin percutnea de este virus es muy elevada, por lo que ante un contacto percutneo accidental del personal sanitario el riesgo de transmisin de
este virus es mayor que para el VHB o el VIH. La nica forma de
transmisin conocida es la percutnea por lo que este virus es muy
prevalente en Espaa dentro del grupo de los ADVP. Sin embargo,
hay que recordar que el grupo ms importante de sujetos que presentan infeccin por el VHC lo constituyen sujetos que no pertenecen a ninguno de los grupos de riesgo tpicos. No se conoce ninguna
medida de profilaxis ni pre-exposicin ni post-exposicin, por lo
que cualquier personal sanitario debe considerar a todo paciente
como potencial transmisor de la enfermedad. Recurdese que los
virus A y E son de transmisin fecal-oral, por lo que no pueden
considerarse como agentes etiolgicos de la hepatitis postrasfusional.
Pregunta 17.- R: 5
La tuberculosis (TBC) es una enfermedad crnica infecciosa muy
prevalente a nivel mundial. Se produce por el contacto del husped
con el M. tuberculosis o M. bovis. Este microorganismo es un bacilo
cido-alcohol resistente que produce el granuloma con necrosis
caseosa como lesin histopatolgica especfica. Recuerda que la mayora de las micobacterias son muy sensibles al medio ambiente por
lo que la tuberculosis o la lepra se transmiten fundamentalmente en
situaciones de hacinamiento. El bacilo tuberculoso (BT) es sensible a
la luz UV por lo que ventilar el hogar donde vive un tuberculoso es
una forma muy eficaz de disminuir el riesgo de transmisin a otros
convivientes. La transmisin es de persona a persona con contacto
estrecho y se produce a travs de las gotas de Pflugge que se producen
durante la tos. La primoinfeccin suele cursar de forma asintomtica,
quedando el BT acantonado en los macrfagos. El riesgo mayor de
infeccin lo presentan los nios y los adultos jvenes que desarrollan
la clnica en forma de tuberculosis secundaria habitualmente 2-3
aos tras la primoinfeccin. Como el contagio es fundamentalmente
va respiratoria las formas de mayor riesgo de transmisin son las que
afectan a la va area (larngea, traqueal, endobronquial o de cavitacin pulmonar extensa).
Pregunta 18.- R: 3
La prueba tuberculnica (PT) consiste en la intradermorreaccin de
Mantoux que, como su nombre indica, consiste en la inoculacin
intradrmica (la inoculacin subcutnea puede inducir a error en la
interpretacin del resultado) del PPD (antgeno purificado). La lectura
del resultado debe hacerse a las 48 horas y no antes ya que la inmunidad frente a la TBC es celular (mediada por linfocitos y por tanto ms
lenta). La inoculacin se realiza en el antebrazo del sujeto y lo que se
mide es el dimetro transverso de la ppula (se mide el dimetro palpable y no el eritema). Si la PT es positiva, esto slo indica que el sujeto ha
estado en contacto con el bacilo (infeccin), lo que permitir diagnosticar a los sujetos que son candidatos a la quimioprofilaxis. Un Mantoux
positivo por tanto slo indica infeccin pero no nos informa de si
adems el sujeto es un individuo enfermo. Para esto ltimo debera
realizarse una placa de trax cuando se est realizando bsqueda de

CTO Medicina C/ Nez de Balboa, 115 28006 MADRID (Espaa) Tfno.: (91) 782 43 32 / Fax: (91) 782 43 27
E-mail: secretaria@ctomedicina.com; iberocto@ctomedicina.com WEB: www.ctomedicina.com; www.iberocto.com

ED Pg. 3

casos secundarios o, de forma definitiva, mediante el cultivo. Recuerda


que la baciloscopia puede ser positiva en sujetos que siguen tratamiento tuberculosttico sin que ello implique resistencias tuberculosas ya
que esos bacilos no suelen ser viables.
Pregunta 19.- R: 4
La quimioprofilaxis (QP) antituberculosa se realiza por definicin
en sujetos no enfermos.
Esta quimioprofilaxis puede ser de dos tipos:
QP primaria: se realiza en sujetos que an no tienen evidencia de
infeccin tuberculosa (Mantoux negativo).
QP secundaria: se realiza en sujetos con evidencia de infeccin
(Mantoux positivo) pero sin enfermedad.
En ambos casos la quimioprofilaxis se realiza en sujetos con especial riesgo de enfermedad (habitualmente convivientes estrechos de
un sujeto bacilfero), aunque tambin estara indicada en aquellos
sujetos con la inmunidad comprometida como son un sujeto que va
a ser sometido a trasplante o bien un individuo que va a recibir
corticoides (diagnstico de LES). No influye en el estado inmunitario
la realizacin de una histerectoma por lo que es en este supuesto en
el que no estara indicado realizar quimioprofilaxis.
El frmaco empleado es la isoniacida. El efecto secundario ms
importante de este frmaco es la hepatitis txica que guarda estrecha
relacin con la edad de los sujetos (a ms edad, mayor riesgo), por lo
que la QP se restringe a partir de los 35 aos salvo en sujetos VIH. Hay
que recordar que el mayor riesgo de infeccin y enfermedad tuberculosa lo tienen nios y adultos jvenes, siendo en este grupo en el
que la hepatoxicidad es menor por isoniacida, por lo que estos sujetos son los que ms se benefician de la QP.
Pregunta 20.- R: 2
Si un adulto con edad avanzada tiene un Mantoux (M) que resulta
negativo, se deber realizar una nueva prueba tuberculnica a la semana. Esto no indica seroconversin reciente.

Pregunta 20. Causas de FP y FN en el Mantoux.


1234546754898
54

123456745897

58548
854
48647
7
74
8
12
7
4
8745897
4
5
8
4
78884
4

2
22577

 !
12"44
#$
87%55897
54!
12&7%5587

854584
4874!
1234546 298
54

'45
45874
57

87886(
12)6568
44744*
88744
#$
57
4%545897

4!
12
7474

+85897
4
8845897
,6
6


-./
22474!
123&".0& 1!
12 8744
87%5587
854

458474!
12345674587
57
86
8
4764!
12)484
876764(
5858
#
!
127%4
72854*
4588*
876%8587584
74
597854*
2255897
854
4*
3
58644*
,6464!
1255837
7458
#
475847!
'45
45874
57
4
656874
#
57
4
357854(
121457487
874564!
12 86587
8745644!
121747
54654!
12&7#55897

5
4747!
12&7#55897
484
%674!
1265
87+!
Desde que el bacilo tuberculoso entra en contacto con el husped
y ste desarrolla la respuesta inmune va a pasar un intervalo de aproximadamente 8 semanas, por lo que hasta que no transcurra este tiempo
Pg. 4 ED

M exico A rgentina
C hile U ruguay

MEDICINA PREVENTIVA

Preparacin Examen de Seleccin 05/06 1 Vuelta

un sujeto infectado va a presentar un M negativo (perodo ventana). Si,


como en el caso de la pregunta, el primer M fue negativo y a la semana
se positiviza, hay que pensar como primera posibilidad en un efecto
Booster, que consiste en que, en los pacientes de edad avanzada, la
inmunidad frente al bacilo TBC se encuentra atenuada y el primer M
sirve para reactivar esta memoria inmunolgica, de forma que ser positivo el segundo M si el sujeto estuvo infectado en el pasado. Si ambos
M realizados en el plazo de una semana son negativos, se descarta
efecto Booster y concluimos definitivamente que el sujeto nunca estuvo
infectado y ya no habra que realizar nuevos M. Si el 2 M hubiera
resultado positivo, indicara infeccin y habra que realizar una placa
de trax para descartar enfermedad tuberculosa.
Tambin podra pensarse (menos frecuente) que el primer M que
se realiz a este individuo fue un falso negativo debido a la tcnica
errnea de aplicacin o a que el PPD fuera inactivo por exposicin
prolongada a la luz.
Pregunta 21.- R: 1
Esta es una situacin de hacinamiento donde, por tanto, el riesgo
de TBC es elevado. Los nios y adultos jvenes son los que presentan
principal riesgo de infeccin y enfermedad, y debe ser en este grupo,
por tanto, en el que ms importante sea la quimioprofilaxis (QP).
Adems, es el grupo donde menos probable es la toxicidad por isoniacida, y por tanto el que se beneficia ms de la QP.
En el caso de nios y adultos jvenes (habitualmente en los menores
de 15 aos) hay que realizar un Mantoux (M). Si el M es negativo, no
quiere decir que el nio no est infectado, sino que puede encontrarse
en el periodo ventana (hay que recordar que la respuesta inmune tarda
unas 8 semanas desde el momento de la infeccin). Es tan alto el riesgo
de infeccin en los nios y tan bajo el riesgo de hepatotoxicidad que se
inicia la QP incluso con M negativo (QP 1). Para confirmar definitivamente si el nio est infectado, habr que repetir el M a las 8 semanas
(habr terminado el periodo ventana). Si el M es negativo en esta segunda ocasin, se suspender la QP 1 (nunca estuvo infectado). Si el segundo M es positivo, entonces habr que descartar la enfermedad mediante
placa de trax. Si la placa es negativa, entonces el nio est infectado,
pero no enfermo. En este ltimo caso, el nio deber finalizar la QP que
inici y prolongarla hasta los 6 meses (QP 2).
Pregunta 22.- R: 4
Si el sujeto es mayor de 15 aos, entonces lo primero que habr que
realizar es un Mantoux (como en cualquier contacto). Si ste es negativo, no tendremos la certeza de no infeccin (periodo ventana), por lo
que habr que repetir el Mantoux a las 8 semanas. La diferencia con el
grupo de nios es que en los mayores de 15 aos no est indicada la
QP 1, por lo que el sujeto no recibe ningn frmaco hasta la realizacin del 2 Mantoux. Si ste 2 es negativo se suspende el seguimiento
(nunca habr estado en contacto con el bacilo). Sin embargo, si el 2 es
positivo lo primero que habr que hacer es una placa de trax para
descartar la presencia de enfermedad. Si finalmente nos encontramos
con 2 Mantoux positivo y placa negativa est indicada la QP 2, que se
realiza como la QP 1 con isoniacida. La QP 2 se prolonga durante 6
meses salvo en dos situaciones: infectados por VIH y portadores de
lesiones fibrticas extensas, en los cuales la QP se prolonga durante 12
meses. La opcin 2 es falsa porque si el Mantoux es positivo en cualquier situacin, antes de iniciar QP 2 o tratamiento hay que descartar
la presencia de enfermedad.
Pregunta 23.- R: 5
En los mayores de 35 aos se restringen las quimioprofilaxis (QP),
salvo en sujetos de especial riesgo como los VIH. En este grupo de
edad, la hepatoxicidad por isoniacida es mayor cuanto ms avanzada
sea la edad del sujeto. El manejo apropiado comienza por realizar un
Mantoux. Si ste resulta negativo, hay que realizar como siempre otro
a las 8 semanas. Si el 2 es negativo se acabar el estudio. Sin embargo,
si el segundo es positivo se descartar la presencia de la enfermedad
mediante una placa de trax. Si la placa resulta negativa y el Mantoux
positivo, no se iniciar ningn tipo de QP, slo se realizar seguimiento
clnico. Podra realizarse QP 2 en mayores de 35 aos si el riesgo de
infeccin es muy grande, como ocurre en el caso de convivientes de
sujetos bacilferos.

CTO Medicina C/ Nez de Balboa, 115 28006 MADRID (Espaa) Tfno.: (91) 782 43 32 / Fax: (91) 782 43 27
E-mail: secretaria@ctomedicina.com; iberocto@ctomedicina.com WEB: www.ctomedicina.com; www.iberocto.com

Comentarios TEST

Seguimiento a distancia

MEDICINA PREVENTIVA

Preparacin Examen de Seleccin 05/06 1 Vuelta


En ancianos, alcohlicos y hepatpatas el riesgo de hepatitis txica
por isoniacida es muy alto. En estos grupos se recomienda la realizacin seriada de analticas. Debera interrumpirse la QP si se
quintuplican los valores basales de transaminasas o aparece clnica de
hepatitis. Si no se pertenece a ninguno de los grupos antes sealados,
se recomienda nicamente una analtica al mes del inicio de la QP y
un seguimiento clnico.

Comentarios TEST

Pregunta 24.- R: 3
En los infectados mayores de 65 aos y en los vacunados con BCG,
la respuesta inmunitaria frente al bacilo tuberculoso puede estar atenuada, de forma que un primer Mantoux puede resultar negativo,
aunque el sujeto hubiera estado expuesto a la infeccin. Para evitar
este falso negativo de la prueba debe hacerse un nuevo Mantoux a la
semana del primero. En este caso, si el Mantoux a la semana es positivo, indicar que el sujeto ha estado en contacto con el bacilo. A esta
reactivacin de la inmunidad frente a la TBC se le llama efecto Booster. Si el 2 Mantoux es tambin negativo entonces hay que descartar
que no se est en periodo ventana por lo que se realizar un nuevo
Mantoux a las 8 semanas del primero. Si este ltimo Mantoux resulta
negativo, se habr finalizado el estudio. Si fuera positivo, habra que
descartar enfermedad tuberculosa mediante placa de trax. Como
vemos, el nico grupo en el que es necesario realizar hasta 3 Mantoux
para concluir el estudio es en el de los ancianos y vacunados con
BCG.
El efecto Booster no es la nica causa de falso negativo de la prueba. Tambin pueden verse falsos negativos en recin nacidos, VIH,
vacunados con triple vrica, enfermedades graves como el cncer, la
sarcoidosis o la deplecin proteica, tuberculosis diseminadas como la
TBC miliar o la que afecta a serosas.
Pregunta 25.- R: 4
El agente etiolgico ms frecuente de meningitis en nuestro medio es
el meningococo. Es la causa ms frecuente en los dos grupos de edad
en los que habitualmente se producen las meningitis: nios pequeos
y nios y adultos jvenes. Sin embargo, rara vez es la causa de las
meningitis del recin nacido (aparecidas en el primer mes de vida:
Listeria, Streptococcus agalactie y E. coli). En los nios hasta los 4 aos de
edad, despus del meningococo el segundo agente etiolgico es el H.
influenzae, que se est haciendo cada vez ms prevalente en nuestro
medio. H influenzae es la causa ms frecuente de meningitis en ese
grupo de edad en EEUU. En los nios mayores y en adultos jvenes, la
causa ms frecuente en nuestro medio sigue siendo el meningococo,
pero el Haemophilus ya no ocupa el segundo lugar, sino que pasa a ser
el neumococo. En mayores de 30 aos, el meningococo pasa a un
segundo lugar, quedando como agente etiolgico ms frecuente en ese
grupo de edad el neumococo. Algunos agentes etiolgicos son frecuentes en determinados grupos de riesgo. Por ejemplo, la Listeria vuelve a
ser una causa frecuente en ancianos y la Klebsiella es habitual causa de
meningitis en alcohlicos (recuerda que la Klebsiella es tambin la causa ms frecuente de neumona en alcohlicos).
Pregunta 26.- R: 1
La transmisin de la meningitis meningoccica se realiza a travs
de las secreciones respiratorias ms frecuentemente de un portador
asintomtico y ms rara vez de un enfermo. Los portadores suelen
ser resistentes a la infeccin porque en la infancia tuvieron en la
rinofaringe N. lactamica, que no es patgena e induce inmunidad
cruzada con otras neiserias mediada por el complemento. La
nasofaringe humana es el nico reservorio conocido, y dado que el
meningococo difcilmente sobrevive en el medio ambiente, para
que se produzca el contagio es necesario un contacto estrecho entre
el reservorio y el husped susceptible. Los contactos estrechos de los
sujetos portadores tienen un riesgo hasta 800 veces mayor de desarrollar la enfermedad (recuerda que el riesgo es an mucho mayor
para los sujetos que presentan dficit del factor final del complemento). El perodo de contagio es mximo en los 10 das posteriores
al contacto con el portador. El sujeto portador o el individuo enfermo dejan de ser contagiosos a las 24 horas de iniciar la
quimioprofilaxis o el tratamiento adecuado, teniendo en cuenta
M exico A rgentina
C hile U ruguay

Seguimiento a distancia

que las penicilinas, que forman parte del arsenal teraputico, pueden suprimir de forma temporal la Neisseria de la rinofaringe, pero
no la erradican. El perodo de incubacin de la enfermedad es de 3
a 4 das.
Pregunta 27.- R: 5
La enfermedad meningoccica est producida por N. meningitidis,
que contiene una cpsula de polisacridos que permite la divisin en
serogrupos (A, B y C los ms importantes). La enfermedad meningoccica tiene distribucin universal, presentndose habitualmente de
modo individual con aumento de las tasas de incidencia cada 10
aos aproximadamente. Los serogrupos A, B y C originan un 90% del
total de casos. El serogrupo B es el ms frecuente en Espaa y suele
originar casos espordicos. El C se manifiesta habitualmente con pequeos brotes y est creciendo su prevalencia en Espaa en los ltimos aos (por esto se ha introducido recientemente en el calendario
vacunal). El serogrupo A habitualmente produce epidemias y es el
ms frecuente en el llamado cinturn de la meningitis.
En Espaa se producen habitualmente unos 3 casos/100000 habs.
y ao, con una letalidad de un 6-10%, algo mayor en las que se
presentan en forma de sepsis fulminante y cuando aparecen en menores de 14 aos. Sin embargo, el ltimo brote epidmico ocurrido
en nuestro pas ha sido debido al serogrupo C, alcanzando incidencias de 6 casos/100000 habs/ao. con una tasa de letalidad especfica
para este serogrupo mayor que para el serogrupo B.
Pregunta 28.- R: 2
Si tenemos en cuenta que el serogrupo ms prevalente de N.
meningitidis en nuestro medio es el B, para el cual no existe vacuna ni
de polisacridos ni conjugada, entenderemos que en Espaa es esencial la quimioprofilaxis para evitar los contagios. Salvo que se demuestre que el brote que aparece en nuestro medio no est producido por
el serogrupo B, se debe administrar a los contactos estrechos
quimioprofilaxis. El frmaco de eleccin es la rifampicina, que se
administra durante 2 das. Alternativas son la ceftriaxona y las quinolonas, administradas en dosis nica. Cuando se sabe que el caso no
est producido por el serogrupo B, adems debe administrarse la
vacuna, ya que sta es ms eficaz que la quimioprofilaxis.
En la embarazada debe evitarse la rifampicina, ya que hay alternativas eficaces. No debe tampoco administrarse quinolonas (recuerda
que tampoco se deben dar a nios, porque en ambos casos deforman
el cartlago de crecimiento). En la embarazada, por tanto, se indica la
administracin de ceftriaxona i.m. en dosis nica.
Es importante recordar que la quimioprofilaxis debe realizarse en
los contactos estrechos, sean stos nios o adultos (a diferencia de la
quimioprofilaxis contra H. influenzae, que se realiza slo en nios).
Pregunta 29.- R: 2
Existen dos tipos de vacunas distintas frente al meningococo:
Vacuna de polisacridos: contiene antgenos de varios serogrupos,
entre los que estn el A y el C. No contiene, sin embargo, el serogrupo B, que es el ms frecuente en nuestro medio. Una desventaja
importante es que la capacidad de desarrollar respuesta inmune
en menores de 2 aos es nula y muy baja entre 2 y 4 aos (40%). En
adultos jvenes, sin embargo, el nivel protector se alcanza hasta en
un 90%.
Vacuna conjugada: la ventaja frente a la anterior es que es muy
inmungena incluso en menores de un ao, por eso es la vacuna
que se ha introducido en el calendario vacunal en la pauta 2-4-6
meses y 1 dosis nica en mayores de 1 ao no vacunados previamente. El inconveniente frente a la anterior es que slo protege
frente al serogrupo C. ste es un inconveniente relativo, ya que el
serogrupo C es el 2 ms frecuente en nuestro medio y la vacuna
de polisacridos tampoco es eficaz frente al serogrupo B.

Recuerda las indicaciones de vacunacin:


Dficit de fraccin final del complemento.
Asplenia.
Inmunodeficiencias humorales.
Contactos de casos debidos al serogrupo C.

CTO Medicina C/ Nez de Balboa, 115 28006 MADRID (Espaa) Tfno.: (91) 782 43 32 / Fax: (91) 782 43 27
E-mail: secretaria@ctomedicina.com; iberocto@ctomedicina.com WEB: www.ctomedicina.com; www.iberocto.com

ED Pg. 5

Pregunta 30.- R: 4
El ttanos est producido por C. tetani, que es un bacilo anaerobio
estricto Gram positivo, esporulado. Esta bacteria es ubicua en forma
de espora, resistiendo condiciones tan adversas como la ebullicin.
En contacto con un medio red-ox bajo (heridas sucias con
coinfecciones por otros grmenes), el bacilo toma su forma activa y es
capaz de liberar la toxina tetnica. Esta toxina migra por los axones
motores hasta llegar al asta lateral de la mdula, donde inhibe a la
neurona encargada de inhibir el tono motor (quitara el freno a las
neuronas motoras, de ah la clnica de espasticidad, rigidez y opisttonos). Como en Espaa la vacunacin frente al ttanos est incluida en
el calendario vacunal, la prevalencia de anticuerpos protectores es
superior al 99% en los menores de 14 aos. Sin embargo, en los pases
subdesarrollados donde la vacunacin no es universal, es frecuente el
ttanos neonatal, forma especialmente grave.
En Espaa se producen unos 0,1-0,2 casos/100000 habs/ao (en
descenso en los ltimos aos). Habitualmente ocurren en mayores de
60 aos porque se abandona la vacunacin recomendada cada 10
aos. En nuestro medio, lo ms frecuente es que no se recuerde el
antecedente de herida tetangena.
Pregunta 31.- R: 2
Aunque la eficacia de la vacuna frente al ttanos es casi del 100%, los
niveles de anticuerpos suelen disminuir con el paso del tiempo, por lo
que se recomienda la vacunacin cada 10 aos a partir de la ltima
dosis del calendario vacunal (14 aos). La vacunacin correcta consiste
en la administracin de 3 dosis de toxoide adsorvido en sal de aluminio
con dosis de toxoide diftrico Td. Si la vacunacin es incompleta, no es
necesario reanudar la vacunacin en todos los casos:
Si se recibieron 3 dosis (vacunacin completa), slo se recomienda
una nica dosis de recuerdo si las 3 dosis se recibieron hace ms
de 10 aos.
Si se recibieron menos de 3 dosis hace menos de 5 aos slo hay
que completar la vacunacin hasta alcanzar las 3 dosis.
Si se recibieron menos de 3 dosis hace ms de 5 aos debe administrarse toxoide hasta un total de 4 dosis.
Es importante recordar que, para evitar el ttanos neonatal, la medida
ms eficaz es vacunar a la mujer embarazada que no estuviera previamente inmunizada. En este caso se considera suficiente con dos dosis de
toxoide, debiendo administrarse la segunda dosis antes del parto.
Pregunta 32.- R: 4
Para contestar correctamente este tipo de preguntas, hay que valorar dos aspectos:
Cunto hace que se vacun?
De qu tipo de herida se trata?
Se considera que una herida es tetangena cuando sta es sucia
(heridas anfractuosas, contaminadas con polvo o tierra, por mordedura de animales...). Si la herida es sucia y el sujeto no est correctamente inmunizado o hace ms de 10 aos que recibi las 3 dosis,
deber administrarse, adems del toxoide, gammaglobulina especfica antitetnica (IGT). Por otra parte, si el sujeto recibi las 3 dosis hace
menos de 5 aos, no deber administrarse ni vacuna ni IGT.
La situacin particular de esta pregunta es un sujeto que ha pasado
la infeccin. Dado que la cantidad de toxoide que produce la enfermedad es muy pequea y que la toxina no entra a la circulacin
(migra por los axones de neurona motora), el haber pasado la enfermedad no asegura anticuerpos, por lo que este sujeto, teniendo en
cuenta que no est vacunado, deber recibir, adems de la pauta
vacunal (3 dosis de toxoide), IGT especfica.
Recuerda este esquema muy general:
Sujeto inmunizado (3 dosis hace menos de 5 aos): nada, independientemente de la herida.
Sujeto no inmunizado (resto de situaciones): asegurar inmunidad
mediante toxoide en heridas limpias y, adems, en heridas sucias,
asegurar inmunidad inmediata mediante IGT.

Pg. 6 ED

M exico A rgentina
C hile U ruguay

MEDICINA PREVENTIVA

Preparacin Examen de Seleccin 05/06 1 Vuelta

Pregunta 32. Profilaxis del ttanos.

Pregunta 33.- R: 3
La gripe es una enfermedad producida por un virus ARN monocatenario muy prevalente en la poblacin general, afectando a un 10%
de sta. Cursa en brotes anuales que se producen en los meses fros y
la transmisin es casi exclusivamente a partir del sujeto enfermo. Para
que se produzca el contagio, el contacto debe ser muy estrecho, ya
que el virus de la gripe es muy lbil en el medio ambiente. Un papel
muy importante en la morbi-mortalidad de los sujetos adultos es el
contagio a partir de los nios. En los meses de mayor contagiosidad
(meses fros), la gripe puede afectar incluso a un 40% de los nios. El
perodo de incubacin de la enfermedad es de 2 das de media,
empezando la clnica de forma abrupta sin prdromos.
Las glicoprotenas superficiales N (neuraminidasa) y H (hemaglutinina) estn en la envoltura lipdica y varan en la gripe tipo A, condicionando la inmunidad de la poblacin, de forma que la vacuna
frente a la gripe se hace teniendo en cuenta las cepas del ao anterior.
Esta variabilidad de las glicoprotenas no ocurre en la gripe tipo B, por
lo que la importancia desde el punto de vista epidemiolgico es menor (poca capacidad de producir infecciones). No obstante debe
recordarse que el mayor riesgo de la gripe B es la capacidad de producir sndrome de Reye en nios que toman salicilatos.
Pregunta 34.- R: 5
La vacuna frente a la gripe que se emplea de manera habitual es una
vacuna trivalente inactiva que contiene dos subtipos de vacuna A que
circularon la temporada anterior (H1N1 y H3N2) y frente al grupo B.
Cuando las cepas del ao en curso coinciden con las de la vacuna,
la eficacia para sujetos menores de 65 aos inmunocompetentes es
de un 70-80%. Esta eficacia es sensiblemente inferior en ancianos,
trasplantados, hemodilisis, y en todos aquellos en los que la respuesta inmune est alterada. Los ttulos de anticuerpos protectores aparecen a los 10-14 das y protegen hasta 6 meses tras la vacunacin. El
momento de la vacunacin es en los meses de otoo en nuestro
medio. Con vacunas que contienen la hemaglutinina porcina se han
descrito casos de Guillain-Barr.
Recuerda las indicaciones:
Mayores de 65 aos.
Portadores de enfermedades crnicas (no se incluyen hipertensos).
Nios mayores de 6 meses que toman salicilatos de forma prolongada.
Mujeres que se encuentran en el tercer trimestre en la poca gripal.
Y tambin las contraindicaciones:
Nios menores de 6 meses.
Anafilaxia a las protenas del huevo.
Pregunta 35.- R: 1
Las vacunas pueden clasificarse segn el tipo de antgeno en vivas
(atenuadas) y muertas (inactivas). Las vacunas vivas estn formadas por
microorganismos viables, pero que presentan prcticamente nula capacidad patgena en sujetos inmunocompetentes. Como es una forma poco virulenta de un microorganismo vivo, la capacidad de gene-

CTO Medicina C/ Nez de Balboa, 115 28006 MADRID (Espaa) Tfno.: (91) 782 43 32 / Fax: (91) 782 43 27
E-mail: secretaria@ctomedicina.com; iberocto@ctomedicina.com WEB: www.ctomedicina.com; www.iberocto.com

Comentarios TEST

Seguimiento a distancia

MEDICINA PREVENTIVA

Preparacin Examen de Seleccin 05/06 1 Vuelta


rar respuesta inmune es similar o incluso mayor a la de la infeccin
natural. El nivel de inmunidad por tanto que se consigue con las
vacunas vivas atenuadas es muy bueno desde la primera dosis (nica
excepcin es la polio oral o tipo Sabin). Como son microorganismos
vivos, se pueden administrar va enteral, como la vacuna frente a la
polio Sabin, condicionando respuesta inmune tambin local (IgA
secretora). Precisamente en esta vacuna hay que vigilar que no existan
sujetos convivientes inmunocomprometidos, ya que pueden eliminarse virus con las heces que causen enfermedad en estos sujetos.
Sin embargo, las vacunas de microorganismos inactivos suelen precisar de varias dosis que se administran de forma percutnea. Como
no son microorganismos viables, la enfermedad no puede transmitirse
a sujetos convivientes inmunocomprometidos. Las vacunas muertas
pueden ser de microorganismos enteros como la antipertussis de clulas enteras, la polio Salk, la antigripal o la de VHA, o fracciones
como antihepatitis B, de toxoide como antidifteria o antitetnica, de
polisacridos como la del meningococo, neumococo o Haemophilus
o acelulares como la antipertussis acelular.

Comentarios TEST

Pregunta 36.- R: 4
Esta pregunta hace referencia a las reacciones adversas producidas
por las vacunas. Habitualmente se produce una reaccin inespecfica
en el lugar de inoculacin en las primeras 48 horas de la vacunacin,
y que consiste en edema, eritema, dolor o induracin. Algunas formas
de reaccin local son ms especficas:
Td (vacunas adsorbidas en aluminio): ndulo cutneo que puede
persistir durante semanas.
Varicela: vesculas localizadas.
BCG: adenopatas locorregionales o adenitis supurada.
Rubola: Exantema generalizado polimorfo que aparece a las dos
semanas de la vacunacin, en el 10% de los sujetos.
Rubola: Artralgias en el 10-20% de las mujeres jvenes vacunadas, 2-3 semanas tras la vacunacin.
Rubola o Sarampin: 20% de los vacunados, adenopatas generalizadas 2-3 semanas tras la vacunacin.
Antitfica-paratfica: afectacin severa del estado general.
Sarampin y parotiditis: encefalitis.
Triple vrica: Convulsiones no febriles, 15 das tras la administracin de la vacuna.
DTP: hipotona y disminucin del estado de alerta.
Ttanos: polineuropata 14 das siguientes a la vacunacin.
Pertussis celular: Sndrome de llanto persistente horas tras la vacunacin.
Pregunta 37.- R: 1
La inmunidad pasiva consiste en la administracin de anticuerpos
preformados. La nica ventaja sobre la inmunizacin activa (vacunas)
es la inmediatez de los anticuerpos. La desventaja fundamental es que
la inmunidad es menos duradera y menos fuerte.
Los sueros se dividen en homlogos o heterlogos. Los primeros
son los ms frecuentes y son los que se obtienen de individuos humanos. Los heterlogos sin embargo se obtienen del suero de animales.
De estos ltimos slo se emplea el suero antibotulismo. Los sueros
heterlogos pueden producir enfermedad del suero (reaccin de
depsito de complejos antgeno-anticuerpo).
Otra clasificacin de los sueros es la que los divide en especficos o
hiperinmunes e inespecficos. Dentro de los primeros estn el de la
hepatitis B, el de varicela, el de ttanos o el de la difteria. Sin embargo,
el suero frente a VHA es inespecfico.
La inmunoglobulina frente al VHA se emplea en la profilaxis preexposicin. Est indicada cuando se necesita inmunidad de forma
inmediata (menos de 2 semanas), ya que de otra forma debera aplicarse la vacuna frente al VHA, que es ms eficaz a largo plazo. Tambin se emplea en la profilaxis post-exposicin. En este caso, la eficacia es alta (90%), previniendo la enfermedad o atenundola.
Pregunta 38.- R: 5
La inmunoglobulina hiperinmune frente al VHB se aplica junto
con la vacunacin (administracin simultnea, aunque en distinta
localizacin) en la profilaxis postexposicin.
M exico A rgentina
C hile U ruguay

Seguimiento a distancia

Actualmente se dispone de una vacuna frente a VHA (virus muertos) que es muy eficaz y se aplica si se va a estar en riesgo de infeccin
(por ejemplo, un viaje a un pas endmico). Si el plazo de tiempo en
el que se va a producir este riesgo es inferior a 2 semanas o ya se ha
producido el contacto, la inmunoglobulina inespecfica frente a VHA
es tambin muy eficaz.
Como la vacuna del sarampin est incluida en el calendario
vacunal, la necesidad de administracin de inmunoglobulina polivalente es escasa. No obstante est indicada la profilaxis pasiva ante un
contacto (en un plazo inferior a 6 das) si:
Nio menor de 1 ao y contacto estrecho.
Inmunodeprimidos no vacunados.
VIH, independientemente del estado vacunal.
Embarazadas no vacunadas.
Tras haber recibido la profilaxis pasiva hay que vacunar, transcurridas 6 semanas desde la aplicacin de la gammaglobulina. Si no han
pasado 3 das desde el contacto, puede realizarse vacunacin como
nica profilaxis postexposicin.
La profilaxis frente a la VVZ se realiza con gammaglobulina hiperinmune. Si recordamos que la vacuna no entra dentro del calendario
vacunal y que est formada por virus vivos, entenderemos que es
relativamente frecuente la administracin de profilaxis pasiva frente al
VVZ. Sus indicaciones son:
Sujetos inmunodeprimidos con contacto estrecho.
RN con varicela materna en el periparto.
Prematuros <28 semanas, independientemente de si la madre tuvo
varicela.
Prematuros >28 semanas, si la madre no tuvo varicela.
Pregunta 39.- R: 3
La vacuna conjugada DTP est introducida en el calendario vacunal. Como est formada a partir de microorganismos vivos, puede
administrarse incluso en sujetos inmunocomprometidos.
En el calendario vacunal infantil se administran 3 dosis en el primer
ao de vida con intervalos de 8 semanas entre cada dosis. Es importante recordar que la vacuna Pertussis puede ser celular (DTP) o acelular
(DTPa). La forma celular est contraindicada en mayores de 1 ao de
edad. Es necesaria una cuarta dosis de esta vacuna que se suele administrar a los 18 meses (DTPa) y otra ms a los 4-6 aos (DTPa /DT). El
calendario vacunal se completa con una 6 dosis a los 14 aos (Td)
con una menor dosis de toxoide diftrico. Recientemente se ha introducido tambin el componente Hib frente a H. influenzae, que se
administra en 4 dosis junto con la vacuna DTP. Si recordamos que H.
influenzae suele afectar exclusivamente a nios pequeos, entenderemos que no es necesaria ninguna dosis por encima de los 18 meses.
Tambin es nueva la incorporacin de la vacuna C conjugada frente a
meningococo C. Se administra en el calendario vacunal a los 2-4-6
meses de edad. Si el nio no fue vacunado en el primer ao de vida, se
administra una dosis nica en nios mayores.
Como ocurre con la mayora de las vacunas, si se ha interrumpido
el calendario vacunal no hace falta reiniciarlo, sino completarlo.
Pregunta 40.- R: 4
La vacuna TV se administra en 2 dosis en el calendario vacunal. En
menores de 1 ao no est contraindicada, si bien no se aconseja su
administracin porque la vacuna podra inactivarse por la presencia
de anticuerpos maternos, siendo este el motivo por el cual se aconseja
la primera dosis a los 15 meses. Esta primera dosis podra adelantarse
si fuese necesario por motivos epidemiolgicos (recuerda que en
menores de 1 ao con contacto muy estrecho se podra administrar
inmunoglobulina). Si se adelanta la primera dosis, como posiblemente se inactive por los anticuerpos maternos, debe administrarse siempre en cualquier caso la dosis de los 15 meses. La segunda dosis se
administra a los 4 aos para asegurar la eficacia de la vacunacin. En
aquellos nios que no han recibido esta segunda dosis, se les administra a los 11 aos.
Como esta vacuna est compuesta por virus vivos est contraindicada
en situaciones de inmunosupresin como el embarazo (en este caso se
podra administrar inmunoglobulina polivalente frente al ttanos).

CTO Medicina C/ Nez de Balboa, 115 28006 MADRID (Espaa) Tfno.: (91) 782 43 32 / Fax: (91) 782 43 27
E-mail: secretaria@ctomedicina.com; iberocto@ctomedicina.com WEB: www.ctomedicina.com; www.iberocto.com

ED Pg. 7

MEDICINA PREVENTIVA

Seguimiento a distancia

Preparacin Examen de Seleccin 05/06 1 Vuelta

Pregunta 41.- R: 3
Si bien el calendario vacunal infantil se cumple en gran medida en
nuestro medio, no ocurre lo mismo con el calendario vacunal del
adulto. Las vacunas recomendadas en los adultos son: Td, TV, gripe y
neumoccica. La vacuna frente al ttanos debe continuarse desde la
ltima dosis del calendario vacunal (14 aos), cada 10 aos, de forma
indefinida. Precisamente en nuestro medio la forma ms frecuente de
ttanos es el ttanos en mayores de 65 aos porque en este grupo de
edad los pacientes no siguen ningn tipo de pauta de vacunacin. En
la mayora de los casos de ttanos en nuestro medio no est clara cul
fue la puerta de entrada para la toxina. La vacuna contiene una pequea dosis de toxoide diftrico.
Desde los 15 a los 65 aos se recomienda la vacunacin contra TV
en aquellos sujetos no inmunizados previamente. Esto es de especial
importancia en las mujeres en edad frtil no vacunadas con la TV, por
el riesgo de rubola fetal. La vacunacin frente a la rubola est
contraindicada si la mujer est embarazada.
En mayores de 65 aos se recomienda la vacunacin anual frente
a la gripe que protege a un 70% aproximadamente de los vacunados
y deben recibir tambin la vacunacin frente al neumococo, con una
eficacia muy similar a la de la gripe.
Pregunta 42.- R: 3
El calendario vacunal en el nio VIH es en todo idntico al del
nio sano, excepto porque se recomienda el empleo de la vacuna
parenteral tipo Salk en los nios VIH y en todos los sujetos que con l
conviven, en vez de la tipo Sabin. La vacuna TV, aunque de virus vivos,
s se aconseja en estos nios VIH porque el riego de contraer el sarampin y la mortalidad que presenta en estos pacientes es muy elevada.
Tambin se recomienda la vacunacin en estos nios frente a VVZ,
por el riesgo de desarrollar zster diseminado. Estas dos ltimas vacunas, por ser de virus vivos, no deben administrase si:
VVZ: no administrarse si CD4<25%.
TV: si CD4< 15%.
Debe recordarse tambin que si un sujeto VIH est expuesto a una
enfermedad para la que exista una forma de profilaxis pasiva eficaz,
sta debe realizarse independientemente del estado vacunal. Esto es
aplicable especialmente para el ttanos, el sarampin y la varicela.
No existe ningn problema por administrar a estos nios la vacuna
DTP (por ser de microorganismos inactivos), al igual que la vacuna

frente a VHB (fraccin antignica) o antineumoccica, antihaemophilus


o antimeningococo (polisacridos o conjugadas).
Pregunta 43.- R: 5
El cncer es una de las causas ms importante de mortalidad en los
pases desarrollados. La mitad de los tumores aparece en los pases
desarrollados y la otra mitad en los pases subdesarrollados. Sin embargo, los distintos tipos de tumor que afectan en una y otra localizacin son distintos. En los pases desarrollados son frecuentes los tumores de pulmn, colon y mama, mientras que en los subdesarrollados,
son ms frecuentes los de estmago, esfago y crvix. En algunos tipos
concretos de tumor est observndose un cambio significativo, de
forma que en los pases desarrollados ha disminuido claramente la
incidencia de cnceres de esfago y estmago, probablemente por la
relacin de estos tumores con el consumo de productos ahumados.
Otro cambio que se est observando es que el cncer de mama, que
es el tumor ms frecuente en la mujer en los pases desarrollados, est
siendo sobrepasado en las mujeres por el cncer de pulmn, lo que
tiene que ver en gran medida con el hbito tabquico en las mujeres.
El cncer de mama no obstante es en nuestro medio el tipo de neoplasia ms frecuente en las mujeres, suponiendo la principal causa de
aos de vida perdidos en este sexo.
Pregunta 44.- R: 5
El principal carcingeno ambiental es el humo del tabaco, que
constituye la principal causa de muerte en nuestro medio al ser un
factor etiolgico clave tanto en las enfermedades cardiovasculares
como en los tumores ms frecuentes. El tabaco se ha relacionado con
uno de cada tres tumores que se producen en el varn y en un 10%
de los que se producen en mujeres. Se han relacionado muchos tipos
de tumor con el tabaco. Aparte de todos los de la esfera ORL y el de
pulmn, tambin ha sido implicado en la gnesis de tumores de estmago y esfago, pncreas, pulmn, vejiga y crvix. Los cnceres de la
cavidad oral tambin se asocian muy frecuentemente al tabaquismo.
Si bien se considera que la forma menos perniciosa de tabaco es el
fumado en pipa, el cncer de labio se ha relacionado especialmente
con el tabaco en pipa. Cuanto mayor es el consumo de tabaco, mayor es el riesgo de cncer. Este riesgo existe tambin para los fumadores pasivos. Se ha estudiado especialmente el riesgo de cncer de
pulmn y los fumadores pasivos, encontrndose un 25% de riesgo
superior en este tipo de fumadores que en los controles no expuestos.
Hay que recordar que nunca es tarde para dejar de fumar. Si bien el
riesgo de cncer de pulmn no se iguala nunca al del no fumador,
este riesgo s se reduce a lo largo del tiempo, siendo muy parecido al
del no fumador a los 10 - 15 aos de abandonar el consumo de
tabaco.
Pregunta 45.- R: 3
Para que se produzca cncer de prstata es absolutamente necesa-

Pregunta 42. Recomendaciones para la vacunacin sistemtica en los pacientes infectados por VIH.
123456789
8675 7947  88  767
48 4

2 363 86

 363 86

122324

#
2
 7 "

&
 '984
&(
79
+
7

7 

56

56
56
56
56
56
56


56

56
56
56
56
56
56


Pg. 8 ED

M exico A rgentina
C hile U ruguay

5789
7  74
 
9 47
9

7
97 47
78 47  
7 
 47
9 47  9 ! 47 
7

   "

$ 797  
97  9747 7  7 4 7 
77 
%

5789
7  74
 
9 47
9

7
5789
7  74
 
9 47
9

7
9747  7  7   
 47 
97 
177 4 7  4  
) 47 *  % ) 47 744
9747 7  7 4   
 47 
97  7  
77 
%

 7  794
4 7 9 47  9 7 7  
%


CTO Medicina C/ Nez de Balboa, 115 28006 MADRID (Espaa) Tfno.: (91) 782 43 32 / Fax: (91) 782 43 27
E-mail: secretaria@ctomedicina.com; iberocto@ctomedicina.com WEB: www.ctomedicina.com; www.iberocto.com

Comentarios TEST

La vacuna TV no debe administrarse en alrgicos a las protenas del


huevo, si bien para estos sujetos existe la posibilidad de vacunacin
con la presentacin cultivada en clulas diploides humanas.
Un hecho particular de la TV es la hipoergia tuberculnica (respuesta disminuida al Mantoux), que puede dar lugar a un falso negativo de la prueba tuberculnica, por lo que si se ha vacunado frente a
la TV, debe posponerse la realizacin del Mantoux varias semanas.

MEDICINA PREVENTIVA

Comentarios TEST

Preparacin Examen de Seleccin 05/06 1 Vuelta

Seguimiento a distancia

ria la presencia de andrgenos. Basta recordar que una forma de


tratamiento de este tumor es la terapia antiandrognica.
Tambin est asociado al cncer de mama la presencia de estrgenos, por lo que slo uno de cada cien tumores aparece en el varn. El
embarazo que se produce a edades tempranas (menores de 30 aos),
se ha relacionado con menor riesgo de cncer de mama. No se ha
encontrado relacin entre el tipo de lactancia y este tumor.
La terapia sustitutiva empleada en la menopausia a base de estrgenos se ha relacionado de forma clara con la aparicin de cncer de
endometrio, por lo que esta forma de teraputica no debe realizarse si
no se asocian progestgenos.
Para el cncer de ovario se puede decir que favorecen este cncer
todos aquellos factores que se relacionan con la ovulacin. Tendran
por tanto efecto protector tanto el embarazo como el empleo de
anticonceptivos orales, mientras que la nuliparidad, la menarquia
precoz y la menopausia tarda favoreceran la aparicin de este tipo
de tumor. Hay que recordar otra asociacin muy tpica y es el consumo por parte de la madre durante la gestacin de dietilestilbestrol es
causa de carcinoma de clulas claras vaginal en las hijas.

producen en menores de 65 aos. El segundo marcador de riesgo


ms importante es el sexo masculino.
La hiperlipidemia es un factor de riesgo muy importante. El principal predictor de riesgo de cardiopata isqumica en los varones de
mediana edad es la hipercolesterolemia. Con la edad se observa un
aumento de los niveles plasmticos de colesterol y triglicridos que
parece mediado por la obesidad. Si bien se evidencia un aumento de
la cardiopata isqumica prematura con unos niveles de colesterol
total superiores a 200 mg/dl, este riesgo es especialmente elevado
cuando se superan los 240 mg/dl. La forma de colesterol que se ha
relacionado con el desarrollo de cardiopata isqumica es la LDL-c y
la VLDL-c. Sin embargo, diversos estudios han relacionado con la
cardiopata isqumica, ms que los niveles elevados de estas dos formas de colesterol, los niveles disminuidos de HDL-c. Los factores que
producen elevacin del HDL-c son el ejercicio fsico, el sexo femenino y la dieta hipolipemiante, mientras que se relacionan con la disminucin, la obesidad, el sedentarismo y el consumo de tabaco. Se
recomienda que el aporte de caloras de la dieta que procede de los
lpidos no supere el 30% del total de caloras.

Pregunta 46.- R: 1
Aunque se ha tratado de buscar alguna forma de deteccin precoz
para el cncer de pulmn, no puede decirse que ninguna prueba por
s sola haya conseguido disminuir la incidencia de este cncer. Ni
siquiera la realizacin de placa de trax o la citologa de esputo, aun
en pacientes fumadores mayores de 50 aos, han conseguido este
beneficio. Si bien el riesgo de cncer nunca alcanzar el del no fumador, este riesgo s se disminuye sensiblemente a partir de los 10 aos de
la supresin del tabaco.
El antgeno prosttico especfico (PSA) es el marcador ms sensible
que hay en el cncer de prstata, pero tiene el inconveniente de que
es muy poco especfico, ya que tambin se encuentra muy elevado
en patologas benignas como la hiperplasia prosttica benigna o en las
prostatitis. Otro marcador que se ha utilizado en el screening de este
tumor ha sido la fosfatasa cida, si bien es poco sensible puesto que
cuando est elevado, suele indicar ya afectacin extraganglionar. Aunque el tacto rectal tampoco es eficaz como prueba de screening, se
recomienda su realizacin fundamentalmente en varones, al ser una
maniobra sencilla y poco costosa.
Si bien la citologa de Papanicolau s es eficaz como screening del
cncer de crvix, no lo es para el adenocarcinoma de endometrio.

Pregunta 49.- R: 4
El principal marcador de riesgo modificable para las enfermedades
cardiovasculares es el tabaco. Este factor de riesgo es causa por s solo
de ms del 20% de las muertes por cardiopata isqumica. El riesgo de
cardiopata isqumica puede llegar a ser tres o cuatro veces mayor en
los fumadores que en los no fumadores, existiendo una clara relacin
dosis-respuesta. Si el abandono del tabaco se asocia a disminucin
del riesgo de cncer de pulmn a los 10-15 aos de dejar el tabaco,
el beneficio es mucho ms rpido para la cardiopata isqumica, ya
que este riesgo aumentado desaparece a los 2-3 aos de dejar el
tabaco. Hay que recordar adems que el tabaco se asocia a un descenso de las cifras de HDL-c. No se asocia, sin embargo, el tabaquismo
con un aumento en las cifras de tensin arterial. El sexo femenino
parece proteger de la accin deletrea del tabaquismo, si bien la
asociacin de tabaquismo en mujeres mayores de 35 aos con la
ingesta de anticonceptivos se asocia a mayor riesgo de cardiopata
isqumica. Tambin el tabaco es un factor de riesgo para la aparicin
de enfermedades cerebrovasculares y para el desarrollo de arteriopata perifrica bien oclusiva, bien aneurismtica.

Pregunta 47.- R: 5
Las enfermedades cardiovasculares (CV) son la principal causa de
mortalidad tanto en los pases desarrollados como en los subdesarrollados, por encima de los 45 aos de edad. En Espaa, el 45% de las
muertes se producen por enfermedades CV, manifestndose como
cardiopata isqumica, accidente cerebrovascular y arteriopata perifrica. En los ltimos aos se est evidenciando una disminucin de la
mortalidad por esta causa tras el aumento importante que se observ
en los aos 70, aunque la repercusin de esta patologa sigue siendo
dramtica, ya que hasta un tercio de las muertes que se producen
ocurren en menores de 65 aos (muertes prematuras).
Los factores ms importantes en el desarrollo de las enfermedades
CV son el sexo masculino, la edad (principal marcador de riesgo), la
hiperlipemia (tanto hipercolesterolemia como hipertrigliceridemia),
la diabetes (insulinodependiente o no), la hipertensin y el tabaquismo.
De las tres formas de EV comentadas (cardiopata isqumica, accidente cerebrovascular y arteriopata perifrica), la ms prevalente es
la cardiopata isqumica, si bien la asociacin de ms de una de las
anteriores es frecuente. Lo ms habitual es que las EV se manifiesten
como cardiopata isqumica, suponiendo adems la forma de presentacin que se asocia a una mayor mortalidad.

Pregunta 50.- R: 4
La prevencin de las enfermedades cardiovasculares se realiza
mediante el control de los factores de riesgo. El primer paso en el
tratamiento de las hiperlipidemias es la dieta. Es recomendable una
dieta que contenga menos de un 30% de caloras provenientes de las
grasas. Tambin es importante el tipo de grasa consumida, ya que las
grasas saturadas producen un aumento de la LDL-c, mientras que lo
contrario ocurre con las grasas monoinsaturadas. Asimismo, el consumo moderado de alcohol tambin se ha relacionado con disminucin del riesgo de mortalidad por cardiopata isqumica. Es importante conseguir el control del peso, ya que la obesidad (fundamentalmente la central o andrgina) se relaciona con hipercolesterolemia,
hipertrigliceridemia y con diabetes y resistencia a la insulina, mientras
que el ejercicio fsico aerbico produce un aumento de los niveles de
HDL-c. Si con la dieta no se consigue disminuir los niveles de colesterol
srico por debajo de 200 mg/dl, debe asociarse un frmaco
hipolipemiante. Para hacer el diagnstico de las hiperlipemias no se
aconseja en nuestro medio su deteccin como prueba de screening,
sino la bsqueda oportunista aprovechando la extraccin por otros
motivos. Para su determinacin no es necesario que el sujeto est en
ayunas. El estudio de las hiperlipidemias est especialmente indicado
en los varones jvenes con historia personal o familiar de cardiopata
isqumica.

Pregunta 48.- R: 1
El principal marcador de riesgo para el desarrollo de enfermedades cardiovasculares es la edad, de forma que son tanto ms prevalentes cuanto mayor sea la edad de los individuos, si bien hay que recordar que hasta un tercio de las muertes por cardiopata isqumica se
M exico A rgentina
C hile U ruguay

CTO Medicina C/ Nez de Balboa, 115 28006 MADRID (Espaa) Tfno.: (91) 782 43 32 / Fax: (91) 782 43 27
E-mail: secretaria@ctomedicina.com; iberocto@ctomedicina.com WEB: www.ctomedicina.com; www.iberocto.com

ED Pg. 9

HEMATOLOGA

Preparacin Examen de Seleccin 05/06 1 Vuelta


1.

1)
2)
3)
4)
5)
2.

Preguntas TEST

3)
4)
5)

10.

11.

Pancitopenia.
Presencia de blastos incrementados en M.O.
Reticulocitopenia.
Hemorragias.
Infecciones recidivantes.

Infeccin por micobacterias atpicas en SIDA.


Hemoglobinuria paroxstica nocturna.
Dficit de vitamina B12.
Mielodisplasia.
Ferropenia.

En los enfermos con anemia aplsica, usted NO esperara


encontrar uno de los siguientes signos de la exploracin fsica:
M exico A rgentina
C hile U ruguay

Saturnismo.
Ferropenia.
Microesferocitosis hereditaria.
Anemia sideroblstica.
Talasemias.

Seale la respuesta FALSA respecto al metabolismo del hierro:


1)
2)
3)
4)
5)

13.

Tuberculosis.
Enfermedad de Gaucher.
Timoma.
Mielofibrosis agnognica.
Carcinoma de prstata.

El incremento en la CHCM eritrocitaria es un dato caracterstico de:


1)
2)
3)
4)
5)

12.

La mejora tras la administracin de globulina antitimocito (GAT) ocurre en las primeras semanas de tratamiento.
Asociando ciclosporina a GAT, se aumenta el nmero de
respuestas, pero no la supervivencia.
Los esteroides en dosis altas pueden ser beneficiosos, pero
no se utilizan aisladamente.
El principal efecto del GAT a largo plazo es la aparicin de
mielodisplasia.
El riesgo de rechazo del TMO es muy inferior en los
pacientes que no han recibido previamente transfusiones
sanguneas.

En el frotis sanguneo de un enfermo con anemia y trombopenia se aprecian clulas en lgrima, normoblastos y clulas
blancas inmaduras. Cul de las siguientes causas le parece
la MENOS probable?:
1)
2)
3)
4)
5)

En el diagnstico diferencial de la pancitopenia, cabran las


siguientes posibilidades, con la EXCEPCIN de:
1)
2)
3)
4)
5)

8.

2)

Talasemias.
Mieloptisis.
Hemlisis traumtica.
Mielodisplasia.
Hipoesplenismo.

En la aplasia de mdula sea, NO debe existir:


1)
2)
3)
4)
5)

7.

Tratamiento quimioteraputico.
Deficiencia de folato.
Hemorragia crnica.
Mielodisplasia.
Alcoholismo.

Los esquistocitos son caractersticos de:


1)
2)
3)
4)
5)

6.

Aplasia.
Mieloptisis.
Anemia de enfermedad crnica.
Hemlisis congnita.
Hemlisis adquirida.

Palidez.
Hemorragias mucocutneas.
Esplenomegalia.
Fiebre.
Ausencia de signos inflamatorios, a pesar de infeccin
activa.

En referencia al tratamiento de la aplasia de mdula sea, NO


es correcto:
1)

Por hipotiroidismo.
Talasemias.
Ferropnica.
Sideroblstica hereditaria.
Saturnismo.

Cul de las siguientes circunstancias NO es causa de macrocitosis eritrocitaria?:


1)
2)
3)
4)
5)

5.

9.

De las siguientes enfermedades que producen anemia, cul


es la ms frecuente como anemia normoctica?:
1)
2)
3)
4)
5)

4.

Aplasia.
Sndrome mielodisplsico.
Hemorragia aguda.
Ferropenia.
Sideroblstica.

Cul de las siguientes anemias NO suele ser microctica?:


1)
2)
3)
4)
5)

3.

1)
2)
3)
4)
5)

Indique cul de las siguientes anemias cursa con elevacin de


reticulocitos en sangre perifrica:

Seguimiento a distancia

La mayor parte del hierro del organismo se encuentra en


los depsitos.
La mayor parte del hierro necesario para la eritropoyesis
basal procede de la destruccin de los hemates viejos.
La absorcin intestinal es superior en forma de hierro hem
de la dieta.
Las prdidas de hierro en la mujer suelen ser el doble que
en el hombre.
Las situaciones de eritropoyesis ineficaz incrementan la
absorcin intestinal.

NO ocurre en la anemia ferropnica:


1)
2)
3)

Bajo VCM.
Baja saturacin de transferrina.
Baja concentracin de transferrina.

CTO Medicina C/ Nez de Balboa, 115 28006 MADRID (Espaa) Tfno.: (91) 782 43 32 / Fax: (91) 782 43 27
E-mail: secretaria@ctomedicina.com; iberocto@ctomedicina.com WEB: www.ctomedicina.com; www.iberocto.com

HM Pg. 1

4)
5)
14.

15.

16.

2)

22.

Sideremia.
Saturacin de transferrina.
Concentracin de transferrina.
VCM disminuido.
HCM.

23.

La produccin de eritropoyetina es inadecuada para el


grado de anemia.
El interfern inhibe la eritropoyesis.
El nmero de sideroblastos medulares est aumentado.
El grado de anemia no suele ser intenso.
La saturacin de transferrina suele estar menos descendida que en la ferropenia.

24.

Ferropenia.
Anemia sideroblstica.
Dficit de folato.
Dficit de B12.
Sndrome de Imerslund.

NO es propio de la hemopoyesis megaloblstica la presencia


de:
1)
2)
3)
4)
5)

Precursores eritroides gigantes.


Metamielocitos gigantes.
Macroovalocitos.
Indice de Herbert <3.
Neutrfilos hipersegmentados.

25.

1)

En la deficiencia de cido flico, NO se encuentra:


1)

5)

Pg. 2 HM

M exico A rgentina
C hile U ruguay

La herencia suele ser autosmica dominante.


Puede no manifestarse hasta la edad adulta.
La prueba de Coombs es positiva.
La autohemlisis disminuye al aadir glucosa.
Los hemates tienen una menor superficie.

Acerca de la deficiencia de glucosa- 6-P deshidrogenasa, una


de las siguientes respuestas NO es correcta:

2)
3)
4)

Hiperplasia de mdula sea.

Esplenomegalia.
Inmunohemlisis.
Microangioptica.
Hemoglobinuria paroxstica nocturna.
Sepsis clostridial.

Respecto a la esferocitosis hereditaria, es FALSO:


1)
2)
3)
4)
5)

26.

Coluria.
Esplenomegalia.
Ictericia.
lceras maleolares.
Litiasis biliar.

Una de las siguientes anemias hemolticas NO tiene causa


extracorpuscular:
1)
2)
3)
4)
5)

La presencia de anemia con incremento de sideremia y


saturacin de transferrina debe hacer pensar en:

Elevacin de LDH srica.


Elevacin de bilirrubina indirecta en suero.
Descenso de haptoglobina srica libre.
Hemoglobinuria.
Reticulocitosis.

Entre los hallazgos clnicos que son caractersticos de las


anemias hemolticas estn los siguientes, EXCEPTO:
1)
2)
3)
4)
5)

En relacin a la anemia asociada a enfermedades crnicas, es


INCORRECTO:

Son ms especficos que los anticuerpos anti-clula parietal.


Su ausencia permite excluir el diagnstico de anemia
perniciosa.
Su importancia patognica es incierta.
Su positividad puede aparecer en diferentes enfermedades autoinmunes.
Pueden verse en algunos casos de mieloma.

En la diferenciacin entre las hemlisis intravasculares y


extravasculares, el parmetro ms fiable es:
1)
2)
3)
4)
5)

Cul de los siguientes parmetros diferencia mejor ferropenia de anemia de enfermedad crnica?:

1)
2)
3)
4)
5)

20.

1)

5)

Nmero de hemates.
Valor de hemoglobina.
Sideremia.
Transferrina srica.
Ferritina srica.

Incremento de LDH srica.


Incremento de bilirrubina srica.
Incremento de homocistena srica.
Incremento de metilmalnico srico.

Respecto a los anticuerpos anti-factor intrnseco, seale la


opcin INCORRECTA:

1)
2)
3)
4)
5)

2)
3)
4)
5)

19.

21.

3)
4)

1)

18.

Saturacin de transferrina.
Ferritina.
Concentracin de transferrina.
Reticulocitos.
VCM.

El tratamiento con hierro en la anemia ferropnica debe


mantenerse hasta la normalizacin de:

1)
2)
3)
4)
5)
17.

2)
3)
4)
5)

Baja HCM.
Baja CHCM.

Cul es el primer parmetro de laboratorio que se altera en


la ferropenia?:
1)
2)
3)
4)
5)

HEMATOLOGA

Preparacin Examen de Seleccin 05/06 1 Vuelta

Es la causa ms frecuente de anemia hemoltica enzimoptica.


La herencia es ligada al X.
Puede ser desencadenada por sulfamidas.
La dosificacin enzimtica correcta debe hacerse en
reticulocitosis.
El favismo es una variedad etiolgica.

CTO Medicina C/ Nez de Balboa, 115 28006 MADRID (Espaa) Tfno.: (91) 782 43 32 / Fax: (91) 782 43 27
E-mail: secretaria@ctomedicina.com; iberocto@ctomedicina.com WEB: www.ctomedicina.com; www.iberocto.com

Preguntas TEST

Seguimiento a distancia

HEMATOLOGA

Preparacin Examen de Seleccin 05/06 1 Vuelta


27.

Qu es INCORRECTO respecto a la anemia de Cooley?:


1)
2)
3)
4)
5)

28.

2)
3)
4)
5)

Preguntas TEST

36.

La activacin del complemento ocurre sobre todo por IgM


y causa hemlisis intravascular.
La hemlisis extravascular tiene lugar por clulas con
receptores Fc de IgG.
El Coombs directo es prueba inequvoca de hemlisis
inmune.
La prueba de Coombs indirecta no siempre es positiva.
La prueba de Coombs directa siempre es positiva.

38.

Casi siempre es por IgM.


La forma idioptica puede ser monoclonal.
Los Ac estn dirigidos frente a Ag I/i.
Cursa con fenmeno de Raynaud.
El inicio de la hemlisis es intravascular.

La asociacin de anemia con frecuentes esquistocitos en el


frotis y trombopenia es tpica de las siguientes enfermedades,
EXCEPTO:
1)
2)
3)

Sndrome de Evans.
Prpura trombopnica trombtica.
Sndrome hemoltico-urmico.
M exico A rgentina
C hile U ruguay

Anomalas en cariotipo.
Presencia de sideroblastos en anillo en mdula.
Grado de citopenias en sangre perifrica.
Presencia de ms de un 5% de blastos en sangre.
Edad avanzada.

El tratamiento ms utilizado para los pacientes con mielodisplasias es:


1)
2)
3)
4)
5)

40.

Panmielopata clonal.
Eritropoyesis ineficaz.
Incremento de clulas sanguneas en sangre perifrica.
Metamorfosis blstica.
Curso inicial indolente.

Entre los factores pronsticos desfavorables de los sndromes mielodisplsicos aparecen todos los siguientes, EXCEPTO:
1)
2)
3)
4)
5)

39.

Hierro.
Esteroides.
Piridoxina.
Transfusiones.
Globulina antitimocito.

Cul de las siguientes caractersticas NO es propia de


sndrome mielodisplsico?:
1)
2)
3)
4)
5)

Es ms frecuente en el sexo femenino.


Puede asociarse a trombopenia.
La especificidad de Ac es frente a Ag Rh.
Es de predominio intravascular.
Siempre hay que descartar como causas LES y LLC-B.

Prueba de Ham.
Prueba de lisis con sacarosa.
Citometra de flujo sangunea.
Estudio de mdula sea.
Cariotipo de precursores hemopoyticos.

En el tratamiento de la hemoglobinuria paroxstica nocturna,


es til todo lo siguiente, con EXCEPCIN de:
1)
2)
3)
4)
5)

37.

Trombocitosis.
Preleucemia.
Hemlisis intravascular.
Trombosis venosa.
Episodios vasooclusivos dolorosos.

La prueba ms sensible y especfica para el diagnstico de


hemoglobinuria paroxstica nocturna es:
1)
2)
3)
4)
5)

Una respuesta es FALSA respecto a la enfermedad por aglutininas fras:


1)
2)
3)
4)
5)

33.

Se produce por polimerizacin y precipitacin de Hb.


Se debe realizar esplenectoma.
Hay isostenuria.
Cursa con lceras maleolares.
Debe hacerse profilaxis de infeccin neumoccica.

Una de las siguientes respuestas, respecto a la anemia por


autoanticuerpos calientes, es INCORRECTA:
1)
2)
3)
4)
5)

32.

35.

Qu es FALSO sobre las anemias inmunohemolticas?:


1)

31.

Anemia.
Esplenomegalia.
Malformaciones seas.
Hemocromatosis.
Microcitosis con presencia de nmero normal o aumentado de hemates.

Coagulacin intravascular diseminada.


Sndrome HELLP.

Cul de las siguientes NO es componente caracterstico de


la hemoglobinuria paroxstica nocturna?:
1)
2)
3)
4)
5)

Qu es FALSO en la hemoglobinopata S?:


1)
2)
3)
4)
5)

30.

34.

El rasgo talasmico se caracteriza por:


1)
2)
3)
4)
5)

29.

Cursa con incremento de HbF.


Deben evitarse las transfusiones sanguneas.
Cursa con malformaciones seas por incremento de
eritropoyetina.
Se asocia a hemocromatosis secundaria.
El factor limitante del pronstico es la siderosis cardaca.

4)
5)

Seguimiento a distancia

Transfusiones sanguneas.
Trasplante de mdula sea.
Piridoxina.
Factores de crecimiento hemopoytico.
Quimioterapia.

Seale la respuesta que mejor caracteriza a un sndrome


mieloproliferativo crnico:
1)

Aumento celular en mdula sea y aumento de clulas


maduras en sangre.

CTO Medicina C/ Nez de Balboa, 115 28006 MADRID (Espaa) Tfno.: (91) 782 43 32 / Fax: (91) 782 43 27
E-mail: secretaria@ctomedicina.com; iberocto@ctomedicina.com WEB: www.ctomedicina.com; www.iberocto.com

HM Pg. 3

2)
3)
4)
5)
41.

2)
3)
4)
5)

45.

47.

1)
2)
3)
4)
5)
48.

2)
3)
4)
5)
50.

1)
2)

1)
2)
3)
4)
5)

5)
51.

Leucemia mieloide crnica.


Leucemia linftica crnica.
Mielodisplasia.
Mieloptisis.
Leucemia linftica aguda.

Uno de los siguientes hallazgos de la exploracin fsica es


fundamental para diferenciar a los enfermos con policitemia
vera de otras formas de poliglobulia:
1)

Pg. 4 HM

Cianosis.
M exico A rgentina
C hile U ruguay

52.

El pico de incidencia de la leucemia aguda linfoide es de


2 a 5 aos de edad.
La leucemia mieloide crnica es ms frecuente por encima de 65 aos.
La leucemia mieloide aguda es ms frecuente en adultos.
La leucemia linftica crnica es la leucemia ms frecuente
en ancianos.
La tricoleucemia predomina en adultos no ancianos.

El factor pronstico ms importante, respecto al aumento de


supervivencia en leucemia aguda mieloide, es:

1)
2)
3)
4)
5)

Clulas blsticas en sangre perifrica NO aparecen en:

Se define por la existencia de ms de un 30% de blastos


en sangre o mdula sea.
Es ms frecuentemente mieloblstica.
El trasplante de mdula sea consigue un elevado porcentaje de remisiones.
Puede presentarse sin fase crnica previa.
Puede asociarse a la existencia de cloromas.

Seale la respuesta INCORRECTA respecto a la incidencia de


las diversas formas de leucemia:

3)
4)

Leucemia mieloide aguda.


Leucemia linfoide aguda.
Leucemia mieloide crnica.
Anemia refractaria con exceso de blastos.
Leucemia aguda en tratamiento.

La eritromelalgia puede ser una manifestacin clnica.


En personas jvenes, puede no ocasionar sntomas.
Una forma de tratamiento es la esplenectoma.
Puede dar accidentes isqumicos transitorios cerebrales.
No siempre existe correlacin entre la clnica y el nmero
de plaquetas.

Respecto a la crisis blstica de la leucemia mieloide crnica,


es INCORRECTO:
1)

Leucemia mieloide crnica.


Leucemia linftica crnica.
Leucemia mieloide aguda.
Anemia refractaria con exceso de blastos.
Sepsis con reaccin leucoeritroblstica.

Incremento de hematocrito.
Incremento de hepatoesplenomegalia.
Mielofibrosis.
Metaplasia mieloide.
Transformacin leucmica.

Respecto a la trombocitemia esencial, es FALSO:


1)
2)
3)
4)
5)

49.

Congestin venosa retiniana.


Esplenomegalia.
Aspecto pletrico.
Hipertensin arterial.

Uno de los siguientes parmetros NO es caracterstico de la


fase de metamorfosis de la policitemia vera:

A qu enfermedad NO corresponde un hemograma con


2.500 leucocitos/mm3 con un 3% de blastos en sangre?:

1)
2)
3)
4)
5)
46.

2)
3)
4)
5)

Leucemia mieloide crnica.


Leucemia mieloide aguda.
Reaccin leucemoide en mieloptisis por cncer.
Anemia refractaria con exceso de blastos.
Anemia refractaria con exceso de blastos en transformacin.

A qu enfermedad corresponde este hemograma: leucocitos 5.500/mm3 con 90% de blastos en sangre?:
1)
2)
3)
4)
5)

44.

Aumento celular en mdula sea y aumento de clulas


maduras en sangre.
Aumento celular en mdula sea y aumento de clulas
inmaduras en sangre.
Aumento de blastos en mdula sea y descenso de clulas
maduras en sangre.
Aumento de blastos en mdula sea y aumento de clulas
maduras en sangre.
Aumento de clulas maduras en mdula sea y presencia
de blastos en sangre.

De qu enfermedad es un hemograma con 150.000 leucocitos por mm3 con un 1% de blastos en sangre perifrica?:
1)
2)
3)
4)
5)

43.

Aumento celular en mdula sea y aumento de clulas


inmaduras en sangre.
Aumento celular en mdula sea y descenso de clulas
maduras en sangre.
Aumento de porcentaje de blastos en mdula sea.
Aumento de porcentaje de blastos en medula sea y
sangre.

Seale la respuesta que mejor define a un sndrome mieloproliferativo agudo:


1)

42.

HEMATOLOGA

Preparacin Examen de Seleccin 05/06 1 Vuelta

Obtencin de remisin completa.


Edad superior a 60 aos.
Grado de leucocitosis.
Subtipo de la FAB.
Alteraciones citogenticas.

Seale cul de los siguientes NO es un factor pronstico


adverso en leucemias agudas linfoblsticas:
1)
2)
3)
4)
5)

Nios mayores de 10 aos.


Adultos mayores de 35 aos.
Inmunofenotipo comn.
Leucocitosis superior a 50.000/mm3.
Afectacin del sistema nervioso central.

CTO Medicina C/ Nez de Balboa, 115 28006 MADRID (Espaa) Tfno.: (91) 782 43 32 / Fax: (91) 782 43 27
E-mail: secretaria@ctomedicina.com; iberocto@ctomedicina.com WEB: www.ctomedicina.com; www.iberocto.com

Preguntas TEST

Seguimiento a distancia

HEMATOLOGA

Preparacin Examen de Seleccin 05/06 1 Vuelta


53.

Respecto a la infiltracin de las leucemias agudas, una respuesta es INCORRECTA:


1)
2)
3)
4)
5)

54.

4)
5)
55.

4)
5)

Preguntas TEST

57.

Habitualmente es de fenotipo B.
Es una forma de linfoma de bajo grado.
Es la forma ms frecuente de leucemia crnica en Occidente.
Casi nunca se acompaa de paraprotena srica.
Puede ser secundaria a quimioterpicos.

4)

Nmero de linfocitos.
Presencia de adenopatas.
Presencia de hepatomegalia.
Presencia de anemia.
Presencia de trombopenia.

5)
63.

Resistencia de fosfatasa cida a tartrato.


Pancitopenia.
Esplenomegalia progresiva.
Inmunofenotipo T.
Respuesta a interfern.

1)
2)
3)
4)
5)

La clula de Sternberg es patognomnica.


La clula de Hodgkin es la variante uninuclear.
La variante de la esclerosis nodular es la clula lacunar.
La esclerosis nodular recidiva con la misma histologa.
El mejor pronstico corresponde a la forma de predominio linfoctico.
M exico A rgentina
C hile U ruguay

66.

Adriamicina - cardiotoxicidad.
MOPP - mielitis transversa.
RT - sarcoma de partes blandas.
Bleomicina - neumonitis.
Vincristina - neuropatas.

Son factores pronsticos desfavorables en la enfermedad de


Hodgkin todos los siguientes, EXCEPTO:
1)
2)
3)
4)
5)

Respecto de la histologa de la enfermedad de Hodgkin, cul


NO es correcta?:

RT en manto.
MOPP.
MOPP + ABVD.
ABVD.
ABVD + RT mediastnica.

Entre las complicaciones producidas por el tratamiento de la


enfermedad de Hodgkin, seale la relacin FALSA:
1)
2)
3)
4)
5)

65.

Presencia de clulas de Sternberg en mdula sea.


Varn de 70 aos, en estadio clnico IIIB.
Presencia de lesiones ocupacionales en bazo, en la TC, y
biopsia heptica positiva.
Mujer joven, sin sntomas B y con adenopatas cervicales
y mediastnicas.
Enfermedad IA cervical alta.

El tratamiento ms adecuado en un paciente con enfermedad de Hodgkin estadio IIA con gran masa mediastnica es:
1)
2)
3)
4)
5)

64.

Curativa en estadios I y II.


Paliativa en estadios III y IV.
Paliativa en todos los estadios.
Curativa en estadio I.
Curativa en todos los estadios.

En cul de las siguientes situaciones cree que es correcta la


laparotoma de estadiaje en la enfermedad de Hodgkin?:
1)
2)
3)

NO es propio de tricoleucosis:
1)
2)
3)
4)
5)

59.

62.

IIA.
IIIA1.
IIIA2.
IIIAS.
IVA.

El tratamiento inicial de la enfermedad de Hodgkin tiene


finalidad:
1)
2)
3)
4)
5)

Leucemia mieloide crnica.


Leucemia linfoblstica T.
Tricoleucosis.
Leucemia aguda promieloctica.
Leucemia megacarioblstica.

En el sistema de estadiaje de RAI para la LLC, el PEOR


pronstico corresponde a:
1)
2)
3)
4)
5)

58.

61.

NO es correcto que la LLC:


1)
2)
3)

Un enfermo con afectacin por enfermedad de Hodgkin en


ganglios cervicales, axilares, mediastnicos y celacos, que no
presenta ningn sntoma B, se encuentra en estadio:
1)
2)
3)
4)
5)

Vincristina + prednisona.
Vincristina + prednisona + L-asparaginasa.
Vincristina + prednisona + L-asparaginasa + daunoblastina.
Arabinsido de citosina + daunoblastina.
6-mercaptopurina + metotrexate.

El cido todo-transretinoico (tretinona) se ha usado como


inductor de remisin leucmica en una de las siguientes:
1)
2)
3)
4)
5)

56.

La afectacin de encas es ms frecuente en la forma


promieloctica.
La CID es ms frecuente en la forma M3.
La afectacin del SNC es ms frecuente en la LAL.
La masa mediastnica es propia de LAL-T.
La afectacin testicular es ms frecuente en LAL.

Cul es el rgimen habitual de tratamiento de la LAM?:


1)
2)
3)

60.

Seguimiento a distancia

Aumento de velocidad de sedimentacin globular.


Hipogammaglobulinemia.
Edad avanzada.
Respuesta lenta al tratamiento.
Hipoalbuminemia.

NO se ha objetivado relacin del VEB con uno de los siguientes linfomas:


1)
2)

Burkitt endmico.
Burkitt no endmico.

CTO Medicina C/ Nez de Balboa, 115 28006 MADRID (Espaa) Tfno.: (91) 782 43 32 / Fax: (91) 782 43 27
E-mail: secretaria@ctomedicina.com; iberocto@ctomedicina.com WEB: www.ctomedicina.com; www.iberocto.com

HM Pg. 5

3)
4)
5)
67.

68.

70.

72.

76.

Melfaln y prednisona.
Ciclofosfamida y prednisona.
Radioterapia.
Esteroides en dosis altas.
Interfern.

Histologa de linfoma inmunoblstico.


Sndrome de hiperviscosidad.
Trastornos neurolgicos.
Presencia de crioaglutininas.
Invasin neoplsica de la mdula sea.

1)

Pg. 6 HM

Con frecuencia tienen sntomas.


M exico A rgentina
C hile U ruguay

80.

Pueden manifestarse con derrames en cavidades.


Predominantemente son de clula T.
Presentan frecuente afeccin de tubo digestivo.
Son de alto grado de agresividad.
Suelen estar diseminados en el diagnstico.

Uno de los siguientes datos es ms frecuente en el linfomaleucemia T asociado al HTLV-I que en otros linfomas:
1)
2)
3)
4)
5)

Cul de las siguientes afirmaciones es FALSA en los linfomas


no hodgkinianos de alto grado?:

Edad.
Estado de rendimiento del paciente.
Inmunofenotipo de la clula tumoral.
LDH srica.
Afeccin extraganglionar.

En relacin con los linfomas asociados al SIDA, seale la


respuesta INCORRECTA:
1)
2)
3)
4)
5)

79.

Linfoma de Burkitt.
Linfoma folicular.
Linfoma MALT gstrico.
Linfoma cerebral del SIDA.
Linfoma postrasplante renal.

Entre los factores pronsticos incluidos en el ndice internacional para linfomas no hodgkinianos, aparecen todos los
siguientes, EXCEPTO:
1)
2)
3)
4)
5)

78.

Linfoma de Burkitt.
Linfoma folicular.
Leucemia linftica crnica.
Linfoma histioctico.
Linfoma inmunoblstico.

Cul de los siguientes linfomas est relacionado con una


infeccin bacteriana crnica?:
1)
2)
3)
4)
5)

77.

Son poco frecuentes en la infancia.


Es frecuente la afeccin retroperitoneal.
Presentan elevadas remisiones completas con quimioterapia.
No suelen invadir el sistema nervioso central.
El paciente puede estar asintomtico.

El sndrome de Richter corresponde a la fase de transformacin de:


1)
2)
3)
4)
5)

Respecto a la enfermedad de Waldenstrm, NO es propio:


1)
2)
3)
4)
5)

73.

4)
5)
75.

El estudio de extensin debe incluir laparotoma.


La base del tratamiento es la poliquimioterapia.
Pueden ser B o T.
Tienen corta supervivencia sin tratamiento.

Respecto a los linfomas no hodgkinianos de bajo grado,


seale la respuesta INCORRECTA:
1)
2)
3)

Beta-2-microglobulina.
Calcemia.
Creatinina srica.
Viscosidad srica.
Uricemia.

El tratamiento ms adecuado del plasmocitoma extramedular sera:


1)
2)
3)
4)
5)

74.

Radiologa sea normal.


Lesin sea solitaria.
Hemoglobina mayor de 10 g/dl.
Calcemia mayor de 12 mg/dl.
Cadenas ligeras en orina menor de 4 g/da.

En el mieloma mltiple, qu parmetro bioqumico se correlaciona mejor con la masa tumoral?:


1)
2)
3)
4)
5)

71.

Adenopatas.
Dolores seos.
Hipercalcemia.
Insuficiencia renal.
Neumona neumoccica.

Todos los siguientes criterios son propios del estadio I del


mieloma mltiple segn Durie-Salmon, con la EXCEPCIN
de:
1)
2)
3)
4)
5)

2)
3)
4)
5)

Inmunocitoma.
Centroctico.
Folicular.
Inmunoblstico.
Leucemia linftica crnica.

Cul de las siguientes es la manifestacin ms frecuente del


mieloma mltiple?:
1)
2)
3)
4)
5)

69.

Linfoma T del adulto.


Linfoma en enfermos de SIDA.
Enfermedad de Hodgkin.

Cul de los siguientes linfomas no Hodgkin se considera de


alto grado en la clasificacin de Kiel?:
1)
2)
3)
4)
5)

HEMATOLOGA

Preparacin Examen de Seleccin 05/06 1 Vuelta

Elevacin de transaminasas.
Hipercalcemia.
Coagulacin intravascular.
Hiperamoniemia.
Inmunodeficiencia humoral.

Uno de los siguientes factores de coagulacin NO es dependiente de vitamina K:

CTO Medicina C/ Nez de Balboa, 115 28006 MADRID (Espaa) Tfno.: (91) 782 43 32 / Fax: (91) 782 43 27
E-mail: secretaria@ctomedicina.com; iberocto@ctomedicina.com WEB: www.ctomedicina.com; www.iberocto.com

Preguntas TEST

Seguimiento a distancia

HEMATOLOGA

Preparacin Examen de Seleccin 05/06 1 Vuelta


1)
2)
3)
4)
5)
81.

2)
3)
4)
5)

Preguntas TEST

3)
4)
5)

3)
4)
5)
89.

Mielodisplasia.
Coagulacin intravascular diseminada.
Prpura trombopnica idioptica.
Prpura trombopnica trombtica.
Esplenomegalia.

90.

Trombopenia.
Aumento del nmero y tamao de megacariocitos en
M.O.
Esplenomegalia.
Anticuerpos antiplaquetarios.
Descartar otras causas de trombopenia.

Esteroides.
Vigilancia sin tratamiento.
Esplenectoma.
Inmunosupresores.
Gammaglobulina.

Cul NO es componente propio de la prpura trombopnica trombtica?:


1)

Trombopenia.
M exico A rgentina
C hile U ruguay

Trombopenia.
Antitrombina III elevada.
Tiempo de protrombina prolongado.
Hemlisis microangioptica.
Productos de degradacin de la fibrina.

La alteracin de laboratorio que guarda una correlacin ms


estrecha en la coagulacin intravascular diseminada con el
sangrado es el nivel de:
1)
2)
3)
4)
5)

92.

Hemoglobinuria paroxstica nocturna.


Policitemia vera.
Trombocitemia esencial.
Enfermedad de Rendu-Osler.
Enfermedad de Moschcowitz.

Cul de los siguientes parmetros hemostticos NO es


propio de la coagulacin intravascular diseminada?:
1)
2)
3)
4)
5)

91.

Es la trombocitopata ms frecuente.
El factor VW se sintetiza en megacariocitos y endotelio
vascular.
La forma ms grave de la enfermedad es la autosmica
dominante.
El sangrado cutneo y ORL es el ms frecuente.
En el defecto cuantitativo, puede utilizarse como tratamiento el acetato de desmopresina.

En cul de las siguientes enfermedades NO es tpica la


trombosis vascular?:
1)
2)
3)
4)
5)

Cul es el tratamiento inicial de la prpura trombopnica


idioptica crnica sintomtica?:
1)
2)
3)
4)
5)

86.

II.
VII.
X.
XII.
XIII.

Esplenectoma.
Esteroides.
Inmunosupresores.
Plasmafresis.
Antiagregantes.

Qu respuesta es FALSA, respecto a la enfermedad de Von


Willebrand?:
1)
2)

Cul de los siguientes NO es criterio diagnstico de prpura


trombopnica idioptica?:
1)
2)

85.

88.

Afectacin miocrdica.
Esquistocitos.
Fiebre.
Alteracin neurolgica.

Cul es el tratamiento de eleccin de la prpura trombopnica trombtica?:


1)
2)
3)
4)
5)

NO es una trombopenia de mecanismo perifrico la asociada


a:
1)
2)
3)
4)
5)

84.

La trombopenia es la causa ms frecuente de prolongacin del tiempo de hemorragia.


La causa ms frecuente de prolongacin del tiempo de
hemorragia con nmero normal de plaquetas es la enfermedad de von Willebrand.
El tiempo de protrombina controla la coagulacin extrnseca.
El control del tratamiento anticoagulante oral se realiza
mediante el tiempo de tromboplastina.
El tiempo de tromboplastina controla la coagulacin
intrnseca.

87.

La deficiencia congnita de uno de los siguientes factores de


coagulacin NO prolonga los tiempos de protrombina ni
tromboplastina:
1)
2)
3)
4)
5)

83.

2)
3)
4)
5)

Respecto a las pruebas de hemostasia, es INCORRECTO:


1)

82.

Protrombina.
Fibringeno.
VII.
IX.
X.

Seguimiento a distancia

Trombopenia.
Prolongacin del tiempo de protrombina.
Prolongacin del TTPa.
Fibringeno plasmtico.
PDF.

Entre los trastornos protrombticos hereditarios se encuentran todos los siguientes, con EXCEPCIN de:
1)
2)
3)
4)
5)

Factor V Leiden.
Dficit de plasmingeno.
Dficit de PAI-I.
Disfibrinogenemia.
Dficit de antitrombina III.

CTO Medicina C/ Nez de Balboa, 115 28006 MADRID (Espaa) Tfno.: (91) 782 43 32 / Fax: (91) 782 43 27
E-mail: secretaria@ctomedicina.com; iberocto@ctomedicina.com WEB: www.ctomedicina.com; www.iberocto.com

HM Pg. 7

Seguimiento a distancia

Respecto a los trastornos protrombticos hereditarios, seale la respuesta FALSA:


1)
2)
3)
4)
5)

94.

95.

1)
2)
3)
4)
5)

Descenso de fibringeno.
Descenso de factor V.
Tiempo de trombina alargado.
Dficit de factor VIII.
Trombopenia.

100.

Hipersensibilidad medicamentosa.
Dosis insuficiente.
Dosis excesiva.
Asociacin con heparina de bajo peso molecular.
Deficiencia de protena C.

1)
2)
3)
4)
5)

Trombosis venosa profunda.


Prtesis valvular cardaca mecnica.
Fibrilacin auricular en estenosis mitral.
Disfuncin severa de ventrculo izquierdo.
Tromboembolismo pulmonar.

Cul de las siguientes situaciones puede producir hipocoagulabilidad adquirida?:


Policitemia vera.
Dficit de hierro.
Ictericia obstructiva.
Dficit de cido flico.
Anemia hemoltica.

Las alteraciones de la hemostasia son caractersticas de los


siguientes procesos, con la EXCEPCIN de:
Adenocarcinoma de prstata.
Leucemia promieloctica.
Mieloma mltiple.
Enfermedad de Hodgkin.
Enfermedad de Waldenstrm.

Un paciente portador de una prtesis valvular cardaca


precisa intervencin quirrgica programada. En relacin a su
tratamiento anticoagulante, lo ms correcto ser:
1)
2)
3)
4)
5)

98.

1)
2)
3)
4)
5)

En las cirrosis hepticas, NO suele encontrarse:

1)
2)
3)
4)
5)
97.

La necrosis cutnea relacionada con anticoagulantes orales


est en relacin con:

Los niveles de INR para control de anticoagulacin oral


deben mantenerse entre 2 y 3, con la EXCEPCIN de una de
las siguientes indicaciones:

1)
2)
3)
4)
5)
96.

Suelen tener herencia autosmica dominante.


No existen buenas pruebas de laboratorio para realizar
despistaje.
Se deben sospechar en casos de tromboembolismo recidivante.
Ocasionan trombosis, sobre todo en la infancia.
Las mujeres deben evitar el uso de anovulatorios orales.

99.

Suspender la anticoagulacin de todo tipo 24 horas antes


de la intervencin.
Suspender anticoagulacin de todo tipo 48 h antes.
Suspender anticoagulacin oral 24 h antes y mantener
heparina intravenosa hasta el momento de la intervencin.
Suspender anticoagulacin oral unos das antes y mantener heparina intravenosa hasta unas horas antes de la
intervencin.
Mantener anticoagulacin oral hasta unas horas antes y
administrar entonces vitamina K y plasma.

La trombocitopenia secundaria a heparina:


1)
2)
3)
4)
5)

Pg. 8 HM

Como otras trombopenias secundarias a frmacos, la


clnica hemorrgica es frecuente.
En esta trombopenia, caractersticamente son ms frecuentes los fenmenos trombticos que los hemorrgicos.
Es ms frecuente con las heparinas de bajo peso molecular.
Se cree que su origen no es inmunolgico.
No existe tal asociacin.

M exico A rgentina
C hile U ruguay

CTO Medicina C/ Nez de Balboa, 115 28006 MADRID (Espaa) Tfno.: (91) 782 43 32 / Fax: (91) 782 43 27
E-mail: secretaria@ctomedicina.com; iberocto@ctomedicina.com WEB: www.ctomedicina.com; www.iberocto.com

Preguntas TEST

93.

HEMATOLOGA

Preparacin Examen de Seleccin 05/06 1 Vuelta

HEMATOLOGA

Preparacin Examen de Seleccin 05/06 1 Vuelta


Pregunta 1.- R: 3
Los reticulocitos traducen el grado de produccin de la mdula
sea. La elevacin de reticulocitos es propia de las anemias
regenerativas, como la hemorragia aguda o los procesos hemolticos.
Por contra, los fallos medulares (aplasias, sndrome mielodisplsico,
anemias sideroblsticas), o los trastornos carenciales (ferropenia) son
anemias hiporregenerativas donde no se elevan los reticulocitos.
Pregunta 2.- R: 1
El hipotiroidismo suele producir anemias macrocticas, siendo necesario introducir dicha enfermedad en el diagnostico diferencial de
anemias macrocticas. Procesos que produzcan disminucin de la
membrana del hemate, como la microesferocitosis hereditaria, o trastornos en los componentes de la hemoglobina (hierro, HEM, inmunoglobulina) producen anemias microcticas como son la ferropnica, la
anemia de los trastornos crnicos, sideroblsticas y talasemias.
Pregunta 3.- R: 3
La anemia de los trastornos crnicos es la causa ms frecuente de
anemia normoctica. Otras enfermedades que cursan con anemias de
tamao normal de hemate son enfermedades medulares como la aplasia y las mieloptisis y la mayor parte de los procesos hemolticos.
Pregunta 4.- R: 3
La hemorragia crnica genera ferropenia, y por tanto, anemia microctica. Las anemias macrocticas pueden ser megaloblsticas (precursores gigantes en la deficiencia de folato y B12), y en general, las
enfermedades de precursores medulares como mielodisplasia pueden producir anemias macrocticas. En la prctica habitual, la causa
ms frecuente de macrocitosis es el alcoholismo.
Pregunta 5.- R: 3
El concepto de enfermedad hemoltica es aquella en la que la vida
media del hemate se encuentra acortada. Si este acortamiento de la
vida eritrocitaria se debe a traumatismo, aparecen caractersticamente
los esquistocitos, que por tanto no son caractersticos de todas las
enfermedades hemolticas, sino de los trastornos traumticos.

Comentarios TEST

Pregunta 6.- R: 2

Pregunta 6.

Caractersticas hematolgicas de la aplasia.

Seguimiento a distancia

El hemograma tpico de la aplasia se caracteriza por pancitopenia


con reticulocitopenia. Esta disminucin de clulas sanguneas es consecuencia de escasez de tejido hematopoytico, no de la invasin de
la medula sea por tejidos anormales, incluyendo blastos. El incremento de blastos en medula sea es propio de las leucemias agudas y
de las mielodisplasias.
Pregunta 7.- R: 5
La pancitopenia es propia de la aplasia, pero puede aparecer en
diversas patologas, como son la hemoglobinuria paroxstica nocturna, la deficiencia de vitamina B12 y la mielodisplasia. Estos trastornos
tienen en comn la posibilidad de producir simplemente anemia, si
bien en formas ms avanzadas los tres trastornos pueden producir
pancitopenia. La invasin de la mdula sea por micobacterias puede cursar tambin con pancitopenia. En la ferropenia, adems de la
anemia puede ser caracterstica la elevacin de la cifra de plaquetas.
Pregunta 8.- R: 3
Las manifestaciones clnicas del paciente con aplasia derivan de la
escasez de clulas sanguneas, por lo que los pacientes tiene sndrome
anmico, infecciones si presentan neutropenias severas y hemorragias
principalmente mucocutneas si hay una cifra muy baja de plaquetas.
En neutropenias muy intensas pueden faltar los signos inflamatorias
acompaantes de procesos infecciosos. La aplasia no cursa con
organomegalias, incluida esplenomegalia.
Pregunta 9.- R: 1
El tratamiento de referencia de la aplasia es el trasplante de mdula
sea. La posibilidad de rechazo de dicho trasplante es inferior en
pacientes que no han sido sensibilizados por transfusiones sanguneas
previas. Como alternativa al trasplante, la eleccin consiste en la asociacin de globulina antitimocito y ciclosporina. Ocasionalmente se
pueden asociar esteroides. Estos tratamientos producen mejora del
pacientes a lo largo de varias semanas, no de forma rpida. Una complicacin del tratamiento con GAT puede ser la aparicin de displasia.
Pregunta 10.- R: 3
La disociacin de clulas rojas en lgrimas (dacriocitos) y la reaccin leucoeritroblstica (clulas inmaduras en sangre perifrica de la
seria blanca y roja) es muy caracterstica, asociada a las citopenias del
paciente, a la mieloptisis. La mieloptisis consiste en la ocupacin de la
medula sea por tejidos anormales, que por proceso ocupacional
disminuyen la produccin de clulas sanguneas normales, al mismo
tiempo que pueden desplazar hacia la sangre perifrica clulas inmaduras. Causas frecuentes de mieloptisis son las metstasis de cncer de
mdula sea. El timoma produce una lesin medular distinta de la
mieloptisis, consistente en la eritroblastopenia o aplasia pura de la
serie roja.
Pregunta 11.- R: 3
Las anemias microcticas se producen fundamentalmente por dos
mecanismos. En el primero de ellos, ms frecuente, se produce una

Pregunta 10. Anemia mieloptsica: reaccin leucoeritroblstica y clulas en lgrima.


M exico A rgentina
C hile U ruguay

CTO Medicina C/ Nez de Balboa, 115 28006 MADRID (Espaa) Tfno.: (91) 782 43 32 / Fax: (91) 782 43 27
E-mail: secretaria@ctomedicina.com; iberocto@ctomedicina.com WEB: www.ctomedicina.com; www.iberocto.com

HM Pg. 1

disminucin de la cantidad de hemoglobina del hemate. Estas patologas estaran representadas por problemas en los componentes de la
hemoglobina (hierro, HEM, y cadenas de globina). Con este mecanismo, no se produce slo disminucin del tamao del hemate, sino
tambin hipocroma eritrocitaria (representada en el hemograma por
el parmetro CHCM). El segundo mecanismo consiste en la disminucin de la superficie de membrana, como es el caso de la esferocitosis
hereditaria, si bien este mecanismo no produce disminucin del contenido de hemoglobina, y por tanto, no ocasiona hipocroma, sino
aumento de la CHCM.
Pregunta 12.- R: 1
La mayor parte del hierro del organismo se encuentra en la molcula de hemoglobina. La mayora del hierro necesario para la eritropoyesis basal procede de la destruccin de los hemates viejos
(hemocateresis). La absorcin intestinal de hierro es mejor en forma
de hierro orgnico, como es el hiero HEM. Las situaciones de eritropoyesis ineficaz incrementan la absorcin intestinal de hierro, pudiendo ocasionar a largo plazo situaciones de hemosiderosis.
Pregunta 13.- R: 3
La situaciones de ferropenia se caracterizan por un patrn bioqumico consistente en disminucin de la sideremia, aumento de la concentracin de transferrina, disminucin de la saturacin de transferrina y disminucin de la ferritina srica. En el hemograma, adems de microcitosis se
ocasiona hipocroma, objetivada por descenso de HCM y CHCM.
Pregunta 14.- R: 2
En la situacin de ferropenia inicial no se produce ni anemia ni
tienen porque aparecer alteraciones en el hemograma. Dado que para
mantener la eritropoyesis el organismo utiliza las reservas de hierro, la
primera alteracin en el laboratorio consiste en disminucin de la ferritina srica, parmetro que se correlaciona con el depsito del hierro.
Pregunta 15.- R: 5
Durante el desarrollo de ferropenia se produce inicialmente el
consumo de los depsitos de hierro. Durante el tratamiento de la
ferropenia se produce inicialmente un incremento de la eritropoyesis, reconocida en el hemograma por el aumento de los reticulocitos
sanguneos y una sucesiva normalizacin del resto de parmetros,
siendo lo ltimo en normalizarse los depsitos de hierro, y por tanto
los niveles de ferritina srica.
Pregunta 16.- R: 3
La hiposideremia es un hallazgo propio tanto de la ferropenia como
de la anemia de los trastornos crnicos, dado que en esta ltima existe
un bloqueo en la salida del hierro de los macrfagos hacia la sangre.
Por tanto, el concepto de hiposideremia no puede hacerse igual a
ferropenia. La saturacin de transferrina est generalmente disminuida en ambos procesos, mientras que la concentracin de transferrina
est elevada en la ferropenia y suele ser normal, o ms frecuentemente disminuida, en la anemia de la enfermedad crnica. Ambos procesos producen anemias que pueden ser microcticas e hipocromas.
Pregunta 17.- R: 3
Los mecanismos de produccin de anemia en las enfermedades
crnicas son mltiples. Por una parte est la inadecuada produccin
de la mdula sea como consecuencia de una inadecuada produccin de eritropoyetina o de la presencia de interfern o factor de necrosis
tumoral alfa, que son inhibidores de la eritropoyesis. Por otra parte, en
lo que se refiere al uso del hierro, existe un bloqueo en la salida del
hierro de depsito, tanto a la sangre como hacia los precursores de la
serie roja a la mdula, por lo que no existe el incremento de precursores
con exceso de hierro (sideroblastos). La intensidad de la anemia en los
trastornos crnicos suele ser moderada.
Pregunta 18.- R: 2
Si bien el descenso de sideremia y de saturacin de transferrina son
hallazgos propios tanto de la anemia ferropnica como la de los trastornos crnicos, el incremento en ambos parmetros es caracterstico
de situaciones de exceso orgnico de hierro, como es el caso de la
Pg. 2 HM

M exico A rgentina
C hile U ruguay

HEMATOLOGA

Preparacin Examen de Seleccin 05/06 1 Vuelta

anemia sideroblstica, que ocasionalmente precisa tratamiento con


quelantes del hierro para evitar sobrecarga frrica.
Pregunta 19.- R: 4
Los hallazgos del hemograma en anemia megaloblstica suelen ser
tpicos: macrocitosis, ocasionalmente muy intensa, hemates de forma ovalada en vez de redonda (macroovalocitos), presencia de hipersegmentacin de los neutrfilos, con existencia de anemia exclusivamente o diversas citopenias, hasta la pancitopenia. A nivel medular se
objetivan precursores de tamao gigante. El ndice de Herbert es una
media de la segmentacin del ncleo de los neutrfilos que, como se
ha dicho, est hipersegmentado en las anemias megaloblsticas. Un
ndice de Herbert inferior a 3 producira una situacin de
hiposegmentacin de los neutrfilos, rasgo que es ms caracterstico
de las mielodisplasias que de las anemias megaloblsticas.
Pregunta 20.- R: 5
La anemia megaloblstica se caracteriza por citopenias en sangre
perifrica, pero mdulas hipercelulares, en las que existe una intensa
hemopoyesis ineficaz, lo que puede ocasionar a nivel srico datos de
destruccin celular excesiva (elevacin de LDH y de bilirrubina).
Ocasionalmente en la anemia megaloblstica los niveles sricos
de folato y B12 pueden ser normales, siendo ms sensibles para detectar carencia de esta sustancia los niveles de homocistena, que se
incrementan tanto en la carencia de folato como en la de B12 y los
de metilmalnico, que slo se incrementan en la deficiencia de B 12.
Pregunta 21.- R: 2
La anemia perniciosa suele ser la forma ms severa de anemia
megaloblstica, caracterizndose histolgicamente por una atrofia de
la mucosa gstrica y por la aparicin de anticuerpos contra clula
parietal (poco especficos) y contra el factor intrnseco, si bien estos
ltimos slo aparecen en dos tercios de los casos, por lo que la ausencia de anticuerpos contra el factor intrnseco no puede descartar una
anemia perniciosa.
Pregunta 22.- R: 4
El hemograma de las anemias hemolticas se suele caracterizar por
anemias normocticas con reticulocitosis sangunea y frotis caracterstico. En gran parte de los pacientes con enfermedades hemolticas se
produce destruccin de hemates tanto intra como extravascular, por
lo que datos como la elevacin de LDH o de bilirrubina indirecta en
suero o el descenso de haptoglobina srica libre se pueden ver tanto
en los procesos hemolticos intra como extravasculares. La existencia
de hemoglobinuria o hemosiderinuria es, sin embargo, caracterstica
de la lisis intravascular.
Pregunta 23.- R: 1
Entre las caractersticas clnicas de los pacientes con enfermedades
hemolticas puede aparecer la existencia de orinas oscuras. Dicha
alteracin es consecuencia de hemoglobinuria y no de bilirrubinuria
o coluria. En ausencia de obstruccin de la va biliar, la bilirrubina
generada en el proceso de destruccin de hemates es conjugada en
el hgado y eliminada por bilis, no por orina. La litiasis biliar puede ser
una complicacin de los pacientes con enfermedades hemolticas
crnicas, por el exceso de produccin de bilirrubina. La
esplenomegalia es un dato caracterstico de muchos pacientes con
enfermedades hemolticas.
Pregunta 24.- R: 4
La destruccin de hemates en las anemias hemolticas puede ser
consecuencia de alteraciones a nivel celular intracorpuscular (como
es el caso de la hemoglobinuria paroxstica nocturna) o extracorpuscular, en cuyo caso el hemate es normal, pudiendo ser destruido por
una hiperfuncin esplnica, anticuerpos antihemates, alteraciones
de la microcirculacin o toxinas como ocurre en la sepsis clostridial a
travs de una hemolisina.
Pregunta 25.- R: 3
La existencia de esferocitos en sangre perifrica es caracterstica
de los procesos hemolticos, fundamentalmente la esferocitosis he-

CTO Medicina C/ Nez de Balboa, 115 28006 MADRID (Espaa) Tfno.: (91) 782 43 32 / Fax: (91) 782 43 27
E-mail: secretaria@ctomedicina.com; iberocto@ctomedicina.com WEB: www.ctomedicina.com; www.iberocto.com

Comentarios TEST

Seguimiento a distancia

HEMATOLOGA

Preparacin Examen de Seleccin 05/06 1 Vuelta


reditaria y las inmunohemlisis. Este ltimo grupo se caracteriza
por positividad en la prueba de Coombs, dado que la destruccin
de los hemates es por efectos de anticuerpos. La aparicin de
esferocitos en la esferocitosis hereditaria no es consecuencia de
ataque de la clula roja por anticuerpos, sino del atrapamiento
esplnico del hemate como consecuencia de alteraciones en la
adaptabilidad celular por fallo en la membrana. La esferocitosis
hereditaria suele ser autosmica dominante, aunque hay formas
recesivas ms graves. La clnica de la esferocitosis hereditaria es
muy variable, pudiendo no manifestarse hasta la edad adulta en
sus formas leves.

Seguimiento a distancia

Pregunta 28.- R: 5

Pregunta 26.- R: 4
La deficiencia de glucosa-6-fosfato deshidrogenasa es la enzimopata ms frecuente eritrocitaria y est ligada al cromosoma X.
Caractersticamente el proceso hemoltico aparece en brotes desencadenados por infecciones, situaciones de acidosis, ingestin
de habas o utilizacin de frmacos como las sulfamidas. En los
reticulocitos, la cantidad de glucosa-6-fosfato deshidrogenasa es
ms elevada que en los hemates ms viejos, por lo que no se
aconseja la cuantificacin enzimtica en situaciones de reticulocitosis, como ocurre durante las crisis hemolticas o inmediatamente
despus de las mismas.

Pregunta 26. Frmacos que producen hemlisis en personas con dficit


de G-6PD

Comentarios TEST

123456768569
2 6 2465
 45
429
4469
2
4
2226
4696296
24669

123456645
6
2495642 6965

12345669
29

 642 646426 64
125
2
7496265642
Pregunta 27.- R: 2
La anemia de Cooley es la forma grave de la talasemia beta y actualmente su nico tratamiento curativo es el trasplante de progenitores hematopoyticos. En ausencia de dicho tratamiento se puede realizar paliacin de la anemia administrando transfusiones sanguneas,
tratamiento que tambin es beneficioso por disminuir el grado de
hiperplasia medular y su alteracin sea consiguiente. Las talasemias
graves se caracterizan por una eritropoyesis ineficaz intensa que facilita la absorcin intestinal de hierro y puede dar lugar a sobrecarga
frrica, siendo la afeccin cardaca la forma ms grave. Por dicho
motivo es necesario tratamiento tambin con quelantes del hierro.

Pregunta 28. Fisiopatologa de la -talasemia.

En el rasgo talasmico, el hemograma se caracteriza por la aparicin de un nmero incrementado de hemates de pequeo tamao.
Hay que tener presente que dicha situacin puede corresponder tambin ocasionalmente a situaciones de ferropenia no intensa, por lo
que el hemograma no permite diferenciar entre ambas patologas,
siendo necesarios estudios de las hemoglobinas eritrocitarias para diagnosticar rasgo talasmico y de ferritina srica para diagnosticar ferropenia. Hay que tener presente tambin que diferenciar entre ambas
situaciones es fundamental, dado que la ferropenia requiere tratamiento con hierro y el rasgo talasmico es una situacin en la que el
hierro est contraindicado.

Pregunta 27. Anemia de Cooley. Alteraciones craneofaciales.


M exico A rgentina
C hile U ruguay

CTO Medicina C/ Nez de Balboa, 115 28006 MADRID (Espaa) Tfno.: (91) 782 43 32 / Fax: (91) 782 43 27
E-mail: secretaria@ctomedicina.com; iberocto@ctomedicina.com WEB: www.ctomedicina.com; www.iberocto.com

HM Pg. 3

HEMATOLOGA

Preparacin Examen de Seleccin 05/06 1 Vuelta

Pregunta 29.- R: 2
La hemoglobinopata S, anemia de clulas falciformes o drepanocitosis, es un proceso en el que la hemoglobina inestable precipita
formando clulas en forma de hoz que, adems de ser destruidas,
ocluyen los vasos sanguneos, produciendo las denominadas crisis
vasooclusivas. La isquemia generada por estos episodios vasooclusivos
puede desencadenar infartos, de los que los ms frecuentes son los
renales, con la consiguiente alteracin en la concentracin urinaria
(isostenuria). Como consecuencia de infartos esplnicos repetidos se
puede llegar a una situacin de hipoesplenismo con inmunodeficiencia asociada. La esplenectoma, por tanto, no es un tratamiento
para esta enfermedad.
Pregunta 30.- R: 5
En las anemias inmunohemolticas, la destruccin del hemate se
produce por la accin de anticuerpos, complemento o ambas sustancias sobre la membrana del hemate. Cuando tiene lugar una intensa
activacin del complemento, como es el caso de la producida por
IgM, se produce una destruccin intravascular y cuando la activacin
del complemento es escasa, como es el caso con anticuerpos del tipo
IgG, la destruccin es predominantemente esplnica. La base del diagnstico de las enfermedades inmunohemolticas es la prueba de
Coombs para detectar anticuerpos antieritrocitarios o la accin del
complemento. La prueba de Coombs directa detecta anticuerpo y/o
complemento sobre la membrana del hemate y la de Coombs indirecta anticuerpo en plasma. Ambas pruebas pueden salir negativas
(anemia inmunohemoltica: Coombs negativo).
Respuesta 31.- R: 4
Un anticuerpo caliente se fija en la membrana del hemate a temperatura fisiolgica y produce una destruccin principalmente
esplnica, tras unin de la fraccin constante de la IgG a los receptores de los macrfagos esplnicos. Ocasionalmente puede asociarse
trombopenia de mecanismo inmune (sndrome de Evans). Aunque
hay formas idiopticas siempre hay que descartar etiologas conocidas (frmacos, infecciones, enfermedades autoinmunes o procesos
linfoproliferativos).
Pregunta 32.- R: 4
La enfermedad de las aglutininas fras est causada por anticuerpos
que se fijan al hemate a temperatura inferior a la fisiolgica, son
habitualmente de tipo IgM y activan intensamente complemento produciendo lisis intravascular. El trmino aglutinina implica que es un
anticuerpo que aglutina hemates, pudiendo producirse atasco en la
microcirculacin con fenmenos isqumicos desencadenados por
exposicin al fro. En este caso se produce una acrocianosis con la
exposicin al fro sin verdadero vasoespasmo o fenmeno de Raynaud.
Pregunta 33.- R: 1
La asociacin de anemia y trombopenia se puede dar en los cinco
procesos especificados en las respuestas, si bien el sndrome de Evans,
por ser un proceso hemoltico inmune, se asocia caractersticamente
con esferocitos en vez de con esquistocitos. Los otros cuatro procesos
ocasionan hemlisis traumtica, y por tanto la existencia de esquistocitos es caracterstica.

sensibles al complemento. Este hecho es consecuencia de la escasez


en la membrana celular de sustancias que neutralizan la actividad del
complemento (CD59 y CD55). La forma ms exacta de diagnosticar la
enfermedad consiste en el estudio de la membrana celular mediante
citometra de flujo, objetivando la escasez de dichos marcadores.
Formas indirectas, menos sensibles y especficas son las prueba de
Ham y la lisis con sacarosa.
Pregunta 36.- R: 3
Como otros procesos hemolticos, la hemoglobinuria paroxstica nocturna puede responder ocasionalmente a esteroides. En situaciones de hemosiderinuria crnica puede desencadenarse una
carencia de hierro, por lo que el tratamiento con hierro puede ser
beneficioso en ciertas circunstancias. Las transfusiones forman parte
del tratamiento paliativo de todo tipo de anemias. Ocasionalmente
la hemoglobinuria paroxstica nocturna puede coexistir con hipoplasia de medula sea, siendo entonces beneficioso el uso de tratamientos para este proceso, como es el caso de la globulina antitimocito.
Pregunta 37.- R: 3
Los sndromes mielodisplsicos son mielopatas clonales que se
caracterizan por eritropoyesis ineficaz con la consiguiente anemia.
En formas severas de mielodisplasia puede existir tambin leucopenia
y trombopenia, caracterizndose por tanto estos procesos por
citopenias sanguneas diversas aisladas o en combinacin. Como todas las mielopatas clonales, estos procesos pueden evolucionar hacia
leucemia aguda.
Pregunta 38.- R: 2
La existencia de sideroblastos en anillo en mdula sea es caracterstica de una forma de sndrome mielodisplsico denominado anemia refractaria con sideroblastos en anillo, que es el trastorno menos
grave. En general, las alteraciones citogenticas en enfermedades de
precursores medulares suelen implicar mal pronstico. Al tratarse de
enfermedades que ocasionan citopenias sanguneas es lgico que la
intensidad de las citopenias implique importancia pronstica, al igual
que la edad avanzada y el incremento de blastos sanguneos, situacin que se da con la evolucin hacia la leucemia aguda.
Pregunta 39.- R: 1
Se han intentado mltiples tratamientos en los pacientes con mielodisplasias, lo que implica que no se ha encontrado todava un tratamiento bueno para la mayor parte de los pacientes. El trasplante de
mdula sea sera un tratamiento definitivo de un grupo minoritario
de pacientes (dada la edad avanzada en gran parte de casos). Quimioterapia con intento de erradicacin del clon premaligno, o frmacos
como los factores de crecimiento hemopoytico son tratamientos
paliativos. La piridoxina ocasionalmente produce un incremento en
la sntesis HEM y mejora de la anemia de los pacientes (anemia refractaria sensible a la piridoxina).
Pregunta 40.- R: 1

Pregunta 34.- R: 1
La hemoglobinuria paroxstica nocturna se caracteriza por la existencia de citopenias hematolgicas, pudiendo existir slo anemia o,
en sus formas ms severas, incluso pancitopenia. Como todos los trastornos clonales de la mdula sea, puede evolucionar hacia leucemia aguda.
El trastorno es consecuencia de un exceso de sensibilidad de los
hemates al complemento, por lo que la lisis es intravascular. Otra
caracterstica de la enfermedad es la existencia de fenmenos trombticos de localizaciones inhabituales, como es el caso de una trombosis mesentrica.
Pregunta 35.- R: 3
La hemoglobinuria paroxstica nocturna es una mielopata clonal
donde las clulas sanguneas del clon anmalo son especialmente
Pg. 4 HM

M exico A rgentina
C hile U ruguay

Pregunta 40. Sndrome mieloproliferativo crnico.

CTO Medicina C/ Nez de Balboa, 115 28006 MADRID (Espaa) Tfno.: (91) 782 43 32 / Fax: (91) 782 43 27
E-mail: secretaria@ctomedicina.com; iberocto@ctomedicina.com WEB: www.ctomedicina.com; www.iberocto.com

Comentarios TEST

Seguimiento a distancia

HEMATOLOGA

Preparacin Examen de Seleccin 05/06 1 Vuelta


Los sndromes mieloproliferativos crnicos son mielopatas clonales caracterizadas por un exceso de proliferacin en las clulas del
clon anormal, que conserva la capacidad de maduracin, por lo que
lo caracterstico desde el punto de vista hematolgico en estos procesos es una hipercelularidad medular y el incremento de clulas sanguneas, pero de tipo maduro.

sarios otros estudios para el diagnstico diferencial con patologas que


presentan blastos en escasa proporcin en la sangre, como pueden
ser la leucemia mieloide crnica, los sndromes mielodisplsicos, y
muy excepcionalmente, procesos mieloptsicos. En la leucemia linftica crnica las clulas sanguneas son de apariencia madura, no
blsticas.

Pregunta 41.- R: 3
Un sndrome mieloproliferativo agudo (leucemia aguda) es un proceso clonal de la mdula sea donde las clulas malignas no son
capaces de madurar mas all de la fase de blasto, condicionando una
ocupacin medular por blastos, que a su vez disminuye el espacio
para la fabricacin de clulas normales, con la consiguiente disminucin de clulas maduras en sangre perifrica.

Pregunta 46.- R: 3
La existencia de esplenomegalia es un hecho caracterstico de los
sndromes mieloproliferativos crnicos, pudiendo aparecer en todos
ellos. Se ha considerado adems un criterio mayor de policitemia
vera, dada la gran importancia de este hallazgo, teniendo presente
adems que la existencia de esplenomegalia en una persona suele ir
aadida a cifras bajas de clulas sanguneas como consecuencia de
hiperesplenismo. Cuando en un paciente se dan a la vez la existencia
de esplenomegalia y elevacin de clulas sanguneas, siempre ha de
considerarse como diagnstico un sndrome proliferativo que llena la
sangre de clulas e infiltra el bazo, ocasionando una asociacin aparentemente paradjica. En muchas manifestaciones, la policitemia
vera coincide con otras formas de poliglobulia, como consecuencia
de los trastornos circulatorios a los que da lugar el incremento de la
viscosidad sangunea. La existencia de eritrocianosis es una caracterstica que puede existir en la policitemia vera.

Pregunta 41. Sndrome mieloproliferativo agudo.

Pregunta 42.- R: 1
Una leucocitosis de la intensidad del paciente es propia de una
leucemia. Las llamadas reacciones leucemoides son situaciones de
leucocitosis que pueden confundirse con leucemias en el hemograma, si bien generalmente no alcanzan cifras tan intensas. Los sndromes mielodisplsicos se caracterizan por leucopenia en vez de leucocitosis. Se entiende que si el 1% de los leucocitos sanguneos son
blastos, el 99% restante son clulas que estn en fases ms maduras,
situacin que es incompatible con la leucemia aguda. En la leucemia
mieloide crnica, a pesar de que existe predominio de clulas maduras, puede haber ocasionalmente una pequea cuanta de blastos en
sangre perifrica.

Comentarios TEST

Seguimiento a distancia

Pregunta 43.- R: 3
La existencia de ms de un 20% de blastos en sangre perifrica es
un criterio definitivo de leucemia aguda. En el hemograma del paciente no se establecen caractersticas de la celularidad blstica, por
lo que no podemos asegurar el tipo de leucemia aguda que tiene este
paciente, de ah que si la respuesta 3 dijese leucemia linfoide aguda,
tambin valdra como contestacin de la pregunta.

Pregunta 47.- R: 1
La denominada fase de metamorfosis de policitemia vera o fase
gastada es consecuencia de un proceso fibrtico de la mdula sea
en el que se va reduciendo progresivamente la formacin de clulas
sanguneas, incluida la serie roja, con la aparicin incluso de anemia.
En esta fase, la policitemia vera es indistinguible de la denominada
mielofibrosis con metaplasia mieloide, pudiendo aparecer en ambos
procesos hepatoesplenomegalia como consecuencia de la
hemopoyesis extramedular en hgado y bazo. Excepcionalmente
puede haber una evolucin a la leucemia aguda.
Pregunta 48.- R: 3
La trombocitosis o trombocitemia esencial se considera el menos
grave de los sndromes mieloproliferativos crnicos clsicos, pudiendo
incluso ser asintomtico en fases iniciales. La clnica propia de la enfermedad consiste en trastornos isqumicos por el exceso de plaquetas
sanguneas, no existiendo una necesaria correlacin entre la gravedad
de la clnica y la cifra plaquetaria en sangre. Como el resto de sndromes
mieloproliferativos crnicos, puede cursar con esplenomegalia. Sin
embargo, la esplenectoma no seria un tratamiento de la enfermedad,
ya que la eliminacin del bazo favorecera el incremento en la cifra de
plaquetas. En el tratamiento de esta enfermedad se suele utilizar
hidroxiurea para disminuir la formacin de clulas a nivel medular.
Pregunta 49.- R: 3

Pregunta 44.- R: 3
Las leucemias crnicas (leucemia mieloide crnica, leucemia mielomonoctica crnica, leucemia linfoide crnica) se caracterizan siempre por leucocitosis sangunea, por lo que la existencia de leucopenia
va en contra del diagnstico de leucemia crnica. En las leucemias
agudas, cuando la cantidad de blastos sanguneos es muy escasa,
puede condicionarse leucopenia como consecuencia de la disminucin de produccin de clulas normales por la medula sea. Los
sndromes mielodisplsicos pueden tener tambin leucopenia con
blastos en sangre.
Pregunta 45.- R: 2
La existencia de blastos en sangre siempre es patolgica, siendo
caracterstica de las leucemias agudas. Si el porcentaje de blastos es
superior al 20%, el diagnstico de leucemia aguda es seguro. Ocasionalmente los pacientes con leucemias agudas pueden tener un porcentaje de blastos en sangre inferior al citado, siendo entonces neceM exico A rgentina
C hile U ruguay

Pregunta 49. Fases en la evolucin de la LMC.

CTO Medicina C/ Nez de Balboa, 115 28006 MADRID (Espaa) Tfno.: (91) 782 43 32 / Fax: (91) 782 43 27
E-mail: secretaria@ctomedicina.com; iberocto@ctomedicina.com WEB: www.ctomedicina.com; www.iberocto.com

HM Pg. 5

HEMATOLOGA

Seguimiento a distancia

Preparacin Examen de Seleccin 05/06 1 Vuelta

La leucemia mieloide crnica es un sndrome mieloproliferativo


crnico en el que la evolucin a leucemia aguda (crisis blstica) es
ms frecuente. Un dato de existencia de ms de un 20% de blastos en
sangre perifrica o mdula sea puede definir la crisis blstica, que es
ms frecuente de tipo mieloide que linfoide. Ocasionalmente esta
fase se puede presentar sin una fase crnica previa. El tratamiento de
esta fase blstica es infructuoso en la mayora de las ocasiones y ni
siquiera un trasplante de mdula consigue porcentajes elevados de
curaciones. Los cloromas son masas slidas de mieloblastos leucmicos, que se pueden ver tanto en leucemias agudas mieloides como en
fase blstica de leucemia mieloide crnica.
Pregunta 50.- R: 2
Las leucemias linfoides son caractersticas de las edades extremas
de la vida, siendo la forma aguda propia de nios y la forma crnica
de ancianos. En la edad intermedia de la vida son ms frecuentes las
leucemias mieloides que las linfoides, predominando las formas agudas sobre la crnica.
Pregunta 51.- R: 1
Aunque existen diversos factores pronsticos en pacientes con
leucemia aguda mieloide, edad avanzada, alteraciones citogenticas
desfavorables, existencia de leucemias agudas secundarias, suelen ser
los factores de peor pronstico.

Pregunta 51. Factores de mal pronstico de las leucemias agudas.

Pregunta 54.- R: 4
El rgimen de induccin a la remisin en la leucemia aguda mieloide suele estar constituido por arabinsido de citosina y derivados
de la adriamicina. Los regmenes que incluyen vincristina, prednisona
y derivados de la adriamicina son caractersticos del tratamiento de
induccin de la leucemia aguda linfoblstica. Mercaptopurina y metotrexate son frmacos empleados en el tratamiento de mantenimiento de la leucemia aguda linfoblstica.

1

123456786934646
8
456

9524


624 242 9

67
2 66
688
6 5 296 6! "252&
4"
96 665 "995

569
6
'6 6(95456 665
6!99"2 4

Pregunta 53.- R: 1
La clnica de las leucemias agudas se caracteriza por dos grandes
grupos de manifestaciones. El primero de ellos corresponde a la existencia de escasas clulas maduras en sangre perifrica en las tres series
hematolgicas con los consiguientes: sndrome anmico, hemorragias por trombopenia e infecciones por neutropenia grave. El segundo
grupo de manifestaciones corresponde a la infiltracin por blastos de
diferentes rganos y tejidos. Aunque puede haber afectacin de todo
tipo de rganos y tejidos en las leucemias agudas linfoides es tpica la
afectacin de rganos linfticos (adenopatas, esplenomegalia, masa
mediastnica por crecimiento del timo) y la infiltracin del sistema
nervioso y en nios, el testculo. En leucemias agudas mieloides, los
monoblastos caractersticamente infiltran sistema nervioso, piel o encas. La caracterstica clnica de la CID es tpica fundamentalmente en
la variedad M3.

9524

8
6 6 6!4"454"956#6$6%
6
6 ) 92&6 6*1+
6,  "2965  9!29

Dada la extraordinaria agresividad de las leucemias agudas, con


la consiguiente velocidad evolutiva, la nica forma de conseguir que
un paciente tenga una prolongacin en su supervivencia es que
exista remisin completa con tratamiento. No obstante, la remisin
completa no indica curacin, dado que estos pacientes pueden
tener recidivas.

Pregunta 55.- R: 4
Los derivados del cido retinoico (tetrinona) son frmacos que
facilitan la maduracin de clulas tumorales en la leucemia aguda
promieloctica (M3). Este frmaco, asociado a quimioterapia, produce altos porcentajes de remisin en la leucemia aguda mieloide M3
con la translocacin 15;17.
Pregunta 56.- R: 5
La leucemia linftica crnica es un proceso neoplsico de linfocitos de inmunofenotipo B y apariencia madura, que es el equivalente
leucmico del linfoma linfoctico bien diferenciado. Es una tipologa
relativamente frecuente en pacientes sobre todo de edad avanzada. A
pesar de que las clulas tumorales forman inmunoglobulina, en la
mayora de los casos esta inmunoglubulina queda en la membrana
celular y no es segregada a la sangre, por lo que la mayora de los
pacientes no presentan paraprotena. La enfermedad es de origen
idioptico.

Pregunta 59. Clasificacin de Rye de la enfermedad de Hodgkin

123456789
3 893 3
2

16 5859
849
878939

5

494

Pg. 6 HM

123454673894
46
748
74 6

7525
748
7864
776
67666
2
5
7$
67 47"56675 4
7
8 2672546
7!4 9764547
7767 6#
6 67, 6
M exico A rgentina
C hile U ruguay

238455

894
46

23 9

 74 

238459

7 4
6745946
776
67666


67666


67666


 6
77%&7'()
7*5424
46754 96



6

!44 745
"45#
56
7!48
7
7+ $
77 474575
6

454  7 64
 

CTO Medicina C/ Nez de Balboa, 115 28006 MADRID (Espaa) Tfno.: (91) 782 43 32 / Fax: (91) 782 43 27
E-mail: secretaria@ctomedicina.com; iberocto@ctomedicina.com WEB: www.ctomedicina.com; www.iberocto.com

Comentarios TEST

123
456789
6
2
6
 6
69 9

Pregunta 52.- R: 3
El denominado inmunofenotipo comn, o preB, recibe este nombre por ser la variedad de leucemia aguda linfoblstica ms frecuente
en nios. Afortunadamente es una leucemia de pronstico favorable
que presenta como marcador tpico el CALLA. El mejor pronstico en
nios corresponde entre las edades de 1 y 9 aos, y en adultos, antes
de los 35 aos. La existencia de leucocitosis, que se traduce en una
elevada masa tumoral, al igual que la afectacin del sistema nervioso
central, son factores pronsticos desfavorables.

HEMATOLOGA

Preparacin Examen de Seleccin 05/06 1 Vuelta


Pregunta 57.- R: 5
Los sistemas de estadificacin de la leucemia linftica crnica se
basan en la historia natural de la evolucin de la enfermedad. Dado
que es una enfermedad de nacimiento medular y de escasa agresividad, su fase inicial corresponde a existencia tumoral en mdula sea
y sangre perifrica, siendo en esta fase los pacientes diagnosticados de
forma casual al realizar hemogramas, donde aparece linfocitosis de
apariencia madura. En fases ms avanzadas se produce crecimiento
de organomegalias (adenopatas, esplenomegalia, hepatomegalia) y
en fases avanzadas, como consecuencia del proceso mieloptsico, la
aparicin de anemia y trombopenia, que en el sistema de Rai implica
la fase ms avanzada de la enfermedad. Hay que recordar, sin embargo, que la leucemia linftica crnica puede producir anemia y/o
trombopenia de mecanismo inmunolgico. Cuando la anemia o la
trombopenia son de dicho mecanismo, este hecho no est necesariamente ligado a la masa tumoral.
Pregunta 58.- R: 4
La tricoleucemia o leucemia de clulas peludas es una neoplasia de
clulas linfoides de inmunofenotipo B casi siempre. Clnicamente se
caracteriza por la existencia de pancitopenia y esplenomegalia. Las
clulas tumorales presentan una caracterstica citoqumica (fosfatasa
cida resistente a tartrato) adems de las caractersticas prolongaciones
de la membrana citoplasmtica. La tricoleucemia presenta buena respuesta a diversos frmacos, fundamentalmente la cladribina.

Comentarios TEST

Pregunta 59.- R: 1
La clula de tipo Sternberg es caracterstica de la enfermedad de
Hodgkin, y en la actualidad se considera que es una clula linfoide
activada. Morfolgicamente son clulas de gran tamao con dos o
mas ncleos y nuclolos muy prominentes. Esta imagen morfolgica
puede aparecer en otras patologas distintas de la enfermedad de
Hodgkin, como son linfomas no Hodgkinianos o procesos linfoproliferativos reactivos, por ejemplo en el seno de viriasis. Dependiendo
de la proporcin de estas clulas malignas en relacin con clulas
reactivas acompaantes en las biopsias tisulares, se reconocen cuatro
histologas en la enfermedad de Hodgkin, de las que la esclerosis
nodular se caracteriza por clulas llamadas lacunares. El predominio
linfocitario y esclerosis nodular se han considerado histologas favorables, y la celularidad mixta y deplecin linfocitaria, histologas desfavorables.
Pregunta 60.- R: 2
La existencia de adenopatas cervicales, axilares, mediastnicas y
celacas, es decir, a ambos lados del diafragma, define un estadio clnico
III, con la letra A por la ausencia de sntomas B y el subndice 1, dado
que la localizacin adenoptica abdominal est en la zona alta del
abdomen. La letra S, inicial de la palabra bazo en ingls (Spleen) se
coloca cuando existe esplenomegalia y se traduce en una mayor posibilidad de diseminacin hematgena de la enfermedad. Un estadio IV
tambin se traduce en enfermedad diseminada hematgenamente.
Pregunta 61.- R: 5
Para la mayor parte de las neoplasias malignas, la existencia de
diseminacin tumoral implica la imposibilidad de curacin. Esta norma oncolgica no es aplicable a tumores hematolgicos como
leucemias y linfomas, enfermedades que, estando extendidas, pueden ser curadas.
Pregunta 62.- R: 4
La laparotoma como tcnica de estadificacin de la enfermedad
de Hodgkin se realiza cada vez con menor frecuencia y est ligada a la
necesidad de asegurar la ausencia o presencia de enfermedad abdominal en pacientes que podran ser tratados con radioterapia (estadio
IA y IIA sin masa voluminosa). Dichos estadios pueden tambin tratarse
con quimioterapia, evitndose en este caso la laparotoma, situacin
que es la habitual en la actualidad. No se aconseja utilizar radioterapia
como nico tratamiento en enfermedad diseminada (respuestas 1, 2 y
3). Los pacientes de las respuestas 4 y 5 podran ser tratados con radioterapia o con quimioterapia. Si se elige radioterapia, se necesitara
laparotoma slo en la respuesta 4. El hecho de que el paciente de la
M exico A rgentina
C hile U ruguay

Seguimiento a distancia

respuesta 5, aun recibiendo radioterapia como nico tratamiento no


necesitase laparotoma, se debe a la peculiar forma de diseminacin
por va linftica de la enfermedad de Hodgkin, que sigue un orden
estricto en la afectacin de territorios linfticos. En dicho paciente, la
existencia de enfermedad aparentemente slo en el cuello hace altamente improbable la presencia de enfermedad abdominal, pudiendo
evitar la laparotoma en esta circunstancia.
Pregunta 63.- R: 5
Como resumen teraputico: la radioterapia como nico tratamiento
se puede aplicar en estadios localizados sin sntomas B ni masa voluminosa. Para todos los estadios es posible la administracin de quimioterapia, independientemente de que presenten sntomas B o no,
si bien la existencia de masa voluminosa aconseja tratamiento combinado.
Pregunta 64.- R: 2
El protocolo MOPP es leucemognico, y entre otras complicaciones, produce elevados ndices de esterilidad definitiva, por lo que no es
un tratamiento aconsejable en pacientes jvenes. Por otra parte, el protocolo ABVD se considera en la actualidad como el protocolo de quimioterapia de referencia para esta enfermedad. Este ltimo tratamiento
puede producir como complicaciones cardiotoxicidad por la
adriamicina o patologa pulmonar por la bleomicina, como dicen las
respuestas 1 y 4. La mielitis transversa es un efecto de la radioterapia.
Pregunta 65.- R: 2
La enfermedad de Hodgkin se caracteriza por producir inmunodeficiencia celular ms que humoral. La edad avanzada, lentitud en
la respuesta teraputica y la hipoalbuminemia son caractersticas
oncolgicas generales de mal pronstico. El aumento de la velocidad
de sedimentacin globular es un parmetro sensible para definir actividad en la enfermedad de Hodgkin.

Pregunta 65. Factores de mal pronstico de la enfermedad de


Hodgkin.

1234567879
2 5 7 8 2272
128648 2  42 6 8464
12 2 !8752 58"4 5
12#!8$%
12&
67' 52#
12()2 *75
12& 72' 5!847
12347 8!'4 '4
122+&,
12 8 2 5! 56 2 826 6 '4 67
Pregunta 66.- R: 3
El virus de EPSTEIN-BARR se ha visto implicado en la patogenia de
diferentes linfomas, como es el tipo Burkitt tanto endmico como no
endmico, los linfomas de los inmunodeprimidos (SIDA y postrasplantados) y enfermedad de Hodgkin. El linfoma o leucemia T del
adulto est relacionado con otro virus, el retrovirus HTLV-I.
Pregunta 67.- R: 4
Los linfomas no hodgkinianos de alto grado de agresividad se caracterizan citolgicamente por presentar clulas de gran tamao o
clulas inmaduras. Las clulas inmaduras o precursoras se denominan obviamente blastos (linfoblastos) y algunos esquemas de clasificacin de linfomas denominan tambin blastos a las clulas grandes.
Como consecuencia se pueden considerar linfomas de alta agresividad todos aquellos que terminan en la denominacin blstico, es
decir, presentan clulas precursoras o clulas de gran tamao. La
excepcin es un peculiar linfoma que presenta dos clulas tumorales,
una clula de ncleo hundido y otra clula grande, que en esta clasificacin recibe el nombre de linfoma centroctico-centroblstico y es
de escasa agresividad. El termino inmunocitoma es equivalente al de
linfoma linfoplasmocitoide.

CTO Medicina C/ Nez de Balboa, 115 28006 MADRID (Espaa) Tfno.: (91) 782 43 32 / Fax: (91) 782 43 27
E-mail: secretaria@ctomedicina.com; iberocto@ctomedicina.com WEB: www.ctomedicina.com; www.iberocto.com

HM Pg. 7

Pregunta 68.- R: 2
A diferencia de los linfomas, donde las adenopatas suelen ser las
manifestaciones clnicas ms frecuentes, en el mieloma mltiple las
manifestaciones clnicas predominantes corresponden al hueso. De
hecho, el mieloma mltiple forma parte de los procesos considerados
como tumores seos. Todo paciente hematolgico con dolores seos
o lesiones osteolticas, en preguntas de MIR, debemos considerar que
presenta un mieloma hasta que se demuestre lo contrario.
Hipercalcemia, anemia, proteinuria, insuficiencia renal, infecciones
fundamentalmente respiratorias y urinarias son otras manifestaciones
tpicas del mieloma, asociadas estas ltimas a la inmunodeficiencia
humoral.
Pregunta 69.- R: 4
A diferencia del sistema de estadificacin de ANN-ARBOR de los
linfomas, el sistema de Durie-Salmon del mieloma mltiple no establece extensin tumoral, sino masa. La existencia de una intensa
hipercalcemia es lgicamente propia de mielomas de alta masa tumoral (estadio III). El mieloma en estadio I ha recibido ocasionalmente el nombre de mieloma quiescente y no suele precisar tratamiento.
Pregunta 70.- R: 1
Todos los parmetros bioqumicos citados en las respuestas estn
relacionados con la masa tumoral, pero el que mejor lo hace es la
beta-2-microglobulina srica. Dicha sustancia no es un marcador patognomnico de la enfermedad, dado que se encuentra tambin elevada en otros procesos linfoproliferativos e incluso situaciones distintas como el SIDA o la insuficiencia renal. Por dicho motivo no vale
para asegurar un diagnstico de mieloma mltiple, pero una vez hecho el diagnstico de la enfermedad s sirve para valorar masa tumoral,
existiendo un sistema de estadificacin exclusivamente basado en el
valor de dicha sustancia en suero.
Pregunta 71.- R: 3
A diferencia del mieloma mltiple, que como su nombre indica es
un tumor diseminado que afecta a la mdula sea hemopoytica de
todos los huesos, y que por tanto, si precisa tratamiento, ste debe ser
sistmico, el plasmocitoma suele ser un tumor solitario que cuando
precisa tratamiento puede ser de tipo local, como la radioterapia.
Melfaln o ciclofosfamida con prednisona suelen ser el tratamiento
clsico del mieloma mltiple que permite mantener en buen control
a muchos pacientes, a pesar de no ser tratamientos curativos. Esteroides, interfern, poliquimioterapia, son otros tipos de tratamiento de
la enfermedad, aunque el trasplante de progenitores hematopoyticos es el nico tratamiento curativo.
Pregunta 72.- R: 1
La macroglobulinemia de Waldenstrm suele tener como base
histolgica un linfoma de tipo linfoplasmocitoide, es decir una
neoplasia de escasa agresividad a diferencia del linfoma inmunoblstico citado en la respuesta 1. Es caracterstica de esta enfermedad la existencia de sndrome de hiperviscosidad en relacin con
la paraprotena de tipo IgM plasmtica. En este sndrome se incluyen trastornos neurolgicos, hemorragias y otras alteraciones circulatorias. La IgM monoclonal puede tener actividad de
crioaglutinina.
Pregunta 73.- R: 2
Los linfomas no hodgkinianos de alta agresividad pueden tener
inmunofenotipo B o T, aunque ms frecuentemente son B en nuestro medio. Al tratarse de tumores altamente agresivos presentan
corta supervivencia sin tratamiento, aunque con tratamientos quimioterpicos agresivos o trasplante de progenitores hematopoyticos
se pueden producir curaciones en muchos casos. La laparotoma
no es una tcnica de estadificacin de linfomas no hodgkinianos.
No debe confundirse la laparotoma de estadificacin con la
laparotoma diagnstica, que ocasionalmente es necesaria para
conocer la enfermedad que un paciente con una masa abdominal
presenta.

Pg. 8 HM

M exico A rgentina
C hile U ruguay

HEMATOLOGA

Preparacin Examen de Seleccin 05/06 1 Vuelta

Pregunta 74.- R: 3
Los linfomas no hodgkinianos de escasa agresividad presentan
fases iniciales asintomticas y en estas fases asintomticas los linfomas de escasa agresividad pueden no requerir tratamiento. A diferencia de la enfermedad de Hodgkin, que puede presentar enfermedad localizada fundamentalmente supradiafragmtica, los linfomas no hodgkinianos de baja agresividad se presentan ocasionalmente en el retroperitoneo. A diferencia de los linfomas altamente
agresivos no suelen invadir el sistema nervioso central y son tumores
de pacientes adultos. En estos linfomas, la proporcin de clulas
tumorales en fase proliferativa es escasa, por lo que la respuesta a
los tratamientos es menor que en los linfomas de alta agresividad.

Pregunta 74. Diferencias entre linfomas por su agresividad.


123456728948

4567
12 872567 

12345676
689
6

12
988
528492452
64 2
56
87
12 6
7257
1249
458
2927
12 82898
5
12 96
456724549 45642
6
72 757
22727
2! 
12"
728948

45672  23#$2447
12567 29484564

123466
689
6

12#
4
58
5287 72 6 24252898
57 
229 24%69
56
87
1249
458
2879624 5272 44
12&452944622!
12'7
2! 2$(')22*+('1&

Pregunta 75.- R: 3
Todos los procesos linfoproliferativos malignos de escasa agresividad pueden, con el paso del tiempo, evolucionar a formas histolgica y clnicamente ms agresivas. Cuando esta situacin ocurre en la
leucemia linftica crnica, este hecho recibe el nombre de sndrome de Richter, complicacin que, aunque tpica, es infrecuente.
Pregunta 76.- R: 3
Los linfomas tipo MALT aparecen en el tejido linfoide asociado a
las mucosas, siendo el ms frecuente de localizacin gstrica. Dicho
linfoma est asociado a la infeccin por Helicobacter pylori. Los linfomas gstricos tipo MALT son tumores generalmente de escasa agresividad que pueden desaparecer tras la erradicacin del Helicobacter.
Como otros linfomas indolentes, pueden tener evolucin a linfoma
agresivo. El linfoma de las respuestas 1, 4 y 5 puede estar relacionado
con el EBV.
Pregunta 77.- R: 3
Existen muchos factores de importancia pronstica en linfomas no
Hodgkinianos, y entre ellos se encuentran todas las respuestas de la
pregunta. De hecho, en general se considera que el inmunofenotipo
T en ciertas circunstancias puede tener peor pronstico que el B. Sin
embargo, el llamado ndice internacional de factores pronsticos para
linfomas no Hodgkinianos est basado en factores puramente clnicos, como la edad, el estado de rendimiento, el estadio de la enfermedad, la afeccin extraganglionar y como parmetros analticos la LDH
srica como medida de masa tumoral. En dicho sistema de factores
pronsticos no aparecen datos relacionados con citogentica ni caractersticas inmunofenotpicas.
Pregunta 78.- R: 2
Los linfomas asociados al SIDA son linfomas no Hodgkinianos y
dentro de ellos predominantemente de clulas B y de alta agresividad.
Como linfomas tpicos se encuentran los de clula pequea
(linfoblsticos) de tipo Burkitt o no Burkitt, de clula grande (como el
linfoma cerebral primario) y los linfomas de cavidades o serosas. Es

CTO Medicina C/ Nez de Balboa, 115 28006 MADRID (Espaa) Tfno.: (91) 782 43 32 / Fax: (91) 782 43 27
E-mail: secretaria@ctomedicina.com; iberocto@ctomedicina.com WEB: www.ctomedicina.com; www.iberocto.com

Comentarios TEST

Seguimiento a distancia

HEMATOLOGA

Comentarios TEST

Preparacin Examen de Seleccin 05/06 1 Vuelta

Seguimiento a distancia

Pregunta 80. Hemostasia secundaria.

frecuente la afectacin extralinftica y la diseminacin en el momento del diagnstico. Con frecuencia se encuentran asociados al EBV.
Pregunta 79.- R: 2
La leucemia-linfoma T del adulto se caracteriza por presentar tanto
enfermedad sangunea (leucemia) como tisular (linfoma). Suele ser un
tumor de alta agresividad caracterstico de zonas geogrficas asociadas
al retrovirus HTLV-I, como es el caso de Japn o Caribe,y presenta con
relativa frecuencia caractersticas que se ven frecuentemente en los
mielomas, como es el caso de la hipercalcemia o las lesiones seas.
Pregunta 80.- R: 2
Los factores de coagulacin dependientes de la vitamina K de
sntesis heptica son la protombina, factor II, y los factores VII, IX y
X (ver esquema de pgina siguiente).
Pregunta 81.- R: 4
El tiempo de hemorragia es una tcnica de estudio de hemostasia
primaria, alterndose en patologas de la plaqueta o del vaso sanguneo. La patologa primaria de la hemostasia primaria ms frecuente es
la trombopenia. En ausencia de trombopenia y de frmacos que interfieran con la hemostasia primaria, hay que considerar la enfermedad de Von Willebrand como causa ms probable. El tiempo de
protrombina valora bsicamente la va extrnseca de la coagulacin y
M exico A rgentina
C hile U ruguay

el tiempo de tromboplastina la va intrnseca. El tratamiento anticoagulante oral se controla fundamentalmente con el tiempo de protrombina, no con el de tromboplastina, a diferencia de la heparina
intravenosa.
Pregunta 82.- R: 5
Para que el tiempo de protrombina y el de tromboplastina sean
normales no debe haber alteraciones en los factores especficos de la
va extrnseca y de la va intrnseca, y tampoco en los factores comunes a ambas vas, es decir factor X, factor V, protrombina y fibringeno.
Por tanto, para cumplir las caractersticas del enunciado de la pregunta, la alteracin debe corresponder a un factor que no est incluido
en ninguna de las vas, como es el factor XIII, un factor que no acta
antes de la formacin de la fibrina, sino despus, para impedir su
degradacin. Si queremos conocer el grado de funcionamiento de
dicho factor, no debemos solicitar tiempos de coagulacin, sino tiempo de lisis del cogulo, que se acorta cuando este factor no funciona
adecuadamente o est en cantidad insuficiente.
Pregunta 83.- R: 1
Las trombopenias pueden clasificarse en dos tipos segn su mecanismo: trombopenias centrales, cuando la medula sea no produce
suficiente cantidad de plaquetas, donde estn patologas como la
aplasia, sndromes mielodisplsicos o leucemias agudas y trombope-

CTO Medicina C/ Nez de Balboa, 115 28006 MADRID (Espaa) Tfno.: (91) 782 43 32 / Fax: (91) 782 43 27
E-mail: secretaria@ctomedicina.com; iberocto@ctomedicina.com WEB: www.ctomedicina.com; www.iberocto.com

HM Pg. 9

HEMATOLOGA

Seguimiento a distancia

Preparacin Examen de Seleccin 05/06 1 Vuelta

Pregunta 84.- R: 3
La PTI es una trombopenia perifrica, por tanto con buen funcionamiento medular, de mecanismo inmunolgico (mediado por anticuerpos antiplaquetarios) cuyo diagnstico exige descartar otras causas de
trombopenia como pueden ser frmacos, infecciones como el VIH,
trastornos autoinmunes o procesos linfoproliferativos. Aunque la destruccin plaquetaria en esta enfermedad tiene lugar en el bazo y la
esplenectoma es la forma de curacin definitiva en gran parte de los
pacientes, paradjicamente no se produce crecimiento esplnico.
Pregunta 85.- R: 1
La PTI crnica puede no producir sntomas si la cifra de plaquetas
no est muy disminuida. En dicha situacin se aconseja la abstencin
teraputica y la vigilancia del paciente. En formas sintomticas el tratamiento inicial, como en otros procesos autoinmunes, suelen ser los
esteroides, que permiten el control de parte de los pacientes. En
pacientes que presenten recidivas tras el tratamiento esteroideo o que
necesiten prolongadamente esteroides, hay que valorar la utilizacin
de esplenectoma, que suele ser un tratamiento definitivo en la mayora de ellos. Para pacientes que no mejoran con los anteriores tratamientos se aconseja medicacin inmunosupresora. La gammaglobulina y la transfusin plaquetaria son tratamientos de emergencia para
formas graves sintomticas.
Pregunta 86.- R: 2
La PTT es una trombopenia perifrica por consumo plaquetaria, en la que la disminucin de la cifra de plaquetas es consecuencia de la produccin de trombos generalizados en la microcirculacin, lo cual se suele acompaar de hemlisis de tipo traumtico
con la consiguiente aparicin de esquistocitos. Entre las manifestaciones clnicas predominan los trastornos isqumicos neurolgicos
y la insuficiencia renal, pudiendo aparecer ocasionalmente fiebre.

Pregunta 86. Diagnstico diferencial entre PTT y SHU.

6787
49
7  7
8
8 
59  
 8

8  
9   8

123

455

12345
9
2 


6789
45
1
52
5
14

Pregunta 87.- R:4


Las cinco respuestas de la pregunta son formas de terapia de la PTT,
aunque el tratamiento ms eficaz y por tanto de eleccin es la
plasmafresis. El motivo es que, en esta enfermedad, existen en el
suero multmeros del factor Von Willebrand que favorecen la agresin
plaquetaria al endotelio vascular, como consecuencia de la falta de
degradacin del factor.
Pregunta 88.- R: 3
La enfermedad de Von Willebrand es la trombocitopata ms
frecuente. Esta enfermedad puede transmitirse de forma autosmica
dominante (tipos I y II) o recesiva (tipo III), siendo esta ltima forma la
ms grave. Este factor se sintetiza en megacariocitos, endotelio vascular y lo presentan las plaquetas en sus grnulos. Como otros defectos
de la hemostasia primaria se caracteriza por hemorragias mucocutneas. El acetato de desmopresina es un tratamiento que permite
incrementar la cantidad de factor, y es til bsicamente en el tipo I.

Pg. 10 HM

M exico A rgentina
C hile U ruguay

Pregunta 89.- R: 4
La trombosis vascular es una manifestacin tpica de ciertas enfermedades clonales de la medula sea, como son la hemoglobinuria
paroxstica nocturna, los sndromes mieloproliferativos crnicos, fundamentalmente policitemia vera y trombocitosis esencial de algunas
leucemias agudas, fundamentalmente la LAM-M3. La enfermedad
de Moschcowitz es el nombre de la PTT. La enfermedad de RenduOsler es la telangiectasia hemorrgica hereditaria, que se caracteriza,
como su nombre indica, por hemorragias y no por trombosis.
Pregunta 90.- R: 2
La CID se caracteriza por el consumo de plaquetas y de factores de
coagulacin para la produccin de cogulos de forma generalizada
en la microcirculacin. Como consecuencia de ello se produce
trombopenia y prolongacin de todos los tiempos de coagulacin.
Asociado a este hecho aparece la destruccin traumtica de hemates, el consumo de los inhibidores de la coagulacin (por tanto, descenso de antitrombina III) y la aparicin de PDF consiguientes a la lisis
del cogulo por el sistema fibrinoltico.
Pregunta 91.- R: 4
Todas las alteraciones citadas en las respuestas son tpicas de la
CID. Dado el consumo plaquetario de factores de coagulacin, en la
CID se produce trombopenia y prolongacin de todos los tiempos de
coagulacin, al mismo tiempo que disminucin de la cantidad de
todos los factores de coagulacin y aparicin de PDF. La CID es una
enfermedad bifsica en la que en la primera fase se producen
indiscriminadamente cogulos y en la fase final se produce sangrado
generalizado. Dicho sangrado tiene fundamentalmente relacin con
la desaparicin de los factores de coagulacin, principalmente fibringeno.
Pregunta 92.- R: 3
Las denominadas trombofilias primarias hereditarias son trastornos en el sistema de coagulacin-fibrinolisis que promueven un exceso de actividad coagulante por varios mecanismos. El primer mecanismo puede ser la aparicin de factores de coagulacin en cantidad
excesiva o con caractersticas anormales (disfibrinogenemia, mutacin del gen de la protrombina y factor V Leiden). El segundo mecanismo puede ser la deficiencia de los inhibidores de la coagulacin
como son la deficiencia de antritrombina-III, de protena C o de
protena S. El tercer mecanismo es consecuencia de fallos en el sistema fibrinoltico, bien por deficiencia en su actividad como consecuencia de deficiencia de plasmingeno o por inhibicin de dicho
sistema por un exceso de inhibidores.
Pregunta 93.- R: 4
Las trombofilias primarias hereditarias son procesos congnitos,
generalmente autosmicos dominantes, que ocasionan tendencia
tromboemblica principalmente en pacientes adultos jvenes. Se
debe sospechar ante situaciones de tromboembolismo de causa incierta en adultos jvenes, o mujeres tras uso de anovulatorios orales
y su diagnstico suele hacerse tras la aparicin de clnica, ya que
efectivamente no existen buenas pruebas de laboratorio para despistaje.
Pregunta 94.- R: 4
En las cirrosis hepticas graves puede producirse un trastorno generalizado de la hemostasia. Por una parte puede haber trombopenia
como consecuencia de hiperesplenismo por hipertensin portal. Por
otra parte, y dado que el hgado sintetiza factores de coagulacin que
estn tanto en la va intrnseca como en la va extrnseca pueden
prolongarse el tiempo de protrombina y de tromboplastina. Adems
la sntesis defectuosa de fibringeno puede producir prolongacin
del tiempo de trombina. Dichas alteraciones son similares a las que
aparecen en la CID diferencindose, entre otras cosas, en el hecho de
que en la CID se produce un consumo de todos los factores de la
coagulacin y en la hepatopata disminuyen los factores relacionados
con la sntesis heptica, fundamentalmente dependientes de la vitamina K, fibringeno y factor V, mantenindose el factor VIII en valores
normales.

CTO Medicina C/ Nez de Balboa, 115 28006 MADRID (Espaa) Tfno.: (91) 782 43 32 / Fax: (91) 782 43 27
E-mail: secretaria@ctomedicina.com; iberocto@ctomedicina.com WEB: www.ctomedicina.com; www.iberocto.com

Comentarios TEST

nias perifricas, donde la mdula sea es ajena a la trombopenia. A su


vez, la trombopenia perifrica puede ser consecuencia de destruccin acelerada de plaquetas, como ocurre con las trombopenias
autoinmunes o en el hiperesplenismo; secuestro plaquetario, como
ocurre en la esplenomegalia, o consumo plaquetario en situaciones
de trombosis excesiva, como es la prpura trombtica trombocitopnica (PTT) o la coagulacin intravascular diseminada (CID).

HEMATOLOGA

Preparacin Examen de Seleccin 05/06 1 Vuelta

Seguimiento a distancia

Pregunta 95.- R: 3
En la ictericia obstructiva severa se produce disminucin de aporte
de bilis al duodeno, con la consiguiente malabsorcin grasa. Dado
que la vitamina K se absorbe con las grasas, en estos pacientes se
puede producir una malabsorcin de vitamina K, con los consiguientes problemas hemorrgicos, fundamentalmente tras tratamientos o
medidas diagnsticas invasivas. Para evitar estos problemas, se aconseja la administracin de vitamina K por va parenteral. La deficiencia
de cido flico puede ocasionalmente producir trombopenia, pero
la trombopenia produce un trastorno de la hemostasia primaria, no
de la coagulacin.
Pregunta 96.- R: 4
Las alteraciones de la hemostasia son caractersticas de la leucemia
promieloctica (CID aguda), las neoplasias diseminadas, fundamentalmente adenocarcinomas (CID crnica), o situaciones de hiperviscosidad srica como ocurre en el mieloma y en la enfermedad de
Waldenstrm.
Pregunta 97.- R: 4
El tratamiento con heparina intravenosa produce un efecto anticoagulante, prcticamente inmediato, dado que la heparina, a travs de la antitrombina-III, produce inhibicin de factores de coagulacin ya existentes. Por contra, el tratamiento anticoagulante oral,
produce anticoagulacin diferida, dado que dicha medicacin no
produce inhibicin de la actividad de factores ya existentes sino de
la sntesis de nuevos factores. Por otra parte, el efecto de la heparina
es de breve duracin, por lo que la suspensin de la heparina provoca la desaparicin del efecto anticoagulante en pocas horas, mientras la anticoagulacin oral puede mantener su efecto varios das. En
la situacin que se establece en la pregunta, dado que el paciente
precisa estar anticoagulado por tener una prtesis valvular cardaca
la actitud ms correcta es la que se sugiere en la respuesta 4.

Comentarios TEST

Pregunta 98.- R: 2
El efecto secundario ms frecuente de la medicacin anticoagulante es el sangrado. Un efecto infrecuente, pero que hay que conocer por su gravedad, es la trombocitopenia secundaria a la heparina.
Dicha situacin, paradjicamente, produce trombosis, y es motivada
por la aparicin de anticuerpos antiheparina que atacan la membrana plaquetaria, produciendo trombocitopenia inmunolgica y poniendo, por mecanismos no del todo conocidos en marcha la coagulacin sangunea. Esta situacin exige la suspensin del tratamiento
con heparina y la administracin de medicacin anticoagulante
antignicamente no relacionada con la heparina.
Pregunta 99.- R: 5
La necrosis cutnea por anticoagulantes orales, est mediada por
un fenmeno trombtico de la microcirculacin cutnea en relacin
con deficiencia de inhibidores de la coagulacin, como protena C o
protena S. La explicacin de este fenmeno paradjico tiene que ver
con el hecho de que la vitamina K es necesaria, no slo para la sntesis
de factores de coagulacin sino tambin para la sntesis de inhibidores
de coagulacin como los citados.
Pregunta 100.- R: 2
El INR es la forma habitual de control de medicacin anticoagulante oral. Bsicamente consiste en una relacin entre el tiempo de
protrombina del paciente anticoagulado por va oral y el tiempo de
protrombina normal para el laboratorio, por lo que un INR de 1
implicara una situacin de no anticoagulacin. Cuanto mayor sea el
resultado numricamente del INR ms intensa es la actividad anticoagulante. En la mayor parte de las situaciones, se precisa un INR en el
intervalo de 2-3 para mantener una adecuada anticoagulacin. Hay
situaciones de elevado riesgo procoagulante como la que el paciente
tiene, en donde el INR debera estar entre 3-4.

M exico A rgentina
C hile U ruguay

CTO Medicina C/ Nez de Balboa, 115 28006 MADRID (Espaa) Tfno.: (91) 782 43 32 / Fax: (91) 782 43 27
E-mail: secretaria@ctomedicina.com; iberocto@ctomedicina.com WEB: www.ctomedicina.com; www.iberocto.com

HM Pg. 11

INMUNOLOGA Y GENTICA

Preparacin Examen de Seleccin 05/06 1 Vuelta


1.

La inmunoglobulina predominante en superficies mucosas y


secreciones externas en la superficie humana es:
1)
2)
3)
4)
5)

2.

Para activarse el linfocito T CD4+, necesita la presentacin del


antgeno en el HLA de clase II de la clula presentadora de
antgeno (CPA), y la coestimulacin de otras molculas como la
CD28. Seale cul es el ligando de sta en la CPA:
1)
2)
3)
4)
5)

3.

Preguntas TEST

Inmunoglobulina de superficie.
Marcador CD4.
Receptor para interleuquina 2.
Antgeno HLA-DR.
Marcador CD3.

La estimulacin antignica de un linfocito Th (helper) en ausencia de coestimulacin, provoca en el mismo:


1)
2)
3)
4)
5)

7.

Eosinfilo.
Basfilo.
Linfocito T.
Clula plasmtica.
Mastocito.

El marcador ms caracterstico de los linfocitos T en el hombre


es:
1)
2)
3)
4)
5)

6.

IgA, IgG, IgM.


IgA, IgM, IgG.
IgM, IgA, IgG.
IgM, IgG, IgA.
IgG, IgM, IgA.

Cul de las siguientes clulas produce IgE?:


1)
2)
3)
4)
5)

5.

CD44.
CD7.
B7 (CD80).
ICAM-1.
LFA-2.

La secuencia por la cual las diferentes inmunoglobulinas alcanzan en los nios niveles iguales a los del adulto es:
1)
2)
3)
4)
5)

4.

La forma monomrica de la IgM, unida al componente S


(secretor).
La forma polimrica de la IgM, unida a la cadena J.
Un dmero IgA, unido a la cadena J y al componente S.
Un dmero IgA unido al componente S, sin cadena J.
IgA mono o polimrica, indistintamente, unida al componente S.

Hipersecrecin de citoquinas.
Hipoestimulacin.
Anergia clonal.
Delecin clonal.
Supresin activa.

La resistencia de la IgA secretora a la protelisis enzimtica es


consecuencia de:
1)

La unin del componente secretor alrededor de la regin


bisagra.
M exico A rgentina
C hile U ruguay

2)
3)
4)
5)
8.

La naturaleza dimrica de la IgA secretora.


Actividad antiproteasa del componente secretor.
Presencia de la cadena J.
Actividad antiproteasa de la cadena J.

Seale cul de las siguientes anomalas es la causa de la forma


adquirida del angioedema:
1)
2)
3)
4)
5)

9.

Seguimiento a distancia

C1 inhibidor deficiente cuantitativa o cualitativamente.


Niveles bajos de IL-10.
Dficit de C1.
Deficiencia de IgE.
Aumento de los niveles de IgE.

La respuesta de anticuerpos independiente de clulas T (timoindependiente) es de un slo tipo de inmunoglobulina. Selela:


1)
2)
3)
4)
5)

Ig G.
Ig M.
Ig A.
Ig D.
Ig E.

10. Cul es el dficit de factores del complemento ms frecuente


en la poblacin mundial?:
1)
2)
3)
4)
5)

C1.
C2.
C3.
C5.
C9.

11. Seale cul de estos tipos celulares NO expresa molculas de


HLA DR:
1)
2)
3)
4)
5)

Osteoclasto.
Clulas de Kupffer.
Clulas de Langerhans.
Linfocitos B.
Neutrfilos.

12. Cmo se denominan las inmunoglobulinas que precipitan a


4C y se redisuelven a 37C?:
1)
2)
3)
4)
5)

Factor reumatoide.
Haptenos.
Crioglobulinas.
Piroglobulinas.
Epsilon.

13. Un nio de 4 aos presenta una historia de infecciones pigenas


de repeticin cuya causa fue siempre bacterias encapsuladas
ricas en polisacridos. Se sospecha deficiencia de un tipo de
inmunoglobulina. Cul de las siguientes sera compatible con
ese cuadro?:
1)
2)
3)
4)
5)

IgE.
IgG1.
IgG (todas las subclases).
IgG2.
IgA2.

CTO Medicina C/ Nez de Balboa, 115 28006 MADRID (Espaa) Tfno.: (91) 782 43 32 / Fax: (91) 782 43 27
E-mail: secretaria@ctomedicina.com; iberocto@ctomedicina.com WEB: www.ctomedicina.com; www.iberocto.com

IG Pg. 1

14. Cules son las molculas de membrana que definen el fenotipo


de la poblacin mayoritaria de los linfocitos T cooperadores?:
1)
2)
3)
4)
5)

TcR+, CD3-, CD4+.


TcR-, CD3+, CD4-.
TcR+, CD3+, CD8+.
TcR+, CD3+, CD4+.
TcR-, CD3+, CD4+.

F(ab)2.
Fab.
Fc.
Regin bisagra.
Componente secretor.

16. Cul es el mecanismo de herencia de los alelos del complejo


principal de histocompatibilidad?:
1)
2)
3)
4)
5)

Ligado al X.
Autosmico dominante.
Autosmico codominante.
Autosmico recesivo.
No mendeliano.

17. Un nio con inmunodeficiencia combinada severa es sometido


a un trasplante de mdula sea. Cul es la complicacin ms
importante que puede aparecer tras este tratamiento?:
1)
2)
3)
4)
5)

Regresin tmica precoz.


Retraso en la produccin de anticuerpos tras ser reinmunizado.
Reaccin de injerto contra husped.
Rechazo de tejido trasplantado.
Ausencia de clulas B maduras en la mdula sea trasplantada.

18. Con respecto a la inmunodeficiencia variable comn, es FALSO


que:
1)
2)
3)
4)
5)

Es un cuadro de inmunodeficiencia primaria por dficit de


anticuerpos.
Cursa con infecciones bacterianas de repeticin.
Es tpico encontrar en el laboratorio panhipogammaglobulinemia.
Debuta en el perodo neonatal inmediato.
Puede asociarse a neoplasias, sobre todo a linfomas o a
cncer gstrico.

19. Una caracterstica del sndrome de DiGeorge es:


1)
2)
3)
4)
5)

Anomalas del desarrollo de la 1 y 2 bolsas farngeas.


Hiperplasia de las glndulas paratiroideas.
Hipercalcemia.
Anomalas cardiovasculares.
Evidencia clara de herencia.

20. Indique la afirmacin FALSA acerca de los linfocitos T colaboradores (T helper):

Pg. 2 IG

M exico A rgentina
C hile U ruguay

1)
2)
3)
4)
5)

15. La zona efectora de la molcula de IgG que media el proceso de


opsonizacin es:
1)
2)
3)
4)
5)

INMUNOLOGA Y GENTICA

Preparacin Examen de Seleccin 05/06 1 Vuelta


Se conocen tres tipos de mecanismo efector, denominados
respectivamente Th1, Th2 y Th3.
Cada mecanismo helper lo realiza un tipo de clulas diferente, por lo que, en realidad, hay dos clases de linfocitos T: los
CD4 y los CD8.
Los linfocitos Th1 potencian la actividad de los macrfagos.
Los Th2 potencian la actividad de los linfocitos B.
El tipo concreto de mecanismo efector T que intervenga en
la respuesta inmune ante una bacteria puede modificar el
cuadro clnico que sta produzca.

21. Un nio de 15 meses es ingresado por un cuadro muy grave de


sarampin varios das despus de ser vacunado. En la analtica
llama la atencin la ausencia de linfocitos T en sangre. Este
cuadro es muy sugerente de:
1)
2)
3)
4)
5)

Inmunodeficiencia variable comn.


Dficit de adenosindeaminasa (ADA).
Hipogammaglobulinemia transitoria de la infancia.
Inmunodeficiencia combinada ligada al cromosoma X.
Sndrome de Di George.

22. Una adolescente de 17 aos es ingresada por una neumona


neumoccica severa. En la historia destaca que, pese a haber
tenido una infancia normal, en los ltimos 2 aos ha tenido
cuadros de diarreas intermitentes, otitis supurada crnica y
cuatro neumonas. En la exploracin destaca una moderada
esplenomegalia. Cul de los siguientes diagnsticos encaja
perfectamente con su cuadro clnico?:
1)
2)
3)
4)
5)

Inmunodeficiencia combinada severa.


Dficit de adenosindeaminasa.
Inmunodeficiencia variable comn.
Aplasia tmica (sndrome de DiGeorge).
Sndrome de Wiskott-Aldrich.

23. En cul de los siguientes procesos es imprescindible el papel


de la inmunidad humoral?:
1)
2)
3)
4)
5)

Eliminacin de clulas infectadas por virus.


Bloqueo de toxinas bacterianas.
Bacterilisis.
Fagocitosis.
Estimulacin del receptor de la clula T.

24. Seale cul de las siguientes alteraciones analticas se observa


de modo caracterstico en los pacientes con sndrome de
Bruton:
1)
2)
3)
4)
5)

Neutropenia marcada.
Ausencia de IgG.
Niveles muy bajos de IgM.
Linfocitos B indetectables.
Linfocitos T indetectables.

25. Slo con una de las siguientes molculas es posible la presentacin del antgeno, por parte de los macrfagos, a los linfocitos
T helper 2 CD4+. Indquela:
1)
2)
3)

Inmunoglobulina de superficie.
HLA-A
HLA- DR.

CTO Medicina C/ Nez de Balboa, 115 28006 MADRID (Espaa) Tfno.: (91) 782 43 32 / Fax: (91) 782 43 27
E-mail: secretaria@ctomedicina.com; iberocto@ctomedicina.com WEB: www.ctomedicina.com; www.iberocto.com

Preguntas TEST

Seguimiento a distancia

INMUNOLOGA Y GENTICA

Preparacin Examen de Seleccin 05/06 1 Vuelta


4)
5)

HLA-C.
Receptor de interleuquina 2.

32. En cul de los siguientes trasplantes es MENOS probable que


aparezca enfermedad injerto contra husped?:

26. Cul de los siguientes NO es un rgano linfoide secundario?:


1)
2)
3)
4)
5)

Bazo.
Timo.
Amgdala palatina.
Placas de Peyer.
Ganglios linfticos.

27. A las molculas que son incapaces de inducir una respuesta


inmune, a menos que se asocien a otra molcula portadora, se
les denomina:
1)
2)
3)
4)
5)

Alergenos.
Paratopos.
Haptenos.
Haplotipos.
Idiotipos.

1)
2)
3)
4)
5)

1)
2)
3)
4)
5)

Sndrome de inmunodeficiencia adquirida.


Mieloma mltiple.
Enfermedad granulomatosa crnica.
Hipogammaglobulinemia ligada al sexo.
Inmunodeficiencia combinada severa.

Preguntas TEST

Un fragmento F(ab)2.
Dos fragmentos Fab y un fragmento Fc.
Dos fragmentos Fd.
Un fragmento F(ab)2 y un fragmento Fc.
Un fragmento Fab y un fragmento Fc.

31. Antes de realizar un trasplante renal se realiza la prueba


cruzada donante-receptor, siendo esta positiva. Seale cul
debe ser la actitud a tomar:
1)
2)
3)
4)
5)

1)
2)
3)
4)
5)

Proceder a realizar normalmente el trasplante.


La prueba cruzada positiva invalida la posibilidad de trasplante, por lo que hay que buscar otro receptor para ese rgano.
Administracin de un bolo suplementario de corticoides
inmediatamente despus de la reperfusin del rgano.
Plasmafresis pre-trasplante.
Realizar la inmunosupresin de base con Tacrolimus (FK506) en vez de ciclosporina.
M exico A rgentina
C hile U ruguay

Aparecer en las primeras 24 horas.


Un rea de induracin de al menos 10 mm de dimetro a las
48-72 horas.
Una zona eritematosa de al menos 10 mm de dimetro a las
48-72 horas.
Estar mediada por complejos antgeno-anticuerpo.
Estar mediada por anticuerpos de la clase IgE.

35. Para distinguir entre linfocitos B, linfocitos T y macrfagos se


recurre a ciertas propiedades de cada poblacin celular. Cul
de las siguientes es FALSA?:
1)
2)
3)
4)
5)

Disgenesia reticular.
Deficiencia de HLA de clase II.
Inmunodeficiencia combinada severa ligada a X.
Inmunodeficiencia combinada severa.
Inmunodeficiencia variable comn.

30. El tratamiento de la molcula de IgG con papana rompe la


molcula en:
1)
2)
3)
4)
5)

Trisoma X.
Trisoma 13.
Microdelecin en el brazo largo del cromosoma X.
Microdelecin en el brazo largo del cromosoma 5.
Microdelecin en el brazo largo del cromosoma 22.

34. Una reaccin de hipersensibilidad retardada a un antgeno se


caracteriza por:

29. Cul de las siguientes inmunodeficiencias enumeradas a continuacin presenta mejor pronstico?:
1)
2)
3)
4)
5)

Mdula.
Rin.
Corazn.
Crnea.
Hgado.

33. Cul es el defecto citogentico que suelen presentar los


pacientes con sndrome de Di George?:

28. Un nio de 7 meses con historia de infecciones pigenas fue


hospitalizado por una infeccin severa por hongos. El paciente
tena ausencia de sombra tmica en Rx, hipogammaglobulinemia y falta de linfocitos B. Esta historia sugiere:
1)
2)
3)
4)
5)

Seguimiento a distancia

Los linfocitos T son CD3+, los B y macrfagos no.


Los linfocitos B tienen Ig de superficie, los T no.
Los macrfagos y algunos linfocitos T son CD4+.
Los macrfagos tienen inmunoglobulina de superficie.
Los T se estimulan con concanavalina A y los B no.

GENTICA
36

En cul de las siguientes enfermedades se ha observado


heterogeneidad de locus?:
1)
2)
3)
4)
5)

Fibrosis qustica.
Inmunodeficiencia combinada severa.
Fenilcetonuria.
Corea de Huntington.
Distrofia de Duchenne.

37. En una ecografa de rutina se diagnostica a un paciente poliquistosis renal del adulto. El paciente es varn y tiene cuatro hijos,
tambin todos ellos varones; estadsticamente, cuntos de
ellos sera lgico que sufriesen la enfermedad?:
1)
2)
3)
4)
5)

0.
1.
2.
3.
4.

CTO Medicina C/ Nez de Balboa, 115 28006 MADRID (Espaa) Tfno.: (91) 782 43 32 / Fax: (91) 782 43 27
E-mail: secretaria@ctomedicina.com; iberocto@ctomedicina.com WEB: www.ctomedicina.com; www.iberocto.com

IG Pg. 3

38. A un segmento de ADN cromosmico que contiene toda la


informacin necesaria para sintetizar una cadena polipeptdica
se le denomina:
1)
2)
3)
4)
5)

Opern.
Gen.
Intrn.
Exn.
Trasposn.

39. Seale la proposicin correcta acerca del patrn de herencia


mitocondrial:
1)
2)
3)
4)
5)

La mitad de los hijos varones de una mujer afecta sufrirn la


enfermedad.
Todas las hijas de un varn afecto son portadoras de la
enfermedad.
Las mujeres afectas solo trasmiten la enfermedad a sus hijas.
Ninguno de los hijos de un varn afecto sufrir la enfermedad.
Tanto los varones como las mujeres trasmiten la enfermedad
a la totalidad de sus hijos con independencia de su sexo.

40. Un nio fallece a las pocas horas del nacimiento por una crisis
de apnea; en la exploracin realizada en el momento de nacer,
destaca labio leporino bilateral, polidactilia, microftalma y
dextrocardia. Cul de los siguientes sndromes encaja con el
fenotipo de este nio?:
1)
2)
3)
4)
5)

Sndrome de Edwards (trisoma 18).


Sndrome de ojo de gato (trisoma parcial 22).
Sndrome de Patau (trisoma 13).
Sndrome de Down (trisoma 21).
Sndrome del maullido de gato (monosoma parcial del
cromosoma 5).

41. Cul es la aneuploida que se observa ms frecuentemente en


la especie humana?:
1)
2)
3)
4)
5)

Sndrome de Down.
Trisoma 16.
Trisoma 13.
Trisoma X.
Monosoma X.

3)
4)
5)

No origina patologas.
Es un fenmeno que se produce por la fusin de dos
cromosomas acrocntricos.
La traslocacin afecta a los brazos largos de dos cromosomas.
Un ejemplo es la alteracin cromosmica del sndrome del
maullido de gato.
Aunque se la denomine as, no es una traslocacin sino una
duplicacin de un segmento gnico.

Talasemia.
Corea de Huntington.
Sndrome de Down.

Pg. 4 IG

M exico A rgentina
C hile U ruguay

Sndrome de Prader-Willi.
Linfoma de Burkitt.

44. En el gen responsable de la fibrosis qustica pueden aparecer


diversas mutaciones diferentes que van traducindose todas
ellas en un mismo fenotipo de enfermedad. A esta situacin se
la denomina:
1)
2)
3)
4)
5)

Heterogeneidad clnica.
Heterogeneidad de locus.
Heterogeneidad allica.
Imprinting gnico.
Fenocopia.

45. Cul de los siguientes patrones de herencia de enfermedad es


el ms frecuente en humanos?:
1)
2)
3)
4)
5)

Autosmica dominante.
Autosmica recesiva.
Ligada al X.
Ligada al Y.
Mitocondrial.

46. Un carcter est ligado totalmente al sexo si su locus gnico est


en:
1)
2)
3)
4)
5)

Slo en el cromosoma X.
Slo en el cromosoma Y.
En el segmento diferencial del X solamente.
Un cromosoma localizado muy prximo a los cromosomas
sexuales.
En el segmento diferencial de los cromosomas sexuales.

47. La inactivacin de un cromosoma X en la mujer:


1)
2)

4)

43. Cul de las siguientes patologas puede originarse por la


presencia de una disoma uniparental?:
1)
2)
3)

4)
5)

3)

42. Seale la afirmacin cierta acerca de la traslocacin robertsoniana:


1)
2)

INMUNOLOGA Y GENTICA

Preparacin Examen de Seleccin 05/06 1 Vuelta

5)

Es un fenmeno que se produce en los primeros estadios de


la gametognesis.
La glucosilacin del ADN desempea un importante papel
en dicha inactivacin.
Se considera un mecanismo de compensacin de la dosis
gnica.
Es visible citolgicamente en el ncleo de las clulas femeninas como cuerpo heterocromtico o corpsculo de Taylor.
La inactivacin se produce al azar, por lo que todas las clulas
tienen inactivados el mismo cromosoma X, bien el de origen
paterno o el de origen materno.

48. Indique la relacin correcta entre las siguientes enfermedades


y sus mecanismos de herencia:
1)
2)
3)
4)
5)

Neuropata ptica de Leber: ligada al X recesiva.


Raquitismo resistente a vitamina D: ligada al X recesiva.
Enfermedad de Huntington: autosmica recesiva.
Enfermedad de Hunter: autosmica recesiva.
Sndrome del cromosoma X frgil: expansin de secuencias.

49. Seale la respuesta FALSA acerca de las traslocaciones Robertsonianas:


1)
2)

Se trata de la fusin de dos cromosomas acrocntricos.


Explican un porcentaje de casos de sndrome de Down.

CTO Medicina C/ Nez de Balboa, 115 28006 MADRID (Espaa) Tfno.: (91) 782 43 32 / Fax: (91) 782 43 27
E-mail: secretaria@ctomedicina.com; iberocto@ctomedicina.com WEB: www.ctomedicina.com; www.iberocto.com

Preguntas TEST

Seguimiento a distancia

INMUNOLOGA Y GENTICA

Preparacin Examen de Seleccin 05/06 1 Vuelta


3)
4)
5)

Seguimiento a distancia

Al ser balanceadas, no originan alteraciones en la descendencia.


Son importantes para realizar consejo gentico.
Influyen en el riesgo de recurrencia de cromosomopatas.

50. En un paciente afecto de una enfermedad de Marfan, cul es


la probabilidad de que al menos uno de sus progenitores tenga
la enfermedad?:
1)
2)
3)
4)

Preguntas TEST

5)

Prcticamente un 100%.
50%.
Los dos padres deben de ser portadores asintomticos.
La probabilidad es baja, dada la alta incidencia de mutaciones espontneas que aparecen en este tipo de herencia.
25%.

M exico A rgentina
C hile U ruguay

CTO Medicina C/ Nez de Balboa, 115 28006 MADRID (Espaa) Tfno.: (91) 782 43 32 / Fax: (91) 782 43 27
E-mail: secretaria@ctomedicina.com; iberocto@ctomedicina.com WEB: www.ctomedicina.com; www.iberocto.com

IG Pg. 5

INMUNOLOGA Y GENTICA

Preparacin Examen de Seleccin 05/06 1 Vuelta

Comentarios TEST

Pregunta 1.- R: 3
La IgA es una IG ampliamente distribuida, la podemos encontrar
tanto en el suero como en las secreciones. Es la predominante en las
secreciones externas: tubo digestivo y bilis, epitelios respiratorios, leche y calostro, saliva, lgrimas, y flujo vaginal.
Aunque la IgA srica es mayoritariamente monomrica, la IgA de
las secreciones es fundamentalmente dimrica, est formada por dos
molculas de IgA unidas covalentemente (puente disulfuro) con la
cadena J y asociada a un polipptido conocido por componente
secretor (CS).
El CS se sintetiza en las clulas epiteliales de la mucosa correspondiente y se sita en el polo basal de la membrana celular, donde acta
como un receptor para IgA dimrica sintetizada por las clulas B de la
submucosa. La IgA se une al receptor e inmediatamente es endocitada
y transportada por el interior de la clula (transcitosis). Cuando llega a
la zona de la membrana celular de la luz del epitelio, el receptor se
escinde y queda libre la IgA unida a un fragmento del mismo (el CS).
La respuesta 1 es falsa porque la IgM no es estable en las secreciones: se degrada inmediatamente. Adems la IgM humana soluble es
pentamrica y slo predominan las formas monomricas en situaciones patolgicas, como la cirrosis biliar primaria.
La respuesta 2 es falsa porque aunque la cadena J se asocia a IgM,
como vimos en el comentario anterior, esta molcula prcticamente
no se ve en las secreciones.
La respuesta 4 y 5 son falsas porque, como ya se ha comentado, la
IgA de secreciones es dimrica y para serlo necesita tener cadena J.
Pregunta 2.- R: 3
El linfocito T slo puede reconocer su antgeno especfico si le es
presentado por otra clula en el interior de una molcula del complejo principal de histocompatibilidad (HLA). Cuando lo reconoce, se
establece un dilogo molecular entre ambas clulas: presentadora
de antgeno y linfocito T, que puede acabar en la activacin del linfocito o la anergia clonal.
De todas las interacciones que tienen lugar en el proceso de activacin del linfocito T, la ms importante es la B7-CD28. Se produce
entre las molculas B7 (CD80, CD86) situadas en la clula presentadora de antgeno y la CD28 del linfocito T CD4+. Regla mnemotcnica: CD4 x B7 = CD28 (4x7=28).
Si se produce esta coestimulacin, que se comporta como una
segunda seal de activacin, el proceso de la respuesta inmune continua, pero si la interaccin no se lleva a cabo, por ejemplo, al no estar
presente la molcula B7 en la superficie de la clula presentadora, el
linfocito T no slo no se activar, sino que acabar entrando en
apoptosis y se desarrollar un estado de tolerancia a ese antgeno que
se denomina anergia clonal.
La nica respuesta que podra plantear alguna confusin es la 2
(CD7), pero la interaccin B7-CD28 es tan importante (la han preguntado varias veces en el MIR) que slo con leerla debemos identificarla
a la primera.

Pregunta 2.

Restriccin histocompatible.

M exico A rgentina
C hile U ruguay

Seguimiento a distancia

Pregunta 3.- R: 4
La primera inmunoglobulina elaborada por el propio organismo
es la IgM, se empieza a producir a partir de los 6 meses y medio de
gestacin. La segunda en aparecer es la IgG, que lo hace ya despus
del nacimiento, en torno a los 7-15 das de vida. IgA aparece ms
tardamente en torno al segundo mes de vida. Tngase en cuenta que
durante los primeros meses de vida la inmunoglobulina mayoritaria es
la IgG de origen materno.
Pregunta 4.- R: 4
Las nicas clulas capaces de producir inmunoglobulinas son los
linfocitos B, y las clulas que derivan de la maduracin de los mismos
(plasmticas). Cuando leemos en una pregunta las siglas IgE, inmediatamente la asociamos con los cuadros clnicos de la alergia, al igual
que los eosinfilos, basfilos o clulas cebadas, pero estas clulas NO
producen inmunoglobulinas.
Pregunta 5.- R: 5
Durante el proceso de maduracin de los linfocitos en el timo
adquieren el receptor antignico especfico de la clula T que les va a
permitir identificar su antgeno. Varias molculas se asocian a dicho
receptor fundamentalmente la denominada CD3 (presente en todos
los linfocitos T).
Adems de CD3 y el receptor antignico, los linfocitos T perifricos se
caracterizan por expresar las siguientes molculas de superficie: CD2 (la
responsable de la formacin de rosetas con hemates de carnero), CD5,
CD7 y adems uno de los siguientes (pero no los dos a la vez):
CD4: los linfocitos T CD4+ son los que reconocen antgenos presentados junto con el HLA de clase II. La mayor parte de los CD4+
desarrollan funciones colaboradoras (HELPER), aunque tambin
existen T CD4+ con actividad citotxica (el 10%).
CD8: los linfocitos T CD8+ reconocen antgenos presentados junto con el HLA de clase I. La mayora son citotxicos, pero tambin
existen T8 colaboradores. Los CD4 predominan sobre los CD8 en
una relacin 2:1.
La inmunoglobulina de superficie (Respuesta1) slo se expresa en
los linfocitos B. EL HLA-DR (Respuesta 4) se expresa en monocitosmacrfagos y linfocitos B. En los T slo se expresa, por un perodo de
tiempo muy corto, durante la activacin del linfocito.
El receptor para interleuquina 2 (Respuesta 3) no es exclusivo de
los linfocitos T.
Pregunta 6.- R: 3
La teora de la segunda seal en la presentacin antignica a los
linfocitos T es uno de los temas preferidos en el MIR en los ltimos
aos, y por eso existen en este test varias preguntas con un cierto
parecido.
Recordad que el reconocimiento de un antgeno por parte de un
linfocito T necesita que le sea presentado por otra clula en el interior
de una molcula del complejo principal de histocompatibilidad (HLA)
y adems otra serie de seales complementarias.
Si se produce coestimulacin, la segunda seal de activacin, el
proceso sigue su curso y el linfocito acabar desarrollando una respuesta inmune frente a ese antgeno. No obstante, si la segunda seal
est ausente, por ejemplo al no estar presente la molcula B7 en la
superficie de la clula presentadora, en vez de una respuesta inmune,
se desarrolla un estado de tolerancia a ese antgeno que se denomina
anergia clonal.
La delecin clonal es el proceso que tiene lugar en el timo para
eliminar los linfocitos autorreactivos.
Pregunta 7.- R: 1
La nica inmunoglobulina que es capaz de mantenerse estable en
las secreciones externas es la IgA debido a su resistencia a la degradacin enzimtica . La IgA dimrica de las secreciones tiene unido el
componente secretor (CS), que oculta las zonas de la inmunoglobulina ms sensibles a la accin de las enzimas (regin bisagra), actuando
como un escudo protector.
Recordad que el CS es una protena de unos 80 kD que es la parte
extracelular (que se liber por protelisis) de un receptor de membrana situado en clulas epiteliales, con gran afinidad para la Fc de la IgA

CTO Medicina C/ Nez de Balboa, 115 28006 MADRID (Espaa) Tfno.: (91) 782 43 32 / Fax: (91) 782 43 27
E-mail: secretaria@ctomedicina.com; iberocto@ctomedicina.com WEB: www.ctomedicina.com; www.iberocto.com

IG Pg. 1

INMUNOLOGA Y GENTICA

Preparacin Examen de Seleccin 05/06 1 Vuelta

. El receptor original tiene un peso molecular de 100 kD y es sintetizado y expresado abundantemente en la membrana de las clulas
epiteliales de las mucosas.
Pregunta 8.- R: 1
El cuadro clnico denominado angioedema o edema angioneurtico se debe a la presencia de niveles muy bajos de C1-inhibidor. Es la
deficiencia de factores del complemento ms frecuente en los pases
desarrollados. Aunque generalmente, se trata de una enfermedad de
herencia autosmica dominante, tambin existen formas adquiridas,
entre las que destacan dos: la asociada a sndromes linfoproliferativos
B y la autoinmune debida a un autoanticuerpo contra el C1-inh que
impide su accin biolgica.
La clnica consiste en la aparicin de edemas de forma aguda y
recurrente. El edema est circunscrito al tejido subcutneo y las
mucosas y las localizaciones ms frecuentes son cara y cuello, extremidades, genitales, laringe y tracto gastrointestinal. El tratamiento es
preventivo con andrgenos atenuados (danazol) que aumentan la
sntesis intraheptica de C1-inh. El C1-inh purificado slo se debe
emplear en crisis agudas y graves.
Pregunta 9.- R: 2
La respuesta inmune humoral (secrecin de anticuerpos) puede ser:
1. Timodependiente (depende de clulas T). En ella todo el proceso
es controlado por lo linfocitos T colaboradores tipo 2 (Helper 2:
TH2). Es la mayoritaria, y por ello cuando se nos hace referencia a
la respuesta de anticuerpos, lo habitual es que se hable slo de la
dependiente de clulas T. Existen dos formas de respuesta:
A) Primaria: aparece la primera vez que se entra en contacto con
un antgeno y como caractersticas principales destacan que:
- Tarda unos cuatro das en comenzar.
- La inmunoglobulina secretada es IgM.
B) Secundaria: es la que aparece cuando ha habido previamente
un contacto con el antgeno (das, meses o aos antes).
- La inmunoglobulina secretada puede ser IgG, IgA, IgD y/o IgE.
- Es rpida debido a que la desarrollan linfocitos de memoria.
La vacunacin se basa en generar linfocitos de memoria para
antgenos concretos.
2. Timoindependiente (independiente de clulas T). No es necesaria
la colaboracin de los linfocitos TH2. Se trata de una respuesta
frente a productos polimricos de difcil degradacin metablica,
que no existen en el organismo humano en condiciones fisiolgicas. Esta respuesta siempre es de clase IgM, puesto que los linfocitos

B responsables de la misma, al no tomar contacto con los TH2, no


pueden cambiar de clase de inmunoglobulina (este proceso es
regulado por los linfocitos TH2). Los principales antgenos son
lipopolisacrido, flagelina y polmeros de d- aminocidos.
Pregunta 10.- R: 5
La deficiencia de factores de complemento ms frecuente a nivel
mundial es la de C9. Es especialmente abundante en Japn, China y
otros pases de Extremo Oriente; no obstante dicha deficiencia es
asintomtica. En los pases desarrollados, la deficiencia ms frecuente
es la de C1 inhibidor. Entre el resto de las deficiencias de componentes de la VA CLSICA, la ms frecuente es la deficiencia de C2 y en
algunos libros hacen especial hincapi en la misma, por lo que conviene recordar que, en general, en las deficiencias de la fase inicial de
la va clsica la sintomatologa ms frecuente es la aparicin de infecciones pigenas y cuadros pseudoautoinmunes, como artritis reumatoide-like o lupus-like (cuadro similar al LES, generalmente con
negatividad de los anticuerpos antinucleares y anti-ADN, pero con
anticuerpos contra el antgeno Ro).
La deficiencia de los componentes terminales (complejo de ataque a membrana), del C5 al C9, se asocia a infecciones de repeticin
por neisserias, tales como las septicemias gonoccicas o la meningitis
meningoccica de repeticin y excepcionalmente enfermedades por
inmunocomplejos.
Pregunta 11.- R: 5
Las molculas HLA de clase I, se encuentran en la membrana de
prcticamente todas las clulas nucleadas y plaquetas. No se expresa
el HLA de clase I clsico (HLA-A, HLA-B y HLA-C) en hemates, sincitiotrofoblasto y algunos escasos timocitos.
Las molculas HLA de clase II (HLA-DR, HLA-DP y HLA-DQ) estn
presentes solamente en la superficie de las clulas presentadoras de
antgenos (monocitos, macrfagos, Von-Kupfer, Langerhans, microgla, etc.) y los linfocitos B.
Comentario a las respuestas falsas:
Los neutrfilos solo tienen HLA de clase I (respuesta 5).
Los osteoclastos, como macrfagos que son, s expresan HLA de
clase 2 (respuesta 1).
Pregunta 12.- R: 3
Las crioglobulinas son inmunoglobulinas que precipitan reversiblemente con el fro (4C) y se redisuelven a 37C. Es posible detectar
en el suero de los enfermos con estados de hipocoagulabilidad (trata-

Pregunta 10. Complejo de ataque a membrana.

Pg. 2 IG

M exico A rgentina
C hile U ruguay

CTO Medicina C/ Nez de Balboa, 115 28006 MADRID (Espaa) Tfno.: (91) 782 43 32 / Fax: (91) 782 43 27
E-mail: secretaria@ctomedicina.com; iberocto@ctomedicina.com WEB: www.ctomedicina.com; www.iberocto.com

Comentarios TEST

Seguimiento a distancia

INMUNOLOGA Y GENTICA

Preparacin Examen de Seleccin 05/06 1 Vuelta


dos con cumarinas), un precipitado similar al colocar el suero a 4C,
pero estos ltimos se diferencian de la crioglobulinemia real porque
el precipitado no se redisuelve a 37C.
Las crioglobulinas suelen aparecer en enfermos que padecen
mielomas o linfomas, infecciones y cuadros autoinmunes, aunque
algunas veces aparecen sin patologa asociada y carecen de relevancia. Pueden desencadenar cuadros graves, entre los que destacan el
fenmeno de Raynaud, la prpura vascular, la urticaria inducida por
el fro y la trombosis arterial distal con gangrena.
Comentario a las respuestas falsas:
Las piroglobulinas son inmunoglobulinas monoclonales que precipitan irreversiblemente a 56C. El 50 % corresponden a pacientes de
mieloma mltiple. El resto son pacientes lpicos o con linfomas.
El factor reumatoide es un anticuerpo que reconoce a la IgG del
propio paciente, generalmente son IgM, pero puede ser cualquier
clase. Aunque la presencia en pequeas cantidades no es un dato
relevante y puede darse en personas normales, unos niveles altos
de FR es tpico que aparezca en la artritis reumatoide.
Los haptenos son los antgenos incompletos incapaces de despertar, por s solos, una respuesta inmune. Slo se detectan si previamente se asocian a otra molcula portadora.
Pregunta 13.- R: 4
Los antgenos polisacridos pueden ser neutralizados por la respuesta de anticuerpos T independiente (IgM) o T dependiente, en
cuyo caso el anticuerpo ms importante es la IgG2. La IgM no est
dentro de las respuestas posibles, por lo que la nica opcin que nos
queda es elegir la respuesta 4 (IgG2).
Pregunta 14.- R: 4
Todos los linfocitos T, por definicin, tienen en su membrana el
receptor antignico de la clula T (TcR) asociado a la molcula CD3.
Por tanto las respuestas 1, 2 y 5 se descartan inmediatamente. Queda
por decidir cual de las dos que restan, 3 y 4, es la verdadera. Si el
enunciado de la pregunta no incluyese el trmino poblacin mayoritaria, ambas seran ciertas porque existen linfocitos T colaboradores tanto CD4+ como CD8+. Pero al incluirlo y puesto que la inmensa mayora de los linfocitos colaboradores son CD4+, queda perfectamente claro que la correcta es la respuesta 4.

Comentarios TEST

Pregunta 15.- R: 3
La diseccin enzimtica de la IgG en la regin bisagra (la ms sensible a la accin enzimtica) nos permite obtener diversos fragmentos
cada uno con una funcin concreta.

Pregunta 15. Digestin enzimtica de inmunoglobulina.

Si digerimos con papana (enzima extrada de la papaya) se obtienen 3 fragmentos:


Un fragmento Fc (Fraccin Comn o cristalizable), formado por las
dos mitades carboxiterminales de las cadenas pesadas. Las funciones comunes a todas las inmunoglobulinas, independientemente
de a qu antgeno estn dirigidas, tales como la opsonizacin, el
paso a travs de placenta o la fijacin del complemento, estn locaM exico A rgentina
C hile U ruguay

Seguimiento a distancia

lizadas en esta fraccin comn. Por tanto la respuesta correcta es la


3.
Dos idnticos llamados Fab (respuesta 2), iniciales de las palabras
inglesas Fraction - Anti Body. Cada uno de ellos contiene la zona de
la molcula responsable de la unin al antgeno. En realidad una
Fab est formada por una cadena ligera y la mitad aminoterminal
de una cadena pesada.
Digiriendo con pepsina obtenemos la F(ab)2 (respuesta 1), que se
trata de una fraccin antibody bivalente, es decir con capacidad de
unir dos antgenos, y diversos restos peptdicos.
La regin bisagra, situada aproximadamente es una zona tremendamente mvil de la cadena pesada que contiene los puentes disulfuro que unen entre s las dos cadenas pesadas.
Pregunta 16.- R: 3
Esta pregunta es mixta de Inmunologa y gentica. Conviene recordar algunos conceptos para entender perfectamente el significado del
enunciado:
Alelos son las distintas formas en las que nos podemos encontrar la
expresin de un gen polimrfico. Por ejemplo, en el caso de los grupos sanguneos humanos, podemos observar tres posibles alelos distintos que son A, B y O. El 80 % de los genes no presentan variabilidad
de una persona a otra, es decir son idnticos para todas las personas.
El 20 % restante varan de unos a otros pero generalmente el producto
que codifica cada alelo vara de los otros slo en uno o dos
aminocidos.
Si las dos copias de un gen determinado, situadas en un cromosoma procedente de cada progenitor, son iguales, el individuo es HOMOCIGOTO (homo=igual), si son distintos el individuo es HETEROCIGOTO (heteros=distinto).
Comportamiento de los alelos: el responsable del fenotipo del
individuo es el grado de expresin de cada alelo respecto a su compaero. Los alelos se definen como dominantes, recesivos o codominantes.
Dominante: para que aparezca el fenotipo es necesario que el
alelo responsable del mismo est presente en uno solo de los 2
cromosomas homlogos. En los heterocigotos, el alelo recesivo
queda silenciado por el dominante. Ejemplo de alelo dominante
es el grupo sanguneo A respecto del O.
Recesivo: slo se expresa el fenotipo de un alelo si est en los dos
cromosomas. Si el otro alelo es dominante, el heterocigoto para un
alelo recesivo no lo expresa. As el genotipo de grupo sanguneo
AO expresa el fenotipo A (O es recesivo, A es dominante).
Codominantes. Los dos alelos tienen la misma fuerza para expresarse, y se expresan a la vez. El heterocigoto para los dos alelos, por
tanto, tendr un fenotipo mezcla de los fenotipos de cada alelo
por separado. Ejemplo: fenotipo AB de grupo sanguneo.
El sistema HLA es completamente codominante, y puesto que adems es muy polimrfico, el estudio de la herencia del mismo es muy
empleado en las pruebas de paternidad. En un individuo siempre
deben expresarse la mitad de los alelos HLA del padre, y si no lo
hacen, dado que es imposible que un HLA se convierta en recesivo, se
puede afirmar la no paternidad.
Conviene recordar que las pruebas de paternidad, ya sean con
HLA o DNA, slo sirven para excluir paternidad; la afirmacin de la
paternidad nunca se puede hacer con una certeza del 100 %.
Pregunta 17.- R: 3
La respuesta 1 (regresin tmica) no tiene nada que ver con un
trasplante de mdula sea. La respuesta 2 no constituye en s ninguna
complicacin. Las respuestas 4 y 5 indicaran que el injerto no ha sido
satisfactorio, pero desde luego no son complicaciones tan graves como
la reaccin de injerto contra husped, que es, sin duda, la peor de las
complicaciones posibles (respuesta 3).
La alorreactividad consiste en la identificacin como extraas de
las clulas de otro individuo de la misma especie y la eliminacin
posterior de las mismas. Ocurre cuando los linfocitos T de un individuo se ponen en contacto con antgenos de histocompatibilidad diferentes a los propios, caso de un injerto, detectando las diferencias y
respondiendo como si stas se debieran a la modificacin que resulta
de la incorporacin de un antgeno extrao a las molculas de

CTO Medicina C/ Nez de Balboa, 115 28006 MADRID (Espaa) Tfno.: (91) 782 43 32 / Fax: (91) 782 43 27
E-mail: secretaria@ctomedicina.com; iberocto@ctomedicina.com WEB: www.ctomedicina.com; www.iberocto.com

IG Pg. 3

histocompatibilidad propias, considerando pues a la clula portadora como si estuviese infectada por un virus.
Debido a la alorreactividad puede producirse tanto el rechazo de
rganos como la enfermedad de injerto contra husped. Esta ltima
consiste en un rechazo del injerto al cuerpo que lo acoje, y para que
se desencadene, son necesarias tres condiciones simultneas:
1. Trasplante de rganos entre personas no histocompatibles.
2. El receptor debe ser una persona inmunocomprometida.
3. En el rgano a trasplantar debe haber linfocitos T viables.
Los poqusimos linfocitos T que estn localizados en el rgano
trasplantado, aunque numricamente inferiores, son funcionalmente
muy superiores al conjunto de todo el sistema inmune del organismo
receptor y acaban destruyndolo.
Pregunta 18.- R: 4
La inmunodeficiencia variable comn es un sndrome de inmunodeficiencia humoral adquirido que suele debutar en la adolescencia
y que est caracterizado por una panhipogammaglobulinemia (las
inmunoglobulinas totales suelen estar por debajo de 500 mg/dL). En
esta enfermedad existe una respuesta de anticuerpos muy dbil lo
que se manifiesta en infecciones bacterianas de repeticin, especialmente respiratorias y digestivas. Existe un nmero normal de linfocitos
B que, in vitro, la mayora de ellos son incapaces de secretar
inmunoglobulinas o lo hacen de forma deficitaria.
Son, por tanto, completamente verdaderas las respuestas 1, 2 y 3.
Asimismo, al igual que el resto de las inmunodeficiencias primarias, se
asocia a una mayor incidencia de tumores, siendo el linfoma no
Hodgking el ms frecuente (respuesta 5 cierta).
Lo que es falso es la edad de aparicin: no aparece en los primeros
meses de vida, sino que debuta a partir de la adolescencia.
Pregunta 19.- R: 4
El sndrome de Di George consiste en una serie de malformaciones
congnitas mltiples que se deben a un anormal desarrollo embrionario de los rganos derivados del tercero y el cuarto arcos farngeos
(la respuesta 1 es falsa) y que afectan fundamentalmente a timo, grandes vasos (la respuesta 4 es verdadera) y glndulas paratiroideas (la
respuesta 2 es falsa).
En la gran mayora de los pacientes se detecta una microdelecin
en el brazo largo del cromosoma 22 (22q11) sin que se conozcan
formas heredadas (la respuesta 5 es falsa).
La clnica de estos enfermos est relacionada con la aplasia o hipoplasia de los rganos afectados: la primera manifestacin a poco de
nacer es la tetania debida a la hipocalcemia (la respuesta 3 es falsa),
que se instaura por la disfuncin de las glndulas paratiroideas (hipoparatiroidismo).
Las anomalas del sistema inmune son variables. La ausencia total
de linfocitos T es rara, y en estos casos, la clnica es la de una inmunodeficiencia combinada severa. Lo habitual es que haya un nmero
bajo de linfocitos T (linfopenia T) y la clnica est relacionada con el
grado de disminucin de la respuesta inmune celular. La presencia de
linfocitos T en la mayora de los enfermos se debe a que la aplasia
tmica total es muy poco frecuente; en la biopsia de estos enfermos es
comn encontrar restos tmicos o tejido tmico ectpico, por lo que
siempre queda algo de funcin inmune celular.
En los casos de ausencia total de timo, las manifestaciones clnicas
son las de gentica.
Otros datos clnicos: hipertelorismo, micrognatia, anomalas en los
grandes vasos e implantacin baja de los pabellones auriculares.
Pregunta 20.- R: 2
Actualmente se acepta que existen tres tipos de linfocitos T colaboradores TH (Helper en ingls): TH1, TH2 y TH3 (respuesta 1 verdadera).
Estos linfocitos, ya sean CD4+ o CD8+, regulan la respuesta inmune ofreciendo su colaboracin, en forma de citoquinas, a otras clulas del sistema inmune. El criterio para clasificarlos son las citoquinas
que secretan cada uno de ellos; las fundamentales son:
TH1 producen IL2 e interfern gamma y controlan las respuestas
de inmunidad celular estimulando a macrfagos y otras clulas T
(respuesta 3 correcta).
Pg. 4 IG

M exico A rgentina
C hile U ruguay

INMUNOLOGA Y GENTICA

Preparacin Examen de Seleccin 05/06 1 Vuelta


TH2 producen IL4, e IL6 y colaboran en las reacciones de inmunidad humoral: linfocitos B y clulas plasmticas (respuesta 4 correcta).
TH3 producen IL10 y TGF Beta y se les considera linfocitos reguladores.
Del tipo de respuesta de linfocitos colaboradores que se desarrolle
frente a un antgeno depende que dicha respuesta concluya en el
desenlace de la enfermedad o en aparicin de formas ms severas de
la misma (respuesta 5 correcta).
La respuesta 2 es falsa, ya que en ella se nos reduce el tipo de
linfocitos T solamente a dos, aunque es cierto que los dos tipos bsicos son los CD4+ y los CD8 positivos, pero stos a su vez se subdividen en otros subgrupos (colaboradores, citotxicos) y estos otros tambin se subdividen.... Adems, la pregunta nos est dando pistas para
que sepamos que la respuesta 2 es incorrecta: la propia respuesta 1
nos est diciendo que, slo entre los T colaboradores, hay tres tipos
(existe una gran variedad de fenotipos de linfocitos T).
En definitiva, la respuesta al decir que solo existen dos tipos de
linfocitos T simplifica excesivamente la clasificacin de un conjunto
de clulas tan heterogneo como los linfocitos T.
Pregunta 21.- R: 4
Las respuestas 1 (inmunodeficiencia variable comn) y 3 (hipogammaglobulinemia transitoria de la infancia), las descartamos inmediatamente. Estas enfermedades consisten en una deficiencia de anticuerpos y nunca dan una clnica tan severa como la descrita en el
enunciado, destacando, por el contrario, las infecciones respiratorias
de repeticin con un nmero de linfocitos T normales.
Las otras tres respuestas s pueden dar una clnica como la descrita,
de inmunodeficiencia combinada severa (ICS). La clave para distinguir una de otra y por tanto acertar la correcta est en la ausencia de
linfocitos T. Descartamos la deficiencia de ADA (respuesta 2) al tratarse
de un problema metablico de la clula que difcilmente acaba en
ausencia de clulas.
Ahora se nos ocurrira que la respuesta es el sndrome de Di George
por la atrofia del timo, que impedira la formacin de clulas T, pero
no lo es porque, como vimos en la pregunta 19, es muy raro que en
este cuadro haya ausencia total de linfocitos T.
Sin embargo, en la inmunodeficiencia combinada severa ligada al
cromosoma X, la ms frecuente de las ICS, s que es tpico encontrar la
ausencia de linfocitos T, por lo que sta es la respuesta verdadera (la 4).
Pregunta 22.- R: 3
Las infecciones respiratorias de repeticin, junto con el sndrome
diarreico intermitente, son las dos principales sintomatologas de los
sndromes de deficiencia de anticuerpos. Otras sintomatologas son
las otitis y sinusitis crnicas. Queda claro, por tanto, que nuestra enferma tiene una deficiencia de anticuerpos, por tanto descartamos inmediatamente las respuestas 1, 2 y 4, que se tratan de sndromes de
inmunodeficiencia combinada severa, que se caracterizan por infecciones de repeticin por hongos y virus, adems de la sintomatologa
descrita anteriormente.
La edad de aparicin (15 aos) nos lleva a descartar el sndrome de
Wiskott-Aldrich, que aparece en el primer ao de vida. La inmunodeficiencia variable comn (respuesta correcta), como ya vimos en otra
pregunta, es un sndrome de inmunodeficiencia humoral adquirido
que suele debutar a partir de la adolescencia.
Pregunta 23.- R: 2
En todas las situaciones descritas en las respuestas, las inmunoglobulinas no tienen ninguna funcin (respuesta 5) o bien realizan la
funcin sealada de modo paralelo a otros mecanismos del sistema
inmune, siendo, en cierta manera, prescindibles.
As es posible eliminar clulas infectadas por virus (respuesta 1) con
clulas T citotxicas o NK adems de con anticuerpos. La bacterilisis
(respuesta 3) puede realizarse fijando el complemento por la va clsica (con anticuerpos) o directamente por la va alterna. La fagocitosis
(respuesta 4) se puede realizar directamente o gracias a la opsonizacin
con anticuerpos.
La nica funcin que realizan los anticuerpos sin que se conozca
por el momento ningn otro mecanismo inmune paralelo, es la neu-

CTO Medicina C/ Nez de Balboa, 115 28006 MADRID (Espaa) Tfno.: (91) 782 43 32 / Fax: (91) 782 43 27
E-mail: secretaria@ctomedicina.com; iberocto@ctomedicina.com WEB: www.ctomedicina.com; www.iberocto.com

Comentarios TEST

Seguimiento a distancia

INMUNOLOGA Y GENTICA

Preparacin Examen de Seleccin 05/06 1 Vuelta


tralizacin de toxinas (botulnica, tetnica, colrica, pertussis, etc.) Por
tanto, la nica respuesta correcta es la 2.
Pregunta 24.- R: 4
La agammaglobulinemia ligada al sexo (sndrome de Bruton) fue la
primera inmunodeficiencia primaria descrita. Se trata de una deficiencia pura de anticuerpos, por lo que las respuestas 1 y 5 quedan
automticamente descartadas.
El gen causante de la enfermedad, situado en el cromosoma X
(como la hemofilia, slo los varones tienen la enfermedad) codifica
una tirosinquinasa (tirosinquinasa de Bruton) expresada en clulas B y
que es necesaria para la maduracin de los linfocitos B.
Caractersticamente, los enfermos carecen de linfocitos B (respuesta 4 verdadera) y de clulas plasmticas. Las dems poblaciones linfocitarias son normales.
Al no haber linfocitos B, los enfermos no tienen nada de IgM (respuesta 3 es falsa, para ser cierto deber decir: IgM indetectable).
Paradjicamente, estos enfermos que carecen de linfocitos B s
tienen IgG (paradoja de Bruton), por tanto la respuesta 2 es falsa.

Comentarios TEST

Pregunta 25.- R: 3
Como ya se ha visto en otras preguntas, los linfocitos T slo son
capaces de reconocer antgenos si stos le son presentados por otras
clulas junto con el complejo principal de histocompatibilidad (molculas HLA). Existen dos tipos de linfocitos, atendiendo a su modo de
detectar los antgenos: los CD4+ reconocen antgenos presentados
junto con el HLA de clase II y los CD8+ reconocen antgenos en el
contexto de molculas HLA clase I.
En el enunciado de la pregunta se nos habla de los CD4+, por
tanto debemos buscar entre las respuestas aquella que contenga un
HLA de clase II. Como regla mnemotcnica, los HLA de clase I tienen
una letra (A, B, C) y los de clase II, dos letras (DR, DP, DQ).
Se descartan las respuestas 1 y 5 por no ser molculas HLA y las 2
y 4 por describir HLA de clase I (una letra). La nica respuesta posible
es, por tanto, la 3.
Pregunta 26.- R: 2
Se denominan rganos linfoides primarios a aquellos en los que se
originan y maduran las clulas del sistema inmune y rganos linfoides
secundarios a aquellos donde se disponen los linfocitos maduros, e
inmunolgicamente competentes. Es en stos ltimos donde se producen las respuestas inmunitarias frente a los estmulos antignicos.
Son rganos linfoides primarios la mdula sea y el timo y secundarios los ganglios linfticos, el bazo y el tejido linfoide asociado a las
mucosas (placas de Peyer, anillo linftico de Waldeyer, etc.).
Las respuestas 1, 3, 4 y 5 hacen referencia, sin ninguna duda, a
rganos linfoides secundarios. Sin embargo, la respuesta 2 (Falsa) hace
referencia al timo, que es un rgano linfoide primario.
Pregunta 27.- R: 3
Esta es una pregunta en la que debemos tener muy claros los principales conceptos de inmunologa bsica y que vamos a recordar:
Epitopo: tambin llamado determinante antignico, es la regin
concreta de un antgeno donde se une el anticuerpo. Un antgeno
suele tener varios epitopos distintos.
Haptenos: son sustancias qumicamente sencillas y de bajo peso
molecular, que por s solas no son inmungenas, pero que pueden
comportarse como tales si se unen covalentemente a otra molcula ms grande (portador o carrier), por lo que algunos autores los
denominan antgenos incompletos. Esta es la respuesta correcta
(3).
Idiotipo: definindolo de una manera grfica y fcilmente
recordable, es la zona del anticuerpo que determina su actividad,
la que interacta directamente con el epitopo del antgeno. En una
definicin ms formal (pero ms fcil de olvidar), es el conjunto de
determinantes antignicos (Idiotopos) situados en las regiones variables de las cadenas pesada y ligera de una inmunoglobulina.
Paratopo: son las zonas concretas del idiotipo responsables del
reconocimiento del determinante antignico, los aminocidos que
interactan con la molcula del antgeno existen. Este ltimo concepto ya no se considera tan importante como hace unos aos y
M exico A rgentina
C hile U ruguay

Seguimiento a distancia

sera difcil que cayese en un examen; aqu est entre las respuestas
falsas con la nica intencin de hacer bulto para rellenar las 4
falsas.
Un haplotipo es un conjunto de genes que estn muy juntos en un
cromosoma y que, por lo general, se heredan como una unidad.
As el conjunto de genes del HLA se considera un haplotipo.
Pregunta 28.- R: 5
La edad del nio y la gravedad de la sintomatologa nos hace
sospechar una inmunodeficiencia primaria y por tanto descartar el
SIDA (ID secundaria a infeccin por VIH). Esta decisin de descartarlo
viene avalada porque en ningn momento se nos habla de positividad de ninguna prueba ni de antecedentes de la madre.
La respuesta 2 la descartamos por los mismos motivos, la edad y
una clnica no compatible.
La respuesta 3 la descartamos porque en ningn sitio nos hacen
referencia a las infecciones por microorganismos catalasa positivos,
caractersticas de esta enfermedad.
La respuesta 4 (sndrome de Bruton) tambin la descartamos porque en ella habra infecciones respiratorias de repeticin y diarreas
intermitentes como sintomatologa ms marcada. Quiz el dato de la
hipogammaglobulinemia y la ausencia de linfocitos B nos hace sospecharla, pero las infecciones severas por hongos, como es el caso, no
aparecen en el Bruton.
Se trata de una inmunodeficiencia combinada severa (respuesta 5
correcta), ya que fallan tanto la inmunidad celular (ausencia de timo
e infecciones severas por hongos) como la humoral (ausencia de
linfocitos B e hipogammaglobulinemia).
Pregunta 29.- R: 5
La disgenesia reticular es la ms grave inmunodeficiencia que se
conoce (respuesta 1 falsa). Los nios que la padecen no tienen ningn
leucocito en la sangre y mueren a las pocas horas de vida, a no ser que
se realice trasplante de mdula sea intratero.
La deficiencia de HLA de clase II constituye una de las formas de la
enfermedad denominada inmunodeficiencia combinada severa (ICS),
de la que hablamos ms adelante.
Las respuestas 3 y 4 hacen referencia a la ICS ligada al X (la forma
ms frecuente) y al sndrome de ICS en general. Se trata de uno de los
cuadros clnicos de inmunodeficiencia ms graves al fallar tanto la
inmunidad humoral como la celular. El pronstico de estos enfermos
era infausto hasta que apareci el trasplante de mdula sea. Actualmente con dicho trasplante recuperamos la funcin inmune, pero no
siempre se llega a la curacin al 100 % porque este tipo de enfermedades suelen deberse a deficiencias metablicas que inciden especialmente en el sistema inmune, pero que afectan tambin a otros
rganos, a los que el trasplante de mdula sea no les es til.
La inmunodeficiencia variable comn (respuesta 5 cierta) es una
deficiencia de anticuerpos que se puede controlar muy bien con
administracin parenteral de inmunoglobulinas, lo que permite que
los pacientes tengan una esperanza de vida slo algo menor que la de
otra persona sana de su edad. Este descenso en la esperanza de vida se
debe, como en todas las inmunodeficiencias, a una mayor incidencia
de tumores que la poblacin general. Por tanto, la menos mala de
las enfermedades aportadas en las respuestas es la ID variable comn.
Pregunta 30.- R: 2
Ya hicimos una pregunta en la que abordamos la diseccin enzimtica de la IgG, pero debido a su importancia y a que ltimamente
parece que se ha vuelto a poner de moda, hemos abordado este
tema con esta otra pregunta con un enfoque diferente a la anterior.
Recordemos que la digestin enzimtica con papana se realiza en
la regin bisagra y nos da varios fragmentos:
Un fragmento Fc (Fraccin Comn), formado por las dos mitades
carboxiterminales de las cadenas pesadas. Aqu estn las zonas de
la molcula responsables de la opsonizacin, el paso de placenta
y la fijacin del complemento.
Dos fragmentos idnticos llamados Fab, que quieren decir FractionAnti Body. Cada uno de ellos es capaz de unirse por s solo a un
antgeno.
Lo que acabamos de recordar est reflejado en la respuesta 2.

CTO Medicina C/ Nez de Balboa, 115 28006 MADRID (Espaa) Tfno.: (91) 782 43 32 / Fax: (91) 782 43 27
E-mail: secretaria@ctomedicina.com; iberocto@ctomedicina.com WEB: www.ctomedicina.com; www.iberocto.com

IG Pg. 5

Del resto de las respuestas, la nica con sentido es la 1, que refleja


el producto de la diseccin con pepsina. El resto de las respuestas se
han rellenado como respuestas falsas inventadas, para hacer bulto.
Pregunta 31.- R: 2
Las pruebas cruzadas inmunolgicas se realizan para detectar, en
la sangre de una persona a la que se plantea trasplantar un rgano,
anticuerpos contra antgenos de histocompatibilidad situados en la
membrana de las clulas del rgano del donante. Si estos anticuerpos
existen, est contraindicado el trasplante porque, de hacerse, estos
anticuerpos se fijaran en las primeras clulas que tomaran contacto
con la sangre (endoteliales) y fijaran sobre ellas el complemento destruyndolas. Al destruirse el endotelio, quedara la membrana basal
de los vasos sanguneos en contacto directo con el plasma, desencadenando progresivamente: agregacin plaquetaria, coagulacin
intravascular, hipoxia del rgano y coagulopata de consumo.
El nico tratamiento posible, por tanto, es el preventivo: no trasplantar.
La presencia de los citados anticuerpos se debe a inmunizaciones
previas frente a antgenos de histocompatibilidad alognicos, debidas
a transfusiones de sangre completa o plaquetas, y embarazos (slo en
mujeres).
Pregunta 32.- R: 4
Ya tratamos en otra pregunta el importante tema de la reaccin de
injerto contra husped (EICH) tras un trasplante. Recordemos que son
necesarias tres circunstancias:
Trasplante de rganos entre personas no histocompatibles.
Receptor del trasplante seriamente inmunodeprimido.
Que existan linfocitos T inmunocompetentes en el rgano trasplantado.
En los trasplantes de mdula, el receptor es una persona
inmunocomprometida; es el trasplante donde la probabilidad de que
aparezca la EICH es mayor (falsa). En los de rganos macizos, como
rin, hgado o corazn, aunque el receptor no sea una persona
inmunodeprimida, siempre quedan linfocitos viables del donante en
el rgano y existe la posibilidad, aunque sea remota, de aparicin de
la EICH (respuestas 2, 3 y 5 falsas) . Pero en el trasplante de crnea,
nunca hay linfocitos del donante y, por tanto, es imposible que se
desencadene dicha enfermedad.
Pregunta 33.- R: 5
El sndrome de Di George es un cuadro clnico consistente en un
anormal desarrollo embrionario de los rganos derivados del tercer y
el cuarto arcos farngeos que da lugar a malformaciones congnitas
mltiples, entre las que destacan la atrofia o hipotrofia del timo, las
glndulas paratiroideas y los grandes vasos de cuello. En la mayor
parte de los enfermos, el cuadro se asocia a una microdelecin en el
brazo largo del cromosoma 22 (22q11).
La clnica que presentan los enfermos est relacionada con el grado de aplasia o hipoplasia de estos rganos, siendo la tetania, debida
a la hipocalcemia por la disfuncin de las glndulas paratiroideas
(hipoparatiroidismo), la primera manifestacin clnica que generalmente presentan estos enfermos. Otros signos clnicos asociados al
sndrome son el hipertelorismo, la micrognatia y la implantacin baja
de los pabellones auriculares.
Sistema inmune: no es frecuente que aparezcan pacientes con
sndrome de Di George que presenten una ausencia total de linfocitos
T, ya que la aplasia tmica total es muy poco frecuente. En estos
escassimos casos las manifestaciones son las de una inmunodeficiencia combinada severa.
Lo habitual es que estos pacientes presenten una hipotrofia del
timo ms o menos marcada o bien tengan tejido tmico ectpico, lo
que supone que casi siempre va a haber funcin tmica en mayor o
menor grado. La mayora de las pacientes, por tanto, presentan una
linfopenia T, ms o menos marcada, y la clnica est relacionada con
el grado de disminucin de la respuesta inmune celular.
Pregunta 34.- R: 2
Coombs y Gell realizaron en 1963 una clasificacin de los mecanismos inmunopatognicos que luego fue modificada en 1975 y que
establece las denominadas reacciones de hipersensibilidad:
Pg. 6 IG

M exico A rgentina
C hile U ruguay

INMUNOLOGA Y GENTICA

Preparacin Examen de Seleccin 05/06 1 Vuelta


Reaccin de tipo I: anafilctica, basada en anticuerpos IgE.
Reaccin de tipo II: citotxica. Anticuerpos que fijan el complemento sobre clulas.
Reaccin de tipo III: lesin por depsito de inmunocomplejos
antgeno-anticuerpo.
Reaccin de tipo IV: retardada, de tipo tuberculnico, mediada por
clulas. A su vez se subdivide en cuatro tipos:
- Reaccin de Jones-Mote. Comienza a las 12-24 h y dura 24 h.
- Dermatitis por contacto, aparece en torno a las 24 h de la exposicin y dura 48-72 h.
- Tipo tuberculina, comienza a las 24 h y dura 72-96 h (es la tipo
IV ms caracterstica).
- Granuloma. Comienza a los 7-14 das y dura varias semanas.
Las pruebas cutneas de hipersensibilidad retardada son el medio
ms simple y eficaz para valorar clnicamente la respuesta inmune
celular frente a un antgeno. Para poder evaluar adecuadamente la
respuesta inmune celular de un individuo mediante estas pruebas, se
deben utilizar, al menos, seis antgenos diferentes (p. e.: candidina,
coccidiocidina, antgeno de estafilococo, tuberculina, estreptoquinasa-estreptodornasa y dermatofitina).
Dependiendo de la cantidad de antgeno inoculada, ser tanto
mayor el grado de reaccin, pero por lo general, la dosis de antgeno
se ajusta para que en una persona normal se considere positiva si se
observa una induracin de ms de 9 mm, 48 h despus de la inyeccin del antgeno.
La negatividad de las pruebas debe ser tratada con cautela, y en
caso de darse, se deben repetir con una mayor concentracin de
antgeno antes de poder afirmar que existe anergia cutnea.
Las pruebas cutneas en nios de corta edad son de interpretacin
dudosa.
Por tanto, en nuestra pregunta, la respuesta 1 es falsa (sera a partir
de las 48 h). La 3 es falsa porque lo que se mide es la induracin y no
el eritema. Las respuestas 4 y 5 son falsas porque este tipo de reaccin
est mediada por clulas y no por anticuerpos.
Pregunta 35.- R: 4
Esta pregunta es un magnfico recordatorio de las caractersticas
que diferencian entre s a las principales clulas del sistema inmune.
La respuesta 1 es cierta, slo los linfocitos T tienen receptor de la
clula T (TcR), el cual se expresa en la membrana de estas clulas
asociado a la molcula CD3, cuya funcin es transmitir la seal de
activacin al interior de la clula cuando el TcR reconoce al antgeno.
Todos los linfocitos T, independientemente de que su receptor sea
alfa-beta (el 95 % de los de sangre perifrica) o gamma-delta, tienen en
su membrana CD3.
La respuesta 2 es cierta, los linfocitos B tienen inmunoglobulina en
la membrana celular.
La respuesta 3 es cierta, efectivamente, adems de un subgrupo de
los linfocitos T (el mayoritario), tambin los monocitos y macrfagos
son CD4+, y por eso son infectados por el VIH.
La respuesta 4 es FALSA, solamente tienen inmunoglobulina de
superficie los linfocitos B. Los macrfagos no tienen IG de superficie,
ni siquiera las tienen las clulas plasmticas (las productoras de grandes cantidades de inmunoglobulinas solubles).
La respuesta 5 es cierta, es un poco difcil, pero el que haya una
nica respuesta con una dificultad mayor no hace compleja el
resto de la pregunta. La falsedad de la respuesta 4 es evidente. El
objeto de la pregunta es dejar bien asentados conceptos fundamentales de inmunologa: todas las clulas con CD3 son linfocitos
T y todas las clulas con inmunoglobulina de superficie son
linfocitos B.
GENTICA
Pregunta 36.- R: 2
Hace aos, cuando se descubri el origen gentico de algunas
enfermedades, se pens que cada gen, si se alteraba, poda dar lugar
a una enfermedad y que, a su vez, cada enfermedad estara originada
por un solo gen. Segn se fue estudiando el genoma, se comprob
que la gentica humana no era tan sencilla y surgi el concepto de
heterogeneidad gentica para tratar de explicar anomalas como, por
ejemplo, que una enfermedad puede estar originada por alteraciones

CTO Medicina C/ Nez de Balboa, 115 28006 MADRID (Espaa) Tfno.: (91) 782 43 32 / Fax: (91) 782 43 27
E-mail: secretaria@ctomedicina.com; iberocto@ctomedicina.com WEB: www.ctomedicina.com; www.iberocto.com

Comentarios TEST

Seguimiento a distancia

INMUNOLOGA Y GENTICA

Preparacin Examen de Seleccin 05/06 1 Vuelta


en genes diferentes. El concepto de heterogeneidad gentica ha ido
evolucionando con los aos y por ello, en preguntas de MIR de los
aos 80, las respuestas que fueron correctas en su momento, ahora
no lo seran.
Actualmente, el concepto de heterogeneidad gentica (HG) tiene
un sentido ms amplio: se dice que existe HG cuando una enfermedad se puede originar por diferentes mecanismos genticos tales como
mutaciones, deleciones, interacciones gnicas, heterogeneidad de
locus, allicas, etc. Es decir, viene a significar algo as como enfermedades con gentica compleja y dentro de ella se engloban tres tipos
de heterogeneidad.
La heterogeneidad de locus se aplica a las situaciones en las que
alteraciones en genes distintos (locus diferentes) pueden dar un cuadro clnico idntico. Un ejemplo lo son la retinitis pigmentaria o la
inmunodeficiencia combinada severa (a esto era a lo que en los aos
80 se le llamaba heterogeneidad gentica).
Son falsas las respuestas 1, 3, 4 y 5 porque slo se conoce un locus
(gen) responsable de esas enfermedades, pero es cierta la respuesta 2
porque la inmunodeficiencia combinada severa se puede deber a
mutaciones en genes diferentes, incluso en cromosomas distintos (formas autosmicas y ligadas al X).

Comentarios TEST

Pregunta 37.- R: 3
La poliquistosis renal del adulto es una de las enfermedades hereditarias ms frecuentes. Se estima que, en Espaa, su frecuencia es
mayor que la de la hipercolesterolemia familiar.
El mecanismo de herencia es autosmico dominante, es decir, que
la presencia de una de las dos copias gnicas alterada va a dar lugar a
la enfermedad.
Conviene recordar que, en las enfermedades con herencia autosmica recesiva, es preciso que estn alteradas las dos copias para que se
manifieste el cuadro clnico. Al ser autosmicas (autosomas son todos
los cromosomas con excepcin de los sexuales), los dos sexos tienen la
misma probabilidad de padecer y transmitir la enfermedad.
La mayora de las enfermedades dominantes muestran dos caractersticas: edad tarda de aparicin y expresin clnica variable.
Patrn de herencia:
1. Transmisin vertical: todos los individuos afectados han heredado
el cuadro de un progenitor tambin afectado, excepto los
escassimos casos debidos a mutaciones espontneas. La probabi-

Seguimiento a distancia

lidad de que un caso concreto se deba a mutacin espontnea es


muy baja (por debajo de 0,00001).
2. No hay portadores sanos, slo existen enfermos (aunque no tengan clnica en el momento del diagnstico) y sanos.
3. Afecta a ambos sexos por igual y un enfermo tendr un 50 % de
hijos afectados y un 50 % de hijos sanos.
4. Los hijos sanos de un enfermo slo tendrn hijos sanos.
Pregunta 38.- R: 2
Se llama GEN o CISTRON al segmento de ADN que contiene la
informacin para sintetizar una cadena polipeptdica. El genoma (el
conjunto de genes de un individuo) se reparte en 46 molculas lineales de ADN, organizadas en torno a un esqueleto proteico y que se
denominan cromosomas. Los distintos genes de un cromosoma se
sitan linealmente en el ADN, uno detrs de otro. Solemos hablar
habitualmente de la protena como producto de la expresin de un
gen, pero en realidad deberamos hablar del pptido producto de ese
gen, puesto que una protena puede estar formada por varias cadenas
peptdicas (polipptido).
Un opern (respuesta 1, falsa) es un segmento de ADN que contiene la informacin para producir varias protenas distintas (relacionadas funcionalmente), se transcribe a un nico ARNm, ste se traduce
a una protena que luego es escindida en varias protenas que tienen
acciones independientes (por ejemplo, enzimas con diferentes
substratos). Un ejemplo de opern es el Lac de la bacteria E. coli, que
codifica tres enzimas de la ruta metablica de la lactosa.
Respuestas 3 y 4 (falsas): la informacin contenida en los genes
eucariotas no es continua, sino que se encuentra repartida en varios
segmentos de ADN codificante (exones), interrumpidos por segmentos de ADN no codificante (intrones). El nmero de exones e intrones
vara de un gen a otro. Conviene recordar que los genes procariotas
no contienen intrones.
Pregunta 39.- R: 4
La herencia mitocondrial es un tema que ha sido preguntado frecuentemente en el MIR y que, por tanto, debemos tener muy claro.
Las mitocondrias son orgnulos que tienen su propio ADN (16,5
kb) con un cdigo gentico distinto del que utiliza el ADN nuclear.
Se considera que las mitocondrias son endosimbiontes: bacterias
primitivas que se fusionaron simbiticamente con una clula pri-

Pregunta 42. Translocacin robertsoniana, un portador asintomtico puede tener hijos con trisoma.
M exico A rgentina
C hile U ruguay

CTO Medicina C/ Nez de Balboa, 115 28006 MADRID (Espaa) Tfno.: (91) 782 43 32 / Fax: (91) 782 43 27
E-mail: secretaria@ctomedicina.com; iberocto@ctomedicina.com WEB: www.ctomedicina.com; www.iberocto.com

IG Pg. 7

mordial, tienen ribosomas (70S como los bacterianos) y sntesis


proteica propia.
En la fecundacin, durante la formacin del cigoto, el ovocito
aporta el proncleo femenino y todo el citoplasma de la nueva
clula. Por el otro lado, el espermatozoide slo aporta el proncleo
masculino, por tanto las mitocondrias se heredan siempre de la
madre.
Las alteraciones en el ADN mitocondrial (ejemplo, la neuropata
ptica atrfica de Leber) dan lugar a enfermedades genticas que se
heredan en lnea directa materna, es decir, una madre enferma trasmitir la enfermedad a todos sus hijos e hijas, y un padre enfermo no
se la trasmitir a ninguno (respuesta 4 correcta).
Pregunta 40.- R: 3
Las trisomas consisten en la existencia de 2n+1 cromosomas, existiendo por tanto uno de ms. Cuando afectan a los cromosomas
sexuales se traducen en retraso mental y alteraciones en la conducta,
y cuando afectan a los autosomas producen gravsimas malformaciones congnitas que suelen impedir el normal desarrollo embrionario.
Se han observado trisomas de todos los pares excepto del 1. Slo
pueden llegar al final de la gestacin y sobrevivir al parto una pequea
proporcin de los afectos de trisomas de los pares 13 y 18 y una
proporcin algo mayor de los del par 21. No obstante, los primeros
sobreviven slo algunas horas o das y a la edad adulta slo llegan los
pacientes del Sndrome de Down (21).
Como sntomas ms destacados de las principales trisomas, completas o parciales, citemos:
Sndrome de Edwards: el pie en mecedora y el occipucio prominente (respuesta 1, falsa).
Trisoma parcial del 22: coloboma del iris y atresia anal (respuesta
2, falsa).
Trisoma del 13: polidactilia y labio leporino (respuesta 3, verdadera) y tambin dextrocardia y microftalma.
La Trisoma del 21 (sndrome de Down): no necesita descripcin
(respuesta 4, falsa).
La respuesta 5 (sndrome del maullido de gato) no corresponde a
una trisoma sino a una monosoma (falta de un cromosoma) parcial.
Las monosomas puras, con excepcin del sndrome de Turner (cromosoma X), son incompatibles con la vida. Entre las trisomas parciales preguntables en el MIR estn la enfermedad del maullido de gato
(cromosoma 5) y el sndrome de Di George (cromosoma 22).
Pregunta 41.- R: 5
El nmero euploide de cromosomas de la especie humana es 46
(diploide). Las anomalas cromosmicas numricas suponen una variacin (ganancia o prdida) del nmero euploide:
Se dice que hay poliploida cuando la clula tiene un nmero de
cromosomas diferente de 46, pero mltiplo de 23 (triploide, 69;
tetraploide, 92).
Por otro lado, se dice que existe aneuploida cuando una clula
tiene un nmero de cromosomas distinto del euploide, pero que
no es mltiplo de 23. Son pues aneuploidas las trisomas (ganancia de un cromosoma) y las monosomas (prdida de un cromosoma).
Las trisomas, consideradas en conjunto, son las aneuploidas ms
frecuentes en la especie humana; no obstante, y si se considera por
separado a todas las trisomas y monosomas, la aneuploida ms frecuente es la monosoma del cromosoma X (sndrome de Turner), y en
segundo lugar, la trisoma del 16.
A veces nos cuesta entender cmo el Turner, menos frecuente en
clnica que el sndrome de Down, sea la aneuploida ms frecuente
en humanos. Debe tenerse en cuenta que cuando se habla de la
incidencia de estas patologas en la especie humana se considera
tambin a los embriones afectados que mueren intratero: la inmensa mayora de los afectados por el Turner no llegan a trmino.
Entendiendo esto tambin nos explicaremos por qu la trisoma ms
frecuente en la especie humana es la del cromosoma 16, y sin embargo nunca hayamos visto un enfermo con dicha patologa. La
trisoma del 16 produce tan graves malformaciones que los afectados mueren a las pocas semanas de desarrollo embrionario y ninguno llega a trmino.
Pg. 8 IG

M exico A rgentina
C hile U ruguay

INMUNOLOGA Y GENTICA

Preparacin Examen de Seleccin 05/06 1 Vuelta


Pregunta 42.- R: 2
La traslocacin robertsoniana es una anomala cromosmica estructural. Puesto que esta patologa ha sido preguntada bastantes veces en el MIR, conviene que antes de seguir hagamos una introduccin que nos sirva de recordatorio.
Las anomalas cromosmicas pueden ser de dos tipos: numricas
(las vimos en la pregunta anterior) y estructurales.
Estas ltimas consisten en una reordenacin de la posicin de los
genes sobre los cromosomas y las dos variedades ms frecuentes, y por
tanto preguntables en el MIR, son las deleciones y traslocaciones.
Delecin: es la prdida de un segmento cromosmico, y por tanto,
de los genes situados (se trata de una monosoma parcial). Si la
delecin es grande o los genes en ella situados son importantes, es
incompatible con la vida.
Traslocacin: se produce una delecin en dos cromosomas, y la
reparacin se hace de modo ineficiente, intercambindose los segmentos. Se la denomina tambin traslocacin balanceada o recproca.
La traslocacin robertsoniana es una situacin intermedia entre las
anomalas numricas y estructurales, se produce por la fusin de dos
cromosomas acrocntricos (se trasloca el cromosoma entero).
La respuesta 1 es falsa porque, al fusionarse los dos cromosomas, el
portador de la traslocacin tendr 45 (en realidad, un cromosoma es
doble). El portador ser fenotpicamente normal (no falta ningn gen),
pero tras la meiosis sus gametos tendrn la informacin gentica de 22
o 24 cromosomas, dependiendo de si lleven o no el cromosoma
doble, pudiendo dar lugar, tras la fecundacin, a embriones
monosmicos o trismicos.
La respuesta 3 es falsa porque afecta a la prctica totalidad del
cromosoma. La respuesta 4 es falsa porque la enfermedad del maullido de gato es una microdelecin. La 5 es falsa porque no es una
duplicacin, sino una fusin.
Pregunta 43.- R: 4
Conviene recordar el concepto de disoma uniparental: la persona que tiene la disoma ha heredado las dos copias de un cromosoma concreto solamente de uno de sus progenitores, y no hereda el
cromosoma homlogo del otro padre. Tendr, por tanto, un cariotipo normal: 46 cromosomas, 24 procedentes de un progenitor y 22
del otro. Si en estos cromosomas no existen genes con imprinting no
habr ningn problema: existen casos en los que el padre le trasmite
al hijo varn los cromosomas X e Y y la madre no trasmite el cromosoma X. Estos casos slo se detectan en las muy poco frecuentes situaciones en las que el padre padece una enfermedad ligada al X, como la
hemofilia A, y que, debido a la disoma uniparental, se trasmite de
padres enfermos a hijos varones.
Pero si algunos de los genes situados en el cromosoma estn
sujetos a imprinting gnico, la cosa se complica.
El imprinting consiste en que, de determinados genes, situados en
cualquier cromosoma, solamente se usa la copia procedente de uno
de los dos progenitores y la otra copia se inactiva.
Las dos enfermedades ms caractersticas (y ms preguntables) debidas a la alteracin de un gen con imprinting son el sndrome de
Prader Willi y el sndrome de Angelman.
En el gen responsable del sndrome de Prader-Willi slo se expresa la
copia paterna; si los dos cromosomas son de origen materno, no se
expresar, y por tanto, no aparecer el cuadro clnico. En el sndrome de
Angelman, por el contrario, slo se expresa la copia materna del gen.
La respuesta 1 es falsa porque la talasemia es una enfermedad con
herencia mendeliana (aunque compleja) y no se debe a disomas. Lo
mismo ocurre con el Huntington (respuesta 2 falsa). El sndrome de
Down es una trisoma, no una disoma uniparental (respuesta 3 falsa).
EL linfoma de Burkitt es una enfermedad compleja en la que se detectan traslocaciones cromosmicas, no disomas (respuesta 5 falsa).
Pregunta 44.- R: 3
En otra pregunta anterior hablamos de la heterogeneidad gentica,
en concreto de la heterogeneidad de locus, y quiz nos supo a poco
porque no qued claro el resto de las heterogeneidades.
Se denomina heterogeneidad allica (heterogeneidad molecular
en algunos textos) a la situacin en la que se producen diferentes

CTO Medicina C/ Nez de Balboa, 115 28006 MADRID (Espaa) Tfno.: (91) 782 43 32 / Fax: (91) 782 43 27
E-mail: secretaria@ctomedicina.com; iberocto@ctomedicina.com WEB: www.ctomedicina.com; www.iberocto.com

Comentarios TEST

Seguimiento a distancia

INMUNOLOGA Y GENTICA

Preparacin Examen de Seleccin 05/06 1 Vuelta


mutaciones en un mismo gen (locus) que producen todas ellas un
mismo fenotipo enfermo, aunque la mutacin sea en zonas distintas
del gen (como en la fibrosis qustica) o bien varios fenotipos distintos
de la misma enfermedad (ejemplo: las betatalasemias).
La heterogeneidad clnica (respuesta 1 falsa) consiste en que diferentes mutaciones en un mismo gen (locus) se traducen en enfermedades completamente distintas (ejemplo: gen de la distrofina y las distrofias musculares de Duchenne y Becker).
La heterogeneidad de locus (respuesta 2 falsa) consiste en que
mutaciones en genes diferentes dan lugar a un cuadro clnico igual, se
explic anteriormente. El imprinting gnico (respuesta 4 falsa) da patologas slo en las disomas uniparentales (no es este el caso).
La fenocopia (respuesta 5 falsa) es una enfermedad ambiental que
es clnicamente muy parecida a otra de origen gentico.
Pregunta 45.- R: 1
De los patrones de herencia de enfermedad, el ms frecuente es el
autosmico dominante. Tngase en cuenta que la probabilidad de
que un enfermo con un cuadro autosmico dominante transmita la
enfermedad a un hijo es del 50 %.
Sin embargo, en las autosmicas recesivas, la probabilidad de transmitir la enfermedad es muy baja, el enfermo transmite el gen alterado
pero para que aparezca la enfermedad, el hijo debe recibir tambin
una copia alterada del otro progenitor (un enfermo solo tendr hijos
sanos si su pareja es sana).

Comentarios TEST

Pregunta 46.- R: 5
Un gen tiene una herencia ligada al sexo cuando su locus (el lugar
que ocupa en el genoma) est situado en uno de los cromosomas
sexuales, el X o el Y.
En los dos cromosomas sexuales existe un segmento homlogo,
igual para ambos y que se recombina en la meiosis, y un segmento
diferencial o heterlogo (no se recombina).
Si el gen est localizado en la regin homloga, tambin denominada pseudoautosmica, el carcter por l codificado est ligado
parcialmente al sexo porque en la meiosis puede pasar de un cromosoma a otro.
Si el gen se sita en la regin diferencial o heterloga, el carcter
que codifica estar totalmente ligado al sexo, pues no hay posibilidad
de intercambio de material gentico en la meiosis.
Las respuestas 1, 2 y 3 son falsas porque puede situarse en ambos
cromosomas. La respuesta 4 es un sinsentido.
Pregunta 47.- R: 3
La inactivacin de un cromosoma X en la mujer (EFECTO LYON)
consiste en la inactivacin en las hembras de uno de los dos cromosomas X, el cual se organiza como cromatina sexual o corpsculo de Barr
(respuesta 4 falsa), que consiste en un cuerpo heteropicntico en el
ncleo celular de las clulas femeninas. Se considera que es un mecanismo de compensacin de la dosis gnica, en el hombre y la mujer.
La inactivacin no ocurre en la gametognesis (respuesta 1 falsa),
sino en los primeros estadios de la embriognesis, sobre el da 16 de
gestacin. Solo se inactiva el segmento diferencial y el proceso es
irreversible por metilacin del ADN (respuesta 2 falsa) y se mantiene
durante toda la vida.
La inactivacin se realiza al azar en todas las clulas, es decir, que
en unas clulas se inactiva el cromosoma X de origen paterno y en
otras el materno (respuesta 5 falsa). Una vez establecida dicha inactivacin, dando origen a que la hembra sea funcionalmente un mosaico para los genes correspondientes al cromosoma X.
El nmero de corpsculos de Barr de una clula es igual al nmero
de cromosomas X inactivados, o lo que es lo mismo, es igual al nmero total de cromosomas X que posea la clula menos uno. As en una
superhembra (XXX) habr dos corpsculos.

Seguimiento a distancia

El raquitismo resistente a la vitamina D se hereda ligado al X dominante (respuesta 2 falsa).


La corea de Huntington se hereda de modo autosmico dominante (respuesta 3 falsa).
La enfermedad de Hunter tiene un patrn de herencia ligado al
cromosoma X recesivo (respuesta 4 falsa).
Pregunta 49.- R: 3
Una traslocacin robertsoniana es una anomala cromosmica
estructural que puede originar una anomala numrica. Por tanto se
trata de una situacin mixta entre ambos tipos de anomalas.
Se produce por la fusin de dos cromosomas acrocntricos en la
que los brazos largos de ambos cromosomas quedan fusionados (respuesta 1 cierta). Esto quiere decir que, tras la fusin, el cromosoma,
originalmente con forma de V, pasa a tener forma de X (dos V
fusionadas).
Los sujetos portadores asintomticos de la traslocacin tienen 45
cromosomas (uno de ellos en realidad es doble).
Los gametos que producen los portadores de la translocacin tras
la fecundacin pueden dan lugar a trisomas o monosomas de un
cromosoma completo (respuesta 3 FALSA). En torno al 5 % de los
casos de sndrome de Down se producen por este mecanismo (respuesta 2 cierta), los enfermos presentan 46 cromosomas, uno de ellos
doble, y es muy importante detectar esos casos de cara al consejo
gentico: la probabilidad de recurrencia de un hijo con sndrome de
Down en una pareja no portadora de la traslocacin es alrededor de
1:700 (similar a la de tener un hijo con Down en la poblacin general) sin embargo, en los portadores la probabilidad es bastante mayor
(respuestas 4 y 5 ciertas) y se puede fijar en cerca del 10 % (vara segn
los estudios).
Pregunta 50.- R: 1
La enfermedad de Marfan, como casi todas las que se deben a
alteracin de una protena estructural, se hereda de modo autosmico dominante. En la mayor parte de los casos observados, el alelo
causante de la enfermedad se hereda de padre o madre enfermos
(respuesta 1 cierta; 2 y 5 falsas) y slo una pequesima proporcin se
deben a mutaciones de novo (respuesta 4 falsa).
Muy importante: en las enfermedades autosmicas dominantes,
slo existen enfermos y sanos, no existen los portadores asintomticos
(respuesta 3 falsa).
Un nio de 3 aos que ha heredado un gen mutado de un progenitor enfermo y que no tiene clnica, no es un portador, se trata de un
enfermo en fase preclnica o silente.

Pregunta 48.- R: 5
El sndrome del cromosoma X frgil es una enfermedad en la que el
mecanismo patognico es la inestabilidad en la longitud de un segmento gnico (expansin de secuencias).
La neuritis de Leber presenta un patrn de herencia mitocondrial
(respuesta 1 falsa).
M exico A rgentina
C hile U ruguay

CTO Medicina C/ Nez de Balboa, 115 28006 MADRID (Espaa) Tfno.: (91) 782 43 32 / Fax: (91) 782 43 27
E-mail: secretaria@ctomedicina.com; iberocto@ctomedicina.com WEB: www.ctomedicina.com; www.iberocto.com

IG Pg. 9

NEFROLOGA

Preparacin Examen de Seleccin 05/06 1 Vuelta


FISIOLOGA.
1.

Cul es el procedimiento ms adecuado para saber si existe


una disminucin de la funcin renal?:
1)
2)
3)
4)
5)

2.

3.

4.

Preguntas TEST

5.

6.

Tbulo proximal.
Asa de Henle.
Tbulo distal.
Tubo colector.
Vasa recta.
11.

Urea.
Insulina.
cido PAH.
Creatinina.
Fenolsulfoftalena.

1)
2)
3)
4)
5)

Tbulo Proximal.
Tbulo Colector.
Asa de Henle.
Tbulo Distal.
Vasa recta.

13.

7.

Qu entendemos por glomerulonefritis aguda?:


1)
2)
3)

La que cursa con hematuria.


Si debuta con oligoanuria.
Si cursa con acidosis metablica.
M exico A rgentina
C hile U ruguay

14.

Alcalosis metablica.
Aumento de LDH.
Hipocomplementemia.
Hipopotasemia .
Aumento del CH5O.

La GN extracapilar Tipo II se caracteriza por:


1)
2)
3)
4)
5)

GLOMERULONEFRITIS.

TBC.
Sfilis.
Sarampin.
Brucelosis.
HIV.

Una mujer de 16 aos de edad tiene diuresis escasas y de color


"coca cola". Nueve das antes, tuvo un cuadro de faringoamigdalitis. En el sistemtico de orina se objetiva proteinuria de
2 g/da. La TA es de 180/120 mmHg. En la exploracin hay
edemas. Qu esperara encontrar en la sangre?:
1)
2)
3)
4)
5)

La reabsorcin del magnesio ocurre fundamentalmente en:

Albmina intravenosa.
Dieta hiperproteica.
Dieta pobre en calcio.
Diurticos tiacdicos.
Diurticos ahorradores de potasio.

Qu cuadro infeccioso puede producir la remisin de la GN


de cambios mnimos?:
1)
2)
3)
4)
5)

12.

AMBG.
IC formados in situ.
Activacin del complemento.
IC circulantes.
Inmunoglobulinas circulantes.

En el tratamiento del edema del sndrome nefrtico, cul se


emplea en el caso de edema refractario?:
1)
2)
3)
4)
5)

Tbulo proximal.
Tbulo distal.
Aparato yuxtaglomerular.
Clulas mesangiales.
Capilares peritubulares.

De las siguientes sustancias, cul se utiliza para medir el


flujo plasmtico renal?:
1)
2)
3)
4)
5)

10.

Aparicin de semilunas.
Proliferacin mesangial difusa.
Depsito mesangial de IgA en la inmunofluorescencia.
ptico normal.
Desdoblamiento de membrana basal.

El mecanismo ms frecuente de produccin de glomerulonefritis es:


1)
2)
3)
4)
5)

La renina se produce en:


1)
2)
3)
4)
5)

9.

Si es irreversible.
Se refiere a la nefropata lpica.

El hallazgo distintivo de la nefropata IgA es:


1)
2)
3)
4)
5)

IL I.
IL 6.
VMA.
AG II.
Endotelina.

De qu estructura de la nefrona depende la concentracin


de la orina?:
1)
2)
3)
4)
5)

8.

Urografa intravenosa.
Cifra de urea en sangre.
Determinar el aclaramiento de creatinina.
Determinar la concentracin de creatinina en sangre.
Determinar el aclaramiento de urea.

Qu sustancia es fundamental en el mecanismo de la


autorregulacin glomerular?:
1)
2)
3)
4)
5)

4)
5)

Seguimiento a distancia

Cursar con poliuria.


Ig E elevada.
Llevar un curso doloroso.
Ser reversible.
Ser hipocomplementmica.

El dao originado en los glomrulos en la endocarditis


infecciosa es?:
1)
2)

Mediado por AMBG.


Mediado por P-ANCA.

CTO Medicina C/ Nez de Balboa, 115 28006 MADRID (Espaa) Tfno.: (91) 782 43 32 / Fax: (91) 782 43 27
E-mail: secretaria@ctomedicina.com; iberocto@ctomedicina.com WEB: www.ctomedicina.com; www.iberocto.com

HM Pg. 1

3)
4)
5)
15.

16.

17.

18.

Se afectan unos glomrulos s y otros no.


Se afecta una parte de glomrulos s y otra no.
Se afectan los glomrulos corticales.
Se afectan los glomrulos yuxtamedulares.
Se afectan los glomrulos ms alejados de la corteza.

25.

Depsitos lineales de IgG.


Depsitos granulosos de IgG.
Depsitos granulosos de IgA.
Depsitos lineales de IgM.
Depsito de C3.

Un varn de 40 aos acude por presentar edemas en piernas


y astenia. En la analtica, se objetiva un aclaramiento de
creatinina del 20%. La proteinuria es rica en albmina y de
3,5 g/da. Qu hara en primer lugar en este paciente?:
1)
2)

Pg. 2 HM

Inmunosupresin.
Corticoterapia.
M exico A rgentina
C hile U ruguay

2)
3)
4)
5)
27.

Glomerulonefritis
Glomerulonefritis
Glomerulonefritis
Glomerulonefritis
Glomerulonefritis

membranosa.
endocapilar difusa.
membranoproliferativa tipo I.
membranoproliferativa tipo II.
lpica.

En un varn de 20 aos sin evidencia de enfermedad


sistmica, que presenta sndrome nefrtico, microhematuria, leve insuficiencia renal y niveles bajos de complemento
(C3) en suero, el diagnstico histolgico ms probable es:
1)

Peor en varones que en mujeres.


Peor en nios que en adultos.
Mejor en nios que en adultos.
No vara con la edad.
Peor en mujeres que en varones.

GN endocapilar difusa.
Nefropata mesangial IgA.
GN rpidamente progresiva.
GN membranoproliferativa.
GN membranosa.

La lipodistrofia es ms frecuente en?:


1)
2)
3)
4)
5)

26.

Glomerulonefritis endocapilar.
Sndrome nefrtico idioptico.
Glomerulonefritis membranosa.
Nefropata lpica.
Glomerulonefritis de la endocarditis infecciosa.

Un varn acude al Servicio de Urgencias por cuadro de


orinas oscuras. Tiene faringitis aguda. Interrogado, refiere
un episodio similar hace 1 ao. La diuresis es de cuanta
normal, y la cifra de creatinina en sangre es de 1,3 mg/dl. En
qu entidad pensara en primer lugar?:
1)
2)
3)
4)
5)

El pronstico de una GN aguda postestreptoccica es:


1)
2)
3)
4)
5)

21.

24.

En la GN rpidamente progresiva por anticuerpos antimembrana basal, la inmunofluorescencia nos presenta:


1)
2)
3)
4)
5)

20.

No existe.
Es propia de la infancia.
Se asocia a hipopotasemia.
Suele cursar con proteinuria.
Tiene buen pronstico.

Sndrome de Goodpasture.
Enfermedad de Berger.
Crioglobulinemia mixta esencial.
Sepsis visceral oculta.
Granulomatosis de Wegener.

En un paciente con carcinoma de pulmn y sndrome


nefrtico, qu cuadro glomerular considera usted ms
probable?:
1)
2)
3)
4)
5)

Una afectacin glomerular focal quiere decir:


1)
2)
3)
4)
5)

19.

23.

Biopsia renal.
Urografa i.v.
TC renal.

En cul de las siguientes entidades es MENOS frecuente la


glomerulonefritis rpidamente progresiva?:
1)
2)
3)
4)
5)

Es lo habitual.
Hay que abandonar los corticoides.
Hay que hacer dilisis.
Que puede tener una hialinosis focal y segmentaria.
Nunca ocurre.

La glomerulonefritis inmunotactoide:
1)
2)
3)
4)
5)

22.

NTA.
Necrosis tbulointersticial aguda.
Aparicin de G.N.R.P.
Vasculitis asociada.
Amiloidosis.

Ante un sndrome nefrtico que no responde a corticoides


en un nio, qu pensaramos?:
1)
2)
3)
4)
5)

3)
4)
5)

Mediado por C-ANCA.


Mediado por inmunocomplejos circulantes.
No se conoce el mecanismo.

Cuando un proceso glomerular desarrolla un cuadro de


oligoanuria, en qu pensaramos en primer lugar?:
1)
2)
3)
4)
5)

NEFROLOGA

Preparacin Examen de Seleccin 05/06 1 Vuelta

Glomerulonefritis mesangiocapilar o membranoproliferativa.


Glomerulonefritis mesangial IgA.
Sndrome nefrtico por mnimos cambios.
Glomeruloesclerosis focal.
Glomerulonefritis membranosa.

La glomerulonefritis esclerosante focal y segmentaria con


hialinosis puede aparecer sobre todo en la evolucin de qu
proceso:
1)
2)
3)

Sndrome de Bartter.
Acidosis tubular renal tipo I.
Nefropata tubulointersticial crnica, asociada a reflujo
vesicoureteral.

CTO Medicina C/ Nez de Balboa, 115 28006 MADRID (Espaa) Tfno.: (91) 782 43 32 / Fax: (91) 782 43 27
E-mail: secretaria@ctomedicina.com; iberocto@ctomedicina.com WEB: www.ctomedicina.com; www.iberocto.com

Preguntas TEST

Seguimiento a distancia

NEFROLOGA

Preparacin Examen de Seleccin 05/06 1 Vuelta


4)
5)
28.

El sndrome nefrtico de la nefropata membranosa se


caracteriza tpicamente:
1)
2)
3)
4)
5)

29.

30.

Preguntas TEST

Depsitos granulares glomerulares de IgG y C3 en IF.


Est mediada por inmunocomplejos.
Proliferacin endocapilar en los glomrulos.
La hematuria microscpica puede persistir durante meses.
Hipocomplementemia persistente.

Pero las remisiones no son nunca completas.


En nios.
En el 20-30% de los casos.
En ms del 80% de los casos.
Prcticamente nunca.

Un varn de 60 aos de edad tiene edemas con hipoalbuminemia. El aclaramiento de creatinina es del 30%. Las cifras
de complemento son normales. De las etiologas secundarias propuestas, en qu entidad pensara probablemente?:
1)
2)
3)
4)
5)

34.

GN mesangial.
GN membranosa.
GN membranoproliferativa.
GN mesangial IgA.
Hialinosis focal y segmentaria.

37.

Respecto a la nefropata IgA, es cierto que:


1)
2)

Cursa con hipocomplementemia.


Ocasionalmente, tiene incidencia familiar.
M exico A rgentina
C hile U ruguay

41.

Nefropata mesangial.
GN membranosa.
GN lpica.
Nefrosis lipoidea.
Enfermedad de Wegener.

Cul es el mecanismo de produccin ms frecuente en la


glomerulonefritis aguda?:
1)
2)
3)
4)
5)

Colagenosis.
Vasculitis.
Neoplasias.
Mieloma mltiple.
Enfermedad inflamatoria intersticial.

Hemlisis.
Dilucional.
Ausencia de eritropoyetina.
Hemorragia.
Hiperesplenismo.

La enfermedad de Hodgkin se asocia frecuentemente a:


1)
2)
3)
4)
5)

40.

Sndrome nefrtico por lesiones mnimas.


GN proliferativa mesangial.
GN esclerosante focal y segmentaria.
GN membranosa.
GN membranoproliferativa.

En la glomerulonefritis aguda, la anemia de la fase aguda es


secundaria a:
1)
2)
3)
4)
5)

39.

Sndrome nefrtico.
Nefritis lpica.
Pielonefritis.
Necrosis tubular aguda.
Sndrome nefrtico.

De las siguientes glomerulonefritis, cul puede recurrir en


horas en el rin trasplantado?:
1)
2)
3)
4)
5)

38.

Glomerulonefritis rpidamente progresiva.


Glomerulonefritis membranosa.
Nefropata lpica.
Amiloidosis renal.
Nefritis intersticial por drogas.

Una mujer de 28 aos presenta, de forma aguda, nuseas,


vmitos, oliguria y macrohematuria. Tiene edemas, TA 180/
110, urea 80 mg/dl, proteinuria y cilindros hemticos en la
orina. Esta paciente tiene:
1)
2)
3)
4)
5)

La nefropata membranosa puede remitir espontneamente:


1)
2)
3)
4)
5)

33.

Coagulopata.
Sndrome nefrtico.
Glomerulonefritis aguda.
Pielonefritis crnica.
Fracaso renal agudo.

Son caractersticas de la glomerulonefritis postestreptoccica todas, EXCEPTO:


1)
2)
3)
4)
5)

32.

36.

Hay depsitos aislados de IgA en el glomrulo.


Se trata con bolos de ciclofosfamida.
Se suele presentar con sndrome nefrtico recidivante.

Mujer de 16 aos, con edemas maleolares y periorbitarios,


proteinuria de 4 g/da, numerosos hemates y cilindros en el
sedimento, C3 40 mg/dl, C4 5 mg/dl. La nefropata ms
probable es:
1)
2)
3)
4)
5)

Un varn con sndrome nefrtico e hipocomplementemia


tendr, como primera posibilidad:
1)
2)
3)
4)
5)

31.

35.

Por el depsito granular de IgE en el examen de inmunofluorescencia.


Por responder frecuentemente al tratamiento con esteroides.
Por ser ms frecuente a partir de los 50 aos de edad.
Por evolucionar rpidamente a la insuficiencia renal, en
la mayora de los casos.
Por ser reversible en el 50 % de los casos.

La presencia de cilindros hemticos en el sedimento urinario


sugiere:
1)
2)
3)
4)
5)

3)
4)
5)

Cristalizacin intratubular de cido rico.


Tratamiento con captopril.

Seguimiento a distancia

Directa por virus.


Directa por inmunoglobulinas.
Inmunocomplejos circulantes.
Por anticuerpos antimembrana basal glomerular.
Inmunocomplejos formados "in situ".

Cul es la glomerulopata en la que los glomrulos afectados son los de las nefronas yuxtamedulares?:
1)

Extracapilar difusa.

CTO Medicina C/ Nez de Balboa, 115 28006 MADRID (Espaa) Tfno.: (91) 782 43 32 / Fax: (91) 782 43 27
E-mail: secretaria@ctomedicina.com; iberocto@ctomedicina.com WEB: www.ctomedicina.com; www.iberocto.com

HM Pg. 3

2)
3)
4)
5)
42.

Hialinosis segmentaria y focal.


Berger.
Epimembranosa.
Exudativa difusa.

4)
5)
48.

Una mujer de 30 aos de edad tiene unas cifras de creatinina


srica de 3 mg/dl. En el sedimento de orina aparece un sedimento rico en todo tipo de cilindros, con una proteinuria rica en
albmina y microhematuria. La cifra de C3 en sangre es de 3 mg/
dl sin oliguria. Qu esperara encontrar en la biopsia renal?:
1)
2)
3)
4)
5)

43.

NEFROLOGA

Preparacin Examen de Seleccin 05/06 1 Vuelta

Nefropata de cambios mnimos.


Glomerulonefritis membranoproliferativa.
Glomerulonefritis mesangial IgA.
Glomerulonefritis endocapilar difusa.
Glomerulonefritis extracapilar.

Una mujer de 30 aos de edad acude al mdico por cuadro


de orinas oscuras. En la exploracin, se objetiva una asimetra corporal, con prdida de panculo adiposo en el miembro inferior izquierdo. En el sedimento hay hematuria y
proteinuria, y las cifras de complemento son bajas. Cul
sera su diagnstico?:
1)
2)
3)
4)
5)

50.

NEFROPATAS INTERSTICIALES.

1)
2)
3)
4)
5)
45.

52.

Rin hipocalimico.
Rin qustico medular.
Rin gotoso.
Rin poliqustico.
Nefropata de Bartter.

53.

Nefropata por analgsicos crnica.


Hialinosis focal.
Nefropata hipopotasmica.
Nefropata membranosa.
Cambios mnimos.

Reversible.
Idiosincrsica.
De etiologa inmunitaria.
Que produce necrosis de papila.
Que nunca lleva al trasplante.

Un varn de 60 aos de edad est diagnosticado de nefropata


intersticial por plomo. Acude a urgencias por estar edematoso,
y tiene proteinuria en rango nefrtico. Qu pensaramos?:
1)
2)
3)
4)
5)

Tiene amiloide renal.


Hialinosis focal y segmentaria.
Nefropata membranosa.
Adenocarcinoma renal.
Cambios mnimos.

La pielonefritis crnica produce principalmente:

Un varn de 30 aos de edad presenta unas cifras de


creatinina en sangre de 3 mg/dl. Refiere poliuria y nicturia.
Qu esperara encontrar en la analtica sangunea?:

1)
2)
3)

1)
2)
3)

Pg. 4 HM

Glomerulopata.
Afectacin intersticial.
Tubulopata proximal.
M exico A rgentina
C hile U ruguay

54.

Disminucin en la capacidad de concentracin.


Proteinuria superior a 3 g/da.
Glucosuria.
Fosfaturia.
Presencia de un aumento de urea mayor que el aumento
de creatinina.

La nefropata por analgsicos es una enfermedad:


1)
2)
3)
4)
5)

En cul de las siguientes patologas hay mayor incidencia de


uroteliomas?:
1)
2)
3)
4)
5)

47.

hematuria.
proteinuria rica en albmina.
hipocalciuria.
poliuria.
hipomagnesemia.

Cul de las siguientes etiologas de nefropata tbulointersticial desarrolla ms precozmente HTA?:


1)
2)
3)
4)
5)

46.

Hay
Hay
Hay
Hay
Hay

En nios menores de 2 aos.


En varones adolescentes.
En adultos que consumen cocana.
Slo en adultos con isquemia renal.
En mujeres entre los 20 y los 40 aos.

El primer dato patolgico detectable en la pielonefritis


crnica es:
1)
2)
3)
4)
5)

En el dao intersticial?:

Linfocitos y clulas plasmticas.


Eosinfilos.
Monocitos.
Fibroblastos.
Polimorfonucleares.

La nefropata por analgsicos crnica es ms frecuente en:


1)
2)
3)
4)
5)

Glomerulonefritis endocapilar difusa.


Glomerulonefritis membranoproliferativa tipo I.
Glomerulonefritis membranoproliferativa tipo II.
Glomerulonefritis extracapilar.
Glomerulonefritis mesangial IgA.

HTA maligna.
Hematuria macroscpica recidivante.
Sndrome nefrtico.
Poliuria y acidosis metablica.
Proteinuria mayor de 5 g/da.

En la nefropata intersticial crnica, la infiltracin intersticial


es fundamentalmente a base de:
1)
2)
3)
4)
5)

51.

44.

El modo ms habitual de presentarse clnicamente una


nefropata intersticial es:
1)
2)
3)
4)
5)

49.

Oliguria.
Eosinofiluria.

Hiperpotasemia.
Hipercalcemia.
Hipernatremia.

CTO Medicina C/ Nez de Balboa, 115 28006 MADRID (Espaa) Tfno.: (91) 782 43 32 / Fax: (91) 782 43 27
E-mail: secretaria@ctomedicina.com; iberocto@ctomedicina.com WEB: www.ctomedicina.com; www.iberocto.com

Preguntas TEST

Seguimiento a distancia

NEFROLOGA

Preparacin Examen de Seleccin 05/06 1 Vuelta


4)
5)
55.

Preguntas TEST

Cilindros hemticos.
Sedimento telescopado.
Cilindros hialinos.
Cristales de estruvita.
Cilindros anchos y creos.

Yersinia spp.
Proteus mirabilis.
Klebsiella pneumoniae.
E. coli.
Enterococo.

Un paciente con antecedentes de podagra tiene un cuadro


de acidosis metablica, acompaado de intensa poliuria y
sed. En el sedimento de orina aparecen gran cantidad de
cristales de cido rico. Qu esperara encontrar en la
exploracin de este paciente?:
1)
2)
3)
4)
5)

60.

Sndrome de Liddle.
Sndrome de Bartter.
Acidosis tubular tipo II.
Nefropata de los Balcanes.
Nefropata intersticial por cido rico.

1)
2)
3)
4)
5)

Eosinofiluria.
Sndrome nefrtico.
Leucocituria.
Cadenas ligeras.
pH urinario de 9.
M exico A rgentina
C hile U ruguay

No existe.
Es de etiologa autoinmune.
Puede confundirse con una neoplasia renal.
Nunca produce fiebre.
Nunca requiere ciruga.

ENFERMEDADES SISTMICAS.
63.

Uno de los siguientes cuadros suele cursar con hematuria


macroscpica recurrente:
1)
2)
3)
4)
5)

64.

65.

Sordera y alteraciones oculares y renales.


Distrofia de uas y displasia de rodilla.
Riones poliqusticos, HTA e insuficiencia renal.
Sndrome de Fanconi y cataratas.
Amiloidosis y polineuropata.

La nefropata por depsito intratubular de cido rico se


puede ver en:
1)
2)
3)
4)
5)

67.

Nefropata por analgsicos.


Intoxicacin por DDT.
Sndrome de Bartter.
Nefropata de los Balcanes.
Mieloma mltiple.

La enfermedad de Alport-Perkoff cursa con:


1)
2)
3)
4)
5)

66.

Sndrome de Alport.
Nefronoptisis.
Bartter.
Glomerulonefritis diabtica.
Amiloidosis.

En cul de los siguientes procesos puede haber una proteinuria mayor de 4 g/da?:
1)
2)
3)
4)
5)

Dolor abdominal.
Poliglobulia.
HTA.
Orinas oscuras.
Masa abdominal.

Un varn de 40 aos de edad tiene un LES con escasa


actividad desde hace varios aos. En el sedimento de orina
aparecen cilindros mixtos. Consulta, porque en los ltimos
2 meses, refiere abundante poliuria y nicturia. Qu esperara encontrar en el sedimento de orina de este paciente?:

No hay proteinuria.
No hay nunca HTA.
Hay hipernatremia.
Puede haber hipopotasemia.
Hay alcalosis metablica.

La pielonefritis xantogranulomatosa:
1)
2)
3)
4)
5)

Cul es el microorganismo causal ms frecuente de las


pielonefritis?:
1)
2)
3)
4)
5)

59.

62.

Un varn de 40 aos, de profesin fontanero, refiere un


cuadro consistente en poliuria y astenia. En la analtica, hay
acidosis metablica hiperclormica. Qu esperara encontrar en el sedimento?:
1)
2)
3)
4)
5)

58.

Nefropata por plomo.


Nefropata por cido rico.
L.E.S.
Nefronoptisis.
Nefropata de los Balcanes.

En las nefropatas intersticiales crnicas:


1)
2)
3)
4)
5)

Cul de estas patologas se asocia ms frecuentemente con


HTA?:
1)
2)
3)
4)
5)

57.

61.

Cul de los siguientes cuadros se asocia con la presencia de


uroteliomas?:
1)
2)
3)
4)
5)

56.

Acidosis metablica.
Hipocloremia.

Seguimiento a distancia

Gota primaria.
Sndrome de Lesch-Nyhan.
Hipercalcemia.
Linfoma tratado con antiblsticos.
Hiperprolactinemia.

La manifestacin clnica cardinal de la nefropata diabtica


es la presencia de:
1)
2)
3)
4)
5)

Hematuria.
Proteinuria.
Piuria.
Sndrome nefrtico.
Insuficiencia renal.

CTO Medicina C/ Nez de Balboa, 115 28006 MADRID (Espaa) Tfno.: (91) 782 43 32 / Fax: (91) 782 43 27
E-mail: secretaria@ctomedicina.com; iberocto@ctomedicina.com WEB: www.ctomedicina.com; www.iberocto.com

HM Pg. 5

68.

76.

5)

Diseccin artica abdominal.


Uropata obstructiva.
Embolismo de la arteria renal.
Glomerulonefritis evolucionada.
Trombosis aguda de la vena renal.

79.

Es adquirida.
Hay un dficit de colgeno tipo III.
El varn no padece la enfermedad.
Puede ser causa de sndrome nefrtico.
Recidiva en el trasplante renal.

Cul de estas sustancias se asocia al adenocarcinoma renal?:


1)

Pg. 6 HM

Mercurio.
M exico A rgentina
C hile U ruguay

Asas de alambre.
Depsitos subendoteliales de inmunoglobulinas.
Depsitos intramembranosos de anti-ADN nativo.
Engrosamiento de la membrana basal por depsitos de ADN.
Cuerpos de hematoxilina.

Qu cuadro histopatolgico lpico renal tiene PEOR pronstico?:


1)
2)
3)
4)
5)

81.

Hay que dar betabloqueantes.


Los diurticos son de eleccin.
Los IECAs pueden revertir la microalbuminuria.
No tiene tratamiento.
Se dan bloqueantes de los canales del calcio.

Un hallazgo casi patognomnico de nefropata lpica es:


1)
2)
3)
4)
5)

80.

Schnlein-Henoch.
Angetis por hipersensibilidad.
Crioglobulinemia mixta esencial.
Panarteritis nodosa.
Enfermedad de Wegener.

En el tratamiento de la proteinuria de la nefropata diabtica:


1)
2)
3)
4)
5)

Crioglobulinas.
AMBG.
ANA.
C-ANCA.
P-ANCA.

Embolia de la arteria renal.


Aneurisma disecante artico, a nivel de arterias renales.
Trombosis de la vena renal.
Clico renal, con bacteriemia y afectacin pulmonar
secundaria.
Sndrome de Goodpasture.

Cul es la vasculitis que afecta a arterias de mediano calibre?:


1)
2)
3)
4)
5)

78.

Cambios mnimos.
Glomerulonefritis membranoproliferativa.
Glomerulonefritis membranosa.
Glomerulonefritis segmentaria y focal.
Glomerulonefritis proliferativa difusa.

Un hombre de 56 aos ingresa para realizar el estudio de un


sndrome nefrtico, presentando durante su estancia dolor
sbito en regin lumbar, con aumento claro de la proteinuria, hematuria y clnica asociada de dolor torcico y disnea.
Cul sera el diagnstico que habra que sospechar?:
1)
2)
3)
4)

77.

Cadmio.
Flor.
Metano.
Etanol.

La lesin glomerular ms frecuentemente encontrada en el


SIDA es:
1)
2)
3)
4)
5)

Qu es correcto en la enfermedad de Alport?:


1)
2)
3)
4)
5)

74.

Sndrome nefrtico idioptico.


GN membranoproliferativa.
Vasculitis.
Amiloidosis.
Nefropata diabtica por esteroides.

Si un paciente presenta dolor lumbar intenso, hematuria


microscpica y aumento de las cifras de LDH y GOT en plasma.
Hay que pensar como primera posibilidad diagnstica?:
1)
2)
3)
4)
5)

73.

Crioglobulinemia mixta esencial.


LES.
Enfermedad de Schnlein-Henoch.
PAN microscpica.
Linfangioleiomiomatosis.

Un varn de 50 aos tiene un episodio de hemoptisis. En el


sedimento de orina hay hematuria. Se realiza una Rx de trax,
encontrndose infiltrados cavitados. Entre sus antecedentes
figura rinitis de repeticin. Qu prueba pedira en primer lugar?:
1)
2)
3)
4)
5)

72.

75.

Una enferma, con artritis reumatoide de veinte aos de


evolucin, que nunca ha recibido sales de oro, desarrolla
proteinuria y sndrome nefrtico. Su diagnstico de sospecha sera:
1)
2)
3)
4)
5)

71.

Glomeruloesclerosis intercapilar difusa.


Glomeruloesclerosis nodular.
Necrosis papilar.
Infeccin urinaria.
Nefritis intersticial crnica.

Un varn de 50 aos de edad es estudiado por haber


presentado un episodio de hemoptisis y hematuria autolimitada. La cifra de complemento es normal. Hay disminucin de
la funcin renal y anticuerpos antimieloperoxidasa en plasma
(con patrn p-ANCA). Cul es la primera posibilidad?:
1)
2)
3)
4)
5)

70.

2)
3)
4)
5)

La lesin ms frecuentemente encontrada en la diabetes es:


1)
2)
3)
4)
5)

69.

NEFROLOGA

Preparacin Examen de Seleccin 05/06 1 Vuelta

GN proliferativa focal lpica.


GN membranosa lpica.
GN cambios mnimos lpica.
GN proliferativa difusa.
GN mesangial lpica.

Un varn de 60 aos tiene antecedentes de bronquiectasias


desde su juventud. Es estudiado por sndrome constitucional, y presenta proteinuria de 4 g/ da, rica en albmina. En
qu proceso pensara inicialmente?:

CTO Medicina C/ Nez de Balboa, 115 28006 MADRID (Espaa) Tfno.: (91) 782 43 32 / Fax: (91) 782 43 27
E-mail: secretaria@ctomedicina.com; iberocto@ctomedicina.com WEB: www.ctomedicina.com; www.iberocto.com

Preguntas TEST

Seguimiento a distancia

NEFROLOGA

Preparacin Examen de Seleccin 05/06 1 Vuelta


1)
2)
3)
4)
5)
82.

Cul es el tratamiento de eleccin en la afectacin grave y


rpida del rin?:
1)
2)
3)
4)
5)

83.

85.

Preguntas TEST

92.

Hemorragias digestivas.
Rinitis.
Cardiopata.
Lesiones ulceradas en dedos.
Fenmeno de Raynaud.

Esclerodermia.
Toxemia gravdica.
Panarteritis nodosa.
Amiloidosis.
Enfermedad de Wegener.

Lupus eritematoso diseminado.


Sndrome de Goodpasture.
Amiloidosis renal.
Panarteritis nodosa.
Ninguna de las anteriores.

Un varn de 35 aos de edad acude a Urgencias por


hemoptisis. La Rx de trax revela un patrn reticulonodular
en bases pulmonares. El enfermo tiene febrcula y artromialgias. En la orina hay microhematuria. Qu prueba diagnstica pedira en primer lugar?:
M exico A rgentina
C hile U ruguay

Eosinofiluria.
Granulomas en el sedimento de orina.
Piuria intensa.
Hematuria y proteinuria asintomticas.
Clulas lipoideas en el sedimento.

La manifestacin ms frecuente de la afectacin renal de la


amiloidosis es:
Hematuria.
Beta2-microglobulinas en orina.
Albuminuria.
Acidosis metablica.
Hipercalciuria.

En el sedimento del lupus, es caracterstico:


1)
2)
3)
4)
5)

Cilindros anchos.
Cilindros piricos.
Eosinofiluria.
Sedimento telescopado.
Cilindros hialinos.

Un paciente acude a consulta por cuadro de eliminacin de


orinas oscuras. Tiene febrcula y eliminacin de esputos
hemoptoicos. En la Rx de trax aparecen infiltrados pulmonares no cavitados. En la anamnesis, refiere aparicin de
fenmeno de Raynaud, y lesiones purpricas en piernas de
forma ocasional. En qu proceso pensara?:
1)
2)
3)
4)
5)

LES.
Crioglobulinemia mixta esencial.
PAN.
Enfermedad de Wegener.
Sndrome de Goodpasture.

INSUFICIENCIA RENAL.
93.

La causa ms frecuente de insuficiencia renal aguda es?:


1)
2)
3)
4)
5)

Las imgenes aneurismticas renales son propias de:


1)
2)
3)
4)
5)

88.

45%.
33%.
20%.
25%.
50%.

91.

Ac anti-MBG.
HLA-A8.
C-ANCA.
P-ANCA.
HLA-B12.

Qu es caracterstico en la afectacin renal de la enfermedad de Wegener?:

1)
2)
3)
4)
5)

Prurito.
Hipertrigliceridemia.
Hipercolesterolemia.
Anorexia.
Calcifilaxia.

La asociacin con trombosis de la vena renal es un dato


frecuente en:
1)
2)
3)
4)
5)

87.

90.

Qu manifestaciones extrarrenales son propias de la granulomatosis de Wegener?:


1)
2)
3)
4)
5)

86.

89.

1)
2)
3)
4)
5)

Qu hematocrito mnimo es el ideal en pacientes con


insuficiencia renal crnica?:
1)
2)
3)
4)
5)

1)
2)
3)
4)
5)

Corticoides.
Ciclosporina A.
Plasmafresis.
Dilisis.
Interfern.

Cul de los siguientes sntomas o signos mejora con la


hemodilisis?:
1)
2)
3)
4)
5)

84.

Nefropata de cambios mnimos.


Glomerulonefritis mesangiocapilar.
Amiloidosis.
Mieloma mltiple.
PAN.

Seguimiento a distancia

94.

Deshidratacin.
Necrosis cortical bilateral.
Colagenosis.
Nefropata por analgsicos aguda.
NTA.

En una hiperpotasemia aguda, cul es la primera medida


teraputica?:
1)
2)
3)
4)
5)

Mg++ oral.
Resn calcio rectal.
Fsforo oral.
Calcio intravenoso.
Dilisis.

CTO Medicina C/ Nez de Balboa, 115 28006 MADRID (Espaa) Tfno.: (91) 782 43 32 / Fax: (91) 782 43 27
E-mail: secretaria@ctomedicina.com; iberocto@ctomedicina.com WEB: www.ctomedicina.com; www.iberocto.com

HM Pg. 7

Seguimiento a distancia
95.

En enfermos agudamente oligricos, lo fundamental en el


tratamiento es:
1)
2)
3)
4)
5)

96.

97.

GN postestreptoccica.
NTA.
PAN.
Shock hipovolmico.
Primeras horas del FRA por obstruccin.

Los frmacos ms frecuentemente implicados en la nefropata tubulointersticial por frmacos, son:


1)
2)
3)
4)
5)

98.

Corregir la hipovolemia.
Digitalizar.
Restringir el aporte proteico.
Administrar furosemida.
Restringir el aporte lquido.

Es causa de fracaso renal agudo, con excrecin fraccional


de sodio alta:
1)
2)
3)
4)
5)

NEFROLOGA

Preparacin Examen de Seleccin 05/06 1 Vuelta

Macrlidos.
Tetraciclinas.
Betalactmicos.
Sulfamidas.
Lincosaminas.

102.

El sndrome de Bartter presenta todo, EXCEPTO:


1)
2)
3)
4)
5)

103.

El sndrome de Gitelman:
1)
2)
3)
4)
5)

104.

Hipopotasemia crnica.
Hipertrofia del aparato yuxtaglomerular.
Aumento de aldosterona.
Acidosis hiperclormica.
Alcalosis hipoclormica.

Cursa con acidosis metablica.


Afecta a los glomrulos.
No produce nefrocalcinosis.
Produce HTA.
Cursa con hiperpotasemia.

En el sndrome de Bartter:
1)
2)
3)
4)
5)

Hay hipoaldosteronismo.
Hay hipermagnesemia.
Puede haber hipernatremia.
Hay aumento de PGE circulante.
Siempre hay HTA.

La causa ms frecuente de FR agudo intrnseco es:


1)
2)
3)
4)
5)

La necrosis tubular aguda.


LES.
HTA.
Pielonefritis.
La necrosis cortical.

NEFROPATAS HEREDITARIAS.
Cul es el sntoma ms frecuente de la poliquistosis renal?:
1)
2)
3)
4)
5)
100.

El sndrome de Liddle:
1)
2)
3)
4)
5)

101.

Dolor en flanco.
Hematuria macroscpica.
Nicturia.
Expulsin de clculos.
Disuria.

Es reversible.
Cursa con hiperpotasemia.
Tiene acidosis metablica.
Es causa de HTA secundaria.
Tiene asociado aumento de renina.

Si en varios miembros de una familia se detectan casos de


nefritis, la exploracin diagnstica ms til es:
1)
2)
3)
4)
5)

Pg. 8 HM

Ecografa renal.
TC.
UIV.
Electromiograma.
Audiometra.

M exico A rgentina
C hile U ruguay

CTO Medicina C/ Nez de Balboa, 115 28006 MADRID (Espaa) Tfno.: (91) 782 43 32 / Fax: (91) 782 43 27
E-mail: secretaria@ctomedicina.com; iberocto@ctomedicina.com WEB: www.ctomedicina.com; www.iberocto.com

Preguntas TEST

99.

NEFROLOGA

Preparacin Examen de Seleccin 05/06 1 Vuelta


Pregunta 1.- R: 3
Se define el aclaramiento de creatinina como el volumen de plasma que se limpia de creatinina al pasar por el rin en un minuto.
Se determina mediante la ecuacin:
CCr=

Volo x [Cr]o
[Cr]p

x 100

Su valor normal es de Ccv =100-120ml/min.


Es el mejor parmetro para determinar el filtrado glomerular, es
decir, la funcin renal.
Pregunta 2.- R: 4
El filtrado glomerular tiene un mecanismo de autorregulacin muy
complejo. La sustancia fundamental que interviene es la angiotensina II
(Ag II), que acta sobre todo en la arteriola eferente.
Cuando la perfusin del glomrulo disminuye, la Ag II constrie la
arteriola eferente, aumentando la filtracin al disminuir el drenaje de
sangre por esta arteriola.
Pregunta 3.- R: 2
El asa de Henle se introduce en la mdula renal, es decir en el gradiente
de concentracin. En la porcin descendente se concentra la orina y en
la rama ascendente se vuelve a diluir, ello permite la formacin del gradiente
y su aprovechamiento para concentrar la orina.
Las nefronas se diferencian por la longitud de su asa de Henle. Cuanto
mas larga es, ms profundiza en el gradiente y puede concentrar ms.
La concentracin final de la orina depende del tbulo colector.

Comentarios TEST

Pregunta 4.- R: 3
La renina es una enzima producida y almacenada en los grnulos de
las clulas yuxtaglomerulares del intersticio medular renal. Actua sobre el
angiotensingeno, que es una globulina producida por el hgado, transformndolo en angiotensina I, que a su vez es convertida por la enzima
conversora de angiotensina (ECA), especialmente abundante en endotelio
vascular pulmonar, en angiotensina II, que estimula la produccin de
aldosterona en la zona glomerular de la corteza suprarrenal.
Pregunta 5.- R: 3
Aproximadamente por los dos riones pasa 1 litro de sangre por minuto
(1/5 gasto cardaco), es decir aproximadamente 550 ml de plasma.
Para su determinacin se utiliza el aclaramiento de PAH (cido
paraaminohiprico). Esta sustancia se filtra en los glomrulos y difunde desde los vasos a los tbulos, siendo nula la cantidad que sale por
la vena renal; por ello permite conocer el volumen plasmtico renal.
Pregunta 6.- R: 3
Hay dos sustancias que tienen su mxima reabsorcin en el asa de
Henle: el Iodo y el Mg.
El resto de sustancias se reabsorben fundamentalmente en el tbulo proximal.
Pregunta 7.- R: 2
La glomerulonefritis se define como aquel proceso de etiologa
inmunitaria en los que hay inflamacin de los glomrulos. Hablamos
de glomerulonefritis aguda cuando existe oligoanuria. Son agudas:
Las que cursan con sndrome nefrtico.
Las GN rpidamente progresivas primarias o secundarias.
Cuando aparece trombosis de la vena renal bilateral.
Pregunta 8.- R: 3
La nefropata mesangial IgA es la ms frecuente en Espaa.
Aunque el 50% de enfermos tiene elevacin de IgA en plasma, el
100% de pacientes tiene depsito mesangial de IgA difuso en la inmunofluorescencia.
Es la nica GN que necesita ser diagnosticada mediante la realizacin de inmunofluorescencia, para demostrar la IgA depositada.

M exico A rgentina
C hile U ruguay

Seguimiento a distancia

Pregunta 8. Distribucin etaria de las glomerulonefritis.


123456

7829
2694 26
64 3 

7829
2694 26

13 

9 



12345

 ! "#!$#%&

12345

2315

'273

15

15345

'6

62

()

'6* %" 4

62

'6* %" 44

62

Pregunta 9.- R: 4
Hay 3 mecanismos de formacin de GN (ver tabla en pgina siguiente):
Anticuerpos antimembrana basal del glomrulo (AMBG): 2% del
total. Caracterstico de la enfermedad de Goodpasture. Produce
un patrn de IF lineal.
Inmunocomplejos: constituye el 97% del total. Hay 2 tipos:
- I.C. circulantes, el ms frecuente.
- I.C. formados "in situ" caracterstico de la GN membranosa.
Patrn IF granular.
Activacin del complemento: 1% del total. No hay inmunoglobulinas y produce un patrn granular en la I.F.
Pregunta 10.- R: 1
El sndrome nefrtico se trata con dieta normoproteica y sin sal.
Hay que aadir diurticos al tratamiento. Se comienza con tiacidas y
espirinolactona, y si no es suficiente, se aade tratamiento con diurticos de asa. Es preciso un control estricto para evitar una deplecin
de volumen y que aparezca una N.T.A.
Cuando el edema es refractario a estas medidas, se utiliza la albmina intravenosa.
Pregunta 11.- R: 3
El sarampin puede producir la remisin de G.N. de cambios mnimos por mecanismos no bien aclarados (posiblemente por modificacin de la inmunidad celular).
Pregunta 12.- R: 3
Este cuadro clnico es compatible con un sndrome nefrtico. El
antecedente de faringoamigdalitis nos debe hacer pensar en la aparicin de una glomerulonefritis postinfecciosa como causa probable
de este cuadro.
Esta glomerulonefritis cursa con hipocomplementemia durante un
perodo no mayor de 8 semanas. Si se prolonga, es preciso realizar
biopsia renal para descartar L.E.S. o una glomerulonefritis membranoproliferativa.
Pregunta 13.- R: 5
Hay 3 tipos de glomerulonefritis rpidamente progresivas (GNRP)
segn el depsito inmunitario:
Tipo I: Anticuerpos antimembrana basal glomerular.
Tipo II: Por inmunocomplejos (puede ser hipocomplementmica).
Tipo III: La ms frecuente, puede tener o no depsito de ANCA.
Pregunta 14.- R: 4
En la endocarditis infecciosa tambin puede haber dao glomerular.
El mecanismo est mediado por inmunocomplejos circulantes (ICC).
En el glomrulo se puede encontrar una GN membranoproliferativa, aparicin de GNRP o una GN postinfecciosa.
Pregunta 15.- R: 3
Cuando en el curso evolutivo de una GN aparece un cuadro de
oligoanuria, hay que plantearse fundamentalmente la aparicin de
una GNRP.

CTO Medicina C/ Nez de Balboa, 115 28006 MADRID (Espaa) Tfno.: (91) 782 43 32 / Fax: (91) 782 43 27
E-mail: secretaria@ctomedicina.com; iberocto@ctomedicina.com WEB: www.ctomedicina.com; www.iberocto.com

IG Pg. 1

NEFROLOGA

Seguimiento a distancia

Preparacin Examen de Seleccin 05/06 1 Vuelta


Pregunta 9. Principales caractersticas de las glomerulonefritis.
GN R P

Proliferacin
endocapilar difusa
ex udativa
(PMN).

Proliferacin
ex tracapilar
semilunas
>50%.

IF

Granulares
subepiteliales.

I: Lineal.
II: Granulares.
III:No depsitos.

ME

Humps (jorobas).

MO

Clnica

Sd. nefrtico.

Infeccin por
estreptococo hemoltico del
Asociaciones
grupo A.
Latencia 1-3
semanas.

Cambios
mnimos

GN membranosa

GEFS

Engrosamiento de la
pared capilar.

Esclerosis
glomerular
segmentaria y
focal.

Granulares
subepiteliales.

I: Subendoteliales.
Mesangiales.
II: Intramembranosos. Si IgA-Berger.

Granulares.

Sd. nefrtico,
hematuria,
Sd. nefrtico.
II: la de peor
pronstico.

Hematuria;
Berger slo
hematuria
recidivante.

A veces
proteinuria
que no llega a
rango
nefrtico.

Berger: infeccin
en resolucin
(latencia de unos
das).

VIH, herona,
reflujo,
rechazo
trasplante.

Fusin de
podocitos.

Spikes
(espigas).

Insuficiencia
renal
rpidamente
progresiva.

Nios (80%)
pronstico
ex celente.

Adulto (35%)
trombosis vena
renal.

I: Goodpasture.
II: Final de
muchos
procesos.

Hodgkin;
atopia (HLAB12).

Carcinomas,
hepatitis B, LES
D -penicilamina,
sales de oro,
captopril.

Pregunta 16.- R: 4
El tratamiento ante la sospecha de un sndrome nefrtico en un
nio es el esteroideo, ya que la causa ms frecuente de este sndrome
en la infancia es la nefropata de cambios mnimos, y responde en un
90% a dicho tratamiento.
Cuando no hay respuesta, hay que plantearse la presencia de un
cuadro de hialinosis focal y segmentaria y realizar una biopsia profunda para coger tejido yuxtamedular, ya que este cuadro suele comenzar en esta zona.
Pregunta 17.- R: 4
La GN inmunotactoide es un cuadro de afectacin glomerular de
causa desconocida que afecta fundamentalmente a varones adultos,
cursando con proteinuria y fracaso renal progresivo.
Hay depsito de material proteico no amiloide en el glomrulo.
Pregunta 18.- R: 1
Cuando nos referimos a una afectacin focal, significa que el dao
afecta a unos glomrulos s y a otros no.
Cuando hablamos de afectacin segmentaria, nos referimos a que
hay dao de una parte del glomrulo, no de todo ste.
Pregunta 19.- R: 1
La afectacin por anticuerpos antimembrana basal produce un
patrn de inmunofluorescencia lineal para IgG, y a veces para C3.
Pregunta 20.- R: 3
La glomerulonefritis postinfecciosa tiene mejor pronstico en nios que en adultos, no habiendo clara diferencia en cuanto al sexo.
Hay que realizar biopsia renal en las siguientes situaciones:
La hematuria dura ms de 6 meses.
La hipocomplementemia dura ms de 8 semanas.
La oligoanuria dura ms de 21 das.
M exico A rgentina
C hile U ruguay

GN mesangial

I: Proliferacin
mesangial
("ral de tren").
Proliferacin
II: engrosamiento
difusa.
membrana basal
(depsitos densos).

Si el enfermo est diagnosticado de un sndrome nefrtico previo,


hay que pensar antes en un cuadro de trombosis venosa renal (TVR),
y si se descarta esta ltima, pensamos en una GNRP.

Pg. 2 IG

GNMP

I: Hepatitis crnica
activa.
II: Lipodistrofia
parcial, hemlisis.

Pregunta 21.- R: 3
Ante un sndrome nefrtico en un adulto hay que realizar una
biopsia de entrada. No se realiza si conocemos una causa en cuya
evolucin clnica pueda aparecer dicho sndrome, por ejemplo, si el
paciente es diabtico.
Sin embargo, si a pesar de conocer una muy probable causa el
curso del sndrome nefrtico es atpico (muy rpido, tienen hematuria, etc), hay que realizar biopsia renal.
Pregunta 22.- R: 2
En la GN mesangial IgA es muy rara la aparicin de un sndrome
extracapilar.
Pregunta 23.- R: 3
Aunque la etiologa ms frecuente de la nefropata membranosa es
idioptica, siempre hay que tener en cuenta la presencia de una
neoplasia slida (cncer de estmago, pulmn, colon), frmacos (Captopril, Penicilamina y sales de oro, etc), metales (Pb, Hg y Cd), los
procesos colgeno-vasculares y la hepatitis B.
Pregunta 24.- R: 2
En este caso clnico, la presencia de hematuria asociada a faringitis
aguda y la existencia de un episodio similar hace un ao, sin oliguria
acompaante, nos habla de la presencia de una nefropata mesangial
IgA probable. Si el paciente tuviese un cuadro de hematuria con
oliguria, nos orientara a una GN endocapilar difusa.
Pregunta 25.- R: 4
Una caracterstica de la GN membranoproliferativa es la probable
asociacin con un cuadro de lipodistrofia, sobre todo asociado al
tipo II (enfermedad por depsitos densos).
A pesar de que haya sospecha de lipodistrofia, hay que realizar
biopsia cutnea y confirmarla.
Pregunta 26.- R: 1
En este caso, el nico cuadro que tiene descenso en el complemento es la GN membranoproliferativa o mesangiocapilar.

CTO Medicina C/ Nez de Balboa, 115 28006 MADRID (Espaa) Tfno.: (91) 782 43 32 / Fax: (91) 782 43 27
E-mail: secretaria@ctomedicina.com; iberocto@ctomedicina.com WEB: www.ctomedicina.com; www.iberocto.com

Comentarios TEST

GN A

NEFROLOGA

Preparacin Examen de Seleccin 05/06 1 Vuelta


Pregunta 27.- R: 3
La etiologa ms frecuente de la GN focal y segmentaria con hialinosis es la idioptica. Causas secundarias que hay que recordar sobre
todo para el MIR son:
El reflujo vesicoureteral.
Las nefropatas tubulointersticiales crnicas.
La evolucin en ocasiones de la nefropata de cambios mnimos.
Nefropata glomerular del paciente con infeccin por HIV, SIDA o
A.D.V.P.
Pregunta 28.- R: 3
La causa ms frecuente de sndrome nefrtico a partir de los 50
aos es la nefropata membranosa. El 20% debuta con proteinuria y
hematuria asintomticas.Este proceso es el ejemplo de dao glomerular
por formacin de IC "in situ".
Con microscopa electrnica en fases iniciales nos encontramos una
imagen caracterstica, los "spikes" o picos, que ayudan al diagnstico.
Pregunta 29.- R: 3
La ausencia de cilindros hemticos no excluye un cuadro de
glomerulonefritis.
Es preciso buscar la presencia de hemates deformados en la orina
ante todo cuadro de hematuria, ya que indican origen glomerular. El
nico falso positivo es la presencia de orinas hipotnicas.
Posibles causas de cilindros hemticos son:
Glomerulonefritis agudas y crnicas.
Vasculitis.
Crioglobulinemia.
Pregunta 30.- R: 3
Aunque el cuadro caracterstico de las GN membranoproliferativas es la presencia de hematuria y proteinuria asintomticas, tambin
pueden cursar con sndrome nefrtico, sndrome nefrtico o un cuadro de oligoanuria por la presencia de GNRP.

Comentarios TEST

Pregunta 31.- R: 5
La hipocomplementemia asociada a la GN endocapilar o postinfecciosa no puede durar mas de ocho semanas, ya que ello induce a
pensar en otros procesos hipocomplementmicos y que pueden cursar con sndrome nefrtico, como son el LES y las GN membranoproliferativas. Por ello, si el complemento bajo dura ms de ocho semanas, est indicada la biopsia renal.
Pregunta 32.- R: 3
Las remisiones espontneas son frecuentes en los nios, mientras
que en los adultos se dan entre un 20 y un 40%. La glomerulonefritis
membranosa progresa a insuficiencia renal progresiva en un 20- 30%
de los pacientes, siendo este hecho ms frecuente en adultos. Si este
deterioro de la funcin renal es rpido debe pensarse en la existencia
de una trombosis de la arteria renal o una glomerulonefritis con
semilunas asociadas.
Esta nefropata puede recidivar en el trasplante.
Pregunta 33.- R: 3
Ante un cuadro de sndrome nefrtico en un adulto mayor de 50
aos hay que pensar como primera posibilidad en una GN membranosa. La etiologa secundaria ms frecuente es una neoplasia.
Pregunta 34.- R: 2
La nefropata IgA puede tener ocasionalmente incidencia familiar
y asociar al HLA Bw35.
Es la GN ms frecuente en Espaa, y por tanto la que ms recidiva
en el trasplante (en nuestro pas).
Pregunta 35.- R: 3
Una mujer joven con cuadro de afectacin renal y complemento
bajo nos debe hacer pensar en un cuadro de nefropata lpica.
Recordad que el cuadro glomerular ms benigno es la GN mesangial lpica y el cuadro ms agresivo y de peor pronstico es la GN
proliferativa difusa lpica.

M exico A rgentina
C hile U ruguay

Seguimiento a distancia

Afecta ms a la mujer
(25-30 aos)

Hipocomplementemia
Aumento de la creatinina
Aumento de la urea
Hipertensin hiperreninmica

Hematuria
Sd. nefrtico
(sedimento telescopado)

La afectacin renal en el lupus


ensombrece el pronstico

Lesiones inflamatorias
- Proliferacin endocapilar.
- Infiltrado leucocitario.
- Depsitos en asa de alambre.
- Semilunas epiteliales.
- Inflamacin del intersticio.

Lesiones irreversibles
- Glomeruloesclerosis.
- Semilunas fibrosas.
- Atrofia tubular.
- Fibrosis intersticial.

Pregunta 35. Afectacin renal en el LES.

Pregunta 36.- R: 5
En este caso clnico, la presencia de oliguria, hematuria y proteinuria con cilindros hemticos nos orienta al diagnstico de sndrome
nefrtico. Este proceso se produce cuando hay una inflamacin aguda del glomrulo. La causa ms frecuente es postinfecciosa, pero procesos inflamatorios que afectan al glomrulo tambin lo pueden producir (LES, etc).
Pregunta 37.- R: 3
La nica GN que puede recurrir en 24/48h es la hialinosis focal y
segmentaria.
Pregunta 38.- R: 2
La anemia que se produce en el fracaso renal agudo y crnico es
de etiologa multifactorial:
Disminucin de Epo.
Hemorragia digestiva.
Hemlisis.
Otros.
En la glomerulonefritis aguda, la presencia de oligoanuria con retencin de volumen de agua y sal en el plasma produce mecanismo
dilucional como causa de anemia.
Pregunta 39.- R: 4
La nefrosis lipoidea o nefropata de cambios mnimos se asocia en
ocasiones a linfoma de Hodgking. No se conoce el motivo de esta
asociacin. Podra obedecer a un trastorno de inmunidad celular.
Pregunta 40.- R: 3
De los 3 mecanismos de produccin de GN el ms frecuente implicado en la produccin de GN aguda son los IC circulantes.
Pregunta 41.- R: 2
La hialinosis focal y segmentaria suele comenzar afectando a los
glomrulos yuxtamedulares.

CTO Medicina C/ Nez de Balboa, 115 28006 MADRID (Espaa) Tfno.: (91) 782 43 32 / Fax: (91) 782 43 27
E-mail: secretaria@ctomedicina.com; iberocto@ctomedicina.com WEB: www.ctomedicina.com; www.iberocto.com

IG Pg. 3

Seguimiento a distancia

NEFROLOGA

Preparacin Examen de Seleccin 05/06 1 Vuelta

Suele manifestarse con hematuria y proteinuria asintomticas o


sndrome nefrtico.
Pregunta 42.- R: 2
En este caso, la cifra de C3 descendido puede estar asociado a GN
membranosa proliferativa o bien una GN endocapilar difusa. La ausencia de oligoanuria nos hace pensar en primer lugar en una GN
membranoproliferativa.
Pregunta 43.- R: 3
La presencia de lipodistrofia asociada a un cuadro de GN nos orienta
a pensar en una GN membranoproliferativa (sobre todo un tipo II).
Pregunta 44.- R: 4
La afectacin del intersticio se caracteriza por la presencia de trastornos de concentracin y acidificacin de la orina. El paciente tiene
poliuria, nicturia y un cuadro de acidosis metablica hiperclormica
(anin gap normal).
En la orina hay leucocituria que indica la presencia de inflamacin en el intersticio. La protena intersticial es la B2 microglobulina.
Pregunta 45.- R: 3
Es rara la hipertensin precoz en el dao intersticial, ya que existe
un fenmeno de prdida de agua y sal, por la dificultad de concentracin de la orina.
La hipertensin puede aparecer en estadios avanzados debido a la
fibrosis intersticial que libera renina.
Sin embargo, el rin gotoso produce hipertensin arterial ms
precoz que otros procesos de intersticio crnico debido a la asociacin de hiperplasia de la ntima arteriolar motivada por los cristales de
cido rico.
Pregunta 46.- R: 1
Hay 2 etiologas de dao intersticial que pueden producir uroteliomas:

Nefropata por analgsicos.


Nefropata de los Balcanes.
Ambas son ms frecuentes en mujeres (por ello aumentan la incidencia de uroteliomas en el sexo femenino).
Respuesta 47.- R: 2
La infeccin del intersticio se denomina pielonefritis. Puede ser
aguda o crnica. La causa ms frecuente es el reflujo de orina y el
germen mas habitual el E. coli.
Pregunta 48.- R: 4
Las nefropatas intersticiales se presentan con dificultad de concentracin y acidificacin de la orina.
Pregunta 49.- R: 1
En las nefropatas intersticiales agudas, el infiltrado es fundamentalmente de polimorfonucleares. Sin embargo, en los procesos crnicos
est formado por linfocitos, monocitos y fibroblastos, con formacin
de fibrosis y retraccin del intersticio.
Pregunta 50.- R: 5
La nefropata por analgsicos crnica es ms frecuente en mujeres,
sobre todo en aquellas que padecen cuadros de depresin con somatizacin
y sndrome fibromilgico (ver figura a pie de pgina). Se requiere la toma de
por lo menos 1 Kg de analgsicos en 1 ao para que se produzca. Es ms
frecuente en pases centroeuropeos. En Espaa es rara.
Pregunta 51.- R: 1
Pregunta frecuente en el examen MIR. El primer dato de la clnica
del proceso intersticial es el trastorno de concentracin de la orina.
Pregunta 52.- R: 4
La segunda causa de necrosis de papila es la nefropata por analgsicos crnica. La causa ms frecuente es la microangiopata diabtica.

Lesin tubulointersticial
difusa en la corteza

Hipertensin arterial
Piuria con cultivos
negativos

Anemia
desproporcionada
con el grado de
Insuficiencia renal

Signo del anillo


en la pielografa
Necrosis crnica de papila

Mujer (ms frecuente anglosajona) que


abusa de los AINES (fenatecinas,...)

Gastritis o lcera pptica

Pregunta 50. Nefropata por analgsicos.

Pg. 4 IG

M exico A rgentina
C hile U ruguay

CTO Medicina C/ Nez de Balboa, 115 28006 MADRID (Espaa) Tfno.: (91) 782 43 32 / Fax: (91) 782 43 27
E-mail: secretaria@ctomedicina.com; iberocto@ctomedicina.com WEB: www.ctomedicina.com; www.iberocto.com

Comentarios TEST

Aumento de la
incidencia de carcinomas
transicionales de
urter y pelvis renal

NEFROLOGA

Preparacin Examen de Seleccin 05/06 1 Vuelta


Pregunta 53.- R: 2
Cuando un paciente tiene diagnosticada una nefropata intersticial
por Pb y tiene en su evolucin un sndrome nefrtico, hay que pensar
en un cuadro de hialinosis focal y segmentaria como causa de dicho
sndrome. En su patogenia est implicada la formacin de IC circulantes con el concurso de antgenos intersticiales.
Pregunta 54.- R: 4
Cuando un paciente tiene un fracaso renal y clnica de poliuria y
nicturia tenemos que pensar en un cuadro intersticial. Existe asociado
un cuadro de acidosis metablica hiperclormica por trastorno de
acidificacin de la orina.
Pregunta 55.- R: 5
La nefropata de los Balcanes est producida por un virus RNA
(coronavirus) que infecta el intersticio. Este germen produce cambios
en el urotelio con aparicin de metaplasia, displasia y cncer urotelial. Por ello es preciso realizar citologas de control en estos pacientes
en las revisiones.
Pregunta 56.- R: 5
Ya comentado con anterioridad (pregunta 45), el cido rico en el
intersticio produce dao en la pared vascular por hiperplasia endotelial, produciendo liberacin de renina.
Pregunta 57.- R: 5
La fibrosis intersticial produce traccin y ensancha la luz tubular,
haciendo mayor el tamao de los cilindros. Son creos por su coloracin (formados por material celular degenerado).
Pregunta 58.- R: 4
Las pielonefritis se producen frecuentemente por la existencia de
reflujo vesicoureteral. El germen ms implicado es el E. coli, ya que es
el microorganismo ms frecuente en las infecciones de orina.

Proteinuria de Bence-Jones.
Sndrome nefrtico por amiloidosis AL asociada.
Pregunta 65.- R: 1
En la enfermedad de Alport-Perkoff existe afectacin de distintos
rganos:
Rin.
Ojos: malformaciones en el cristalino (esferofaquia, lenticono).
Odos: sordera o hipoacusia de percepcin.
Pregunta 66.- R: 4
Cuando se trata con quimioterapia una neoplasia hematolgica se
libera gran cantidad de purinas que dan lugar a cido rico que
puede filtrarse y obstruir los tbulos de la nefrona.
Es preciso prevenir este proceso con abundantes lquidos, bicarbonato y alopurinol.
Pregunta 67.- R: 2
Cuando la diabetes afecta al glomrulo, aparece microalbuminuria
por aparicin de un cuadro de glomeruloesclerosis difusa intercapilar.
Es preciso confirmar esta proteinuria dos veces ms en 6 meses, ya
que su confirmacin es indicacin de tratamiento con IECAS.
EVOLUCIN

NEFROPATA

Inicio
hiperglucemia

Estadio I
+ tamao renal

Comentarios TEST

Pregunta 61.- R: 4
En los procesos de intersticio crnico, la dificultad de reabsorcin
a nivel proximal junto con el mayor aporte de sodio al tbulo distal y
mayor intercambio de Na+ por K+ conlleva la aparicin en ocasiones
de deplecin de potasio.
Pregunta 62.- R: 3
La pielonefritis xantogranulomatosa es una patologa muy poco
frecuente. Ocurre en enfermos con infecciones de orina por grmenes urealticos (como por ejemplo el Proteus mirabilis) y con mecanismo de reflujo, junto con una dificultad de los macrfagos del intersticio renal para eliminar lpidos que hacen que se llenen de grasa.
Como consecuencia, el parnquima renal se deforma dando morfologa pseudotumoral. El cuadro clnico a veces se confunde con
una neoplasia (fiebre, leucocitosis, VSG , dolor).
Su tratamiento es la nefrectoma.
Pregunta 63.- R: 1
La enfermedad de Alport es gentica. Suele estar ligada la herencia
al cromosoma X. Hay defecto del colgeno tipo IV. La membrana
basal esta laminada como un "hojaldre". Por ello es frecuente la hematuria y la proteinuria rica en albmina.
Pregunta 64.- R: 5
El mieloma mltiple puede dar lugar a proteinuria abundante por
2 mecanismos:
M exico A rgentina
C hile U ruguay

TERAPUTICA

10-15 aos

Estadio II
Microalbuminuria
intermitente
Estadio III
M. Persistente

Pregunta 59.- R: 3
Ya explicado anteriormente (pregunta 56).
Pregunta 60.- R: 3
La leucocituria en orina es indicativo de la existencia de procesos
inflamatorios, pero no necesariamente de etiologa infecciosa; es por
ello que, sea cual sea la causa de la inflamacin del intersticio, se produce leucocituria, apareciendo en ocasiones cilindros leucocitarios.
La piuria es la leucocituria de origen pigeno.

Seguimiento a distancia

Control metablico
Disminuir HTA (eleccin IECA)
Restriccin proteica

3-7 aos

Proteinuria manifiesta
(Macroproteinuria >300 mg/d)

Estadio IV

Nefropata clnica

Control estricto
de la HTA (eleccin IECA)

5-7 aos

I.R. Terminal

Estadio V

Dilisis
Trasplante

Pregunta 67. Dao renal en la DM.

Pregunta 68.- R: 1
La lesin ms frecuente en el glomrulo del rin diabtico es la
glomeruloesclerosis difusa intercapilar. Sobre sta puede aparecer en
el 15% de los pacientes un cuadro de glomeruloesclerosis nodular
que suele manifestarse como sndrome nefrtico.
Pregunta 69.- R: 4
La asociacin hematuria-hemoptisis se denomina sndrome de
Goodpasture. La causa ms frecuente de este cuadro es la Enfermedad de Goodpasture.
El segundo cuadro en frecuencia es la vasculitis de Wegener (cANCA positivo en el 97% de los casos).
Cuando el paciente presenta en la analtica p-ANCA pensamos en
un cuadro de PAN microscpica.
Pregunta 70.- R: 4
Algunos pacientes con procesos inflamatorios crnicos (AR, EA,
etc.) y portadores de procesos infecciosos crnicos pueden desarrollar un sndrome nefrtico secundario a la presencia de amiloidosis
con afectacin glomerular.

CTO Medicina C/ Nez de Balboa, 115 28006 MADRID (Espaa) Tfno.: (91) 782 43 32 / Fax: (91) 782 43 27
E-mail: secretaria@ctomedicina.com; iberocto@ctomedicina.com WEB: www.ctomedicina.com; www.iberocto.com

IG Pg. 5

Seguimiento a distancia

NEFROLOGA

Preparacin Examen de Seleccin 05/06 1 Vuelta

Pregunta 77.- R: 4
La PAN clsica afecta en el 90% de pacientes al rin y suele debutar
con HTA vasculorrenal. Es frecuente la aparicin de microaneurismas a
nivel renal que pueden ayudar al diagnstico (arteriografa).
Pregunta 78.- R: 3
Cuando aparece proteinuria en la nefropata diabtica hay que
iniciar tratamiento con IECA, ya que han demostrado que pueden
remitir la proteinuria.
Cuando existen contraindicaciones para su uso (tos severa, angioedema, artritis por IECAs), se deben usar ARA II.
Pregunta 79.- R: 5
Los cuerpos de hematoxilina son restos nucleares que se tien, (con
tcnicas de hematoxilina-eosina) de rojo. Son patognomnicos de LES.
Pregunta 80.- R: 4
El cuadro que tiene peor pronstico es la GN proliferativa difusa
lpica. El cuadro con mejor pronstico es la GN mesangial lpica.
Pregunta 81.- R: 3
Pacientes con cuadros de osteomielitis crnica, bronquiectasias o
portadores de sonda vesical permanente, o enfermos parapljicos
con escaras tienen mayor probabilidad de desarrollar un cuadro de
sndrome nefrtico secundario a una amiloidosis AA.
Pregunta 82.- R: 3
La plasmafresis con recambio del plasma es de mucha utilidad en
procesos como:
Enfermedad de Goodpasture.
Crioglobulinemia.
Rin de mieloma.

Pregunta 71.- R: 4
Se trata de una enfermedad de Wegener por la presencia del
cuadro de hematuria y hemoptisis, asociado a infiltrados cavitados
y rinitis de repeticin (afectacin ORL en el 97% de los pacientes).
El c-ANCA est presente en un 97% de los pacientes y es un
marcador diagnstico y pronstico de la enfermedad. No es causa
de la enfermedad. Sirve para el seguimiento y pronstico del enfermo.
Pregunta 72.- R: 3
La presencia de LDH elevada en sangre y/o orina indica necrosis
tisular. Cuando ocurre unido a un cuadro de lumbalgia y hematuria
nos plantearemos el diagnstico de oclusin arterial trombtica o
emblica con infarto isqumico del parnquima. En la trombosis de
la vena renal no aumenta la LDH.
Pregunta 73.- R: 4
La enfermedad de Alport raramente produce un sndrome nefrtico. El resto de las opciones son incorrectas.
Pregunta 74.- R: 2
El cadmio puede producir adenocarcinoma renal y uroteliomas.
A nivel glomerular puede dar un cuadro de nefropata membranosa
y tambin puede producir una nefropata intersticial crnica.
Pregunta 75.- R: 4
La afectacin glomerular ms frecuente encontrada en el SIDA es
la hialinosis focal y segmentaria.
Pregunta 76.- R: 3
La aparicin de un cuadro de lumbalgia y hematuria en el seno de
un sndrome nefrtico nos har pensar en la aparicin de trombosis
de la vena renal (TVR). El 15% se asocia a TEP, y por ello la TVR es
causa de sndrome hematuria-hemoptisis.

Pg. 6 IG

M exico A rgentina
C hile U ruguay

Pregunta 83.- R: 4
El sntoma que siempre mejora con dilisis es la anorexia. Sin embargo no se modifica el prurito, al estar mediado por PTH y calcificaciones subcutneas.
Pregunta 84.- R: 2
El enfermo con I.R. crnica tiene aterosclerosis generalizada, y por
ello el riesgo de eventos cardiovasculares (IAM y ACV) es ms frecuente. Por esta razn, hay que asegurarse una adecuada oxigenacin
mediante una adecuada cifra de hemoglobina 10-11g. Para ello utilizaremos E.P.O. y/o transfusiones.
Pregunta 85.- R: 2
La vasculitis de Wegener afecta en el 100% de los pacientes al
pulmn, aunque la Rx de trax sea normal.
El sistema ORL se afecta en el 97% de los casos, con rinitis, sinusitis,
mastoiditis, etc.
El rin aparece afectado en un 70% de los casos. La lesin predominante es la GN membranoproliferativa, apareciendo hematuria y
proteinuria en el sedimento de orina.
Pregunta 86.- R: 4
La causa principal de TVR presenta diferencias etarias:
En los neonatos, la causa ms frecuente es la deshidratacin hipertnica.
En la infancia, la causa ms frecuente es la GN de cambios mnimos.
En el adulto, las causas ms frecuentes son la GN membranosa y la
amiloidosis.
Pregunta 87.- R: 4
Los microaneurismas suelen ser caractersticos de la panarteritis
nodosa.
Pregunta 88.- R: 1
La causa ms frecuente de sndrome de Goodpasture es la Enfermedad de Goodpasture, por ello la primera prueba que hay que
pedir es Ac anti-MBG. Son positivos en el 90% de los enfermos.

CTO Medicina C/ Nez de Balboa, 115 28006 MADRID (Espaa) Tfno.: (91) 782 43 32 / Fax: (91) 782 43 27
E-mail: secretaria@ctomedicina.com; iberocto@ctomedicina.com WEB: www.ctomedicina.com; www.iberocto.com

Comentarios TEST

Pregunta 70. Amiloidosis en la AR de larga evolucin.

NEFROLOGA

Preparacin Examen de Seleccin 05/06 1 Vuelta


1) Insuficiencia prerrenal (descenso de perfusin renal).

Disminucin absoluta del volumen de sangre efectivo.


- Hemorragia.
- Prdidas cutneas: quemaduras, sudor.
- Prdidas gastrointestinales: diarrea, vmitos.
- Prdidas renales: diurticos.
- Tercer espacio: peritonitis, quemaduras.
Disminucin relativa del volumen de sangre efectivo.
- Insuficiencia cardaca congestiva.
- Arritmias.
- Cirrosis heptica: ascitis, sndrome hepatorrenal.
- Sndrome nefrtico.
- Hipotensin de cualquier etiologa.
- Fracaso hemodinmico intraglomerular:
Antiinflamatorios no esteroideos.
Inhibidores del enzima de conversin.
Hipercalcemia.
Aminas vasoconstrictoras.
Oclusin arterial.
- Tromboembolismo bilateral.
- Tromboembolismo de un rin solitario.
- Aneurisma de la arteria renal o aorta.

1
3

2) Insuficiencia parenquimatosa.

3) Insuficiencia postrenal.

Comentarios TEST

Intraluminal.
- Litiasis: rica, clcica, infecciosa.
- Cogulos.
- Necrosis papilar.
- Tumores: hipernefroma, urotelioma.
Intraparietal.
- Malacoplaquia.
- Estenosis: congnita, postinfecciosa,
postraumtica.
- Tumores.
- Anomalas congnitas.
Compresin extrnseca.
- Malformacin congnita: urter retrocavo, rin
en herradura, bridas vasculares.
- Hiperplasia prosttica.
- Adenocarcinoma prosttico.
- Fibrosis retroperitoneal: metisergida, neoplasias.
- Tumores: prstata, pelvis congelada.
- Ligadura yatrgena de urter.
Disfuncin neurgena.
- Vejiga neurgena.
espstica
automtica
Lesin
Lesinsobre
sobreL1-L3:vejiga
L1-L3: vejiga
tona ooflccida.
Lesin bajo L1-L3: vejiga tona o flccida.
- Disfuncin de la unin pieloureteral.
- Reflujo vesicoureteral: con el mismo efecto
hidrodinmico que una obstruccin real.
Obstruccin venosa.
- Trombosis venosa renal.
- Neoplasia.

Seguimiento a distancia

Alteraciones vasculares.
- Vasculitis.
- Hipertensin arterial maligna.
- Sustancias vasoconstrictoras.
- Eclampsia.
- Microangiopata trombtica.
- Estados de hiperviscosidad.
- Antiinflamatorios no esteroideos.
- Hipercalcemia.
Alteraciones glomerulares.
- Glomerulonefritis agudas.
Alteraciones tubulares.
- Necrosis tubular isqumica.
Hipotensin profunda: hemorragia, shock, ICC.
Rin trasplantado.
Frmacos vasoconstrictores.
- Necrosis tubular txica.
Nefrotxicos.
Antibiticos (aminoglucsidos, anfotericina B, etc.)
Metales (mercurio, bismuto, arsnico, plata, cadmio,
hierro, antimonio).
Disolventes (tetracloruro de carbono, glicol,
tetracloruro de etileno).
Contrastes radiolgicos yodados.
Pigmentos intratubulares.
Hemoglobinuria.
Mioglobinuria.
Bilirrubinuria.
Protenas intratubulares.
Mieloma mltiple.
Cristales intratubulares.
Acido rico.
Oxalatos.
Alteraciones tubulointersticiales.
- Por frmacos.
- Por infecciones.
- Por radiacin.

Pregunta 93. Clasificacin del fracaso renal agudo.

Pregunta 89.- R: 4
Los hallazgos ms frecuentes en la orina de la enfermedad de
Wegener son la hematuria y proteinuria asintomticas.
Pregunta 90.- R: 3
La afectacin glomerular por amiloide condiciona la aparicin de
albuminuria.
Pregunta 91.- R: 4
El sedimento telescopado es caracterstico del LES y algunas vasculitis. Es un sedimento con todo tipo de cilindros.
Pregunta 92.- R: 2
La crioglobulinemia afecta al rin en ms del 85% de los pacientes (principalmente el tipo II o crioglobulinemia mixta esencial, asoM exico A rgentina
C hile U ruguay

ciada en el 8% de los casos a hepatitis por virus C).


Se manifiesta en el sistemtico de orina por hematuria y proteinuria asintomticas.
El cuadro ms caracterstico de afectacin glomerular es la GN
membranoproliferativa.
Al microscopio da una imagen caracterstica de pseudotrombos de
globulinas.
Pregunta 93.- R: 1
La causa ms frecuente de azoemia prerrenal es la deshidratacin.
La insuficiencia renal prerrenal es la causa ms frecuente de insuficiencia renal aguda.
En la tabla puedes ver otras causas de fracaso renal segn el nivel de
afectacin de la nefrona.

CTO Medicina C/ Nez de Balboa, 115 28006 MADRID (Espaa) Tfno.: (91) 782 43 32 / Fax: (91) 782 43 27
E-mail: secretaria@ctomedicina.com; iberocto@ctomedicina.com WEB: www.ctomedicina.com; www.iberocto.com

IG Pg. 7

NEFROLOGA

Seguimiento a distancia

Preparacin Examen de Seleccin 05/06 1 Vuelta

Pregunta 95.- R: 1
En los enfermos agudamente oligricos, lo fundamental en el
tratamiento es corregir la hipovolemia mediante expansin de volumen.
En el caso de insuficiencia cardaca, no se puede expandir y hay que
tratar al paciente con dobutamina para aumentar el gasto cardaco.
La nica indicacin de expansin de volumen en enfermos oligricos con clnica de ICC es el infarto masivo del VD.
Pregunta 96.- R: 2
La excrecin fraccional de sodio es el mejor parmetro para diferenciar un fracaso agudo renal prerrenal (<1) de un fracaso parenquimatoso (>1).
Es un cociente de aclaramiento de sodio y creatina; relaciona el
filtrado de sodio con respecto a la funcin renal.
Pregunta 97.- R: 3
Los frmacos ms implicados en la nefropata tubulointersticial
aguda por frmacos son los Betalactmicos y la Rifampicina.
Es un cuadro idiosincrsico y reversible que cursa con fracaso
renal agudo polirico con clnica alrgica asociada.
El 80% de los enfermos tienen eosinofilia y/o eosinofiluria.

Pregunta 100.- R: 4
La enfermedad de Liddle es una tubulopata distal; es una causa de
HTA secundaria, sobre todo en varones jvenes. En esta patologa el
tbulo distal retiene sodio en exceso y elimina K+ en la orina, de tal forma
que siempre cursa con alcalosis metablica y cifras de potasio bajas en el
plasma. Su tratamiento es con Triamtirene y en su defecto amiloride.
Pregunta 101.- R: 5
La enfermedad de Alport tambin se denomina nefritis familiar y es
un cuadro de herencia ligada al cromosoma X.
Existe una alteracin del colgeno tipo IV y se manifiesta por alteraciones renales, oculares y auditivas.
En el ojo puede aparecer lenticono o esferofaquia y en el odo sordera
de percepcin, por ello una prueba muy til es la audiometra.
En el rin lo caracterstico es la aparicin de una membrana basal
en "capa de hojaldre" con microscopio electrnico.
Suele manifestarse con proteinuria y raramente con sndrome
nefrtico.
Pregunta 102.- R: 4
La enfermedad de Bartter es un proceso autosmico recesivo que
afecta al funcionamiento del asa de Henle. Hay disfuncin del
intercambiador Na+: 2 Cl: K+. Ello origina grandes prdidas de potasio por la orina y aparicin de hipopotasemia y alcalosis metablica.
Los pacientes tienen poliuria y pueden tener nefrocalcinosis debido a la presencia de hipercalciuria.
Sndrome de Bartter
Porcin ascendente gruesa del asa de Henle
Aparato
yuxtaglomerular

Pregunta 98.- R: 1
La causa ms frecuente de insuficiencia renal aguda parenquimatosa es la NTA de etiologa isqumica y/o txica.
La excrecin fraccional de sodio en este cuadro clnico es mayor
de 1 y el sodio en orina > 30 mEq/L.

Pregunta 98. Diagnstico diferencial de la IR prerrenal vs intrnseca.


NDICE DIAGNSTICO

FRA PRERRENAL

FRA INTRNSECA

F.E. Na

<1

>1

Concentracin de sodio en
orina (mmol/l)

<10

>20

Proporcin de creatinina en
orina y plasma

>40

<20

Proporcin de nitrgeno
ureico en orina y plasma

>8

<3

Densidad urinaria

>1,018

<1,012

Osmolalidad urinaria

>500

<250

Proporcin entre BUN y


creatinina en plasma

>20

<10-15

ndice de insuficiencia renal

>1

<1

Sedimento urinario

Cilindros hialinos.

Cilindros granulosos
marrn turbios.

Pregunta 99.- R: 1
La poliquistosis renal o enfermedad qustica hepatorrenal es la
quinta causa ms frecuente de I.R. crnica.
Tienen 2 tipos de herencia: autosmica dominante y autosmica
recesiva.
La forma del adulto es autosmica dominante y empieza a manifestarse aproximadamente a la edad de 40 aos.
El sntoma ms frecuente es el dolor en flancos y el que sigue ms
frecuente es la presencia de hematuria.
Es importante recordar la asociacin de esta enfermedad con insuficiencias valvulares cardacas, aneurismas intracraneales, divertculos
intestinales y hernias inguinales.

Pg. 8 IG

M exico A rgentina
C hile U ruguay

Renina
Prostaglandinas

Angiotensina II

Mcula densa

Na+
-

Cl

Mg++

Aldosterona
Tensin arterial
normal

Alcalosis
hipopotasmica

Pregunta 102.Fisiopatologa del sndrome de Bartter.

Pregunta 103.- R: 3
La enfermedad de Gitelman es muy parecida a la enfermedad de
Bartter. En ella no hay hipercalciuria, y por lo tanto, no hay nefrocalcinosis.
Pregunta 104.- R: 4
La hipopotasemia induce la sntesis de prostaglandinas E y F por las
clulas del intersticio medular. Son las responsables de que estos
enfermos tengan normotensin a pesar del hiperaldosteronismo secundario que padecen y que no tengan respuesta hipertensiva al
darles agentes vasoconstrictores.

CTO Medicina C/ Nez de Balboa, 115 28006 MADRID (Espaa) Tfno.: (91) 782 43 32 / Fax: (91) 782 43 27
E-mail: secretaria@ctomedicina.com; iberocto@ctomedicina.com WEB: www.ctomedicina.com; www.iberocto.com

Comentarios TEST

Pregunta 94.- R: 4
Ante un cuadro de hiperpotasemia aguda, el nico frmaco que
puede antagonizar el efecto txico del K+ a nivel cardaco es el
calcio i.v (cloruro o gluconato).
Para disminuir las cifras de K+ hay que utilizar glucosado ms
insulina, salbutamol, bicarbonato 1/6 M intravenoso (para corregir la
acidosis), furosemida y/o dilisis, si es preciso.

NEUMOLOGA Y CIRUGA TORCICA

Preparacin Examen de Seleccin 05/06 1 Vuelta

1.

FISIOLOGA.

4)

Seale la respuesta FALSA:

5)

1)
2)
3)
4)
5)

2.

Seale la relacin FALSA:


1)
2)
3)
4)
5)

3.

2)
3)
4)
5)

Preguntas TEST

2)
3)
4)
5)
5.

El volumen residual se eleva, aunque en algunas enfermedades la elevacin es ms marcada.


La CVF suele estar disminuida.
El patrn obstructivo se acompaa en la mayora de los casos
de una disminucin en la DLCO.
El VEF1 es menor del 80% del terico.
La CPT puede aumentar.

La ventilacin alveolar es:


1)

El volumen de gas contenido en la zona comprendida entre


la trquea y los bronquiolos terminales.
El volumen de gas comprendido en la zona entre bronquiolos
respiratorios y alveolos.
El volumen de gas que no participa del intercambio de gases
por unidad de tiempo.
El volumen de gas intrapulmonar en la posicin de inspiracin mxima.
El volumen de gas que participa del intercambio de gases por
unidad de tiempo.

NO es correcto en relacin a la hipoxemia:


1)
2)
3)

6.

2)
3)
4)
5)
7.

Las alteraciones en la difusin son una causa de hipoxemia


frecuente.
La hipoxemia de las grandes alturas se acompaa de normo
o hipocapnia.
Son causa de hipoxemia las anomalas vasculares con cortocircuito arteriovenoso, que pueden ser hereditarias o adquiridas.
M exico A rgentina
C hile U ruguay

3)
4)
5)

La PaO2 60 mmHg corresponde a una saturacin de Hb del 90%.


Cuando la Hb alcanza una saturacin del 90%, la curva
adquiere un aspecto de meseta, de tal forma que el aumento
de PaO2 modifica escasamente la saturacin.
El desplazamiento de la curva a la derecha implica una mayor
cesin de O2 a los tejidos por parte de la Hb.
El aumento de hidrogeniones desplaza la curva a la izquierda.
La acidosis respiratoria y las situaciones febriles desplazan la
curva a la derecha.

Uno de los siguientes NO constituye un mecanismo compensador de hipoxemia:


1)
2)
3)
4)
5)

9.

Los vrtices pulmonares estn mejor perfundidos que las


bases.
Las bases pulmonares estn mejor ventiladas que los vrtices.
La relacin ventilacin/perfusin es algo mayor en los vrtices
pulmonares.
De todas las causas de hipoxemia, los trastornos de ventilacin/perfusin intervienen en la mayora de los casos.
La sangre que procede de los vrtices tiene una PaO2 algo
mayor que la procedente de las bases.

Respecto a la curva de saturacin de la hemoglobina (Hb),


seale lo FALSO:
1)
2)

8.

La administracin de oxgeno corrige la hipoxemia producida por trastornos de ventilacin/perfusin.


En un mismo paciente pueden existir varios mecanismos
causantes.

De las siguientes afirmaciones indique la INCORRECTA:


1)

Capacidad vital (CV): volumen espirado mximo despus de


una inspiracin mxima.
Capacidad pulmonar total (CPT): volumen que contienen
los pulmones en la posicin de inspiracin mxima.
Volumen espiratorio de reserva (VER): volumen espirado
despus de una espiracin normal.
Volumen residual (VR): volumen inspirado y/o espirado en
cada movimiento respiratorio.
Capacidad funcional residual (CFR): volumen de gas que
contienen los pulmones tras una espiracin normal.

Con relacin al patrn ventilatorio obstructivo, todo es cierto,


EXCEPTO:
1)

4.

La zona de transicin est constituida por los bronquiolos


respiratorios.
El espacio muerto anatmico se denomina tambin zona de
conduccin.
El acino es la unidad anatmica situada distal al bronquiolo
terminal.
El lobulillo secundario est formado por sacos alveolares y
ductus alveolares.
La cantidad de moco bronquial producido por el rbol
bronquial es de 100 ml/da.

Seguimiento a distancia

Aumento del gasto del VD.


Aumento de la eritropoyesis.
Aumento de bicarbonato en orina.
Aumento de PCO2 en sangre arterial.
Aumento de la frecuencia respiratoria.

La hipoxemia que aparece en un embolismo pulmonar agudo


se relaciona con:
1)
2)
3)
4)
5)

Aumento del espacio muerto anatmico, en relacin con la


presencia de reas mal perfundidas.
Aumento de las reas de cortocircuito.
Es secundaria a una hiperventilacin perifrica.
Aumento del espacio muerto fisiolgico.
No se corrige con O2 al 100% ya que se debe a un shunt
intrapulmonar.

10. De las siguientes situaciones clnicas, indique en cul NO est


disminuida la DLCO:
1)
2)
3)
4)
5)

Fibrosis pulmonar.
Enfisema.
Bronquitis crnica.
Hipertensin pulmonar primaria.
Embolismos pulmonares recurrentes.

11. Respecto al control de la ventilacin, es FALSO:


1)

El principal regulador de la ventilacin es la PCO2, que


estimula a los quimiorreceptores centrales y tambin a los
perifricos.

CTO Medicina C/ Nez de Balboa, 115 28006 MADRID (Espaa) Tfno.: (91) 782 43 32 / Fax: (91) 782 43 27
E-mail: secretaria@ctomedicina.com; iberocto@ctomedicina.com WEB: www.ctomedicina.com; www.iberocto.com

NM Pg. 1

2)
3)
4)
5)

La respuesta de PCO2 disminuye con la edad y durante el


sueo.
El nivel de PO2 altera la sensibilidad al CO2. As, si hay
hipoxemia se es ms sensible al nivel de PCO2.
La ingesta de barbitricos baja la sensibilidad tanto al CO2
como al O2.
Las variaciones de PO2 y pH estimulan a los receptores
centrales y perifricos, con la misma intensidad.

12. Un paciente con insuficiencia respiratoria muestra una gasometra arterial con PO2 59 mmHg, PCO2 38 mmHg, pH 7,36 y
una diferencia alveolo-arterial de oxgeno de 35 mmHg. Se
administra O2 al 100% y no se corrigen los valores de los gases
arteriales. El cuadro subyacente MENOS probable es:
1)
2)
3)
4)
5)

Enfermedad vascular pulmonar.


Atelectasia.
Edema agudo de pulmn.
Shunt intracardaco.
Shunt intrapulmonar.

13. Un paciente de 50 aos acude a Urgencias por disnea de curso


progresivo. La gasometra arterial muestra los siguientes datos:
pH 7,47, PaO2 62 mmHg, PaCO2 30 mmHg, bicarbonato 26
mEq/l. Indique, de las siguientes posibilidades, la que mejor
explicara su situacin:
1)
2)
3)
4)
5)

Hipoventilacin alveolar primaria.


Poliomielitis bulbar.
Cifoescoliosis.
Enfermedad pulmonar obstructiva crnica.
Insuficiencia cardaca.

14. Un paciente con EPOC presenta en situacin de estabilidad una


gasometra arterial con los siguientes valores: pH 7,40, PaO2 58
mmHg, PaCO2 46 mmHg. Ante una descompensacin presumiblemente infecciosa acude a Urgencias y se obtiene una gasometra que demuestra: pH 7,36, PaO2 50 mmHg, PaCO2 60
mmHg, bicarbonato 35 mEq/l. Una de las siguientes respuestas
es correcta:
1)
2)
3)
4)
5)

La gasometra presenta una acidosis respiratoria aguda.


La gasometra demuestra una acidosis mixta.
En este caso, existe insuficiencia ventilatoria crnica con
hipoxemia.
El gradiente (A-a) de O2 probablemente es de 10 mmHg.
En este caso est indicada la ventilacin mecnica.

ENFERMEDAD PULMONAR OBSTRUCTIVA CRNICA.


15. Qu dato caracteriza, de entre los siguientes, a los pacientes
con enfermedad pulmonar obstructiva crnica (EPOC)?:
1)
2)
3)
4)
5)

Exposicin al tabaco durante al menos 10 aos, ms de 20


cig/da.
Presencia de disnea de moderados esfuerzos de un ao de
evolucin.
Hipersecrecin bronquial.
Limitacin al 50% del flujo areo reversible.
Obstruccin bronquial crnica no reversible con tratamiento.

Pg. 2 NM

M exico A rgentina
C hile U ruguay

NEUMOLOGA Y CIRUGA TORCICA

Preparacin Examen de Seleccin 05/06 1 Vuelta

16. Uno de los siguientes datos NO es tpico de los pacientes EPOC


tipo bronquitis crnica:
1)
2)
3)
4)
5)

Frecuentes reagudizaciones infecciosas.


Tendencia al desarrollo de poliglobulia.
Tendencia al desarrollo de cor pulmonale crnico.
Hiperinsuflacin pulmonar.
Aspecto abotargado, cianosis labial.

17. Uno de los siguientes datos NO aparece en la Rx de trax de un


paciente con enfisema:
1)
2)
3)
4)
5)

Aplanamiento diafragmtico.
Oligoemia vascular.
Aumento de la trama broncovascular.
Corazn en gota.
Imgenes qusticas de pared fina.

18. Un paciente de 65 aos, fumador de 25 cigarrillos diarios desde


su juventud, presenta tos y expectoracin crnicas, que a veces
ha sido ligeramente hemoptoica, sobre todo coincidiendo con
agudizaciones infecciosas. En los ltimos 6 meses nota disnea
progresiva, que es en la actualidad de medianos esfuerzos. De los
siguientes enunciados, indique el que NO esperara encontrar:
1)
2)
3)
4)
5)

Indice de Tiffeneau del 60%.


Hipocapnia.
Frecuentes descompensaciones infecciosas.
PaO2 55 mmHg.
Aumento del ndice cardiotorcico.

19. Respecto al tratamiento de la EPOC, es FALSO:


1)
2)
3)
4)
5)

Los antitusgenos estn contraindicados.


La hidratacin del esputo, no ha demostrado ningn beneficio.
La vacuna anti-influenza reduce la morbimortalidad de los
EPOC durante las epidemias gripales, y es eficaz en 3/4 partes
de los pacientes.
La rehabilitacin mejora la tolerancia al ejercicio.
Los anticolinrgicos son los broncodilatadores de eleccin,
presentando una accin ms duradera que los adrenrgicos
beta 2 de accin corta.

20. Indique lo que NO es cierto en relacin al enfisema por dficit


de alfa-1-antitripsina:
1)
2)
3)
4)
5)

La enfermedad se hereda de modo autosmico codominante.


Los homocigotos Z y S tienen concentraciones reducidas del
enzima, pero los ltimos suelen tener cantidad suficiente para
conseguir la proteccin.
El enfisema predomina en lbulos inferiores.
En el mismo paciente no se ha descrito la existencia de
enfisema y bronquiectasias.
Hay controversia sobre si el heterocigoto se asocia a alteracin de la funcin pulmonar.

21. Seale lo que NO es cierto en relacin a la oxigenoterapia


continua domiciliaria:
1)
2)
3)
4)
5)

Mejora la tolerancia al ejercicio.


Mejora la funcin neuropsicolgica.
Alivia la hipertensin pulmonar.
Una vez conseguida la estabilizacin clnica puede suspenderse.
El tratamiento incluye un mnimo de 16 horas al da.

CTO Medicina C/ Nez de Balboa, 115 28006 MADRID (Espaa) Tfno.: (91) 782 43 32 / Fax: (91) 782 43 27
E-mail: secretaria@ctomedicina.com; iberocto@ctomedicina.com WEB: www.ctomedicina.com; www.iberocto.com

Preguntas TEST

Seguimiento a distancia

NEUMOLOGA Y CIRUGA TORCICA

Preparacin Examen de Seleccin 05/06 1 Vuelta


22. Respecto al cor pulmonale y su tratamiento, es FALSO de entre
las siguientes opciones:
1)
2)
3)
4)
5)

La correccin de la hipoxemia es el mejor tratamiento del cor


pulmonale.
Los diurticos se emplean para tratamiento del edema, aunque
tienen poco efecto sobre el problema primario.
Si se instaura tratamiento diurtico es necesario un control
riguroso de iones sricos, ya que la aparicin de alcalosis
metablica disminuira el estmulo del centro respiratorio.
Los vasodilatadores muestran una mejora de la relacin V/Q.
La digoxina slo est indicada si existe fracaso del ventrculo
izquierdo.

23. Paciente varn de 60 aos, fumador desde hace 30 aos,


diagnosticado de EPOC tipo enfisema, acude a urgencias
porque, tras un episodio de infeccin de va area superior,
presenta empeoramiento de su disnea habitual, agitacin e
insomnio. Se le realiza gasometra arterial con PO2 50 mmHg,
PCO2 55 mmHg, pH 7,25 y bicarbonato de 31 mEq/l. Ante la
situacin de insuficiencia respiratoria se le administra oxgeno,
y a las pocas horas comienza con cuadro de estupor y coma.
En la gasometra arterial con O2 se objetiva pH 7,09, PO2 70
mmHg y PCO2 75 mmHg. El paso siguiente es:
1)
2)
3)
4)
5)

Intubacin orotraqueal y ventilacin mecnica.


Retirar el oxgeno, porque es la causa del aumento de CO2.
Aumento del oxgeno, pues el estupor es secundario a la
hipoxia cerebral.
Iniciar tratamiento antibitico.
Teofilinas para estimular el centro respiratorio.

24. Un paciente con EPOC presenta disnea de medianos esfuerzos.


La espirometra muestra un VEF1 del 60%. De los siguientes
tratamientos, indique el ms adecuado:
1)
2)
3)
4)
5)

Bromuro de ipratropio y betaadrenrgicos.


Teofilina.
Corticoides inhalados y bromuro de ipratropio.
Oxigenoterapia domiciliaria y betaadrenrgicos.
Betaadrenrgicos orales.

Preguntas TEST

ASMA.

2)
3)
4)
5)

El asma extrnseco est mediado generalmente por una


reaccin inmunolgica semirretardada.
El asma intrnseco presenta en la mayora de los casos una
elevacin importante de la IgE.
El asma extrnseco no suele tener en la familia miembros con
alguna enfermedad alrgica.
En el asma intrnseco podemos encontrar algunos pacientes
que tengan intolerancia a la aspirina.
La eosinofilia es mayor en el asma extrnseco.

26. Respecto al diagnstico del asma bronquial, es FALSO:


1)

3)
4)

5)

Ante una historia clnica compatible y una espirometra


obstructiva, hemos de realizar un test broncodilatador para
demostrar la reversibilidad de la obstruccin bronquial.
M exico A rgentina
C hile U ruguay

Si la espirometra es normal, podemos descartar el diagnstico de asma.


En un paciente con tos crnica y espirometra normal hemos
de realizar un test de metacolina.
En un paciente con tos y opresin torcica tras el ejercicio,
en el que se haya descartado una cardiopata como causa de
su opresin, hemos de realizar un test de metacolina o un test
de ejercicio.
El test de metacolina y el test de histamina son test de
provocacin que se realizan ante la sospecha clnica de asma
con espirometra basal normal.

27. Respecto al diagnstico diferencial del asma, es FALSO entre las


siguientes opciones:
1)
2)
3)
4)
5)

En el asma de predominio nocturno evidente, es necesario


descartar episodios de aspiraciones digestivas.
En los lactantes es de enorme importancia plantearse como
diagnstico diferencial la aspiracin de cuerpo extrao.
En personas ansiosas hay que descartar que se trate de una
disfuncin gltica. Estos casos suelen presentar una mala
respuesta al tratamiento convencional.
Conviene examinar si existen datos cutneos, neurolgicos
o de otros rganos, que nos hiciesen pensar que se trata de
una vasculitis.
Si no hay antecedentes familiares de asma o eosinofilia, es
raro que sea realmente un caso de asma por lo que hay que
insistir en el diagnstico diferencial.

28. De los siguientes hallazgos en el paciente con crisis de asma,


seale el que se relaciona con obstruccin grave:
1)
2)
3)
4)
5)

Sibilancias difusas.
Hiperinsuflacin acentuada.
Tos con expectoracin.
Taquicardia.
Disnea.

29. Indique lo FALSO en relacin a una crisis asmtica:


1)
2)
3)
4)

25. A propsito de las diferencias entre el asma intrnseco y extrnseco, seale la afirmacin correcta:
1)

2)

Seguimiento a distancia

5)

Uno de los indicadores de gravedad de la obstruccin es el


pulso paradjico.
El pico de flujo es un importante indicador de la gravedad.
En situaciones extremas las sibilancias disminuyen de forma
considerable.
La aparicin de neumomediastino es una complicacin muy
frecuente.
Pueden producirse atelectasias.

30. De los siguientes frmacos, indique cules son de eleccin en


el tratamiento de los episodios agudos de asma:
1)
2)
3)
4)
5)

Glucocorticoides inhalados.
Metilxantinas.
Simpaticomimticos subcutneos.
Simpaticomimticos inhalados.
Glucocorticoides sistmicos.

31. En la evolucin de un paciente con crisis de asma se decide


trasladarle a una UVI. Seale de los siguientes el parmetro que
ha hecho con mayor probabilidad que esta actuacin sea la
recomendada:
1)
2)

PEF 60%
Taquipnea.

CTO Medicina C/ Nez de Balboa, 115 28006 MADRID (Espaa) Tfno.: (91) 782 43 32 / Fax: (91) 782 43 27
E-mail: secretaria@ctomedicina.com; iberocto@ctomedicina.com WEB: www.ctomedicina.com; www.iberocto.com

NM Pg. 3

3)
4)
5)

Hiperinsuflacin torcica.
PaO2 61 mmHg.
PaCO2 46 mmHg.

32. Un paciente presenta clnica de disnea con sibilancias con una


frecuencia aproximada de 3 veces/semana. De los siguientes
tratamientos, cul sera el ms apropiado?:
1)
2)
3)
4)
5)

Corticoides sistmicos de base y betaadrenrgicos durante los


episodios de clnica.
Cromoglicato de base y betaadrenrgicos durante los episodios de clnica.
Corticoides inhalados de base y betaadrenrgicos cuando
tenga clnica.
Betaadrenrgicos de modo continuo.
Nedocromil sdico y corticoides sistmicos.

33. Paciente con diagnstico de asma desde hace dos aos y que
realiza tratamiento de forma estable con corticoides inhalados
y betaadrenrgicos inhalados de corta duracin a demanda.
Acude a Urgencias porque en los ltimos dos das presenta
aumento de su disnea, y en la ltimas 24 horas se ha administrado 12 pulsaciones del betaadrenrgico. Su pico de flujo (PEF)
nada ms llegar es 90 l/min (<50%). Esto indicara:
1)
2)
3)
4)
5)

Que la crisis es moderada (PEF <50% del terico), y deben


utilizarse nicamente 4 pulsaciones del betaadrenrgico
inhalado.
Que en este caso existe indicacin de ventilacin mecnica.
Que la crisis es severa, y deben utilizarse betaadrenrgicos
inhalados de accin corta y corticoides sistmicos.
Que no precisa oxigenoterapia.
Que no precisar aminofilina intravenosa.

34. Un paciente con historia de asma de 5 aos de evolucin tiene


sntomas continuos con limitacin de su actividad fsica habitual. En perodos de aparente estabilidad su PEF y/o VEF1 est
entre el 60-80%. En situacin de estabilidad, el tratamiento ms
adecuado sera:
1)
2)
3)
4)
5)

Bromuro de ipratropio pautado y betaadrenrgicos de accin corta a demanda.


Prednisona oral en dosis de 10 mg/da y betaadrenrgicos de
accin corta a demanda.
Corticoides inhalados (budesonida o beclometasona) a dosis
>800-1.000 microg/da y betaadrenrgicos inhalados de
accin larga a demanda.
Corticoides inhalados en la dosis del punto anterior, betaadrenrgicos inhalados de accin larga y betaadrenrgicos
inhalados de accin corta a demanda.
Cromoglicato disdico, teofilinas de accin retardada y
betaadrenrgicos inhalados de accin corta empleados a
demanda.

BRONQUIECTASIAS.

En Espaa, una causa frecuente es la post-tuberculosis.


La clnica ms frecuente es tos y expectoracin purulenta
aunque en ocasiones los pacientes se encuentran asintomticos.

Pg. 4 NM

M exico A rgentina
C hile U ruguay

3)
4)
5)

Las inmunodeficiencias con dficit de produccin de anticuerpos suelen asociar sinusitis a la presencia de bronquiectasias.
El dficit de IgA es la inmunodeficiencia que ms comnmente presenta bronquiectasias.
Las bronquiectasias post-tuberculosis son causantes de hasta
el 40% de los casos de hemoptisis masivas.

36. Una de las siguientes enfermedades que se asocian con bronquiectasias tiene tambin asma mediado por una reaccin
inmunolgica tipo I. Indique cul:
1)
2)
3)
4)
5)

Sndrome de Williams-Campbell.
Sndrome de Mounier-Kuhn (traqueobroncomegalia).
Aspergilosis broncopulmonar alrgica.
Sndrome de Kartagener.
Panhipogammaglobulinemia.

37. De los siguientes grmenes, indique cul coloniza a ms del 90%


de los pacientes con fibrosis qustica (FQ):
1)
2)
3)
4)
5)

Staphylococcus aureus.
Haemophilus influenzae.
Pseudomonas aeruginosa variedad mucoide.
Mycoplasma.
Legionella.

38. Respecto a la fibrosis qustica, es FALSO:


1)
2)
3)
4)
5)

Es una enfermedad de herencia autosmica recesiva.


Es la enfermedad letal ms frecuente en la raza blanca.
El 90% debuta al nacimiento con leo meconial.
El cromosoma afecto es el 7 en su brazo largo.
La causa ms frecuente de muerte son las complicaciones
pulmonares.

39. Ante las repetidas sinusitis, otitis y bronquitis que padece un


paciente de 16 aos, su mdico de familia le solicita un TC
torcico que demuestra la presencia de espacios areos qusticos en toda la va area, junto con ubicacin cardaca en
hemitrax derecho. Por todo ello, usted piensa en:
1)
2)
3)
4)
5)

Fibrosis qustica.
Sndrome de Young.
Sndrome de Kartagener.
Dficit de alfa-1-antitripsina.
Aspergilosis broncopulmonar alrgica.

BRONQUIOLITIS.
40. Respecto a la bronquiolitis, es FALSO:
1)
2)

35. Es FALSO, respecto a las bronquiectasias:


1)
2)

NEUMOLOGA Y CIRUGA TORCICA

Preparacin Examen de Seleccin 05/06 1 Vuelta

3)
4)
5)

La bronquiolitis simple presenta un infiltrado inflamatorio


en la pared bronquiolar, mientras que la obliterante asocia,
adems, mltiples tapones fibrosos que "obliteran" la luz.
La bronquiolitis constrictiva muestra fibrosis en la muscular
con epitelio intacto.
Si en la bronquiolitis obliterante se asocia fibrosis en los
alveolos cercanos, el proceso se denomina B.O.N.O. (bronquiolitis obliterante con neumona organizada).
La bronquiolitis infecciosa suele deberse al VRS.
La bronquiolitis infecciosa suele afectar a adultos y ser
irreversible.

CTO Medicina C/ Nez de Balboa, 115 28006 MADRID (Espaa) Tfno.: (91) 782 43 32 / Fax: (91) 782 43 27
E-mail: secretaria@ctomedicina.com; iberocto@ctomedicina.com WEB: www.ctomedicina.com; www.iberocto.com

Preguntas TEST

Seguimiento a distancia

NEUMOLOGA Y CIRUGA TORCICA

Preparacin Examen de Seleccin 05/06 1 Vuelta


41. Respecto a las bronquiolitis, una de las siguientes afirmaciones
NO es correcta:
1)
2)
3)
4)
5)

La bronquiolitis obliterante (B.O.) idioptica se sospecha


ante una obstruccin no reversible rpidamente progresiva
con ms de un 25% de neutrfilos en el LBA y sin causa clara.
La B.O.N.O. idioptica suele tener una clnica subaguda
consistente en malestar general, fiebre, tos, expectoracin y
prdida de peso.
La B.O. idioptica generalmente presenta Rx normal o con
discreta hiperinsuflacin, siendo rara la existencia de infiltrados.
La B.ON.O. idioptica suele presentar una Rx tpica con
infiltrados perifricos bilaterales.
La B.O.N.O. idioptica suele manifestarse con crepitantes y
acropaquias.

3)
4)
5)

42. Respecto al tumor carcinoide, es FALSO:


1)
2)
3)
4)
5)

Comprenden el mayor porcentaje en el grupo de adenomas


bronquiales.
Pueden comportarse como tumores malignos metastatizando por va hematgena y ganglionar.
Suelen ser tumores centrales que presentan sintomatologa
durante meses o aos.
El diagnstico de confirmacin se realiza con puncin aspiracin con aguja fina.
Su tratamiento es la reseccin quirrgica.

43. Es cierto, respecto del tumor de Pancoast:


1)
2)
3)

Preguntas TEST

4)
5)

El tipo histolgico que se asocia con ms frecuencia es el


carcinoma de clulas pequeas.
No est indicada la mediastinoscopia como procedimiento
de estadificacin ganglionar.
La radioterapia preoperatoria est indicada dentro de la
planificacin teraputica.
Aparece sndrome de Horner en todos los pacientes.
Es tpico el dolor escapular y en regin radial de miembros
superiores.

44. En relacin a las manifestaciones clnicas locales y a distancia


del carcinoma pulmonar son ciertos todos los enunciados,
EXCEPTO:
1)
2)
3)
4)
5)

El carcinoma escamoso y el oat cell, al ser preferentemente


centrales, suelen manifestarse con tos y expectoracin hemoptoica.
Cuando el tumor central alcanza un tamao capaz de
obstruir un bronquio lobar, puede producirse neumonitis
obstructiva.
El adenocarcinoma pulmonar, al ser perifrico, a veces es
descubierto de modo accidental, antes de producir sntomas.
El pulmn contralateral es metastatizado con mucha frecuencia.
Cerebro, hueso, hgado y glndulas suprarrenales son rganos metastatizados con frecuencia.

45. Indique la asociacin INCORRECTA:


1)
2)

Carcinoma microctico - hiponatremia.


Adenocarcinoma - tumor que asienta sobre zonas cicatrizales.
M exico A rgentina
C hile U ruguay

Carcinoma epidermoide - tumor que puede cavitarse.


Carcinoma epidermoide - hipercalcemia por secrecin ectpica de PTH.
Carcinoma microctico - variedad histolgica de carcinoma de
pulmn que ms frecuentemente produce derrame pleural.

46. En el estudio de extensin del cncer de pulmn, una de las


siguientes exploraciones NO est indicada:
1)
2)
3)
4)

TUMORES PULMONARES.

Seguimiento a distancia

5)

Gammagrafa sea si hay sntomas o aumento de la fosfatasa


alcalina.
Toracoscopia en pacientes con derrame pleural con citologa negativa.
Mediastinoscopia si hay parlisis completa del nervio recurrente larngeo.
TAC abdominal para valorar hgado y las suprarrenales si hay
signos de afectacin.
TAC craneal en el adenocarcinoma.

47. Uno de los siguientes NO es criterio de irresecabilidad en el


cncer de pulmn:
1)
2)
3)
4)
5)

Sndrome de vena cava superior.


Metstasis heptica nica.
Adenopatas en mediastino contralateral.
Invasin de cuerpos vertebrales.
Carcinoma microctico en estadio I.

48. NO se considera criterio de inoperabilidad en el cncer de


pulmn:
1)
2)
3)
4)
5)

VEMS preoperatorio <1000 cc.


IAM en los tres meses previos a la ciruga.
PCO2 > 45 mmHg.
CV > 45%.
VEMS postoperatorio predicho 670 ml.

49. Enfermo de 48 aos con carcinoma epidermoide de 4 cm de


dimetro en LID. En la FBC se observa lesin endobronquial en
el bronquio del LID situada a 4 cm de la carina principal y en el
TAC se observan adenopatas hiliares ipsilaterales mayores de
1cm. Cul es el estadio TNM del paciente?:
1)
2)
3)
4)
5)

IIa.
IIb.
IIIa.
IIIb.
IV.

50. Un paciente de 60 aos, ex fumador desde hace 10, presenta


en una Rx de trax realizada por una cada casual, un ndulo
pulmonar de 2 cm de dimetro con bordes muy ntidos. La
actitud ms correcta sera:
1)
2)
3)

Ya que es ex fumador, se dar de alta al paciente, no sin antes


indicarle que consulte en el caso de presentar sintomatologa
respiratoria.
Indicar una Rx de trax a los 3 meses para controlar el tamao,
y si hubiera algn cambio, realizar entonces broncoscopia.
Realizar un TC torcico, y si el ndulo tuviera calcificacin,
dar el alta al paciente, ya que cuando los ndulos tienen
calcio son benignos.

CTO Medicina C/ Nez de Balboa, 115 28006 MADRID (Espaa) Tfno.: (91) 782 43 32 / Fax: (91) 782 43 27
E-mail: secretaria@ctomedicina.com; iberocto@ctomedicina.com WEB: www.ctomedicina.com; www.iberocto.com

NM Pg. 5

4)
5)

Realizar broncoscopia con biopsia transbronquial.


Realizar de entrada toracotoma diagnstica, ya que a un
ndulo de ese tamao es prcticamente imposible acceder
con biopsia transbronquial.

51. Est usted valorando la operabilidad de un paciente con carcinoma anaplsico de clulas grandes y se encuentra con un
VEMS de 1.500 ml. Esto indicara que:
1)
2)
3)
4)
5)

Ya es inoperable el paciente.
Se debe mirar ahora el valor de la CPT.
Se debe mirar el valor de la DLCO.
Se debe realizar una gammagrafa de perfusin y calcular el
VEMS predicho postoperatorio.
No hay problema para la operabilidad.

52. Con respecto a la clasificacin TNM del cncer de pulmn,


seale cul de las siguientes opciones le parece correcta:
1)
2)
3)
4)
5)

Tumor de 2 cm de dimetro: T2.


Derrame pleural tumoral: M1.
Metstasis ganglionares subcarnicas: N1.
Invasin del nervio frnico: T4.
Parlisis recurrencial: T4.

53. Paciente diagnosticado de carcinoma broncognico tipo oat


cell que presenta hinchazn de cara, cuello y circulacin
colateral desde hace varias semanas, con adenopatas subcarnicas tumorales y sin evidencia clnica de metstasis a
distancia. Seale el estadio correcto:
1)
2)
3)
4)
5)

Ia ( T1N0M0 ).
IIIb ( T4N3M0 ).
IIIb ( T4N2M0 ).
IIb ( T2N1M0 ).
IV ( T4N0M1).

Gammagrafa sea.
ECO-TC abdominal.
TC cerebral.
RM torcica.
Broncoscopia con cepillado bronquial y citologa del aspirado.

55. Seale cul es el tratamiento de eleccin de un paciente con


carcinoma epidermoide en LID, de 7 cm de dimetro mayor,
con invasin de diafragma derecho y sin afectacin de ganglios
linfticos locorregionales ni evidencia clnica de metstasis a
distancia:
1)
2)
3)

RT torcica, por tratarse de un caso irresecable.


QT, pues se trata de una enfermedad T4.
Ciruga con reseccin pulmonar ampliada a diafragma y
linfadenectoma.

Pg. 6 NM

M exico A rgentina
C hile U ruguay

4)
5)

QT neoadyuvante y posterior intervencin quirrgica.


QT de induccin, seguida de RT.

ENFERMEDADES DE LA PLEURA, EL MEDIASTINO Y EL DIAFRAGMA.


56. En relacin a la etiologa del derrame pleural, son ciertos todos
los enunciados, EXCEPTO:
1)
2)
3)
4)
5)

La causa ms frecuente de derrame pleural es la insuficiencia


cardaca.
El derrame paraneumnico es la causa ms frecuente de
exudado.
El derrame pleural producido por el mesotelioma est en
clara relacin con exposicin al asbesto.
El derrame tuberculoso es una causa frecuente de derrame
pleural, sobre todo en mayores de 65 aos.
El embolismo pulmonar puede producir derrame pleural.

57. Indique el signo radiolgico ms frecuente en Rx de trax en


bipedestacin en un paciente con derrame pleural:
1)
2)
3)
4)
5)

Aumento de distancia entre cmara gstrica y margen pulmonar izquierdo inferior.


Menisco en borde lateral de uno de los hemitrax.
Ensanchamiento cisural.
Opacificacin de un hemitrax.
Borramiento de ngulo costofrnico.

58. El derrame pleural en relacin con la artritis reumatoide presenta todas las siguientes caractersticas, EXCEPTO:
1)
2)
3)

54. Paciente de 61 aos de edad, diagnosticado de carcinoma


epidermoide en LSI. En la broncoscopia se identific que la
tumoracin se originaba en un bronquio subsegmentario del
culmen. Rx y TC: masa de 2 cm de dimetro en LSI, sin
evidenciarse adenopatas mediastnicas de tamao significativo. Analtica: Ca 9,8, FA 105, Na 130, K 3,9, bilirrubina 1,4,
GOT 258, GPT 450. Seale cul de las siguientes exploraciones estara indicada en este caso:
1)
2)
3)
4)
5)

NEUMOLOGA Y CIRUGA TORCICA

Preparacin Examen de Seleccin 05/06 1 Vuelta

4)
5)

Predominio por el sexo masculino.


Es generalmente unilateral y de predominio derecho.
Puede aparecer antes que las manifestaciones articulares de
la enfermedad.
Es un exudado con niveles de glucosa <30 mg/dl en la
mayora de los casos.
El nivel del pH es generalmente >7,35.

59. En relacin al derrame pleural del lupus eritematoso sistmico,


todas las caractersticas son ciertas, EXCEPTO:
1)
2)
3)
4)
5)

En la mitad de los casos el derrame es bilateral.


Cuando es unilateral, predomina en el lado izquierdo.
La combinacin de derrame pleural bilateral y aumento del
tamao de la silueta cardaca sugiere el diagnstico, sobre
todo en mujeres jvenes.
El pH en lquido pleural es tpicamente <7,2.
La glucosa en lquido pleural es aproximadamente igual al
valor en sangre.

60. Un paciente con derrame pleural unilateral presenta en el estudio


del lquido de toracocentesis un hematocrito del 2%. De los
siguientes diagnsticos, indique el ms probable:
1)
2)
3)
4)
5)

Derrame pleural tuberculoso.


Derrame pleural paraneumnico.
Derrame pleural por embolismo pulmonar.
Insuficiencia cardaca.
Linfangioleiomiomatosis.

CTO Medicina C/ Nez de Balboa, 115 28006 MADRID (Espaa) Tfno.: (91) 782 43 32 / Fax: (91) 782 43 27
E-mail: secretaria@ctomedicina.com; iberocto@ctomedicina.com WEB: www.ctomedicina.com; www.iberocto.com

Preguntas TEST

Seguimiento a distancia

NEUMOLOGA Y CIRUGA TORCICA

Preparacin Examen de Seleccin 05/06 1 Vuelta


61. Un paciente presenta un derrame pleural de las siguientes
caractersticas protenas LP/P 0,65, glucosa 60, pH 7,4, linfocitos 60% y ADA 15 U/L. NO es probable:
1)
2)
3)
4)
5)

Derrame pleural maligno.


TBC Pleural.
Lupus eritematoso sistmico.
Artritis reumatoide.
Empiema.

62. Seale lo FALSO en relacin al mesotelioma maligno:


1)
2)
3)
4)
5)

Es el tumor pleural ms frecuente.


En la mayora de los casos se relaciona con exposicin previa
al asbesto.
Uno de los sntomas es el dolor torcico.
En muchos casos, el diagnstico no se confirma hasta la
autopsia.
Con frecuencia hay derrame pleural hemorrgico.

63. Todos los enunciados siguientes referidos al neumotrax son


correctos, EXCEPTO:
1)
2)
3)
4)
5)

El neumotrax espontneo primario ocurre sin traumatismo


previo.
El neumotrax espontneo secundario ocurre con traumatismo previo.
El neumotrax espontneo primario suele ocurrir por ruptura de bullas apicales.
El anterior neumotrax puede ocurrir en fumadores.
La causa ms frecuente de neumotrax espontneo secundario es la EPOC.

64. Con relacin a la parlisis diafragmtica, seale lo FALSO:


1)
2)
3)
4)

Preguntas TEST

5)

La parlisis unilateral puede deberse a infiltracin tumoral


por carcinoma broncognico.
La parlisis unilateral suele tener intensa disnea.
El diagnstico de lo anterior se confirma mediante fluoroscopia o radioscopia dinmica.
En la parlisis bilateral, la capacidad vital est marcadamente
reducida.
En la parlisis bilateral, el tratamiento de eleccin es la
respiracin asistida.

65. Con relacin a las hernias diafragmticas, seale lo FALSO:


1)
2)
3)
4)
5)

Las del agujero de Bochdalek se localizan en la parte posterolateral de trax.


Generalmente aparecen en el lado izquierdo.
Son ms frecuentes en pacientes obesos las de Bochdalek.
La hernia de Morgagni aparece con ms frecuencia en la
parte anterior del trax.
Si dan sntomas, se deben intervenir.

66. Un paciente se realiza una Rx de trax para incorporarse al


servicio militar, y se aprecia una masa en el mediastino anterior
con calcificaciones. El paciente est asintomtico. El diagnstico ms probable ser:
1)
2)

Ganglioneuroma.
Linfoma.
M exico A rgentina
C hile U ruguay

3)
4)
5)

Seguimiento a distancia

Teratoma.
Neurofibroma.
Paraganglioma.

67. Cul de las siguientes NO es indicacin de intervencin quirrgica en un neumotrax?:


1)
2)
3)
4)
5)

Neumotrax espontneo bilateral simultneo.


Primer episodio de neumotrax con fuga area durante ms
de 7 das.
Neumotrax espontneo contralateral a uno previo.
Segundo episodio de neumotrax espontneo ipsilateral.
Neumotrax yatrgeno.

68. Mujer de 52 aos de edad, con antecedente de mastectoma


derecha por adenocarcinoma de mama. Acude a Urgencias
por presentar disnea progresiva hasta hacerse de reposo. La
Rx de trax muestra la presencia de derrame pleural izquierdo
en cantidad moderada. Se realiza una toracocentesis diagnstica, obteniendo un exudado mononuclear con citologa positiva para adenocarcinoma. Seale la respuesta correcta:
1)
2)
3)
4)
5)

El derrame pleural maligno es una contraindicacin de


drenaje pleural con tubo, dado el mal pronstico que tienen
estos pacientes.
Si no existe atelectasia asociada, debera colocarse tubo de
trax para realizar pleurodesis.
Abstencin teraputica, pues la disnea suele desaparecer con
el tiempo.
Se realizar biopsia pleural percutnea, pues la citologa
positiva no es diagnstica de derrame maligno.
Debe realizarse toracotoma con extirpacin de las metstasis
pleurales.

69. Un paciente de 50 aos, fumador de 30 cigarrillos diarios desde


los 20 aos, presenta tos desde hace 2 meses y sintomatologa
constitucional. En el ltimo mes observa disnea que se ha hecho
progresiva, siendo en el momento de la consulta de medianos
esfuerzos. Se realiza una Rx de trax, donde se aprecia un
derrame pleural izquierdo. La espirometra muestra una CVF
del 60%. Se realiza una toracocentesis, donde se aprecia un
lquido serohemtico, con una relacin de protenas lquido
pleural/suero 0,7 y una glucosa 70 mg/dl. Se realiza una biopsia
pleural cerrada donde no se objetivan granulomas, y la citologa
es negativa para proceso tumoral. La actitud ms correcta en
este caso sera:
1)
2)
3)
4)
5)

Iniciar tratamiento con un antibitico de amplio espectro, ya que


se ha descartado de modo razonable un proceso neoplsico.
Ya que la causa ms frecuente de derrame pleural es la
insuficiencia cardaca, deberamos iniciar tratamiento con
un diurtico y repetir la Rx de trax a los 7 das.
Ya que el lquido es serohemtico, esto hace que la causa ms
probable sea un tromboembolismo pulmonar.
No se ha descartado un proceso tumoral, por lo tanto el
siguiente proceder debera ser la toracoscopia y biopsia con
control visual.
Debera ponerse de inmediato un tubo para drenar la
cavidad pleural.

70. Ante el diagnstico de carcinoma epidermoide de esfago, se


realiza una reseccin del tercio superior del esfago a un varn

CTO Medicina C/ Nez de Balboa, 115 28006 MADRID (Espaa) Tfno.: (91) 782 43 32 / Fax: (91) 782 43 27
E-mail: secretaria@ctomedicina.com; iberocto@ctomedicina.com WEB: www.ctomedicina.com; www.iberocto.com

NM Pg. 7

de 62 aos, transcurriendo el postoperatorio sin incidencias,


y siendo dado de alta a los 6 das de la intervencin. Una semana
despus, el paciente acude al hospital por disnea de moderados
esfuerzos. Se realiza una Rx trax, objetivndose un derrame
pleural izquierdo. Seale la cierta:
1)
2)
3)
4)
5)

NEUMOLOGA Y CIRUGA TORCICA

Preparacin Examen de Seleccin 05/06 1 Vuelta

Probablemente se trata de un hemotrax, dado el antecedente quirrgico reciente.


Debe tratarse de un derrame tumoral metastsico, ya que los
tumores pleurales suelen ser de origen metastsico.
Se debe sospechar un quilotrax. Un nivel de triglicridos
>110 mg/dl apoyar este diagnstico.
Se trata de un pseudoquilotrax de larga evolucin.
El tratamiento es quirrgico de entrada.

3)
4)
5)

74. Con respecto al TEP, una de las siguientes afirmaciones NO es


correcta. Cul?:
1)
2)
3)
4)

EMBOLISMO PULMONAR E HIPERTENSIN PULMONAR


PRIMARIA.

5)

71. Es FALSO sobre la hipertensin pulmonar primaria (HPP):


1)
2)

3)

4)
5)

Para su diagnstico se requiere una presin media de la


arteria pulmonar mayor de 25 mmHg en reposo y mayor de
30 mmHg con el esfuerzo.
Para diagnosticar una hipertensin pulmonar como primaria, es preciso haber descartado patologa cardaca o pulmonar que pueda producir una forma secundaria, as como
enfermedad del tejido conectivo.
La infeccin por VIH, la inhalacin de cocana y la ingesta de
frmacos supresores del apetito, as como la hipertensin
portal, pueden producir una enfermedad vascular pulmonar
con caractersticas clnicas y patolgicas similares a la HPP.
Es una enfermedad rara que predomina en mujeres en la 2
y 4 dcada.
Una forma anatmica llamada venooclusiva predomina sin
embargo a partir de los 60 aos.

72. Indique lo FALSO en relacin al tratamiento de la HPP:


1)
2)

3)

4)
5)

La enfermedad es de mal pronstico, con una supervivencia


media sin tratamiento de 2,5 aos.
Tienen ms probabilidad de responder a los vasodilatadores a largo plazo por va oral los pacientes que respondieron
al test de vasorreactividad aguda con vasodilatadores de
accin corta y potente (prostaciclina i.v.) con una disminucin de la presin de la arteria pulmonar, un aumento del
gasto cardaco y pocos cambios en la presin sistmica.
Cuando el test de vasorreactividad es como el descrito
anteriormente, el uso de vasodilatadores como los bloqueantes del calcio producen una mejora sostenida en la
mayora de los pacientes.
Cuando los pacientes dejan de responder a los vasodilatadores orales, se podra utilizar la prostaciclina i.v. de modo
crnico como puente al trasplante.
El trasplante sera la opcin teraputica cuando los pacientes
tienen la enfermedad ms avanzada.

73. Seale lo FALSO en relacin al embolismo pulmonar:


1)
2)

La disnea de instauracin brusca es generalmente el sntoma


ms frecuente.
Cuando el embolismo se acompaa de una zona de infarto,
puede haber dolor pleurtico.

Pg. 8 NM

M exico A rgentina
C hile U ruguay

Ante un cuadro clnico compatible es preciso una historia


detallada para buscar factores de riesgo.
El derrame pleural unilateral es la manifestacin radiolgica
ms frecuente.
El sncope suele ser un sntoma de embolismo masivo.

La sospecha clnica alta es muy valiosa para tomar decisiones.


La radiografa de trax puede mostrar elevacin de un
hemidiafragma.
La pletismografa de impedancia en pacientes con sntomas
de trombosis venosa profunda tiene una sensibilidad del
90% aproximadamente cuando se evalan las pantorrillas.
La ultrasonografa es en la actualidad una de las tcnicas ms
utilizadas entre las no invasivas para evaluar las extremidades
inferiores.
En algunos casos es preciso realizar una arteriografa pulmonar
para asegurar o descartar el tromboembolismo pulmonar.

75. Con relacin al tratamiento anticoagulante, seale lo FALSO:


1)
2)
3)
4)
5)

La heparina sdica por va intravenosa es el anticoagulante


de eleccin.
Para conseguir el efecto deseado, el tiempo parcial de
tromboplastina activado (TTPA) se establece entre 1,5-2,5 el
tiempo control.
Las HBPM han demostrado ser tan eficaces y seguras en el
tratamiento del TEP como las HNF, por lo que cada vez son
ms utilizadas.
La principal complicacin del tratamiento con heparina es
la hemorragia, que aparece aproximadamente en un tercio
de los pacientes.
Son tambin efectos secundarios de la heparina la plaquetopenia y osteoporosis.

76. Paciente varn de 75 aos, con antecedentes de fumador y


bebedor habitual. Ha sido intervenido hace 5 das de una
tumoracin en el sistema nervioso central y presenta disnea
brusca. A la exploracin, el paciente est taquicrdico y taquipneico, y en la GAB presenta como nico dato relevante hipoxemia moderada. Se realiza una gammagrafa ventilacin-perfusin con hallazgo de alta probabilidad de TEP. La actitud ms
correcta es:
1)
2)
3)
4)
5)

Evaluar las EEII con tcnicas invasivas para valorar la etiologa


del TEP.
Iniciar anticoagulacin con heparina i.v. los primeros das y
posteriormente cambiar el tratamiento por ACO durante 6
meses.
Colocarle un filtro de cava.
Iniciar tratamiento con terapia tromboltica.
Realizar tratamiento con HBPM durante 3 meses.

77. En relacin a las formas clnicas de la neumonitis por hipersensibilidad, todo es cierto, EXCEPTO:
1)
2)

La forma aguda aparece a las 4-8 horas de la exposicin, lo cual


est en relacin con un mecanismo inmunolgico tipo III.
La clnica aguda se caracteriza por fiebre, escalofros, tos no
productiva, disnea, mialgias, malestar general.

CTO Medicina C/ Nez de Balboa, 115 28006 MADRID (Espaa) Tfno.: (91) 782 43 32 / Fax: (91) 782 43 27
E-mail: secretaria@ctomedicina.com; iberocto@ctomedicina.com WEB: www.ctomedicina.com; www.iberocto.com

Preguntas TEST

Seguimiento a distancia

NEUMOLOGA Y CIRUGA TORCICA

Preparacin Examen de Seleccin 05/06 1 Vuelta


3)
4)
5)

Los sntomas pueden durar hasta 3 das, tras cesar la exposicin, pero generalmente duran 1 da.
El diagnstico puede confundirse con una neumona viral o
bacteriana.
La forma aguda y subaguda se caracterizan por un patrn
intersticial nodular, a veces acompaado de zonas de condensacin del espacio areo, con un claro predominio en
lbulos superiores.

78. Una de las siguientes afirmaciones en relacin con las enfermedades producidas por polvos inorgnicos NO es verdadera.
Seale cul:
1)
2)
3)
4)
5)

Las placas pleurales, que son lesiones localizadas sobre todo


en la pleura parietal, son especficas de la exposicin a asbesto.
La atelectasia redonda es una complicacin rara de la
exposicin a asbesto.
Las formas de silicosis que ms predisponen a tuberculosis
son la aguda y la acelerada.
La enfermedad intersticial producida por exposicin a asbesto es de predominio en lbulos inferiores.
El derrame pleural benigno en relacin con la exposicin a
asbesto tiene un perodo de latencia menor que el mesotelioma.

79. Con respecto a la silicosis, es FALSO:


1)
2)
3)
4)
5)

La silicosis simple predomina en lbulos superiores, presentando lesiones micronodulares menores de 1 cm.
La silicosis complicada presenta conglomerados mayores de
1 cm en lbulos superiores.
La silicosis acelerada predomina en campos medios.
La silicosis aguda presenta un patrn alveolar en lbulos
inferiores.
Se han descrito adenopatas hiliares calcificadas en la
silicosis, pero no son patognomnicas de esta enfermedad.

80. Uno de los siguientes datos NO esperara encontrarlo en un


paciente en el que se sospecha una fibrosis pulmonar idioptica
(alveolitis fibrosante criptogentica):

Preguntas TEST

1)
2)
3)
4)
5)

Predominio de la enfermedad alrededor de los 50 aos.


Crepitantes teleinspiratorios en las bases pulmonares.
Patrn reticular en la radiografa de trax de predominio en
campos inferiores junto a la existencia de placas pleurales.
Disminucin de la CPT.
Disminucin de la difusin.

81. En la afectacin pulmonar en las enfermedades del colgeno


todo es cierto, EXCEPTO:
1)
2)
3)
4)
5)

Es ms frecuente en el lupus la enfermedad intersticial que la


pleural.
El lupus puede presentar hemorragia pulmonar.
La afectacin pulmonar por la artritis reumatoide es ms
frecuente en hombres.
El Sndrome de Sjgren puede comportarse como linfoma.
La esclerodermia es la que con ms frecuencia desarrolla
enfermedad intersticial.

82. Con respecto a las eosinofilias pulmonares, es FALSO:

M exico A rgentina
C hile U ruguay

1)
2)
3)
4)
5)

Seguimiento a distancia

La neumona eosinfila crnica presenta infiltrados migratorios.


La neumona eosinfila crnica puede asociar asma.
La toxicidad aguda por nitrofurantona presenta infiltrados
basales.
La eosinofilia pulmonar simple se trata en ocasiones con
mebendazol.
El sndrome hipereosinfilo afecta sobre todo a varones de
edad media.

83. La aspergilosis broncopulmonar se caracteriza por todo lo


siguiente, EXCEPTO:
1)
2)
3)
4)
5)

Al depositarse el hongo en el rbol bronquial, prolifera y


provoca la produccin de Ig G y E especficas, as como Ig E total.
Los anticuerpos especficos se unen a antgenos del hongo y se
producen inmunocomplejos, los cuales daan las paredes
bronquiales producindose bronquiectasias centrales.
En la patogenia se implica una reaccin inmunolgica tipo
I y tipo III.
El cultivo del Aspergillus fumigatus es un criterio principal de
enfermedad.
La eosinofilia sangunea (generalmente ms de 1.000 eosinfilos/mm3) es un criterio principal.

84. Con respecto a la sarcoidosis, cul de las siguientes afirmaciones es cierta?:


1)
2)
3)
4)
5)

El sndrome de Lfgren presenta tpicamente adenopatas


hiliares unilaterales y patrn intersticial nodular.
El sndrome de Heerfordt presenta entre sus caractersticas
uvetis posterior.
La sarcoidosis es algo ms frecuente en mujeres, y la mayor
incidencia ocurre entre los 20-40 aos.
Es tpica de pacientes fumadores.
El derrame pleural aparece aproximadamente en la mitad de
los casos.

85. Indique lo verdadero en relacin a la sarcoidosis:


1)
2)
3)
4)
5)

El eje hipotlamo-hipofisario es la parte del sistema endocrino que se afecta con ms frecuencia.
Los rganos reproductores, cuando se afectan, suelen producir esterilidad.
Con el embarazo, las manifestaciones de sarcoidosis suelen
empeorar.
Cuando hay afectacin de la glndula partida, suele ser
unilateral.
La afectacin de la partida no es caracterstica de la sarcoidosis.

86. Indique la relacin INCORRECTA en la sarcoidosis:


1)
2)
3)
4)
5)

Estadio I: adenopatas hiliares.


Estadio II: adenopatas hiliares y paratraqueales.
Estadio III: patrn intersticial reticulonodular.
Estadio IV: patrn en panal.
Sndrome de Lfgren: eritema nodoso ms adenopatas
hiliares bilaterales.

87. La histiocitosis X primaria pulmonar o granuloma eosinfilo


pulmonar se caracteriza por todo lo siguiente, EXCEPTO:

CTO Medicina C/ Nez de Balboa, 115 28006 MADRID (Espaa) Tfno.: (91) 782 43 32 / Fax: (91) 782 43 27
E-mail: secretaria@ctomedicina.com; iberocto@ctomedicina.com WEB: www.ctomedicina.com; www.iberocto.com

NM Pg. 9

1)
2)
3)
4)
5)

Es una enfermedad infrecuente que produce una enfermedad pulmonar intersticial y que puede acompaarse de
lesiones osteolticas solitarias.
Se afectan sobre todo varones fumadores con edad fundamentalmente entre 20-40 aos.
El pulmn presenta una infiltracin por clulas de Langerhans (clulas muy diferenciadas), que son histiocitos dendrticos que pertenecen a la serie monocito-macrfago.
La proliferacin de clulas de Langerhans es patognomnica
de esta enfermedad.
Las clulas de Langerhans se distinguen de otros monocitos
porque al microscopio electrnico tienen unos grnulos
intracitoplsmicos que se llaman de Birbeck.

88. La linfangioleiomiomatosis se caracteriza por todo lo siguiente,


EXCEPTO:
1)
2)
3)
4)
5)

Proliferacin de clulas musculares inmaduras en los pulmones y en los linfticos del trax y abdomen.
Formacin de quistes de pared fina en pulmones.
Adenopatas retroperitoneales.
Angiomiolipomas renales.
Derrame pleural hemtico.

89. Con relacin a la granulomatosis de Wegener, todos los siguientes enunciados son correctos, EXCEPTO:
1)
2)
3)
4)
5)

Es una vasculitis necrotizante granulomatosa que afecta fundamentalmente a vasos pequeos (arteriolas, vnulas y capilares),
aunque a veces puede afectar tambin a arterias y venas.
Los granulomas pueden ser intra y extravasculares.
Afecta generalmente a personas alrededor de los 50-60 aos
de edad.
En la biopsia renal es tpico encontrar tanto la vasculitis
caracterstica como los granulomas.
La biopsia de senos paranasales y nasofarngeos suele mostrar
granulomas pero no vasculitis.

90. En relacin al sndrome de Goodpasture, es FALSO que:


1)
2)
3)
4)
5)

El aumento de la DLCO refleja la ocupacin alveolar por la


hemorragia.
La biopsia renal es diagnstica.
Los corticoides mejoran la glomerulonefritis.
La causa principal de muerte en estos pacientes es la hemorragia pulmonar.
La biopsia renal tiene implicacin pronstica.

91. Seale, de todos los enunciados, el que NO es correcto en


relacin a las neumonas:

2)
3)

Las formas tpicas suelen cursar con tos, expectoracin, fiebre


alta y dolor pleurtico.
El aumento de las vibraciones vocales y el soplo tubrico en
la auscultacin pulmonar es tpico sobre todo de las neumonas virales.
Las neumonas atpicas suelen tener manifestaciones generales y poca focalidad pulmonar.

Pg. 10 NM

M exico A rgentina
C hile U ruguay

4)
5)
92

La radiografa de las formas tpicas suele cursar con patrn


alveolar y broncograma.
El derrame pleural acompaa con ms frecuencia a las
neumonas bacterianas.

Respecto a la clnica de las neumonas atpicas, es FALSO:


1)
2)
3)
4)
5)

La Legionella spp en ocasiones cursa con hiponatremia (que


puede ser causa de confusin mental), diarrea o hematuria.
La Chlamydia pneumoniae ocasiona broncoespasmo, con
sibilancias en la auscultacin pulmonar.
La Chlamydia psittaci (causante de la psitacosis) se relaciona
con la exposicin a pjaros y cursa con esplenomegalia.
La Coxiella burnetti (causante de la fiebre Q), al ser una
rickettsiosis, cursa con exantema.
En la fiebre Q, la manifestacin extrapulmonar ms frecuente
es la hepatitis.

93. Paciente de 47 aos de edad, no fumador, con antecedente de


contacto profesional con ganado, clnicamente asintomtico,
en el que en estudio radiolgico rutinario mediante Rx y TAC
torcico se detecta una lesin qustica de contenido lquido de
6 cm de dimetro en LID. En la analtica sangunea destaca una
leve eosinofilia y serologa positiva para hidatidosis. Cul de las
siguientes opciones le parece adecuada:
1)
2)
3)
4)
5)

Seguimiento radiolgico del paciente, ya que el quiste hidatdico pulmonar no suele presentar complicaciones durante
su evolucin.
Iniciar tratamiento con albendazol, ya que la hidatidosis
pulmonar rara vez requiere ciruga y con tratamiento mdico
suele controlarse.
Realizar broncoscopia y biopsia de la lesin para descartar
la posibilidad de carcinoma broncognico.
PAAF de la lesin, si no es accesible mediante broncoscopia.
Toracotoma y extirpacin.

94. Un paciente de 35 aos, sano previamente y fumador ocasional,


presenta despus de una gripe un cuadro de fiebre de 38C, tos,
expectoracin herrumbrosa y dolor pleurtico derecho. En la
analtica hay leucocitosis y la Rx de trax muestra una condensacin alveolar en el lbulo superior derecho. Se realiza una
gasometra arterial, que es normal. Indique la respuesta correcta:
1)
2)
3)

NEUMONAS.

1)

NEUMOLOGA Y CIRUGA TORCICA

Preparacin Examen de Seleccin 05/06 1 Vuelta

4)
5)

Como se trata de una neumona severa, hay que ingresar al


paciente y tratarle con penicilina i.v.
Como es una neumona probablemente por Legionella, le
ingresaramos y pautaramos tratamiento con eritromicina
intravenosa.
Como se trata de una neumona severa, ingresaramos al
paciente y le administraramos una cefalosporina de 3
generacin intravenosa.
Como lo ms probable es que la etiologa sea neumoccica,
el tratamiento sera con penicilina oral durante 7-10 das.
La sospecha de C. psittacci es alta, por lo que le trataramos
con tetraciclina.

95. Una de las siguientes situaciones en un paciente con neumona


NO rene criterios de ingreso hospitalario:
1)

Paciente de 26 aos, previamente sano, con 3.500 leucocitos/mm3.

CTO Medicina C/ Nez de Balboa, 115 28006 MADRID (Espaa) Tfno.: (91) 782 43 32 / Fax: (91) 782 43 27
E-mail: secretaria@ctomedicina.com; iberocto@ctomedicina.com WEB: www.ctomedicina.com; www.iberocto.com

Preguntas TEST

Seguimiento a distancia

NEUMOLOGA Y CIRUGA TORCICA

Preparacin Examen de Seleccin 05/06 1 Vuelta


2)
3)
4)
5)

Paciente de 45 aos, con diabetes mellitus tipo II bien


controlada.
Paciente de 23 aos, sin antecedentes de inters, con TA 70/
40 mmHg, 115 lpm y ms de 28 rpm.
Paciente de 62 aos, sin antecedentes, con 14.000 leucocitos/mm3, 16 rpm y PO2 64 mmHg.
Paciente de 31 aos, diagnosticado hace 3 das. Se inici
tratamiento antibitico, pero acude a Urgencias nuevamente
por persistencia de la clnica (fiebre y disnea de moderados
esfuerzos).

96. En los sndromes de hipoventilacin, NO es cierto que:


1)
2)
3)
4)
5)

Seguimiento a distancia

100. Con respecto al trasplante de pulmn, seale la opcin FALSA:


1)
2)
3)
4)
5)

La mayora de los trasplantados sufre al menos un episodio


de rechazo agudo en los primeros tres meses.
En el rechazo agudo, los pacientes pueden presentar leucocitosis, febrcula e infiltrados pulmonares.
El rechazo agudo es la primera causa de mortalidad.
En el rechazo agudo la sensibilidad y especificidad de la
biopsia transbronquial es alta (80-100% respectivamente).
La manifestacin principal del rechazo crnico es la bronquiolitis obliterante, que ocurre generalmente entre el 1-2
aos postrasplante.

Los pacientes con defecto a nivel neuromuscular y del


aparato ventilatorio son capaces de hiperventilar voluntariamente.
Los pacientes con defecto a nivel del sistema de control
metablico tienen mediciones normales de resistencia y
distensibilidad.
La presin inspiratoria y espiratoria mximas estn conservadas cuando hay una alteracin a nivel del sistema de control
metablico.
Cuando el defecto es a nivel neuromuscular, las presiones
que se pueden generar en la va area estn disminuidas.
Cuando hay un defecto a nivel del aparato ventilatorio, los
volmenes y flujos son tpicamente anormales y est deteriorada la respuesta ventilatoria a los estmulos qumicos.

97. Con relacin a las apneas, NO es cierto:


1)
2)
3)
4)
5)

En la apnea central cesa el impulso a los msculos respiratorios.


En gente sana puede haber episodios de apnea central.
Un ndice apnea-hipopnea de 10 o ms sugiere apnea
obstructiva del sueo.
En la apnea obstructiva no cesa el impulso respiratorio.
El mtodo definitivo que confirma la apnea obstructiva es la
oximetra nocturna.

Preguntas TEST

98. En el sndrome de dificultad respiratoria aguda, NO esperara


encontrar:
1)
2)
3)
4)
5)

Compliance pulmonar disminuida.


Presin capilar enclavamiento alta.
Shunt intrapulmonar.
Membranas hialinas intraalveolares.
Infiltrados alveolointersticiales.

MALFORMACIONES Y TRASPLANTE PULMONAR.


99. Es FALSO, con relacin al secuestro broncopulmonar:
1)
2)
3)
4)
5)

No presenta comunicacin con la va area.


Habitualmente su aporte sanguneo procede de una arteria
intercostal.
El secuestro extralobar tiene un drenaje venoso anmalo
hacia la circulacin sistmica.
El secuestro intralobar suele asentar en el ngulo costofrnico
posterior izquierdo.
El tratamiento es la extirpacin quirrgica, aun cuando se
trate de un hallazgo casual.

M exico A rgentina
C hile U ruguay

CTO Medicina C/ Nez de Balboa, 115 28006 MADRID (Espaa) Tfno.: (91) 782 43 32 / Fax: (91) 782 43 27
E-mail: secretaria@ctomedicina.com; iberocto@ctomedicina.com WEB: www.ctomedicina.com; www.iberocto.com

NM Pg. 11

NEUMOLOGA Y CIRUGA TORCICA

Preparacin Examen de Seleccin 05/06 1 Vuelta


Pregunta 1.- R: 4
La pregunta hace referencia a la divisin de la va area y sus unidades anatmicas. La va area se divide en tracto respiratorio superior,
cuya funcin fundamental es acondicionar el aire inspirado para que
llegue en perfectas condiciones a la zona de intercambio gaseoso. Este
ltimo se divide en:
Zona de conduccin: desde la trquea hasta los bronquiolos terminales. Como dice la respuesta 2, se denomina tambin espacio
muerto anatmico y contiene un volumen aproximado de 150ml.
Zona de transicin: formada por los bronquiolos respiratorios
(Resp. 1).
Zona respiratoria: constituida por los ductus y sacos alveolares.
Realizan la funcin fundamental del aparato respiratorio, el intercambio de gases.

Seguimiento a distancia

Pregunta 3.- R: 3
Los patrones ventilatorios anormales vienen definidos en funcin
de las alteraciones en los volmenes pulmonares estticos y dinmicos. Hay dos alteraciones ventilatorias fundamentales, las restrictivas y
las obstructivas.

Las unidades anatmicas son:


Acino: unidad anatmica distal al bronquiolo terminal, es decir
ductus y sacos alveolares.
Lobulillo primario: sacos alveolares dependientes de un ductus
alveolar (por ello es falsa la respuesta 4, ya que define al lobulillo
primario).
Lobulillo secundario: mnima porcin de parnquima pulmonar
independiente de los lobulillos vecinos y rodeada de tejido conjuntivo.

Comentarios TEST

La ltima respuesta hace referencia a la cantidad de moco bronquial formado por da en el aparato respiratorio. En la trquea y
bronquios es producido en su mayor parte por las glndulas submucosas y en los bronquiolos por las clulas de Clara.
Pregunta 2.- R: 4
En esta pregunta se repasan los parmetros que evalan la funcin
ventilatoria.
Hay dos tipos de volmenes, los estticos y los dinmicos.
La mayora de los volmenes y capacidades pulmonares (suma de
distintos volmenes) se determinan mediante una espirometra simple
en el caso de los estticos, y forzada en los dinmicos. Sin embargo,
tanto el volumen residual como las capacidades que dependen de l
(capacidad pulmonar total y capacidad funcional residual) requieren
otras tcnicas como la pletismografa o la dilucin con Helio.
El volumen corriente es el volumen inspirado o espirado que se
moviliza en cada movimiento respiratorio (aproximadamente 500ml).
Esto es lo que define la respuesta 4, haciendo referencia al volumen
residual, por lo que es la respuesta falsa. El volumen residual es el
volumen de gas que contienen los pulmones despus de una espiracin mxima. Es muy importante para que sea posible el intercambio
gaseoso en todas las fases de la ventilacin. La capacidad vital, la
capacidad pulmonar total, el volumen de reserva espiratorio y la capacidad funcional residual son volmenes estticos correctamente
definidos en esta pregunta.

Pregunta 3. Alteraciones ventilatorias.

El patrn ventilatorio obstructivo se caracteriza por una dificultad


en la espiracin, lo que se pone de manifiesto en las pruebas de
funcin respiratoria con una disminucin de los flujos espiratorios y
un aumento del volumen residual. El primer parmetro que se altera
es el FEF 25-75%, y a ste le sigue la disminucin del FEV1 y del ndice
de Tiffeneau, junto con el aumento progresivo del volumen residual.
La CPT puede mantenerse normal o aumentar a medida que aumenta el VR. Nunca disminuye, dato caracterstico de las enfermedades
restrictivas. La CV, sin embargo, permanece normal o disminuida, al
igual que en las enfermedades restrictivas, motivo por el que no es un
parmetro muy til en el diagnstico diferencial. La CVF es menor
que la CV puesto que encontramos un colapso precoz de la va area
durante la espiracin en estos enfermos.
La DLCO slo estara disminuida si hubiese una destruccin acompaante de la zona de intercambio gaseoso, como ocurre en el enfisema, pero no en un gran grupo de enfermos con patrn obstructivo,
como son los bronquticos crnicos.
Pregunta 4.- R: 5
La ventilacin es el volumen de aire que se moviliza en la respiracin por unidad de tiempo. De todo ese volumen de aire, hay una
parte que no interviene en el intercambio gaseoso. Por ello, cuando
hablamos de ventilacin, podemos hacer referencia a:
Ventilacin Total (VT): volumen total de aire movilizado por unidad de tiempo (minuto).
- VT = Volumen corriente (VC) x Frecuencia resp reposo.
- VT = 500 ml x 12-16 rpm (7L/m).
Ventilacin Alveolar (VA): volumen de aire que interviene en el
intercambio gaseoso por unidad de tiempo (minuto).
- VA = (volumen corriente - Volumen espacio muerto) x
frecuencia respiratoria en reposo.
- VA=(500 ml - 150 ml) x 12-16 rpm (5L/m).

Pregunta 2. Volmenes pulmonares estticos.


M exico A rgentina
C hile U ruguay

Ventilacin del Espacio Muerto (VEM): volumen de gas que no


interviene en el intercambio gaseoso (volumen del espacio muerto)
por unidad de tiempo.
- VEM =Volumen Espacio Muerto x F.R. reposo.
- VEM = 150 x 12-16 rpm (2L/m).

CTO Medicina C/ Nez de Balboa, 115 28006 MADRID (Espaa) Tfno.: (91) 782 43 32 / Fax: (91) 782 43 27
E-mail: secretaria@ctomedicina.com; iberocto@ctomedicina.com WEB: www.ctomedicina.com; www.iberocto.com

NM Pg. 1

Seguimiento a distancia

NEUMOLOGA Y CIRUGA TORCICA

Preparacin Examen de Seleccin 05/06 1 Vuelta

Pregunta 5.- R: 1
Se define hipoxemia como una PaO2 menor de 80mmHg. Hay
varios mecanismos causantes de hipoxemia:
Disminucin de la PO2 en el aire inspirado. Es la hipoxemia que
aparece en las grandes alturas. Segn la intensidad de la hipoxemia, el individuo aumentar su frecuencia respiratoria ms o menos, por lo que puede acompaarse de normocapnia o hipocapnia por hiperventilacin compensadora (respuesta 2).
Hipoventilacin. Cursa de modo caracterstico con un aumento
de la PaCO2 y mejora con la administracin de oxgeno. Generalmente el gradiente alvolo arterial de oxgeno es normal, salvo que
haya sobreaadido otro mecanismo causante de hipoxemia como
el shunt o las alteraciones ventilacin-perfusin.
Alteraciones de la relacin V/Q. Son las ms frecuentes (resp 1 es
incorrecta). La PaCO2 es variable, depende de la enfermedad subyacente, y el gradiente de O2 suele estar aumentado, ya que la
lesin est en el pulmn, y es entonces cuando se altera este parmetro. Es caracterstica la respuesta a la administracin de oxgeno
a alto flujo.
Efecto Shunt. Los alveolos estn perfundidos, pero no ventilados.
Como en el caso anterior, hay una alteracin del gradiente de
oxgeno, pero sin respuesta a la administracin de O2 , ya que al
estar lesionada la zona ventilatoria, ste no llega a la zona de intercambio gaseoso.
Alteraciones de la difusin. Produce hipoxemia en ejercicio. Si
aparece en reposo, hay sobreaadido otro mecanismo como las
alteraciones V/Q.

correcta de la tcnica denominada pulsioximetra, ya que mediante


su utilizacin obtenemos valores de saturacin de la hemoglobina
por el oxgeno. La curva de disociacin de la hemoglobina tiene
forma sigmoidea y nos interesa diferenciar en ella dos partes en las
que el comportamiento es completamente diferente. Una zona de
pendiente en la que pequeas variaciones en la PaO2 condicionan
grandes cambios en la saturacin de hemoglobina, y una zona de
meseta donde grandes cambios en la PaO2 casi no condicionan variaciones en la saturacin de la hemoglobina. En condiciones normales,
el punto donde cambia la curva de pendiente a meseta es lo que
corresponde con una PO2 de 60 mmHg y una saturacin aproximadamente del 90%. Este valor y todos los valores superiores aseguran
una oxigenacin adecuada.
Esta curva puede ser desplazada hacia la derecha o hacia la izquierda en funcin de distintos factores que modifican la afinidad de la
hemoglobina por el oxgeno. La disminucin del pH (o aumento del
hidrogeniones), el aumento de la PaCO2, el aumento del 2-3 DPG y el
aumento de la temperatura desplazan la curva a la derecha, con lo que
la afinidad de la Hb es menor y se libera el O2 a los tejidos con mayor
facilidad. Situaciones inversas desplazan la curva hacia la izquierda.
% SATURACIN
HEMOGLOBINA

Sangre
arterial

Sangre
venosa

80

60

AUMENTO HIDROGENIONES
(disminuye pH)
AUMENTO CO2
AUMENTO TEMPERATURA
AUMENTO DPG

40

20

20

40

50

60

Muestra A

80

100

120
PO2 (mmHg)

Muestra B

Pregunta 7. Curva de disociacin de la hemoglobina y factores modificadores.

Pregunta 5. Algoritmo diagnstico de la hipoxemia.

Pregunta 6.- R: 1
Para asegurar un correcto intercambio gaseoso debe existir una
adecuada relacin entre la ventilacin y la perfusin. La situacin
ideal es la concordancia completa entre la ventilacin y la perfusin,
con lo que el cociente V/Q tiende a 1. Sin embargo, en bipedestacin
y por efecto de la gravedad, se establece tanto un gradiente de ventilacin como un gradiente de perfusin. En ambos casos, las bases estn
mejor ventiladas y perfundidas que los vrtices, pero el gradiente creciente que se establece vrtice-base es mayor en caso de la perfusin
que en el de la ventilacin, motivo por el que la relacin V/Q es mayor
en los vrtices.
La mayor relacin V/Q de los vrtices conlleva que la sangre procedente de esa zona tenga una mayor PO2 y una menor PCO2, puesto
que el intercambio es casi perfecto.
Pregunta 7.- R: 4
En esta pregunta repasamos la curva de disociacin de la oxihemoglobina, que es indispensable conocer para hacer una interpretacin
Pg. 2 NM

M exico A rgentina
C hile U ruguay

Pregunta 8.- R: 4
La hipoxemia condiciona la activacin de mecanismos compensadores que son tanto ms eficaces cuanto ms lenta y progresiva es su
instauracin. Sus mecanismos compensadores son los siguientes:
Aumento del gasto cardaco.
Aumento de la eritropoyesis. Se estimula la secrecin de eritropoyetina y aparece poliglobulia.
Aumento de la ventilacin. El estmulo hipoxmico acta sobre los
quimiorreceptores articos y carotdeos condicionando un aumento de la frecuencia respiratoria que induce la aparicin de
alcalosis respiratoria por una disminucin de la PaCO2 secundaria
a la hiperventilacin (la respuesta 4 es incorrecta). Posteriormente
se intenta compensar la alcalosis respiratoria perdiendo bicarbonato a nivel renal.
Aumento del 2-3 DPG para desviar la curva de disociacin de la
hemoglobina a la derecha y liberar ms oxgeno a los tejidos.
Vasodilatacin tisular.
Pregunta 9.- R: 4
Para asegurar un correcto intercambio gaseoso se necesita una
adecuada relacin ventilacin perfusin. Cuando hablamos de trom-

CTO Medicina C/ Nez de Balboa, 115 28006 MADRID (Espaa) Tfno.: (91) 782 43 32 / Fax: (91) 782 43 27
E-mail: secretaria@ctomedicina.com; iberocto@ctomedicina.com WEB: www.ctomedicina.com; www.iberocto.com

Comentarios TEST

NEUMOLOGA Y CIRUGA TORCICA

Preparacin Examen de Seleccin 05/06 1 Vuelta


boembolismo pulmonar hablamos de una alteracin ventilacin-perfusin tipo espacio muerto. Lo que ocurre es que la unidad est correctamente ventilada y pobremente perfundida porque hay mbolos en el lecho vascular. Esto condiciona que la relacin V/Q tienda a
infinito a expensas de un aumento del espacio muerto alveolar (despreciable en condiciones de normalidad) que conlleva un aumento
del espacio muerto fisiolgico. La respuesta correcta es por tanto la 4.
La respuesta 1 es incorrecta, porque el espacio muerto anatmico
es el volumen de aire contenido en la va area que no participa en el
intercambio de gases, y por tanto no se modifica cuando se lesionan
las zonas de intercambio gaseoso.
La respuesta 2 hace referencia al otro tipo de alteracin de la
ventilacin-perfusin, el efecto shunt o cortocircuito, en el que la
unidad de intercambio est bien perfundida e incorrectamente ventilada, por lo que la relacin V/Q tiende a cero y la mezcla de sangre
que llega a la unidad es similar a la que sale al no realizarse el intercambio de gases. En este caso, la administracin de O2 no corrige la
hipoxemia mientras que en el efecto espacio muerto s.
Pregunta 10.- R: 3
La determinacin de la capacidad de difusin de monxido de
carbono (DLCO) hace una distincin del estado funcional de la membrana alveolocapilar. La DLCO slo aumenta en 2 situaciones:
Fases iniciales de la ICC por un aumento del volumen de sangre
que llegue al capilar de la zona de intercambio, uno de los factores
que condicionan la DLCO.
Hemorragia alveolar, pues la hemoglobina de los hemates vertidos
al alveolo tambin capta monxido de carbono.

Comentarios TEST

Generalmente la DLCO disminuye de forma tpica en todas las


situaciones en las que hay alteracin de alguno de los factores que
condicionan la DLCO, como la superficie de intercambio, la concentracin de hemoglobina en la sangre, la relacin V/Q y el grosor de la
membrana alveolocapilar. Esto ocurre tpicamente en el enfisema, las
enfermedades intersticiales, el TEP recurrente y la HT Pulmonar.
En el caso de la bronquitis crnica, no se producen alteraciones de
la DLCO hasta fases muy avanzadas de la enfermedad.
Pregunta 11.- R: 5
Hay 2 sistemas de control de la ventilacin: el control voluntario,
localizado en la corteza cerebral y responsable de estimular o inhibir
el impulso cerebral, y el control involuntario. En el control involuntario intervienen varios factores:
Quimiorreceptores (QR): hay 2 tipos: los centrales y los perifricos. Los QR centrales estn localizados fundamentalmente a nivel
bulbar y son sensibles a las variaciones de la PaCO2 , que es el
principal regulador de la ventilacin. Su aumento condiciona un
aumento de la frecuencia respiratoria (hiperventilacin). Las variaciones de PaO2 no actan como estmulo directo a este nivel.
Los quimiorreceptores perifricos estn localizados en el seno
carotdeo y cuerpo artico. Estos son sensibles a las variaciones
tanto de PaCO2 como de PaO2. La respuesta a las variaciones de
PaCO2 disminuye con la edad, durante el sueo y con la ingesta de
barbitricos. La hipoxemia, sin embargo, aumenta la sensibilidad a
las variaciones de la PaCO2.
Mecanorreceptores: protegen al pulmn de la sobredistensin
mediante el reflejo de Hering-Breuer. Cuando se activan inhiben la
inspiracin.
Centros respiratorios: generan el impulso ventilatorio y son regulados tanto por las aferencias que provienen desde los QR centrales como las que provienen de los QR perifricos.
En condiciones normales, el principal regulador de la ventilacin
es la PaCO2; pero en situaciones de hipercapnia crnica es la PaO2 el
principal estmulo respiratorio. Por este motivo, en los pacientes con
EPOC no se debe corregir bruscamente la hipoxemia.
Pregunta 12.- R: 1
Estamos ante el manejo habitual de un paciente en insuficiencia
respiratoria. En los casos clnicos es importante conocer exactamente
a qu hace referencia la gasometra. En este caso, lo que tenemos es
un paciente con una PaO2 < 60, es decir, hipoxemia en rango de
M exico A rgentina
C hile U ruguay

Seguimiento a distancia

insuficiencia respiratoria, sin hipercapnia (PaCO2 normal). Adems


hacen mencin al GA-a de O2, que est alterado (>15mmg), lo que
indica una alteracin a nivel del parnquima pulmonar o de la va
area, puesto que en las alteraciones de la regulacin de la ventilacin y de caja torcica est en rango de normalidad. Por ltimo, nos
dicen que no hay respuesta a la administracin de O2 a alto flujo para
corregir la hipoxemia. Una vez excluidas las causas de hipoventilacin, al tener la PCO2 normal, el diagnstico diferencial se va a realizar en funcin de la respuesta al O2 de la hipoxemia:
Hay respuesta alteracin V/Q efecto espacio muerto. Por ello la
respuesta 1 es la menos probable.
No hay respuesta shunt o cortocircuito (resp. 2-5).
Pregunta 13.- R: 5
En este caso estamos ante un paciente que presenta en la GAB una
alcalosis respiratoria, es decir, est hiperventilando, por lo que las dos
primeras respuestas son falsas, ya que hacen referencia a enfermos
incapaces de hiperventilar. La alcalosis respiratoria aparece cuando el
enfermo aumenta su frecuencia respiratoria y elimina ms CO2 de lo
habitual. La prdida de CO2 condiciona que el pH aumente por encima de 7,45 y d lugar a una situacin de alcalosis de origen respiratorio. En el caso de las respuestas 3 y 4, estamos hablando de enfermos cuya situacin basal es de acidosis respiratoria compensada, es
decir, pacientes que retienen CO2 y compensan el pH reteniendo
bicarbonato por va renal. Sera necesario una hiperventilacin excesiva para eliminar ese exceso de PCO2 por debajo del rango de normalidad, y adems en ningn caso encontraramos unas cifras de
bicarbonato inferiores a lo normal, ya que existe el mecanismo de
compensacin en estas enfermedades.
En resumen:
pH
PCO2 HCO3 Alcalosis Resp
>7,45
<35 Normal
Acidosis Resp
- Aguda
<7,45
>45 Normal
- CR/Compensada
normal >45 Aumentada
Pregunta 14.- R: 3
En esta pregunta tenemos que valorar 2 gasometras en un mismo
paciente con EPOC como enfermedad de base.
La primera gasometra hace referencia a la situacin habitual del
enfermo en fase estable y la segunda al momento de la reagudizacin.
En ambas hay que valorar 2 cosas:
Oxigenacin: en ambas hay una situacin de hipoxemia, puesto
que la PO2 es inferior a 80 mmHg. Adems, estn en rango de
insuficiencia respiratoria al ser la PO2 <60 mmHg.
Equilibrio cido-base: la situacin es diferente segn el momento.
En fase estable, el paciente tiene una acidosis respiratoria crnica,
es decir, tiene una hipercapnia que compensa reteniendo bicarbonato por va renal para mantener el pH compensado. En el
momento de la reagudizacin la hipercapnia aumenta y el bicarbonato retenido por va renal no es suficiente para compensar el
pH, por lo que los encontramos en un valor prximo a la acidosis.
Es lo que se denomina acidosis respiratoria parcialmente compensada o crnica descompensada, si el pH es < a 7,35.
Las respuestas 1 y 2 son incorrectas, ya que el paciente tiene datos
de insuficiencia respiratoria crnica en ambas gasometras, como expresa la respuesta 3.
La respuesta 4 es incorrecta, ya que la lesin est en la va area y
el parnquima pulmonar, por lo que el GA-a O2 debe ser patolgico
(>15 mmHg) y la 5 tambin, porque no hay datos que indiquen la
ventilacin mecnica.
Pregunta 15.- R: 5
La EPOC es una patologa que se caracteriza por una obstruccin
crnica al flujo areo que generalmente es progresiva y no revierte
completamente con tratamiento (respuesta 5). El principal factor de
riesgo es el tabaco. El efecto depende del nmero de cigarrillos, de la
edad de inicio y del tiempo que se fuma. Esto se valora con un
parmetro que se denomina paquetes/ao.
Paquetes/ao = n de paquetes/da por aos fumando.
Si se superan los 20 paquetes/ao, hay mayor riesgo para desarrollar una EPOC, y si se superan los 40 paquetes/ao, mayor riesgo de
cncer de pulmn.

CTO Medicina C/ Nez de Balboa, 115 28006 MADRID (Espaa) Tfno.: (91) 782 43 32 / Fax: (91) 782 43 27
E-mail: secretaria@ctomedicina.com; iberocto@ctomedicina.com WEB: www.ctomedicina.com; www.iberocto.com

NM Pg. 3

La disnea suele aparecer alrededor de los 60 aos y tiene un desarrollo progresivo a lo largo de aos. Generalmente antes de la aparicin de la disnea suelen referir tos y expectoracin.
El estudio de la funcin pulmonar es til para establecer tanto el
diagnstico de obstruccin bronquial (FEV1 < 80% respecto al valor
terico) como la gravedad de la enfermedad (EPOC grave: FEV1 <
40% del valor terico).
Pregunta 16.- R: 4
Cuando hablamos de EPOC, hablamos de 2 grandes grupos de
enfermos que se comportan de forma diferente:
Bronquitis crnica: son enfermos con un hbito pcnico (obesos y
con cuello corto) y presentan una clnica habitual de disnea progresiva no muy importante que toleran bien, acompaada de tos con
expectoracin habitual. Tienen infecciones bronquiales con frecuencia, lo que condiciona episodios repetidos de insuficiencia respiratoria. Son enfermos que en situacin basal tienen una buena tolerancia
a la hipercapnia, por lo que no hiperventilan y mantienen una insuficiencia respiratoria crnica con acidosis respiratoria compensada. La
presencia de hipoxemia crnica conlleva la aparicin de poliglobulia
e hipertensin pulmonar (HTP). Esto se manifiesta clnicamente en su
aspecto ciantico abotargado. Todas estas caractersticas se definen en
las respuestas 1, 2, 3 y 5.
Enfisema: en estos enfermos la manifestacin clnica mas importante es la disnea. No toleran bien la hipercapnia, por lo que hiperventilan para compensarla. Por esta razn en su GAB presentan una
hipoxemia leve sin hipercapnia, lo que hace que la aparicin de
HTP y poliglobulia sea rara. Lo caracterstico en esta alteracin es la
destruccin de los septos alveolares y el aumento de los espacios
alveolares con la hiperinsuflacin consiguiente (respuesta 4).
Pregunta 17.- R: 3
La radiologa del enfisema es bastante expresiva en casos evolucionados, pero puede ser normal en estadios iniciales. Los datos ms
relevantes son:
Hiperclaridad pulmonar por un aumento del volumen de aire y
ausencia de vasos en las zonas ms perifricas al perderse los septos
alveolares (resp. 2).
Hiperinsuflacin que se manifiesta por un corazn pequeo y
alargado (corazn en gota, resp. 4), horizontalizacin de las costillas con aumento de los espacios intercostales, aplanamiento y
descenso diafragmtico (resp. 1) y aumento de los espacios retroesternal y retrocardaco.
Presencia de bullas, evidenciables como imgenes qusticas de
pared fina (resp 5).
Datos de hipertensin pulmonar. Solamente se van a apreciar en
casos muy avanzados.

Pregunta 17. Radiografa de trax en un paciente con EPOC tipo enfisema.

Pg. 4 NM

M exico A rgentina
C hile U ruguay

NEUMOLOGA Y CIRUGA TORCICA

Preparacin Examen de Seleccin 05/06 1 Vuelta

En la radiologa de la bronquitis crnica no hay un patrn caracterstico, aunque podemos encontrar habitualmente:
Aumento de la trama bronquial con imgenes en ral de tranva por
engrosamiento y fibrosis de las paredes bronquiales.
Signos de hipertensin pulmonar precapilar con aumento de la
trama vascular perihiliar y cardiomegalia por aumento del tamao
de la aurcula derecha y del ventrculo derecho.
En casos avanzados aparecern datos radiolgicos de enfisema
centroacinar, con aumento de los espacios retroesternal y
retrocardaco y aplanamiento diafragmtico. Es ms raro el hallazgo de imgenes compatibles con bullas.
Pregunta 18.- R: 2
Estamos ante un paciente fumador desde su juventud, con clnica
caracterstica de EPOC tipo bronquitis crnica. Refiere tos con expectoracin que a veces es hemoptoica y frecuentes reagudizaciones
infecciosas. En este momento ya tiene disnea de moderados esfuerzos, lo que nos tiene que hacer pensar que es un paciente que al
menos tiene una EPOC moderada.
En sus pruebas de funcin respiratoria lo que vamos a encontrar es
un patrn ventilatorio obstructivo con un VEMs y un ndice de
Tiffeneau disminuidos, junto con un aumento del volumen residual.
La capacidad de difusin de monxido de carbono, sin embargo,
estar poco afectada. Como en este momento de la enfermedad la
insuficiencia respiratoria ya est consolidada, lo habitual es encontrar
hipoxemia asociada a hipercapnia (no hipocapnia como dice la respuesta 2), encontrando datos en la gasometra de acidosis respiratoria
crnica con pH normal, PCO2 y HCO3- altos.
En la radiografa de trax encontraremos un aumento de la trama
bronquial como consecuencia del engrosamiento y fibrosis de las paredes de los bronquios, y un aumento del ndice cardiotorcico y de las
ramas vasculares perihiliares, ya que en este momento el paciente ya
tendr asociada a la hipoxemia crnica una hipertensin pulmonar.
Pregunta 19.- R: 2

Pregunta 19. Algoritmo teraputico del EPOC.

CTO Medicina C/ Nez de Balboa, 115 28006 MADRID (Espaa) Tfno.: (91) 782 43 32 / Fax: (91) 782 43 27
E-mail: secretaria@ctomedicina.com; iberocto@ctomedicina.com WEB: www.ctomedicina.com; www.iberocto.com

Comentarios TEST

Seguimiento a distancia

NEUMOLOGA Y CIRUGA TORCICA

Preparacin Examen de Seleccin 05/06 1 Vuelta

Comentarios TEST

El tratamiento de la EPOC comprende varios aspectos:


Tratamiento especfico: tiene tres objetivos fundamentales, supresin del hbito tabquico, evitar la contaminacin ambiental y
control de la infeccin. El tabaco es el factor de riesgo fundamental
en el desarrollo de EPOC. Su abandono, junto con oxigenoterapia
domiciliaria cuando est indicada, son las nicas medidas que
mejoran la supervivencia. Para conseguir la deshabituacin
tabquica se han creado unidades de apoyo en las que se hace un
tratamiento psicolgico y mdico, con terapia sustitutiva y otros
frmacos ms recientes como el bupropin. En el control de la
infeccin se van a emplear las vacunas antigripal y antineumoccica. La vacuna antigripal disminuye el riesgo de infeccin en aproximadamente 2/3 de los pacientes.
Tratamiento sintomtico: va dirigido a controlar el trastorno ventilatorio y la insuficiencia respiratoria y a conseguir una readaptacin al ejercicio y una reinsercin laboral. El tratamiento del trastorno ventilatorio comprende una serie de medidas destinadas a
facilitar la eliminacin de secreciones; son la estimulacin de la tos
voluntaria y la hidratacin correcta del esputo. El tratamiento
broncodilatador es un pilar fundamental. Los ms utilizados en
estos enfermos son los anticolinrgicos, ya que actan tanto a nivel
de la pequea va area como en las zonas de mayor calibre. Hasta
el momento slo exista un compuesto, el bromuro de ipratropio.
En la actualidad existe el bromuro de tiotropio, cuya ventaja principal es la administracin cada 24h frente a la administracin cada
8h del bromuro de ipratropio. Los frmacos utilizados para disminuir la inflamacin son los corticoides, cuya indicacin fundamental son las reagudizaciones. Los programas de rehabilitacin
han demostrado mejorar la tolerancia al ejercicio y la calidad de
vida, aunque no mejoran los parmetros de funcin pulmonar.
Pregunta 20.- R: 4
El dficit de 1 antitripsina (AAT) es una enfermedad hereditaria
que se caracteriza por una disminucin de las concentraciones plasmticas de AAT.
La AAT es una protena sintetizada en el hgado y localizada fundamentalmente en los pulmones. Inactiva a la elastasa de los neutrfilos
contrarrestando as su accin destructora sobre la matriz estructural del
pulmn. Es codificada por un nico gen en el cromosoma 14. La
enfermedad se hereda de forma autosmica codominante, ya que el
fenotipo se determina genticamente por la expresin independiente
de los 2 alelos paternos. La mayora de los enfermos tienen un fenotipo
PiZZ con niveles de enzima por debajo del 16% de lo normal (se establece como nivel protector el 35%). El fenotipo de la poblacin normal
es el PiMM y el de aquellos pacientes con dficit de la enzima (AAT
54%) sin manifestaciones clnicas es el Pizz.
Existe controversia sobre si la heterocigosis asocia manifestaciones.
El dficit de AAT se asocia fundamentalmente a enfisema panacinar, aunque tambin podamos encontrar otras lesiones del parnquima pulmonar como las bronquiectasias (la respuesta 4 es incorrecta).
La afectacin heptica es menos frecuente que la pulmonar y raras
veces coexisten. En nios se ha descrito hepatitis neonatal con marcada colestasis y en adultos cirrosis heptica.
Las manifestaciones de la enfermedad pulmonar son las del enfermo EPOC tipo enfisema cuyo sntoma fundamental es la disnea. La
principal caracterstica es la localizacin radiolgica en las bases (en
el enfisema del fumador es en los vrtices) y la precoz aparicin, en la
4 dcada de la vida, si el paciente es adems fumador. El tratamiento
de la enfermedad heptica es el trasplante. Para la afectacin pulmonar
inicialmente se intenta el tratamiento sustitutivo junto con las medidas
generales y si no va bien, se recurre al trasplante.
Pregunta 21:- R: 4
En estadios avanzados de la enfermedad, cuando existe insuficiencia respiratoria clnica, se ha demostrado que la OCD aumenta la supervivencia de los enfermos. Adems mejora las funciones
neuropsicolgicas, reduce los perodos de hospitalizacin y consigue una mayor tolerancia al ejercicio. Estas acciones se obtienen
como resultado de la correcin de la hipoxia tisular que conlleva
una disminucin de la hipertensin pulmonar y de la poliglobulia.

M exico A rgentina
C hile U ruguay

Seguimiento a distancia

El objetivo de la OCD es corregir la hipoxemia y mantener una


PaO2 aproximada de 65 mmHg, lo que se consigue generalmente con
flujos de 1,5-2 lpm administrados mediante gafas nasales un mnimo
de 16 horas al da, incluyendo el perodo del sueo.
La OCD siempre se plantea con el enfermo en fase estable (resp 4.
incorrecta), una vez abandonado el hbito tabquico y recibiendo
un tratamiento correcto y completo. Los criterios de OCD son una
PO2 persistente inferior a 55mmHg o una PaO2 entre 55mmHg y
60mmHg con evidencia de hipertensin pulmonar, Cor Pulmonale
crnico, insuficiencia cardaca congestiva, arritmias o poliglobulia. La
indicacin se considera definitiva tras 3 meses de tratamiento. Una
indicacin no plenamente aceptada es la presencia de una PaO2 >
60mmHg que presente importante descenso durante el ejercicio o
durante el sueo.
Nos podemos replantear el tratamiento en aquellos enfermos en
los que en la evolucin encontramos cifras de PaO2 superiores a
60mmHg respirando aire ambiental de forma persistente.
Pregunta 22.- R: 4
El tratamiento del Cor Pulmonale (CP) supone actuar sobre la enfermedad de base y la insuficiencia respiratoria que provoca, y sobre
la hipertensin pulmonar. El tratamiento de la enfermedad de base
supondr en la mayora de los casos actuar sobre la EPOC.
La correccin de la hipoxemia, que es la causa ms importante de
hipertensin pulmonar, es el tratamiento ms adecuado del cor pulmonale. Si iniciamos el tratamiento con oxgeno en la fase reversible de la
circulacin pulmonar, obtenemos grandes mejoras hemodinmicas
como la disminucin de la resistencia vascular pulmonar (resp. 1).
Los diurticos tienen un papel en el paciente con cor pulmonale y
edema perifrico. Pero hay que recordar que la expansin de volumen permite al corazn derecho mantener el G.C. cuando las RVP
estn aumentadas. Una diuresis excesiva puede provocar una mala
perfusin por disminucin del G.C. y una alcalosis metablica que a
su vez inducir de forma compensadora una mayor retencin de
CO2. Por ello es necesario un control riguroso de los iones sricos
(resp. 2 y 3). La digoxina no est indicada en estos pacientes, salvo si
hay asociada una taquiarritmia o una insuficiencia ventricular izquierda. Este frmaco, aunque mejora la contractilidad del VD, produce
una vasoconstriccin pulmonar, lo que empeora la hipertensin
pulmonar (resp. 5).
Los vasodilatadores no han demostrado beneficios a largo plazo.
La utilizacin de estos frmacos produce fundamentalmente dos efectos
adversos: la hipotensin sistmica, derivada de que su accin es fundamentalmente sistmica, y empeoramiento de la relacin V/Q, al
mejorar el flujo sanguneo de zonas mal ventiladas (por ello la resp. 4
es la incorrecta).
Pregunta 23.- R: 1
Estamos ante un paciente diagnosticado de EPOC. El enfermo acude a urgencias por empeoramiento de su situacin clnica y con datos
clnicos de encefalopata hipercpnica. En la GAB, el paciente muestra una marcada acidosis respiratoria (pH 7,25) y una hipoxemia importante (pO2 50mmHg). Se instaura tratamiento con oxigenoterapia
que consigue mejorar la oxigenacin. Sin embargo, el paciente no
slo no mejora clnicamente sino que progresa a un cuadro de estupor y coma. En este momento, la GAB muestra una acidosis an ms
importante con unas cifras de PCO2 muy altas.
La respuesta 4 es correcta, hay que tratar la infeccin que ha desencadenado la reagudizacin, pero no es el objetivo prioritario en
este momento. Las teofilinas, como dice la respuesta 5, son estimulantes respiratorios, pero no lo suficientemente potentes como para resolver la situacin en la que se encuentra el paciente.
Son incorrectas tanto la respuesta 2 como la 3. No es posible retirar
el O2, porque se producir una hipoxemia tisular brusca sobre la
situacin de acidosis que ya tiene el paciente que podra provocarle
la muerte y tampoco es necesario aumentar el flujo, porque el estupor
es secundario a la hipercapnia y no a la hipoxemia.
La actitud adecuada, como expresa la respuesta 1, es la instauracin de la ventilacin mecnica. El deterioro del nivel de conciencia
es el dato clnico fundamental para indicar la ventilacin mecnica
tanto en la insuficiencia respiratoria aguda como en la crnica.

CTO Medicina C/ Nez de Balboa, 115 28006 MADRID (Espaa) Tfno.: (91) 782 43 32 / Fax: (91) 782 43 27
E-mail: secretaria@ctomedicina.com; iberocto@ctomedicina.com WEB: www.ctomedicina.com; www.iberocto.com

NM Pg. 5

Pregunta 24.- R: 1
El tratamiento de la EPOC en fase estable se establece en funcin
de la gravedad de la enfermedad. La gravedad de la enfermedad depende a su vez del FEV1:
EPOC leve (FEV1 60-80% del valor terico).
El dato clnico ms frecuente es la "tos del fumador" y la disnea o no
existe o es mnima. El tratamiento depende fundamentalmente del
tipo de disnea que presente el paciente. Si es continua, se instaurar
tratamiento con anticolinrgicos y se reservarn los agonistas 2
para las crisis como medicacin de rescate. Si la disnea es intermitente, se utilizarn los agonistas 2 a demanda.
EPOC moderada (FEV1 40-59% del valor terico).
Hay disnea moderada, tos acompaada o no de expectoracin y
ruidos patolgicos en la auscultacin pulmonar. Este grupo de enfermos recibe tratamiento con anticolinrgicos y agonistas 2 de
larga duracin para mantener una broncodilatacin sostenida. Los
agonistas 2 de corta duracin permanecen como medicacin de
rescate. Si no hay control sintomtico, se aaden las teofilinas y si
hay insuficiencia respiratoria se instaura la OCD. En la pregunta
estamos en este grupo de enfermos, por lo que la respuesta correcta es la 1.
EPOC grave (FEV1 del 40% del valor terico).
Todos reciben tratamiento con anticolinrgicos, agonistas 2 de
corta y larga duracin y teofilinas. Si no hay buena respuesta se
plantea la utilidad del tratamiento con corticoides inhalados. Al
igual que en el grupo anterior, si hay insuficiencia respiratoria se
instaura la OCD.
Pregunta 25.- R: 4
Hay distintas clasificaciones de asma. En esta pregunta se hace
referencia a la clasificacin etiolgica. Segn la etiologa, distinguimos
entre asma extrnseco o atpico y asma intrnseco. El asma extrnseco
es la que se desencadena por un agente inmunolgico determinado.
La reaccin inmune est mediada por la IgE. Los linfocitos B segregan
IgE especfica cuando son activados por los linfocitos Th2, los cuales
se activan al contacto con el antgeno. Los alergenos se unen a la IgE,
que est unida a la membrana de clulas productoras de sustancias
proinflamatorias que van a desencadenar la sintomatologa. La reexposicin aumenta la sntesis de IgE especfica. Suele iniciarse en la
infancia o en la juventud y cursa de forma intermitente en relacin a
la exposicin a los antgenos. Es frecuente la historia familiar o personal de alergia, por lo que son tiles las pruebas cutneas que son
positivas. Por todo lo anterior podemos descartar las respuestas 1 y 3.
En el asma intrnseco no se puede demostrar la existencia de un
agente etiolgico ni un mecanismo inmunolgico responsable. Los
linfocitos pueden liberar citoquinas, que activan los leucocitos. Esto se
produce tambin en respuesta a algn antgeno, pero sin intervencin
de la IgE. Los eosinfilos son las clulas caractersticas de la reaccin
asmtica activados por la interleuquina 5 y liberan un gran nmero de
sustancias inflamatorias responsables de la sintomatologa. La eosinofilia es mayor en este tipo de pacientes (las respuestas 2 y 5 son incorrectas). De forma caracterstica, comienza en la edad adulta, se asocia a
veces a poliposis nasal e intolerancia a los salicilatos y tiene peor evolucin, con sntomas ms persistentes y peor respuesta al tratamiento. Por
lo dicho, la respuesta 4 es la respuesta correcta.
Pregunta 26.- R: 2
La sospecha de asma bronquial se basa siempre en la historia clnica y en los hallazgos de la exploracin fsica. La trada clsica es la
disnea, sibilancias y tos, y a veces opresin torcica. Pueden aparecer
de forma simultnea o aislada y son muy importantes las situaciones
ante las que aparecen estos sntomas.
Para confirmar el diagnstico se realizan unas pruebas de funcin
respiratoria. Si demostramos un patrn obstructivo, el paso siguiente es
realizar un test de broncodilatacin para demostrar la reversibilidad de
la obstruccin. Se considera que el test es positivo cuando hay una
mejora del FEV1 igual o mayor al 15%. Si la espirometra es normal, no
se puede excluir el diagnstico de asma y hay que continuar el estudio
realizando un test de provocacin bronquial. Por ello, la respuesta
incorrecta es la 2. Los test de provocacin demuestran la hiperreactividad de la va area. Se define como hiperreactividad a la disminucin
Pg. 6 NM

M exico A rgentina
C hile U ruguay

NEUMOLOGA Y CIRUGA TORCICA

Preparacin Examen de Seleccin 05/06 1 Vuelta

del FEV1 un 20% o ms de su lnea basal tras la realizacin de una


prueba de provocacin con agentes inhalados (metacolina o histamina)
o estmulos fsicos (ejercicio o hiperventilacin). Esto se debe a que hay
una respuesta exagerada a estos agentes por parte de la va area. Las
respuestas 3, 4 y 5 son correctas. Generalmente, despus de realizar el
test de provocacin se realiza el test de broncodilatacin para demostrar la reversibilidad de la obstruccin.
Pregunta 27.- R: 5
En el diagnstico diferencial del asma hay que tener en cuenta
mltiples procesos. La respuesta 1 hace referencia al reflujo gastroesofgico. Durante la noche, con el decbito, el reflujo aumenta y son
ms fciles las microaspiraciones digestivas. Esto produce una irritacin sostenida en la va area que conduce a una situacin de hiperreactividad fundamentalmente de predominio nocturno. El tratamiento con inhibidores de la bomba de protones y medidas posturales suprime las manifestaciones. En nios de corta edad hay que tener
en cuenta, como dice la respuesta 2, la posibilidad de aspiracin de
un cuerpo extrao. En ese caso las manifestaciones suelen ser localizadas en la zona del pulmn donde se encuentra el cuerpo extrao.
La respuesta tres hace referencia a las crisis de ansiedad. El diagnstico
diferencial se basa en la ausencia de alteraciones en el intercambio de
gases y la mala respuesta al tratamiento convencional. Otros procesos
con los que se puede plantear el diagnstico diferencial son la EPOC,
el asma cardial, el tromboembolismo pulmonar, fibrosis qustica y
bronquiolitis. Tambin hay que descartar estenosis en las VAS. El diagnstico diferencial se basa fundamentalmente en los ruidos respiratorios. En el caso del asma aparecen sibilancias, que son ruidos respiratorios de predominio espiratorio. En las estenosis de las VAS aparece el
estridor, ruido de predominio inspiratorio.
Por ltimo, al valorar un a paciente asmtico hay que considerar
aquellos procesos que pueden cursar con asma, como la neumona
eosinfila crnica, la aspergilosis broncopulmonar alrgica, las vasculitis (respuesta 4) y el sndrome carcinoide.
Para terminar, la ltima respuesta es incorrecta, puesto que los
antecedentes familiares aparecen en el caso del asma extrnseca, pero
no son caractersticos de la intrnseca, y la eosinofilia puede no aparecer en enfermos diagnosticados de asma.
Pregunta 28.- R: 2
Las crisis de asma se clasifican en funcin del grado de obstruccin. A mayor obstruccin bronquial, ms grave es la crisis. No obstante en el momento de la crisis la valoracin se basa, adems de en
los datos de pico flujo, en la situacin clnica del enfermo.
Son signos de gravedad la presencia de disnea de reposo, la utilizacin de la musculatura accesoria, las sibilancias intensas, la diaforesis,
taquicardia mayor a 120 lpm, taquipnea mayor a 30 rpm y el pulso
paradjico mayor a 25mmHg.
Existen tambin unos signos llamados de extrema gravedad, que son
a los que hace referencia el enunciado de la pregunta. En estos enfermos el signo que ms se relaciona con la gravedad de la obstruccin es
la hiperinsuflacin, ya que a medida que la broncoconstriccin aumenta, el aire va quedando atrapado en el interior del pulmn. Al no
salir el aire a travs de una va cerrada, no encontraremos sibilancias
(ruido que se produce al salir el aire por una va estenosada) sino silencio auscultatorio, otro de los datos de extrema gravedad. En este momento aparecen signos de compromiso hemodinmico como la
bradicardia y la hipotensin. Por ltimo, la caracterstica en el patrn
respiratorio es la disnea intensa acompaada de una incapacidad para
hablar. La tos y la expectoracin no son posibles en este momento
porque el paciente es incapaz de generar un flujo espiratorio suficiente.
Pregunta 29.- R: 4
La crisis asmtica se manifiesta por la aparicin aguda de disnea,
sibilancias, opresin torcica, tos, etc. Estos sntomas y signos pueden
aparecer de forma aislada o combinada. Adems hay que conocer
los signos de gravedad y los de extrema gravedad para tomar una
actitud teraputica adecuada.
Son signos de gravedad la disnea de reposo, las sibilancias intensas,
la diaforesis, la taquicardia >120 lpm, la taquipnea >30 rpm y el
pulso paradjico > 25mmHg.

CTO Medicina C/ Nez de Balboa, 115 28006 MADRID (Espaa) Tfno.: (91) 782 43 32 / Fax: (91) 782 43 27
E-mail: secretaria@ctomedicina.com; iberocto@ctomedicina.com WEB: www.ctomedicina.com; www.iberocto.com

Comentarios TEST

Seguimiento a distancia

NEUMOLOGA Y CIRUGA TORCICA

Preparacin Examen de Seleccin 05/06 1 Vuelta


A medida que la crisis es ms grave, cambia la auscultacin pulmonar. En las fases de menor obstruccin encontramos una disminucin
del murmullo vesicular y sibilancias difusas. Cuando progresa la obstruccin, las sibilancias van disminuyendo hasta llegarse al silencio
auscultatorio en crisis muy graves. Acompaan al silencio auscultatorio
otros signos de extrema gravedad como la cianosis, bradicardia, hipotensin y deterioro del nivel de conciencia.
La medicin del pico flujo espiratorio (PEF) es el parmetro de
mayor valor para evaluar el grado de obstruccin al flujo areo y la
respuesta al tratamiento. El PEF corresponde al flujo mximo obtenido en una espiracin forzada y tiene una buena correlacin con el
volumen espirado en el primer segundo (FEV1). Idealmente debe valorarse con un espirmetro, pero en el entorno de las urgencias se
utilizan medidores de flujo espiratorio ambulatorios (peak flow meter,
o PFM).
Es importante tambin identificar las posibles complicaciones en el
contexto de una crisis asmtica como el neumotrax, neumomediastino y atelectasias por impactacin de tapones mucosos. Son manifestaciones infrecuentes que aparecen generalmente en crisis graves empeorando la situacin del enfermo (la resp. 4 es la incorrecta).

Comentarios TEST

Pregunta 30.- R: 4
Los agonistas 2 de accin rpida inhalados representan el tratamiento inicial de eleccin en la crisis asmtica. Proporciona una broncodilatacin inmediata pero breve (menos de tres horas). Estos frmacos
tambin pueden administrarse por va parenteral, subcutnea o intravenosa, aunque no se consigue una mejor respuesta. Por ello, esta va de
administracin es para aquellos pacientes que no responden a la va
inhalada, o cuando, por algn motivo, sta no es posible.
Los corticoides sistmicos se utilizan en pacientes que no responden al tratamiento inicial con agonistas 2. Esto generalmente ocurre
en crisis moderadas y graves. El efecto beneficioso de estos frmacos
ocurre aproximadamente a las 6-12 horas de su administracin. La
eficacia de los corticoides administrados por va intravenosa y por va
oral es similar.
Los corticoides inhalados no tienen una utilidad demostrada en el
momento agudo, pero su uso debe iniciarse antes de suspenderse la
terapia sistmica, si el paciente lo tena como tratamiento de base. Esto
se debe a que el inicio de su accin es gradual.
Las metilxantinas se utilizan como coadyuvantes de los agonistas
2. Como monoterapia es un tratamiento menos eficaz. Se utilizan
como tercera lnea de tratamiento en las crisis graves. Tras el momento
inicial en el que se administran por va intravenosa se aportan por va
oral.
Pregunta 31.- R: 5
En los pacientes asmticos, los criterios de ingreso en UCI en una
reagudizacin son clnicos, gasomtricos y de funcin pulmonar.
Entre los datos clnicos est la necesidad de ventilacin mecnica
por deterioro del nivel de conciencia o parada cardiorrespiratoria y el
agotamiento fsico.
Los criterios gasomtricos son la hipoxemia <60 mmHg refractaria
al tratamiento y la hipercapnia refractaria al tratamiento. Nunca se
indicar intubacin orotraqueal y ventilacin mecnica antes de comprobar la respuesta al tratamiento, ya que ms del 80% mejoran con
la terapia broncodilatadora y no requieren esta tcnica cruenta. Pero
hay que tener en cuenta que una PaCO2 dentro de los valores de
normalidad indica que el paciente se est fatigando, ya que lo habitual en el contexto de una crisis asmtica es que la PaCO2 est disminuida por hiperventilacin.
Por ltimo, el criterio de funcin pulmonar que indica ingreso en
UCI es el PEF menor del 33% a pesar del tratamiento.
La taquipnea y la hiperinsuflacin torcica son signos de crisis
asmtica grave, pero no de ingreso en UCI.
Por tanto, la respuesta 5, que nos habla de una PCO2>45mmHg,
es la opcin adecuada.
Pregunta 32.- R: 3
Segn el estado basal del paciente y la frecuencia de las crisis,
podemos establecer la clasificacin clnica del asma bronquial. Es la
ms til desde un punto de vista teraputico, ya que en ella se basa el
M exico A rgentina
C hile U ruguay

Seguimiento a distancia

esquema de tratamiento. Utiliza criterios clnicos y funcionales para


definir cuatro escalones:
Asma intermitente: los sntomas diurnos aparecen menos de una
vez por semana con crisis nocturnas muy infrecuentes (menos de
dos veces por mes). Su funcin pulmonar es normal (PEF, o FEV1
>80%) y la variabilidad del PEF es menor del 20%.
Asma persistente leve: hay ms de una crisis semanal, pero menos
de una diaria. Los sntomas nocturnos aparecen ms de 2 veces al
mes. La funcin pulmonar es normal, pero a diferencia del grupo
anterior, la variabilidad est entre el 20-30%.
Este es el escaln en el que se sita el enfermo del enunciado de la
pregunta. En el tratamiento se utilizan corticoides inhalados en
dosis bajas asociados a un agonista 2 de accin corta de rescate. Si
aparecen sntomas nocturnos se pueden aadir agonistas 2 de
accin prolongada. En los nios se pueden sustituir los corticoides
inhalados por inhibidores de las clulas cebadas. Este tratamiento
es el que aparece en la respuesta 3.
Asma persistente moderada: sntomas continuos durante el da
con crisis nocturnas semanales. Su PEF / FEV1 est entre el 60-80%
del terico con una variabilidad mayor del 30%.
Asma persistente grave: sntomas diurnos y nocturnos continuos con
crisis graves que le limiten su actividad normal y el sueo. PEF o FEV1
<60% del valor terico con una variabilidad mayor del 30%.

Pregunta 32. Tratamiento del asma.

1234

567689
7 9286

92694726

176962
94

6

1234
567893578678



4954554


1234
892527867888
4954554


975 582
56442

1234
8925278678
3894

!

4954554

975 582
56442
"
156442
4 5#6494

12342889

!

4954554

975 582
56442
"
156442
4 5#6494

176962
94


882
14652742

564424
83464

14652742

564424
83464

14652742

564424
83464

146527424
83464$256
88%949
6 975 582
9482&

Pregunta 33.- R: 3
A todo enfermo, en cualquier fase de la evolucin, se le deben
suministrar instrucciones para iniciar tratamiento inmediatamente, ante
una reagudizacin y sobre cundo y cmo debe solicitar atencin
mdica. La valoracin de la gravedad y la respuesta al tratamiento se
basan en los sntomas y en la cada del PEF. En funcin de la gravedad
de la crisis, el tratamiento comprende agonistas 2 nebulizados,
oxigenoterapia y corticoides sistmicos fundamentalmente.
Crisis leve: los sntomas slo aparecen con el ejercicio. Es capaz de
hablar normalmente y hay buena respuesta al tratamiento inicial.
El FEV1 o PEF es > 70-80%. La PaO2 es > 80 mmHg y la PaCO2 <35
mmHg. Generalmente es suficiente el tratamiento con agonistas 2
en nebulizacin o mediante inhalacin con cmara espaciadora.
La reevaluacin inicial se hace a los 30 minutos, y si hay mejora
completa, puede darse de alta a las 4-6 horas.
Crisis moderada: el paciente refiere disnea de reposo y es capaz
de hablar con frases cortas. Hay respiracin sibilante y la frecuencia cardaca supera los 100 lpm. La respuesta al tratamiento habitual es slo parcial. El FEV1 o PEF se sita entre el 50-80%. La PO2
est entre 60-80 mmHg y la PCO2 < 42mmHg. Se iniciar tratamiento con oxigenoterapia y agonistas 2 mediante nebulizacin o
inhalacin con cmara. Se aadir tambin tratamiento con

CTO Medicina C/ Nez de Balboa, 115 28006 MADRID (Espaa) Tfno.: (91) 782 43 32 / Fax: (91) 782 43 27
E-mail: secretaria@ctomedicina.com; iberocto@ctomedicina.com WEB: www.ctomedicina.com; www.iberocto.com

NM Pg. 7

corticoides sistmicos. Deben ingresar al menos unas horas, y si es


preciso, se pueden aadir otros broncodilatadores.
Crisis grave: el paciente sufre una importante disnea y tiene un
habla entrecortada. Presenta signos de gravedad como diaforesis,
agitacin, uso de la musculatura accesoria, frecuencia respiratoria
>30 rpm, frecuencia cardaca >120 lpm, pulso paradjico > 25
mmHg y respiracin sibilante. El FEV1 o PEF es < 50% o es imposible de realizar. La PaO2 es < 60 mmHg y la PaCO2 >42 mmHg. Se
considera muy grave con criterios de ingreso en UCI cuando aparece deterioro del nivel de conciencia, cianosis, silencio auscultatorio, bradicardia e incapacidad para hablar, la PaO2 es <60mmHg
y la PCO2 >45 mmHg, su PEF es <33% y puede complicarse con
PCR. En este momento estara indicada la intubacin orotraqueal
y la ventilacin mecnica. En la pregunta se hace referencia a un
enfermo con una crisis grave pero sin criterios de UVI. Su tratamiento es con agonistas 2 inhalados o nebulizados ms corticoides sistmicos. Si no hay mejora se pueden aadir teofilinas i.v. y
sulfato de magnesio. La respuesta correcta por tanto es la 3.
Pregunta 34.- R: 4
Para establecer el tratamiento de asma bronquial estable, lo ms til
es utilizar la clasificacin clnica. Se utilizan para establecer dicha clasificacin criterios clnicos y funcionales. Se establecen 4 escalones.
Asma intermitente: la sintomatologa es infrecuente y la funcin
pulmonar es normal.
Asma persistente leve: los pacientes refieren ms de una crisis
semanal, pero menos de una al da. Hay sntomas durante la noche ms de 2 veces al mes. La funcin pulmonar es normal, pero la
variabilidad se sita entre el 20-30%.
Asma persistente moderada: tienen sntomas continuos que limitan su actividad fsica, y durante la noche aparecen sntomas con
mucha frecuencia, lo que altera la calidad del sueo. La funcin
pulmonar (VEF1 o PEF) se sita entre el 60-80% y la variabilidad es
>30%. En el tratamiento se utilizan corticoides inhalados en altas
dosis (800-1600 mg/d) ms agonistas 2 inhalados de larga duracin. Los de corta duracin se utilizan como medicacin de rescate.
El paciente del enunciado est situado en este escaln diagnstico
y el tratamiento correcto es el descrito en la respuesta 4.
Asma persistente grave: sntomas diurnos y nocturnos continuos
con crisis graves que pueden condicionar ingresos hospitalarios de
repeticin. El FEV1 o PEF es < 60% con una variabilidad >30%.
Pregunta 35.- R: 4
Las bronquiectasias se definen como la dilatacin anormal e irreversible del rbol bronquial causada por la destruccin de los componentes elsticos y musculares de las paredes bronquiales. Aunque
nuestro pas debe considerarse como un pas desarrollado, la tuberculosis sigue siendo el factor predisponente que se asocia a bronquiectasias con mayor frecuencia. A ello contribuyen la prescripcin
de tratamientos incorrectos, el retraso en el diagnstico y la alta incidencia en pacientes VIH seropositivos.
Causas de bronquiectasias son:
Infecciones: sarampin, tosferina, neumonas necrotizantes, infecciones virales.
Obstruccin bronquial: cuerpo extrao, estenosis postuberculosas, adenopatas, tumoraciones.
Neumonitis inflamatorias por aspiracin de contenido gstrico o
inhalacin de gases.
EPID como sarcoidosis.
Alteraciones de la inmunidad:
- Por hiperrespuesta: ABPA, post-trasplante pulmonar.
- Inmunodeficiencias: variable comn, enfermedad Bruton, enfermedad granulomatosa crnica, VIH, dficit de inmunoglobulinas (dficit de subclases de IgG, con o sin dficit de IgA). La
IgA deficitaria de forma aislada no condiciona la aparicin de
bronquiectasias (respuesta 4 incorrecta).
Malformaciones congnitas traqueobronquiales: sndrome de
Williams-Campbell y sndrome Mounier-Khun.
Alteraciones de la escalera mucociliar: FQ, discinesia ciliar primaria, sndrome de Young.
Pg. 8 NM

M exico A rgentina
C hile U ruguay

NEUMOLOGA Y CIRUGA TORCICA

Preparacin Examen de Seleccin 05/06 1 Vuelta

La tos y la expectoracin mucopurulenta son los sntomas principales, aunque algunos pacientes pueden estar asintomticos. La disnea, la anorexia y la prdida de peso son sntomas tardos que indican enfermedad evolucionada. La hemoptisis moderada es una complicacin relativamente frecuente que habitualmente se asocia a
infecciones del tracto respiratorio inferior. La hemoptisis masiva no
es frecuente, y en nuestro medio las bronquiectasias secundarias a
tuberculosis son responsables de esta complicacin en el 40% de los
casos. La sinusitis se asocia con frecuencia a bronquiectasias de
predisposicin gentica como la F.Q., sndrome de Young, sndrome
de Kartagener y las inmunodeficiencias con dficit de produccin
de anticuerpos.
Pregunta 36.- R: 3
Hay mltiples enfermedades que se asocian a bronquiectasias.
El sndrome de Williams-Campbell y el sndrome de MounierKhun son entidades que cursan con bronquiectasias debido a una
alteracin congnita del rbol traqueobronquial. El sndrome de
Kartagener es una alteracin congnita donde hay una reduccin o
ausencia del trasporte mucociliar debido a un mal movimiento de los
cilios de las clulas respiratorias. Es por tanto una discinesia ciliar
constituida por bronquiectasias, sinusitis y situs inverso.
La ausencia de los brazos de denena es la responsable ultraestructural de esta anomala, que afecta al epitelio ciliado del aparato respiratorio, al de los senos paranasales, trompas auditivas, conducto deferente
y flagelo de los espermatozoides. Junto con la F.Q. y el sndrome de
Young son las 3 entidades que cursan con esterilidad y bronquiectasias.
La panhipogammaglobulinemia, tambin denominada inmunodeficiencia variable comn, puede pasar desapercibida en la infancia y
debutar en la edad adulta con infecciones pulmonares recurrentes. Es
importante establecer su diagnstico, ya que es susceptible de tratamiento sustitutivo con gammaglobulinas, con buena respuesta en el
control de las infecciones. La Aspergilosis brocopulmonar alrgica es un
sndrome asmtico secundario a una reaccin de hipersensibilidad
tipo I y tipo III frente a antgenos del Aspergillus. Las bronquiectasias en
estos enfermos tienen un aspecto caracterstico, ya que la dilatacin
tiene lugar en la porcin proximal del bronquio (bronquiectasias centrales). La respuesta 3 es la correcta.
Pregunta 37.- R: 3
En el adulto, el grado de afectacin respiratoria secundaria a las
bronquiectasias y a la fibrosis pulmonar es el responsable de la elevada mortalidad. La morbimortalidad de estos pacientes est condicionada por las infecciones bacterianas broncopulmonares. Al principio, los grmenes habituales son el Staphylococcus aureus y el
Haemophilus influenzae. La prevalencia de S. aureus es mucho ms
acentuada durante los primeros aos de vida (30-40%) y disminuye
con la edad. Con el transcurso del tiempo es la Pseudomonas aeruginosa
el germen que las coloniza crnicamente, siendo muy difcil su erradicacin. Su prevalencia aumenta con la edad alcanzando a ms del
90% de los pacientes adultos. Inicialmente la colonizacin corresponde a cepas no mucoides y ms adelante a cepas mucoides (la
resp. 3 es la correcta). La presencia de B. cepacia se asocia generalmente a un rpido progreso de la enfermedad respiratoria. A veces, se
encuentra A. fumigatus en los cultivos de esputo. En general, no implica la aparicin de sntomas y no es necesario el tratamiento.
Pregunta 38.- R: 3
La fibrosis qustica (F.Q.) es la enfermedad hereditaria letal ms
frecuente en la raza blanca (1/2500 RN vivos) y es una de las entidades
que ms casos est aportando como causa de bronquiectasias en la
edad adulta. El defecto gentico produce una alteracin en el regulador de conductancia transmembrana (CFTR). El gen del CFTR se localiza en el brazo largo del cromosoma 7 y se transmite por una herencia autosmica recesiva. Se han descrito ms de 600 mutaciones que
pueden producir el fenotipo F.Q.
Las manifestaciones ms frecuentes en el adulto y el adolescente
son la enfermedad pulmonar secundaria a bronquiectasias y fibrosis
pulmonar, la insuficiencia pancretica con esteatorrea, prdida de
peso y, en menor frecuencia, diabetes, azoospermia obstructiva. Una
forma de presentacin clsica en el recin nacido es el leo meconial.

CTO Medicina C/ Nez de Balboa, 115 28006 MADRID (Espaa) Tfno.: (91) 782 43 32 / Fax: (91) 782 43 27
E-mail: secretaria@ctomedicina.com; iberocto@ctomedicina.com WEB: www.ctomedicina.com; www.iberocto.com

Comentarios TEST

Seguimiento a distancia

NEUMOLOGA Y CIRUGA TORCICA

Preparacin Examen de Seleccin 05/06 1 Vuelta


Aparece inmediatamente despus del nacimiento y cursa como una
obstruccin intestinal. Su incidencia se sita aproximadamente entre
el 10-15% (la respuesta incorrecta es, por tanto, la 3). El tratamiento es
quirrgico, aunque ocasionalmente responden al enema con
gastrografn, sobre todo si se utiliza precozmente.

Comentarios TEST

Pregunta 39.- R: 3
Estamos ante un paciente joven que presenta manifestaciones respiratorias tanto de va area superior (sinusitis y otitis) como de va
area inferior (bronquitis). El diagnstico diferencial en este enfermo
es muy amplio, aunque podemos reducirlo a aquellas entidades que
cursan con una alteracin en la motilidad ciliar. Esto condiciona una
reduccin o ausencia del transporte mucociliar en distintos epitelios,
como el del aparato respiratorio, los senos paranasales y las trompas
auditivas. En este grupo de enfermos podemos incluir la F.Q., el sndrome de Young y el sndrome de Kartagener.
El dficit de -1-antitripsina es una enfermedad hereditaria que
como manifestaciones respiratorias tiene fundamentalmente enfisema y bronquiectasias. La ABPA cursa con sintomatologa asmtica y
bronquiectasias. En ninguna de las dos suele existir afectacin de los
senos paranasales o los conductos auditivos, por lo que se pueden
descartar las respuestas 4 y 5.
El diagnstico definitivo nos lo dan los hallazgos del TAC torcico.
Adems de imgenes compatibles con bronquiectasias (stas aparecen en las 3 entidades enunciadas en las 3 primeras respuestas) nos
hablan de la existencia de un situs inversus. Esto, junto con la presencia de bronquiectasias y sinusitis, nos tiene siempre que hacer pensar
en el sndrome de Kartagener (respuesta 3).
Pregunta 40.- R: 5
La bronquiolitis simple es la inflamacin de la pared bronquiolar.
Esa inflamacin inicial puede conducir a un proceso de reparacin
en el que se produce una proliferacin excesiva de tejido de granulacin y posteriormente una fibrosis, apareciendo a nivel histolgico
mltiples tapones fibrosos que "obliteran" la luz bronquiolar. Esta entidad es la que se denomina bronquiolitis obliterante.
La bronquiolitis obliterante se divide en dos grupos:
Bronquiolitis obliterante, sin neumona organizada. Afecta de forma exclusiva o predominante al bronquiolo terminal. La afectacin puede ser constrictiva o proliferativa segn la afectacin histolgica. La proliferativa muestra lesiones de tejido fibroso ocupando
la luz y la constrictiva fibrosis concntrica en la muscular acompaada de infiltrado inflamatorio en la pared bronquiolar.
Bronquiolitis obliterante con neumona organizada. En ella la proliferacin ocurre tambin a nivel de los bronquiolos respiratorios o
a nivel alveolar. Si no hay afectacin del bronquiolo terminal, se
denomina neumona organizada.
Esta clasificacin tiene una traduccin clnica, ya que desde un
punto de vista funcional, la primera se expresa como un trastorno
obstructivo y la segunda como uno restrictivo o con menor frecuencia
mixto.
La bronquiolitis infecciosa es una de las enfermedades pulmonares que con mayor frecuencia requiere hospitalizacin durante la
infancia. Durante los dos primeros aos de vida, el VRS es el responsable de la mayora de los casos. Pasada esta edad, los virus siguen
siendo los agentes ms frecuentemente implicados, aunque otros grmenes como el Mycoplasma pneumoniae son ocasionalmente responsables. Los adenovirus tipos 3, 7 y 21 ocasionan las formas ms
graves, pudiendo desencadenar la aparicin de una bronquiolitis
obliterante en el 60% de los casos que se recuperan. La respuesta 5 es
la incorrecta, porque en adultos el nmero de casos es escaso. Por
tanto, la bronquiolitis infecciosa es una enfermedad tpicamente infantil.
Pregunta 41.- R: 5
La bronquiolitis obliterante se clasifica fundamentalmente en:
Bronquiolitis obliterante sin neumona organizada: debe sospecharse en pacientes con tos seca, disnea de esfuerzo y obstruccin
bronquial no reversible rpidamente progresiva. La sospecha diagnostica es ms fcil si no hay otros factores de riesgo para la obstruccin bronquial como el tabaquismo. Hoy en da se admite que
M exico A rgentina
C hile U ruguay

Seguimiento a distancia

la mayora de estas formas de bronquiolitis son errneamente incluidas entre otras formas ms comunes de obstruccin crnica al
flujo areo. La radiografa de trax puede ser normal o estar discretamente hiperinsuflada. La exploracin funcional respiratoria muestra una disfuncin obstructiva con una DLCO conservada. En el
TACAR se observan datos indirectos de obstruccin bronquiolar
como dilatacin de bronquios y bronquiolos terminales, areas de
mosaico (distinta atenuacin) e hiperinsuflacin. En el LBA es caracterstica la presencia de neutrfilos en un porcentaje superior al
25%. Se tratan con corticoides sistmicos en altas dosis, siendo
pobre la respuesta.
Bronquiolitis obliterante con neumona organizada: afecta por
igual a ambos sexos, preferentemente entre los 40-60 aos. Los
pacientes suelen referir los sntomas a partir de un cuadro pseudogripal que se contina con tos persistente, fiebre, malestar general,
disnea y prdida de peso. Suele haber crepitantes en la exploracin fsica y son raras las acropaquias (resp. 5 incorrecta). Desde un
punto de vista funcional presenta un trastorno restrictivo con disminucin de la DLCO. La Rx trax muestra en la mayora de los
pacientes infiltrados alveolares mltiples, parcheados y bilaterales.
En el LBA se ha descrito un aumento de linfocitos con disminucin
del cociente CD4/CD8. Una BTB de buena calidad permite el diagnstico por un patlogo experimentado. La mayor parte de los
pacientes responde a la corticoterapia.
Pregunta 42.- R: 4
El tumor carcinoide representa el 80-90% de los adenomas bronquiales. Aunque son los tumores primitivos distintos al cncer de
pulmn ms frecuentes, slo representan el 1-2% de todos los tumores pulmonares. Son tumores neuroendocrinos que se originan
en las clulas del sistema APUD. Desde el punto de vista histolgico
se dividen en carcinoide tpico y atpico. El tpico representa el
90% de todos los carcinoides bronquiales. Son tumores de baja
malignidad. El atpico representa el 10% de estos tumores y tienen
un comportamiento agresivo con metstasis por va ganglionar y
hematgena.
El 80% de los carcinoides bronquiales son centrales. Su sintomatologa se relaciona con la ocupacin y/o la obstruccin bronquial. Sus
manifestaciones ms frecuentes son la disnea, fiebre, dolor torcico y
hemoptisis (son tumores muy vascularizados). Es frecuente que por su
lento crecimiento la sintomatologa aparezca durante meses o aos.
Los tumores perifricos (20%) son, en general, asintomticos. El sndrome carcinoide es una entidad clnica constituida por manifestaciones cutneas (enrojecimiento facial), gastrointestinales (diarrea, hiperperistaltismo), respiratorias (broncoespasmo) y cardiovasculares (colapso vasomotor). Se asocia con mayor frecuencia a los carcinoides
intestinales. Slo aparece en el 2% de los carcinoides bronquiales, y
es caracterstico que aparezca sin presencia de metstasis distales. El
diagnstico de confirmacin se realiza mediante fibrobroncoscopia.
Su aspecto muy vascularizado es caracterstico, al igual que el sangrado con la toma de biopsia. La puncin aspiracin aguja fina se reserva
para aquellos que escapan al diagnstico por fibrobroncoscopia.
El tratamiento de eleccin es la reseccin quirrgica. Se realiza
ciruga conservadora en los tpicos y se reserva la ciruga agresiva para
los atpicos. En aquellos pacientes que presentan una enfermedad
ms all de las posibilidades de la ciruga se utiliza la quimioterapia.
Para el tratamiento del sndrome carcinoide se utilizan los anlogos de
la somatostatina.
Pregunta 43.- R: 3
Los tumores del sulcus superior (tumor de Pancoast) son tumores
que por su localizacin e invasin de estructuras vecinas producen
manifestaciones caractersticas. El tipo histolgico que se asocia con
ms frecuencia es el carcinoma epidermoide. Suele comenzar con
dolor en el hombro o en la regin vertebroescapular por invasin
directa de las costillas y msculos. El tumor progresa produciendo
una afectacin de las races C8, T1 y T2 del plexo braquial, lo que
conlleva un dolor irradiado por el brazo en su regin cubital y una
atrofia de la musculatura de la mano. Cuando invade el ganglio
estrellado y afecta a la cadena simptica causa un sndrome de Horner
(ptosis, miosis, enoftalmos y anhidrosis facial). El diagnstico se basa

CTO Medicina C/ Nez de Balboa, 115 28006 MADRID (Espaa) Tfno.: (91) 782 43 32 / Fax: (91) 782 43 27
E-mail: secretaria@ctomedicina.com; iberocto@ctomedicina.com WEB: www.ctomedicina.com; www.iberocto.com

NM Pg. 9

NEUMOLOGA Y CIRUGA TORCICA

Seguimiento a distancia

Preparacin Examen de Seleccin 05/06 1 Vuelta

Pregunta 44.- R: 4
Las manifestaciones clnicas locales estn en funcin de la localizacin tumoral. Los tumores de localizacin preferentemente central,
como el carcinoma epidermoide y el oat cell, presentan sntomas como
consecuencia de la irritacin, ulceracin y obstruccin bronquial. Tambin pueden aparecer manifestaciones como resultado de la infeccin
del parnquima pulmonar distal al tumor. La tos con expectoracin
hemoptoica es el sntoma ms frecuente. El dolor torcico y la disnea
aparecen con menos frecuencia. La neumonitis obstructiva o las neumonas de repeticin en la misma localizacin deben hacer sospechar
la existencia de una obstruccin bronquial que favorece las complicaciones infecciosas en el parnquima distal. Los tumores de localizacin
perifrica, como el adenocarcinoma, pueden ser diagnosticados de
forma casual en un examen radiolgico. Dan sntomas ms tardamente y suelen hacerlo por la afectacin pleural.
Respecto a las manifestaciones a distancia por las metstasis, dependen del rgano afecto. Ocurren con mayor frecuencia en el sistema
nervioso central (SNC), huesos, hgado y glndulas suprarrenales. El
pulmn contralateral es una afectacin poco frecuente (resp. 4 incorrecta). En el momento del diagnstico existen metstasis en el SNC en
aproximadamente el 10% de los casos, dando lugar a nuseas, vmitos,
cambios de conducta, cefalea y focalidad neurolgica. Las metstasis
seas ocurren con preferencia en la columna vertebral, pelvis y fmur,
produciendo dolor intenso con mala respuesta a los analgsicos. Las
metstasis hepticas y suprarrenales suelen ser asintomticas.
Ver tabla en la parte inferior.
Pregunta 45.- R: 5
Aproximadamente el 2% de los pacientes tienen sntomas y signos
sistmicos no relacionados con la propagacin tumoral. Son los sndromes paraneoplsicos. La mayor parte de las manifestaciones estn
en relacin con liberacin de sustancias.
El tumor que asocia con mayor frecuencia la presentacin de
sndromes paraneoplsicos es el carcinoma de clulas pequeas.
Pueden ser la primera manifestacin, aparecer de forma simultnea a
otros signos y sntomas o ser el dato de recidiva tumoral en pacientes
ya tratados.
El carcinoma microctico puede asociar hiponatremia (por secrecin inadecuada de ADH), hipopotasemia (por secrecin inadecuada

de ACTH), y con menor frecuencia, sndrome de Lambert-Eaton y ceguera retiniana (resp. 1). El carcinoma epidermoide puede asociar hipercalcemia (secrecin inadecuada de PTH) (resp 4) y el carcinoma de
clulas grandes ginecomastia (produccin de gonadotropinas).
Adems de los sndromes paraneoplsicos, hay determinadas formas de presentacin que se asocian ms a una estirpe histolgica que
a otra. La cavitacin es ms frecuente en el carcinoma epidermoide
(>20%) y en el carcinoma de clulas grandes (<20%), mientras que el
derrame pleural y la asociacin a cicatrices previas es caracterstico
del adenocarcinoma (resp 2 correcta y 5 incorrecta), que es adems
la extirpe histolgica que se diagnostica con mayor frecuencia en
pacientes no fumadores.
Pregunta 46.- R: 3
Una vez establecido el diagnstico, tipificacin histlogica del tumor y situacin endoscpica, es preciso realizar un estudio de extensin tumoral.
La realizacin de una TAC torcica y de la porcin superior del
abdomen es en la actualidad una exploracin rutinaria. La TAC torcica nos aporta informacin sobre la lesin primaria y su relacin con
estructuras vecinas. Es el mtodo no invasivo ms eficaz para la evaluacin ganglionar mediastnica. La TAC abdominal sirve para evaluar el
hgado y las glndulas suprarrenales (resp 4 correcta).
La mediastinoscopia es la tcnica ms precisa para evaluar la
afectacin mediastnica de forma bilateral. Unos autores la indican
en todo paciente que vaya a ser sometido a ciruga y otros slo en
aquellos cuyas adenopatas sean mayores de 1cm en el TAC o sean
tumores hiliares de gran tamao. La mediastinotoma se inicia cuando hay sospecha de afectacin ganglionar en la ventana aortopulmonar, es decir, en tumores de lbulo superior izquierdo y bronquio principal izquierdo. Estas dos tcnicas son invasivas y no se
realizan en pacientes con criterios de irresecabilidad (parlisis del
nervio recurrente larngeo). Por este motivo, la respuesta 3 es incorrecta. La toracoscopia es til en el estudio del derrame pleural con
citologa negativa para comprobar realmente la ausencia de afectacin pleural.
La gammagrafa sea est indica ante la presencia de dolores seos,
hipercalcemia o aumento de la fosfatasa alcalina. La TAC craneal
debe hacerse de rutina en el adenocarcinoma o carcinoma de clulas grandes por el alto riesgo de metstasis en SNC. En el resto de
estirpes histolgicas slo se realizar si aparece clnica sugestiva de
lesin subyacente.
Pregunta 47.- R: 5
La resecabilidad hace referencia a la posibilidad de que el tumor
sea oncolgicamente resecado, es decir, que el cirujano sea capaz de
quitar toda la masa tumoral.
No son pacientes quirrgicos los pacientes en estadio IIb y IV en la
clasificacin TNM para el cncer de pulmn de clulas no pequeas
y el carcinoma microctico con estadio TNM superior a I (la respuesta

Pregunta 44. Caractersticas diferenciales de los carcinomas pulmonares.


Relacin con tabaco

L ocalizacin y cavitacin

Epidermoide

+++

Centr a l
++

- Sndr om e de Pa ncoa st
- Aum enta PTH

Microctico

++

Centr a l
-

Adenocarcinoma
(incluy e
bronquioloalveolar)

Per ifr ico


-

Osteoa r tr opa ta hiper tr fica

Clulas grandes

Per ifr ico


+

Pg. 10 NM

M exico A rgentina
C hile U ruguay

Sndromes paraneoplsicos

Aum enta ADH


Aum enta ACTH
Dedos pa lillo ta m bor
Sndr om es neur olgicos m iop ticos

Caractersticas especficas

Tum or m a s m eta st tico


- Der r a m e pleur a l m a ligno
- Segundo tum or m s
m eta st tico
- Cr ecim iento sobr e cica tr ices
- Aum ento secr eciones
Ginecom a stia

CTO Medicina C/ Nez de Balboa, 115 28006 MADRID (Espaa) Tfno.: (91) 782 43 32 / Fax: (91) 782 43 27
E-mail: secretaria@ctomedicina.com; iberocto@ctomedicina.com WEB: www.ctomedicina.com; www.iberocto.com

Comentarios TEST

en la historia clnica, la radiologa de trax y la broncoscopia o la


puncin aspiracin aguja fina. En el estudio de extensin tumoral se
utilizarn el TAC toracoabdominal, la mediastinoscopia y la mediastinotoma. El TAC cerebral y la gammagrafa sea se realizan cuando
se sospecha una posible afectacin en estos rganos. La radioterapia
no aumenta la supervivencia en pacientes con cncer de pulmn
pero est indicada, de forma preoperatoria, en el tumor de Pancoast
para incrementar las posibilidades de reseccin. La ciruga que se
realiza es una reseccin en bloque (la respuesta 3 es la correcta). La
supervivencia a los 5 aos flucta entre el 25-50%.

NEUMOLOGA Y CIRUGA TORCICA

Preparacin Examen de Seleccin 05/06 1 Vuelta


5 es la incorrecta). Por lo tanto, son irresecables los tumores que
invaden el mediastino, corazn, grandes vasos, trquea o carina
traqueal, esfago y cuerpos vertebrales, y los que ocasionan derrame
pleural maligno, metstasis en los ganglios mediastnicos contralaterales
o supraclaviculares y metstasis a distancia.

Afectacin
NLR
Sdme. VCS

Afectacin
Traqueal extensa
o < 2 cm de la carina

Pregunta 49.- R: 2
En esta pregunta hacen referencia a un carcinoma epidermoide y
su clasificacin TNM. Nos dan datos de tamao y su localizacin en
funcin de los hallazgos de fibrobroncoscopia. Segn estos datos es
un T2 , ya que su tamao supera los 3cms, est situado a ms de 2cms
de la carina traqueal y no hay aparente invasin de estructuras vecinas. En la TAC tenemos hallazgos de adenopatas hiliares homolaterales, lo que corresponde a un N1. Finalmente la ausencia de metstasis
es un M0.
El estadio tumoral T2N1M0 corresponde a un estadio TNM IIb (respuesta 2). Podemos resumir los estadios TNM como aparecen en el
siguiente esquema:

Pregunta 49. Estadiaje TNM del cncer pulmonar no microctico.

25
Cncer microctico
estadio > 1

Seguimiento a distancia

11

12

13

14

12

13

113

1113

21

112

113

1112

1113

22

1112

1112

1112

1113

23

1113

1113

1113

1113

4567862898
1

Derrame
pleural maligno

Metstasis
a distancia

Afectacin del
nervio frnico

Comentarios TEST

Pregunta 47. Criterios de irresecabilidad.

No obstante, y a pesar de darse las circunstancias enunciadas, hay


un grupo de pacientes estrictamente seleccionados que pueden ser
sometidos a ciruga. En algunos tumores que invaden el frnico inferior puede hacerse una exresis completa. Pacientes jvenes con tumores localizados que invaden carina pueden ser subsidiarios de
carinoplastia y las metstasis nicas en SNC pueden ser extirpadas
junto al tumor primario en su totalidad.
En relacin con el estadio IIIa, la invasin ganglionar mediastnica
N2 es un dato de mal pronstico. Por ello, antes de considerar los
resecables hay que someterlos a quimioterapia de induccin. Si hay
buena respuesta se operan, y si no, se tratan con quimioterapia.
Pregunta 48.- R: 4
La operabilidad se refiere a la situacin funcional y fisiolgica del
paciente que le har resistir la ciruga dejando suficiente parnquima
como para mantener un adecuado intercambio gaseoso. La edad por
s misma no es una contraindicacin quirrgica, aunque es conocida
la peor evolucin de los pacientes mayores de 70 aos a los que se les
realiza una neumonectoma.
Los criterios de inoperabilidad se dividen en:
Globales: el estado clnico (Karnofsky < 40%) y las enfermedades
sistmicas graves e incontrolables (insuficiencia heptica o renal
severas, diabetes, coagulopatas, malnutricin, alteraciones psquicas o cerebrales).
Cardacas: arritmias incontrolables, insuficiencia cardaca grave
refractaria al tratamiento e IAM en los 3 meses previos (en las ltimas revisiones se reduce a 4-6 semanas).
Pulmonares: el principal criterio es el FEV1. Si es superior a 2l, el
paciente es operable, y si es inferior a 1l, inoperable. Si se sita
entre 1 y 2l, hay que calcular el FEV1 postoperatorio predicho
mediante una gammagrafa de perfusin. Si es mayor de 800cc, es
operable, si es menor de 08l, no. La CV <45%, la PaCO2 >45mmHg
y la hipertensin pulmonar grave son tambin criterios de inoperabilidad. (La respuesta incorrecta es la 4).

M exico A rgentina
C hile U ruguay

Pregunta 50.- R: 4
En esta pregunta valoramos la actitud ante el Ndulo Pulmonar
Solitario (NPS). Se define as el hallazgo de una densidad radiolgica
rodeada de parnquima pulmonar sano, con margen circunscrito
que mide de 1 a 6 cms en un paciente asintomtico. Lo primero que
hay que hacer es determinar si hay criterios de malignidad o de benignidad. Son criterios de benignidad: edad menor de 35 aos y no
fumador, ausencia de crecimiento en 2 aos, duplicacin en menos
de un mes, margen ntido, lesiones satlites, calcificacin en "ojo de
buey" o en "palomitas de maz".
En estos casos se hace un seguimiento radiolgico cada 3 meses
durante 1 ao, y luego anualmente. Cuando no hay criterios de
benignidad se considera un NPS con criterios de malignidad. En
estos casos, el primer paso es la realizacin de una fibrobroncoscopia (FBC) con biopsia. Si se llega al diagnstico de malignidad, hay
que hacer el estudio de extensin para indicar el tratamiento correcto. Cuando no se llega al diagnstico con la FBC se valora la
localizacin. Si es central, se repite la fibrobroncoscopia. Si es perifrico, se pasa al siguiente escaln diagnstico, puncin aspiracin
aguja fina (PAAF) guiada por TAC. Esta misma prueba diagnstica se
utiliza cuando hay un NPS central con 2 fibrobroncoscopias negativas. Cuando la PAAF es negativa hay que realizar una toracotoma
diagnstica. En este caso tenemos un paciente mayor de 35 aos
con antecedentes de tabaquismo, por tanto tiene criterios de malignidad. Lo primero que hay que realizar entonces, es una fibrobroncoscopia con biopsia (resp. 4).
Pregunta 51.- R: 4
Cuando se habla de operabilidad se hace referencia a la situacin
funcional y fisiolgica del enfermo. Se valora si podr resistir la ciruga,
si quedar suficiente parnquima sano para poder realizar correctamente el intercambio de gases.
Se valoran distintas facetas:
Situacin global del paciente: donde hay que tener en cuenta su
estado general y las enfermedades de base.
Si el Karnofsky es menor del 40% o hay enfermedades graves asociadas, no ser subsidiario de tratamiento quirrgico.
Situacin cardaca: las arritmias mal controladas o los antecedentes de infarto agudo de miocardio en las semanas previas tambin
contraindican la ciruga.
Situacin pulmonar: el principal criterio de inoperabilidad se basa
en el FEV1 del paciente, que se evala de la siguiente manera:
- FEV1 < 1l: inoperable.
- FEV1 1-2l: es necesario valorar la situacin en la que el paciente
quedar tras la ciruga. El FEV1 postoperatorio predicho se reali-

CTO Medicina C/ Nez de Balboa, 115 28006 MADRID (Espaa) Tfno.: (91) 782 43 32 / Fax: (91) 782 43 27
E-mail: secretaria@ctomedicina.com; iberocto@ctomedicina.com WEB: www.ctomedicina.com; www.iberocto.com

NM Pg. 11

NEUMOLOGA Y CIRUGA TORCICA

Preparacin Examen de Seleccin 05/06 1 Vuelta

za mediante una gammagrafa de perfusin. Esta es la situacin


en la que se encuentra nuestro paciente, por lo que la respuesta
correcta es la 4. Si en este clculo obtenemos un FEV1 >08l, el
paciente podr operarse.
- FEV1 >2l: operable.

ECO-TAC abdominal: se plantea ante la sospecha de metstasis


heptica o suprarrenal. Este es nuestro caso; encontramos aumentada tanto la bilirrubina como las enzimas hepticas, lo que nos
obliga a descartar la existencia de dao heptico secundario a
metstasis hepticas (resp. 2 correcta).

Pregunta 52.- R: 5
Cuando hay situaciones especiales debemos conocer a qu categora TNM pertenecen.
Podemos definir las siguientes situaciones:
Invasin del nervio frnico T3.
Parlisis de la cuerda vocal (resultante de la invasin del nervio
recurrente larngeo) T4.
Afectacin grandes vasos T4.
Sndrome de vena cava superior T4.
Extensin pericardio visceral T4.
Extensin pericardio parietal T3.
Tumor de Pancoast T3.
Tumor de Pancoast con afectacin de cuerpos vertebrales T4.
Afectacin traqueal o esofgica T4.
Derrame pleural o pericrdico con citologa positiva T4.
Cncer bronquioloalveolar difuso bilateral M1.
Segn esto, la respuesta 5 es correcta; adems, los tumores < 3 cms
se clasifican como T1, el derrame pleural maligno como T4, las metstasis ganglionares subcarinales como N2 y la invasin del nervio frnico como T3, por lo que las otras respuestas no son ciertas.

Pregunta 55.- R: 3
Estamos ante un paciente con un carcinoma de clulas no pequeas. Para poder decidir cul es el tratamiento de eleccin hay que
conocer el estadio TNM, ya que en funcin de ste decidimos la
actitud teraputica.
En este caso, tenemos un tumor de 7cms que afecta a estructuras
de la pared inferior de la caja torcica. Por ello se corresponde con un
tumor T3. No hay afectacin ganglionar locorregional linftica ni metstasis a distancia. Es por tanto un N0, M0. Un tumor T3 N0 M0 se
clasifica como estado TNM IIb. La reseccin quirrgica es el tratamiento indicado en los pacientes en estadio I, II, y IIIA, siempre que el
tumor y su drenaje linftico sea complementariamente extirpado. La
lobectoma y la neumonectoma son los procedimientos de eleccin.
Cuando hay afectacin de estructuras de la pared torcica, como es
nuestro caso, hay que ampliar la ciruga haciendo reseccin de la
pared torcica, o el diafragma. Si hablamos de un tumor de sulcus,
estara indicada la radioterapia preoperatoria. Otra situacin especial
son los tumores IIIb con N2. En estos pacientes es necesario realizar
poliquimioterapia previa para controlar la enfermedad ganglionar antes
de la ciruga. Nuestro paciente tiene un tumor IIb con extensin
diafragmtica, por tanto la respuesta correcta es la que incluye la
ciruga. Hay que elegir entre la 3 y la 4. La respuesta 4 no es correcta,
ya que nuestro paciente no tiene afectacin ganglionar, por lo que no
estara indicada la quimioterapia neoadyuvante. La respuesta 3 permite la reseccin completa del tumor y su drenaje linftico, es la
correcta. No son candidatos a ciruga los pacientes en estadio IIIb y IV.
Estos pacientes recibirn tratamiento con poliquimioterapia y radioterapia.
CIRUGA:
- IA.
- IB.
- IIA.
- IIB.
- IIIA.
- (T3 N1 M0).

Pregunta 53.- R: 3
Esta pregunta hace referencia a un carcinoma microctico. Habitualmente en esta histologa tumoral se diferencian dos estadios:
Enfermedad localizada: indica que la enfermedad puede ser abarcada por un campo de radioterapia torcica, es decir est limitada
al trax, ganglios linfticos regionales (mediastnicos, hiliares y
supraclaviculares, ipsi y contralaterales), afectacin del larngeo
recurrente y sndrome de vena cava superior (SVCS).
Enfermedad avanzada: no abarcable por radioterapia.
Sin embargo, a veces tambin se emplea la clasificacin TNM en
estos pacientes. Esto es importante sobre todo si hablamos de estadios
tumorales Ia Ib, ya que son resecables.
Nuestro paciente tiene clnica de SVCS, por tanto corresponde a
una clasificacin tumoral T4; tiene adenopatas mediastnicas, lo que
corresponde a un estadio ganglionar N2 , y no tiene metstasis (M0). El
estadio TNM que corresponde al T4N2M0 es el IIIb (respuesta 3). En la
clasificacin habitual sera un tumor de clulas pequeas en estadio
localizado no subsidiario de ciruga. Recuerda que el carcinoma
microctico con estadio TNM>1 es un criterio de irresecabilidad.
Pregunta 54.- R: 2
Estamos ante un paciente con 61 aos, diagnosticado de carcinoma epidermoide en LSI. Por los hallazgos radiolgicos y de fibrobroncoscopia nos encontramos con una masa tumoral <3 cm que est a
> 2 cm de la carina traqueal. Es, por tanto, un estadio tumoral T1.
Adems no hay evidencia radiolgica de existencia de metstasis ganglionares (N0). La respuesta 5 no tiene ningn sentido, ya que la clasificacin histolgica del tumor ya la conocemos. La respuesta 4 tambin es incorrecta ya que la RMN rara vez es necesaria, al no aportar
informacin adicional al TAC. Salvo en los tumores de Pancoast o en
los ntimamente relacionados con el mediastino (no es nuestro caso),
donde la RMN torcica nos ayuda a dilucidar la relacin del tumor
con el plexo braquial y los grandes vasos mediastnicos, esta prueba
no est indicada. La bsqueda exhaustiva de metstasis en pacientes
asintomticos y sin anormalidades analticas no est indicada ante la
baja probabilidad de deteccin de metstasis silentes. Sin embargo,
cuando existen sntomas o signos sugestivos hay que indicar las exploraciones pertinentes.
TAC cerebral: se realiza cuando hay sntomas neurolgicos, en el
oat cell (hay un 10% de metstasis asintomticas) y en algunos
casos de adenocarcinoma y carcinoma de clulas grandes.
Gammagrafa sea: est indicada cuando hay un aumento de la
fosfatasa alcalina, hipercalcemia o sntomas articulares.

Pg. 12 NM

M exico A rgentina
C hile U ruguay

TRATAMIENTO NEOADYUVANTE + CIRUGA:


- IIIA.
- T 1-3 N2 M0.
QUIMIOTERAPIA-RADIOTERAPIA:
- IIIb.
- IV.
Pregunta 56.- R: 4
Las causas de derrame pleural son mltiples y el mecanismo de
produccin vara de unas a otras. Precisamente, atendiendo a estos
mecanismos, los derrames pleurales pueden dividirse en dos grandes
grupos exudados y trasudados.
En general, cada patologa produce uno u otro tipo de derrame,
aunque algunas enfermedades, como el tromboembolismo pulmonar o el mixedema, pueden originar ambos (resp. 5).
Los exudados se producen cuando hay enfermedad pleural. Los
trasudados cuando los factores sistmicos que influyen en la formacin o absorcin de lquido pleural se alteran (aumento de la presin
hidrosttica o disminucin de la presin onctica).
La separacin entre exudados y trasudados se establece en funcin de los criterios de Light:
Protenas en LP / Protenas plasmticas > 0,5.
LDH en LP / LDH plasmtica > 0,6.
LDH superior 2/3 del lmite superior plasmtico.
Si no se cumple ningn criterio, se considera trasudado. La causa
ms frecuente es la insuficiencia cardaca congestiva (resp. 1). Otras
causas son el tromboembolismo pulmonar y la cirrosis heptica.

CTO Medicina C/ Nez de Balboa, 115 28006 MADRID (Espaa) Tfno.: (91) 782 43 32 / Fax: (91) 782 43 27
E-mail: secretaria@ctomedicina.com; iberocto@ctomedicina.com WEB: www.ctomedicina.com; www.iberocto.com

Comentarios TEST

Seguimiento a distancia

NEUMOLOGA Y CIRUGA TORCICA

Preparacin Examen de Seleccin 05/06 1 Vuelta


Si se cumple alguno de los criterios, hablamos de exudados. Son
causa de exudado:
Derrame pleural paraneumnico: es la causa ms frecuente de
derrame tipo exudado (resp. 2). La etiologa ms frecuente son los
anaerobios, neumococo y gram negativos. El tratamiento es el de la
infeccin de base.
Tuberculosis pleural: suele ser una primoinfeccin. Los pacientes
afectados suelen ser jvenes y con Mantoux negativo (resp. 4 incorrecta).
Derrame neoplsico: la causa ms frecuente son los tumores pulmonares. Los tumores pleurales malignos son poco frecuentes y
estn en relacin con el asbesto (resp. 3).
Derrame asociado a enfermedades gastrointestinales: pancreatitis,
rotura esofgica, enfermedad de Whipple.
Otras: colagenosis, neumotrax, quilotrax, mixedema, embolismo pulmonar, sndrome de Meigs, etc.

Seguimiento a distancia

Pregunta 58.- R: 5
La afectacin pleural es posiblemente la afectacin pleuropulmonar ms frecuente de la artritis reumatoide. Su prevalencia se ha estimado en menos del 5%. El derrame pleural puede aparecer antes de
las manifestaciones articulares o coincidir con las fases de artritis activa. Suele aparecer en pacientes varones mayores de 40 aos y con
ndulos subcutneos. Puede cursar asintomtico y descubrirse en
una Rx de trax, o debutar con dolor pleurtico acompaado o no de
disnea. Puede acompaarse de enfermedad parenquimatosa
pulmonar y con menos frecuencia de pericarditis. Suele ser de escasa
o moderada cuanta, generalmente unilateral y de predominio derecho. Suele persistir durante semanas o meses y, finalmente, resolverse
de forma espontnea. En algunos casos persiste durante aos, evolucionando a pseudoquilotrax. Es un lquido con caractersticas de
exudado, con niveles altos de protenas y LDH, y tpicamente presenta cifras bajas de glucosa (<30mg/dl), pH (por debajo de 7,20, la
respuesta 5 es incorrecta) y complemento. El recuento celular suele
mostrar predominio linfoctico. Un dato interesante es la presencia de
cristales de colesterol cuando es de larga evolucin. Otro hallazgo son
ttulos altos de FR, aunque hay que tener en cuenta la relativa inespecificidad de este dato, ya que ttulos altos de FR pueden encontrarse
en derrames neoplsicos, tuberculosos o lpicos.
El tratamiento no est estandarizado; el uso de frmacos inductores de remisin de la enfermedad de base tambin induce a la resolucin de la afectacin pleural. Si evolucionan a fibrotrax extenso, es
necesaria la ciruga.

Pregunta 58. Diferencias entre exudado tuberculoso y reumatoide.



Comentarios TEST



Pregunta 56. Causas de trasudado pleural.

Pregunta 57.- R: 5
Desde el punto de vista radiolgico puede manifestarse como derrame pleural libre o encapsulado. El derrame pleural encapsulado se
produce por acmulo de lquido en una zona de la pleura con amplias adherencias entre ambas hojas que impiden su despegamiento.
Puede simular una condensacin del parnquima o una tumoracin.
El derrame pleural libre es el que ocupa una cavidad pleural sin
adherencias o tabicaciones y se sita, por tanto, en la zona declive,
movilizndose con los cambios de posicin. Puede presentarse como
derrame pleural tpico. Aqu la distribucin del lquido con el paciente
en bipedestacin se acumula entre la superficie inferior del pulmn y el
diafragma, sobre todo en las zonas posteriores. Lo primero que se ocupa es el seno costofrnico, por ello se borra el seno en la Rx de trax. Es
el signo ms frecuente (resp. 5 correcta). A medida que aumenta la
cantidad de lquido, el lmite superior adopta la forma de una curva de
concavidad superior y vrtice externo. Esto se denomina lnea de
damoisseau, y es el signo ms caracterstico. Otra forma de presentacin es el derrame pleural subpulmonar. El acmulo de lquido simula
la elevacin del hemidiafragma. En el lado izquierdo se sospecha cuando la distancia entre el borde inferior del pulmn y la cmara gstrica es
superior a 2cms. Por ltimo, puede presentarse como un derrame atpico (en las cisuras), o un derrame masivo (opacificacin completa de un
hemitrax).

M exico A rgentina
C hile U ruguay

123245637859428
4 68

123245637859428
963649268

12343567
89


  343
3
23

3
 8 3
3
 8 3

Pregunta 59.- R: 4
Las manifestaciones pleuropulmonares en el lupus eritematoso
sistmico (LES) aparecen en alrededor del 50% de los pacientes en
algn momento de su evolucin. La afectacin pleural es el cuadro
ms frecuente (45% de los pacientes). Aunque es habitual que aparezca en fases iniciales de la enfermedad, slo en aproximadamente
el 2% de los casos es la forma de presentacin. Suele manifestarse
como dolor pleurtico acompaado de roce pleural a la auscultacin. Los brotes de pleuritis suelen durar pocos das o semanas,
remitiendo espontneamente o con tratamiento y tendiendo a la
recidiva. Suele ser bilateral, y cuando es unilateral, es de localizacin izquierda. Es un exudado rico en protenas, con pH alto (>7,35;
la respuesta 4 es incorrecta), glucosa similar a la srica (>60mg/dl) y
recuento celular con moderada leucocitosis de predominio mononuclear. Puede ser de carcter neutroflico si el derrame es reciente.
Podemos encontrar adems ttulos altos de ANA, bajos de complemento y clulas con fenmeno LE. El tratamiento es sencillo, con
buena respuesta y con escasa formacin de paquipleuritis. Se suele
comenzar con AINEs, y si no hay respuesta, se recurre a los
corticoesteroides. No es rara la asociacin de pleuritis y pericarditis
en mujeres jvenes, lo que sugiere en este grupo de enfermos la
etiologa lpica de la serositis.
Pregunta 60.- R: 3
Se ha comprobado que aproximadamente el 15% de los trasudados y el 40% de los exudados tienen aspecto serosanguinolento. Cuando esto ocurre, es importante la realizacin de un hematocrito para
diferenciar entre las distintas etiologas:
< 1% del hematocrito perifrico: suele estar en relacin con la
realizacin traumtica de la tcnica de toracocentesis. Si esto ocurre, la coloracin no es uniforme a lo largo de la extraccin y los
macrfagos no contienen inclusiones de hemoglobina.

CTO Medicina C/ Nez de Balboa, 115 28006 MADRID (Espaa) Tfno.: (91) 782 43 32 / Fax: (91) 782 43 27
E-mail: secretaria@ctomedicina.com; iberocto@ctomedicina.com WEB: www.ctomedicina.com; www.iberocto.com

NM Pg. 13

NEUMOLOGA Y CIRUGA TORCICA

Seguimiento a distancia

Preparacin Examen de Seleccin 05/06 1 Vuelta

> 50% del hematocrito perifrico: es una entidad especfica denominada hemotrax. Las causas ms frecuentes son las traumticas. El tratamiento es la colocacin de un tubo de drenaje para
cuantificar el sangrado.
1 - 50% del hematocrito perifrico: las causas ms frecuentes en
este grupo son los derrames pleurales de origen neoplsico, secundarios a un tromboembolismo pulmonar y los traumatismos
torcicos. En esta pregunta no estn las neoplasias, por lo que la
respuesta correcta ser el embolismo pulmonar (respuesta 3). Recuerda que el tromboembolismo pulmonar puede producir derrames pleurales exudados o trasudados, siendo ms frecuentes los
exudados asociados a pequeos infartos pulmonares.
Pregunta 61.- R: 5
Estamos ante un paciente con un derrame pleural con criterios de
exudado, ya que el cociente protenas LP/SP es mayor de 0,5. En
funcin del pH podemos determinar que, al ser superior a 7,2, es
poco probable que ocurra en derrames complicados. Es probable
que sea un lupus eritematoso y poco probable que sea un cncer,
una tuberculosis pleural, un empiema o una artritis reumatoide, donde habitualmente el pH es menor de 7,20. Por esto la respuesta 3
queda excluida. Lo mismo ocurre con las cifras de glucosa; son normales en el lupus y estn bajas en el resto. El predominio de linfocitos

en el recuento celular aparece, sin embargo, en todas las etiologas


descritas en las cuatro primeras respuestas y es poco probable en el
empiema, donde lo habitual es el predominio de neutrfilos. Adems, el empiema se define como presencia de pus en el lquido pleural.
Para ello se necesita que se complique un derrame pleural
paraneumnico, y esto va definido en funcin del pH. Si est entre 7
y 7,2, hay alto riesgo; si es < 7, se define como empiema. Con un pH
de 7,4, no es posible la existencia de un empiema.
Ver tabla en la pgina siguiente.
Pregunta 62.- R: 1
Es un tumor que deriva de las clulas mesoteliales que tapizan la
cavidad pleural. Es un tumor raro (la resp. 1 es incorrecta, son las
metstasis) que tiene una clara relacin con la exposicin a asbesto,
aunque tambin se ha asociado a otros factores concomitantes como
las radiaciones. El tiempo desde la primera exposicin al asbesto y la
presencia del tumor es variable, y puede oscilar entre los 20 a 40
aos. Dos tercios de los casos tienen edad entre 40 y 70 aos. La
clnica es muy insidiosa, con dolor torcico de caractersticas pleurticas y disnea. Posteriormente presentan prdida de peso, febrcula y
tos seca. Puede asociar hipoglucemia por consumo excesivo de glucosa por parte del tumor y osteopata hipertrfica. El hallazgo radiolgico ms frecuente es el derrame pleural, y se acompaa en 1% de

Pregunta 61. Caractersticas diferenciales de los derrames pleurales.


Bioqumica

Neumona

Exudado.
Bact = Pm n.
Virus: m ononucleares.
Cuanto m s inflam acin LDH y glucosa.
pH<7,2 riesgo evolucin em piem a.

Neoplasia

Hem tico, seroso o serohem tico.


Exudado.
Hto >1% y <50%.
Citologa lq. pleural.

Otras caractersticas

Tratamiento

Etiologa + frecuente: bacteriana


(neum ococo, BGN)

El de la neum ona

Es la causa m s frecuente de
derram e sanguinolento.
Ca pulm n > m am a >linfom a.
Sintom tica.

T EP

Exudado (75%).
Trasudado (25%) si existe fracaso hem odinm ico.
Hem tico, Hto >1% y <50%.

T BC

Exudado.
linfocitos.
ADA.
IFN (lo m s sensible y especfico).
Glucosa <60 m g/dl.
Ausencia de clulas m esoteliales.

< 35a .
Prim oinfeccin tuberculosa:
m a n to u x - .
Con frecuencia unilateral.
Diagnstico con biopsia pleural
ciega.

L ES

Exudado seroso.
Clulas lupus.
Glucosa norm al.
pH >7,2 (norm al).
C' .
ANA lquido pleural.

Bilat 50%.
Unilat 50%.
Con m ayor frecuencia el lado
izquierdo.

AR

Exudado.
pH <7,2.
Glucosa <30 m g/dl (si <15m g/dl
diagnstico confirm acin).
C' .
FR en lquido pleural.

5% casos.
U n il a t e r a l .
M s frecuente en lado derecho.
Varn con ndulos subcutneos y
ttulos altos de FR.

Rotura esofgica

Exudado.
am ilasa pancretica.

Clulas epiteliales, partculas de


a l im e n t o .
pH cido.

Pancreatitis

Exudado.
am ilasa pancretica.

Localizacin.
Lado izquierdo.
15-20% de las pancreatitis.

Pg. 14 NM

M exico A rgentina
C hile U ruguay

25-50% de los TEP

Paliativo/sintom tico=
evacuacin del derram e
El del TEP.
Peligro de transform ar el derram e
en hem otrax por el tratam iento
anticoagulante.
Tto TBC.
Si el paciente:
- < 35 a .
- No factores de riesgo de neoplasia.
- ADA >70.
Iniciar tto antituberculoso em prico.

Tto del LES (corticoides)

Tratam iento AR

Tto etiolgico
y
Tto de la m ediastinitis
Tto etiolgico

CTO Medicina C/ Nez de Balboa, 115 28006 MADRID (Espaa) Tfno.: (91) 782 43 32 / Fax: (91) 782 43 27
E-mail: secretaria@ctomedicina.com; iberocto@ctomedicina.com WEB: www.ctomedicina.com; www.iberocto.com

Comentarios TEST

Proceso

NEUMOLOGA Y CIRUGA TORCICA

Preparacin Examen de Seleccin 05/06 1 Vuelta


casos de placas pleurales. En el TAC se observa una pleura engrosada
e irregular. En el anlisis del lquido pleural encontramos un exudado
seroso o serosanguinolento con cifras de pH y glucosa disminuidas. A
mayor disminucin, peor pronstico. Adems suele ser un lquido
rico en cido hialurnico. El diagnstico suele requerir toracoscopia
e incluso toracotoma. La biopsia pleural cerrada no puede distinguir
entre mesotelioma y metstasis de adenocarcinoma. No es infrecuente que el diagnstico se haga en la autopsia. El tratamiento de los
mesoteliomas localizados es la ciruga. En los extensos se puede proponer la quimioterapia y radioterapia. De todas formas, la supervivencia no es muy prolongada (supervivencia media de 7 meses tras el
diagnstico), ya que cualquier forma de tratamiento es paliativo.

Comentarios TEST

Pregunta 63.- R: 2
El neumotrax se define por la presencia de aire en el espacio pleural,
con la consiguiente prdida de la presin subatmosfrica pleural y
colapso pulmonar parcial o total. Habitualmente el neumotrax se
clasifica segn su etiologa. Se distinguen los neumotrax traumticos,
iatrognicos y espontneos. A su vez, los neumotrax espontneos pueden ser primarios (en ausencia de enfermedad pulmonar previa) o
secundarios (en el curso de otra enfermedad pulmonar); siempre sin
traumatismo previo (la respuesta 2, por tanto, es incorrecta).
El neumotrax espontneo primario se produce preferentemente
en individuos jvenes, talla alta y delgados. No se ha demostrado su
relacin con el consumo de tabaco. Alguna teora establece como
causa la sobredistensin de los alveolos apicales del pulmn, especialmente en los pulmones muy alargados, que acaban por romperse
dando lugar a zonas localizadas de enfisema paraseptal (blebs
subpleurales). La rotura de estos blebs son la causa inmediata del
neumotrax.
El neumotrax traumtico es el que aparece en el contexto de un
traumatismo torcico, y el iatrgeno el debido a maniobras diagnsticas o teraputicas.
Pregunta 64.- R: 2
La parlisis diafragmtica es una patologa que cursa con un aspecto clnico muy amplio. Afecta al principal msculo implicado en la
inspiracin, y segn su afectacin, el enfermo puede estar desde completamente asintomtico hasta en insuficiencia respiratoria franca. Hay
dos tipos segn la localizacin de la lesin.
En la parlisis diafragmtica unilateral se afecta slo el nervio frnico de un lado. Se sospecha al encontrar un hemidiafragma elevado
en la Rx de trax, ya que habitualmente el enfermo est asintomtico
o paucisintomtico (la respuesta 2, por tanto, es incorrecta). La causa
ms frecuente es la infiltracin tumoral del nervio frnico (generalmente por extensin local de un carcinoma pulmonar). Otras causas
son el traumatismo quirrgico y la idioptica. Su diagnstico se confirma por radioscopia dinmica o fluoroscopia. La parlisis diafragmtica
bilateral puede ser secundaria a enfermedades neurodegenerativas y
a traumatismos cervicales o dorsales altos con compromiso medular.
Estos enfermos desarrollan un problema ventilatorio restrictivo de origen extratorcico, comportndose como sndromes de hipoventilacin graves. La mayora de los enfermos refiere intensa disnea que
empeora con el decbito. El tratamiento de eleccin es la ventilacin
mecnica, y si el nervio frnico est intacto se puede emplear marcapasos diafragmtico.
Pregunta 65.- R: 3
Con el nombre de hernia diafragmtica se entiende la profusin de
vsceras abdominales en la cavidad torcia a travs de una abertura
normal o patolgica del diafragma. Pueden ser de etiologa traumtica o atraumtica.
Las hernias traumticas aparecen en el contexto de traumatismos
con lesin diafragmtica y no son verdaderas hernias, ya que carecen
de saco herniario. La mayora tienen indicacin quirrgica.
Las hernias atraumticas son tres:
Hernia de hiato esofgico: una parte del estmago pasa a travs
del hiato esofgico del diafragma al trax. La pirosis y el dolor
torcico a nivel del apndice xifoides son los sntomas ms habituales. Casi siempre responden a tratamiento mdico.

M exico A rgentina
C hile U ruguay

Seguimiento a distancia

Hernia de Bochdalek o lumbocostal: es casi siempre una hernia


sin saco herniario. Se produce a travs de un defecto congnito
situado en la zona posterolateral del trax, sobre todo en el lado
izquierdo. La forma neonatal es la ms frecuente. Hay un paso del
contenido abdominal a la caja torcica y al nacimiento el RN tiene
un pulmn total o parcialmente colapsado. Segn la gravedad de
insuficiencia respiratoria siempre tienen indicacin quirrgica, y
en el momento actual se realiza incluso tratamiento intratero.
Hernia de Morgagni: es poco frecuente. La puerta herniaria se
encuentra en la hendidura situada entre la pared esternal y la parte
costal de la musculatura diafragmtica. Se localizan en la zona
anterior del trax, fundamentalmente en el lado derecho Ocurren
con mayor frecuencia en adultos obesos (la respuesta 3 es incorrecta). Tienen saco herniario y pueden contener grasa y/o vsceras
abdominales. Son quirrgicas cuando producen sntomas, se han
estrangulado o hay riesgo de estrangulacin.
Pregunta 66.- R: 3
Las masas mediastnicas se clasifican en funcin de su localizacin
en los distintos compartimientos mediastnicos. Segn donde se encuentren es ms orientativo de una u otra etiologa.
Mediastino anterior: aqu podemos encontrar una masa tmica, la
ms frecuente del compartimento. Asocia con frecuencia sndromes generales, como el sndrome de Cushing, la miastenia gravis o
la aplasia de la serie roja. Otras lesiones son las tiroideas, como el
bocio endotorcico, y los tumores germinales, como el seminoma
o el coriocarcinoma. El teratoma es un tumor de origen dermoide
que afecta con mayor frecuencia a adultos jvenes. Raramente son
sintomticos y generalmente su hallazgo es casual en un Rx de
trax, donde se presentan como masas bien definidas con calcificaciones en su interior. Este es el tumor al que hace referencia la
pregunta, por lo tanto la respuesta correcta es la 3.
El resto de los tumores de las otras cuatro respuestas se localizan en
mediastino posterior, a excepcin del linfoma, que puede aparecer en cualquier compartimento mediastnico.
Mediastino medio: los tumores ms frecuentes en esta localizacin son los quistes del desarrollo. Otras masas son derivadas de la
afectacin ganglionar (metstasis, tuberculosis, linfoma).
Mediastino posterior: el grupo principal de esta regin lo constituyen los tumores neurognicos como el neurofibroma, ganglioneuroma y paraganglioma. Con menos frecuencia aparecen lesiones
esofgicas, pseudoquistes pancreticos o hernia de Bochdalek.
Pregunta 67.- R: 5
El tratamiento del neumotrax viene definido fundamentalmente
en funcin de su cuantificacin y la presencia de compromiso respiratorio. En pacientes con un neumotrax < 20-30% y sin compromiso respiratorio, la actitud teraputica es la analgesia para controlar el
intenso dolor y la observacin. En pacientes con compromiso respiratorio y en pacientes con neumotrax >20-30%, la actitud teraputica
indicada es la colocacin de un tubo de drenaje endotorcico. Si no
hay reexpansin o a pesar de la reexpansin persiste la fuga area ms
de 7 das, es necesario recurrir al tratamiento quirrgico, al igual que
en las siguientes situaciones:
Fuga persistente ms de 7-10 das tras la colocacin de tubo de
drenaje endotorcico.
Segundo episodio de neumotrax.
Neumotrax bilateral simultneo.
Neumotrax contralateral a uno previo.
Complicaciones del neumotrax como hidroneumotrax, fstula
broncopleural, empiema, neumotrax a tensin, fracaso respiratorio o edema de reexpansin.
Enfermedad de base susceptible de tratamiento quirrgico como
quiste hidatdico, absceso neoplsico o bullas de gran tamao.
Situacin socioprofesional peculiar (pilotos, buceadores).
La pleurodesis qumica, generalmente realizada con talco o tetraciclinas, se utiliza en enfermos con pulmn ya expandido y en mal
estado, que por algn otro motivo no pueden tolerar la actitud
quirrgica.
Todas las respuestas son indicacin de ciruga, a excepcin del
neumotrax iatrgeno (respuesta 5). Cuando se produce un neumo-

CTO Medicina C/ Nez de Balboa, 115 28006 MADRID (Espaa) Tfno.: (91) 782 43 32 / Fax: (91) 782 43 27
E-mail: secretaria@ctomedicina.com; iberocto@ctomedicina.com WEB: www.ctomedicina.com; www.iberocto.com

NM Pg. 15

trax en el contexto de la realizacin de una tcnica diagnstica o


teraputica hay que valorar igualmente la repercusin respiratoria y
su cuantificacin para decidir si es susceptible de ciruga.
Pregunta 68.- R: 2
En esta pregunta se plantea la actitud ante un derrame pleural
neoplsico. La causa ms frecuente de derrame pleural maligno
son las metstasis de tumores extrapleurales. El cncer de pulmn,
de mama y los linfomas suponen el 75% de los casos, y hay un 6%
de casos en los que se desconoce el tumor primitivo. Desde un
punto de vista histolgico las metstasis ms frecuentes son las del
adenocarcinoma de cualquier origen. Es el segundo tipo de exudado ms frecuente (tras los derrames paraneumnicos). La forma de
presentacin ms tpica es un derrame pleural de cuanta importante con dolor torcico y disnea. El diagnstico se realiza por
citologa (entre el 50-80% de los casos), biopsia pleural ciega o
toracoscopia. El derrame pleural maligno indica enfermedad sistmica y se debe valorar la quimioterapia si el tumor primario es
sensible. Si produce sintomatologa respiratoria debe ser evacuado.
Inicialmente se realiza una toracocentesis evacuadora, y si recidiva,
hay que plantearse la pleurodesis qumica tras colocacin de drenaje, endotorcico (respuestas 1 y 3 incorrectas). Los tumores no
quimiosensibles slo son subsidiarios de tratamiento paliativo (respuesta 5 incorrecta). En los derrames pleurales malignos secundarios a tumor pulmonar, antes de colocar un tubo de drenaje endotorcico hay que ver si hay lesin endobronquial, ya que si la hay,
es imposible la reexpansin pulmonar y la resolucin del derrame.
En este caso es un tumor de mama diseminado; es posible tambin
que exista afectacin pulmonar endobronquial por metstasis, por
tanto habr que descartarla antes de colocar el tubo de drenaje endotorcico, como dice la respuesta 2.
Pregunta 69.- R: 4
Estamos ante un paciente con factores de riesgo para desarrollar
cncer de pulmn (es >35 aos y fumador) que presenta un sndrome constitucional en el contexto de un derrame pleural.
Las caractersticas bioqumicas del lquido pleural confirman que
es un exudado porque cumple uno de los criterios de Light (protenas
LP/SP >0,5). Esto descarta la respuesta 2, que habla de un derrame
pleural tipo trasudado.
El aspecto serohemtico nos obliga a hacer el diagnstico diferenciado entre derrame de origen neoplsico, tromboemblico y traumtico fundamentalmente. Lo ms frecuente es el derrame pleural
maligno, por lo que la respuesta 3 tambin es incorrecta.
El hallazgo en la biopsia pleural cerrada de ausencia de granulomas indica la poca probabilidad de que sea una tuberculosis, pero en
ningn caso la citologa negativa descarta la existencia de proceso
tumoral. La respuesta 1 es por tanto incorrecta, ya que no se puede
asumir que es infeccioso, tratarlo con antibioterapia y no seguir adelante en el algoritmo diagnstico. La respuesta 5 tambin es incorrecta
porque no concurre ninguna indicacin de tubo de drenaje. No hay
criterios de empiema (glucosa <50) ni repercusin respiratoria suficiente para indicar drenaje inmediato.
Estamos, por tanto, ante un derrame pleural exudado donde an
no se ha descartado el proceso tumoral. Hay que realizar la siguiente
prueba diagnstica que, como dice la respuesta correcta (la 4), es la
biopsia guiada por toracoscopia.
Pregunta 70.- R: 3
La presencia de derrame pleural 1 semana despus de que el paciente sea sometido a una ciruga de cavidad torcica debe hacer
pensar en rotura iatrgena del conducto torcico, sobre todo si se
tiene en cuenta la proximidad anatmica del campo quirrgico y
dicha estructura.
Adems el tiempo de evolucin hasta que aparece el derrame es
amplio (1 semana), lo que hace ms probable que sea secundario al
acmulo de linfa en el espacio pleural tras la rotura del conducto
torcico, y no a una lesin vascular donde el tiempo de presentacin
del hemotrax sera menor (respuesta 1 incorrecta).
Igualmente es poco probable que sea un derrame pleural maligno,
ya que debera tener un mayor tiempo de evolucin, no tiene por qu
Pg. 16 NM

M exico A rgentina
C hile U ruguay

NEUMOLOGA Y CIRUGA TORCICA

Preparacin Examen de Seleccin 05/06 1 Vuelta

estar en relacin con la ciruga y se hubieran visto los implantes pleurales durante la ciruga (respuesta 2 incorrecta). Tampoco es probable el
pseudoquilotrax porque stos son derrames pleurales de larga evolucin, secundarios fundamentalmente a artritis reumatoide y tuberculosis pleural (respuesta 4 incorrecta). Lo ms probable es, como decamos
al principio, el quilotrax, que se caracteriza por ser un lquido lechoso
con cifras altas de triglicridos y grasas totales.

Pregunta 70. Conducta a seguir ante un derrame pleural.

El tratamiento es la nutricin rica en triglicridos de cadena media


y pobre en el resto de grasas. Si es secundario a traumatismo, se colocar un tubo de drenaje endotorcico, y slo si no mejora, se proceder a la ciruga (respuesta 5 incorrecta).
Pregunta 71.- R: 5
La hipertensin pulmonar primaria (HPP) es una rara enfermedad
que predomina en mujeres entre la 2-4 dcada. Se asume el diagnstico cuando el paciente tiene una presin media en la arteria
pulmonar mayor de 25mmHg en reposo o mayor de 30mmHg en
ejercicio, con una presin de enclavamiento normal.
Para diagnosticar una hipertensin pulmonar como primaria hay
que haber excluido otras causas ms frecuentes de hipertensin pulmonar secundarias como la EPOC, el TEP de repeticin, la infeccin
VIH, la inhalacin de cocana, los frmacos estimuladores del apetito,
la hipertensin portal.
Hay varios tipos histolgicos:
Arteriopata pulmonar plexognica: es la forma ms frecuente.
Hay hipertrofia de la media, lesiones en la ntima y lesiones plexiformes que consisten en destruccin segmentaria de la media con
plexos de microvasos proliferantes. Las lesiones estn en arteriolas
y arterias de pequeo calibre.
Arteriopata pulmonar trombtica: los hallazgos son similares a la
anterior con signos de trombos recanalizados.
Enfermedad venoclusiva: representa aproximadamente el 10%
de los casos de hipertensin pulmonar primaria. Hay lesin de las
arteriolas y fundamentalmente de las vnulas donde se produce
una arterializacin de las venas y vnulas. Predomina en la infancia, y si aparece en adultos, lo hace con ms frecuencia en varones
(respuesta 5 falsa).
Hemangiomatosis capilar: es una enfermedad poco frecuente donde proliferan microvasos de paredes finas que infiltran el intersticio
y el parnquima.

CTO Medicina C/ Nez de Balboa, 115 28006 MADRID (Espaa) Tfno.: (91) 782 43 32 / Fax: (91) 782 43 27
E-mail: secretaria@ctomedicina.com; iberocto@ctomedicina.com WEB: www.ctomedicina.com; www.iberocto.com

Comentarios TEST

Seguimiento a distancia

NEUMOLOGA Y CIRUGA TORCICA

Preparacin Examen de Seleccin 05/06 1 Vuelta


Pregunta 72.- R: 3
La hipertensin pulmonar primaria es una enfermedad progresiva
para la que no hay curacin. La supervivencia media sin tratamiento
es inferior a 2,5 aos.

Comentarios TEST

Pregunta 72. Tratamiento de la hipertensin pulmonar primaria.

Hay 3 medidas fundamentales en el tratamiento:


Vasodilatadores: en el momento del diagnstico de la hipertensin pulmonar primaria se plantea la realizacin de un test de
vasorreactividad para predecir la respuesta del paciente al tratamiento con vasodilatadores va oral (antagonistas del calcio). Se
realiza con prostaciclina i.v. y se considera que la respuesta es
positiva si la presin media de la arteria pulmonar disminuye un
20%. Si esto ocurre, se administrarn vasodilatadores va oral. Estos
producen una mejora sostenida en un tercio de los pacientes (la
respuesta 3 es incorrecta). En los pacientes en los que no hay respuesta utilizaremos vasodilatadores de accin corta y potente como
la prostaciclina i.v.; siempre y cuando tengan un grado de ICC III IV de la NYHA.
Anticoagulantes orales: el tratamiento con estos frmacos aumenta
la supervivencia al prevenir la trombosis in situ. Estn indicados en
todos los pacientes con hipertensin pulmonar primaria.
Transplante pulmonar. Est indicado en pacientes con enfermedad muy avanzada con insuficiencia cardaca derecha a pesar del
tratamiento vasodilatador. No se ha descrito recidiva de la enfermedad en los rganos trasplantados.
Pregunta 73.- R: 4
El sntoma ms frecuente cuando hay un tromboembolismo pulmonar (TEP) es la disnea, y el signo ms frecuente la taquipnea.
Sin embargo hay que realizar una correcta anamnesis y exploracin fsica para identificar qu sntomas y signos acompaan con
mayor frecuencia a las distintas formas de presentacin del TEP.
El tromboembolismo de localizacin central en arterias pulmonares
proximales se caracteriza por producir importantes alteraciones en el
intercambio de gases. Tiene poco riesgo de infarto parenquimatoso, al
ser posible la vascularizacin del parnquima por circulacin colateral.
El TEP perifrico, en arterias segmentarias y subsegmentarias, produce pocas alteraciones del intercambio gaseoso, pero tiene un mayor riesgo de asociar infarto pulmonar. Cuando esto ocurre, aparece
generalmente dolor pleurtico, hemoptisis y febrcula.
Cuando hay un TEP muy importante pueden existir alteraciones
hemodinmicas asociadas. A la disnea sbita se suele aadir hipotensin, sncope o cianosis.
Las pruebas complementarias habitualmente son normales, aunque podemos encontrar en ocasiones datos que ayuden al diagnstico de TEP. En la Rx de trax puede existir derrame pleural, generalM exico A rgentina
C hile U ruguay

Seguimiento a distancia

mente asociado a una condensacin parenquimatosa triangular perifrica en la zona inferior. Es lo que se denomina "joroba de Hampton"
y aparece cuando hay infarto pulmonar asociado. La resp. 4 es incorrecta porque, aunque ste es un dato caracterstico, la forma de
presentacin radiolgica ms frecuente es la Rx de trax normal.
Pregunta 74.- R: 3
El diagnstico de TEP es complicado. Se basa en la combinacin
de una anamnesis metdica, una correcta exploracin fsica y una
serie de tcnicas invasivas y no invasivas.
La sospecha clnica es muy importante. Dada la gravedad del proceso, la presencia de clnica compatible, junto con la presencia de
factores de riesgo, es suficiente para iniciar tratamiento anticoagulante
(respuesta 1 correcta).
Las pruebas complementarias habituales no muestran generalmente
alteraciones. En ocasiones en la Rx de trax podemos encontrar elevacin de un hemidiafragma, oligohemia perifrica , atelectasias
laminares o derrame pleural (respuesta 2 correcta). En el ECG las
alteraciones ms comunes son la taquicardia sinusal y las anomalas
inespecficas del ST. En la GAB suele encontrarse hipoxemia con
hipocapnia.
La gammagrafa de perfusin es el test de screening ms til para
descartar un TEP clnicamente importante, pero en ocasiones es necesario realizar una angiografa pulmonar. Es la tcnica gold standard,
y puede precisarse cuando la sospecha clnica es muy alta y la gammagrafa y el resto de tcnicas diagnsticas no son concluyentes (respuesta 5 correcta).
Cuando tenemos sospecha clnica de TEP, si se demuestra la existencia de TVP se puede asumir el diagnstico de TVP + TEP (enfermedad tromboemblica). En el estudio de la TVP se utiliza fundamentalmente la ultrasonografa venosa, ya que detecta enfermedad tanto en
el sistema venoso profundo proximal como distal (la respuesta 4 es
correcta).
Otras tcnicas diagnsticas son la pletismografa de impedancia,
que detecta trombos en el sistema venoso proximal (popltea o superior) y la flebografa isotpica con fibringeno marcado, que detecta
trombos en el sistema venoso distal (pantorrilla). Por tanto, la respuesta
incorrecta es la 3.
Pregunta 75.- R: 4
Una vez diagnosticado el paciente de TEP, el tratamiento de eleccin, si no hay contraindicaciones, es la anticoagulacin. En el momento inicial, el tratamiento debe realizarse con heparina. Este frmaco acelera la accin de la antitrombina III, por lo que adems de
prevenir la formacin de nuevos trombos, permite que la fibrinlisis
endgena disuelva parcialmente el trombo ya formado.
La heparina sdica intravenosa se utiliza la mayora de las veces
como anticoagulante de eleccin (respuesta 1 correcta). Requiere
monitorizacin de los tiempos de coagulacin para controlar su accin. El efecto deseado se consigue cuando el tiempo parcial de tromboplastina activada (TTPa) se establece alrededor del doble del tiempo control (respuesta 2 correcta).
Las heparinas de bajo peso molecular han demostrado ser tan
eficaces y seguras como la heparina intravenosa, por lo que en el
momento actual se utilizan de forma habitual en el tratamiento inicial
del TEP (respuesta 3 correcta). Se administran por va subcutnea y no
requieren monitorizacin del TTPa, ya que su respuesta es predecible
en funcin de la dosis.
Son frmacos bastante seguros, aunque pueden asociar efectos
secundarios como la osteopenia, plaquetopenia o hemorragia. La
hemorragia es una complicacin poco frecuente que suele aparecer
si hay otros factores sobreaadidos como trombopenia o alteraciones
de la coagulacin (respuesta 4 incorrecta).
Se ha puesto de manifiesto que las complicaciones hemorrgicas
aparecen con menos frecuencia en las heparinas de bajo peso molecular en la heparina sdica intravenosa.
Ver esquema en la pgina siguiente.
Pregunta 76.- R: 3
Estamos ante un paciente que, tras ser sometido a una ciruga de larga
duracin, comienza con clnica muy sugestiva de embolismo pulmonar
que se confirma en una gammagrafa de ventilacin perfusin.

CTO Medicina C/ Nez de Balboa, 115 28006 MADRID (Espaa) Tfno.: (91) 782 43 32 / Fax: (91) 782 43 27
E-mail: secretaria@ctomedicina.com; iberocto@ctomedicina.com WEB: www.ctomedicina.com; www.iberocto.com

NM Pg. 17

NEUMOLOGA Y CIRUGA TORCICA

Preparacin Examen de Seleccin 05/06 1 Vuelta

Pregunta 75. Algoritmo diagnstico- teraputico del TEP

Pg. 18 NM

M exico A rgentina
C hile U ruguay

CTO Medicina C/ Nez de Balboa, 115 28006 MADRID (Espaa) Tfno.: (91) 782 43 32 / Fax: (91) 782 43 27
E-mail: secretaria@ctomedicina.com; iberocto@ctomedicina.com WEB: www.ctomedicina.com; www.iberocto.com

Comentarios TEST

Seguimiento a distancia

NEUMOLOGA Y CIRUGA TORCICA

Preparacin Examen de Seleccin 05/06 1 Vuelta


La presencia de factores de riesgo (ciruga prolongada) ms clnica
sugestiva (disnea brusca con hipoxemia) y el hallazgo de alta probabilidad de TEP nos permite hacer un diagnstico de certeza en este paciente.
El estudio de las extremidades inferiores para valorar el origen del embolismo
pulmonar es recomendable, pero no es necesario realizar tcnicas
invasivas. Es suficiente con un Eco doppler (resp. 1 incorrecta).
El tratamiento anticoagulante descrito en la respuesta 2 es correcto,
pero en este caso el paciente tiene una contraindicacin de anticoagulacin al estar recin intervenido de una tumoracin en el sistema nervioso central. El tratamiento anticoagulante que aparece en la respuesta
5 es poco probable porque la duracin habitual de la anticoagulacin
suele ser 6 meses. Aunque en este caso da igual, puesto que la
anticoagulacin est contraindicada. Son contraindicaciones absolutas de anticoagulacin las ditesis y procesos hemorrgicos, la hipertensin arterial severa, los aneurismas y hemorragias intracraneales y las
cirugas recientes del sistema nervioso central y ocular. El embarazo es
contraindicacin absoluta slo de anticoagulacin oral.
La terapia tromboltica a la que se hace referencia en la respuesta 4
slo est indicada en el TEP y TVP masivos.
La mejor alternativa para este paciente es el filtro de cava (resp. 3).

Comentarios TEST

Pregunta 77.- R: 5
Segn la intensidad y frecuencia de la exposicin a polvos inorgnicos se describen 3 formas clnicas de alveolitis alrgica o neumonitis
por hipersensibilidad. La reaccin inmunolgica ms relevante es el
dao tisular mediado por inmunocomplejos (reaccin tipo III).
Forma aguda: se produce ante exposiciones breves e intermitentes a dosis bajas de antgeno. Su clnica puede confundirse con
una neumona infecciosa (respuesta 4 correcta). A las pocas horas de exposicin el paciente presenta fiebre, escalofros, disnea,
tos seca, mialgias y malestar general. En la exploracin fsica hay
crepitantes en ambos pulmones, taquipnea y cianosis. Los sntomas suelen cesar tras la exposicin en menos de 24 horas, aunque a veces pueden durar 2-3 das. Radiolgicamente se caracteriza por infiltrados nodulares mal definidos localizados en lbulos inferiores (respuestas 1, 2, 3, correctas; respuesta 5 incorrecta).
Forma subaguda: es similar a la anterior, con un comienzo ms
insidioso y una sintomatologa menos florida. La Rx trax es igual
que en la forma aguda.
Forma crnica: aparece ante exposiciones prolongadas a dosis
bajas de antgenos. Presenta durante meses o aos sntomas como
disnea progresiva, tos seca, prdida de peso. No es infrecuente la
presencia de COR PULMONALE. La clnica es parecida a la bronquitis crnica. En la radiografa de trax se encuentran los hallazgos definidos en la respuesta 5. Es muy raro que revierta.

Seguimiento a distancia

Pregunta 78.- R: 1
La inhalacin y depsito de polvos inorgnicos provoca reacciones tisulares que con el paso del tiempo conllevan un dao pulmonar importante.
La exposicin a asbesto puede producir distintas manifestaciones:
Asbestosis: produce una fibrosis pulmonar similar a la fibrosis pulmonar idioptica, es decir, una enfermedad intersticial de predominio bibasal. Suele aparecer por encima de 10 aos de exposicin y
su curso es variable (respuesta 4 correcta). Las placas pleurales en la
Rx trax slo indican exposicin a asbesto, pero no asbestosis. Se
localizan preferentemente en pleura diafragmtica, y aunque cuando aparecen en el contesto de una enfermedad intersticial son muy
orientativas de asbestosis, no son, como dice la respuesta 1, que es
incorrecta, especficas de enfermedad por asbesto. Pueden aparecer en otras patologas como la tuberculosis pleural.
Cncer de pulmn: aumenta el riesgo a partir de 15 aos de exposicin. Dicho riesgo aumenta si el paciente es fumador, puesto que
el asbesto y el tabaco son cocarcingenos. La histologa ms frecuente es el carcinoma epidermoide y el adenocarcinoma.
Mesotelioma maligno: aumenta el riesgo por encima de 20-30
aos de exposicin.
Derrame pleural benigno y atelectasia redonda: son complicaciones que pueden aparecer en cualquier momento de la evolucin de la exposicin a asbesto por encima de los 10 aos.
Pregunta 79.- R: 3
Se diferencian cuatro formas de silicosis segn la intensidad de la
exposicin, el periodo de latencia y la historia natural. Cada una de
ellas tiene un patrn radiolgico caracterstico, que es a lo que hace
referencia esta pregunta. La silicosis simple o clsica suele aparecer
tras 15 aos o ms de la exposicin. Radiolgicamente presenta lesiones micronodulares menores de 1 cm, de predominio en lbulos
superiores, como dice la respuesta 1. Pueden existir tambin adenopatas hiliares calcificadas en "cscara de huevo". Este hallazgo es frecuente y caracterstico, pero no patognomnico de esta enfermedad
(la respuesta 5 es correcta). La silicosis acelerada es similar a la simple,
pero aparece con exposiciones ms intensas y tiene un tiempo de
latencia menor, entre 5 y 10 aos. Radiolgicamente presenta las
mismas alteraciones que la simple; por tanto, la respuesta 3 es la
incorrecta, ya que las lesiones predominan en lbulos superiores y no
en medios como se dice en ella. La silicosis complicada ocurre como
evolucin de otras formas de silicosis. La imagen radiolgica tpica es
la fibrosis masiva progresiva (conglomerados silicticos mayores de
1cm localizados en lbulos superiores). Hay compromiso clnico y de
funcin respiratoria importante, y el riesgo de tuberculosis est aumentado (respuesta 2 correcta). Por ltimo, la silicosis aguda es conse-

Pregunta 79. Patrones radiolgicos de afectacin en la silicosis.


M exico A rgentina
C hile U ruguay

CTO Medicina C/ Nez de Balboa, 115 28006 MADRID (Espaa) Tfno.: (91) 782 43 32 / Fax: (91) 782 43 27
E-mail: secretaria@ctomedicina.com; iberocto@ctomedicina.com WEB: www.ctomedicina.com; www.iberocto.com

NM Pg. 19

cuencia de una exposicin muy intensa. La radiografa de trax muestra un patrn miliar o patrn alveolar difuso que predomina en lbulos inferiores. La respuesta 4 tambin es correcta.
Pregunta 80.- R: 3
La fibrosis pulmonar idioptica (FPI) es una enfermedad intersticial que aparece generalmente entre los 50 y 70 aos, aunque puede aparecer a cualquier edad. Inicialmente el paciente comienza
con clnica tpica de tos seca y disnea al ejercicio. A medida que la
enfermedad progresa la disnea se hace ms intensa. En la exploracin fsica lo caracterstico es la aparicin de acropaquias y los crepitantes teleinspiratorios bibasales. En fases avanzadas pueden aparecer sntomas de hipertensin pulmonar secundaria y cor pulmonale. En el estudio de funcin pulmonar inicialmente no hay hallazgos patolgicos, pero a medida que progresa la enfermedad aparece
un patrn restrictivo (disminucin de la CPT, la CV, el VR y el FEV1)
con descenso de la DLCO. A veces este parmetro se ve afectado
antes que los datos de la espirometra. En la Rx de trax aparece,
como dice la respuesta 3, un patrn reticular o reticulonodular de
predominio bibasal. Sin embargo no hay, como tambin dice la
respuesta 3, placas pleurales. Estas aparecen fundamentalmente con
la exposicin a asbesto. Por tanto, la respuesta 3 es la incorrecta.
Recuerda que la asbestosis es casi igual a la FPI, salvo por los antecedentes de exposicin a asbesto, la presencia de placas pleurales y el
hallazgo de fibras de asbesto en material histolgico. La TC de alta
resolucin es muy til en el diagnstico precoz, aunque el diagnstico definitivo se hace siempre sobre una muestra histolgica (de
biopsia transbronquial en el 25% de los pacientes y de biopsia
toracotmica en el resto).
Pregunta 81.- R: 1
Las enfermedades del tejido conectivo son un grupo heterogneo
de trastornos inflamatorios de origen multifactorial mediados inmunolgicamente. Los pulmones se afectan frecuentemente, dada la
abundancia de tejido conectivo. Las manifestaciones pulmonares
pueden, en ocasiones, preceder o anunciar el comienzo de la expresin sistmica de estas enfermedades. Las enfermedades del colgeno
que hay que conocer son:
Artritis reumatoide: puede presentar varias manifestaciones. La
afectacin pleuropulmonar ms frecuente es la pleuritis. La afectacin intersticial aparece fundamentalmente en varones con ttulos
altos de factor reumatoide, y a veces precede a la enfermedad
articular (respuesta 3 correcta). Otras manifestaciones son los ndulos reumatoideos o la bronquiolitis obliterante.
Lupus eritematoso sistmico: tambin es la afectacin pleural ms
frecuente y aparece aproximadamente en el 50% de los casos. La
afectacin intersticial, por el contrario, es muy poco frecuente y
aparece en lupus de larga evolucin (respuesta 1 incorrecta). En
ocasiones puede aparecer en cuadro de hemorragia alveolar secundario al depsito pulmonar de inmunoglobulinas y complemento.
Puede alcanzar tasas de mortalidad de hasta el 80%.
Esclerodermia: la afectacin pulmonar es frecuente (alrededor
del 40%) y es la principal causa de mortalidad. Es la colagenosis
que con mayor frecuencia desarrolla enfermedad intersticial (respuesta 5 correcta).
Espondilitis anquilosante: es la nica que presenta afectacin en
vrtices, el resto la tiene en bases.
Pregunta 82.- R: 1
La eosinofilia pulmonar en un sndrome clnico que engloba a
diversos procesos que cursan con infiltrados pulmonares con eosinfilos y eosinofilia perifrica (a veces no aparece). Los infiltrados pulmonares suelen manifestarse radiolgicamente en forma de opacidades bilaterales de distribucin segmentaria, de aspecto no homogneo y carcter migratorio; salvo la neumona eosinfila crnica donde los infiltrados no son migratorios, y si recurren, lo hacen en la
misma localizacin (por tanto, la respuesta 1 es incorrecta).
Se clasifican en funcin de si se conoce o no su etiologa. Son de
etiologa conocida la ABPA, las neumonas por parsitos y las secundarias a frmacos. Es caracterstica la afectacin por nitrofurantona
que presenta en su forma aguda fiebre, escalofros, tos y disnea. Ra-

Pg. 20 NM

M exico A rgentina
C hile U ruguay

NEUMOLOGA Y CIRUGA TORCICA

Preparacin Examen de Seleccin 05/06 1 Vuelta

diolgicamente hay infiltrados bibasales, como dice la respuesta 3.


Son de etiologa desconocida la esosinofilia pulmonar simple (a veces
es secundaria a parsitos y est indicado el tratamiento con mebendazol, la respuesta 4 es correcta), la neumona eosinfila crnica, que es
caracterstica de mujeres de mediana edad con historia de rinitis,
atopia y asma (como dice la respuesta 2), y el sndrome hipereosinfilo, que como veis en la respuesta 5, afecta fundamentalmente a
varones de edad media. Hay una infiltracin difusa de rganos por
eosinfilos y el aparato respiratorio se afecta slo en la mitad de los
casos. La enfermedad cardaca es la principal causa de muerte. Sin
tratamiento, la supervivencia es menor de un ao, pero si tratamos
con corticoides en altas dosis, supera los 10 aos.
Pregunta 83.- R: 4
La Aspergilosis broncopulmonar alrgica (ABPA) es una eosinofilia
pulmonar de origen conocido. Est producida por el hongo Aspergillus fumigatus como consecuencia de una reaccin inmunolgica
tipo I y tipo III frente a antgenos del Aspergillus. Al depositarse el hongo
en el rbol traqueobronquial, se producen IgG e IgE especficos, as
como un aumento de la IgE total. Estos anticuerpos se unen a los
antgenos del hongo y forman inmunocomplejos circulantes que se
depositan en la pared bronquial dandola. Esto conlleva la aparicin de bronquiectasias de localizacin preferentemente central. A
todo esto se aade la presencia de infiltrados pulmonares por el
acmulo de clulas inflamatorias, principalmente eosinfilos (las respuestas 1 a 3 son correctas). Clnicamente se presenta de forma tpica
como asma de difcil control, disnea, sibilancias, febrcula y tos productiva con tapones marronceos.
Sin embargo, para el diagnstico existen unos criterios mayores y
menores, que son los siguientes:

Criterios mayores:
Asma.
Eosinofilia sangunea (>1000/mm3) (Resp. 5).
Precipitinas IgG contra Aspergillus.
Test cutneo de hipersensibilidad inmediata positivo para Aspergillus.
IgE total srica elevada.
Infiltrados pulmonares transitorios.
Bronquiectasias centrales.
IgG e IgE especficas elevadas.

Criterios menores:
Expectoracin marroncea.
Aspergillus en esputo (la respuesta 4 no es correcta, porque dice
que es un criterio mayor).
Reaccin cutnea retardada.
Pregunta 84.- R: 3
La sarcoidosis es una enfermedad multisistmica, de etiologa desconocida, que se caracteriza por la presencia de granulomas no
caseificados en los tejidos afectados. Prcticamente, cualquier rgano
puede verse involucrado, aunque son el pulmn y los ganglios linfticos torcicos los ms frecuentemente afectados (hasta el 90% de los
casos). Es rara la afectacin pleural y el derrame es excepcional (respuesta 5 incorrecta). Es una enfermedad de distribucin mundial ms
frecuente en la raza negra. El 80% de los casos ocurre en personas entre
20-40 aos de edad y es algo ms habitual en las mujeres. Por tanto, la
respuesta que buscamos es la 3. La respuesta cuatro es incorrecta, ya
que es ms frecuente en pacientes no fumadores donde no hay afectacin de la inmunidad pulmonar y se pueden perpetuar los procesos
inflamatorios que ocurren en esta enfermedad. Se describen dos
sndromes clnicos que se enuncian en las respuestas 1 y 2 cuyas manifestaciones no son las descritas en dichas respuestas. El sndrome de
Lfgren es un cuadro de fiebre, poliartralgias, eritema nodoso y
adenopatas hiliares bilaterales y simtricas. El sndrome de Heerfordt
presenta fiebre, uvetis anterior, parotiditis y parlisis facial.
Pregunta 85.- R: 1
Las manifestaciones extrapulmonares a las que hace referencia
esta pregunta son muy variadas y confieren gran pleomorfismo a la
enfermedad. Destacan por su frecuencia y relevancia:

CTO Medicina C/ Nez de Balboa, 115 28006 MADRID (Espaa) Tfno.: (91) 782 43 32 / Fax: (91) 782 43 27
E-mail: secretaria@ctomedicina.com; iberocto@ctomedicina.com WEB: www.ctomedicina.com; www.iberocto.com

Comentarios TEST

Seguimiento a distancia

NEUMOLOGA Y CIRUGA TORCICA

Comentarios TEST

Preparacin Examen de Seleccin 05/06 1 Vuelta

Seguimiento a distancia

Cutneas: la ms frecuente es el eritema nodoso y la ms especfica el lupus pernio.


Oculares: la afectacin ms frecuente es la uvetis anterior, aunque
se pueden presentar escleritis o queratitis.
Neurolgicas: aparecen con mayor frecuencia en las formas crnicas. Son mononeuritis (parlisis facial o multineuritis).
Articulares: generalmente son artritis errticas.
Hepticas: hay afectacin histolgica en el 60-90% de los pacientes, pero slo hay sntomas como hepatomegalia o alteraciones
enzimticas en 1/3 de los pacientes.
Cardacas: son poco frecuentes y generalmente asintomticas.
Renales: son muy raras.
Sistema endocrino metablico: el eje hipotlamo-hipofisario es
lo ms afectado (la respuesta 1 es correcta). Clnicamente se manifiesta como una diabetes inspida.
Glndulas exocrinas. Es caracterstica la afectacin bilateral de las
partidas, que slo da clnica en el 10% de los pacientes (respuestas 4 y 5 incorrectas).
Aparato reproductor. En el varn, lo ms frecuente es el aumento
del tamao testicular. Es rara la esterilidad. En la mujer, es caracterstica la mejora con el embarazo (respuestas 2 y 3 incorrectas).

Pregunta 87.- R: 4
La histiocitosis X es una enfermedad de etiologa desconocida incluida dentro del grupo de desrdenes proliferativos del sistema
mononuclear fagoctico. Es una afeccin poco frecuente, aunque no
excepcional, que incide fundamentalmente en varones fumadores
entre 20 y 40 aos, como dice la respuesta 2. Aunque la lesin predominante es la pulmonar, como enfermedad intersticial, puede afectar
tambin a huesos e hipfisis (respuesta 1 correcta). Se desconoce el
mecanismo patognico mediante el cual se produce una proliferacin y activacin de los macrfagos alveolares y las clulas de
Langerhans. Estas clulas son clulas muy diferenciadas que pertenecen a la serie monocito-macrfago y presentan de forma caracterstica grnulos intracitoplasmticos de Birbeck al estudio con microscopio electrnico. Estas clulas son caractersticas, pero no patognomnicas de la enfermedad (resp. 4 incorrecta), ya que tambin las encontramos aumentadas en los pacientes fumadores sin histiocitosis X.
La enfermedad intersticial se inicia en los bronquiolos ms pequeos,
donde se asocia un infiltrado inflamatorio. Es de distribucin broncovascular y se afectan tambin arteriolas y vnulas. Al avanzar el proceso aparece la fibrosis intersticial y los quistes areos, dato muy caracterstico de estos enfermos.

Pregunta 86.- R: 2
Las manifestaciones radiolgicas difieren de unos casos a otros y se
han utilizado para establecer distintas situaciones que se correlacionan con la severidad, pronstico y evolucin de la enfermedad. A
esto hacen referencia las cuatro primeras respuestas. Son correctas la
1, 3 y 4, mientras que es incorrecta la respuesta 2. La clasificacin
radiolgica es la siguiente:
Estadio 0: la radiologa es normal. Ocurre en menos de un 5% de
las formas subagudas.
Estadio I: se presenta fundamentalmente en las formas agudas.
Ocurre aproximadamente en un 75% de los pacientes. Es la forma
ms benigna por su escasa repercusin funcional y su tendencia a
la remisin espontnea. Slo se evidencian adenopatas hiliares y
mediastnicas bilaterales y simtricas. No se pone de manifiesto
afectacin pulmonar.
Estadio II: se manifiesta por la presencia de adenopatas y patrn
intersticial bilateral difuso. Se presenta tanto en las formas agudas
como crnicas. La tendencia a la regresin con tratamiento con
corticoides es ms lenta y menos constante.
Estadio III: generalmente aparece en formas crnicas. Se manifiesta como un patrn intersticial difuso con tendencia a la evolucin
fibrtica. No suelen existir adenopatas.
Estadio IV: fibrosis pulmonar difusa (pulmn en panal).

Pregunta 88.- R: 5
La linfangioleiomiomatosis es una rara enfermedad que se caracteriza por la proliferacin de clulas musculares lisas alrededor de las estructuras broncovasculares y el intersticio, unido a dilatacin qustica
de los espacios areos terminales que en el TAC de alta resolucin se
identifican como quistes de pared fina (respuestas 1 y 2 correctas).
Tambin hay afectacin linftica con dilatacin y tortuosidad de los
vasos linfticos, incluyendo el conducto torcico. Los linfticos pueden
verse daados tanto a nivel torcico como extratorcico, por lo que
podemos visualizar adenopatas en distintas regiones (respuesta 3 correcta). Los angiomiolipomas renales son otra manifestacin extratorcica que hay que estudiar (respuesta 4 correcta). El sntoma ms frecuente es la disnea. El neumotrax espontneo por ruptura de quistes
aparece en el 50% de los casos y el derrame pleural, que es raro al
diagnstico de la enfermedad, ocurre en 1/3 de los enfermos a lo largo
de su evolucin. Las caractersticas del derrame pleural son de un
quilotrax, ya que es secundario a la lesin del conducto torcico o los
vasos linfticos pleurales. Encontraremos, por tanto, ttulos altos de grasas totales y triglicridos con cifras normales de colesterol. Esto justifica
que la respuesta 5 sea incorrecta.

Pregunta 86. Estadiaje radiolgico de la sarcoidosis pulmonar.

El sndrome de Lfgren es una forma aguda de sarcoidosis que


presenta eritema nodoso, fiebre, adenopatas hiliares bilaterales y simtricas y poliartralgias (respuesta 5 correcta).
M exico A rgentina
C hile U ruguay

Pregunta 89.- R: 4
La granulomatosis de Wegener es una vasculitis necrosante granulomatosa de pequeo vaso que afecta tanto al tracto respiratorio superior como inferior. En ocasiones tambin afecta a arterias y a venas
(respuesta 1 correcta). La sinusitis es el sntoma de presentacin ms
frecuente. La afectacin pulmonar vara desde infiltrados asintomticos o acompaados de tos, disnea, dolor torcico y de forma caracterstica hemoptisis. En la Rx de trax es frecuente el hallazgo de infiltrados nodulares bilaterales y tpico la cavitacin de estos infiltrados.
Podemos encontrar tambin afectacin ocular, articular, cutnea y
con mayor frecuencia renal. Una asociacin frecuente es la
glomerulonefritis rpidamente progresiva. El diagnstico ms rentable
es la biopsia pulmonar, donde en la muestra histolgica encontramos
hallazgos de vasculitis y granulomas intra y extravasculares (respuesta
2 correcta). Sin embargo en la biopsia de otros rganos no encontramos todos los hallazgos histolgicos de esta vasculitis. Como dice la
respuesta 5, en la biopsia de senos paranasales hay granulomas pero
no vasculitis, y en la biopsia renal hay vasculitis, pero no granulomas.
La respuesta 4 es la incorrecta.
Pregunta 90.- R: 3
El sndrome de Goodpasture es un sndrome de etiologa desconocida que puede manifestarse a cualquier edad, aunque predomina
en varones jvenes. Se caracteriza por la trada de hemorragia alveolar, glomerulonefritis (que puede ser progresiva y conducir a la insuficiencia renal) y anticuerpos circulantes frente a antgenos de la membrana basal. El cuadro clnico inicial suele manifestarse con hemoptisis (80%), disnea (70%), tos y debilidad. Estos sntomas se acompaan

CTO Medicina C/ Nez de Balboa, 115 28006 MADRID (Espaa) Tfno.: (91) 782 43 32 / Fax: (91) 782 43 27
E-mail: secretaria@ctomedicina.com; iberocto@ctomedicina.com WEB: www.ctomedicina.com; www.iberocto.com

NM Pg. 21

NEUMOLOGA Y CIRUGA TORCICA

Preparacin Examen de Seleccin 05/06 1 Vuelta

Pregunta 90. Enfermedades intersticiales con afectacin radiolgica de lbulos superiores e inferiores.

Pg. 22 NM

M exico A rgentina
C hile U ruguay

CTO Medicina C/ Nez de Balboa, 115 28006 MADRID (Espaa) Tfno.: (91) 782 43 32 / Fax: (91) 782 43 27
E-mail: secretaria@ctomedicina.com; iberocto@ctomedicina.com WEB: www.ctomedicina.com; www.iberocto.com

Comentarios TEST

Seguimiento a distancia

NEUMOLOGA Y CIRUGA TORCICA

Preparacin Examen de Seleccin 05/06 1 Vuelta


en el 80% de los pacientes de infiltrados alveolares perihiliares. La
afectacin renal inicial consiste en proteinuria y microhematuria. La
insuficiencia renal manifiesta aparece posteriormente. Es caracterstico que curse en brotes. Es caracterstico encontrar la DLCO aumentada cuando hay hemorragia alveolar aguda (respuesta 1) y en la exploracin funcional el hallazgo ms frecuente es un patrn restrictivo.
Para el diagnstico, la biopsia renal es el proceder de eleccin, ya que
desde fases precoces el depsito de inmunoglobulinas es constante,
teniendo adems implicacin pronstica (respuestas 2 y 5 correctas).
La causa principal de muerte es la hemorragia pulmonar, que se trata
con corticoides intravenosos a altas dosis (respuesta 4 correcta). El
tratamiento de la enfermedad se hace con inmunosupresores y
plasmafresis, siendo esta ltima la que ha aumentado la tasa de supervivencia.
Ambos tratamientos y la hemodilisis son necesarios para controlar
la afectacin renal (respuesta 3 es incorrecta, porque los corticoides
solos no bastan para controlar la glomerulonefritis).
Ver esquema en la pgina 23.

Comentarios TEST

Pregunta 91.- R: 2
Las neumonas se pueden clasificar segn su forma de presentacin clnica en tpicas y atpicas. La neumona tpica se presenta de
forma brusca con fiebre alta, escalofros, tos con expectoracin purulenta y dolor torcico de caractersticas pleurticas. Es tambin frecuente la asociacin de herpes labial (respuesta 1 correcta).
En la exploracin, la semiologa es de condensacin (crepitantes,
aumento de las vibraciones vocales y soplo tubrico) y en la Rx de
trax el hallazgo suele ser un patrn alveolar con broncograma. A
veces se asocia derrame pleural. Los patgenos que de forma caracterstica se presentan as son las bacterias como el neumococo, el estafilococo, el Haemophilus o los gram negativos (respuestas 4 y 5 correctas).
Las neumonas atpicas tienen un curso ms larvado con predominio de sntomas generales, como dice la respuesta 3. Puede existir
febrcula y tos seca. En la exploracin fsica son raros los hallazgos
patolgicos. La respuesta 2 es incorrecta, ya que describe la semiologa de una condensacin. En la Rx de trax el hallazgo ms caracterstico es un infiltrado intersticial y es rara la presencia de derrame paraneumnico asociado (respuesta 5 correcta).
Patgenos que producen estas manifestaciones son el Mycoplasma,
la Coxiella y los virus.
Pregunta 92.- R: 4
La Legionella pneumophila es ubicua en ambientes hmedos y clidos, siendo el agua su hbitat natural. La infeccin de individuos
debilitados o inmunocomprometidos es consecuencia de la inhalacin de la bacteria en aerosol. Puede originar neumonas espordicas
o en brotes epidmicos. La confusin puede ser un rasgo prominente
y en la mitad de los pacientes aparece cefalea importante, confusin
y delirio. Entre las manifestaciones extrapulmonares podemos encontrar pericarditis, miocarditis, afectacin digestiva, afectacin renal y
encefalitis. Sospechamos Legionella ante todo paciente grave con neumona sin respuesta a betalactmicos, fiebre alta, hiponatremia, manifestaciones neurolgicas, alteraciones digestivas y LDH >700 u/ml. La
respuesta 1 es, por tanto, correcta. La Chlamydia, tanto la pneumoniae
como la psittaci, cursan como neumonas atpicas y pueden manifestarse como broncoespasmos con sibilancias en la auscultacin
pulmonar. En la C. psittaci hay que buscar tambin en la exploracin
esplenomegalia. Este germen es causante de la psitacosis y se relaciona
con la exposicin a pjaros (respuestas 2 y 3 correctas). La respuesta 4
es incorrecta porque, aunque es cierto que la Coxiella burnetti es una
rickettsiosis, es la nica que cursa sin exantema. Es el agente responsable de la Fiebre Q, enfermedad febril aguda que se acompaa de
neumona en casi la mitad de los pacientes. La transmisin al hombre
suele ser resultado de la inhalacin de polvo contaminado con
coxiellas procedentes de heces secas, orina, lecho o aerosoles en
mataderos. Como resiste mucho tiempo en el medio ambiente, a
veces no se conoce el antecedente epidemiolgico del contacto animal. Como dice la respuesta 5, la manifestacin extrapulmonar ms
frecuente es la hepatitis.

M exico A rgentina
C hile U ruguay

Seguimiento a distancia

Pregunta 93.- R: 5
La hidatidosis pulmonar es una antropozoonosis producida por el
desarrollo, en dicha vscera, de un gusano llamado Echinococcus. El
hombre se contagia cuando ingiere aguas o verduras contaminadas
por heces de crnidos que portan los huevecillos de este parsito.
Inicialmente la infeccin se localiza a nivel heptico, y desde all el
germen migra por las venas suprahepticas hasta la circulacin pulmonar. La clnica caracterstica en sus primeros estadios evolutivos es
la que presenta el paciente de nuestra pregunta, es decir, asintomtico. El diagnstico de sospecha es radiolgico (Rx trax, TC, y RMN) y
se confirma mediante serologa. El quiste ir progresando en sus distintos estadios evolutivos, siendo frecuentes las complicaciones. Estas son
ms probables a mayor evolucin de la lesin. Por este motivo, las
respuestas 1 y 2 son incorrectas. El tratamiento farmacolgico se utiliza generalmente asociado a ciruga. Tanto la biopsia mediada por
fibrobroncoscopia como la PAAF guiada por TAC estn contraindicadas, ya que el riesgo con estas tcnicas de rotura del quiste es muy alta.
Si esto ocurre, puede aparecer un sndrome sptico o una anafilaxia
grave. El tratamiento de eleccin es la exresis mediante toracotoma,
como dice la respuesta 5.
Pregunta 94.- R: 4
El cuadro clnico del paciente es compatible con una neumona
de origen bacteriano, una neumona tpica. El germen ms frecuentemente implicado es el neumococo. Las respuestas 1 y 3 son incorrectas porque no es un enfermo que cumpla criterios de gravedad para
neumona extrahospitalaria. Los criterios de gravedad son: taquipnea
importante (FR>30), insuficiencia respiratoria, signos de fatiga muscular, disminucin del nivel de conciencia, afectacin radiolgica bilateral y/o multilobar, shock sptico (TA< 90/60) y oliguria. La respuesta
2 es incorrecta, porque no nos dan ningn dato que nos oriente a que
el paciente tiene una neumona por Legionella como hiponatremia,
diarrea o sndrome confusional. La respuesta 5 tambin es incorrecta
porque, como decamos al principio, es una neumona tpica y la C.
psitacci se pone de manifiesto como una neumona atpica. La respuesta 4 es, por tanto, la respuesta correcta. Al ser de probable etiologa neumoccica, el tratamiento inicial en un paciente sin factores de
riesgo es la penicilina oral durante 7-10 das. Slo si la clnica no
mejora o hay datos de progresin radiolgica, valoraremos cambiar el
tipo de antibitico. Esto suele ocurrir en pacientes infectados por
neumococos resistentes a la penicilina, ms frecuentes en las otitis
peditricas que en las neumonas adquiridas en la comunidad.
Pregunta 95.- R: 4
En esta pregunta, lo que necesitamos conocer son los criterios de
ingreso en los pacientes con neumona adquirida en la comunidad.
Son los siguientes:
Edad superior a los 65 aos.
Enfermedades subyacentes crnico-debilitantes, principalmente
EPOC, diabetes mellitus, insuficiencia cardaca o renal crnicos,
alcoholismo, esplenectoma, malnutricin, cirrosis heptica, demencia y sospecha de aspiracin.
Cavitacin o derrame paraneumnico.
Recuento leucocitario <4000 o > 30000/l.
Hematocrito < 30%.
Sospecha de etiologa poco habitual, como S. aureus, bacilos gram
negativos o anaerobios.
Complicaciones spticas extrapulmonares, por ejemplo, meningitis, artritis o endocarditis.
Neumona grave (FR >30 rpm, insuficiencia respiratoria, fatiga
muscular, deterioro del nivel de conciencia, afectacin radiolgica bilateral y/o multilobar, TA < 90/60 y oliguria).
Falta de respuesta al tratamiento inicial, imposibilidad de ingesta
oral o dificultad para el manejo ambulatorio.
El paciente de la respuesta 1 tiene una cifra de leucocitos <4000/
ml; el de la respuesta 2 presenta comorbilidad asociada; el de la
respuesta 3 est hipotenso (TA < 90/60), y el de la respuesta 5 presenta
un fallo teraputico. El nico paciente sin criterios de ingreso es el
paciente de la respuesta 4.

CTO Medicina C/ Nez de Balboa, 115 28006 MADRID (Espaa) Tfno.: (91) 782 43 32 / Fax: (91) 782 43 27
E-mail: secretaria@ctomedicina.com; iberocto@ctomedicina.com WEB: www.ctomedicina.com; www.iberocto.com

NM Pg. 23

Pregunta 96.- R: 1
La hipoventilacin se produce cuando la PaCO2 aumenta por encima de los valores normales, es decir, hay hipercapnia.
Los sndromes de hipoventilacin crnica pueden producirse por
3 mecanismos: alteracin del impulso respiratorio, defectos en el sistema neuromuscular o alteracin del aparato ventilatorio.
Las alteraciones del impulso respiratorio se localizan en los quimiorreceptores centrales y perifricos y en las neuronas respiratorias
del tronco enceflico. En estos enfermos no hay respuesta de hiperventilacin a los estmulos qumicos y la Po.1 y el EMGdi estn alterados. Son capaces de hiperventilar de forma voluntaria y las presiones
mximas, volmenes, flujos, resistencia y distensibilidad son normales
(respuestas 2 y 3 correctas).
Los defectos en el sistema neuromuscular se localizan en la mdula
y nervios perifricos, y en los msculos respiratorios. La respuesta a los
estmulos qumicos, la Po. 1 y el EMGdi son patolgicos. En estos
enfermos es caracterstica la incapacidad para hiperventilar voluntariamente (respuesta 1 incorrecta) y la disminucin de las presiones
mximas (respuesta 4 correcta).
Los defectos en el aparato ventilatorio se localizan en la pared
torcica, en el pulmn y en las vas respiratorias. La respuesta ventilatoria a los estmulos qumicos est deteriorada, pero la Po.1 y el EMGdi
estn conservados. Lo tpico en estos defectos es la disminucin de los
volmenes y flujos, el aumento de las resistencias, la disminucin de
la distensibilidad y el aumento del gradiente alveolo-arterial de oxgeno.
Pregunta 97.- R: 5
Una apnea consiste en el cese completo del flujo inspiratorio superior a 10 segundos de duracin. Hay tres tipos de apnea: la obstructiva, la central y la mixta. En la apnea obstructiva hay una oclusin de
la va area superior a nivel de la orofaringe. Como consecuencia de
esto, el flujo cesa mantenindose el esfuerzo muscular ventilatorio, ya
que el impulso respiratorio continua. Son las apneas ms frecuentes.
En la apnea central, el flujo cesa al abolirse transitoriamente el impulso a los msculos respiratorios. No hay, por tanto, esfuerzo muscular.
Las apneas mixtas consisten en episodios centrales seguidos de un
componente obstructivo. Las apneas obstructivas casi siempre son
patolgicas, las centrales pueden aparecer de forma aislada en el
sueo REM de gente sana (respuestas 1, 2 y 4 correctas).
Una hipopnea consiste en el cese del flujo areo de al menos un 50%,
acompaado de una desaturacin de la hemoglobina de al menos 4%.
El ndice de apnea-hipopnea (IAH) es el nmero total de apneas e
hipopneas dividido por los minutos totales de sueo y multiplicado
por 60. Se considera patolgico un IAH >10 (respuesta 3 correcta).
El diagnstico definitivo para confirmar el SAOS es la polisomnografa, en la que se evidencian los episodios de cese de flujo areo con
mantenimiento del esfuerzo respiratorio con IAH mayor o igual a 10.
La oximetra domiciliaria se utiliza como prueba de screening (respuesta 5 incorrecta).
Pregunta 98.- R: 2
El sndrome de dificultad respiratoria del adulto (SDRA) se sospecha cuando aparece una insuficiencia respiratoria aguda de rpida
evolucin, junto con un patrn radiolgico con infiltrados alveolointersticiales difusos sin cardiomegalia. La ausencia de cardiomegalia, al igual que la presencia de una presin capilar de enclavamiento (PCP) normal, son muy importantes, ya que sitan el problema en
el pulmn. Si hay aumento de la PCP o cardiomegalia la afectacin
pulmonar es por hipertensin pulmonar postcapilar secundaria a
un problema cardaco en cavidades izquierdas. Por tanto, la respuesta 2 es incorrecta.
En el SDRA aparece un edema pulmonar por aumento de la permeabilidad de la membrana alveolocapilar cuando es lesionada. Esto
hace que el lquido intraalveolar sea muy rico en protenas e interfiera
con el surfactante, lo que conlleva el colapso alveolar. Se forman
entonces membranas hialinas en las zonas ms declives y queda solamente una pequea parte de parnquima sano que debe realizar
todo el trabajo. Estas alteraciones ventilacin/perfusin condicionan
la aparicin en fases avanzadas de la enfermedad de shunt
intrapulmonar secundario al colapso y atelectasias que aparecen. Si
Pg. 24 NM

M exico A rgentina
C hile U ruguay

NEUMOLOGA Y CIRUGA TORCICA

Preparacin Examen de Seleccin 05/06 1 Vuelta

la situacin es sostenida en el tiempo, aparece la fibrosis en 2-3 semanas. La sospecha clnica se confirma con criterios gasomtricos (PO2
< 55mmHg con FiO2 >50%; PaO2 <50mmHg con FiO2 > 60%) y
con la presencia de una compliance pulmonar disminuida (<50ml/
cm M2O).
La causa ms frecuente de este cuadro es la sepsis, y el tratamiento
consiste en solucionar el problema de base, oxigenoterapia y ventilacin mecnica.

Pregunta 98. Fisiopatologa del SDRA.

Pregunta 98. Diagnstico diferencial SDRA vs EAP

5789
9

8

3
66
9
8 

 7 879

1234

546

12342565789
39 95 54944
23 92556589

4 925889425
 789
93
 25259 79892

25238
3253982
2582595 1
3982

 7 824529 984 


294 "5 7 8245292894
734459589 5!28 55598!2894
"5128922
#82

18898595949
29 2 9
$%9  9822
&5 5945437
9 9'
())(

"5$%9  9822
"5&372 59587 2
"538492
"5&5(*&5+,--'
555.5#/
555.5# 88342 
"5  80
454 4

Pregunta 99.- R: 2
El secuestro broncopulmonar es una masa de tejido pulmonar
separada del parnquima sano, sin comunicacin con la va area y

CTO Medicina C/ Nez de Balboa, 115 28006 MADRID (Espaa) Tfno.: (91) 782 43 32 / Fax: (91) 782 43 27
E-mail: secretaria@ctomedicina.com; iberocto@ctomedicina.com WEB: www.ctomedicina.com; www.iberocto.com

Comentarios TEST

Seguimiento a distancia

NEUMOLOGA Y CIRUGA TORCICA

Preparacin Examen de Seleccin 05/06 1 Vuelta

Seguimiento a distancia

cuyo aporte sanguneo proviene de una arteria sistmica (lo ms frecuente es la aorta torcica o abdominal y en ocasiones una arteria
intercostal). De la definicin de esta malformacin pulmonar extraemos la informacin necesaria para decir que la respuesta falsa es la 2
y que la respuesta 1 es correcta.
Hay 2 tipos de secuestros: intralobar y extralobar. El secuestro
intralobar carece de pleura propia y suele localizarse en el ngulo
costofrnico posterior izquierdo. El secuestro extralobar tiene pleura
propia, est en relacin con el hemidiafragma izquierdo en ms de un
90% de los casos y su drenaje venoso es anmalo a la circulacin
sistmica, creando un cortocircuito izquierda - derecha (respuestas 3
y 4 correctas).
Suelen ser asintomticos hasta que se infectan. En este momento
cursan con clnica de hemoptisis y expectoracin purulenta.
El tratamiento siempre es quirrgico, aun cuando todava no hayan aparecido los sntomas, para evitar las complicaciones. En el estudio preoperatorio es necesario realizar una arteriografa, para ver de
donde proviene el aporte sanguneo, y un esofagograma, para descartar comunicacin con el esfago.

Comentarios TEST

Pregunta 100.- R: 3
Una de las complicaciones del trasplante pulmonar es el rechazo.
Hay dos tipos de rechazo: agudo y crnico. El rechazo agudo lo
presentan tres cuartas partes de los pacientes, generalmente en los dos
o tres primeros meses. Clnicamente se presenta como fiebre, tos, disnea, crepitantes..., simulando un proceso infeccioso. Hay afectacin
de la funcin pulmonar con hipoxemia y disminucin del FEV1. El
diagnstico se realiza con biopsia transbronquial (tiene una alta sensibilidad y especificidad) y la respuesta al tratamiento con corticoides
intravenosos es bueno (respuestas 1, 2, y 4 correctas). El rechazo crnico aparece en el 50% de los pacientes y cursa como una bronquiolitis obliterante (respuesta 5 correcta). El diagnstico es clnico (infeccin respiratoria de curso subagudo en pacientes con antecedentes
de rechazo agudo y cada importante y progresiva del FEV1), ya que la
biopsia transbronquial es poco sensible. El tratamiento consiste en
aumentar los inmunosupresores, aunque no siempre se consigue frenar la enfermedad. Es la principal causa de muerte tarda, pero no la
causa ms frecuente de muerte. La primera causa de mortalidad son
las infecciones (respuesta 3 incorrecta). La neumona ms frecuente es
la neumona por CMV. Aparece tras el primer mes postrasplante y
antes de los 6 meses. Clnicamente es indistinguible del rechazo agudo y se trata con ganciclovir. Otras causas de neumonas son el estafilococo, los bacilos gram negativos y el Aspergillus.

M exico A rgentina
C hile U ruguay

CTO Medicina C/ Nez de Balboa, 115 28006 MADRID (Espaa) Tfno.: (91) 782 43 32 / Fax: (91) 782 43 27
E-mail: secretaria@ctomedicina.com; iberocto@ctomedicina.com WEB: www.ctomedicina.com; www.iberocto.com

NM Pg. 25

NEUROLOGA

Preparacin Examen de Seleccin 05/06 1 Vuelta


SEMIOLOGA.
1.

Un paciente con lesin en el rea motora puede presentar


todas, EXCEPTO una:
1)
2)
3)
4)
5)

2.

Preguntas TEST

Cpsula interna izquierda.


Mesencfalo izquierdo.
Mesencfalo derecho.
Protuberancia izquierda.
Conducto auditivo interno izquierdo.

8.

9.

2)
3)
4)
5)

La lesin del tercer par craneal cursa con pupila midritica


arreactiva homolateral.
Una lesin de la arteria cartida externa puede cursar con
pupila mitica reactiva homolateral.
La presencia de pupilas puntiformes y reactivas puede
deberse a una intoxicacin por opiceos.
La presencia de pupilas arreactivas a la luz, pero que se
contraen al acomodarse a la visin cercana es sugerente
de lesin cordonal posterior.
La va aferente del reflejo fotomotor es el nervio ptico,
mientras que la eferente es el tercer par.

Un paciente acude a su mdico por haber observado descenso de su prpado derecho con disminucin de la hendidura
palpebral. La exploracin revela una pupila derecha mitica.
En el diagnstico diferencial incluir todos, EXCEPTO uno de
los siguientes:
M exico A rgentina
C hile U ruguay

Polineuropata de fibras finas.


Polirradiculopata.
Lesin centromedular.
Lesin cordonal anterior.
Lesin cordonal posterior.

Mielopata cervical.
Neuroles.
Degeneracin subaguda combinada de la mdula.
Sndrome espinal anterior.
Esclerosis lateral amiotrfica.

Seale la definicin correcta:


1)

2)
3)
4)
5)

11.

medular anterior.
medular posterolateral.
medular central.
de hemiseccin medular derecha.
de hemiseccin medular izquierda.

Un varn de 60 aos refiere, desde hace seis meses, dficit


motor progresivo en miembros inferiores, acompaado de
sensacin elctrica cuando flexiona el cuello, que se inicia a
nivel cervical y se irradia distalmente. A la exploracin se
objetivan amiotrofias en musculatura intrnseca de las manos, reflejos bicipital y estilorradial normales, hiperreflexia
en miembros inferiores y respuesta cutneo plantar extensora bilateral. Cul de los siguientes diagnsticos le parece ms
probable?:
1)
2)
3)
4)
5)

10.

Sndrome
Sndrome
Sndrome
Sndrome
Sndrome

La prdida de sensibilidad trmica y dolorosa bilateral entre los


niveles D3 y D6, con sensibilidad conservada por encima y
debajo de la lesin, nos debe hacer sospechar el diagnstico de:
1)
2)
3)
4)
5)

Corteza parietal izquierda.


Tlamo izquierdo.
Mesencfalo izquierdo.
Bulbo izquierdo.
Ganglio de Gasser izquierdo.

Con respecto a la exploracin de la pupila, seale el enunciado INCORRECTO:


1)

6.

Hemimdula derecha.
Bulbo medial derecho.
Protuberancia izquierda.
Cpsula interna derecha.
Cpsula interna izquierda.

Sndrome bulbar lateral o de Wallenberg.


Siringomielia cervical.
Tumor apical pulmonar.
Patologa carotdea.
Neuroles.

Paciente con disminucin de fuerza en miembro inferior


derecho, con disminucin de sensibilidad trmica en miembro inferior izquierdo y dficit de sensibilidad posicional en
miembro inferior derecho. Se trata de:
1)
2)
3)
4)
5)

Paciente con hipoestesia hemifacial izquierda y hemicorporal derecha. Seale la localizacin ms probable de su lesin:
1)
2)
3)
4)
5)

5.

7.

La exploracin de un paciente que consulta por diplopa


revela limitacin en la mirada hacia fuera del ojo izquierdo,
paresia hemifacial izquierda y disminucin de fuerza en
extremidades derechas. La lesin se sita en:
1)
2)
3)
4)
5)

4.

Ausencia de reflejos miotticos, en fases muy precoces.


Reflejos miotticos exaltados.
Ausencia de reflejos cutneo abdominales.
Respuesta cutneo plantar flexora en fases avanzadas del
cuadro.
Disminucin de la masa muscular del territorio afectado.

Un paciente presenta hemiparesia derecha que afecta a cara,


brazo y pierna. Seale, de entre las siguientes opciones, la
localizacin ms probable de la lesin:
1)
2)
3)
4)
5)

3.

1)
2)
3)
4)
5)

Seguimiento a distancia

Distonas: movimientos continuos, reptantes, que pueden


afectar a cualquier rea corporal y que suelen aumentar
si el paciente intenta realizar un movimiento voluntario o
hablar.
Mioclonas: movimientos repetitivos, irregulares y estereotipados que pueden afectar a diversos grupos musculares.
Corea: movimientos involuntarios de torsin relacionados con contracciones musculares mantenidas.
Temblor: movimiento oscilatorio rtmico debido a contracciones de tipo alternante de msculos agonistas y
antagonistas.
Atetosis: movimientos rpidos de amplitud variable debidos a contracciones simultneas de msculos agonistas y
antagonistas.

Varn de 72 aos que presenta a la exploracin tono muscular aumentado de forma generalizada, con resistencia con-

CTO Medicina C/ Nez de Balboa, 115 28006 MADRID (Espaa) Tfno.: (91) 782 43 32 / Fax: (91) 782 43 27
E-mail: secretaria@ctomedicina.com; iberocto@ctomedicina.com WEB: www.ctomedicina.com; www.iberocto.com

NR Pg. 1

NEUROLOGA

Preparacin Examen de Seleccin 05/06 1 Vuelta

tinua a la movilizacin pasiva y reflejos miotticos normales.


De todas las siguientes, seale cul le parece la definicin
correcta de la alteracin motora que presenta el paciente:
1)
2)
3)
4)
5)
12.

de reposo.
postural.
cintico.
intencional.
rbrico.

2)
3)
4)
5)

19.

2)
3)
4)
5)

Por tratarse de un temblor postural, insistira sobre sus


antecedentes familiares.
Realizara urgentemente una prueba de imagen para
descartar una lesin cerebelosa hemisfrica.
Pensara que probablemente en los prximos aos desarrollar una enfermedad de Parkinson.
Lo tranquilizara, ya que probablemente sean sus nervios.
Buscara signos que apunten a una ingesta alcohlica
importante.

Pg. 2 NR

Disartria.
M exico A rgentina
C hile U ruguay

temporal izquierdo.
temporal derecho.
parietal izquierdo.
parietal derecho.
ptica izquierda.

No reconocimiento de objetos a travs del tacto (astereognosia).


Negligencia del hemicuerpo contralateral en lesiones no
dominantes.
No discriminacin entre dos estmulos cutneos en hemicuerpo contralateral.
Anestesia dolorosa en el hemicuerpo contralateral.
Cuadrantanopsia homnima inferior contralateral.

Una mujer de 72 aos, hipertensa, presenta bruscamente un


trastorno del lenguaje, con habla incoherente, sustituyendo
una palabra por otra, siendo incapaz de repetir palabras que
se le dicen. Sin embargo, obedece rdenes verbales. La
localizacin ms probable de la lesin es:
1)
2)
3)
4)
5)

Lbulo frontal izquierdo.


Girus supramarginalis izquierdo.
Lbulo temporal izquierdo.
rea prefrontal izquierda.
En torno al rea de Wernicke.

CUADROS CONFUSIONALES. ENCEFALOPATAS METABLICAS. COMA.

De entre los siguientes sntomas y signos, seale cul NO


corresponde a una lesin cerebelosa:
1)

Lbulo
Lbulo
Lbulo
Lbulo
Cintilla

Cul de los siguientes datos clnicos NO es caracterstico de


la lesin del lbulo parietal?:
1)

21.

Prosopagnosia.
Astereognosia.
Simultanagnosia.
Hemianopsia homnima con respeto macular.
Apraxia ptica.

Seale dnde localizara la lesin ante un paciente con


cuadrantanopsia homnima superior izquierda:
1)
2)
3)
4)
5)

20.

Temblor esencial.
Lesin ocupante de espacio en fosa posterior.
Atrofia aislada de vermis cerebeloso.
Lesin cordonal posterior.
Lesin mesenceflica.

Seale cul de los siguientes sntomas NO corresponde a una


lesin de la corteza occipital:
1)
2)
3)
4)
5)

Hipoglucmica.
Hipercpnica.
Heptica.
Urmica.
Txica por anticomiciales.

Paciente de 55 aos que acude a su consulta porque en los


ltimos meses refiere un temblorcillo en la mano izquierda
cuando tiene la mano apoyada; si realiza cualquier movimiento,
este temblor desaparece. Cul sera su actitud?:
1)

16.

Temblor
Temblor
Temblor
Temblor
Temblor

Temblor intencional.
Temblor cintico.
Marcha atxica.
Hipertona.

Paciente que presenta de forma progresiva bipedestacin y


marcha inestable, con pasos desiguales, sin clara lateralizacin.
No presenta dismetras, disdiadococinesias ni oscilaciones en
extremidades. Seale el diagnstico ms probable:
1)
2)
3)
4)
5)

18.

Un movimiento involuntario, caracterizado por sacudidas


rpidas y arrtmicas secundarias a periodos silentes en la
actividad electromiogrfica, se puede encontrar asociado a
cualquiera de estas encefalopatas, SALVO:
1)
2)
3)
4)
5)

15.

Anticolinrgicos.
Benzodiacepinas.
Toxina botulnica.
Barbitricos.
Tenotoma.

Un paciente de 57 aos presenta oscilaciones en las manos


cuando realiza actividades como intentar enhebrar una
aguja o sujetar un vaso mientras bebe. No presenta movimientos involuntarios cuando no realiza actividad con las
manos. Seale cul de estos trminos le parece ms correcto
para definir este trastorno:
1)
2)
3)
4)
5)

14.

17.

Seale cul es el tratamiento de eleccin en las distonas


focales crvico-faciales:
1)
2)
3)
4)
5)

13.

Discinesia.
Distona.
Rigidez.
Espasticidad.
Paresia.

2)
3)
4)
5)

22.

Un paciente de 70 aos es trado a urgencias por sus familiares por presentar un cuadro de instauracin progresiva en

CTO Medicina C/ Nez de Balboa, 115 28006 MADRID (Espaa) Tfno.: (91) 782 43 32 / Fax: (91) 782 43 27
E-mail: secretaria@ctomedicina.com; iberocto@ctomedicina.com WEB: www.ctomedicina.com; www.iberocto.com

Preguntas TEST

Seguimiento a distancia

NEUROLOGA

Preparacin Examen de Seleccin 05/06 1 Vuelta


el curso de varias horas consistente en inatencin, desorientacin temporoespacial, incapacidad para retener informacin, lenguaje incoherente y alucinaciones. Seale la etiologa ms probable de este proceso:
1)
2)
3)
4)
5)
23.

24.

Preguntas TEST

1)

1)
2)
3)
4)
5)

3)

Encefalopata urmica.
Encefalopata heptica.
Lesin compresiva mesenceflica.
Lesin vascular protuberancial extensa.
Lesin bulbar.

La exploracin de un paciente en coma revela una postura en


extensin y adduccin de miembros inferiores, con extensin, adduccin y rotacin interna de miembros superiores,
que aparece con la estimulacin dolorosa. Cul de los
siguientes hallazgos es ms compatible con la situacin
clnica descrita?:
Patrn respiratorio rtmico con pausas de apnea y rachas
de hiperventilacin.
Reflejos oculoceflicos conservados.
Pupilas de tamao y reactividad normales.
Hematoma en fosa posterior.
Respuesta a rdenes verbales.

4)
5)
29.

4)
5)
30.

Lesin hemisfrica izquierda.


Lesin protuberancial derecha.
Lesin bulbar derecha.
Hipoglucemia severa.
Encefalopata hipxica.

1)
2)
3)
4)
5)

Lesin
Lesin
Lesin
Lesin
Lesin

bulbar.
pontina.
mesenceflica.
dienceflica.
hemisfrica bilateral.

Un paciente es trado al hospital en situacin clnica de coma.


A la exploracin se le objetiva patrn respiratorio de hiperventilacin rtmica y pupilas isocricas de tamao medio y reactividad normal. El diagnstico ms probable es:
1)
2)
3)

Lesin hemisfrica profunda.


Lesin mesenceflica.
Lesin pontina.
M exico A rgentina
C hile U ruguay

2)
3)
4)
5)
32.

Asterixis.
Temblor de reposo.
Edema de papila.
Cefalea.
Disminucin del nivel de conciencia.

Cul de los siguientes datos NO es caracterstico del coma


hipoglucmico?:
1)

Ante un paciente en coma que presenta una respiracin


taquipneica sin aparentes hallazgos en la auscultacin pulmonar, pensara en primer lugar:

Buena regeneracin posterior de las reas afectadas.


Su origen puede ser un infarto agudo de miocardio.
Afectar de forma precoz el crtex cerebeloso, los ganglios
basales y el rea hipocampal.
Causar daos tisulares cerebrales en 5 minutos tras el inicio
de la anoxia.
Aparicin de sntomas das despus de sufrir el proceso
isqumico.

Un paciente EPOC es trado a Urgencias por descompensacin respiratoria. Seale cul de los sntomas NO esperara
encontrar a la exploracin:
1)
2)
3)
4)
5)

31.

Debe realizarse TC craneal para descartar lesin dienceflica ocupante de espacio.


Debe realizarse RM cerebral centrada en fosa posterior
por ser sta la prueba de imagen de eleccin.
Debe tratrsele de forma inmediata con vitamina B1
parenteral.
Deben revisarse los hallazgos analticos al ingreso y la
sueroterapia pautada.
La causa ms probable es coma de origen conversivo.

La encefalopata anoxicoisqumica NO se caracteriza por:


1)
2)
3)

Un paciente en coma presenta desviacin tnica de la mirada


hacia la izquierda y disminucin de la movilidad en hemicuerpo izquierdo ante estmulos dolorosos. Como causa de su
coma sospechar:
1)
2)
3)
4)
5)

27.

Hematoma subdural subagudo.


Tumoracin hemisfrica no dominante.
Estatus parcial complejo.
Encefalitis herptica.
Trastorno metablico.

Lesin bulbar.
Coma acidtico.

Usted est valorando a un paciente que ingres hace 24 horas


por un cuadro confusional con lenguaje incoherente y ataxia.
A la exploracin se le objetiva estado alerta con incapacidad
para realizar cualquier movimiento voluntario, salvo parpadeo dbil y movimientos verticales oculares. Su opinin sera:

2)

2)
3)
4)
5)

26.

28.

NO se considera causa de estado comatoso:

1)

25.

4)
5)

Seguimiento a distancia

Aparicin de los primeros sntomas al cabo de minutos de


hipoglucemia severa mantenida.
La ingesta de antidiabticos orales puede ser causa de
coma hipoglucmico.
Puede cursar con hemiparesia, especialmente en ancianos.
Se presenta como una desconexin brusca del medio, sin
sntomas previos.
La sobredosis de insulina es una causa frecuente.

Un paciente es trado a Urgencias por presentar diplopa,


importante inestabilidad en la marcha y disminucin del nivel
de conciencia con lenguaje incoherente. Entre sus antecedentes
destaca alcoholismo. De las siguientes opciones, cul le parece
la medida teraputica inicial ms adecuada?:
1)
2)
3)

Observacin clnica sin tratamiento.


Puncin lumbar y tratamiento con aciclovir emprico.
Administracin de vitamina B 1 parenteral.

CTO Medicina C/ Nez de Balboa, 115 28006 MADRID (Espaa) Tfno.: (91) 782 43 32 / Fax: (91) 782 43 27
E-mail: secretaria@ctomedicina.com; iberocto@ctomedicina.com WEB: www.ctomedicina.com; www.iberocto.com

NR Pg. 3

4)
5)

Sueroterapia con sueros glucosados abundantes.


Administracin de neurolpticos parenterales.

AMNESIAS. DEMENCIAS.
33.

2)
3)
4)
5)

2)
3)
4)
5)

39.

Enfermedad de Binswanger.
Enfermedad de Huntington.
Enfermedad de Alzheimer.
Estadio lacunar.
Demencia multiinfarto.

2)
3)
4)
5)

3)
4)
5)

2)
3)
4)
5)

Cul de los siguientes es el frmaco ms til para la enfermedad de Alzheimer?:


1)
2)
3)
4)

Pg. 4 NR

Agonistas de receptores dopaminrgicos.


Inhibidores de la acetilcolinesterasa cerebral.
Inhibidores de la MAO-B.
Antioxidantes (idebenone).
M exico A rgentina
C hile U ruguay

Enfermedad de Gerstmann-Straussler-Scheinker.
Enfermedad de Creutzfeldt-Jakob.
Enfermedad de Strachan.
Insomnio familiar fatal.
Encefalopata espongiforme.

Sobre la enfermedad de Binswanger, seale la FALSA:


1)

Es la causa de demencia ms frecuente en Occidente.


La prdida neuronal es difusa, con predominio en lbulos
frontales y temporales.
El diagnstico de certeza es anatomopatolgico.
Las placas seniles y de degeneracin neurofibrilar son
patognomnicas.
Cursa con reduccin de neurotransmisores en SNC.

La mitad de los pacientes afectados muestran signos de


afectacin de primera motoneurona, como hiperreflexia
e hipertona elstica.
El deterioro severo se produce en la mayora de los pacientes antes de un ao desde el inicio de los sntomas.
Ms de la mitad de los pacientes suelen presentar algn
signo parkinsoniano.
El EEG no es til para el diagnstico.
Pueden existir antecedentes familiares en un reducido
nmero de casos.

Una de estas patologas NO est producida por priones:


1)
2)
3)
4)
5)

42.

Enfermedad de Pick.
Enfermedad de Huntington.
Enfermedad de Friedreich.
Enfermedad de Binswanger.
Enfermedad de Parkinson.

Paciente de 65 aos consulta por un cuadro progresivo de


olvidos frecuentes, trastorno de la marcha y sacudidas musculares breves y arrtmicas desencadenadas por sobresaltos.
Seale, en relacin con la patologa que probablemente sufre
el paciente, la respuesta INCORRECTA:
1)

41.

Deterioro del estado de alerta y atencin.


Deterioro de la memoria de fijacin.
Apraxias.
Agnosias.
Afasias.

Seale cul de las siguientes enfermedades cursa con demencia de caractersticas corticales:
1)
2)
3)
4)
5)

40.

Factores neurotrpicos.

Un paciente ha sido diagnosticado de demencia cortical.


Usted NO esperara encontrar en la exploracin:
1)
2)
3)
4)
5)

La enfermedad de Alzheimer se caracteriza por todos los


siguientes enunciados, EXCEPTO uno:
1)
2)

37.

38.

Iniciara tratamiento antiepilptico, por tratarse de una


crisis parcial compleja.
Tranquilizara a la familia, porque probablemente en menos
de 6 horas el cuadro remita sin ningn problema.
Pedira una RM para descartar una encefalitis herptica
temporal.
Recomendara una valoracin psiquitrica.
Iniciara antiagregacin con clopidogrel por tratarse de un
fenmeno isqumico cerebral.

El deterioro de funciones superiores caracterizado por deterioro de memoria, trastornos de concentracin y enlentecimiento en los procesos de pensamiento, con ausencia de
francas apraxias, agnosias o afasia, es caracterstico de todas,
MENOS una de estas patologas:
1)
2)
3)
4)
5)

36.

La memoria inmediata depende fundamentalmente del


nivel de atencin.
La amnesia global transitoria afecta bsicamente al recuerdo de datos recientes.
El tratamiento de los cuadros de amnesia se basa en el
bloqueo de los receptores gabargicos.
La amnesia antergrada es uno de los hallazgos caractersticos del sndrome de Korsakoff.
Una lesin bilateral extensa de lbulos temporales puede
cursar con un sndrome amnsico.

Acude a urgencias una paciente de 65 aos que de manera


brusca presenta prdida de la memoria reciente, preguntando continuamente "dnde estoy?". En la exploracin no se
objetiva ningn signo de focalidad neurolgica. Cul sera
su actitud?:
1)

35.

5)

Sobre los defectos de memoria, seale la opcin FALSA:


1)

34.

NEUROLOGA

Preparacin Examen de Seleccin 05/06 1 Vuelta

Aparece en pacientes hipertensos mal controlados de


larga evolucin.
Presenta leucoaraiosis en las pruebas de imagen cerebral,
que aunque no es patognomnica, resulta muy caracterstica de esta patologa.
Existen alteraciones en la marcha, con una marcha a
pequeos pasos.
Cursa con demencia subcortical.
El tratamiento antihipertensivo adecuado puede revertir
los sntomas.

EPILEPSIA.
43.

La diferencia entre crisis parciales simples y complejas es:


1)
2)

Presencia o no de manifestaciones psquicas.


Duracin de la crisis.

CTO Medicina C/ Nez de Balboa, 115 28006 MADRID (Espaa) Tfno.: (91) 782 43 32 / Fax: (91) 782 43 27
E-mail: secretaria@ctomedicina.com; iberocto@ctomedicina.com WEB: www.ctomedicina.com; www.iberocto.com

Preguntas TEST

Seguimiento a distancia

NEUROLOGA

Preparacin Examen de Seleccin 05/06 1 Vuelta


3)
4)
5)
44.

Un paciente de 25 aos ha comenzado a presentar episodios


repetidos de prdida de conciencia consistentes en sensacin epigstrica ascendente seguida de desconexin del
medio, mirada fija, automatismos deglutorios y falta de
respuesta a estmulos externos. Se sigue de corta confusin
posterior. La duracin de estos episodios es de 2-3 minutos.
Se trata probablemente de:
1)
2)
3)
4)
5)

45.

5)

Preguntas TEST

2)
3)
4)
5)
47.

La causa ms probable del caso es una lesin ocupante de


espacio a nivel cortical.
Est indicada la realizacin urgente de una TC craneal en
busca del diagnstico.
El origen ms probable de la crisis es un traumatismo
craneoenceflico de baja intensidad, que el paciente
haya olvidado.
El episodio tnico-clnico generalizado puede deberse a
una generalizacin a partir del foco hemisfrico izquierdo.
En el caso de tratarse de una lesin tumoral, lo ms
probable es que se trate de una metstasis.

2)
3)
4)
5)
49.

3)
4)
5)
50.

4)
5)

Parestesias hemicorporales.
Alucinaciones visuales en forma de luces centelleantes.
Elevacin del brazo con desviacin ceflica hacia ese
brazo.
Sensacin de bolo gstrico que asciende seguido de
alteracin del nivel de conciencia.
Parpadeo bilateral con mirada fija.

2)
3)
4)
5)

M exico A rgentina
C hile U ruguay

El tratamiento en monoterapia consigue el control de las


crisis en el 20% de los pacientes aproximadamente.
La carbamacepina es el frmaco de segunda eleccin en
las crisis de ausencia tpicas.
La etosuximida es el tratamiento de eleccin para las crisis
parciales complejas.
El fenobarbital en nios puede producir un sndrome de
hiperactividad.
Es habitual el tratamiento profilctico de las crisis febriles
a largo plazo con bajas dosis de valproico.

El tratamiento de eleccin en el estatus epilptico es:


1)
2)
3)
4)
5)

53.

Esperar a una segunda crisis para instaurar tratamiento.


Iniciar tratamiento con cido valproico.
Iniciar tratamiento con etosuximida.
Iniciar tratamiento con vigabatrina.
Iniciar tratamiento en caso de presentar antecedentes
familiares.

Es correcto, en relacin al tratamiento de la epilepsia:


1)

52.

Las tpicas duran escasos segundos.


Son frecuentes los antecedentes familiares de crisis febriles
o de epilepsia.
El EEG intercrtico muestra descargas de punta-onda lenta
sin repercusin clnica.
Precisan tratamiento anticomicial continuado hasta los 910 aos, edad a la que las crisis ceden espontneamente.
Se asocian con bajo rendimiento escolar.

Un paciente de 25 aos ha presentado una crisis comicial de


tipo tnico-clnico generalizado, sin claro factor desencadenante. La exploracin es normal; el EEG interictal y la RM
cerebral son normales. La actitud teraputica recomendada
sera:
1)
2)
3)
4)
5)

51.

Conservacin del desarrollo psicomotor a lo largo de la


enfermedad.
Existencia de patologa cerebral subyacente en la mayora
de los casos.
Crisis generalizadas mioclnicas.
Hipsarritmia durante las crisis.
Buen control teraputico con fenitona.

Seale la respuesta correcta con respecto a las crisis febriles


infantiles:
1)
2)

Seale cul de las siguientes es un patrn caracterstico de


crisis originadas en el lbulo frontal:
1)
2)
3)

En el sndrome de West es caracterstico:


1)

Se trata de una crisis clnica secundariamente generalizada.


Se trata de una crisis simple motora.
Es una crisis parcial compleja.
El foco epilptico parece localizarse en el lbulo parietal
izquierdo, junto a la hoz.
Supone un signo inequvoco de recidiva tumoral.

Un varn de 45 aos comienza de modo brusco con movimientos tnicos en miembro superior derecho, de pocos
segundos de duracin, que en una ocasin son seguidos de
prdida brusca del nivel de conciencia junto con movimientos tnico-clnicos generalizados. Refiere no haber sufrido
nunca accidentes o contusiones de importancia. Sobre el
caso, es INCORRECTO:
1)

48.

Crisis generalizada no convulsiva.


Crisis parcial simple.
Crisis parcial compleja.
Crisis de ausencia.
No presenta semiologa de crisis comicial.

Un paciente de 70 aos, con antecedente de meningioma


operado hace un ao, refiere desde hace 2 semanas episodios
repetidos consistentes en movimientos clnicos involuntarios
que comienzan en la mano derecha y progresan por miembro
superior hasta afectar a todo el hemicuerpo derecho. Duran
unos 3 minutos y ceden sin secuelas. No hay deterioro de
conciencia asociado. Podemos afirmar que:
1)
2)
3)
4)

46.

Conservacin de la conciencia y percepcin del ambiente.


Foco en lbulo temporal.
Etiologa subyacente.

Seguimiento a distancia

Primero clonacepam i.v. y si no cede, valproato sdico i.v.


Primero diacepam i.v. y si no cede, difenilhidantona i.v.
Primero fenobarbital i.v.; si no cede, fenitona i.v.
Primero valproato i.v. y si no cede, fenitona i.v.
Fenobarbital i.v. y si no cede, penthotal sdico i.v.

Una mujer de 30 aos, diagnosticada de epilepsia generalizada


tnico-clnica desde los 20, consulta por estar embarazada de
4 semanas. Se encuentra en tratamiento con carbamacepina
desde los 20 aos y no ha presentado crisis desde los 25 aos
de edad. La actitud teraputica recomendada sera:
1)

Suspender progresivamente el tratamiento.

CTO Medicina C/ Nez de Balboa, 115 28006 MADRID (Espaa) Tfno.: (91) 782 43 32 / Fax: (91) 782 43 27
E-mail: secretaria@ctomedicina.com; iberocto@ctomedicina.com WEB: www.ctomedicina.com; www.iberocto.com

NR Pg. 5

2)
3)
4)
5)
54.

5)

1)
2)
3)
4)

Mantener el tratamiento a la dosis actual y vigilar los niveles


plasmticos del frmaco.
Aadir cido valproico, por ser ste el frmaco de eleccin
para su tipo de epilepsia.
Aumentar la dosis de carbamacepina, por ser habitual una
cada de los niveles plasmticos del frmaco durante el
embarazo.
Se recomienda aborto teraputico.

Seale cul de los siguientes efectos adversos NO es caracterstico de cada frmaco:


1)
2)
3)
4)

NEUROLOGA

Preparacin Examen de Seleccin 05/06 1 Vuelta

5)
60.

Fenitona: hiperplasia gingival.


Valproato: cada de pelo.
Fenobarbital: sedacin.
Carbamacepina: reduccin concntrica del campo visual.
Felbamato: aplasia medular.

Con respecto al tratamiento para modificar el curso de la


enfermedad en la esclerosis mltiple, seale la respuesta
INCORRECTA:
1)
2)
3)
4)

ENFERMEDADES DESMIELINIZANTES.
55.

Seale cul de los siguientes sntomas es el ms frecuente


como debut de una esclerosis mltiple:
1)
2)
3)
4)
5)

56.

57.

5)

Lesin fascicular del tercer par derecho.


Lesin nuclear del tercer par derecho.
Lesin del fascculo longitudinal medial derecho.
Lesin del fascculo longitudinal medial izquierdo.
Oftalmoplejia internuclear bilateral.

Bandas oligoclonales en el LCR.


Aumento del ndice de IgG en el LCR.
Enlentecimiento de los potenciales evocados visuales.
RM cerebral normal.
Lesiones periventriculares en la RM cerebral.

3)
4)

Sobre el tratamiento de la esclerosis mltiple, seale la


respuesta correcta:

Pg. 6 NR

M exico A rgentina
C hile U ruguay

5)
63.

Es una enfermedad de curso monofsico.


Es habitual que las lesiones desmielinizantes presenten
captacin de contraste.
Clsicamente se asocia con la infeccin por el virus del
sarampin.
En pases desarrollados, la varicela es el agente etiolgico
ms frecuentemente asociado.
El interfern beta es til para el control sintomtico de la
enfermedad.

Usted es consultado para valorar a un nio de 10 aos que


presenta retraso escolar, crisis generalizadas y ataxia progresiva. En las exploraciones complementarias destaca un EEG
con descargas de ondas lentas de alto voltaje seguidas de
perodo de aplanamiento de la actividad cerebral. En la RM
cerebral realizada se aprecian mltiples lesiones en sustancia
blanca y moderada atrofia cortical. En relacin con la enfermedad que probablemente presenta, seale la respuesta
correcta:
1)
2)

Depresin.
Crisis epilpticas.
Neuralgia del trigmino.
Movimientos coreoatetsicos involuntarios.
Signo de Lhermitte.

Seale cul de los siguientes hallazgos en las pruebas complementarias es MENOS sugerente de esclerosis mltiple:
1)
2)
3)
4)
5)

59.

4)

Seale cul de los siguientes sntomas es ms frecuente en el


curso de una esclerosis mltiple:
1)
2)
3)
4)
5)

58.

3)

62.

No hay tratamiento especfico para las formas progresivas


primarias.
En las formas remitentes-recurrentes puede llegar a reducirse un 30% de los brotes.
Estn disponibles los interferones beta 1a y beta 1b y el
acetato de glatiramer.
El tratamiento con interferon puede ayudar a la recuperacin de secuelas de brotes previos.
La administracin de estos frmacos es por va parenteral.

Es FALSO sobre la encefalomielitis diseminada aguda:


1)
2)

Hipoestesias.
Paresia.
Disminucin de la agudeza visual.
Disfasia.
Disfuncin esfinteriana.

Una joven con antecedente de trastorno sensitivo hemicorporal hace 1 ao, presenta diplopa de 1 semana de evolucin. La exploracin muestra limitacin en la aproximacin
del ojo derecho con nistagmo en ojo izquierdo cuando
intenta la mirada lateral a la izquierda. Se trata de:
1)
2)
3)
4)
5)

5)
61.

Los corticoides ayudan a mejorar secuelas previas.


Los corticoides disminuyen el nmero de brotes.
El uso de interfern no est indicado en el primer brote.
El tratamiento concomitante con corticoides e interfern
est contraindicado.
No existe tratamiento que modifique el curso clnico de
la enfermedad.

Se trata probablemente de un caso de esclerosis mltiple.


La causa es una infeccin vrica, la ms probable, la
varicela, por ser sta la enfermedad vrica nativa ms
frecuente en pases occidentales.
En el LCR esperara encontrar bandas oligoclonales con
aumento de las gammaglobulinas.
Si se aplica en fases iniciales, el tratamiento especfico es
altamente efectivo.
Es un cuadro frecuente y habitualmente autolimitado.

Un paciente con antecedentes de alcoholismo es trado al


hospital tras ser encontrado en la calle con alteracin del
nivel de conciencia. En la exploracin presenta apertura
ocular espontnea, tetraparesia, ausencia de movimientos
faciales, con conservacin de motilidad ocular vertical voluntaria y parpadeo, que parece hacer de forma voluntaria
en respuesta a orden verbal. Seale cul de entre los siguientes le parece el diagnstico ms probable:

CTO Medicina C/ Nez de Balboa, 115 28006 MADRID (Espaa) Tfno.: (91) 782 43 32 / Fax: (91) 782 43 27
E-mail: secretaria@ctomedicina.com; iberocto@ctomedicina.com WEB: www.ctomedicina.com; www.iberocto.com

Preguntas TEST

Seguimiento a distancia

NEUROLOGA

Preparacin Examen de Seleccin 05/06 1 Vuelta


1)
2)
3)
4)
5)

Brote agudo de esclerosis mltiple.


Mielinolisis central pontina
Intoxicacin etlica aguda
Encefalopata de Wernicke.
Psicosis de Korsakoff.

3)
4)
5)
68.

PATOLOGA EXTRAPIRAMIDAL.
64.

Un paciente de 47 aos presenta un cuadro progresivo de


decaimiento, tristeza, trastornos de memoria, y movimientos
continuos involuntarios de forma generalizada. No tiene
antecedentes personales destacables. Sabe que su abuelo
paterno "perdi la cabeza" y que tena movimientos extraos.
Su padre falleci joven en un accidente. Seale, sobre el
diagnstico ms probable de este paciente, la respuesta FALSA:
1)
2)
3)
4)
5)

65.

2)
3)
4)
5)

Preguntas TEST

Asocia movimientos coreicos, demencia subcortical y


trastornos psiquitricos.
El diagnstico puede establecerse con la clnica y los
antecedentes familiares.
Presenta una dilatacin selectiva de las astas frontales de
los ventrculos laterales.
Existe una alteracin gentica en el cromosoma X.
No existe medicacin que detenga la progresin de la
enfermedad, aunque s puede pautarse tratamiento sintomtico.

71.

Fluoxetina.
Reserpina.
Haloperidol.
Bromocriptina.
Tetrabenacina.

1)
2)

Enfermedad de Parkinson.
Enfermedad de Huntington (variante rgida o enfermedad
de Westphal).
M exico A rgentina
C hile U ruguay

2)
3)
4)
5)
73.

Temblor de reposo.
Bradicinesia.
Reflejos miotticos normales.
Conservacin de funciones corticales.
Cadas frecuentes.

Con respecto a la enfermedad de Parkinson idioptica, seale


la respuesta INCORRECTA:
1)

Paciente de 71 aos remitido a consulta de neurologa por


cambios de comportamiento de instauracin progresiva en
pocos meses, junto a alucinaciones visuales ocasionales y
cadas al suelo en varias ocasiones, sin prdida del nivel de
conciencia. En la exploracin se aprecia hipomimia facial,
aumento del tono muscular de forma global, con resistencia
continua a los movimientos pasivos, reflejos presentes algo
hipoactivos y simtricos. No se objetiva movimientos involuntarios ni temblor de reposo. Ante estos signos, seale cul de
los siguientes diagnsticos le parece ms probable:

Enfermedad de Parkinson.
Sndrome de Steele-Richardson-Olszewski.
Atrofia olivopontocerebelosa.
Enfermedad de Shy-Drager.
Hidrocefalia normotensa.

Cul de entre los siguientes datos clnicos, cuando se presentan desde el inicio del cuadro sindrmico, NO es sugerente
de enfermedad de Parkinson idioptica?:
1)
2)
3)
4)
5)

72.

Enfermedad de Parkinson.
Sndrome de Steele-Richardson-Olszewski.
Atrofia olivopontocerebelosa.
Enfermedad de Shy-Drager.
Hidrocefalia normotensa.

Un varn de 65 aos consulta por dificultad para iniciar la


miccin, incontinencia e impotencia. En la historia clnica
destaca la presencia de sncopes posturales desde hace aos,
junto con dificultad para la marcha, rigidez y bradicinesia.
Cul es su sospecha diagnstica?:
1)
2)
3)
4)
5)

Cul de los siguientes frmacos NO es til en el tratamiento


de la enfermedad de Huntington?:
1)
2)
3)
4)
5)

67.

70.

Enfermedad de Parkinson.
Sndrome de Steele-Richardson-Olszewski.
Atrofia olivopontocerebelosa.
Enfermedad de Shy-Drager.
Hidrocefalia normotensa.

Ante un paciente que presenta rigidez, bradicinesia, escaso


temblor y marcha atxica con aumento de base de sustentacin, su diagnstico de presuncin ser:
1)
2)
3)
4)
5)

Es INCORRECTO sobre la enfermedad de Huntington:


1)

66.

En una TC craneal se podr objetivar atrofia de la cabeza


del ncleo caudado.
El pronstico de esta enfermedad es de una supervivencia
de entre 10 y 25 aos.
El tratamiento de eleccin es con neurolpticos (bloqueantes de receptores dopaminrgicos).
Si el padre del paciente hubiera vivido lo suficiente, con
toda probabilidad habra presentado un cuadro similar.
El nico aminocido que se encuentra alterado en el
sistema nervioso central es la dopamina.

Enfermedad por cuerpos de Lewy difusos.


Enfermedad de Creutzfeldt-Jakob.
Enfermedad de Alzheimer.

Varn de 75 aos, con clnica progresiva de trastornos en la


marcha, cadas frecuentes, rigidez cervical en extensin,
parlisis de la mirada conjugada vertical y demencia. El
trastorno ms probable es:
1)
2)
3)
4)
5)

69.

Seguimiento a distancia

El marcador anatomopatolgico ms caracterstico son


los cuerpos de Lewy, que se encuentran especialmente en
el tronco del encfalo.
El temblor de reposo caracterstico se asocia en numerosas
ocasiones a un temblor postural.
Los cambios en la personalidad son frecuentes en las fases
evolucionadas de la enfermedad.
A la exploracin es frecuente encontrar una hipertona
elstica.
En el tratamiento farmacolgico no se incluyen los antagonistas dopaminrgicos.

Seale cul de los siguientes neurotransmisores se encuentra


ms alterado en la enfermedad de Parkinson:

CTO Medicina C/ Nez de Balboa, 115 28006 MADRID (Espaa) Tfno.: (91) 782 43 32 / Fax: (91) 782 43 27
E-mail: secretaria@ctomedicina.com; iberocto@ctomedicina.com WEB: www.ctomedicina.com; www.iberocto.com

NR Pg. 7

1)
2)
3)
4)
5)
74.

75.

1)

Amantadina.
Lisuride.
Bromocriptina.
Clorpromacina.
Selegilina.

2)
3)
4)

Cursa con afectacin de primera y segunda motoneuronas, de forma asimtrica.


En fases iniciales no se suele afectar la musculatura extraocular.
En fases avanzadas es comn la afectacin intelectual.
La causa ms frecuente de muerte est en relacin con la
debilidad de la musculatura respiratoria.
La media de supervivencia desde el inicio de los sntomas
se sita en torno a los tres aos.

1)

PATOLOGA VASCULAR CEREBRAL.

4)
5)

El fenmeno wearing-off es la reaparicin de sntomas


parkinsonianos por descenso de las concentraciones
plasmticas de L-dopa.
El fenmeno on-off son fluctuaciones motoras que no
tienen relacin con los niveles plasmticos de L-dopa.
El tratamiento del fenmeno wearing-off se basa en mantener concentraciones plasmticas estables.
El tratamiento del fenmeno on-off se basa en aumentar
de forma sostenida las concentraciones plasmticas de Ldopa.
Estos fenmenos no suelen aparecer en las primeras fases
de la enfermedad.

80.

2)
3)
4)
5)

Con frecuencia se encuentran en la historia antecedentes


familiares de sntomas similares.
Dada la existencia de rueda dentada, lo ms probable es
que desarrolle un parkinsonismo en los prximos aos.
El tratamiento con beta-bloqueantes puede ser una buena
opcin como terapia sintomtica.
Si se encuentra temblor cintico en la exploracin, habr
que descartar patologa de fosa posterior.
Hay que descartar que el paciente est siguiendo algn
tratamiento que pueda justificar la sintomatologa.

81.

2)
3)
4)
5)
82.

Es la ataxia hereditaria ms frecuente.


Los sntomas comienzan antes de los 25 aos de edad.
Suele haber hipo-arreflexia generalizada.
Las respuestas cutneo plantares son extensoras.
La principal causa de muerte son las infecciones respiratorias de repeticin.

Seale cul de las siguientes patologas NO se asocia con la


enfermedad de Friedreich:

Pg. 8 NR

M exico A rgentina
C hile U ruguay

Crisis comicial afectando al rea frontal derecha asociando parlisis de Todd postcrtica.
Embolismo cardaco sobre la arteria basilar con lesin
isqumica de la hemiprotuberancia derecha.
Embolismo sobre territorio de la arteria cerebral media
izquierda.
Embolismo sobre el territorio de la arteria cerebral posterior izquierda.
Embolismo sobre el territorio de la arteria cerebral posterior derecha.

Cul de los siguientes signos y sntomas NO esperara


encontrar en un infarto de la arteria cerebral anterior?:
1)
2)
3)
4)
5)

83.

HTA.
Hipercolesterolemia.
Tratamiento con anticonceptivos orales.
Tabaquismo.
Diabetes mellitus.

Un paciente de 56 aos present hace dos meses un IAM


anteroseptal. Es trado hoy a Urgencias por cuadro brusco de
prdida de fuerza en miembros derechos. La exploracin
revela desviacin conjugada de los ojos a la izquierda y
hemianopsia derecha por amenaza. Se trata de:
1)

En relacin con la enfermedad de Friedreich, seale la respuesta INCORRECTA:


1)
2)
3)
4)
5)

Seale cul de los siguientes es el factor de riesgo ms


importante de la patologa cerebrovascular isqumica:
1)
2)
3)
4)
5)

Paciente de 65 aos que presenta oscilaciones en las manos


cuando realiza determinadas tareas como escribir, usar cubiertos o afeitarse. Mejora con la ingesta moderada de alcohol.
La exploracin neurolgica es normal, salvo una discreta
rueda dentada en mano derecha. No presenta temblor de
reposo, limitacin en la amplitud o rapidez de los movimientos,
ni trastornos de la marcha. Seale la opcin que le parece
INCORRECTA con respecto al caso clnico descrito:
1)

78.

Sobre la evolucin de la ELA, seale la respuesta FALSA:

5)

3)

77.

79.

Deterioro cognitivo.
Miocardiopata.
Trastornos de conduccin cardaca.
Malformaciones esquelticas.
Diabetes mellitus.

Seale la respuesta FALSA con relacin a los efectos secundarios del tratamiento a largo plazo con L-dopa en la enfermedad de Parkinson:

2)

76.

1)
2)
3)
4)
5)

Noradrenalina.
Dopamina.
Serotonina.
Acetilcolina.
Somatostatina.

Seale cul de los siguientes frmacos NO es til en el


tratamiento de la enfermedad de Parkinson:
1)
2)
3)
4)
5)

NEUROLOGA

Preparacin Examen de Seleccin 05/06 1 Vuelta

Hemianopsia homnima contralateral.


Abulia.
Desinhibicin o moria.
Hemiparesia contralateral de predominio crural.
Apraxia de la marcha en lesiones bilaterales.

Paciente de 58 aos que acude a Urgencias por prdida de


visin en el hemicampo derecho, de instauracin brusca. En
la exploracin no se aprecia dficit motor y la comprensin
y expresin del lenguaje estn conservadas. Destaca una
hipoestesia hemicorporal derecha que incluye hemicara.
Con respecto al diagnstico ms probable de este enfermo,
seale la respuesta correcta:

CTO Medicina C/ Nez de Balboa, 115 28006 MADRID (Espaa) Tfno.: (91) 782 43 32 / Fax: (91) 782 43 27
E-mail: secretaria@ctomedicina.com; iberocto@ctomedicina.com WEB: www.ctomedicina.com; www.iberocto.com

Preguntas TEST

Seguimiento a distancia

NEUROLOGA

Preparacin Examen de Seleccin 05/06 1 Vuelta


1)
2)
3)
4)
5)
84.

Paciente de 76 aos que sufre de forma brusca dificultad para


sostener cosas con la mano izquierda, asociando dificultad
para hablar. Entre sus antecedentes personales destacan:
hipercolesterolemia leve en tratamiento diettico, HTA de
difcil control teraputico y sustitucin de la cabeza femoral
izquierda por prtesis hace 5 aos. A la exploracin presenta
desviacin de la comisura bucal hacia la derecha con capacidad conservada para cerrar el ojo izquierdo. La exploracin sensitiva es normal. Seale el diagnstico ms probable:
1)
2)
3)
4)
5)

85.

3)
4)
5)

Preguntas TEST

87.

88.

89.

Paciente de 58 aos que acude por presentar episodio de


prdida de visin en ojo izquierdo, de inicio en campo
superior, de duracin total de unos dos minutos. Fondo de
ojo: sin hallazgos. Refiere haber presentado das antes episodio de escasos minutos de duracin de hormigueos en la
mano derecha, que le cedieron con un masaje. A la exploracin, auscultacin cardaca rtmica sin soplos. Cartidas
rtmicas y simtricas, sin soplos. Seale la respuesta INCORRECTA:
1)
2)
3)
4)
5)

La primera prueba diagnstica a realizar es un estudio ecoDoppler de troncos supraarticos.


Si se aprecia estenosis en el eco-Doppler, se debe realizar
angiografa posteriormente.
Si en el estudio angiogrfico se aprecia estenosis de menos
del 30% en la cartida sintomtica, el tratamiento de
eleccin es antiagregacin.
Si se detecta estenosis entre el 30 y el 50% de la cartida
sintomtica, el tratamiento se realiza con anticoagulacin
oral.
En el caso de encontrarse una estenosis de ms del 70 %
en la cartida sintomtica, hay que plantearse la posibilidad quirrgica.
M exico A rgentina
C hile U ruguay

Putamen.
Cpsula interna.
Tlamo.
Hemisferios cerebrales.
Hemisferios cerebelosos.

La TC craneal de un paciente muestra un hematoma talmico


de 1,5 cm de dimetro. Seale la causa ms probable de esta
patologa:
1)
2)
3)
4)
5)

Hemorragia intraparenquimatosa corticosubcortical de


localizacin temporoparietal derecha.
Lesin isqumica en territorio de arteria cerebral posterior
izquierda.
Infarto lacunar en ganglios basales derechos.
Infarto lacunar en cpsula interna derecha.
Infarto isqumico carotdeo derecho extenso.

Permite detectar infartos lacunares.


Una TC craneal urgente normal puede aparecer en fases
muy precoces de las hemorragias intraparenquimatosas.
Algunos infartos lacunares pueden no ser visibles.
En fase aguda, la isquemia cerebral puede no ser visible.
reas hipodensas en territorios frontera sugieren origen
hemodinmico.

La localizacin ms frecuente de los hematomas intraparenquimatosos espontneos de causa hipertensiva es:


1)
2)
3)
4)
5)

Angiopata amiloide.
Malformacin vascular.
Rotura de aneurisma sacular.
Hipertensin.
Rotura de aneurisma fusiforme.

Seale cul es la causa ms probable de isquemia en el


territorio de la arteria cerebral media:
1)
2)
3)
4)
5)

En cuanto al uso de la tomografa computadorizada para el


diagnstico de la patologa cerebrovascular aguda, seale la
respuesta FALSA:
1)
2)

86.

Es preciso la realizacin de una RM cerebral ante la


sospecha de patologa de fosa posterior.
Presenta probablemente isquemia en territorio de arteria
cerebral media izquierda, de ramas parietales.
Los datos clnicos sugieren afectacin en el territorio de la
arteria cerebral posterior izquierda.
Si la prueba de imagen cerebral urgente es normal, lo ms
probable es que se trate de un estatus epilptico parcial
Es urgente descartar patologa ateromatosa carotdea.

Seguimiento a distancia

Aterotrombosis de grandes vasos intracraneales.


Aterotrombosis de grandes vasos extracraneales.
Embolia cardiognica.
Embolia paradjica a travs de un foramen oval permeable.
Trombosis in situ.

SISTEMA NERVIOSO PERIFRICO.


90.

En el diagnstico diferencial de las neuropatas, seale el


enunciado correcto:
1)
2)
3)
4)
5)

91.

Seale cul de las siguientes neuropatas perifricas NO se ha


relacionado con la infeccin por VIH:
1)
2)
3)
4)
5)

92.

La disminucin en la amplitud de los potenciales de


accin es tpica de la afectacin axonal.
La dispersin de los potenciales de accin es caracterstica
de la afectacin axonal.
La velocidad de conduccin se encuentra disminuida en
las neuropatas axonales y en las desmielinizantes.
Los bloqueos de conduccin son sugerentes de lesin
axonal.
No existe lesin axonal y desmielinizante concomitantemente en un mismo nervio.

Polineuropata desmielinizante aguda.


Neuropata sensitiva distal.
Mononeuritis mltiple.
Neuropata amiotrfica proximal.
Polirradiculitis.

Seale el enunciado INCORRECTO con respecto a las neuropatas diabticas:


1)
2)
3)

La ms frecuente es la polineuropata distal simtrica.


La mononeuropata craneal ms frecuente es el III par
incompleto.
Sntomas como la hipotensin ortosttica, la impotencia o
las diarreas son sugerentes de neuropata vegetativa.

CTO Medicina C/ Nez de Balboa, 115 28006 MADRID (Espaa) Tfno.: (91) 782 43 32 / Fax: (91) 782 43 27
E-mail: secretaria@ctomedicina.com; iberocto@ctomedicina.com WEB: www.ctomedicina.com; www.iberocto.com

NR Pg. 9

4)
5)
93.

2)
3)
4)
5)
94.

3)
4)
5)

4)
5)

5)
97.

4)
5)
98.

4)
5)

Se hereda con carcter recesivo ligado a X.


Afecta casi exclusivamente a varones.
El gen defectuoso se encuentra en el brazo corto del
cromosoma X (Xp21).
Aparece en la segunda dcada de la vida.
La miocardiopata se presenta en un alto porcentaje de
pacientes.

La lesin est mediada por anticuerpos.


Los anticuerpos actan a nivel presinptico.
Se produce un aplanamiento de los pliegues postsinpticos.
El 10% se asocia a timoma.
La musculatura craneofacial es la ms frecuentemente
afectada.

Es INCORRECTO en relacin con la miastenia gravis:


1)
2)
3)
4)
5)

100.

Dficit motor de predominio distal en extremidades.


Alteraciones sensitivas, aunque poco intensas.
Frecuente compromiso de la reactividad pupilar por
afectacin eferente.
Reflejos de estiramiento muscular conservados.
Disfuncin autonmica asociada.

Seale el enunciado FALSO sobre la miastenia gravis:


1)
2)
3)

99.

Asocia cataratas subcapsulares, cardiopata con bloqueos


AV y deterioro intelectual a lo largo de la evolucin.

La miastenia gravis se caracteriza por:


1)
2)
3)

El tratamiento con corticoides no est indicado.


La administracin de gammaglobulina policlonal intravenosa es til en fases iniciales.
La plasmafresis es til en fases iniciales.
El suero antitoxina puede bloquear la extensin de la
enfermedad.
Lo que garantiza la supervivencia del paciente es el mantenimiento de las funciones vitales a medio plazo.

En la enfermedad de Duchenne, NO es cierto:


1)
2)
3)

96.

Es la forma ms frecuente de polineuropata desmielinizante aguda.


Puede tener alguna sintomatologa sensitiva.
Se ha asociado a la infeccin previa por Campylobacter
jejuni.
El diagnstico puede establecerse por la clnica y el estudio
electrofisiolgico.
El hallazgo de un LCR normal en fases iniciales descarta la
enfermedad.

Un paciente presenta clnica de debilidad en miembros


inferiores de instauracin progresiva en el curso de escasos
das tras un proceso gastroentertico. En la exploracin
destaca la presencia de debilidad en extremidades, ms
acusada en miembros inferiores, abolicin generalizada de
reflejos miotticos y conservacin de la sensibilidad. Con
respecto al tratamiento ms adecuado de la patologa que
probablemente presenta este paciente, seale la respuesta
INCORRECTA:
1)
2)

95.

Son potencialmente reversibles si se tratan de forma agresiva.


La amiotrofia diabtica es de predominio proximal en
cintura plvica.

Sobre el sndrome de Guillain-Barr, NO es cierto:


1)

NEUROLOGA

Preparacin Examen de Seleccin 05/06 1 Vuelta

La aparicin de ptosis y debilidad fluctuantes a lo largo del


da deben hacer sospechar el diagnstico.
La clnica y los hallazgos neurofisiolgicos pueden ser
suficientes para establecer el diagnstico.
Es preciso la realizacin de una RX de trax para confirmar
el diagnstico.
La alteracin tmica ms frecuentemente asociada es la
hiperplasia.
En las formas oculares puras el tratamiento de eleccin son
los anticolinestersicos.

En relacin con el botulismo, es correcto:


1)
2)
3)
4)
5)

En lactantes se produce tpicamente por la ingesta de la


toxina preformada.
En adultos, la ingesta de esporas es la causa ms frecuente.
El debut del cuadro clnico es ms rpido en el causado
por heridas contaminadas.
Pupilas puntiformes reactivas.
Es frecuente la presencia de oftalmoplejia.

Mujer de 29 aos que presenta de forma progresiva, debilidad en la musculatura facial, claudicacin mandibular y
debilidad en ambas manos. No presenta dificultad para la
marcha ni para subir escaleras. La paciente refiere que tras
un movimiento, los msculos se le quedan contrados, tardando varios segundos en relajarse; este fenmeno le desaparece tras unos minutos de ejercicio continuado y le
reaparece por las maanas al levantarse. Con respecto al
cuadro que probablemente presenta la paciente, seale la
opcin INCORRECTA:
1)
2)
3)
4)

Son tpicos los antecedentes familiares.


Una biopsia muscular mostrar atrofia, preferentemente
de las fibras tipo I.
El tratamiento de eleccin de las miotonas es la fenitona.
El proceso patolgico primario es una alteracin de la
sntesis de los receptores de acetilcolina en la membrana
muscular.

Pg. 10 NR

M exico A rgentina
C hile U ruguay

CTO Medicina C/ Nez de Balboa, 115 28006 MADRID (Espaa) Tfno.: (91) 782 43 32 / Fax: (91) 782 43 27
E-mail: secretaria@ctomedicina.com; iberocto@ctomedicina.com WEB: www.ctomedicina.com; www.iberocto.com

Preguntas TEST

Seguimiento a distancia

NEUROLOGA

Preparacin Examen de Seleccin 05/06 1 Vuelta


Pregunta 1.- R: 4
El rea motora ocupa la circunvolucin prerrolndica o rea 4 de
Brodmann en el lbulo frontal. All se encuentran los cuerpos
neuronales de las primeras motoneuronas encargadas de inervar el
hemicuerpo contralateral de forma voluntaria. Las lesiones de las primeras motoneuronas, si se preservan las segundas, dan lugar a una
paresia espstica, con reflejos de estiramiento muscular aumentados
(opcin 2), hipertona elstica, clonus y respuesta cutneo-plantar
extensora (signo de Babinski), por lo que la opcin incorrecta es la 4.
En fases muy precoces de lesin de primera motoneurona los reflejos estn abolidos (opcin 1), sobre todo en lesiones de la va piramidal
(mdula), y tardan das o semanas en exaltarse.
En fases avanzadas puede encontrarse disminucin de la masa
muscular (opcin 5), dependiente del grado de prdida de fuerza,
condicionada por la menor actividad muscular. No sucede igual en
las lesiones de la segunda motoneurona, en las que la amiotrofia es
precoz y ms intensa.
Los reflejos de estiramiento muscular o miotticos se desencadenan por una conexin monosinptica (entre una neurona sensitiva y
una segunda motoneurona), mientras que los reflejos cutneos suelen
ser polisinpticos, y no se comportan de la misma manera que los
anteriores. Los reflejos cutneos abdominales reflejan la respuesta
medular al estmulo en la piel abdominal, con una contraccin de la
pared del abdomen; se dividen en superiores (D6-9), medios (D9-11)
e inferiores (D11-L1) y suelen abolirse en lesiones de las primeras
motoneuronas, especialmente en las lesiones medulares.

Comentarios TEST

Pregunta 1. Diagnstico diferencial de lesin de 1 y 2 motoneurona.


1234565789
57

234565789
57

123456789
36782 7 4
7

7 4678 574 678683 67


9
36782 7 4
7

68 256
65 8684836
87 76
53
6 873 75 5 

1256
65 85236
 

467853
567
73 7 8  634 

!76
8" #57$5

4678536 567868 77


73 7 8  634 
84!6

68 7 5 4 567

% 8!575
8 7 5 4 567

Pregunta 2.- R: 5
Cuando un paciente presenta un dficit motor del que no sabemos sus caractersticas exploratorias (reflejos, tono, etc.) y debemos
basarnos exclusivamente en la distribucin, hay que recordar los distintos patrones:
Lesiones medulares: no afectan a la cara, y el paciente presenta
una hemiparesia ipsilateral a la lesin. Aunque existe una parte de
axones de primeras motoneuronas que descienden por el cordn
anterior y se decusan en la mdula, la mayor parte de la va corticoespinal se decusa en el bulbo y se sita en el cordn lateral
ipsilateral al hemicuerpo que va a inervar.
Lesiones troncoenceflicas: tpicamente dan lugar a los sndromes
cruzados, en los que aparece sintomatologa en un hemicuerpo y
en la hemicara opuesta. La afectacin de las extremidades se produce por la lesin de la va corticoespinal en su trayecto por el
tronco del encfalo, an sin decusar, por lo que dan lugar a defectos contralaterales, mientras que la afectacin facial se produce
por lesin de los ncleos motores, es decir, las segundas
motoneuronas, que se sitan en el mismo lado del msculo al que
inervan, por lo que producen defectos ipsilaterales. En los sndromes
cruzados, la hemicara indica el lado de la lesin.
Lesiones por encima del tronco del encfalo (cpsula interna, corteza frontal): dan lugar a defectos hemicorporales contralaterales
que incluyen la hemicara y las extremidades.
En el caso de la pregunta, al tener un defecto en la hemicara, brazo
y pierna en el mismo hemicuerpo, sabemos que se trata de una lesin
M exico A rgentina
C hile U ruguay

Seguimiento a distancia

superior al tronco del encfalo, y contralateral, puesto que an no ha


tenido lugar ninguna decusacin, y por eso es la 5 la respuesta correcta.
Pregunta 3.- R: 4
Aunque el motivo de la consulta es la diplopa, en realidad es un
dato que no es imprescindible para responder correctamente, dado
que el resto de la exploracin que presenta el paciente indica un
sndrome cruzado, con afectacin facial izquierda y de extremidades
derechas.
Los sndromes cruzados son la presentacin caracterstica de las
lesiones del tronco del encfalo, donde se lesiona la va piramidal
antes de decusarse en el bulbo, y los ncleos motores del tronco del
encfalo.
Ante la presencia de un sndrome cruzado, la localizacin de la
lesin viene definida por la afectacin craneofacial, tanto para el
nivel lesional como para la lateralidad. La lesin es homolateral a la
cara afectada, en este caso izquierda, y el nivel lo indica el par
craneal afectado, en este caso el VII, que se sita en la protuberancia, con lo que tenemos suficientes datos para dar con la opcin
correcta.
Las diplopas que se describen como una limitacin en la motilidad del globo ocular en una direccin concreta apuntan a lesin
neuromuscular de un msculo concreto de los encargados de mover
el globo ocular (lo que se denomina motilidad ocular extrnseca o
extraocular). El III par se encarga de la motilidad vertical superior e
inferior del ojo, y del movimiento de adduccin (hacia la lnea media); el IV par contribuye a un movimiento similar al de mirar a la
punta de la nariz (infraversin y rotacin interna), y el VI par realiza
una accin de abduccin casi pura. Si el paciente tiene limitacin en
la mirada externa con el ojo izquierdo, el origen ms probable es la
afectacin del VI par izquierdo, que en el caso concreto de la pregunta, apunta a una lesin nuclear en la protuberancia izquierda, junto al
ncleo del VII par izquierdo y la va corticoespinal.
Pregunta 4.- R: 4
Ante la presencia de un sndrome cruzado, sea ste motor o sensitivo, la lesin estar situada en el tronco del encfalo. En el caso de los
sndromes cruzados sensitivos, ser una afectacin baja, ya que la
porcin central del trigmino (el ncleo sensitivo central del trigmino) se sita en su mayor parte en el bulbo y en la parte ms inferior de
la protuberancia. Dado que entre las respuestas no est la opcin de
la protuberancia, la respuesta no puede ser ms que la 4.
Hay que recordar, con respecto a los sndromes sensitivos, que una
lesin parietal no produce anestesia hemicorporal, a pesar de encontrarse all la corteza sensitiva primaria, sino trastornos en las sensibilidades combinadas, esto es, en el reconocimiento adecuado de los
estmulos sensitivos (agnosias sensitivas).
Las lesiones talmicas s producen anestesia hemicorporal completa,
que incluye la hemicara, y que si se establece de forma definitiva,
suele acompaarse de episodios lancinantes de dolor espontneo en
el hemicuerpo anestesiado (dolor talmico), y de movimientos
pseudoatetoides de la mano afectada, probablemente en relacin
con la desaferentizacin sensitiva.
Por lo que respecta a las lesiones bulbares, como es el caso de la
pregunta, hay que recordar que en el bulbo la va espinotalmica,
portadora de la informacin del dolor y de la temperatura, discurre
por la porcin lateral, mientras que la informacin posicional y vibratoria que asciende por los cordones posteriores, se sita medial. Esta
disposicin anatmica hace que en la lesin bulbar lateral se vea
afectada la sensibilidad termoalgsica y no la propioceptiva, y en el
sndrome bulbar medial suceda al contrario.
Pregunta 5.- R: 2
El equilibro en el dimetro pupilar viene determinado por la accin permanente del estmulo parasimptico, que produce constriccin, y el estmulo simptico, que provoca dilatacin. La presencia,
por tanto, de unas pupilas medias, es diagnstica de un equilibrio
simptico-parasimptico.
La existencia de una pupila midritica indica lesin de la va parasimptica con conservacin de la simptica. Hay que pensar en lesiones a nivel del ncleo parasimptico o de Edinger-Westphal (mesence-

CTO Medicina C/ Nez de Balboa, 115 28006 MADRID (Espaa) Tfno.: (91) 782 43 32 / Fax: (91) 782 43 27
E-mail: secretaria@ctomedicina.com; iberocto@ctomedicina.com WEB: www.ctomedicina.com; www.iberocto.com

NR Pg. 1

flico posterior), de la porcin mesenceflica del III par, o del trayecto


del III par (seno cavernoso, rbita). Las lesiones del ncleo pueden tener
afectacin exclusivamente pupilar; las otras se acompaarn de dificultad en los movimientos del globo ocular y de ptosis.
El hallazgo de una pupila mitica acompaada de ptosis es un
sndrome de Horner e indica lesin del simptico cervical. Debido a su
especial disposicin anatmica, las lesiones del simptico pueden
producirse en distintos niveles: hipotlamo, tronco del encfalo, mdula cervical (columna intermediolateral), vrtice torcico (ganglio
simptico superior cervical), cartida interna y rbita. No se relaciona
con la cartida externa, y por eso la opcin falsa es la 2.
Cualquier alteracin de la motilidad pupilar puede estar originada
por el uso de sustancias de aplicacin externa (colirios), con resultados que dependern de la naturaleza del frmaco.
La exploracin de la motilidad pupilar se realiza fundamentalmente con el reflejo fotomotor, por el cual un estmulo luminoso es
conducido por el nervio ptico-quiasma-cintilla ptica hasta el mesencfalo, y sin detenerse en el cuerpo geniculado lateral, como hace
el resto de la va ptica, conecta con los dos ncleos de EdingerWestphal, produciendo miosis bilateral.
El reflejo de acomodacin es ms complejo: la imagen de un objeto
acercndose hacia el ojo llega hasta la corteza occipital, y un impulso
descendente llega hasta los ncleos oculomotores mesenceflicos,
originando miosis, convergencia (los dos pares III realizan adduccin)
y acomodacin del cristalino. En la neurosfilis, este reflejo permanece conservado mientras que el fotomotor est abolido. Por esta razn
puede considerarse correcta la opcin 4, ya que la lesin de cordones posteriores no genera una pupila de Argyll- Robertson, pero la
presencia de esta alteracin de las pupilas sugiere fuertemente que
estamos ante una neurosfilis, que se acompaar de tabes dorsal.
Pregunta 6.- R: 5
La presencia de ptosis y miosis es altamente sugerente de sndrome de
Horner por lesin del simptico cervical. La inervacin simptica se encarga de la dilatacin pupilar, de la musculatura tarsal (contribuye a la
elevacin del prpado) y de las glndulas sudorparas y los vasos sanguneos de la hemicara. El sndrome de Horner completo incluira pues:
Disminucin de hendidura palpebral
Miosis
Enoftalmos (aparente para algunos autores, por paresia del msculo orbitario)
Anhidrosis
Vasodilatacin
Si la lesin es distal a la bifurcacin carotdea, la anhidrosis y la
vasodilatacin no estarn presentes, dado que dicha inervacin
discurre por separado de la pupila a partir de ese punto.
Dado el complejo trayecto del simptico hasta alcanzar la pupila,
en el diagnstico diferencial del sndrome de Horner hay que incluir
las lesiones de la cartida interna (diseccin), del vrtice pulmonar
(tumores pulmonares), de la mdula cervical (lesiones centromedulares) y del tronco del encfalo. En el bulbo la va se sita lateral, por lo
que se ve afectada en el sndrome de Wallenberg y no en el sndrome
bulbar medial.
De las opciones de la pregunta, la que no se relaciona con el
sndrome de Horner es la neuroles; esta patologa asocia otro tipo de
alteracin pupilar: la pupila de Argyll-Robertson.
Pregunta 7.- R: 4
Aunque algunos sndromes medulares son muy caractersticos y
constantes, lo fundamental es recordar las principales vas y su disposicin en la mdula, dado que lesiones traumticas o compresivas
pueden dar sndromes aproximados que no encajen por completo
en lo que conocemos previamente.
Cordones posteriores: conducen la sensibilidad posicional, vibratoria, de presin y tacto hasta el tlamo (pasando por unos ncleos
en el bulbo raqudeo, donde se encuentran las segundas neuronas
sensitivas, y decusndose para alcanzar el tlamo contralateral).
Cada cordn posterior contiene la informacin del hemicuerpo
ipsilateral.
Va piramidal: los axones de las primeras motoneuronas descienden en su mayor parte por el cordn lateral medular hasta alcanPg. 2 NR

M exico A rgentina
C hile U ruguay

NEUROLOGA

Preparacin Examen de Seleccin 05/06 1 Vuelta

zar el nivel correspondiente de la segunda motoneurona. Su decusacin se produce en el bulbo raqudeo, por lo que cada cordn
lateral lleva las vas motoras para el hemicuerpo ipsilateral.
Va espinotalmica: las fibras sensitivas del nervio perifrico
que recogen las seales del dolor y la temperatura se decusan
en la mdula, con un trayecto ascendente, tras encontrar su
segunda neurona sensitiva en el asta posterior medular, y se
sitan en la porcin anterior-lateral medular. Por lo tanto, la
va espinotalmica en la mdula conduce informacin sensitiva contralateral.
Conociendo las principales vas medulares, no es difcil adivinar la
sintomatologa que presentar un paciente del que conozcamos la
lesin, o interpretar la distribucin de la misma si lo que se nos describe es cada uno de los defectos neurolgicos.

Pregunta 7. Sndromes medulares.


Mielopata
transversa

Sd. Brown-Sequard
(hemiseccin medular)

Sd. medular central

Sd. medular posterolateral

Sd. cordonal posterior

Sd. medular anterior

Pregunta 8.- R: 3
La prdida de sensibilidad trmica y dolorosa nos debe hacer sospechar una lesin de la va espinotalmica, por la que discurre esta
sensibilidad. Una lesin de esta va a nivel medular cursara con alteraciones sensitivas termoalgsicas que comenzaran dos segmentos
por debajo del nivel lesional (debido al trayecto ascendente de los
axones que acceden a la va en el interior de la mdula) y que afectaran a toda la parte inferior del hemicuerpo desde el nivel inicial. El
hecho de que se trate de un nivel lesional suspendido (con reas
ntegras por encima y por debajo de la lesin) descarta que se trate de
una lesin funicular (del cordn).
La sensibilidad termoalgsica tambin puede verse afectada en el
nervio perifrico, dado que es recogida por unas fibras independientes
del resto de sensibilidad tctil. Son fibras III y IV, finas, pobremente
mielinizadas algunas, que tras entrar por la raz posterior medular, encuentran su segunda neurona sensitiva en el asta posterior medular.
El axn de esta segunda neurona sensitiva de la va se decusa con
un trayecto ascendente, pasando por la comisura anterior medular,
muy cerca del centro de la mdula, hasta alcanzar el cordn anterior
y lateral contralaterales.
La lesin de las neuronas III y IV perifricas dara lugar a un defecto
con patrn polineuroptico, en guante y calcetn, siendo extrao que
presente un defecto proximal y no distal, como lo sera entre D3 y D6.

CTO Medicina C/ Nez de Balboa, 115 28006 MADRID (Espaa) Tfno.: (91) 782 43 32 / Fax: (91) 782 43 27
E-mail: secretaria@ctomedicina.com; iberocto@ctomedicina.com WEB: www.ctomedicina.com; www.iberocto.com

Comentarios TEST

Seguimiento a distancia

NEUROLOGA

Preparacin Examen de Seleccin 05/06 1 Vuelta


La clnica aislada de anestesia termoalgsica bilateral y simtrica
debe hacer sospechar una lesin centromedular. La causa ms frecuente de lesin centromedular es la Siringomielia, con dilatacin del
conducto ependimario y lesin de las vas adyacentes.

Comentarios TEST

Pregunta 9.- R: 1
Tenemos un paciente con dficit motor en miembros inferiores,
que se acompaa de reflejos vivos y signo de Babinski, lo que indica
lesin de la va piramidal. El hecho de que en miembros superiores no
haya asimismo hiperreflexia sugiere que la lesin es medular, dado
que si fuera por encima de los segmentos medulares cervicales, lo ms
probable es que cursara con hiperreflexia generalizada.
En miembros superiores se objetivan amiotrofias, lo que sugiere
lesin de segunda motoneurona.
En un paciente con clnica de afectacin de primeras y segundas
motoneuronas hay que considerar la posibilidad de una esclerosis
lateral amiotrfica (ELA): de curso progresivo en la edad adulta, en
este caso no es compatible, dado que la ELA debe cursar sin alteraciones cognitivas, de esfnteres ni sensitivas, y el paciente presenta signo
de Lhermitte, lo que indica afectacin de los cordones posteriores
medulares a nivel cervical.
La degeneracin subaguda combinada de la mdula presenta deterioro de cordones laterales y posteriores medulares, de curso ascendente, motivado por un dficit de vitamina B12. No cursa con afectacin de segundas motoneuronas, por lo que la amiotrofia en manos
no apoya el diagnstico.
El sndrome espinal anterior se produce en la mayora de las ocasiones por una lesin isqumica en el territorio de la arteria espinal
anterior, que irriga los dos tercios anteriores medulares. No afecta los
cordones posteriores y cursa con una plejia bilateral aguda, de mal
pronstico en lo que respecta a su recuperacin.
La mielopata cervical se produce por una disminucin del espacio en el canal medular cervical, segmento en que la mdula alcanza
su mximo grosor, con lo que se produce una compresin de curso
lentamente progresivo, afectando a las vas ascendentes y descendentes medulares (lo que justifica el Lhermitte cordones posteriores- y la
espasticidad en miembros inferiores cordones laterales-) y a las
neuronas motoras presentes a ese nivel, lo que explica la amiotrofia
coincidente con la lesin.
Pregunta 10.- R: 4
Las descripciones de los trastornos del movimiento que cursan con
exceso de movimiento (hipercinesias) no siempre hacen fcilmente
identificable el movimiento en cuestin. Unas pistas para enfocar estos trastornos:
Las contracciones musculares mantenidas tpicas son las distonas:
tienen una instauracin lenta y progresiva en el tiempo, hasta que se
establecen como fijas, limitando la actividad funcional del sujeto.
Los movimientos repetitivos estereotipados son los tics: predominan en el rea facial, de inicio tpicamente en la edad infantil. La
inmensa mayora son simples y benignos, y no precisan tratamiento farmacolgico.
Los movimientos continuos son coreoatetsicos: los atetsicos son
movimientos continuos, lentos, que predominan distalmente, aunque pueden afectar a cualquier rea corporal; los movimientos
coreicos son rpidos, de amplitud variable, que tambin pueden
afectar a cualquier parte del cuerpo y que cuando son generalizados aparecen como un baile (chorea = baile)
Las sacudidas musculares, arrtmicas, breves y saltatorias son tpicas
de las mioclonas: pueden tener origen cortical, medular o perifrico, y son arrtmicas. Todo el mundo tiene mioclonas hipnaggicas (al coger el sueo) que suelen coincidir con imgenes onricas
de cada, tropezn, etc.
Las oscilaciones rtmicas, por actividad alternante agonista-antagonista, son el temblor. Los temblores se distinguen en distintas clasificaciones, segn la forma de presentacin, la frecuencia de las
oscilaciones, o las patologas de base asociadas. Todo el mundo
tiene un cierto grado de temblor (temblor fisiolgico) que se exacerba en situaciones de cansancio, estrs, consumo de estimulantes, etc.

M exico A rgentina
C hile U ruguay

Seguimiento a distancia

Pregunta 11.- R: 3
El tono muscular viene definido por la resistencia del msculo a la
movilizacin pasiva. Los aumentos del tono muscular (hipertonas)
pueden estar en relacin con:
Espasticidad: presente cuando se pierde la funcin inhibitoria que
ejerce la va piramidal sobre la segunda neurona motora. Es mayor
al comienzo del movimiento y en movimientos rpidos; si se ejerce
una fuerza continua, se produce una relajacin del msculo (fenmeno de navaja). Se acompaa de reflejos miotticos vivos y
respuesta cutaneoplantar extensora. Si la espasticidad es muy intensa y mantenida puede llegar a dar lugar a una contraccin
mantenida (distona espstica).
Rigidez: existe un aumento de tono muscular mantenido en musculatura flexora y extensora, con lo que aparece una resistencia continua a la movilizacin pasiva en cualquier direccin. Los reflejos de
estiramiento muscular son normales y la respuesta cutneo-plantar
es flexora, salvo que coexista una lesin de la va piramidal. En el
parkinsonismo, la rigidez se acompaa de interrupciones rtmicas de
la resistencia (fenmeno de rueda dentada).
La disminucin del tono muscular (hipotona) es una prdida del
tono normal en la que los msculos aparecen flccidos y blandos,
con una resistencia inferior a la normal al movimiento pasivo. Las
causas ms frecuentes son las lesiones de la segunda motoneurona o
las miopatas. Las lesiones cerebelosas pueden cursar con una
hipotona algo ms leve que la descrita anteriormente. El desuso tambin produce hipotona.
Pregunta 12.- R: 3
La distona es un trastorno del movimiento que se define por
contracciones musculares sostenidas que producen posturas anormales, contorsiones y, en ocasiones, movimientos anormales.
Se han utilizado distintas clasificaciones, siendo las ms tiles las
que marcan la edad de comienzo (precoces si lo hacen antes de los 25
aos y tardas a las que aparecen ms tarde), la distribucin anatmica (focales, segmentarias, hemidistonas, generalizadas), la causa (primarias, hereditarias, secundarias) y, ms recientemente, las basadas en
los defectos genticos asociados.
El tratamiento de las distonas depende de la distribucin y la intensidad, y va encaminado a la mayor recuperacin funcional posible, ms que a una recuperacin completa. Se dispone de tres escalones teraputicos:
Frmacos: agentes dopaminrgicos (agonistas dopaminrgicos,
levodopa), anticolinrgicos, benzodiacepinas, baclofn, litio, antiepilpticos y un largo etctera.
Toxina botulnica: es la mejor opcin para la distona cervical, el
blefaroespasmo y la distona espstica. Se inyecta directamente en
el msculo seleccionado, obtenindose un beneficio que persiste
semanas-meses. No es til en las distonas que afectan a muchos
msculos.
Ciruga: en las distonas que no responden a los tratamientos
descritos arriba, pueden ser subsidiarios de procedimientos quirrgicos como la lesin o estimulacin cerebral profunda seleccionada en determinados ganglios basales, similar a la que se
realiza en los casos de enfermedad de Parkinson
farmacorresistente.
Pregunta 13.- R: 2
El temblor es un trastorno del movimiento definido por oscilaciones rtmicas debidas a la actividad alternante de la musculatura agonista y antagonista. Es el trastorno del movimiento ms comn. Puede
clasificarse siguiendo varios criterios (ver figura en pgina siguiente); el
ms objetivo es segn la frecuencia del temblor, pero lo ms til es,
probablemente, englobarlo dentro de un sndrome clnico:
El temblor de reposo es visible cuando la parte afectada est en
reposo (con apoyo o sometida a efecto de la gravedad) y hay que
pensar en un sndrome parkinsoniano.
El temblor postural aparece cuando se intenta mantener una postura contra gravedad. Se presenta tpicamente en el temblor esencial benigno; es la forma ms frecuente de temblor de causa farmacolgica; puede aparecer en enfermos de Parkinson asociado al

CTO Medicina C/ Nez de Balboa, 115 28006 MADRID (Espaa) Tfno.: (91) 782 43 32 / Fax: (91) 782 43 27
E-mail: secretaria@ctomedicina.com; iberocto@ctomedicina.com WEB: www.ctomedicina.com; www.iberocto.com

NR Pg. 3

NEUROLOGA

Preparacin Examen de Seleccin 05/06 1 Vuelta

temblor de reposo tpico, y es el temblor fisiolgico que todo el


mundo presentamos.
El temblor cintico aparece en los movimientos con amplio recorrido, y es tpico de la patologa cerebelosa y mesenceflica (temblor rbrico), con la peculiaridad de que en los casos cerebelosos
es aislado, mientras en las lesiones mesenceflicas coexisten temblor de reposo, postural y cintico en el mismo paciente. El temblor intencional es el empeoramiento o la aparicin del temblor al
final de un movimiento, y puede ser una primera sintomatologa
de un temblor cintico.
En esta pregunta, con las descripciones de situaciones en las que
aparece el temblor, cabe la duda de si se trata de un temblor cintico,
dado que hay accin, o un temblor postural, dado que las acciones
son mantenimiento de posturas casi fijas, con mnimo recorrido. Como
ayuda, recordar que el temblor cintico e intencional se suele describir
en maniobras de exploracin de ataxia (dedo-nariz, taln-rodilla) mientras que el temblor postural se describe en posiciones fijas (manos adelante) o con mnimo recorrido (escribir, usar cubiertos, servir agua).
Pregunta 14.- R: 1
Dentro de los movimientos involuntarios, los que vienen definidos
como sacudidas musculares deben hacernos pensar en mioclonas,
ms si se especifica que son rpidas y arrtmicas. Slo el hecho de que
se especifique que aparecen periodos silentes (o de silencio elctrico, es
decir, que no hay actividad muscular en determinados momentos) en
el electromiograma, nos da el diagnstico sindrmico de asterixis.
La asterixis se explora pidiendo al paciente que mantenga una
posicin fija contra gravedad (generalmente las manos en dorsiflexin), observndose las sacudidas porque es incapaz de mantener la
postura.
Inicialmente descrita en relacin con la insuficiencia heptica,
denominndose flapping, en realidad puede aparecer en el contexto
clnico de otras encefalopatas txico-metablicas, aunque ocasionalmente puede presentarse en pacientes con lesiones hemisfricas
cerebrales.
La encefalopata hipoglucmica caractersticamente tiene una fase
inicial, marcada por el aumento de catecolaminas circulantes, con un
cuadro de inquietud psicomotora, taquicardia, sudoracin y midriasis
que precede a una disminucin del nivel de conciencia y entrada
progresiva en coma hipoglucmico, si no se corrige a tiempo.

Pregunta 15.- R: 3
La descripcin de un temblor que slo aparece cuando no se
ejerce accin muscular en la parte afectada (dejada caer a favor de
gravedad, o apoyada) es diagnstica de temblor de reposo, y ste a su
vez conlleva inmediatamente la sospecha de un sndrome parkinsoniano.
De los sndromes que cursan con rigidez, bradicinesia y temblor de
reposo, el ms frecuente es la Enfermedad de Parkinson. Es, por otra
parte, en el que predomina esta forma de debut, con un temblor de
reposo, de predominio en manos, asimtrico (en fases iniciales slo
tiembla una mano) e inicialmente sin otros signos o sntomas, con lo
cual, la mera aparicin de un temblor de reposo en una mano levanta la sospecha de enfermedad de Parkinson.
De los distintos tipos de temblor, el ms frecuente es el temblor
postural; si el paciente refiere un temblor postural de instauracin
lentamente progresiva, en efecto hay que preguntar por los antecedentes familiares, ya que la sospecha sera un temblor esencial, que es
en la mayora de los casos familiar, con distintos tipos de herencia (la
ms frecuente, autosmica dominante con alta penetrancia).
En pacientes con antecedente de alcoholismo se pueden encontrar temblores de tipo cintico por lesin cerebelosa crnica, aunque
es ms caracterstica la atrofia vermiana, y temblores de carcter postural,
en fases de deprivacin alcohlica.
Todos los temblores posturales se acentan en situaciones de inquietud emocional y mejoran con la tranquilidad, independientemente de cul sea su origen.
Pregunta 16.- R: 5
Las lesiones cerebelosas cursan con ataxia, es decir, mala organizacin de los movimientos. Aparecen errores en la amplitud, la direccin, la fuerza y la frecuencia empleada en los distintos movimientos.
Entran en accin distintos componentes del movimiento en el momento equivocado, y las correcciones que se hacen suelen ser excesivas, por lo que el movimiento resultante resulta irregular, pendular y
espasmdico (asinergia).
Una lesin cerebelosa puede presentar un temblor que no es de
reposo (postural o cintico) que caractersticamente se acenta al
final (temblor intencional); parece relacionarse con la lesin de vas
dentatorrubrotalmicas, por lo que la lesin de las mismas a nivel
mesenceflico, cerca del ncleo rojo, puede cursar con un temblor
muy similar (temblor rbrico) que se diferencia del cerebeloso por

Prenguta 13. Clasificacin y tratamiento del temblor

Pg. 4 NR

M exico A rgentina
C hile U ruguay

CTO Medicina C/ Nez de Balboa, 115 28006 MADRID (Espaa) Tfno.: (91) 782 43 32 / Fax: (91) 782 43 27
E-mail: secretaria@ctomedicina.com; iberocto@ctomedicina.com WEB: www.ctomedicina.com; www.iberocto.com

Comentarios TEST

Seguimiento a distancia

NEUROLOGA

Preparacin Examen de Seleccin 05/06 1 Vuelta


asociar un componente de temblor de reposo del que aqul carece.
En reposo no se pueden objetivar alteraciones del movimiento; en
la valoracin del tono muscular pueden no encontrarse signos patolgicos, a lo sumo una disminucin del tono muscular que es leve,
menos intensa de la que presentan las lesiones de segunda
motoneurona; esto justifica que la respuesta falsa sea la 5.
Hay que recordar que la marcha atxica por lesin cerebelosa se
caracteriza por una aparente debilidad en miembros inferiores: la
extremidad que avanza lo hace con movimientos errticos y al apoyar
puede golpear en el suelo, con lo que puede resultar difcil diferenciar esta marcha de la que se presenta en el dficit de sensibilidad
profunda. El paciente con lesin cerebelosa presenta mayor dificultad al intentar caminar con ojos cerrados; esto no es un signo de
Romberg positivo, ya que slo debe considerarse como tal a la inestabilidad que aparece al cerrar los ojos y desaparece al abrirlos.

Comentarios TEST

Pregunta 17.- R: 3
Ante la descripcin de un paciente con marcha atxica tenemos
que establecer el diagnstico diferencial bsicamente entre unos cuadros concretos:
Ataxia sensitiva: por prdida de la sensibilidad posicional; tpicamente en relacin con lesin de cordones posteriores medulares.
Considerarla si se refieren trastornos sensitivos asociados (sobre todo
de sensibilidad posicional, tctil profunda o vibratoria), en pacientes con alteraciones en la vitamina B12 (pensando en una degeneracin subaguda combinada de la mdula: operados de estmago, iletis terminal, anemia macroctica o megaloblstica), antecedentes de sfilis (tabes dorsal). Cursa con Romberg positivo, y con
ojos abiertos la marcha no tiene claras lateralizaciones, sino slo
problemas con el final del movimiento: los pies golpean fuertemente en el suelo (marcha taloneante). El caso de la pregunta podra serlo si no es por la descripcin de bipedestacin inestable
(que no hace coincidir con el cierre ocular, para pensar en un
Romberg) y los pasos desiguales.
Ataxia cerebelosa: es una marcha insegura, con pasos desiguales,
inestable con ojos abiertos y cerrados (aunque algo peor con ojos
cerrados). Si se acompaa de signos atxicos de extremidades, pensaremos en una lesin cerebelosa completa, como podra generar
una lesin ocupante de espacio en fosa posterior, pero si no se
encuentran ms signos cerebelosos que en la bipedestacin y en la
marcha, hay que pensar en una atrofia aislada de vermis.
Ataxia vestibular: la marcha es inestable, pero aparece lateralizacin de forma constante hacia el mismo lado. Generalmente se

Seguimiento a distancia

acompaar de vrtigo, con sensacin de giro de objetos. El Romberg


ser positivo hacia el mismo lado de la lateralizacin de la marcha.
Pregunta 18.- R: 2
Las lesiones en el lbulo occipital pueden dar lugar a distintos tipos
de clnica segn la localizacin exacta de la lesin:
Lesiones de corteza visual primaria: son exclusivamente visuales;
pueden cursar con cuadrantanopsias o hemianopsia; si son bilaterales, con visin en tnel (hemianopsia con respeto macular bilateral) o con ceguera cortical, que se acompaa habitualmente de
alucinaciones y de anosognosia (el paciente niega que no ve).
Lesiones de reas asociativas visuales: incluyen sndrome visuales
complejos con alteraciones en la percepcin o nominacin de los
colores, prosopagnosia con dificultad para reconocer las caras, dificultad para mantener la mirada fija en un punto (apraxia ptica),
dificultad para ver objetos de forma simultnea (simultanagnosia).
Las lesiones occipitales que afectan en parte a la porcin posterior
del cuerpo calloso, pueden originar sndromes de desconexin: el
paciente puede ser capaz de escribir pero no de leer (alexia sin
agrafia, asociada a lesiones occipitales izquierdas), de ver los objetos pero no de cogerlos con la mano (desconexin visuomanual) o
incapaz de dar nombre a un objeto o a un color que se le muestra
de forma visual (por desconexin del rea visual con el rea sensitiva del lenguaje).
La astereognosia o incapacidad para reconocer objetos por medio
del tacto es tpico de lesiones de la corteza parietal.
Pregunta 19.- R: 2
Conviene tener claros algunos conceptos en cuanto al campo visual:
La descripcin del dficit visual siempre hace referencia al rea
que el paciente deja de ver en su campo visual; esto supone que el
defecto que lo origina ser siempre contralateral (lesiones derechas
dficit izquierdo y al revs) e invertido (lesiones inferiores dficit
superior y al revs).
La homonimia y la heteronimia hacen referencia a si el defecto en el
campo visual coincide o no en ambos ojos en la misma parte del
campo; en el caso de la pregunta, describe un paciente que si
cierra el ojo izquierdo tiene prdida de visin superior izquierda y
si cierra el ojo derecho tiene prdida de visin superior izquierda:
defecto homnimo. Si al cerrar un ojo el defecto es superior izquierdo y al cerrar el ojo contralateral, la prdida de visin est en
otro cuadrante, es un defecto heternimo.

Pregunta 19. Defectos campimtricos y lesiones de la va ptica paraquiasmstica


M exico A rgentina
C hile U ruguay

CTO Medicina C/ Nez de Balboa, 115 28006 MADRID (Espaa) Tfno.: (91) 782 43 32 / Fax: (91) 782 43 27
E-mail: secretaria@ctomedicina.com; iberocto@ctomedicina.com WEB: www.ctomedicina.com; www.iberocto.com

NR Pg. 5

Los defectos heternimos son tpicos de las lesiones quiasmticas


(o de un paciente con dos lesiones distintas) mientras que los dficits homnimos son siempre retroquiasmticos.
La congruencia e incongruencia tienen valor en los defectos homnimos. Aunque el dficit visual coincida en ambos ojos, no siempre
es idntico: si el rea que se pierde en el campo superior izquierdo
(en el caso de la pregunta) es mayor en un ojo que en otro, tenemos una cuadrantanopsia homnima incongruente; si el defecto
es idntico, es congruente. Las alteraciones campimtricas heternimas no son superponibles, con lo que hablar de congruencia o
incongruencia no tiene sentido.
La congruencia entre los campos visuales de ambos ojos se va
ganando de forma progresiva a medida que nos acercamos a la
corteza occipital, de modo que un defecto plenamente congruente sugiere una lesin cortical; las lesiones de las radiaciones pticas
son menos congruentes que las occipitales, pero ms que las de
cintillas pticas.
Pregunta 20.- R: 4
La corteza sensitiva del lbulo parietal es la encargada de recibir los
estmulos sensitivos combinados; sera la encargada de la percepcin
global de la sensibilidad ms all del dato aislado de temperatura o
dolor o forma. De ah que una exploracin sensitiva completa deba
incluir el anlisis de datos concretos (recepcin de estmulo tctil
superficial roces-, tctil profundo toques-, dolor, temperatura, vibracin, posicin) y el anlisis de datos combinados (reconocer un
objeto, reconocer un dibujo hecho sobre la piel, reconocer varios
estmulos simultneos), siendo estos ltimos los que suponen una exploracin cortical parietal, y los primeros la informacin sobre las vas
sensitivas.
Otras funciones perceptivas del lbulo parietal hacen que sus lesiones, ms all del trastorno concreto del reconocimiento de datos
aislados, puedan conllevar trastornos ms amplios de la percepcin, a
lo que a veces se le denomina negligencia: el paciente puede no
reconocer el hemicuerpo afectado como suyo, no hacer caso a estmulos que vienen del lado afectado e incluso no ser claramente conscientes del dficit. Estos sntomas son ms frecuentes en las lesiones no
dominantes (derechas).
Por el contrario, cualquier referencia a alteraciones en una sensibilidad concreta (tctil superficial, vibratoria, etc; dolorosa en el caso de
la pregunta) nos obliga a pensar en lesiones infracorticales: si se habla
de anestesia, hay que pensar en el tlamo en los casos de hemianestesia
completa o en lesiones medulares en casos de anestesias con un nivel
bilateral. En la opcin 4, que habla de anestesia dolorosa podramos pensar en una lesin talmica o en una lesin aislada de la
sensibilidad termoalgsica (va espinotalmica).
Pregunta 21.- R: 2
La localizacin del lenguaje dentro del hemisferio izquierdo (en el
que se encuentran las funciones del lenguaje en la mayor parte de la
poblacin) no es de una certeza anatmica indiscutible. Se sita en
torno a la cisura de Silvio: reas temporales posterosuperiores
perisilvianas seran el rea posterior del habla y zonas posterobasales
del lbulo frontal, junto al lbulo parietal y sobre la cisura de Silvio, el
rea anterior del lenguaje. Estas reas se extienden ms all de las
clsicas reas de Wernicke y de Broca respectivamente.
En los pacientes en los que est alterada la produccin del lenguaje,
con una emisin de palabras escasa o nula se habla de afasia no
fluente, e indica lesin en reas anteriores del lenguaje; si el mismo
paciente conserva la capacidad de comprender rdenes que se le
dan, siempre que no tenga que responder verbalmente, indica que la
lesin es slo de reas anteriores: afasia motora.
En los pacientes en los que la principal alteracin es la comprensin
de las rdenes que se les pide verbalmente que ejecuten, se sospecha
lesin de reas posteriores del lenguaje; si adems se acompaa de un
lenguaje fluente (indicando que no hay afectacin de reas anteriores
del lenguaje), tenemos una afasia sensitiva. El lenguaje, a pesar de ser
fluente, ser ininteligible, con importantes alteraciones semnticas y
sintcticas.
Algunos pacientes, como en el caso de la pregunta, presentan un
habla fluida con parafasias, como el paciente con afasia sensitiva,
Pg. 6 NR

M exico A rgentina
C hile U ruguay

NEUROLOGA

Preparacin Examen de Seleccin 05/06 1 Vuelta

pero la comprensin est conservada, lo que no corresponde con


este diagnstico; tienen, sin embargo importantes alteraciones de la
repeticin: a esta afasia se la denomina de conduccin, la lesin suele
afectar a la circunvolucin supramarginal, al final de la cisura de
Silvio.
En las afasias, los dficits en el lenguaje escrito suelen coincidir con
los del lenguaje oral, aunque tienden a ser ms graves.
Pregunta 22.- R: 5
La descripcin del caso clnico es la caracterstica de un cuadro
confusional. Lo que determina fundamentalmente la existencia de
dicho cuadro es la inatencin, la baja interaccin del paciente con el
entorno. Puede presentar un cuadro de exceso de actividad, con
agitacin, o con cierta disminucin del nivel de conciencia.
El resto de los sntomas en realidad pueden relacionarse directamente con la dificultad de contacto con el explorador, con lo que la
desorientacin, el lenguaje incoherente y la incapacidad para retener
nuevos datos no indican que el paciente tenga un deterioro de funciones superiores de base, sino que est alterada la elaboracin de
procesos mentales, incluyendo la memoria, el lenguaje, el clculo y la
orientacin.
El cuadro confusional es una clnica comn a una infinidad de
etiologas, descritas en la tabla, siendo la ms frecuente los trastornos
txico-metablicos.
Consideracin aparte merece el delirium, cuya definicin corresponde a un cuadro confusional con hiperactividad vegetativa
(taquicardia, sudoracin, hipertensin arterial) y cuyo representante
tpico es el delirium tremens por deprivacin alcohlica.

Pregunta 22. Causas de cuadro confusional

123456478749
9  5 89
589529
2459 9
5 9 9
9  89
123456478749
245 89

123456789
26 8567 768 92826 764 2 7882 86 76 9
222 6 6789
12 789
2 6786  67892 69689278 67869
12!696
2769692 76 9278" 929 89289 7# 6789
12$ 6 8 7% 6 69
12&978 9 764 22' 2 76 26 766 
12(8 6 927 929'  7 86 29'' 
12) %  82 8 2782 88* 6 26+'68282687766 
123456478749
9 96 89

12,969
12(68 '76 2   768 92687 8# 67 9
12) 8567 768 9
Pregunta 23.- R: 5
El coma es una situacin de mala funcin cerebral global. El nivel
de conciencia depende del normal funcionamiento de la sustancia
reticular activadora (SRA) troncoenceflica y su proyeccin difusa a
toda la corteza cerebral. Cuando encontramos a un paciente en estado de coma hay que pensar en un trastorno difuso generalizado o
multifocal de la corteza o una lesin directa de la SRA.
En las lesiones difusas corticales, la tnica es la ausencia de focalidad neurolgica. Excepcin notable es la hipoglucemia severa en
ancianos, que puede cursar con hemiparesia. Suele haber una cierta
hipotona, puede haber movimientos horizontales pendulares de los
ojos, las funciones y reflejos del tronco cerebral estn conservadas, y
el patrn respiratorio es muy variable: puede ser normal, de tipo
Cheyne-Stokes, con hipoventilacin en algunas intoxicaciones (depresores del SNC) o hiperventilatorio (acidosis).
Las lesiones del tronco del encfalo son muy ricas
semiolgicamente: pueden tener alteraciones pupilares, alteracin o
abolicin de los reflejos del tronco (corneales, oculoceflicos, oculovestibulares), trastornos del ritmo respiratorio y con frecuencia reaccin de descerebracin (extensora) a estmulos dolorosos.
Aunque todas las lesiones del tronco del encfalo, a cualquier
nivel, pueden generar una situacin de coma, el trmino estado

CTO Medicina C/ Nez de Balboa, 115 28006 MADRID (Espaa) Tfno.: (91) 782 43 32 / Fax: (91) 782 43 27
E-mail: secretaria@ctomedicina.com; iberocto@ctomedicina.com WEB: www.ctomedicina.com; www.iberocto.com

Comentarios TEST

Seguimiento a distancia

NEUROLOGA

Preparacin Examen de Seleccin 05/06 1 Vuelta


comatoso, que aunque no muy correcto sugiere un paciente estabilizado en situacin de coma, no es muy compatible con una lesin
bulbar, dado que un dao a este nivel, en el que ya no hay SRA, lo
que produce es una alteracin primaria del ritmo respiratorio, altamente inestable, que puede ser mortal en pocas horas. Si la lesin no
afecta a los ncleos respiratorios, el paciente tendr focalidad deficitaria sin alteracin del nivel de conciencia, como puede suceder en el
Sndrome de Wallenberg.

Comentarios TEST

Pregunta 24.- R: 4
Las respuestas motoras anmalas son movimientos reflejos que
aparecen en lesiones del SNC. No siempre permiten la localizacin
exacta de la lesin que las origina, pero s pueden ser tiles en el
abordaje diagnstico.
La descerebracin o postura extensora cursa con extensin y
adduccin de las cuatro extremidades, con hiperpronacin de las
superiores. Aparece en lesiones mesenceflicas o protuberanciales
altas.
La decorticacin cursa con extensin de miembros inferiores y
flexin de miembros superiores, con adduccin; algunos autores hablan de hiperpronacin asociada y otros de supinacin, con lo que
asumimos que el dato ms importante es la flexin, que sirve para
distinguir la decorticacin de la descerebracin, dado que por lo que
respecta a los miembros inferiores son iguales.
Si el coma es muy profundo, puede no encontrarse ningn tipo de
movimiento, existiendo una hipotona generalizada. En casos en los que
no haya respuestas motoras a los estmulos, pero el nivel de conciencia
est relativamente conservado, hay que sospechar lesiones de las vas
motoras (tpicamente protuberanciales: sndrome de cautiverio).
En el caso de la pregunta, en el que se describe una postura de
descerebracin, esperaramos encontrar una situacin de coma bastante profundo (respuesta 5 falsa) con alteraciones de los reflejos y las
funciones del tronco (falsas la 2 y la 3) y alteraciones en el patrn
respiratorio que no corresponderan a un ritmo de Cheyne-Stokes
como est descrito en la opcin 1.
Una lesin ocupante de espacio en la fosa posterior puede causar
una compresin de tronco de forma directa o indirecta (herniaciones).
Pregunta 25.- R: 2
Las desviaciones forzadas de la mirada son tpicas de dos localizaciones distintas: la frontal y la protuberancial, no resultando difcil
distinguir entre ambas:
La lesin frontal cursa con una desviacin oculoceflica hacia el
lado del lbulo frontal lesionado (con una lesin frontal izquierda
el paciente viene mirando hacia la izquierda); lo que suele corresponder con el lado contrario de la hemiparesia (una lesin frontal
izquierda cursa con una hemiparesia derecha). En resumen: el
paciente con lesin frontal viene mirando hacia el hemicuerpo
que puede mover.
La lesin protuberancial cursa con desviacin ocular (ms que
oculoceflica) hacia el lado contrario a la lesin (una lesin protuberancial derecha presenta desviacin ocular hacia la izquierda),
lo que suele corresponder con el hemicuerpo deficitario (la lesin
de la va piramidal a su paso por la protuberancia genera una
hemiparesia contralateral, ya que se decusar en el bulbo raqudeo:
una lesin protuberancial derecha cursa con una hemiparesia izquierda). En resumen: el paciente con lesin protuberancial viene mirando hacia el hemicuerpo partico.
Por supuesto hay otra serie de datos que nos ayudan a diferenciar
entre ambas: la lesin protuberancial es probable que deje al paciente
en coma, mientras que la frontal no tiene por qu alterar el nivel de
conciencia; la distribucin del dficit generalmente es de predominio
faciobraquial en las lesiones frontales; las lesiones protuberanciales asociarn probablemente alteraciones pupilares; las lesiones frontales pueden asociar otros dficits (afasia motora en hemisferio dominante).
Pregunta 26.- R: 3
Lo cierto es que definir una respiracin como taquipneica no es
sugerente de ninguna lesin en ninguna zona concreta, pero si es lo
M exico A rgentina
C hile U ruguay

Seguimiento a distancia

que tenemos como dato en un paciente en coma, hay que interpretarlo literalmente: el paciente respira rpido.
Qu patrones respiratorios conocemos en el coma?:
Respiracin bulbar: atxica, irregular, con paradas de apnea. No
podemos considerarla taquipneica, aunque haga ciertas fases de
respiracin rpida y superficial.
Respiracin protuberancial: apnustica, con pausas entre inspiracin y espiracin y la siguiente inspiracin; casi justo lo contrario
de taquipneico.
Respiracin dienceflica: de Cheyne-Stokes, hemisfrica profunda, en la que hay fases de hiperventilacin seguidas de pausas de
apnea, que se suceden a intervalos regulares. Tpica de las lesiones
hemisfricas bilaterales profundas, o en algunos casos de afectacin encefaloptica difusa.
Respiracin mesenceflica: hiperventilacin rtmica, que s consideramos taquipneica. Probablemente encontraremos en la exploracin otros signos de afectacin mesenceflica, porque si no habra que pensar en la acidosis como causa (en ese caso, a la respiracin rpida y rtmica relacionada con la acidosis, se le denomina
respiracin de Kussmaul).
Pregunta 27.- R: 5
Ante un paciente comatoso, la exploracin de las pupilas y su
reactividad es especialmente til, ya que se alteran en lesiones del
tronco del encfalo y aportan informacin de dnde puede estar
situada la lesin. La existencia de pupilas isocricas y normorreactivas
indica que el tronco cerebral del paciente permanece sin lesin.

Pregunta 27. Respuestas pupilares caractersticas de las lesiones enceflicas

Ante un paciente que presenta una situacin de coma sin afectacin del tronco cerebral, nos queda la posibilidad de una lesin
hemisfrica profunda o una afectacin difusa de toda la corteza, lo
que conocemos como encefalopatas.
El hecho de que este paciente presente un patrn respiratorio de
hiperventilacin rtmica nos debe hacer pensar en una encefalopata,
ya que el patrn ms frecuentemente asociado a la lesin hemisfrica
profunda es la respiracin de Cheyne-Stokes, que cursa con ritmo
regular de hiperventilacin-apnea, por aumento de sensibilidad de
los centros inspiratorios a la concentracin de CO2.
La hiperventilacin rtmica es un signo clsico del coma acidtico,
denominndose respiracin de Kussmaul; no es exclusiva de la acidosis, pero es la acidosis metablica de cualquier origen la que se
relaciona tpicamente con ella.
Pregunta 28.- R: 4
La descripcin del cuadro clnico es lo que nos debe llevar la
mxima atencin:
Incapacidad para realizar cualquier movimiento voluntario salvo
parpadeo y movimientos oculares verticales: los movimientos
oculares horizontales conjugados tienen su rea de coordinacin en la protuberancia, y desde all se dirigen las rdenes a los
III pares para los movimientos de adducin (por los fascculos
longitudinales mediales), mientras que los movimientos oculares
verticales se organizan en el mesencfalo, donde se sitan los III
pares. A nivel mesenceflico se origina tambin el movimiento de

CTO Medicina C/ Nez de Balboa, 115 28006 MADRID (Espaa) Tfno.: (91) 782 43 32 / Fax: (91) 782 43 27
E-mail: secretaria@ctomedicina.com; iberocto@ctomedicina.com WEB: www.ctomedicina.com; www.iberocto.com

NR Pg. 7

convergencia, en el que, a pesar de ser horizontal, slo intervienen los terceros pares, con adduccin de ambos ojos
Estado alerta: esto implica que es capaz de interaccionar con el
entorno; probablemente significa que sigue objetos con la mirada
en el plano vertical, que se produce cierre palpebral ante la aproximacin de un objeto al campo visual (reflejo de amenaza) o incluso que es capaz de mantener una conversacin si se le da la oportunidad de contestar s o no con un determinado nmero de
parpadeos. Este dato induce a pensar en una conservacin de la
sustancia reticular activadora (SRA) troncoenceflica.
Cmo un paciente con un dficit motor que incluye la parte alta
de la protuberancia como lmite superior puede no tener lesin de
la SRA, que est situada en el tronco del encfalo, es la respuesta
que hay que dar: el sndrome de cautiverio.
El sndrome de cautiverio est originado por una lesin desmielinizante que afecta a las vas motoras descendentes de la protuberancia,
sin afectar al nivel de conciencia. Una RM cerebral puede mostrar
desmielinizacin difusa protuberancial.
Las lesiones que pueden dar esta clnica pueden ser de mltiples
etiologas, incluyendo la idioptica, pero el antecedente de una paciente que ha presentado antes un cuadro confusional que ha sido
atendido mdicamente debe hacer pensar en una hiponatremia corregida demasiado rpidamente, que puede provocar una lesin de
sustancia blanca protuberancial.
Pregunta 29.- R: 1
El cerebro es el tejido con mayor sensibilidad a la falta de perfusin, comenzando a producirse lesiones irreversibles tras escasos minutos de hipoxia, aunque no todas las zonas del encfalo sufren en la
misma medida. Las reas ms sensibles son las descritas en la opcin
3, de tal manera que un paciente que se recupere de forma incompleta y le quede alguna secuela tras una hipoxia cerebral es ms
frecuente que presente ataxia, alteraciones de memoria y trastornos
motores extrapiramidales.
La hipoxia cerebral puede producirse por cualquier causa que
impida la llegada de flujo sanguneo oxigenado al cerebro, sea por
parada cardiaca de cualquier origen, asfixia, o compresin vascular.
Es la posibilidad de sufrir un dao cerebral irreversible, lo que obliga
a realizar desde el primer momento de una parada cardiaca maniobras de resucitacin, porque si se recupera flujo pasados 10-15 minutos, el paciente probablemente permanezca en situacin de coma.
El cuadro denominado encefalopata hipxica retardada se presenta en pacientes que, tras sufrir el proceso de hipoxia aguda, recuperan aparentemente sin secuelas, y stas aparecen en los das siguientes, debido a una desmielinizacin de las reas afectadas, que se
instaura de forma subaguda.
Las reas cerebrales lesionadas no tienen una regeneracin en lo
que respecta a tejido neuronal: se produce una cicatriz glial que
puede dar lugar a crisis epilpticas tardas.
Pregunta 30.- R: 2
Un paciente EPOC con descompensacin respiratoria presentar
problemas ms frecuentemente por la retencin excesiva de CO2 que
por la hipoxia, debido a los cambios que se producen en los pacientes con hipoxia crnica, que permiten tolerar presiones bajas de O2.
La hipercapnia produce una vasodilatacin cerebral con el consiguiente aumento de presin intracraneal, con lo que la retencin de
CO2 cursar con toda la clnica que acompaa a la hipertensin
intracraneal. En la exploracin se puede encontrar edema de papila
en toda patologa que aumente la presin intracraneal.
El paciente puede presentar una cefalea holocraneal, opresiva no
especfica, pero quiz sea ms sugerente de hipertensin intracraneal
la presencia de vmitos en escopetazo.
La asterixis, aunque clsicamente descrita asociada a la encefalopata heptica, puede presentarse asimismo en algunas otras encefalopatas, entre ellas la hipercpnica. El paciente tendr alteracin del
nivel de conciencia.
El temblor de reposo es tpico de los sndromes parkinsonianos. Su
aparicin es de curso lentamente progresivo; es difcil asociarlo a un
cuadro brusco y menos a una descompensacin respiratoria, salvo
que se tenga una patologa previa.
Pg. 8 NR

M exico A rgentina
C hile U ruguay

NEUROLOGA

Preparacin Examen de Seleccin 05/06 1 Vuelta

Pregunta 31.- R: 4
El coma hipoglucmico es una encefalopata metablica, con afectacin difusa de la corteza. Es exclusiva de diabticos tratados, dado
que en un organismo sin patologa, la gluconeognesis impide que se
llegue a esa situacin de forma fisiolgica. El cerebro es un rgano
selectivo y slo utiliza glucosa como fuente de energa, salvo en condiciones de ayuno prolongado, en el que se producen los cambios
metablicos suficientes para permitir que se consuman cuerpos
cetnicos.
Tanto los antidiabticos orales como la insulina pueden producir
una hipoglucemia lo suficientemente mantenida como para producir
una situacin de coma.
De forma tpica, por muy rpidamente que se produzca el descenso de los niveles de glucemia, el cuerpo tiene una primera reaccin de
stress que consiste en la liberacin de catecolaminas; esto lleva a un
cuadro clnico hiperadrenrgico, con sudoracin, taquicardia, boca
seca, midriasis y agitacin, que es seguido de la disminucin del nivel
de conciencia, generalmente en minutos-horas (dependiendo del ritmo de accin de la terapia hipoglucemiante).
Ante un paciente en coma del que no podemos obtener cifras de
glucemia, se trate o no de un diabtico conocido en tratamiento (pero
especialmente si se conoce este antecedente) una de las primeras medidas a tomar es la administracin de glucosa por va endovenosa.
Pregunta 32.- R: 3
La trada de oftalmoplejia externa (alteraciones de la motilidad del
globo ocular sin afectacin pupilar o de la acomodacin), ataxia y
cuadro confusional es la tpica de la encefalopata de Wernicke por
dficit de vitamina B1 (tiamina). La teora dice que la trada se instaura
en ese orden y que tras la terapia de reposicin el orden de recuperacin de los sntomas tambin es el mismo: primero la oftalmoparesia,
posteriormente la ataxia y por ltimo el cuadro confusional.
En la prctica lo que nos interesa es no dejar pasar ninguna encefalopata de Wernicke sin sospecharla. Habr que estar atentos a la
presencia de signos de malnutricin o de alcoholismo previos a la
presentacin de la nueva clnica.
Ante la sospecha de sndrome de Wernicke est indicada la administracin de 100 mg de tiamina por va parenteral; la disponibilidad
de la vitamina por va intramuscular es prcticamente inmediata, no
teniendo en principio claras ventajas la intravenosa, pero s la va
parenteral sobre la enteral, dado que estamos hablando con un paciente malnutrido que probablemente asociar problemas de
malabsorcin digestiva.
Hay que intentar evitar la administracin de glucosa intravenosa en
un paciente con sospecha de Wernicke antes de la tiamina, ya que
puede desencadenar o empeorar la encefalopata.
Lo ms indicado en un paciente con dficit nutricional combinado es la administracin de un combinado polivitamnico, ya que la
carencia de otras vitaminas tambin puede producir sntomas neurolgicos.
AMNESIAS. DEMENCIAS.
Pregunta 33.- R: 3
La memoria es la capacidad para almacenar datos (y rescatarlos
cuando es preciso). En principio se puede asumir que es una funcin
repartida de forma difusa por todo el cerebro, aunque hay algunas
reas (hipocampo-cerebro basal) cuyas lesiones provocan importantes alteraciones de la memoria.
Aunque hay distintas clasificaciones, es prctico recordar algunos
datos bsicos:
Memoria inmediata: se valora pidiendo al paciente que repita lo
que se le dice; depende sobre todo de la atencin; en pacientes
con alteraciones importantes del resto de funciones mnsicas, la
memoria inmediata se conserva si mantienen un buen nivel de
conciencia.
Memoria reciente: se valora pidiendo al paciente que retenga algn dato durante varios minutos, mientras se le encarga alguna otra
tarea, y pidindole que lo repita ms tarde.
Memoria remota: se le pide al paciente que aporte datos de hace
aos, biogrficos o histricos.

CTO Medicina C/ Nez de Balboa, 115 28006 MADRID (Espaa) Tfno.: (91) 782 43 32 / Fax: (91) 782 43 27
E-mail: secretaria@ctomedicina.com; iberocto@ctomedicina.com WEB: www.ctomedicina.com; www.iberocto.com

Comentarios TEST

Seguimiento a distancia

NEUROLOGA

Preparacin Examen de Seleccin 05/06 1 Vuelta


Otra clasificacin dividira las amnesias en retrgrada, cuando se
olvidan datos que previamente se conocan, y antergrada, cuando el
paciente no es capaz de retener nueva informacin.
Lo cierto es que actualmente no existe un tratamiento para los dficits
aislados de memoria, fuera de los tratamientos globales para enfermedades degenerativas que se acompaen de amnesias (demencias).

sencia de claros dficits neurolgicos focales. El prototipo es la


enfermedad de Alzheimer.
Subcorticales: hay fundamentalmente un enlentecimiento global
de los procesos mentales, sin claras alteraciones corticales (afasia,
apraxia, agnosia) y con fallos de memoria inicialmente menos llamativos.

Pregunta 34.- R: 2
La amnesia global transitoria es lo que presenta el paciente. Hay
que tener claro que el cuadro tpico tiene las siguientes caractersticas:
Aparicin brusca y duracin menor de 24 horas
El paciente suele repetir las mismas preguntas, pero no olvida quin
es, no est afsico ni confuso, puede realizar acciones complejas
(vestirse, conducir) sin aparente dificultad
Predomina el defecto amnsico antergrado: el paciente no retiene datos durante ms de 1-2 minutos; puede presentar alguna
laguna retrgrada, menos llamativa.
Se desconoce la etiologa de la amnesia global transitoria. Se ha
postulado origen isqumico (pero el paciente no presenta ms riesgo
antes ni despus de ictus que la poblacin de su mismo rango de
edad, lo que no se correspondera con ataque isqumico transitorio)
o epilptico (pero la duracin de las crisis es menor, el EEG realizado
durante la crisis no muestra alteraciones que la justifiquen y el paciente no tiene otro tipo de crisis epilpticas).
El factor desencadenante del episodio puede ser muy variable:
esfuerzos fsicos intensos, estrs emocional, coito, ataque de migraa,
ingesta de frmacos.
Tratamiento no tiene, puesto que se desconoce la etiologa. Lo que
s se conoce es el buen pronstico: al paciente le queda como secuela una laguna de memoria correspondiente al tiempo de la crisis, pero
la incidencia de recurrencias es baja.

El enunciado de la pregunta describe una demencia subcortical,


entre las cuales no se encuentra la enfermedad de Alzheimer, que es,
por el contrario, la demencia cortical por excelencia (y por frecuencia).

Pregunta 35.- R: 3
Hay que recordar que las apraxias, las agnosias y las afasias son
sndromes clnicos que se presentan nicamente en procesos que
afecten a la corteza hemisfrica, a diferencia de otros sndromes (prdida de fuerza, disartria, anopsias) que pueden ser clnicamente comunes pese a poder estar causados por lesiones de distintas localizaciones.

Pregunta 35. Diagnstico diferencial entre


demencia cortical y subcortical

Comentarios TEST

Seguimiento a distancia

424967
 529 85 466
16569
42
965 65 9
25667
4 379
3  29
296 429

1232456789
5 657

1232456785 657

123245465728979

4 8932

425

5869

87794724626589472
54859

62 9499 949899


9 5 9

9947282 

282 48222

 98894 78562
54 9354249 2 492
82685 8 75 95
! 897544 965

"582#949 5939

La definicin de demencia incluye un deterioro global (con afectacin de distintas reas) de las funciones intelectuales previas, con conservacin del nivel de conciencia y en ausencia de una lesin orgnica
cerebral definida.
Se quedan por tanto fuera de esta definicin los trastornos de alguna funcin intelectual aislada (memoria, afasia, acalculia, etc), los
defectos en la adquisicin de dichas funciones (oligofrenias), los comas y los procesos secundarios (encefalopatas).
Una vez diagnosticado el paciente de demencia, tambin existen
varios subtipos: la principal diferencia a recordar est entre las demencias:
Corticales: el sntoma inicial es la prdida de memoria, y se van
desarrollando progresivamente afasia, apraxias y agnosias en auM exico A rgentina
C hile U ruguay

Pregunta 36.- R: 4
La enfermedad de Alzheimer es en efecto la causa ms frecuente
de demencia en el mundo occidental. Su etiologa es hasta hoy desconocida, habindose postulado numerosos factores que podran
influir en su aparicin.
Los hallazgos anatomopatolgicos son los que proporcionan el
diagnstico de certeza, por lo que en realidad no se alcanza in vivo.
Pruebas complementarias funcionales, tales como el SPECT, que
muestren una hipoperfusin frontotemporal pueden apoyar el diagnstico clnico.
Las lesiones histolgicas bsicas son:
La degeneracin neurofibrilar: acmulos de neurofilamentos degenerados; su componente principal son protenas fosforiladas,
ubiquitina y amiloide. La intensidad de esta degeneracin es lo
que ms parece correlacionarse con el grado de demencia.
Las placas neurticas: acmulo de neuritas distrficas con clulas
microgliales que con el paso del tiempo tienen un depsito central
de amiloide (placas maduras). Se encuentra numerosa protena
tau.
Depsito de amiloide: el amiloide se forma a partir de una protena
codificada en el cromosoma 21; se deposita en el centro de las
placas y en las paredes de los vasos de pequeo calibre cerebrales.
Las lesiones anatomopatolgicas bsicas no son patognomnicas
de la Enfermedad de Alzheimer (opcin falsa), y es slo su hallazgo de
forma difusa y la densidad de las lesiones lo que es diagnstico.
En la Enfermedad de Alzheimer se desarrolla un dficit de todos los
neurotransmisores, si bien los sistemas colinrgicos parecen los ms
afectados, pero en este caso no slo por la reduccin presinptica de
neurotransmisor, sino tambin por la prdida de receptores.
Pregunta 37.- R: 2
Hoy existen en el mercado frmacos comercializados para el tratamiento de la Enfermedad de Alzheimer. Todos comparten el mecanismo de accin de ser inhibidores de la acetilcolinesterasa a nivel cerebral, dado que ha sido el nico mecanismo farmacolgico que ha
demostrado eficacia teraputica in vivo.
Estn indicados en las fases leve y moderada de la enfermedad,
dado que en fases avanzadas el deterioro es tan acusado que una
potenciacin de la actividad colinrgica no consigue unos beneficios
tangibles frente a los potenciales efectos secundarios.
Obtienen mejora en todos los aspectos de la enfermedad: el deterioro de funciones superiores, las alteraciones del comportamiento y
la capacidad funcional global del sujeto. La dosis debe aumentarse de
forma creciente, dependiendo de la tolerancia de los efectos secundarios, que generalmente se producen a nivel digestivo.
Otras terapias no han demostrado eficacia, a pesar de que se ha
intentado actuar sobre los distintos mecanismos que en principio influyen en la enfermedad: formacin de depsitos amiloides, otros
neurotransmisores, fenmenos oxidativos, etc.
Pregunta 38.- R: 1
El diagnstico de demencia se establece cuando existe afectacin
de distintas reas de funcionamiento cerebral, de forma progresiva,
sin que existan enfermedades subyacentes que puedan influir en ese
proceso, y con conservacin del nivel de conciencia. Si no existe un
buen nivel de conciencia no se puede evaluar la afectacin de las
funciones superiores, y no es posible conocer si el deterioro simplemente es transitorio por una causa que, una vez corregida, no deje
una alteracin establecida.

CTO Medicina C/ Nez de Balboa, 115 28006 MADRID (Espaa) Tfno.: (91) 782 43 32 / Fax: (91) 782 43 27
E-mail: secretaria@ctomedicina.com; iberocto@ctomedicina.com WEB: www.ctomedicina.com; www.iberocto.com

NR Pg. 9

Esto es vlido para todos los tipos de demencia, corticales y subcorticales.


En el caso de las demencias corticales, principalmente tendremos
problemas de memoria, en especial y en fases iniciales se altera ms la
memoria reciente o de fijacin, mientras que el paciente an es capaz
de recordar datos biogrficos antiguos. En fases avanzadas incluso esa
memoria remota se altera y se pierde definitivamente.
Las apraxias pueden describirse como que el paciente tiene dificultad para vestirse, para realizar acciones simples que antes saba
(abrir una puerta con una llave, manejar un mando a distancia). Las
agnosias pueden ser evidentes en la incapacidad para reconocer los
rostros de los familiares, u objetos cotidianos (un reloj, una llave). La
afasia inicialmente es nominal: el paciente tiene un lenguaje espontneo fluente, pero cuando se le pide que nombre alguna cosa es incapaz de extraer ese nombre de su memoria, aunque lo haya usado de
forma espontnea.
Pregunta 39.- R: 1
Las demencias corticales se diferencian de las subcorticales en el
contexto clnico por una mayor alteracin de las funciones superiores: memoria, clculo, lenguaje, acciones motoras complejas; mientras que las demencias subcorticales conservan esas funciones, en el
contexto de un funcionamiento globalmente enlentecido. Hay, no
obstante, muchas situaciones clnicas en las que resulta muy difcil
diferenciar entre ambas entidades.
Por lo general, los trastornos denominados extrapiramidales, cuando asocian demencia, suele ser de caractersticas subcorticales, y lo
mismo sucede con las demencias de origen vascular.
La enfermedad de Pick es una demencia muy similar clnicamente
a la enfermedad de Alzheimer, en la que predominan las alteraciones
de caractersticas frontales: desinhibicin, alteracin de la personalidad previa, lenguaje verborreico, incontinencia de esfnteres. En fases evolucionadas se produce un deterioro global.
En las pruebas de imagen aparece una atrofia selectiva frontal y
temporal inferior, ms selectiva que en el Alzheimer, sin que se sepa la
razn. La anatoma patolgica es la diagnstica, con la aparicin de
neuronas degeneradas (clulas de Pick) o con inclusiones intraneuronales de filamentos anmalos, similares a los de la degeneracin
neurofibrilar del Alzheimer (grnulos de Pick).
No existe tratamiento para esta enfermedad.
Pregunta 40.- R: 4
La primera premisa para responder la pregunta es descifrar el cuadro clnico:
Olvidos frecuentes de forma progresiva en paciente mayor: sugiere
demencia incipiente (otra cosa sera un olvido global de forma
brusca: amnesia global transitoria).
Trastorno de la marcha: aunque no est especificado el tipo de
trastorno, una demencia con trastorno de la marcha de tipo aprxico
ayudara a pensar en una demencia cortical, mientras que una
marcha a pequeos pasos apunta ms a patologa subcortical.
Sacudidas musculares breves y arrtmicas: definicin de mioclonas; desencadenadas por sobresaltos? mioclonas reflejas.
Una demencia progresiva con mioclonas debe hacer pensar automticamente en una enfermedad de Creutzfeldt-Jakob. De hecho, la
trada tpica es una demencia con mioclona y un EEG caracterstico
con las denominadas ondas trifsicas, aunque el cuadro suele completarse con signos cerebelosos, piramidales, extrapiramidales, crisis
epilpticas y alteraciones de segunda neurona motora.
El diagnstico de esta enfermedad se basa en el contexto clnico
coherente, con el EEG compatible; las pruebas complementarias suelen ser negativas. En el LCR, que puede mostrar un aumento inespecfico de las protenas, se detecta la protena 14-3-3, que no es especfica ni sensible al 100%, pero que aparece en la mayor parte de los
casos.
El curso clnico es fatal en el transcurso de 6-12 meses, con muerte
generalmente producida en relacin con complicaciones respiratorias.
El diagnstico de certeza es el anatomopatolgico, con el hallazgo
de una espongiosis con escasa actividad inflamatoria.
Pg. 10 NR

M exico A rgentina
C hile U ruguay

NEUROLOGA

Preparacin Examen de Seleccin 05/06 1 Vuelta

Pregunta 41.- R: 3
Hay que recordar que los priones son partculas de naturaleza
proteica y que se comportan como agentes infecciosos. Todo el mundo posee la protena precursora de la protena prinica (PrP), de la
que an no se conoce claramente su funcin. Su cdigo gentico est
en el cromosoma 20.
En algunas familias, una mutacin en el cromosoma 20 codifica de
forma anmala la protena precursora, con lo que se produce la
enfermedad, con debut generalmente en la vida adulta (5 dcada):
son la enfermedad de Gerstmann-Straussler-Scheinker, el insomnio
familiar fatal y alguna variante familiar de la enfermedad de CreutzfeldtJakob.
Una modificacin en la estructura terciaria de la protena normal
(sin cambios en la codificacin gentica) puede hacer que dicha protena se agregue en forma de un depsito amiloide resistente a la actividad de las proteasas, generando escasa actividad inflamatoria y una
destruccin de caractersticas espongiformes. Esta modificacin la puede conseguir el contacto con la protena prinica; estas son las variantes
infecciosas: las variantes adquiridas de la enfermedad de C-J clsica, la
nueva variante del CJ y el kuru.
El mecanismo por el que la protena consigue llegar hasta el sistema
nervioso central y al propio interior de las clulas hasta conseguir la
transformacin de las protenas precursoras en protenas prinicas
permanece desconocido.
La enfermedad de Strachan o neuritis jamaicana es una enfermedad de carcter adquirido, de origen carencial, aunque sin corresponder a ninguno de los sndromes carenciales clsicos por dficit de
una vitamina (beri-beri, pelagra), que presenta polineuropata (con
dolor en extremidades y abolicin de reflejos), ataxia, disminucin de
visin y audicin, y que responde a la terapia polivitamnica.
Pregunta 42.- R: 5
La enfermedad de Binswanger en realidad debera diagnosticarse
nicamente post mortem, ya que en su diagnstico incluye que exista
una desmielinizacin difusa subcortical con ausencia de infartos extensos o de una gran nmero de infartos lacunares, junto con
lipohialinosis y esclerosis de arteriolas.
Se trata de pacientes con factores de riesgo vascular, bsicamente
hipertensin, que presentan en las pruebas de imagen una alteracin
en la densidad o la seal de la sustancia blanca que sugiere
desmielinizacin y que se denomina leucoaraiosis.
En la prctica, ante un paciente con deterioro cognitivo subcortical y la presencia de una prueba de imagen (TC o RM) sugerente, se
considera que estamos ante el mismo cuadro, se denomine enfermedad de Binswanger o no.
De modo que un paciente que presente:
- deterioro de caractersticas subcorticales con parkinsonismo, rigidez, urgencia miccional, trastornos de la marcha;
- factores de riesgo arteriosclertico (hipertensin arterial, diabetes,
isquemia coronaria)
- infartos lacunares previos, y
- leucoaraiosis bilateral extensa en las pruebas de imagen;
queda diagnosticado de encefalopata isqumica subcortical.
No existe un tratamiento que revierta los sntomas, ya que cuando
estamos en fase sintomtica, indica que el dao cerebral est establecido. El control estricto de los factores de riesgo aspira a prevenir una
progresin ms acusada.
De todos modos, conviene recordar que la forma ms frecuente de
demencia vascular es la que se produce por infartos mltiples, generalmente de predominio subcortical, y que en ocasiones no es fcil de
separar de una enfermedad de Alzheimer. Para distinguir entre ambas
se ha utilizado la escala isqumica de Hachinski, que valora datos
clnicos que apoyen ms un tipo u otro de demencia.
EPILEPSIA.
Pregunta 43.- R: 3
Las crisis parciales son crisis focales elctricamente, aunque algunas puedan tener sntomas que obligan al diagnstico diferencial con
crisis generalizadas; el ms importante es la desconexin del medio,
que por definicin comparten las crisis generalizadas, y que tambin

CTO Medicina C/ Nez de Balboa, 115 28006 MADRID (Espaa) Tfno.: (91) 782 43 32 / Fax: (91) 782 43 27
E-mail: secretaria@ctomedicina.com; iberocto@ctomedicina.com WEB: www.ctomedicina.com; www.iberocto.com

Comentarios TEST

Seguimiento a distancia

NEUROLOGA

Preparacin Examen de Seleccin 05/06 1 Vuelta


pueden presentar algunas crisis focales. El diagnstico ms difcil est
entre las crisis de ausencia con las crisis parciales complejas, que
pueden ser clnicamente muy similares.
Otro lmite especialmente difcil se sita entre las crisis parciales
simples con sntomas psquicos y las crisis con alteracin del nivel de
conciencia, dado que en muchas ocasiones los propios sntomas psquicos pueden conducir a trastornos en el nivel de alerta, o ser ya
parte de la crisis parcial compleja. En cualquier caso, a efectos de un
caso terico, se debe especificar que hay alteracin del nivel de conciencia para diagnosticar de crisis parciales complejas y no simples.
La mayora de las crisis parciales complejas tienen foco en el lbulo
temporal, lo que no implica que todas las crisis que tienen su origen en
dicho lbulo cursen con alteracin del nivel de conciencia.
De igual modo, la etiologa subyacente no es determinante en el
curso clnico de los episodios comiciales, ni el tipo exacto de los
mismos implica mejor o peor pronstico de la patologa de base.

Comentarios TEST

Pregunta 44.- R: 3
El paciente presenta desconexin del medio, lo que inicialmente
nos reduce el diagnstico diferencial, de entre las opciones, a la 3 o la
4. Las crisis generalizadas no convulsivas no existen como tales en las
clasificaciones actuales de las crisis epilpticas, salvo que se les quiera
aplicar ese nombre a lo que conocemos como ausencias, y las crisis
parciales simples no alteran el nivel de conciencia.
No es sencillo distinguir entre crisis de ausencia y las crisis parciales
complejas que cursan casi como ausencias, y bsicamente nos tiene
que hacer descartar la ausencia el hecho de que aparezca algn
sntoma incompatible con la misma.
De entre los datos del caso clnico, la duracin es poco sugerente
de ausencias, que suelen durar segundos, pero no es un signo incompatible, como lo son la sensacin epigstrica previa a la crisis y la
confusin posterior al episodio. La presencia de estos sntomas nos
hacen descartar el diagnstico de ausencias tpicas; tendramos que
conformarnos con el de ausencias atpicas, que es un trmino menos
definido en el que se incluyen las crisis que cursan predominantemente con prdida del nivel de conciencia y escasos sntomas motores. Pero preferiblemente habr que elegir la opcin de las crisis parciales complejas por ser mucho ms exacta.
Pregunta 45.- R: 2
Los episodios repetidos de movimientos involuntarios sugieren crisis motoras. Si no conllevan deterioro del nivel de conciencia, se
tratarn de crisis parciales simples, ya que tanto las crisis generalizadas
como las crisis parciales complejas asocian alteracin del nivel de
conciencia. El hecho de que se desplacen o extiendan a lo largo del
mismo hemicuerpo indica que la actividad elctrica tambin se desplaza o extiende, pero a lo largo de la circunvolucin motora primaria del lbulo frontal. A este tipo de crisis, descritas por Jackson, se le
llama jacksoniana, pero no hay generalizacin con afectacin del
resto de la corteza.
En cuanto a la causa, dado el antecedente de un meningioma
intervenido, habr que descartar una recidiva del tumor, pero no es
imprescindible la existencia de lesin ocupante de espacio. Dado
que est intervenido, es posible que haya habido una lesin quirrgica o por compresin por el tumor previo que puede generar la crisis,
sin necesidad de ms tumor.
Por ltimo, dado que los sntomas motores comienzan en la mano
derecha, localizamos el punto de inicio en el lbulo frontal izquierdo,
en el rea correspondiente a las primeras motoneuronas encargadas de
la extremidad superior, que no estn en la hoz (donde se sita la raz del
muslo), sino a un nivel inferior, en la circunvolucin motora.
Pregunta 46.- R: 3
Las causas de crisis epilpticas de debut en adulto joven son fundamentalmente los traumatismos previos y las lesiones ocupantes de
espacio, de las cuales la ms frecuente es la metstasis de un tumor
primario en otra localizacin. De stos, se necesita un traumatismo al
menos moderado para poder considerarlo causante de un foco epileptgeno, mientras que no necesitamos antecedente tumoral para
encontrar una metstasis; de hecho, no es infrecuente que algunos
tumores den sus primeros sntomas por metstasis intracraneales. ReM exico A rgentina
C hile U ruguay

Seguimiento a distancia

cordar que se tratarn de lesiones con captacin de contraste en


anillo, con edema digitiforme o vasognico alrededor, con importante efecto de masa.
Ante la sospecha de una lesin ocupante de espacio, est indicada
la realizacin de una prueba de imagen. No necesariamente tiene
que ser urgente si lo entendemos como sinnimo de inmediata,
pero no puede dilatarse mucho, puesto que pensamos en un tumor.
La prueba de imagen que ms datos aportar ser la RM, pero una TC
puede ser tambin diagnstica.
El sndrome clnico que presenta el paciente parecen ser crisis
parciales tnicas (se detalla que hay alteracin del nivel de conciencia, luego son crisis parciales complejas) y el hecho de que en una
ocasin se presente un episodio tonicoclnico generalizado simplemente indica extensin de la actividad elctrica anmala a lo largo de
toda la corteza.
Pregunta 47.- R: 3
Los sntomas que presentan las crisis epilpticas dependen, lgicamente, de la funcin del rea cerebral en el que se asienta la anomala
elctrica paroxstica. Podemos tambin, conociendo la sintomatologa,
suponer a la inversa dnde situar el foco epilptico.
En el lbulo frontal se sita la corteza motora primaria, y por eso
encontraremos movimientos involuntarios en el hemicuerpo contralateral; asimismo cada lbulo frontal coordina la desviacin oculoceflica hacia el lado contrario, que se produce cuando se estimula la
corteza; por eso el paciente mira al hemicuerpo que tiene los movimientos involuntarios.
Las parestesias hemicorporales paroxsticas sugieren un foco en
corteza parietal contralateral.
Las luces centelleantes paroxsticas en un hemicampo sugieren
focalidad occipital; hay que establecer el diagnstico diferencial entre crisis y episodios de aura migraosa, que es ms frecuentemente
occipital. Las crisis suelen tener inicio y fin bruscos, y suelen ser de
corta duracin (segundos, escasos minutos), mientras que el aura visual migraosa suele durar varios minutos (menos de media hora) y se
siguen de cefalea hemicraneal en el lado contrario a las fotopsias
(aunque pueden existir auras sin migraa posterior).
La sensacin de bolo epigstrico ascendente puede ser
premonitoria de una crisis generalizada, aunque no es privativa de las
mismas, y pueden aparecer en las crisis parciales complejas y en las
denominadas ausencias atpicas.
Pregunta 48.- R: 2
Hay que recordar que el sndrome de West debuta ms frecuentemente entre los 3 y los 6 meses y siempre en el primer ao. En la mayor
parte de los casos (60%) existe patologa subyacente, y es en estos casos
en los que el pronstico a medio-largo plazo es peor: el paciente suele
quedar con una afectacin psicomotriz importante y suelen ser epilepsias crnicas de difcil tratamiento. En los casos (40%) en los que no se
encuentra patologa subyacente el pronstico es mejor.
Las crisis de espasmos se presentan generalmente al despertar, sin
que se necesite un estmulo especial para desencadenarlas, y conviene no olvidar que la hipsarritmia (el EEG que se considera parte de la
clnica caracterstica del sndrome) es interictal, es decir, cuando el
paciente no tiene crisis (opcin 4, falsa). Los espasmos que se consideran ms frecuentes varan segn las series, pero probablemente sean
los flexores y los mixtos (flexo-extensores).
El diagnstico no es difcil desde un punto de vista sindrmico; hay
que realizar un amplio despistaje para la patologa de base.
Elegir entre los tratamientos que han demostrado eficacia en el
sndrome de West no es fcil, ya que hay varios frmacos tiles: el
clsico es con ACTH, aunque el valproato y el clonacepam tambin
puede ser eficaces. La vigabatrina da mejor resultado que los otros en
el control de los sndromes de West asociados a esclerosis tuberosa,
pero su uso generalizado se ha seguido del hallazgo de lesiones en la
retina, que comienzan en la retina perifrica (con la consiguiente
reduccin perifrica del campo visual) y que obliga a realizar
campimetras de control a los pacientes tratados, dado que el dao es
irreversible. En principio, el uso de este frmaco est limitado a casos
seleccionados, por periodos concretos de tiempo y con seguimiento
campimtrico.

CTO Medicina C/ Nez de Balboa, 115 28006 MADRID (Espaa) Tfno.: (91) 782 43 32 / Fax: (91) 782 43 27
E-mail: secretaria@ctomedicina.com; iberocto@ctomedicina.com WEB: www.ctomedicina.com; www.iberocto.com

NR Pg. 11

NEUROLOGA

Preparacin Examen de Seleccin 05/06 1 Vuelta

Pregunta 49.- R: 2
Las crisis febriles tpicas son un fenmeno muy frecuente en la
edad infantil, debido a la especial sensibilidad que tiene el cerebro a
esas edades para presentar crisis generalizadas en relacin con las
subidas trmicas. Con la madurez completa de la corteza, que se va
alcanzando a lo largo de la infancia, aumenta la resistencia a que los
cambios de temperatura produzcan alteraciones elctricas.
Se desconocen por el momento los mecanismos bsicos por los
cuales se es ms sensible a los cambios de temperatura, pero se ha
visto estadsticamente que se presentan antecedentes familiares en un
gran nmero de casos.
En las crisis tpicas, tanto el desarrollo, como el comportamiento,
rendimiento y funciones cerebrales son normales fuera de los episodios crticos, as como el EEG. No es preciso el tratamiento de base
con anticomiciales, dado que es suficiente con el control de los ascensos trmicos bruscos.
En casos seleccionados, se puede pautar tratamiento con diacepam oral o rectal en los dos primeros das de los procesos febriles
como profilaxis de las crisis, pero no existe un consenso de actuacin
para todos los pacientes, por lo que no es un tratamiento que pueda
aceptarse como generalizado.
Las denominadas crisis febriles atpicas sugieren patologa de
base que tiene una repercusin clnica en el contexto de un sndrome febril, ms que de una corteza cerebral normal con mayor sensibilidad a la hipertermia, por lo que hay quien prefiere hablar de
crisis con fiebre, lo que significa que el paciente tiene crisis no
causadas nicamente por la fiebre, sino por la suma de sta a otra
patologa.
Pregunta 50.- R: 1
Hay que recordar que epilepsia significa tener crisis epilpticas
de forma recurrente. Cualquier cerebro puede tener, bajo determinadas circunstancias, una actividad elctrica paroxstica que clnicamente conlleve una crisis epilptica, sin que esto signifique mayor
riesgo de presentar crisis ms adelante.
La decisin de instaurar un tratamiento anticomicial est basada
en el riesgo estimado de presentar nuevas crisis en el futuro; cuando es
bajo o no es posible determinarlo, no est indicado iniciar un tratamiento, dado el no desestimable nmero de casos en los que se presenta una crisis aislada con un estudio complementario normal o
negativo.
Los sndromes epilpticos merecen una consideracin aparte,
pero ya estamos en el contexto de una patologa hereditaria de la
que se puede estimar el riesgo de presentar nuevas crisis por los
antecedentes familiares y lo que conocemos de la enfermedad, y
por eso puede estar indicado iniciar el tratamiento a la primera
crisis.
De modo que, ante una primera crisis: NO TRATAR.

Pregunta 50. Indicacin de frmaco antiepilptico


en funcin del tipo de crisis.

12345675892
2

27323 284

1234526

789
1




5446542 326522

1
1
789
8

452  !542

1
"

452 2 !542

1



546542

89#
1

" 3$  %

8
1
89#
7

" 3$  &22

1
 
89#

542

1
 
89#

1234567897
76 7545  75456 57 8 6523459  76 7
 47
8 ! 75"45 !7978 75#$45 97
75$% 45 &'(
75#245 9789 7 1

Pg. 12 NR

M exico A rgentina
C hile U ruguay

Pregunta 51.- R: 4
Se trata de una serie de generalidades que hay simplemente que
recordar:
1) El tratamiento en monoterapia (una vez seleccionado un frmaco
de primera lnea para el tipo de crisis) consigue un buen control de
las mismas en un alta porcentaje de pacientes (60-80%)
2) En las crisis de ausencia tpicas hay dos frmacos que, segn los
autores, se sitan en primer o segundo lugar: la etosuximida y el
cido valproico. La fenitona y la carbamacepina pueden empeorar las crisis de ausencia
3) La etosuximida es un frmaco cuya nica indicacin hoy da son
las crisis de ausencia tpicas, sin que controle ningn otro tipo. En
las crisis parciales complejas, el tratamiento (o uno de ellos ) de
primera lnea sera la carbamacepina
4) El fenobarbital cuenta entre sus efectos secundarios ms frecuentes
la sedacin y la somnolencia; en nios puede producirse el efecto
paradjico de desencadenar un cuadro de hiperactividad. Esto no
es muy distinto de lo que sucede con el sndrome de hiperactividad e hipercinesia primario, en el que parece que la alteracin
bsica es una tendencia al sueo con dificultad para mantener el
nivel de alerta y por eso, para conseguirlo, el paciente tiene la
hiperactividad y es preciso tratarlo con estimulantes para que no
tenga la necesidad de ser hiperactivo
5) Las crisis febriles son un proceso frecuente en los primeros aos de
vida (1-5 aos) y se relaciona con antecedentes familiares de crisis
febriles infantiles en los padres y con algunas formas de epilepsia.
No est indicado el tratamiento profilctico con anticomiciales a
largo plazo, y s el control de los ascensos de temperatura en los
procesos febriles. En algunos casos concretos, se puede administrar
diacepam al principio de cada proceso febril, pero no es una
prctica generalizada.
Pregunta 52.- R: 2
Hay distintas pautas de actuacin ante un estatus epilptico, pero
generalmente se establece una actuacin por escalones:
Benzodiacepina: intravenosa en adultos, rectal en nios. En nuestro medio se utiliza habitualmente el diacepam; en otros pases
disponen de loracepam por va intravenosa y se utiliza en lugar del
diacepam. En cualquier caso, hay que tener cuidado con la dosis,
dado que puede producir depresin respiratoria. El efecto de las
benzodiacepinas es de corta duracin, con lo que en muchas
ocasiones no se espera a que el paciente tenga una segunda crisis
para pasar al segundo escaln.
Fenitona (difenilhidantona) intravenosa: de inicio de accin algo
ms lento, pero de vida media larga, permite que posteriormente se
mantenga el tratamiento a largo plazo con el mismo frmaco. Tambin est admitido el cido valproico intravenoso, aunque no en
todos los pases. Si con la administracin de fenitona en dosis mxima (hasta 25 mg/kg) no ceden las crisis, pasar al siguiente escaln.
Barbitricos: el fenobarbital como primera opcin. Su administracin generalmente ha de ser en la UCI o con posibilidad inmediata
de intubacin, si es preciso. En algunos casos se precisa otro tipo de
barbitricos, con anestesia general si es preciso.
Pregunta 53.- R: 2
En principio, un paciente epilptico con buen control con un
frmaco durante aos, si est cinco aos sin crisis (como en este caso)
con pruebas complementarias negativas, podra ser candidato a un
intento de lenta retirada de medicacin; en este caso, si no apareciera
el tema del embarazo, la opcin correcta sera la 1.
Un embarazo con tratamiento farmacolgico en general tiene mayor riesgo de malformaciones fetales. Estas se producen en la fase de
organognesis, es decir, las primeras semanas, con lo que si queremos
evitar ese riesgo hay que suspender el frmaco al menos durante la
primera fase del embarazo; eso obligara a una suspensin brusca del
tratamiento: si hacemos una lenta retirada, no hemos evitado las malformaciones.
La retirada de un frmaco anticomicial debe hacerse de forma
muy lenta (meses), porque una retirada brusca puede hacer que el
paciente presente de nuevo crisis, incluso de una forma ms agresiva
que las que motivaron el inicio del tratamiento.

CTO Medicina C/ Nez de Balboa, 115 28006 MADRID (Espaa) Tfno.: (91) 782 43 32 / Fax: (91) 782 43 27
E-mail: secretaria@ctomedicina.com; iberocto@ctomedicina.com WEB: www.ctomedicina.com; www.iberocto.com

Comentarios TEST

Seguimiento a distancia

NEUROLOGA

Preparacin Examen de Seleccin 05/06 1 Vuelta


Visto lo anterior, tenemos que decidirnos entre asumir el riesgo de
malformaciones o el riesgo de crisis durante el embarazo. Esta balanza
se inclina hacia lo primero: es menor el riesgo para el feto de estar
sometido a un frmaco, en especial si es en monoterapia, que el de
que la madre embarazada sufra una crisis.
Es cierto que puede haber una disminucin de niveles del frmaco
durante el embarazo por aumento del metabolismo y del volumen de
distribucin, pero no se suele precisar un aumento de dosis, y menos
a priori.
Pregunta 54.- R: 4

Pregunta 54. Efectos secundarios de la medicacin anticomicial.


12345678349
6

  4679376793 37 6 94

76385 8785 4 87 85 58


7
944
73
54 
7 8 73 9 
71
53 
7
53 23 3
4 746
75 
8
7776869 38
7!3 4
786
63
4 7 5  
7"3 56#
388
7$ 895867%3  8 7
 
777 3  5587998&'3
7$3 6
3 7 634 
7776 38 7
4 3 558

7"6 68
7"6 68
7"6 68
7"6 68
7"6 68
7"6 68
73
56(

7)  646

Comentarios TEST

ENFERMEDADES DESMIELINIZANTES.
Pregunta 55.- R: 1
El sntoma de comienzo ms frecuente de la EM es la alteracin de
la sensibilidad, lo que se produce hasta en el 45% de los casos. Puede
consistir en parestesias (descritas como pinchazos u hormigueo) o
hipoestesias (acorchamientos).
El dficit motor tambin es frecuente como sntoma de debut, que
puede presentarse como franca paresia focal o como debilidad generalizada, que a la exploracin se acompaa de signos de lesin piramidal.
Las alteraciones visuales son caractersticas, aunque sean menos
frecuentes al debut: lo ms frecuente es un escotoma central con
disminucin de la agudeza visual. El fondo de ojo puede presentar
edema de papila (papilitis) o ser normal (neuritis retrobulbar), siendo
ms frecuente lo ltimo. Pasadas unas semanas, en el fondo de ojo se
apreciar una atrofia de la papila, con palidez de la misma.
La disfasia aislada no encaja dentro del curso clnico de la esclerosis mltiple, salvo que est asociada a deterioro cognitivo por numerosas lesiones diseminadas.
Los trastornos del control de los esfnteres son muy infrecuentes al
debut, aunque no sucede lo mismo a lo largo del curso de la enfermedad, donde ms de la mitad de los pacientes sufre algn tipo de
alteracin en este sentido.
Pregunta 56.- R: 3
La oftalmoplejia internuclear es muy caracterstica de la esclerosis
mltiple; se dice que si es bilateral y en un paciente joven, puede
considerarse casi patognomnico.
El cuadro clnico se produce por la interrupcin del fascculo
longitudinal medial, que recorre la protuberancia, poniendo en comunicacin la zona de organizacin de la motilidad horizontal conjugada, que se encuentra junto a los sextos pares, con el rea mesenceflica prxima a los terceros pares.
El fascculo longitudinal medial de cada lado es el que lleva la
informacin para la adducin del ojo ipsilateral, de tal forma que
ante una lesin del fascculo derecho se encuentra una imposibilidad
para la adducin del ojo derecho cuando se intenta la mirada conjugada. Es un movimiento (el del recto interno) que induce el III par,
pero no hay que confundir la oftalmoplejia con la lesin del III: el ojo
puede hacer movimientos verticales y no est desviado hacia fuera en
reposo, como sucedera con una lesin del par III.
M exico A rgentina
C hile U ruguay

Seguimiento a distancia

Asimismo, la informacin ascendente por el fascculo longitudinal


medial facilita el movimiento de abduccin del ojo contralateral, por lo
que, al producirse la lesin, aparece un nistagmo en el ojo que s abduce.
En resumen: ante la presencia, como nico defecto de motilidad
ocular, de alteracin en la mirada horizontal hacia un lado, en el que
aparece un ojo que no adduce (no mira hacia adentro) y un ojo que
abduce (mira hacia fuera) pero con nistagmo, podemos situar la lesin
en el fascculo longitudinal medial del ojo que no mira hacia adentro,
o, si no se menciona el fascculo, en la protuberancia de ese lado.
Pregunta 57.- R: 1
Entre los sntomas menos frecuentes en el curso de una esclerosis
mltiple se encuentran los fenmenos paroxsticos: dolores o parestesias lancinantes que recuerdan las neuralgias (como la neuralgia del
trigmino) en principio asumidas como originadas por lesiones y reas
de desaferentizacin sensitiva, aunque a veces no hay forma de justificar la causa. Generalmente responden bien al tratamiento con
inmunomoduladores (antiepilpticos, antidepresivos tricclicos, como
las neuralgias esenciales).
La existencia de cuadros de actividad neuronal anmala (como las
crisis epilpticas) son infrecuentes en la EM (un 3%) y se producen en
relacin con lesiones activas que se siten prximas a la sustancia gris,
postulndose que la irritacin cortical que produce la inflamacin
activa sera la causa de las crisis. Lo mismo sucedera en los movimientos coreoatetsicos, en lesiones prximas a los ganglios basales, bien
por interrupcin de vas inhibidoras, bien por irritacin.
El signo de Lhermitte es un signo clsico de afectacin de cordones
posteriores a nivel cervical, con la aparicin de un dolor agudo (tipo
descarga elctrica) que recorre la columna vertebral en sentido descendente. No es exclusivo de la EM, sino que puede presentarse en
lesiones medulares cervicales de otro origen, y en aquella aparece
cuando hay lesin medular, con lo que puede significar un signo de
mal pronstico si aparece en el debut de la enfermedad.
Los trastornos afectivos, especialmente la depresin, son comunes
en la EM. Valorada la incidencia de depresin en otras patologas que
producen un grado similar de discapacidad, se objetiva un mayor
nmero de casos en los causados por la esclerosis mltiple, y se ha
relacionado con la carga lesional o la existencia de lesiones en reas
concretas. Tambin pueden aparecer otros sntomas psiquitricos
causados por la terapia corticoidea, aunque no se recomienda el
tratamiento preventivo con litio u otros estabilizadores del nimo,
dado que no es lo ms frecuente.
Pregunta 58.- R: 4
Los hallazgos en las pruebas complementarias no son por s mismos diagnsticos ni descartan el diagnstico de esclerosis mltiple,
limitndose a apoyar o a alejar el diagnstico.
Las prueba ms til para el diagnstico es hoy por hoy la resonancia magntica, puesto que es capaz de distinguir un mayor nmero de
lesiones e incluso diferenciar entre lesiones desmielinizantes activas o
antiguas, bien por el uso de determinadas secuencias especficas o
por la administracin de contraste. De este modo, el hallazgo de una
RM cerebral normal, ante la presencia de una clnica por lo dems
sugerente de EM, si bien no descarta el diagnstico, puesto que podra
ser normal en fases muy precoces, s es la menos sugerente de la
enfermedad y obliga a plantear antes otros diagnsticos.
Los hallazgos del LCR apuntan hacia una actividad inflamatoria
dentro del SNC cuando aparecen anticuerpos que han sido sintetizados dentro del SNC: esto se demuestra mediante dos mediciones:
Aumento del ndice de IgG: en el total de protenas de un lquido
normal aparecen ms inmunoglobulinas de lo habitual, aunque la
proteinorraquia total no aumente; esto implica una sntesis excesiva de IgG
Bandas oligoclonales: al estudiar las IgG en el LCR y en el suero del
mismo paciente, se detectan anticuerpos en el LCR que no estn
presentes en el suero, lo que indica que se han sintetizado dentro
del SNC. Este es el hallazgo que aparece de forma ms constante en
la enfermedad.
Los potenciales evocados se basan en la deteccin de la actividad
cortical que se produce con la estimulacin de distintos sistemas

CTO Medicina C/ Nez de Balboa, 115 28006 MADRID (Espaa) Tfno.: (91) 782 43 32 / Fax: (91) 782 43 27
E-mail: secretaria@ctomedicina.com; iberocto@ctomedicina.com WEB: www.ctomedicina.com; www.iberocto.com

NR Pg. 13

perifricos (visual, auditivo, sensitivo, motor). Si se detecta enlentecimiento en alguno de los sistemas, se diagnostica de forma indirecta de
desmielinizacin en algn punto de la va. Hoy por hoy, las que se
consideran ms tiles son los visuales, dado que para el resto de los
sistemas la RM es ms sensible, mientras que para la valoracin del
nervio ptico lo es el estudio del potencial evocado visual.
Pregunta 59.- R: 3
El tratamiento de la esclerosis mltiple se basa en dos puntos:
La disminucin de la intensidad del brote: desde un punto de vista
conceptual, si conseguimos disminuir la actividad inflamatoria lo
antes posible, conseguiramos un menor dao y una mejor recuperacin. Parece que el tratamiento con corticoterapia intravenosa en altas dosis conseguira este efecto, aunque a largo plazo el
grado de recuperacin es el mismo en tratados y no tratados, pero
en los tratados esa recuperacin se produce antes. Incluso se ha
valorado que el tratamiento con corticoides en un primer brote de
neuritis ptica retrasara la aparicin del segundo brote, sin que se
sepa la razn y sin que se hayan repetido los resultados en brotes
de otro tipo
La disminucin del nmero de brotes: en las formas recurrentesremitentes existen tratamientos capaces de disminuir el nmero de
brotes, con lo que a largo plazo tendramos una menor carga
lesional y por lo tanto menos secuelas. El interfern beta, el acetato
de glatiramer y algunos inmunosupresores han obtenido este efecto, y hoy por hoy, por menos efectos secundarios relativos, se administra el interfern. No est indicado el tratamiento con frmacos para disminuir el nmero de brotes cuando an no se sabe el
nmero de brotes que espontneamente presentar el paciente,
por lo que no se trata a partir de un nico brote.
Un paciente que est sometido a tratamiento para modificar el
curso de la enfermedad, con interfern beta o con otro frmaco, en el
caso de presentar un nuevo brote tiene indicacin de ser tratado con
corticoterapia intravenosa, no siendo incompatibles ambos tratamientos.
La existencia de secuelas establecidas en un paciente con esclerosis mltiple habla de lesin axonal residual aadida a la desmielinizacin, y hoy por hoy no hay tratamiento que consiga una regeneracin neuronal, con lo que no se dispone de ningn tratamiento que
ayude a mejorar secuelas establecidas desde brotes antiguos.
Pregunta 60.- R: 4
El tratamiento modificador de la historia natural de la esclerosis
mltiple hoy por hoy est basado en la reduccin del nmero de
brotes. En este objetivo han demostrado eficacia varios frmacos, siendo los ms usados en la actualidad el interfern beta y el acetato de
glatiramer, que han demostrado reduccin de un 30% de los brotes
previstos en comparacin con grupos control.
Se administran por va subcutnea o intramuscular con distintas
pautas y dosis, a pacientes con formas recurrentes-remitentes de la
enfermedad, que hayan presentado dos brotes en los dos ltimos
aos; con menos frecuencia no existe indicacin de tratar.
En las formas monosintomticas (un nico brote) y las formas progresivas primarias (en las que hay una lesin continua, sin brotes) no
est indicado el tratamiento, ya que no hay brotes que reducir.
Las secuelas de brotes previos indican lesiones residuales ms all
de la desmielinizacin, por lo que no se recuperan con ningn tratamiento. Se ha descrito mejora de las lesiones desmielinizantes en la
resonancia magntica tras el tratamiento con interfern, pero sin cambios en la situacin funcional del sujeto, con lo que probablemente
signifique una reduccin de las lesiones aparentes, pero que estaban
funcionando a pesar de la desmielinizacin.
Pregunta 61.- R: 5
Para contestar esta pregunta basta conocer cul es la utilidad del
interfern para dejar reducidas a dos las opciones a pensar.
El interfern beta es un frmaco que se administra en las esclerosis
mltiples remitentes-recurrentes para disminuir el nmero de brotes.
Es un inmunomodulador, que hace que el paciente tenga menos
episodios, sin modificar la intensidad de stos, por lo que, en caso de
presentar brote, el paciente debe recibir terapia con corticoides.
Pg. 14 NR

M exico A rgentina
C hile U ruguay

NEUROLOGA

Preparacin Examen de Seleccin 05/06 1 Vuelta

Por lo tanto, ninguna enfermedad monofsica se beneficiar de un


tratamiento de este tipo, sino que ser preciso tratar con corticoterapia i.v. como cualquier otra inflamacin asptica activa del SNC.
En el caso de la encefalomielitis diseminada aguda, que se presenta
entre das y escasas semanas despus de una infeccin o una vacunacin, se produce una desmielinizacin difusa de la sustancia blanca,
probablemente por activacin de la inmunidad celular, con una
reactividad cruzada con antgenos de la mielina.
Las vacunaciones que se han asociado con este cuadro son las de
virus vivos atenuados, y tras la administracin de suero antitetnico
(no del toxoide); la ms frecuente, la vacuna del sarampin. Esto hace
que en pases de baja prevalencia de sarampin, se sustituya la vacuna de virus atenuados por una sin virus vivos.
Hoy las infecciones espontneas con las que se asocia de forma
ms frecuente esta patologa son las infecciones respiratorias de etiologa desconocida, seguidas de la varicela. Clsicamente era la infeccin por el virus del sarampin.
La clnica puede variar desde un sndrome meningo-encefalomieltico grave con afectacin de muchos sistemas (cerebeloso, motor; crisis epilpticas), hasta un cuadro leve similar a un primer brote
de esclerosis mltiple.
El tratamiento, como se ha dicho, es con dosis altas de corticoides.
Pregunta 62.- R: 3
En un paciente de esta edad que previamente est bien y comienza
con un cuadro de deterioro progresivo hay que pensar en enfermedades degenerativas que, dado que se describe con afectacin de la
sustancia blanca, debera hacernos pensar en alteraciones degenerativas de la sustancia blanca: las leucodistrofias.
En efecto, las leucodistrofias son cuadros degenerativos propios de la
edad infantil, que, de forma genrica, presentan ceguera, sordera, deterioro mental y tetraplejia de instauracin progresiva, pero no suelen
presentar crisis epilpticas, mioclonas, ni alteraciones en el EEG.
En este paciente en el que expresamente se nos describe una alteracin caracterstica del EEG hay que pensar en una panencefalitis
esclerosante subaguda. Se produce en pacientes que han sufrido el
sarampin (tambin es posible en vacunados, pero con una incidencia muchsimo menor) y cursan de la forma descrita en el caso clnico.
Se produce una encefalitis difusa de las sustancias gris y blanca, con
pocos signos inflamatorios. En el LCR se detectan ttulos de anticuerpos
antisarampin altos, pero no virus. Tambin hay bandas oligoclonales.
No existe ningn tratamiento y la evolucin es mortal en el curso
de 1-4 aos.
Pregunta 63.- R: 2
La descripcin clnica de un paciente con aparente buen nivel de
conciencia, con el que se puede mantener una conversacin en la
que las contestaciones son a base de parpadeos, es la imagen caracterstica de un sndrome de cautiverio.
Este sndrome se produce por lesiones protuberanciales extensas
que respeten la sustancia reticular, con lo que el paciente no est
comatoso, como ocurrira de tratarse de una lesin ocupante de espacio, pero no tiene motilidad voluntaria por debajo de ese nivel, lo
que incluye los movimientos oculares horizontales, la musculatura
craneofacial y el resto del cuerpo.
Las causas son mltiples, y hay que recordar como curiosa la
mielinlisis central pontina yatrognica por una recuperacin demasiado rpida de una hiponatremia, lo que da lugar a una deshidratacin y destruccin de las clulas a dicho nivel.
Puede aparecer una destruccin espontnea de la mielina o asociado a otras lesiones desmielinizantes.
No tiene buen tratamiento, salvo que se identifique la patologa de
base, y lo ms importante es distinguirlo de un cuadro compresivo, que
puede causar el mismo dficit motor, pero que generalmente ir acompaado de coma, alteraciones pupilares y en los reflejos de tronco.
PATOLOGA EXTRAPIRAMIDAL.
Pregunta 64.- R: 5
Aunque la presencia de trastornos anmicos y de la memoria pueda
ser patolgica, el hecho de que se describa un paciente con movimien-

CTO Medicina C/ Nez de Balboa, 115 28006 MADRID (Espaa) Tfno.: (91) 782 43 32 / Fax: (91) 782 43 27
E-mail: secretaria@ctomedicina.com; iberocto@ctomedicina.com WEB: www.ctomedicina.com; www.iberocto.com

Comentarios TEST

Seguimiento a distancia

NEUROLOGA

Preparacin Examen de Seleccin 05/06 1 Vuelta

Seguimiento a distancia

Comentarios TEST

Pregunta 69. Diagnstico diferencial del parkinsonismo

tos continuos involuntarios es el dato ms significativo. Los movimientos


continuos generalizados sin especificar rea predominante ni velocidad de ejecucin los podramos denominar coreoatetsicos.
Un paciente adulto en el que debutan movimientos continuos
involuntarios hay que pensar que se trate de movimientos coreicos, ya
que los movimientos atetsicos con mayor frecuencia se asocian a
patologa congnita y no degenerativa de inicio tardo.
Estamos por lo tanto ante un paciente con demencia, trastorno
afectivo y movimientos involuntarios: esto es suficiente para el diagnstico de la enfermedad de Huntington, aunque hoy da puede
establecerse con certeza en ausencia de antecedentes o incluso de
clnica, mediante estudio gentico.
La herencia es autosmica dominante, por lo que las posibilidades
de que los hijos de un paciente afecto padezcan la enfermedad son el
50%, pero ante la presencia de antecedentes dos generaciones atrs
se tiene prcticamente la certeza de que el padre del paciente de este
caso tambin habra padecido la enfermedad.
En esta enfermedad se produce una alteracin de la protena
huntingtina, de la que an no se conoce su funcin normal, y que
conlleva una destruccin neuronal en distintos sistemas, predominando la atrofia del estriado, pero con afectacin en fases ms avanzadas de la corteza cerebral, el locus ceruleus, el ncleo subtalmico
y otras reas
Desde el punto de vista bioqumico, el principal neurotransmisor
relacionado es la dopamina, existiendo una hiperfuncin dopaminrgica; pero tambin se altera la neurotransmisin gabargica, la
colinrgica, la somatostatina y la sustancia P.
M exico A rgentina
C hile U ruguay

Pregunta 65.- R: 4
Conviene recordar con respecto a esta enfermedad:
Que es de herencia autosmica dominante, por lo tanto, afecta a
hombres y mujeres.
Que aunque el diagnstico se puede establecer por la clnica y los
antecedentes familiares compatibles, hoy da el estudio gentico,
en busca de las repeticiones del triplete CAG en el cromosoma 4
establece el diagnstico de certeza incluso en fase sintomtica; esto
no quiere decir que sea preciso, se puede seguir diagnosticando
como siempre si no se dispone del estudio gentico. Un mayor
nmero de tripletes parece correlacionarse con el inicio ms precoz, pero no con la clnica psiquitrica ni con la cognitiva.
Que presenta el fenmeno de anticipacin: en la duplicacin
gentica, para la formacin de las clulas sexuales se produce un
aumento en el nmero de copias CAG, con lo que, si el descendiente toma ese cromosoma, la enfermedad debutar antes en l
que en su antecesor.
Que la atrofia del ncleo caudado (ms acusada en la cabeza)
hace que ese espacio se ocupe por el ventrculo lateral prximo.
Eso en la TC semeja una dilatacin ventricular como las hidrocefalias, pero en este caso es selectiva: slo est aumentada de tamao
el asta frontal, que es la que est junto al caudado, mientras que el
resto del ventrculo (astas temporales y occipitales) es normal.
Tener en la memoria tambin dos variantes respecto a la forma tpica:
1) variante juvenil (de Westphal): presenta un cuadro rigido-acintico
ms llamativo que el coreico, y crisis epilpticas. Son casos heredados del padre y no de la madre, y tienen peor pronstico.

CTO Medicina C/ Nez de Balboa, 115 28006 MADRID (Espaa) Tfno.: (91) 782 43 32 / Fax: (91) 782 43 27
E-mail: secretaria@ctomedicina.com; iberocto@ctomedicina.com WEB: www.ctomedicina.com; www.iberocto.com

NR Pg. 15

2) variante de inicio tardo: la corea como signo predominante, evolucin ms benigna.


Pregunta 66.- R: 4
El tratamiento de la enfermedad tiene un objetivo meramente sintomtico, ya que hasta el momento no se dispone de ninguna molcula capaz de detener la muerte de las neuronas afectadas, con lo que
ningn tratamiento consigue detener la evolucin de la enfermedad.
Los sntomas coreicos se controlan con antagonistas dopaminrgicos, entre los que se encuentran los neurolpticos clsicos (haloperidol, tiapride, sulpiride) y los depletores presinpticos de dopamina
(reserpina, tetrabenacina).
Se ha intentado actuar sobre otros sistemas de neurotransmisin
afectados (gabargico, serotoninrgico, colinrgico, somatostatina) sin
claros efectos beneficiosos.
La base neuroqumica de la depresin en la enfermedad de
Huntington se desconoce, ya que los sistemas serotoninrgicos parecen estar afectados tanto en los pacientes deprimidos como en los
que no lo estn. En los pacientes que presenten sndrome depresivo se
puede iniciar un tratamiento emprico, con inhibidores selectivos de
la recaptacin de serotonina (fluoxetina, paroxetina) o con otro tipo
de antidepresivos (tricclicos).
En principio, los agonistas dopaminrgicos no tienen papel alguno
que jugar en una enfermedad con una hiperactividad dopaminrgica, salvo para empeorarlos, y por eso se considera correcta la opcin
4. Sin embargo, en algunas ocasiones se pueden obtener beneficio
con dosis bajas de agonistas dopaminrgicos, por un efecto paradjico que se postula se produce al actuar sobre los receptores
presinpticos, limitando la liberacin de ms dopamina.
Pregunta 67.- R: 3
Para no caer en la confusin, separar los datos del caso clnico en
las distintas reas de la exploracin:
Motor: hipertona plstica (resistencia continua), reflejos presentes,
algo hipoactivos. Todo ello compatible con enfermedad de
Parkinson. Cadas frecuentes que no parecen crisis epilpticas (conservacin de la conciencia): no sugerente de enf. de Parkinson.
Extrapiramidal: hipomimia facial, hipertona plstica, sin temblor de
reposo (lo que esperaramos en una enfermedad de Parkinson) ni
movimientos involuntarios (lo que presentara una variante de
Westphal, adems de ser ms joven que el paciente de la pregunta)
Funciones corticales: cambios de comportamiento, alucinaciones
visuales, instauracin subaguda
Son demasiados los datos que hacen ms que dudoso el diagnstico de enfermedad de Parkinson. Ni el curso clnico ni la edad del
paciente acompaan a la variante rgida de la enf. de Huntington
Igualmente son demasiados signos motores extrapiramidales para
una enfermedad de Creutzfeldt-Jakob o de Alzheimer, que cursan de
una forma menos llamativa en lo motor (salvo las mioclonas en la
primera).
En ocasiones es especialmente difcil el diagnstico diferencial entre la demencia por cuerpos de Lewy y un Parkinson-demencia, dado
que el diagnstico definitivo se establece post-mortem y que incluso
entonces, dada la aparicin de datos anatomopatolgicos similares,
puede no quedar claro.
De forma arbitraria se ha establecido que, si los sntomas parkinsonianos preceden a los cognitivos en un ao, estaremos ante un
Parkinson-demencia, mientras que si la demencia precede o coexiste
al comienzo con el cuadro motor, lo consideraremos una Enfermedad por Cuerpos de Lewy.
Pregunta 68.- R: 2
Existe una serie de datos clnicos muy sugerentes de la existencia de
una parlisis supranuclear progresiva o enfermedad de SteeleRichardson, aunque al principio del cuadro pueda confundirse y
diagnosticarse errneamente de Enfermedad de Parkinson. De hecho, un nmero no pequeo de pacientes (hasta un 40-50%) presentan una cierta mejora con la administracin de levodopa.
Entre el 60 y el 90 % de los pacientes presenta inestabilidad postural al inicio, dando lugar a cadas frecuentes, dato poco compatible
con el diagnstico de Enfermedad de Parkinson.
Pg. 16 NR

M exico A rgentina
C hile U ruguay

NEUROLOGA

Preparacin Examen de Seleccin 05/06 1 Vuelta

La afectacin tpica de los movimientos oculares supranucleares no


suele presentarse hasta transcurridos cuatro-cinco aos del comienzo
de los sntomas, consistiendo en una limitacin para los movimientos
oculares verticales con conservacin de los reflejos oculoceflicos.
Puede haber una rigidez axial intensa, lo que es ms llamativo en la
hiperextensin forzada del cuello que suelen presentar estos pacientes, con un tono relativamente conservado en las extremidades.
Existe muy frecuentemente un deterioro cognitivo asociado, que
no suele ser una demencia completa, pero que conlleva bradipsiquia, irritabilidad, incluso labilidad emocional.
La parlisis supranuclear progresiva tiene un rango de presentacin
muy similar al de la enfermedad de Parkinson, ms frecuentemente en
torno a los 50-60 aos, aunque existen casos de debut ms precoz.
La muerte se produce por complicaciones asociadas (respiratorias,
cadas).
Pregunta 69.- R: 3
La presencia de un cuadro parkinsoniano con rigidez y bradicinesia nos debe hacer pensar siempre en primer lugar en una enfermedad de Parkinson, que es la causa ms frecuente de sndrome
parkinsoniano (ver figura en pgina siguiente).
Si aparecen signos de afectacin de otros sistemas, hay que pensar
en principio que no se trata de una enfermedad de Parkinson.
En el caso de la pregunta, ya se nos presenta la existencia de escaso
temblor, cuando el temblor de reposo es el signo ms frecuente de
inicio de la enf. de Parkinson. Pero la presencia de marcha atxica
automticamente nos lleva a la lista de diagnsticos diferenciales en la
que no cabe el Parkinson, puesto que no puede presentar sntomas
de afectacin cerebelosa.
Siempre que aparezca un cuadro parkinsoniano con sntomas incompatibles, hay con considerar la presencia de una atrofia multisistmica: este es un cuadro que engloba la coexistencia de sntomas
parkinsonianos, vegetativos, cerebelosos y piramidales en distinto grado en el mismo paciente.
En un principio se consideraban distintas enfermedades dependiendo de la sintomatologa preponderante, hasta que se demostr
que en realidad la anatoma patolgica era la misma y que se trataban
de presentaciones distintas de un mismo proceso.
En el caso de la atrofia multisistmica que presenta de forma predominante sntomas cerebelosos, la denominamos atrofia olivo-ponto-cerebelosa, por ser stas las localizaciones en las que en un principio se crea estaba confinada la degeneracin
Pregunta 70.- R: 4
Nos estn describiendo un cuadro de afectacin vegetativa, con
sntomas predominantemente genitourinarios: podramos encontrarnos ante un cuadro de disautonoma perifrica o bien de afectacin
medular con afectacin de segmentos inferiores.
La presencia del antecedente de sncopes no acompaa para el
diagnstico de patologa medular, por lo que podra corresponder a
un sndrome perifrico si no fuera por los signos parkinsonianos.
Ante un sndrome parkinsoniano con disautonoma, podra tratarse
de una enfermedad de Parkinson si el cuadro motor hubiera sido compatible y previo a la aparicin de los sntomas vegetativos. Generalmente los signos disautonmicos no suelen ser muy marcados.
En este paciente, sin embargo, lo que parece menos llamativo es el
cuadro parkinsoniano, y ms limitante los sncopes de largo tiempo
de evolucin. Este es el patrn tpico de la enfermedad de Shy-Drager,
que se encuadra dentro de la atrofia multisistmica, en la que predomina el cuadro vegetativo sobre el motor.
El tratamiento de los sntomas motores, como en el resto de atrofias
multisistmicas, es poco satisfactorio, y en este caso es preciso un
buen control de los sntomas vegetativos, especialmente de los sncopes, que pueden producir problemas por las cadas. Se recomienda
evitar hipotensiones y situaciones en que stas pueden producirse y
hay algunos autores que recomiendan tratamiento hipertensor, aunque este extremo no est comnmente aceptado.
Pregunta 71.- R: 5
La presentacin tpica de la enfermedad de Parkinson idioptica es
el temblor de reposo, unilateral, en una mano, que con el tiempo se
extiende a la mano contralateral.

CTO Medicina C/ Nez de Balboa, 115 28006 MADRID (Espaa) Tfno.: (91) 782 43 32 / Fax: (91) 782 43 27
E-mail: secretaria@ctomedicina.com; iberocto@ctomedicina.com WEB: www.ctomedicina.com; www.iberocto.com

Comentarios TEST

Seguimiento a distancia

NEUROLOGA

Preparacin Examen de Seleccin 05/06 1 Vuelta

Comentarios TEST

En fases ms avanzadas es cuando aparece la bradicinesia caracterstica, junto con el aumento de tono muscular y la inestabilidad
postural.
Los reflejos miotticos suelen conservarse normales o mnimamente disminuidos a lo largo de toda la evolucin de la enfermedad; unos reflejos vivos o exaltados obligara a replantearse el diagnstico, ya que en la enfermedad de Parkinson idioptica no hay
afectacin de la va piramidal, salvo que haya adems otras lesiones
asociadas.
La capacidad cognitiva de los pacientes se conserva tambin durante largo tiempo, siendo en fases avanzadas cuando no es infrecuente
que stas se alteren, dando lugar a lo que se denomina parkinsondemencia, y que obliga al diagnstico diferencial con la enfermedad
por cuerpos de Lewy si la demencia aparece en fases iniciales.
A pesar de las alteraciones posturales, el paciente generalmente no
sufre cadas frecuentes hasta fases avanzadas de la enfermedad, y si
aparecen en fases iniciales hay que considerar otros diagnsticos (parlisis supranuclear progresiva, enfermedad por cuerpos de Lewy, etc).
Pregunta 72.- R: 4
Hay que recordar que el diagnstico de enfermedad de Parkinson
idioptica es eminentemente clnico, y que las pruebas complementarias son especialmente tiles para descartar otras patologas, salvo las
pruebas de imagen funcionales (PET, SPECT) que detecten alteraciones en la funcin de los ganglios basales.
El estudio anatomopatolgico muestra cambios degenerativos similares a los que se producen en la enfermedad por cuerpos de Lewy,
quedando en discusin si ambas entidades no forman parte de una
misma patologa, con presentaciones diferentes. La diferencia es que los
datos de afectacin estructural se encuadran en ganglios basales y tronco del encfalo en la enfermedad de Parkinson, mientras que en la otra,
tanto la corteza como el rea subcortical tiene afectacin. Es la afectacin cortical la que se relaciona con la demencia, con lo cual es esperable
que el Parkinson, que suele conservar las funciones superiores hasta
fases avanzadas, en las que aparecen cambios de personalidad y
cognitivos, no presente degeneracin cortical.
El temblor caracterstico de la enf. de Parkinson es el temblor de
reposo; durante aos puede ser incluso el nico dato. Pero a lo largo
de la evolucin es frecuente que se asocie con un cierto componente
de temblor postural, que, si es muy intenso, puede llegar a precisar
tratamiento con betabloqueantes.
El tratamiento busca, de forma directa o indirecta, el aumento de la
transmisin dopaminrgica, por lo que en efecto no hay que incluir
antagonistas dopaminrgicos, que empeoraran los sntomas.
Por ltimo, recordar que los signos de afectacin piramidal (reflejos vivos, hipertona elstica, respuesta cutneo-plantar extensora)
obligan a replantear el diagnstico de enfermedad de Parkinson, debiendo considerarse en primer lugar una atrofia multisistema (degeneracin estro-ngrica) o que haya otras lesiones distintas afectando a las
primeras motoneuronas (isquemia lacunar?).
Pregunta 73.- R: 2
Existen distintos neurotransmisores alterados en la enfermedad de
Parkinson, y a algunos de ellos se les achaca alguno de los sntomas
caractersticos de la enfermedad:
Dopamina: es el ms afectado; disminuye sobre todo en el estriado, lo que se correlaciona con el cuadro motor (bradicinesia, temblor y rigidez). Tambin parece involucrado en los cuadros
disautonmicos, por deplecin en reas hipotalmicas.
Noradrenalina: disminuye sobre todo en la corteza hemisfrica, y
se sospecha involucrado en las alteraciones cognitivas.
Serotonina: su disminucin en la corteza frontal y en el estriado se
ha postulado en relacin con los cuadros depresivos que pueden
aparecer en el curso de la enfermedad.
Acetilcolina: disminuye en crtex, y est clara su relacin con la
demencia; este hecho hace que el tratamiento con anticolinrgicos en el Parkinson est limitado a pacientes jvenes, con la enfermedad en fases iniciales, con temblor como nico sntoma o predominante, dado que en fases avanzadas el deterioro de los sistemas colinrgicos puede empeorarse con dichos frmacos.
Somatostatina: disminuye en corteza, y su significado patolgico es
incierto
M exico A rgentina
C hile U ruguay

Seguimiento a distancia

Pregunta 74.- R: 4
Los frmacos tiles en la enfermedad de Parkinson son los que de
forma lgica esperaramos que sustituyan la neurotransmisin defectuosa, ms algunos de accin desconocida o mltiple:
Anticolinrgicos: frenan el predominio relativo de la transmisin
colinrgica en el estriado que se produce por la disminucin de la
dopamina. Contraindicados en pacientes con glaucoma, prostatismo y en personas mayores por el riesgo de alteraciones cognitivas.
Tampoco se usan en fases avanzadas de la enfermedad; su indicacin son pacientes jvenes en los que predomina el temblor
Levodopa con inhibidor de descarboxilasa perifrica: es el tratamiento ms eficaz para la enfermedad en todos sus aspectos clnicos, y la limitacin de su uso depende fundamentalmente de los
efectos secundarios a corto y largo plazo
Agonistas dopaminrgicos: estimulan directamente los receptores;
inducen con frecuencia a sntomas digestivos, que pueden ser controlados con antidopaminrgicos perifricos (antiemticos). Sus
indicaciones son pacientes en fases iniciales, intentando reservar la
levodopa para ms adelante, y el control de las fluctuaciones motoras en fases avanzadas de la enfermedad que se ha tratado con
levodopa
Amantadina: antivrico cuya accin parkinsoniana se descubri
de modo casual, es de accin leve y se utiliza en fases iniciales o
para el control de las discinesias
Selegilina: inhibidor de la mono-amino-oxidasa, permite que haya
ms dopamina disponible en la terminal sinptica. Se ha postulado su mecanismo neuroprotector para las neuronas dopaminrgicas, pero en este sentido no hay evidencia completa
La clorpromazina es un neurolptico clsico, de efecto antidopaminrgico, no muy aconsejable en el tratamiento de enfermos con
Parkinson.
Pregunta 75.- R: 4
A las complicaciones derivadas del tratamiento con levodopa de
forma prolongada algunos autores las denominan sndrome de
levodopaterapia crnica, y suele aparecer tras 3-5 aos de tratamiento con este frmaco. Bsicamente aparecen fluctuaciones en la
movilidad y discinesias.
Las fluctuaciones motoras pueden ser simples, en las que el paciente presenta de nuevo dficit de movilidad a las 3-4 horas de la
ltima administracin de levodopa y recupera su situacin funcional
tras la nueva dosis. Fluctuaciones complejas son las que se presentan
de modo errtico, sin una clara relacin temporal con la ingesta de
dosis, y pueden deberse a mltiples causas, generalmente combinadas
Las discinesias pueden clasificarse en discinesias de beneficio de
dosis (aparecen en el pico mximo de levodopa, y generalmente son
de tipo coreico), discinesias bifsicas (movimientos rtmicos alternantes
de las extremidades inferiores que aparecen al inicio y/o al final de la
accin de la levodopa) y distona off (postura distnica que aparece
coincidiendo con el fin del efecto de la levodopa)
El manejo de las fluctuaciones motoras simples y de las discinesias
de beneficio de dosis se basa en la distribucin de la dosis de levodopa a lo largo del da, asegurarse de que no hay interferencias con la
absorcin, o el uso de medicacin con liberacin lenta, con objeto
de evitar altibajos en los niveles plasmticos del frmaco.
El control de las fluctuaciones motoras errticas requiere generalmente la combinacin de distintos frmacos, con objeto de mantener
un estmulo dopaminrgico continuo sin fluctuaciones.
Pregunta 76.- R: 2
En ocasiones es difcil determinar desde el punto de vista terico,
ante qu tipo de temblor nos encontramos, y nos tenemos que imaginar las situaciones en las que aparece el temblor:
El temblor de reposo aparece en apoyo y sin hacer ninguna actividad.
El temblor de actitud es visible al mantener una postura o con
mnimos movimientos.
El temblor cintico aparece al realizar movimientos en los que
interviene toda la extremidad.

CTO Medicina C/ Nez de Balboa, 115 28006 MADRID (Espaa) Tfno.: (91) 782 43 32 / Fax: (91) 782 43 27
E-mail: secretaria@ctomedicina.com; iberocto@ctomedicina.com WEB: www.ctomedicina.com; www.iberocto.com

NR Pg. 17

En el caso de la pregunta, aunque cupiera la duda de si se trata de


un temblor postural o cintico, la ausencia de alteraciones en la marcha no apunta a un cuadro cerebeloso.
La presencia de una rueda dentada podra hacer sospechar una
enfermedad de Parkinson, pero se especifica la ausencia de temblor
de reposo, que es lo caracterstico de este cuadro.
Por lo tanto, estamos ante un temblor de caractersticas posturales,
con fenmeno de rueda dentada, que mejora con alcohol: son los
datos caractersticas del temblor esencial.
Esta es una enfermedad de etiologa no determinada, pero con un
claro componente familiar, con lo que son frecuentes los antecedentes familiares en la anamnesis. Antes de dar este diagnstico hay que
descartar fundamentalmente dos cosas: la ingesta de algn frmaco
que pueda causar temblor (el temblor farmacolgico es ms frecuentemente de tipo postural) y la existencia de focalidad neurolgica.
Puede presentar rueda dentada (de hecho, la rueda dentada en la
enfermedad de Parkinson se achaca al temblor de reposo, ms que
considerarse un signo independiente).
El tratamiento de esta patologa es meramente sintomtico, y se
utilizan los betabloqueantes o la primidona (un precursor del fenobarbital). El momento de comenzar el tratamiento lo da la limitacin
que tenga el paciente para su vida diaria.
Pregunta 77.- R: 5
La enfermedad de Friedreich es, en efecto, la ataxia hereditaria
ms frecuente, con herencia autosmica recesiva ligada al cromosoma 9. Los sntomas siempre comienzan en las dos primeras dcadas
de la vida, tras un desarrollo normal, y una vez que aparecen, se
produce un desarrollo completo de la enfermedad.
El defecto anatomopatolgico fundamental es la degeneracin de
los cordones posteriores, aunque posteriormente tambin se afectan
los cordones laterales, con vas descendentes corticoespinal y
espinocerebelosa ascendente: esto da lugar a la ataxia, aunque el
cerebelo no est afectado desde el punto de vista anatmico.
Adems del cuadro clnico ms llamativo, que es la ataxia, en la
exploracin se aprecian alteraciones de la sensibilidad profunda (la
que depende de cordones posteriores) y arreflexia (a pesar de la degeneracin de la va corticoespinal, que debera cursar con hiperreflexia),
ya que la degeneracin de las neuronas sensitivas que forman los cordones posteriores interrumpe la va aferente de los reflejos de estiramiento muscular. Sin embargo, la respuesta cutneo-plantar es extensora, como corresponde a la degeneracin de la va corticoespinal, ya
que la recepcin del estmulo superficial cutneo s se conserva.
En casi todos los casos hay afectacin cardiaca, que es la que ms
suele limitar el pronstico vital, aunque tpicamente se describe una
miocardiopata hipertrfica; en numerosas ocasiones se detecta dilatacin, con trombos intracavitarios o anomalas de la conduccin,
con lo que los problemas cardiacos pueden estar asociados a problemas restrictivos, emblicos o por arritmias.
Pregunta 78.- R: 1
El curso evolutivo de la Enfermedad de Friedreich es completo una
vez que comienzan los sntomas, llevando al fallecimiento de los pacientes en la cuarta-quinta dcada, debido a complicaciones cardiacas
(ms frecuente) o respiratorias.
En prcticamente todos los enfermos hay miocardiopata, que
puede ser hipertrfica o dilatada, y que con frecuencia asocian trastornos de la conduccin, detectables en el EKG. Es la causa ms frecuente de muerte.
Existen alteraciones esquelticas adquiridas por la disfuncin neurolgica, bsicamente cifoescoliosis y pies cavos, aunque la limitacin
en la marcha la da la ataxia y no estas anomalas.
Suele presentarse diabetes, pero prcticamente nunca hay alteracin de las funciones superiores.
Pregunta 79.- R: 3
La esclerosis lateral amiotrfica es la enfermedad degenerativa de
la motoneurona ms frecuente. Hay que sospecharla ante un paciente con clnica de prdida de fuerza, progresiva, subaguda, y que en la
exploracin muestre en unas reas corporales signos de prdida de
motoneurona superior (reflejos vivos, respuesta cutneo-plantar
Pg. 18 NR

M exico A rgentina
C hile U ruguay

NEUROLOGA

Preparacin Examen de Seleccin 05/06 1 Vuelta

extensora) y en otras lesin de motoneurona inferior (atrofia muscular marcada, fasciculaciones, reflejos disminuidos).
En ningn momento de la evolucin presenta alteracin del control de esfnteres (esas neuronas, sin que se sepa la razn, no se degeneran), ni con menoscabo de las funciones superiores, ni con alteraciones sensitivas.
El diagnstico diferencial ms frecuente que hay que hacer es con
la mielopata cervical, en la que el paciente tambin tiene signos de
afectacin de primera motoneurona (con hiperreflexia y Babinski en
miembros inferiores) y segunda motoneurona en brazos, con atrofia y
reflejos disminuidos. Esta distribucin caracterstica y la posible afectacin sensitiva y de esfnteres por compresin de otros cordones
medulares sirven para el diagnstico.
En las fases iniciales, la musculatura extraocular est preservada,
aunque en fases avanzadas tambin se afecta.
La supervivencia espontnea de la enfermedad puede alargarse de
forma discreta con el tratamiento con riluzol.
PATOLOGA VASCULAR CEREBRAL
Pregunta 80.- R: 1
De entre los factores de riesgo del ictus isqumico (ver tabla), el ms
importante, por ser el ms frecuente y por el hecho de ser modificable, es la hipertensin arterial.
El riesgo relativo de padecer un ictus isqumico est directamente
relacionado con las cifras elevadas de tensin arterial, tanto la diastlica como la sistlica, por lo que los pacientes con hipertensin sistlica
aislada tambin tienen ms riesgo de sufrir un ictus isqumico.
El grado y la progresin de la aterosclerosis carotdea se relaciona
con los niveles de colesterol total y LDL-colesterol, mientras que sucede a la inversa con el HDL-colesterol. Debido a esto es especialmente
importante el control de la hipercolesterolemia, tanto en prevencin
primaria como en secundaria del ictus isqumico.
El tratamiento con anticonceptivos orales aumenta el riesgo, sobre
todo si se asocia con otros factores (HTA, tabaquismo), especialmente
en anticonceptivos con alto contenido de estrgenos. Est en tela de
juicio si la terapia hormonal sustitutiva en la menopausia incrementa
o no el riesgo.
El tabaquismo promueve la formacin de placas de ateroma, disminuye el flujo sanguneo cerebral y favorece la hiperviscosidad sangunea, lo que conlleva a multiplicar por dos el riesgo de ictus isqumico. A los cinco aos de dejar de fumar, el riesgo se iguala con los no
fumadores.
La diabetes aumenta el riesgo de padecer infarto principalmente
de tipo aterotrombtico, asociado a la microangiopata y al aumento
de la aterosclerosis cerebral. Adems, los ictus sufridos por pacientes
diabticos tienen mayor morbimortalidad.

Pregunta 80. Factores de riesgo del ictus isqumico.

12
324565789

1234546
1275856
129
6
12

56

24565789

12 

2
2225
56
125

2
6
125  5254556
1255 6

12
56
123  6
12 5
2! 2
2225  6
1255
 2"  56

12# 6
12#
4546
12$5%454
222&56
12'5(5

Pregunta 81.- R: 3
Una prdida de fuerza de instauracin brusca sugiere un origen
vascular. Las prdidas de fuerza asociadas a crisis epilpticas aparecen
tras una crisis motora (parlisis postcrtica o de Todd) o en las crisis
atnicas, que son generalizadas.
Cuando sospechamos un cuadro vascular, se trata de establecer el
rea de lesin dependiendo de los sntomas: un cuadro motor puede
aparecer por diversas lesiones. La desviacin conjugada de los ojos se
presenta en lesiones frontales y en lesiones protuberanciales, diferencindose (entre otros datos) en que los ojos miran hacia el hemicuerpo
partico en el caso de las lesiones protuberanciales y hacia el sano en
las frontales.

CTO Medicina C/ Nez de Balboa, 115 28006 MADRID (Espaa) Tfno.: (91) 782 43 32 / Fax: (91) 782 43 27
E-mail: secretaria@ctomedicina.com; iberocto@ctomedicina.com WEB: www.ctomedicina.com; www.iberocto.com

Comentarios TEST

Seguimiento a distancia

NEUROLOGA

Preparacin Examen de Seleccin 05/06 1 Vuelta


En este caso, la presencia de hemianopsia asociada al defecto motor
automticamente nos debe hacer pensar en una lesin hemisfrica, y
nos permite descartar un proceso lacunar, que cursara con un defecto motor aislado. La exploracin del campo visual en un paciente no
colaborador no es sencilla; nos servimos del reflejo de amenaza, que
consiste en aproximar rpidamente hacia los ojos un objeto por el
campo visual que queremos valorar. Esto provoca un cierre inconsciente de los prpados; si no se produce, deducimos que existe una
prdida de visin por esa rea.
En un paciente del que conocemos como antecedente un IAM
reciente, podemos apostar por una etiologa cardioemblica como
ms probable si aparecieran en otras opciones infarto en la arteria
cerebral media por otras causas.
Hay que recordar que los mbolos cardiacos tienen como destino
ms frecuente la arteria cerebral media.

Comentarios TEST

Pregunta 82.- R: 1
La arteria cerebral anterior irriga la porcin medial de los lbulos
frontal y parietal. Por lo que respecta a las funciones sensitiva y motora, un infarto en este territorio provoca una afectacin contralateral
de predominio crural; en el caso del dficit sensitivo ser de sensibilidades combinadas, como corresponde a un dficit cortical, y no
cursar con anestesia ni con prdida de sensibilidades aisladas.

Seguimiento a distancia

inters, mutismo acintico) o con desinhibicin (moria; menos frecuentemente).


Pueden aparecer los reflejos de liberacin frontal (prensin, succin, palmomentoniano).
La incontinencia de esfnteres es ms frecuente en las lesiones bilaterales, aunque tambin puede encontrarse en lesiones unilaterales.
Un infarto frontal bilateral tambin puede producirse dado que la
malformacin ms frecuente del polgono de Willis es el origen comn de ambas arterias cerebrales anteriores, faltando en una de ellas
el segmento arterial entre la cartida interna y la comunicante anterior, lo que cursa de forma asintomtica debido a la suplencia de riego
por esta ltima.
Las alteraciones del campo visual de causa vascular pueden estar
originadas por una lesin en territorio de la arteria cerebral media, de
la coroidea anterior o de la cerebral posterior, pero no de cerebral
anterior.
Las lesiones frontales bilaterales pueden cursar con un cuadro clnico que semeja el de la hidrocefalia a presin normal, con apraxia
de la marcha, incontinencia urinaria y deterioro de funciones superiores.
Pregunta 83.- R: 3
Ante una hemianopsia de instauracin brusca, con lo cual, de
probable origen vascular, es el resto de sntomas asociados lo que nos
da el diagnstico.
Si la hemianopsia no se acompaa de otros dficits, lo ms probable es que sea en territorio de la arteria cerebral posterior. En los casos
de lesin del territorio completo de esta arteria, aparece hemianopsia
y hemianestesia, por lesin talmica, como sucede en este caso.
Si la hemianopsia se acompaa de hemiparesia y hemihipoestesia,
podemos estar ante una lesin de arteria cerebral media o de coroidea
anterior. La primera es ms frecuente, presenta un dficit de predominio faciobraquial y probablemente se acompaar de desviacin
oculoceflica, mientras que la coroidea da una hemiparesiahemihipoestesia completas, sin desviacin de la mirada.
En el caso de la pregunta, la opcin de la cerebral media est
prcticamente descartada al conservar el paciente un lenguaje normal, dado que el dficit visual es derecho, y por lo tanto la lesin es
izquierda: un paciente con lesin de cerebral media que le produce
hemianopsia asociara trastornos del lenguaje.
Dado que nuestra sospecha clnica es isquemia en lbulo occipital y
tlamo, la prueba de imagen de eleccin ser una TAC (la RM es en
efecto para fosa posterior, pero no es este caso) que si aparece normal en
un primer momento, ser porque an no se han producido cambios
estructurales tras la isquemia, lo que sucede pasadas las primeras horas.
El mecanismo ms frecuente de oclusin del flujo sanguneo en el
territorio de la arteria cerebral posterior es el emblico, y aunque en
ocasiones estas arterias pueden originarse a partir de las comunicantes
posteriores, lo ms frecuente es que se originen en la bifurcacin de la
basilar, por lo que la patologa ateromatosa carotdea que pueda estar
asociada no justifica isquemia en este territorio.
Pregunta 84.- R: 4
Atencin a la descripcin clnica del cuadro, porque no es muy
detallada y hay que considerar distintos datos:
Dificultad para sostener cosas con la mano: esto podra corresponder a una paresia, o bien a una ataxia, con torpeza por incoordinacin, y sta a su vez de etiologa cerebelosa o sensitiva (lesin talmica
o en cordones posteriores).
Dificultad para hablar: esto puede referirse a una afasia motora
(con dificultad para emitir lenguaje) o a una disartria, tambin con
dificultad para la emisin, pero por mala articulacin, sea de causa
cerebelosa o por algn dficit motor. Quiz tambin una disfona
podra describirse de esta manera, pero parece ms apropiado
que para una disfona se refieran alteraciones en la voz.

Pregunta 82. A) Territorios vasculares cerebrales. B) Vascularizacin del tronco


del encfalo y del polgono de Willis.

Por afectacin de reas prefrontales se puede acompaar de trastornos del comportamiento, bien con inhibicin (abulia, apata, desM exico A rgentina
C hile U ruguay

Los antecedentes en los que se describen distintos factores de riesgo cerebrovascular no se corresponden con un cuadro vascular, de
inicio brusco.
La exploracin describe una desviacin de la comisura hacia la
derecha, esto es, una paresia facial izquierda. El hecho de encontrar

CTO Medicina C/ Nez de Balboa, 115 28006 MADRID (Espaa) Tfno.: (91) 782 43 32 / Fax: (91) 782 43 27
E-mail: secretaria@ctomedicina.com; iberocto@ctomedicina.com WEB: www.ctomedicina.com; www.iberocto.com

NR Pg. 19

dficit de fuerza facial apunta que la dificultad del habla ser una disartria
motivada por esta debilidad. Si es capaz de cerrar el ojo izquierdo es
que no es una parlisis completa, es decir, tiene una paresia facial
izquierda central (o supranuclear), no nuclear ni perifrica.
Una vez etiquetado el dficit en el lenguaje como disartria por
paresia facial, la dificultad con la mano izquierda parece menos probable que se trate de una ataxia (ya que el habla no es por lesin
cerebelosa y se describe una exploracin sensitiva normal), y pensaremos antes en una paresia.
Tenemos un cuadro tpico que asocia dificultad para hablar y
dificultad con una mano: la disartria - mano torpe, causada por una
infarto lacunar capsular, en este caso derecho, puesto que el dficit es
izquierdo.
De las otras opciones, quiz merezca la pena recordar que los
hematomas pueden cursar clnicamente con un sndrom,e lacunar,
siempre que tengan la misma localizacin que esperamos del infarto
lacunar: un hematoma en cpsula interna derecha podra dar este
cuadro clnico, pero no el hematoma descrito en la primera opcin.
Pregunta 85.- R: 2
La TC es la prueba de imagen de eleccin en patologa cerebrovascular en el momento agudo. Ya sabemos que cuanto ms precoz sea,
hay menos posibilidades de ver reas isqumicas, ya que la
hipodensidad del parnquima refleja el grado de edema y destruccin del tejido, que no empieza inmediatamente, por lo que es posible encontrar una TC normal cuando en realidad hay infarto. Pero lo
que nos interesa en el momento agudo es descartar que se trate de un
proceso hemorrgico o de etiologa distinta de la vascular, y para ver
sangre, la TC es la prueba de imagen de eleccin.
Limitaciones del TAC: el momento hiperagudo (explicado antes);
la definicin (lesiones de milmetros de dimetro pueden no distinguirse en algunos TC, dependiendo de la calidad del aparato) y la fosa
posterior, porque en esos cortes se incluye la base del crneo y el
hecho de que haya mucho hueso empeora la definicin.
Por lo tanto, si se especifica cul es la prueba de eleccin para ver
infartos lacunares, es la RM, lo que no quiere decir que la mayor parte
no sean visibles tambin con la TC, pero alguno se escapara.
Lo que es visible desde el primer momento son las hemorragias, de
tal forma que, ante la sospecha de patologa hemorrgica (clnica de
HSA, subdural, antecedentes de hematomas espontneos previos) la
prueba de imagen de eleccin sera la TC.
Las reas hipodensas (correspondientes a infarto) suelen dibujar
bien el territorio vascular de la arteria obstruida: son infartos en cua,
porque generalmente suelen tener ms trozo de corteza que de rea
subcortical. Hay procesos en que la distribucin del infarto no corresponde con un territorio vascular arterial:
Los infartos venosos, en los que se produce isquemia por dificultad
en el retorno venoso, y las reas hipodensas no tiene un patrn
comn en todos los pacientes; y
Los infartos hemodinmicos: cuando no hay presin suficiente, se
produce una isquemia en las reas ms distales de los vasos afectados, que suelen corresponder a los mrgenes de los territorios vasculares (denominados territorios frontera).
Pregunta 86.- R: 4
El episodio descrito corresponde a una amaurosis fugaz del ojo
izquierdo, lo que apunta directamente a patologa carotdea izquierda. En el fondo de ojo se podran haber apreciado, quiz, cristales de
colesterol, resto del mbolo fragmentado para acabar de completar la
definicin.
El episodio de hormigueos en la mano derecha de das antes es
muy posible que corresponda a un ataque isqumico transitorio hemisfrico izquierdo, adonde podra ir un fragmento de trombo que se
desprendiera de la cartida izquierda.
Luego estamos ante un paciente con dos AIT que suponemos de
origen aterotrombtico carotdeo izquierdo.
El proceso diagnstico a seguir con una cartida sintomtica es el
descrito:
Eco-doppler de TSA: si fuera normal o sugerente de estenosis no
significativa, se antiagregara ante el cuadro clnico. Si sugiere estenosis significativa, estara indicada la realizacin de una angiografa
Pg. 20 NR

M exico A rgentina
C hile U ruguay

NEUROLOGA

Preparacin Examen de Seleccin 05/06 1 Vuelta

para determinar con la mayor exactitud posible el grado de estenosis. Probablemente dentro de poco se pueda determinar el grado con ms exactitud en angiografas no convencionales (angioTC, angio-RM) pero de momento la prueba gold standard es la
arteriografa de troncos supraarticos.
Arteriografa: con el grado de estenosis de certeza, caben dos opciones: por encima del 70% de estenosis hay indicacin de intervencin (endarterectoma carotdea); con menos estenosis est indicado mantener la antiagregacin.
En estenosis carotdeas altas pero menores del 70%, sintomticas,
hay dudas sobre qu hacer, pero la anticoagulacin slo se contempla en cartidas que estn a la espera de ciruga, no como tratamiento
de mantenimiento a medio plazo.
La angioplastia carotdea como sustitucin endovascular de la
endarterectoma abierta est dando buenos resultados, pero an es
una tcnica pendiente de consolidarse.
Pregunta 87.- R: 1
La etiologa de los sangrados intracraneales puede sugerirse por la
localizacin de los mismos:
Los hematomas epidurales se producen por rotura de ramas arteriales dependiendo de la arteria menngea media: nunca son espontneos.
Los hematomas subdurales son por rotura de venas subdurales;
generalmente traumticos, pero a veces, pueden aparecer sin antecedente claro, sea por el tiempo de latencia desde el traumatismo
o por la escasa energa que se precisa y que hace que se olvide.
Las hemorragias subaracnoideas se producen por rotura de aneurismas saculares (ms frecuente) o malformaciones vasculares (ms
frecuentes en jvenes de menos de 20 aos de edad).
Los hematomas intraparenquimatosos lobares, en el lmite entre
la sustancia gris y la blanca de los lbulos hemisfricos, se producen por rotura de malformaciones vasculares en personas jvenes
y asociado a rotura de vasos con depsitos amiloides (angiopata
congfila) en personas mayores.
Los hematomas profundos se producen por rotura de vasos perforantes; estos vasos sufren de forma crnica debido a los factores de
riesgo vascular, especialmente la hipertensin y la diabetes, y su
rotura puede relacionarse con un aumento de presin arterial que
a veces no es excesivo, si no fuera porque se parte de una situacin
previa delicada. Dentro de los territorios profundos, la localizacin
ms frecuente son los ganglios basales (putamen, caudado, tlamo), tronco del encfalo y cpsula interna.
Pregunta 88.- R: 4
La angiopata amiloide es un proceso de personas mayores, en las
que se forman depsitos de material amiloide en las paredes arteriolares.
Puede asociarse a la Enfermedad de Alzheimer, que tambin presenta
depsitos de amiloide extravasculares. La rotura de estas arteriolas de
pared rgida por estos depsitos origina hematomas lobares, en el lmite
cortico-subcortical de los hemisferios. No tiene tratamiento, porque los
depsitos ya no se movilizan, y su curso es presentar hematomas espontneos de repeticin en esas localizaciones.
Las malformaciones vasculares, teniendo en cuenta su distinta naturaleza, disposicin, tamao y localizacin, pueden ser la causa de sangrados subaracnoideos o intraparenquimatosos (superficiales o profundos). Son ms frecuentes en pacientes jvenes, aunque pueden debutar a cualquier edad. El pronstico, tanto de recurrencias como de un
primer sangrado, ante el hallazgo de una malformacin en principio
asintomtica, debe establecerse de forma individualizada.
Los aneurismas saculares se forman en las bifurcaciones vasculares, ms frecuentemente en arterias del polgono de Willis, y dentro
de ste son ms frecuentes en los vasos que dependen de las cartidas (la localizacin ms frecuente: bifurcacin cerebral anterior
comunicante anterior). Se denominan congnitos, aunque lo que
en realidad lo es la debilidad en las paredes arteriales en esas bifurcaciones, que con los aos dan lugar a las dilataciones saculares.
Distintos son los aneurismas fusiformes: su localizacin tpica es la
arteria basilar, y se pueden considerar una dilatacin anmala de la
misma, que no se rompe ni sangra, y que puede dar problemas como

CTO Medicina C/ Nez de Balboa, 115 28006 MADRID (Espaa) Tfno.: (91) 782 43 32 / Fax: (91) 782 43 27
E-mail: secretaria@ctomedicina.com; iberocto@ctomedicina.com WEB: www.ctomedicina.com; www.iberocto.com

Comentarios TEST

Seguimiento a distancia

NEUROLOGA

Preparacin Examen de Seleccin 05/06 1 Vuelta


otra lesin ocupante de espacio a ese nivel. La indicacin de intervenir depende del tamao.
La hipertensin de forma crnica es la causa ms frecuente de
hemorragia intraparenquimatosa, y su localizacin ms habitual es en
territorios vasculares de arterias perforantes, cuyas paredes son las que
ms sufren la presin arterial elevada.
Pregunta 89.- R: 2
Cuidado con la nomenclatura! Lo importante es tener los conceptos claros:
El mecanismo de gnesis de trombo a nivel vascular es por la agregacin de plaquetas y fibrina sobre una estenosis previa, que generalmente est a su vez condicionada por el depsito de colesterol.
A este fenmeno le podemos denominar atero- (placa de ateroma)
trombosis. Lo ms frecuente es que el trombo formado sobre la
placa de ateroma no obstruya el vaso en ese punto (aunque esto
puede suceder), sino que el agregado se libere y circule hasta obstruir un vaso distalmente: a este fenmeno lo podemos llamar
embolia artero-arterial o aterotromboembolia, pero el mecanismo
de gnesis es la aterotrombosis, y el tratamiento debe ir encaminado a evitar la aterotrombosis.
El mecanismo de formacin de un cogulo es la activacin de
factores de coagulacin generalmente en una zona vascular de
flujo lento (venas profundas, corazn) y la isquemia cerebral slo
puede hacerlo mediante su circulacin y la oclusin distal.
En la prctica, a la hora de abordar el diagnstico etiolgico de un
proceso isqumico cerebral en un paciente, no se suele plantear a
priori la causa, ya que hay que completar el estudio con pruebas en
ambos sentidos.
Ante el hallazgo de patologa cardiaca embolgena, lo ms frecuente es que el ictus haya tenido un origen emblico cardiaco, y ante la
existencia de lesiones carotdeas estenosantes, lo ms probable es que
se trate de un mecanismo aterotrombtico.
Existen ictus indeterminados, cuando el paciente tiene ambas cosas, o cuando no se encuentra justificacin al proceso. Estadsticamente
la mayor parte de los procesos cerebrovasculares isqumicos tienen
en su gnesis mecanismos aterotrombticos con suelta de trombos a
distancia, por eso es correcta la opcin 2.

Comentarios TEST

SISTEMA NERVIOSO PERIFRICO.


Pregunta 90.- R: 1
El estudio neurofisiolgico es bsico para determinar las alteraciones estructurales bsicas del nervio perifrico: fundamentalmente
encaminado a distinguir las afectaciones axonales de las desmielinizantes.
La afectacin axonal produce una disminucin de la amplitud del
potencial nervioso, que en principio se conducir con una velocidad
normal si se conserva la mielina.
La afectacin desmielinizante produce un enlentecimiento de las
velocidades de conduccin; en ocasiones el enlentecimiento se produce en puntos concretos en los que se detecta una dificultad en la
conduccin desproporcionadamente marcada respecto al resto (bloqueo de conduccin). La desmielinizacin produce otro efecto aadido: el potencial que conduce el nervio generalmente es nico,
mientras que si la conduccin se produce por un segmento
desmielinizado, las velocidades de conduccin son variables, detectndose distintos potenciales en lugar de uno solo tras un estmulo
(dispersin).
La mayor parte de las polineuropatas de causa metablica tienen
afectacin mixta axonal y desmielinizante, con disminucin de las
velocidades de conduccin y de las amplitudes.
Pregunta 91.- R: 4
Las manifestaciones neurolgicas asociadas al VIH son mltiples y
no pocas corresponden al nervio perifrico. Pueden producirse por
afectacin directa debida al propio virus, por infecciones oportunistas, o por efectos secundarios del tratamiento.
De forma general, en las fases iniciales de la infeccin, la afectacin
del sistema nervioso perifrico suele tener un mecanismo inmunolM exico A rgentina
C hile U ruguay

Seguimiento a distancia

gico, con lo que puede responder a terapias en ese sentido


(gammaglobulinas, plasmafresis, corticoides). En este contexto estara
la forma ms frecuente de afectacin en el comienzo de la infeccin:
la polineuropata desmilinizante aguda, que recuerda a un sndrome
de Guillain-Barr.
En fases avanzadas predominan las neuropatas carenciales y txicas, y la ms frecuente es la polineuropata sensitiva simtrica distal,
con distribucin en guante y calcetn, que probablemente es de
causa multifactorial.
En fases avanzadas puede aparecer tambin la polirradiculopata
que se asocia a la infeccin concomitante por CMV, que se puede
demostrar en la biopsia, y que puede responder al ganciclovir. Tambin el VIH de forma directa puede causar una polirradiculitis.
La amiotrofia proximal en miembros inferiores es un cuadro tpico
dentro de las afectaciones del sistema nervioso perifrico asociada a la
diabetes mellitus, en especial la tipo 2. Cursa con dolor en raz de
miembros inferiores, seguido de prdida de fuerza y de masa muscular, y puede ser reversible si se trata de forma agresiva.
Pregunta 92.- R: 4
La diabetes mellitus es responsable de un gran nmero de patologas en el sistema nervioso perifrico, mediadas por microangiopata y
microinfartos en los nervios perifricos.
Los patrones de afectacin del sistema nervioso perifrico por culpa de la DM son bsicamente de tres tipos:
Mononeuropatas: son caractersticas las mononeuropatas craneales, aunque tambin pueden aparecer en extremidades. La
ms frecuente es la parlisis incompleta del III par (con conservacin de la actividad pupilar). Generalmente tienen buen pronstico y se recuperan sin secuelas.
Neuropata motora proximal (denominada amiotrofia diabtica):
generalmente en DM tipo 2, presentan dolor en miembros inferiores, junto con prdida de fuerza y de masa muscular, de predominio proximal. Estos pacientes suelen responder al tratamiento
insulnico, incluso aunque no tuvieran un mal control con antidiabticos orales, y recuperan en gran medida los sntomas. Se ha
postulado que esta forma de neuropata tendra un mecanismo
distinto al del resto de las polineuropatas asociadas a la DM
Neuropata autonmica: hipotensin ortosttica, alteraciones de
la sudoracin, vejiga neurgena, diarreas y disautonoma cardiaca,
con trastornos del ritmo y posible muerte sbita.
Polineuropata difusa: existe alteracin de las fibras sensitivas perifricas, con alteracin de las fibras finas (dolor; parestesias, quemazn), las fibras gruesas (alteraciones en la posicin, pie neuroptico
por torceduras recurrentes) y las autonmicas (trastornos trficos
cutneos, lceras). Es la forma ms frecuente y generalmente presenta una mezcla de estos sntomas en distintos grados, con una
distribucin en guante y calcetn.
Salvo escasas excepciones, el tratamiento intensivo de la diabetes
no consigue revertir sintomatologa, y lo que ms puede hacer es
prevencin.
Pregunta 93.- R: 5
El diagnstico de sndrome de Guillain-Barr es eminentemente
clnico, ante la presencia de una debilidad ascendente con abolicin
de los reflejos de estiramiento muscular, incluso en reas corporales
en las que an no hay debilidad significativa.
Se produce una desmielinizacin muy proximal en las races anteriores medulares (las races motoras), con lo que el cuadro es casi
nicamente motor, pero no obstante pueden aparecer sntomas de
otros sistemas (sensitivo, vegetativo), siempre en menor intensidad que
la paresia.
El curso clsico es ascendente, aunque existe el denominado sndrome de Miller-Fisher, en el que predomina la debilidad superior
(craneofacial). En el Guillain-Barr clsico, la musculatura que menos
se afectar ser la ocular, aunque en los casos ms graves tambin
puede verse afectada.
La prueba complementaria que establece el diagnstico son los
hallazgos neurofisiolgicos: el estudio del nervio perifrico no suele
mostrar hallazgos patolgicos, dado que la desmielinizacin es muy

CTO Medicina C/ Nez de Balboa, 115 28006 MADRID (Espaa) Tfno.: (91) 782 43 32 / Fax: (91) 782 43 27
E-mail: secretaria@ctomedicina.com; iberocto@ctomedicina.com WEB: www.ctomedicina.com; www.iberocto.com

NR Pg. 21

proximal, pero hay una onda (denominada onda F) que se detecta en


el nervio perifrico tras un estmulo que recorre el nervio hacia atrs y
de nuevo hacia adelante, y que est abolida en el sndrome de GuillainBarr, debido a la lesin. El estudio neurofisiolgico es, por otra parte,
el primero en ser positivo.
En el LCR se puede encontrar un aumento de protenas, sin que
haya aumento de clulas inflamatorias (disociacin albmino-citolgica) porque est producido por la desmielinizacin. Es sugerente del
diagnstico, pero si no se encuentra no lo descarta.
Pregunta 94.- R: 4
Ante un cuadro clnico tpico de sndrome de Guillain-Barr (infeccin previa, arreflexia, debilidad de curso ascendente, ausencia de
sntomas no motores) debera establecerse el diagnstico con estudio
neurofisiolgico y/o con estudio del LCR.
Establecido el diagnstico, deberamos recordar que se trata de un
proceso autoinmune en el que circulan autoanticuerpos contra la
mielina del sistema nervioso perifrico. Cuando tenemos clnica es que
se ha producido la sntesis de dichos anticuerpos, en el tiempo de
latencia entre el proceso infeccioso y el debut del cuadro neurolgico.
Los corticoides y otros inmunosupresores pueden disminuir e incluso frenar la sntesis de anticuerpos, pero en el caso del GuillainBarr no son eficaces, porque a la aparicin de los sntomas ya se han
producido los anticuerpos; por tanto, los corticoides no son eficaces.
La medida ms til es retirar de la circulacin los anticuerpos circulantes: hay varios sistemas, siendo el clsico la plasmafresis (sustitucin del suero por un compuesto artificial), pero hay otros sistemas
menos agresivos para el paciente e igualmente eficaces (inmunoadsorcin, administracin de gammaglobulina policlonal). Estos mtodos son tanto ms eficaces cuanto ms precoces, ya que estn intentando impedir la gnesis de ms lesin. En fases avanzadas no consiguen beneficios.
La medida que ms ha influido en la supervivencia de estos enfermos es el soporte vital, dado que, al tratarse de un proceso desmielinizante con poco dao axonal, puede producirse una remielinizacin
si el paciente no fallece por complicaciones asociadas (dificultad respiratoria o en la deglucin).
No hay toxina involucrada en el sndrome de Guillain-Barr; los
sueros antitoxina son tiles en el botulismo o en el ttanos.
Pregunta 95.- R: 4
La distrofia muscular de Duchenne es en efecto una enfermedad
ligada al X, con un gen alterado en el brazo corto de dicho cromosoma, que debera codificar una protena presente en la membrana
muscular, denominada distrofina, y que no existe en estos pacientes.
Hay que pensar en esta enfermedad en un paciente que, a los 2-4
aos de edad, tras un desarrollo normal, empieza a presentar dificultad para correr, levantarse del suelo y subir escaleras. En ese momento
ya puede ser evidente a la exploracin la hipertrofia de pantorrillas,
por sustitucin del msculo por grasa.
El paciente tendr una marcha dificultosa, bambolendose hacia
los lados por insuficiencia muscular en cintura plvica, y para levantarse del suelo lo tiene que hacer apoyndose. En torno a los 10 aos
de edad se es incapaz de caminar. No hay tratamiento; la muerte se
produce alrededor de los 20 aos por problemas respiratorios; slo
en un 10 % de los casos es por problemas cardiacos.
El cuadro completo de esta enfermedad incluye la miopata de
msculo estriado, la miocardiopata y un bajo cociente intelectual.
No hay alteraciones sensitivas, y los reflejos estn abolidos.
Es especialmente importante el diagnstico de mujeres portadoras,
dado que la mayor parte son asintomticas (la coincidencia de los dos
cromosomas X anmalos es letal intratero). Un estudio que incluya
EKG, CPK seriadas, biopsia muscular y una buena historia familiar,
permite asegurar el diagnstico de portadora al 95%; tambin puede
realizarse el estudio gentico de los cromosomas X.
Pregunta 96.- R: 4
Hay que tener presente el diagnstico de un paciente que debute en
la edad adulta con fenmeno miotnico y debilidad facial y en manos
de forma progresiva, dado que la distrofia miotnica de Steinert es la
distrofia ms frecuente de la edad adulta, y la primera manifestacin
Pg. 22 NR

M exico A rgentina
C hile U ruguay

NEUROLOGA

Preparacin Examen de Seleccin 05/06 1 Vuelta

muscular es precisamente la miotona de predominio en manos, que


no suele ser incapacitante, que mejora con el calentamiento (tras unas
pocas contracciones voluntarias) y empeora en los das fros.
Con la progresin de la enfermedad se afecta la musculatura facial,
de la masticacin, del cuello, antebrazos y piernas. Pero se trata de
una enfermedad sistmica, con lo que el paciente puede tener disfagia, trastornos esfinterianos, hipogonadismo, trastornos de conduccin cardiaca (en el 70% de los pacientes), alteraciones valvulares
cardiacas y deterioro cognitivo progresivo.
El aspecto de estos pacientes es caracterstico: ptosis palpebral,
sonrisa invertida con boca entreabierta (por debilidad facial), voz
gangosa, cataratas y calvicie.
El diagnstico se establece por la clnica, el estudio electromiogrfico
(que muestra un patrn mioptico con descargas miotnicas), y la biopsia
muscular, que muestra en efecto atrofia de fibras tipo I. La CPK est
normal o levemente elevada. El estudio gentico (cromosoma 19) es
diagnstico.
No hay tratamiento salvo el sintomtico, intentando evitar infecciones intercurrentes; control de trastornos cardiacos, con marcapasos si es preciso, y la miotona, que rara vez requiere tratamiento,
puede controlarse con fenitona o procainamida.
Pregunta 97.- R: 4
La miastenia gravis se produce por un bloqueo de los receptores de
acetilcolina de la placa muscular. Este bloqueo no es completo, en
cuyo caso el paciente no podra realizar ningn movimiento, pero
impide el normal funcionamiento de la placa neuromuscular, que se
hace ms evidente a medida que se repite la transmisin de potenciales.
Este empeoramiento progresivo relacionado con la llegada repetida de
estmulos tiene su correlato clnico en la fatigabilidad: la fuerza y los
reflejos estn inicialmente conservados, pero una repeticin del acto
motor va empobreciendo la transmisin y aparece la debilidad.
Este mecanismo se objetiva con el estudio neurofisiolgico: el primer potencial de accin es normal (como lo son los reflejos, un solo
estmulo) pero la estimulacin repetitiva (como si se hiciera un ejercicio) conlleva una prdida de la intensidad de la transmisin (respuesta
decremental).
Dado que la afectacin es exclusivamente de la placa motora, en
un paciente miastnico no deben aparecer signos de otros sistemas
afectados: autonmico, sensitivo, cerebelo... Slo un dficit motor
con fatigabilidad; si aparecen otros datos en la exploracin, habr
que replantear el diagnstico.
La distribucin anatmica de la afectacin en la miastenia gravis es
preferentemente ocular, y si hay una generalizacin a tronco y extremidades, la afectacin es predominantemente proximal.
Pregunta 98.- R: 2
Los anticuerpos responsables de la miastenia gravis se sitan en la
hendidura sinptica, en la membrana muscular, sobre los receptores
de acetilcolina.
Cuando un receptor de acetilcolina es bloqueado por anticuerpos, queda inutilizado de forma definitiva, con lo que tiene que ser
reabsorbido y sustituido. Este recambio se hace por recaptacin de
segmentos de membrana en los que van incluidos los receptores, por
lo que, si el recambio es muy intenso porque haya muchos receptores
afectados, la reposicin no se hace con la misma velocidad, y empieza a faltar membrana en la sinapsis.
La sinapsis habitualmente tiene pliegues complicados, con el fin de
tener ms superficie donde situar receptores de acetilcolina; si se quita mucha membrana, queda poco espacio para receptores, con lo
que el funcionamiento de la sinapsis no vuelve a ser correcto por
cambios estructurales y no slo por el bloqueo de los receptores.
La miastenia se asocia frecuentemente con alteraciones tmicas,
siendo la ms frecuente la hiperplasia, pero en un 10% de los casos se
asocia a franca tumoracin de origen tmico, lo que automticamente
convierte la reseccin del timo en una medida indicada. En los otros
casos en los que se detecta bien una hiperplasia o bien un timo de
apariencia normal (el hecho de que se detecten restos tmicos en una
persona adulta hay quien lo considera patolgico, puesto que debe
atrofiarse antes) la indicacin quirrgica la da el cuadro clnico: debut precoz, mala respuesta al tratamiento mdico, evolucin rpida.

CTO Medicina C/ Nez de Balboa, 115 28006 MADRID (Espaa) Tfno.: (91) 782 43 32 / Fax: (91) 782 43 27
E-mail: secretaria@ctomedicina.com; iberocto@ctomedicina.com WEB: www.ctomedicina.com; www.iberocto.com

Comentarios TEST

Seguimiento a distancia

NEUROLOGA

Preparacin Examen de Seleccin 05/06 1 Vuelta


Pregunta 99.- R: 3
El diagnstico de miastenia gravis debe sospecharse ante una clnica sugerente (opcin 1). Toda debilidad fluctuante es sugerente de
miastenia gravis, como causa ms frecuente de sndrome miasteniforme. Si el paciente es un varn, mayor, con tumor conocido o sndrome constitucional, quiz habr que pensar primero en un sndrome
de Eaton-Lambert.
Ante la sospecha clnica existen una serie de pruebas complementarias que confirman el diagnstico:
Test de edrofonio: una recuperacin inmediata y transitoria de
fuerza tras la administracin de este anticolinestersico de vida
media corta i.v. apoya la sospecha de que la debilidad la cause un
bloqueo en la placa motora.
Estimulacin repetitiva: el estudio basal es normal, pero puede
apreciarse como los potenciales de accin disminuyen de tamao
cuando se somete al msculo a una salva de estmulos.
Estudio de fibra aislada: con un electrodo especial, se valoran los
tiempos de latencia entre el estmulo y el potencial de accin muscular en una misma fibra muscular; en una persona normal, el
tiempo de latencia permanece constante, mientras que si existe
bloqueo de la placa motora, se detecta amplia variabilidad en esos
intervalos de tiempo.
Deteccin de anticuerpos antirreceptor de acetilcolina: no es especfico al 100%, dado que pueden aparecer asociados a otros
tipos de miastenia (inducida) o a otras patologas autoinmunes.

Seguimiento a distancia

curso de semanas. A partir de esa edad es excepcional que haya una


colonizacin por la bacteria, por los cambios del pH gstrico y la flora
bacteriana, por lo que en nios mayores y adultos el cuadro se produce por ingesta de toxina preformada: con una latencia que va de
horas a escasos das (media de 48 horas) aparecen los sntomas: pupilas fijas, boca seca, leo paraltico y debilidad generalizada, de predominio en musculatura craneal (oftalmoplejia externa, ptosis, debilidad facial) y bulbar (disfagia, disartria, disfona) pero que puede llegar
a ser generalizada.
La latencia del botulismo adquirido por heridas va de 4 a 51 das,
ms prolongada que la de la va digestiva.

Comentarios TEST

Ninguna de estas pruebas complementarias es diagnstica per se, y


el hecho de encontrar alguna de ellas negativa no descarta el diagnstico de miastenia gravis.

Pregunta 99. Tratamiento de las distintas formas de miastenia gravis.

Pregunta 100.- R: 5
El botulismo es un cuadro mediado por la toxina botulnica, que
bloquea la liberacin de acetilcolina de las terminales presinpticas.
No atraviesa la barrera hematoenceflica, por lo que la transmisin
colinrgica central no se ve afectada.
Hay dos vas por las que se puede adquirir la toxina: por heridas
contaminadas (no una herida simple, sino profunda, o con muy mal
curso clnico, dado que la produccin de la toxina se produce en
medios anaerobios) o digestiva, con paso de toxina a travs de la
pared del tubo digestivo.
La va digestiva es la ms frecuente. En lactantes (menos de un ao)
que ingieran esporas de C. botulinum, stos pueden proliferar en el
tracto digestivo y producir toxina; el cuadro clnico es progresivo en el
M exico A rgentina
C hile U ruguay

CTO Medicina C/ Nez de Balboa, 115 28006 MADRID (Espaa) Tfno.: (91) 782 43 32 / Fax: (91) 782 43 27
E-mail: secretaria@ctomedicina.com; iberocto@ctomedicina.com WEB: www.ctomedicina.com; www.iberocto.com

NR Pg. 23

NEUROCIRUGA

Preparacin Examen de Seleccin 05/06 1 Vuelta


HIPERTENSIN INTRACRANEAL E HIDROCEFALIA
1.

Ante un caso de HTIC en fase avanzada, sera MENOS frecuente


la aparicin de:
1)
2)
3)
4)
5)

2.

Preguntas TEST

Sndrome de Reiter.
Enfermedad de Behet.
Sfilis terciaria.
Infeccin por criptococo.
Dficit del complejo vitamnico B.

La primera medida teraputica a considerar en una paciente


diagnosticada de "pseudotumor cerebri" idioptico sera:
1)
2)
3)
4)
5)

6.

Ateromatosis carotdea.
Absceso cerebeloso de origen otolgico.
Enfermedad de Lermitte-Duclos.
Astrocitoma piloctico de cerebelo.
Diseccin de arteria vertebral.

Un varn de 35 aos viene sufriendo cefaleas, vmitos, visin


borrosa y diplopa en varias ocasiones desde hace pocas semanas.
La exploracin fsica evidencia lesiones aftosas en mucosa oral y
en glande. Fondo de ojo: edema de papila. TC cerebral sin
hallazgos. Que diagnstico le parece ms probable?:
1)
2)
3)
4)
5)

5.

Anisocoria.
Debilidad motora en extremidades izquierdas.
Malformacin arteriovenosa como patologa de base.
Hidrocefalia.
Defecto campimtrico inferior izquierdo.

7.

8.

Administracin de corticoides.
Fenestracin del nervio ptico.
Realizacin de punciones lumbares repetidas.
Derivacin lumboperitoneal.
Restriccin hidrosalina y diurticos.

Cul de las siguientes patologas se puede asociar con hidrocefalia comunicante con mayor probabilidad?:
M exico A rgentina
C hile U ruguay

Carcinoma microctico de pulmn.


Vasculopata hipertensiva.
Enfermedad de Dandy-Walker.
Quiste coloide.
Tumor de clulas germinales.

Un anciano de 70 aos presenta un cuadro consistente en


trastornos mnsicos, incontinencia urinaria y torpeza al caminar que ha ido apareciendo a lo largo de varios meses. El
diagnstico ms probable de entre los siguientes es:
1)
2)
3)
4)
5)

Una paciente de 30 aos acude a urgencias por cuadro de dolor


occipitocervical y vrtigo de aparicin brusca, acompaado de
vmitos e inestabilidad en la marcha. Tras pocas horas en
observacin, presenta deterioro progresivo en el nivel de
conciencia. Se realiza TAC cerebral que muestra una hipodensidad difusa en hemisferio cerebeloso izquierdo, con efecto de
masa y compresin del IV ventrculo, originando herniacin
transtentorial inversa e hidrocefalia obstructiva. Indique el
diagnstico etiolgico ms probable de entre los siguientes:
1)
2)
3)
4)
5)

4.

Alteraciones del patrn respiratorio.


Ondas A o "plateau" en el registro de PIC.
Lesin del III par craneal.
lceras de Cushing.
Taquicardia supraventricular.

Un varn de 24 aos acude a Urgencias por crisis epilptica de


pocos segundos de duracin y cefalea hemicraneal. A su ingreso
se realiza TC cerebral, que muestra un hematoma en lbulo
temporal derecho, con efecto de masa y abierto a ventrculo
lateral. Indique el hallazgo MENOS probable de entre los siguientes en relacin con este paciente:
1)
2)
3)
4)
5)

3.

1)
2)
3)
4)
5)

Seguimiento a distancia

Encefalopata de Binswanger.
Hidrocefalia crnica del adulto.
Enfermedad de Alzheimer.
Carcinoma de prstata con metstasis cerebrales.
Mielopata cervical espondiloartrsica.

Un paciente de 5 aos de edad, con derivacin ventriculoperitoneal colocada a los pocos das del nacimiento por hidrocefalia secundaria a estenosis del acueducto de Silvio, ha estado
bien hasta hace 2 das, cuando comenz con un cuadro de
cefalea, vmitos y disminucin del nivel de conciencia. Cul de
estas medidas NO sera adecuada?:
1)
2)
3)
4)
5)

Realizar una TC craneal para descartar hidrocefalia por


malfuncin valvular.
Estudio del fondo de ojo.
Pedir radiografa del trayecto valvular para observar una
posible rotura.
Iniciar tratamiento con dexametasona.
Ingreso hospitalario para observacin.

TUMORES
9.

Seala lo FALSO en relacin a las neoplasias cerebrales secundarias o metastsicas:


1)
2)
3)
4)
5)

Suponen la neoplasia intracraneal ms frecuente.


Son ms frecuentes a nivel supratentorial.
El primario sistmico ms frecuente es el carcinoma broncopulmonar.
La tendencia al sangrado intralesional es mayor que en los
tumores primarios del SNC.
El tratamiento es fundamentalmente quirrgico en la mayora
de estas lesiones.

10. Un varn de 55 aos presenta una clnica de hemiparesia


izquierda y deterioro cognitivo de un mes de evolucin. En el
estudio RM se observa una lesin expansiva cerebral frontal
derecha, subcortical, que alcanza cuerpo calloso, de morfologa irregular, captacin de contraste en anillo y edema perilesional. Analtica general y Rx de trax sin hallazgos patolgicos.
Diagnstico ms probable:
1)
2)
3)
4)
5)

Linfoma.
Glioblastoma.
Toxoplasmosis.
Metstasis.
Absceso.

11. Varn de 50 aos con crisis comiciales de 2 aos de evolucin


que ltimamente no responden adecuadamente a la medica-

CTO Medicina C/ Nez de Balboa, 115 28006 MADRID (Espaa) Tfno.: (91) 782 43 32 / Fax: (91) 782 43 27
E-mail: secretaria@ctomedicina.com; iberocto@ctomedicina.com WEB: www.ctomedicina.com; www.iberocto.com

NC Pg. 1

NEUROCIRUGA

Preparacin Examen de Seleccin 05/06 1 Vuelta

cin. En la TC cerebral aparece una lesin frontal derecha con


zonas calcificadas y que no capta contraste. Cul sera el
diagnstico y actitud teraputica ms aconsejable?:

2)

1)
2)
3)
4)
5)

4)

Glioblastoma: ciruga + RT.


Ependimoma: vigilancia con RM anuales.
Oligodendroglioma: ciruga.
Meduloblastoma: ciruga + RT craneoespinal.
Metstasis: descartar tumor sistmico.

12. Es FALSO, en relacin al ependimoma intracraneal:


1)
2)
3)
4)
5)

Puede diseminarse por el espacio subaracnoideo.


La localizacin ms frecuente es el suelo del cuarto ventrculo.
Suele debutar con hidrocefalia.
La variante mixopapilar tpicamente aparece a nivel supratentorial.
El dato histolgico tpico son las formaciones en roseta.

13. Sobre los tumores enceflicos en nios, es ERRNEO:


1)
2)
3)
4)
5)

Suponen la neoplasia ms frecuente despus de las leucemias.


El tumor supratentorial ms frecuente es el astrocitoma cerebral.
Suelen debutar con clnica de HTIC.
Las metstasis son excepcionales.
El meduloblastoma suele ser de localizacin vermiana.

14. La existencia de una tumoracin qustica con calcificacin


marginal, de localizacin supraselar, en un joven de 16 aos que
desde hace 3 meses refiere prdida de visin en ambos campos
temporales, sugiere el diagnstico de:
1)
2)
3)
4)
5)

Teratoma.
Quiste dermoide.
Craneofaringioma.
Adenoma hipofisario.
Astrocitoma qustico hipotalmico.

15. Seale lo FALSO en relacin al linfoma del SNC:


1)
2)
3)
4)
5)

Es una neoplasia de clulas B de bajo grado.


Asocia afectacin menngea al inicio o afectacin ocular en
ms de un 10 % de los casos.
Aparece con mayor frecuencia en pacientes trasplantados o
con SIDA.
En inmunodeprimidos, se asocia invariablemente con una
infeccin de las clulas tumorales por el virus de Epstein-Barr.
La radioterapia es la base del tratamiento.

16. Seale la opcin FALSA sobre los meningiomas:


1)
2)
3)
4)
5)

Son ms frecuentes en mujeres en la 5 y 6 dcadas de la vida.


La localizacin ms frecuente es la base craneal.
A veces presentan alteraciones genticas en el cromosoma 22.
La radioterapia es eficaz ante restos o recidivas que progresan.
Son tumores que derivan de la aracnoides.

17. Cul de las siguientes opciones es FALSA respecto al hemangioblastoma?:


1)

Es el tumor primario intraaxial de fosa posterior ms frecuente


del adulto.

Pg. 2 NC

M exico A rgentina
C hile U ruguay

3)

5)

El tratamiento consiste fundamentalmente en la exresis


quirrgica del tumor.
Hasta un 50% de los casos se asocian a enfermedad de Von
Hippel-Lindau.
La imagen radiolgica ms habitual es una lesin qustica con
un ndulo mural.
Puede cursar con alteraciones hematolgicas.

18. Seale las localizaciones ms frecuentes de los neurinomas


intracraneales:
1)
2)
3)
4)
5)

VIII par y VII par.


VIII par y V par.
V par y VI par.
VI par y VII par.
VIII par y IX par.

19. Un varn de doce aos es estudiado en Endocrinologa por


diabetes inspida y cefaleas. La exploracin neurolgica
pone de manifiesto una leve dificultad en la elevacin de la
mirada. Los niveles sricos de HCG y AFP son normales.
Usted realizara una RM cerebral para descartar principalmente uno de los siguientes diagnsticos:
1)
2)
3)
4)
5)

Craneofaringioma.
Glioma hipotalmico.
Colesteatoma en cisternas de la base.
Germinoma de regin pineal.
Macroadenoma hipofisario.

20. Indique lo FALSO sobre el quiste coloide cerebral:


1)
2)
3)
4)
5)

Se origina en la regin anterior del tercer ventrculo.


Es un derivado de la parfisis.
Presenta una pseudocpsula de tejido glial.
Su contenido es PAS +.
Puede producir hidrocefalia aguda con el cambio de posicin de la cabeza.

21. La asociacin de hamartomas del iris, pseudoartrosis de la tibia


y glioma de vas pticas es propia de uno de los siguientes
sndromes neurocutneos:
1)
2)
3)
4)
5)

Von Hippel-Lindau.
Sturge-Weber.
Neurofibromatosis tipo I.
Neurofibromatosis tipo II.
Esclerosis tuberosa.

22. NO incluye tumores cerebrales una de las siguientes entidades:


1)
2)
3)
4)
5)

MEN tipo I.
Neurofibromatosis tipo I.
Enfermedad de Bourneville.
Enfermedad de Lafora.
Sndrome de Turcot.

TRAUMATISMOS CRANEOENCEFLICOS (TCE)


23. Paciente con TCE leve con prdida de conocimiento inicial,
herida en cuero cabelludo y fractura lineal en la Rx de crneo.
Cuando usted lo ve, est asintomtico y la exploracin neuro-

CTO Medicina C/ Nez de Balboa, 115 28006 MADRID (Espaa) Tfno.: (91) 782 43 32 / Fax: (91) 782 43 27
E-mail: secretaria@ctomedicina.com; iberocto@ctomedicina.com WEB: www.ctomedicina.com; www.iberocto.com

Preguntas TEST

Seguimiento a distancia

NEUROCIRUGA

Preparacin Examen de Seleccin 05/06 1 Vuelta


lgica es normal. Cul de las siguientes actitudes es la ms
indicada?:
1)
2)
3)
4)
5)

Alta a su domicilio.
TC craneal y, si es normal, alta.
TC craneal, para descartar lesiones asociadas e ingresar 24
horas para observacin.
TC craneal y ciruga de la fractura, realizando fijacin interna.
TC craneal y, si es normal, dar antibiticos profilcticos por
el riesgo elevado de infeccin intracraneal.

24. Seale la FALSA respecto a las fracturas del peasco:


1)
2)
3)
4)
5)

La fractura longitudinal puede afectar al nervio facial en hasta


un 20% de los casos.
La fractura transversal suele asociar hipoacusia de conduccin.
Es frecuente la lesin del tmpano en las fracturas longitudinales.
Las fracturas oblicuas cursan casi invariablemente con parlisis facial.
El signo de Battle sugiere fractura de la base del crneo.

25. Tras un intervalo asintomtico, un joven de 25 aos que ha


sufrido un accidente de motocicleta comienza a presentar
deterioro progresivo del nivel de conciencia hasta alcanzar un
GCS de 10/15, acompaado de anisocoria y prdida de fuerza
en hemicuerpo derecho. La proyeccin lateral de una placa de
crneo evidenci una fractura lineal de escama temporal izquierda. Cul de las siguientes opciones es cierta respecto al
diagnstico que debe sospechar?:
1)
2)
3)
4)
5)

El pronstico de recuperacin funcional de este paciente es


muy malo incluso con ciruga.
El tratamiento de eleccin es la ciruga mediante realizacin
de trpano.
Lo ms probable es que se deba a rotura de venas puente
corticales.
Hay signos en la exploracin que sugieren herniacin transtentorial.
Existe riesgo alto de infeccin intracraneal asociada a la
fractura.

Preguntas TEST

26. Es correcto, en relacin al dao axonal difuso:


1)
2)
3)
4)
5)

Presenta siempre alteraciones en la TC cerebral.


Habitualmente ocurre en TCE con flexoextensin brusca
cervical.
El deterioro neurolgico aparece a partir de la primera
semana del TCE.
El punteado hemorrgico que aparece en la TC suele localizarse en los hemisferios cerebrales y el cuerpo calloso.
Tiene muy buen pronstico.

27. Una mujer de 40 aos que consulta por exoftalmos unilateral


pulstil, estrabismo convergente de ojo derecho y sensacin
de soplo retroocular, con el antecedente de traumatismo
craneal hace 24 horas, llevar a pensar en el diagnstico de:
1)
2)
3)
4)
5)

Neumoencfalo a tensin.
Aneurisma postraumtico de arteria oftlmica.
Meningocele intraorbitario.
Trombosis de seno cavernoso.
Fstula cartido-cavernosa.
M exico A rgentina
C hile U ruguay

Seguimiento a distancia

HEMORRAGIA SUBARACNOIDEA (HSA)


28. En relacin con la hemorragia subaracnoidea, es FALSO de
entre las siguientes opciones:
1)
2)
3)
4)
5)

Su etiologa no traumtica ms frecuente es la rotura de un


aneurisma cerebral.
Se detectan aneurismas mltiples en menos de un 5% de los
casos.
Los aneurismas fusiformes suelen aparecer en circulacin
vertebrobasilar.
La tcnica de eleccin para el diagnstico de HSA es la TC
cerebral.
La isquemia subendocrdica puede ser una complicacin
que se vea dentro del contexto clnico de una hemorragia
subaracnoidea.

29. El aneurisma mictico cerebral se relaciona con:


1)
2)
3)
4)
5)

Infeccin candidisica en un enfermo VIH +.


Absceso cerebral.
Hemorragia subaracnoidea de repeticin.
Diseminacin hemtica de una aspergilosis pulmonar.
Endocarditis bacteriana.

30. Una mujer de 35 aos presenta un cuadro clnico de hemorragia subaracnoidea asociado a ptosis palpebral derecha y midriasis ipsilateral. Indique la etiologa que le parece ms probable:
1)
2)
3)
4)
5)

Aneurisma de arteria comunicante posterior.


Aneurisma de arteria oftlmica.
Malformacin arteriovenosa en mesencfalo.
Aneurisma de arteria comunicante anterior.
Fstula cartido-cavernosa.

31. En un paciente que al sptimo da de sufrir una HSA presenta


un cuadro de disfasia y deterioro del nivel de conciencia, si
descartamos resangrado en una TC cerebral, el tratamiento
ms recomendable sera:
1)
2)
3)
4)
5)

Restriccin hidrosalina.
Reducir la presin arterial con diurticos y betabloqueantes.
Administrar suero hipertnico.
Practicar clipaje urgente del aneurisma causante de su hemorragia.
Aumentar dosis de calcioantagonistas, inducir hipotensin
arterial y realizar la embolizacin urgente del aneurisma.

32. La asociacin con aneurisma cerebral se ha descrito ms


frecuentemente en una de estas cardiopatas congnitas:
1)
2)
3)
4)
5)

Tetraloga de Fallot.
Coartacin artica.
Transposicin de los grandes vasos.
Anomalas de los cojinetes subendocrdicos.
Ductus arterioso persistente.

PATOLOGA RAQUIMEDULAR
33. Una mujer de 50 aos acude a la consulta por lumbocitica
derecha de 2 meses de evolucin. El dolor aumenta con las

CTO Medicina C/ Nez de Balboa, 115 28006 MADRID (Espaa) Tfno.: (91) 782 43 32 / Fax: (91) 782 43 27
E-mail: secretaria@ctomedicina.com; iberocto@ctomedicina.com WEB: www.ctomedicina.com; www.iberocto.com

NC Pg. 3

maniobras de Valsalva y la flexin anterior de tronco. La


exploracin neurolgica evidencia reflejo patelar homolateral
dbil, con fuerza y sensibilidad conservadas. Realizara una RM
para descartar una hernia discal ms probablemente localizada
a nivel del espacio:
1)
2)
3)
4)
5)

1)
2)
3)
4)
5)

Hiporreflexia bicipital.
Hiporreflexia estilorradial.
Debilidad a la extensin del codo.
Hipoestesia en segundo dedo de la mano.
Aumento de la cervicobraquialgia en la maniobra de Spurling.

35. El tratamiento de eleccin para un paciente de 65 aos de edad


que consulta por dolor lumbar irradiado a miembros inferiores,
claudicacin neurgena intermitente y dolor a la extensin del
tronco es:
1)
2)
3)
4)
5)

Discectoma.
Laminectoma.
Flavectoma.
Corpectoma.
Artrodesis.

2)
3)
4)
5)

En dficits medulares est indicado el tratamiento con metilprednisolona en altas dosis en las 8 primeras horas del
traumatismo.
La ciruga descompresiva est sobre todo indicada en dficits
medulares completos que no mejoran en las primeras 24 h.
Sndrome de Brown-Squard: dficit motor y sensibilidad
propioceptiva ipsilateral + defecto en la sensibilidad termoalgsica contralateral por debajo de la lesin.
Sndrome medular central: presencia de paresia de miembros superiores de intensidad mayor que en miembros
inferiores.
El tratamiento inicial de una listesis cervical traumtica con
dficit neurolgico es la traccin cervical.

37. Seale lo INCORRECTO en relacin con la patologa de columna cervical:


1)
2)
3)
4)
5)

La fractura de Jefferson compromete los arcos anterior y


posterior del atlas.
La enfermedad de Morquio predispone a luxacin atloaxoidea por hipoplasia de la apfisis odontoides del axis.
La fractura de pedculos de C3 tambin es conocida como
fractura de Hangman.
El esguince cervical es la patologa ms frecuente de columna
cervical.
La presencia de dficit neurolgico en un traumatismo
cervical sin lesin sea es indicacin absoluta de estudio
mediante RM.

Pg. 4 NC

M exico A rgentina
C hile U ruguay

Duracin del dolor > 1 mes.


Reposo en cama sin alivio.
Dficit motor en la pierna.
Toma crnica de glucocorticoides.
Edad menor de 50 aos.

39. El tumor intrarraqudeo ms frecuente es una metstasis.


Indique, de entre las siguientes neoplasias slidas, cul disemina con MENOS frecuencia al canal vertebral:
1)
2)
3)
4)
5)

Mama.
Pulmn.
Prstata.
Rin.
Colon.

MALFORMACIONES DEL SNC


40

Cul de los siguientes binomios es FALSO?:


1)
2)
3)
4)
5)

36. Es FALSO sobre las lesiones medulares traumticas:


1)

38. En la valoracin inicial de un dolor lumbar agudo no suelen estar


indicados estudios complementarios de laboratorio o radiogrficos, excepto ante la sospecha de una etiologa grave. Indique
cul de los siguientes NO se considera factor de riesgo para este
posible origen grave de lumbalgia:

L1-L2.
L2-L3.
L3-L4.
L4-L5.
L5-S1.

34. En una hernia discal cervical localizada en el espacio C5-C6,


sera ms DIFCIL encontrar en la exploracin:
1)
2)
3)
4)
5)

NEUROCIRUGA

Preparacin Examen de Seleccin 05/06 1 Vuelta

Escafocefalia - Sutura sagital.


Trigonocefalia - Sutura metpica.
Plagiocefalia - Sutura pterional.
Paquicefalia - Sutura lambdoidea.
Braquicefalia - Sutura coronal.

41. Con respecto a la malformacin de Chiari es FALSA:


1)
2)
3)
4)
5)

El tipo I se asocia con frecuencia a siringomielia y suele


debutar al nacimiento.
El tipo II se asocia con hidrocefalia y mielomeningocele.
El tipo III se asocia con meningoencefalocele.
El tipo IV es una hipoplasia de cerebelo.
El sntoma ms frecuente es la cefalea suboccipital.

42. Seale cul de los siguientes defectos del tubo neural es una
espina bfida manifiesta:
1)
2)
3)
4)
5)

Seno drmico congnito.


Diastematomielia.
Quiste perineural de Tarlov.
Sndrome de regresin caudal.
Raquisquisis.

43. Seale lo FALSO referente al mielomeningocele:


1)
2)
3)
4)
5)

Cerca del 90% de los pacientes desarrollan hidrocefalia.


La posicin del nio debe ser en decbito prono.
Se ha relacionado con el dficit de cido flico.
Suelen detectarse niveles anormalmente bajos de AFP en
lquido amnitico.
La mayor seguridad diagnstica prenatal la aporta la ecografa.

44. Una joven de 15 aos presenta fiebre y rigidez de nuca desde


hace unas horas. La exploracin fsica evidencia una lesin
angiomatosa con hipertricosis en la vecindad de una pequea

CTO Medicina C/ Nez de Balboa, 115 28006 MADRID (Espaa) Tfno.: (91) 782 43 32 / Fax: (91) 782 43 27
E-mail: secretaria@ctomedicina.com; iberocto@ctomedicina.com WEB: www.ctomedicina.com; www.iberocto.com

Preguntas TEST

Seguimiento a distancia

NEUROCIRUGA

Preparacin Examen de Seleccin 05/06 1 Vuelta


depresin en la piel de la lnea media lumbosacra. Qu diagnstico le sugiere?:
1)
2)
3)
4)
5)

Enfermedad de Mollaret.
Quiste aracnoideo.
Mielocele.
Seno drmico congnito.
Infeccin diseminada por criptococo en enferma con SIDA.

OTROS TEMAS
45. Una paciente de 60 aos presenta desde hace varios meses
episodios de dolor muy intenso, lancinante, de presentacin
sbita, de escasa duracin y carcter recidivante, de distribucin
en mejilla y regin perioral de hemifacies izquierda. En ocasiones
se desencadena con la masticacin o el simple bostezo. Instaurara tratamiento a corto plazo con:
1)
2)
3)
4)
5)

1)
2)
3)
4)
5)

Seguimiento a distancia

La Rx simple de columna cervical es imprescindible en el


estudio del paciente politraumatizado.
La TC tiene un valor insuperable en las urgencias neuroquirrgicas.
La RM supera a la TC en el estudio de las fracturas basilares.
La RM es insustituible en el estudio de los tumores de lnea
media.
La RM es de eleccin para el estudio de las lesiones de fosa
posterior.

50. El sndrome del canal de Guyn se relaciona con la compresin


del nervio:
1)
2)
3)
4)
5)

Cubital.
Radial.
Mediano.
Peroneo lateral.
Interdigital metatarsiano.

Fluoxetina.
Sumatriptn.
Derivados mrficos.
Carbamacepina.
Corticoides ms indometacina.

46. La radiociruga se ha demostrado til en todas, EXCEPTO una


de las siguientes patologas:
1)
2)
3)
4)
5)

Meningioma.
Malformaciones arteriovenosas.
Aneurismas.
Metstasis.
Neurinoma del acstico.

47. En cuanto al tratamiento del absceso cerebral, es FALSO:


1)
2)
3)

Preguntas TEST

4)
5)

La evacuacin quirrgica constituye el tratamiento de eleccin cuando hay signos de HTIC.


Las mejores respuestas teraputicas se obtienen en los abscesos nicos en fase de encapsulacin.
La cobertura antibitica abarca desde el momento de la
sospecha de absceso cerebral hasta 7-10 das tras la ciruga.
El absceso cerebeloso de origen otolgico supone una urgencia quirrgica en la mayora de los casos.
La combinacin antibitica con metronidazol-cefotaxima es
una de las ms eficaces.

48. La ciruga funcional estereotxica logra buenos resultados en


ciertos trastornos del movimiento. Indique la FALSA entre las
siguientes asociaciones enfermedad - tcnica quirrgica indicada:
1)
2)
3)
4)
5)

Enfermedad de Parkinson idioptica estimulacin crnica


del ncleo subtalmico.
Discinesias por L-dopa palidotoma.
Distona generalizada idioptica estimulacin de globo
plido interno.
Parlisis supranuclear progresiva estimulacin crnica de
ncleo ventral intermedio del tlamo.
Temblor esencial talamotoma ventrolateral.

49. Con respecto a los estudios de imagen del SNC, es FALSO:


M exico A rgentina
C hile U ruguay

CTO Medicina C/ Nez de Balboa, 115 28006 MADRID (Espaa) Tfno.: (91) 782 43 32 / Fax: (91) 782 43 27
E-mail: secretaria@ctomedicina.com; iberocto@ctomedicina.com WEB: www.ctomedicina.com; www.iberocto.com

NC Pg. 5

NEUROCIRUGA

Preparacin Examen de Seleccin 05/06 1 Vuelta


HIPERTENSIN INTRACRANEAL E HIDROCEFALIA.
Pregunta 1.- R: 5
El sndrome de hipertensin intracraneal (HTIC) es muy importante en Neurociruga, y es comn a un gran nmero de las patologas que vas a estudiar en esta asignatura, por lo que si lo comprendes bien, entenders mejor la clnica y el tratamiento de todas ellas.
Hay una serie de conceptos importantsimos que debemos recordar, aunque resulte un tanto extensa la explicacin de la pregunta:
El cerebro es un rgano que consume grandes cantidades de oxgeno y glucosa para desarrollar sus funciones. Para que le lleguen
stos es imprescindible que se mantenga constante el flujo sanguneo cerebral (FSC), que como sabes depende de la tensin arterial
(TA) y la presin intracraneal (PIC) en funcin de la frmula recogida en la figura a continuacin, de modo que un aumento de esta
ltima condiciona un descenso del FSC. Por tanto, en los pacientes
con sndrome de HTIC est comprometido el aporte de oxgeno y
glucosa al cerebro, y secundariamente su funcin.
FSC = PPC / RV = (TAM-PIC) / RV

Comentarios TEST

Pregunta 1.

El flujo sanguneo cerebral (FSC) se calcula como la presin de


perfusin cerebral (PPC = TAM - PIC) dividida por las resistencias vasculares cerebrales.

Otro concepto importante para entender el sndrome de HTIC es


que el cerebro est incluido dentro de una cavidad rgida, el crneo.
Dentro del crneo hay fundamentalmente parnquima cerebral,
sangre y lquido cefalorraqudeo (LCR). Puesto que el crneo no se
puede distender, cualquier aumento de uno de esos tres componentes va a producir una elevacin de la PIC, cuya manifestacin
clnica ser el sndrome de HTIC. Esto simplifica mucho el estudio de
la clnica de las enfermedades neuroquirrgicas que se desarrollarn posteriormente a lo largo del test, puesto que todos aquellos
procesos intracraneales en los que aumenta el volumen de alguno
de estos componentes (tumores, hemorragias, hidrocefalias, abscesos cerebrales, etc.) tendrn en comn los sntomas de este sndrome, y la clnica variar en funcin de los dficits neurolgicos focales
asociados segn la localizacin de la lesin.
La clnica asociada con la elevacin de la PIC es cefalea, ms
intensa durante la noche, que suele acompaarse de nuseas y/o
vmitos de predominio matutino. Estos sntomas son los primeros
en aparecer. El signo exploratorio caracterstico es el edema de
papila, y en ocasiones puede apreciarse una parlisis del VI par
craneal que da lugar a diplopa (recuerda que el VI par es el que
ms frecuentemente se afecta en los pacientes con HTIC).
Cuando la PIC contina elevndose, los mecanismos de compensacin del cerebro pueden ser insuficientes y puede comprometerse
el flujo cerebral y por tanto el aporte de oxgeno y glucosa. En estas
circunstancias, a los sntomas anteriores se asocian otros que se traducen en mal funcionamiento del cerebro. Se produce entonces
una disminucin progresiva del nivel de conciencia del paciente
que se acompaa de signos de disfuncin del tronco del encfalo.
As, hay mal control de la respiracin, de la tensin arterial y del
latido cardaco. Esta trada clnica, consistente en alteraciones del
patrn respiratorio, bradicardia e hipertensin arterial, se conoce
como trada de Cushing y aparece en fases avanzadas de la HTIC. La
respuesta incorrecta es la 5, porque en estas circunstancias hay un
enlentecimiento de la frecuencia cardaca, no taquicardia. La opcin 1 es cierta, porque es uno de los componentes de la trada.
Si se monitoriza la PIC mediante sensores especiales de presin
colocados por el neurocirujano dentro del crneo, se pueden ver
alteraciones de la morfologa normal de la curva de PIC. La ms
importante es la onda A o plateau, que se recoge en la opcin 2
del enunciado de la pregunta.
La opcin 3 es tambin cierta. En fases avanzadas de la HTIC, el
aumento de presin provoca desplazamiento de las estructuras
cerebrales, que quedan comprimidas contra otras estructuras intracraneales como la duramadre. Estos desplazamientos se conocen como herniaciones cerebrales. La ms frecuente es la herniacin uncal, uno de cuyos sntomas es la parlisis del III par. No
olvides repasar la clnica caracterstica de esta entidad.
M exico A rgentina
C hile U ruguay

Seguimiento a distancia

Las lceras de Cushing mencionadas en la opcin 4 son lceras


gstricas que pueden verse en pacientes graves con HTIC.
Pregunta 2.- R: 5
Las lesiones intracraneales producen una clnica comn a todas
ellas por elevacin de la PIC, el sndrome de hipertensin intracraneal
(HTIC). Sin embargo otros sntomas difieren segn la localizacin de las
lesiones; son los llamados signos neurolgicos focales. Para entenderlos debes repasar la semiologa de las lesiones lobares cerebrales en el
apartado de Semiologa del captulo del Manual CTO.
En esta pregunta se describe el caso clnico de un paciente de 24
aos con un hematoma en el lbulo temporal derecho que ha cursado con crisis epilpticas y cefalea. En el estudio de imagen se aprecia
que el hematoma tiene volumen suficiente como para provocar efecto de masa, y que est abierto al ventrculo, lo que significa que hay
sangre en el interior del sistema ventricular.
En cualquier caso clnico en el que nos pregunten el hallazgo ms
o menos probable, debemos hacer un anlisis como el que hemos
explicado del caso para saber qu sntomas podramos esperar. En
este caso concreto podemos esperar sntomas de HTIC (por el volumen de la lesin), sntomas focales (segn la localizacin de la lesin,
en este caso temporal) y otros relacionados con caractersticas especiales de la lesin que nos describen (en nuestro caso se puede esperar una dificultad para la circulacin del LCR porque los ventrculos
contienen sangre en su interior).
Si acudimos ahora a las opciones, encontramos fcilmente la respuesta errnea. Las opciones 1 y 2 son ciertas porque un hematoma
temporal puede provocar fcilmente un desplazamiento del uncus
temporal por la hendidura del tentorio, es decir, una herniacin uncal, que cursar con parlisis del III par (y por tanto con midriasis de
ese lado que motivar una anisocoria) y hemiparesia contralateral
(debilidad motora en miembros izquierdos, contralateral al hematoma, por compresin del pednculo cerebral mesenceflico, por el
que discurre la va corticoespinal).
La opcin 3 es cierta porque en un paciente joven, sin antecedente traumtico conocido y con un hematoma intraparenquimatoso
lobar, la causa ms frecuente de la hemorragia es la rotura de una
malformacin arteriovenosa. Aprovechamos para recordarte que los
hematomas intraparenquimatosos pueden ser profundos o superficiales (tambin llamados lobares). Debes conocer que la causa ms
frecuente de hematomas intraparenquimatosos profundos es la hipertensin arterial, y que en estos casos suelen localizarse en los ganglios
basales y el tlamo. Si la rotura de una malformacin arteriovenosa es
la causa ms frecuente de hematomas intraparenquimatosos lobares
en jvenes, en los ancianos lo es la angiopata amiloide.
La opcin 4 es tambin posible porque el hematoma que se describe en el enunciado est abierto al sistema ventricular, y por tanto puede
producirse una hidrocefalia por obstruccin del flujo normal del LCR.
La opcin menos probable es la 5, porque por el lbulo temporal
discurren las fibras pticas responsables de la informacin de la parte
superior del campo visual, y por tanto una lesin temporal produce
una cuadrantanopsia superior (no inferior). La figura de la pgina
siguiente es un esquema de los defectos campimtricos asociados
con lesiones en las distintas partes de la va ptica, que debes repasar
en el apartado de Semiologa del Manual CTO.
La clave en esta pregunta estaba en que el dficit neurolgico focal
que se describe no corresponde con la localizacin de la lesin que
se indica en el enunciado.
Pregunta 3.- R: 5
Vamos a analizar el enunciado de la pregunta. Se describe el caso
clnico de una paciente joven con clnica cerebelosa y dolor cervical que sufre un deterioro progresivo del nivel de conciencia. Se
realiza una TAC craneal y se aprecia una hipodensidad difusa en el
hemisferio cerebeloso izquierdo con efecto de masa que produce
herniacin transtentorial, es decir, desplazamiento del cerebelo y
el tronco del encfalo hacia arriba a travs de la hendidura tentorial.
Lo que se pretende es determinar cul es la causa ms probable de
este cuadro.
La opcin 1 no puede ser, porque una ateromatosis de la arteria
cartida interna puede provocar una isquemia en el territorio de la
circulacin anterior (y por tanto supratentorial), mientras que la lesin

CTO Medicina C/ Nez de Balboa, 115 28006 MADRID (Espaa) Tfno.: (91) 782 43 32 / Fax: (91) 782 43 27
E-mail: secretaria@ctomedicina.com; iberocto@ctomedicina.com WEB: www.ctomedicina.com; www.iberocto.com

NC Pg. 1

Seguimiento a distancia

NEUROCIRUGA

Preparacin Examen de Seleccin 05/06 1 Vuelta

Pregunta 2. Defectos campimtricos y lesiones de la va ptica.

Pregunta 4.- R: 2
El cuadro clnico descrito es muy caracterstico, y muy frecuentemente preguntado en el MIR. Corresponde con un sndrome de
hipertensin intracraneal benigna, tambin conocida como pseudotumor cerebri.
Habitualmente lo preguntan describiendo un sndrome de hipertensin intracraneal tpico (cefalea, vmitos, diplopa por parlisis del
VI par y visin borrosa por edema de papila), que cursa en brotes
autolimitados (dicen en el enunciado que le ha pasado en varias
Pg. 2 NC

M exico A rgentina
C hile U ruguay

ocasiones en pocas semanas). Ante este cuadro clnico debemos sospechar HTIC, y la prueba diagnstica de eleccin en los pacientes
con HTIC es la TAC craneal. Lo que caracteriza al pseudotumor cerebri
es que estos pacientes tienen un cuadro clnico de HTIC, pero las
pruebas de imagen en ellos no muestran ninguna alteracin (TAC
craneal sin hallazgos).
Ante un paciente con sospecha de sndrome de HTIC benigna hay
que descartar posibles etiologas que se recogen en la tabla a continuacin. Se ha relacionado con una mayor frecuencia en mujeres
obesas, ciertas enfermedades sistmicas y la toma de ciertos frmacos.
La descripcin en el enunciado de aftas en mucosa oral y genitales es
caracterstica de la enfermedad de Behet (opcin 2), una de las entidades asociadas con este diagnstico.

Pregunta 4. Etiologa del sndrome de hipertensin


intracraneal benigna.

Alteraciones del drenaje venoso.


Embarazo y anticonceptivos orales.
Obesidad.
Hipo e hipertiroidismo.
Hipoparatiroidismo.
Insuficiencia suprarrenal.
Sndrome de Cushing.
Anemia ferropnica severa.
Sarcoidosis.
Lupus eritematoso sistmico.
Enfermedad de Behet
Frmacos:
- Vitamina A y derivados (retinoides).
- Tetraciclinas.
- cido nalidxico.
- Nitrofurantona.
- Sulfamidas.
- Litio.
- Indometacina.
- Fenitona.

CTO Medicina C/ Nez de Balboa, 115 28006 MADRID (Espaa) Tfno.: (91) 782 43 32 / Fax: (91) 782 43 27
E-mail: secretaria@ctomedicina.com; iberocto@ctomedicina.com WEB: www.ctomedicina.com; www.iberocto.com

Comentarios TEST

que nos estn describiendo en el enunciado est en la fosa posterior


(y por tanto en territorio vertebrobasilar).
La opcin 2, el absceso cerebeloso otolgico, podra dar un cuadro clnico y una imagen radiolgica similar. Sin embargo, en contra
est que no se describe clnica previa de infeccin tica y no explica
el dolor occipitocervical. Recuerda que la fiebre no siempre est presente en los pacientes con absceso cerebral, y por tanto no es un
criterio de exclusin.
La enfermedad de Lhermitte-Duclos (opcin 3) es un gangliocitoma difuso de cerebelo, es decir, un tumor, cuya evolucin clnica es
ms progresiva. Lo mismo podramos decir del astrocitoma piloctico
de cerebelo (opcin 4), un tumor benigno que habitualmente tiene
aspecto qustico en la TAC.
La respuesta correcta es la 5. El inicio del cuadro con dolor
occipitocervical y vrtigo de aparicin brusca es muy sugestivo de
diseccin de la arteria vertebral (que tiene un recorrido cervical a
travs de las apfisis transversas de las vrtebras cervicales y despus,
tras rodear por detrs las masas laterales del atlas, se introduce en el
interior del crneo por el foramen occipital para irrigar estructuras
enceflicas de la fosa posterior). El deterioro posterior del nivel de
conciencia del paciente se debe a que la diseccin de la arteria ha
producido un descenso del flujo sanguneo a travs de la misma y por
tanto un infarto isqumico de cerebelo, que es la lesin radiolgica
que se describe en el enunciado. El rea de infarto ha producido
efecto de masa y compresin del tronco, lo que explicara la disminucin del nivel de conciencia. Recuerda que las lesiones isqumicas
cerebrales aparecen en la TAC como lesiones hipodensas (negras),
mientras que las hemorragias son hiperdensas (blancas). La razn es
que la sangre tiene hierro, que se ve blanco en las radiografas).

NEUROCIRUGA

Preparacin Examen de Seleccin 05/06 1 Vuelta


Pregunta 5.- R: 5
El sndrome de hipertensin intracraneal benigna o pseudotumor cerebri se caracteriza por sntomas clnicos de HTIC, con hallazgos exploratorios compatibles (edema de papila) en ausencia de
lesiones objetivables en las pruebas de imagen (TAC craneal normal).
Se ha asociado con algunas enfermedades sistmicas y la toma de
ciertos frmacos (ver tabla anterior).
En esta pregunta se pretende que reflexionemos acerca del manejo
teraputico de este sndrome.
Dado que en muchas ocasiones el pseudotumor cerebri se asocia
con la ingesta de frmacos, la primera medida teraputica en estos
pacientes es suprimir el agente etiolgico probable, y por tanto suspender el frmaco que lo puede estar desencadenando. Sin embargo
esta opcin no est recogida entre las posibles respuestas.
Todas las respuestas que se enumeran en las opciones de esta pregunta son medidas eficaces y tiles en el tratamiento de esta entidad,
pero el manejo debe ser escalonado (ver figura). Por tanto, aunque
todas valdran, la respuesta correcta es la 5, porque la restriccin
hidrosalina y la administracin de diurticos (sobre todo acetazolamida) debe ser la primera medida teraputica de entre las citadas.
El resto se iran realizando a medida que fracasan las medidas anteriores. Tras la restriccin hidrosalina y los diurticos se pueden administrar corticoides. Si estas medidas farmacolgicas no son eficaces se realizan punciones lumbares evacuadores para sacar LCR y reducir la PIC.
Si el paciente precisa mltiples punciones para controlar los sntomas,
se coloca una derivacin lumboperitoneal a fin de sacar LCR de forma
continua y no tener que pinchar al paciente reiteradamente.
Cuando todas estas medidas fracasan hay que recurrir a otras tcnicas quirrgicas ms agresivas, como la descompresin quirrgica de
la vaina del nervio ptico o las craniectomas descompresivas, aunque rara vez es necesario practicarlas.
No olvides que el objetivo del tratamiento de estos pacientes es
conservar la visin, ya que la HTIC repetida produce edema de papila
bilateral que puede conducir a la ceguera.

Seguimiento a distancia

LCR se acumula en el interior de los ventrculos cerebrales, dando


lugar a una dilatacin de los mismos observable en las pruebas de
imagen, que se conoce como hidrocefalia (ver figura).
Hay dos tipos de hidrocefalia. Por un lado estn aquellas en las que
el LCR no es capaz de llegar al espacio subaracnoideo porque hay
una obstruccin en el trayecto intraventricular; son las hidrocefalias
obstructivas o no comunicantes. Un segundo tipo son las hidrocefalias comunicantes, en las que el LCR producido en los ventrculos
puede llegar a los espacios subaracnoideos, pero no circula por ellos
hasta las granulaciones de Paccioni, o si lo hace no se reabsorbe en
estas ltimas.

Pregunta 6. TAC craneal de un neonato con hidrocefalia. Se observa gran


dilatacin del sistema ventricular.

Comentarios TEST

En esta pregunta nos piden determinar la causa ms probable de


una hidrocefalia comunicante (ver esquema a continuacin). Hay
que eliminar por tanto todas las opciones que supongan una obstruccin a la circulacin del LCR por el sistema ventricular. Esto elimina la
opcin 3 (la enfermedad de Dandy-Walker es una atresia de los agujeros de Luschka y Magendie, y por tanto el LCR queda atrapado
antes de salir al espacio subaracnoideo).

Pregunta 5.

Manejo teraputico del Sndrome de Hipertensin Intracraneal


Benigna (Pseudotumor cerebrii).

Pregunta 6.- R: 1
Esta pregunta hace referencia a la etiologa de la hidrocefalia.
La hidrocefalia se produce como consecuencia de un disbalance
entre la formacin y la reabsorcin del LCR. El LCR se forma en el
plexo coroideo de los ventrculos cerebrales y circula a travs de estos
hasta alcanzar el espacio subaracnoideo, para ser reabsorbido en las
granulaciones de Paccioni y pasar al torrente sanguneo. Se producen
diariamente unos 500 cc de LCR, que deben ser reabsorbidos. Si la
produccin es superior a la capacidad de reabsorcin, ese exceso de
M exico A rgentina
C hile U ruguay

Pregunta 6.

Tipos de hidrocefalia y ejemplos de las causas ms frecuentes.

CTO Medicina C/ Nez de Balboa, 115 28006 MADRID (Espaa) Tfno.: (91) 782 43 32 / Fax: (91) 782 43 27
E-mail: secretaria@ctomedicina.com; iberocto@ctomedicina.com WEB: www.ctomedicina.com; www.iberocto.com

NC Pg. 3

NEUROCIRUGA

Preparacin Examen de Seleccin 05/06 1 Vuelta

Las opciones 4 y 5 son tumores que tambin obstruyen las vas de


circulacin normal del LCR (el quiste coloide es un tumor tpico del
tercer ventrculo, y los tumores de clulas germinales se localizan con
ms frecuencia en la regin pineal y pueden obstruir la circulacin en
el tercer ventrculo o en el acueducto de Silvio).
La vasculopata hipertensiva (opcin 2) no es una causa habitual
de hidrocefalia, aunque pueden verse cambios de seal radiolgica
periventricular en estos pacientes.
La respuesta correcta es la 1, porque el carcinoma microctico de
pulmn puede metastatizar al sistema nervioso central. Aunque lo
ms probable es que produzca metstasis cerebrales (masas tumorales
intracerebrales), puede producir tambin metstasis en las meninges
(carcinomatosis menngea). La carcinomatosis menngea es una causa frecuente de hidrocefalia comunicante en la que el LCR llega al
espacio subaracnoideo, pero no puede circular por l porque est
ocupado por clulas tumorales. El mismo mecanismo es responsable
de la hidrocefalia arreabsortiva asociada con infecciones (meningitis)
o con la hemorragia subaracnoidea.
Pregunta 7.- R: 2
Este es uno de los cuadros clnicos ms frecuentemente preguntados en el examen MIR, y siempre lo preguntan del mismo modo
(forma de presentacin clnica), por lo que no se puede fallar. Aprovechamos para recordarte que, aunque clsicamente se ha llamado
hidrocefalia normotensiva o a presin normal (y as la han preguntado en el examen MIR), un nombre ms utilizado por los
neurocirujanos actualmente es hidrocefalia crnica del adulto, y
tambin podran preguntarla de esta otra manera. No en una entidad
nueva; es exactamente lo mismo.
La hidrocefalia crnica del adulto se define por una trada clnica
(trada de Hakim) que incluye demencia con alteraciones de memoria, incontinencia urinaria y alteraciones de la marcha, y es un cuadro
tpico de ancianos (ver figura). Todas las preguntas que han cado en
el examen MIR haciendo alusin a esta entidad se referan a la trada
tpica, preguntada expresamente o incluida en el enunciado para
hacer diagnstico diferencial con otras demencias.

Demencia

Incontinencia urinaria

Apraxia de la marcha

Pregunta 7. Hidrocefalia normotensiva.

Pg. 4 NC

M exico A rgentina
C hile U ruguay

Ante la sospecha clnica se realizar una prueba de imagen cerebral, en la que debe observarse una hidrocefalia comunicante, es
decir, una dilatacin de todo el sistema ventricular sin que se aprecie
una obstruccin a la circulacin del LCR.
Dado que la hidrocefalia crnica del adulto es una causa reversible de demencia, la sospecha clnica exige determinar si el paciente
es candidato a tratamiento. El tratamiento de eleccin es la derivacin ventriculoperitoneal (vlvula).
Antes de la intervencin se realizan una serie de pruebas clnicas
para intentar averiguar si el paciente responder o no al tratamiento
(punciones lumbares evacuadoras, test de infusin, etc.). Todas estas
pruebas intentan predecir si el paciente mejorar o no tras la colocacin de la derivacin. Con las punciones lumbares se pretende sacar
LCR (15 o 20cc) y ver si mejora la capacidad intelectual del paciente
y la marcha, en cuyo caso se colocar la vlvula. Los test de infusin,
algo ms complejos, hacen lo contrario; introducen lquido a travs
de una puncin lumbar y miden las variaciones de presin para ver la
tolerancia del sistema y si es necesaria la vlvula. Si el test de infusin
es positivo, la probabilidad de mejora tras la derivacin es mayor.
Pregunta 8.- R: 4
Nos presentan a un paciente de 5 aos de edad, portador de una
derivacin ventriculoperitoneal para el tratamiento de una hidrocefalia secundaria a estenosis de acueducto (hidrocefalia obstructiva)
detectada al nacimiento, que desde hace 2 das ha empeorado y
presenta una clnica compatible con hipertensin intracraneal. Es
obligatorio sospechar que existe una malfuncin de la vlvula y realizar todos los esfuerzos necesarios para diagnosticarla (ver tabla) y
tratarla a tiempo, porque una malfuncin en un paciente con vlvula
puede ser mortal.
Las derivaciones de LCR son catteres colocados en el interior del
sistema ventricular, que llevan un sistema valvular para regular el paso
de LCR y que lo derivan hacia otras cavidades del organismo (habitualmente el peritoneo, pero tambin la pleura o las cavidades cardacas pueden ser utilizadas). Por tanto, pueden malfuncionar por problemas en cualquier punto de su recorrido, y es obligatorio estudiar todo el trayecto de la vlvula.
La opcin 1 est indicada ante la sospecha de malfuncin para
valorar el tamao del sistema ventricular (si hay una dilatacin respecto a los estudios previos puede indicar que la derivacin no est funcionando, o lo est haciendo mal, y adems permite ver la posicin
del extremo ventricular del catter, que debe estar colocado dentro
del ventrculo).
La opcin 2, el examen del fondo de ojo, debe realizarse en todo
paciente con sospecha de HTIC, porque permite objetivar el edema
de papila caracterstico de estos enfermos.
Es obligatorio tambin realizar un estudio del trayecto valvular (opcin 3) mediante Rx simple de todo el recorrido (crneo, cuello, trax
y abdomen), con el fin de comprobar la integridad del sistema de
derivacin (que no hay roturas ni desconexiones entre sus elementos)
y la correcta ubicacin de los extremos ventricular y distal (debe estar
en el interior de la cavidad elegida).
Aunque no est recogida como opcin en la pregunta, otra causa de
malfuncin de la vlvula es la infeccin, por lo que ante una clnica
sugestiva de mal funcionamiento ms fiebre debe descartarse la infeccin mediante estudio bioqumico y microbiolgico del LCR (obtenido
por puncin directa del reservorio valvular o por puncin lumbar).
Si se encuentra la causa del mal funcionamiento de la vlvula (hay
roturas, desconexiones, infecciones o se ha salido el catter del ventrculo
o de la cavidad peritoneal), debe ser revisada quirrgicamente. Ningn
paciente con sospecha de malfuncin valvular puede ser dado de alta;
como mnimo debe ser ingresado para observar la evolucin (opcin 5)
y debe recibir tratamiento especfico si lo requiere.
No est indicado el tratamiento con dexametasona (opcin 4) ante
la sospecha de malfuncin valvular. La dexametasona es un tratamiento muy eficaz para la HTIC secundaria a tumores cerebrales con
edema vasognico, pero no sirve para tratar la HTIC secundaria a
hidrocefalia. El tratamiento de eleccin ante una malfuncin valvular es la revisin quirrgica de la misma.
No obstante, aprovechamos para recordar que, aunque las derivaciones de LCR son las intervenciones indicadas para el tratamiento

CTO Medicina C/ Nez de Balboa, 115 28006 MADRID (Espaa) Tfno.: (91) 782 43 32 / Fax: (91) 782 43 27
E-mail: secretaria@ctomedicina.com; iberocto@ctomedicina.com WEB: www.ctomedicina.com; www.iberocto.com

Comentarios TEST

Seguimiento a distancia

NEUROCIRUGA

Preparacin Examen de Seleccin 05/06 1 Vuelta


permanente de la hidrocefalia, actualmente el tratamiento de eleccin para la hidrocefalia obstructiva secundaria a estenosis del acueducto de Silvio (el paciente de la pregunta) es la ventriculostoma
endoscpica del suelo del tercer ventrculo, que puede resolver la
hidrocefalia sin necesidad de derivaciones de LCR y por tanto evita los
reingresos por malfuncin valvular. Esta tcnica utiliza un endoscopio
para navegar por el interior del sistema ventricular hasta el suelo del
tercer ventrculo y crear all una comunicacin artificial entre el sistema
ventricular y el espacio subaracnoideo, sin necesidad de que el paciente sea portador de prtesis, por lo que actualmente se considera que
debe intentarse antes de la colocacin de una derivacin en todos los
pacientes con estenosis de acueducto. Si en el nio del caso clnico se
hubiera realizado lo anterior, en lugar de colocar la vlvula, no tendramos que plantearnos ahora todas las medidas del enunciado.

Pregunta 8. Manejo diagnstico del paciente con sospecha


de malfuncin valvular.
Se sospecha por historia clnica.
Exploracin fsica:
- Nivel de conciencia.
- Fondo de ojo.
- Exploracin neurolgica.
- Fiebre y rigidez nucal.

TAC craneal.
Rx del trayecto valvular.
Anlisis bioqumico y microbiolgico del LCR.
Ingreso para observacin.
Revisin quirrgica si lo requiere.

Comentarios TEST

TUMORES.
Pregunta 9.- R: 5
Esta pregunta hace referencia a las metstasis cerebrales.
Las respuestas 1 a 4 son correctas. Las metstasis cerebrales suponen la neoplasia intracraneal ms frecuente en el adulto, y habitualmente son de localizacin supratentorial, a nivel corticosubcortical,
aunque tambin pueden producirse en la fosa posterior. Son excepcionales en nios.
En las lneas siguientes te describimos los conceptos ms importantes que hay que conocer respecto a las metstasis cerebrales para el
examen MIR:
Origen. Si vemos a un paciente con una imagen en la TAC sugestiva
de metstasis cerebral (lesin hipodensa que capta contraste perifricamente o en anillo, rodeada de abundante edema vasognico), el tumor primario sistmico que ms probablemente ser responsable de la metstasis es el carcinoma de pulmn. Sin embargo, si consideramos por separado los tumores primarios, el que
con ms frecuencia metastatiza en el SNC es el melanoma.
Tendencia al sangrado. Aunque hay muchos tumores cerebrales
primarios con tendencia al sangrado (glioblastoma, ependimoma,
oligodendroglioma, adenoma de hipfisis, etc.), las metstasis tienen mayor tendencia a sangrar que los tumores primarios, sobre
todo las procedentes de carcinoma de pulmn, coriocarcinoma,
melanoma, rin y tiroides. Como otras lesiones hemorrgicas,
cuando han sangrado son hiperdensas (blancas) en la TAC.
Indicaciones de tratamiento. La respuesta incorrecta es la 5.
Aunque la ciruga est indicada ante una lesin metastsica nica
con efecto de masa en una regin accesible del cerebro sin riesgo
de secuelas neurolgicas importantes, el tratamiento de eleccin
de las metstasis cerebrales es la radioterapia holocraneal. Hay que
tener en cuenta que, con mucha frecuencia, las metstasis cerebrales son mltiples, y el hecho de que en la TAC slo veamos una
no quiere decir que no haya ms no visibles, por lo que debe
radiarse todo el cerebro.
Por tanto, y para concretar, el tratamiento de eleccin de las metstasis cerebrales es la radioterapia holocraneal, aunque deben operarM exico A rgentina
C hile U ruguay

Seguimiento a distancia

se aquellas nicas y accesibles que producen HTIC o efecto de masa


significativo.
Pregunta 10.- R: 2
Se describe el caso clnico de un varn de edad media con clnica
neurolgica focal (hemiparesia), en el que se encuentra en un estudio
de RM una lesin expansiva cerebral frontal, de morfologa irregular,
que se extiende a travs del cuerpo calloso, y que se realza en anillo
tras administrar medio de contraste. Hay dos datos importantes para el
diagnstico, el patrn de captacin de contraste y el hecho de que
sea una lesin infiltrante (una lesin irregular en el lbulo frontal que
se extiende por la sustancia blanca hasta el cuerpo calloso).
Un dato que suele ayudar en el diagnstico diferencial es la imagen radiolgica tpica de captacin en anillo (ver tabla). Sin embargo no es til en esta pregunta, porque todas las opciones descritas
(linfoma, glioblastoma, toxoplasmosis, metstasis y absceso cerebral)
pueden dar esta imagen. Por tanto, debemos llegar a la respuesta
correcta a travs de otros datos del enunciado.

Pregunta 10. Lesiones cerebrales con captacin de contraste


en anillo en la TAC craneal.

Glioblastoma multiforme.
Metstasis.
Linfoma cerebral primario.
Toxoplasmosis.
Absceso pigeno (bacteriano).

La opcin ms probable es que se trate de un tumor primario


maligno, un glioblastoma multiforme (opcin 2). Recuerda que los
gliomas son los tumores cerebrales primarios ms frecuentes en el
adulto. Son tumores intraparenquimatosos, que suelen localizarse a
nivel subcortical. Se trata de tumores infiltrantes, es decir, que no
tienen un plano de separacin claro con el cerebro normal. Pueden
extenderse por la sustancia blanca, y por ello los de localizacin
frontal alcanzan frecuentemente el cuerpo calloso y pueden llegar
hasta el otro hemisferio (tumores en alas de mariposa). Se clasifican
segn su agresividad histolgica en gliomas de bajo grado, gliomas
anaplsicos y glioblastoma multiforme (el ms agresivo de todos). Este
ltimo es el que caractersticamente capta en anillo y puede cruzar
por el cuerpo calloso al otro lado.

Pregunta 10. Captacin de contraste en anillo. Glioblastoma multiforme

Las metstasis (opcin 4) son muy frecuentes, y tienen una imagen


similar en anillo, pero no suelen ser infiltrantes, y tienen un plano
ms o menos definido de separacin con el cerebro. No dan el patrn en alas de mariposa. Adems, el hecho de que la Rx de trax y la
analtica sean normales, aunque no es definitivo, ayuda a excluir esta
opcin.

CTO Medicina C/ Nez de Balboa, 115 28006 MADRID (Espaa) Tfno.: (91) 782 43 32 / Fax: (91) 782 43 27
E-mail: secretaria@ctomedicina.com; iberocto@ctomedicina.com WEB: www.ctomedicina.com; www.iberocto.com

NC Pg. 5

NEUROCIRUGA

Preparacin Examen de Seleccin 05/06 1 Vuelta

Los abscesos cerebrales (opcin 5) son lesiones purulentas de origen bacteriano generalmente, que producen clnica de HTIC y focalidad
neurolgica. No necesariamente cursan con fiebre o alteraciones analticas (leucocitosis), pero suelen dar este dato en las preguntas MIR.
Como son lesiones encapsuladas, tienden a tener un plano de separacin con el cerebro (no se comportan como lesiones infiltrantes).
La toxoplasmosis (opcin 3) da un patrn radiolgico similar en
pacientes inmunocomprometidos. Es la principal causa de lesin ocupante de espacio intracerebral en pacientes infectados por VIH, y el
hallazgo de una lesin cerebral en estos pacientes es indicacin de
tratamiento emprico antitoxoplasma (sulfadiacina y pirimetamina).
Son lesiones localizadas, no infiltrantes.
Los linfomas cerebrales (opcin 1) pueden ser tumores primarios o
secundarios (metstasis neurolgicas de un linfoma sistmico). Estas s
son lesiones infiltrantes, de elevada malignidad, pero se ven habitualmente en pacientes inmunodeprimidos (aunque cada vez son ms frecuentemente diagnosticadas en inmunocompetentes). Entre los primarios, los ms frecuentes son los linfomas de alto grado de clulas B. Es
caracterstica una distribucin periventricular, aunque no obligada. Frecuentemente la imagen radiolgica mejora tras la administracin de
corticoides, por lo que se han llamado tumores fantasma. El tratamiento de eleccin es la radioterapia y quimioterapia con metotrexate.
Por tanto, por lo considerado hasta ahora lo ms probable es que
la lesin descrita sea un glioblastoma multiforme.

jan por tanto con el enunciado. El glioblastoma y las metstasis s son


tumores del adulto y supratentoriales, pero tienen una historia natural
ms corta (este paciente tiene sntomas de 2 aos de evolucin) y
captan contraste intravenoso intensamente.

Pregunta 11.- R: 3
Como siempre, hay que analizar el enunciado cuidadosamente.
Nos piden llegar a un diagnstico, y segn ste, indicar el tratamiento
ms adecuado.
Se trata de un varn de edad media, epilptico, que no responde
bien al tratamiento, y que tiene una imagen expansiva en la TAC
cerebral. Los tumores son una causa relativamente frecuente de epilepsia farmacorresistente, y en las opciones nos dan a elegir entre 5
tumores distintos. Habr que fijarse en otros datos del enunciado para
llegar a la respuesta.
Los otros datos que nos dan son: es un tumor de localizacin
frontal, muestra calcificaciones y no capta contraste (lo que traducido
significa que no parece muy maligno, pues los tumores que captan
contraste intravenoso son habitualmente los malignos, por rotura de
la barrera hematoenceflica, o los muy vascularizados). Otro dato a
favor de que no sea un tumor muy maligno es el largo tiempo de
evolucin (2 aos).
Resumiendo, se trata de un varn de edad media con crisis epilpticas de 2 aos de evolucin y un tumor frontal calcificado, que no
parece muy maligno. Entre las opciones destaca una por encima de
las dems como la ms probable: el oligodendroglioma (opcin 3).
Fjate en que el oligodendroglioma encaja con todos los datos del
enunciado. De hecho deberas recordarlo precisamente por ellos. Es
el tumor ms epileptognico de todos los tumores primarios; con
frecuencia se diagnostica en pacientes con crisis rebeldes al tratamiento (y nuestro paciente tiene crisis de larga evolucin). Su localizacin ms habitual es el lbulo frontal. Caractersticamente tiene tendencia a calcificarse, y con frecuencia muestra reas hemorrgicas.
Habitualmente no capta contraste intravenoso porque es un tumor de
bajo grado de malignidad (aunque hay una variante anaplsica). Recuerda que la caracterstica anatomopatolgica tpica de este tumor
son las clulas en huevo frito (clulas con ncleo hipercromtico
rodeado de un citoplasma de escasa apetencia tintorial). El tratamiento de eleccin del oligodendroglioma es la ciruga, como recoge la
opcin 3.
Aprovechamos para recordar otros tumores primarios del SNC que
se parecen al oligodendroglioma en el hecho de que tambin son
muy epileptognicos, y tambin tienden a presentar calcificaciones,
pero cuya localizacin ms tpica es el lbulo temporal. Se trata del
gangliocitoma y ganglioglioma.
Los otros tumores descritos en las opciones son el glioblastoma, las
metstasis, el meduloblastoma y el ependimoma, todos ellos tumores
muy malignos. Aunque no supieras reconocer el oligodendroglioma
por las caractersticas descritas en el enunciado, hay datos que permiten excluir los otros 4. El meduloblastoma y el ependimoma son tumores ms frecuentes en nios y tpicos de la fosa posterior; no enca-

Pregunta 11. Oligodendroglioma frontal derecho.

Pg. 6 NC

M exico A rgentina
C hile U ruguay

Pregunta 12.- R: 4
Una pregunta directa sobre el ependimoma que requiere conocer
algunas de sus caractersticas.
Los ependimomas son tumores derivados de clulas ependimarias,
y por tanto pueden estar localizados a nivel intracraneal, pero tambin
a nivel espinal. En concreto son ms frecuentes en esta ltima localizacin, y especialmente en el filum terminale, que es el nico sitio en el
que se da una variante histolgica de ependimoma caracterstica, la
variante mixopapilar. Por tanto, la respuesta falsa es la 4, ya que el
ependimoma mixopapilar se localiza caractersticamente en el filum
terminale. Los ependimomas intracraneales no son mixopapilares.
Respecto a los intracraneales, debes recordar que son mucho ms
frecuentes en la fosa posterior (en el suelo del 4 ventrculo opcin
2) que supratentoriales. Los intracraneales son tpicos de nios, mientras que los espinales afectan ms a adultos jvenes.
Son tumores benignos y son caractersticas en el estudio anatomopatolgico las formaciones en roseta (opcin 5).

Pregunta 12. Ependimoma del suelo de IV ventrculo.

Puesto que su localizacin intracraneal ms habitual es el suelo del


4 ventrculo, no es extrao que produzcan hidrocefalia (opcin 3).
Como ya habrs pensado, se trata de una hidrocefalia obstructiva,
porque bloquean la circulacin normal del LCR.

CTO Medicina C/ Nez de Balboa, 115 28006 MADRID (Espaa) Tfno.: (91) 782 43 32 / Fax: (91) 782 43 27
E-mail: secretaria@ctomedicina.com; iberocto@ctomedicina.com WEB: www.ctomedicina.com; www.iberocto.com

Comentarios TEST

Seguimiento a distancia

NEUROCIRUGA

Preparacin Examen de Seleccin 05/06 1 Vuelta


Por su localizacin intraventricular se puede explicar tambin su
tendencia a diseminar por el LCR y por tanto por el espacio subaracnoideo (opcin 1). Esta es una caracterstica que comparten con otros
tumores muy malignos del SNC (los tumores neuroectodrmicos primitivos o PNET, y a la cabeza de ellos el meduloblastoma, que se
localiza ms frecuentemente en el techo del 4 ventrculo, por lo que
exige diagnstico diferencial con el ependimoma). Recuerda que en
el tratamiento del ependimoma (y tambin del meduloblastoma) deben incluirse una RM espinal y un estudio del LCR (citologa) para
descartar diseminaciones de clulas tumorales por el espacio
subaracnoideo, y que cuando se realiza radioterapia, sta debe ser
craneoespinal por el mismo motivo.
Pregunta 13.- R: 2
Esta pregunta hace referencia a los tumores enceflicos en nios.
Como sabes, son tumores muy frecuentes en este grupo de edad. En
concreto son los tumores slidos ms frecuentes, y slo estn superados en frecuencia por las leucemias (opcin 1).
Una caracterstica importante que debe recordarse de los tumores
cerebrales de los nios es que suelen ser primarios; las metstasis
cerebrales en nios son excepcionales (opcin 4).
Otro dato importante es que, a diferencia de los adultos, en los
nios los tumores cerebrales son ms frecuentes en la fosa posterior, es
decir, infratentoriales.
Los tumores cerebrales ms habituales en los nios son, por orden
de frecuencia (ver tabla en la pgina siguiente): los gliomas (en concreto el astrocitoma de cerebelo, que suele ser muy benigno), el
meduloblastoma (un tumor muy maligno) y el craneofaringioma.
Fjate que los dos primeros son de fosa posterior, y el tercero supraselar
(y por tanto supratentorial). Esto se presta a preguntas en las que se
utilicen juegos de palabras, por ejemplo:

Comentarios TEST

1- El tumor cerebral ms frecuente en los nios es?


El astrocitoma de cerebelo.
2- El tumor cerebral maligno ms frecuente en nios es?
El meduloblastoma (porque el astrocitoma de cerebelo en nios es
un tumor muy benigno).
3- El tumor supratentorial ms frecuente en nios es?
El craneofaringioma (porque los otros dos, que son ms frecuentes,
no se localizan a nivel supratentorial).
Fjate en que una sola palabra en el enunciado cambia todo el
sentido de la pregunta. Evidentemente la respuesta errnea en esta
pregunta es la 2, ya que el tumor supratentorial ms frecuente en los
nios no es el astrocitoma cerebral, sino el craneofaringioma.
Como ya has aprendido, los tumores cerebrales en los nios son ms
frecuentes a nivel infratentorial, y por eso suelen debutar con clnica de
HTIC (opcin 3) y no con clnica focal (que es la forma de debut ms
frecuente en los adultos, en los que suelen ser supratentoriales). Los
tumores de fosa posterior (infratentoriales) dan clnica de HTIC rpidamente porque la fosa posterior es muy pequea, y cualquier crecimiento en su interior eleva muy deprisa la presin intracraneal.
Finalmente, la opcin 5 es correcta porque el meduloblastoma,
un tumor tpico de los nios como ya hemos visto, suele ser un tumor
de lnea media que afecta al vermis y al techo del 4 ventrculo (recuerda que hay que hacer diagnstico diferencial con el ependimoma, que se localiza en el suelo del 4 ventrculo).

Pregunta 13. Tumores cerebrales ms frecuentes en nios y adultos.

12345
4 
  
 
9

6789
45

123456789
79 44
892 1234767 6 62
234 9 679 2
2349 679 2
234  9 2
2348 9 8  9 2
4 8 7798 2
!4"8 7798 2

M exico A rgentina
C hile U ruguay

#4 8 7798 2
4"8 7798 2

Seguimiento a distancia

Pregunta 14.- R: 3
Atencin a esta pregunta. Hace referencia a uno de los tumores
cerebrales ms preguntados en el MIR, y por suerte siempre nos lo
presentan igual.
Describen un paciente adolescente con un defecto campimtrico
caracterstico (se trata de una prdida de visin en ambos campos
temporales, es decir, una hemianopsia bitemporal). Desde un punto
de vista semiolgico la hemianopsia bitemporal se debe a una compresin del quiasma ptico (ver figura de pregunta 2), y efectivamente, en el enunciado nos dan un dato que apoya esto; el paciente tiene
una lesin qustica con calcificaciones en localizacin supraselar (y
por encima de la silla turca tenemos el quiasma ptico). No olvides
repasar la semiologa de los defectos campimtricos, pues los preguntan con mucha frecuencia.
No hay ninguna duda posible. El paciente que nos describen tiene
probablemente un craneofaringioma (opcin 3).
Siempre que han preguntado el craneofaringioma en el MIR lo
han hecho de esta forma: paciente joven con lesin supraselar qustica y calcificada que presenta un defecto visual (hemianopsia bitemporal) o una disfuncin endocrina (obesidad habitualmente).
Recuerda que el craneofaringioma es el tumor supratentorial
ms frecuente en los nios y que su localizacin es supraselar. Tiene
gran tendencia a las calcificaciones marginales (se describen las calcificaciones en parntesis), y por su localizacin supraselar comprime
el quiasma ptico y provoca disfuncin endocrina (compresin del
tallo hipofisario e hipotlamo). Es tpico que se trate de una lesin con
quistes en cuyo interior hay una sustancia oleosa que se describe
como aceite de motor.
Pregunta 15.- R: 1
Los linfomas en el sistema nervioso central (SNC) pueden ser primarios o secundarios (metstasis en el sistema nervioso de un linfoma
sistmico). Esta pregunta hace referencia al linfoma cerebral primario, un tumor que caractersticamente afecta a pacientes muy inmunodeprimidos. Suele diagnosticarse en pacientes trasplantados o infectados por VIH y afectados de SIDA (opcin 3). No obstante est
aumentando la frecuencia de casos diagnosticados en pacientes inmunocompetentes, aunque de cara al MIR debes recordarlo como
tpico de inmunodeprimidos y relacionado con infecciones por el
virus de Epstein-Barr (opcin 4).
Se trata de un tumor muy maligno, con un mal pronstico. Desde
un punto de vista histolgico suele ser una neoplasia de alto grado
compuesta en su mayora por clulas B. Por tanto, la opcin 1 es
falsa; el linfoma cerebral primario suele ser un linfoma de clulas B,
pero de alto grado.
El tratamiento con corticoides est indicado en todos los pacientes
con tumores cerebrales porque son muy eficaces para reducir la HTIC
secundaria al edema vasognico que estos producen. Pues bien, en el
caso del linfoma cerebral primario, la respuesta a los corticoides (dexametasona) es espectacular, hasta el punto de llegar a desaparecer la
imagen radiolgica tras su administracin, lo que no quiere decir que se
curen con corticoides. Se han llamado por esto tumores fantasma.
Sin embargo el tratamiento de eleccin del linfoma cerebral primario
es la radioterapia (opcin 5) y actualmente suele combinarse con quimioterapia, siendo el agente de eleccin el metotrexate.
No olvides que el linfoma cerebral es una de las lesiones que capta
contraste en anillo en la TAC, y que debes incluir en el diagnstico
diferencial de las lesiones con este patrn de imagen junto al
glioblastoma multiforme, las metstasis y los abscesos cerebrales (pigenos o bacterianos y por toxoplasma).
Pregunta 16.- R: 2
El meningioma es uno de los tumores intracraneales importantes
de cara al MIR. Debes conocer su frecuencia, localizacin ms habitual y tratamiento. Otros datos como la clnica dependen de la localizacin y los podrs deducir.
Los meningiomas son los terceros tumores intracraneales en frecuencia en el adulto, despus de las metstasis y los gliomas. Sin embargo, como estos dos ltimos son intraparenquimatosos, el meningioma es el tumor intracraneal extraparenquimatoso ms frecuente.

CTO Medicina C/ Nez de Balboa, 115 28006 MADRID (Espaa) Tfno.: (91) 782 43 32 / Fax: (91) 782 43 27
E-mail: secretaria@ctomedicina.com; iberocto@ctomedicina.com WEB: www.ctomedicina.com; www.iberocto.com

NC Pg. 7

Debes recordar que el meningioma es un tumor derivado de clulas aracnoideas (opcin 5) y por tanto de la aracnoides (leptomeninge), no de la duramadre. Por ello podemos encontrarlo en todas
aquellas localizaciones craneales y espinales en las que se encuentra
aracnoides y, aunque tambin puede aparecer en la base craneal, es
mucho ms frecuente a nivel de la convexidad (particularmente la
convexidad parasagital), por lo que la opcin falsa es la 2.
Aunque pueden verse a distintas edades y en ambos sexos, son ms
frecuentes en mujeres que en hombres, y sobre todo en las dcadas
5-6 de la vida (opcin 1).
La respuesta 3 es cierta, y hace referencia a los raros casos en que
los meningiomas se asocian con sndromes neurocutneos, tambin
conocidos como facomatosis. En concreto, el meningioma puede
asociarse con la neurofibromatosis tipo II, trastorno caracterizado
genticamente por una alteracin en el cromosoma 22. En estos casos
los meningiomas son con frecuencia mltiples y aparecen a edades
ms tempranas. Recuerda sin embargo que el tumor intracraneal tpicamente asociado con la neurofibromatosis tipo II es el neurinoma
del VIII par (habitualmente bilateral en estos pacientes, siendo criterio
diagnstico de esta enfermedad).
Algunas preguntas recientes del examen MIR han hecho referencia
al tratamiento del meningioma. Partimos de la base de que la mayor
parte de los meningiomas son tumores benignos, y que por el hecho
de estar localizados extraparenquimatosos, el cerebro no est infiltrado habitualmente, sino desplazado. Por tanto, si se extirpa el tumor
completamente el paciente est curado. As, el tratamiento de eleccin del meningioma es la extirpacin quirrgica, y generalmente
no requiere tratamientos complementarios. El factor ms importante
para evitar recidivas es la extensin de la ciruga. Sin embargo, debemos tener en cuenta que, por su localizacin, a veces de difcil abordaje quirrgico, algunos meningiomas no son extirpables por completo.
Hay que tener claro que el meningioma se trata con ciruga. Sin
embargo, en algunos casos puede estar indicada la radioterapia, en
concreto en los casos de restos o recidivas que continan creciendo
(opcin 4) o cuando la histologa es muy agresiva (meningiomas malignos).

Pregunta 16. Meningioma de la hoz cerebral.

Pregunta 17.- R: 3
El hemangioblastoma es otro de los tumores intracraneales que
son preguntados con cierta frecuencia en el examen MIR, y te adelantamos que habitualmente lo preguntan como caso clnico y el dato
clave suele ser que es un tumor de fosa posterior y se asocia con
elevacin del hematocrito.
Vamos a aprovechar la opcin 1 para hacer juegos de palabras
con los tumores de la fosa posterior. Fjate bien, porque cambiando
una sola palabra del enunciado, te estaran preguntando por distintos
tumores. Por ejemplo:
Pg. 8 NC

M exico A rgentina
C hile U ruguay

NEUROCIRUGA

Preparacin Examen de Seleccin 05/06 1 Vuelta

1- Cul es el tumor ms frecuente en la fosa posterior del adulto?


Las metstasis.
2- Cul es el tumor primario ms frecuente en la fosa posterior del
adulto?
El hemangioblastoma.
3- Cul es el tumor primario intraaxial ms frecuente en la fosa
posterior del adulto?
El hemangioblastoma.
4- Cul es el tumor primario extraaxial ms frecuente en la fosa
posterior del adulto?
El neurinoma del VIII par (despus iran, por orden de frecuencia,
el meningioma y el epidermoide).
5- Cul es el tumor ms frecuente en la fosa posterior del nio? El
astrocitoma de cerebelo (recuerda que las metstasis son excepcionales en nios).
6- Cul es el tumor primario ms frecuente en la fosa posterior del
nio?
El astrocitoma de cerebelo.
7- Cul es el tumor primario maligno ms frecuente en la fosa
posterior del nio?
El meduloblastoma (recuerda que el astrocitoma de cerebelo suele
ser muy benigno).
Volvemos ahora al hemangioblastoma. Efectivamente, como ya
has visto, la opcin 1 es verdadera; es el tumor primario intraparenquimatoso de fosa posterior ms frecuente en el adulto. Concretamente se localiza a nivel de hemisferios cerebelosos. Recuerda esta
caracterstica; el hemangioblastoma (en el adulto) y el astrocitoma (en
los nios) son tumores cerebelosos pero hemisfricos, mientras que el
meduloblastoma (tpico de nios) es vermiano o de lnea media.
Debes recordar la asociacin entre hemangioblastoma de cerebelo y enfermedad de von Hippel-Lindau, una facomatosis que caractersticamente se asocia con este tumor y con la presencia de angiomas
retinianos. Sin embargo, la respuesta 3 es falsa porque la gran mayora de los hemangioblastomas son espordicos (slo un 20% se asocian con von Hippel-Lindau).
La otra caracterstica a recordar de este tumor es su capacidad
para producir eritropoyetina (EPO), por lo que es frecuente que
los pacientes que lo padecen tengan una elevacin del hematocrito,
que es la alteracin hematolgica tpica a la que se refiere la respuesta 5, y a la que han hecho referencia todas las preguntas MIR
sobre este tumor.
Ya has visto que el hemangioblastoma se localiza en los hemisferios
cerebelosos. Su imagen tpica en la TAC es una lesin qustica, con un
ndulo mural (respuesta 4). Esto significa que en alguna parte de la
pared del quiste hay un engrosamiento, que es la parte slida del
tumor, que habr que resecar en la ciruga, que es el tratamiento de
eleccin (respuesta 2).
En resumen, debes quedarte con la idea de que el hemangioblastoma te lo preguntarn en casos clnicos como un paciente con clnica cerebelosa, que tiene el hematocrito alto y un tumor qustico en un
hemisferio cerebeloso.
Pregunta 18.- R: 2
Los neurinomas son tumores derivados de las clulas de Schwann,
y por eso se llaman tambin schwannomas. A nivel intracraneal los
podemos encontrar como tumores de los nervios craneales, pero son
ms frecuentes a nivel espinal como tumores de las races nerviosas
raqudeas.
La respuesta correcta es la 2 porque los neurinomas intracraneales ms frecuentes son los neurinomas del VIII par, seguidos de los del
V par; el resto son muy raros.
El neurinoma del VIII par es un tumor importante en el MIR. Habitualmente se conoce como neurinoma del acstico, aunque esto es
incorrecto, porque estos tumores crecen a partir del componente
vestibular del VIII par, no del acstico. Debes recordar este dato
porque te puede inducir a error.
Del neurinoma del VIII par es importante conocer que se trata del
tumor ms frecuente del ngulo pontocerebeloso (el segundo es el

CTO Medicina C/ Nez de Balboa, 115 28006 MADRID (Espaa) Tfno.: (91) 782 43 32 / Fax: (91) 782 43 27
E-mail: secretaria@ctomedicina.com; iberocto@ctomedicina.com WEB: www.ctomedicina.com; www.iberocto.com

Comentarios TEST

Seguimiento a distancia

NEUROCIRUGA

Preparacin Examen de Seleccin 05/06 1 Vuelta


meningioma y el tercero el epidermoide). Tambin te lo pueden preguntar como el tumor extraaxial de la fosa posterior ms frecuente.
Clnicamente produce sintomatologa ORL: hipoacusia neurosensorial, acfeno y vrtigo, por lo que suele ser diagnosticado por esos
especialistas, pero en su crecimiento puede afectar a otros pares
craneales (el V y el VII fundamentalmente), e incluso al cerebelo, al
tronco del encfalo y pares bajos.
Debes conocer la asociacin entre neurinoma del VIII par y neurofibromatosis tipo II (en estos casos suelen ser neurinomas bilaterales).
El tratamiento de eleccin es la ciruga o la radiociruga estereotctica.

Comentarios TEST

Pregunta 18. Neurinoma del VIII par.

Pregunta 19.- R: 4
De nuevo nos enfrentamos a un caso clnico que debemos interpretar. Podramos resumirlo como un nio con alteraciones endocrinolgicas (diabetes inspida), alteraciones focales (dificultad para la
elevacin de la mirada) y clnica de HTIC (cefalea).
De todos estos datos, las alteraciones endocrinolgicas nos llevaran a pensar en un tumor que est afectando al hipotlamo o alrededores (otras estructuras muy mediales y basales). Todos los tumores
descritos, especialmente 1, 2, 4 y 5 cursan con alteraciones endocrinolgicas. Por tanto, este dato no es muy til para hacer el diagnstico
diferencial.
Tampoco lo es la cefalea, sntoma comn a la mayor parte de los
tumores cerebrales.
Sin embargo hay un dato definitivo, en este caso el sntoma focal,
que como siempre tiene un valor localizador. El nio tiene una dificultad para la elevacin de la mirada, lo que nos obliga a pensar en
un sndrome de Parinaud, y dirige las sospechas hacia la regin pineal.
Recuerda que en las lesiones de la regin pineal, y con frecuencia tambin en la hidrocefalia, se puede producir este sndrome,
tambin conocido como sndrome mesenceflico dorsal, que cursa
con parlisis de la elevacin de la mirada con ausencia de reflejo
fotomotor, parlisis de la convergencia y reflejo de acomodacin
conservado.
Por tanto, la respuesta ms probable es la opcin 4: germinoma
de la regin pineal.
De los tumores de regin pineal debes conocer que son ms frecuentes en nios y pacientes jvenes que en adultos. Hay que diferenciar dos tipos de tumores en esta regin: los tumores derivados
del parnquima pineal (pineocitoma y pineoblastoma), que son ms
raros, y los tumores derivados de clulas germinales (mucho ms
frecuentes).
Los tumores de clulas germinales son los ms frecuentes en esta
regin, especialmente el germinoma. Hay otros ms raros que reciben tambin el nombre de tumores germinales no germinomatosos
(es decir, los que no son germinomas). Es importante conocer que en
estos tumores pueden encontrarse elevados en sangre y LCR ciertos
M exico A rgentina
C hile U ruguay

Seguimiento a distancia

marcadores tumorales. Los ms tpicos son la gonadotropina corinica en el coriocarcinoma y la alfafetoprotena en el tumor del seno
endodrmico. Recuerda que, en el caso del germinoma, los marcadores tumorales son habitualmente negativos.
Te preguntars el motivo por el que se separa el germinoma de los
llamados tumores germinales no germinomatosos. La razn es que el
germinoma tiene una extraordinaria radiosensibilidad, por lo que su
tratamiento de eleccin es la radioterapia. Para el resto, que no responden bien a RT, la alternativa es la ciruga. Una vez ms llamamos tu
atencin sobre los marcadores tumorales (si son negativos, hay que
sospechar germinoma y puede curarse con radioterapia sin necesidad de ciruga).
Pregunta 20.- R: 3
Esta es una pregunta directa acerca del quiste coloide cerebral.
El quiste coloide es un tumor disembrioplsico, como lo es el
craneofaringioma. El primero se supone derivado de la parfisis, y se
localiza en la porcin anterior del tercer ventrculo (opciones 1 y 2).
El craneofaringioma, mucho ms preguntado en el MIR, se origina a
partir de restos embrionarios de la bolsa de Rathke, y se localiza en la
regin supraselar.
Respecto al quiste coloide, debes recordar que es un tumor tpico
del tercer ventrculo, como ya hemos visto, y por tanto puede producir obstruccin para la circulacin del LCR, originando una hidrocefalia obstructiva. Lo que es tpico de este tumor es que, al comportarse
como una lesin qustica dentro del tercer ventrculo, puede desplazarse con los movimientos de la cabeza, de modo que la obstruccin
puede variar segn la posicin, y puede ser causa de hidrocefalias
agudas intermitentes por bloqueo de los agujeros de Monro (opcin
5). Ahora es infrecuente porque con la TAC se diagnostica ms
precozmente, pero antes se describa como una causa de muerte
sbita por este motivo.
Por tanto, debes recordar que es un tumor qustico del tercer
ventrculo que puede originar hidrocefalia intermitente. Muy caracterstico de este tumor es que en las tinciones histolgicas, como dice la
opcin 4, se aprecia un contenido glucoproteico que se tie con
cido peridico de Schiff (PAS +).
La respuesta falsa es la 3, porque no se trata de un tumor glial, sino
que tiene una cpsula verdadera de origen epitelial.

Pregunta 20. Quiste coloide del III ventrculo.

Pregunta 21.- R: 3
Esta es una pregunta importante, porque en ocasiones han preguntado en el examen por la asociacin entre determinadas enfermedades neurocutneas (tambin conocidas como facomatosis) y tumores
del sistema nervioso central.
Las ms importantes se recogen en la tabla siguiente, y debes recordarlas:

CTO Medicina C/ Nez de Balboa, 115 28006 MADRID (Espaa) Tfno.: (91) 782 43 32 / Fax: (91) 782 43 27
E-mail: secretaria@ctomedicina.com; iberocto@ctomedicina.com WEB: www.ctomedicina.com; www.iberocto.com

NC Pg. 9

Pregunta 21. Facomatosis y tumores del Sistema Nervioso Central.


Esclerosis tuberosa.
Neurofibromatosis tipo I.
Neurofibromatosis tipo II.
Sturge-Weber.
Von Hippel-Lindau.
Klippel-Trenaunay.

Astrocitoma gigantocelular
subependimario.
Glioma de vas pticas.
Neurinoma bilateral del VIII par.
Meningiomas.
Angiomas leptomenngeos.
Hemangioblastoma cerebeloso.
Angioma cavernoso de la
mdula espinal.

Como ya habrs adivinado, la respuesta correcta es la opcin 3:


neurofibromatosis tipo I, que caractersticamente asocia tumores de las
vas visuales, y en concreto el glioma del nervio ptico. Las malformaciones de los huesos largos (pseudoartrosis de la tibia) y los hamartomas de
iris (que originan una heterocroma, o diferencia de color entre los dos
ojos) son otros de los criterios diagnsticos, junto con las manchas color
caf con leche, las eflides (pecas) axilares, los neurofibromas cutneos,
etc. Los criterios diagnsticos de neurofibromatosis tipo I se recogen en la
tabla a continuacin (debe reunir 2 o ms de los siguientes):

Pregunta 21. Criterios diagnsticos de neurofibromatosis tipo I.


1- 6 o ms manchas caf con leche de ms de 5 mm de dimetro
(prepuberal) o ms de 15 mm (postpuberal).
2- 2 o ms neurofibromas de cualquier tipo, o un neurofibroma
plexiforme.
3- Eflides axilares o inguinales.
4- Glioma del nervio ptico.
5- Ndulos de Lisch (hamartomas de iris).
6- Anomalas seas como displasia del ala del esfenoides o adelgazamiento de la cortical de los huesos largos con o sin pseudoartrosis
(tibia y peron).
7- Un familiar de primer grado con diagnstico de neurofibromatosis
tipo I segn los criterios anteriores.
No es imprescindible que memorices todos los criterios (no los han
preguntado nunca), pero conviene familiarizarse con ellos, sobre todo
con la relacin con el glioma del nervio ptico.
Por su parte, la neurofibromatosis tipo II tambin se relaciona
con tumores del SNC, en concreto con el neurinoma bilateral del
VIII par (tambin llamado neurinoma del acstico, aunque es un
trmino incorrecto porque como sabes se origina en la porcin vestibular del VIII par, no en la acstica). Los criterios diagnsticos de
neurofibromatosis tipo II estn recogidos en la tabla:

Pregunta 21. Criterios diagnsticos de neurofibromatosis tipo II.


1- Neurinoma bilateral del VIII par o
2- Un familiar de primer grado con diagnstico de neurofibromatosis
tipo II y uno de los siguientes:
a. Neurinoma unilateral del VIII par o
b. Dos de los siguientes:
Neurofibroma.
Meningioma.
Glioma.
Schwannoma
Catarata subcapsular juvenil
Tampoco han preguntado nunca estos criterios, y lo importante es
que recuerdes la asociacin con el neurinoma bilateral del VIII par.
Otro glioma que se asocia a las facomatosis es el astrocitoma
gigantocelular subependimario, un tumor de bajo grado de malignidad que se ve asociado a la esclerosis tuberosa.
Finalmente, recuerda la asociacin entre enfermedad de von
Hippel Lindau y hemangioblastoma cerebeloso (tumor de hemisfe-

Pg. 10 NC

M exico A rgentina
C hile U ruguay

NEUROCIRUGA

Preparacin Examen de Seleccin 05/06 1 Vuelta

rio cerebeloso que caractersticamente secreta eritropoyetina, por lo


que suele elevar el hematocrito) y angiomas retinianos.
Recuerda que la enfermedad de Sturge-Weber se conoce tambin como angiomatosis encfalo-trigeminal porque presenta angiomas (nevus flameus) en el territorio sensitivo del trigmino (primera
rama generalmente) y con frecuencia retinianos, con calcificaciones
cerebrales y crisis epilpticas.
Pregunta 22.- R: 4
Como ya hemos visto, son frecuentes y caractersticas las asociaciones entre sndromes neurocutneos (facomatosis) y tumores cerebrales.
La neurofibromatosis tipo I (opcin 2) se asocia frecuentemente
con gliomas de las vas visuales. La enfermedad de Bourneville, tambin conocida como esclerosis tuberosa, se asocia con el astrocitoma
gigantocelular subependimario, un glioma de bajo grado de malignidad.
Pero no slo hay asociaciones con las facomatosis, tambin se
pueden ver con otras enfermedades sistmicas.
La opcin 1 hace referencia a la neoplasia endocrina mltiple
tipo I (MEN I), que es una enfermedad sistmica que asocia hiperparatiroidismo primario familiar, tumores hipofisarios (generalmente
adenomas), tumores pancreticos e hipergastrinemia con enfermedad ulcerosa pptica (sndrome de Zollinger-Ellison). No la confundas con la neoplasia endocrina mltiple tipo II (MEN II), en la que el
hiperparatiroidismo primario se asocia con feocromocitoma y carcinoma medular de tiroides, pero no con tumores hipofisarios.
El sndrome de Turcot (opcin 5) es una rara variante de poliposis
colnica familiar que se asocia con tumores cerebrales malignos.
La respuesta correcta a esta pregunta es la opcin 4, porque la
enfermedad de Lafora no incluye tumores cerebrales entre sus hallazgos. Se trata de una forma hereditaria de epilepsia mioclnica progresiva, con herencia autosmica recesiva.
TRAUMATISMOS CRANEOENCEFLICOS (TCE).
Pregunta 23.- R: 3
Nos presentan un caso clnico de un paciente que ha sufrido un
traumatismo craneoenceflico, y nos piden que decidamos sobre la
actitud ms correcta con este paciente. Ojo! con estas preguntas,
porque se han puesto de moda en los ltimos exmenes MIR y,
aunque no suelen ser difciles, requieren analizar con cuidado la
situacin.
Antes de contestar la pregunta debes recordar que, a la hora de
valorar a un paciente con un traumatismo craneoenceflico, el dato
ms importante (el de mayor valor pronstico) es el nivel de conciencia, y para valorarlo de manera homognea en todos los sitios se
describi una escala aceptada universalmente, que es la Escala de
Coma de Glasgow (GCS) recogida en la tabla a continuacin. Aunque no hay ninguna pregunta en este test sobre ella, debes repasarla
porque la han preguntado en alguna ocasin. Lo ms importante es
conocer que valora tres parmetros: la apertura ocular, la respuesta
motora y la respuesta verbal, y que punta el nivel de conciencia de
3 a 15 (siendo 15 la puntuacin mxima).

Pregunta 23. Escala de Coma de Glasgow.


APERTURA
DE OJOS

RESPUESTA
MOTORA

Obedece rdenes

Localiza el dolor

Orientado

Espontnea

Retira al dolor

Confuso

A la voz

Flexora
(decorticacin)

Inapropiado

Al dolor

Extensin
(descerebracin)

Incomprensible

No

No

No

PUNTUACIN

RESPUESTA
VERBAL

CTO Medicina C/ Nez de Balboa, 115 28006 MADRID (Espaa) Tfno.: (91) 782 43 32 / Fax: (91) 782 43 27
E-mail: secretaria@ctomedicina.com; iberocto@ctomedicina.com WEB: www.ctomedicina.com; www.iberocto.com

Comentarios TEST

Seguimiento a distancia

NEUROCIRUGA

Preparacin Examen de Seleccin 05/06 1 Vuelta

Comentarios TEST

Volviendo a la pregunta que nos ocupa, describen a un paciente


que ha tenido un golpe en la cabeza, con prdida de conciencia
inicial aunque ahora est consciente, que tiene una exploracin
neurolgica normal, y al que se ha realizado una Rx de crneo que
muestra una fractura lineal.
Lo primero que hay que pensar es que si ha perdido la conciencia
inicialmente, y tiene una fractura en el crneo, el golpe ha sido de
suficiente intensidad como para que el paciente permanezca en observacin. No sera una buena medida darle de alta a su domicilio,
como dice la opcin 1, sin saber si adems de la fractura tiene alguna
otra lesin intracraneal. Por tanto, descartamos esta opcin.
Las otras 4 opciones insisten en que debe realizarse una TAC craneal, medida que est justificada para saber si por debajo de la fractura hay alguna lesin intracraneal asociada.
Descartamos la opcin 2 porque el hecho de que la TAC craneal sea
normal inicialmente no asegura que pueda aparecer alguna hemorragia intracraneal de forma diferida. Con los hallazgos descritos, y aunque
la exploracin neurolgica sea normal y la TAC craneal no muestre
alteraciones, el paciente debe quedar en observacin unas horas.
La opcin 4 es tambin incorrecta, porque las fracturas lineales del
crneo no precisan tratamiento quirrgico. Aunque suene informal,
el crneo no se puede escayolar, y por tanto una fractura lineal no
desplazada se trata exclusivamente con analgsicos, dejando que consolide espontneamente. Slo se operan las fracturas con hundimiento de la bveda craneal, pero no las lineales.
La opcin 5 tambin es incorrecta. Una fractura lineal no implica
mayor riesgo de infeccin intracraneal, aunque exista una herida en
el cuero cabelludo. Slo est aumentado el riesgo de infeccin si hay
fuga de lquido cefalorraqudeo (lo que significa que la duramadre
est abierta), y en esos casos s que se asocian antibiticos, pero no son
necesarios en el caso descrito.
Por tanto, la respuesta correcta es la 3. Realizaremos una TAC craneal para descartar lesiones intracraneales asociadas y dejaremos al
paciente en observacin durante 24 horas para estar seguros de que no
aparece ninguna complicacin diferida no visible en la primera TAC.
Por cierto, recuerda que la complicacin que deberas esperar en
este paciente, aunque no necesariamente tiene que ocurrir, es la aparicin de un hematoma epidural por sangrado de la fractura lineal
que presenta. Si adems te dijeran que la fractura es de la escama del
hueso temporal, la sospecha sera mayor, porque debajo de dicha
escama se encuentra la arteria menngea media con sus ramas, cuya
rotura es la principal causa del hematoma epidural.
Pregunta 24.- R: 2
Por su localizacin, podemos distinguir distintos tipos de fracturas
craneales, esencialmente las de bveda craneal (lineales o fracturahundimiento) y las de base del crneo.
Recientemente han hecho algunas preguntas en el examen MIR
sobre fracturas de crneo. Lo importante es que conozcas las implicaciones que tienen, si requieren tratamiento quirrgico o no, y si necesitan o no profilaxis antibitica por riesgo de infeccin intracraneal.
Intentaremos, antes de contestar la pregunta, dar unas pinceladas
sobre los puntos que hemos mencionado.
Una cosa general a todas las fracturas es que el pronstico no
depende de la lesin sea (si se rompe el hueso, no pasa nada), depende de las lesiones intracraneales asociadas. Por tanto, en todo
paciente con lesiones seas hay que indagar sobre si existen otras
lesiones, explorando al paciente y haciendo pruebas de imagen (la
TAC es la de eleccin en los traumatismos).
Las fracturas lineales no precisan tratamiento quirrgico. El pronstico, como en todas las fracturas, depende de las lesiones asociadas. No precisan tratamiento antibitico profilctico porque no
hay mayor riesgo de infeccin intracraneal (este riesgo slo est
aumentado si hay salida de LCR, es decir, si la dura est abierta).
Otra cosa es que presenten heridas anfractuosas en cuero cabelludo, con prdida de sustancia, en cuyo caso se asocian antibiticos,
pero no por la fractura sino por la herida.
Las fracturas hundimiento de la bveda craneal son ms graves
que las lineales, porque el fragmento de hueso hundido puede
afectar a estructuras intracraneales. Por ello estas fracturas suelen
requerir tratamiento quirrgico para retirar el fragmento deprimiM exico A rgentina
C hile U ruguay

Seguimiento a distancia

do y para reconstruir la duramadre y reparar las lesiones cerebrales


subyacentes. En este caso s se suelen aadir antibiticos para prevenir infecciones, porque la duramadre suele estar rota.
El tercer tipo son las fracturas de base de crneo, a las que se
refiere esta pregunta. Las fracturas de la base del crneo se ven con
mucha dificultad en las Rx simples de crneo, por lo que la prueba
de eleccin para diagnosticarlas suele ser la TAC craneal, que adems permite ver las lesiones intracraneales asociadas, de las que
depender el pronstico. Recuerda que hay ciertos signos en la
exploracin fsica del paciente que nos llevan a sospechar que
puede tener una fractura en la base del crneo: hematoma
periorbitario en ojos de mapache, hematoma retroauricular (signo de Battle), o salida de LCR o sangre por el odo o la nariz.
De las fracturas de base de crneo, debes recordar las
frontoetmoidales (que suelen asociar rinolicuorrea, es decir, salida
de LCR por las fosas nasales) y las de peasco, a las que se refiere esta
pregunta (que pueden asociar otolicuorrea y otorragia, es decir, salida
de LCR o sangre por el CAE).
Las fracturas de peasco se dividen segn la direccin del trazo de
fractura, visible en la TAC craneal, en: longitudinales, transversales y
oblicuas. Si te preguntan algo acerca de ellas, ser en relacin con el
riesgo de otorragia (es decir, de lesin del tmpano), hipoacusia (que
puede ser neurosensorial, por lesin del odo interno, o de conduccin, por lesin del odo medio o del tmpano) y parlisis facial.
En la tabla de la pgina siguiente tienes recogidas las manifestaciones ms habituales de cada una de ellas:
La manera ms sencilla para recordarlas es imaginarse el trazo de
fractura:
Las fracturas longitudinales se llaman tambin timpnicas, porque afectan al odo medio tmpano CAE. Por tanto, habitualmente muestran desgarro timpnico y otorragia, hipoacusia de
conduccin, y es raro que afecten al nervio facial (slo el 20% de
ellas asocia parlisis facial).
Las fracturas transversales se llaman tambin neurosensoriales,
porque afectan al odo interno CAI agujero rasgado posterior.
Por tanto habitualmente el tmpano est intacto, aunque pueden
mostrar hemotmpano, tienen hipoacusia neurosensorial, y es frecuente la lesin del nervio facial (hasta en un 50%).
Las fracturas oblicuas se llaman tambin timpanolaberntica, y
muestran caractersticas mixtas entre las dos anteriores. En ellas es
casi constante la lesin del nervio facial.
Por tanto, la respuesta falsa es la 2, porque en las fracturas transversales del peasco se afecta el odo interno, y en consecuencia la
hipoacusia es neurosensorial, no de conduccin.
El resto de opciones, como ya hemos explicado, son ciertas.

Pregunta 24. Fracturas del peasco.


12345623789
56
387 87349
7325 4 34 
722
57 879 97 9

1234567859
6
12 4
62759 6
123456

242
12
 2
2 4242
2759

12
 24 2422 26 2
7
2962594 242 6
2225


12396

2426 
12!
2

2"#$ %2 46
9
&
262
&6 675429


12'7664
2 44 74
123456237523
2387 87349
325 4 34 
722
57 879 97
52
9

12(859
62 7
7626 2)4567859
6
122 7
76
12396

2 464 6

12*+76
12,7
562496 7 462)

2426862
6
12!
2

2-#$ %2
&6 6754262 46754
12.64
2674
2/424
2962
27659
24 249
 4

CTO Medicina C/ Nez de Balboa, 115 28006 MADRID (Espaa) Tfno.: (91) 782 43 32 / Fax: (91) 782 43 27
E-mail: secretaria@ctomedicina.com; iberocto@ctomedicina.com WEB: www.ctomedicina.com; www.iberocto.com

NC Pg. 11

Pregunta 25.- R: 4
De nuevo se trata de un caso clnico, y debes fijarte especialmente
en l porque es una de las preguntas ms habituales de la asignatura
en el examen MIR.
Se trata de un paciente joven que, tras un accidente de trfico, y
con un intervalo asintomtico previo, presenta deterioro del nivel de
conciencia y unas anomalas en la exploracin neurolgica compatibles con herniacin uncal del lado izquierdo (tiene anisocoria, probablemente por midriasis izquierda secundaria a compresin del III
par craneal, y prdida de fuerza en hemicuerpo derecho por compresin del pednculo cerebral izquierdo, que es el lugar por el que
discurren las fibras motoras en el mesencfalo). En la placa de crneo
tiene una fractura de la escama del hueso temporal izquierda.
En resumen: paciente joven con traumatismo craneoenceflico,
inicialmente asintomtico, que ahora presenta deterioro de conciencia secundario a herniacin uncal. Hay que sospechar que el paciente tiene un hematoma epidural temporal izquierdo.
Hay varios datos en la historia que orientan a este diagnstico. La
localizacin de la fractura es de alto riesgo. Como sabes, las fracturas
lineales no tienen ninguna gravedad por s solas, sino que el pronstico depende de las lesiones intracraneales asociadas. Siempre que un
paciente tiene una fractura debe realizarse una TAC craneal para
descartar estas lesiones. La complicacin intracraneal ms frecuentemente asociada con las fracturas lineales es el hematoma epidural,
sobre todo si estas afectan a la escama del hueso temporal, porque
justo debajo de ella se encuentran la arteria menngea media y sus
ramas. Recuerda que la rotura de la arteria menngea media es la
causa ms frecuente del hematoma epidural, y esto ha sido frecuentemente motivo de pregunta MIR.
Otro dato a favor de que se trate de un hematoma epidural es la
evolucin clnica. No olvides que la secuencia clsica es: prdida de
conciencia inicial con recuperacin posterior, seguida de un intervalo
lcido, con deterioro posterior secundario generalmente a herniacin
del uncus del lbulo temporal (lo que provoca anisocoria por compresin del III par ipsilateral con hemiparesia contralateral). Es la misma
secuencia que describen en el enunciado. Sin embargo, debes saber
que no siempre se ve esta secuencia (a veces no hay intervalo lcido),
aunque en las preguntas MIR suelen ponerla siempre as.
Por tanto, se trata de un hematoma epidural. Si hiciramos una
TAC craneal, veramos una lesin hiperdensa (blanca) con forma de
lente biconvexa (a diferencia del hematoma subdural, que tiene forma de semiluna).
Recuerda que denominamos hematoma epidural al que se encuentra entre el hueso y la duramadre, y que por tanto no contacta
con el cerebro, frente al hematoma subdural, que se localiza por
debajo de la duramadre, en contacto con el cerebro, y suele acompaarse de lesiones del parnquima cerebral subyacente.
La respuesta cierta es evidentemente la 4. Hay signos de herniacin transtentorial, que es otra forma de llamar a la herniacin uncal.
La respuesta 1 es errnea porque el hematoma epidural, si se
opera a tiempo (se trata de una emergencia neuroquirrgica), tiene un
extraordinario pronstico de recuperacin funcional, con una mortalidad muy baja (inferior al 10%). Todo lo contrario del hematoma
subdural agudo, que tiene una mortalidad y morbilidad muy altas
(porque suele acompaarse de lesiones del parnquima cerebral subyacente).
La respuesta 2 tambin es incorrecta. Tanto el hematoma epidural
como el subdural son eso.... hematomas. Por tanto son slidos; es
sangre coagulada. El tratamiento de eleccin de ambos es quirrgico,
pero para quitarlos es necesario hacer una craneotoma (es decir,
levantar una parte del hueso para poder extirpar el hematoma). No es
suficiente con la realizacin de un trpano porque esto es un pequeo agujero de menos de 1 cm de dimetro en el hueso, y por ese
agujero no sale un cogulo slido. Sin embargo, el trpano es la tcnica de eleccin para evacuar un hematoma subdural crnico, porque
estos no son slidos, sino lquidos (se trata de sangre evolucionada), y
por tanto se pueden aspirar a travs de un pequeo agujero. Para que
lo recuerdes nos permitimos un pequeo smil: para comerse el flan
(slido) hay que quitar la tapa (craneotoma), pero para tomarse un
zumo (lquido) es suficiente con hacer un pequeo agujero (trpano)
y meter una pajita para aspirar (drenaje).
Pg. 12 NC

M exico A rgentina
C hile U ruguay

NEUROCIRUGA

Preparacin Examen de Seleccin 05/06 1 Vuelta

La opcin 3 no es cierta porque el hematoma epidural, como ya


sabes, se debe a rotura de arterias de la duramadre o arterias menngeas (generalmente la arteria menngea media). Sin embargo, el hematoma subdural s que se suele producir por rotura de las pequeas
venas puente corticales (son venas entre el cerebro y la duramadre, y
por tanto su rotura produce sangrado subdural).
Finalmente la opcin 5 es falsa porque, una vez ms, no existe
aumento del riesgo de infeccin intracraneal en las fracturas lineales
(solamente en los casos en que est rota la duramadre y sale LCR al
exterior).
En la tabla siguiente se recogen las principales diferencias entre
hematoma epidural y subdural agudo.

Pregunta 25. Diagnstico diferencial entre hematoma epidural


y subdural agudo.

Origen

C lnica

HEMATOMA EPIDURAL

HEMATOMA SUBDURAL

Lesin arterial (la ms


frecuente es la arteria
menngea media).

R otura de venas corticales, y


puede ser:
- Agudo (en la primera semana,
que es el que nos ocupa).
- Subagudo (7-10 das despus).
- Crnico (tpico de alcohlicos y
ancianos).

Tpicamente, perodo breve de


inconsciencia seguido de
No presenta intervalo lcido.
intervalo lcido y,
Disminucin de conciencia desde
posteriormente, coma de
el inicio.
rpida evolucin (herniacin
uncal).

TAC

Imagen hiperdensa en forma


de lente B ICONVEXA.

Imagen hiperdensa
en forma de SEMILUNA.

Lesin
directa del
parnquima

En general menor y ms tarda


por compresin.

Mayor y desde el inicio,


por estar la sangre en contacto
con el parnquima cerebral.

Mortalidad

Menor,con diagnstico y
tratamiento precoz casi del
0%.

25-90% segn el grado de


afectacin parenquimatosa
directa.

Tratamiento

Craneotoma y evacuacin.

Craneotoma y evacuacin.

Pregunta 26.- R: 4
El dao o lesin axonal difusa es una lesin postraumtica que, a
diferencia del hematoma subdural, el hematoma epidural o las contusiones hemorrgicas intraparenquimatosas, no tiene tratamiento
quirrgico. Como su propio nombre indica, se trata de una lesin de
los axones de las neuronas, y por tanto no es reparable con ciruga.
Del nombre puedes deducir tambin que las lesiones se encontrarn
principalmente en los lugares por los que discurren los axones, y en
concreto en la sustancia blanca. Adems, tambin puedes deducir
que no es una lesin focal, sino difusa por todo el cerebro.
La primera opcin que podemos descartar por tanto es la opcin
5. La lesin axonal difusa, que es un dao irreversible de los axones
de mltiples neuronas y de forma difusa en el cerebro, tiene muy mal
pronstico funcional.
No es cierta tampoco la opcin 2. El mecanismo principal de lesin
en estos pacientes son los movimientos de aceleracin-desaceleracin,
que cizallan los axones de las neuronas en la sustancia blanca.
Podemos descartar tambin la opcin 1 si pensamos en lo que
significa lesin axonal difusa. Se trata de una lesin difusa de los axones,
que son de pequeo tamao, no de una lesin focal, por eso en
muchos pacientes no se ven alteraciones en la TAC cerebral. No obstante, cuando se ven alteraciones en la TAC, lo que vemos es un
pequeo punteado hemorrgico en la sustancia blanca de los hemisferios cerebrales y en el cuerpo calloso (precisamente los sitios por
donde discurren los axones). Por tanto, la respuesta correcta en esta
pregunta es la opcin 4.
Finalmente descartamos tambin la opcin 3. Los pacientes con
lesin axonal difusa presentan un deterioro neurolgico importante

CTO Medicina C/ Nez de Balboa, 115 28006 MADRID (Espaa) Tfno.: (91) 782 43 32 / Fax: (91) 782 43 27
E-mail: secretaria@ctomedicina.com; iberocto@ctomedicina.com WEB: www.ctomedicina.com; www.iberocto.com

Comentarios TEST

Seguimiento a distancia

NEUROCIRUGA

Preparacin Examen de Seleccin 05/06 1 Vuelta

Comentarios TEST

desde el mismo momento del traumatismo, porque la rotura de los


axones se produce justamente en ese momento. De hecho, los
neurocirujanos sospechamos lesin axonal difusa en un paciente con
mala situacin neurolgica inmediatamente tras un traumatismo en el
que la TAC no muestra alteraciones o muestra slo las pequeas anomalas que hemos descrito. En otras palabras, hay que sospechar lesin axonal difusa en un paciente con un traumatismo que est muy
malito desde el principio, con una TAC craneal no tan mala.
Pregunta 27.- R: 5
Nos preguntan sobre un cuadro clnico muy caracterstico, que si
cae en el MIR lo har precisamente contando los mismos datos que
en este enunciado.
Se trata de una mujer que ha tenido un golpe en la cabeza hace
unas 24 horas y que ahora consulta porque, de forma brusca, ha
aparecido un exoftalmos pulstil unilateral. Adems resulta que el ojo
derecho est metido hacia adentro, por lo que la seora probablemente ver doble, y se escucha un soplo detrs del globo ocular. No
nos lo dicen, pero probablemente el ojo correspondiente le duele
mucho y se encuentra congestivo.
Nos ofrecen cinco diagnsticos, de los cuales el ms probable, por
la exploracin clnica de la paciente y por el antecedente traumtico,
es la fstula cartido-cavernosa (opcin 5), lesin que se ve con ms
frecuencia en pacientes que presentan fracturas de la base del crneo.
La fstula cartido-cavernosa es una comunicacin directa entre la
arteria cartida y el seno cavernoso (que es una estructura venosa). Se
trata por tanto de una fstula arteriovenosa, lo que supone un paso
directo de sangre a gran presin desde una arteria a una vena, y por
tanto la congestin del sistema venoso. Eso explica el motivo por el
que la paciente tiene un soplo (caracterstico de las fstulas arteriovenosas, en este caso retroocular, porque ese es el lugar que ocupa el
seno cavernoso). Tambin explica por qu la paciente ha desarrollado bruscamente un exoftalmos en ese lado y tiene el globo ocular y
los prpados congestivos (por sobrecarga del sistema venoso).
Las alteraciones de los pares craneales oculomotores, que producen diplopa, se deben a compresin de los mismos en el interior del
seno cavernoso. El ms frecuentemente afectado es el VI par, porque
es el nico que discurre junto a la arteria cartida interna en el interior
del seno cavernoso (por eso esta paciente tiene el ojo metido hacia
adentro, por lesin del VI par). El resto de pares craneales oculomotores
(III y IV), que tambin pueden afectarse, se localizan en la pared lateral
del seno cavernoso junto a las ramas primera y segunda del V par.
El diagnstico de sospecha que se establece por la clnica y la
exploracin se confirmar mediante angiografa cerebral. El tratamiento
de eleccin es endovascular en los casos en los que la fstula presenta
alto flujo.
HEMORRAGIA SUBARACNOIDEA (HSA).
Pregunta 28.- R: 2
Esta es una pregunta directa sobre hemorragia subaracnoidea (HSA),
una patologa con gran importancia en el examen por el nmero de
preguntas que genera. Intentaremos explicarte el manejo de la HSA
paso a paso, aunque la explicacin necesariamente debe ser un poquito extensa.
El primer concepto importante que hay que conocer para el examen es el recogido en la opcin 1. La causa ms frecuente de hemorragia subaracnoidea como tal (es decir, de sangrado en el espacio
subaracnoideo) son los traumatismos. Sin embargo, cuando en el MIR
preguntan por hemorragia subaracnoidea, si no dicen lo contrario,
suelen referirse a la espontnea, y la causa ms frecuente de hemorragia subaracnoidea espontnea es la rotura de un aneurisma cerebral. Por tanto la opcin 1 es cierta.
El segundo concepto importante que hay que distinguir es que no
es lo mismo hemorragia subaracnoidea que aneurisma cerebral; la
rotura de un aneurisma es una de las causas de HSA, la ms frecuente, pero no la nica.
Por tanto, cuando tenemos sospecha de que un paciente tiene una
HSA, hay que estudiarlo para conocer la causa de la misma, y hay que
tratar por un lado la HSA y por otro lado la causa.
M exico A rgentina
C hile U ruguay

Seguimiento a distancia

Comenzamos explicando el manejo inicial del paciente con sospecha de HSA, que es muy importante (ver figura). Sospechamos que
un paciente tiene una HSA cuando refiere la aparicin muy brusca de
un terrible dolor de cabeza (dicen los libros que es el peor dolor de
cabeza que existe) acompaado de cortejo vegetativo (sudoracin,
nuseas, vmitos). A la exploracin el paciente presentar signos
menngeos por la existencia de sangre en el espacio subaracnoideo (el
mismo mecanismo por el que hay signos menngeos en las meningitis
al existir pus en el espacio subaracnoideo). No siempre el paciente
tiene prdida de conciencia.

Pregunta 28. Manejo diagnstico del paciente con HSA espontnea.

Si un paciente acude a Urgencias con un cuadro clnico como el


que hemos descrito, ante la sospecha de HSA, la primera prueba
diagnstica que hay que realizar es una TAC craneal sin contraste
(nunca con contraste, porque el contraste se ve blanco como la sangre y nos enmascarara la hemorragia). Por tanto, la opcin 4 es correcta.
La TAC craneal por s sola diagnostica la mayor parte de las HSA
(ver figura a continuacin). Slo en aquellos casos en los que la sospecha clnica es alta pero la TAC es normal, recurriremos a la puncin
lumbar, que es una manera ms directa de saber si hay sangre o no en
el LCR. De hecho la puncin lumbar es la tcnica diagnstica ms
sensible para HSA, pero no es la primera que hacemos. Esto ha sido
objeto de pregunta MIR.
Si por TAC craneal, o por puncin lumbar, se confirma la presencia de sangre en el espacio subaracnoideo, tenemos diagnosticada la
HSA (llammoslo diagnstico sindrmico). Este es el momento de
comenzar a tratarla. La sangre subaracnoidea no se puede extirpar
quirrgicamente (no es un hematoma localizado, sino que se trata de
sangre repartida difusamente por los espacios subaracnoideos craneal y espinal. Por tanto, un concepto importante es que la hemorragia subaracnoidea no se opera (otra cosa es que pueda ser necesario
el tratamiento quirrgico de la causa de la HSA). El tratamiento de la
HSA es mdico, y consiste en administrar al paciente frmacos que
controlen la sintomatologa (analgsicos y antiemticos), junto con
otros frmacos que ayudan a prevenir o evitar un nuevo sangrado
(control de la tensin arterial, sedantes, laxantes, etc.). Adems se ad-

CTO Medicina C/ Nez de Balboa, 115 28006 MADRID (Espaa) Tfno.: (91) 782 43 32 / Fax: (91) 782 43 27
E-mail: secretaria@ctomedicina.com; iberocto@ctomedicina.com WEB: www.ctomedicina.com; www.iberocto.com

NC Pg. 13

Seguimiento a distancia

NEUROCIRUGA

Preparacin Examen de Seleccin 05/06 1 Vuelta

ministra nimodipino, un antagonista del calcio que es til para la


prevencin de una de las complicaciones ms terribles de la HSA, el
vasoespasmo. Generalmente el tratamiento de la HSA se realiza en
Unidades de Cuidados Intensivos para una mejor monitorizacin del
paciente. Si el nivel de conciencia del paciente est muy afectado,
puede ser necesario el soporte respiratorio, pero no es una norma.

en colocar un clip metlico en el cuello del aneurisma para que no


entre sangre en su interior), o el tratamiento endovascular (tambin
llamado embolizacin, que consiste en realizar un cateterismo para
rellenar el aneurisma con unas espirales de platino, de modo que la
sangre se trombose en su interior y deje de entrar sangre). Cualquiera
de los dos es un tratamiento vlido, y segn los casos concretos, se
prefiere uno u otro.

Bifurcacin de la a.
cerebral media

Unin de la a. comunicante
posterior con la
a. cartida interna

Unin de la a. comunicante
anterior con la
a. cerebral anterior

Origen de la a.
cerebral media
Top de la a. basilar

Pregunta 28. Localizaciones ms frecuentes de los aneurismas cerebrales.

Hasta ahora te hemos explicado cules son los pasos que damos
para diagnosticar una HSA (confirmar que hay sangre en el espacio
subaracnoideo), y una vez diagnosticada, cul es el tratamiento mdico inicial que debe realizarse.
Sin embargo, el manejo de estos pacientes no termina aqu. Es
necesario llegar a un diagnstico etiolgico (cul es la causa de la
HSA?). Para ello deben realizarse otras pruebas diagnsticas. Dado
que hemos visto que la causa ms frecuente de HSA espontnea es la
rotura de un aneurisma intracraneal (opcin 1), la primera tcnica
que realizaremos para buscar la causa ser una arteriografa cerebral,
con la que pretendemos ver el aneurisma que se ha roto. Sin embargo
es importante recordar aqu que en no pocas ocasiones los aneurismas cerebrales son mltiples (hasta un 20% de los casos), ms frecuentemente que lo que dice la opcin 2, que por tanto es la incorrecta
en esta pregunta. Esto tiene una implicacin muy importante: el
hecho de encontrar un aneurisma en un vaso cerebral no excluye la
presencia de otros y, por tanto, la angiografa cerebral debe incluir
los 4 vasos (las dos arterias cartidas internas y las dos vertebrales).
Slo de esta manera estaremos seguros del nmero de aneurismas
que tiene el paciente.
Es importante saber que la mayor parte de los aneurismas cerebrales,
a diferencia de los de la aorta abdominal, son aneurismas saculares.
Los aneurismas fusiformes son muy raros a nivel cerebral, pero cuando
aparecen lo hacen ms frecuentemente a nivel de la circulacin posterior (vertebrobasilar). Por tanto, la opcin 3 es cierta.
Recuerda que la localizacin ms frecuente de los aneurismas cerebrales es a nivel del polgono de Willis, y concretamente en la unin
de la arteria comunicante anterior con la arteria cerebral anterior (ver
figura).
Si se confirma la existencia de un aneurisma cerebral responsable
de la HSA, habr que tratarlo para que no se produzca un resangrado
(tratamiento etiolgico). Ya hemos visto que la HSA se trata mdicamente, pero el tratamiento de su causa ms frecuente, la rotura de un
aneurisma, es quirrgico. Hay dos tcnicas disponibles para tratar los
aneurismas cerebrales, y ambas tienen como objetivo la exclusin del
aneurisma de la circulacin general (dicho con otras palabras, que
deje de entrarle sangre para que no vuelva a romperse). Puede tratarse
un aneurisma cerebral con ciruga (clipaje quirrgico, que consiste
Pg. 14 NC

M exico A rgentina
C hile U ruguay

Como ves, no es lo mismo el diagnstico de la HSA (TAC y puncin lumbar) que el de la causa de la HSA (arteriografa), y tampoco
es lo mismo el tratamiento de la HSA (mdico) que el de su causa
(ciruga o embolizacin). Esperamos que estos conceptos te hayan
quedado claros, porque son muy importantes.
Pregunta 29.- R: 5
Nos preguntan ahora sobre un tipo muy especial de aneurismas
cerebrales, los aneurismas micticos.
Como sabes, los aneurismas cerebrales son dilataciones anormales
de la pared de una arteria cerebral, y generalmente son aneurismas
saculares de origen congnito. La rotura de un aneurisma cerebral es
la causa ms frecuente de HSA espontnea.
No todos los aneurismas cerebrales son de origen congnito. Algunos de ellos son adquiridos por diversas causas. Los aneurismas
micticos son un ejemplo de esta situacin.
Los aneurismas micticos se deben a la llegada de mbolos spticos a las arterias cerebrales, concretamente mbolos procedentes del
corazn en un paciente con endocarditis bacteriana. Por tanto, la
respuesta correcta en esta pregunta es la opcin 5.
Contrariamente a lo que su nombre sugiere en un principio, no
tienen nada que ver con los hongos; son de etiologa bacteriana.
Como habrs estudiado en Neurologa, el lugar ms frecuente de
embolias cerebrales es el territorio de la arteria cerebral media, porque es la arteria cerebral que recibe un mayor flujo. Por el mismo
motivo, la localizacin ms frecuente de los aneurismas micticos es
la arteria cerebral media.
Adems, contrariamente a los aneurismas saculares, que suelen
estar en los vasos del polgono de Willis, los aneurismas micticos
suelen tener una localizacin ms distal (su localizacin tpica es la
bifurcacin de la arteria cerebral media).
Pregunta 30.- R: 1
Esta pregunta hace referencia a signos con valor localizador en la
hemorragia subaracnoidea (HSA).
Se trata de una mujer con HSA que, en la exploracin, presenta
ptosis palpebral derecha y midriasis del mismo lado. Se trata por tanto
de una parlisis del III par craneal derecho, que como sabes es el par
craneal por el que llegan las fibras parasimpticas hasta la pupila, de
modo que, si se lesiona, produce una midriasis por aumento del tono
simptico. La ptosis palpebral se debe a hipofuncin del msculo

CTO Medicina C/ Nez de Balboa, 115 28006 MADRID (Espaa) Tfno.: (91) 782 43 32 / Fax: (91) 782 43 27
E-mail: secretaria@ctomedicina.com; iberocto@ctomedicina.com WEB: www.ctomedicina.com; www.iberocto.com

Comentarios TEST

Pregunta 28. TAC craneal de hemorragia subaracnoidea (rotura de aneurisma de arteria cerebral media derecha).

NEUROCIRUGA

Comentarios TEST

Preparacin Examen de Seleccin 05/06 1 Vuelta


elevador del prpado superior, tambin inervado por el III par craneal.
Como ya te hemos explicado, la rotura de un aneurisma cerebral
es la causa ms frecuente de HSA. Sin embargo, no todos los aneurismas cerebrales se rompen; algunos pueden aumentar progresivamente de tamao sin romperse, dando lugar a sntomas compresivos que
dependen de la localizacin del aneurisma.
Por tanto, debes saber que los aneurismas cerebrales pueden originar sntomas de dos clases: derivados de su rotura (hemorragia
subaracnoidea) o por compresin de estructuras vecinas. En algunos
casos, como el descrito en la pregunta, pueden producir los dos tipos
de sntoma a la vez. En otros, la aparicin de estos ltimos se interpreta
como sntomas centinela que anteceden a la rotura del aneurisma.
Cuando un aneurisma produce sntomas compresivos, da una
oportunidad al neurocirujano de sospechar la localizacin del mismo sin necesidad de angiografa, aunque evidentemente siempre es
necesaria la confirmacin radiolgica.
De estos signos clnicos con valor localizador, el ms importante,
y el que debes recordar, es el que sealan en esta pregunta. Cuando
un paciente con HSA tiene una parlisis del III par sugiere fuertemente la presencia de un aneurisma de la arteria comunicante posterior,
pues existe una importante relacin anatmica entre esta arteria y el III
par craneal.
Evidentemente ese no es el nico signo localizador, pero es el ms
caracterstico. Los aneurismas de arteria comunicante anterior, cuando alcanzan un tamao suficiente, pueden producir defectos campimtricos por compresin del quiasma.
No todos los aneurismas producen estos signos localizadores. En
muchas ocasiones no existe esta clnica. Podemos recurrir entonces a
signos radiolgicos con valor localizador, en concreto a la distribucin de la sangre subaracnoidea. Dicho de otra forma, podemos aventurar la localizacin ms probable del aneurisma en funcin de en
qu lugar del cerebro encontramos ms sangre subaracnoidea. De
entre ellos, merece la pena destacar la presencia de un hematoma
interhemisfrico, que sugiere la presencia de una aneurisma de la
arteria comunicante anterior o la arteria pericallosa, o la presencia de
un hematoma intrasilviano (en la cisura de Silvio), que sugiere un
aneurisma de la arteria cerebral media.
De todas formas, no debes olvidar que, aunque por la clnica o
por la radiologa podamos sospechar la presencia de un aneurisma
en alguna localizacin concreta, siempre es necesaria la confirmacin radiolgica mediante arteriografa, sin olvidar que hasta un
20% de los aneurismas cerebrales son mltiples, y por tanto es imprescindible estudiar los 4 vasos cerebrales (las dos arterias cartidas internas y las dos vertebrales) para identificar otras posibles anomalas
vasculares asociadas.
Pregunta 31.- R: 3
Esta pregunta hace referencia a las complicaciones de la hemorragia subaracnoidea (HSA). Antes de contestar la pregunta, permtenos
explicarte brevemente cules son esas complicaciones y la forma prctica de enfocarlas de cara al examen MIR.
En primer lugar te recordaremos que la HSA es una patologa muy
grave, con un ndice de mortalidad y complicaciones muy elevado.
Sirva como dato que hasta un 10% de los pacientes que la presentan
fallecen antes de llegar al hospital, y que la mortalidad dentro del
primer mes es prxima al 50%.
La HSA puede tener tres complicaciones neurolgicas principalmente: la hidrocefalia, el resangrado y el vasoespasmo. Debes aprender a identificarlas y conocer su manejo.
HIDROCEFALIA.
Como sabes la hidrocefalia es un disbalance entre la produccin y
la absorcin de LCR, y hay dos tipos de hidrocefalia (comunicante y
no comunicante, en funcin de que el LCR sea capaz de salir del
sistema ventricular al espacio subaracnoideo o no). En la HSA pueden
darse los dos tipos de hidrocefalia, y pueden aparecer de forma precoz (inmediatamente tras el sangrado) o tarda.
Puede haber hidrocefalia obstructiva o no comunicante si hay
hemorragia intraventricular asociada a la HSA. Tambin puede proM exico A rgentina
C hile U ruguay

Seguimiento a distancia

ducirse una hidrocefalia comunicante debido a que el espacio subaracnoideo est lleno de sangre y el LCR no puede circular libremente
por l.
Cuando la hidrocefalia aparece inmediatamente tras la HSA producir un cuadro clnico de hipertensin intracraneal (HTIC), con
cefalea, vmitos y deterioro rpido del nivel de conciencia. Si un
paciente con HSA se deteriora de nivel de conciencia y se confirma la
dilatacin del sistema ventricular en la TAC, el tratamiento de eleccin
es la colocacin de un drenaje ventricular externo (se trata de una
medida temporal, pues cuando se reabsorba la sangre subaracnoidea
puede restablecerse la circulacin normal del LCR).
Sin embargo, en ocasiones el paciente se recupera de la hemorragia, se va de alta, y con el paso del tiempo comienza a presentar
sntomas de hidrocefalia crnica (alteraciones cognitivas, trastorno de
la marcha, incontinencia, etc.). Estos sntomas se deben al desarrollo
de una hidrocefalia comunicante y, como en estos casos no es previsible la reversibilidad de los sntomas con el paso del tiempo, el tratamiento de eleccin es la derivacin permanente del LCR (una vlvula).
RESANGRADO.
El resangrado es la principal causa de mortalidad en los primeros
das. Evidentemente, cualquier aneurisma que se ha roto una vez
puede volver a hacerlo hasta que sea tratado y excluido de la circulacin general. Esta es la razn por la que es importante tratar el aneurisma lo antes posible. Si te ponen un caso clnico de resangrado, lo
identificars por un rpido deterioro de la situacin clnica del paciente y sntomas sugestivos de una nueva HSA (los mismos que se
produjeron la primera vez). Se trata de un evento muy grave, y la nica
forma de tratarlo (si no se trata el aneurisma) es prevenirlo, utilizando
medidas mdicas que reduzcan los factores que lo pueden provocar
(control de la tensin arterial, evitar maniobras de Valsalva mediante
laxantes, sedantes, etc.).
VASOESPASMO.
El vasoespasmo es la principal causa de morbimortalidad en los
pacientes que sobreviven a los primeros das de la HSA.
No es otra cosa que una contraccin de la musculatura de una
arteria como consecuencia de la irritacin que produce la sangre a su
alrededor. El resultado es una reduccin del flujo a travs de esa arteria,
y la aparicin de sntomas isqumicos en el territorio dependiente de
esa arteria. Por tanto, lo identificars en los casos clnicos ante la aparicin de nuevos dficits focales unos das despus del sangrado. Es importante este ltimo dato; es necesario el paso de ciertos das para la
aparicin de los sntomas, ya que la arteria debe ser irritada, cerrarse y
la reduccin del flujo producir la isquemia, y esto lleva su tiempo. Por
ello no suele verse antes del 4 da postsangrado.
En resumen, se identifica por la aparicin de nueva focalidad
neurolgica en el periodo de riesgo (entre el 4 y 12 da).
En el caso que nos ocupa hay que sospechar vaoespasmo porque,
7 das despus del sangrado, el paciente ha desarrollado una nueva
focalidad, la disfasia, que sugiere vasoespasmo de la arteria cerebral
media izquierda. Como consecuencia de la isquemia de un territorio
tan amplio como el de esta arteria, la disfasia se acompaa de un
deterioro del nivel de conciencia del paciente.
El tratamiento de eleccin del vasoespasmo consiste en aumentar el
flujo por la arteria espstica. Para ello utilizamos la terapia triple H:
aumentar la tensin arterial para vencer la resistencia que ofrece el vaso
de menor calibre (hipertensin), diluir la sangre para facilitar su paso
por ese vaso (hemodilucin) y aumentar la volemia para aumentar la
cantidad de sangre que pasa por esos vasos (hipervolemia). Recuerda
que la profilaxis del vasoespasmo se realiza desde el primer da del
ingreso tras el sangrado con antagonistas del calcio (nimodipino).
De entre las opciones indicadas, slo una recoge la filosofa del
tratamiento del vasoespasmo que te hemos explicado.
La opcin 1 no es vlida, porque si restringimos el aporte de lquidos, reduciremos la volemia. La opcin 2 es incorrecta, porque al
reducir la tensin arterial se reducir el flujo sanguneo cerebral. No
vale tampoco la opcin 4, porque clipar el aneurisma sera el tratamiento de la causa de la HSA, y evitara el resangrado, pero no afecta
en nada al vasoespasmo una vez producido. Tampoco vale la opcin

CTO Medicina C/ Nez de Balboa, 115 28006 MADRID (Espaa) Tfno.: (91) 782 43 32 / Fax: (91) 782 43 27
E-mail: secretaria@ctomedicina.com; iberocto@ctomedicina.com WEB: www.ctomedicina.com; www.iberocto.com

NC Pg. 15

5, porque aunque la primera parte del enunciado es correcta (aumentar la dosis de nimodipino), la segunda no lo es, ya que no podemos reducir la tensin arterial.
Por todo ello, y aunque la opcin 3 es incompleta, es la nica que
se ajusta a los principios del tratamiento del vasoespasmo (el salino
hipertnico aumentar la volemia y elevar la tensin arterial).
Pregunta 32.- R: 2
Esta pregunta no requiere mucha explicacin. Hay algunas patologas sistmicas que se asocian a una mayor incidencia de aneurismas
cerebrales. En muchos casos se trata de enfermedades del tejido
conectivo en las que existe una debilidad de la pared vascular, pero
hay otras patologas en las que tambin est aumentada la incidencia,
entre ellas la coartacin de aorta recogida en la opcin 2 de esta
pregunta.
Algunas enfermedades en las que est aumentada la incidencia de
aneurismas cerebrales se recogen en la tabla.

Pregunta 32. Enfermedades con aumento de riesgo


de aneurismas cerebrales.

Sndrome de Marfan.
Sndrome de Ehlers-Danlos (defecto del colgeno tipo III).
Pseudoxantoma elstico.
Displasia fibromuscular.
Poliquistosis renal.
Anemia de clulas falciformes.
Coartacin de aorta.
Sndrome de Rendu-Osler-Weber.

PATOLOGA RAQUIMEDULAR.
Comenzamos ahora con uno de los bloques ms importantes de
esta asignatura en trminos de nmero de preguntas MIR, el bloque
de la patologa raquimedular y, en concreto, el de las lumbociatalgias.
Antes de nada nos permitimos una reflexin sobre la orientacin
que estas preguntas han tenido en los ltimos exmenes MIR. Hasta
hace unos dos aos, todas las preguntas pedan averiguar en qu nivel
de la columna lumbar o cervical se localiza una hernia discal que
produce unos sntomas determinados. Es importantsimo que seas
capaz de reconocer el nivel de lesin segn los sntomas que te exponen en el enunciado.
Sin embargo, desde hace dos aos muchas preguntas hacen referencia a la actitud ms adecuada ante estas enfermedades. Estas preguntas son relativamente recientes en la historia del MIR, pero no por
ello menos importantes, y en las prximas preguntas te daremos pistas
para poder contestarlas mejor.
Pregunta 33.- R: 3
Como sta han sido las preguntas que clsicamente ponan en el
examen; hay que averiguar el nivel de la hernia discal en base a los
sntomas que te presentan.
Como consejo, te diremos que es necesario que conozcas los sntomas motores, sensitivos y los reflejos afectados con la lesin de cada
raz nerviosa lumbar o cervical. Sin embargo, en la prctica, todas las
preguntas que han puesto en el examen, relativas a hernias discales
lumbares, se contestaban sabiendo el reflejo que se altera en cada raz
(incluso si no te acordaras de los sntomas motores o sensitivos, aunque lgicamente conocer stos te ayudar mucho). Por tanto, en todas las preguntas de hernias discales lumbares hay que fijarse en el
reflejo afectado.
En este caso clnico, el paciente, de mediana edad, tiene citica
desde hace dos meses, la exploracin de la sensibilidad y la fuerza en
miembros inferiores es normal (datos que por tanto no ayudan a
localizar el nivel), pero tiene un reflejo patelar o rotuliano disminuido
en el lado de la citica. La alteracin del reflejo, como siempre, nos
localiza el nivel de la lesin. El reflejo rotuliano se afecta en las lesiones
de la raz L4, y sta es la raz que se afecta en las hernias discales L3-L4
(recuerda que en las hernias discales posterolaterales, las que habiPg. 16 NC

M exico A rgentina
C hile U ruguay

NEUROCIRUGA

Preparacin Examen de Seleccin 05/06 1 Vuelta

tualmente preguntan en el MIR, la raz que se afecta es la que lleva el


nombre de la vrtebra de abajo).

Pregunta 33. Semiologa de la lesin de las races del plexo lumbosacro.


12314

14315

15367

89

9 9
999

12

13

45

8
99

123456782

4 5 2

67  9 7


6789
 7 7 77 7

 77
77

77  
6789
 7512
77 




67 9
77
7 7
7 
 !

67"7! 7
67"
67#$
777  
77

7
77 9
 
67#$
777  
67& 7
7'
77
 
67 7 7
67"7! 
77%
75127 
77 7 

Todas las preguntas de hernias discales lumbares del MIR se han


podido contestar localizando el reflejo alterado. Ya sabes que el rotuliano se afecta en las hernias discales L3-L4 (raz L4) y el aquleo en las
L5-S1 (raz S1). Recuerda que la raz L5 (hernias L4-L5) no tiene
ningn reflejo asociado. Slo queremos llamar tu atencin sobre
una pregunta MIR que hizo referencia a una lesin de la raz L5 y
deca que el aquleo poda estar abolido, dando esta opcin como
cierta. Aunque habitualmente la raz L5 no asocia ningn reflejo (y
eso es lo que tienes que aprenderte), ocasionalmente puede asociarse
con el aquleo (este dato est recogido en el Harrison, tratado de
Medicina Interna, pero se trata de una excepcin y no debes tomarlo
como un dato importante). En definitiva, lo que debes recordar es que
la raz L5 no asocia ningn reflejo.
Pregunta 34.- R: 3
Esta pregunta es el equivalente a la pregunta anterior, pero a nivel
de la columna cervical. Se trata de localizar la sintomatologa que no
aparecera en una hernia discal cervical C5-C6.
Como ya hemos mencionado, en el caso de las preguntas sobre
hernias discales lumbares, la clave est siempre en mirar el reflejo afectado. No es as en las hernias discales cervicales, porque los reflejos
osteotendinosos de miembros superiores pueden ser compartidos por
varias races. En el caso de las hernias cervicales, no te recomendamos
mirar el reflejo afectado; sin embargo, todas las preguntas del MIR que
hacan referencia a ellas se contestaban mirando el territorio sensitivo
afectado, y ms concretamente, los dedos de la mano en los que el
paciente presentaba dolor, hipoestesia o parestesias.
Recuerda que las hernias C5-C6 (raz C6) afectan a la sensibilidad
de 1 y 2 dedos, las hernias C6-C7 (raz C7) al 3 dedo, y las hernias
C7-D1 (raz C8) a 4 y 5 dedos de la mano. Con esta informacin se
contestaban prcticamente todas las preguntas MIR, aunque no fueras capaz de recordar las alteraciones motoras o el reflejo asociado
con la lesin de las races cervicales.
Lo que te hemos dicho es aplicable a la mayora de las preguntas del
examen MIR, pero lamentablemente no a la pregunta que nos ocupa,
que nos obliga a saber la clnica motora, sensitiva y el reflejo afectado en
las hernias discales C5-C6 (por tanto, asociados a la lesin de la raz C6).
La opcin falsa es la 3, puesto que la debilidad para la extensin
del codo, y en general para todos los movimientos extensores del
miembro superior, es consecuencia de la lesin de la raz C7 (por
tanto aparece en las hernias C6-C7).
Las opciones 1, 2 y 4 son ciertas porque la lesin de la raz C6
produce alteraciones sensitivas en 1 y 2 dedo, alteracin de los
reflejos bicipital y estilorradial, y debilidad para la flexin del codo.

CTO Medicina C/ Nez de Balboa, 115 28006 MADRID (Espaa) Tfno.: (91) 782 43 32 / Fax: (91) 782 43 27
E-mail: secretaria@ctomedicina.com; iberocto@ctomedicina.com WEB: www.ctomedicina.com; www.iberocto.com

Comentarios TEST

Seguimiento a distancia

NEUROCIRUGA

Preparacin Examen de Seleccin 05/06 1 Vuelta


Respecto a la opcin 5, la maniobra de Spurling es el equivalente
en la exploracin de las hernias cervicales a la maniobra de Lasgue
en las lumbares. Consiste en presionar en el vrtex craneal con la
cabeza extendida y girada hacia el lado de la cervicobraquialgia; si se
reproduce el dolor, esta maniobra es positiva.

Comentarios TEST

Pregunta 34. Semiologa de la lesin de las races del plexo braquial.


C 4-C 5

C 5-C 6

C 6-C 7

C 7-D1

Raiz
habitualmente
afectada

C5

C6

C7

C8

Reflejo
alterado

B icipital

B icipital
Estilorradial

Tricipital

Tricipital

- Extensin
codo.
- Flexin
mueca.

- Flexin dedos.
- Musculatura
intrnseca de
la mano.

- 3 dedo.

- 4 y 5 dedos.
- Cara medial
del antebrazo.

Dficit motor

- Separacin
hombro.
- Flexin
codo.

- Flexin codo.
- Extensin
mueca.

Dficit
sensitivo

- Hombro.
- Cara
lateral del
brazo.

- Cara lateral
del antebrazo.
- 1 y 2 dedos.

Pregunta 35.- R: 2
El diagnstico de sospecha que debes establecer en el paciente
descrito es una raquiestenosis lumbar.
La estenosis del canal vertebral lumbar produce clnica dolorosa
en miembros inferiores, que generalmente se desencadena al caminar (claudicacin neurognica), cediendo con el reposo. A diferencia
de la hernia discal lumbar, en la que el dolor de espalda es mayor con
la flexin de la columna, en el caso de la estenosis del canal, el dolor
aumenta con la extensin del tronco. Se dice clsicamente que los
pacientes con raquiestenosis lumbar pueden montar en bicicleta,
porque les duele menos la espalda al estar flexionada.
Es importante realizar diagnstico diferencial entre claudicacin
neurognica y claudicacin vascular, que se produce por insuficiencia del sistema arterial de los miembros inferiores (ver tabla).
Por tanto, estamos ante un paciente con posible raquiestenosis
lumbar, y nos preguntan el tratamiento de eleccin. La respuesta
correcta es la opcin 2; el tratamiento de eleccin de la estenosis de
canal lumbar es la ampliacin del dimetro del canal medular, que se
consigue al extirpar los elementos posteriores de la vrtebra (el arco
posterior, o lo que es lo mismo las lminas). La extirpacin de la
apfisis espinosa y las lminas de la vrtebra se conoce como laminectoma.
La discectoma, recogida en la opcin 1, consiste en la extirpacin
del disco intervertebral, y e s la tcnica quirrgica de eleccin para el
tratamiento de las hernias discales. A nivel lumbar se realiza generalmente por va posterior, pero a nivel cervical se hace por va anterior
(ya que es ms difcil llegar al disco desde atrs, por estar en el trayecto
la mdula cervical). Generalmente, para tratar las hernias lumbares se
realiza una hemilaminectoma y discectoma, mientras que para las
hernias cervicales se suele hacer una discectoma y artrodesis
intersomtica por va anterior.
Se denomina flavectoma (opcin 3) a la extirpacin de los ligamentos amarillos, corpectoma (opcin 4) a la extirpacin de un
cuerpo vertebral y sustitucin del mismo por hueso o injertos metlicos. Finalmente artrodesis es una fusin entre elementos de dos vrtebras que se consigue por diversos medios (interponiendo hueso,
tornillos y barras, injertos metlicos, alambres, etc.).
Recuerda que el tratamiento inicial de eleccin de la hernia discal,
tanto cervical como lumbar, no es la ciruga, sino el tratamiento mdico con analgsicos, antiinflamatorios y relajantes musculares. Solamente deben ser intervenidos los pacientes en los que fracasa el tratamiento conservador, o en los que aparecen signos de lesin radicular
con prdida de fuerza objetivable clnicamente o con EMG, o signos
de compresin de la cola de caballo o mielopata (ver tabla a continuacin).
M exico A rgentina
C hile U ruguay

Seguimiento a distancia

Pregunta 35. Diagnstico diferencial entre claudicacin


neurognica y claudicacin vascular.

D istribucin del
dolor

Claudicacin
neurognica

Claudicacin
vascular

Territorio de un nervio
(dermatoma)

Grupo muscular con


irrigacin comn

Ejercicio de
intensidades
variables.
Mantenimiento
Factores
prolongado de una
desencadenantes
postura.
Al ponerse en pie,
antes de comenzar la
marcha.

D istancia al
caminar para
aparicin
Alivio con el
reposo

Variable

Ejercicio con
intensidad constante,
menor conforme
progresa la enfermedad.
Raro en pie sin
caminar.

Constante

Lento.
Inmediato.
Dependiente de la
No depende de la
postura (mejor en
postura.
flexin de la columna).

Pulsos
perifricos

Conservados

Disminuido o ausentes

Palidez cutnea
al elevar los
MMII

No

Marcada

Temperatura en
los MMII

Normal

Disminuida

En este sentido, han aparecido preguntas en los ltimos exmenes


MIR que hacen referencia a la actitud ms adecuada ante un paciente
con lumbocitica o cervicobraquialgia de reciente aparicin, y la
respuesta correcta en todos los casos ha sido que debe tratarse mdicamente, sin necesidad de realizar pruebas diagnsticas inicialmente,
y solamente en caso de que pasado un mes no haya mejorado, o
cuando presenta factores de riesgo, deben realizarse estas pruebas
para descartar una causa que requiera tratamiento quirrgico.

Pregunta 35. Indicaciones de ciruga de la hernia discal.


Lesin de la raz que produce una prdida aguda o progresiva de
fuerza objetivable clnicamente o por EMG. Es indicacin de ciruga urgente.
Signos clnicos sugestivos de sndrome de cola de caballo o lesin
medular (disfuncin de esfnteres, anestesia perineal en silla de
montar, etc.). Es indicacin de ciruga urgente.
Fracaso del tratamiento conservador, es decir, dolor incapacitante
de caractersticas radiculares que no responde a tratamiento mdico durante un perodo mnimo de 4 semanas.
Incapacidad recidivante a pesar del tratamiento mdico.
Pregunta 36.- R: 2
Esta pregunta hace referencia a las lesiones medulares traumticas.
Desde el punto de vista clnico, la semiologa de las lesiones medulares traumticas no difiere de la que se puede encontrar en otros
tipos de afectacin medular (tumoral, infecciosa, esclerosis mltiple,
etc.). El estudio de la clnica de estas lesiones se recoge en el apartado
de Semiologa del Manual CTO, que debes repasar, y puede simplificarse mucho si recordamos unas breves nociones de la anatoma
medular y su implicacin clnica:
Como sabes, en la mdula hay sustancia gris (donde estn los
cuerpos de las neuronas) y sustancia blanca (por donde discurren

CTO Medicina C/ Nez de Balboa, 115 28006 MADRID (Espaa) Tfno.: (91) 782 43 32 / Fax: (91) 782 43 27
E-mail: secretaria@ctomedicina.com; iberocto@ctomedicina.com WEB: www.ctomedicina.com; www.iberocto.com

NC Pg. 17

NEUROCIRUGA

Preparacin Examen de Seleccin 05/06 1 Vuelta

las fibras nerviosas de las vas ascendentes o sensitivas y descendentes o motoras). Cualquier lesin medular (tambin las traumticas)
va a afectar a estas dos estructuras en el nivel de la lesin, pero
tambin va a producir una interrupcin de las vas que ascienden
y descienden, y por lo tanto va a interrumpir la inervacin motora
y sensitiva por debajo de la lesin.
Por tanto, para simplificar el estudio de la clnica de las lesiones
medulares debes recordar que habr sntomas sensitivos y motores
(de 2 motoneurona) en el territorio correspondiente al nivel de la
lesin, y clnica sensitiva y motora (pero de 1 motoneurona) por
debajo de la lesin.
En el nivel de la lesin encontrars por tanto una anestesia de ese
territorio y una parlisis flccida con arreflexia e hipotona. Ejemplo: una lesin medular C7 producir una anestesia en dermatoma C7 y parlisis flccida de la musculatura extensora de miembros superiores.
Si embargo, por debajo de la lesin tambin encontraremos una
hipoestesia, pero el trastorno motor es de 1 motoneurona, y por
tanto se objetivar parlisis espstica con hiperreflexia, hipertona,
clonus y respuesta plantar extensora (signo de Babinski). Ejemplo:
la lesin medular C7 que hemos descrito ms arriba produce parlisis flccida de la musculatura extensora de miembros superiores,
pero tambin habr parlisis de miembros inferiores, aunque en
este caso espstica con la exploracin que te hemos descrito.
Respecto a las vas largas, ascendentes y descendentes, debes recordar que se afecta todo el territorio localizado por debajo de la
lesin. Para simplificar el estudio te recomendamos que recuerdes
slo tres de estas vas, y es importante que sepas dnde se cruzan
para entender si la clnica es ipsilateral o contralateral:
- La va corticoespinal (descendente), por la que discurren las
primeras motoneuronas, se cruza a nivel del bulbo, y por tanto
ya viene cruzada en la mdula, as que los sntomas sern del
mismo lado de la lesin. Se localiza en los cordones laterales de
la mdula. Su afectacin produce parlisis espstica ipsilateral
por debajo de la lesin, como ya hemos descrito.
- La va espinotalmica (ascendente) lleva las fibras correspondientes a la sensibilidad termoalgsica (dolor y temperatura), y
se cruza a nivel medular (por el centro de la mdula), por lo que
su lesin produce sntomas del lado contrario al de la lesin. Se
localiza en los cordones anteriores de la mdula. Su lesin producir prdida de la sensibilidad a la temperatura y al dolor
contralateral por debajo de la lesin.
- La va lemniscal (ascendente) discurre por los cordones posteriores de la mdula y lleva fibras correspondientes a la sensibilidad propioceptiva (posicin y vibracin), y se cruza a nivel del
tronco, por lo que su lesin produce sntomas del mismo lado
de la lesin. La clnica, cuando se afecta, es prdida de la sensibilidad propioceptiva ipsilateral por debajo de la lesin, lo que
implica la aparicin de una ataxia sensitiva.

Una vez recordados estos conceptos, podemos contestar la pregunta.


La opcin 3 se refiere al sndrome de Brown-Squard, tambin
conocido como sndrome de hemiseccin medular. De su nombre se
deduce que se lesiona la mitad de la mdula (los cordones posterior,
lateral y anterior de un lado), y por tanto las vas lemniscal,
corticoespinal y espinotalmica respectivamente de un mismo lado,
por lo que se producir prdida de la sensibilidad propioceptiva
ipsilateral, parlisis ipsilateral y prdida de la sensibilidad termoalgsica contralateral por debajo de la lesin. Por tanto, esta respuesta es
correcta.
La opcin 4 hace referencia al sndrome medular central. Este es
un sndrome muy caracterstico en el que se interrumpen las vas que
cruzan por el centro de la mdula, y por tanto las fibras sensitivas
termoalgsicas, por lo que suele producir una alteracin suspendida
de la sensibilidad para la temperatura y el dolor en los niveles afectados. Esta alteracin se conoce tambin como dficit disociado de la
sensibilidad (se altera la del dolor y la temperatura, pero se conserva
la propioceptiva), y es tpica de la siringomielia (una enfermedad en la
que se crea una cavidad rellena de lquido en el centro de la mdula
que interrumpe las fibras descritas). Este es el rasgo clnico ms tpico
del sndrome centromedular, pero con l no se contesta la pregunta
porque han preguntado algo ms rebuscado. Cuando la lesin
centromedular es grande, puede afectar tambin a otras vas largas de
la mdula, en este caso a la corticoespinal, que lleva fibras motoras, y
puesto que en la va corticoespinal las fibras nerviosas correspondientes a los miembros superiores van colocadas ms medialmente o centralmente que las de los miembros inferiores, es ms probable que el
dficit motor afecte ms a brazos que a piernas. Por tanto, esta opcin
tambin es cierta; en el sndrome centromedular, la paresia es mayor
en miembros superiores que inferiores y hay una alteracin suspendida y disociada de la sensibilidad.
La clnica de otras lesiones medulares se deduce conociendo la
localizacin de la lesin (ver tabla a continuacin).
Las otras tres opciones se refieren al tratamiento de las lesiones
medulares traumticas.
La opcin 5 es cierta; si existe una listesis cervical, es decir, un
desplazamiento de una vrtebra cervical sobre otra, con el riesgo consiguiente de lesin medular, la primera medida teraputica debe ser
conseguir una correcta alineacin de la columna para reducir el dao
medular, y por tanto el tratamiento inicial es la traccin cervical. Despus, en funcin del tipo de lesin se recurrir a la ciruga para fijar
esa columna. El tratamiento de la posible lesin medular es el mismo
que en otros tipos de lesin traumtica.
La opcin 1 tambin es cierta; en todo paciente con lesin medular traumtica debe iniciarse tratamiento con dosis muy altas de
metilprednisolona en las primeras 8 horas del traumatismo, que se
mantienen durante 8 horas. El motivo es que ciertos estudios (NASCIS)
demostraron un mejor pronstico funcional de los pacientes tratados
de este modo comparados con los que no reciban corticoides.

Cordn posterior.
Alt. sensibilidad propioceptiva y ataxia sensitiva ipsilaterales
Va piramidal o corticoespinal.
Lesin de 1 motoneurona (parlisis espstica) ipsilateral

Asta anterior.
Lesin de 2 motoneurona (parlisis flccida) ipsilateral
Va espinotalmica.
Alt. sensibilidad termoalgsica contralateral
Pregunta 36. Vas ms importantes de la mdula espinal y clnica asociada a su lesin.

Pg. 18 NC

M exico A rgentina
C hile U ruguay

CTO Medicina C/ Nez de Balboa, 115 28006 MADRID (Espaa) Tfno.: (91) 782 43 32 / Fax: (91) 782 43 27
E-mail: secretaria@ctomedicina.com; iberocto@ctomedicina.com WEB: www.ctomedicina.com; www.iberocto.com

Comentarios TEST

Seguimiento a distancia

NEUROCIRUGA

Preparacin Examen de Seleccin 05/06 1 Vuelta


La respuesta falsa es la 2, porque la persistencia de un dficit
medular completo ms all de las primeras 24 horas indica que no se
va a producir recuperacin neurolgica, y por tanto el tratamiento
descompresivo no ser til. Recuerda que se denomina lesin
completa a aquella en la que no existe ninguna funcin neurolgica
conservada ms de tres segmentos por debajo del nivel de la lesin, y
que es un signo de mal pronstico, pues menos de un 3% de los
pacientes con lesiones completas en la primera exploracin presentarn alguna mejora en las primeras 24 horas.

Pregunta 36. Sndromes medulares.


Mielopata
transversa

Sd. Brown-Sequard
(hemiseccin medular)

Sd. medular central

Sd. medular posterolateral

Sd. cordonal posterior

Seguimiento a distancia

Pregunta 38.- R: 5
Atencin a esta pregunta, porque en los ltimos aos estn preguntando sobre los factores de riesgo que pueden indicar una etiologa
grave del dolor lumbar. Como ya te hemos explicado, hasta hace
unos dos aos, todas las preguntas sobre lumbociticas hacan referencia al nivel en el que se encontraba la posible hernia discal en base
a los datos clnicos y a la exploracin del paciente. Sin embargo, en
los ltimos exmenes han preguntado sobre la actitud diagnstica y
teraputica con estos pacientes. Es sencillo contestar estas cuestiones,
pero hay que tener algunos conceptos claros:
El manejo inicial de todo paciente con lumbalgia o lumbocitica de
reciente comienzo sin factores de riesgo que sugieran una etiologa
grave del dolor debe ser conservador. Lo que han querido que
contestemos en las preguntas MIR que han aparecido sobre este tema
es que a estos pacientes hay que recomendarles tratamiento mdico
con analgsicos, antiinflamatorios no esteroideos y relajantes musculares, no hacerles ninguna prueba de imagen, y reevaluarles al cabo
de 15 das. Es muy importante que recuerdes esto.
Cundo solicitaremos pruebas diagnsticas?. Como te hemos
explicado, ante una lumbocitica o lumbalgia de reciente comienzo sin factores de riesgo se iniciar tratamiento mdico y se
reevaluar al paciente a los 15 das. Si es necesario, se corregir el
tratamiento en ese momento. Slo si persiste la clnica con un
tratamiento correcto al cabo de un mes, y por supuesto, si hay
factores de riesgo de etiologa grave del dolor, se solicitarn estudios de imagen u otras pruebas diagnsticas (analticas,
electrofisiologa, etc).
Cules son los factores de riesgo que sugieren una etiologa
grave del dolor lumbar o la citica?. Si en la historia clnica o en la
exploracin se recoge algn dato que sugiera que la etiologa del
dolor puede ser infecciosa o tumoral, cuando existe el antecedente de un traumatismo, o en los casos en que la exploracin
neurolgica muestra un dficit neurolgico, es necesario realizar
estudios diagnsticos desde un primer momento. La siguiente tabla
recoge los factores de riesgo que debes recordar.

Pregunta 38. Factores de riesgo de etiologa grave del dolor lumbar.

Comentarios TEST

Sd. medular anterior

Pregunta 37.- R: 3
Esta pregunta hace referencia a lesiones de las primeras vrtebras
cervicales, que encontrars mejor explicadas en Traumatologa. No
obstante, recordaremos aqu las ms frecuentes.
La fractura de Jefferson (opcin 1) es una fractura de los arcos
posterior y anterior del atlas, generalmente inestable, que no suele
acompaarse de dficit neurolgicos si no hay otras lesiones asociadas debido a que el canal raqudeo es muy amplio a ese nivel.
La fractura de Hangman o fractura del ahorcado, producida por
hiperextensin del crneo sobre la columna cervical, es una fractura
bilateral de la pars interarticularis de los pedculos de C2, y por tanto
la respuesta 3 es falsa. Suele acompaarse de subluxacin anterior
de C2 sobre C3. Generalmente son fracturas estables y no requieren
fijacin quirrgica, pero s inmovilizacin externa prolongada.
La luxacin atloaxoidea (opcin 2) suele ser secundaria a traumatismos o aparece en relacin con enfermedades sistmicas. La asociacin
ms frecuente es con la artritis reumatoide como consecuencia del
pannus inflamatorio que se produce alrededor de la articulacin
odontoidea. Sin embargo, puede aparecer en el contexto de la enfermedad de Morquio por hipoplasia de la apfisis odontoides. Generalmente provoca dolor suboccipital. Si no produce sntomas y la luxacin es pequea, puede tratarse con collarn cervical. Si da sntomas
importantes o hay gran desplazamiento requiere ciruga.
Finalmente haremos un breve comentario para recordar que la
ausencia de lesiones seas en un traumatismo cervical no excluye
que pueda existir un dao neurolgico asociado. Hay lesiones medulares cervicales sin lesin sea asociada (en ingls se conocen con las
siglas SCIWORA Spinal Cord Injuries Without Radiographic
Abnormality). En estos casos es obligada la realizacin de una RM
cervical, por lo que es cierta la opcin 5.
M exico A rgentina
C hile U ruguay

Edad mayor de 50 aos.


Diagnstico previo de cncer.
Diagnstico previo de enfermedad sistmica grave.
Antecedentes de traumatismo espinal.
Historia de ciruga reciente (espinal o no).
Infeccin crnica pulmonar, urinaria o cutnea.
Consumo de drogas por va parenteral.
Historia de inmunosupresin (trasplante, VIH, etc).
Tratamiento prolongado con glucocorticoides.
Duracin del dolor superior a 1 mes.
Ausencia de alivio con reposo en cama.
Aparicin de incontinencia urinaria o fecal, o nicturia reciente.
Hallazgos exploratorios:
- Fiebre inexplicada.
- Prdida de peso llamativa inexplicada.
- Masa abdominal.
- Alteraciones neurolgicas sugestivas de sndrome de cauda
equina (prdida de fuerza en miembros inferiores, anestesia en
silla de montar, incontinencia urinaria o fecal, etc...).

La respuesta falsa es la 5, porque los individuos jvenes no tienen


mayor riesgo. Sin embargo son ciertas todas las dems. La falta de
alivio del dolor con el reposo en cama, o su persistencia durante ms
de un mes, son criterios para profundizar en la etiologa del dolor, al
igual que la aparicin de defectos neurolgicos. Finalmente, la toma
crnica de glucocorticoides favorece la infeccin y la osteoporosis,
por lo que la aparicin de dolor lumbar en estos pacientes obliga a
descartar estas causas.
Pregunta 39.- R: 5
Vamos a aprovechar esta pregunta para recordarte algunos conceptos bsicos sobre los tumores espinales.

CTO Medicina C/ Nez de Balboa, 115 28006 MADRID (Espaa) Tfno.: (91) 782 43 32 / Fax: (91) 782 43 27
E-mail: secretaria@ctomedicina.com; iberocto@ctomedicina.com WEB: www.ctomedicina.com; www.iberocto.com

NC Pg. 19

Seguimiento a distancia

NEUROCIRUGA

Preparacin Examen de Seleccin 05/06 1 Vuelta

La clnica de estas lesiones va a depender del nivel en el que se


encuentren, y se deduce fcilmente si tienes en cuenta los conceptos
que te hemos explicado sobre las lesiones medulares.
Los tumores raquimedulares se clasifican en tres grandes grupos
(no estn incluidos los tumores primarios del hueso, que estudiars en
Traumatologa):
Tumores extradurales o epidurales. En este grupo se recogen todos los tumores que se localizan en el canal raqudeo por fuera de
la duramadre. De todos ellos, los ms frecuentes son las metstasis,
aunque la mdula puede comprimirse tambin por tumores vertebrales que en su crecimiento alcancen el canal raqudeo.
Las metstasis epidurales espinales pueden tener su origen en diversos tumores primarios, pero lo ms frecuente es que sean de origen
hematolgico (linfomas), o tengan su origen en el pulmn, la mama
o la prstata (opciones 1, 2 y 3). Generalmente las metstasis vertebrales son destructivas, pero pueden ser osteoblsticas (forman hueso) las de mama y las de prstata, y debes recordar este dato.
De entre todos los tumores citados, el que menos frecuentemente
produce metstasis raqudeas es el de colon. Por tanto, la respuesta correcta es la 5.
Tumores intradurales extramedulares. Los tumores intradurales
son menos frecuentes que los epidurales. Se subdividen a su vez
segn se localicen dentro o fuera de la mdula.
Entre los tumores intradurales extramedulares hay que destacar
dos: los neurinomas (los tumores primarios espinales ms frecuentes) y los meningiomas. Esto es lgico, porque entre la duramadre
y la mdula se encuentran la aracnoides (lugar de origen de los
meningiomas) y las races nerviosas (origen de los neurinomas).
Otros tipos histolgicos son mucho menos frecuentes a este nivel,
incluidas las metstasis.
Tumores intradurales intramedulares. Debes recordar en este
grupo solamente dos tipos histolgicos: los gliomas (tumores
derivados de las clulas gliales) y los ependimomas (derivados
de tejido ependimario medular). Entre los ependimomas merece ser recordado el ependimoma del filum terminale, porque
tiene una histologa muy caracterstica que solamente se ve en
los ependimomas de esta localizacin: son ependimomas
mixopapilares.

Pregunta 40. Craneoestenosis y sutura implicada.


Escafocefalia
Sutura sagital
Trigonocefalia
Sutura metpica
Plagiocefalia
Sutura coronal unilateral
Braquicefalia
Sutura coronal bilateral
Paquicefalia (plagiocefalia posterior) Sutura lambdoidea
Turricefalia (oxicefalia)
Todas las suturas
Pregunta 41.- R: 1
El trmino malformacin de Chiari se refiere a un conjunto de entidades que probablemente se deben a distintos mecanismos patognicos.
Aunque se numeran del tipo I al tipo IV, no son distintos estadios de una
misma enfermedad, es decir, no se pasa de un tipo al otro. No obstante, se
cumple que a mayor nmero en el nombre mayor es la gravedad, menor
es la edad de aparicin de la sintomatologa y menos frecuentes son.
Es importante que recuerdes en qu consisten, la edad de aparicin y si asocian algunas otras patologas neurolgicas (en concreto,
cul asocia hidrocefalia, siringomielia o mielomeningocele). Fjate en
estos datos cuando las estudies.
La ms importante de todas es la malformacin de Chiari tipo I,
porque es la ms frecuente. Consiste en un descenso de las amgdalas
cerebelosas por debajo del foramen magno, debido a un menor tamao de la fosa posterior (ver figura). Es la que aparece ms tarde
cronolgicamente, generalmente en la edad adulta, por lo que la
respuesta falsa es la 1. Con frecuencia se asocia con siringomielia
(cavidades rellenas de lquido en el interior de la mdula espinal), y
esto ha sido objeto de pregunta MIR. El sntoma ms frecuente del
Chiari tipo I es la cefalea suboccipital (opcin 5). El tratamiento quirrgico de esta entidad consiste en aumentar el tamao de la fosa
posterior, extirpando el hueso que la recubre y abriendo la duramadre que luego se cierra con un injerto de mayor tamao (craniectoma
suboccipital y duraplastia).

MALFORMACIONES DEL SNC.

Pregunta 40.- R: 3
Todas las malformaciones recogidas en las opciones de esta pregunta se engloban bajo el trmino comn de craneoestenosis o
craneosinostosis, y se producen como consecuencia del cierre precoz de las suturas craneales recogidas en el segundo componente de
los binomios propuestos.
Respecto a las craneoestenosis, que no han sido preguntadas en el
examen MIR, hay que recordar la patogenia (cierre precoz de la sutura correspondiente) y, en general, saber que se trata de malformaciones que tienen una importancia esencialmente cosmtica, porque en
general los nios no suelen presentar alteraciones de su capacidad
intelectual.
Por tanto, el tratamiento busca conseguir un resultado esttico adecuado en la mayora de los casos. Para ello recurrimos a reconstrucciones quirrgicas mediante craneotomas, remodelacin del reborde orbitario, etc.
En la siguiente tabla se recogen las craneoestenosis ms frecuentes
y la sutura afectada. La respuesta falsa es la 3, porque la plagiocefalia
se produce por el cierre precoz de la sutura coronal unilateral, no de
la sutura pterional. Aprovechamos para aclarar que, cuando se habla
de plagiocefalia sin decir nada ms, nos referimos a la plagiocefalia
anterior, porque existe otra forma de plagiocefalia posterior por cierre
precoz de la sutura lambdoidea que tambin se denomina paquicefalia
y se recoge en la opcin 4. El cierre precoz de la sutura coronal
bilateralmente da lugar a la braquicefalia (opcin 5).

Pg. 20 NC

M exico A rgentina
C hile U ruguay

Pregunta 41. Malformacin de Chiari tipo I.

Respecto al Chiari tipo II, lo que desciende por debajo del foramen magno no son slo las amgdalas cerebelosas (como en el tipo I).
En esta entidad, gran parte del contenido de la fosa posterior se hernia
hacia abajo (bulbo, protuberancia, IV ventrculo y vermis). En consecuencia, cursa con clnica ms grave, y con frecuencia provoca la
muerte por alteraciones respiratorias (disfuncin troncoenceflica).
Debuta antes que el tipo I (en la infancia) y muy frecuentemente se
asocia con mielomeningocele e hidrocefalia (opcin 2).
El Chiari tipo III es incompatible con la vida y se diagnostica al
nacimiento por la inclusin de todo el contenido de la fosa posterior
en un meningoencefalocele cervical alto (opcin 3).

CTO Medicina C/ Nez de Balboa, 115 28006 MADRID (Espaa) Tfno.: (91) 782 43 32 / Fax: (91) 782 43 27
E-mail: secretaria@ctomedicina.com; iberocto@ctomedicina.com WEB: www.ctomedicina.com; www.iberocto.com

Comentarios TEST

Se trata de un tema poco relevante en el examen MIR, del cual


debes conocer slo conceptos bsicos que se recogen en las siguientes preguntas.

NEUROCIRUGA

Preparacin Examen de Seleccin 05/06 1 Vuelta


Finalmente, el Chiari tipo IV es una hipoplasia del cerebelo (opcin 4), pero no hay herniacin de estructuras de fosa posterior por el
foramen magno.
Pregunta 42.- R: 5
Como ya sabes, se denomina espina bfida a una malformacin
raqudea consistente en un defecto de cierre de los arcos vertebrales
posteriores, a travs del cual pueden quedar expuestos al exterior o
visibles algunas estructuras neurales (espina bfida manifiesta) o no
(espina bfida oculta). En algunos casos pueden asociarse anomalas
del tejido nervioso subyacente, pero no es obligado.
La respuesta correcta es la 5. La raquisquisis es un defecto de
cierre completo de todo el tubo neural, generalmente incompatible
con la vida, en el que todas las estructuras neurales raqudeas quedan
expuestas al exterior. Puede acompaarse tambin de defecto de cierre del crneo, llamndose craneorraquisquisis, y en muchos casos
es motivo de aborto espontneo.
El resto de las enfermedades mencionadas en las opciones de esta
pregunta son formas de espina bfida oculta.
El seno drmico congnito (opcin 1) es una de las formas de
espina bfida oculta en la que hay un tracto fibroepitelial que pone en
comunicacin la piel con el tejido celular subcutneo, o en los casos
ms graves con el canal medular, debido a un defecto de cierre del
arco posterior vertebral. Ninguna estructura neural queda expuesta o
es visible desde el exterior, pero pueden verse un punto deprimido en
la piel (como un agujerito) siempre en la lnea media o muy cerca de
ella, y en ocasiones signos que se asocian con la espina bfida oculta
(fundamentalmente mechones de pelo, angiomas o lipomas alrededor del punto deprimido). Generalmente no tienen trascendencia
clnica a no ser que produzcan un anclaje medular o se infecten,
siendo una causa potencial de meningitis.
La diastematomielia (opcin 2) es una malformacin de la mdula
espinal que consiste en la presencia de dos hemimdulas, cada una
de ellas en el interior de un tubo dural independiente, separadas por
un septo osteocartilaginoso.
Finalmente, recuerda la mayor frecuencia del sndrome de regresin caudal (opcin 4), con agenesia del sacro, en los hijos de madres
diabticas.

Pregunta 42. Defectos de cierre del tubo neural.

Comentarios TEST

123456547589
76

12324356786
928324356786

623 6 8 88


 678

243567 4373
17686484 2286

1234562789
86 82 88 52862
926 62 82
9287 9

3 5
325
  43 8 3 

873
32  84 4 228 
2 334 2 62476 
8636 83786
8 67 64 
 686 67
!3282 437364
826586 476 623 8 382 664 
"83#23 32 84 $
6 642 83 $3 823 67
%&6 7 '(
&3 6 664 4 3 
2 33 3 38)2
4667 6323863764
%*8+ 36 386846(
!837 3282 4373%,
5 34323(
!3282 4373%3+
826586
6285836
-6 8 88 837 4373
%3  2)84 (

M exico A rgentina
C hile U ruguay

Seguimiento a distancia

Pregunta 43.- R: 4
El mielomeningocele es la forma ms frecuente de espina bfida
manifiesta, y consiste en un defecto de cierre del arco posterior vertebral a travs del cual se hernian las meninges y estructuras nerviosas
(races nerviosas de la cola de caballo y placa neural), por lo que
asocia generalmente disfuncin de estas estructuras.
De todas las formas de espina bfida es la ms importante para el
examen MIR. Debes recordar algunas cosas sobre esta entidad, recogidas en las opciones de esta pregunta:
Su localizacin ms frecuente es a nivel lumbar.
Es muy frecuente su asociacin con hidrocefalia (opcin 1), y en
estos casos requerir tratamiento mediante derivacin ventriculoperitoneal). Es frecuente tambin su aparicin en los pacientes con
Chiari tipo II.
Al igual que otros defectos de cierre del canal neural, se ha relacionado con el dficit de cido flico en la madre (opcin 3), por lo
que se recomienda la administracin de suplementos de esta vitamina al menos dos meses antes de iniciar la gestacin para prevenir
su aparicin. La profilaxis con cido flico ha sido objeto de pregunta MIR.
La opcin falsa es la 4, porque al estar expuestas las estructuras
neurales al exterior en el saco amnitico, se detectan niveles anormalmente elevados de alfafetoprotena en el lquido amnitico y
en el suero materno. Esta determinacin puede utilizarse para hacer diagnstico prenatal, aunque la ecografa es una tcnica
diagnstica ms segura y especfica (opcin 5).
El tratamiento de esta entidad es quirrgico (cierre del mielomeningocele) y debe hacerse precozmente. Puesto que la localizacin es a nivel lumbar, la posicin ptima para estos nios es en
decbito prono (opcin 2).
Pregunta 44.- R: 4
Nos presentan un caso clnico de una paciente de 15 aos de
edad, con cuadro clnico de fiebre y rigidez de nuca desde hace unas
horas, que en ausencia de ms datos nos debe hacer pensar en una
meningitis infecciosa.
El resto de la exploracin parece normal, con excepcin del hallazgo de una lesin angiomatosa y un mechn de pelo en la vecindad de
una pequea depresin en la piel de la lnea media lumbosacra.
El primer diagnstico en el que debemos pensar ante los hallazgos
descritos es que se trate de una meningitis bacteriana en un paciente
con un seno drmico congnito.
Como ya sabes, el seno drmico congnito es una malformacin
englobada dentro del grupo de las espinas bfidas ocultas, y consiste
en un tracto fibroso recubierto de epitelio que pone en comunicacin la piel con estructuras profundas (desde el tejido celular subcutneo a, en los casos ms graves, el interior del canal medular como
consecuencia de un defecto de cierre del arco posterior del tubo
neural). La nica anomala que podemos ver desde el exterior es la
presencia de esa pequea depresin cutnea que se describe en el
enunciado, que en ocasiones se acompaa de otros estigmas tpicos
de los pacientes con espina bfida oculta (mechones de pelo, angiomas
y lipomas como en el caso clnico que nos plantean). Sin embargo no
existe ninguna estructura neural expuesta al exterior, por lo que esta
malformacin se clasifica como una forma de espina bfida oculta.
La mayor parte de las formas de espina bfida oculta no tienen
trascendencia clnica, pero el seno drmico congnito tiene un riesgo
potencial de infeccin que se traducira en la aparicin de cuadros
de meningitis, por lo que se recomienda la extirpacin quirrgica
profilctica del tracto. En caso de que produzca infeccin primero
debe tratarse sta, y despus se operar.
Recuerda que la meningitis de Mollaret (opcin 1) es una forma
de meningitis asptica recidivante que se asocia con la rotura de tumores dermoides o epidermoides y liberacin del contenido tumoral
al espacio subaracnoideo. No es una meningitis infecciosa.
Pregunta 45.- R: 4
Nos presentan un caso clnico de dolor facial en una paciente de
60 aos. La paciente describe el dolor como muy intenso y de caractersticas lancinantes (como un calambre). El dolor aparece sbitamente y en ocasiones se desencadena al masticar o bostezar (con los

CTO Medicina C/ Nez de Balboa, 115 28006 MADRID (Espaa) Tfno.: (91) 782 43 32 / Fax: (91) 782 43 27
E-mail: secretaria@ctomedicina.com; iberocto@ctomedicina.com WEB: www.ctomedicina.com; www.iberocto.com

NC Pg. 21

movimientos de la boca). Aparece y desaparece sin previo aviso, los


episodios de dolor son de corta duracin, pero es muy recidivante.
Adems, la paciente informa sobre una distribucin del dolor muy
caracterstica en la regin perioral y la mejilla del lado izquierdo (pero
no del lado derecho, slo afecta a una hemicara).
El enunciado de la pregunta est describiendo un caso tpico de
neuralgia del trigmino, afectando al territorio de la segunda (mejilla)
y tercera ramas (regin perioral y mandibular) del nervio en el lado
izquierdo. Adems se describen muy bien las caractersticas del dolor:
lancinante, aparece como una corriente elctrica y con frecuencia al
hacer ciertas maniobras o tocar en determinado punto de la cara
(reas gatillo). No se describe que exista alteracin de la sensibilidad
en las ramas del trigmino afectadas, lo que tambin es tpico de esta
enfermedad (la sensibilidad suele ser normal).
No hay ninguna duda de que este paciente presenta una neuralgia
del trigmino, y el tratamiento de eleccin es la carbamacepina. La
respuesta correcta es la 4. Slo en los casos en los que falla el tratamiento mdico se recurre a tratamiento quirrgico por diversas tcnicas que est recogidas en el Manual CTO.
En la mayor parte de los pacientes con neuralgia del trigmino no
se encuentra una anomala estructural que cause el dolor. Se denominan entonces neuralgias esenciales.
Sin embargo, en un pequeo grupo pueden existir causas estructurales que motiven la compresin o la lesin del nervio; se habla entonces de neuralgias secundarias. La ms frecuente de ellas es que exista
una compresin de la raz del nervio por un vaso sanguneo (generalmente la arteria cerebelosa superior o una arteria trigeminal persistente),
pero hay otras causas (tumores, esclerosis mltiple, traumatismos, infartos
pontinos que afecten al ncleo del V par, etc.).

Pregunta 45. Clasificacin etiolgica de la neuralgia del trigmino.

12345678592
2 856
988 8 5
9 5


123456785

2 3 54859

8  8 5

12345678967
7 925 
492996  649
12928972 2297
12724462 546472 52 45264 8459

1272 4 5492 2992 669

124778 4529
92 2
929 2
25 47
222!99282  56 "
12#87 2! 4 874 $25 45789$28 545 4789"
129
78945296 47 579
12% 452 84
454 956 2! 
7428&
64
"
12 796 9
12#9896487
12% 452
25&
72
2'292!
475 
2227567(
(9 "

Pregunta 46.- R: 3
Esta pregunta hace referencia a una tcnica neuroquirrgica especfica, que no ha sido preguntada antes en el examen MIR, pero que
cuenta con un amplio campo actualmente, en expansin, y con unas
indicaciones que muy brevemente y de manera simplificada te comentaremos. Es ms correcto denominarla radiociruga estereotctica.
La radiociruga estereotctica, para explicarlo de manera muy simple, es una tcnica que permite aplicar una dosis muy alta de radiacin, generalmente en una nica sesin, de manera muy precisa y
focalizada sobre una lesin cerebral, minimizando la radiacin que
recibe el tejido cerebral sano perilesional. Para conseguir esa precisin se utiliza el marco de estereotaxia (un sistema que permite determinar con extraordinaria seguridad las coordenadas de cualquier
estructura o punto del cerebro). Esta excelente precisin en la distribucin de la radiacin, junto con la posibilidad de administrar el
tratamiento en una nica sesin, son las principales diferencias con la
radioterapia convencional.
No es necesario que conozcas muchas cosas ms sobre la radiociruga, pero debes tener algunas ideas sobre sus indicaciones. En general, y de forma muy simple, la radiociruga es til para tratar lesiones
muy bien circunscritas, de tamao igual o inferior a 3 centmetros.
Las principal indicacin de la radiociruga en el momento actual
es el tratamiento de malformaciones arteriovenosas (MAVs) con nido
Pg. 22 NC

M exico A rgentina
C hile U ruguay

NEUROCIRUGA

Preparacin Examen de Seleccin 05/06 1 Vuelta

de menos de 3 cm de dimetro (opcin 2), en reas en las que la


ciruga no es aconsejable (localizadas en zonas profundas del cerebro
o cerca de reas funcionales elocuentes del cerebro). Una limitacin
de la radiociruga es que, una vez administrada, la MAV tarda en
obliterarse un periodo de tiempo de entre 1 y 3 aos, durante el cual
persiste el riesgo de sangrado de la malformacin.
Entre los tumores, se acepta actualmente que la radiociruga puede
ser de utilidad en el tratamiento de determinadas metstasis,
meningiomas (sobre todo del seno cavernoso) y neurinomas del VIII
par en los que se descarte tratamiento quirrgico (opciones 1, 4 y 5).
En general, no est indicada para el tratamiento de tumores infiltrantes
como los gliomas.
En ningn caso se utiliza la radiociruga para el tratamiento de
aneurismas cerebrales (la respuesta falsa es la 3). Como ya sabes, el
tratamiento de eleccin de los aneurismas cerebrales es su exclusin
de la circulacin general mediante clipaje quirrgico o embolizacin
(tratamiento endovascular). La radiociruga no es una opcin en estos
pacientes en ningn caso.
Pregunta 47.- R: 3
Esta pregunta hace referencia al tratamiento de los abscesos cerebrales, que como sabes son colecciones purulentas intracerebrales
que la mayor parte de las veces tienen una etiologa bacteriana (recuerda que lo ms frecuente es que se trate de infecciones polimicrobianas, con flora mixta en la que predominan los grmenes aerobios).
Otro dato que debes recordar es que el sntoma ms frecuente de
presentacin es la cefalea, y que no siempre asocian fiebre.
Los abscesos cerebrales son colecciones de pus intracerebral. El
tratamiento ideal es el que combina la antibioterapia y la ciruga (si
esta es posible, porque se trate de un absceso nico en una localizacin apropiada).
Las opciones 3 y 5 se refieren a la utilizacin de antibiticos para el
tratamiento del absceso cerebral. Puesto que los abscesos cerebrales
suelen tener flora mixta con aerobios y anaerobios, la pauta antibitica
debe incluir frmacos que cubran ambos tipos de grmenes. La pauta
recogida en la respuesta 5 es la ms habitualmente utilizada, e incluye
cefotaxima (u otra cefalosporina de 3 generacin) y metronidazol.
Cuando se sospecha que puede haber estafilococos implicados (sobre
todo en los abscesos postquirrgicos) se asocia tambin vancomicina.
La opcin 3 es la falsa porque el diagnstico de absceso cerebral obliga
a mantener el tratamiento antibitico durante un mnimo de 4-6 semanas, independientemente de que el paciente sea intervenido o no.
Las opciones 1, 2 y 4 se refieren al papel de la ciruga.
Como sabes, la formacin del absceso se produce en varias etapas.
En la primera fase, conocida como cerebritis, se organiza la respuesta
inflamatoria alrededor del foco necrtico. En una segunda fase se
organiza una cpsula de colgeno y gliosis que limita el absceso. La
evacuacin quirrgica es ms eficaz (tanto por puncin como la ciruga abierta con reseccin del absceso y su cpsula) cuando el absceso
est encapsulado, y no est indicada en fases de cerebritis. Lgicamente, si el absceso es nico y est bien delimitado, el xito de la
ciruga es ms probable (opcin 2).
La respuesta 1 es cierta. Todos los pacientes con absceso cerebral
reciben tratamiento antibitico, y la mayora son operados. La ciruga
est indicada de eleccin en lesiones que producen clnica y signos
de hipertensin intracraneal (HTIC), efecto de masa en las pruebas de
imagen, o edema importante, sobre todo si se trata de lesiones nicas
y accesibles mediante ciruga.
Finalmente, la opcin 4 tambin es cierta. Los abscesos cerebrales
localizados en el cerebelo suelen producirse por contigidad, siendo
el foco primario ms frecuente una infeccin otolgica. Dado que el
cerebelo se localiza en el interior de la fosa posterior, que anatmicamente es un espacio ya reducido, la existencia de una coleccin
purulenta en el cerebelo incrementa rpidamente la presin en la
fosa posterior, comprimiendo las estructuras troncoenceflicas, por lo
que generalmente un absceso cerebral en la fosa posterior es una
urgencia quirrgica.
Pregunta 48.- R: 4
Esta pregunta aborda el tema de la ciruga funcional estereotctica
aplicada al tratamiento de los trastornos del movimiento. Es un tema

CTO Medicina C/ Nez de Balboa, 115 28006 MADRID (Espaa) Tfno.: (91) 782 43 32 / Fax: (91) 782 43 27
E-mail: secretaria@ctomedicina.com; iberocto@ctomedicina.com WEB: www.ctomedicina.com; www.iberocto.com

Comentarios TEST

Seguimiento a distancia

NEUROCIRUGA

Comentarios TEST

Preparacin Examen de Seleccin 05/06 1 Vuelta


muy especfico de la Neurociruga, que nunca han preguntado en el
examen MIR, pero que est en alza, motivo por el que queremos
aportarte algunas nociones.
Como sabes, la ciruga estereotctica utiliza un sistema de coordenadas para poder localizar con extraordinaria precisin cualquier
estructura o punto del cerebro, y dirigir los instrumentos quirrgicos a
esa estructura con mnimo margen de error. Con ayuda del marco de
estereotaxia podemos, por ejemplo, tomar una biopsia de una lesin
cerebral de muy pequeo tamao (con precisin prxima al milmetro). Del mismo modo podemos localizar y dirigir un electrodo a
cualquier ncleo profundo del cerebro, y esta es precisamente la
finalidad de la ciruga de los trastornos del movimiento.
Sabemos que los trastornos del movimiento se deben a alteraciones en el balance de los neurotransmisores en los ganglios de la base
(ncleo caudado, putamen, globo plido medial y lateral, ncleo
subtalmico, sustancia negra compacta, etc....). Se ha descubierto que
la lesin de algunos de estos ncleos puede controlar los trastornos
del movimiento que se asocian a estas enfermedades.
Existen dos tipos de tcnicas para el tratamiento de los trastornos
del movimiento. Las tcnicas ablativas realizan una lesin por radiofrecuencia que destruye las clulas y fibras nerviosas del ncleo elegido como diana. La estimulacin cerebral profunda lo que hace es
colocar un electrodo permanente en el ncleo y estimularlo de forma
continua a alta frecuencia; se ha visto que esa estimulacin continua
tiene el mismo efecto que la lesin del ncleo, pero es reversible si se
retira el electrodo.
Los ncleos diana de estas tcnicas dependen del tipo de trastorno del movimiento.
En el tratamiento de la enfermedad de Parkinson, la tcnica de
eleccin es la estimulacin crnica del ncleo subtalmico, porque
es la tcnica que consigue un mejor control de todos los sntomas
(opcin 1).
En algunos pacientes en los que el sntoma predominante de la
enfermedad de Parkinson es el temblor, o en los pacientes con temblor esencial, la diana de eleccin es el tlamo (talamotoma ventrolateral) como se recoge en la opcin 5.
En pacientes con distona generalizada, y en pacientes con enfermedad de Parkinson en los que existen discinesias invalidantes en
relacin con la administracin de L-dopa, se utiliza como diana el
globo plido medial (opciones 2 y 3).
La respuesta falsa es la 4. La parlisis supranuclear progresiva (enfermedad de Steele-Richardson-Olszewski) es una enfermedad tpica
de ancianos caracterizada por la presencia de signos extrapiramidales
en conjuncin con otros sntomas entre los que destacan cadas frecuentes, rigidez cervical con distona y una parlisis de la mirada
vertical (signo clnico ms caracterstico de la enfermedad), y no se
beneficia de ninguna de las tcnicas de neurociruga funcional.

Seguimiento a distancia

tante, no debemos olvidar que la Rx simple de la columna cervical es


imprescindible en todo paciente politraumatizado para descartar lesiones de la columna cervical (opcin 1). Tiene la ventaja sobre los
cortes axiales de la TC de que permite ver la alineacin de los cuerpos
vertebrales.
La RM, sin embargo, es superior a la TC en el estudio de ciertas
patologas. Precisamente porque la TC ve muy bien el hueso, en espacios pequeos rodeados de hueso se crean artefactos que impiden
ver con buena resolucin los tejidos blandos. Este es el motivo por el
que la RM es superior a la TAC en el estudio de lesiones de la mdula
espinal y de la fosa craneal posterior (cerebelo y tronco del encfalo), como dice la opcin 5.
Indudablemente la resolucin de las imgenes del cerebro que se
consigue con la RM es muy superior a la conseguida con la TC, por
ello, aunque para el diagnstico inicial de los tumores cerebrales
suele utilizarse la TC por su mayor disponibilidad, el estudio de la
morfologa del tumor, sus relaciones anatmicas, distribucin del edema perilesional, etc. se realiza mejor con RM, especialmente en los
tumores de lnea media que guardan relaciones muy importantes con
estructuras anatmicas cruciales como el tronco del encfalo, la
hipfisis y tallo hipotalmico, el quiasma ptico, el tercer ventrculo,
el tlamo e hipotlamo, la glndula pineal, etc. (opcin 4).
Pregunta 50.- R: 1
El sndrome del canal de Guyn es el nombre que recibe la compresin del nervio cubital (opcin 1) a su paso por el carpo en direccin a la palma de la mano. El nervio cubital queda atrapado en su
recorrido por este canal anatmico localizado en el borde cubital del
carpo, entre el hueso pisiforme y el gancho del hueso ganchoso. El
suelo del canal de Guyon es el retinculo de los msculos flexores, y
el techo son fibras procedentes del ligamento volar transverso del
carpo. Por el interior del canal de Guyon discurren la arteria cubital
(lateralmente) y el nervio cubital (medialmente).
La compresin del nervio cubital en el carpo ocasiona una alteracin sensitiva (dolor y/o parestesias o hipoestesia en el borde cubital
de la palma de la mano, 5 dedo y borde cubital del 4 dedo), pero
no se afecta el dorso de la mano porque la rama sensitiva del nervio
cubital para el dorso sale por encima del canal de Guyon (diferencia
con la compresin en el codo). El trastorno sensitivo puede acompaarse de alteraciones motoras que afectan fundamentalmente a los
msculos de la eminencia hipotenar (oponente, flexor y separador
del 5 dedo), pero tambin al aproximador del pulgar y a los msculos interseos, todos ellos msculos inervados por el nervio cubital.
El tratamiento consiste en la seccin quirrgica del ligamento volar
transverso del carpo por encima del nervio cubital para aumentar el
espacio en el interior del canal y liberar el nervio.

Pregunta 49.- R: 3
Han aparecido algunas preguntas en el examen MIR sobre la indicacin de las tcnicas de imagen aplicadas al estudio de las patologas
neurolgicas. Por este motivo merece la pena dedicar unas lneas a
aclarar en qu patologas se prefiere la tomografa computerizada craneal (TC), y cundo es mejor la resonancia magntica cerebral (RM).
La respuesta falsa es la 3. La RM es inferior a la TC para el estudio
de las lesiones seas en todos los casos, simplemente porque los rayos
X ven mejor el hueso. La TC craneal es superior para el estudio de
todas las lesiones seas (fracturas craneales y vertebrales) y las lesiones
calcificadas.
Otra ventaja de la TC sobre la RM es su mayor disponibilidad
(accesible en casi todos los hospitales) y rapidez en la exploracin,
que la hacen insustituible en la valoracin de las urgencias neuroquirrgicas (opcin 2). La TC craneal es la prueba diagnstica de eleccin en el estudio de los traumatismos craneoenceflicos, muy por
encima de la RM. Igualmente permite diagnosticar con mucha facilidad los problemas hemorrgicos (la sangre se ve blanca o hiperdensa
en la TC), mientras que en la RM la sangre se ve con apariencia distinta
segn las secuencias y el tiempo de evolucin.
Por tanto, podemos decir que la TC es superior a la RM para el
estudio del hueso, calcificaciones y urgencias neuroquirrgicas (sobre todo traumatismos craneoenceflicos y hemorragias). No obsM exico A rgentina
C hile U ruguay

CTO Medicina C/ Nez de Balboa, 115 28006 MADRID (Espaa) Tfno.: (91) 782 43 32 / Fax: (91) 782 43 27
E-mail: secretaria@ctomedicina.com; iberocto@ctomedicina.com WEB: www.ctomedicina.com; www.iberocto.com

NC Pg. 23

PEDIATRA

Preparacin Examen de Seleccin 05/06 1 Vuelta


1.

Seale la afirmacin correcta respecto al test de Apgar:


1)
2)
3)
4)
5)

2.

2)
3)
4)

Preguntas TEST

2.
3.
4.
5.
6.

Fiebre de sed.
Colagenosis.
Neoplsica.
Infecciosa.
Facticia.

3)

5)

4)
5)
8.

4)
5)

Recin nacido de cinco das de vida, EG 36 semanas y peso


2.500 g, es llevado al servicio de Urgencias porque, mientras
dorma, sus padres han notado que durante 5 segundos
dejaba de mover el trax, volviendo a respirar de forma muy
rpida posteriormente. No refieren cambios en la coloracin de la piel. Seale la respuesta FALSA:
1)
2)

3)

Es ms frecuente en RNPT.
Estamos ante un patrn de respiracin peridica, que
consiste en pausas apneicas de cinco a diez segundos,
seguidas de tandas de respiracin rpida durante diez o
quince segundos.
Lo ms frecuente es que se acompaen de cianosis y
cambios en la FC.
M exico A rgentina
C hile U ruguay

Administracin de surfactante endotraqueal.


Antibioterapia.
Administracin de oxgeno y ventilacin mecnica, si es
preciso.
Prostaglandinas.
Control del aporte de lquidos.

Un nio de 42 semanas de gestacin, arrugado, plido,


hipotnico, apneico y cubierto de lquido amnitico verdoso. Respecto al cuadro que puede padecer este paciente,
seale la opcin FALSA:
1)
2)
3)
4)
5)

10.

Normal.
Infiltrado de patrn reticulogranular con broncograma
areo.
Hiperinsuflacin y aumento de las marcas vasculares, sin
broncograma.
Patrn de esponja.
Borde de ambos pulmones colapsados.

En el tratamiento de la EMH se incluyen las siguientes


medidas, EXCEPTO:
1)
2)
3)

9.

Enfermedad de membrana hialina.


Taquipnea transitoria del recin nacido.
Sndrome de aspiracin meconial.
Persistencia de la circulacin fetal.
Sndrome de Wilson-Mikity.

RN de 35 semanas de edad gestacional que en las primeras


12 horas de vida, presenta taquipnea, quejido audible y
cianosis progresiva, refractaria a la administracin de oxgeno. A la auscultacin encontramos crepitantes, sobre todo en
bases pulmonares. En la gasometra aparece hipoxemia progresiva, aumento de la PCO2 y acidosis metablica. Qu
radiografa de trax esperara encontrar en este momento?:
1)
2)

El cefalohematoma es una hemorragia subperistica, y


est delimitado por las suturas craneales.
La clavcula es el hueso que se fractura con mayor
frecuencia durante el parto.
El llamado ndulo de Stroemayer suele requerir tratamiento quirrgico.
La rotura heptica no siempre precisa tratamiento quirrgico.
Las fracturas craneales ms frecuentes en el parto son las
lineales.

Generalmente persiste de forma intermitente hasta alcanzar las treinta y seis semanas de edad postconcepcional.
Puede reducirse el nmero de episodios apneicos mediante una transfusin de concentrado de hemates o
estimulacin fsica externa.

Ante un neonato a trmino que tras cesrea presenta


precozmente taquipnea y cianosis que desaparece con
pequeas cantidades de oxgeno, en el que la auscultacin
pulmonar es normal y en el que la radiografa muestra
marcas vasculares prominentes y lquido en las cisuras sin
broncograma areo, sospecharemos:
1)
2)
3)
4)
5)

7.

Seale la afirmacin FALSA respecto a las lesiones producidas por traumatismos obsttricos:
1)

5.

6.

Cul es la etiologa ms frecuente del sndrome febril en el


lactante?:
1)
2)
3)
4)
5)

4.

5)

RN pretrmino que, a las 6 h de vida, inicia cuadro compatible con un cuadro de distress respiratorio tipo I, y que a
la exploracin presenta quejido espiratorio a la auscultacin, ausencia de retraccin esternal, tiraje intercostal leve
con movimientos toracoabdominales sincrnicos y sin aleteo nasal. Su puntuacin en el Score de Silverman ser:
1)
2)
3)
4)
5)

3.

Se debe practicar nicamente a los RN patolgicos o


supuestamente patolgicos.
Entre los parmetros que se valoran se encuentra la
frecuencia respiratoria.
Se debe realizar a los 1-5-10 minutos de vida.
Un test de Apgar de 4 al minuto de vida implica un alto
riesgo de parlisis cerebral.
La palidez cutnea generalizada supone una puntuacin de 1 en el test de Apgar.

4)

Seguimiento a distancia

La primera maniobra en su reanimacin debe ser la


aspiracin de trquea bajo visin laringoscpica.
Es un patologa tpica de RNPT y RNT.
El E. coli es el germen ms frecuente de infeccin bacteriana en estos pacientes.
En la Rx de trax es tpico encontrar un patrn de
atrapamiento areo.
Esta patologa produce una llamativa hipertensin pulmonar.

Un nio a trmino recibe al nacer asistencia respiratoria


con bolsa y mscara con una presin mxima superior a

CTO Medicina C/ Nez de Balboa, 115 28006 MADRID (Espaa) Tfno.: (91) 782 43 32 / Fax: (91) 782 43 27
E-mail: secretaria@ctomedicina.com; iberocto@ctomedicina.com WEB: www.ctomedicina.com; www.iberocto.com

PD Pg. 1

20 cmH2O por un perodo de 3 minutos. Al llegar a planta,


presenta taquipnea, ronquidos y cianosis con desviacin
a la derecha del choque de la punta. El trax es asimtrico,
y a la auscultacin se evidencia hipoventilacin en el
hemitrax izquierdo. El diagnstico ms probable es:
1)
2)
3)
4)
5)
11.

2)
3)
4)
5)

12.

14.

Se consideran factores de riesgo para su desarrollo tener


distress respiratorio severo que precise largos perodos de
administracin de O2.
Las principales causas de muerte de estos pacientes son
la bronquiolitis necrotizante y fallo cardaco derecho.
El tratamiento incluye broncodilatadores, diurticos y
dexametasona.
La nefrolitiasis puede ser una complicacin de estos
pacientes, secundaria al uso de diurticos y a la alimentacin parenteral.
A pesar de que pueda retirarse la administracin de O2
antes de salir de la unidad de cuidados intensivos el
pronstico para la mayora de estos pacientes es malo,
presentando en la adolescencia obstruccin de las vas
respiratorias e hiperreactividad bronquial.

3)
4)
5)
15.

Eventracin diafragmtica.
Malformacin adenomatosa del pulmn.
Quiste por duplicacin del esfago.
Hernia diafragmtica.
Fstula traqueoesofgica.

Respecto a la retinopata del prematuro, seale la respuesta


FALSA:
1)
2)
3)

Pg. 2 PD

La administracin de oxgeno en altas concentraciones


es un factor de claro riesgo.
El tiempo de administracin tambin influye.
En el 90% de los afectados, la enfermedad progresa hacia
una fibroplasia retrolental.
M exico A rgentina
C hile U ruguay

Antibioterapia.
Supositorios de glicerina.
Enema de solucin hiperosmolar.
Estimulacin rectal.
Ciruga.

Seale la afirmacin FALSA de entre las siguientes respecto


a la ictericia fisiolgica:
1)
2)
3)
4)
5)

17.

Enema de solucin hiperosmolar.


Alimentacin enteral a dbito continuo a travs de una
sonda nasogstrica.
Actitud expectante y vigilancia cuidadosa, ante la posibilidad de perforacin intestinal.
Se debe suspender la alimentacin enteral y pautar
fluidoterapia y antibioterapia i.v.
La presencia de neumatosis intestinal aconseja la intervencin quirrgica urgente.

RN de 38 semanas de edad gestacional presenta distensin


abdominal y vmitos biliosos durante el primer da de vida.
En el examen fsico se palpan cordones duros que siguen el
marco clico. Una radiografa en bipedestacin muestra
una masa en pompa de jabn en cuadrante inferior derecho. El tratamiento inmediato ms apropiado es:
1)
2)
3)
4)
5)

16.

La vitamina C no parece tener relacin con ella.


Hay otros factores cooperantes con la administracin de
oxgeno para su desarrollo.

Un RN pretrmino de 1.800 g de peso, con antecedentes de


hipoxia perinatal, a los 10 das de vida presenta distensin
abdominal importante, vmitos y deposiciones hemorrgicas. En la Rx de abdomen se observa edema de asas intestinales, con un patrn en "miga de pan" y presencia de gas
en la pared intestinal. Cul sera la opcin teraputica ms
adecuada en este paciente?:
1)
2)

Un varn nacido a trmino de una hora de vida, est


ciantico y presenta quejido respiratorio. El examen fsico
revela dificultad respiratoria moderada, asimetra torcica
y abdomen excavado. Los ruidos cardacos se auscultan
sobre hemitrax derecho y aparece hipoventilacin en
hemitrax izquierdo. El diagnstico ms compatible con
estos hallazgos es:
1)
2)
3)
4)
5)

13.

4)
5)

Sepsis por estreptococo del grupo B.


Sndrome de corazn izquierdo hipoplsico.
Neumomediastino.
Neumotrax.
Fstula traqueoesofgica.

Un RN de 32 semanas, con antecedente de enfermedad de


membrana hialina sometido a ventilacin mecnica con
altas concentraciones de oxgeno, tiene actualmente un
mes de vida y sigue dependiendo del respirador para
mantener adecuadas saturaciones de oxgeno. En los ltimos das presenta adems oliguria con edemas en miembros inferiores y se palpa el hgado a 5 cm bajo el reborde
costal derecho. En la Rx de trax se observa una imagen de
pequeas zonas redondas y claras que alternan con otras
de mayor densidad. En relacin con la patologa que presenta este paciente, seale la respuesta FALSA:
1)

PEDIATRA

Preparacin Examen de Seleccin 05/06 1 Vuelta

Las cifras mximas de bilirrubina suelen alcanzarse a los


3-4 das.
La ictericia en los RNPT suele ser de inicio algo ms tardo
y ms prolongada que en los RNT.
Puede comenzar en las primeras 24 horas de vida en el
RNT.
Su duracin suele ser inferior a 10-15 das.
En los RNPT pueden alcanzarse cifras de hasta 14 mg/dl.

En relacin a la ictericia por lactancia materna o sndrome


de Arias es FALSO que:
1)
2)
3)
4)
5)

Su frecuencia es aproximadamente de 1 por cada 200


RN a trmino que reciben lactancia materna.
Es una ictericia que aparece a la mitad o final de la
primera semana, alcanzando su mximo en la tercera.
Se pueden alcanzar cifras de hasta 30 mg/dl, de ah el
riesgo de kernicterus en estos neonatos.
En la patogenia se implican varias sustancias presentes en
la leche materna como pregnanodiol, cidos grasos de
cadena larga y glucuronidasas.
Cuando se suspende la lactancia materna durante 2-4
das se produce una rpida disminucin de la bilirrubina
srica.

CTO Medicina C/ Nez de Balboa, 115 28006 MADRID (Espaa) Tfno.: (91) 782 43 32 / Fax: (91) 782 43 27
E-mail: secretaria@ctomedicina.com; iberocto@ctomedicina.com WEB: www.ctomedicina.com; www.iberocto.com

Preguntas TEST

Seguimiento a distancia

PEDIATRA

Preparacin Examen de Seleccin 05/06 1 Vuelta


18.

En relacin a la enfermedad hemoltica por isosensibilizacin ABO, seale la FALSA:


1)
2)
3)
4)
5)

19.

Un lactante de 1 semana es llevado a Urgencias en estado de


coma con hemorragias en la retina y palidez importante.
Entre sus antecedentes destaca que el parto tuvo lugar en el
domicilio, no present complicaciones posteriores y est
recibiendo lactancia materna. Seale lo FALSO respecto a
este cuadro:
1)
2)
3)
4)
5)

20.

2)
3)
4)
5)

Preguntas TEST

Hipoglucemia neonatal.
Hipocalcemia neonatal.
Policitemia.
Alteracin del sistema nervioso central.
Sndrome de abstinencia a opiceos.

Comparando los RN de madres adictas a herona y adictas


a metadona, es cierto que en los hijos de heroinmanas:
1)
2)
3)
4)

23.

24.

4)
5)

La asistencia prenatal es mejor.


Es mayor la incidencia de consumo de mltiples drogas
por la madre.
El peso medio de los neonatos al nacimiento es mayor.
La incidencia de convulsiones es menor.
M exico A rgentina
C hile U ruguay

2)
3)
4)
5)

Es la segunda causa ms frecuente de infeccin viral


congnita.
El 90% de los recin nacidos infectados estarn asintomticos al nacer.
La manifestacin ms tarda y caracterstica es una
hipoacusia de transmisin.
La afectacin ocular tpica es una retinitis pigmentosa.
Estos nios suelen desarrollar macrocefalia evolutivamente.

Cul de las siguientes manifestaciones NO forma parte de


la ttrada de Sabin?:
1)
2)
3)
4)
5)

28.

Observar la aparicin de los sntomas y luego tratar.


Extraer sangre de cordn para estudiar transaminasas.
Exanguinotransfusin inmediata, si la madre tiene el
antgeno HBe positivo.
Poner una dosis bimensual de gammaglobulina.
Iniciar vacunacin y gammaglobulina en paritorio.

Un RN de bajo peso para su edad gestacional presenta un


exantema petequial, hepatoesplenomegalia y coriorretinitis. En la TC craneal se observan calcificaciones periventriculares sin otros hallazgos. Con respecto al cuadro que
parece presentar este paciente, seale la verdadera:
1)

27.

Klebsiella.
E. coli y estreptococos del grupo A.
E. coli y estreptococos del grupo B.
Estafilococo.
Gardnerella vaginalis.

Qu hara usted ante un RN de madre con hepatitis B activa


en el momento del nacimiento?:
1)
2)
3)

26.

Megalencefalia.
Sndrome asfctico.
Hiperbilirrubinemia.
Sndrome de regresin caudal.
Cardiomiopata.

En la sepsis del RN, los grmenes ms frecuentes son:


1)
2)
3)
4)
5)

25.

El inicio de las manifestaciones clnicas es ms tardo.

NO es propio del RN hijo de madre diabtica:


1)
2)
3)
4)
5)

La anemia fisiolgica es ms intensa y ms precoz en los


RNT que en los RNPT.
El cuadro de anemia fisiolgica neonatal est en relacin
con la hemlisis de glbulos rojos y con un dficit de
eritropoyetina.
En los RNPT est indicada la administracin de suplementos de hierro a partir de los 2 meses de vida.
La anemia fisiolgica puede precisar transfusin en
algunos casos.
La betatalasemia no se manifestar antes de los 6 meses
de vida.

Seale el diagnstico que le parece ms probable para un


RN hijo de madre diabtica, que, tras un parto sin incidencias, present a las 12 horas de vida temblores, letargia y
rechazo de las tomas as como ligera acrocianosis. A los 2
das de vida presenta ictericia que requiere fototerapia:
1)
2)
3)
4)
5)

22.

Se debe a un dficit de los factores dependientes de


vitamina K.
Suele iniciarse a partir de las primeras 24 horas.
Es ms frecuente en hijos de madres que han tomado
fenitona o fenobarbital.
Las complicaciones hemorrgicas son menos frecuentes
en nios que toman LM que en los que reciben bibern.
Se trata con la administracin de vitamina K y/o plasma
fresco congelado.

Respecto a la anemia en el perodo neonatal, seale cul de


las siguientes afirmaciones es FALSA:
1)

21.

La madre suele ser O.


Los anticuerpos causantes suelen ser de la clase Ig G.
Pueden resultar afectos los productos del primer embarazo.
El antgeno ms frecuentemente implicado es A1.
La enfermedad se agrava en los embarazos siguientes, si
no hacemos profilaxis.

5)

Seguimiento a distancia

Coriorretinitis.
Cataratas.
Crisis convulsivas.
Hidrocefalia.
Calcificaciones cerebrales.

Un RN de 38 semanas de edad gestacional y 1.750 g,


presenta erupcin prpuro-petequial, hepato-esplenomegalia y se ausculta un soplo continuo spero en 2 espacio
intercostal izquierdo. En la Rx de trax se objetiva cardiomegalia con aumento de la vascularizacin pulmonar as
como imgenes lineales radiotransparentes con aumento
de densidad sea en la zona metafisaria del hmero sin
reaccin peristica. En relacin con el cuadro que presenta
este paciente, seale la respuesta FALSA:

CTO Medicina C/ Nez de Balboa, 115 28006 MADRID (Espaa) Tfno.: (91) 782 43 32 / Fax: (91) 782 43 27
E-mail: secretaria@ctomedicina.com; iberocto@ctomedicina.com WEB: www.ctomedicina.com; www.iberocto.com

PD Pg. 3

1)
2)
3)
4)
5)
29.

Hepatoesplenomegalia.
Rinitis.
Pnfigo sifiltico.
Osteocondritis con reaccin peristica.
Queratitis.

Infeccin por VIH.


Infeccin por CMV.
Infeccin por VHS tipo I.
Infeccin por VHS tipo II.
Exantema txico del RN.

2)
3)
4)
5)
34.

2)
3)
4)
5)

35.

La sonrisa referencial aparece ente el 1 y 2 mes.


El sostn ceflico se inicia a los tres meses.
La sedestacin estable se consigue a los 6-8 meses.
Emite sus primeros fonemas con 3-4 semanas.
Realiza frases cortas con 2 aos.

Nia de 6 aos que presenta desde hace 3 das fiebre de 38


C que no cede con antitrmicos, tos seca y facies congestiva.
Hoy acude por aparecer exantema maculopapuloso, rojo
intenso, no puntiforme, confluente, que no se blanquea a
la presin en la parte superior del trax, cara y parte
proximal de EESS. A la exploracin. T 40,3C, adenopatas
bilaterales en ngulo mandibular y lesiones blanquecinas

Pg. 4 PD

M exico A rgentina
C hile U ruguay

El signo ms caracterstico del perodo prodrmico son


las adenopatas.
El exantema suele aclararse al tercer da con mnima
descamacin.
La artritis afecta sobre todo a mujeres y nios preadolescentes, principalmente a grandes articulaciones.
El bazo puede estar aumentado de tamao.
Las complicaciones neurolgicas en forma de encefalitis
son mucho menos frecuentes que en un caso de sarampin.

Nos traen a Urgencias a un escolar de 7 aos de edad, que


estando totalmente bien, presenta fiebre de 40C, acompaada de vmitos, cefaleas, pulso rpido y algo dbil, con
sensacin de enfermedad grave. A la exploracin destacan
amgdalas hipertrficas con exudado blanco-amarillento, as
como enantema en paladar blando. Adems presenta adenitis satlite en ngulo mandibular izquierdo. A las 14 horas
aparece exantema puntiforme a la palpacin que respeta el
ngulo nasogeniano. A los 3-4 das se inicia descamacin
furfurcea. El tratamiento etiolgico de este proceso es:
1)
2)
3)
4)
5)

36.

El exantema comienza por la cara, tiene evolucin


descendente, y desaparece en el mismo orden en el que
apareci.
La neumona de clulas gigantes de Hecht es menos
frecuente que la neumona por sobreinfeccin bacteriana.
La gravedad de la enfermedad est directamente relacionada con la intensidad y confluencia del exantema.
El prurito suele ser intenso.
Entre las posibles complicaciones se encuentra la anergia cutnea y la reactivacin de una TBC preexistente.

Seale cul de los siguientes afirmaciones respecto a la


rubola es FALSA:
1)

Onfalocele.
Granuloma umbilical.
Quiste o persistencia del uraco.
Gastrosquisis.
Ombligo amnitico.

En relacin con el desarrollo psicomotor en el nio, seale


la respuesta FALSA:
1)
2)
3)
4)
5)

33.

1)

Un recin nacido de tres semanas de vida que desprendi


el cordn umbilical a los diecisis das y cuyo ombligo no ha
cicatrizado, presenta una pequea tumoracin sonrosada
con secrecin seromucosa con un pH de 7,4. Cul de los
siguientes diagnsticos le parece ms probable?:
1)
2)
3)
4)
5)

32.

sobre base eritematosa en mucosa subyugal. Respecto a la


enfermedad que padece esta paciente, seale la afirmacin
FALSA:

Un RN hijo de madre prostituta presenta en el 7 da de vida


un cuadro de fiebre con letargia y rechazo de las tomas as
como convulsiones. A la exploracin destaca hepatoesplenomegalia con fontanela anterior abombada y lesiones
vesiculares en calota craneal. Cul sera la causa ms
probable de este cuadro?:
1)
2)
3)
4)
5)

31.

Cuando la madre contrae la infeccin en el tercer


trimestre aumenta el riesgo de infeccin del feto, pero el
cuadro clnico es ms leve.
Si la infeccin se produce despus de la 12 semana de
gestacin, es poco probable la asociacin de una cardiopata congnita.
La presencia de IgM especfica anti-rubola refleja la
produccin en tero de anticuerpos por el feto.
El lactante puede permanecer con infeccin crnica
durante meses despus del nacimiento.
El virus puede aislarse en sangre.

Cul de las siguientes manifestaciones es ms frecuente en


la forma precoz de sfilis congnita?:
1)
2)
3)
4)
5)

30.

PEDIATRA

Preparacin Examen de Seleccin 05/06 1 Vuelta

Midecamicina por 5 das.


Tratamiento sintomtico.
Penicilina oral durante 10 das.
Penicilina benzatina en 3 dosis consecutivas.
Asociar corticoides al antibitico elegido.

Un nio de 4 aos acude al pediatra por aparicin de


lesiones papulosas rojas, algunas con vesculas blanquecinas no umbilicadas en tronco y mucosa oral, muy pruriginosas. Dos das antes presentaba cuadro catarral con fiebre
moderada. Respecto al cuadro que presenta este nio, todo
lo siguiente es cierto, EXCEPTO:
1)
2)
3)
4)
5)

Est causado por un poxvirus, el virus de la varicela zoster.


Las lesiones costrosas no contienen virus viables.
La complicacin ms frecuente es la sobreinfeccin de
las lesiones cutneas.
La neumona varicelosa es poco frecuente en la infancia.
La encefalitis postvaricela que se presenta con signos
cerebelosos tiene mejor pronstico que si lo hace con
signos cerebrales.

CTO Medicina C/ Nez de Balboa, 115 28006 MADRID (Espaa) Tfno.: (91) 782 43 32 / Fax: (91) 782 43 27
E-mail: secretaria@ctomedicina.com; iberocto@ctomedicina.com WEB: www.ctomedicina.com; www.iberocto.com

Preguntas TEST

Seguimiento a distancia

PEDIATRA

Preparacin Examen de Seleccin 05/06 1 Vuelta


37.

1)
2)
3)
4)
5)
38.

2)
3)
4)
5)

Preguntas TEST

Si apareciera una erupcin cutnea 24 horas despus de


comenzar la fiebre, sera probable el diagnstico de
rosola o exantema sbito.
En la rosola infantil, el diagnstico diferencial puede ser
difcil, puesto que en las primeras 36 horas cursa con
leucocitosis, con predominio de neutrfilos.
La fiebre podra corresponder a los prdromos de una
rubola.
Si la fiebre dura 3 das, desaparece luego rpidamente
y aparece una erupcin cutnea en ese momento, sera
probable el diagnstico de eritema infeccioso.
Es probable una escarlatina, pues es una enfermedad
propia de nios de 6 a 18 meses.

Jennifer tiene 16 meses de edad y acude a Urgencias por


presentar desde hace 2 das fiebre de 39C que su pediatra
atribuy a cuadro respiratorio. A la exploracin presenta
fiebre de 40C y contractura antilgica en flexin de extremidad inferior izquierda. Queda ingresada para estudio, y
4 das ms tarde persiste el cuadro febril, junto con edema
indurado en ambos pies, fisuras labiales, conjuntivitis no
purulenta y exantema polimorfo en tronco. El diagnstico
ms probable ser:
1)
2)
3)
4)
5)

40.

Es producida por el parvovirus B19.


El perodo de incubacin es de 7 das.
El perodo prodrmico se caracteriza por fiebre alta.
El exantema caracterstico aparece en tres etapas.
Como complicacin puede producirse una crisis aplsica grave.

41.

Un lactante de 7 meses presenta sbitamente fiebre de


40C. No existe ningn otro sntoma y la exploracin fsica
es normal, salvo el hallazgo de una coriza ligera y adenopatas cervicales posteriores. El nio parece encontrarse bastante bien. Cul de las siguientes afirmaciones describe
mejor los factores diagnsticos que deben considerarse en
este caso?:
1)

39.

1)
2)
3)
4)
5)

Nio de 7 aos presenta rash eritematoso en ambas mejillas,


tronco y zona proximal de extremidades, con tendencia a
aclararse en la zona central. Con respecto a esta enfermedad, seale la afirmacin FALSA:

4)
5)

Paciente varn de 5 aos de edad que acude por presentar


cuadro febril y odinofagia con exudado blanquecino en
ambas amgdalas que su pediatra trat con penicilina oral.
Dos das despus, presenta petequias y prpura palpables,
principalmente en miembros inferiores, as como artralgias en ambos tobillos que le impiden la marcha y dolor
abdominal de tipo clico. Los estudios complementarios
revelan 13 g/dl de Hb; leucocitos 10.500/mm3 con frmula
normal; plaquetas 485.000/mm3; tiempo de protrombina
de 95% y tiempo parcial de tromboplastina de 27 segundos (control 25 segundos). La causa ms probable de estos
hallazgos es:

2)
3)
4)

M exico A rgentina
C hile U ruguay

5)
44.

Hasta en el 80% hay aumento de transaminasas.


En la analtica aparece leucopenia.
Ttulos elevados de anticuerpos frente al virus de EpsteinBarr.
Existe un sndrome linfoproliferativo en varones que han
sufrido la infeccin por este virus.
La complicacin ms temida es la rotura esplnica.

Un beb de 3 meses, que recibe lactancia materna, presenta


una historia de accesos de tos con congestin facial, lagrimeo y acaba vomitando. Hace 2 semanas present cuadro
catarral de vas reas altas. En la Rx trax se aprecian breves
infiltrados perihiliares. Con respecto a este cuadro, seale
la respuesta que NO sea cierta:
1)

Rubola.
Mononucleosis infecciosa.
Prpura de Schnlein-Henoch.
Enfermedad de Kawasaki.
Eritema infeccioso.

Herpangina.
Escarlatina.
Sarampin.
Rubola.
Mononucleosis infecciosa.

Respecto a la mononucleosis infecciosa, es FALSO que:


1)
2)
3)

43.

Reaccin a la penicilina.
Prpura de Schnlein-Henoch.
Mononucleosis infecciosa.
Enfermedad de Kawasaki.
Prpura trombocitopnica idioptica.

Nio de 9 aos que acude a Urgencias por presentar


tumefaccin cervical bilateral sin fiebre ni otros sntomas.
En la exploracin fsica presenta: BEG, febrcula, palpacin
de varias adenopatas dolorosas de 1,5 cm, bilaterales en
regin submandibular, y un exudado blanco-amarillento
sobre amgdalas hipertrficas e hipermicas. En el frotis
nasofarngeo aparece cultivo positivo para estreptococo
beta hemoltico del grupo A. Se instaura tratamiento con
amoxicilina v.o., a pesar de lo cual la paciente regresa al 5
da por persistencia del cuadro, fatiga intensa, exantema
maculopapuloso en tronco y extremidades, y fiebre de
39C. Cul es el diagnstico ms probable?:
1)
2)
3)
4)
5)

42.

Seguimiento a distancia

Puede aparecer al principio de la vida por falta de


anticuerpos maternos.
La fase catarral precede al perodo paroxstico.
La tos paroxstica puede durar de 1 a 4 semanas.
El recuento de leucocitos suele elevarse en forma considerable con predominio de polimorfonucleares.
El perodo de incubacin oscila entre 5 y 10 das.

Un varn de 4 aos presenta fiebre, tumefaccin parotdea


y cefalea. Respecto al cuadro que presenta este paciente,
seale la opcin FALSA:
1)
2)
3)
4)
5)

La tumefaccin parotdea es bilateral, pero asincrnica.


La orquitis y la meningoencefalitis pueden aparecer en
ausencia de parotiditis.
La inmunidad transplacentaria probablemente explica
su ausencia durante los 6 primeros meses de vida.
La enfermedad est causada por un virus.
La sordera nerviosa unilateral es una complicacin
frecuente de esta enfermedad.

CTO Medicina C/ Nez de Balboa, 115 28006 MADRID (Espaa) Tfno.: (91) 782 43 32 / Fax: (91) 782 43 27
E-mail: secretaria@ctomedicina.com; iberocto@ctomedicina.com WEB: www.ctomedicina.com; www.iberocto.com

PD Pg. 5

45.

Con respecto al SIDA en la infancia, seala la opcin


correcta:

4)

1)

5)

2)
3)
4)
5)
46.

2)
3)
4)
5)

3)
4)
5)

Una IgG+ en un lactante de 10 meses indica infeccin


activa por el VIH.
La mayora de los nios infectados presenta hipergammaglobulinemia policlonal precoz.
La linfopenia e inversin del cociente CD4/CD8 son
menos llamativas que en el adulto.
No es til la determinacin de IgA contra el VIH, porque
no suele aparecer hasta el 6 mes de vida.
Los resultados positivos en la PCR han mostrado una
correlacin positiva con el aislamiento del virus en
cultivo.

2)
3)
4)
5)
51.

Es una vacuna de microorganismos atenuados.


La alergia al huevo ha dejado de ser una contraindicacin absoluta para su administracin.
Est contraindicada su utilizacin en embarazadas y
pacientes VIH+.
Su administracin en el calendario actual se realiza a los
15 meses y a los 4 aos.
En zonas de elevada morbimortalidad puede administrarse la primera dosis a los 6 - 12 meses, con una segunda
dosis de refuerzo despus del ao de vida.

2)
3)
4)
5)

4)
5)

3)
4)
5)

La vacuna tipo Sabin es de virus vivos atenuados.


La vacuna oral produce inmunidad local (IgA) y general
(IgG).
En el calendario actual se administra a los 2-4-6-18
meses y entre los 4 y 6 aos.
La vacuna tipo Salk est contraindicada en familiares de
inmunodeprimidos.
En caso de diarrea o vmitos sera conveniente retrasar
la vacunacin con virus atenuados.

Seale la afirmacin cierta respecto a la vacuna DTP:


1)
2)
3)

Pg. 6 PD

La vacuna de la tos ferina est contraindicada en mayores de 18 meses.


No puede administrarse conjuntamente con otras vacunas.
La vacuna DTPa (acelular) tiene mayor riesgo de efectos
secundarios que la DTP, por lo que no se utiliza de forma
rutinaria.
M exico A rgentina
C hile U ruguay

Realizar quimioprofilaxis con ceftriaxona durante 2 das.


No existe quimioprofilaxis eficaz.
Administracin de la vacuna antimeningoccica en
nios menores de 5 aos.
Administracin de rifampicina durante 2 das.
Administracin de rifampicina durante 4 das.

Cul sera la actitud a seguir ante un nio de 4 aos


asintomtico con Mantoux negativo, hijo de un tuberculoso
bacilfero?:
1)
2)
3)
4)
5)

54.

La vacuna de VHB est realizada por ingeniera gentica


con efectos adversos mnimos.
La pauta de administracin en caso de riesgo inmediato
como contacto con una jeringuilla sera 0, 1, 2 meses, y
al ao.
La vacuna VHB no est recomendada en mujeres embarazadas.
No est indicada su administracin en nios mayores de
7 aos.
Se puede administrar con la gammaglobulina anti VHB,
siempre que se administre en distinto punto.

Cul sera la actitud ms adecuada a seguir en los contactos


de un nio con meningitis meningoccica?:
1)
2)
3)

53.

Las nuevas vacunas conjugadas son aptas para la inmunizacin en lactantes mayores de 2 meses.
Es muy inmungena.
Sus efectos adversos son mnimos.
Es necesario aplicarla despus de los 5 aos.
En el calendario vacunal debera administrarse a los 2,
4, 6 y 18 meses.

Seale la afirmacin FALSA respecto a la vacuna del VHB:


1)

52.

A los 14 aos es preciso vacunar con Td (ttanos y


difteria adultos), y no son necesarios nuevas dosis de
recuerdo.
No est contraindicada en pacientes inmuno-deprimidos.

Con respecto a la vacuna contra el Haemophilus influenzae


tipo B, es FALSO que:
1)

Seale la afirmacin FALSA respecto a la vacuna de la polio:


1)
2)

49.

50.

Respecto a la vacuna triple vrica, seale la afirmacin


FALSA:
1)
2)

48.

El SIDA precoz es menos frecuente que el tardo y la


clnica predominante es infecciosa.
El SIDA tardo es ms frecuente y predomina la clnica
neurolgica.
La neumona por MAI es la infeccin oportunista ms
frecuente en el SIDA peditrico.
La manifestacin ms frecuente en el SIDA infantil es la
Hepatitis B.
La parotiditis es ms frecuente en el SIDA infantil que en
el del adulto.

Sobre el diagnstico de un nio VIH+, es FALSO que:


1)

47.

PEDIATRA

Preparacin Examen de Seleccin 05/06 1 Vuelta

Observacin y repetir Mantoux en 6 meses.


Realizar quimioprofilaxis primaria durante 2 meses con
INH y repetir el Mantoux posteriormente.
Realizar quimioprofilaxis primaria durante 6 meses con
INH.
Administrar tratamiento completo con tres frmacos
durante 6 meses.
Vacunacin con BCG.

Respecto a las diferencias entre la leche materna y la leche


de vaca, seale la afirmacin correcta:
1)
2)
3)

La leche de vaca tiene un contenido energtico 2 veces


superior al de la leche materna.
La leche de vaca contiene una cantidad de calcio
superior a la leche materna.
La leche materna contiene mayor cantidad de vitamina
K que la leche de vaca.

CTO Medicina C/ Nez de Balboa, 115 28006 MADRID (Espaa) Tfno.: (91) 782 43 32 / Fax: (91) 782 43 27
E-mail: secretaria@ctomedicina.com; iberocto@ctomedicina.com WEB: www.ctomedicina.com; www.iberocto.com

Preguntas TEST

Seguimiento a distancia

PEDIATRA

Preparacin Examen de Seleccin 05/06 1 Vuelta


4)
5)
55.

2)
3)
4)
5)

3)
4)
5)

Preguntas TEST

59.

No suele haber antecedentes familiares de atopia.


La protena comnmente implicada es la betalactoglobulina.
Las pruebas cutneas sern positivas.
El diagnstico definitivo se realiza mediante un test de
supresin-provocacin.
Suele ser un proceso transitorio.

2)
3)
4)
5)
60.

4)
5)

3)
4)
5)

Oligohidramnios materno.
La sonda utilizada en el paritorio no se puede introducir
en estmago.
Exceso de secreciones orales.
Cuando aparecen tos y cianosis con los intentos de
alimentacin.
Ante un abdomen lleno de aire, timpanizado.

1)
2)
3)
4)

Cambiar su frmula por un hidrolizado de protenas de


vaca.
Hacer endoscopia para descartar esofagitis.
Solicitar ecografa abdominal y/o trnsito digestivo.
Realizar manometra esofgica.

2)

M exico A rgentina
C hile U ruguay

3)
4)
5)
63.

Afecta al 1-2% de la poblacin.


Se localiza en leon a unos 75 cm de la vlvula ileocecal.
Se denomina hernia de Littr cuando el divertculo de
Meckel se aloja en una hernia inguinal indirecta.
Su manifestacin clnica ms frecuente es una hemorragia
gastrointestinal acompaada de dolor abdominal.
El mtodo diagnstico ms til es la gammagrafa con
Tc 99.

Un varn de 14 meses, previamente sano, de forma brusca


presenta crisis de llanto con encogimiento de piernas y
palidez, separados por perodos asintomticos de duracin
variable en los que permanece decado. Ha presentado dos
vmitos y una deposicin semilquida sanguinolenta. Exploracin fsica: afebril; palpacin de una masa alargada y mal
definida, localizada en hipocondrio derecho. En la Rx de
abdomen aparece silencio areo en hipocondrio derecho.
Seale cul de las siguientes afirmaciones es cierta respecto
al cuadro que presenta este paciente:
1)

Varn de 3 meses que consulta por vmitos desde hace 1


mes. Vomita despus de todas las tomas, sin fuerza; come
siempre con buen apetito y presenta adecuado desarrollo
ponderoestatural, sin prdida de peso. Realiza dos deposiciones diarias con heces de consistencia, aspecto y volumen
normal. Entre sus antecedentes destaca estar alimentado
con frmula artificial desde el mes de vida. Su actitud sera:

Estenosis hipertrfica de ploro.


Enfermedad de Hirschprung.
Divertculo de Meckel.
Atresia duodenal.
Invaginacin intestinal.

Con respecto al divertculo de Meckel, es FALSO que:


1)
2)
3)

62.

El signo radiolgico frecuente es el llamado "signo de la


cuerda".
Es ms frecuente en nias.
Presenta alcalosis metablica hiperclormica con hipopotasemia.
La prueba diagnstica de eleccin es la Rx de abdomen.
El tratamiento consiste en la pilorotoma submucosa de
Ramstedt.

RN con sndrome de Down presenta vmitos biliosos a las


36 horas de vida. A la exploracin destaca abdomen excavado. El diagnstico ms probable es:
1)
2)
3)
4)
5)

61.

Indicar a la madre medidas posturales y usar espesante


de la leche.

Seale la respuesta verdadera respecto a la estenosis hipertrfica de ploro:


1)

Debemos sospechar una atresia de esfago en todos los


siguientes casos, EXCEPTO:
1)
2)

58.

El agente causal ms frecuente en la infancia es el


rotavirus.
Suele ser un proceso autolimitado, que cede en 3-10
das.
Puede causar deshidratacin isotnica con acidosis.
Es frecuente que tienda a la cronificacin.
El diagnstico puede hacerse mediante deteccin rpida de antgeno de rotavirus en heces.

Lactante de 3 meses de edad, alimentado al pecho hasta hace


1 semana en que, debido a una hipogalactia, se decide
suplementar con una frmula artificial. Hoy acude a Urgencias por presentar anorexia, vmitos, prdida de peso as
como exantema urticarial en tronco. Entre sus antecedentes
destaca RNT, nacido por cesrea y haber iniciado alimentacin oral a las 4 horas de vida con frmula artificial. Con
respecto a la patologa actual, es FALSO que:
1)
2)

57.

5)

Varn de 13 meses que desde hace 2 das presenta fiebre de


38 C y sntomas catarrales. Hoy acude por presentar
vmitos no biliosos y 6 deposiciones en las ltimas 2 horas,
lquidas, no ftidas, sin moco ni pus ni sangre. Respecto al
cuadro que presenta este paciente, seale cul de las
siguientes afirmaciones es FALSA:
1)

56.

La leche de vaca es ms rica en hidratos de carbono que


la leche materna.
La leche materna tiene una relacin casena/seroprotenas de 70/30.

Seguimiento a distancia

Debe retrasarse el tratamiento durante 4-6 horas debido


a la alta incidencia de reduccin espontnea.
La recidiva es menos frecuente despus de la reduccin
hidrosttica que tras la reduccin quirrgica.
Si hay signos de obstruccin intestinal de 48 horas de
evolucin, es preferible la reduccin quirrgica.
Una invaginacin asociada a divertculo de Meckel
suele reducirse satisfactoriamente con enema opaco.
La mortalidad es muy elevada, aunque la reduccin
tenga lugar en las primeras 24 horas.

Nio de 4 meses es llevado a consulta por estreimiento


crnico desde el nacimiento. Entre sus antecedentes personales destacan tapn meconial al nacimiento que se resolvi
con enemas. Alimentacin materna exclusiva. Talla y peso

CTO Medicina C/ Nez de Balboa, 115 28006 MADRID (Espaa) Tfno.: (91) 782 43 32 / Fax: (91) 782 43 27
E-mail: secretaria@ctomedicina.com; iberocto@ctomedicina.com WEB: www.ctomedicina.com; www.iberocto.com

PD Pg. 7

PEDIATRA

Preparacin Examen de Seleccin 05/06 1 Vuelta

<P3. Cul de los siguientes datos NO apoyara el diagnostico?:


1)
2)
3)
4)
5)
64.

2)
3)
4)
5)
65.

1)
2)
3)
4)
5)
68.

69.

Aumento del amonio.


Ictericia con hepatomegalia.
Hipoglucemia.
Convulsiones.
Elevacin de las enzimas hepticas.

5)

Se puede asociar a poliesplenia.


El tratamiento definitivo es la hepatoportoenterostoma.
En la biopsia es posible hallar clulas gigantes.
Es infrecuente la aparicin de varios casos en una misma
familia.
En el sondaje duodenal se suele observar ausencia de
bilis.

Paciente de 5 meses que acude por presentar, desde hace


2 das, hasta 10 deposiciones al da, lquidas sin moco ni
sangre, as como dolor medio abdominal sin relacin temporal. Entre sus antecedentes destaca diarrea sanguinolenta hace 1 semana. En la exploracin destaca: afebril, buen
estado general, distensin abdominal con abundantes rui-

Pg. 8 PD

M exico A rgentina
C hile U ruguay

Laringotraquetis aguda.
Laringitis espasmdica.
Epiglotitis aguda.
Difteria.
Traquetis bacteriana.

Varn de 5 aos de edad que, estando previamente bien,


despierta por la noche con fiebre de 39,8C, voz apagada,
intenso babeo, estridor inspiratorio con escasa tos. A la
exploracin destaca mal estado general, estridor inspiratorio e intenso tiraje supraesternal y subcostal, as como
tendencia a la hiperextensin del cuello. Se realiza radiografa lateral de faringe, observando ocupacin de espacio
supragltico. En el hemograma destaca una leucocitosis
con desviacin izquierda. Qu medida teraputica NO
estara aconsejada en este caso?:
1)
2)
3)
4)
5)

71.

Traquetis bacteriana.
Laringitis supragltica.
Laringitis aguda.
Laringitis estridulosa.
Epiglotitis.

Ante un nio de 2 aos de edad que, estando previamente


bien, se despierta bruscamente por la noche con tos perruna, estridor y dificultad respiratoria, pulso acelerado y
afebril, pensaremos en:
1)
2)
3)
4)
5)

70.

Dficit de sacarasa-isomaltasa.
Dficit aislado de isomaltasa.
Dficit de lactasa.
Dficit de enteroquinasa.
Gastroenteritis por rotavirus.

Nia de 2 aos acude a urgencias por cuadro de disnea,


estridor y tos perruna. En la exploracin destaca rinorrea
intensa, amgdalas hipertrficas con exudado puntiforme.
Fiebre de 38.5. A la auscultacin pulmonar se aprecia
hipoventilacin bilateral y estridor inspiratorio. El diagnstico ms probable ser:
1)
2)
3)
4)
5)

Neonato varn, de tres semanas de vida, acude a su consulta


por ictericia con coluria e hipocolia. A la exploracin se
palpa hepatomegalia. En la gammagrafia con HIDA, la captacin es normal, pero la excrecin es nula. Sobre la entidad
que sospecha, seale la opcin FALSA:
1)
2)
3)
4)

67.

Existe una predisposicin gentica, relacionada con los


HLA B8, DR7, DR3 y DQ2.
Hay una mayor prevalencia de la enfermedad en nios
con dficit selectivo de IgA o diabetes mellitus.
El perodo ms comn de presentacin es entre los 6
meses y los 2 aos de edad.
Los anticuerpos antiendomisio Ig A tienen una alta
especificidad y sensibilidad.
El trigo, arroz, cebada y centeno deben ser excluidos de
la dieta para toda la vida.

Paciente de 6 aos de edad que hace una semana present


cuadro compatible con varicela, acude hoy por presentar
vmitos incoercibles junto con clnica neurolgica de obnubilacin, hiperventilacin e hiperreflexia. Seale cul de
las siguientes manifestaciones NO es compatible con este
cuadro:
1)
2)
3)
4)
5)

66.

Biopsia rectal patolgica.


Heces acintadas.
Ampolla sin heces en el tacto rectal.
Relajacin del esfnter anal interno.
Permanencia del bario en colon durante ms de 24
horas al realizar un trnsito digestivo.

Paciente de 12 meses que, de forma progresiva, inicia un


cuadro de inapetencia, mal carcter, retraso del crecimiento ponderoestatural, as como diarrea abundante, ftida y
de aspecto grasiento. Exploracin fsica: afebril, regular
estado general, palidez cutaneomucosa, prdida de masa
muscular proximal y abdomen distendido. Seale lo FALSO
respecto a la enfermedad celaca:
1)

dos hidroareos y eritema en regin del paal. En el examen


macroscpico de heces se detecta presencia de azcares
reductores. Este cuadro es:

Aplicar oxgeno e intentar obtener una va area artificial


en quirfano.
Iniciar tratamiento antibitico con ceftriaxona.
Aplicar adrenalina racmica y corticoides.
Usar las medidas generales para descender la hipertermia que sufre el paciente.
Mantener tranquilo al paciente.

Lactante varn de 3 meses de edad que desde hace dos das


presenta cuadro de rinorrea serosa con estornudos y tos
seca acompaado de rechazo de las tomas. Hoy acude a
Urgencias por cuadro de dificultad respiratoria con taquipnea y tos sibilante paroxstica. Exploracin fsica:
37,7C, regular estado general, aciantico, con aleteo
nasal y tiraje sub e intercostal. Auscultacin pulmonar:
sibilancias espiratorias difusas con espiracin alargada. Se
realiza hemograma con 3,5 millones de hemates; Hb de 10

CTO Medicina C/ Nez de Balboa, 115 28006 MADRID (Espaa) Tfno.: (91) 782 43 32 / Fax: (91) 782 43 27
E-mail: secretaria@ctomedicina.com; iberocto@ctomedicina.com WEB: www.ctomedicina.com; www.iberocto.com

Preguntas TEST

Seguimiento a distancia

PEDIATRA

Preparacin Examen de Seleccin 05/06 1 Vuelta


g/dl; Hto de 32%; 7.000 leucocitos/mm3 con 50% neutrfilos, 35% linfocitos y 5% monocitos. En la Rx de trax
destaca hiperinsuflacin pulmonar. Seale la FALSA respecto a la evolucin clnica de esta enfermedad:
1)
2)
3)
4)
5)
72.

Paciente de 2 aos de edad consulta por estancamiento


ponderoestatural (<P3). Entre los antecedentes personales
destacan en el ltimo ao episodios repetidos de bronquiolitis con regular recuperacin posterior, persistiendo una
tos crnica. Adems presenta diarrea intermitente sin acompaarse de fiebre ni vmitos. Cul sera el diagnstico ms
probable?:
1)
2)
3)
4)
5)

73.

2)
3)
4)
5)

Preguntas TEST

76.

3)
4)
5)
77.

Enfermedad celaca.
Tuberculosis.
Asma.
Fibrosis qustica.
Dficit de alfa 1 antitripsina.

La incidencia es inversamente proporcional a la edad


gestacional.
La tasa de esterilidad es similar a la de la poblacin general
cuando se trata de una criptorquidia unilateral.
En los casos no tratados o tratados ms all de la pubertad,
existe riesgo de desarrollar gonadoblastoma.
Se debe realizar un test de HCG en los casos bilaterales
para descartar una anorquia.
El tratamiento hormonal no sustituye al quirrgico.

4)
5)

Tratamiento con antitrmicos y ver evolucin.


Realizar puncin lumbar.
Diagnstico de infeccin de orina y administrar antibiticos v.o.
Realizar puncin suprapbica e iniciar antibiticos.
Descartar reflujo gastroesofgico.

2)
3)
4)
5)

2)
3)
4)
5)

1)
2)
3)

Puede sospecharse intratero al ver dilatacin pielocalicial en la ecografa obsttrica.


Suele dar lugar a infecciones urinarias de repeticin.
La indicacin quirrgica parte del nmero anual de
recidivas de infeccin urinaria.
M exico A rgentina
C hile U ruguay

80.

Es la cardiopata congnita ms frecuente en lactantes y


nios.
Se suele auscultar un soplo pansistlico desde el nacimiento.
La Rx de trax puede mostrar signos de pltora pulmonar.
Puede precisar profilaxis antimicrobiana contra la endocarditis.
En la mayora de los casos se produce un cierre espontneo del defecto.

La persistencia del ductus arterioso se asocia a todo lo


siguiente, EXCEPTO a:
1)
2)
3)
4)
5)

Es FALSO respecto al reflujo vesicoureteral en el nio:

En el Eco-Doppler se observara disminucin del flujo


sanguneo en ese testculo.
El tratamiento consiste en antibiticos, antiinflamatorios
y tras la fase aguda, ciruga.
Es la causa ms frecuente de escroto agudo en los nios
menores de 6 aos.
En mayores de 13 aos se debe hacer diagnstico
diferencial con una epididimitis.
En el periodo neonatal generalmente tiene mal pronstico.

Seale la opcin FALSA al hablar de las CIVs:


1)

79.

Es la causa ms frecuente de IRA en los nios pequeos.


Existe anemia hemoltica microangioptica y trombopenia dentro del cuadro clnico.
La trombosis bilateral de las venas renales puede provocar un cuadro similar.
El empleo de corticoides supone el tratamiento de
eleccin.
La aparicin de recidivas es muy poco frecuente.

Varn de 5 aos presenta de forma sbita dolor en teste


derecho, sin antecedente de traumatismo previo. Exploracin: afebril, teste tumefacto, doloroso a la exploracin con
ausencia del reflejo cremastrico. Seale la afirmacin
FALSA:
1)

78.

Puede ser til hacer profilaxis antibitica si se han


repetido varias infecciones urinarias.
Si el reflujo es masivo, puede llevar a IRC en la infancia
y puede requerir ciruga.

Varn de 2 aos en cuya historia clnica destaca haber


padecido un proceso diarreico en los das previos. En la
exploracin destaca palidez, estupor y hematuria. Ha presentado adems una convulsin focal. Respecto al cuadro
que usted sospecha, seale cul de las siguientes opciones
es FALSA:
1)
2)

Lactante de 18 meses consulta por cuadro de vmitos y


febrcula de 48 horas de evolucin con progresiva prdida
del apetito. En la exploracin no se objetiva foco infeccioso.
Hemograma con leucocitosis y desviacin a la izquierda.
Sistemtico de orina: leucocitos y nitritos positivos. Cul
sera la actitud ms adecuada?:
1)
2)
3)

75.

5)

Respecto a la criptorquidia, seale cul de las siguientes


opciones NO es cierta:
1)

74.

La fase ms crtica son las primeras 48-72 horas desde el


comienzo de la tos y la disnea.
En menores de un mes puede cursar como apnea con
escasez de clnica respiratoria.
Despus de la fase crtica mejoran muy rpidamente.
La ribavirina podra usarse en lactantes con cardiopatas
congnitas o displasia broncopulmonar.
Las complicaciones bacterianas como la bronconeumona u OMA son frecuentes.

4)

Seguimiento a distancia

Pulsos perifricos saltones.


Mayor frecuencia en prematuros.
Soplo continuo en el rea pulmonar.
Puede ser efectivo el tratamiento con indometacina.
Las prostaglandinas son el mejor tratamiento mdico
para conseguir su cierre.

Ante un recin nacido de dos das con cianosis, taquipnea


sin soplo a la auscultacin y que, en la radiografa de trax,
muestra ligera cardiomegalia, con un pedculo cardaco

CTO Medicina C/ Nez de Balboa, 115 28006 MADRID (Espaa) Tfno.: (91) 782 43 32 / Fax: (91) 782 43 27
E-mail: secretaria@ctomedicina.com; iberocto@ctomedicina.com WEB: www.ctomedicina.com; www.iberocto.com

PD Pg. 9

81.

estrecho y aumento del flujo sanguneo pulmonar, el diagnstico ms probable es:

1)
2)

1)
2)
3)
4)
5)

3)

3)
4)
5)

2)
3)
4)
5)

86.

Desplazamiento inferior de la vlvula tricspide anormal.


En el electrocardiograma puede aparecer un patrn de
Wolff-Parkinson-White.
Soplo sistlico y diastlico.
Aurcula derecha pequea y ventrculo derecho muy
hipertrofiado.
Valva tricuspdea anterior agrandada y abombada.

El tipo ms frecuente es la coartacin de localizacin


yuxtaductal.
La mayor parte de los casos se muestran asintomticos.
La tensin arterial en los miembros inferiores es mayor
que en los superiores.
En ocasiones, la tensin arterial en el brazo derecho es
mayor que en el izquierdo.
En la radiografa de trax, con el tiempo, se visualizan
escotaduras en los bordes inferiores de las costillas.

87.

TGA.
Fallot.
DAP.
Sndrome de Eisenmenger.
Anomala de Ebstein.

3)
4)
5)

El agujero oval persistente no es una forma de CIA.


La CIA tipo ostium secundum produce un segundo ruido
ampliamente desdoblado y que se modifica con la
respiracin.
Es frecuente que asocie retraso de la conduccin ventricular derecha.
La endocarditis infecciosa es extremadamente rara, por
lo que no es necesario realizar profilaxis.
El canal A-V comn suele evolucionar a ICC y requiere
un tratamiento quirrgico precoz.

2)
3)
4)
5)

Recin nacida de 3 semanas de vida, que presenta valores de


TSH de 75 mcgr/ml ( 0,3 5 ) en el screening neonatal de
hipotiroidismo. Con respecto a esta situacin, es FALSO:

Pg. 10 PD

M exico A rgentina
C hile U ruguay

Si fuera varn, la causa ms probable de pubertad


precoz central sera un hamartoma hipotalmico.
La aparicin de los caracteres sexuales secundarios, sin
activacin gonadal previa, permite diagnosticar una
pubertad precoz perifrica.
En mujeres, la causa ms frecuente de pubertad precoz
central son las lesiones del SNC.
Si fuera un varn con tamao testicular inferior a 4 cc,
habra que pensar, entre otras causas, en un hepatoblastoma.
Si fuera una nia con lesiones cutneas hiperpigmentadas y lesiones seas, habra que pensar en un sndrome
de McCune-Albright.

La estatura de un nio de 4 aos est por debajo del P3. Su


velocidad de crecimiento es normal. La edad sea es de 2
aos. El padre present un retraso en la pubertad, aunque
actualmente su talla est dentro de los lmites normales. Este
proceso se denomina:
1)
2)
3)
4)
5)

89.

Estenosis pilrica.
Sndrome adrenogenital congnito pierde sal.
Hipotiroidismo congnito.
Panhipopituitarismo.
Hiperaldosteronismo.

Paciente que inicia desarrollo de los caracteres sexuales


secundarios a una edad inferior a la considerada normal.
Con respecto a esta entidad, es FALSO:
1)

88.

Su causa ms frecuente es la disgenesia tiroidea.


Situaciones de dismorfognesis pueden asociarse a sordera neurosensorial.
Un tratamiento precoz supone un mejor pronstico
neurolgico.
Los signos clnicos del hipotiroidismo estn ya presentes
inmediatamente al nacimiento.
En situaciones de mal control pueden acabar desencadenando una pubertad precoz.

Un nio de 7 das es ingresado para evaluacin de cuadro


de vmitos y deshidratacin. La exploracin fsica es normal. La concentracin de Na y K en plasma es 120 mEq/l y
9 mEq/L respectivamente. El diagnstico ms probable ante
este cuadro es:
1)
2)
3)
4)
5)

La comunicacin interauricular es la cardiopata congnita


ms frecuente a partir de los 15 aos. Sus distintos tipos se
caracterizan por lo siguiente, EXCEPTO:
1)
2)

85.

5)

Ante un recin nacido ciantico con pulsos normales, soplo


sistlico en borde esternal izquierdo, radiografa que muestra un tamao cardaco normal con elevacin de la punta
cardaca y disminucin de la vascularizacin pulmonar, y
un electrocardiograma con signos de hipertrofia del ventrculo derecho, pensara en:
1)
2)
3)
4)
5)

84.

4)

Con respecto a la coartacin de aorta, seale la FALSA:


1)

83.

Tetraloga de Fallot.
Coartacin de aorta.
Anomala de Ebstein.
Trasposicin simple de las grandes arterias.
Atresia tricuspdea.

Los nios con enfermedad de Ebstein presentan todo lo


siguiente, EXCEPTO:
1)
2)

82.

PEDIATRA

Preparacin Examen de Seleccin 05/06 1 Vuelta

Hipopituitarismo primario.
Hipopituitarismo secundario.
Retraso constitucional del crecimiento.
Talla baja gentica.
Sndrome de Laron.

Varn de 4 aos, y con antecedentes personales de haber


nacido con retraso de crecimiento intrauterino, que actualmente, y en el contexto de un cuadro diarreico, presenta
valores de glucemia plasmtica de 30 mg/dl, con intensa
reaccin a cuerpos cetsicos en orina. La respuesta correcta es:
1)
2)

Por la reaccin de cuerpos cetnicos, el diagnstico ms


probable es un hiperinsulinismo.
El ayuno prolongado puede desencadenar esta situacin.

CTO Medicina C/ Nez de Balboa, 115 28006 MADRID (Espaa) Tfno.: (91) 782 43 32 / Fax: (91) 782 43 27
E-mail: secretaria@ctomedicina.com; iberocto@ctomedicina.com WEB: www.ctomedicina.com; www.iberocto.com

Preguntas TEST

Seguimiento a distancia

PEDIATRA

Preparacin Examen de Seleccin 05/06 1 Vuelta


3)
4)
5)
90.

3)
4)
5)

2)
3)
4)
5)

Preguntas TEST

97.

Determinacin del nivel madurativo.


EEG.
Calcemia.
Glucemia.
Examen visual.

Se trata de una epilepsia generalizada secundaria, que


suele comenzar a los 4 - 8 meses de edad.
Suele clasificarse en idiopticos, criptognicos y secundarios.
En el EEG aparece un patrn de ondas lentas hipervoltadas, bilaterales y asincrnicas.
El tratamiento de eleccin es ACTH o prednisona.
Los casos idiopticos tienen peor pronstico que los
secundarios.

Administracin de bolo de glucosa.


Administracin de antitrmicos y diacepam.
Administracin de fenitona.
Realizar puncin lumbar.
Administracin de piridoxina.

4)
5)

Seale la opcin FALSA respecto al sndrome de LennoxGastaut:


1)
2)
3)
4)
5)

Suele asociar varios tipos de crisis, entre ellos, tnicas,


atnicas y ausencias atpicas.
Son frecuentes los episodios de status epilptico.
Las crisis responden bien al tratamiento farmacolgico.
En la mayora de los casos es un proceso secundario.
Suele asociar retraso psicomotor.
M exico A rgentina
C hile U ruguay

Es el tumor extracraneal ms frecuente en la infancia.


La edad ms frecuente al diagnstico son los 2 aos.
La presentacin con un sndrome de mioclonus- opsoclonus ensombrece el pronstico.
Las catecolaminas en orina estarn elevadas.
Se asocia con delecin del cromosoma 1 y anomalas
del 17.

Seale la opcin FALSA respecto al tumor de Wilms:


1)
2)
3)
4)
5)

100.

Tumores del SNC.


Osteosarcoma.
Leucemias.
Tumor de Wilms.
Neuroblastoma.

Nia de 2 aos presenta movimientos mioclnicos de


extremidades y sacudidas desordenadas de los ojos. En la
exploracin se palpa masa abdominal en lnea media y
flanco derecho. Se le realiza una TC abdominal, evidencindose una masa de consistencia mixta y calcificaciones en su
interior. Seale la afirmacin FALSA:
1)
2)
3)

99.

Su pico de incidencia est entre los 5 y 8 aos de edad.


Suelen ser tumores slidos con escasa cavitacin.
Hay clnica de hipertensin intracraneal.
Es el tumor de fosa posterior ms frecuente en la niez.
La supervivencia a los 5 aos es del 90%.

Cul es el tipo de cncer ms frecuente en la niez?:


1)
2)
3)
4)
5)

98.

Crisis de ausencia.
Epilepsia mioclnica juvenil.
Epilepsia rolndica.
Sd. de West.
Sd. de Lennox- Gastaut.

Todo lo siguiente es cierto al hablar del astrocitoma del


cerebelo, EXCEPTO:
1)
2)
3)
4)
5)

Llega a Urgencias un nio de 18 meses con fiebre de 39, que


ha presentado en su domicilio un episodio tnico-clnico
generalizado de 3 minutos de duracin, con posterior
somnolencia. Lo correcto a su llegada a Urgencias sera:
1)
2)
3)
4)
5)

94.

96.

Seale la respuesta FALSA, de entre las siguientes, con


respecto a los espasmos infantiles:
1)

93.

Los RN con convulsiones suelen tener mal pronstico.


El tratamiento consiste en medidas de sostn y fenobarbital o diacepam.
Las crisis tnico-clnicas generalizadas son raras durante
el primer mes de vida.
La clnica es variable y el EEG puede ser la nica forma
para su diagnstico.
La causa ms frecuente de crisis en el perodo neonatal
es la encefalopata hipxico-isqumica.

La forma de epilepsia mas frecuente en la infancia es:


1)
2)
3)
4)
5)

Un varn de 7 aos tiene mal rendimiento escolar. Su maestro


comunica que a veces parece preocupado, presenta episodios de parpadeo y, ocasionalmente, tarda en responder o sus
respuestas son incorrectas. El examen fsico es normal. El
estudio que ms ayuda al diagnstico es:
1)
2)
3)
4)
5)

92.

95.

Respecto a las convulsiones neonatales, seale la afirmacin FALSA:


1)
2)

91.

Los valores de alanina en plasma estn elevados.


En la RM pancretica se demostrar la existencia de una
lesin focal.
Este cuadro empeora con la edad.

Seguimiento a distancia

Se asocia a delecin del cromosoma 11, ya sea en las


clulas tumorales o en todas las clulas del organismo.
Se asocia a aniridia y anomalas genitourinarias.
La HTA es un hallazgo frecuente.
El diagnstico definitivo nos lo da la biopsia.
Suele tratarse de una masa abdominal palpable asintomtica, con o sin metstasis pulmonares.

Respecto al retinoblastoma, seale la respuesta FALSA:


1)
2)
3)
4)
5)

Las metstasis o recidivas pueden asociarse a elevaciones


de AFP y CEA.
Las formas bilaterales suelen diagnosticarse a una edad
ms precoz que las unilaterales.
Si el tumor es bilateral debe procederse siempre a la
enucleacin.
La supervivencia global es de aproximadamente un
90%, sobre todo en las formas intraoculares.
Rara vez existen metstasis en el momento del diagnstico.

CTO Medicina C/ Nez de Balboa, 115 28006 MADRID (Espaa) Tfno.: (91) 782 43 32 / Fax: (91) 782 43 27
E-mail: secretaria@ctomedicina.com; iberocto@ctomedicina.com WEB: www.ctomedicina.com; www.iberocto.com

PD Pg. 11

PEDIATRA

Preparacin Examen de Seleccin 05/06 1 Vuelta


Pregunta 1.- R: 3
El test de Apgar es un test consensuado para documentar el estado
del RN en momentos puntuales. Lo ms frecuente es que en un RN
sano se mida al minuto de vida y a los 5 minutos, pero si a los 5
minutos la puntuacin es inferior a 7 se contina midiendo a los 1015 y 20 minutos. Se debe realizar a cualquier RN.
Es importante conocer los parmetros que se puntan y su valoracin.
Este test no sirve para valorar el riesgo de mortalidad perinatal ni la
probabilidad de dao neurolgico.

Seguimiento a distancia

cir ictericia precoz prolongada, sin otra repercusin clnica. Recordar


para el MIR el cuadro clnico tpico de la hemorragia suprarrenal:
nio grande, parto de nalgas, que en las primeras horas de vida presenta clnica de hipotensin, sangrado y shock.
Respuesta 5: las fracturas craneales lineales generalmente no precisan tratamiento quirrgico, salvo que se traten de fracturas deprimidas
que produzcan lesiones en el parnquima.

Pregunta 1. Test de Apgar.


3DUiPHWURV

(VIXHU]R
UHVSLUDWRULR

$XVHQWH

'pELO

/ODQWR

)&

$XVHQWH

OSP

!OSP

7RQRPXVFXODU

+LSRWRQtD

/LJHUDIOH[LyQ

0RYLPLHQWRVDFWLYRV

,UULWDELOLGDGUHIOHMD

1R

0XHFDV

7RV

&RORU

3DOLGH]

$FURFLDQRVLV

5RVDGR

Pregunta 2.- R: 1
Para valorar la dificultad respiratoria en los RN se emplea el score
de Silverman.

Comentarios TEST

Pregunta 2. Test de Silverman

9$/25$&,1

'LVRFLDFLyQ
WyUD[
DEGRPHQ

1RUPDO

7yUD[ILMRVHPXHYH
HODEGRPHQ

5HVSLUDFLyQHQ
EDODQFHR

7LUDMH

$XVHQWH

,QWHUFRVWDO

,QWHUFRVWDOVXSUDH
LQIUDHVWHUQDO

5HWUDFFLyQ
[LIRLGHD

$XVHQWH

'LVFUHWD

,QWHQVD

$OHWHRQDVDO

$XVHQWH

'LVFUHWR

4XHMLGR
UHVSLUDWRULR

$XVHQWH

,QWHQVR
6HR\HFRQIRQHQGR 6HR\HDGLVWDQFLD
6LQIRQHQGR

Pregunta 3.- R: 4
La causa ms frecuente de sndrome febril en el lactante es infecciosa y fundamentalmente vrica.
Pregunta 4.- R: 3
La respuesta 1 es cierta: es importante para el MIR tener claro las
diferencias entre caput (edema de partes blandas, no limitado por
suturas) y cefalohematoma (hematoma entre hueso y periostio, limitada por suturas).
La respuesta 2 es cierta: el traumatismo obsttrico ms frecuente es
la fractura de clavcula. A la exploracin lo ms llamativo es la crepitacin y el reflejo de Moro asimtrico.
La respuesta 3 es falsa: el llamado ndulo de Stromayer es un
hematoma palpable a nivel del ECM. La mayora de las veces se
reabsorbe espontneamente, pero en ocasiones se puede fibrosar
acortando el msculo. El tratamiento se basa en la fisioterapia y muy
rara vez requiere ciruga.
La respuesta 4 es cierta: la vscera que con mayor frecuencia se
lesiona es el hgado, pero la mayora de las veces son pequeos hematomas subcapsulares limitados que secundariamente pueden produM exico A rgentina
C hile U ruguay

Pregunta 4.

Cefalohematoma y caput succedaneum.

Pregunta 5.- R: 3
Estamos ante un cuadro de respiracin peridica del RN. Lo ms
caracterstico son pequeos episodios de apnea que NO se acompaan de cianosis ni de bradicardia. Esto ltimo es el dato ms importante para diferenciarlo de las pausas de apnea idiopticas.

CTO Medicina C/ Nez de Balboa, 115 28006 MADRID (Espaa) Tfno.: (91) 782 43 32 / Fax: (91) 782 43 27
E-mail: secretaria@ctomedicina.com; iberocto@ctomedicina.com WEB: www.ctomedicina.com; www.iberocto.com

PD Pg. 1

PEDIATRA

Preparacin Examen de Seleccin 05/06 1 Vuelta

Generalmente no precisa tratamiento, a diferencia de las apneas


idiopticas, que suelen requerir la administracin de cafena o teofilina y el uso de CPAP.
Ambas se solucionan conforme el nio va creciendo y alcanza las
36 semanas de edad postconcepcional.

Pregunta 5. Diagnstico diferencial de apneas en el neonato.

$SQHDLGLRSiWLFD
(GDG

5137

5137\517

$SQHDVVHJ
&OtQLFD

5HVSLUDFLyQSHULyGLFD

&LDQRVLVEUDGLFDUGLD
$XPHQWDGXUDQWHHO
VXHxR5(0

'HVDSDUHFHDODV
3URQyVWLFR VHPSRVWFRQFHSFLRQDO
1RULHVJRGHPXHUWH
V~ELWD

$SQHDVVHJVHJXLGDV
GHUHVSLUDFLRQHVUiSLGDV
GXUDQWHVHJ
12FLDQRVLVQLEUDGLFDUGLD
$XPHQWDQGXUDQWHODV
IDVHVGHOVXHxR
'HVDSDUHFHDODVVHP
SRVWFRQFHSFLRQDO
1RULHVJRGHPXHUWH
V~ELWD

Pregunta 6.- R: 2
El cuadro clnico se refiere a una taquipnea transitoria del RN, Sd
de Avery, pulmn hmedo o SDR tipo II.
La presentacin tpica es un RN nacido por cesrea o por parto
vaginal rpido, lo que supone que el trax no ha estado sometido a la
presin positiva que exprime de lquido los alveolos del pulmn. La
clnica se inicia en las primeras horas de vida, que cede y mejora con
medidas poco agresivas (oxgeno en incubadora). Suelen alcanzar la
resolucin clnica en un plazo mximo de 3 das.
Lo que establece el diagnstico, aparte de la evolucin clnica, es
la Rx de trax, donde se aprecia aumento de marcas vasculares, lquido en las cisuras, SIN broncograma areo. Esto ltimo es imprescindible para distinguirlo de la Rx de la EMH (R1).
R3: El SAM es tpico de RN postrmino, con antecedentes de estrs
en el momento del parto.
R4: La PCF suele cursar con clnica marcada de cianosis que no
responde a la administracin de oxgeno. No hay una Rx de trax
caracterstica.
R5: el Sd de Wilson-Mikity es a efectos clnicos semejante al trmino de DBP.
Pregunta 7.- R: 2
Estamos ante un cuadro tpico de enfermedad de membranas hialinas: RNPT con dificultad respiratoria intensa que aparece en las primeras horas de vida, que no mejora tras la administracin de oxgeno.
En la gasometra es llamativa la importante hipoxia. Para el diagnstico
es necesario realizar una Rx de trax, y en ocasiones, la medicin del
cociente lecitina/ esfingomielina en secreciones bronquiales, que debe
ser inferior a 2.
No existe una imagen radiolgica patognomnica. Lo ms tpico es
encontrar un infiltrado retculo granular con broncograma areo. Pero
esto mismo nos lo podemos encontrar en una neumona, por lo que
a la hora de instaurar tratamiento debemos cubrir esta posibilidad.
La R3 corresponde al pulmn hmedo.
La R4: El patrn de esponja en la Rx es tpico de la DBP.
La R5: corresponde a un neumotrax bilateral.
Pregunta 8.- R: 4
El tratamiento de la EMH consiste en:
Monitorizacin estrecha, control de lquidos.
Intentar mantener una aceptable oxigenacin (pO2 50-70 mmHg).
Si no se consigue con oxgeno indirecto, ser necesario intubar al
paciente e iniciar ventilacin mecnica.
Administracin de surfactante intratraqueal en las primeras 24 horas de vida, pudiendo recibir hasta 4 dosis segn evolucin. Esta
medida se ha visto que mejora la clnica de la EMH, pero que no
disminuye los casos de DBP.
Pg. 2 PD

M exico A rgentina
C hile U ruguay

Administracin de antibiticos: ampicilina ms gentamicina, porque la clnica y la Rx pueden ser semejantes en el caso de una
sepsis con neumona.
La respuesta incorrecta es la 4. Las prostaglandinas en los neonatos
estn indicadas en caso de cardiopatas ductus dependientes, en las
que es IMPRESCINDIBLE mantener el ductus abierto.
Pregunta 9.- R: 2
El caso clnico es el de un nio que ha sufrido un SAM. Lo tpico es
que se produzcan en RN postrmino, con un estrs en el momento
del parto, que estimula el peristaltismo intestinal y se produce la eliminacin de meconio intratero.
Al nacimiento, el meconio se encuentra en las vas areas altas; si
no se elimina a tiempo, este pasar al pulmn con las primeras respiraciones, produciendo un cuadro de alveolitis con riesgo de sobreinfeccin bacteriana, especialmente por E. coli, atelectasias y atrapamiento areo con riesgo de neumotrax y neumomediastino.
En los casos ms graves se asocia con una alta mortalidad debido a
la hipertensin pulmonar que desarrollan.
El tratamiento consiste en ventilacin mecnica, antibiticos y
medidas para disminuir la hipertensin pulmonar: alcalinizar, tolazolina y oxido ntrico inhalado. Algunos nios se pueden beneficiar de
la ECMO (oxigenacin por membrana extracorprea).
Pregunta 10.- R: 4
Estamos ante un nio con antecedentes de haber estado sometido
a altas presiones respiratorias para ventilarles durante la reanimacin,
producindole una hiperinsuflacin pulmonar. Esto es un factor de
riesgo claro para el desarrollo de neumotrax. Otras causas que pueden favorecerlo es patologa pulmonar que disminuya la compliance:
EMH, SAM, hipoplasia pulmonar,...
La clnica es tpica: disminucin del murmullo vesicular con
hiperresonancia. El diagnstico lo da la Rx de trax donde se aprecia
el pulmn colapsado.
El tratamiento, en caso de que el nio estuviera asintomtico, sera
administracin de oxgeno al 100%. Si produjera clnica o estuviera a
tensin, habra que evacuarlo mediante puncin.
Entre las opciones del caso clnico, ninguna plantea realmente
problemas:
R1: la sepsis en un RN tiene una clnica y analtica tpicas, y generalmente la afectacin pulmonar es bilateral.
R2: en el Sd del corazn izquierdo hipoplsico, lo tpico es un RN
con distrs severo, auscultacin pulmonar normal y cianosis intensa.
R3: el neumomediastino suele dar tonos cardacos apagados y
auscultacin pulmonar normal.
R5: en las fstulas T-E, la clnica es dificultad respiratoria intensa al
iniciar la alimentacin por episodios de aspiracin masiva.
Pregunta 11.- R: 5
Estamos ante un RN con displasia broncopulmonar. La definicin
ya aparece en el caso clnico: necesidad de oxgeno para mantener
adecuadas saturaciones. Dentro de los factores de riesgo para este
cuadro figuran: la toxicidad del oxgeno en altas concentraciones,
inmadurez pulmonar y barotrauma.
El diagnstico se establece por la historia clnica y la Rx de trax,
donde es tpico encontrar el llamado patrn de esponja (aparecen
reas ms claras que alternan con otras de mayor densidad).
El tratamiento se basa en el uso de diurticos y broncodilatadores;
el uso de corticoides actualmente est muy debatido.
R5: la mayora de los nios suelen tener un curso favorable, alcanzando la normalidad de la funcin pulmonar hacia los 2 aos de
vida. Hay un pequeo porcentaje que evolucionan hacia un cuadro
de hipertensin pulmonar persistente. Las dos causas ms frecuentes
de muerte en estos nios son la ICC derecha y la bronquiolitis necrotizante.
Pregunta 12.- R: 4
El diagnostico ms compatible es la hernia diafragmtica. Los dos
datos claves para el diagnstico son:

CTO Medicina C/ Nez de Balboa, 115 28006 MADRID (Espaa) Tfno.: (91) 782 43 32 / Fax: (91) 782 43 27
E-mail: secretaria@ctomedicina.com; iberocto@ctomedicina.com WEB: www.ctomedicina.com; www.iberocto.com

Comentarios TEST

Seguimiento a distancia

PEDIATRA

Preparacin Examen de Seleccin 05/06 1 Vuelta


Dificultad respiratoria inmediata.
Abdomen excavado.
El diagnstico definitivo lo obtendremos mediante la Rx de trax
donde se aprecia en el hemitrax izquierdo imgenes de asas intestinales con desplazamiento del mediastino hacia el lado contralateral.
Lo ms decisivo en el tratamiento de este nio va a ser la estabilizacin de la funcin respiratoria, ya que en las mayora de las ocasiones
se asocia con una severa hipoplasia pulmonar. Una vez que el paciente est estabilizado respiratoriamente, se realizar la intervencin quirrgica consistente en descender las asas intestinales y poner un parche en el hemidiafragma.

Comentarios TEST

Pregunta 13.- R: 3
Dentro de los factores de riesgo para el desarrollo de retinopata en
los grandes prematuros (peso < 1500 g) figuran: el oxgeno en altas
concentraciones, variaciones en la TA. La vitamina E parece tener un
factor protector (R4) por su papel antioxidante.
En las UCIs neonatales se ha establecido de forma rutinaria estudios oftalmolgicos en todos los nios menores de 1500 grs. Si se
aprecian zonas de proliferacin de vasos con riesgo de desprendimiento de retina, se realizar laserterapia o crioterapia.
El pronstico para la mayora de los casos es bueno (R3 falsa). Las
secuelas suelen ser pequeos defectos de la refraccin: miopa o
astigmatismo, o en ocasiones nistagmus. La displasia retrolental es una
complicacin rara y poco frecuente, pero muy grave, puesto que
puede dejar al nio ciego.
Pregunta 14.- R: 4
Estamos ante un RN con un cuadro de enterocolitis necrotizante
(NEC). La presentacin tpica es en un RNPT que a la semana inicia
cuadro de vmitos, distensin abdominal y deposiciones con sangre.
El primer diagnstico que nos debemos plantear es una NEC.
Dentro de los factores de riesgo figuran, aparte de la prematuridad,
la administracin precoz y en altas concentraciones de leche y factores infecciosos.
La prueba diagnstica indicada sera una Rx de abdomen donde
se apreciara el dato ms caracterstico, la neumatosis intestinal.
El tratamiento se debe instaurar ante la mnima sospecha, puesto
que de eso va a depender el pronstico.
Si el nio no tiene signos de perforacin intestinal, se dejar a dieta
absoluta durante semanas, con antibioterapia. Si en algn momento
de la evolucin se produjera una perforacin intestinal o sepsis refractaria a tratamiento antibitico, sera necesario realizar tratamiento
quirrgico.
Las complicaciones a largo plazo son la estenosis intestinales y el
sndrome de intestino corto.
Pregunta 15.- R: 3
El cuadro clnico se refiere a un leo meconial. La forma de presentacin ms habitual en los neonatos es como un cuadro de obstruccin intestinal. El dato que apoyara este diagnostico sera palpar unos
cordones duros que siguen el marco clico, puesto que el lugar donde con mayor frecuencia se impacta el meconio es la zona de leon
distal y coln proximal.
Una vez establecido el diagnstico es siempre obligatorio descartar una fibrosis qustica (un 15% debutan como leo meconial)
con la realizacin de un tripsingeno srico, que estar aumentado.
El tratamiento, si el nio no tiene signos de perforacin intestinal,
se realiza mediante la administracin de enemas hiperosmolares. Si
con esta medida no se soluciona el problema, sera necesario recurrir
a la ciruga, realizando una reseccin de la zona de impactacin y
anastomosis termino-terminal.
Pregunta 16.- R: 3
Respecto a las ictericias neonatales, es importante tener claro dos
aspectos.
Las causas ms frecuentes segn el momento de aparicin aparecen reflejadas en la siguiente tabla:

M exico A rgentina
C hile U ruguay

Seguimiento a distancia

Pregunta 16. Causas de ictericia neonatal.


1as 24 horas

Hemlisis.
Infecciones: sepsis, TORCH.
Fisiolgica.
Infecciones: sepsis, TORCH.
Anemias hemolticas.
Sepsis.
TORCH.
Obstruccin intestinal.
Lactancia materna.
Galactosemia, hipotiroidismo, lactancia materna,
metabolopatas, ictericia obstructiva, Gilbert,
Crigler-Najjar...

2-3er da
4-7 da

>1 mes

Caractersticas de la ictericia fisiolgica vs no fisiolgica:


- Inicio en las primeras 24 horas: SIEMPRE patolgico.
- Predominio de bilirrubina directa: SIEMPRE patolgico.
Otros datos de la no fisiolgica:
- Incremento mayor de 5 mg/24 horas.
- Bilirrubina en sangre de cordn mayor de 3 mg/dl.
- RNT > 12 y RNPT > 14.
- Duracin mayor de 14 das.
Pregunta 17.- R: 3
La ictericia por lactancia materna (LM) es la causa ms frecuente de
ictericia tarda. Su frecuencia es de aproximadamente 1/200 (R1).
Se debe a que en la leche materna existen unas sustancias
(pregnanodiol, cidos grasos de cadena larga) que inhiben la glucoronil
transferasa, dando como resultado un aumento de la bilirrubina a
expensas de fraccin indirecta (R4).
En ocasiones, cuando existen dudas diagnsticas, se puede interrumpir la LM transitoriamente durante unos das, observndose un claro
descenso en la bilirrubina, pero sta no es indicacin de suspender definitivamente la LM, puesto que no se han descrito casos de kernicterus
(R3).
Pregunta 18.- R: 5
Para el MIR es importante saberse bien las diferencias entre la
incompatibilidad Rh y ABO:

Pregunta 18. Diferencias entre incompatibilidad Rh y AB0.

5K

$%

0DGUH5K +LMR5K 
1HFHVDULDLQPXQL]DFLyQSUHYLD
/DLQFRPSDWLELOLGDGGHJUXSR
SURWHJHIUHQWHDOD
LQFRPSDWLELOLGDG5K
6HDJUDYDHQORVVLJXLHQWHV
HPEDUD]RV

0DGUH+LMR$
3XHGHSURGXFLUVHHQHOSULPHU
HPEDUD]R
1RVHDJUDYDFRQORVVLJXLHQWHV

,FWHULFLD
$QHPLD
+LGURSVIHWDOHQFDVRVJUDYHV

,FWHULFLD
$QHPLD
1RKLGURSV

3UHQDWDO&,DODPDGUHGXUDQWH
7\7
3RVWQDWDO&' 

3UHQDWDOQRWLHQHQXWLOLGDG
3RVWDQDWDO&' GpELO&, FRQ
HVIHURFLWRVLV



7UDWDPLHQWRIRWRWHUDSLD
H[DQJXLQRWUDQVIXVLyQ
3UHYHQFLyQ*DPPDHQ7\HQODV
SULPHUDVKRUDVWUDVHOSDUWRVLOD
PDGUHWLHQHXQWHVWGH&,QHJDWLYR



,GHP
1RWLHQH

CTO Medicina C/ Nez de Balboa, 115 28006 MADRID (Espaa) Tfno.: (91) 782 43 32 / Fax: (91) 782 43 27
E-mail: secretaria@ctomedicina.com; iberocto@ctomedicina.com WEB: www.ctomedicina.com; www.iberocto.com

PD Pg. 3

PEDIATRA

Preparacin Examen de Seleccin 05/06 1 Vuelta

Seguimiento a distancia

Pregunta 19.- R: 4
El caso clnico hace referencia a una enfermedad hemorrgica del
RN. La causa es un dficit de vitamina K, debido a que durante el
embarazo el paso transplacentario de vitamina es escaso, la LM es
pobre en vitamina K (R4) y en el intestino no hay bacterias que sinteticen vitamina. Todas estas circunstancias obligan a administrar vitamina K (1 mg im ) a todos los RN en las primeras 24 horas de vida. En
algunas ocasiones va a ser necesario ms de una dosis.
La clnica consiste en sangrado a distintos niveles: umbilical, digestivo, nasal,...
El tratamiento consiste en la administracin de nuevas dosis de
vitamina K o plasma fresco congelado.
Es importante recordar que los hijos de madres que durante el
embarazo han tomado fenitona o fenobarbital tienen ms riesgo de
padecer este trastorno.
Pregunta 20.- R: 1
La llamada anemia fisiolgica se debe a un dficit transitorio de
EPO, que unido a la hemlisis de glbulos rojos durante los primeros
meses de vida, dejan al nio anmico.
Las cifras ms bajas se producen a los 2 meses en los RNPT y a los
3 en los RNT (R1).
El tratamiento consiste en suplementos de hierro a partir de los 2
meses en los RNPT, ya que antes los depsitos estn llenos por la
hemlisis de los glbulos rojos, y en casos seleccionados sera necesario la trasfusin de concentrados de hemates.
En los neonatos no se manifiesta la betatalasemia, porque la cade-

na beta de la hemoglobina se empieza a sintetizar a partir de los 6


meses.
Pregunta 21.- R: 3
Por la clnica de temblores, letargia y rechazo de las tomas, podra
corresponder a cualquiera de las opciones. Pero los datos de acrocianosis e ictericia a los 2 das hablan a favor de la policitemia (el hijo de
madre diabtica tiene riesgo de tener policitemia).
Se define policitemia como Hto venoso mayor o igual a 65%. El
Hto capilar es por definicin un 5-20% mayor que el venoso.
Las consecuencias clnicas derivan de la hiperviscosidad de la sangre: distrs respiratorio, riesgo de NEC, trombosis de la vena renal,
disminucin de flujo a nivel del SNC,... y tambin pueden presentar
hipoglucemia (el sustrato metablico de los glbulos rojos es la glucosa; al haber muchos GR, consumen mucha glucosa).
El tratamiento consiste en la exanguinotrasfusin parcial, que se
realiza extrayendo un volumen de sangre y reponindolo con SSF,
para que disminuya la viscosidad.
Pregunta 22.- R: 4
Recordamos brevemente las diferencias entre los hijos de madre
adictas a herona y las adictas a metadona (ver tabla).
Estos nios estn sometidos a un estrs crnico intratero, por lo
que es menos frecuente la hiperbilirrubinemia y el desarrollo de
EMH.
Durante la reanimacin neonatal estos nios no deben recibir
naloxona por el riesgo de desarrollar un Sd. de abstinencia brusco.

Caractersticas.

Comienzo precoz.

Comienzo tardo.

Nosocomial.

Comienzo.

Nacimiento; menos de 7 das


normalmente menos de 3 das.

8-28 das, en ocasiones hasta


60 das.

De la primera semana al alta


Hospitalaria.

Riesgos obsttricos.

Colonizacin, amnionitis,

Poco frecuentes.
premadurez.

Premadurez; ingreso en UCI


neonatales, reseccin intestinal.

Presentacin.

Dificultad respiratoria, neumona


Shock.

Fiebre, signos
1 nerviosos centrales
o focales.

Apnea, bradicardia, letargia,


inestabilidad trmica.

Meningitis.

20%.

75%.

10-20%.

Otros sistemas.

Raro.

Pielonefritis, osteomielitis, artritis


sptica, celulitis.

Neumona, pielonefritis, endoftalmitis,


trombos spticos, flebitis, infecciones
cutneas, sepsis de lneas centrales,
enterocolitis necrotizante.

Estreptococo del grupo B tipos


Ia, Ib, Ia/c, II, III, E. coli,
Klebsiella, Listeria monocytogenes,
enterococos, Haemophilus
influenzae no tipables,
S. pneumoniae.

Estreptococo del grupo B tipo III,


E. coli Ag K L. monocytogenes,
herpes simple.

Staphylococcus epidermidis, S. aureus,patgenos.


Candida albicans, Pseudomonas
aeruginosa, E. coli, herpes simple,
Klebsiella, Serratia.

Tratamiento.

Ampicilina y gentamicina o
cefotaxima.

Ampicilina y gentamicina o
cefotaxima.

Depende de los agentes nosocomiales


Presentes en la sala de neonatologa;
cefotaxima, vancomicina y gentamicina.

Medidas de apoyo.

Ventilacin mecnica, frmacos


vasoactivos, reposicin
hidroelectroltica, ECMO.

Ventilacin mecnica, frmacos


vasoactivos, reposicin
hidroelectroltica.

Ventilacin mecnica, frmacos


vasoactivos, reposicin
hidroelectroltica.

Mortalidad.

15-70%.

10-20%.

5-10%.

Microorganismos
patgenos.

Pg. 4 PD

M exico A rgentina
C hile U ruguay

CTO Medicina C/ Nez de Balboa, 115 28006 MADRID (Espaa) Tfno.: (91) 782 43 32 / Fax: (91) 782 43 27
E-mail: secretaria@ctomedicina.com; iberocto@ctomedicina.com WEB: www.ctomedicina.com; www.iberocto.com

Comentarios TEST

Pregunta 24. Caractersticas de la sepsis neonatal.

PEDIATRA

Preparacin Examen de Seleccin 05/06 1 Vuelta


Pregunta 22. Diferencias entre hijos de madres adictas a herona y a
metadona.

+HURtQD

0HWDGRQD

0iVULHVJRGHDERUWRV\&,5
1RGHDQRPDOtDVFRQJpQLWDV

,GHP

3HRUFRQWUROGHHPEDUD]R

(PEDUD]RPHMRUFRQWURODGR
5LHVJRGHSROLGURJDGLFFLyQ

6tQGURPHGHDEVWLQHQFLD
WHPEORUHVLUUULWDELOLGDGGLDUUHD $SDUHFHPiVWDUGH\GXUDPiVWLHPSR
YyPLWRVODELOLGDGYDVRPRWRUD 0iVULHVJRGHFRQYXOVLRQHV
)HQREDUELWDO

,GHP

Pregunta 23.- R: 1
El embarazo del hijo de madre diabtica es considerado un embarazo de alto riesgo, debido al gran nmero de problemas que puede
presentar el RN. Entre ellos figuran:
Macrosoma con visceromegalia (pero no megalencefalia), excepto las madres con diabetes avanzadas, y vasculopata, que suelen
tener nios CIR.
Cardiomiopatas: lo ms frecuente es la hipertrofia asimtrica del
tabique interventricular, que clnicamente se manifiesta como una
estenosis subartica.
Sndrome asfctico: son nios ms grandes y con ms complicaciones durante el parto.
Riesgo de EMH: la insulina en altas concentraciones inhibe la sntesis de surfactante.
Policitemia, ictericia y trombosis de la vena renal.
Malformaciones: las ms frecuentes malformaciones cardacas.
Otras: agenesia lumbosacra y sd. del coln izquierdo hipoplsico.

Seguimiento a distancia

Alteraciones metablicas:
- Hipoglucemia: ms frecuente en las primeras horas. Es necesario un control (venoso o capilar) pues la mayora de las veces es
asintomtico. El tratamiento consiste en el aporte de glucosa iv
mediante perfusin continua, evitando los bolos por el riesgo
de hiperglucemia de rebote.
- Hipocalcemia.
Pregunta 24.- R: 3
Mediante el esquema de la parte inferior de la pgina recordamos
las diferencias entre sepsis precoz y tarda
Pregunta 25.- R: 5
La madre que puede contagiar una hepatitis B al RN es aquella
que tiene el Ag s +, (hepatitis crnica activa, hepatitis aguda, portadora). Si adems el Age es positivo, el riesgo asciende hasta un 90%.
El momento de mayor riesgo es el momento del parto.
La mayora de las veces, la enfermedad en el neonato es asintomtica, pero tiene una alta probabilidad de evolucionar hacia una forma
crnica y sufrir degeneracin maligna. Para evitar este curso es fundamental realizar profilaxis con vacuna frente a hepatitis B (0,1,6) y
gammaglobulina especfica en el mismo momento. Si la profilaxis se
realiza correctamente, podr recibir lactancia materna.
Pregunta 26.- R: 2
El caso clnico nos presenta a un RN con clnica de CMV congnita. Los datos claves que nos permiten llegar al diagnstico son: coriorretinitis y calcificaciones periventriculares (en el caso de la TXP, estas
son difusas). La infeccin congnita por CMV es la infeccin congnita ms frecuente. Lo ms frecuente es que estn asintomticos, pero si
tienen clnica pueden presentar ictericia, hepatoesplenomegalia, calcificaciones, hepatitis,... La neumonitis es la forma ms caracterstica
de infeccin postnatal. No existe tratamiento eficaz; en casos de alta
replicacin al nacimiento se puede emplear Ganciclovir. La secuela
ms frecuente es la hipoacusia neurosensorial.

Pregunta 27. Diferencias entre toxoplasma y citomegalovirus.


CITOMEGALOVIRUS
Se reactiva asintomtica 1-2%
Se infecta sintomtica 1-2

Se infecta

Transmisin

Placentaria 1e r trimestre ms frecuente, ms grave


Canal del parto
Lactancia

Placentario
- 1e r trimestre, ms grave
- 3e r trimestre: ms frecuente

Asintomtico ms frecuente

5-18% secuelas tardas, hipoacusia neurosensorial


bilateral y severa

Sin tratamiento coriorretinitis

Coriorretinitis
Microcefalia
Calcificaciones periventriculares

SABIN
Coriorretinitis
Hidrocefalia
Calcificaciones intracraneales difusas
Convulsiones

Prenatal Ac antiCMV (no se hace prevencin)

1 determinacin:
Negativo: vigilar
Positivo: hacer una 2 determinacin:
- Disminuido o igual: infeccin pasada
- Aumentado: tratar

Aislamiento y cultivo en orina "inclusiones en ojo de


buho"
IgM
IgG o estable a las 6 semanas

Aislamiento en placenta (sangre?)


IgM
IgG o estable a las 6 semanas

No
Ganciclovir a veces

Siempre:
Sintomtico: pirimetadina + sulfadicina 6 meses
pirimetadina + sulfamicina/espiromicina 6
meses
Asintomtico. espiromicina y valorar serologa

Infeccin materna

Comentarios TEST

TOXOPLASMA

Sntoma

Screening prenatal

Diagnstico

Tratamiento

M exico A rgentina
C hile U ruguay

CTO Medicina C/ Nez de Balboa, 115 28006 MADRID (Espaa) Tfno.: (91) 782 43 32 / Fax: (91) 782 43 27
E-mail: secretaria@ctomedicina.com; iberocto@ctomedicina.com WEB: www.ctomedicina.com; www.iberocto.com

PD Pg. 5

Seguimiento a distancia

PEDIATRA

Preparacin Examen de Seleccin 05/06 1 Vuelta

Pregunta 27.- R: 2
Ver tabla en la pgina anteror.
Pregunta 28.- R: 1
Estamos ante un nio RN CIR (edad gestacional de 38 semanas y
peso < 2500gr) con clnica de erupcin petequial y hepatoesplenomegalia. Esto lo puede presentar cualquier infeccin connatal.

El neonato contrae la infeccin cuando pasa por el canal del


parto, por lo tanto lesiones genitales activas en el momento del parto
contraindican un parto vaginal y es indicacin de cesrea.
El tratamiento en este nio sera la administracin de aciclovir iv.
(Ver tabla).
Pregunta 31.- R: 2
En relacin al ombligo del RN, es importante recordar que consta
de 2 arterias (en ocasiones una sola arteria se asocia con anomalas
renales y trisoma del 18) y 1 vena.
Pueden existir alteraciones como el onfalocele (dd con gastrosquisis) y hernia umbilical (ver tabla).
Sobre secreciones por el ombligo han preguntando en el MIR:
Persistencia del uraco: sale por el ombligo un lquido amarillento
similar a la orina con un pH cido.
Persistencia del conducto onfalomesentrico (fstula intestinal): sale
un lquido amarillento con un pH alcalino.
El ombligo amnitico se refiere a un ombligo con mucha gelatina.

Pregunta 28. Manifestaciones clnicas de la rubola congnita.

A continuacin nos describen la clnica tpica del DAP: soplo


continuo en 2 espacio intercostal izquierdo con pulsos femorales
saltones. Las lesiones seas son las tpicas de la rubola, las que nos
encontraramos en la sfilis seran las mismas, pero con reaccin
peristica.
Aparte de esto, para el MIR debemos recordar:
Al contrario que en todas las infecciones connatales, la rubola
tiene ms riesgo de trasmisin en el 1 trimestre.
La trada de Gregg se caracteriza por:
- Sordera de percepcin.
- Cardiopata: la ms frecuente el DAP.
- Afectacin ocular: lo ms frecuente son las cataratas.
Un RN infectado por rubola congnita puede eliminar el virus
en las secreciones hasta ao y medio despus del nacimiento,
por lo que es obligatorio aislarlo de mujeres embarazadas.
Pregunta 29.- R: 1
La clnica de la sfilis congnita se subdivide en 2 grupos:
Precoz: los dos primeros aos.
Tarda: ms all de los dos aos.
La sfilis precoz viene a ser parecida a una sfilis 2 del adulto.
Recordar que la trasmisin de la sfilis en el RN es por va transplacentaria, por lo tanto no deberemos encontrar chancro de inoculacin.
La clnica ms frecuente de la sfilis precoz es la hepatoesplenomegalia. Otras manifestaciones son: lesiones cutneo-mucosas (pnfigo,
rash, rinorrea intensa), lesiones seas (imgenes radiolucentes en
metfisis de huesos largos con reaccin peristica). La sfilis tarda es
semejante a la sfilis terciaria del adulto: lesiones cutneas (rgades
sifilticas), lesiones seas (nariz en silla de montar, tibias en sable), lesiones del SNC (tabes dorsal y paresia). Recordar los elementos de la
trada de Hutchinson: sordera, queratitis y anomalas dentarias.
El tratamiento consiste en la administracin de penicilina.
Pregunta 30.- R: 4
Todo el caso clnico hace referencia a la infeccin por el virus
herpes simple, siendo el ms frecuente el tipo II.
La clnica suele comenzar a la semana con grave afectacin del
estado general, fontanela abombada y vesculas en la zona de presentacin (en la prctica clnica slo aparecen en un 70% de los
casos). Se asocia con una elevada mortalidad.

Pg. 6 PD

M exico A rgentina
C hile U ruguay

HERNIA
UMBILICAL

ONFALOCELE

GASTROSQUISIS

DEFECTO DE
CIERRE

Umbilical.

Umbilical.

Paramedio
(lat. dcho. + frec.).

CUBIERTAS

Peritoneo y piel.

Peritoneo.

Sin peritoneo.

- Anomalas
cromosmicas.
- Extrofia vesical.
- Sd. Beckwith
(macrosoma e
hipoglucemia).

Atresia intestinal.

Correcin
quirrgica
precoz
(peor
pronstico).

Correcin
quirrgica precoz
(buen
pronstico).

- Estrangulacin
(rara).
COMPLICACIONES
- Reduccin
espontnea.

TRATAMIENTO

Quirrgico, si:
- Estrangulacin.
- Crecimiento
progresivo.
- Persiste a los
3-5 aos.

Pregunta 32.- R: 4
Recordar los siguientes datos de forma muy simplificada:
1,5 m: sonrisa social.
3 m: inicio sostn ceflico.
6 m: inicia sedestacin.
9 m: movimiento de pinza.
10-12 m. Inicia bipedestacin.
12 m: emite bislabos referenciales.
15-22 m: realiza torres de cubos de 2 hasta seis.
16-19 m: corre y realiza combinaciones de 2 palabras.
Pregunta 33.- R: 4
Estamos ante un cuadro de sarampin. El caso clnico va relatando
detalladamente las fases de esta enfermedad: Fase prodrmica: catarro con tos y fiebre, manchas de Koplik (lesiones blanquecinas sobre
base eritematosa en mucosa subyugal), es un dato patognomnico de
esta enfermedad. Fase exantemtica: inicio del exantema por la cara
y descenso. El exantema es maculopapular, no pruriginoso. La temperatura sube hasta 39-40. Fase de resolucin :el exantema desaparece en el mismo orden que apareci.
Entre las complicaciones figuran:
Cuadros ORL: complicacin ms frecuente.
Neumonas: en los nios es ms frecuente la sobreinfeccin bacteriana, ms que la neumona de clulas gigantes de Hecht.
Afectacin del SNC: encefalitis aguda y PEES (panencefalitis esclerosante subaguda): forma de encefalitis por virus lentos, con un
pronstico psimo, el diagnostico se hace detectando en LCR aumento de Acs frente al sarampin.

CTO Medicina C/ Nez de Balboa, 115 28006 MADRID (Espaa) Tfno.: (91) 782 43 32 / Fax: (91) 782 43 27
E-mail: secretaria@ctomedicina.com; iberocto@ctomedicina.com WEB: www.ctomedicina.com; www.iberocto.com

Comentarios TEST

Pregunta 31. Patologa umbilical.

PEDIATRA

Preparacin Examen de Seleccin 05/06 1 Vuelta

Seguimiento a distancia

Pregunta 30. Infecciones connatales.

Infeccin
materna

VARICELA

SFILIS

RUBEOLA

Se infecta

Se infecta

Se infecta sintomtica 70%

Placentaria:
er
Transmisin 1 y 2 trimestre, ms frecuente.
Prx imo a parto: ms grave.
er

Sntomas

Transmisin 1 y 2 trimestre:
atrofia miembros, cicatrices y
malformaciones.
5 a 21 das preparto: varicela
leve en 4 primeros das
postparto.
5 das preparto a 2 das
postparto: varicela grave en
das 5 y 10 postparto.

Screaning
prenatal

ADN del virus.


IgM en sangre de cordn
Diagnstico
(descenso rpido).
Aislamiento del virus en
el lquido vesicular.

Comentarios TEST

Si infeccin materna entre 5 das


preparto y 2 postparto:
Tratamiento Ig anti VVZ.
Aislamiento estricto.
Aciclovir si es grave.

er
Placentaria: 3 trimestre:
ms frecuente si madre con
sfilis primaria o secundaria
no tratadas

Se reactiva (VHS II)


Se infecta: poco frecuente

Placentaria
Placentaria, 1er trimestre ms frecuente,
Canal del parto
ms grave
Postparto

Precoz (<2 aos):


Transitoria: CIR, hepatoesplenomegalia, Generalizada (1 semana)
- Hepatomegalia.
meningitis...
sepsis, encefalitis...
- Pnfigo sifiltico, coriza,
Permanente (Gregg):
Localizada
neumona alta de Hetcht.
- Cataratas
(2 y 3 semanas):
Tarda (>2 aos):
- Sordera de percepcin
- Queratoconjuntivitis
- Sordera de percepcin.
- Cardiopata
- Vesculas cutneas
- Queratitis intersticial.
- Retraso mental, fenmenos
- Encefalitis del lbulo
- Anomalas dentarias.
autoinmunes.
temporal
1er trimestre y antes del
parto.
VDRL y FTA-ABS .
Si infeccin activa tratar a
la madre.
VDRL.
FTA-ABS tratar con
penicilina.

Penicilina 15 das

Pregunta 34.- R: 3
La rubola es una enfermedad banal por su curso, pero de gran
gravedad si afecta a la mujer embarazada, por las graves secuelas que
puede producir en el feto.
Clnicamente es muy parecida al sarampin, pero con un curso
ms leve y recortado. Suele cursar con febrcula y el exantema desaparece al 3 da. A la hora de contarnos un caso clnico nos deben
poner siempre en la fase prodrmica, las adenopatas retroauriculares
dolorosas a la palpacin que tardan semanas en resolverse.
La complicacin ms frecuente es la artritis de pequeas articulaciones, ms frecuentes en mujeres jvenes, que se resuelve sin dejar secuelas.
Pregunta 35.- R: 3
Estamos ante un caso de escarlatina. El agente ms frecuente es el
Streptococcus pyogenes (alfa hemoltico). Afecta a nios entre 5-15
aos de edad. La fase prodrmica suele ser una faringoamigdalitis
bacteriana aguda (fiebre alta, exudado faringoamigdalar). Posteriormente aparece un exantema, de inicio en tronco , que se hace ms
confluente en los pliegues (lneas de Pastia), afectando a la cara y
respetando el tringulo nasogeniano. Afecta a palmas y plantas, y
puede dejar descamacin residual al resolverse.
El diagnstico de eleccin es un frotis de exudado farngeo. El
tratamiento consiste en la administracin de penicilina oral durante
10 das. En los casos en los que no es posible realizar el tratamiento
oral completo estar indicado la administracin de penicilina benzatina (im) 1 dosis.
Entre las complicaciones ms importantes figuran:
Glomerulonefritis postestreptoccica.
Fiebre reumtica: slo si la puerta de entrada ha sido un foco ORL.
M exico A rgentina
C hile U ruguay

HERPES SIMPLE

Aislamiento del virus.


IgM.
IgG o estable a las 6
semanas.

No

Cultivo y aislamiento virus

Aciclovir

Pregunta 36.- R: 1
El caso que se nos presenta es una varicela. Ya sabemos que el
agente causal es el virus de la varicelazoster, de la familia de los
herpes virus. La fase prodrmica consiste en un catarro de vas altas
con febrcula. Lo caracterstico del exantema es que suele afectar a
tronco y parte proximal de extremidades, respetando las zonas distales. Las lesiones aparecen en distintos estadios evolutivos (o lesiones en
cielo estrellado); estas son lesiones eritematosas con vesculas y otras
en fase de costra. Las lesiones pican mucho. La complicacin ms
frecuente es la sobreinfeccin bacteriana de las lesiones.
Otras complicaciones son:
Neumona varicelosa: en la infancia es ms frecuente la secundaria a sobreinfeccin bacteriana, ms que la producida por el propio virus.
Afectacin del SNC: lo ms frecuente es en forma de cerebelitis,
que evoluciona de forma favorable.
La varicela en los nios sanos no precisa tratamiento con aciclovir.
Son indicaciones para su uso: neonatos e inmunodeprimidos.
Pregunta 37.- R: 3
La pregunta hace referencia al llamado eritema infeccioso,
megaloeritema o 5 enfermedad. En el MIR lo suelen preguntar para
confundirlo con el exantema sbito, roseola infantil o 6 enfermedad.
El eritema infeccioso tiene como agente causal el Parvovirus B 19.
El cuadro se caracteriza por febrcula y aparicin de un exantema en
encaje o cartogrfico en mejillas y zona proximal de extremidades. Se
resuelve espontneamente, pero en ocasiones puede recidivar cuando el nio se estresa, tiene fiebre,...

CTO Medicina C/ Nez de Balboa, 115 28006 MADRID (Espaa) Tfno.: (91) 782 43 32 / Fax: (91) 782 43 27
E-mail: secretaria@ctomedicina.com; iberocto@ctomedicina.com WEB: www.ctomedicina.com; www.iberocto.com

PD Pg. 7

Entre las complicaciones figuran la aplasia medular en nios susceptibles (con enfermedades hematolgicas).
Pregunta 38.- R: 2
El cuadro al que se refiere este caso clnico es un exantema sbito,
roseola infantil o 6 enfermedad. Lo tpico es que se produzca en
lactantes, cursando con fiebre alta de 39-40 y algn signo catarral,
pero con un estado general bueno.
Con respecto a la R1, es falsa porque lo tpico es que la fiebre alta
persista durante 3 das, no slo 24 horas.
La respuesta 2 es la cierta: si en las primeras 24 horas le realizramos una analtica, encontraramos una importante leucocitosis con
neutrofilia. Si esperaramos 36 horas ms, se apreciara un cambio
significativo en la analtica con neutropenia y linfocitosis.
La 3 es falsa, porque la rubola suele cursar con febrcula o incluso sin fiebre.
La 4 es falsa: OJO no confundir con el eritema infeccioso,
megaloeritema o 5 enfermedad.
La 5 es falsa, porque la escarlatina es una enfermedad tpica de
nios ms mayores, entre los 5 y 15 aos de edad.
Pregunta 39.- R: 4
Recordemos los criterios diagnsticos de la enfermedad de Kawasaki
o sd. mucocutneo ganglionar.

Pregunta 39. Criterios diagnsticos de la enfermedad de Kawasaki.

$
%

PEDIATRA

Preparacin Examen de Seleccin 05/06 1 Vuelta

'JFCSFEFBMNFOPTEBTEFEVSBDJO
1SFTFODJBEFEFMPTDJODPTJHVJFOUFTTJHOPT
 $POKVOUJWJUJTCJMBUFSBMOPQVSVMFOUB
 "MUFSBDJPOFTFOMBNVDPTBEFMBPSPGBSJOHF DPO
JOZFDDJOGBSOHFBMBCJPTTFDPTDPOGJTVSBT
JOZFDUBEPTPBNCPT ZMFOHVBFOGSFTB
 "MUFSBDJPOFTFOMBT[POBTQFSJGSJDBTEFMBT
FYUSFNJEBEFT DPNPFEFNBZFSJUFNBFONBOPTP
QJFT EFTDBNBDJOEFJOJDJPQFSJVOHVFBM
 &YBOUFNB EFJOJDJPFOFMUSPODPQPMJNPSGP OP
WFTJDVMBS

Pregunta 41.- R: 5
Estamos ante un cuadro de mononucleosis infecciosa, caracterizado por fiebre, adenopatas y faringoamigdalitis.
El agente causal ms frecuente es el virus Epstein-Barr. Hasta en un
5% se puede asociar con exantema, pero si le administramos antibiticos (amoxicilina) ante la sospecha de una infeccin bacteriana, el
riesgo de aparecer un exantema maculopapular pruriginoso aumenta
hasta en un 90% de los casos. En este caso clnico, el S. pyogenes
aislado en el frotis no significa una infeccin activa por ste, si no que
se traduce en un estado de portador crnico.
Dentro de las complicaciones, la ms grave es la rotura esplnica.
Otras seran la hipertrofia amigdalar, que puede producir obstruccin
de la va area y hepatitis subclnica con elevacin de las transaminasas en hasta un 70-90% de los casos.
La herpangina es un cuadro caracterizado por fiebre alta, adenopatas y lceras en los pilares anteriores del paladar. El tratamiento es
sintomtico.
Pregunta 42.- R: 2
Para el diagnstico de mononucleosis infecciosa, aparte de la sospecha clnica, podemos encontrar:
Leucocitosis (10000-20000) con ms de un 20-40% de linfocitos
atpicos.
Aumento de las transaminasas.
Acs heterfilos: son especficos frente al VEB. Es til en mayores de
4 aos; por debajo de esta edad, la sensibilidad es < del 20%.
Puede permanecer positivo hasta 9 meses despus, por lo que no
es til para el diagnstico de infeccin activa.
Acs especficos contra el VEB:
- Fase aguda:
IgM + - IgG: frente a la cpside viral : CVA
EA + (Ag temprano)
EL VEB se ha asociado con distintos tumores como el linfoma Burkitt,
carcinoma nasofarngeo, sd. de Duncan y diversos sds. linfoproliferativos.
El tratamiento consiste en reposo; en algunos casos puede ser til la
administracin de corticoides (obstruccin de la va area alta, anemia hemoltica autoinmune, convulsiones,...).

Adems de estos sntomas se puede encontrar patologa muy variada: artritis , pericarditis, iritis,... Es muy llamativa la trombocitosis, especialmente en la fase subaguda de la enfermedad.
El tratamiento consiste en el empleo de AAS y Gammaglobulina en
la fase aguda, manteniendo el AAS en dosis antiagregantes posteriormente. El empleo de la gammaglobulina en la fase aguda previene el
desarrollo de aneurismas coronarios en la fase subaguda.

Pregunta 43.- R: 4
El cuadro del que vamos a hablar a continuacin es de una tos ferina.
La edad con ms riesgo es el periodo de lactante (la madre no le da Acs
protectores, porque estos van disminuyendo a lo largo de la vida).
La clnica consta de una fase prodrmica: fase catarral, con febrcula, rinorrea y tos blanda. Este es el periodo de mxima contagiosidad. La fase paroxstica: consiste en episodios de golpes o accesos
de tos, seguido en muchas ocasiones de vmitos. En los nios mayores es caracterstico el gallo inspiratorio. Y por ltimo, la fase de
convalecencia: esta fase se caracteriza por encontrarse el nio afebril.
El diagnstico se basa en el cultivo del moco nasal en medio de
Bordet-Gengou.
En la analtica es tpico encontrar una linfocitosis absoluta. Para el
tratamiento se utiliza la eritromicina.

Pregunta 40.- R: 2
Este paciente presenta una prpura de Schlein-Henoch. Como
ya recordis, es una vasculitis de pequeo vaso.
La fase prodrmica suele consistir en proceso ORL. Posteriormente
puede aparecer, en orden de frecuencia:
Afectacin cutnea: prpura palpable, en mmii, que suele cursar
en brotes.
Artritis de rodillas y tobillos.
Dolor abdominal clico, vmitos y deposiciones con sangre.
Afectacin renal: hematuria con o sin proteinuria. Este es el factor
que marca el pronstico.
Slo en menos de un 1% se produce afectacin del SNC.
El tratamiento consiste en antiinflamatorios para las molestias articulares y en ciclos cortos de corticoides, en caso de dolor abdominal
intenso.

Pregunta 44.- R: 5
La parotiditis est producida por un paramixovirus. Tiene mayor
incidencia entre los 515 aos, provocando ms complicaciones en
los pacientes post-pberes. Los Acs maternos protegen los primeros 68 meses de vida. La clnica consiste en una inflamacin de las glndulas partidas de comienzo brusco, asincrnico, siendo tpico el desplazamiento del pabelln auricular hacia delante con borramiento
del ngulo mandibular.
Entre las complicaciones figuran:
Meningitis asptica: es la complicacin ms frecuente en la infancia, siendo el 50% subclnica.
Orquitis: 25% de varones postpberes, el 90% es unilateral. La
esterilidad es una secuela poco frecuente.
Otros: ooforitis, pancreatitis, sordera neurosensorial, etc.
El tratamiento consiste en medidas generales y analgsicos.

 -JOGBEFOPQBUBDFSWJDBMVOJMBUFSBM

&

Pg. 8 PD

-BFOGFSNFEBEOPTFQVFEFFYQMJDBSQPSOJOHVOBPUSB
DBVTBDPOPDJEB

M exico A rgentina
C hile U ruguay

CTO Medicina C/ Nez de Balboa, 115 28006 MADRID (Espaa) Tfno.: (91) 782 43 32 / Fax: (91) 782 43 27
E-mail: secretaria@ctomedicina.com; iberocto@ctomedicina.com WEB: www.ctomedicina.com; www.iberocto.com

Comentarios TEST

Seguimiento a distancia

PEDIATRA

Preparacin Examen de Seleccin 05/06 1 Vuelta

Pregunta 45.- R: 5
Respecto al SIDA en Pediatra, hay que recordar:
La principal va de transmisin es vertical, durante el parto. El porcentaje de infectados es aproximadamente del 5-8%.
El periodo de latencia es menor que en adulto. Se subdivide en
SIDA precoz (periodo de latencia de pocos meses, clnica antes del
ao de vida, predominando las manifestaciones neurolgicas, pronostico malo) y tardo (ms parecido al del adulto, predominando
la clnica infecciosa).
Las manifestaciones clnicas ms frecuentes en pediatra, comparndolo con los adultos son: cuadros ORL, parotiditis, neumona
intersticial linfoide y manifestaciones neurolgicas (calcificaciones
de los ganglios basales, atrofia cerebral,...). La causa ms frecuente
de muerte es la neumona por P. Carinii.
Son infecciones menos frecuentes en los nios: TBC, hepatitis B,
linfomas y sarcoma de Kaposi.

Pregunta 46.- R: 1
Es importante realizar quimioprofilaxis con AZT durante el embarazo, el momento del parto y luego al RN.
El tratamiento se basa en la triple terapia, al igual que en los adultos.
Para evaluar la evolucin, se emplea la carga viral y el cociente
CD4/CD8, recordando que la cifra de linfocitos hay que extrapolarla
segn la edad del nio.

Pregunta 46. Diagnstico de SIDA en la infancia.

GHPHVHV
'RVUHVXOWDGRVSRVLWLYRV
3&5
&XOWLYRYLUDO
$JS

!PHVHV
(/,6$\VHFRQILUPDSRU:HVWHUQ%ORW
2ORVFULWHULRVGHODSDUWDGRDQWHULRU

Comentarios TEST

2FOtQLFDGHO6,'$
Pregunta 47.- R: 3
La triple vrica protege contra el virus del sarampin, rubola y
parotiditis. Es una vacuna de virus vivos atenuados.
Indicaciones: a los 15 meses y una dosis de recuerdo a los 4 aos,
a nios y nias.
Efectos adversos: cuadro de fiebre y exantema, ms atenuados que
las infecciones frente a las que protege (son virus vivos).
Contraindicaciones:
Inmunodeprimidos: excepto nio HIV+.
Embarazo.
La alergia al huevo ha dejado de ser una contraindicacin absoluta.

Seguimiento a distancia

7 aos, porque a partir de esa edad hay ms riesgos que beneficios.


Indicaciones: en el calendario vacunal se administra a los 2,4,6 y
18 meses y a los 4 aos se puede administrar DT o DTPa , luego
cada 10 aos dT. Las primeras dosis se pueden administrar en un
mismo preparado con otras vacunas (polio inactivada, Hib, VHB),
para minimizar el nmero de pinchazos.
En pacientes inmunodeprimidos se puede administrar sin riesgo.
Pregunta 50.- R: 4
La vacuna del Hib es una vacuna conjugada, muy segura y eficaz.
Protege frente a infecciones invasivas (meningitis y epiglotitis). En el
calendario vacunal se incluye a los 2-4-6 y 18 meses. Estara indicada
solamente en nios menores de 5 aos.
Pregunta 51.- R: 4
La vacuna frente a la hepatitis B est realizada por ingeniera gentica. Produce en las personas que la recibe Acs frente al Ag s. En el
calendario vacunal se puede administrar con diversas pautas: (0-1-6),
(0,2,6) y (2, 4,6) y una cuarta a los 18 meses. Para prevenir la trasmisin
de la hepatitis B en el neonato, en el caso que la madre sea portadora,
se debe asociar con gammaglobulina especifica al nacimiento.
Est indicada a cualquier edad si el nio no ha sido vacunado
previamente. Sus contraindicaciones son muy escasas. Recordar que
el embarazo no es una contraindicacin absoluta, pero se aconseja
evitar todo tipo de vacunas siempre que sea posible.
Pregunta 52.- R: 4
De forma simplificada, vamos a exponer la quimioprofilaxis frente
a los agentes ms frecuentes de meningitis.
Meningococo: a todos los contactos, independiente de la edad,
con rifampicina en dosis de 10 mg/K/dosis cada 12 horas 4 dosis +/
- vacuna en caso del meningococo C.
Hib: nios menores de 5 aos sin vacunar y adultos que conviven
con menores de 5 aos sin vacunar (stos pueden ser portadores):
rifampicina en dosis de 20 mg/K/dosis cada 24 horas 4 dosis +7vacuna.
Neumococo: no existe quimioprofilaxis activa. Se puede indicar la
vacuna del neumococo de 7 serotipos para prevenir futuros contactos.
Pregunta 53.- R: 2
Vamos a esquematizar a continuacin la actitud que hay que llevar a
cabo con un nio que ha tenido un contacto con un bacilfero positivo.

Pregunta 48.- R: 4
Existen actualmente dos tipos de vacunas: Polio oral (Sabin) y Polio
im (Salk).
La polio Sabin es una vacuna de virus vivos atenuados que produce
inmunidad local tipo IgA y vacunacin comunitaria por contacto fecaloral. Como efecto adverso ms importante destacan algunos casos aislados de polio en nios sanos, lo que ha motivado en los ltimos aos
que se estn potenciando las vacunas de virus inactivados.
Est contraindicada en personas inmunodeprimidas y personas
que conviven con stos.
La vacuna tipo Salk (im) est elaborada con virus inactivados, es
menos eficaz, pero ms segura. Es la que actualmente recomienda la
AEP. La pauta de administracin es a los 2-4-6 18 meses y a los 4 aos.
Pregunta 49.- R: 5
Lo bsico que tenis que recordar de la vacuna DTP es lo siguiente:
Componentes:
- Difteria: toxoide.
- Ttanos: toxoide.
- Pertussis: dos tipos: celular y acelular. Esta ltima con muchos
menos efectos adversos a nivel neurolgico. Recomendada por
la AEP. La Pertussis celular estaba contraindicada en mayores de
M exico A rgentina
C hile U ruguay

Pregunta 53. Actitud ante un nio que ha tenido contacto con un paciente
bacilfero positivo.

CTO Medicina C/ Nez de Balboa, 115 28006 MADRID (Espaa) Tfno.: (91) 782 43 32 / Fax: (91) 782 43 27
E-mail: secretaria@ctomedicina.com; iberocto@ctomedicina.com WEB: www.ctomedicina.com; www.iberocto.com

PD Pg. 9

PEDIATRA

Preparacin Examen de Seleccin 05/06 1 Vuelta

Seguimiento a distancia

Pregunta 59. Vmitos del lactante.


Reflujo gastroesofgico

Estenosis hipertrfica de ploro

Momento de aparicin

1 semana

2 a 3 semana

Vmitos

Alimentarios sin fuerza

Alimentarios a chorro

Diagnstico

Tratamiento

Sntomas respiratorios.
Disminucin del crecimiento.
Clnica + respuesta a tratamiento

pHmetra 24h

endoscopia
Medidas posturales.
Cisapride.
Ciruga (funduplicatura de Nissen).

Evolucin.
R x : "signo de la cuerda".
1 burbuja gstrica.

Rx:
R X:
1 burbuja gstrica.
1 burbuja gstrica.
Ausencia de gas distal. 1 burbuja duodenal.

Ciruga:
Piloromiotoma.
Fredet R amsted.

/(&+(+80$1$

/(&+('(9$&$

NFDOO
J
&DVHtQD
6HURSURWHtQDV
JODFWRVD\RWUDV
J
FJUDVRVHVHQFLDOHV
FJUDVRVFDGHQDODUJD
LQVDWXUDGRV
&ROHVWHURO





NFDOO
J
&DVHtQD
6HURSURWHtQDV
JODFWRVD
J
(VFDVRViFJUDVRV
HVHQFLDOHV
FJUDVRVVDWXUDGRV

5HODFLyQ
&iOFLR)yVIRUR

9LWDPLQD$






(VFDVD




(VFDVD
(VFDVD




+LGUDWRVGH&DUERQR

*UDVDV

0LQHUDOHV
+LHUUR
&REUH
)O~RU

9LWDPLQD%
9LWDPLQD&
9LWDPLQD'
9LWDPLQD(
9LWDPLQD.
1LWUyJHQRQRSURWpLFR

 YHFHV



Pregunta 55.- R: 4
La causa ms frecuente de GEA en nuestro medio es el rotavirus. Es
una enfermedad tpica de lactantes que produce una diarrea lquida,
SIN sangre. El diagnstico se basa en la realizacin de un test de ELISA
en las heces para detectar el Ag.
El tratamiento es sintomtico.
Se est estudiando una vacuna (semejante a la polio oral) para
intentar evitar esta enfermedad en los lactantes.
Pregunta 56.- R: 1
Estamos ante un cuadro de alergia a protenas de leche de vaca. El
cuadro tpico consiste en inicio de la clnica tras la ingesta de un
bibern de leche de vaca. La sintomatologa suele ser digestiva (vmiPg. 10 PD

M exico A rgentina
C hile U ruguay

B il io s o s
Abdomen ex cavado.
Asociar al Down.

Pregunta 54. Diferencias entre la leche materna y la de vaca.

3URWHtQDV

Alimentarios

Masa palpable.
Alcalosis hipoclormica.

Pregunta 54.- R: 2
El calostro es la primera secrecin mamaria, las principales diferencias con la leche materna madura es que contiene ms protenas y
ms minerales que la leche madura.

&DORUtDV

Atresia duodenal

Ciruga

Ciruga

tos, diarrea), cutnea (urticaria) y en ocasiones respiratoria (broncoespasmo, edema de glotis).


El cuadro est mediado por un mecanismo inmunolgico tipo Ig E.
Para el diagnstico se emplean las pruebas cutneas, que sern
positivas.
El tratamiento consiste en la suspensin de la leche de vaca, reemplazndola por frmulas especiales (hidrolizado de protenas). Suele
ser un proceso autolimitado, la mayora de los nios pueden tomar
leche de vaca a partir de los 2 aos.
Pregunta 57.- R: 1
El tipo ms frecuente de fstula es la tipo 3 (atresia esofgica con
fstula distal).
A nivel proximal nos encontraremos la siguiente clnica:
- Polihidramnios: el feto no traga adecuadamente el lquido amnitico.
- La sonda se queda enrollada en el bolsn superior.
- Salivacin excesiva, aspiracin pulmonar masiva (fstula proximal) o por rebosamiento.
A nivel distal:
- Abdomen excavado si la fstula es proximal.
- Abdomen distendido, si la fstula es distal.

Pregunta 57. Tipos de fstulas traqueoesofgicas.

Pregunta 58.- R: 5
Estamos ante un caso de RGE leve, caracterizado por pequeos vmitos sin fuerza despus de las tomas, por lo dems el nio se encuentra
asintomtico con peso y talla en percentiles normales. El RGE es muy
frecuente en el primer ao de vida, hasta un 85%. Generalmente desaparece hacia los 2 aos, aunque en ocasiones puede persistir hasta los 4.
En un 10% de los casos, el RGE se asocia con escasa ganancia
ponderal, esofagitis, anemia ferropnica y clnica respiratoria.
El diagnostico se basa en la historia clnica y en ocasiones en la
realizacin de pruebas complementarias: transito superior con bario

CTO Medicina C/ Nez de Balboa, 115 28006 MADRID (Espaa) Tfno.: (91) 782 43 32 / Fax: (91) 782 43 27
E-mail: secretaria@ctomedicina.com; iberocto@ctomedicina.com WEB: www.ctomedicina.com; www.iberocto.com

Comentarios TEST

Otros sntomas

Atresia pilrica

PEDIATRA

Preparacin Examen de Seleccin 05/06 1 Vuelta


y una pHmetra de 24 horas (para cuantificar el grado de reflujo, es la
prueba ms sensible y especfica).
El tratamiento en los casos leves consiste en medidas posturales y
espesantes de las tomas. En casos de reflujos ms graves se administraran estimulantes del peristaltismo (cisapride, domperidona) o tratamiento quirrgico (funduplicatura de Nissen).
Pregunta 59.- R: 5
La clnica tpica de un cuadro de estenosis hipertrfica de ploro
consiste en vmitos proyectivos tras las tomas, tras el vmito el nio se
queda irritable y hambriento (ver tabla en pgina siguiente).
Debido a los vmitos persistentes, es tpico encontrar una alcalosis
metablica con normo o hipo K.
La prueba diagnstica a realizar ante la sospecha de este cuadro
sera una ecografa abdominal. En ocasiones se debe recurrir a un
trnsito digestivo, donde aparece de forma tpica la imagen del signo
de la cuerda.
El tratamiento es quirrgico: pilorotoma extramucosa de Ramsted.

Seguimiento a distancia

La clnica se caracteriza por un cuadro de malabsorcin, de


inicio ms frecuente entre los 6 meses y los 2 aos. Suelen presentar esteatorrea, estancamiento de la curva ponderal, disminucin
de la masa muscular, irritabilidad y distensin abdominal importante.
El diagnstico se basa en los test serolgicos (Acs antigliadina, antirreticulina, antiendomisio y antitransglutaminasa, estos ltimos los ms
sensibles y especficos de todos, y en la realizacin de biopsias intestinales.
El tratamiento consiste en la exclusin de por vida del trigo,
cebada, centeno y +/- avena. Pueden tomar libremente maz y
arroz.

Pregunta 60.- R: 4
Lo tpico de la atresia duodenal son los vmitos biliosos en las
primeras 24- 48 horas, con abdomen excavado. Con mayor frecuencia la atresia se localiza a nivel de la 3 porcin del duodeno. En la Rx
de abdomen es tpico encontrar la imagen de doble burbuja.
La R1: lo tpico sera encontrar vmitos a las 2-3 semanas de vida.
R2: lo tpico es estreimiento y en ocasiones vmitos fecaloideos.
R3: sangrado indoloro rectal sin vmitos.
R5: la clnica de la invaginacin intestinal suele comenzar con
episodios de dolor intenso, siendo muy raro en el periodo neonatal.

Comentarios TEST

Pregunta 61.- R: 4
El divertculo de Meckel es un resto del conducto onfalomesentrico. Es la malformacin digestiva ms frecuente (R1), localizndose a
unos 50- 70 cm de la vlvula ileocecal.
La clnica ms frecuente suele consistir en una hemorragia rectal
indolora (R4) e intermitente debido a la ulceracin de la mucosa ileal
adyacente al divertculo con mucosa ectpica (gstrica o pancretica).
La tcnica diagnstica ms sensible es la gammagrafa con Tc 99.
El tratamiento es quirrgico.
Pregunta 62.- R: 3
Estamos ante un cuadro tpico de invaginacin intestinal. Nio que
presenta episodios repetidos de llanto y encogimiento de piernas con
posterior decaimiento. Si la clnica evoluciona hasta un 60%, puede
presentar deposiciones en jalea de grosella, heces con sangre roja
fresca y moco.
Para el diagnstico inicial se suele realizar una ecografa abdominal.
Una invaginacin intestinal es siempre una urgencia. Si han trascurrido < de 48 horas y no hay signos de perforacin, se realiza tratamiento con enemas de bario o con aire, para intentar la reduccin.
Un 5% puede recidivar (frecuente en aquellos casos en los que se
realiza reduccin hidrosttica).
Pregunta 63.- R: 4
La enfermedad de Hirschprung se caracteriza por estreimiento
crnico desde el nacimiento asociado a un estancamiento o retraso
ponderal (por malabsorcin).
Para el diagnstico es til el tacto rectal, donde se aprecia una
ampolla rectal vaca de heces. En la manometra se aprecia un aumento paradjico del tono del esfnter anal. El diagnstico definitivo
lo dar la biopsia: en el segmento afectado se observa ausencia de
clulas ganglionares con aumento de la acetilcolinesterasa y aumento de las terminaciones nerviosas.
Pregunta 64.- R: 5
Los factores que intervienen en la patogenia de la celiaca son:
HLA: DR3, DR4, DR7 , DQ y W2.
Factores inmunolgicos: los linfocitos de la lmina propia se inmunizan frente a la gliadina.
Factores ambientales.
M exico A rgentina
C hile U ruguay

Pregunta 64. Diagnstico de enfermedad celaca.

Pregunta 65.- R: 2
El Sd. de Reye se asocia con una infeccin vrica los das previos.
A la semana inicia un cuadro progresivo de vmitos persistentes con
deterioro del nivel de conciencia que evoluciona al coma.
En el laboratorio aparecen parmetros de fallo heptico agudo:
aumento de GOT/GPT, de LDH, CPK, amonio e hipoglucemia. Es
caracterstico del Sd de Reye que la glutamato DH (enzima
mitocondrial) est muy elevada y que la cifra de bilirrubina sea
normal.
El tratamiento consiste en tratar la alteracin heptica (glucosa,
VitK,...) y el edema cerebral para evitar el enclavamiento cerebral.
El pronstico depende de la evolucin neurolgica.
Pregunta 66.- R: 2
Estamos ante un cuadro de atresia de vas biliares extrahepticas; lo
tpico es un cuadro de colestasis en un lactante de 2-3 semanas (predominio de bilirrubina directa, coluria, acolia).
La prueba diagnostica sera la realizacin de un HIDA, donde se
aprecia captacin del contraste por el hgado, pero que no llega al
intestino (ausencia de excrecin).
El tratamiento, aparte de las medidas de mantenimiento, consiste en
la realizacin de una hepatoportoenterostoma o tcnica de Kasai,
para intentar restablecer el flujo biliar y disminuir el dao heptico. En
la mayora de los casos, el tratamiento definitivo es el trasplante heptico.

CTO Medicina C/ Nez de Balboa, 115 28006 MADRID (Espaa) Tfno.: (91) 782 43 32 / Fax: (91) 782 43 27
E-mail: secretaria@ctomedicina.com; iberocto@ctomedicina.com WEB: www.ctomedicina.com; www.iberocto.com

PD Pg. 11

PEDIATRA

Preparacin Examen de Seleccin 05/06 1 Vuelta

Seguimiento a distancia

La clnica suele cursar de forma brusca, por la noche, acompandose de intensa dificultad respiratoria y apenas sin tos, con fiebre alta
y aspecto sptico.
En el tratamiento, lo prioritario es asegurarse la va area (mediante
intubacin orotraqueal o traqueostoma) y administracin de antibiticos, generalmente cefalosporinas de 3 generacin. Con el tratamiento adecuado, el cuadro suele remitir en 2448 horas.
En los casos dudosos nunca hay que intentar visualizar la epiglotis,
sino realizar una Rx lateral de faringe donde se apreciar una epiglotis
engrosada.

Pregunta 67.- R: 3
Es un cuadro de malabsorcin a hidratos de carbono. Tiene como
antecedente una GEA que lesiona el borde del enterocito, produciendo un dficit transitorio de lactasa. La clnica se caracteriza por diarrea,
dolor clico, heces espumosas y cidas (pH< 5.5) que excorian la zona
del paal. Ante la sospecha se debe realizar un Clinitest en las heces que
ser positivo. El tratamiento consiste en la exclusin de la lactosa de la
dieta, que la mayora de las veces es transitoria.
La R1 es falsa porque, aunque la clnica es igual, el Clinitest es
negativo.
La R2: no da clnica.
R4: produce una malabsorcin de protenas con clnica de desnutricin y edemas.
R5: la clnica del rotavirus es vmitos y diarrea lquida, no cida.
Pregunta 68.- R: 3
Este es un cuadro de laringitis aguda caracterizado por un antecedente previo de CVA con fiebre de 38-38.5 y tos perruna.
El tratamiento consiste en humedad ambiental, corticoides (en aerosol o sistmicos) y adrenalina en aerosol.
Pregunta 69.- R: 2
Se resume en el cuadro inferior.
Pregunta 70.- R: 3
La epiglotitis es una enfermedad tpica de nios menores de 5 aos;
el agente causal ms frecuente es el Hib. Actualmente es excepcional
ver una epiglotitis debido a la vacunacin universal frente al Hib.

Pregunta 72.- R: 4
El cuadro que presenta este nio es claramente una fibrosis qustica.
Las manifestaciones clnicas suelen ser:
Respiratorias: bronquiolitis recurrentes, tos crnica, neumona,
plipos nasales, sinusitis,...
Digestivas: leo meconial, esteatorrea, malabsorcin, prolapso rectal, cirrosis biliar,...
Otras: diabetes, azoospermia.
Para el diagnstico se requiere al menos dos test positivos del sudor,
junto con manifestaciones clnicas compatibles y/o presencia de mutaciones en el estudio gentico. El estudio gentico no puede desplazar al test del sudo debido a la gran variedad de mutaciones que
existen.
Para el tratamiento de estos pacientes es fundamental el cuidado
del pulmn. Se debe realizar diariamente fisioterapia respiratoria y
tratamiento agresivo de las infecciones pulmonares.
Pregunta 73.- R: 3
Se define criptorquidia como la ausencia del testculo en el escroto, situndose en un trayecto fisiolgico del descenso, pudiendo ser o
no palpable.
La frecuencia es del 4% en el RN, aumentando de forma importante en los RNPT. En la mayora de los casos descienden antes del ao
de edad (rara vez despus del ao descienden fisiolgicamente).

Pregunta 69. Crup infeccioso.


/$5,1*,7,6$*8'$ HVWULGXORVD

/$5,1*275$48(7,69,5$/

(3,*/27,7,6$*8'$

(WLRORJtD

$OHUJLDSVLFROyJLFR

9LUXVSDUDLQIOXHQ]DH HOPiVIUHFXHQWH

+LQIOXHQ]DHE

$QWHFHGHQWHV

1RKD\

&DWDUURYtDVDOWDV SDFLHQWH\RIDPLOLD

&OtQLFD

(VSDVPRODUtQJHRUHFRUWDGR
JHQHUDOPHQWHQRFWXUQR 1RILHEUH

)LHEUHWRVGLVQHDDOWDHVWULGRULQVSLUDWRULR

)LHEUHDOWDEDEHRGLVIDJLD
GLVQHDFDEH]DH[WHQGLGD

'XUDFLyQ

QRFKHV

'tDVVHPDQDV

)XOPLQDQWH

7UDWDPLHQWR

Pg. 12 PD

$PELHQWHWUDQTXLORKXPLGLILFDU
&RUWLFRLGHV
$GUHQDOLQDUDFpPLFDHQDHURVRO
M exico A rgentina
C hile U ruguay

,QWXEDUR[tJHQR
&HIDORVSRULQDGHJHQHUDFLyQ
7WRHQ89,

CTO Medicina C/ Nez de Balboa, 115 28006 MADRID (Espaa) Tfno.: (91) 782 43 32 / Fax: (91) 782 43 27
E-mail: secretaria@ctomedicina.com; iberocto@ctomedicina.com WEB: www.ctomedicina.com; www.iberocto.com

Comentarios TEST

Pregunta 66. Colestasis neonatal.

Pregunta 71.- R: 5
Estamos ante un cuadro de bronquiolitis aguda. El agente ms frecuente es el VRS. Es una enfermedad estacional de los meses de invierno y primavera.
Afecta a nios menores de 2 aos, principalmente lactantes. Comienza por un cuadro catarral, con tos blanda, mocos y febrcula, y
en los das siguientes evoluciona hacia un cuadro de obstruccin de
vas areas distales con sibilancias y espiracin alargada. Esta fase crtica suele durar 2- 3 das con posterior mejora.
El tratamiento consiste en fisioterapia y beta 2 agonistas inhalados
(terbutalina, adrenalina,...).
La ribavirina estara indicada en nios con patologa pulmonar o
cardiaca de base; ni los corticoides ni los antibiticos estaran indicados. Actualmente se intenta prevenir su aparicin administrando gammaglobulina especfica mensualmente en los meses de riesgo a los
lactantes con patologa de base (cardipatas, neumpatas).
La secuela a largo plazo ms frecuente es la hiperreactividad bronquial.

PEDIATRA

Preparacin Examen de Seleccin 05/06 1 Vuelta


Entre las complicaciones figuran:
Problemas psicolgicos.
Infertilidad: se producen cambios histolgicos que alteran la
espermiognesis, y estos dependen de la edad. El pronostico de
fertilidad es mejor cuanto ms baja sea la localizacin del teste.
Riesgo de degeneracin maligna: el testculo no descendido tiene
un riesgo de sufrir degeneracin maligna entre 10 y 40 veces ms,
siendo el seminoma el tumor ms frecuente en estos casos.
Para el diagnstico de testes impalpables, bilateralmente se determina la LH, FSH y testosterona basales, y luego la testosterona despus
de la estimulacin con HCG durante 3 das. Si existe una ausencia en
la elevacin de la testosterona con elevacin de la LH y FSH, estamos
ante una anorquia (ausencia bilateral de testculos).
Aproximadamente al ao de edad si la criptorquidia persiste se
debe realizar tratamiento hormonal. El xito se sita en torno al 20%
en nios menores de 6 aos, y si falla se realizar orquidopexia.

Comentarios TEST

Pregunta 74.- R: 4
El caso clnico de esta pregunta es una infeccin de orina (ITU). En
los lactantes la clnica de ITU suele ser muy sutil e inespecfica (anorexia, nuseas, vmitos, febrcula, estancamiento ponderal,...)
El agente ms frecuente es el E. coli. El diagnstico se realiza mediante
el cultivo de orina. El mtodo de recogida en ocasiones es problemtico (bolsa autoadhesiva en rea genital) porque tiende a contaminarse
con las bacterias de la regin anal. Actualmente se prefiere la puncin
suprapbica. En caso de dudas diagnsticas en la fase aguda se puede
emplear el DMSA (tambin til para ver cicatrices post pielonefritis).
El tratamiento se realiza preferentemente con Gentamicina iv. Es
necesario en todo nio/a menor de 5 aos descartar la existencia de
un reflujo vesicoureteral.
Pregunta 75.- R: 3
Respecto al RVU, hay que saber que es la anomala congnita ms
frecuente de la unin ureterovesical. Se suele diagnosticar al estudiar
las ITUs.
Se clasifican segn la intensidad, el grado de dilatacin ureteral y la
deformidad calicial (grados I-V y reflujo intrarrenal).
La principal complicacin es la nefropata por reflujo intrarrenal,
que es la causa de hasta un 20% de las insuficiencias renales y la 1
causa de HTA en la infancia
Para el diagnstico se realiza una cistografa miccional. Se deben
descartar malformaciones anatmicas, evaluar el tamao renal y descartar cicatrices con el DMSA.
El tratamiento quirrgico se establece en funcin del grado de
reflujo. Habitualmente los reflujos grados I y II se resuelven espontneamente, en el grado III un 50% precisa tratamiento por va endoscpica, y los del grado IV y V precisarn tratamiento quirrgico.
Pregunta 76.- R: 4
El sndrome hemoltico urmico (SHU) suele presentarse en nios
menores de 4 aos. En los das previos el nio tiene una GEA enteroinvasiva (el agente ms frecuente es el E. coli O-156). El patgeno
produce unas toxinas que favorecen por la lesin a nivel endotelial la
agregacin plaquetaria y la formacin de trombos y secundariamente
una anemia hemoltica microangioptica.
El rgano ms afectado en este cuadro es el rin produciendo un
cuadro de IRA. Otros en menor frecuencia son: intestino, SNC,...
El diagnstico se basa en:
Anemia hemoltica microangioptica: haptoglobina descendida
con esquistocitos en el frotis de sangre perifrica.
Plaquetopenia.
IRA: por ecografa se debe diferenciar de la trombosis venosa bilateral.
El tratamiento consiste en dilisis peritoneal y medidas de sostn.
En los casos algo ms complicados se puede realizar plasmafresis o
administracin iv de prostaglandinas.
El pronstico es bueno, siendo raras las recidivas, excepto en los
casos familiares.
Pregunta 77.- R: 2
Estamos ante un cuadro de torsin testicular. Se debe realizar diagnostico diferencial con la orquioepididimitis.
M exico A rgentina
C hile U ruguay

Seguimiento a distancia

La clnica consiste en un dolor agudo intenso, con tumefaccin


escrotal, sin fiebre ni traumatismo previo. El reflejo cremastrico suele
estar anulado.
El diagnstico se basa fundamentalmente en la clnica, en caso de
duda se puede recurrir a la gammagrafa de flujo testicular o a la
realizacin de una eco-Doppler.
El tratamiento consiste en la reduccin manual o fijacin quirrgica del teste afecto y del contralateral. El tiempo de viabilidad del
testculo torsionado es de 46 horas, por lo que siempre se debe
considerar una urgencia quirrgica.

Pregunta 77. Patologa testicular aguda.

7RUVLyQ

2UTXLRHSLGLGLPLWLV

'RORUEUXVFR
LQWHQVR


0iVSURJUHVLYR

+LGURFHOH

6t

0iVIUHFXHQWH

5HIOHMR
FUHPDVWpULFR

$QXODGR

3HUPDQHFH

0&
(GDG
$QWFSHUVRQDOHV
(VFURWR
7HVWH
3UHKQ

(FR'RSSOHU

0D\RUHV
(SLVRGLRVSDUHFLGRV 3URVWDWLVPRFLUXJtD
EDMDVRQGDMHV
SUHYLRV
SDURWLGLWLV
,QIODPDGR"
,QIODPDGR
'RORURVRHOHYDGR\ 'RORURVRPyYLO
KRUL]RQWDOL]DGR
1R"
6t

0HQRVIOXMRTXHHO ,JXDORPD\RUTXHHO
FRQWUDODWHUDO
FRQWUDODWHUDO

Pregunta 78.- R: 2
La CIV es la cardiopata congnita ms frecuente en la infancia. La
mayora suelen ser pequeas, situadas en la porcin membranosa del
tabique y que con el tiempo sufrirn un cierre espontneo. En cuanto
a la clnica y tratamiento, es necesario diferenciar entre CIVs pequeas y grandes:

Pregunta 78. Diferencias entre CIV pequeas y grandes.


3HTXHxD
&OtQLFD

'LDJQyVWLFR

*UDQGH

6RSORSDQVLVWyOLFRIXHUWH 6RSORSDVLVWyOLFROHYH
,&&
$VLQWRPiWLFDV
&OtQLFDUHVSLUDWRULD
5[WyUD[OHYHSOpWRUD
(&*1

5[GHWyUD[FDUGLRPHJDOLD
\SOpWRUDSXOPRQDU
(&*FUHFLPLHQWRGH
FDYLGDGHVL]TXLHUGD

&LUXJtDDQWHVGHODxRSDUD
HYLWDUHOGHVDUUROORGHKWS
3URILOD[LVGHHQGRFDUGLWLV 3URILOD[LVGHHQGRFDUGLWLV
6XHOHQFHUUDUVH

7UDWDPLHQWR HVSRQWiQHDPHQWH

Pregunta 79.- R: 5
El ductus arterioso persistente es una cardiopata tpica de RNPT,
en los que la sntesis local de PGE1 mantiene el ductus abierto en la
vida extrauterina, unido a la sobrecarga de volumen.
Esta cardiopata congnita tambin se asocia a la rubola congnita.

CTO Medicina C/ Nez de Balboa, 115 28006 MADRID (Espaa) Tfno.: (91) 782 43 32 / Fax: (91) 782 43 27
E-mail: secretaria@ctomedicina.com; iberocto@ctomedicina.com WEB: www.ctomedicina.com; www.iberocto.com

PD Pg. 13

La clnica consiste en un soplo continuo, en maquinaria o de


Gibson, ms audible en 2 espacio intercostal izquierdo; los pulsos
femorales son saltones.
El nio puede presentar clnica de ICC.
En la Rx de trax se aprecia pltora pulmonar con aumento de
cavidades izquierdas. El diagnstico definitivo lo obtenemos por ecografa.
En los RNPT, si no existen contraindicaciones, se comienza con la
administracin de indometacina, y si no se soluciona, tratamiento
quirrgico. En caso de no ser un RNPT, el tratamiento de entrada es
quirrgico.
Pregunta 80.- R: 4
La TGA es la cardiopata ciangena ms frecuente de inicio en el
RNPT (recordad que la c. ciangena ms frecuente en conjunto es la
T. De Fallot).
LA TGA ms frecuente es la que cursa con un tabique interventricular ntegro.
La clnica suele consistir en un RN que en las primeras horas de
vida presenta taquipnea y cianosis intensa (conforme se va cerrando
el ductus), no tiene soplo y los pulsos son dbiles. En la Rx de trax se
aprecia un pedculo cardaco estrecho con pltora pulmonar. El ECG
suele ser de caractersticas normales para un RN. El diagnstico se
confirma mediante la ecografa.
El tratamiento consiste en la perfusin de PGE de forma urgente
para mantener abierto el ductus, y cuanto antes realizar la correccin
quirrgica. La tcnica de eleccin es el switch arterial o Jatene, que
consiste en seccionar la salida de los grandes vasos e intercambiarlos.
Pregunta 81.- R: 4
La anomala de Ebstein consiste en la insercin anmala (ms inferior) de la valva medial de la tricspide, causando una obstruccin a
la salida del ventrculo derecho, y produciendo anatmicamente una
AD grande y un VD pequeo.
La clnica va a consistir en cianosis, cuyo momento de aparicin va
a depender del grado de obstruccin a la salida del VD.
Es frecuente la aparicin de arritmias; la ms tpica es un patrn de
Wolf-Parkinson-White.
Pregunta 82.- R: 3
La CoA es ms frecuente en nias con sd de Turner. El tipo ms
frecuente es yuxtaductal. Los nios suelen estar asintomticos. En la
exploracin es tpico encontrar un soplo sistodiastlico interescapular y pulsos femorales dbiles con HTA en mmss. En la Rx de trax
aparecen escotaduras costales en los bordes inferiores de las costillas
y a veces el llamado signo del 3. En el ECG aparecen signos de hipertrofia de VI. El tratamiento se realiza mediante arteriografa con dilatacin con baln o mediante ciruga colocando un parche.
Pregunta 83.- R: 2
Se trata de una tetraloga de Fallot (TF). La TF es la cardiopata
ciangena ms frecuente. Sus componentes son:
Hipertrofia de VD.
CIV grande.
Acabalgamiento de la Ao.
Estenosis infundibular de la pulmonar.
La clnica consiste en cianosis que en ocasiones empeora, produciendo crisis de cianosis secundarias a la contraccin del rodete de
msculo cardiaco que produce la estenosis infundibular. La cianosis,
como en cualquier cardiopata ciangena, produce policitemia y
riesgo de formacin de trombos o infartos a nivel del SNC.
En la Rx de trax se aprecia un corazn en forma de zueco e
isquemia pulmonar. En el ECG aparece signos de hipertrofia de VD. El
tratamiento es quirrgico; si no es posible por la edad y peso del nio,
se puede realizar un shunt sistmico-pulmonar en espera de la correccin completa.
Pregunta 84.- R: 2
Las CIA es la forma ms frecuente de cardiopata congnita en la
edad adulta y la segunda en la infancia.
Pg. 14 PD

M exico A rgentina
C hile U ruguay

PEDIATRA

Preparacin Examen de Seleccin 05/06 1 Vuelta


El tipo ms frecuente es la CIA ostium 2, que se caracteriza por un
segundo ruido amplio y fijo que no se modifica con la respiracin. La
Rx de trax suele ser normal o con leves alteraciones (pltora, leve
cardiomegalia). En el ECG suelen aparecer bloqueos de rama derecha, con un patrn rSR en precordiales derechas. El tratamiento es
necesario si el shunt es 2:1 y se prefiere realizar durante la infancia,
debido al riesgo de desarrollar hipertensin pulmonar en la edad
adulta. Otro tipo de CIA menos frecuente es la CIA Ostium 1; una
variedad es el llamado canal AV: CIA 1 +CIV+ alteraciones de los
cojinetes endocrdicos. Es tpica del sd. de Down.
Pregunta 85.- R: 4
Estamos ante un RN con hipotiroidismo congnito. La causa ms
frecuente es la disgenesia tiroidea. La clnica tpica del hipotiroidismo
no est presente al nacimiento, sino que aparece en el transcurso de
semanas. Suele presentar, aparte de la sintomatologa comn con el
adulto (somnolencia, estreimiento, hipotermia, mixedema ...), hernia umbilical, ictericia prolongada, fontanelas amplias y macroglosia.
Si no se instaura tratamiento, con el tiempo desarrollan retraso mental. El tratamiento consiste en la administracin de hormona tiroidea.
Pregunta 86.- R: 2
El cuadro clnico es un nio con hiperplasia suprarrenal congnita
por dficit de 21 hidroxilasa forma pierde sal. En este tipo de alteracin se produce una disminucin de la produccin de aldosterona,
cursando la analtica con un sodio srico bajo y un potasio alto.
La estenosis hipertrfica de ploro tambin cursa con vmitos y
puede producir deshidratacin, pero el patrn inico consiste en
una alcalosis metablica con hipocloremia y potasio bajo.
El panhipopituitarismo no tiene porqu tener alteraciones inicas,
puesto que la produccin de aldosterona por la suprarrenal no se
encuentra alterada.
El hipotiroidismo puede presentar hiponatremia, pero con niveles
de potasio normales.
En el hiperaldosteronismo, los iones se encuentran alterados en
sentido contrario.
Pregunta 87.- R: 3
En cuanto a la pubertad precoz (inicio de caracteres sexuales en las
nias antes de los 8 aos y en los varones antes de los 9), se debe hacer
diagnstico diferencial entre dos situaciones: pubertad precoz verdadera o central (activacin del eje hipotlamo-hipfiso-gonadal) y la
pseudopubertad precoz o perifrica (no activacin gonadal inicial). La
causa ms frecuente de la primera en las nias es la forma idioptica, y
en los varones suelen ser los tumores del SNC. El tratamiento consiste en
anlogos de GnRH para frenar la pubertad. La pubertad precoz verdadera tiene una etiologa muy diversa (ver tabla en pgina siguiente).
Pregunta 88.- R: 3
La causa ms frecuente de talla baja en el nio son las dos variantes
de la normalidad.

Pregunta 88. Tipos de talla baja no patolgica.

&DUDFWHUtVWLFDV

7DOODEDMDIDPLOLDU

5HWUDVRFRQVWLWXFLRQDO

&DXVDV

7DOODEDMDHQORVSDGUHV

$QWHFHGHQWHVIDPLOLDUHV

'LDJQyVWLFR

9HORFLGDGFUHFLPLHQWR
QRUPDO
(2 (&(WDOOD
7DOODILQDO WDOODGLDQD

7UDWDPLHQWR

1RSUHFLVD

9HORFLGDGFUHFLPLHQWR
OHQWD
3XEHUWDGUHWUDVDGD
(2 (WDOOD(&
7DOODILQDO WDOODGLDQD
1RSUHFLVD

Pregunta 89.- R: 2
Estamos ante un cuadro de hipoglucemia cetsica. Es la causa ms
frecuente de hipoglucemia en la infancia. La edad de aparicin es
entre los 18 meses y 5 aos, siendo un proceso autolimitado. Estos

CTO Medicina C/ Nez de Balboa, 115 28006 MADRID (Espaa) Tfno.: (91) 782 43 32 / Fax: (91) 782 43 27
E-mail: secretaria@ctomedicina.com; iberocto@ctomedicina.com WEB: www.ctomedicina.com; www.iberocto.com

Comentarios TEST

Seguimiento a distancia

PEDIATRA

Preparacin Examen de Seleccin 05/06 1 Vuelta

Seguimiento a distancia

Comentarios TEST

Pregunta 87. Diagnstico de la pubertad precoz.

nios sufren hipoglucemia por mala tolerancia al ayuno prolongado.


En la analtica encontramos hipoglucemia hipoinsulinmica y alanina
srica disminuida. En orina hay cuerpos cetnicos positivos. El tratamiento consiste en comidas frecuentes.

ticas muy variadas con una mala evolucin clnica hacia el retraso
psicomotor y trastornos psquicos. El EEG muestra un patrn basal muy
desestructurado, y el tratamiento consiste en el uso de mltiples
anticomiciales con difcil control de las crisis. El pronstico es malo.

Pregunta 90.- R: 1
La causa ms frecuente de convulsiones en el RN es la encefalopata
hipxico-isqumica. Las crisis suelen ser focales, no generalizadas, puesto
que el SNC del neonato no est completamente mielinizado. El diagnstico se basa en el EEG. El fenobarbital es el frmaco de primera
eleccin. El pronstico depender de la causa desencadenante.

Pregunta 95.- R: 3
La epilepsia rolndica o de paroxismos centrotemporales es el tipo
de crisis ms frecuente en la infancia. La clnica consiste en sntomas
orofarngeos, de predominio nocturno, que en ocasiones se pueden
generalizar. En el EEG aparece un foco temporal. En la mayora de las
ocasiones no precisa tratamiento y su pronstico es excelente.

Pregunta 91.- R: 2
Estamos ante un cuadro de petit mal o crisis de ausencia. Se inicia
alrededor de los seis aos y la clnica consiste en episodios de breve
desconexin con el medio de 5 a 20 segundos con algn automatismo. En ocasiones se pueden generalizar a crisis de gran mal. En el EEG
aparecen punta onda a 3 ciclos/seg de inicio y final brusco. Las crisis
se pueden desencadenar con la hiperventilacin. Para su tratamiento
se emplea valproato o etosuximida.

Pregunta 96.- R: 2
El astrocitoma del nio es el tumor de fosa posterior ms frecuente.
Suelen ser de bajo grado, qustico y tienden a localizarse en hemisferios cerebelosos. La clnica habitual es la hipertensin intracraneal en
el momento del diagnstico. El pronstico es bueno, a diferencia del
meduloblastoma, que suele afectar al vermis cerebeloso y ser de mal
pronstico.

Pregunta 92.- R: 5
Los espasmos infantiles o s. de West suelen aparecer alrededor de
los 4 u 8 meses. Consisten en episodios de flexo-extensin de miembros y cabeza cuando el nio se est empezando a dormir o a despertar. Se acompaa adems de un enlentecimiento del desarrollo
psicomotor. En el EEG es tpico el patrn de hipsarritmia. En la etiologa existe un grupo sintomtico, donde hay un dao cerebral previo o
una sintomatologa conocida, siendo stos los de peor pronstico. Un
segundo grupo criptognico donde el nio es normal hasta el inicio
de las crisis. Para el tratamiento se usa ACTH.

Pregunta 97.- R: 3
El tumor ms frecuente en la infancia es la LLA.
Tumor slido ms frecuente: cerebrales. Fosa anterior craneofaringioma, posterior astrocitoma.
Tumor slido extracraneal ms frecuente: neuroblastoma.
Tumor abdominal ms frecuente: Wilms.
Tumor de partes blandas ms frecuentes: rabdomiosarcoma.
Tumor orbitario ms frecuente: rabdomiosarcoma.
Tumor ocular ms frecuente: retinoblastoma.

Pregunta 93.- R: 2
El cuadro clnico hace referencia a un nio con una convulsin
febril. La clnica tpica consiste en fiebre alta, crisis tnico-clnica
generalizada de breve duracin con un periodo postcrtico corto. La
edad de presentacin es entre los 6 meses y 5 aos. Existen antecedentes familiares hasta en un 30% de los casos. El tratamiento en la
fase aguda consiste en la administracin de diacepam intrarrectal y
medidas para disminuir la hipertermia.

Pregunta 98.- R: 3
El cuadro que se expone es una nia con un neuroblastoma. La
edad de aparicin tpica es en menores de 2 aos. Se suele localizar en
el abdomen a nivel de las glndulas suprarrenales y la clnica ms frecuente es la masa abdominal que cruza lnea media. Entre los sndromes
paraneoplsicos (no modifican el pronstico) figura el opsoclonusmioclonus (caso clnico) y la diarrea secretora. Para el diagnstico se
utiliza la TC craneal, catecolaminas en orina aumentadas y la
gammagrafa con MIBG. El tratamiento depende del estadio, realizndose ciruga, quimioterapia y radioterapia. La supervivencia es del 50%.

Pregunta 94.- R: 3
Respecto al sndrome de Lennox-Gastaut, hay que decir que tiene un
pico de incidencia entre uno y siete aos. Consiste en crisis de caracters-

Pregunta 99.- R: 4
El tumor de Wilms se asocia con anomalas genitourinarias, hemihipertrofia y aniridia. Se han encontrado deleciones en el cromoso-

M exico A rgentina
C hile U ruguay

CTO Medicina C/ Nez de Balboa, 115 28006 MADRID (Espaa) Tfno.: (91) 782 43 32 / Fax: (91) 782 43 27
E-mail: secretaria@ctomedicina.com; iberocto@ctomedicina.com WEB: www.ctomedicina.com; www.iberocto.com

PD Pg. 15

Seguimiento a distancia

PEDIATRA

Preparacin Examen de Seleccin 05/06 1 Vuelta

ma 11. La edad al diagnstico es en nias menores de tres aos. La


clnica ms frecuente es la masa abdominal asintomtico. Otros como
HTA, hematuria, etc. La localizacin ms frecuente de las metstasis es
el pulmn. El diagnstico se basa en la TC y la PAAF, no aconsejndose la realizacin de biopsia.

Pg. 16 PD

M exico A rgentina
C hile U ruguay

CTO Medicina C/ Nez de Balboa, 115 28006 MADRID (Espaa) Tfno.: (91) 782 43 32 / Fax: (91) 782 43 27
E-mail: secretaria@ctomedicina.com; iberocto@ctomedicina.com WEB: www.ctomedicina.com; www.iberocto.com

Comentarios TEST

Pregunta 100.- R: 3
El retinoblastoma es un tumor tpico de nios pequeos. La clnica
habitual es la presentacin de leucocoria. Los casos familiares se asocian con afectacin bilateral. El tratamiento en estos casos consiste en
enucleacin de ojo ms afectado y lser o radioterapia local en el
otro. En casos de tumor unilateral, el tratamiento de eleccin es la
enucleacin. Se asocia con el cromosoma 13, y ello confiere la posibilidad de aparicin de otros tumores posteriormente como el
osteosarcoma.

PSIQUIATRA

Preparacin Examen de Seleccin 05/06 1 Vuelta


TRASTORNOS NEURTICOS.
1.

Una paciente presenta desde su adolescencia episodios recurrentes de palpitaciones y mareo con sensacin de prdida de
estabilidad; ocasionalmente ha sufrido desvanecimientos; tras
repetidas exploraciones somticas negativas comienza tratamiento psiquitrico. Cul de los siguientes cuadros es RARO
que complique el trastorno que sufre la paciente?:
1)
2)
3)
4)
5)

2.

Preguntas TEST

Metro.
Centros comerciales.
Hablar en pblico.
Ascensores.
Aglomeraciones.

Trastorno esquizoide.
Distimia.
Personalidad evitativa.
Agorafobia.
Esquizofrenia residual.

NO es frecuente que los pacientes obsesivos estn preocupados con:


1)
2)
3)
4)
5)

6.

Buspirona.
Loracepam sublingual.
Control de la respiracin.
Diacepam oral.
Explicacin de la benignidad del cuadro.

La fobia social es ms DIFCIL de diferenciar de:


1)
2)
3)
4)
5)

5.

8.

Cul de las situaciones que se mencionan es MENOS probable


que desencadene miedo en un agorafbico?:
1)
2)
3)
4)
5)

4.

Abuso de sedantes.
Trastorno obsesivo.
Depresin.
Alcoholismo.
Ansiedad crnica.

La contaminacin.
La duda.
La simetra.
La soledad.
La enfermedad.

2)
3)
4)
5)

Existe una asociacin fuerte entre el trastorno de la Tourette


y el TOC.
Algunos casos infantiles se relacionan con infecciones estreptoccicas.
Hay una disfuncin de los circuitos corticosubcorticales
frontales.
El elemento psicopatolgico ms tpico es la sensacin de
duda.
Se ha demostrado una alteracin selectiva de la neurotransmisin noradrenrgica.
M exico A rgentina
C hile U ruguay

Uno de los siguientes sntomas NO forma parte de los criterios


diagnsticos del trastorno por estrs postraumtico:
1)
2)
3)
4)
5)

9.

Psicoanlisis.
Sertralina.
Fenelcina.
Loracepam.
Clomipramina.

Pesadillas recurrentes con el suceso traumtico.


Amnesia de detalles importantes del acontecimiento.
Estado de nimo depresivo.
Respuesta de sobresalto exagerada.
Evitacin de los lugares relacionados con el suceso.

La presencia de mltiples sntomas fsicos para los que no se


encuentra una causa evidente define al trastorno conocido
como:
1)
2)
3)
4)
5)

Dismorfofobia.
Hipocondra.
Sndrome de Briquet.
Conversin histrica.
Trastorno facticio.

10. En la clasificacin DSM, NO se incluye dentro de los trastornos


disociativos:
1)
2)
3)
4)
5)

Amnesia.
Fuga.
Personalidad mltiple.
Despersonalizacin.
Convulsiones.

11. Las benzodiacepinas ejercen su accin:


1)
2)
3)
4)
5)

Activando el receptor serotoninrgico 5HT1a.


Modificando la permeabilidad neuronal al calcio.
Potenciando la funcin GABArgica.
Interfiriendo con los sistemas de segundo mensajero.
Bloqueando la recaptacin noradrenrgica.

12. Ante un paciente en el que sospechamos una intoxicacin por


alprazolam, el tratamiento de eleccin es:

En la etiologa del trastorno obsesivo NO es cierto:


1)

Cul de las opciones de tratamiento parece la ms potente para


un paciente dominado por la necesidad de comprobar cada
pequeo acto cotidiano un nmero fijo de veces, hecho ste
que le provoca gran ansiedad, pero que dice ser incapaz de
controlar?:
1)
2)
3)
4)
5)

Ante un paciente que est en plena crisis de angustia, una de las


siguientes medidas es INEFICAZ:
1)
2)
3)
4)
5)

3.

7.

Seguimiento a distancia

1)
2)
3)
4)
5)

Naloxona.
Flumacenil.
N-acetilcistena.
Trihexifenidilo.
Fisostigmina.

13. En los pacientes con hepatopata, uno de los siguientes ansiolticos est CONTRAINDICADO:
1)
2)
3)

Oxacepam.
Loracepam.
Clormetiazol.

CTO Medicina C/ Nez de Balboa, 115 28006 MADRID (Espaa) Tfno.: (91) 782 43 32 / Fax: (91) 782 43 27
E-mail: secretaria@ctomedicina.com; iberocto@ctomedicina.com WEB: www.ctomedicina.com; www.iberocto.com

PQ Pg. 1

Seguimiento a distancia
Tiapride.
Cloracepato dipotsico.

14. Una de las benzodiacepinas hipnticas que se relacionan a


continuacin se utiliza exclusivamente para el insomnio de
conciliacin por su corta vida media. Selela:
1)
2)
3)
4)
5)

15. La forma ms frecuente de trastorno del estado de nimo es, de


entre las siguientes:
Enfermedad bipolar.
Distimia.
Trastorno ciclotmico.
Depresin mayor.
Melancola.

Depresin
Depresin
Depresin
Depresin
Depresin

melanclica.
psictica.
atpica.
neurtica.
puerperal.

17. Seale el trastorno afectivo que sufre una paciente de 35 aos


que desde su adolescencia experimenta de forma casi constante sntomas depresivos de intensidad leve, asociados con una
sensacin de cansancio enorme y una clara tendencia al sueo,
y cuyos sntomas apenas desaparecen unas semanas al ao,
empeorando de forma dramtica cuando la paciente sufre un
problema sentimental:
1)
2)
3)
4)
5)

Depresin mayor.
Trastorno mixto ansioso-depresivo.
Personalidad depresiva.
Neurastenia.
Distimia atpica.

18. Dentro de los cuadros depresivos del anciano, NO es frecuente


que encontremos:
1)
2)
3)
4)
5)

Predominio de los sntomas somticos.


Quejas cognitivas.
Alto riesgo de suicidio.
Frecuente ideacin delirante.
Mala respuesta al electrochoque.

19. De las alteraciones del sueo, cul es la que refieren los


pacientes con depresin atpica?:
1)
2)

Insomnio de conciliacin.
Despertar precoz.

Pg. 2 PQ

M exico A rgentina
C hile U ruguay

Por la maana.
Con IMAOs ms que con tricclicos.
En relacin con factores externos.
Al terminar el da.
En verano.

21. La presencia de un sndrome depresivo leve y autolimitado tras


la prdida de un ser querido recibe el nombre de:
1)
2)
3)
4)
5)

Melancola.
Depresin reactiva.
Duelo no complicado.
Depresin mayor.
Neurosis depresiva.

22. Uno de los siguientes delirios NO se considera congruente con


las enfermedades afectivas:

16. El concepto de depresin mayor incluye en el DSM-IV diversos


subtipos de depresiones, entre los cuales NO se incluye:
1)
2)
3)
4)
5)

Somnolencia excesiva diurna.


Frecuentes despertares nocturnos.
Sueo no reparador.

20. Se describe clsicamente que los pacientes depresivos "endgenos" mejoran:


1)
2)
3)
4)
5)

Triazolam.
Brotizolam.
Lormetacepam.
Loprazolam.
Flunitracepam.

TRASTORNOS AFECTIVOS, FRMACOS ANTIDEPRESIVOS Y


ESTABILIZADORES DEL HUMOR.

1)
2)
3)
4)
5)

3)
4)
5)

1)
2)
3)
4)
5)

Ruina.
Influencia.
Grandeza.
Enfermedad.
Culpa.

23. Un paciente de 62 aos lleva un mes prcticamente sin realizar


ninguna actividad, no tiene ganas ms que de estar en la cama y
dice no tener fuerzas ni para acabar con su vida. Asimismo, est
convencido de estar arruinado a pesar de que, segn su familia,
mantienen una economa desahogada. El personal de enfermera
ha observado que est algo ms activo por la noche y que coge
bien el sueo, pero se despierta a las 3 de la madrugada. Tras
recibir tratamiento antidepresivo con imipramina empieza a
estar ms inquieto e irritable, hablar durante horas seguidas y con
tendencia a la procacidad y a la desinhibicin sexual. Qu
debemos considerar en primer lugar?:
1)
2)
3)
4)
5)

Que presenta una reaccin psicolgicamente normal tras la


recuperacin de la depresin.
Que padece un trastorno bipolar, desconocido hasta ese
momento.
Que el tratamiento antidepresivo est induciendo un cuadro
maniforme.
Que estn apareciendo rasgos patolgicos de personalidad,
ocultos por la depresin.
Que est simulando, pues esos sntomas no son congruentes
en un trastorno afectivo.

24. Dentro de las diferencias entre el trastorno bipolar y la depresin recurrente, NO encontramos:
1)
2)
3)
4)
5)

Inicio a una edad ms tarda en el depresivo.


Ms antecedentes familiares afectivos en el bipolar.
Duracin ms larga de las recadas en el depresivo.
Alto riesgo de suicidio en el bipolar.
Equilibrio entre sexos en el depresivo.

CTO Medicina C/ Nez de Balboa, 115 28006 MADRID (Espaa) Tfno.: (91) 782 43 32 / Fax: (91) 782 43 27
E-mail: secretaria@ctomedicina.com; iberocto@ctomedicina.com WEB: www.ctomedicina.com; www.iberocto.com

Preguntas TEST

4)
5)

PSIQUIATRA

Preparacin Examen de Seleccin 05/06 1 Vuelta

PSIQUIATRA

Preparacin Examen de Seleccin 05/06 1 Vuelta


25. NO sugiere "bipolaridad" en un paciente depresivo:
1)
2)
3)
4)
5)

Debut en el puerperio.
Antecedentes familiares de mana.
Presencia de ideacin delirante incongruente.
Inicio en la adolescencia.
Provocacin de hipomanas con antidepresivos.

26. Se llama trastorno bipolar 2 a la siguiente combinacin de


sndromes afectivos:
1)
2)
3)
4)
5)

Depresin mayor y mana.


Depresin mayor sobre una distimia.
Depresin mayor e hipomana.
Mana "unipolar" (sin depresin asociada).
Depresin invernal y episodios hipertmicos en verano.

27. Cul de los siguientes pacientes presenta un mayor riesgo de


suicidio?:
1)
2)
3)
4)
5)

Varn, 25 aos, con distimia de inicio precoz, sin antecedentes de suicidio.


Mujer, 46 aos, con depresin endgena unipolar.
Mujer, 85 aos, con depresin reactiva tras su reciente
mudanza.
Varn, 72 aos, con melancola delirante.
Mujer, 19 aos, con depresin bipolar y antecedentes
familiares de suicidio.

28. Las listas de frmacos capaces de causar depresin son muy


largas; sin embargo, uno de los siguientes NO se asocia con
depresin sino con mana:
1)
2)
3)
4)
5)

Propranolol.
Isoniacida.
Neurolpticos.
Reserpina.
Anticonceptivos hormonales.

Preguntas TEST

29. En una de las siguientes indicaciones NO se utilizan por el


momento como estrategia teraputica los antidepresivos:
1)
2)
3)
4)
5)

Trastorno por angustia.


Narcolepsia-catapleja.
Dolor crnico neuroptico.
Cefalea tensional.
Abstinencia alcohlica.

30. A la hora de seleccionar un antidepresivo para el tratamiento de un


paciente, cul termina siendo el principal factor implicado?:
1)
2)
3)
4)
5)

Los antecedentes familiares de respuesta a un frmaco.


El subtipo sintomtico de depresin.
El antecedente de una respuesta personal al frmaco.
El perfil potencial de efectos adversos.
Las preferencias del paciente.

31. Una vez superada la fase aguda de una depresin, se recomienda:


1)
2)

Retirar el tratamiento antidepresivo.


Reducir la dosis a la mitad y mantener el tratamiento 6
semanas ms.
M exico A rgentina
C hile U ruguay

3)
4)
5)

Seguimiento a distancia

Mantener al menos 6 meses la misma dosis que produjo la


mejora.
Dejar una dosis mnima y eficaz para prevenir recadas de
forma indefinida.
Comentar al paciente que tome el frmaco segn se encuentre.

32. Los antidepresivos tricclicos renen las siguientes caractersticas, EXCEPTO:


1)
2)
3)
4)
5)

Control a travs de niveles plasmticos.


Bajo precio.
Importantes efectos anticolinrgicos.
Alta letalidad en sobredosis.
Inicio del tratamiento a dosis teraputicas.

33. Los ISRS son los antidepresivos ms usados en la actualidad, entre


otras razones porque no producen apenas efectos secundarios
graves; sin embargo, muchos pacientes se van a quejar de:
1)
2)
3)
4)
5)

Disfunciones sexuales.
Somnolencia.
Estreimiento.
Poliuria.
Sequedad de piel y mucosas.

34. Los IMAOs son el grupo de antidepresivos menos utilizado en


la actualidad; sin embargo existen algunas indicaciones para las
que son el frmaco de eleccin; seale una de ellas:
1)
2)
3)
4)
5)

Narcolepsia.
Trastorno obsesivo.
Pnico.
Depresin atpica.
Bulimia.

35. El tratamiento electroconvulsivo est especialmente indicado


en uno de los siguientes pacientes:
1)
2)
3)
4)
5)

Distimia, refractaria al tratamiento farmacolgico.


Depresin mayor delirante.
Trastorno bipolar en fase mixta.
Ciclador rpido.
Depresin "doble".

36. A la hora de iniciar un tratamiento con litio, una de las pruebas


siguientes resulta INNECESARIA:
1)
2)
3)
4)
5)

Test de embarazo.
Sodio y potasio.
Electrocardiograma.
Electroencefalograma.
Creatinina y urea.

37. Una de las siguientes afirmaciones acerca de las cifras de litemia


es FALSA:
1)
2)
3)
4)

Niveles inferiores a 0,4 mEq/L son, en general, ineficaces.


Niveles superiores a 1,5 mEq/L se asocian con toxicidad.
Durante el perodo de mantenimiento suele bastar con
litemias inferiores a 1,0 mEq/L.
En las fases agudas manacas se recomiendan litemias superiores a 1,0 mEq/L.

CTO Medicina C/ Nez de Balboa, 115 28006 MADRID (Espaa) Tfno.: (91) 782 43 32 / Fax: (91) 782 43 27
E-mail: secretaria@ctomedicina.com; iberocto@ctomedicina.com WEB: www.ctomedicina.com; www.iberocto.com

PQ Pg. 3

Seguimiento a distancia

En las fases agudas depresivas se recomiendan litemias inferiores a 1,0 mEq/L.

38. Un paciente bipolar en tratamiento con litio ingresa en Urgencias por un sndrome confusional; al determinar la litemia se
obtiene una cifra de 3,1 mEq/l, cuando haca una semana su
litemia era de 1,0 mEq/l; suponiendo que el paciente no haya
variado la dosis prescrita, cul de las siguientes circunstancias
explicara esta cifra?:
1)
2)
3)
4)
5)

Tratamiento concomitante con carbamacepina.


Uso de nifedipino como antihipertensivo.
Sobreingesta hdrica.
Toma de indometacina por un esguince.
Prescripcin de teofilina para una enfermedad pulmonar
obstructiva crnica.

39. Una paciente de 43 aos, diagnosticada desde los 22 aos de


trastorno bipolar y sin recadas hasta el ltimo ao tomando
exclusivamente litio, durante este ao ha presentado 5 episodios afectivos que su psiquiatra relaciona con la instauracin
de paroxetina ante fases depresivas leves y finalmente ha
diagnosticado a la paciente de cicladora rpida. En estos momentos est ligeramente deprimida. Cul de las siguientes
opciones teraputicas te parece ms adecuada?:
1)
2)
3)
4)
5)

Suspender el litio e instaurar el topiramato como base de su


tratamiento.
Mantener el litio y aadir carbamacepina o valproico.
Quitar el litio y aadir el valproico y un ISRS, pues la paciente
est deprimida.
Mantener el litio y aadir olanzapina.
Quitar el litio y realizar terapia de mantenimiento con TEC.

40. Uno de los siguientes pacientes NO presenta una mala respuesta al litio:
1)
2)
3)
4)
5)

Mana disfrica.
Recadas estacionales.
Mana secundaria.
Ciclacin rpida.
Fase bipolar mixta.

41. Uno de los siguientes datos acerca de la epidemiologa de la


esquizofrenia es FALSO:

4)
5)

La prevalencia-vida es del 1%.


En varones se describe un comienzo ms precoz.
Hay una mayor frecuencia de enfermedad en clases sociales
bajas.
Existe un exceso de esquizofrnicos que han nacido en los
meses fros.
La esquizofrenia duplica el riesgo de muerte.

42. El concepto actual de esquizofrenia se apoya en los siguientes


datos SALVO:
1)
2)
3)

Curso habitualmente crnico y deteriorante.


Presencia de crisis alucinatorias y delirantes.
Duracin superior a 6 meses.

Pg. 4 PQ

M exico A rgentina
C hile U ruguay

Ausencia de factores psicosociales precipitantes.


Cambios afectivos y reduccin de la sociabilidad.

43. Cul de los siguientes factores NO se incluye dentro de las


causas de vulnerabilidad a la esquizofrenia?:
1)
2)
3)
4)
5)

Antecedentes familiares.
Problemas obsttricos.
Consumo de txicos.
Infecciones virales.
Enfermedades infantiles.

44. Un varn de 24 aos es llevado a urgencias por presentar gran


angustia. Sus padres comentan que llevan notndole raro desde
hace unos 8 meses; ha dejado de salir con sus amigos y de
estudiar, y parece "encerrado en s mismo". Los dos ltimos meses
tiene una mirada "rara", segn su madre, quien lo ha encontrado
hablando slo en numerosas ocasiones. En Urgencias el paciente
te comenta angustiado que est siendo sometido a una vigilancia
y persecucin asfixiante por todas aquellas personas que van en
los coches con matrcula acabada en M, pues esa letra significa
"Morirs pronto". Cmo se denomina este ltimo fenmeno
psicopatolgico?:
1)
2)
3)
4)
5)

Bloqueo del pensamiento.


Descarrilamiento.
Pensamiento disgregado.
Percepcin delirante.
Idea delirante secundaria.

45. Seala cul de los siguientes sntomas NO se incluye entre los


sntomas de primer rango de la esquizofrenia:
1)
2)
3)
4)
5)

Eco del pensamiento.


Alucinaciones visuales.
Robo del pensamiento.
Vivencias de influencia.
Delirios de control.

46. Cul de los sntomas siguientes aparece con mayor claridad en


la fase residual de la esquizofrenia?:

TRASTORNOS PSICTICOS. FRMACOS ANTIPSICTICOS.

1)
2)
3)

4)
5)

1)
2)
3)
4)
5)

Descarrilamientos.
Percepciones delirantes.
Aplanamiento afectivo.
Pseudoalucinaciones auditivas.
Conducta catatnica.

47. El debut precoz de la esquizofrenia se asocia con un peor


pronstico de la enfermedad, como todos los datos siguientes,
EXCEPTO uno:
1)
2)
3)
4)
5)

Sexo masculino.
Antecedentes familiares de depresin.
Personalidad previa paranoide.
Escasa productividad psictica.
Ausencia de desencadenantes.

48. La probabilidad de recada en un paciente esquizofrnico NO


est en relacin con:
1)
2)

Su conciencia de enfermedad.
Los efectos secundarios del tratamiento.

CTO Medicina C/ Nez de Balboa, 115 28006 MADRID (Espaa) Tfno.: (91) 782 43 32 / Fax: (91) 782 43 27
E-mail: secretaria@ctomedicina.com; iberocto@ctomedicina.com WEB: www.ctomedicina.com; www.iberocto.com

Preguntas TEST

5)

PSIQUIATRA

Preparacin Examen de Seleccin 05/06 1 Vuelta

PSIQUIATRA

Preparacin Examen de Seleccin 05/06 1 Vuelta


3)
4)
5)

El tipo de esquizofrenia.
El cumplimiento de la medicacin.
La "emocin expresada" en la familia.

49. A diferencia de la esquizofrenia, la paranoia presenta entre sus


caractersticas:
1)
2)
3)
4)
5)

Inicio en edad juvenil.


Escaso deterioro.
Tendencia a la cronicidad.
Delirios de perjuicio.
Alucinaciones.

50. Un compaero de tu trabajo comienza a sospechar de la


fidelidad de su novia, quien trabaja en otro departamento de la
empresa, cercano al tuyo; cada vez que coincids en la cafetera
y os ve hablando, tiene la certeza de que estaris quedando para
ms adelante y si no hablis cree que se debe a que os habis
sentido vigilados; cmo se denomina este fenmeno psicopatolgico?:
1)
2)
3)
4)
5)

Formacin reactiva.
Anulacin.
Interpretacin delirante.
Aislamiento del afecto.
Fantasa.

51. Cul es el mecanismo de accin principal de los antipsicticos


"clsicos"?:
1)
2)
3)
4)
5)

Bloqueo serotoninrgico 5HT1a.


Estmulo GABArgico.
Inhibicin de la COMT.
Antidopaminrgico D2.
Anticolinrgico muscarnico.

52. Una de las siguientes afirmaciones sobre los antipsicticos es


FALSA:
1)
2)

Preguntas TEST

3)
4)
5)

Los antipsicticos "tpicos" son todos igual de eficaces.


La potencia de un antipsictico se refiere a su accin antidopaminrgica.
Cuanto ms potente es un antipsictico ms hipotensin
produce.
Los efectos extrapiramidales de los antipsicticos de baja
potencia son escasos.
La galactorrea se produce por afectacin del sistema tuberoinfundibular.

53. El mecanismo de accin que parece aumentar la eficacia de la


risperidona y reducir sus efectos extrapiramidales es:
1)
2)
3)
4)
5)

Bloqueo de la recaptacin de dopamina.


Antagonismo del receptor H1.
Potenciacin de la funcin GABArgica.
Bloqueo D2 y antagonismo 5HT2a.
Estimulacin D1 y D4.

54. Tras una inyeccin de un psicofrmaco un paciente presenta


un movimiento involuntario de los ojos hacia arriba, que le
resulta doloroso y se mantiene a pesar de sus esfuerzos por
evitarlo; qu tratamiento precisa?:
M exico A rgentina
C hile U ruguay

1)
2)
3)
4)
5)

Seguimiento a distancia

Biperideno.
Amantadina.
Dantrolene.
Loracepam.
Propranolol.

55. Dentro de las estrategias actuales de tratamiento de la esquizofrenia, el TEC sigue teniendo un papel en:
1)
2)
3)
4)
5)

Formas hebefrnicas.
Esquizofrenia catatnica.
Primeros brotes.
Pacientes con graves sntomas extrapiramidales.
Falta de cumplimiento del tratamiento.

56. Dentro de las opciones de abordaje de tipo psicosocial de la


esquizofrenia, se considera DESCARTADO en el momento
actual:
1)
2)
3)
4)
5)

Terapia de modificacin de conducta.


Psicoeducacin.
Grupos de autoayuda de familias.
Psicoterapias dinmicas.
Centros de rehabilitacin psicosocial.

TRASTORNOS RELACIONADOS CON SUSTANCIAS.


57. Dentro de las complicaciones asociadas al consumo agudo de
alcohol, hay una que est expresada de forma INCORRECTA:
1)
2)
3)
4)
5)

La intoxicacin idiosincrsica se produce tras un consumo


mnimo de alcohol.
Existe buena correlacin entre la alcoholemia y los efectos
neuropsiquitricos.
El tratamiento de la intoxicacin es sintomtico.
La sobredosis de alcohol puede ser letal.
Con frecuencia vemos hiperglucemia como consecuencia
de la intoxicacin.

58. Un paciente ingresado por un traumatismo craneal presenta en


la noche del primer da de ingreso un episodio de agitacin
psicomotriz, encontrndose sudoroso y con temblor grosero;
dice estar viendo cientos de insectos subiendo por las sbanas
de la cama y en la exploracin se encuentra desorientado en
tiempo y espacio; uno de los hallazgos siguientes sera INCOMPATIBLE con el cuadro que padece:
1)
2)
3)
4)
5)

VCM elevado.
GGT mayor de 300.
Descenso de la CDT (desialotransferrina).
Hipertermia.
Convulsiones tnico-clnicas.

59. Dentro de las alternativas farmacolgicas para el tratamiento


de deshabituacin a largo plazo del paciente alcohlico, NO
utilizamos:
1)
2)
3)
4)
5)

Naltrexona.
Diacepam.
Tiapride.
Acamprosato.
Cianamida.

CTO Medicina C/ Nez de Balboa, 115 28006 MADRID (Espaa) Tfno.: (91) 782 43 32 / Fax: (91) 782 43 27
E-mail: secretaria@ctomedicina.com; iberocto@ctomedicina.com WEB: www.ctomedicina.com; www.iberocto.com

PQ Pg. 5

Seguimiento a distancia

60. De las relaciones siguientes entre frmacos y su uso en la


dependencia de opiceos, una es FALSA:

5)

Naloxona tratamiento de la intoxicacin (sobredosis).


Metadona tratamiento de mantenimiento con opiceos.
Naltrexona tratamiento de deshabituacin con antagonistas.
Clonidina tratamiento de la dependencia mixta de herona
y cocana.
Guanfacina tratamiento de la abstinencia (desintoxicacin)
sin opiceos.

61. Un paciente aparece en su consulta demandando tratamiento


pues presenta dolores generalizados, sudoracin, hipertermia,
nuseas y vmitos; a la exploracin detectamos marcas cutneas en forma de ral y cicatrices de abscesos subcutneos en
brazos y piernas; no parecen existir antecedentes mdicos
relevantes, qu actitud deberemos tomar?:
1)
2)
3)
4)
5)

Remitir al paciente a la Unidad de Hospitalizacin Psiquitrica ms cercana.


Derivar al paciente a un centro especializado en drogodependientes.
Administrarle midazolam intravenoso como medicacin
sedante.
Ponerle un goteo de naloxona tras dos ampollas i.v. iniciales.
Iniciar tratamiento con naltrexona y clonidina.

62. Cul de los siguientes frmacos NO se usa para intentar reducir


el riesgo de recadas en consumidores crnicos de cocana?:
1)
2)
3)
4)
5)

Bromocriptina.
Desipramina.
Propranolol.
L-dopa.
Amantadina.

63. Un varn de 75 aos es ingresado para una reseccin prosttica


transuretral; la primera noche tras la operacin se muestra
inquieto, creyendo reconocer en las enfermeras a los compaeros que tuvo durante el Servicio Militar e intentando levantarse de su cama; al ser retenido por la fuerza comienza a gritar
llamando a la Legin pues dice estar secuestrado; qu NO ser
lgico encontrar acompaando a este sndrome?:
Empeoramiento nocturno.
Alucinaciones visuales.
Agitacin psicomotriz.
Preservacin de la capacidad de atencin.
Fluctuaciones de la clnica.

64. Qu antipsictico de los siguientes se recomienda para el


control de la agitacin psicomotriz en los casos de delirium?:
1)
2)
3)
4)
5)

Tioridacina.
Sulpiride.
Zuclopentixol.
Haloperidol.
Clorpromacina.

Pg. 6 PQ

M exico A rgentina
C hile U ruguay

1)
2)
3)
4)
5)

Hemograma.
TAC craneal.
Test de Rorscharch.
Anlisis de orina.
ECG.

66. Una paciente de 70 aos acude a su consulta refiriendo frecuentes prdidas de objetos cotidianos y ocasional desorientacin en calles poco familiares; qu prueba de entre las siguientes NO utilizara en el estudio inicial de este caso de deterioro
cognitivo?:
1)
2)
3)
4)
5)

Hemograma y bioqumica plasmtica.


Niveles de hormonas tiroideas.
TC craneal.
Polisomnografa (estudio de la latencia REM).
Mini-Examen Cognoscitivo (MEC) de Lobo.

67. La prdida de funciones superiores que aparece asociada a la


enfermedad de Huntington se clasifica como:
1)
2)
3)
4)
5)

Reversible.
Irreversible, pero con tratamiento.
Subcortical.
Cortical.
Axial.

68. La presencia de demencia, alteraciones de la marcha, parkinsonismo e incontinencia urinaria orienta hacia el diagnstico de:

TRASTORNOS COGNOSCITIVOS: DELIRIUM, DEMENCIAS Y


AMNESIAS.

1)
2)
3)
4)
5)

65. En un hospital numerosos enfermos se encuentran en riesgo de


sufrir un delirium; en el estudio de este sndrome NO resulta til
una de las siguientes pruebas:

1)
2)
3)
4)
5)

Enfermedad de Pick.
Hidrocefalia normotensiva.
Enfermedad de Alzheimer.
Demencia vascular.
Demencia con cuerpos de Lewy.

69. En el diagnstico diferencial entre depresin y demencia resulta


FALSO que:
1)
2)
3)
4)
5)

Las alteraciones de las pruebas de neuroimagen sean altamente inespecficas en ambas enfermedades.
Los pacientes depresivos tiendan a mejorar por la noche,
mientras que los dementes suelen mejorar por la maana.
Los pacientes con demencia suelen exagerar sus fallos cognitivos y los depresivos minimizarlos.
Los cambios en las pruebas neuropsicolgicas son incongruentes en la depresin y consistentes en las demencias.
La mitad de las llamadas pseudodemencias depresivas desarrollarn a medio o largo plazo una verdadera demencia.

OTROS TRASTORNOS MENTALES: PERSONALIDAD, ALIMENTACIN, SUEO, INFANTILES.


70. Una chica de 17 aos es llevada al servicio de urgencias por sus
padres porque la han encontrado en su habitacin realizndose
cortes en los antebrazos. La paciente dice hacerse los cortes
para sentir que existo y expresa sentimientos crnicos de

CTO Medicina C/ Nez de Balboa, 115 28006 MADRID (Espaa) Tfno.: (91) 782 43 32 / Fax: (91) 782 43 27
E-mail: secretaria@ctomedicina.com; iberocto@ctomedicina.com WEB: www.ctomedicina.com; www.iberocto.com

Preguntas TEST

1)
2)
3)
4)

PSIQUIATRA

Preparacin Examen de Seleccin 05/06 1 Vuelta

PSIQUIATRA

Preparacin Examen de Seleccin 05/06 1 Vuelta


vaco. Es conocida en el Servicio de Urgencias por los numerosos
intentos autolticos que ha realizado mediante la ingestin de
pastillas. Refiere tener atracones ocasionalmente y sus padres
estn asustados por su promiscuidad sexual. Durante la entrevista, dice en ocasiones que se quiere ir de vacaciones, para poco
despus ponerse a llorar y expresar deseos de suicidio. Cul es
el trastorno de personalidad que padece?:
1)
2)
3)
4)
5)

Personalidad
Personalidad
Personalidad
Personalidad
Personalidad

obsesiva.
antisocial.
pasivo-agresiva.
lmite.
depresiva.

71. Una persona que lleva una vida bastante marginal, con un
repertorio limitado de intereses, una llamativa ausencia de
relaciones sociales y con una respuesta emocional escasa,
presenta una personalidad:
1)
2)
3)
4)
5)

Esquizoide.
Esquizoafectiva.
Esquizofrnica.
Esquizotpica.
Esquizofreniforme.

72. Una adolescente se desmaya en clase de gimnasia, tras lo que


es llevada al botiqun del instituto; all se detecta una tensin
arterial de 70/45 mmHg; la chica est muy delgada, detalle que
no habamos notado hasta la exploracin fsica; confrontada
con estos datos, cul de las siguientes conductas NO sera
lgica en esta paciente?:
1)
2)
3)
4)
5)

1)
2)
3)
4)
5)

Seguimiento a distancia

Betabloqueantes.
Clonacepam.
Carbamacepina y valproico.
Clomipramina.
Sistemas de ventilacin positiva.

76. En uno de los siguientes trastornos de la infancia, el tratamiento


con antipsicticos es de eleccin:
1)
2)
3)
4)
5)

Dficit de atencin con hiperactividad.


Autismo.
Mutismo selectivo.
Sndrome de la Tourette.
Enuresis nocturna.

77. Carlos es un nio de 7 aos que sufre con frecuencia castigos


en clase por no aceptar la disciplina; pasa las horas de colegio
levantndose continuamente del pupitre y molestando a sus
compaeros, que han comenzado a hacerle el vaco pues dicen
que es muy bruto; en casa se comporta de forma parecida,
habiendo destrozado todos sus juguetes y sufriendo con frecuencia accidentes, pues no mide correctamente el riesgo; qu
problema presenta con mayor probabilidad?:
1)
2)
3)
4)
5)

Retraso mental.
Trastorno disocial de la personalidad.
Conducta oposicionista.
Dficit de atencin con hiperactividad.
Depresin enmascarada.

Preocupacin por el aspecto fsico y la esttica.


Alto rendimiento acadmico.
Tendencia a vestirse con ropas ajustadas y ligeras.
Uso excesivo de laxantes.
Evitacin de las reuniones sociales en donde pueda verse
forzada a comer en pblico.

Preguntas TEST

73. Dentro de las caractersticas del atracn bulmico, NO se


encuentra:
1)
2)
3)
4)
5)

Tensin creciente antes del atracn.


Relacin desproporcionada ingesta/tiempo.
Sensacin de prdida de control.
Seleccin de alimentos hipocalricos.
Aumento de la frecuencia por la noche.

74. Al revisar la polisomnografa de su paciente encuentra un


perodo con ondas muy lentas en el EEG, asocindose con un
tono muscular bajo y sin movimientos oculares; qu trastorno
del sueo se asocia a esta fase?:
1)
2)
3)
4)
5)

Bruxismo.
Apnea obstructiva del sueo.
Enuresis.
Sonambulismo.
Pesadillas.

75. La prdida brusca del tono muscular en respuesta a una


emocin es un sntoma casi patognomnico de un trastorno;
cul es el tratamiento del mismo?:
M exico A rgentina
C hile U ruguay

CTO Medicina C/ Nez de Balboa, 115 28006 MADRID (Espaa) Tfno.: (91) 782 43 32 / Fax: (91) 782 43 27
E-mail: secretaria@ctomedicina.com; iberocto@ctomedicina.com WEB: www.ctomedicina.com; www.iberocto.com

PQ Pg. 7

PSIQUIATRA

Preparacin Examen de Seleccin 05/06 1 Vuelta


Pregunta 1.-R: 2
Los trastornos primarios por ansiedad se pueden presentar de dos
formas:
1. Continua, como en el trastorno por ansiedad generalizada.
2. Episdica, en forma de crisis de ansiedad; cuando las crisis tienen
lugar siempre al enfrentarse a una determinada situacin decimos
que el paciente sufre una fobia; otros pacientes tienen crisis espontneas (llamadas crisis de angustia o ataques de pnico), sin relacin aparente con estmulo alguno; cuando estas crisis espontneas se repiten y repercuten en la vida de las personas hablamos de
trastorno por angustia (o trastorno de pnico).
De una forma algo artificial tambin se van a incluir dentro de los
trastornos por ansiedad los trastornos obsesivo-compulsivos (en donde la ansiedad es secundaria a los fenmenos obsesivos y compulsivos) y los trastornos por estrs agudo y postraumtico (en donde
la ansiedad es uno de los muchos sntomas que aparecen ante una
situacin estresante de intensidad mxima).
No es raro que los trastornos por ansiedad se vayan confundiendo
entre s cuando se cronifican o complican; el ejemplo ms claro lo
vemos en las complicaciones que surgen cuando el trastorno por
angustia se prolonga en el tiempo:
Inicialmente el paciente comienza a preocuparse de forma CONTINUA por la posibilidad de tener nuevas crisis (ANSIEDAD ANTICIPATORIA).
Luego EVITA determinados lugares en donde no se siente seguro
por el miedo a no recibir ayuda si sufre una crisis (AGORAFOBIA).
No es raro que desarrolle una intensa PREOCUPACIN por la
posibilidad de tener una ENFERMEDAD fsica que explique sus
sntomas (ANSIEDAD HIPOCONDRACA).
Algunos pacientes recurrirn a sustancias sedantes para aliviar su
ansiedad (ABUSO del ALCOHOL o de BENZODIACEPINAS).
Finalmente, no son infrecuentes los sntomas depresivos que se
asocian a un aumento del riesgo de suicidio (DEPRESIN y SUICIDIO).

Comentarios TEST

En el trastorno obsesivo-compulsivo, los pacientes pueden sufrir


crisis de ansiedad (sobre todo cuando se les impide realizar los rituales
compulsivos), pero no vemos lo contrario (un paciente con crisis de
angustia no desarrolla fenmenos obsesivo-compulsivos).
Pregunta 2.-R: 1
En el trastorno por angustia (pnico) diferenciamos tres necesidades de tratamiento:
El control de los sntomas agudos de una crisis de angustia (ataque de
pnico) se realiza con benzodiacepinas (diacepam, cloracepato,
loracepam o alprazolam son las ms usadas); utilizarlas por va oral
o sublingual carece de importancia, pues su farmacocintica no
vara y adems en Espaa NO se comercializa ninguna presentacin sublingual de benzodiacepinas. Es igualmente necesario ayudar al paciente a frenar la hiperventilacin y explicarle la ausencia
de consecuencias fsicas de la crisis. La buspirona es un ansioltico
NO benzodiacepnico (agonista parcial serotoninrgico) con un
perfil ms adecuado para el trastorno por ansiedad generalizada;
carece de efectos sedantes (no sirve para cortar una crisis de ansiedad), no produce dependencia ni abstinencia y no interacciona
con el alcohol; tiene un inicio de accin lento (semanas) y no ha
demostrado efectos preventivos anti-pnico.
La prevencin de nuevas crisis se realiza con antidepresivos, siendo
hoy en da de eleccin los ISRS (paroxetina o citalopram son los
ms utilizados) por su mejor perfil de efectos secundarios; esto no
quiere decir que no sean de utilidad los antidepresivos clsicos
(tricclicos o IMAOs). Tambin son eficaces las benzodiacepinas de
alta potencia (alprazolam, clonacepam, loracepam), pero el miedo a la posible dependencia ha hecho que se trate de limitar su uso
a las primeras semanas del tratamiento, mientras el antidepresivo
alcanza su mxima eficacia.
Finalmente resulta imprescindible evaluar las distintas complicaciones del trastorno por angustia/pnico y ofrecer un tratamiento especfico si persisten una vez controladas las crisis.
M exico A rgentina
C hile U ruguay

Seguimiento a distancia

Pregunta 2. Ventajas y desventajas de los tratamientos


preventivos para el pnico.

12345657

82792345657
12 67 2
2 9
62

94

12342564789

34 2 27 97

127
5
7
9 
126  2 254 

127
56
  2 49 7 

12
 
128
7
2564789 628
12 8
42 9 27
2
12 
7 72
5
 62
1274 2  228 6
12
 



129 27
246

23 52 357

126 5 7 27! 


12"9 42

94
12 7#
642
7 9 

12$
%427
2 842
127
5
7
9 

126  46 

Pregunta 3.-R: 3
La mayora de los pacientes con agorafobia presentan adems crisis
de angustia espontneas (ataques de pnico); se considera por tanto,
que su agorafobia es secundaria. Pero tambin hay formas de agorafobia
primaria en las que no detectamos crisis espontneas. El tratamiento de
la agorafobia en s no es distinto: tcnicas psicolgicas de modificacin
de conducta (sobre todo la exposicin en vivo). La diferencia vendr
dada por la necesidad de aadir tratamiento farmacolgico para prevenir las crisis de angustia en el primer caso. Las tcnicas psicolgicas basadas en la modificacin de conducta son el tratamiento de eleccin en
todas las fobias. Suele realizarse primero un listado de todas las situaciones temidas y una ordenacin en funcin del miedo asociado
(jerarquizacin); posteriormente se programan ejercicios de exposicin
en vivo, comenzando en las situaciones ms fciles (desensibilizacin sistemtica). Se han desarrollado otras tcnicas ms agresivas que
buscan la exposicin brusca a la situacin fbica en su mxima intensidad (inundacin o implosin), pero suelen dar peores resultados.
Pregunta 4.-R: 3
El paciente con fobia social tiene miedo de determinadas situaciones en las que se expone a la opinin de otras personas. Lo que le
preocupa no es por tanto el lugar en el que se encuentra, sino la
reaccin de los dems ante su comportamiento. No es raro que estos
pacientes limiten sus actividades sociales, pudiendo confundirse con
otros pacientes que tambin presentan retraccin social (agorafbicos
que no salen de sus casas por el miedo a los desplazamientos, esquizofrnicos o depresivos que restringen sus relaciones sociales por la
prdida de inters asociada a sus enfermedades, personalidades esquizoides sin inters real por la conducta social). Quizs lo ms complicado sea diferenciarles de las personalidades evitativas o fbicas,
con quienes comparten los mismos errores de pensamiento, si bien se
supone que la fobia social comienza en un momento dado (en torno
a los veinte aos) y las personas evitativas siempre han sido as.
En las fobias sociales los frmacos pueden ayudar a los tratamientos psicolgicos de dos formas distintas; en las formas de fobia social
restringidas a situaciones poco frecuentes (p.ej. miedo a hablar en
pblico delante de un auditorio) los bloqueantes de los receptores
beta-adrenrgicos (propranolol, atenolol) disminuyen la respuesta
ansiosa perifrica y pueden permitir un mayor autocontrol de la ansiedad; en la forma generalizada de fobia social se propone el tratamiento con IMAOs como coadyuvante de la psicoterapia. En los
ltimos aos los ISRS tambin se presentan como alternativa para
estos pacientes, habindose convertido quizs en los frmacos de
primera opcin, aun cuando los estudios sobre su eficacia son menos
claros. Los problemas asociados al uso de IMAOs (necesidad de dieta,
riesgo de interacciones medicamentosas) les han relegado a un segundo plano, a pesar de su indiscutible eficacia.
Pregunta 5.-R: 4
Las dos principales formas de presentacin del trastorno obsesivocompulsivo son:

CTO Medicina C/ Nez de Balboa, 115 28006 MADRID (Espaa) Tfno.: (91) 782 43 32 / Fax: (91) 782 43 27
E-mail: secretaria@ctomedicina.com; iberocto@ctomedicina.com WEB: www.ctomedicina.com; www.iberocto.com

PQ Pg. 1

1. La combinacin de ideas obsesivas relacionadas con la posible


contaminacin o contagio de enfermedades y rituales compulsivos de lavado excesivo o de evitacin del contacto con posibles
fuentes contaminantes.
2. La combinacin de ideas obsesivas de duda o incertidumbre y
rituales compulsivos de comprobacin.
Menos frecuentes son las obsesiones centradas en la necesidad de
orden y simetra, que dan lugar a compulsiones de colocacin y ordenacin. Sin embargo no es raro encontrar pacientes con otros fenmenos ms extraos, destacando las llamadas ideas de contraste (ideas de
contenidos agresivos o sexuales totalmente inapropiadas para la situacin en la que se encuentra el paciente y en desacuerdo con sus deseos
o creencias) que pueden conducir a la aparicin de fobias de impulsin (en las que el paciente tiene miedo de llevar a cabo esas ideas tan
desagradables y evita situaciones potencialmente peligrosas como el
uso de cuchillos, el acercarse a las ventanas o el aproximarse a los
andenes del tren por el miedo a presentar un impulso frente al que no
se pueda resistir). Aunque en muchos casos los fenmenos compulsivos
guardan una relacin ms o menos lgica con las ideas obsesivas, en
otros pacientes veremos rituales totalmente independientes, con una
relacin ms bien mgica (por ejemplo, la necesidad de colocar las
cosas en un determinado orden para que no suceda nada malo).
Pregunta 6.-R: 5
El trastorno obsesivo-compulsivo idioptico suele aparecer al final
de la adolescencia o principio de la juventud, de una forma insidiosa,
y evoluciona con carcter crnico y de forma fluctuante, agravndose
en situaciones de estrs. Es raro que desaparezca por completo, pero
tambin es raro que alcance una gravedad invalidante. La mayora de
los pacientes acaban por acostumbrarse a los fenmenos obsesivos y
compulsivos, lo que puede explicar su escasa presencia en consulta a
pesar de su elevada prevalencia en poblacin general (cerca del 2%).
Cuando debuta en la infancia se puede asociar con los trastornos por
tics (tanto los tics motores crnicos como el trastorno de la Tourette).
Recientemente se ha descrito una asociacin entre algunas formas infantiles de inicio agudo y fenmenos autoinmunes cercanos a la fiebre
reumtica (secundarios por tanto a infecciones estreptoccicas
[estreptococo beta hemoltico o SGA] farngeas); se ha denominado
PANDAS (siglas en ingls del Trastorno Neuropsiquitrico Peditrico de
origen Autoinmune Asociado al Estreptococo) y se supone que comparte un mismo mecanismo patognico con el corea de Sydenham
(lesin mediada por autoanticuerpos frente a los ganglios basales).

Pregunta 6.

Esquema del sndrome PANDAS.

Cuando el trastorno obsesivo-compulsivo aparece en adultos o


ancianos es necesario buscar problemas neurolgicos como causa
del mismo (corea de Huntington, accidentes vasculares cerebrales,
Pg. 2 PQ

M exico A rgentina
C hile U ruguay

PSIQUIATRA

Preparacin Examen de Seleccin 05/06 1 Vuelta

etc.). En cualquier caso, tanto en las formas idiopticas como en las


formas secundarias se implica a las mismas regiones cerebrales, los
circuitos de conexin entre corteza prefrontal, ganglios basales y
tlamo. De hecho, la posible utilidad de la psicociruga en algunos
casos seleccionados se basa en la interrupcin de alguna de las vas de
estos circuitos. El neurotransmisor ms relacionado con el trastorno
obsesivo-compulsivo es la serotonina, lo cual va a ser utilizado en el
tratamiento farmacolgico.
Pregunta 7.-R: 5
Uno de los datos ms sorprendentes del trastorno obsesivo-compulsivo es su respuesta selectiva a medicacin serotoninrgica; el tratamiento farmacolgico de eleccin son los antidepresivos serotoninrgicos, pudiendo en ocasiones potenciarse su efecto con otras sustancias (litio, buspirona). En los casos refractarios y en aquellos en los
que coexistan tics puede ser til aadir un antipsictico (pimocide,
haloperidol). Dentro de los antidepresivos el mejor estudiado es el
antidepresivo tricclico clorimipramina (o clomipramina), si bien sus
efectos adversos le han relegado a un segundo plano tras la aparicin
de los ISRS. stos se han convertido en la medicacin de primera
eleccin, teniendo en cuenta adems que en este trastorno suelen
necesitarse dosis dos o tres veces superiores a las dosis habituales para
la depresin y tiempos de tratamiento muy prolongados, lo que disminuye la efectividad del tratamiento con clorimipramina al aumentar los abandonos. Los IMAOs se han usado para pacientes resistentes,
mientras que las benzodiacepinas carecen de efecto anti-obsesivo
especfico. Junto con el tratamiento farmacolgico es necesario un
tratamiento psicolgico basado (como en las fobias) en las tcnicas de
modificacin de conducta; en concreto, la tcnica preferida se denomina exposicin (a la situacin obsesiva temida) con prevencin de
respuesta (compulsiva). El psicoanlisis no obtiene resultados en los
pacientes con este trastorno. En casos refractarios al tratamiento farmacolgico, la psicociruga (cingulotoma, capsulotoma bilateral anterior, tractotoma subcaudada) puede ser eficaz.
Pregunta 8.-R: 3
Los sntomas del trastorno por estrs postraumtico se agrupan en
cuatro tipos:
Reexperimentacin del acontecimiento: pesadillas, imgenes tipo
flash-back.
Evitacin voluntaria (tipo fbica) o involuntaria (amnesia psicgena) de todo lo relacionado con el acontecimiento.
Estado de hiperalerta: reacciones bruscas ante los ruidos, problemas de concentracin, insomnio.
Cambios emocionales: embotamiento, sensacin de corte vital.
Cuando el trastorno por estrs no ha alcanzado un mes de duracin
se denomina trastorno por estrs agudo, y suelen predominar los sntomas emocionales y disociativos. Se diagnostica un trastorno de estrs
postraumtico cuando los sntomas superan el mes de duracin, pudiendo verse formas crnicas (de ms de 6 meses de duracin) e incluso
formas de inicio demorado (que comienzan al cabo de 6 meses desde
el acontecimiento). Dentro de las complicaciones de estos trastornos
destacan los sndromes depresivos y el abuso de sustancias.
De las posibles situaciones traumticas, aquellas que se deben a
fenmenos naturales (catstrofes como terremotos o inundaciones)
producen menos problemas que las que se deben a la intervencin
humana; dentro de stas, los accidentes son menos traumticos que
los delitos, en los que una persona provoca voluntariamente un dao
sobre otra; los delitos sexuales y los secuestros producen estrs postraumtico en un nmero muy elevado de casos (ms del 80% de las
vctimas de violacin).
No existe un tratamiento especfico del estrs postraumtico; los
antidepresivos (IMAOs, ISRS) parecen obtener mejores resultados cuando predominan los fenmenos de reexperimentacin y los cambios
emocionales; las conductas de evitacin precisarn de un tratamiento conductual adecuado; para reducir el nivel de alerta se pueden
usar benzodiacepinas (cuidado con el abuso de sustancias) y tcnicas
de relajacin. Lo que s parece clara es la necesidad de detectar y
tratar precozmente a estos pacientes, pues la cronificacin de los sntomas predice una mala respuesta al tratamiento.

CTO Medicina C/ Nez de Balboa, 115 28006 MADRID (Espaa) Tfno.: (91) 782 43 32 / Fax: (91) 782 43 27
E-mail: secretaria@ctomedicina.com; iberocto@ctomedicina.com WEB: www.ctomedicina.com; www.iberocto.com

Comentarios TEST

Seguimiento a distancia

PSIQUIATRA

Preparacin Examen de Seleccin 05/06 1 Vuelta

Comentarios TEST

Pregunta 9.-R: 3
Dentro de los pacientes que se presentan con sntomas fsicos para
los que no se encuentra una explicacin mdica (SOMATOMORFOS) tenemos:
Somatizadores: se quejan de numerosos sntomas (digestivos, neurolgicos, genitales, cardiorrespiratorios); tienden a cambiar de
mdico cuando se les confronta con la posibilidad de un origen
psicolgico de sus molestias; pueden abusar de la
automedicacin (analgsicos) y corren el riesgo de someterse a
pruebas diagnsticas innecesarias y peligrosas; en su mayora
son mujeres, que suelen comenzar a tener sntomas en la juventud, siguiendo un curso crnico y fluctuante; se utiliza a veces en
estos casos el epnimo de sndrome de Briquet. Una variante de
somatizador ms limitada en su expresin clnica es el paciente
que se queja de dolor, siendo ste desproporcionado para los
hallazgos exploratorios.
Hipocondracos y dismorfofbicos: estn preocupados por padecer una enfermedad o sufrir un defecto fsico, respectivamente;
aunque tambin pueden cambiar de mdico si no obtienen la
atencin que creen necesaria, es ms raro en ellos la tendencia a la
automedicacin o la realizacin de pruebas complementarias de
riesgo; en el caso de los dismorfofbicos, el peligro fundamental
son las posibles intervenciones estticas (mdicas o quirrgicas)
que pueden dejar secuelas; en ambos casos no veremos un predominio de ningn sexo y el inicio suele ser ms tardo; los pacientes
oscilan en la intensidad de sus preocupaciones, desde formas cercanas a los trastornos obsesivo-compulsivos hasta formas casi delirantes; se propone para estos pacientes un tratamiento similar al de
los trastornos obsesivo-compulsivos (ISRS, clorimipramina), asociado a veces a antipsicticos (pimocide).
Conversivos: se suelen presentar en Urgencias con signos neurolgicos incongruentes a la exploracin fsica (convulsiones, parlisis, anestesias, cegueras); casi siempre se identifica un factor estresante relacionado temporalmente con el inicio de los sntomas; tienden a la recuperacin espontnea pero tambin a la recurrencia; en algunos raros
casos veremos formas de evolucin crnica; como en el caso de los
somatizadores, suelen debutar en la juventud y hay un claro predominio de las mujeres; en los pases desarrollados son cada vez menos
frecuentes; parece que con el desarrollo cultural se produce una
mayor tendencia a la somatizacin como expresin fsica del malestar psicolgico; ambas, conversin y somatizacin, junto con las variantes disociativas, componen las distintas formas de presentacin
de la histeria, una enfermedad clsica dentro de la Psiquiatra que
parece traducir a sntomas fsicos o psiquitricos incongruentes la
respuesta de determinadas personas ante el malestar psicolgico.

Pregunta 9.

Caractersticas de los fenmenos histricos.

Conviene NO confundir a estos pacientes (en donde los sntomas


son involuntarios y no existe enfermedad fsica real) con los pacientes
PSICOSOMTICOS (que presentan una enfermedad fsica real e invoM exico A rgentina
C hile U ruguay

Seguimiento a distancia

luntaria, en donde los factores psicolgicos influyen en su origen o


evolucin) ni con los pacientes que fingen VOLUNTARIAMENTE tener
una enfermedad. Dentro de estos ltimos diferenciamos los simuladores, en los que existe una motivacin evidente de tipo econmico o
legal, de los trastornos facticios, en los que la motivacin parece ser de
tipo psicolgico, la necesidad de asumir la identidad de enfermo,
incluso a costa de perjuicios para su salud o su economa. La forma ms
espectacular de trastorno facticio es el sndrome de Mnchausen, en el
que los sntomas inventados y los signos autoprovocados abarcan diferentes rganos y sistemas; la mayora de los facticios, sin embargo, son
formas limitadas a sntomas ms concretos, siendo ms difciles de detectar, pues los pacientes se preocupan de no ser descubiertos y de
parecerse lo ms posible a la enfermedad real. Desde el punto de vista
legal la simulacin es un delito evidente; los trastornos facticios pueden
convertirse en un problema legal cuando los pacientes provocan la
enfermedad en terceras personas (hijos, esposo) para poder asumir la
identidad de cuidador (trastornos facticios por poderes). En la mayora de estos pacientes encontramos graves alteraciones de la personalidad, siendo refractarios al tratamiento.
Pregunta 10.-R: 5
En los trastornos disociativos los pacientes presentan sntomas psicolgicos (NO fsicos) extraos (cambios de comportamiento, amnesias, viajes sin sentido) que no encajan en las enfermedades psiquitricas o neurolgicas habituales y se comportan de forma extraa ante los tratamientos. Como en el caso de los pacientes conversivos,
es frecuente encontrar un factor estresante relacionado temporalmente con el sntoma y una tendencia a la desaparicin espontnea
y a la recurrencia. Los sntomas disociativos pueden aparecer en el
contexto de otras enfermedades psiquitricas, destacando su asociacin con los trastornos por estrs (agudo y postraumtico) y con
el trastorno de personalidad lmite. En los ltimos aos se insiste en
la frecuencia de antecedentes de abuso sexual en la infancia de
estos pacientes. Dependiendo de la cultura del pas encontramos
diferentes formas de trastornos disociativos; as, las formas ms frecuentes en Espaa parecen ser los episodios de amnesia, siendo
excepcionales las fugas y anecdticos los casos de alteraciones disociativas de la personalidad (personalidad mltiple); en pases menos
desarrollados veremos numerosos casos de trances y posesiones disociativas, y en EE.UU. llama la atencin la elevada frecuencia de
casos referidos con alteraciones disociativas de la personalidad.
La clasificacin DSM (no la CIE-10) incluye dentro de los trastornos disociativos a los cuadros de despersonalizacin y desrealizacin
crnicas; no hay que olvidar que estos sntomas de extraeza ante
uno mismo o el entorno pueden aparecer en numerosas enfermedades psiquitricas (trastornos por ansiedad, sobre todo en crisis,
sndromes depresivos, psicosis agudas, intoxicaciones por drogas) y en
algunos trastornos neuropsiquitricos, sobre todo los que afectan al
lbulo temporal (epilepsia parcial compleja, tumores, encefalitis); los
casos en donde aparecen de forma aislada y con evolucin crnica
son excepcionales.
Pregunta 11.-R: 3
Las benzodiacepinas (BZD) poseen un sistema de receptores propios (receptores benzodiacepnicos u omega), con al menos dos subtipos (omega-1 y omega-2); estos receptores estn formados por diversas subunidades proteicas, para las cuales existen numerosas variantes, sin que se conozca todava la importancia funcional de cada una
de ellas. Desde el punto de vista farmacodinmico, todas las BZD
hacen lo mismo: se unen a su receptor y modifican la afinidad del
receptor gabargico de tipo A por el GABA, aumentando su actividad
(modificacin alostrica positiva o potenciacin). Las diferentes BZD
tendrn mayor o menor potencia, pero a las dosis adecuadas todas
comparten los efectos anticonvulsivantes, ansiolticos, hipnticos y
miorrelajantes. Sus principales diferencias son farmacocinticas (velocidad de absorcin, semivida de eliminacin, presencia o no de
metabolizacin heptica).
Pregunta 12.-R: 2
Una de las ventajas de que las BZD tengan un sistema propio de
receptores ha sido la posibilidad de disear un antagonista especfico

CTO Medicina C/ Nez de Balboa, 115 28006 MADRID (Espaa) Tfno.: (91) 782 43 32 / Fax: (91) 782 43 27
E-mail: secretaria@ctomedicina.com; iberocto@ctomedicina.com WEB: www.ctomedicina.com; www.iberocto.com

PQ Pg. 3

PSIQUIATRA

Seguimiento a distancia

Preparacin Examen de Seleccin 05/06 1 Vuelta

Los antipsicticos tienen un riesgo cardiolgico considerablemente menor, dependiendo sus efectos en sobredosis de otras caractersticas (bloqueo histaminrgico, bloqueo alfa-adrenrgico, efectos anticolinrgicos) que varan enormemente de unos a otros; en el caso de
presentarse graves efectos extrapiramidales agudos, el tratamiento de
eleccin son los anticolinrgicos (biperideno, trihexifenidilo); en el
caso de un exceso de accin anticolinrgica se puede recurrir al uso
de fisostigmina (anticolinestersico de accin corta).
Dentro de los estabilizadores del humor, el litio es el ms peligroso
en el caso de sobredosis, aunque la principal causa de intoxicacin
por litio no va a ser la toma voluntaria o accidental de un exceso de
medicacin, sino las interacciones con frmacos que alteran su eliminacin renal o la concurrencia de problemas hidroelectrolticos que
tienen el mismo efecto; el tratamiento de eleccin en caso de intoxicacin grave es la hemodilisis.
Pregunta 13.-R: 5
La mayora de las BZD se eliminan por va heptica mediante un
doble mecanismo: metabolizacin y conjugacin; el primero de estos procesos se afecta gravemente en el caso de hepatopata, permaneciendo el segundo casi inalterado. Por eso las BZD, que slo se
conjugan, sin metabolizarse en el hgado, son las de eleccin en el
caso de hepatopata (loracepam, oxacepam, temacepam); estas BZD
tienen una semivida de eliminacin corta, cercana a las 8 horas.

Pregunta 13. Ventajas e inconvenientes de las benzodiacepinas


en funcin de su vida-media.

1234567324

123435678
2688
34686

836234364
83
 2
16882
48
284
8688
34686
24
32
42 2
1
6882
48
34

48
32364
23
332 22
4
4 2!38 8!
43"268 8 4


8
48
23

12342

56372

9
4 8

4

2

7

2

7

2

7

7

12

7

2

Pregunta 14.-R: 1
Existen otras BZD, de alta potencia, que tampoco se metabolizan de
forma sustancial en el hgado (alprazolam, triazolam, midazolam) y cuya
semivida de eliminacin es quizs demasiado corta; en el caso de triazolam y midazolam es de menos de 6 horas por lo que no se usan como
ansiolticos; triazolam se utiliza para tratar el insomnio de conciliacin
(siempre en tandas cortas de pocas semanas para no producir tolerancia y dependencia) y midazolam se usa como preanestsico y en pro-

Pg. 4 PQ

M exico A rgentina
C hile U ruguay

cedimientos instrumentales potencialmente desagradables (cateterismos,


colonoscopias) para relajar a los pacientes.

Pregunta 14. Alternativas a las benzodiacepinas en sus


diferentes indicaciones.

4 59256 7
12324567829
8 27 478265
46 87827486 9724424
46 878  8 27
4!2983
4!28 9265
12324 86 8 27
4!59926
46 8 87 53
68 27
46 878274"456 878  8 274756 7
1232438295#56 7 418 926$5865
4%6258 59
4%68 2
65
4& 8245928 2
1232
56 8 2697856 7 415535 865
4'556 865
4(532 8)865
4*28535 2
Pregunta 15.-R: 4
Clasificamos los sndromes afectivos en funcin de dos criterios: su
gravedad y su duracin. La gravedad se evala con diferentes escalas
de sntomas (depresivos o manacos), aunque el principal criterio de
gravedad ser la repercusin que el sndrome clnico tenga sobre el
funcionamiento del paciente. Atendiendo a la gravedad, diferenciamos los sndromes depresivos o manacos en MAYORES y MENORES.
Para que un sndrome depresivo mayor sea considerado relevante se
exige una duracin mnima de 2 semanas (EPISODIO DEPRESIVO
MAYOR), mientras que en el caso de la mana la duracin se reduce a
1 semana o menos, si precisa hospitalizacin por su intensidad (EPISODIO MANACO). En los sndromes menores se exige una mayor
duracin, debiendo persistir los sntomas depresivos 2 aos para aceptar el diagnstico de DISTIMIA; en los sndromes manacos s se acepta
la importancia de los cuadros menores de duracin recortada, diagnosticndose de HIPOMANA a los sndromes manacos menores que
duran al menos 4 das; la alternancia de sntomas hipomanacos y
sntomas depresivos menores durante un perodo de al menos 2 aos
recibe el diagnstico de CICLOTIMIA.
Los cuadros depresivos son mucho ms frecuentes que los manacos, encontrndose en los primeros un claro predominio femenino,
mientras que en los segundos hay una cierta igualdad. De igual forma
los sndromes episdicos (de semanas o meses de duracin) son ms
frecuentes que los sndromes crnicos (de aos de duracin). De
ah que el trastorno afectivo ms frecuente sea el episodio depresivo
mayor.
La melancola (o depresin endgena) es una variante de episodio
depresivo mayor que se identifica por la presencia de unos sntomas
caractersticos; por tanto, todos los casos de melancola son episodios
depresivos mayores, pero tan slo una pequea parte de los episodios
depresivos mayores presentan sntomas melanclicos.
Pregunta 16.-R: 4
Los episodios depresivos mayores (EDM) reciben diferentes adjetivos calificativos en funcin de distintas caractersticas:
Segn la gravedad de los sntomas, podemos ver EDM de intensidad leve, moderada o severa; las formas ms graves de EDM llegan
a presentar sntomas psicticos (delirios sobre todo), por lo que se
habla de EDM con sntomas psicticos o depresin psictica (o
depresin delirante).
Segn el patrn de sntomas tenemos EDM con sntomas melanclicos (o depresin endgena) y EDM con sntomas atpicos; sin
embargo la mayora de los EDM no encajan en ninguno de estos
dos patrones, presentando sntomas inespecficos; parece que con

CTO Medicina C/ Nez de Balboa, 115 28006 MADRID (Espaa) Tfno.: (91) 782 43 32 / Fax: (91) 782 43 27
E-mail: secretaria@ctomedicina.com; iberocto@ctomedicina.com WEB: www.ctomedicina.com; www.iberocto.com

Comentarios TEST

capaz de revertir sus efectos en el caso de sobredosis; el flumacenilo es


el tratamiento de eleccin en las intoxicaciones por BZD y se ha
convertido, junto con la naloxona (antagonista de los receptores opioides y tratamiento de eleccin en las intoxicaciones por opiceos), en
una medicacin bsica de los servicios de urgencias. En el caso del
resto de psicofrmacos carecemos de antdotos especficos. Las sobredosis de antidepresivos varan en su gravedad en funcin del tipo
de antidepresivo:
Los ISRS son muy seguros, pues carecen de cardiotoxicidad y sus
efectos sobre el sistema nervioso son escasos.
Los IMAOs ocupan un lugar intermedio.
Los antidepresivos tricclicos son los ms peligrosos, al producir
alteraciones de la conduccin cardaca potencialmente letales.

PSIQUIATRA

Preparacin Examen de Seleccin 05/06 1 Vuelta


las recadas sucesivas de la enfermedad los sntomas van adquiriendo un carcter ms endgeno y pueden alcanzar con mayor
facilidad una gravedad psictica; lo mismo parece ocurrir en los
EDM que debutan en la edad avanzada. Tambin podemos ver un
EDM con sntomas catatnicos (o depresin estuporosa).
Cuando el inicio del EDM guarda una relacin con el puerperio se
habla de EDM de inicio puerperal o depresin puerperal (o depresin posparto).
La depresin neurtica (trmino clsico) se corresponde ms o
menos con la distimia de la clasificacin actual, no con los EDM.
Pregunta 17.-R: 5
Los sntomas depresivos atpicos incluyen tres caractersticas que, en
teora, predeciran una mala respuesta a medicacin antidepresiva:
1. La presencia de rasgos patolgicos de personalidad, en la lnea
dependiente o histrinica.
2. La existencia de un factor estresante asociado al inicio de los sntomas (frecuentemente un problema sentimental).
3. La ausencia de sntomas melanclicos, observndose una inversin de los sntomas biolgicos (aparecen somnolencia excesiva y
aumento del apetito).

Comentarios TEST

Sin embargo se obtiene una respuesta razonable al tratamiento con


IMAOs, no con tricclicos. Esta selectividad farmacolgica parece estar
relacionada con una mayor frecuencia de alteraciones serotoninrgicas en estos pacientes (los IMAOs tienen un efecto ms serotoninrgico
que noradrenrgico). En general, los sntomas atpicos no implican una
gravedad excesiva; de hecho, podemos ver sntomas atpicos tanto en
los EDM como en las distimias. En cambio los sntomas melanclicos s
se asocian a una mayor gravedad (y a un mayor riesgo de suicidio, por
tanto); en estos pacientes melanclicos se encuentran ms alteraciones
neurobiolgicas, por lo que su respuesta a los tratamientos biolgicos es
superior (tanto a antidepresivos como a electrochoque).
Pregunta 18.-R: 5
En los ancianos no es raro que se oculte una depresin tras una
serie de quejas fsicas o de memoria, lo cual en parte puede reflejar la
mayor preocupacin por estos sntomas en este grupo de edad. Tambin es destacable la mayor frecuencia de sntomas psicomotores
(inhibicin, agitacin) y de sntomas psicticos; se han relacionado
ambos sntomas con alteraciones en los ganglios basales, que podran
deberse al deterioro cerebral asociado a la edad o a la presencia de
factores de riesgo cerebrovascular. La presencia de sntomas psicticos implica la necesidad de modificar el tratamiento, aadiendo antipsicticos a los antidepresivos o recurriendo directamente al electrochoque. No hay que olvidar que la edad es por s misma un factor que
aumenta el riesgo de suicidio, lo cual, aadido a la presencia de un
sndrome depresivo y a la gravedad psictica, que con frecuencia
alcanza explica el enorme riesgo de suicidio que tienen las depresiones graves del anciano.
En los nios, los sndromes depresivos van a ocultarse tras quejas
fsicas (los nios pequeos carecen de lenguaje emocional y tienden a expresarse somticamente) o problemas escolares; en los adolescentes, los trastornos de conducta, incluido el abuso de sustancias,
pueden enmascarar una depresin. No hay que olvidar que, en nios
y adolescentes el estado de nimo predominante durante una depresin va a ser con frecuencia irritable ms que triste, lo que dificultar
el diagnstico.
Pregunta 19.-R: 3
Dentro de los sndromes depresivos, los sntomas fsicos son los ms
fciles de diagnosticar, pues tienen un carcter objetivo; sin embargo
carecen de especificidad diagnstica al presentarse tambin en casi
todas las enfermedades psiquitricas y mdicas; por esto, en los pacientes con enfermedades psiquitricas o mdicas coexistentes, habr
que recurrir a sntomas ms subjetivos (estado de nimo, pensamientos) para asegurar el diagnstico. La alteracin del sueo ms frecuente en una depresin es el insomnio de caractersticas inespecficas
(problemas de conciliacin con despertares nocturnos); cuando aparece tendencia al despertar precoz sugiere una depresin melanclica
M exico A rgentina
C hile U ruguay

Seguimiento a distancia

(EDM con sntomas melanclicos o depresin endgena), sobre todo


si se asocia con una variacin diurna del estado de nimo en la forma
de empeoramiento matutino y mejora vespertina. Los pacientes con
sntomas atpicos suelen referir una desagradable necesidad de dormir durante el da, quejndose adems de un cansancio extremo. En
lo que hace referencia al apetito y al peso, lo ms frecuente es ver una
disminucin de ambos; en los pacientes atpicos habr aumento del
apetito y del peso, con una tendencia a comer mayores cantidades de
hidratos de carbono (dulces) de lo habitual para el paciente.
Pregunta 20.-R: 4
Solamente en las depresiones ms graves el estado de nimo permanece inalterable todo el da, con una ausencia de reactividad ante
lo que le rodea (anestesia emocional). En los cuadros melanclicos el
paciente puede encontrarse algo mejor por la noche (mejora vespertina), pero lo que s va a referir con frecuencia es que est mucho peor
por la maana, nada ms despertarse (empeoramiento matutino); la
asociacin entre despertar precoz y empeoramiento matutino es altamente especfica de melancola (o depresin endgena), y se ha relacionado con alteraciones en los marcapasos endgenos pues sigue
un ritmo muy parecido al ritmo del cortisol (que con frecuencia est
alterado en estos pacientes). En los sndromes depresivos menores los
pacientes conservan cierta capacidad de mejorar cuando las circunstancias externas les son favorables (preservan la reactividad); as tienden a notarse mejor cuanto ms activos estn, lo que puede dar el
aspecto de una mejora matutina, opuesta a la melanclica; sin embargo en estos casos la variacin del humor NO tiene un carcter
neurobiolgico, sino psicosocial.
La respuesta selectiva a IMAOs frente a tricclicos es una de las
caractersticas de las depresiones atpicas.
La asociacin entre depresin y estaciones no es tan frecuente como
se cree popularmente; tan slo un 15% de los pacientes con enfermedades afectivas recurrentes (depresivas o bipolares) presentan un ritmo
estacional claro, que se repite de ao en ao. Lo habitual en estos casos
es que los EDM aparezcan en otoo y primavera, y los episodios manacos en verano. En los pases nrdicos se ha descrito una variacin del
ritmo estacional en los pacientes bipolares: habra EDM en invierno
(frecuentemente con sntomas atpicos) y episodios manacos en verano (en realidad son ms frecuentemente hipomanacos, es decir, bipolares-2). Este cambio se ha puesto en relacin con la duracin del
fotoperodo (muy corta en invierno, muy larga en verano), proponindose el tratamiento con fototerapia con carcter preventivo o curativo.
Pregunta 21.-R: 2
Uno de los problemas ms frecuentes ante el diagnstico de depresin es la tendencia casi automtica a buscar un factor psicosocial
estresante que consideramos desencadenante. Para empezar, no
hay que olvidar que muchas personas se enfrentan a diario a situaciones estresantes muy similares, pero slo una parte de ellas desarrollan
un sndrome depresivo; hay una serie de factores individuales que
aumentan el riesgo de presentar una depresin, entre los que destaca
la presencia de antecedentes familiares, los antecedentes personales
de depresin y determinados rasgos de personalidad. Los factores
estresantes parece que actan de una forma inespecfica, aumentando la probabilidad de presentar un sndrome psiquitrico en funcin de la predisposicin individual. Adems no existe una buena
correlacin entre la presencia o ausencia de un factor estresante y la
duracin, la intensidad o las caractersticas del sndrome depresivo
asociado; la mitad de los pacientes con EDM no refieren acontecimiento estresante alguno en las semanas previas al inicio de su depresin, y su evolucin no es diferente de la evolucin de los pacientes
que s refieren un acontecimiento causal. Hoy en da se reserva el
trmino trastorno adaptativo con sntomas depresivos para aquellos pacientes que presentan sntomas depresivos de intensidad menor en respuesta a una situacin estresante que produce sntomas
similares en la mayora de las personas; estos sntomas se resuelven
espontneamente en muchos casos y no suelen precisar tratamiento
farmacolgico, salvo que su intensidad aumente; correspondera con
el concepto de depresin reactiva, aunque podemos ver depresiones reactivas (es decir, relacionadas con problemas vitales) de intensidad tal que permita el diagnstico de EDM.

CTO Medicina C/ Nez de Balboa, 115 28006 MADRID (Espaa) Tfno.: (91) 782 43 32 / Fax: (91) 782 43 27
E-mail: secretaria@ctomedicina.com; iberocto@ctomedicina.com WEB: www.ctomedicina.com; www.iberocto.com

PQ Pg. 5

PSIQUIATRA

Seguimiento a distancia

Preparacin Examen de Seleccin 05/06 1 Vuelta

123456789
7

213249 6

123456

78696
86 886 56

4584 4 

524 

649

6 4 

66 46 56

524 

Pregunta 22.-R: 2
En los sndromes afectivos ms graves los pacientes pueden presentar sntomas psicticos; en general estos sntomas son ms probables
en los cuadros manacos que en los depresivos; los delirios son mucho ms frecuentes que las alucinaciones, por lo que usamos los
trminos depresin (o mana) psictica o delirante de forma similar.
En la mayora de los pacientes con estos sntomas encontramos una
relacin lgica entre el contenido de los delirios y alucinaciones y su
estado de nimo (sntomas psicticos congruentes, ideas deliroides o
ideas delirantes secundarias); si estn deprimidos, su visin pesimista
se extiende hacia el pasado (ideas de culpa), el presente (ideas de
minusvala y enfermedad) y el futuro (ideas de ruina); si estn manacos, su optimismo y autoconfianza puede llegar a los delirios de grandeza. En algunos casos los delirios y alucinaciones se alejan de los
contenidos lgicos (delirios de perjuicio, delirios de control), planteando el diagnstico diferencial con la esquizofrenia; no hay un
acuerdo acerca de la importancia de estos sntomas psicticos incongruentes con el estado de nimo, llegando incluso a proponer la
Clasificacin Internacional de Enfermedades el diagnstico de trastorno esquizoafectivo para estos pacientes (la DSM americana slo acepta este diagnstico cuando los pacientes cumplen POR COMPLETO
tanto los criterios de esquizofrenia como los del trastorno afectivo).

de fase (a diferencia de los pacientes depresivos, en donde se mantiene el mismo frmaco 6 meses a la misma dosis). Sin embargo, vemos
pacientes que tienen respuestas maniformes al tratamiento con antidepresivos sin que hayan presentado nunca episodios manacos espontneos; oficialmente no se pueden catalogar como bipolares, pero
debera tenerse en cuenta en ellos la posibilidad de tratamiento de
prevencin de recadas con litio.
Pregunta 24.-R: 5
A mediados del siglo XX, se pudo diferenciar la enfermedad bipolar
de la enfermedad depresiva atendiendo a la mayor frecuencia de antecedentes familiares afectivos en la primera; de hecho, hay autores que
ante un paciente depresivo con antecedentes familiares de enfermedad
bipolar proponen el tratamiento profilctico con litio, entendiendo
que ser cuestin de tiempo el que presenten un episodio maniforme.
Otras diferencias entre la enfermedad bipolar y la depresiva son de tipo
epidemiolgico (predominio de mujeres en la enfermedad depresiva,
inicio ms temprano en la enfermedad bipolar, mayor riesgo [relativo]
de suicidio en la enfermedad bipolar). Se han realizado numerosas
comparaciones entre las recadas depresivas de ambas enfermedades;
parece que las depresiones bipolares tienden a ser algo ms cortas que
las unipolares (6 meses frente a 12 meses), aunque con una mayor
frecuencia de recidivas a lo largo de la vida (6-9 frente a 2-3). En general,
los sntomas son muy parecidos, aunque se describen ms sntomas
atpicos en las formas bipolares (sobre todo en las bipolares-2) y una
mayor tendencia a la agitacin psicomotora en las depresiones
unipolares. No hay que olvidar que en la enfermedad bipolar podemos ver cualquier variante de EDM (melanclico, atpico, psictico,
catatnico) y diferentes formas de episodios manacos (mana clsica
eufrica, mana atpica disfrica, hipomana, episodios mixtos).

Pregunta 24. Espectro bipolar.


Pregunta 22. Escalera de intensidad de la depresin y adaptacin del tratamiento.

Pregunta 23.-R: 3
La respuesta habitual de un paciente deprimido al tratamiento con
antidepresivos es la vuelta a la normalidad. De hecho, si una persona
que no est deprimida toma estos medicamentos NO va a notar un
aumento del estado de nimo, sino simplemente efectos secundarios
(los antidepresivos NO son euforizantes y los euforizantes, como la
cocana, NO tienen efecto antidepresivo). En algunos casos los pacientes sufren un cambio radical de su estado de nimo en respuesta
al tratamiento, alcanzando una intensidad hipomanaca o manaca a
gran velocidad. Esta respuesta tan rpida es tpica de los pacientes
bipolares, recomendndose en ellos la retirada del frmaco antidepresivo en cuanto se consigue la normalizacin para evitar el cambio

Pg. 6 PQ

M exico A rgentina
C hile U ruguay

Pregunta 25.-R: 3
La mayora de los pacientes bipolares debutan con un EDM, siendo de especial inters la posibilidad de predecir una evolucin bipolar
desde el primer episodio. El principal factor de riesgo para ser bipolar
es tener antecedentes familiares de esa enfermedad. En las mujeres un
EDM puerperal evolucionar hacia una enfermedad bipolar en cerca
del 80% de los casos. El debut en la adolescencia tiende a ser ms
frecuente en las formas bipolares (pero NO es determinante). La respuesta anormalmente rpida al tratamiento antidepresivo y la aparicin de cuadros maniformes secundarios al mismo son tambin dos
caractersticas que sugieren bipolaridad. Aunque algunos trabajos encuentran una mayor frecuencia de ideacin delirante en las formas
bipolares, NO se ha podido comprobar que esa ideacin tenga en las
formas bipolares un carcter incongruente con ms frecuencia que
en las formas unipolares.

CTO Medicina C/ Nez de Balboa, 115 28006 MADRID (Espaa) Tfno.: (91) 782 43 32 / Fax: (91) 782 43 27
E-mail: secretaria@ctomedicina.com; iberocto@ctomedicina.com WEB: www.ctomedicina.com; www.iberocto.com

Comentarios TEST

Pregunta 21. Relacin entre los distintos tipos de depresin


y los rasgos de personalidad previos.

PSIQUIATRA

Preparacin Examen de Seleccin 05/06 1 Vuelta


Cuando los pacientes debutan con un episodio manaco, el diagnstico es inmediato; los varones tienen una mayor tendencia a presentar este tipo de comienzo, lo que plantea problemas de diagnstico con la esquizofrenia (varones, jvenes, conducta desorganizada y
agitada, delirios y alucinaciones) y las conductas disociales (varones,
jvenes, conductas arriesgadas, abuso de sustancias).

Comentarios TEST

Pregunta 26.-R: 3
Actualmente se aceptan dos variantes mayores de trastorno afectivo bipolar (TAB). En el TAB-1, los pacientes tienen episodios manacos
o mixtos, asociados casi siempre a episodios depresivos (EDM o depresiones menores); como se ve, cabe la posibilidad de que un paciente tenga slo episodios manacos o mixtos, pero es algo excepcional y no parece que evolucionen de forma diferente a los pacientes
manaco-depresivos. En el TAB-2 veremos EDM junto a episodios hipomanacos; estas hipomanas son difciles de diagnosticar y no es
raro que estos pacientes se clasifiquen errneamente como depresivos, privndoles del tratamiento con estabilizadores, lo que explica
su peor pronstico a largo plazo (mayores tasas de suicidio, ms tendencia a la ciclacin rpida); en los pacientes con TAB-2 vemos un
ligero predominio femenino, mientras que en los TAB-1 hay un equilibrio entre sexos. Se acepta slo una forma menor de trastorno bipolar,
la ciclotimia, definida por la alternancia de sntomas hipomanacos y
sntomas depresivos menores durante un perodo mnimo de 2 aos.
La alternancia de episodios depresivos invernales y episodios maniformes en verano caracteriza a la forma estacional nrdica del TAB;
suelen ser TAB-2 (EDM ms hipomana).
La aparicin de un EDM sobre una distimia previa se conoce a
veces como depresin doble. Implica una mayor gravedad que los
EDM aislados, con menor tendencia a la recuperacin.
Pregunta 27.-R: 4
En Psiquiatra diferenciamos dos conceptos referidos al suicidio
que popularmente se mezclan de forma inadecuada. En primer
lugar hablamos de SUICIDIO cuando una persona se autolesiona de
forma voluntaria y deliberada con la intencin de acabar con su vida.
Por otra parte, hablamos de AUTOLESIN (sin ms) cuando una persona se provoca algn tipo de dao fsico sin que existiera un deseo
explcito de muerte. Estas autolesiones son mucho ms frecuentes que
los verdaderos intentos de suicidio (100 veces o ms) y en general se
pueden dividir en aquellas que tienen una finalidad MANIPULADORA (distraer o llamar la atencin del entorno, conseguir algn tipo de
beneficio) y aquellas que tienen un carcter ms IMPULSIVO (como
las reacciones en cortocircuito de los pacientes con personalidad lmite). Ante cualquier autolesin no accidental debe solicitarse una valoracin psiquitrica que determinar la intencionalidad de la misma.
La mayora de las autolesiones no suicidas se presentan en jvenes,
sobre todo mujeres, que se enfrentan a una situacin estresante ms o
menos grave, careciendo de otros recursos psicolgicos para abordarla (con frecuencia hay rasgos patolgicos de personalidad); suelen
elegirse mtodos de baja letalidad (intoxicacin con medicamentos,
cortes con armas blancas) y no se prepara el acto con antelacin,
siendo relativamente fcil el rescate (a veces se llevan a cabo en presencia de la propia familia); tras su atencin mdica en Urgencias y la
valoracin psiquitrica se les remite a los dispositivos ambulatorios,
siendo excepcional la necesidad de ingreso. En cambio, la mayora
de los intentos de suicidio se dan en personas mayores de 50 aos
(una excepcin notable son los intentos de suicidio que presentan los
jvenes con esquizofrenia), varones, que sufren enfermedades psiquitricas bien definidas (depresin, abuso de sustancias,
esquizofrenia), asociadas o no a otros factores adversos de tipo mdico (enfermedades crnicas, dolorosas, incapacitantes) o social (viven
solos, han perdido una relacin significativa recientemente, pasan
por dificultades econmicas); suelen utilizarse mtodos ms agresivos
(precipitacin desde gran altura, ahorcamiento, armas de fuego) y se
planifica mejor el acto, evitando ser descubiertos; si sobreviven al
intento de suicidio, suelen ingresar y es necesario adoptar una serie de
precauciones para que no vuelvan a intentar suicidarse durante el
ingreso (sedacin, contencin mecnica, vigilancia constante).
Sin embargo, conviene tener en cuenta que algunos libros hablan
de suicidio consumado para referirse a los pacientes autnticamente
M exico A rgentina
C hile U ruguay

Seguimiento a distancia

suicidas y de intento de suicidio para referirse a los pacientes que se


autolesionan por otras razones.
Pregunta 28.-R: 2
Se llama depresin secundaria o exgena a todo sndrome depresivo para el que se encuentra una causa mdica, txica o farmacolgica. En la prctica destacaremos 3 causas especialmente frecuentes:
Frmacos y drogas: aquellos frmacos que alteran la funcin noradrenrgica central (beta-bloqueantes, reserpina, alfa-metil-DOPA,
clonidina), frmacos con actividad esteroidea (estrgenos, progestgenos, corticoides, ACTH), intoxicacin crnica por drogas sedantes (alcohol, opiceos, benzodiacepinas) y abstinencia de drogas estimulantes (cocana, anfetaminas).
Enfermedades endocrinolgicas: sobre todo el hipotiroidismo, pero
tambin el hipertiroidismo (sobre todo en ancianos), el hiperparatiroidismo o los trastornos adrenales (Cushing, Addison).
Enfermedades neurolgicas: tanto las enfermedades degenerativas
que afectan de forma ms o menos difusa a todo el Sistema Nervioso Central (Alzheimer, Parkinson) como algunas enfermedades localizadas (tumores, accidentes vasculares, abscesos) que lesionan
sobre todo las regiones frontales.
La isoniacida fue, en realidad, el primer medicamento antidepresivo utilizado en Psiquiatra, a principios de los aos 50, dando origen
a los primeros IMAOs; por tanto, comparte con ellos la capacidad de
producir cuadros maniformes.
Pregunta 29.-R: 5
La utilidad de los antidepresivos va mucho ms all de los sndromes depresivos, de ah que en la actualidad se prefiera denominarles
por su mecanismo de accin (ISRS, IMAOs, inhibidores no selectivos
de la recaptacin).

Pregunta 29. Otros usos de los antidepresivos.


En Psiquiatra:
Trastornos de ansiedad:
- Pnico: ISRS, ADT, IMAOs.
- TOC: ISRS, ADT (clomipramina).
- Fobia social: IMAOs, ISRS.
- TEPT: ISRS, IMAOs, ADT.
- TAG: IRSN, ISRS, ADT.
Trastornos somatomorfos:
- Hipocondra / dismorfofobia.
- Somatizacin / dolor.
Trastornos psicticos:
- Depresin postesquizofrenia.
Trastornos por uso de sustancias:
- Abstinencia de la cocana: ADT (desipramina).
- Prevencin de la neurotoxicidad por xtasis (MDMA): ISRS.
Trastornos infantiles:
- Dficit de atencin: ADT.
- Enuresis: imipramina.
- Mutismo selectivo: ISRS, IMAOs?
Trastornos del sueo:
- Insomnio crnico: ADT sedantes, mirtazapina.
- Narcolepsia (catapleja): ADT, IMAOs, ISRS?
Trastornos de la alimentacin:
- Bulimia: ISRS en dosis altas, IMAOs?
- Obesidad: sibutramina, ISRS?
Trastornos de la personalidad:
- Lmite: ISRS.
Fuera de la Psiquiatra:
Dolor crnico neuroptico: ADT (amitriptilina, clomipramina).
Fibromialgia reumtica: ADT (amitriptilina).
Cefaleas:
- Migraas.
- Cefaleas tensionales (amitriptilina).

CTO Medicina C/ Nez de Balboa, 115 28006 MADRID (Espaa) Tfno.: (91) 782 43 32 / Fax: (91) 782 43 27
E-mail: secretaria@ctomedicina.com; iberocto@ctomedicina.com WEB: www.ctomedicina.com; www.iberocto.com

PQ Pg. 7

PSIQUIATRA

Preparacin Examen de Seleccin 05/06 1 Vuelta

Dentro de los trastornos psiquitricos van a usarse antidepresivos en


casi todos los trastornos por ansiedad (en el pnico para prevenir las
crisis, en la fobia social grave, en el trastorno obsesivo-compulsivo, en el
trastorno por estrs postraumtico y, recientemente, en el trastorno por
ansiedad generalizada). Tambin los utilizamos para frenar la impulsividad en pacientes con trastornos de la conducta alimentaria (atracones
bulmicos), trastornos del control de los impulsos (juego patolgico o
ludopata), personalidades tipo lmite o trastornos por abuso de sustancias (alcoholismo). Dentro de estos ltimos, los antidepresivos van a
tener cierta capacidad de aliviar el sndrome de abstinencia de la cocana y parece que protegen de la neurotoxicidad inducida por xtasis
y otras anfetaminas de diseo. En algunos trastornos psiquitricos propios de la infancia (enuresis, trastorno por dficit de atencin) pueden
utilizarse, aunque no sean el tratamiento de primera eleccin. Por la
capacidad sedante de algunos antidepresivos se proponen stos como
alternativa a las benzodiacepinas en algunos casos de insomnio.
Ya en un terreno ms neuropsiquitrico, vemos cmo los antidepresivos tricclicos se utilizan para la prevencin de la cefalea tensional y el tratamiento de dolores neuropticos crnicos. Tienen adems
la capacidad de reducir la frecuencia de episodios de catapleja en la
narcolepsia. El efecto antihistamnico de la doxepina supera a la mayora de los antihistamnicos oficiales, y no es raro que se use para el
tratamiento del prurito idioptico.
Pregunta 30.-R: 3
A la hora de enfrentarnos al tratamiento de un sndrome depresivo,
vamos a decidirnos por un tipo de tratamiento u otro en funcin de la
intensidad de los sntomas. Cuando un sndrome depresivo se acerca
a la intensidad de EDM, la probabilidad de respuesta a frmacos
antidepresivos (AD) supera el 60%, siendo mayor an en los casos de
EDM con sntomas melanclicos. En las formas extremas de depresin (depresin psictica) ser necesario combinar AD con antipsicticos o recurrir al tratamiento con electrochoque. En las formas menores de depresin la respuesta a frmacos ronda el 40%, logrndose
ms el alivio sintomtico que la curacin completa.
Una vez decidida la necesidad de poner un AD, la eleccin entre
los distintos compuestos depender de:
Los antecedentes de respuesta en episodios previos.
La presencia de otras enfermedades que contraindiquen el uso de
un AD en concreto (por ejemplo, una cardiopata contraindicara
el uso de AD tricclicos).
La toma de alguna medicacin que interaccione con el AD.
El perfil de efectos adversos del AD que puedan provocar su rechazo por el paciente.
Los antecedentes de respuesta a un AD en concreto en un familiar
de primer grado.
El patrn de sntomas dentro de la depresin (por ejemplo, la presencia de un patrn atpico indicara el uso de IMAOs).
La comodidad de su uso.
El precio del frmaco.

todo este proceso de tratamiento se simplifica, pues la dosis inicial suele


ser la dosis suficiente (no hay que subir poco a poco, para que el
paciente se acostumbre a los efectos adversos como con los AD tricclicos),
tanto para el tratamiento agudo y de continuacin, como para el tratamiento de mantenimiento a largo plazo.
Se discute si alguno de los AD ms modernos (venlafaxina o
mirtazapina) tendran un efecto algo ms rpido.
Pregunta 32.-R: 5
Los AD tricclicos (ADT) son los AD ms antiguos y, por tanto, aquellos de los que se dispone de ms experiencia. De hecho, la imipramina se considera el AD patrn con el que deben compararse los dems
antidepresivos. Los ADT logran su efecto antidepresivo aumentando
la disponibilidad de las principales monoaminas (noradrenalina, serotonina y dopamina) al inhibir su recaptacin. Sin embargo no lo
hacen de una forma selectiva, sino que tambin bloquean numerosos
receptores, cada uno de los cuales va a producir una serie de efectos
secundarios. No todos los ADT son iguales en esta caracterstica; los
hay con potentes efectos anticolinrgicos (amitriptilina, clorimipramina)
y los hay con menos problemas a ese nivel (nortriptilina, desipramina);
estos efectos muscarnicos son muy mal tolerados, especialmente por
pacientes ancianos. Asmismo tenemos ADT muy sedantes por su efecto
antihistamnico (amitriptilina, doxepina, clorimipramina) y otros ms
activadores con escasa capacidad sedante (imipramina). Lo que suele
ser necesario en todos los ADT es comenzar con dosis muy bajas e ir
subiendo poco a poco, para buscar una cierta tolerancia a los efectos
adversos. Un problema adicional de los ADT son sus efectos cardiovasculares; por su capacidad de bloqueo adrenrgico pueden causar
hipotensin ortosttica y por su accin parecida a la quinidina sobre
la conduccin cardaca cabe la posibilidad de que produzcan arritmias
potencialmente letales. La intoxicacin con ADT supone un serio
problema en los pacientes depresivos con ideas suicidas. Sin embargo, todava vamos a ver utilizarlos atendiendo a tres ventajas:
Su precio (7-10 veces inferior al precio de los nuevos AD).
La posibilidad de controlar el cumplimiento a travs de los niveles
plasmticos (sobre todo con imipramina).
Su posible mayor potencia antidepresiva (que se explicara por el
hecho de que la mayora de los ADT actan a un doble nivel:
noradrenrgico y serotoninrgico).

Teniendo en cuenta que la mayora de los pacientes se presentan sin


haber tomado nunca medicacin AD o no la recuerdan, los ISRS renen
las caractersticas idneas para convertirse en los AD de primera eleccin.
Pregunta 31.-R: 3
Los antidepresivos no curan la depresin rpidamente, tienen un
efecto a medio plazo, tardando varias semanas (4-6) en conseguir la
remisin de los sntomas, aunque previamente ya se hayan detectado
signos de respuesta. Una vez alcanzada la remisin, se propone la
necesidad de continuar con el mismo tratamiento (el mismo AD, a la
misma dosis) un mnimo de 6 meses para prevenir la recada del episodio. Pasado ese tiempo (que dependiendo de diversos factores, como la
mayor edad, la gravedad del episodio o el nmero de episodios previos, podr alargarse), se considera que el paciente se ha recuperado
del episodio depresivo. Entonces se podr retirar el AD (casi siempre de
forma progresiva, en unas semanas para minimizar posibles sntomas
abstinenciales), salvo a los pacientes que hayan sufrido numerosos episodios, a los que se les propondr tratamiento indefinido de mantenimiento (que suele realizarse en la dosis mnima posible para reducir los
efectos adversos y mejorar as el cumplimiento). Con los modernos ISRS
Pg. 8 PQ

M exico A rgentina
C hile U ruguay

Pregunta 32. Esquema de efectos de los antidepresivos tricclicos y posibles


efectos secundarios.

Pregunta 33.-R: 1
A principios de los 80, los ISRS irrumpen en el mercado psicofarmacolgico con tres ventajas evidentes:
Un perfil de efectos secundarios mucho ms benfico.
Una mayor comodidad de uso, al no tener que aumentar lentamente la dosis, ni fraccionarla en varias tomas diarias.
La casi ausencia de letalidad en el caso de sobredosis.

CTO Medicina C/ Nez de Balboa, 115 28006 MADRID (Espaa) Tfno.: (91) 782 43 32 / Fax: (91) 782 43 27
E-mail: secretaria@ctomedicina.com; iberocto@ctomedicina.com WEB: www.ctomedicina.com; www.iberocto.com

Comentarios TEST

Seguimiento a distancia

PSIQUIATRA

Preparacin Examen de Seleccin 05/06 1 Vuelta


Pregunta 33. Ventajas y desventajas de los antidepresivos
tricclicos y los ISRS.

12345678694

623  9
123456765727895
572

1



69
623  9
12 2 4 692

22
9 7
7

22 68
7 5

12 65

12 975
7
2 82
5

1289 2
285  2

12726  7

22 7956255 7587

2276 8 24 7665875 89

123672 4 692

12 652  7


22 68
7 5

12!89 76658 2

1236772 975
7
2 82

2247 76658"9567

22
5

12#3456765727895
572

12
5
7
2
2

22 8 $

Comentarios TEST

En apenas 10 aos desplazan casi por completo a los ADT y a los


IMAOs en el terreno de la depresin, y poco a poco van avanzando
hacia otros campos (pnico, trastorno obsesivo, bulimia, fobia social).
En la actualidad se han convertido en los AD de primera eleccin en
la depresin y en la mayora de esas otras indicaciones. Sin embargo,
tienen tres aspectos discutibles:
Su precio es 7-10 veces superior al precio de los ADT, aunque los
estudios de coste-efectividad han demostrado que la diferencia de
precio del frmaco se compensa por otros costes relacionados con
la depresin (das de baja laboral, necesidad de ingreso, necesidad
de medicacin adicional para tratar efectos adversos).
Aparentemente tienen una menor potencia antidepresiva (debida
quizs a su efecto exclusivamente serotoninrgico), aunque los estudios sobre efectividad en condiciones reales (no de ensayo clnico) demuestran que los pacientes tienden a abandonar el tratamiento con ADT o no alcanzan dosis adecuadas por culpa de los
efectos adversos, con lo que la mayor eficacia de los ADT se pierde,
igualndose su efectividad a la de los ISRS.
Van a presentar numerosas interacciones farmacocinticas en los
citocromos P450 hepticos, a los que inhiben en mayor o menor
medida (parece que citalopram y sertralina menos que los dems
ISRS) modificando al alza los niveles plasmticos de diversos frmacos, algunos potencialmente peligrosos por sus efectos cardiotxicos
(como los ADT, algunos antihistamnicos modernos como la terbinafina o el astemizol y algn procintico como el cisapride).
Pregunta 34.-R: 4
Los IMAOs tenan un perfil de tolerancia ligeramente mejor que los
ADT, pero dos importantes problemas prcticos:
La necesidad de seguir una dieta para evitar posibles crisis hipertensivas secundarias a compuestos vasopresores derivados de las
aminas (tiramina).
Algunas interacciones farmacolgicas potencialmente peligrosas por
sus efectos noradrenrgicos (vasoconstrictores nasales) o serotoninrgicos (ISRS, meperidina).
Aunque la dieta no es complicada (bsicamente se trata de evitar
todos los alimentos que han sufrido procesos de fermentacin, como el
queso, las bebidas alcohlicas fermentadas como el vino o la cerveza,
los ahumados, las conservas y los salazones) lentamente se han ido
abandonando; en la actualidad slo nos queda un IMAO clsico en
Espaa (tranilcipromina), aunque en otros pases (Francia, Inglaterra,
EE.UU.) an podemos encontrar otros en uso. Estrictamente hablando,
habra 2 indicaciones oficiales para el uso de IMAOs en primera instancia: la depresin con sntomas atpicos y la fobia social grave. Sin embargo, incluso en esos casos es cada vez ms frecuente el uso de ISRS.
Pregunta 35.-R: 2
El tratamiento electroconvulsivo (TEC) fue la primera estrategia realmente eficaz para el tratamiento de la depresin; se comenz a utilizar
a finales de los aos 30, y hasta la revolucin farmacolgica de los aos
50-60 fue el instrumento preferido para el tratamiento agudo de la
depresin. Tras unos aos de rechazo ms por motivos polticos que
M exico A rgentina
C hile U ruguay

Seguimiento a distancia

por razones cientficas, desde finales de los 80 asistimos a un renacimiento de la TEC. El TEC tiene tres ventajas sobre los frmacos AD:
Es ms potente en su accin antidepresiva, lo que explica su principal indicacin, la depresin resistente a AD, y el que, en ocasiones, se utilice como primera opcin en la forma ms grave de
depresin, la depresin psictica o delirante.
Logra su efecto antidepresivo ms rpido que los frmacos (en 2-3
semanas cuando se aplica a das alternos), lo que explica su utilidad en las depresiones estuporosas o catatnicas y en las depresiones con graves ideas suicidas.
Tiene escasos efectos secundarios fsicos, lo que explica su utilidad
en casos de intolerancia o contraindicacin para el uso de ADT, en
pacientes polimedicados o con patologa orgnica compleja y en las
psicosis graves durante el embarazo.
Slo existe una contraindicacin absoluta para el TEC: la presencia de hipertensin intracraneal (pues durante cualquier crisis epilptica aumenta la produccin de lquido cefalorraqudeo y cabra la
posibilidad de una herniacin cerebral). Otras contraindicaciones
dependen del procedimiento anestsico y cada vez son ms raras.
El principal efecto secundario es de tipo neuropsicolgico: la produccin de una amnesia antergrada. Depende del estado cognitivo
previo (ms frecuente en ancianos), del nmero de sesiones recibidas,
de la carga elctrica suministrada y de la disposicin de los electrodos.
Al cabo de unos meses NO existen secuelas del tratamiento (salvo
ciertas lagunas de memoria consecuencia del perodo de amnesia) y
el funcionamiento cognitivo de los pacientes es incluso mejor que si
hubieran recibido frmacos, pues la recuperacin de la depresin
llega a ser ms completa.
La mala fama del TEC procede de la poca inicial, en la que se
aplicaba sin anestesia (por estar poco desarrollados los procedimientos anestsicos) y en la que se usaba de forma indiscriminada para casi
cualquier tipo de paciente psiquitrico. No hay que olvidar que fuera
de la depresin mayor el TEC apenas tiene indicaciones; se puede
usar en los episodios manacos, pero es raro que stos sean resistentes
a los frmacos; tambin se va a usar en episodios esquizofrnicos
agudos resistentes a antipsicticos, sobre todo si existen sntomas catatnicos o si hay clnica depresiva asociada.
Pregunta 36.-R: 4
El litio es el estabilizador de primera eleccin en la enfermedad
bipolar. Su principal beneficio es la reduccin del nmero de recadas (sobre todo manacas); tambin consigue atenuar la intensidad de
las mismas, caso de que se produzcan. Ms discutibles son sus efectos
sobre los episodios agudos; como antidepresivo, el litio es poco potente y no es raro que se deban usar AD asociados (con las precauciones debidas para no inducir un cambio de fase); como antimanaco,
el litio es muy eficaz pero tiene un efecto retardado (varias semanas)
que puede resultar inaceptable para pacientes manacos graves; adems no se dispone de otra presentacin aparte de la oral, lo que
condiciona su uso a la aceptacin del tratamiento por parte del paciente (que no es lo habitual en las formas graves); el litio es especialmente eficaz en los cuadros de mana clsica (eufrica), disminuyendo su eficacia en las manas disfricas (irritables). Su efecto en las fases
mixtas es tambin bastante discreto, prefirindose en estos casos otros
frmacos (valproico, carbamacepina, antipsicticos atpicos).
Adems, el litio es un medicamento con numerosos efectos secundarios, hasta tal punto que si un paciente no refiere ningn problema durante el tratamiento, probablemente no lo est tomando. Para prevenir los
efectos adversos ms graves se realizan una serie de controles mdicos
antes y durante el tratamiento con litio; con diferencia, lo ms importante
ser el control de los niveles plasmticos de litio (litemia), pues guardan una
relacin bastante estrecha con la eficacia y la toxicidad. Dado que el litio
se elimina por el rin, utilizando los mismos sistemas de transporte que el
sodio o el potasio, ser necesario disponer de parmetros que nos informen del estado de la funcin renal (creatinina, BUN) y los iones mencionados. Uno de los efectos ms peligrosos es la alteracin de la conduccin
cardaca; las alteraciones en el ECG permiten controlar hasta qu punto el
litio produce problemas a este nivel (normalmente se van a limitar a cambios en la onda T). El litio es un medicamento teratgeno; se ha relacionado con malformaciones cardacas (anomala de Ebstein), por lo que antes

CTO Medicina C/ Nez de Balboa, 115 28006 MADRID (Espaa) Tfno.: (91) 782 43 32 / Fax: (91) 782 43 27
E-mail: secretaria@ctomedicina.com; iberocto@ctomedicina.com WEB: www.ctomedicina.com; www.iberocto.com

PQ Pg. 9

de iniciar el tratamiento con litio en una mujer en edad frtil conviene


asegurarse de que no est embarazada e informarla de la necesidad de
que adopte medidas de anticoncepcin eficaces; ante cualquier sospecha de embarazo habr que repetir la prueba diagnstica de gestacin
pues, de confirmarse, se debera interrumpir el tratamiento (nunca de
forma brusca por el riesgo de recada). La ltima prueba obligatoria en el
control rutinario del tratamiento con litio es la determinacin de la TSH
para detectar problemas tiroideos (que en ocasiones explican la resistencia
al tratamiento). Otras determinaciones (glucemia, calcemia, pruebas de
concentracin de la orina) dependern de la evolucin y de la aparicin
de efectos secundarios.
El electroencefalograma NO es una prueba necesaria para el control del litio ni de ningn otro psicofrmaco; cuando aparecen alteraciones en el EEG es que ya se ha provocado neurotoxicidad, que con
frecuencia ser irreversible.
Pregunta 37.-R: 5
Los niveles de litio deben estar entre 0,4 y 1,5 mEq /L. Lgicamente
en los episodios agudos sern necesarias litemias ms altas (superiores
a 1,0 mEq/L) que en los perodos de estabilidad (inferiores a 1,0 mEq/
L); la mayora de los pacientes van a estar un poco por arriba o por
debajo de esta cifra (1,0 mEq/L), para no provocar una excesiva toxicidad o perder la eficacia.
Si el episodio manaco o depresivo no es demasiado grave quizs
baste con una ligera correccin de la dosis del estabilizador, pero en
los casos graves ser necesario aadir tratamiento especfico; en los
casos de mana grave la falta de colaboracin del paciente suele
obligar al tratamiento inicial con antipsicticos (intramusculares),
dejando la adicin del estabilizador para ms adelante. Una vez
conseguido el control de la recada se retirar el frmaco especfico
de la fase en cuanto sea posible, para no inducir un cambio de
polaridad.
Pregunta 38.-R: 4
Otro problema adicional del litio es la posibilidad de que los niveles
plasmticos se modifiquen por factores que afectan la eliminacin renal, sin que el paciente haya variado la dosis. Dado que el litio utiliza los
mismos sistemas de transporte renal que el sodio, cualquier circunstancia que aumente la reabsorcin de este ion (hipovolemia, deshidratacin, hiponatremia) producir un aumento de los niveles de litio, superando el umbral de toxicidad. Numerosos frmacos van a producir los
mismos problemas, bien por sus efectos sobre la vascularizacin renal
(AINEs, inhibidores de la COX-2), bien por sus efectos sobre el tbulo
renal (diurticos, IECAs, ARA-2). En teora podramos ver una disminucin de la litemia, con la consiguiente prdida de eficacia, en las situaciones hidroelectrolticas inversas (hipernatremia, ingesta hdrica excesiva) y por algunos frmacos (metilxantinas, diurticos osmticos e inhibidores de la anhidrasa carbnica), pero son situaciones excepcionales.
Pregunta 39.-R: 2
La carbamacepina y el cido valproico son las dos grandes alternativas al litio (estabilizadores de segunda lnea); ambos tienen una toxicidad parecida (hematolgica y heptica), pero la carbamacepina es ms
txica y adems tiene una capacidad notable de induccin enzimtica, lo que explica el mayor uso del valproico (en EE.UU. se ha puesto a
la altura del litio, como frmaco de primera lnea en mana aguda). Se
controlan de una forma similar a su uso en la epilepsia, aunque no se ha
demostrado que los niveles plasmticos guarden relacin con su eficacia (s con la toxicidad). En general tienen ms efecto como antimanacos,
siendo su efecto antidepresivo mnimo.
Otros estabilizadores, ya de tercera lnea, son:
Algunas benzodiacepinas de alta potencia, como el clonacepam,
que se usan en casos de ciclacin rpida.
Antipsicticos atpicos, como la clozapina, la olanzapina o la risperidona, que se usan en el control de las fases mixtas, las manas
disfricas y en mana resistente al tratamiento.
Antiepilpticos de ltima generacin, como el topiramato (estudiado en mana aguda), la lamotrigina (estudiado en depresin bipolar
y como frmaco auxiliar en la prevencin de recadas) y la gabapentina (estudiado como frmaco auxiliar en la prevencin de reca-

Pg. 10 PQ

M exico A rgentina
C hile U ruguay

PSIQUIATRA

Preparacin Examen de Seleccin 05/06 1 Vuelta

das).
Antagonistas de los canales de calcio, como el verapamil o el nifedipino, estudiados en casos de ciclacin rpida.
Pregunta 40.-R: 2
El litio es el medicamento de eleccin en la enfermedad bipolar
pero no es la solucin definitiva. Hay pacientes que no toleran los
efectos adversos, mientras que en otros est contraindicado (insuficiencia renal, psoriasis, enfermedad del nodo sinusal). Adems hay un
porcentaje de pacientes (30-40%) que no responden bien al litio:
Las manas atpicas o disfricas (irritables).
Los episodios mixtos.
Los cuadros manacos secundarios a enfermedades mdicas.
Los pacientes con ciclacin rpida.
La ciclacin rpida se ve con ms frecuencia en mujeres, sobre
todo con TAB-2. En ocasiones se detectan alteraciones tiroideas que
necesitan ser corregidas para frenar la ciclacin. En general no responden bien a la monoterapia con litio; quizs respondan algo mejor
al tratamiento con carbamacepina o valproico, pero lo ms habitual
es utilizar como mnimo dos estabilizadores. Una de las causas ms
frecuentes de ciclacin rpida es el tratamiento farmacolgico incorrecto, fundamentalmente el abuso de AD y la no utilizacin de estabilizadores (lo cual es frecuente en los pacientes con TAB-2, por las
dificultades para el diagnstico de las hipomanas).
Pregunta 41.-R: 3
La esquizofrenia es una enfermedad frecuente, afectando a cerca del
1% de la poblacin en algn momento de su vida; teniendo en cuenta
que tan slo 1/3 de los casos se pueden considerar de buen pronstico,
con una recuperacin ms o menos completa, se calcula que en Espaa
hay cerca de 250.000 personas con la enfermedad. Tiende a aparecer
en la adolescencia o juventud (de los 15 a los 35 aos), aunque se han
descrito casos de inicio en la infancia y en la vejez. Afecta por igual a
hombres y mujeres, si bien en las mujeres tiene un inicio algo ms tardo
(quizs por el efecto protector de los estrgenos contra la actividad
dopaminrgica excesiva), lo que determina un mejor pronstico. No se
han descrito variaciones de su incidencia en funcin de la clase social, la
cultura o la raza; sin embargo la mayora de los pacientes tienen un nivel
socioeconmico bajo, como consecuencia del deterioro que produce
la enfermedad (hiptesis del descendimiento social); el pronstico parece ms favorable cuanto menor es el nivel de desarrollo de la regin (p.ej.
mejor en zonas rurales que en reas urbanas). Estadsticamente los pacientes tienen un mayor riesgo de muerte tanto por causas no naturales
(suicidio) como por causas naturales.
Pregunta 42.-R: 4
En la actualidad, el diagnstico de esquizofrenia se basa en los
siguientes criterios:
Presencia de unos sntomas caractersticos que se han dividido de
una forma algo artificial en positivos (los que predominan en las
fases agudas o brotes) y negativos (que son ms evidentes en las
fases residuales).
Ausencia de otras enfermedades psiquitricas, enfermedades mdicas, frmacos o txicos que los justifiquen.
Produccin de un deterioro en el funcionamiento habitual del
paciente (acadmico, social, laboral).
Duracin superior a 6 meses, comprendiendo los prdromos, el
brote psictico (que debe durar como mnimo 1 mes, salvo que se
trate precozmente) y la fase residual de recuperacin. (Ver tabla en
la pgina siguiente).
Pregunta 43.-R: 3
Desconocemos cules son las causas precisas de esta enfermedad,
pero se maneja un modelo terico que incluye factores que aumentaran el riesgo para padecerla (factores predisponentes o de vulnerabilidad) y otros que contribuiran a su aparicin en individuos predispuestos (factores precipitantes o desencadenantes).
Dentro de los primeros tenemos:
Presencia de antecedentes familiares de esquizofrenia (quizs el
ms importante).

CTO Medicina C/ Nez de Balboa, 115 28006 MADRID (Espaa) Tfno.: (91) 782 43 32 / Fax: (91) 782 43 27
E-mail: secretaria@ctomedicina.com; iberocto@ctomedicina.com WEB: www.ctomedicina.com; www.iberocto.com

Comentarios TEST

Seguimiento a distancia

PSIQUIATRA

Preparacin Examen de Seleccin 05/06 1 Vuelta


Haber nacido en los meses fros del ao (se ha relacionado con
infecciones por el virus de la gripe durante el embarazo).
Antecedentes de problemas obsttricos.
Enfermedades graves de la primera infancia (sobre todo las que
afectan directamente al sistema nervioso central, como meningoencefalitis, epilepsia grave, traumatismos craneales severos).
Los factores desencadenantes NO producen esquizofrenia en personas sin vulnerabilidad; su influencia es ms evidente en las recadas
de la enfermedad:
Txicos (sobre todo alucingenos, cannabis y estimulantes como
la cocana o las anfetaminas).
Estrs social (relacionado sobre todo con el abandono del tratamiento).
Enfermedades graves que afectan al sistema nervioso central.

Pregunta 42. Sntomas positivos y negativos de la esquizofrenia.


123454623

789
54623



123456761 845649
3 35 3 4354 


434 5455

393
35456


9 6 4 56


3   


6 76 841 9  6 18
1  76 1



8

4234
64
373846 

4$
56 843  3 435

4 3 434 35 


4

4443 4 396

4447 6



42 % 3 4347 6





4

42 
395 4 35
3

543
6435
3

4443
35 3 435
37 6
3 

44435
37 6
3 

4!33
544 6 4 3

4!33
54 4 6

4441 9
3
4
561 434"#533


4419

6 1434 & 3 3


&!"84"'84 "(2"843 

4( 5653 

8 

423 
6 

()""84 "()"
46
3.4 3 4 35/3
4(9 535 64 3 76
4( 6 8

4$  65%
4$65  43
* 4+,-
42 
354+,-

Comentarios TEST

23 
6 84
3  3

4445#365

enfermo y su vida (no, por ejemplo, voces que insultan, que son con
mucho las alucinaciones ms frecuentes de toda la Psiquiatra).
En los delirios destac tanto su contenido (delirios de control e
influencia frente a los mucho ms frecuentes, pero ms inespecficos
delirios de perjuicio) como la forma de aparicin (percepciones delirantes sin sentido, frente a las mucho ms frecuentes interpretaciones
delirantes, ms o menos conectadas con la realidad).
Ejemplo de percepcin delirante: veo una persona apoyada en una
farola de la calle, iluminada por su luz; esa luz quiere decir que soy
el nuevo Mesas, y ese es el Demonio que viene a impedir que me
manifieste.
Ejemplo de interpretacin delirante: veo una persona apoyada en
una farola de la calle, iluminada por su luz; comienzo a pensar si
no lleva mucho tiempo all y si no estar esperando a que salga
para hacerme dao.
Pregunta 46.-R: 3
En las fases agudas o brotes predominan los sntomas positivos y en
las fases residuales los sntomas negativos. El aplanamiento afectivo es
un sntoma negativo.
Vamos a detenernos un poco en el diagnstico diferencial de los
sntomas negativos; stos pueden deberse a diferentes causas, cada
una de ellas con una solucin especfica:
En ocasiones, los sntomas se deben a que el enfermo est deprimido
(es frecuente al principio de la enfermedad, cuando tienen todava
conciencia de su cambio), respondiendo al tratamiento antidepresivo.
Los frmacos antipsicticos tienen la capacidad de producir sntomas extrapiramidales que mimetizan los sntomas negativos y responden a una reduccin o un cambio de la medicacin.
La repercusin social de la enfermedad (aislamiento, rechazo) conduce a muchos enfermos a una situacin similar a la descrita en los
cuadros deficitarios que mejora radicalmente al comenzar un tratamiento de tipo rehabilitador.
Por ltimo, la persistencia de sntomas positivos intensos puede
hacer que algunos enfermos rechacen el contacto social o se muestren retrados como consecuencia del miedo que tienen a ser daados por los dems.

42 434  355


4(3 6459
6964+,-

Pregunta 44.-R: 4
Los trastornos del pensamiento son quizs (con las alteraciones de
la percepcin) los sntomas ms estudiados en la esquizofrenia; los
podemos dividir en trastornos del contenido del pensamiento (ideas
delirantes) y trastornos del curso o la forma del pensamiento.
Dentro de las ideas delirantes, hablamos de ideas delirantes primarias cuando parecen surgir de forma autnoma, independientes
del resto de fenmenos psicopatolgicos, siendo tpicas de la esquizofrenia y la paranoia; las ideas delirantes secundarias (o ideas deliroides) proceden de fenmenos de tipo afectivo (depresin, mana,
angustia).
Los pacientes tienen problemas para dirigir su pensamiento hacia
unas conclusiones lgicas, presentando con frecuencia prdidas de
sentido (descarrilamientos) que acaban por producir una sensacin
de disgregacin del pensamiento; en otras ocasiones se van a quejar
de la interrupcin del curso del pensamiento (bloqueo).
Pregunta 45.-R: 2
Schneider se plante cules de todos los sntomas de la esquizofrenia eran ms especficos de la enfermedad, siendo a la vez frecuentes
y de fcil diagnstico; buscaba lo que ahora llamaramos sntomas de
alto valor predictivo positivo; aunque se equivoc al creer que eran
frecuentes (aparecen en slo un tercio de los pacientes), s hay que
reconocer que son altamente sugerentes de esquizofrenia, pero NO
patognomnicos. No olvidar que en la esquizofrenia son mucho ms
frecuentes los sntomas negativos, que por desgracia son totalmente
inespecficos.
Dentro de las alucinaciones destac las auditivas, en concreto aquellas en las que se escuchan voces humanas que hablan entre s del
M exico A rgentina
C hile U ruguay

Seguimiento a distancia

Pregunta 46. Historia natural de la esquizofrenia.

Pregunta 47.-R: 2
Los factores pronsticos de la esquizofrenia han sido preguntados
en varias ocasiones; podemos dividirlos de una forma sencilla en:
Factores previos al inicio de la enfermedad:
- Sexo (mujeres mejor que varones).
- Antecedentes familiares (mejor si NO hay antecedentes de esquizofrenia).
- Nivel de adaptacin previo (mejor cuanto mayor fuera).
- Personalidad previa (mejor si era normal).
- Nivel de inteligencia previo (mejor si NO era bajo).
Factores relacionados con el debut de la enfermedad:
- Edad de inicio (uno de los fundamentales; mejor cuanto ms
tarda).
- Presencia de factores precipitantes (mejor si los hay).
- Presencia de sntomas depresivos o manacos (mejor si los hay).
- Presencia de sntomas confusionales (mejor si lo hay).
- Inicio rpido (mejor que si el inicio es insidioso).
- Patrn de sntomas (mejor si dominan los positivos).
- Rapidez a la hora de iniciar un tratamiento.
Factores evolutivos:
- Nmero de recadas (fundamentalmente en los 5 10 primeros
aos).

CTO Medicina C/ Nez de Balboa, 115 28006 MADRID (Espaa) Tfno.: (91) 782 43 32 / Fax: (91) 782 43 27
E-mail: secretaria@ctomedicina.com; iberocto@ctomedicina.com WEB: www.ctomedicina.com; www.iberocto.com

PQ Pg. 11

- Gravedad de los sntomas negativos (determinantes para el pronstico social y laboral).


- Resistencia al tratamiento de los sntomas positivos.
Pregunta 48.-R: 3
En la prevencin de las recadas de la enfermedad esquizofrnica
resulta fundamental el mantenimiento de un tratamiento antipsictico a largo plazo; diversos factores influyen en el abandono del mismo
(que es bastante frecuente y es la principal causa de recada):
El perfil de efectos secundarios del medicamento: para evitar el
abandono por esta razn debemos individualizar el tratamiento
segn las preferencias del paciente; teniendo en cuenta que los
efectos extrapiramidales son muy mal tolerados, los antipsicticos
atpicos parecen tener una mejor aceptacin.
La conciencia de enfermedad: puede mejorarse mediante tcnicas de psicoeducacin en la enfermedad, sobre todo a travs de
terapias grupales.
La gravedad del estrs psicosocial sufrido por el paciente, sobre todo
el debido al ambiente familiar cuantificado por el concepto de la
expresin emocional: puede disminuirse mediante tcnicas de intervencin familiar, el uso de recursos como centros de da u hospitales
de da y, en ltima instancia, mediante alternativas residenciales.
Pregunta 49.-R: 2
La paranoia o trastorno delirante es una enfermedad sorprendente
en la que los pacientes tienen bsicamente un nico sntoma (un
delirio), con una relativa preservacin del resto del funcionamiento
psicolgico y un escaso deterioro de su personalidad. Podemos ver
cuadros paranoicos de inicio agudo, en respuesta a un estrs identificable (como los descritos en presos o inmigrantes), pero la paranoia
clsica tiene un inicio insidioso, casi siempre entrelazado con una
personalidad previa peculiar (paranoide, demasiado sensible). Cuando el delirio toma importancia el paciente suele encontrarse en la
edad adulta, sufriendo consecuencias en los terrenos social, familiar o
laboral segn los contenidos del delirio. Tiene un curso crnico, con
una respuesta discreta del delirio al tratamiento; con la medicacin se
va a buscar ms un control conductual que una disolucin del delirio
(encapsulacin).
Las clasificaciones actuales aceptan 5 formas de delirios crnicos
paranoicos:
Delirios de perjuicio (los ms frecuentes, pero muy poco especficos).
Delirios de celos (relacionados en ocasiones con el alcoholismo).
Delirios de enfermedad (hipocondra delirante).
Delirios de grandeza.
Delirios de enamoramiento.
Pregunta 50.-R: 3
El mecanismo por el cual los pacientes paranoicos llegan al delirio
es la interpretacin delirante de la realidad; con frecuencia basan sus
delirios en hechos que han sucedido, pero a los que dan un significado nuevo dentro de lo razonable (a diferencia de los delirios esquizofrnicos, que pueden ser increbles). Psicoanalticamente se dice que
estos pacientes recurren fundamentalmente a la proyeccin como
mecanismo psicolgico de defensa (colocar en el exterior la causa de
mis problemas, por ejemplo, si no encuentro algo es porque me lo
han quitado).
Clsicamente no se admite la existencia de alucinaciones en los
trastornos delirantes (s son frecuentes las ilusiones), aunque podemos
ver alteraciones de la percepcin (sobre todo tctiles u olfativas) que
se encuentren al margen de lo alucinatorio.
Pregunta 51.-R: 4
Los antipsicticos clsicos ejercen diferentes acciones sobre los
sistemas de neurotransmisin cerebrales:
Poseen la capacidad de bloquear los receptores dopaminrgicos; el
bloqueo D2 se ha relacionado tanto con su efecto antipsictico
como con sus efectos secundarios extrapiramidales (por accin sobre la va nigroestriada) y endocrinos (hiperprolactinemia por su
accin sobre la va tuberoinfundibular); adems se relaciona con su
capacidad para inhibir el vmito y el hipo inducidos por quimioterapia.
Pg. 12 PQ

M exico A rgentina
C hile U ruguay

PSIQUIATRA

Preparacin Examen de Seleccin 05/06 1 Vuelta

Los ms antiguos tienen importantes efectos antihistamnicos (que


producen somnolencia y aumento del apetito y del peso), efectos
antiadrenrgicos (con hipotensin ortosttica como consecuencia) y efectos anticolinrgicos (produciendo el tpico sndrome
muscarnico: sequedad de boca, estreimiento, visin cercana borrosa, retencin urinaria, alteraciones de memoria).
Pregunta 52.-R: 3
Dentro de los antipsicticos tpicos o neurolpticos encontramos
diferencias en su potencia (capacidad de bloqueo dopaminrgico) que
pueden ser compensadas con un aumento de dosis; la eficacia antipsictica es similar en todos ellos. Al aumentar la dosis van a aparecer ms
efectos sistmicos como consecuencia de los bloqueos colaterales descritos (menor potencia, ms dosis, mayor sedacin, ms hipotensin,
ms acciones anticolinrgicas); en los antipsicticos menos potentes la
presencia de un bloqueo anticolinrgico significativo va a disminuir la
tasa de efectos extrapiramidales (puesto que as en la va nigroestriada
los dos neurotransmisores estarn bloqueados).
Las consecuencias de la hiperprolactinemia inducida por los antipsicticos son bien conocidas: en mujeres veremos amenorrea, ginecomastia, galactorrea y disminucin de la libido; en varones,
disfunciones sexuales y disminucin de la libido sobre todo. Dentro de
los antipsicticos clsicos, destaca la capacidad del sulpiride para inducir aumentos enormes de la prolactina, incluso cuando se usa en dosis
bajas.

Pregunta 52. Relacin entre dosis y potencia en los antipsicticos clsicos.

Pregunta 53.-R: 4
Los modernos antipsicticos atpicos se diferencian tericamente
de los antipsicticos clsicos en tres caractersticas:
Producen menos efectos extrapiramidales.
Tienen un mayor efecto antipsictico que los clsicos (tomando
como referencia el haloperidol).
Tienen cierto efecto sobre los sntomas negativos de la enfermedad
(accin de la que carecen los antipsicticos tpicos).
Cmo consiguen esta eficacia diferencial?:
Algunos de ellos tienen la capacidad de bloquear mltiples receptores (dopaminrgicos, serotoninrgicos, histaminrgicos, adrenrgicos, colinrgicos), sin que se pueda atribuir a alguno de ellos en
concreto su eficacia: clozapina, olanzapina, quetiapina.
Otros combinan acciones antidopaminrgicas con acciones
antiserotoninrgicas: risperidona, sertindol, ziprasidona.
Unos pocos buscan el bloqueo dopaminrgico selectivo de las vas
tericamente implicadas en los sntomas de la enfermedad (vas

CTO Medicina C/ Nez de Balboa, 115 28006 MADRID (Espaa) Tfno.: (91) 782 43 32 / Fax: (91) 782 43 27
E-mail: secretaria@ctomedicina.com; iberocto@ctomedicina.com WEB: www.ctomedicina.com; www.iberocto.com

Comentarios TEST

Seguimiento a distancia

PSIQUIATRA

Preparacin Examen de Seleccin 05/06 1 Vuelta


mesolmbicas y vas mesocorticales), respetando la va nigroestriada, responsable de los efectos extrapiramidales: amisulpride. Sin
embargo estos frmacos no estn exentos de problemas.
Carecemos por el momento de presentaciones parenterales, por lo
que cuando el paciente rechaza el tratamiento hemos de recurrir a
antipsicticos clsicos (tanto en urgencias [haloperidol] como en
tratamiento de mantenimiento [preparados depot]).
Tienen un precio muy superior a los frmacos tradicionales, sin que se
haya podido demostrar en estudios de coste-efectividad que compensen su precio con una reduccin de otros costes de la enfermedad.
Algunos de sus efectos secundarios siguen produciendo problemas
de tolerancia:
- La risperidona produce una tasa de efectos extrapiramidales
importante cuando se usa en dosis altas; adems dispara los
niveles de prolactina incluso en dosis bajas, con las consecuencias endocrinas conocidas.
- La clozapina, la olanzapina y la quetiapina pueden producir un
aumento del apetito y del peso muy llamativos, adems de efectos
sobre la tensin arterial y somnolencia; el aumento de peso se ha
relacionado con alteraciones metablicas (diabetes mellitus tipo II,
dislipemias) y aumento de la morbimortalidad cardiovascular.

Seguimiento a distancia

del tronco o las extremidades produciendo una seria limitacin en


el paciente; su respuesta al tratamiento es pobre, pudiendo considerarse casi irreversibles.
Cada efecto secundario tiene un tratamiento especfico:
Anticolinrgicos para las distonas agudas (por va parenteral) y el
parkinsonismo (por va oral): biperideno, trihexifenidilo.
Betabloqueantes (propranolol) o benzodiacepinas (loracepam) para
la acatisia.
Depleccionantes dopaminrgicos (tetrabenacina) para las discinesias tardas graves.
Amantadina para el parkinsonismo (no es habitual en Espaa).
Un efecto secundario especialmente grave y con componente extrapiramidal es el sndrome neurolptico maligno; en l se combinan:
Elementos extrapiramidales (rigidez extrema).
Hipertermia.
Alteraciones vegetativas (sudoracin, labilidad de la tensin arterial
o la frecuencia cardaca).
Sntomas confusionales.
Tiene una importante mortalidad cuando se deja sin tratamiento,
fundamentalmente por las repercusiones renales de la necrosis muscular masiva (por la rigidez y la hipertermia), siendo bsico su diagnstico precoz (analticamente veremos leucocitosis, aumento de la
CPK y de otros enzimas musculares, hiperfosfatemia, hiperpotasemia) y su tratamiento inmediato; se suele indicar el ingreso del paciente en una UCI para un control mdico estricto, precisando de
una serie de medidas fsicas para frenar la hipertermia y de determinados frmacos (dantroleno [relajante muscular], bromocriptina [agonista dopaminrgico]) para revertir los sntomas principales.

Pregunta 54. Aparicin de los diferentes sntomas


extrapiramidales en funcin del tiempo.
Efectos extrapiramidales
Agudos
Caractersticas
Inicio

Comentarios TEST

Clnica

F. R iesgo

Distona

Subagudos
Parkinsonismo

1 semana

En 3-4 meses

Crisis
oculgiras
Tortcolis

Temblor
Acinesia
Rigidez
(parkinsonismo)

Pregunta 54.-R: 1
De todos los efectos secundarios que pueden presentar los antipsicticos, los ms preguntados son los extrapiramidales; stos van cambiando en funcin del tiempo:
En las primeras horas o das podemos ver distonas agudas; son
contracciones sostenidas de grupos musculares que pueden resultar dolorosas o peligrosas (distonas larngeas); son especialmente
frecuentes en varones, tanto ms cuanto ms jvenes.
Al cabo de varios das o semanas aparecen dos tipos de efectos
aparentemente opuestos:
- Una reduccin de los movimientos en la forma de sndrome
parkinsoniano.
- Un aumento de la movilidad en la forma de inquietud (objetiva
y subjetiva) o acatisia.
Cuando un paciente ha tomado medicacin antipsictica durante
meses o aos puede presentar movimientos de tipo coreoatetsico
(los ms frecuentes de la musculatura faciobucolingual) denominados discinesias tardas; en raras ocasiones afectan a msculos
M exico A rgentina
C hile U ruguay

Anticolinrgicos

Discinesia
Tras aos de
tratamiento

Inquietud

Movimientos
faciobucolinguales
15% grave
Mujeres, ancianos
Deterioro previo
del SNC
Sntomas
afectivos
Tto.
anticolinrgicos?

Jvenes
Varones
NL incisivos

Pregunta 53. Evolucin de los antipsicticos.


Tratamiento

Tardos
Acatisia

Anticolinrgicos
Amantadina

Betabloqueantes
BZD

Clozapina o
depleccionantes de
dopamina
(tetrabenacina)

Pregunta 55.-R: 2
El tratamiento con TEC fue inicialmente probado en pacientes con
esquizofrenia, aunque pronto se comprob su extraordinaria eficacia
en pacientes afectivos (depresin, mana). En la actualidad su uso en
esquizofrenia es muy limitado:
Pacientes con formas catatnicas.
Pacientes con esquizofrenia y depresin grave asociada o ideas
graves de suicidio.
Pacientes con brotes psicticos resistentes al tratamiento en los que
se ha demostrado el cumplimiento del mismo.
En general suelen ser necesarias ms sesiones de tratamiento que
en la depresin y los resultados (salvo en las formas catatnicas) no
son espectaculares.
No hay que olvidar que la nica opcin de tratamiento que, adems del TEC, ha demostrado cierta eficacia en los brotes resistentes es

CTO Medicina C/ Nez de Balboa, 115 28006 MADRID (Espaa) Tfno.: (91) 782 43 32 / Fax: (91) 782 43 27
E-mail: secretaria@ctomedicina.com; iberocto@ctomedicina.com WEB: www.ctomedicina.com; www.iberocto.com

PQ Pg. 13

el uso de los antipsicticos atpicos modernos (no son eficaces el uso


de dosis masivas de neurolpticos, la utilizacin de dosis de carga al
principio del tratamiento o las asociaciones de neurolpticos).
Pregunta 56.-R: 4
Cuando hablamos del tratamiento de la enfermedad no debemos pensar que todo se reduce al uso de frmacos; en ese caso
bastara con asegurarnos el cumplimiento (con formas intramusculares depot) para controlar la enfermedad. Tan importante como el
tratamiento farmacolgico van a ser todas las medidas psicosociales
destinadas a mejorar el cumplimiento (psicoeducacin en la enfermedad, terapias e intervenciones familiares) y a la rehabilitacin y
reinsercin social de los pacientes (tcnicas de modificacin de conducta, tcnicas de afrontamiento del estrs). La disponibilidad y gestin de los llamados recursos intermedios (hospitales de da, centros
de da, centros de rehabilitacin psicosocial, centros de rehabilitacin laboral, recursos residenciales) condicionar el pronstico de
la enfermedad. De aqu que en los ltimos aos se est hablando de
nuevos problemas en el manejo clnico de este tipo de pacientes,
como el sndrome de la puerta giratoria (reingresos repetidos de los
mismos pacientes por la ausencia de recursos externos al hospital) o
el problema de los pacientes jvenes crnicos, especialmente grave
en las grandes ciudades.
Las terapias dinmicas (derivadas del psicoanlisis) carecen en la
actualidad de indicacin en este tipo de pacientes, puesto que precisan de una capacidad de introspeccin importante y se enfrentan a
una enfermedad con condicionantes neurobiolgicos evidentes.
Pregunta 57.-R: 5
El alcohol es un sedante del Sistema Nervioso Central y sus efectos
dependen de la dosis ingerida, la capacidad de metabolizarlo (tolerancia farmacocintica) y la adaptacin cerebral al consumo crnico
(tolerancia farmacodinmica). En general encontramos una buena
correlacin entre la alcoholemia y los efectos clnicos, de tal forma
que si una persona tiene una alcoholemia elevada y no manifiesta
signos de intoxicacin podemos asegurar que tiene una dependencia
del alcohol. De forma inversa vemos pacientes que con alcoholemias
muy bajas sufren graves alteraciones conductuales (agresividad, agitacin); son las llamadas intoxicaciones idiosincrsicas o borracheras
patolgicas, y su origen parece ser la mayor sensibilidad de algunas
personas (ancianos, nios, pacientes con dao cerebral) a las sustancias sedantes (alcohol, benzodiacepinas, barbitricos). Cuando se ingiere una cantidad masiva de alcohol puede producirse la afectacin
del tronco del encfalo, apareciendo somnolencia y llegando en ocasiones a la depresin cardiorrespiratoria. En general, el tratamiento de
las intoxicaciones alcohlicas es puramente sintomtico, evitando las
complicaciones comunes a cualquier coma (neumonas por aspiracin, sndromes compartimentales por decbito prolongado, hipotermia por exposicin al aire libre); una caracterstica diferencial del
coma etlico es la aparicin de hipoglucemia como consecuencia de
la inhibicin de la gluconeognesis y la glucogenlisis; por eso es
frecuente que a estos pacientes se les pauten soluciones glucosadas,
con la precaucin de poner previamente tiamina para no precipitar
una encefalopata de Wernicke; en casos excepcionales se puede
recurrir a la dilisis para eliminar cantidades extremas de alcohol.

Pregunta 57. Efectos fisiolgicos de las intoxicaciones y las abstinencias.

Pg. 14 PQ

M exico A rgentina
C hile U ruguay

PSIQUIATRA

Preparacin Examen de Seleccin 05/06 1 Vuelta

Pregunta 58.-R: 3
Una de las complicaciones ms graves del alcoholismo es el sndrome de abstinencia severo o delirium tremens. Asocia una importante mortalidad cuando se deja sin tratamiento, fundamentalmente
por las complicaciones hidroelectrolticas secundarias a los vmitos,
la sudoracin profusa o la diarrea. Para que se produzca un sndrome
de abstinencia tiene que existir tolerancia, y sta se adquiere como
consecuencia del consumo crnico de cantidades elevadas de alcohol; parece que en su fisiopatologa intervienen mecanismos gabargicos y glutamatrgicos, lo que quizs explique la elevada incidencia
de convulsiones generalizadas tnico-clnicas.

Pregunta 58. Marcadores del consumo excesivo de alcohol.

123
345678944
65 298

96 9
96 154
45 7
97
5 45678944
65
3
55  5

345679
575
$1%
$96 147 54
%9
 9 7 95 &97
1
96 9 )3*3
"95
+6 96 49
57 45675
65757
34
0
14
7 2
 /4


 965 7967


5  979
8

5
 965 45
9 9 5
  795657
 49
 995
 965 7967


5 5!5 979
8

5
 "5
9 9 5
 9455 # 795657
 5! 49

 5!5 7967


5 9!4 979
8

5
 % 96  96 5
54 '( &57
 5! 49

 ,
49 545 8

54 65 95

7 5 +
5 -
 . 97 /
 7
6 5 95

7

   7967
97    979 &8
7
.
997 9 '  8+
  %54 5497
6979 &8
7 9 56 4

+6
6 9 5 
7

Pregunta 58. Fisiopatologa de la dependencia y abstinencia del alcohol.

Pregunta 59.-R: 2
En la prevencin de recadas del alcoholismo destacamos dos fenmenos psicolgicos que pueden ser parcialmente controlados con
medicacin:
Por un lado, los sntomas de abstinencia condicionados a estmulos ambientales: ACAMPROSATO.
Por otro, la prdida de control asociada al consumo de una pequea cantidad del txico: NALTREXONA.
Totalmente distinto es el mecanismo de accin de los interdictores
(disulfiram, cianamida); stos bloquean el metabolismo del alcohol
en uno de sus pasos intermedios (aldehdo deshidrogenasa), provocando la acumulacin de acetaldehdo, el cual desencadena una
reaccin histaminrgica. La base del tratamiento es conductual; se
trata de disminuir una conducta problemtica mediante el miedo a
un castigo potencial; por tanto, es necesario que el paciente est al
tanto de su tratamiento, siendo conveniente que firme un consenti-

CTO Medicina C/ Nez de Balboa, 115 28006 MADRID (Espaa) Tfno.: (91) 782 43 32 / Fax: (91) 782 43 27
E-mail: secretaria@ctomedicina.com; iberocto@ctomedicina.com WEB: www.ctomedicina.com; www.iberocto.com

Comentarios TEST

Seguimiento a distancia

PSIQUIATRA

Comentarios TEST

Preparacin Examen de Seleccin 05/06 1 Vuelta

Seguimiento a distancia

Pregunta 64. Caractersticas clnicas del sndrome confusional agudo.

miento informado como precaucin legal, por si decide beber a pesar de tomarlo.
Otros frmacos como los ISRS o el tiapride parecen jugar un papel
ms limitado.
M exico A rgentina
C hile U ruguay

Pregunta 60.-R: 4
En la dependencia de la herona, el tratamiento farmacolgico es
ms sencillo al existir frmacos especficos para el sistema opioide
endgeno, tanto agonistas como antagonistas.

CTO Medicina C/ Nez de Balboa, 115 28006 MADRID (Espaa) Tfno.: (91) 782 43 32 / Fax: (91) 782 43 27
E-mail: secretaria@ctomedicina.com; iberocto@ctomedicina.com WEB: www.ctomedicina.com; www.iberocto.com

PQ Pg. 15

PSIQUIATRA

Seguimiento a distancia

Preparacin Examen de Seleccin 05/06 1 Vuelta


Pregunta 62.Fases de la dependencia de la cocana y su tratamiento.

123456787692

12345678479

7
738
1234567847

7465733645
7
3
765645

746445
7 4567847!"645#
7
"533657 456
7847365645#
74$6834"57 84
7443375745744#

362 2768

%&378479

74"457'"3
%&37847

75&67(45
745467"67365(

7
84"4565
73'33657 845"(
7"6"#

4"437847

7)*565(6
7"4)+,7 364743-#
745467"67365(7 3#

7)4356457

7 , .)
#
7
6557

7 6(63"#

123456784793
89876 6 84
1234564789
2647586 68

123456787692

12 



87 12 58 847284

4433 2
 


128 787
1234564789
2 68  7 6

1234567847


87 72 1

122 8 68

64
!68 72
88


124 6

362 2768


" 8


12868 787

 3
784793
89876 6 84#
128 8 7
1238 664

12$847 242
7 25
 

  27692
 788


87 72 58 847
4 
256 5 8"4




87 72 58 847

 64


12% 84
47264"6  68 72 2
2


87 72 58 847

1256 4 72!8 72 7 8  7

4 


Pregunta 61.-R: 2
La abstinencia de la herona es un cuadro clnico espectacular,
pero sin complicaciones importantes; por tanto, y a diferencia de la
abstinencia del alcohol, NO es una urgencia mdica; los pacientes no
deben recibir tratamiento de forma puntual en un servicio de urgencias, salvo que deban quedar ingresados por otro motivo; la actitud
ms correcta es derivarles a centros especializados para que inicien
un tratamiento reglado destinado no slo a la abstinencia sino a la
prevencin de recadas.
Para favorecer el mantenimiento de la abstinencia se han probado
diferentes frmacos. La naltrexona (antagonista opioide) puede bloquear
los efectos de una dosis puntual de herona; sin embargo un bloqueo
puede ser revertido con una dosis suficientemente alta de herona, por lo
que el paciente debe comprometerse a no intentar vencerlo (de nuevo
aparece la necesidad del consentimiento informado), pues se provocara
una intoxicacin grave al estar desintoxicado (haber perdido la tolerancia);
por este motivo solamente se propone para pacientes altamente motivados,
sin caractersticas impulsivas y sin problemas mdicos o psiquitricos asociados. La metadona (agonista opioide) consigue disminuir de forma ms
eficaz las complicaciones legales y mdicas del consumo de herona, permitiendo la progresiva normalizacin del paciente en todas las esferas afectadas por la drogodependencia; se plantea como un tratamiento a largo
plazo, pudiendo necesitarse aos hasta que se pueda plantear su retirada;
es el tratamiento recomendado para los pacientes refractarios a otros tratamientos, los heroinmanos con problemas psiquitricos o mdicos graves,
las embarazadas y los politoxicmanos.
Pregunta 62.-R: 3
En el tratamiento de la dependencia de la cocana, las opciones de
apoyo farmacolgico son mucho ms limitadas; la accin fundamental de esta sustancia es catecolaminrgica (dopaminrgica sobre todo)
y todos los frmacos utilizados actan en estos sistemas. (Ver tabla).
Pregunta 63.-R: 4
Este paciente sufre un delirium o sndrome confusional agudo; esta
alteracin del estado mental aparece frecuentemente en enfermos
ingresados en hospitales generales, como consecuencia de los procesos mdicos, las intervenciones quirrgicas o los tratamientos farmacolgicos; el sustrato fisiopatolgico es una encefalopata aguda cuyo
origen pueden ser problemas originados en el cerebro o fuera de l
(procesos metablicos, txicos, frmacos, etc.). El sntoma inicial del
delirium es una alteracin de la atencin y la concentracin, que con
frecuencia conducen a la desorientacin (primero en el tiempo, ms
adelante en el espacio); es frecuente asistir a grandes fluctuaciones de
la clnica, en ocasiones relacionadas con factores externos (p.ej. empeoran al llegar la noche, pues la menor iluminacin hace que interpreten peor el entorno).

Pg. 16 PQ

M exico A rgentina
C hile U ruguay

  27692
  788

Pregunta 64.-R: 4
Describimos tres formas de delirium: agitados, estuporosos y mixtos.
En realidad en un mismo enfermo o en un mismo proceso patolgico
pueden verse todas las formas. Hay dos aspectos clnicamente importantes del delirium. En primer lugar, la AGITACIN de algunos delirium
puede suponer un riesgo para la salud del paciente y de las personas
que le rodean; por esto llaman al psiquiatra, como experto en sedacin, para elegir el frmaco que controle el comportamiento agresivo,
sin empeorar el estado fsico. Ver figura en la pgina siguiente.
El haloperidol es el frmaco de eleccin para el control de cualquier cuadro de agitacin, pues carece de efectos cardiovasculares
y respiratorios; su margen de seguridad es elevadsimo, y con esas
dosis curiosamente no se producen los temidos efectos extrapiramidales de estos frmacos. Los antipsicticos ms sedantes (clorpromacina, levomepromacina, zuclopentixol) tienen el riesgo de producir
una hipotensin severa; otros antipsicticos sedantes tienen importantes efectos anticolinrgicos (tioridacina) que agravaran la confusin mental. Las benzodiacepinas a dosis elevadas pueden producir
depresin respiratoria, sobre todo si el paciente est tomando otros
frmacos con efectos depresores. Sin embargo son el tratamiento de
eleccin en los sndromes de abstinencia alcohlica grave (delirium
tremens), pues corrigen de forma especfica la fisiopatologa del sndrome.
Pregunta 65.-R: 3
No hay que olvidar que todo delirium tiene una CAUSA que conviene
aclarar cuanto antes, pues implica una morbimortalidad notable; por eso
realizamos diferentes pruebas para diagnosticar el origen del mismo (anlisis de sangre [hemograma, bioqumica completa] y de orina, ECG, pruebas
de imagen [Rx de trax, TAC], puncin lumbar). El test de Rorscharch (test
de las manchas de tinta) es una prueba proyectiva que explora los aspectos
inconscientes de la personalidad; es totalmente absurdo aplicarlo a estos
pacientes, con una alteracin de su nivel de conciencia.
Pregunta 66.-R: 4
Cuando la combinacin de fallos de memoria, desorientacin,
disnomia y dispraxias y la demostracin de alteraciones en la exploracin
neuropsicolgica elemental (MEC o Mini-mental) hace que sospechemos
una posible demencia debemos seguir un sistema de diagnstico diferencial para descartar posibles enfermedades que tienen un carcter reversible o al menos algo ms benigno que las demencias degenerativas.
El primer paso es descartar que no exista una causa externa que
est influyendo en el funcionamiento cerebral hasta el punto de simular una demencia: intoxicaciones crnicas con frmacos sedantes, abuso de txicos, alteraciones metablicas (insuficiencia renal,
hipotiroidismo, encefalopata heptica), anemias severas, lesiones
ocupantes de espacio intracraneal (tumores, quistes, abscesos), enfermedades infecciosas o inflamatorias con tratamiento eficaz (sfilis, esclerosis mltiple), dficits vitamnicos, etc.
Por tanto, al paciente con sospecha de demencia deben realizrsele pruebas analticas (hemograma, bioqumica plasmtica completa,
serologas [sfilis, VIH], sistemtico de orina, estudio funcional tiroideo,

CTO Medicina C/ Nez de Balboa, 115 28006 MADRID (Espaa) Tfno.: (91) 782 43 32 / Fax: (91) 782 43 27
E-mail: secretaria@ctomedicina.com; iberocto@ctomedicina.com WEB: www.ctomedicina.com; www.iberocto.com

Comentarios TEST

Pregunta 60. Fases del tratamiento y los frmacos implicados


en la dependencia de la herona.

PSIQUIATRA

Preparacin Examen de Seleccin 05/06 1 Vuelta


niveles de B12 y flico), ECG y pruebas de imagen (Rx trax, TAC),
para descartar las principales causas de demencia reversible.
La polisomnografa slo puede ayudar a diferenciar la demencia de
la depresin, pues en la primera aumenta la latencia REM y en la segunda se acorta; sin embargo es una prueba compleja y con niveles de
sensibilidad y especificidad bajos, por lo que no se suele utilizar.

Seguimiento a distancia

Pregunta 68.-R: 2
Son frecuentes las preguntas que combinan demencia con diferentes sntomas, preguntndonos el diagnstico; repasemos los ms
importantes:

Pregunta 68. Demencias asociadas a sntomas especiales.

Pregunta 67.-R: 3
111213456789
6 1614818798
11121 9 5
6 9
817
87
8
1112187


111217 17
86 5617 76

11121
94 8


7 9684
8
7 56
8

76564528

111217 16 984 8
11121!6695 1"87968 "

12324567

!66 9
818 9487

11121#
 1 67 4$
9 1 9846
11121%7696 56 1988
11121349
89
6 1
846

!66 9
819

1112187



967 161'6()

11121& 58 19 
846
111218
16167 84
8
11121 958 1 6 
9 5
98
111213858*18

4
8*1

676 9
8

111213456789
6 16148184
6 589
$

!66 9
8 17 5 2
56 7846 1+
9,

1112167 6678 9
8

Pregunta 69.-R: 3

Pregunta 69. Diferencias entre demencia y depresin.

Comentarios TEST

Pregunta 67. Esquema de diagnstico de la demencia.

Una vez que hemos descartado las demencias reversibles, buscaremos otras demencias cuya causa puede al menos ser controlada,
impidiendo que avance el deterioro, sin que se pueda recuperar lo
perdido (demencias vasculares, demencias postraumticas, demencia pugilstica, demencias txicas): demencias irreversibles, pero con
tratamiento capaz de detener la progresin.
Lo que resta son las demencias degenerativas irreversibles o progresivas; algunas de stas son secundarias a enfermedades para las cuales
no tenemos tratamiento en la actualidad (demencia asociada a la
enfermedad de Creutzfeldt-Jakob, demencia asociada a la infeccin
VIH, demencia asociada a la dilisis crnica), pero la mayora son
demencias degenerativas primarias, es decir, debidas a enfermedades
neurodegenerativas cuyo origen puede ser gentico-hereditario
(Huntington) o desconocido (Alzheimer).
Separamos las demencias degenerativas primarias en dos grupos,
en funcin del patrn de sntomas:
Cuando predomina el enlentecimiento tanto motor como psquico, asociado a movimientos anormales y alteraciones emocionales hablamos de un patrn SUBCORTICAL; siguen este patrn las enfermedades degenerativas extrapiramidales (Parkinson,
Huntington).
Cuando predominan los fallos de memoria, la alteracin del lenguaje, los problemas en el reconocimiento y las dificultades en la
realizacin de tareas cotidianas hablamos de un patrn CORTICAL; en stas diferenciamos un patrn FRONTO-TEMPORAL, cuando la enfermedad debuta con una transformacin de la personalidad (que refleja el inicio en las regiones anteriores, como en la
enfermedad de Pick), y un patrn TEMPORO-PARIETAL, cuando
dominan las alteraciones afsicas, aprxicas y las agnosias (como
en la enfermedad de Alzheimer).
M exico A rgentina
C hile U ruguay

6
8
1


7!!"!
""
5!
"!$"!
%! 
2!!

6!&
7!$!
!! 
1&!"
)&
2*!!
!!!
!+
",
!..!

12324567
5

892
1 12324567
128 296 7

123454636

78956

45

8

 

6 

  342

  24

   463

5 
!

2#



6

6

%!
  346
75 9 56

 6

4!4254  2

"#545  6$

2#
7  5 3
76&4 5 3
 4 2  ' 6

%623  5 3

'4&'
!(!

  3

( 9 2 3



6

%6289 2

) 4

3- 

* 6 4 2 6

%629342

%6824462 3 5  34 3

CTO Medicina C/ Nez de Balboa, 115 28006 MADRID (Espaa) Tfno.: (91) 782 43 32 / Fax: (91) 782 43 27
E-mail: secretaria@ctomedicina.com; iberocto@ctomedicina.com WEB: www.ctomedicina.com; www.iberocto.com

PQ Pg. 17

PSIQUIATRA

Seguimiento a distancia

Preparacin Examen de Seleccin 05/06 1 Vuelta

Pregunta 71.-R: 1
Los trastornos de personalidad aumentan el riesgo de presentar trastornos mentales, existiendo algunas relaciones especialmente claras:
Los esquizotpicos sufren con frecuencia episodios psicticos breves; en ellos encontramos una elevada frecuencia de antecedentes
familiares de esquizofrenia; en algunas clasificaciones se consideran formas latentes de esquizofrenia (pues sus sntomas recuerdan
en ocasiones a los de esta enfermedad).
Los esquizoides tambin se relacionan con las enfermedades psicticas, siendo la personalidad anormal que con ms frecuencia
se relaciona con la aparicin de esquizofrenia; de hecho sus rasgos
se solapan con los sntomas negativos de la esquizofrenia en sus
formas simple o residual (pero a diferencia de esos pacientes, los
esquizoides siempre han sido as).
Los paranoides se relacionan muy fuertemente con los trastornos
delirantes, ya sean agudos (psicosis reactivas breves) o crnicos
(paranoia).
Pregunta 72.-R: 3
En los trastornos alimentarios fundamentales (anorexia y bulimia
nerviosa) el sntoma central es el mismo: una preocupacin PATOLGICA por la posibilidad de engordar. Este miedo condiciona su relacin con la comida, a la que ven ms como una enemiga que como
un medio necesario para conseguir una actividad normal. A qu se
debe ese miedo? Muchas veces no llegamos a saberlo, pero subyacen
una serie de pensamientos distorsionados y sobrevalorados (pero NO
delirantes) acerca de las consecuencias del aumento de peso en sus
vidas. No es raro encontrar adems distorsiones en la percepcin de
su esquema corporal, pero algunas pacientes, reconociendo que estn delgadas, no soportan la idea de recuperar peso.
Cuando este trastorno aparece en una chica con una buena capacidad de autocontrol (generalmente con rasgos obsesivos de personalidad) veremos cmo va a ser capaz de:
1. Seguir una dieta restrictiva, aguantando su deseo de comer (la palabra anorexia sugiere la falta de apetito, lo cual no es cierto hasta
que no se alcanza la emaciacin);

2. Como consecuencia de esa dieta se produce:


- Una prdida de peso (se considera importante una prdida del
15% del peso oficialmente aceptado como ideal o un IMC inferior a 17,5).
- Aparece amenorrea.
3. Esto es la anorexia nerviosa restrictiva.

Pregunta 72. Esquema de los trastornos de la conducta alimentaria.

Cuando el trastorno afecta a chicas con un menor autocontrol


(con rasgos ms impulsivos) la dieta va a verse interrumpida por:
1. Los atracones bulmicos en los que:
- Se ingiere una enorme cantidad de alimento.
- En un tiempo llamativamente corto.
- Con una sensacin de prdida de control.
2. Para compensar los atracones aparecen una serie de conductas
destinadas a favorecer la prdida de peso, que van desde las conductas purgantes (vmitos autoprovocados, abuso de laxantes, uso
de diurticos) a otras menos agresivas (alternancia de ayuno y atracones, ejercicio compulsivo).
3. Esto es la bulimia nerviosa.
Sin embargo, vemos tambin algunas formas mixtas; lo ms frecuente
es que con la cronificacin de una anorexia nerviosa las pacientes
comiencen a presentar conductas purgantes SIN tener atracones, para no
absorber lo poco que coman (anorexia restrictiva-purgativa). Adems,

Pregunta 70. Caractersticas bsicas de los trastornos de la personalidad.

1234567
859
2
1 8 2 5




Pg. 18 PQ

1
2345167897923451
1 1 428


 1578

12759
586
8

1268 784
12458 8 4
128 58687
125678965
7
12345678965
7
12&42 7848497
12&42 7848497
12994
9459
1234 994
9459 26
4
 
12
4' 497 2
78764
6
(
12
4' 497 2
78764
6
(
22286
4

22)57 *286
+5
87
12,6
54889492454267 2 88-587 12.54628 88949
1272
2798)8462762
28
7
123
9
62
48845
2#72
'7545%2762
28
7
1234546789

12 8787
12 8789

M exico A rgentina
C hile U ruguay


42257878
12 8487
12
87
12

698
6

1234 87!4"5
872#$%
12345678965
7
12&42 7848497
12994
9459
12
4' 497 2
78764
6
(
229786497 2752
28
97
123
5 7648949
2/6
584 /
120

625768)845

CTO Medicina C/ Nez de Balboa, 115 28006 MADRID (Espaa) Tfno.: (91) 782 43 32 / Fax: (91) 782 43 27
E-mail: secretaria@ctomedicina.com; iberocto@ctomedicina.com WEB: www.ctomedicina.com; www.iberocto.com

Comentarios TEST

Pregunta 70.-R: 4
El trastorno lmite de la personalidad es probablemente el que ms
complicaciones psiquitricas produce; en l se mezclan sntomas de
todas las lneas de la psicopatologa (ansiosos, emocionales, psicticos, alimentacin, impulsividad, abuso de sustancias, etc.), en general
de una intensidad importante pero de una duracin breve, y, casi
siempre, en respuesta a alguna circunstancia vivida como frustrante.
Probablemente por esto se trate del trastorno de la personalidad que
ms puede responder al tratamiento farmacolgico (antidepresivos,
antipsicticos, estabilizadores), teniendo el cuidado de elegir frmacos con un elevado nivel de seguridad en el caso de intoxicacin (no
es raro que estos pacientes protagonicen autolesiones de carcter
impulsivo). Los autores norteamericanos han relacionado este trastorno con el antecedente de abusos sexuales en la infancia, si bien esta
relacin es muy cuestionada fuera de EE.UU.

PSIQUIATRA

Preparacin Examen de Seleccin 05/06 1 Vuelta


con el paso de los aos, las pacientes con anorexia llegan a perder el
control sobre su alimentacin y comienzan a presentar atracones (lo que
algunos llaman bulimarexia). Ms raro es que una chica con bulimia
llegue a enganchar un perodo suficientemente largo de autocontrol como
para que pierda peso y presente amenorrea prolongada.
Pregunta 73.-R: 4
Los atracones bulmicos guardan una estrecha relacin con la capacidad de autocontrol y los rasgos impulsivos de personalidad; por eso no es
raro encontrar en estas pacientes una mayor frecuencia de abuso de
drogas, autolesiones, sntomas depresivos, promiscuidad, etc. Suelen ser
ms frecuentes por la noche (cuando es menos probable que sean descubiertas) y tras un perodo de ayuno. Los alimentos deben ser fciles de
tragar y no es raro que se seleccionen sobre todo dulces (la urgencia del
atracn no hace posible la seleccin de alimentos hipocalricos).
De todas las conductas alimentarias anormales, slo los atracones
son susceptibles de cierto tratamiento farmacolgico; la medicacin
de eleccin son los ISRS, utilizndose dosis mucho ms altas que las
habituales para una depresin.
En general el tratamiento de estas pacientes es multidisciplinar,
implicndose especialistas en endocrinologa y nutricin (el bajo peso
es la principal causa de ingreso), en ginecologa (para prevenir las
consecuencias metablicas de la amenorrea), en modificacin de
conducta (para ayudar en el autocontrol de los atracones y los vmitos) y en psicoterapia (para explorar los motivos que subyacen a esa
preocupacin por la alimentacin).
Pregunta 74.-R: 4
Ver tabla.
Pregunta 75.-R: 4
La narcolepsia (sndrome de Gelineau) es una enfermedad poco
frecuente en la cual se combinan:
Ataques incoercibles del sueo (casi siempre sueo REM).
Con la aparicin de caractersticas del sueo REM en la vigilia:
- Catapleja (prdida brusca del tono muscular).
- Parlisis del sueo (falta de recuperacin del tono muscular tras
el despertar).
- Alucinaciones hipnaggicas e hipnopmpicas (aparicin o persistencia de las imgenes de los sueos en la transicin entre
vigilia y sueo).

Seguimiento a distancia

En su tratamiento se combinan una serie de medidas higinicas


que buscan respetar los perodos naturales de sueo e interrumpir la
actividad con siestas programadas para disminuir la frecuencia de
ataques de sueo con diversos medicamentos destinados a aliviar la
somnolencia (estimulantes anfetamnicos, modafinil) o los fenmenos
REM (antidepresivos, modafinil).
Pregunta 76.-R: 4
El uso de psicofrmacos en los trastornos mentales de la infancia y
la adolescencia est muy poco estudiado (por motivos ticos suelen
hacerse pocos ensayos clnicos) y es muy controvertido (por las hiptesis dominantes que atribuyen la mayora de los problemas mentales
de este grupo de edad a conflictos psicolgicos). Sin embargo, en
algunas entidades est claramente establecida la utilidad de la medicacin, sin que por ello se descarten los abordajes psicolgicos o
sociales:
Trastorno por dficit de atencin con hiperactividad:
- De primera eleccin: estimulantes anfetamnicos (metilfenidato, dextroanfetamina, pemolina).
- De segunda eleccin: antidepresivos tricclicos.
Trastorno de La Tourette:
- De primera eleccin: antipsicticos tpicos incisivos (pimocide,
haloperidol).
- De segunda eleccin: clonidina.
Enuresis nocturna:
- De primera eleccin: tcnicas de modificacin de conducta
(sistemas de alarma).
- De segunda eleccin: desmopresina nasal, imipramina oral.
Mutismo selectivo:
- De primera eleccin: psicoterapias.
- tiles: ISRS (como en las fobias sociales).
Pregunta 77.-R: 4
El trastorno por dficit de atencin con hiperactividad es uno de
los diagnsticos ms frecuentes en Psiquiatra Infantil. Estos nios no
consiguen mantener la atencin un tiempo prolongado, siendo fcilmente distrables por cualquier estmulo y mostrndose sumamente
inquietos. Suele debutar en la primera infancia, pero se hace ms
evidente al iniciar la escolarizacin. De sus tres esferas sintomticas
(dficit de atencin, hiperactividad, impulsividad), las dos primeras

Comentarios TEST

Pregunta 74. Fases del sueo y fenmenos fisiolgicos y patolgicos asociados.



12324256578215 45 572


9


9


5 28

75


6669 
6 27
25

Fase 1 (5%)

3

Beta (>12Hz)
frontal

Alfa
(8-12Hz)
occipital

3

Activo

Descenso


3

Movimientos
rpidos

 !"#"$"
%7&685&6%'()*
(+ ,$-#.!
/0!$,
%(!.0(!)
1)#-)
82
$$)0!#$)
M exico A rgentina
C hile U ruguay

Fase 2 (45%)

Fase 3 (12%)

Theta (4-8 Hz)

Fase 4 (13%)

Sueo "delta" (<4Hz)

Complejos K
husos del sueo

<50%

>50%

6
9 

 27
25

Beta (>12Hz)
ondas "en dientes de
sierra"

Disminuido

Movimientos lentos rotatorios


"en balancn"
Superficial

Atona

Ausentes
Media

Rpidos, conjugados

Sueo "profundo"

Media
Inestabilidad,
arritmias, apneas

Descenso, estabilidad
PRL (+)
GH (+), TSH (-), ACTH (-)

"Sueos",
erecciones peneanas
Homeotermia
Bruxismo

Poiquilotermia
Sonambulismo, terror nocturno

CTO Medicina C/ Nez de Balboa, 115 28006 MADRID (Espaa) Tfno.: (91) 782 43 32 / Fax: (91) 782 43 27
E-mail: secretaria@ctomedicina.com; iberocto@ctomedicina.com WEB: www.ctomedicina.com; www.iberocto.com

Pesadilla

PQ Pg. 19

Seguimiento a distancia

PSIQUIATRA

Preparacin Examen de Seleccin 05/06 1 Vuelta

Pg. 20 PQ

M exico A rgentina
C hile U ruguay

CTO Medicina C/ Nez de Balboa, 115 28006 MADRID (Espaa) Tfno.: (91) 782 43 32 / Fax: (91) 782 43 27
E-mail: secretaria@ctomedicina.com; iberocto@ctomedicina.com WEB: www.ctomedicina.com; www.iberocto.com

Comentarios TEST

suelen mejorar rpidamente con el tratamiento farmacolgico; los


estimulantes anfetamnicos mejoran la capacidad atencional y disminuyen la distraibilidad, permitiendo que el nio aproveche las horas
escolares y reduzca los comportamientos problemticos. Cuando no
se diagnostica a tiempo o predomina la impulsividad es frecuente que
se vayan sumando conductas antisociales (robos, mentiras, peleas),
encontrndose en los estudios evolutivos una clara tendencia al mantenimiento de esos comportamientos y al diagnstico de trastorno
antisocial de la personalidad. Tampoco es raro que se produzca el
abuso de sustancias (a veces como auto-tratamiento) o que persistan
sntomas en la edad adulta.
Conviene tener en cuenta este diagnstico cuando nos enfrentamos a un nio con problemas de comportamiento en el medio escolar, si bien es necesario que encontremos sntomas de las dos esferas
principales (dficit de atencin, hiperactividad), pues existen otras
muchas causas de mal comportamiento en un nio (inicio de un
trastorno antisocial, depresin con predominio de la irritabilidad, problemas de adaptacin en pacientes con discapacidad intelectual).

REUMATOLOGA

Preparacin Examen de Seleccin 05/06 1 Vuelta


1.

Una mujer de 44 aos desarrolla a lo largo de dos meses


inflamacin articular en ambas rodillas, tobillos, carpos, metacarpofalngicas e interfalngicas proximales. Se procede al
estudio del lquido sinovial con artrocentesis de una rodilla con
el siguiente resultado: lquido algo turbio con viscosidad baja,
45.000 clulas/ mm3 (70% neutrfilos), glucosa de 64 mg/dl
(glucemia en sangre de 112 mg/dl) y protenas de 48 g/l. Con los
datos aportados, el diagnstico ms probable es:
1)
2)
3)
4)
5)

2.

3.

Preguntas TEST

Enfermedad mixta del tejido conectivo.


Sndrome de Sjgren.
Lupus inducido por frmacos.
Sndrome antisintetasa.
Lupus eritematoso sistmico.

Factor reumatoide.
Anticuerpos anticardiolipina IgG.
Anticuerpos anticardiolipina IgM.
VDRL.
Test de tromboplastina parcial activado.

Respecto a la enfermedad renal de la panarteritis nodosa


clsica, seale la respuesta FALSA:
1)
2)
3)
4)
5)

6.

Artritis reumatoide.
Tuberculosis.
Sarcoidosis.
Sinovitis por cuerpo extrao.
Sarcoma sinovial.

Una mujer de 34 aos es evaluada en el servicio de ginecologa


por haber presentado tres abortos en el primer trimestre de
embarazo. Como antecedente present hace 4 aos una tromboflebitis popltea izquierda. Ante la sospecha de sndrome
antifosfolpido NO es de utilidad para orientar este diagnstico
la determinacin de:
1)
2)
3)
4)
5)

5.

8.

Se produce en la mayora de los pacientes.


La forma ms habitual de presentacin es la glomerulonefritis.
Suele producir deterioro de la funcin renal.
En la arteriografa se aprecian microaneurismas.
La hipertensin arterial es una manifestacin habitual.

Seale qu manifestacin es comn a la panarteritis nodosa


clsica y a la panarteritis microscpica:
1)

Presencia de aneurismas.
M exico A rgentina
C hile U ruguay

Panarteritis nodosa.
Granulomatosis de Wegener.
Granulomatosis alrgica (enfermedad de Churg-Strauss).
Sndrome de poliangetis de superposicin.
Vasculitis por hipersensibilidad.

Mujer de 59 aos con fiebre, artralgias y deformidad nasal con


obstruccin de vas areas superiores y secrecin hemorrgica.
Despus de 6 meses de iniciado el proceso se percibe hundimiento de la pirmide nasal, dificultad para tragar, dolor farngeo y
ronquera. En la exploracin de la faringe aparece enrojecida con
encas tumefactas. En las fosas nasales se aprecia un tejido
blanquecino que las obstruye parcialmente. Analtica: Hb 12,4
g/dl, leucocitos 10.100/mm3 (frmula normal), Cr 2 mg/dl; factor
reumatoide positivo; crioglobulinas negativas. Orina 50-100
hemates/campo con cilindros granulosos e hialinos y proteinuria de 850 mg/l. NO espera encontrar en esta enfermedad:
1)
2)
3)
4)
5)

9.

Afectacin de vnulas.
Respuesta al tratamiento con ciclofosfamida.
Afectacin pulmonar.
Glomerulonefritis.

Mujer de 34 aos que consulta por fiebre y dolor torcico. En


la radiologa de trax se observan infiltrados pulmonares
bilaterales y a nivel cutneo prpura palpable. En la analtica
destaca la presencia de 1.200 eosinfilos/mm3. Cul es el
diagnstico ms probable?:
1)
2)
3)
4)
5)

En una mujer de 36 aos, con artritis intermitente de articulaciones de manos, rodillas y tobillos se detecta en el laboratorio
la presencia de anticuerpos antinucleares a ttulo 1/640, antiADNss, anti-ADNds, y antihistona positivos, el diagnstico ms
probable es:
1)
2)
3)
4)
5)

4.

7.

Artritis psorisica.
Artritis reactiva.
Artritis por cristales de cido rico.
Lupus eritematoso sistmico.
Artritis reumatoide.

Una biopsia sinovial informada por el patlogo como sinovitis


crnica inespecfica es compatible con:
1)
2)
3)
4)
5)

2)
3)
4)
5)

Seguimiento a distancia

Ndulos pulmonares.
Granulomas larngeos.
Destruccin sea orbitaria.
Glomerulonefritis.
Proptosis.

El diagnstico diferencial de la granulomatosis de Wegener NO


incluye:
1)
2)
3)
4)
5)

Granulomatosis linfomatoide.
Sndrome de Goodpasture.
Granulomatosis alrgica (sndrome de Churg-Strauss).
Enfermedad de Buerger.
Neumona por Legionella.

10. Mujer de 22 aos que ingresa por ictus cerebral. En la exploracin no se detecta pulso arterial en ambas extremidades superiores y se ausculta un soplo de insuficiencia artica. Analtica:
discreta anemia con elevacin de VSG. Indique el diagnstico
ms probable:
1)
2)
3)
4)
5)

Panarteritis nodosa.
Arteritis de Takayasu.
Lupus eritematoso con anticoagulante lpico.
Sndrome antifosfolpido catastrfico
Vasculitis del sistema nervioso central.

11. En relacin con la arteritis de clulas gigantes, NO es cierto que:


1)
2)
3)

La biopsia normal de la arteria temporal descarta el diagnstico.


Se acompaa de elevacin marcada de la VSG.
Afecta a pacientes de edad avanzada.

CTO Medicina C/ Nez de Balboa, 115 28006 MADRID (Espaa) Tfno.: (91) 782 43 32 / Fax: (91) 782 43 27
E-mail: secretaria@ctomedicina.com; iberocto@ctomedicina.com WEB: www.ctomedicina.com; www.iberocto.com

RM Pg. 1

4)
5)

A menudo hay sntomas de polimialgia reumtica.


Responde espectacularmente al tratamiento con corticoides.

12. Seale cul de las siguientes caractersticas de la enfermedad de


SchnleinHenoch NO es comn a las vasculitis predominantemente cutneas (de hipersensibilidad):
1)
2)
3)
4)
5)

Afectacin de capilares y vnulas.


Fenmeno de leucocitoclasia.
Prpura palpable como manifestacin ms frecuente.
Buen pronstico.
Afectacin visceral.

13. Respecto a las vasculitis predominantemente cutneas, indique


la respuesta correcta:
1)
2)
3)
4)
5)

Se relaciona con infecciones por virus hepatotropos.


No se observa relacin entre estas vasculitis y los linfomas.
La panarteritis microscpica se incluye en este grupo.
La prpura palpable desaparece con la vitropresin.
A excepcin del sndrome de Sjgren, el resto de conectivopatas se relacionan con estas vasculitis.

14. Mujer de 43 aos que presenta desde hace tres meses cefalea
pulstil parietotemporal derecha acompaada de nuseas. De
forma brusca presenta un episodio de prdida de fuerza en
hemicuerpo izquierdo que se recupera en 10 minutos y unas
horas despus, prdida de memoria y dificultad para la expresin verbal. En el TC no se encuentran alteraciones relevantes
y en la RM se observan cambios de seal periventriculares
extensos y mltiples lesiones perifricas corticales y en sustancia blanca. Qu prueba elegira para confirmar el diagnstico?:
1)
2)
3)
4)
5)

c-ANCA.
Arteriografa mesentrica.
TC toracoabdominal.
Biopsia de arteria temporal.
Biopsia cerebral.

15. Un varn de 23 aos viene presentando lceras orales dolorosas de forma recurrente en los ltimos siete meses. En las ltimas
semanas adems presenta lceras escrotales, lesiones de eritema nodoso y dolor, tumefaccin e impotencia funcional en
rodillas y tobillos. Presenta as mismo elevacin de la VSG. El
factor reumatoide y los ANA son negativos. No presenta sntomas oculares y la exploracin oftalmolgica es normal. Para el
tratamiento de este paciente, en este momento se podran
incluir todas, EXCEPTO una de las siguientes medidas:
1)
2)
3)
4)
5)

REUMATOLOGA

Preparacin Examen de Seleccin 05/06 1 Vuelta

Colchicina.
AINEs.
Ciclosporina.
Corticoides en dosis bajas.
Corticoides tpicos orales.

16. Varn de 42 aos que hace 7 meses comenz con fatiga,


artralgias y congestin nasal, seguido de un episodio de hemoptisis. La analtica mostr elevacin de la VSG, leucocitosis,
anemia, alteraciones en el sedimento urinario as como infiltrados pulmonares. El ttulo de c-ANCA fue de 1:128. Qu
tratamiento recomendara en este paciente?:

Pg. 2 RM

M exico A rgentina
C hile U ruguay

1)
2)
3)
4)
5)

Ciclosporina 0,5 mg/kg/semanal.


Metotrexate 1 mg/semanal.
Prednisona 1 mg/kg/da y ciclofosfamida 2 mg/kg/da.
Colchicina 1 mg/da.
Prednisona 0,5 mg/kg/da.

17. Una mujer de 46 aos desarrolla de forma progresiva en los


ltimos meses parestesias en pie izquierdo, mano derecha y
posteriormente izquierda, junto con fiebre, prdida cuantificada de 10 kilos de peso, dolor abdominal difuso que predomina despus de comer con rectorragia ocasional y artromialgias. En la exploracin destaca la presencia de hipoestesia
del dorso del pie izquierdo, tres primeros dedos de la mano
derecha e imposibilidad para la dorsiflexin de pie izquierdo
con signos de irritacin peritoneal en la palpacin abdominal.
Hemoglobina 9,6 gr/dl, 13.230 leucocitos/mm3 (frmula normal) y creatinina de 3,4 mg/dl. Radiografa de trax normal.
Los hallazgos referidos son compatibles con el diagnstico de:
1)
2)
3)
4)
5)

Enfermedad de Goodpasture.
Sndrome de Schnlein-Henoch.
Panarteritis nodosa clsica.
Enfermedad de Wegener.
Sndrome poliangetico de superposicin.

18. En una de las siguientes enfermedades la hiperuricemia NO se


debe al aumento del catabolismo de las purinas:
1)
2)
3)
4)
5)

Policitemia vera.
Anemia hemoltica.
Psoriasis extensa.
Leucemia linftica crnica.
Enfermedad de Lesch-Nyhan.

19. Un varn de 76 aos acude al servicio de urgencias por


presentar dolor en flanco derecho de instauracin brusca
asociado a nuseas y vmitos. Refiere episodios similares que
han sido etiquetados de clicos nefrticos as como episodios
de podagra de repeticin. En el estudio analtico presenta una
uricemia de 9,3 mg/dl y una uricosuria en 24 horas de 356 mg.
Una vez resuelto el episodio agudo, qu actitud teraputica
adoptara sobre la hiperuricemia?:
1)
2)
3)
4)
5)
20.

No precisa tratamiento, salvo beber abundante lquido.


Alopurinol y colchicina.
Benzobromarona.
Colchicina aislada.
Sulfinpirazona.

Un paciente de 42 aos, diagnosticado de enfermedad por


depsito de cristales de pirofosfato clcico, requiere la realizacin de las siguientes determinaciones bioqumicas, EXCEPTO:
1)
2)
3)
4)
5)

PTH.
Magnesio.
Ferritina.
Fosfatasa alcalina.
ACTH.

21. La aparicin de artrosis en una de las siguientes articulaciones


debe hacerle sospechar la presencia de una artropata crnica
por microcristales:

CTO Medicina C/ Nez de Balboa, 115 28006 MADRID (Espaa) Tfno.: (91) 782 43 32 / Fax: (91) 782 43 27
E-mail: secretaria@ctomedicina.com; iberocto@ctomedicina.com WEB: www.ctomedicina.com; www.iberocto.com

Preguntas TEST

Seguimiento a distancia

REUMATOLOGA

Preparacin Examen de Seleccin 05/06 1 Vuelta


1)
2)
3)
4)
5)

Rodilla.
Cadera.
Codo.
Interfalngica proximal.
Interfalngica distal.

22. Una paciente de 81 aos presenta un episodio de monoartritis


aguda en tobillo desde hace 24 h. En los ltimos aos ha
presentado episodios similares en carpo y rodilla. La radiologa
muestra imgenes compatibles con condrocalcinosis tanto en
la rodilla como en el tobillo y el estudio del lquido sinovial
cristales de forma rectangular con estudio microbiolgico
negativo. En el tratamiento de esta paciente estaran indicadas
todas estas medidas, SALVO:
1)
2)
3)
4)
5)

AINEs.
Reposo de la articulacin.
Drenaje de la articulacin y administracin de corticoides
intraarticulares.
Corticoides sistmicos.
Colchicina.

23. El depsito de hidroxiapatita NO se relaciona con una de las


siguientes manifestaciones clnicas:
1)
2)
3)
4)
5)

Hombro de Milwaukee.
Periartritis escapulohumeral.
Artrosis.
Monoartritis aguda.
Poliartritis simtrica.

24. El depsito de cristales de oxalato clcico se asocia habitualmente a:

Preguntas TEST

1)
2)
3)
4)
5)

Oxalosis primaria.
Insuficiencia renal terminal.
Insuficiencia cardaca.
Hemocromatosis.
Hiperparatiroidismo.

25. Mujer de 76 aos, sin antecedentes reumatolgicos, presenta


3 das despus de una apendicectoma inflamacin articular
en rodilla derecha y fiebre de 38,5 C. El lquido sinovial
muestra 75.000 clulas/mm3, siendo el 98% PMN. El Gram y
cultivo son negativos. En el microscopio de luz polarizada se
observan algunos cristales de forma rectangular. Su diagnstico es:
1)
2)
3)
4)
5)

Artritis sptica.
Artritis tuberculosa reagudizada.
Gota.
Pseudogota.
Artritis reactiva.

26. Entre las siguientes manifestaciones clnicas es ms habitual en


el lupus eritematoso sistmico:
1)
2)
3)
4)
5)

Artritis.
Anemia hemoltica.
Sndrome cognitivo leve.
Pericarditis.
Glomerulonefritis proliferativa focal.
M exico A rgentina
C hile U ruguay

Seguimiento a distancia

27. Como criterio diagnstico de lupus eritematoso sistmico se


incluye:
1)
2)
3)
4)
5)

Anticuerpos anti-ADNss.
Anticuerpos anti-RNP.
Psicosis lpica.
Anemia microangioptica.
Leucopenia farmacolgica.

28. Cul de los siguientes factores de predisposicin gentica es


FALSO en el lupus eritematoso sistmico?:
1)
2)
3)
4)
5)

Aumento de concordancia de la enfermedad en gemelos


monocigticos.
Predisposicin familiar a padecer LES.
Asociacin entre ttulos elevados de anti-ADN IgG y nefritis
lpica con DR2 y DR3.
Asociacin de ac anti-Ro y lupus eritematoso cutneo subagudo y DR4.
Asociacin del anticoagulante lpico con DR4 y DR7.

29. La presencia de anticoagulante lpico significa un mayor riesgo


de presentar las siguientes manifestaciones, SALVO:
1)
2)
3)
4)
5)

Pericarditis.
Trombocitopenia.
Trombosis venosas.
Falsa positividad de la serologa de les.
Livedo reticularis.

30. Uno de los hallazgos histolgicos en los pacientes con afectacin renal en el lupus eritematoso sistmico es reversible:
1)
2)
3)
4)
5)

Necrosis glomerular.
Esclerosis glomerular.
Fibrosis intersticial.
Atrofia tubular.
Semilunas fibrosas.

31. En relacin al LES y el embarazo, seale la respuesta FALSA:


1)
2)
3)
4)
5)

Los antipaldicos y los inmunosupresores estn contraindicados.


Los corticoides de eleccin son los de larga vida media.
El mtodo anticonceptivo de eleccin son las medidas fsicas.
La amenorrea es frecuente.
Son frecuentes los brotes de la enfermedad en el puerperio.

32. En relacin a las alteraciones hematolgicas del lupus eritematoso sistmico, seale la respuesta correcta:
1)
2)
3)
4)
5)

La hemlisis es la causa ms frecuente de anemia.


La esplenectoma est contraindicada.
La trombopenia no suele ser grave.
La leucopenia obliga a administrar cobertura antibitica
profilctica.
La mayora de los pacientes con test de Coombs directo
positivo presentan hemlisis.

33. Dentro de las manifestaciones neurolgicas en el lupus eritematoso NO espera encontrar:


1)

Anticuerpos antineuronales.

CTO Medicina C/ Nez de Balboa, 115 28006 MADRID (Espaa) Tfno.: (91) 782 43 32 / Fax: (91) 782 43 27
E-mail: secretaria@ctomedicina.com; iberocto@ctomedicina.com WEB: www.ctomedicina.com; www.iberocto.com

RM Pg. 3

2)
3)
4)
5)

Anticuerpos antimielina.
Polineuropata.
Mielitis transversa.
Afectacin ms habitual dentro de los pares craneales del
facial.

34. El anticuerpo ms habitual en los pacientes con lupus eritematoso es:


1)
2)
3)
4)
5)

Antifosfolpido.
Anti-RNP.
Antihistona.
Anti-ADN.
Antinucleares.

2)
3)
4)
5)

Inicio de los sntomas de forma inmediata al iniciar el tratamiento.


Serositis.
Sndrome orgnico cerebral.
Anticuerpos anti-Sm.
Glomerulonefritis proliferativa focal.

36. Una paciente de 23 aos, diagnosticada de LES y actualmente sin


tratamiento, presenta en las ltimas semanas astenia, fiebre
vespertina, artritis migratoria y dolor de caractersticas pleurticas en la inspiracin profunda. El tratamiento recomendado es:
1)
2)
3)
4)
5)

Corticoides en dosis altas (1 mg/kg/24 h).


Azatioprina.
Azatioprina y corticoides.
AINEs.
Bolos mensuales de ciclofosfamida.

37. Mujer de raza negra de 37 aos, con historia de tres meses de


rash persistente que afecta a la cara, parte anterior del trax,
palmas y plantas de pies. A la exploracin se observan mltiples
lesiones hipopigmentadas y atrficas. La biopsia de la piel del
trax indica un infiltrado denso en la dermis profunda compatible con lupus discoide. Qu tratamiento recomendara?:
1)
2)
3)
4)
5)

AINEs.
Bolos de esteroides i.v.
Hidroxicloroquina oral.
Ciclofosfamida.
Sales de oro.

38. Mujer de 22 aos, diagnosticada de LES con antecedente de rash


facial, poliartritis, pericarditis, y anticuerpos antinucleares y antiADN. Refiere deterioro de las funciones superiores con labilidad
emocional, falta de concentracin y de memoria en su trabajo.
Tambin refiere cefalea recurrente refractaria al uso de analgsicos. Recibe tratamiento con AINEs y corticoides tpicos cutneos. Analtica: ANA y anti-ADN, anti-Ro y niveles bajos de
complemento. En la RM muestra atrofia ligera, as como reas de
captacin de alta intensidad, ms evidentes en regin parietotemporal. Recomendara para su tratamiento:
1)
2)

Corticoides orales.
Suspender los AINEs.

Pg. 4 RM

M exico A rgentina
C hile U ruguay

3)
4)
5)

Suspender los corticoides tpicos.


Carbamacepina.
Antidepresivos.

39. En una paciente de 30 aos con lupus eritematoso sistmico que


presenta trombopenia autoinmune severa con clnica hemorrgica asociada, NO es de utilidad una de las siguientes
medidas:
1)
2)
3)
4)
5)

35. En un paciente diagnosticado de lupus inducido por frmacos


espera encontrar:
1)

REUMATOLOGA

Preparacin Examen de Seleccin 05/06 1 Vuelta

Esteroides.
Gammaglobulina.
Ciclofosfamida.
Penicilamina.
Esplenectoma.

40. NO es uno de los criterios diagnsticos de la artritis reumatoide:


1)
2)
3)
4)
5)

lceras orales.
Ndulos subcutneos.
Factor reumatoide positivo.
Osteoporosis yuxtaarticular.
Artritis simtrica.

41. Con respecto al factor reumatoide, indique la respuesta INCORRECTA:


1)
2)
3)
4)
5)

Aparece de forma constante en pacientes con ndulos


reumatoides o vasculitis.
Su frecuencia disminuye con la edad en la poblacin general.
Se trata de ac contra la fraccin Fc de IgG.
Las pruebas habituales suelen detectar factor reumatoide IgM.
Aparece ocasionalmente tras una transfusin o vacunacin.

42. Indique cul de los siguientes factores NO indica mal pronstico en los pacientes con artritis reumatoide:
1)
2)
3)
4)
5)

Afectacin de las manos.


Ttulos elevados de factor reumatoide.
Ndulos subcutneos.
Elevacin de la VSG.
Sndrome de Felty.

43. Una mujer de 62 aos, con artritis reumatoide de larga evolucin, presenta desde hace 72 horas dolor e inflamacin en el
codo derecho con febrcula. La artrocentesis muestra un lqui3
do de aspecto purulento con ms de 100.000 leucocitos/mm
y marcado descenso de la glucemia. El germen responsable de
esta situacin con ms frecuencia es:
1)
2)
3)
4)
5)

Staphylococcus epidermidis.
Streptococcus viridans.
Staphylococcus aureus.
Gonococo.
Mycobacterium tuberculosis.

44. La subluxacin atloaxoidea antero-posterior se define por:


1)
2)
3)

Presencia de erosiones en la apfisis odontoides.


Aumento de distancia entre la apfisis odontoides y el cuerpo
del atlas.
Impresin basilar.

CTO Medicina C/ Nez de Balboa, 115 28006 MADRID (Espaa) Tfno.: (91) 782 43 32 / Fax: (91) 782 43 27
E-mail: secretaria@ctomedicina.com; iberocto@ctomedicina.com WEB: www.ctomedicina.com; www.iberocto.com

Preguntas TEST

Seguimiento a distancia

REUMATOLOGA

Preparacin Examen de Seleccin 05/06 1 Vuelta


4)
5)

Erosiones en odontoides y en articulaciones interapofisarias


con movilidad patolgica.
Desplazamiento lateral de la apfisis odontoides respecto al
atlas.

45. La deformidad en "cuello de cisne" que aparece en la artritis


reumatoide se caracteriza por:
1)
2)
3)
4)
5)

Hiperextensin de la interfalngica proximal con flexin de


la interfalngica distal.
Flexin de la interfalngica proximal con extensin de la
interfalngica distal.
Desviacin cubital de la mueca.
Flexin de las interfalngicas distales.
Flexin de interfalngicas proximal y distal.

46. Una paciente de 54 aos, diagnosticada de artritis reumatoide


seronegativa de larga evolucin, ha recibido a lo largo de su
enfermedad mltiples tratamientos de fondo. Actualmente
sigue tratamiento con AINEs y corticoides en dosis bajas. En los
ltimos 6 meses ha desarrollado proteinuria de intensidad
creciente, sin hematuria, hasta alcanzar el rango nefrtico. En
las ltimas determinaciones analticas ha comenzado adems
a presentar deterioro de la funcin renal. La afectacin renal
ms probable en esta paciente ser:
1)
2)
3)
4)
5)

Glomerulonefritis membranosa tarda por sales de oro.


Nefritis intersticial por AINEs.
Trombosis de la vena renal.
Amiloidosis secundaria.
Vasculitis reumatoide.

47. La afectacin ocular ms frecuente en la artritis reumatoide es:


1)
2)
3)
4)
5)

Escleritis.
Escleromalacia perforante.
Queratoconjuntivitis seca.
Uvetis anterior aguda.
Uvetis posterior.

Preguntas TEST

48. El derrame pleural que se produce en la artritis reumatoide se


caracteriza por:
1)
2)
3)
4)
5)

Recuento celular bajo.


Caractersticas de trasudado.
Adenosindeaminasa baja.
Aparecer siempre acompaado de fibrosis pulmonar intersticial.
Concentracin baja de glucosa.

49. En la artritis reumatoide, la fibrosis pulmonar intersticial en sus


fases iniciales NO se caracteriza por:
1)
2)
3)
4)
5)

Descenso de la PO2 con el ejercicio.


Descenso del ndice VEF1/CVF.
Discreto aumento de la trama reticular en la radiologa de trax.
Descenso de la capacidad de difusin pulmonar.
Ausencia de cianosis en reposo.

50. Un varn de 37 aos presenta desde hace 6 meses inflamacin


articular que incluye articulaciones interfalngicas proximales,
carpos, rodillas y tobillos que cede parcialmente al tomar AINEs
M exico A rgentina
C hile U ruguay

Seguimiento a distancia

junto con rigidez matutina en ambas manos de varias horas de


evolucin. En las ltimas semanas ha desarrollado lceras dolorosas en piernas, as como polineuropata de predominio sensitivo. Sobre esta situacin, seale la respuesta correcta:
1)
2)
3)
4)
5)

El diagnstico de artritis reumatoide se debe descartar por los


nuevos sntomas aparecidos.
El CH50 en suero estar elevado.
El paciente cumple criterios para el diagnstico de sndrome
de Felty.
La biopsia cutnea mostrar probablemente datos de vasculitis necrotizante.
Debemos sospechar el desarrollo de un linfoma.

51. Paciente de 36 aos, diagnosticada hace 6 meses de artritis


reumatoide. Presenta factor reumatoide positivo, anticuerpos
antinucleares negativos y HLA DR3. Tras seguir tratamiento con
sales de oro parenterales desarrolla proteinuria de rango
nefrtico. Tras suspender las sales de oro, NO se debera usar
como tratamiento en esta paciente:
1)
2)
3)
4)
5)

Hidroxicloroquina.
Sulfasalacina.
D-penicilamina.
Metotrexate.
Corticoides.

52. En qu grupo de pacientes con artritis crnica juvenil aparece


con mayor frecuencia la uvetis crnica?:
1)
2)
3)
4)
5)

Sistmico.
Poliarticular seropositivo.
Oligoarticular de inicio tardo.
Poliarticular seronegativo.
Oligoarticular de inicio precoz.

53. En una nia de 4 aos con inflamacin de la rodilla izquierda


y tobillo derecho de 6 meses de evolucin en cuyo lquido
sinovial se hallan 34.450 clulas/mm3 con predominio de polimorfonucleares, sin microorganismos en el cultivo, que no ha
presentado fiebre ni lesiones cutneas, espera encontrar en el
estudio analtico:
1)
2)
3)
4)
5)

AAN.
Anticardiolipina.
VSG normal.
DR4.
B27.

54. En la artritis crnica juvenil, el tratamiento con esteroides est


especialmente indicado en:
1)
2)
3)
4)
5)

Presentacin sistmica.
Oligoarticular precoz.
Oligoarticular tarda.
Poliarticular seropositiva.
Poliarticular seronegativa.

55. Indique la caracterstica que NO suele aparecer en las espondiloartropatas inflamatorias seronegativas:
1)

Afectacin poliarticular simtrica.

CTO Medicina C/ Nez de Balboa, 115 28006 MADRID (Espaa) Tfno.: (91) 782 43 32 / Fax: (91) 782 43 27
E-mail: secretaria@ctomedicina.com; iberocto@ctomedicina.com WEB: www.ctomedicina.com; www.iberocto.com

RM Pg. 5

2)
3)
4)
5)

Curso evolutivo crnico.


Entesitis.
Ausencia de ndulos subcutneos.
Uvetis anterior aguda.

56. Una mujer de 30 aos, sana hasta hace 6 meses y sin antecedentes epidemiolgicos relevantes, presenta en este ltimo periodo
dolor e inflamacin en la rodilla derecha y articulacin coxofemoral izquierda, as como dolor y rigidez de predominio
nocturno en la zona lumbar. En la radiografa de articulaciones
perifricas se observa disminucin del espacio articular, que es
ms marcada en la articulacin coxofemoral, mientras que en
la radiografa de pelvis se observan erosiones en ambas articulaciones sacroilacas con esclerosis marginal del hueso ilaco. En
la exploracin fsica presenta dolor y limitacin a la movilidad
en las articulaciones perifricas afectadas as como maniobras
sacroilacas de apertura positivas. No se encuentran lesiones
cutneas ni ungueales y el ojo derecho aparece enrojecido sin
disminucin de la agudeza visual. Analtica: anemia normoctica y normocrmica con VSG elevada y factor reumatoide
negativo. Con estos datos, el diagnstico ms probable es:
1)
2)
3)
4)
5)

Artritis reumatoide.
Artritis reactiva.
Artritis psorisica.
Artritis brucelar.
Espondilitis anquilosante.

57. Seale cul de las siguientes NO es una manifestacin extraarticular de la espondilitis anquilosante:
1)
2)
3)
4)
5)

Nefropata IgA.
Fibrosis apical pulmonar bilateral.
Uvetis anterior.
Meningoencefalitis.
Insuficiencia artica.

58. En el tratamiento de la espondilitis anquilosante, NO es de


utilidad:
1)
2)
3)
4)
5)

AINEs.
Sulfasalazina.
Corticoides intralesionales.
Rehabilitacin.
Sales de oro.

59. Varn de 30 aos que consulta por artritis de rodilla derecha


de inicio brusco y talalgia bilateral. Las caractersticas del
lquido sinovial son: 40.000 leucocitos/mm3, protenas elevadas y glucosa normal, sin microorganismos en el Gram. Qu
diagnstico le parece ms probable?:
1)
2)
3)
4)
5)

Lupus eritematoso sistmico.


Artritis reactiva.
Artritis reumatoide.
Artritis gonoccica.
Artritis infecciosa.

60. Uno de los siguientes grmenes NO est implicado en la aparicin


de artritis reactiva de origen gastrointestinal. Selelo:
1)

Campylobacter.

Pg. 6 RM

M exico A rgentina
C hile U ruguay

REUMATOLOGA

Preparacin Examen de Seleccin 05/06 1 Vuelta


2)
3)
4)
5)

Pseudomonas.
Yersinia.
Shigella.
Salmonella.

61. Indique la manifestacin cutnea que NO es tpica en pacientes


con artritis reactiva:
1)
2)
3)
4)
5)

lceras orales.
Queratodermia blenorrgica.
Balanitis circinada.
Oniclisis de las uas.
Pioderma gangrenoso.

62. Para establecer el diagnstico diferencial entre la artritis reactiva y la artritis gonoccica, es correcto que:
1)
2)
3)
4)
5)

En las dos enfermedades hay tenosinovitis.


El dolor lumbosacro es caracterstico de la artritis reactiva
pero no de la gonoccica.
El hallazgo de cultivo positivo en crvix de gonococo excluye
el diagnstico de artritis reactiva.
En ambas enfermedades puede haber cultivo positivo del
lquido sinovial.
En ambos casos la base del tratamiento son los AINEs.

63. Mujer de 54 aos con psoriasis de varios aos de evolucin


tratada tpicamente, presenta desde hace 7 meses inflamacin
marcada de las articulaciones interfalngicas proximales y
distales, carpos, rodilla y cadera izquierda. En la radiografa de
manos se observa destruccin articular marcada de varias
articulaciones con tendencia a la luxacin. En la exploracin
cutnea se observan placas psorisicas extensas que cubren
gran parte del abdomen y de extremidades inferiores. En la
analtica destaca elevacin marcada de VSG y PCR con FR a
ttulos bajos. En este momento, el tratamiento ms adecuado es:
1)
2)
3)
4)
5)

Metotrexate.
Cloroquina.
Sales de oro.
Penicilamina.
Sulfasalazina.

64. Respecto a la afectacin articular de la enfermedad inflamatoria


intestinal, seale la respuesta FALSA:
1)
2)
3)
4)
5)

La artritis perifrica es una afectacin ms frecuente que la


espondilitis.
El 70% de los pacientes con espondilitis son HLA B27
positivos.
La artritis perifrica puede ser oligoarticular.
La artritis suele ser clnicamente de comienzo agudo y no
destructiva.
La mitad de los pacientes con artritis perifrica, sin espondilitis, son HLA B27 positivos.

65. Un varn de 82 aos consulta por presentar dolor de caractersticas mecnicas en la cadera derecha. En el estudio radiolgico
se observa un patrn mixto (esclertico y ltico) en pelvis y hueso
femoral; en la analtica presenta una marcada elevacin de la
hidroxiprolinuria y de la fosfatasa alcalina. Como complicacin
de esta enfermedad NO espera encontrar:

CTO Medicina C/ Nez de Balboa, 115 28006 MADRID (Espaa) Tfno.: (91) 782 43 32 / Fax: (91) 782 43 27
E-mail: secretaria@ctomedicina.com; iberocto@ctomedicina.com WEB: www.ctomedicina.com; www.iberocto.com

Preguntas TEST

Seguimiento a distancia

REUMATOLOGA

Preparacin Examen de Seleccin 05/06 1 Vuelta


1)
2)
3)
4)
5)

Hipercalciuria.
Artropata coxofemoral.
Fractura patolgica.
Insuficiencia cardaca.
Hipovitaminosis D.

66. NO se ha relacionado con la patogenia de la enfermedad de


Paget:
1)
2)
3)
4)
5)

Virus del sarampin.


Virus respiratorio sincitial.
Virus del moquillo canino.
Agregacin familiar.
HLA DR4.

67. La degeneracin sarcomatosa es la complicacin ms grave en


la enfermedad de Paget. Indique la respuesta FALSA respecto a
la misma:
1)
2)
3)
4)
5)

Las localizaciones ms habituales son el fmur, hmero,


crneo y pelvis.
El pronstico es mejor que en los osteosarcomas primarios.
Son causa habitual de sarcoma seo en los ancianos.
Los signos sugestivos de su aparicin son la fractura patolgica
y la tumefaccin de partes blandas.
Puede existir un aumento de fosfatasa alcalina sobre la basal.

68. Cul NO se considera una indicacin para el tratamiento de la


enfermedad de Paget?:
1)
2)
3)
4)
5)

Dolor persistente en el hueso pagtico.


Compresin nerviosa.
Niveles de fosfatasa alcalina superiores al 10% de la normalidad.
Hipercalcemia.
Preparacin para ciruga ortopdica.

69. La definicin actual de osteoporosis postmenopusica es:


1)

Preguntas TEST

2)
3)
4)
5)

Reduccin de la masa sea suficiente como para producir


una fractura.
Osteopenia radiolgica con o sin fractura asociada.
Descenso de masa sea igual o superior a 2,5 DE respecto a
la media de las mujeres de la edad de la paciente.
Descenso de masa sea igual o superior a 2,5 DE respecto a
la media de las mujeres jvenes.
Fractura vertebral o de cuello de fmur en mujer postmenopusica independientemente de la densidad sea.

70. Una de las siguientes enfermedades NO se asocia a la aparicin


de osteoporosis:
1)
2)
3)
4)
5)

Artrosis.
Hipogonadismo.
Osteognesis imperfecta.
Hiperparatiroidismo.
Hipertiroidismo.

71. Respecto a la osteoporosis tipo II o del anciano, seale la


respuesta INCORRECTA:
1)

Afecta a varones y mujeres con una edad por encima de los


75 aos.
M exico A rgentina
C hile U ruguay

2)
3)
4)
5)

Seguimiento a distancia

La concentracin circulante media de 1,25-(OH)2-D3 est


elevada.
Se asocia en la clnica con la aparicin de fracturas del cuello
femoral.
Los niveles de PTH tienden a estar elevados.
Se detecta por densitometra una disminucin de hueso
cortical.

72. La osteognesis imperfecta tipo I NO incluye:


1)
2)
3)
4)
5)

Sordera.
Ectopia del cristalino.
Esclerticas azules.
Aumento de incidencia de fracturas.
Herencia autosmica dominante.

73. Entre los tratamientos en la osteoporosis postmenopusica NO


incluimos:
1)
2)
3)
4)
5)

Risedronato.
Alendronato.
Estrgenos.
Raloxifeno.
Mitramicina.

74. En una anciana ingresada en un hospital de crnicos con inadecuada alimentacin y exposicin solar se sospecha que haya
desarrollado una osteomalacia ante la persistencia de dolores
seos generalizados y debilidad muscular. En el estudio analtico
NO espera encontrar para confirmar el diagnstico:
1)
2)
3)
4)
5)

Hiperfosforemia.
Hiperparatiroidismo secundario.
Hipocalcemia.
Aumento de la depuracin renal de fosfato.
Descenso de la 25(OH)D3.

75. En la osteomalacia secundaria a insuficiencia renal crnica, a


diferencia de la causada por dficit nutricional, NO encontraremos:
1)
2)
3)
4)
5)

Fracturas de Looser.
Descenso de 25-(OH)-D3.
Descenso de 1,25-(OH)2-D3.
Hiperparatiroidismo secundario.
Normocalcemia o hipocalcemia.

76. Los anticuerpos antitopoisomerasa se asocian en la esclerosis


sistmica a:
1)
2)
3)
4)
5)

Fenmeno de Raynaud intenso.


Esclerodactilia.
Fibrosis pulmonar.
Hipertensin pulmonar.
Cirrosis biliar primaria.

77. Existe relacin entre ciertos factores ambientales y la esclerosis


sistmica desde el punto de vista patognico. Cul de los
siguientes NO se ha relacionado?:
1)
2)

Polvo de slice.
Silicona (implantes quirrgicos).

CTO Medicina C/ Nez de Balboa, 115 28006 MADRID (Espaa) Tfno.: (91) 782 43 32 / Fax: (91) 782 43 27
E-mail: secretaria@ctomedicina.com; iberocto@ctomedicina.com WEB: www.ctomedicina.com; www.iberocto.com

RM Pg. 7

3)
4)
5)

Cloruro de vinilo.
Tricloroetileno.
Asbesto.

78. El estudio bsico de un paciente con fenmeno de Raynaud


incluye las siguientes pruebas complementarias MENOS una,
que es FALSA. Sealar esta:
1)
2)
3)
4)
5)

Crioglobulinemia.
Anticuerpos anti-SCL 70 y anticentrmero.
Capilaroscopia.
Radiografa de trax.
Anticuerpos antihistonas.

79. Mujer de 34 aos, diagnosticada hace unos 10 aos de sndrome de CREST. En los ltimos meses presenta disnea progresiva.
En la radiografa de trax se observa aumento del cono de la
arteria pulmonar y oligohemia perifrica; el ECG muestra
signos de crecimiento de cavidades derechas. Cul le parece
el diagnstico ms probable?:
1)
2)
3)
4)
5)

Fibrosis pulmonar.
Broncoaspiracin por reflujo gstrico.
Hipertensin pulmonar.
Carcinoma de clulas alveolares.
Tromboembolismo pulmonar.

80. Uno de los siguientes tipos de anemia NO aparece en relacin


con la esclerosis sistmica progresiva:
1)
2)
3)
4)
5)

Anemia megaloblstica.
Anemia hemoltica microangioptica.
Anemia de trastornos crnicos.
Anemia refractaria con exceso de blastos.
Anemia ferropnica.

81. Mujer de 37 aos que consulta por poliartritis de pequeas


articulaciones, tumefaccin de manos, palidez ocasional de los
pulpejos de los dedos, mialgias y disfagia. En la analtica tiene
elevacin de CK y aldolasa. Cul es el diagnstico ms probable?:
1)
2)
3)
4)
5)

Enfermedad mixta del tejido conectivo.


Artritis reumatoide.
Esclerosis sistmica.
Sndrome mialgia-eosinofilia.
Lupus eritematoso sistmico.

82. Elija la respuesta FALSA en las artritis spticas:


1)
2)
3)
4)
5)

Las infecciones por BGN tienen un curso menos agudo que


las infecciones por CGP.
La infeccin sobre prtesis articular suele diagnosticarse en
la primera semana despus de la ciruga.
La infeccin por Pseudomonas afecta con frecuencia a la
articulacin esternoclavicular en los pacientes ADVP.
Los sntomas inflamatorios pueden enmascararse en los
pacientes tratados con corticoides.
El Haemophilus influenzae produce infecciones frecuentes en
los primeros aos de vida.

83. Seale la relacin teraputica INADECUADA para el tratamiento de una artritis sptica:

Pg. 8 RM

M exico A rgentina
C hile U ruguay

REUMATOLOGA

Preparacin Examen de Seleccin 05/06 1 Vuelta


1)
2)
3)
4)
5)

Haemophilus influenzae, doxiciclina.


Staphylococcus aureus, cloxacilina.
Neisseria gonorrhoeae, ceftriaxona.
Pseudomonas aeruginosa, ceftacidima.
Streptococcus pneumoniae, ceftriaxona.

84. Un paciente de 22 aos presenta de forma aguda una inflamacin poliarticular simtrica con preferencia en pequeas articulaciones, as como intensa astenia. Los sntomas desaparecen
a los 5 das, apareciendo ictericia conjuntival. La causa ms
probable de este cuadro es:
1)
2)
3)
4)
5)

Infeccin por el virus de Epstein-Barr.


Artritis reumatoide de inicio.
Poliartritis inespecfica e ictericia por AINEs.
Rubola.
Hepatitis B.

85. Las manifestaciones asociadas a la infeccin por el virus de la


inmunodeficiencia humana NO incluyen:
1)
2)
3)
4)
5)

Artralgias.
Artritis reumatoide.
Espondiloartropata indiferenciada.
Sndrome de Reiter.
Artritis psorisica.

86. En el curso clnico de la enfermedad de Lyme NO es caracterstico:


1)
2)
3)
4)
5)

Respuesta inicial al tratamiento con tetraciclinas.


Artritis perifrica de predominio en pequeas articulaciones.
Neuritis craneal.
Bloqueo auriculoventricular.
Meningitis linfocitaria.

87. La prueba ms especfica para el diagnstico de la xerostoma


en el sndrome de Sjgren es:
1)
2)
3)
4)
5)

Biopsia de glndula salival menor.


Sialometra.
Gammagrafa.
Sialografa.
Ecografa de partida.

88. Uno de los siguientes hallazgos NO es caracterstico del sndrome seco que aparece en la infeccin por el virus VIH:
1)
2)
3)
4)
5)

Infiltrado linfocitario en glndulas salivares por CD8.


Asociacin con DR5.
Presencia de ac anti-Ro.
Test de Schirmer positivo.
Ulceraciones puntiformes en crnea.

89. Una mujer de 65 aos presenta desde hace dos meses intensa
debilidad muscular que actualmente le impide levantarse de la
cama, con dolor muscular de predominio proximal. Ante esta
sintomatologa, que no ha mejorado con el tratamiento de
AINEs, se decide su derivacin al servicio de urgencias, donde
se objetiva la prdida de fuerza bilateral y simtrica de predominio proximal en extremidades superiores e inferiores, as
como elevacin de CK muy significativa. En el dorso de las

CTO Medicina C/ Nez de Balboa, 115 28006 MADRID (Espaa) Tfno.: (91) 782 43 32 / Fax: (91) 782 43 27
E-mail: secretaria@ctomedicina.com; iberocto@ctomedicina.com WEB: www.ctomedicina.com; www.iberocto.com

Preguntas TEST

Seguimiento a distancia

REUMATOLOGA

Preparacin Examen de Seleccin 05/06 1 Vuelta


manos presenta lesiones cutneas sobre las articulaciones.
Como estudios complementarios a realizar en esta paciente,
tendr MENOS utilidad:
1)
2)
3)
4)
5)

Radiografa de trax.
TC craneal.
Hemograma con frotis.
Ecografa abdominal.
Mamografa.

90. La enfermedad pulmonar intersticial en enfermos con polimiositis se asocia a la presencia de:
1)
2)
3)
4)
5)

Anticuerpos anti-ARNt sintetasa.


Presencia de cuerpos de inclusin en la biopsia.
Anticuerpos anti-Mi2.
Anticuerpos anti-SRP.
Anticuerpos anti-ADN.

91. Seale la respuesta correcta en la polimiositis por cuerpos de


inclusin:
1)
2)
3)
4)
5)

Afecta a personas ancianas, con predominio de varones.


El inicio de los sntomas es extremadamente rpido.
Se asocia habitualmente a cardiopata.
Presencia casi constante de autoanticuerpos.
La CK siempre est elevada.

92. Un joven de 14 aos, emigrante de origen turco, ha presentado


en los ltimos 15 meses episodios recurrentes de fiebre, dolor
abdominal, artralgias y dificultad respiratoria. Segn refiere su
familia, un hermano mayor presentaba episodios similares
antes de fallecer por insuficiencia renal terminal. Recomendara para su tratamiento:
1)
2)
3)
4)
5)

Preguntas TEST

3)
4)
5)

La disfuncin endocrina es poco frecuente.


Puede producir afectacin neurovegetativa con hipotensin
postural.
La artropata amiloide se ve con frecuencia en el mieloma
mltiple.
La amiloidosis heredofamiliar no afecta al sistema nervioso.
La afectacin cardaca cursa con insuficiencia cardaca
congestiva y cardiomegalia.

94. Un varn de 64 aos consulta en el servicio de urgencias por


inflamacin articular en rodillas y tobillos de carcter simtrico
y sensacin de quemazn en la superficie externa de las espinas
tibiales. Desde hace meses refiere deformidad de los dedos de
ambas manos en palillo de tambor con engrosamiento y reblandecimiento de las bases ungueales. En la radiografa articular no
se observan hallazgos llamativos y a nivel de la tibia calcificacin
peristica. Dentro de las patologas que hay que descartar en
este paciente NO se incluye:
1)

Tumor broncognico.
M exico A rgentina
C hile U ruguay

Sarcoidosis.
Endocarditis bacteriana.
Hemocromatosis.
Enfermedad inflamatoria intestinal.

95. Una mujer de 55 aos presenta desde hace aos dolor intermitente en la rodilla derecha que se acenta con la bipedestacin.
El estudio analtico es anodino sin elevacin de VSG. La paciente
consulta porque en las ltimas semanas el dolor se ha incrementado notando cierta inflamacin en la articulacin. En la exploracin fsica la rodilla est aumentada de tamao con rebote
positivo de la patela compatible con la presencia de derrame
articular. La radiologa demuestra una disminucin del espacio
articular con esclerosis marginal. Se realiza una artrocentesis,
obteniendo 35 ml de lquido sinovial de aspecto claro, con 630
clulas/mm3 con predominio de mononucleares y sin microcristales. Seale el tratamiento que NO utilizara:
1)
2)
3)
4)
5)

Reposo relativo de la articulacin.


Corticoides sistmicos.
AINEs sistmicos.
cido hialurnico intraarticular.
Condroitn sulfato oral.

96. NO es un hallazgo radiolgico propio de la artrosis:


1)
2)
3)
4)
5)

Esclerosis subcondral.
Osteofitos.
Disminucin del espacio articular.
Osteoporosis yuxtaarticular.
Geodas.

97. La localizacin actualmente ms frecuente de artropata neuroptica es:


1)
2)
3)
4)
5)

AINEs.
Ciclosporina.
Colchicina.
Corticoides orales.
Metronidazol.

93. Seale la respuesta FALSA en la amiloidosis:


1)
2)

2)
3)
4)
5)

Seguimiento a distancia

Articulacin glenohumeral.
Cadera.
Rodilla.
Sacroilacas.
Tarso y metatarsianas.

98. En la policondritis recidivante, aparece en casi todos los enfermos:


1)
2)
3)
4)
5)

Artritis.
Condritis auricular.
Glomerulonefritis.
Conjuntivitis.
Insuficiencia artica.

99. Mujer de 47 aos que refiere en los ltimos meses sensacin de


dolor muscular generalizado que afecta a las extremidades
superiores e inferiores y a la musculatura de la espalda. La
palpacin de la columna vertebral no resulta especialmente
dolorosa en ninguna localizacin, pero presenta dolor selectivo a nivel de la insercin de la musculatura suboccipital,
segmento central del trapecio, junto a ambos epicndilos, cara
interna de ambas rodillas y en la musculatura adyacente a
ambos trocnteres femorales. No refiere cuadro constitucional
salvo astenia intensa. Seale entre las siguientes caractersticas
la que NO espera encontrar en esta paciente:
1)

Alteracin del sueo.

CTO Medicina C/ Nez de Balboa, 115 28006 MADRID (Espaa) Tfno.: (91) 782 43 32 / Fax: (91) 782 43 27
E-mail: secretaria@ctomedicina.com; iberocto@ctomedicina.com WEB: www.ctomedicina.com; www.iberocto.com

RM Pg. 9

Seguimiento a distancia
2)
3)
4)
5)

REUMATOLOGA

Preparacin Examen de Seleccin 05/06 1 Vuelta

Colon irritable.
CPK elevada.
Aldolasa normal.
VSG normal.

100. Varn de 74 aos que desde hace dos meses presenta gran
impotencia funcional para elevar los brazos, manteniendo
movilidad pasiva normal. En la anamnesis no refiere alteraciones visuales, cefalea ni claudicacin mandibular. La VSG es de
115 mm/1 hora con enzimas musculares normales. La exploracin fsica muestra un pulso temporal bilateral normal, sin
dolor a la palpacin ni engrosamiento arterial. En el tratamiento
de este paciente, debemos elegir:
Prednisona 1 mg/kg/24 h.
Prednisona 10-15 mg/24 h.
Paracetamol.
Prednisona y azatioprina.
No precisa tratamiento, si no existe afectacin histolgica de
la arteria temporal.

Pg. 10 RM

M exico A rgentina
C hile U ruguay

CTO Medicina C/ Nez de Balboa, 115 28006 MADRID (Espaa) Tfno.: (91) 782 43 32 / Fax: (91) 782 43 27
E-mail: secretaria@ctomedicina.com; iberocto@ctomedicina.com WEB: www.ctomedicina.com; www.iberocto.com

Preguntas TEST

1)
2)
3)
4)
5)

REUMATOLOGA

Preguntas TEST

Preparacin Examen de Seleccin 05/06 1 Vuelta

M exico A rgentina
C hile U ruguay

Seguimiento a distancia

CTO Medicina C/ Nez de Balboa, 115 28006 MADRID (Espaa) Tfno.: (91) 782 43 32 / Fax: (91) 782 43 27
E-mail: secretaria@ctomedicina.com; iberocto@ctomedicina.com WEB: www.ctomedicina.com; www.iberocto.com

RM Pg. 11

REUMATOLOGA

Preparacin Examen de Seleccin 05/06 1 Vuelta

Comentarios TEST

Pregunta 1.- R: 5
Nos encontramos ante una paciente que presenta por un lado un
patrn inflamatorio poliarticular (afectacin de ms de 4 articulaciones) y con un estudio de lquido sinovial que muestra datos de lquido
inflamatorio: el nmero de clulas se encuentra entre 3.000 y 50.000
por mm3, la glucosa se encuentra discretamente descendida respecto
a la encontrada en plasma, y existe un aumento de protenas respecto
a la normalidad (recuerda que en el lquido sinovial las protenas son
parte de las encontradas en el plasma, por lo que se sitan en unos
2 gramos/dl, o lo que es lo mismo, 20 gramos/litro). Sin embargo, la
presencia de lquido inflamatorio es comn a cualquiera de las respuestas ofrecidas, por lo que la clave para resolver la pregunta se
encuentra en el patrn articular:
La artritis psorisica presenta con ms frecuencia un patrn oligoarticular (2-4 articulaciones) asimtrico con frecuente afectacin
de interfalngicas distales.
La artritis reactiva es igualmente otra espondiloartropata con afectacin oligoarticular asimtrica, de preferencia en extremidades
inferiores.
La artritis por cristales de cido rico es habitualmente monoarticular.
En el LES suele haber artralgias, con frecuencia migratorias.
La respuesta por lo tanto es la 5, ya que las localizaciones articulares incluidas son tpicas de la artritis reumatoide.
Pregunta 2.- R: 1
Lo primero que debes conocer es que la biopsia sinovial es un
procedimiento til para el diagnstico diferencial de enfermedades
articulares de etiologa no filiada por la valoracin clnica y otros
mtodos menos invasivos, como el estudio del lquido sinovial. Se
indica fundamentalmente en los cuadros de monoartritis crnica (superior por lo tanto a 6 semanas de evolucin), que no tiene hasta el
momento diagnstico preciso, especialmente si se sospecha la presencia de una infeccin. Podemos encontrar resultados especficos
en la biopsia de las siguientes enfermedades:
Infecciones: en el caso de las artritis tuberculosas, fngicas o gonoccica la sensibilidad del cultivo tisular de la membrana sinovial es
superior al 90%.
Artritis por microcristales: presencia de cristales de urato monosdico o de pirofosfato clcico en la membrana sinovial.
Neoplasias: en la reticulocitosis multicntrica, inflamacin sinovial
con clulas gigantes e histiocitos espumosos, con citoplasma eosinfilo y tincin PAS positiva; en la sinovitis villonodular pigmentada, proliferacin de la sinovial, clulas gigantes, histiocitos espumosos y depsitos de hemosiderina; en tumores primarios y metstasis infiltracin de clulas malignas.
Enfermedades por depsito como la amiloidosis con depsito de
material amiloide que se tie con rojo congo, ocronosis con depsito de pigmento ocrontico, hemocromatosis con depsito de
hemosiderina en la sinovial y en los macrfagos, sarcoidosis con
granulomas no caseificantes, artritis por cuerpo extrao con partculas ajenas a la sinovial como espinas vegetales o granulomas por
cuerpo extrao.
La respuesta correcta es la artritis reumatoide, ya que en esta enfermedad encontramos hiperplasia sinovial e hipervascularizacin, infiltrados de clulas inflamatorias crnicas y necrosis. Estos hallazgos son
comunes a otras colagenosis como el lupus eritematoso sistmico y
otras patologas inflamatorias como las espondiloartropatas. Los
ndulos reumatoides son raramente identificados en la sinovial.
Pregunta 3.- R: 5
En esta pregunta la clave para encontrar la respuesta correcta no
est en el patrn de afectacin articular, ya que la presencia de artritis
intermitente, aunque caracterstica del LES, puede aparecer en gran
parte de enfermedades inflamatorias articulares y colagenosis, sino en
la combinacin de anticuerpos que tiene la paciente.
La primera respuesta, la enfermedad mixta del tejido conectivo,
presenta de forma caracterstica ttulos elevados de anticuerpos
antinucleares que son anti-RNP a titulacin igual o superior a
1:1.600 (recuerda que la titulacin de anticuerpos es mayor cuanM exico A rgentina
C hile U ruguay

Seguimiento a distancia

to ms elevado sea el denominador, ya que la cifra expresada


indica la ltima dilucin a la que resultan positivos).
En el sndrome de Sjgren, los anticuerpos caractersticos son los
anti-Ro (o SS-A) y los anti-La (o SS-B).
En el lupus inducido por frmacos aparecen anticuerpos antinucleares y anti-histona, pero no aparecen los anticuerpos especficos del lupus eritematoso sistmico, como son los anti-DNAds ni
los anti-Sm. Pueden aparecer sin embargo anti-DNAss.
El sndrome antisintetasa es una patologa recientemente descrita
que identifica a los pacientes con polimiositis, anticuerpos
antisintetasa (anti-Jo1) y enfermedad pulmonar intersticial.
La respuesta correcta es, por lo tanto, el lupus eritematoso sistmico. Debemos recordar que en esta patologa dentro de sus criterios
diagnsticos se incluye la presencia de dos caractersticas que tiene
esta paciente: la presencia de anticuerpos antinucleares y los antiDNAds. Ten en cuenta que en el LES tambin puede haber, como
en el lupus inducido por frmacos, anti-histona.

Pregunta 4.- R: 1
El sndrome antifosfolpido se define por la presencia de criterios
clnicos y analticos:
Clnicos:
Morbilidad en el embarazo:
- 3 o ms abortos (< 10 semanas de gestacin) espontneos, consecutivos y no justificados por otros mecanismos y excluidas
causas cromosmicas maternas o paternas, u hormonales o anatmicas maternas.
- Una o ms muertes fetales (>10 semanas de gestacin) inexplicadas con fetos normales.
- Parto prematuro en la semana 34 del embarazo o anteriores por
pre-eclampsia o insuficiencia placentaria.
Laboratorio:
Anticuerpos anticardiolipina- IgG (ttulos positivos moderados o
altos).
Anticuerpos anticardiolipina- IgM (ttulos positivos moderados o
altos).
Anticoagulante lpico positivo. Se realizan a partir de plasma pobre en plaquetas con pruebas coagulomtricas (tiempo de tromboplastina parcial activado, tiempo de tromboplastina tisular, tiempo de veneno de vbora de Rusell). En caso de alargamiento de
alguna de ellas, se intenta su correccin con plasma normal o con
la adicin de fosfolpidos.
Para la clasificacin definitiva se requiere la presencia de un criterio
clnico y uno de laboratorio. Se recomienda adems que los anticuerpos sean positivos en dos ocasiones separadas por un periodo
mnimo de 6 semanas.
La presencia de VDRL falsamente positivo es habitual en estos pacientes, ya que el sustrato para la prueba tiene abundantes fosfolpidos,
por lo que se da una reaccin cruzada positiva. De esta forma nos
queda como nica respuesta posible el factor reumatoide. En los
ltimos aos se ha descrito un cofactor que incrementa el riesgo de
presentar trombosis en los pacientes con sndrome antifosfolpido,
que son los anticuerpos anti-beta-2-glicoproteina.
Pregunta 5.- R: 2
La panarteritis nodosa es una vasculitis necrotizante con afectacin
de arterias musculares pequeas y medianas que se caracteriza por
afectar a las arterias renales y viscerales. La lesin predominante a
nivel renal es una arteritis sin glomerulonefritis, por lo que parece
claro que la respuesta tiene que ser la segunda. Desde el punto de vista
clnico es caracterstico el desarrollo progresivo de insuficiencia renal
e hipertensin arterial (respuestas 3 y 5).
A diferencia de ella, la poliarteritis microscpica es una vasculitis
necrotizante con afectacin de pequeos vasos (capilares, vnulas o
arteriolas) que cursa con escasos o nulos depsitos de complejos
inmunitarios (pauciinmune). Como tambin puede aparecer arteritis
necrosante de arterias de pequeo y mediano tamao, este proceso
muestra manifestaciones comunes con la PAN clsica, salvo que la

CTO Medicina C/ Nez de Balboa, 115 28006 MADRID (Espaa) Tfno.: (91) 782 43 32 / Fax: (91) 782 43 27
E-mail: secretaria@ctomedicina.com; iberocto@ctomedicina.com WEB: www.ctomedicina.com; www.iberocto.com

RM Pg. 1

glomerulonefritis es muy frecuente en la poliarteritis microscpica, y a


menudo aparece capilaritis pulmonar.
El diagnstico de la PAN clsica se basa en la presencia de los
hallazgos caractersticos en la biopsia de los tejidos afectados. Si no
existe un tejido fcilmente accesible a la biopsia, es suficiente con
demostrar por angiografa la afectacin vascular, especialmente en
forma de aneurismas de arterias de pequeo y mediano calibre en la
circulacin renal, heptica y visceral (respuesta 4). Sin embargo los
aneurismas no son patognomnicos de la PAN clsica; adems no
siempre tienen que aparecer, quedando reducidos los signos angiogrficos a estenosis y obliteracin de los vasos.
Pregunta 6.- R: 3
La respuesta 1 se refiere a una caracterstica angiogrfica caracterstica de la PAN nodosa clsica que no solemos observar en la poliangetis microscpica, ya que en esta los vasos afectados suelen ser de
menor dimetro. La presencia de microaneurismas no es patognomnica de la PAN clsica, ya que se han encontrado en otras patologas
como la granulomatosis de Wegener, mixoma auricular, endocarditis,
lupus eritematoso sistmico o prpura trombtica trombocitopnica.
La respuesta 2 es una caracterstica de la poliangetis microscpica,
ya que esta vasculitis afecta a vasos de pequeo calibre.
La respuesta 4, la afectacin pulmonar, es de nuevo tpica de la
panarteritis microscpica en forma de capilaritis pulmonar. Las arterias pulmonares no estn afectadas en la PAN clsica, y la afectacin
de las arterias bronquiales es poco habitual.
La presencia de enfermedad renal es caracterstica de ambas enfermedades, pero en la PAN clsica se ven afectadas las arterias de mediano calibre y en la microscpica los capilares, produciendo
glomerulonefritis.
La nica respuesta posible, por lo tanto, es la tercera, ya que ambas
enfermedades sin tratamiento tienen un pronstico muy desfavorable. El esquema teraputico incluye la combinacin de prednisona 1
mg/kg/da con ciclofosfamida 2 mg/kg/da. Con este tratamiento se
consiguen remisiones en el 90% de los casos.
Pregunta 7.- R: 3
El planteamiento a la hora de establecer el diagnstico en esta
paciente se orienta por los hallazgos clnicos y analticos, donde destaca la presencia de eosinofilia perifrica. La respuesta 1 la podemos
descartar rpidamente, ya que en la PAN en su forma clsica no hay
afectacin pulmonar y en la microscpica no existe eosinofilia.
La granulomatosis de Wegener (respuesta 2) puede producir infiltrados pulmonares bilaterales, pero de nuevo en esta enfermedad no
es caracterstica la presencia de eosinofilia.
La respuesta nmero 4 habla de un cuadro, la poliangetis de superposicin, caracterizada por la presencia en un mismo paciente de
hallazgos clnicos y/o histolgicos propios de ms de una vasculitis
incluyendo PAN clsica, granulomatosis alrgica, Wegener, arteritis de
Takayasu o vasculitis por hipersensibilidad. Sin embargo, todos los
hallazgos encontrados en esta paciente, incluyendo la afectacin cutnea en forma de prpura palpable, se describen en el contexto de la
enfermedad de Churg-Strauss (respuesta correcta 3). Esta enfermedad
incluye para su diagnstico la presencia de una serie de criterios:
Asma.
Eosinofilia perifrica superior al 10% o mayor de 1.500/mm3.
Historia previa de alergia (excepto medicamentosa).
Afectacin pulmonar (infiltrados migratorios o transitorios) atribuibles a vasculitis sistmica.
Mono o polineuropata.
Afectacin de senos paranasales. Historia de dolor agudo o crnico paranasal o velamiento radiolgico de senos paranasales.
Eosinfilos extravasculares. Biopsia de arteria, arteriola o vnula
que muestra acumulacin de eosinfilos extravasculares.
La presencia de 4 criterios confiere una elevada sensibilidad y
especificidad para el diagnstico de la enfermedad.
Pregunta 8.- R: 3
Esta paciente presenta las caractersticas tpicas de la granulomatosis
de Wegener para cuyo diagnstico utilizamos los siguientes criterios:
Pg. 2 RM

M exico A rgentina
C hile U ruguay

REUMATOLOGA

Preparacin Examen de Seleccin 05/06 1 Vuelta


Inflamacin nasal u oral: desarrollo de lceras dolorosas o indoloras en la mucosa oral o secrecin nasal purulenta o hemtica.
Alteraciones en la radiologa de trax: ndulos, infiltrados no migratorios o cavitacin.
Alteraciones del sedimento urinario: microhematuria o presencia
de cilindros hemticos.
Inflamacin granulomatosa en la biopsia: inflamacin granulomatosa dentro de la pared vascular o en regin perivascular.
La presencia de al menos dos de estos criterios supone una especificidad/sensibilidad para el diagnstico de la granulomatosis de Wegener.
En este paciente, la determinacin de factor reumatoide positivo
no debe hacerte dudar sobre el diagnstico, ya que aunque es caracterstica su determinacin en la artritis reumatoide, puede aparecer en
otras enfermedades inflamatorias del tejido conectivo como el sndrome de Sjgren o vasculitis, infecciones y neoplasias.
Respecto a otras manifestaciones clnicas, puede aparecer proptosis por el desarrollo de granulomas retrooculares, incluyndose como
otras manifestaciones oculares la conjuntivitis, dacriocistitis y escleritis. La respuesta que nos queda es la nmero 3, destruccin sea
orbitaria, ya que la granulomatosis de Wegener afecta a tejidos blandos, pero no suele producir afectacin sea. Cuando ocurre sta,
debemos pensar en otros procesos destructivos, como el granuloma
de la lnea media o tumores.
Pregunta 9.- R: 4
La granulomatosis de Wegener se considera un sndrome
renopulmonar por la frecuente afectacin que ocurre a estos dos
niveles (junto con la afectacin nasal u oral supone la trada clnica
tpica de la enfermedad). Otras patologas con frecuente afectacin a
estos dos niveles se incluyen en su diagnostico diferencial:
La granulomatosis linfomatoide afecta al pulmn, la piel, sistema
nervioso central y el rin, rganos donde se encuentran las clulas plasmocitoides y linfocitoides atpicas que infiltran los tejidos
con carcter angioinvasor. En la mitad de los casos se desarrolla un
autntico linfoma maligno.
El sndrome de Goodpasture se caracteriza por la presencia de
anticuerpos antimembrana basal que producen hemorragias pulmonares y glomerulonefritis necrosante con depsito lineal de
inmunocomplejos y complemento.
La granulomatosis alrgica es una vasculitis necrotizante caracterizada por la presencia de asma, alergia y eosinofilia perifrica en la
mayora de los pacientes. Recuerda que en esta enfermedad los
infiltrados pulmonares son migratorios o transitorios.
En algunas neumonas como las producidas por neumococo,
Legionella o Mycoplasma se produce glomerulonefritis proliferativa
con depsito granular de inmunocomplejos y complemento.
Otros sndromes renopulmonares, que no aparecen dentro de las
respuestas de esta pregunta, son la poliarteritis microscpica, el LES
o la prpura trombtica trombocitopnica.
Como ves, excepto la respuesta nmero 4 (enfermedad de Buerger),
el resto de las entidades se incluyen en el diagnstico diferencial con
la granulomatosis de Wegener. La enfermedad de Buerger o tromboangetis obliterante es un proceso caracterizado por la isquemia de las
extremidades, en la que la formacin de trombos arteriales y venosos
es el mecanismo fundamental. Afecta sobre todo a pacientes jvenes,
varones y fumadores.
Pregunta 10.- R: 2
La paciente de este caso clnico presenta algunas de las caractersticas tpicas de la arteritis de Takayasu que se recogen en sus criterios
diagnsticos:
Edad inferior a los 40 aos.
Claudicacin de las extremidades (desarrollo o empeoramiento
de cansancio en los msculos de una o ms extremidades durante
su uso, especialmente de extremidades superiores).
Diferencia de tensin arterial entre ambos brazos > 10 mmHg (en
la tensin sistlica).
Soplos en arterias subclavias y aorta: soplos audibles a la auscultacin sobre una o ambas arterias subclavias o en la aorta abdominal.

CTO Medicina C/ Nez de Balboa, 115 28006 MADRID (Espaa) Tfno.: (91) 782 43 32 / Fax: (91) 782 43 27
E-mail: secretaria@ctomedicina.com; iberocto@ctomedicina.com WEB: www.ctomedicina.com; www.iberocto.com

Comentarios TEST

Seguimiento a distancia

REUMATOLOGA

Preparacin Examen de Seleccin 05/06 1 Vuelta


Anomalas en la angiografa: estrechamiento u oclusin de la aorta
entera, sus ramas proximales o grandes arterias, en la zona proximal
de las extremidades superiores o inferiores, no debida a
arteriosclerosis, displasia muscular, o causas similares. Los cambios
habitualmente son focales o segmentarios.
La primera respuesta, panarteritis nodosa, la podemos descartar por
varios motivos: la edad de presentacin es demasiado precoz para esta
enfermedad y produce con ms frecuencia otra sintomatologa, como
afectacin del sistema nervioso perifrico, gastrointestinal y renal.
La respuesta 3, el LES con anticoagulante lpico, puede producir
ictus cerebral, pero no es tpica la alteracin del pulso en extremidades superiores. Lo mismo podemos decir de la respuesta 4, el sndrome antifosfolpido catastrfico.
En la vasculitis del sistema nervioso central, respuesta 5, la sintomatologa aparece exclusivamente a este nivel, estando ausente la alteracin de pulsos en extremidades superiores y la insuficiencia artica.

Comentarios TEST

Pregunta 11.- R: 1
Para el diagnstico de la arteritis de la temporal, igual que como
hemos visto en otras vasculitis, nos basamos en la presencia de una
serie de manifestaciones clnicas tpicas. En este caso son:
Edad igual o superior a los 50 aos (respuesta nmero 3).
Cefalea de reciente comienzo.
Alteraciones de la arteria temporal (dolor a la palpacin, descenso
del pulso en las arterias temporales no relacionado con arteriosclerosis de las arterias carotdeas).
VSG igual o superior a 50 mm/1 hora (respuesta nmero 2).
Biopsia de la arteria temporal anormal (mostrando vasculitis caracterizada por un predominio de clulas mononucleares o inflamacin granulomatosa, generalmente con clulas gigantes
multinucleadas.
Desde el punto de vista clnico, en la mitad de los pacientes aparecen sntomas de polimialgia reumtica, caracterizada por la presencia
de dolor moderado o intenso y rigidez matinal superior a los 30 minutos y de ms de un mes de evolucin al menos en dos de las siguientes
tres reas: cuello, cintura escapular y /o cintura pelviana (respuesta
nmero 4).
El tratamiento de la enfermedad se basa en la administracin de
corticoides en dosis altas (1 mg/kg/da de prednisona) (respuesta 5).
La nica opcin posible despus de lo que hemos visto es la primera respuesta. Recuerda que la afectacin vascultica suele ser
segmentaria, por lo que la presencia de un resultado normal en la
biopsia no descarta por completo el diagnstico sospechado. Para
intentar que el resultado no sea negativo se debe hacer una biopsia
amplia de la arteria temporal (unos 2 centmetros), estudiando el
patlogo mltiples cortes de la muestra.
Pregunta 12.- R: 5
La enfermedad de Schnlein-Henoch es una vasculitis que suele
aparecer en la infancia, entre los 4 y 7 aos, caracterizada por la
presencia de prpura palpable (distribuida principalmente en nalgas
y en miembros inferiores), artralgias, signos y sntomas gastrointestinales y glomerulonefritis. La presencia de estos ltimos sntomas de afectacin visceral no es caracterstica de las vasculitis predominantemente cutneas, por lo que debemos elegir la respuesta nmero 5. Sin
embargo, el resto de caractersticas que aparecen en las respuestas son
comunes a ambas enfermedades. Los vasos predominantemente afectados son los de menor calibre (capilares y vnulas; respuesta 1). El
fenmeno de leucocitoclasia (respuesta 2) supone la presencia de
residuos nucleares que quedan de los neutrfilos que han infiltrado
los vasos y sus alrededores durante la fase aguda. La prpura palpable
es la norma en ambas enfermedades y se debe a la extravasacin de
hemates a nivel cutneo por la rotura de pequeos vasos.
En el caso de la vasculitis de Schnlein-Henoch se sospecha la
presencia de mltiples antgenos desencadenantes, como infecciones
de vas areas superiores, diversos frmacos, alimentos, picaduras de
insectos o inmunizaciones. Los anticuerpos formados suelen ser IgA,
cuya presencia se ha demostrado tambin en las biopsias renales de
estos pacientes. La mortalidad de esta vasculitis es escasa, evolucioM exico A rgentina
C hile U ruguay

Seguimiento a distancia

nando la mayora de los pacientes de forma favorable igual que en las


vasculitis cutneas (respuesta 4). La escasa mortalidad suele estar relacionada con la aparicin de insuficiencia renal.
Pregunta 13.- R: 1
El trmino de vasculitis predominantemente cutneas se refiere a un
cuadro clnico que suele estar dominado por las manifestaciones cutneas, aunque no de forma exclusiva. De hecho, en este tipo de vasculitis
cualquier rgano puede estar afectado, aunque la clnica extracutnea
suele ser mucho menos intensa que la provocada por las vasculitis
necrosantes generalizadas. Otros trminos sinnimos son el de vasculitis
por hipersensibilidad o el de vasculitis leucocitoclstica.
Las vasculitis predominantemente cutneas pueden dividirse en dos
grupos, segn el antgeno responsable de la hipersensibilidad. Dentro
de los antgenos de origen exgeno se incluyen frmacos, infecciones
(respuesta 1) o una protena extraa. La crioglobulinemia mixta esencial produce cuadros de vasculitis cutnea y se ha asociado al virus de
la hepatitis C (por lo tanto se confirma que la respuesta correcta es la 1).
Dentro de los antgenos endgenos incluimos las situaciones relacionadas con enfermedades autoinmunes como el lupus eritematoso
sistmico, la artritis reumatoide o el sndrome de Sjgren (respuesta 5), o
algunos tumores como los linfomas (respuesta 2).
La respuesta 3 debemos descartarla porque la panarteritis microscpica se incluye, al igual que la panarteritis nodosa clsica y la enfermedad de Churg-Strauss, dentro de las vasculitis necrotizantes
sistmicas.
La respuesta 4 es incorrecta, ya que es caracterstico que la prpura de las vasculitis cutneas no desaparezca con la vitropresin.
Pregunta 14.- R: 5
El cuadro descrito en este caso clnico es una vasculitis aislada del
sistema nervioso central caracterizada por la afectacin inflamatoria
vascular exclusivamente a este nivel, sin que aparezcan signos de
vasculitis en otras localizaciones. Afecta sobre todo a arteriolas, aunque se pueden ver afectados vasos de cualquier tamao. El proceso
inflamatorio suele estar integrado por infiltrados de clulas mononucleares, acompaados o no de la formacin de granulomas. En la
mayora de los casos no se encuentra ningn factor patognico
desencadenante, aunque se han visto casos relacionados con infecciones virales y bacterianas, enfermedad de Hodgkin o consumo de
anfetaminas.
El motivo de consulta habitual, como aparece descrito en el caso
clnico, es la presencia de cefalea, trastornos de las funciones superiores
y defectos neurolgicos focales, sin sntomas generales. El diagnstico
generalmente se establece demostrando en la arteriografa las alteraciones vasculares caractersticas, y se confirma por biopsia del parnquima
cerebral y las leptomeninges (respuesta correcta nmero 5).
El pronstico de esta enfermedad es muy desfavorable, aunque
algunos pacientes responden a corticoides solos o en combinacin a
ciclofosfamida.
Debes tener en cuenta que la duda que en ocasiones se tiene al
leer el caso clnico con el diagnstico de la arteritis de la temporal,
queda despejada por la edad de la paciente, ya que esta patologa
aparece por encima de los 50 aos (respuesta 4).
Pregunta 15.- R: 3
El diagnstico de este paciente se corresponde con un sndrome
de Behet con datos suficientes como para confirmarlo segn los
criterios diagnsticos que se utilizan:
lceras recidivantes en boca, ms dos de las siguientes manifestaciones:
lceras genitales recidivantes.
Lesiones oculares (uvetis anterior o posterior o vasculitis retiniana).
Lesiones cutneas (eritema nodoso, pseudofoliculitis, o lesiones
papulo-pstulas, o ndulos acneiformes).
Prueba de patergia positiva (observada por un mdico a las 48
horas).
En el caso de esta pregunta, el paciente tiene lceras orales, genitales y eritema nodoso, por lo que cumple el criterio mayor y dos
menores.

CTO Medicina C/ Nez de Balboa, 115 28006 MADRID (Espaa) Tfno.: (91) 782 43 32 / Fax: (91) 782 43 27
E-mail: secretaria@ctomedicina.com; iberocto@ctomedicina.com WEB: www.ctomedicina.com; www.iberocto.com

RM Pg. 3

El tratamiento del sndrome de Behet es sintomtico y emprico y


debe estar relacionado con los sntomas que presenta el paciente. Recuerda que en este caso hay sntomas a nivel cutneo, de mucosas y
articular. La afectacin de mucosas puede mejorar con la aplicacin
local de corticoides o de forma sistmica en dosis bajas (respuesta 4 y 5).
La colchicina (respuesta 1), los AINEs (respuesta 2) o el interfern alfa
pueden ser beneficiosos para el tratamiento de la artritis.
En el caso de que el paciente presente uvetis o afectacin del
sistema nervioso central, es necesaria la administracin de corticoides
en dosis altas (1 mg/kg/da) y azatioprina (2-3 mg/kg/da) o ciclosporina
(5-10 mg/kg/da). La ausencia de sintomatologa del paciente a este
nivel hace innecesaria la administracin de ciclosporina, por lo que
la respuesta es la 3.
Pregunta 16.- R: 3
La presentacin de este caso clnico es caracterstica de la enfermedad de Wegener. En esta vasculitis la afectacin clnica ocurre a tres
niveles:
Inflamacin nasal u oral: desarrollo de lceras dolorosas o indoloras en la mucosa oral o secrecin nasal purulenta o hemtica.
Alteracin en la radiologa de trax: ndulos, cavitacin o infiltrados no migratorios ni fugaces en el parnquima pulmonar.
Alteraciones en el sedimento urinario: microhematuria (> de 5
hemates por campo) o presencia de cilindros hemticos.
Adems se manifiesta la presencia de anticuerpos c-ANCA positivos, que son muy sensibles y especficos para el diagnstico de la
granulomatosis de Wegener, aunque no patognomnicos, ya que tambin se observan en otras vasculitis necrotizantes como la PAN clsica,
la P.A. microscpica, enfermedad de Churg-Strauss y algunas vasculitis
por frmacos.
Desde el punto de vista del pronstico, este es fatal sin tratamiento.
Los glucocorticoides solos producan alguna mejora sobre la enfermedad, pero no modificaban de forma sustancial el pronstico. Se ha
demostrado claramente que el tratamiento de eleccin es la ciclofosfamida en dosis de 2 mg/kg/da, por va oral, junto con glucocorticoides. Durante el tratamiento, hay que vigilar de cerca el contaje de
leucocitos y ajustar la dosis de ciclofosfamida para mantenerlos por
encima de 3000/ mm3. Al principio del tratamiento hay que incorporar prednisona en dosis de 1 mg/kg/da, con toma diaria durante el
primer mes, pasando gradualmente a una pauta de corticoides a das
alternos; por ltimo se disminuyen progresivamente hasta interrumpirlos aproximadamente a los 6 meses.
Pregunta 17.- R: 3
En este caso clnico debes fijarte en las manifestaciones clnicas y
analticas que de forma progresiva te llevan a la respuesta correcta,
que es la panarteritis nodosa clsica. Se trata de una mujer de mediana edad donde destaca la presencia de afectacin del sistema nervioso perifrico en forma de mononeuritis mltiple confirmada en la
exploracin fsica, dolor abdominal con signos de irritacin peritoneal y rectorragia ocasional e insuficiencia renal con elevacin de
productos nitrogenados en la sangre.
Desde el punto de vista analtico presenta adems leucocitosis sin
eosinofilia, por lo que la posibilidad de que se trate de una enfermedad de Churg-Strauss es pequea, mxime si recordamos que no
tiene clnica pulmonar ni antecedente de asma o alergia.
Otras respuestas ofrecidas debemos descartarlas por los siguientes
motivos:
La enfermedad de Goodpasture se caracteriza por la presencia de
anticuerpos anti-membrana basal, apareciendo las manifestaciones
clnicas a nivel pulmonar (hemorragia alveolar) y renal (glomerulonefritis), sin presentar afectacin en el sistema nervioso perifrico.
El sndrome de Schnlein-Henoch es una vasculitis que suele afectar a nios entre 4 y 7 aos, aunque en ocasiones aparece tambin
en adultos. Aunque puede producir dolor abdominal y
glomerulonefritis, el cuadro viene dominado por la clnica cutnea (prpura palpable) y la artritis.
En la granulomatosis de Wegener, la afectacin preferente ocurre
en las vas areas superiores, pulmn y rin, estando ausente en
esta paciente las dos primeras localizaciones.
Pg. 4 RM

M exico A rgentina
C hile U ruguay

REUMATOLOGA

Preparacin Examen de Seleccin 05/06 1 Vuelta


El sndrome poliangetico de superposicin se describe cuando en
un paciente aparecen datos clnicos y/o anatomopatolgicos compatibles con ms de una vasculitis. En este caso, los sntomas son
todos tpicos de una sola vasculitis, la panarteritis nodosa clsica.
Pregunta 18.- R: 5
Las causas de hiperuricemia es una de las cuestiones ms preguntadas sobre este tema junto con su tratamiento. Dentro de la etiologa
de la hiperuricemia solemos distinguir las siguientes posibilidades:
Hiperuricemia por sntesis de urato aumentada:
- Por aumento del catabolismo de las purinas: dentro de este grupo
se encuentran las enfermedades mielo y linfoproliferativas, mieloma y otros tumores, especialmente si son tratados con quimioterapia, por la gran destruccin celular que se deriva. Tambin se
puede encontrar en la hemlisis, psoriasis, policitemia vera y enfermedad de Paget. La dieta, aunque es una fuente de purinas
que se pueden metabolizar a cido rico, tiene mucha menor
importancia. De hecho, la restriccin estricta de purinas reduce
la concentracin plasmtica de cido rico en solo 1 mg/dl.
- Defectos hereditarios del metabolismo: aumento de la actividad de la PRPP sintetasa y el dficit de HPRT, que cuando es
parcial, produce el sndrome de Kelley-Seegmiller y cuando es
completo, el sndrome de Lesch-Nyhan. Esta ltima enfermedad (respuesta 5) se caracteriza por la presencia de hiperuricemia, hiperaciduria, clculos de cido rico y gota por sobreproduccin de uratos. Asocia tambin automutilacin, coreoatetosis y otros trastornos neurolgicos.
Excrecin renal disminuida: este mecanismo supone el 90% de las
hiperuricemias. Est relacionada con mltiples situaciones, entre
las que podemos encontrar la toma de frmacos: diurticos, salicilatos en dosis bajas, ciclosporina, tuberculostticos y algunos
antirretrovirales; insuficiencia renal y situaciones que producen
acidosis como la cetoacidosis diabtica y la acidosis lctica.
Como puedes comprobar, la respuesta correcta es la enfermedad
de Lesch-Nyhan, ya que aunque se trata de una patologa que produce hiperuricemia por aumento de sntesis de uratos igual que las dems respuestas, no est relacionada como el resto con el aumento del
catabolismo de las purinas.
Pregunta 19.- R: 2
Esta pregunta permite que recuerdes cules son las manifestaciones clnicas y la actitud teraputica que debes adoptar en un paciente
con hiperuricemia.
Hiperuricemia asintomtica: en aquellos pacientes que presentan
hiperuricemia sin clnica no est indicado tratarla. La excepcin la
suponen los pacientes con neoplasias, especialmente los que vayan a recibir quimioterapia, que deben ser adecuadamente hidratados, alcalinizando la orina para facilitar la excrecin renal del
cido rico y administrando alopurinol para disminuir su sntesis.
Artritis aguda por cido rico: debemos tratar al paciente con AINEs
y/o colchicina mientras presente los sntomas. Si el paciente no
reciba previamente tratamiento para la hiperuricemia, no debe
incorporarse hasta que el episodio agudo ceda.
Gota intercrtica: supone la situacin que padece el paciente de
este caso, es decir, episodios recidivantes de artritis aguda pero sin
clnica entre ellos. En este caso debemos tratar la hiperuricemia y la
eleccin entre alopurinol o uricosricos depender del resultado
del balance de cido rico. Si la excrecin renal de cido rico en
24 horas es superior a 700 mg, administraremos alopurinol, y si es
menor, uricosricos (benzobromarona, sulfinpirazona o
probenecid). Sin embargo hay que tener en cuenta que los
uricosricos estn contraindicados si el paciente tiene antecedentes de nefrolitiasis (como en este paciente) o padece insuficiencia
renal. En estos casos, la alternativa teraputica es la utilizacin de
alopurinol.
En la gota tofcea crnica, cuando los tofos son de gran tamao se
utiliza como frmaco de eleccin el alopurinol, pero si son pequeos tambin podemos utilizar los uricosricos, como veamos en el
apartado anterior.

CTO Medicina C/ Nez de Balboa, 115 28006 MADRID (Espaa) Tfno.: (91) 782 43 32 / Fax: (91) 782 43 27
E-mail: secretaria@ctomedicina.com; iberocto@ctomedicina.com WEB: www.ctomedicina.com; www.iberocto.com

Comentarios TEST

Seguimiento a distancia

REUMATOLOGA

Preparacin Examen de Seleccin 05/06 1 Vuelta


Siempre que utilizamos alopurinol o uricosricos debemos aadir
colchicina durante unos meses para prevenir la aparicin de nuevos
episodios de artritis aguda al descender los niveles de cido rico.

Pregunta 19. Artritis gotosa.

Comentarios TEST

Pregunta 20.- R: 5
El depsito de cristales de pirofosfato ocurre en la mayora de los
casos en pacientes mayores de 70 aos y sin ninguna enfermedad
metablica que induzca su depsito. Sin embargo, cuando se demuestra la presencia de una artropata por estos cristales en una
persona por debajo de los 55 aos, es necesario descartar la presencia de una serie de patologas que favorecen su depsito como son:
Hiperparatiroidismo (respuesta nmero 1, PTH)
Hipomagnesemia (respuesta nmero 2, magnesio).
Hemocromatosis (respuesta 3, ferritina).
Hipofosfatasia (respuesta 4, fosfatasa alcalina).
Como ves, la respuesta correcta es la 5, ya que no hay relacin
con patologa asociada a la glndulas suprarrenales. La asociacin
con hipotiroidismo, aunque probable, no est demostrada.
Existen tambin formas familiares. Suelen iniciarse entre la 3 y 5
dcada de la vida, con afectacin poliarticular severa. El mecanismo
hereditario parece ser autosmico dominante, observndose en algunas familias defectos genticos de los cromosomas 5p y 8q.
Pregunta 21.- R: 3
La artrosis es la patologa articular ms frecuente. En la artrosis primaria o idioptica no existen factores predisponentes. En la artrosis secundaria el proceso es anatomopatolgicamente indistinguible del primario, pero existe algn factor subyacente que favorece el desarrollo del
proceso. En la pregunta que nos realizan existen una serie de localizaciones que son caractersticas de la artrosis primaria, como son:
Manos: nodular (ndulos de Heberden y ndulos de Bouchard),
artrosis interfalngica erosiva (sin ndulos), de la primera articulacin carpometacarpiana.
Pies: hallux valgus, hallux rigidus, dedos contrados (en martillo),
articulacin astragalonavicular.
Rodilla: compartimento interno, compartimento externo, compartimento femororrotuliano.
Cadera: excntrica (superior), concntrica (axial, interna), difusa.
Columna vertebral: articulaciones apofisarias, articulaciones intervertebrales (discos), espondilosis (osteofitos), ligamentosa (hiperostosis vertebral anquilosante).
Otras localizaciones nicas: glenohumeral, acromioclavicular,
tibioastragalina, sacroiliaca, temporomandibular.
M exico A rgentina
C hile U ruguay

Seguimiento a distancia

Como puedes comprobar, dentro de las localizaciones habituales


de la artrosis primaria se encuentran todas las respuestas, excepto la
articulacin del codo. En caso de aparecer en esta articulacin artrosis deberemos pensar en una causa secundaria, como son las enfermedades microcristalinas por depsito de cristales de calcio (pirofosfato clcico o hidroxiapatita). Otras posibilidades son los traumatismos, alteraciones congnitas, metablicas (ocronosis, hemocromatosis, enfermedad de Wilson), endocrinas (acromegalia, hiperparatiroidismo, hipotiroidismo), enfermedades seas y articulares o artropata
neuroptica.
Pregunta 22.- R: 4
En este caso clnico debemos en primer lugar establecer el diagnstico. Se trata de una paciente mayor que presenta un cuadro de monoartritis aguda. Como antecedente relevante nos indican que ya ha
presentado episodios similares en otras articulaciones, siendo el estudio radiolgico caracterstico de condrocalcinosis. Este hallazgo no
asegura el tipo de cristal que tiene la paciente, ya que cualquier
microcristal que tenga calcio (pirofosfato clcico, oxalato clcico, hidroxiapatita clcica) puede dar una imagen de condrocalcinosis. Para
establecer el diagnstico de forma definitiva siempre debemos hacer
un estudio de microcristales en el lquido sinovial. En este caso nos
indican que presenta cristales de forma rectangular que son caractersticos de la artropata por pirofosfato clcico. Adems nos descartan,
con el estudio microbiolgico, la presencia de infeccin articular.
Una vez establecido de forma definitiva el diagnstico debemos elegir la actitud teraputica y sealar la alternativa de tratamiento que no se
utiliza.
La respuesta 1 (AINEs) y 5 (colchicina) son los frmacos que habitualmente utilizamos en la artritis aguda por microcristales. La respuesta 2
(reposo de la articulacin) es una medida correcta en las fases agudas de
la inflamacin articular, independientemente de la causa que la produce. Sin embargo, hay que intentar comenzar la movilizacin articular
en cuanto el dolor lo permita.
En algunos pacientes puede ser una buena medida la administracin intraarticular de corticoides, pero es obligatorio, como se ha realizado en este caso, confirmar el diagnstico antes de realizar la infiltracin. Sin embargo, la administracin sistmica de corticoides no es una
medida teraputica adecuada, por lo que la respuesta es la 4.
Pregunta 23.- R: 5
La hidroxiapatita clcica es el mineral principal del hueso y de los
dientes. La mayora de las calcificaciones de partes blandas del organismo son producidas por hidroxiapatita. En la mayora de los casos,
como ocurre con los cristales de pirofosfato, el depsito es idioptico,
pero existen una serie de enfermedades asociadas que debemos recordar, entre las que incluimos:
Enfermedades del tejido conectivo: LES, dermatomiositis o esclerodermia.
Enfermedades metablicas: hiperparatiroidismo, hiperfosfatemia,
intoxicacin por vitamina D, insuficiencia renal crnica, diabetes
mellitus.
Trastornos neurolgicos: ACV y traumas medulares.
Desde el punto de vista clnico los depsitos suelen ser articulares o
periarticulares y frecuentemente asintomticos, pero tambin se describen los siguientes cuadros sintomticos relacionados:
Periartritis calcificante, siendo el hombro la localizacin ms habitual (respuesta 2).
Artrosis: se identifican cristales en el 50% de los cuadros de artrosis
(respuesta 3).
Artritis: inflamacin aguda monoarticular (respuesta 4).
Artropata destructiva: afecta preferentemente a la rodilla y al hombro (hombro de Milwaukee, respuesta 1).
La respuesta que nos queda como correcta es la poliartritis simtrica (respuesta 5), que es caracterstica de la artritis reumatoide.
Pregunta 24.- R: 2
El oxalato es el producto final del metabolismo del cido ascrbico
y de algunos aminocidos. Se alcanzan concentraciones sricas de

CTO Medicina C/ Nez de Balboa, 115 28006 MADRID (Espaa) Tfno.: (91) 782 43 32 / Fax: (91) 782 43 27
E-mail: secretaria@ctomedicina.com; iberocto@ctomedicina.com WEB: www.ctomedicina.com; www.iberocto.com

RM Pg. 5

sobresaturacin como consecuencia de una excrecin renal disminuida y otros factores como la ingesta elevada de vitamina C. Aunque
existe una forma de oxalosis primaria (respuesta 1) caracterizada por la
presencia de hiperoxalemia, nefrolitiasis, insuficiencia renal y muerte
alrededor de los 20 aos, la mayora de los casos se asocian a insuficiencia renal terminal (respuesta correcta 2). Otras enfermedades que
aparecen en esta pregunta, como la hemocromatosis o el
hiperparatiroidismo, se relacionan con el depsito de cristales de
pirofosfato.
Desde el punto de vista clnico, el depsito de cristales de oxalato
afecta con frecuencia a las articulaciones de rodillas y manos, produciendo en la articulacin calcificacin (condrocalcinosis) al igual que
otros cristales de calcio. Los derrames articulares suelen ser no
inflamatorios con un nmero de clulas inferior a las 2000/mm3. El
aspecto y la birrefringencia de los cristales de oxalato es variable,
siendo frecuentemente de forma bipiramidal y con birrefringencia
muy positiva.
Desde el punto de vista teraputico hay que tener en cuenta que
en la oxalosis primaria se indica el trasplante heptico. En los pacientes con insuficiencia renal se evitan los suplementos con vitamina C.
Cuando se produce clnica se utilizan la colchicina, los AINEs o los
corticoides intraarticulares.
Pregunta 25.- R: 4
En este caso clnico debemos plantearnos el diagnstico diferencial en una monoartritis aguda. Las causas ms habituales de esta
sintomatologa son las infecciones articulares, las artritis por microcristales y tambin se puede considerar la artritis reactiva, aunque en este
ltimo caso suelen afectarse nuevas articulaciones de forma aditiva en
el curso evolutivo.
La paciente de esta pregunta ha sido sometida recientemente a una
intervencin quirrgica; esta situacin puede favorecer por un lado la
aparicin de una bacteriemia, y por lo tanto de una artritis sptica, y
debido a la situacin de stress que supone, favorecer la artritis por
microcristales.
El estudio del lquido sinovial, tcnica ms importante para poder
definir el diagnstico, nos muestra un nmero de clulas muy elevado, por encima de las 50.000/mm3, lo cual suele ser tpico de las
infecciones, pero ocasionalmente tambin se observa en procesos
inflamatorios muy activos. La ausencia de microorganismos en la
tincin de GRAM y sobre todo en el cultivo hace poco probable la
infeccin, ya que la sensibilidad de esta tcnica es del 90%. Finalmente la presencia positiva de cristales de forma rectangular, caractersticos de la artropata por cristales de pirofosfato, confirma como respuesta correcta la nmero 4, la artropata aguda por cristales de
pirofosfato clcico o pseudogota.
Pregunta 26.- R: 1
El lupus eritematoso sistmico es el prototipo de enfermedad autoinmune caracterizada por la presencia de clnica inflamatoria en mltiples rganos y sistemas. Para poder contestar correctamente a esta
pregunta debemos tener en cuenta la clasificacin de las manifestaciones de la enfermedad y la frecuencia de su aparicin:
Manifestaciones sistmicas (95%) como fatiga, fiebre, anorexia.
Musculoesquelticas (95%) como artralgias, artritis (60%, respuesta
correcta 1), mialgias.
Cutneas (80%) como erupcin malar, fotosensibilidad, lceras
orales, erupcin discoide.
Hematolgicas (85%): anemia de trastornos crnicos, leucopenia,
linfopenia o trombopenia. La presencia de anemia hemoltica es
poco habitual (10%, respuesta 2).
Neurolgicas (60%): disfuncin cognitiva (50%, respuesta 3), sndromes orgnicos cerebrales.
Cardiopulmonares (60%): pleuritis (50%) y pericarditis (30%; respuesta 4).
Renales (50%): proteinuria, cilindros celulares. La forma ms frecuente de afectacin renal activa es la glomerulonefritis proliferativa difusa ms que la focal (respuesta 5).
Gastrointestinales (45%) como dolor, nuseas, alteraciones de las
enzimas hepticas.

Pg. 6 RM

M exico A rgentina
C hile U ruguay

REUMATOLOGA

Preparacin Examen de Seleccin 05/06 1 Vuelta


Como puedes comprobar, todas las manifestaciones que aparecen
en las respuestas de la pregunta aparecen en el lupus eritematoso
sistmico, pero debemos considerar como la ms habitual de ellas la
aparicin de artritis.
Pregunta 27.- R: 3
El diagnstico del lupus eritematoso se basa en la utilizacin de 11
criterios diagnsticos. Se precisa la presencia de al menos 4 de ellos,
no existiendo ningn criterio obligatorio. Recientemente, en el ao
1997, se han editado unas pequeas modificaciones respecto a los
criterios aceptados desde 1982, quedando actualmente de la siguiente forma:
1. Eritema malar (eritema fijo, plano o elevado sobre los pmulos).
2. Eritema discoide (placas elevadas de eritema con descamacin
queratsica).
3. Fotosensibilidad (la exposicin a rayos ultravioleta causa erupcin
cutnea).
4. lceras bucales (lceras bucales o nasofarngeas observadas por el
mdico).
5. Artritis (artritis erosiva con afectacin de dos o ms articulaciones
perifricas, que se caracterizan por dolor, tumefaccin o derrame).
6. Serositis (pleuritis o pericarditis).
7. Enfermedad renal (proteinuria mayor de 500 mg/24 horas o cilindros celulares).
8. Enfermedad neurolgica (convulsiones o psicosis sin ninguna otra
causa).
9. Enfermedad hematolgica (anemia hemoltica, leucopenia, linfopenia o trombopenia, descartando induccin farmacolgica de la
citopenia).
10.Trastornos inmunolgicos (anti-DNAds, anti-Sm o anti-fosfolpido).
11.Anticuerpos antinucleares (ttulo de AAN elevado por inmunofluorescencia).
Como ves, la respuesta 3 es la nica correcta, al estar incluida la
psicosis lpica en el criterio nmero 8, dentro de la enfermedad
neurolgica.
Pregunta 28.- R: 4
En esta pregunta se manifiestan algunos de los factores genticos
que se han descrito asociados al lupus eritematoso sistmico.
La respuesta 1 recuerda que los gemelos monocigticos desarrollan cerca del 50% del lupus cuando uno de ellos est afectado. Esta
asociacin hereditaria tambin se observa en los familiares de primer
grado, de los que el 10% desarrollan la enfermedad (respuesta 2).
A continuacin se describe en varias respuestas la asociacin que
algunos anticuerpos tienen con distintas manifestaciones clnicas de
la enfermedad. Por ejemplo, en la respuesta 3 se recuerda que los
anticuerpos anti-DNA tienen relacin con la aparicin de nefritis
lpica, al demostrarse su depsito formando parte de inmunocomplejos en los glomrulos afectados. Su aparicin est relacionada con
el HLA-DR2 y DR3. La presencia de anticuerpos anti-Ro tiene una
clara relacin con la aparicin de diversas manifestaciones clnicas
como el lupus cutneo subagudo. Sin embargo los anticuerpos antiRo se relacionan con el HLA-DR3, y no DR-4, como aparece en la
respuesta 4, por lo que esta es la respuesta que debemos elegir.
En la respuesta 5 se recuerda que el anticoagulante lpico se relaciona con DR4 y DR7. El anticoagulante se define por la presencia de un
alargamiento patolgico del tiempo de tromboplastina parcial activado
(TTPA), no corregible con la administracin de plasma fresco, y se debe
a la presencia de anticuerpos antifosfolpido en sangre.
Pregunta 29.- R: 1
El alargamiento del tiempo de tromboplastina parcial activado
(TTPA) es un fenmeno conocido desde hace muchos aos en los
pacientes con lupus eritematoso sistmico y denominado anticoagulante lpico. En los ltimos aos esta alteracin coagulomtrica no
corregible con la administracin de plasma fresco se sabe que es
debida a la presencia de anticuerpos antifosfolpido. Estos pacientes
muestran con frecuencia falsa positividad en el test de sfilis (VDRL), al

CTO Medicina C/ Nez de Balboa, 115 28006 MADRID (Espaa) Tfno.: (91) 782 43 32 / Fax: (91) 782 43 27
E-mail: secretaria@ctomedicina.com; iberocto@ctomedicina.com WEB: www.ctomedicina.com; www.iberocto.com

Comentarios TEST

Seguimiento a distancia

REUMATOLOGA

Preparacin Examen de Seleccin 05/06 1 Vuelta


estar formado el sustrato utilizado para esta prueba en gran parte por
fosfolpidos y producirse una reaccin cruzada al tener el paciente en
suero anticuerpos antifosfolpidos (respuesta 4). La determinacin de
anticuerpos antifosfolpido se realiza actualmente mediante ELISA diseado para la determinacin del fosfolpido ms habitual, que es la
cardiolipina (anticuerpos anticardiolipina). Estos anticuerpos pueden
ser de tipo IgG o IgM. Adems se han ido conociendo una serie de
manifestaciones clnicas asociadas a la presencia de estos anticuerpos, como son:
Morbilidad en el embarazo: abortos, muertes fetales y partos prematuros.
Trombosis: arteriales, venosas y de pequeo vaso (respuesta 3).
Trombocitopenia perifrica (respuesta 2).
Lesiones valvulares (endocarditis de Libman-Sachs).
Livedo reticularis (respuesta 5).

Comentarios TEST

Como podemos comprobar, la nica manifestacin no relacionada es la respuesta 1, la pericarditis.


Pregunta 30.- R: 1
La glomerulonefritis se debe a la presencia de depsitos de inmunocomplejos circulantes o por la formacin de inmunocomplejos in
situ en el mesangio y en la membrana basal del glomrulo. La clasificacin de las glomerulonefritis en el LES comprende las siguientes
formas:
I. Glomrulo normal:
a) Normal (por todas las tcnicas).
b) Normal por microscopa ptica, pero con depsitos en microscopa electrnica o inmunofluorescencia.
II. Alteraciones mesangiales puras:
a) Ensanchamiento mesangial y/o leve hipercelularidad.
b) Moderada hipercelularidad.
III. Glomerulonefritis proliferativa focal o segmentaria (asociada a alteraciones mesangiales leves o moderadas):
a) Lesiones activas necrotizantes.
b) Lesiones activas y esclerosantes.
c) Lesiones esclerosantes.
IV. Glomerulonefritis proliferativa difusa:
a) Sin lesiones segmentarias.
b) Con lesiones activas necrotizantes.
c) Con lesiones activas y esclerosantes.
d) Con lesiones esclerosantes.
V. Glomerulonefritis membranosa:
a) Membranosa pura.
b) Asociada a lesiones de categora II.
c) Asociada a lesiones de categora III.
d) Asociada a lesiones de categora IV.
VI. Glomerulonefritis esclerosante avanzada.
A su vez, se han establecido unos ndices histolgicos de actividad
y de cronicidad:

Pregunta 30. Tipos de lesiones en el LES.

64267 6  8 6

6426
9 4296 94548 6

12345673678594
4383

12345673675 2454383

4296
8 7

4296
8 7

1234567839
6
85593
18
346 7636 468
1865 9
855938
1344
6956 499 8
95938
1 765394 8
58
4
64

1
58346 5483593
1865 9 76349

1 765394 8
559
4 4 
58938

16346 6 836
695
134769 593

M exico A rgentina
C hile U ruguay

Seguimiento a distancia

Cada variable se punta de 0 a 3 (ausente, leve, moderada, grave).


El valor de las variables necrosis fibrinoide y semilunas celulares se
multiplica por dos.
Como puedes comprobar, la nica lesin aguda (reversible) es la
respuesta 1; las dems corresponden a lesiones crnicas (irreversibles).
Pregunta 31.- R: 2
En esta pregunta se repasan una serie de conceptos importantes del
LES y el embarazo. Hay que recordar que la enfermedad aparece
mayoritariamente en mujeres en edad frtil, por lo que es necesario
conocer las interacciones de la enfermedad con la gestacin.
La amenorrea es frecuente (respuesta 4), aunque la tasa de fertilidad en las pacientes con LES es normal. Existe una frecuencia elevada
de aborto espontneo y muerte intratero (10-30%), especialmente
en las que tienen anticuerpos antifosfolpido. El tratamiento de eleccin de las mujeres con abortos previos y anticuerpos antifosfolpido
es la administracin de heparina de bajo peso molecular.
El embarazo produce efectos variables sobre el LES. En ocasiones
la enfermedad se activa, sobre todo durante las 6 semanas posteriores
al parto (respuesta 5). Si no se observa enfermedad renal ni cardiaca
grave y la actividad del LES se encuentra bajo control, la mayora de
las mujeres tiene embarazos a trmino con recin nacidos normales.
Los glucocorticoides (excepto los de vida media larga, dexametasona y betametasona) son inactivados por enzimas placentarias y no
producen anomalas fetales (respuesta 2). Los inmunosupresores y
los antipaldicos no pueden utilizarse durante el embarazo (respuesta 1).
Queda por lo tanto la respuesta nmero 3 como correcta. Ante la
relacin patognica de los estrgenos en el desarrollo del LES se desaconseja su utilizacin como mtodo anticonceptivo, siendo de eleccin los mtodos de barrera (preservativos, diafragma).
Pregunta 32.- R: 3
En esta pregunta se repasan algunas de las manifestaciones hematolgicas que aparecen en el LES.
La primera afirmacin es falsa, ya que la causa ms frecuente de
anemia es la de trastornos crnicos, siendo sin embargo la anemia
hemoltica menos habitual, pero ms caracterstica (criterio diagnstico).
La esplenectoma es una medida necesaria en ocasiones cuando
se produce una citopenia (sobre todo trombocitopenia) que no responde al tratamiento habitual con corticoides en dosis altas. Otras
medidas que pueden emplearse en caso de trombocitopenia grave
son la administracin de gammaglobulina intravenosa, los citotxicos
o el danazol. Sin embargo, la mayora de los pacientes tienen
trombocitopenia leve (respuesta correcta 3).
La leucopenia es otro hallazgo habitual en la enfermedad, pero
casi nunca se asocia a infecciones recidivantes y no precisa tratamiento (respuesta 4).
Finalmente se observa que el 50% de los pacientes presentan test
de Coombs directo positivo de tipo complemento (no Ig), que no es
indicativo de hemlisis (respuesta 5). La anemia hemoltica aparece
en el 10% de los pacientes y est mediada por anticuerpos IgG, que
dan una prueba de Coombs directa positiva para Ig y complemento.
La prueba definitiva es la deteccin de los anticuerpos en el fluido de
las clulas sanguneas.
Pregunta 33.- R: 5
El LES puede afectar a cualquier localizacin neurolgica, incluyendo encfalo, mdula, sistema nervioso perifrico y pares craneales. La disfuncin cognitiva leve es la manifestacin mas frecuente. En
esta pregunta se repasan varias de estas manifestaciones, as como
algunas de las relaciones patognicas que se conocen.
La respuesta 1, la presencia de anticuerpos antineuronales, se describe hasta en el 60% de los casos, situndose el antgeno en la superficie neuronal y linfocitaria. En algunos estudios, los ttulos elevados
de IgG se relacionan con la afectacin difusa del SNC.
Tambin se han descrito los anticuerpos antimielina que pueden
producir dao en la conduccin nerviosa a nivel perifrico (respuesta 2).

CTO Medicina C/ Nez de Balboa, 115 28006 MADRID (Espaa) Tfno.: (91) 782 43 32 / Fax: (91) 782 43 27
E-mail: secretaria@ctomedicina.com; iberocto@ctomedicina.com WEB: www.ctomedicina.com; www.iberocto.com

RM Pg. 7

La polineuropata sensitiva o mixta es la manifestacin ms frecuente de afectacin del sistema nervioso perifrico (10%) (respuesta
3), siendo menos habitual la mononeuritis mltiple o la polineuropata motora en relacin a vasculitis. Los pares craneales afectados mas
frecuentemente son los motores oculares, el nervio ptico (produciendo oftalmoplejias, ptosis, escotomas, alucinaciones visuales o ceguera central) y el trigmino, respetando habitualmente la rama motora (respuesta correcta, la 5).
La mielitis trasversa se asocia con frecuencia a la presencia de anticuerpos antifosfolpidos (respuesta 4), por mecanismo isqumico.
Pregunta 34.- R: 5
El LES es una enfermedad autoinmune donde se ha descrito la
presencia de mltiples anticuerpos antinucleares. Debes tener en cuenta que, aunque algunos antgenos no se encuentran en el ncleo, sino
en el citoplasma, de forma genrica a todos estos anticuerpos se les
denomina antinucleares (aunque, como ves, algunos son
anticitoplasmticos). Por este motivo, la respuesta correcta es la 5. Los
AAN se encuentran en el 98% de los pacientes con LES y su presencia
se demuestra enfrentando el suero del paciente a clulas de cultivo
(clulas Hep 2) y detectados mediante inmunofluorescencia. Se consideran positivos a titulaciones por encima de 1:40.
Los anticuerpos anti-DNAds (de doble cadena) son relativamente
especficos del LES, apareciendo en el 70% de los casos (respuesta 4).
Son todava ms especficos los anti-Sm, pero estos se observan solo
en el 30% de los enfermos.
Los anticuerpos antihistona aparecen con ms frecuencia en el lupus inducido por frmacos (95%) que en el LES (30-70%) (respuesta 3).
Los anticuerpos anti-RNP aparecen en mltiples enfermedades autoinmunes como polimiositis, esclerodermia, enfermedad mixta del
tejido conectivo y LES (40%) (respuesta 2). Cuando aparecen en el LES
sin anticuerpos anti-DNA, el riesgo de nefritis es bajo.
Por ltimo, los anticuerpos antifosfolpido descritos en la respuesta
1 se observan en el 20-50% de los casos.
Pregunta 35.- R: 2
El lupus inducido por frmacos es un sndrome similar al LES
inducido por la toma de distintos medicamentos como la procainamida, hidralazina, isoniazida, clorpromazina, D-penicilamina, metildopa, quinidina o interfern alfa, y posiblemente por hidantonas,
etosuximida y anticonceptivos orales. Este sndrome ocurre con poca
frecuencia entre los frmacos citados, excepto con la procainamida,
el medicamento ms frecuente, y la hidralazina. Existe una predisposicin gentica en los sujetos acetiladores lentos hepticos. Los
sntomas ms frecuentes son los generales (astenia, febrcula) y las
artralgias. La pleuropericarditis aparece en el 25-50% (repuesta correcta 2). Los sntomas severos que podemos observar en el LES son
aqu raros, como la afectacin del SNC (respuesta 3) o la renal (respuesta 5).
La aparicin de los sntomas no es inmediata desde el inicio de la
toma del frmaco, pasando habitualmente varios meses hasta que se
detecta la aparicin de los anticuerpos antinucleares y el inicio de la
enfermedad (respuesta 1).
En la mayora de los pacientes se detectan anticuerpos antinucleares tipo anti-histona, pero no se encuentran los anticuerpos especficos del lupus eritematoso sistmico, como son los anti-DNAds o los
anti-Sm (respuesta 4).
Finalmente el abordaje teraputico en esta enfermedad consiste en
la retirada de la medicacin responsable, con lo que la mayora mejora
en pocas semanas. Si los sntomas son graves, est indicada la administracin de corticoides durante poco tiempo (de 2 a 10 semanas).
Pregunta 36.- R: 4
En esta pregunta se plantea el caso de una paciente con LES con los
siguientes sntomas: manifestaciones generales como astenia y febrcula, artritis y serositis leve, y te plantea distintas posibilidades teraputicas. Debes tener en cuenta que se trata de una paciente con sntomas leves de la enfermedad, por lo que la intensidad del tratamiento
debe ir en relacin con la gravedad y la respuesta lgica es la utilizacin de AINEs (respuesta correcta nmero 4), y en caso de no responder, utilizar corticoides en dosis bajas.
Pg. 8 RM

M exico A rgentina
C hile U ruguay

REUMATOLOGA

Preparacin Examen de Seleccin 05/06 1 Vuelta


El resto de las alternativas planteadas son la utilizacin de corticoides en dosis elevadas e inmunosupresores o la combinacin de ambos frmacos. Este tipo de tratamiento tiene sentido si las manifestaciones del paciente fueran severas (por ejemplo glomerulonefritis proliferativa difusa, afectacin del sistema nervioso central, neumonitis lpica, citopenia severa como anemia hemoltica o trombocitopenia).
Recientemente se ha incluido dentro del uso de inmunosupresores la administracin de un nuevo frmaco denominado micofenolato, en dosis de 1 a 2,5 gr/da en varias dosis. La azatioprina se utiliza
en dosis de 2-3 mg/kg/da y la ciclofosfamida en bolos mensuales con
dosis de 10 a 15 mg/kg, reduciendo de esta forma su habitual toxicidad vesical cuando se administra por va oral.
Pregunta 37.- R: 3
En esta paciente se plantean algunas lesiones cutneas que aparecen en el lupus eritematoso sistmico y la actitud teraputica que
debemos tomar. De forma esquemtica, lo que podemos recordar es
lo siguiente:
Lesiones cutneas en el contexto de manifestaciones sistmicas
(anemia, nefropata, serositis): suelen mejorar con el tratamiento
que se establece para estas ltimas, generalmente corticoides sistmicos.
Lesiones cutneas aisladas especficas (lupus discoide): aplicacin
de corticoides tpicos (triamcinolona o fluocinolona) dos o tres
veces al da. En la cara es aconsejable utilizar preparados de hidrocortisona, a fin de evitar las lesiones atrficas residuales, el acn o
las telangiectasias. Si la respuesta a las dos semanas es insuficiente,
se asociarn antimalricos (respuesta correcta nmero 3). En caso
de mala respuesta a las pautas anteriores, se puede aadir
prednisona en dosis de 0,5 mg/kg/da durante 15 das y despus
reducir progresivamente la dosis.
Lupus cutneo subagudo: en este caso, el tratamiento es con antimalrico junto a corticoide tpico. Si a las 4 semanas no hay evidencia de mejora, aadir corticoide sistmico (prednisona 0,5 mg/kg/
da). Una alternativa en caso de mala respuesta a la pauta anterior
es la talidomida en dosis de 25-50 mg/da, si bien es un frmaco no
comercializado en nuestro pas (su indicacin es de uso compasivo) y sus efectos secundarios ms importantes son la teratogenicidad y la neuropata perifrica.
El caso planteado es el de un lupus cutneo discoide demostrado
por la biopsia cutnea, por lo que al no haber dentro de las respuestas
la posibilidad de utilizar corticoides tpicos debemos elegir la
hidroxicloroquina.
Pregunta 38.- R: 1
En este caso nos preguntan por la actitud teraputica en una paciente con lupus eritematoso sistmico con clnica preferente a nivel
del sistema nervioso central, habindose realizado la tcnica de imagen ms til para su diagnstico, la RNM. En este caso nos muestra
reas de captacin de alta intensidad compatible con una zona inflamatoria mas evidente a nivel parietotemporal. En trminos generales
las manifestaciones neuropsiquitricas se dividen en dos grupos a la
hora de enfocar el tratamiento:
Manifestaciones difusas, como psicosis, depresin profunda, sndrome orgnico cerebral, convulsiones, meningitis, etc, probablemente mediadas por anticuerpos especficos (antineuronales,
antiprotena P ribosmica), inmunocomplejos y/o fenmenos de
vasculitis: el tratamiento debe individualizarse segn el tipo y gravedad del cuadro clnico. En general, adems de intentar controlar
la actividad lpica con dosis altas de corticoides (1-2 mg/kg/da)
(respuesta correcta 1), se deben utilizar frmacos sintomticos como
anticonvulsivantes o antipsicticos.
Accidentes isqumicos cerebrales, bien por trombosis secundaria
a sndrome antifosfolpido, por vasculitis o por combinacin de
ambos factores: el tratamiento fundamental es la anticoagulacin.
Si adems coexisten signos de actividad inflamatoria, aadiremos
corticoides, asociados o no a inmunosupresores.
En esta paciente hay que incorporar corticoides orales a su tratamiento, como indica la respuesta correcta. La administracin de car-

CTO Medicina C/ Nez de Balboa, 115 28006 MADRID (Espaa) Tfno.: (91) 782 43 32 / Fax: (91) 782 43 27
E-mail: secretaria@ctomedicina.com; iberocto@ctomedicina.com WEB: www.ctomedicina.com; www.iberocto.com

Comentarios TEST

Seguimiento a distancia

REUMATOLOGA

Preparacin Examen de Seleccin 05/06 1 Vuelta


bamacepina (respuesta 4) tendra sentido si presentara crisis convulsivas, y la administracin de antidepresivos si presentara depresin,
asociados, como comentamos en el punto 1 a la administracin de
corticoides.

Comentarios TEST

Pregunta 39.- R: 4
De las manifestaciones hematolgicas en el LES, solo requieren tratamiento especfico la anemia hemoltica clnicamente significativa y la
trombopenia grave. La leucopenia slo precisa tratamiento con prednisona en el caso excepcional de neutropenia autoinmune grave.
El tratamiento inicial de la anemia hemoltica y la trombopenia grave
es la prednisona en dosis de 1-2 mg/kg/da (respuesta 1). En el caso de la
anemia, si no se controla a las 3 semanas de tratamiento, debe considerarse la esplenectoma o el tratamiento con inmunosupresores. En el
caso de la trombopenia, si el paciente presenta hemorragias se debe
considerar la administracin de gammaglobulina intravenosa (400 mg/
kg/da durante 5 das), que permite una elevacin rpida de plaquetas
(respuesta 2), aunque suele ser transitoria. Si el control con la prednisona no es bueno, como alternativa a la esplenectoma pueden utilizarse
bolos mensuales de ciclofosfamida (750-1000 mg/m2 de superficie corporal) durante 6 meses (respuesta 3), y si no hay respuesta, pueden
ensayarse antes de la esplenectoma pulsos de dexametasona, eficaces
en la prpura trombtica idioptica.
Tras la esplenectoma (respuesta 5) hay pacientes que no se controlan con una dosis razonable de corticoides. En ellos se puede intentar
vincristina, danazol, gammaglobulina intravenosa o plasmafresis.
Como puedes comprobar, dentro del arsenal teraputico tiene
cabida cualquiera de los frmacos sealados excepto la D-penicilamina (respuesta 4). Este frmaco lo utilizamos en la artritis reumatoide
en la segunda lnea teraputica y tambin como antifibrtico en la
esclerosis sistmica.
Pregunta 40.- R: 1
En esta pregunta debes recordar los criterios diagnsticos de la artritis reumatoide para encontrar la respuesta adecuada. Ten presente
que, de los 7 criterios utilizados para el diagnstico de esta enfermedad, el paciente debe presentar al menos 4. Los criterios son:
1) Rigidez matutina: rigidez en y alrededor de las articulaciones que
dura una hora antes de que se produzca la mejora funcional
mxima.
2) Artritis de 3 o ms reas articulares: al menos tres reas observadas
de forma simultnea por el mdico, con tumefaccin de partes
blandas o derrame articular. Las 14 reas que se pueden afectar de
forma bilateral son IFP, MCF, carpo, codo, rodilla, tobillo y MTF.
3) Artritis de las articulaciones de las manos: incluyendo carpo, MCF
e IFP.
4) Artritis simtrica: afectacin simultnea de las mismas reas articulares en ambos lados del cuerpo.
5) Ndulos reumatoides: ndulos subcutneos sobre las prominencias seas, superficies extensoras o regiones yuxtaarticulares, observados por un mdico.
6) Factor reumatoide srico: demostrado por cualquier mtodo en
que el resultado haya sido positivo en menos de un 5% de la
poblacin sana.
7) Alteraciones radiolgicas: en radiografa de carpos y manos, con
erosiones u osteopenia en banda.
Como puedes comprobar, la presencia de lceras orales (respuesta
1, correcta) no se incluye dentro de los criterios diagnsticos en la
artritis reumatoide. Ten presente que s son criterio en el LES y en la
enfermedad de Behet.
Pregunta 41.- R: 2
Esta pregunta repasa algunos de los conceptos ms importantes en
relacin al factor reumatoide.
En primer lugar podemos recordar que el factor reumatoide es una
inmunoglobulina (habitualmente IgM, pero puede ser de cualquier
tipo) dirigida contra la Fc de una IgG. De esta forma podemos comprobar cmo las respuestas 3 y 4 son dos afirmaciones correctas. El factor
reumatoide IgM es detectado por tcnicas de aglutinacin (aglutinacin en ltex: partculas de ltex recubiertas por IgG humana; WaalerRose: hemates de carnero recubiertos por IgG de conejo).
M exico A rgentina
C hile U ruguay

Seguimiento a distancia

La aparicin del factor reumatoide, aunque tpica de la artritis


reumatoide, no es exclusiva de esta enfermedad y se ha comprobado
su presencia en mltiples situaciones como: sndrome de Sjgren y
otras enfermedades del tejido conectivo, cirrosis biliar primaria, infecciones, neoplasias, enfermedades pulmonares y otras enfermedades
inflamatorias agudas o crnicas. Incluso se demuestra su presencia
despus de vacunaciones o transfusiones (repuesta 5).
Se acepta que entre el 1 y el 8% de los individuos sanos presentan
factor reumatoide IgM, aunque la frecuencia es mayor por encima de
los 60 aos (respuesta correcta: 2, ya que no disminuye su presencia,
sino que aumenta con la edad).
Los pacientes con artritis reumatoide que presentan concentraciones importantes de factor reumatoide IgM tienen mayor riesgo de
presentar manifestaciones extraarticulares, en particular ndulos reumatoides y vasculitis (respuesta 1), y una enfermedad erosiva grave.
Pregunta 42.- R: 1
La evolucin de la artritis reumatoide es variable. La mayora
tienen una actividad mantenida, fluctuante con un grado variable
de deformidad articular. Las remisiones son ms probables durante
el primer ao. La progresin de la enfermedad es mayor durante los
6 primeros aos y luego tiende a remitir. Tienen peor pronstico los
pacientes con ttulos altos de factor reumatoide, elevacin de reactantes de fase aguda como VSG , PCR y haptoglobina, ndulos subcutneos, erosiones radiolgicas en la primera valoracin, presencia de ms de 20 articulaciones afectadas o nivel socioeconmico
bajo.
Con lo que acabamos de comentar puedes estar seguro de que las
respuestas 2 (ttulos elevados de factor reumatoide), 3 (ndulos subcutneos), 4 (elevacin de VSG) son todos ellos parmetros de mal pronstico en la enfermedad.
La respuesta nmero 5, el sndrome de Felty, es una situacin clnica dentro de la artritis reumatoide que se acompaa de esplenomegalia y neutropenia. Estos pacientes suelen tener ttulos elevados de factor reumatoide, que ya hemos visto que por s solo es un dato de mal
pronstico, pero adems tienen por la neutropenia tendencia a las
infecciones de repeticin, por lo que el pronstico se agrava.
La respuesta correcta es la afectacin de manos. Este hallazgo es
prcticamente constante en la artritis reumatoide, y por lo tanto no
puede discriminar a los pacientes con peor pronstico.
Pregunta 43.- R: 3
En este caso clnico se plantea la aparicin de un cuadro de monoartritis aguda en una paciente con artritis reumatoide como enfermedad de base. Siempre que se plantean preguntas en las que la sintomatologa es la inflamacin monoarticular aguda, la exploracin clave para orientar el diagnstico es la realizacin de la artrocentesis y el
examen del lquido sinovial. En el caso que nos ocupa es lo que se ha
realizado, encontrando un lquido de aspecto purulento (con ms de
50.000 clulas/mm3 y marcado descenso de la glucosa) caracterstico
de las infecciones articulares agudas. Respecto a la pregunta que se
realiza, debes realizar el siguiente razonamiento:
El Staphylococcus epidermidis es el germen que aparece relacionado con mas frecuencia en las infecciones de prtesis articulares, y
despus del aureus en las infecciones articulares secundarias a inoculacin directa intraaarticular (p.e. artrocentesis).
Las infecciones por Streptococcus se suelen observan con ms frecuencia en nios, por debajo de los 5 aos.
El Staphylococcus aureus se considera, de forma global, el germen
que con ms frecuencia produce artritis sptica. Su prevalencia a
partir de los 40 aos es mayor que ningn otro germen. Por este
motivo debes considerar esta la respuesta correcta. Hay que recordar que la artritis reumatoide, por la destruccin articular que produce, predispone a las infecciones articulares, siendo el aureus el
microorganismo ms habitualmente hallado.
La infeccin gonoccica ocurre con ms frecuencia en adultos
jvenes (entre los 15-40 aos), con contactos sexuales mltiples sin
proteccin (preservativo).
La infeccin tuberculosa produce una artritis lentamente progresiva monoarticular, con lquido sinovial con menor nmero de clulas que el presentado, unas 20.000 clulas/mm3.

CTO Medicina C/ Nez de Balboa, 115 28006 MADRID (Espaa) Tfno.: (91) 782 43 32 / Fax: (91) 782 43 27
E-mail: secretaria@ctomedicina.com; iberocto@ctomedicina.com WEB: www.ctomedicina.com; www.iberocto.com

RM Pg. 9

Seguimiento a distancia

REUMATOLOGA

Preparacin Examen de Seleccin 05/06 1 Vuelta

Pregunta 44.- R: 2
La artritis reumatoide es una enfermedad inflamatoria articular
que produce afectacin preferente de pequeas articulaciones de
manos y pies (excepto las interfalngicas distales), aunque se puede
observar inflamacin en cualquier articulacin sinovial. Respecto a
la afectacin del esqueleto axial (columna vertebral y articulaciones
sacroiliacas), la enfermedad produce clnica en la columna cervical
en el 40% de los casos, siendo poco habitual la afectacin de la
columna dorsal, lumbar o de las articulaciones sacroiliacas.
Dentro de la columna cervical la manifestacin ms caracterstica
de la enfermedad es la subluxacin atloaxoidea, definida por un
aumento de distancia patolgico entre la superficie anterior de la
apfisis odontoides de la segunda vrtebra y el atlas (primera vrtebra).
Cuando esta distancia supera los 4 milmetros, definimos esta alteracin. Para poder ser visualizada se realiza una radiologa lateral en
flexin del cuello. Esta alteracin radiolgica puede ser asintomtica,
producir dolor en la zona del occipucio o alteraciones neurolgicas.
En el caso de que produzca manifestaciones leves se puede colocar
collarn cervical durante los periodos sintomticos. Si produce clnica
neurolgica, es necesaria la realizacin de reparacin neuroquirrgica (fijacin de la columna cervical al crneo).
El resto de las respuestas que aparecen en la pregunta son tambin
manifestaciones que podemos encontrar en la columna cervical en la
artritis reumatoide. Ten en cuenta que las articulaciones interapofisarias son sinoviales y que se pueden inflamar y tener erosiones como
las articulaciones perifricas.

pender el frmaco. En el caso que discutimos presenta de forma


aislada proteinuria, sin otras alteraciones de la funcin renal, por
lo que esta posibilidad es poco probable.
La trombosis de la arteria renal ocurre en pacientes con alguna
situacin precipitante como traumatismos, compresin extrnseca,
deshidratacin, sndrome nefrtico, embarazo o toma de anticonceptivos. En los casos agudos se produce prdida repentina de la
funcin renal, acompaada de fiebre y dolor lumbar. Como ves,
nuestra paciente ni tiene factores precipitantes ni clnica compatible con este diagnstico.
La amiloidosis AA complica un 10-20% de los casos con artritis
reumatoide, pero produce manifestaciones clnicas en el 3-10%
de los casos, en forma inicial como sndrome nefrtico y posteriormente como insuficiencia renal. La amiloidosis es ms comn en
enfermos con artritis reumatoide de larga evolucin (>10 aos),
con factor reumatoide positivo y artropata destructiva. Como puedes comprobar, esta es la respuesta correcta.
La vasculitis reumatoide puede afectar a cualquier rgano. En la
forma ms agresiva (vasculitis necrosante) puede producir polineuropata o mononeuritis mltiple, afectacin cutnea (necrosis y ulceracin) o visceral (infarto visceral). La vasculitis renal es
rara.

A nivel de los pies tambin podemos encontrar distintas deformidades. La lesin mas caracterstica es el hundimiento del antepi,
pero tambin se observa ensanchamiento del metatarso, hallux valgus, subluxacin plantar de la cabeza de los metatarsianos, dedos en
martillo con desviacin lateral, de forma que el primero se sita por
debajo del segundo.
Pregunta 46.- R: 4
En esta pregunta se plantean varios mecanismos de dao renal que
pueden aparecer en un paciente con artritis reumatoide.
La primera respuesta habla de la glomerulonefritis membranosa
que puede aparecer por la toma de sales de oro. La administracin de este frmaco produce proteinuria en el 5-25% de los
pacientes. La proteinuria aparece a los 4-6 meses de tratamiento
y hasta el 33% desarrollan un sndrome nefrtico florido. La proteinuria cede cuando se suspende el frmaco. Esta posibilidad la
debes descartar, ya que la paciente actualmente no recibe este
tratamiento.
Los AINEs producen distintos efectos sobre el rin, entre ellos
insuficiencia renal aguda de causa hemodinmica, retencin de
sal y agua, hiponatremia, hiperpotasemia, necrosis papilar, nefritis
intersticial, sndrome nefrtico. El sndrome nefrtico y la insuficiencia renal coexisten a menudo por la combinacin de nefritis
intersticial y lesin glomerular de cambios mnimos. Esta entidad
afecta principalmente a los pacientes que toman derivados del
cido propinico, ibuprofeno y naproxeno, pero puede ocurrir
con otros AINEs. La enfermedad renal remite, en general, al susPg. 10 RM

M exico A rgentina
C hile U ruguay

Pregunta 45. Deformacin articular en la artritis reumatoide.

Pregunta 47.- R: 3
La afectacin ocular ms frecuente en la artritis reumatoide es la
queratoconjuntivitis seca que aparece en el 20% de los pacientes
(respuesta correcta 3). Debes recordar que la causa secundaria ms
frecuente de sndrome de Sjgren secundario es la artritis reumatoide.
Con mucha menor frecuencia aparece otra manifestacin ocular
que es la epiescleritis o escleritis, que aparecen en menos del 1% de los
casos. Suele tratarse de pacientes con artritis reumatoide de larga evolucin, con ndulos reumatoides. La epiescleritis suele ser leve y transitoria, mientras que la escleritis afecta a capas ms profundas del ojo y
constituye un proceso inflamatorio ms grave. Desde el punto de vista
anatomopatolgico, la lesin es similar a un ndulo reumatoide y puede causar adelgazamiento y perforacin ocular (escleromalacia
perforante).

CTO Medicina C/ Nez de Balboa, 115 28006 MADRID (Espaa) Tfno.: (91) 782 43 32 / Fax: (91) 782 43 27
E-mail: secretaria@ctomedicina.com; iberocto@ctomedicina.com WEB: www.ctomedicina.com; www.iberocto.com

Comentarios TEST

Pregunta 45.- R: 1
La persistencia del proceso inflamatorio en la artritis reumatoide
favorece la aparicin de destruccin articular y deformidades, siendo
la mano una de las localizaciones habituales. En ella se describen las
siguientes deformidades:
Dedo en cuello de cisne: hiperextensin de las interfalngicas proximales con flexin de las interfalngicas distales (respuesta correcta 1).
Dedo en ojal: presentan una alteracin inversa a la descrita en el
cuello de cisne, flexin de interfalngica proximal con extensin
de interfalngica distal (respuesta 2).
Dedo en martillo: flexin de interfalngica distal (respuesta 4).
Deformidad del primer dedo en Z: hiperextensin de la metacarpofalngica con flexin de la interfalngica.
Desviacin en rfaga cubital: desviacin de los dedos desde las
articulaciones interfalngicas proximales hacia el lado cubital del
carpo.

REUMATOLOGA

Preparacin Examen de Seleccin 05/06 1 Vuelta


Por ltimo, debes recordar que en la artritis reumatoide, a diferencia de las espondiloartropatas, no aparece uvetis anterior. La uvetis
posterior, propia de otras enfermedades como el Behet, tampoco
aparece.
Desde el punto de vista teraputico, si el paciente presenta un
sndrome seco, el tratamiento ser tpico (lgrimas artificiales). Si presenta epiescleritis, se recomiendan los AINEs o corticoides tpicos, y si
se trata de escleritis necrosante, se administrarn corticoides sistmicos en dosis altas (1 mg/kg/da). En el caso de la escleromalacia
perforante, puede ser necesaria la reparacin quirrgica.

Comentarios TEST

Pregunta 48.- R: 5
La afectacin pleural es la manifestacin pulmonar ms comn en
la artritis reumatoide. Se estima que un 20% de los pacientes tienen
dolor pleurtico, pero solo el 5% presentan derrame. Aparece con
ms frecuencia en varones y est asociado a la presencia de factor
reumatoide.
El lquido pleural de la artritis reumatoide es un exudado de aspecto pajizo, verdoso o lechoso, segn su cronicidad. El recuento celular
es inferior a 5000 leucocitos/mm3, con predominio linfocitario. Las
cifras de glucosa son bajas (respuesta correcta 5), habitualmente menores a 30 mg/dl, el factor reumatoide IgM est elevado (ttulo superior
a 1/320), la LDH est aumentada y el complemento descendido. La
enzima adenosindeaminasa (ADA) est elevada.
El estudio histolgico de la pleura parietal muestra ndulos reumatoides y fibrosis, y en la pleura visceral se ven cambios inflamatorios
crnicos inespecficos.
El patrn citolgico del lquido pleural consiste en clulas gigantes
y un fondo de material necrtico granular.
El derrame pleural suele ser bilateral. Se pueden producir empiemas de inicio insidioso, como complicacin del derrame, atribuidos
a diferentes mecanismos, como sobreinfeccin de derrames previos o
pequeos neumotrax en pacientes con ndulos subpleurales.
Pregunta 49.- R: 2
En esta pregunta debes recordar los conceptos que se explican en
neumologa sobre patrn restrictivo y patrn obstructivo en el estudio
espiromtrico.
La enfermedad intersticial aparece en la artritis reumatoide con
mayor frecuencia en varones, observndose alteraciones radiolgicas
compatibles en un 2% de los casos. Sin embargo, los estudios
necrpsicos muestran un porcentaje bastante superior (20%).
Los sntomas comunes son la tos no productiva y la disnea progresiva, con estertores crepitantes bibasales, pudindose observar acropaquias e incluso osteoartropata hipertrfica.
El estudio radiolgico puede ser normal en fases iniciales, detectndose la neumopata por estudios funcionales, o presentar el tpico
patrn de infiltrados reticulonodulares basales (respuesta 3) o pulmn
en panal en fases avanzadas. El TAC torcico de alta resolucin presenta una mayor sensibilidad que la radiologa convencional en la
deteccin de enfermedad intersticial.
Las pruebas de funcin respiratoria muestran un patrn restrictivo,
por lo que la respuesta correcta es la 2, ya que en este patrn, a diferencia del obstructivo, no desciende el ndice VEF1/CVF. En las fases iniciales
de la enfermedad se produce desaturacin y cianosis con el ejercicio
(respuestas 1 y 5). El test de difusin mostrar una alteracin intersticial
(respuesta 4), con alteracin de la difusin pulmonar.
El lavado bronquioalveolar es til para detectar la afectacin subclnica, as como para cuantificar la actividad inflamatoria.
Pregunta 50.- R: 4
En este caso clnico se plantea el caso de un paciente varn con
una poliartritis de curso crnico con rigidez matutina prolongada y en
el que se desarrolla un cuadro de polineuropata y lesiones cutneas
en forma de lceras dolorosas. Del anlisis de las respuestas planteadas podemos decir:
La primera respuesta plantea la duda de que se trate de una artritis
reumatoide (AR) por los sntomas no articulares aparecidos. Sin
embargo, el diagnstico de AR es muy probable, al cumplir varios
criterios para este diagnstico, como son la rigidez matutina de ms
de una hora de evolucin, la afectacin de 3 o ms articulaciones
M exico A rgentina
C hile U ruguay

Seguimiento a distancia

y la artritis de articulaciones de las manos. La aparicin de clnica


neurolgica perifrica en forma de polineuropata o mononeuritis
mltiple y lceras cutneas es sugestiva del desarrollo dentro de la
AR de una vasculitis reumatoide.
El CH50 (complemento hemoltico medio) suele estar normal en la
AR en sangre perifrica, excepto cuando se produce una vasculitis
reumatoide donde desciende, al formarse inmunocomplejos circulantes que consumen complemento.
La presencia de un sndrome de Felty se confirma cuando el paciente tiene la trada clsica: AR, esplenomegalia y neutropenia. Al
no aparecer los tres datos en el paciente, este diagnstico se descarta.
La respuesta correcta es la 4. El caso planteado es caracterstico de
una vasculitis reumatoide de tipo necrotizante sistmica, por lo
que la biopsia de un rgano afectado (piel o sistema nervioso perifrico), mostrar los hallazgos tpicos histolgicos.
En la artritis reumatoide, la incidencia de linfomas parece algo
superior a la esperada en la poblacin general. Sin embargo, los
sntomas del paciente son compatibles con una vasculitis necrotizante sistmica.

Pregunta 51.- R: 3
En esta pregunta es necesario recordar la actitud teraputica y
alguno de los efectos secundarios que pueden aparecer cuando tratamos a un paciente con artritis reumatoide (AR).
La primera escala teraputica incluye la administracin de AINEs,
asociados o no a corticoides en dosis bajas por va oral. Sin embargo,
con este tratamiento el proceso inflamatorio se controla en pocos
pacientes, por lo que es necesario incorporar un frmaco de segunda
lnea en la mayora de los casos. Dentro de este grupo se incluyen una
serie de frmacos clsicos (sales de oro, D-penicilamina, sulfasalazina
y antipaldicos), inmunosupresores (metotrexate, ciclosporina),
inmunomoduladores (leflunomida) y frmacos anti-TNF alfa
(infliximab, etanercep).
La administracin de estos frmacos no est exenta de efectos
secundarios y en la pregunta nos plantean el caso de un paciente
que tras recibir sales de oro desarrolla un sndrome nefrtico. Este
frmaco produce como toxicidad ms habitual lesiones mucocutneas, pero algunos pacientes desarrollan una glomerulonefritis membranosa con sndrome nefrtico. Se ha comprobado que la aparicin de esta toxicidad es ms habitual en los que presentan HLADR3, como ocurre en nuestro paciente. La aparicin de toxicidad
importante obliga a la suspensin del tratamiento y a plantearnos
otra alternativa dentro de los frmacos de segunda lnea. Sin embargo, no podemos optar dentro de ellos en este caso a la utilizacin de
D-penicilamina, ya que este frmaco tambin produce la misma
toxicidad renal que las sales de oro, por lo que la respuesta correcta
es la 3.
Pregunta 52.- R: 5
La artritis crnica juvenil, tambin denominada artritis idioptica
juvenil, se define por la presencia de un cuadro de artritis de curso
crnico (superior a las 6 semanas), en nios o jvenes menores de 16
aos, de etiologa autoinmune. Dentro de las respuestas que aparecen en esta pregunta se recogen las distintas formas evolutivas de la
enfermedad:
La forma sistmica aparece por igual en nios y nias, a una edad
muy precoz (por debajo de los 5 aos), presentando un cuadro
sistmico de fiebre, rash, linfoadenopatas y hepatoesplenomegalia acompaando a la artritis.
La forma poliarticular se define por la afectacin de ms de 4
articulaciones, distinguindose la forma seropositiva (con factor
reumatoide positivo) y seronegativa (factor reumatoide negativo).
La forma seropositiva, igual que ocurre en los adultos, es ms grave
desde el punto de vista articular y adems puede presentar diversas
manifestaciones extraarticulares. Sin embargo no aparece uvetis,
igual que no apareca uvetis en la artritis reumatoide del adulto. La
forma poliarticular seronegativa es menos agresiva que la seropositiva, no asocindose a la aparicin de manifestaciones sistmicas.
La forma oligoarticular de inicio tardo es la nica que predomina
en varones y aparece por encima de los 10 aos. Est asociado a

CTO Medicina C/ Nez de Balboa, 115 28006 MADRID (Espaa) Tfno.: (91) 782 43 32 / Fax: (91) 782 43 27
E-mail: secretaria@ctomedicina.com; iberocto@ctomedicina.com WEB: www.ctomedicina.com; www.iberocto.com

RM Pg. 11

REUMATOLOGA

Seguimiento a distancia

Preparacin Examen de Seleccin 05/06 1 Vuelta

Pregunta 53.- R: 1
La presentacin de este caso clnico rene las caractersticas de
una artritis crnica juvenil: se trata de un cuadro de inflamacin articular de curso crnico (superior a las 6 semanas) en el que se descarta
en el cultivo del lquido sinovial la presencia de enfermedad bacteriana
articular.
Dentro de la clasificacin de la artritis crnica juvenil corresponde a la forma de presentacin ms habitual de la enfermedad, la
oligoarticular de inicio precoz: presentacin antes de los 6 aos,
afectacin de 4 o menos articulaciones (en este caso rodilla y tobillo). Analizando las respuestas, debemos razonar de la siguiente forma:
La respuesta 1 es la correcta, ya que en esta forma de presentacin al menos el 75% de los casos tienen anticuerpos antinucleares. Recuerda que en esta forma de enfermedad aparece en un
20% de los casos uvetis crnica. Sin embargo, el pronstico articular es el ms favorable de todas las formas de artritis crnica
juvenil.
Los anticuerpos anticardiolipina no aparecen asociados a la artritis
crnica juvenil.
La VSG estar elevada, como en cualquier proceso inflamatorio.
La asociacin de esta forma de enfermedad es con HLA DR5 y
DR8, y no con DR4, que es tpico de los pacientes con forma
poliarticular seropositiva.
El HLA-B27 se asocia a las formas oligoarticulares de inicio tardo.
Pregunta 54.- R: 1
La actitud teraputica en la artritis crnica juvenil pasa por la identificacin de su forma de presentacin, ya que cada una de ellas
precisa distintas medidas farmacolgicas. De forma general, en cualquiera de las presentaciones de la enfermedad la primera escala teraputica a ensayar es la administracin de aspirina en dosis altas o
AINEs. A continuacin se recuerdan los frmacos ensayados segn la
forma de presentacin:
En los pacientes con formas sistmicas, la primera escala teraputica suele ser insuficiente para el control de la inflamacin articular
y de los sntomas generales, por lo que suele ser preciso aadir
corticoides. Por lo tanto, la respuesta correcta es la 1. Dado que los
corticoides pueden producir retraso del crecimiento en los nios,
cuando se administran de forma prolongada se debe valorar su
combinacin o sustitucin por inmunosupresores.
En los pacientes con formas poliarticulares debes plantear el tratamiento igual que en la artritis reumatoide del adulto; si no hay
buena respuesta con la primera escala teraputica debemos aadir un frmaco de segunda lnea como sales de oro, D-penicilamina, hidroxicloroquina, sulfasalazina, metotrexate o ciclosporina A. Tambin se utilizan actualmente los frmacos anti-TNF.

Las formas oligoarticulares son las de mejor pronstico y menor


agresividad en el dao articular, por lo que la primera escala de
tratamiento suele ser suficiente.
Pregunta 55.- R: 1
Las espondiloartropatas son un grupo de enfermedades inflamatorias donde se incluyen como entidades ms importantes la espondilitis anquilosante, la artritis psorisica, la artritis reactiva y la artritis
asociada a enfermedad inflamatoria intestinal (colitis ulcerosa y enfermedad de Crohn). Las caractersticas de estas enfermedades se contraponen a las que aparecen en la artritis reumatoide y las podemos
resumir en la tabla a pie de pgina.
Como podemos comprobar, la respuesta nmero 1 es la correcta,
ya que la afectacin poliarticular simtrica es caracterstica de la artritis reumatoide y no de las espondiloartropatas, que suelen tener presentacin oligoarticular asimtrica.
Pregunta 56.- R: 5
Los datos caractersticos que presenta esta paciente son compatibles con el diagnstico de espondilitis anquilosante (respuesta 5): dolor y rigidez de predominio nocturno en la zona lumbar (es decir,
dolor lumbar de caractersticas inflamatorias), radiologa de pelvis donde se observa la presencia de erosiones en ambas articulaciones sacroilacas y esclerosis del margen del hueso ilaco (inicialmente en la
regin inferior de la articulacin) (sacroiletis grado II). Recuerda que
para el diagnstico de esta enfermedad nos basamos en la presencia
de una serie de criterios diagnsticos:
Sacroiletis radiolgica y al menos la presencia de un criterio menor:
Dolor inflamatorio en la columna lumbar.
Limitacin en los tres planos en la movilidad de la columna lumbar.
Limitacin de la expansin torcica.
La paciente, como podemos comprobar, cumple el criterio mayor
y uno de los menores, por lo que podemos confirmar el diagnstico
de espondilitis anquilosante. Las otras opciones pueden descartarse
por los siguientes motivos:
La artritis reumatoide no tiene sacroiletis.
No existen antecedentes epidemiolgicos de artritis reactiva.
No existe enfermedad cutnea compatible con psoriasis.
La artritis brucelar suele producir una sacroiletis unilateral y no
bilateral, como en este caso.
Pregunta 57.- R: 4
En la espondilitis anquilosante se describe la presencia de mltiples manifestaciones extraarticulares (ver tabla en la pgina siguiente, donde se compara con la artritis reumatoide), entre las que se
encuentran varias de las respuestas de esta pregunta. Las manifestaciones ms relevantes son:
Uvetis anterior aguda (respuesta 3): es una de las manifestaciones
ms habituales. Suele presentarse de forma unilateral y se manifiesta por dolor, fotofobia, lagrimeo y visin borrosa. Los ataques de
uvetis no suelen durar ms de 2 3 meses y en la mayora de los
casos curan sin dejar secuelas.
Afectacin cardiovascular: inflamacin de la raz artica produciendo insuficiencia artica (respuesta 5) en el 10% de los pacien-

Pregunta 55. Diferencias entre las espondiloartropatas y la AR.


1234567849

439 92

123456789
2 62
8 264
9 6
123456789
2 627 45264
9 65
123 68
89729
 984
2
59 98

12
896 9522
68
8972 6265466 2
9

12
79 65
8976526 

 984
6528476524884 76
5
222679 497
9
5297 65 97
65284
65
123589
8972872 3!"#$
123 68
8972%95 98
28

8 65 98
267 659 952&97 

8972 6
222
297568972 67 975
267262%465'
Pg. 12 RM

M exico A rgentina
C hile U ruguay



7 729 476

12(
8 264
9 62&$)*'
12+ 45264
9 652&#)*'
12,9
 9 95259 98

12+25
896 9522
68
89728
59268459-
2 62
2847
286-98

12
79 65
8976526 

 984
6527 45254.84 7652/649 95
222-
5849 95257 62 62(6 26 8
123589
8972872 3!012
123 68
8972%95 98
28

8 65 98
23/
774532597-9
2&97 

897
222 62
26.
7
2597-9
'

CTO Medicina C/ Nez de Balboa, 115 28006 MADRID (Espaa) Tfno.: (91) 782 43 32 / Fax: (91) 782 43 27
E-mail: secretaria@ctomedicina.com; iberocto@ctomedicina.com WEB: www.ctomedicina.com; www.iberocto.com

Comentarios TEST

HLA-B27 y pueden observarse episodios de uvetis anterior aguda,


y no uvetis crnica, como nos preguntan.
La respuesta correcta es la forma oligoarticular de inicio precoz.
Esta es la forma de presentacin ms frecuente de artritis crnica juvenil, y se asocia en la mayora de los casos a la presencia
de anticuerpos antinucleares. En un 20% de los casos se desarrolla uvetis de curso crnico como complicacin de la enfermedad.

REUMATOLOGA

Preparacin Examen de Seleccin 05/06 1 Vuelta

Seguimiento a distancia

tes con ms de 30 aos de evolucin de la enfermedad. Tambin


se pueden encontrar trastornos de la conduccin por fibrosis.
Manifestaciones pleuropulmonares: poco frecuentes y de aparicin tarda. La ms tpica es la fibrosis de los lbulos pulmonares
superiores, que pude ser colonizada por Aspergillus.
Manifestaciones neurolgicas: fracturas-luxaciones vertebrales cervicales, subluxacin atloaxoidea con clnica cervical o sndrome
de cola de caballo secundario a aracnoiditis crnica. Sin embargo,
no observamos meningoencefalitis, por lo que la respuesta correcta es la 4.
Manifestaciones genitourinarias: la ms frecuente es la prostatitis
crnica y la nefropata IgA (respuesta 1).

Pregunta 57. Manifestaciones extraarticulares de la AR y la EA.

1232434562789
3 4 7



782
6689

 2

743 824
24

Comentarios TEST

47534

5  44345

84 5
37

1234567879
296 6
385
 36 59634 74
 834 6 
29 56345
 8347558 5
34745963
344533245
 47 437 95963
 47 437 95963
395 3 
397  "46 83
27 279543
 94733559
479!2 7 6 
3445334 8#4 87$ %9 2 8 398 5546 85
34 854 6 
& 7 7 
347565'734254 (47 656 6 849 85
347565%'&
74#4587 
%9558 9* 67' 85
3 58 9339) 5 +9789 85+,  545
393435
*3#6 85 
955674 5 963 6 95

Pregunta 58.- R: 5
Respecto a la actitud teraputica que debemos tomar ante un paciente con espondilitis anquilosante, debemos tener en cuenta las
siguientes pautas:
No existe ningn tratamiento especfico que pueda detener el curso evolutivo de la enfermedad, por lo que nuestra actitud debe
dirigirse a controlar la inflamacin y prevenir el desarrollo de deformidades.
La base del tratamiento mdico es la utilizacin de AINEs. Dentro de este grupo la fenilbutazona es el frmaco ms eficaz para
el control sintomtico, pero su uso est restringido para los brotes de gran inflamacin y durante un tiempo limitado al producir en ocasiones agranulocitosis. Por este motivo debemos optar
para el tratamiento crnico por otros AINEs, como la indometacina.
Cuando la afectacin perifrica (artritis perifrica) es importante,
podemos utilizar algunos de los tratamientos que se utilizan en la
artritis reumatoide, como la sulfasalazina o el metotrexate. Sin embargo, estos frmacos no modifican el curso de la enfermedad
axial. Otros frmacos de segunda lnea no han mostrado eficacia,
como las sales de oro (respuesta correcta 5), D-Penicilamina o los
antipaldicos.
Los corticoides sistmicos no son de utilidad, pero s cuando se
utilizan de forma intralesional en la entesopata o en la sinovitis
persistente que no responde a AINEs.
M exico A rgentina
C hile U ruguay

Pregunta 58. Afectacin vertebral en la espondilitis anquilosante.

Pregunta 59.- R: 2
Este cuadro clnico presenta muchas de las caractersticas de la
respuesta que elegimos, la artritis reactiva. La enfermedad aparece
con ms frecuencia en varones jvenes (entre los 20-40 aos), en un
periodo entre 1-4 semanas despus de presentar un cuadro infeccioso intestinal o uretral que en ocasiones pasa desapercibido. La clnica
articular caracterstica es la de una artritis asimtrica que predomina
en extremidades inferiores de inicio brusco, aunque aditiva (se van
sumando articulaciones afectas). Al formar parte de las espondiloartropatas es caracterstica la aparicin de entesitis, por este motivo el
paciente tiene talalgia bilateral (inflamacin de los tendones de la
fascia plantar o del aquleo).
Desde el punto de vista del estudio del lquido sinovial, este es de
caractersticas inflamatorias y asptico. Este ltimo dato es imprescindible, ya que si hay crecimiento de algn germen a nivel articular, la
artritis sera sptica y no reactiva.
El resto de respuestas las descartamos por los siguientes motivos:
El lupus eritematoso sistmico es ms habitual en mujeres jvenes,
produce artralgias o artritis migratoria sin entesitis.
La artritis reumatoide produce un cuadro de poliartritis simtrica
sin entesitis.
La artritis gonoccica puede producir una artritis aguda, pero suele
estar precedida de un cuadro febril con bacteriemia, con artritis
migratoria y tenosinovitis.
La artritis infecciosa no produce entesitis, el lquido sinovial es de
caractersticas infecciosas (ms de 50.000 clulas/mm3 y consumo
de glucosa) y no tiene entesitis. El GRAM es positivo en un nmero
elevado de pacientes. (Ver tabla en la pgina siguiente)
Pregunta 60.- R: 2
La artritis reactiva o sndrome de Reiter es una espondiloartropata
que aparece despus de una infeccin uretral o entrica en el plazo
de 1 a 4 semanas. Todas las bacterias involucradas en la aparicin de
una artritis reactiva son intracelulares y afectan caractersticamente a
mucosas, surgiendo una respuesta autoinmune anmala en stas. Probablemente el mecanismo es la aparicin de una respuesta antibacteriana local que posteriormente se reproducira en la sinovial, al existir
en sta componentes que guardaran similitud morfolgica con alguna estructura bacteriana.
En el momento de la aparicin de la artritis, los pacientes de forma
mayoritaria han curado de la infeccin inicial. Los cuadros de artritis
reactiva de origen entrico son ms prevalentes en Europa y los de
origen uretral en los pases anglosajones (EE.UU. y Reino Unido).
Los principales grmenes responsables de la infeccin gastrointestinal son la Shigella flexneri, Salmonella, Yersinia y Campylobacter, por
lo que la respuesta a esta pregunta es la 2, Pseudomonas.

CTO Medicina C/ Nez de Balboa, 115 28006 MADRID (Espaa) Tfno.: (91) 782 43 32 / Fax: (91) 782 43 27
E-mail: secretaria@ctomedicina.com; iberocto@ctomedicina.com WEB: www.ctomedicina.com; www.iberocto.com

RM Pg. 13

REUMATOLOGA

Seguimiento a distancia

Preparacin Examen de Seleccin 05/06 1 Vuelta

Pregunta 59. Diagnstico diferencial entre la Enfermedad


de Behet y la artritis reactiva.
Behet

Concepto

Afectacin
articular

Piel

Artritis reactiva

Es una vasculitis que


afecta a vasos de
pequeo calibre.

Es una
espondiloartropata
s e r o n e g a t iv a , e n
general, secundaria a
una infeccin entrica
o g e n it o u r in a r ia .

Oligoartritis.
N o e r o s iv a .
No espondilitis.

Poliartritis erosiva y
m uy dolorosa.
Dactilitis, entesitis
aqulea o fascitis
plantar.
Espondilitis en m enos
del 50%.

Diversas lesiones:
papulopstulas en cara y
Queratoderm ia
tronco, eritem a nodoso;
blenorrgica.
pseudofoliculitis.
Prueba de patergia +.

Anejos
cutneos

No onicopata.

Mucosas

lceras orales dolorosas. lceras orales


lceras genitales
indoloras.
dolorosas.
Balanitis circinada.

Ojos

Uvetis posterior.

Onicopata .

Conjuntivitis.
Uvetis anterior.

Pregunta 61.- R: 5
Dentro de la artritis reactiva, como ocurre en la mayora de las enfermedades articulares inflamatorias, existen una serie de manifestaciones
extraarticulares, destacando dentro de ellas las mucocutneas.
En la boca hay lceras transitorias, superficiales, y en la mayora de
los casos, asintomticas (respuesta 1).
La lesin cutnea caracterstica es la queratodermia blenorrgica
(respuesta 2), constituida por vesculas que se vuelven hiperqueratsicas y acaban formando costras antes de desaparecer. Son ms frecuentes en las palmas de manos y plantas de pies, pero pueden aparecer en cualquier otra parte. En los pacientes infectados por el virus
VIH, estas lesiones suelen ser muy extensas y sumamente graves, dominando el cuadro clnico.
A menudo hay lesiones en el glande, denominadas balanitis circinada (respuesta 3), que consisten en vesculas que rpidamente se
rompen para formar erosiones superficiales indoloras que pueden
formar costras parecidas a las de la queratodermia blenorrgica.
Tambin son frecuentes las lesiones ungueales en forma de oniclisis (respuesta 4), coloracin amarillenta de la parte distal e hiperqueratosis en capas superpuestas.
Como puedes comprobar, la nica manifestacin que no aparece
es el pioderma gangrenoso. Esta lesin cutnea s se puede relacionar
con otras formas de espondiloartropata: la artritis asociada a enfermedad inflamatoria intestinal.
Pregunta 62.- R: 2
Despus de un cuadro de uretritis infecciosa, un paciente puede
presentar dos cuadros articulares diferentes: si la infeccin ha sido

Pg. 14 RM

M exico A rgentina
C hile U ruguay

producida por gonococo, una artritis sptica gonoccica (es tericamente la causa ms frecuente de infeccin articular en adultos jvenes) o una artritis reactiva, si la infeccin ha sido por Chlamydia. Si
analizamos las respuestas que tenemos, analizamos as las diferencias
clnicas:
La tenosinovitis es caracterstica de la artritis gonoccica. En la artritis reactiva hay entesitis (inflamacin de la insercin del tendn en
el hueso).
El dolor lumbosacro, efectivamente, es caracterstico de la artritis
reactiva y no de la gonoccica, ya que al formar parte del grupo de
las espondiloartropatas puede producirse sacroiletis.
Si encontramos un cultivo positivo a nivel genital de gonococo, no
podemos estar del todo seguros de que sea el germen productor
del cuadro articular. En ocasiones hay infecciones mixtas, incluyendo Chlamydia, y el paciente desarrolla una artritis reactiva. El
diagnstico definitivo de artritis gonoccica se establece si el paciente tiene cultivo positivo a nivel del lquido sinovial.
En la artritis reactiva nunca puede haber cultivo positivo a nivel
articular, ya que en este caso se tratara de una artritis sptica, y no
de una artritis reactiva.
En el caso de la artritis gonoccica, la base de su tratamiento, lgicamente, es la administracin de antibiticos (penicilina o mejor
cefalosporina de tercera generacin). En el caso de la artritis reactiva, sern los AINEs.
Pregunta 63.- R: 1
En este caso clnico se plantea la actitud teraputica ms adecuada en un paciente con artritis psorisica con enfermedad cutnea relevante. El esquema que utilizamos es muy similar al que se
utiliza en la artritis reumatoide y lo podemos resumir de la siguiente forma:
Si el paciente tiene una forma de artritis leve, utilizamos la primera
escala teraputica, que incluye la administracin de AINEs, y si es
preciso, aadir corticoides orales en dosis bajas.
En el caso de que la enfermedad sea ms severa, aadir frmacos
de segunda lnea para reducir la progresin de la enfermedad.
Dentro de estos frmacos es de especial utilidad el metotrexato,
que es eficaz para la enfermedad articular y cutnea. Los antipaldicos como la hidroxicloroquina pueden mejorar los sntomas,
pero no se suelen utilizar porque pueden empeorar las lesiones
cutneas. Frmacos de segunda lnea como las sales de oro y la
sulfasalazina pueden resultar igualmente eficaces.
Otro frmaco que se ha ensayado es la ciclosporina A, que tambin
puede mejorar la enfermedad reumtica y cutnea. La utilizacin de
retinoides, como el etetrinato, puede tambin ser eficaz pero suele
estar limitado su uso por los efectos secundarios que produce.
Como puedes comprobar, el tratamiento que resulta ms eficaz
para tratar la enfermedad articular y la enfermedad cutnea es la
utilizacin del metotrexate, que es la respuesta que debemos elegir.
Pregunta 64.- R: 5
En esta pregunta se plantean dos cuestiones: por un lado, las caractersticas clnicas de la artritis relacionada con la enfermedad inflamatoria intestinal, y por otro, la relacin del HLA-B27 con las distintas
espondiloartropatas.
La manifestacin reumatolgica ms frecuente es la artritis perifrica (respuesta 1), que se encuentra en relacin con la extensin de la
enfermedad intestinal. El porcentaje de pacientes con EII que la padecen se sita entre el 10-25%. La afectacin axial (sacroiletis y espondilitis) vara segn los estudios entre el 1-20%. En general, es ms frecuente en la colitis ulcerosa.
La artritis suele ser mono u oligoarticular asimtrica (respuesta 3),
con predominio de grandes articulaciones de miembros inferiores
(rodillas y tobillos), y en menor medida en miembros superiores
(carpos, codos y pequeas articulaciones de manos). El episodio articular es de instauracin brusca. Su curso se relaciona con la extensin e intensidad de la enfermedad intestinal. La artritis es autolimitada y generalmente no erosiva (respuesta 4). Su patrn evolutivo es
cclico y recidivante en las mismas articulaciones o migratorio en sucesivas articulaciones.

CTO Medicina C/ Nez de Balboa, 115 28006 MADRID (Espaa) Tfno.: (91) 782 43 32 / Fax: (91) 782 43 27
E-mail: secretaria@ctomedicina.com; iberocto@ctomedicina.com WEB: www.ctomedicina.com; www.iberocto.com

Comentarios TEST

En las formas genitourinarias el agente desencadenante suele ser la


Chlamydia trachomatis, y con menor frecuencia, el Ureaplasma
urealyticum.

REUMATOLOGA

Preparacin Examen de Seleccin 05/06 1 Vuelta

Prevalencia de B-27

Pregunta 67.- R: 2
La degeneracin sarcomatosa es la complicacin ms grave que
puede aparecer en la evolucin de la enfermedad de Paget, aunque su
prevalencia es baja (1%).

90%
60-80%
50-70%

Pregunta 67. Diferencias entre el osteosarcoma primario


y el osteosarcoma sobre hueso pagtico.

Pregunta 64. Relacin entre el HLA-27 y las espondiloartropatas.


Espondiloartropata
Espondilitis anquilosante
Artritis Reactiva
Espondilitis psorisica y espondilitis en
enfermedad inflamatoria intestinal (EII)
Artritis perifrica psorisica
Artritis perifrica en EII

14%
7%

Como podemos comprobar, la respuesta es la 5, ya que los pacientes con artritis perifrica en la enfermedad inflamatoria intestinal no
presentan una prevalencia superior a la de la poblacin general de
HLA-B27 (7%).

Comentarios TEST

Pregunta 65.- R: 5
El paciente que nos plantean en este caso presenta las caractersticas
radiolgicas clnicas y analticas tpicas de la enfermedad de Paget.
Desde el punto de vista radiolgico, la presencia de un patrn
mixto, esclertico y ltico (patrn en mosaico) en localizaciones seas
tpicas (pelvis y hueso femoral) debemos considerarlo como tpico. La
afectacin a este nivel puede producir artropata degenerativa, al alterar la enfermedad el hueso intraarticular, produciendo dolor de caractersticas mecnicas.
Los pacientes con enfermedad de Paget suelen ser mayores de 50
aos, y en la analtica se observa elevacin de los parmetros de
formacin sea relacionados con la actividad de los osteoblastos (fosfatasa alcalina) y parmetros de resorcin sea relacionados con la
actividad de los osteoclastos (hidroxiprolinuria).
Las complicaciones que pueden aparecer en la enfermedad de
Paget son:
Hipercalcemia e hipercalciuria por aumento del recambio seo
(respuesta 1).
Artropata coxofemoral por afectacin de esta articulacin (respuesta 2).
Debilitamiento del hueso afecto con fracturas patolgicas (respuesta
3).
Aumento del gasto cardiaco por aumento de vascularizacin de
los huesos afectos por la enfermedad, produciendo en ocasiones
insuficiencia cardiaca (respuesta 4).
Como podemos ver, la respuesta es la 5. No tiene por qu presentar
hipovitaminosis D, a no ser que tenga asociado un dficit de toma de
esta vitamina o de exposicin solar.
Pregunta 66.- R: 5
Respecto a la etiopatogenia de la enfermedad de Paget, aunque se
desconozca la causa exacta que produce la enfermedad, existen una
serie de datos que debes conocer:
En los aos 70 se comunic el hallazgo,mediante microscopa
electrnica, de unas inclusiones que aparecan en el ncleo de los
osteoclastos de pacientes con enfermedad de Paget y que guardaban gran similitud con estructuras virales. Ms tarde se consigui
identificar en los osteoclastos protenas del virus del sarampin y
del virus respiratorio sincitial, as como del virus distemper (del
moquillo canino). En un principio estas inclusiones se consideraron especficas de la enfermedad de Paget, pero posteriormente se
han encontrado partculas similares en otras enfermedades (tumor
de clulas gigantes y ostelisis expansiva familiar), as como en la
picnodisostosis, la osteopetrosis y la oxalosis primaria. Recientemente se ha demostrado que los pacientes con enfermedad de
Paget expresan el gen de la IL-6 en los osteoclastos y presentan un
aumento de IL-6 en el plasma. Esta produccin excesiva de IL-6,
que puede ser inducida por un virus, actuara como estmulo local
del remodelado seo.
Cerca del 40% de los pacientes con enfermedad de Paget tienen
un familiar de primer grado que tambin padece la enfermedad,
por lo que se especula con la presencia de algn factor hereditario
que favorezca el desarrollo de la enfermedad. Sin embargo, el
estudio del HLA no ha arrojado resultados concluyentes (respuesta
correcta, 5).
M exico A rgentina
C hile U ruguay

Seguimiento a distancia

586 68  3 86


6 4 667 8 
 86

7523 85

123452678596
123452678596 25 74

7 967 5
425 63 85
12345676789
6 82
 
8 2

366867388285 


42 889267
3 88
6
2 8832
2

!6 7

"
6#8#72
"
6#828
6  
8 85
3

93 # 6
18  %8
36
2
88'(()
65638*6


6
2 8273$ %89
2 &
9
8828
289
#736  6&6#8 73
3
$8 8 23473767
9 
27

Aunque no siempre ocurre, se puede producir elevacin de la fosfatasa alcalina respecto a la que el paciente ya tiene elevada por la
enfermedad de Paget. Como podemos ver, la respuesta es la 2.
Pregunta 68.- R: 3
De forma genrica, debemos recordar que la indicacin para el
tratamiento de la enfermedad de Paget se produce en los casos en que
la enfermedad es sintomtica. Sin embargo, en algunas ocasiones tambin puede recomendarse en pacientes asintomticos, como veremos a continuacin. Los frmacos que vamos a utilizar son aquellos
que reducen la actividad de los osteoclastos, es decir, la calcitonina y
sobre todo los bifosfonatos.
Las indicaciones absolutas para el tratamiento son:
Dolor seo primario del hueso pagtico.
Deformidad sea rpidamente progresiva.
Hipercalcemia e hipercalciuria.
Insuficiencia cardiaca.
Preparacin para ciruga ortopdica (tratamiento previo para reducir la actividad del hueso, su vascularizacin y hacer que el
sangrado sea menor).
Podemos considerar como indicaciones relativas el tratamiento de
las complicaciones evolutivas, como son:
Sndromes neurolgicos compresivos (tratar a los pacientes con
enfermedad en vrtebras o base del crneo).
Fisuras y fracturas corticales (tratar a los pacientes con lesiones
lticas en huesos sustentadores de carga).
En los pacientes con discreta elevacin de la fosfatasa alcalina sin
manifestaciones clnicas asociadas como se indica en la respuesta 3,
no existe indicacin de tratamiento.
Pregunta 69.- R: 4
La definicin actual de la osteoporosis postmenopusica es la siguiente: densidad mineral sea que disminuye 2,5 desviaciones estndar (DE) por debajo del pico de masa sea que se produce en mujeres
jvenes (a los 30-35 aos), medido mediante el T-score o puntuacin
T. Las posibilidades que podemos encontrar en una mujer postmenopusica al medir su masa sea son las siguientes:
Densitometra normal: prdida de masa sea hasta 1 DE respecto
al T-score.

CTO Medicina C/ Nez de Balboa, 115 28006 MADRID (Espaa) Tfno.: (91) 782 43 32 / Fax: (91) 782 43 27
E-mail: secretaria@ctomedicina.com; iberocto@ctomedicina.com WEB: www.ctomedicina.com; www.iberocto.com

RM Pg. 15

REUMATOLOGA

Preparacin Examen de Seleccin 05/06 1 Vuelta

Osteopenia: prdida de masa sea entre 1 DE y 2,5 DE respecto


al T-score.
Osteoporosis: prdida de masa sea igual o superior a 2,5 DE
respecto al T-score.
Osteoporosis grave: prdida de masa sea igual o superior a 2,5
DE respecto al T-score asociada a la presencia de fractura.
Los valores de la densitometra tambin vers que en ocasiones son
comparados con el denominado Z-score o puntuacin Z. En este
caso, la variacin en desviaciones estndar se realiza entre la densitometra de la paciente y la media de masa sea que tienen las mujeres
de su misma edad. Sin embargo, como ya hemos sealado, la definicin actual se realiza utilizando el T-score y no el Z-score.
Pregunta 70.- R: 1
Las causas de osteoporosis las podemos recordar en la siguiente
tabla:
Osteoporosis primaria:
- Postmenopusica.
- Senil.
- Idioptica juvenil y del adulto joven.
Osteoporosis secundaria:
- Enfermedades endocrinas y metablicas: hipogonadismo, hiperparatiroidismo, hipertiroidismo, hipercorticismo (exgeno o
endgeno), hipofosfatasia y asociada a otras enfermedades endocrinas como la osteomalacia.
- Genticas: osteognesis imperfecta, homocistinuria, sndrome
de Ehlers-Danlos, sndrome de Marfan.
- Hematolgicas: mieloma.
- Frmacos: corticoides, heparina, antiestrgenos.
- Otras causas: mastocitosis, escorbuto, desnutricin, alcoholismo, mastocitosis sistmica.
Podemos comprobar como la artrosis, que es la respuesta que
debemos elegir, no aparece asociada a la aparicin de osteoporosis.
Pregunta 71.- R: 2
La osteoporosis tipo II o del anciano presenta una serie de caractersticas analticas y clnicas que podemos recordar precisamente estableciendo el diagnstico diferencial con la osteoporosis tipo I o postmenopusica.

Pregunta 71. Diagnstico diferencial entre osteoporosis tipo I y tipo II.

123456575282938659

123456575282938659

123456789

456789

 2

77
77

776697

 2 967886 

97
76 7!

9"#7

89 49 9


75 89 4
 !"1#1$
%25789 49 85
834238 
&5 8' 89 49 2
(7 37 2

$#7667
76!
76967
766%779
9

79

&
!

'9

&
!6
77

&
!677

(7)7

(7)7

*#7
6677+9

7

*#7
6,7879
-.6677


4 8
479574
8659 49425
647 8 5

En la osteoporosis tipo II, como se puede comprobar en la tabla,


hay una reduccin primaria de la produccin de vitamina D3 (respuesta correcta 2). El motivo es que esta vitamina se produce en el
rin, y en los ancianos esta funcin se vuelve ms deficitaria, por lo
que los niveles de esta vitamina suelen estar disminuidos, de ah que se
Pg. 16 RM

M exico A rgentina
C hile U ruguay

absorba menos calcio a nivel intestinal y se estimule la produccin de


PTH (hiperparatiroidismo secundario).
Pregunta 72.- R: 2
Dentro de las causas de osteoporosis de base hereditaria se incluyen una serie de enfermedades como la osteognesis imperfecta, la
homocistinuria, el sndrome de Ehlers-Danlos y el sndrome de Marfan.
De esta ltima enfermedad es tpica la ectopia o luxacin del cristalino, respuesta que debemos elegir.
La osteognesis imperfecta es una enfermedad congnita del tejido conectivo, muy heterognea desde el punto de vista clnico, bioqumico y gentico. El defecto bioqumico primario, determinado
genticamente, es el resultado de un dficit cuantitativo y cualitativo
del colgeno tipo I. Por ello, no solo se afecta el esqueleto, sino
tambin otros tejidos, como la piel, ligamentos, tendones, dientes,
vasos sanguneos, vlvulas cardiacas y huesos timpnicos. Constituye la enfermedad congnita ms comn, que se traduce en
osteopenia.
La forma tipo I es la ms benigna y menos deformante, y tambin la
que se encuentra con mayor frecuencia en los adultos (60% de los casos). Normalmente no existen deformaciones, y si existen, son mnimas.
La historia clnica consiste en la existencia de mltiples fracturas, despus
de traumatismos mnimos desde el periodo neonatal (respuesta 4). La
incidencia de fracturas disminuye despus de la pubertad.
Las esclerticas se encuentran azuladas en todas las edades (respuesta 3). Los trastornos auditivos aparecen en el 70% de los casos
(respuesta 1). Su herencia es autosmica dominante (respuesta 5).
Pregunta 73.- R: 5
Las posibilidades teraputicas en la osteoporosis tipo I son actualmente las siguientes:
Frmacos moduladores de los receptores estrognicos como el
Raloxifeno. Los efectos de este frmaco ms relevantes son la reduccin del cncer de mama invasor de un 70% de las pacientes
que lo toman. Al contrario del tamoxifeno, el raloxifeno no se
asocia con un incremento del riesgo de cncer uterino. El raloxifeno
aumenta los sofocos. Reduce el colesterol y el LDL, las lipoprotenas y el fibringeno, por lo que es posible que reduzca el riesgo
cardiovascular. Aumenta la masa sea y reduce la aparicin de
fracturas.
Bifosfonatos, como el alendronato o el risedronato. Ambos han
demostrado que aumentan la masa sea y reducen la aparicin de
fracturas.
Estrgenos: los estudios epidemiolgicos indican que las mujeres
que toman estrgenos de sustitucin presentan una reduccin del
50% de las fracturas osteoporticas, incluidas las de cadera. Los
efectos beneficiosos son mximos entre las mujeres que empiezan
pronto con el tratamiento de sustitucin y lo continan de forma
prolongada. Su empleo a largo plazo se puede asociar con un
incremento del riesgo de tromboembolia venosa y de cncer de
vescula biliar, uterino y de mama. En los estudios de observacin,
los estrgenos se han asociado con una reduccin significativa de
la incidencia de infarto de miocardio.
Otras posibilidades teraputicas incluyen la calcitonina, la PTH, el
fluoruro y otros frmacos experimentales.
Como podemos ver, la mitramicina, frmaco que se utiliz hace
aos en la enfermedad de Paget, no se incluye para el tratamiento de
la osteoporosis.
Pregunta 74.- R: 1
La osteomalacia por dficit de aporte de vitamina D es una situacin relativamente frecuente en los ancianos, preferentemente en
aquellos ingresados en centros de crnicos con enfermedades que
les impiden la movilizacin y una adecuada alimentacin. Las dos
fuentes de las que disponemos para la formacin de vitamina D3 son
la exposicin a los rayos ultravioleta con la trasformacin en la piel
de la provitamina D3 (7-dihidrocolesterol) en vitamina D3 y la alimentacin. Este dficit lleva asociado una reduccin de la produccin de 25 (OH)D en el hgado, que es el parmetro bioqumico de
medicin del dficit de vitamina D, y por lo tanto el que utilizamos

CTO Medicina C/ Nez de Balboa, 115 28006 MADRID (Espaa) Tfno.: (91) 782 43 32 / Fax: (91) 782 43 27
E-mail: secretaria@ctomedicina.com; iberocto@ctomedicina.com WEB: www.ctomedicina.com; www.iberocto.com

Comentarios TEST

Seguimiento a distancia

REUMATOLOGA

Preparacin Examen de Seleccin 05/06 1 Vuelta


para realizar el diagnstico de osteomalacia por dficit de aporte de
vitamina D.
En fase avanzada del dficit, la transformacin de 25(OH)D a
1,25(OH)2 D se ve reducida, y con ello la absorcin de calcio intestinal,
y por lo tanto se produce hipocalcemia. Esta estimula la produccin de
PTH, que aumenta la resorcin sea y corrige la calcemia y aumenta la
depuracin renal de fosfatos, produciendo hipofosfatemia (respuesta
correcta 1). Por lo tanto, el patrn bioqumico resultante ser: descenso
de 25(OH)D y en fases avanzadas de 1, 25(OH)2 D, calcemia descendida o normal, hiperparatiroidismo secundario, aumento de la depuracin renal de fosfato e hipofosforemia.

Seguimiento a distancia

Pregunta 76. Manifestaciones clnicas de la esclerosis sistmica.


Manifestacin clnica
Raynaud
Engrosamiento de piel
Calcinosis subcutnea
Telangiectasias
Artritis
Miopata
Motilidad esofgica alterada
Fibrosis pulmonar
Hipertensin pulmonar aislada
Insuficiencia cardiaca
Crisis renales

Forma difusa (%) Forma limitada (%)


90-95
100
10
40
70
50
80
40
<1
30
15

95-100
98
50
85
40
5
80
35
< 10
<1
<1

Pregunta 76. Esclerosis sistmica difusa versus


esclerosis sistmica limitada.

Comentarios TEST

Pregunta 74. Principales osteopatas metablicas.

Pregunta 75.- R: 2
En el caso de un paciente con insuficiencia renal crnica, a diferencia de la debida al dficit nutricional, no tiene por qu existir
descenso en los niveles de 25 (OH)D, ya que en esta ocasin el trastorno se produce en el paso a nivel renal de la 25 (OH)D a 1,25(OH)2 D.
Ser esta ltima molcula la que descender de forma significativa al
estar enfermo el tejido que la produce. Podemos comprobar que la
respuesta correcta es la 2.
El patrn bioqumico que esperamos encontrar en esta situacin
ser el siguiente: 25 (OH) D normal, con descenso marcado de
1,25(OH)2 D. A partir de este momento el esquema es similar al del
dficit de aporte de la vitamina D, pero con una diferencia final: al
reducirse la molcula ms activa de la vitamina D, la 1,25(OH)2 D, la
absorcin de calcio a nivel intestinal se reduce, producindose hipocalcemia y estimulndose la produccin de PTH (hiperparatiroidismo secundario). La PTH aumenta la resorcin de calcio en el hueso y
tiende a corregir la calcemia, pero como el rin est enfermo no se
produce el aumento de la depuracin renal de fosfatos, sino disminucin. Por este motivo, los pacientes con insuficiencia renal crnica
tienen hiperfosfatemia y no hipofosfatemia.
Pregunta 76.- R: 3
Dentro de la esclerosis sistmica se distinguen dos formas de presentacin, que son la esclerosis sistmica difusa y la esclerosis sistmica limitada. Las diferencias fundamentales entre ambas formas de
presentacin aparecen en la tabla de la pgina siguiente.
Podemos ver que los anticuerpos antitopoisomerasa se relacionan
con la forma ms grave de enfermedad, la esclerosis sistmica difusa, y
en ella aparece con mayor frecuencia la fibrosis pulmonar.
La fibrosis pulmonar aparece con mayor frecuencia en la forma
difusa (40%), pero en la limitada tambin puede aparecer (35%), aunque habitualmente con menor gravedad.
M exico A rgentina
C hile U ruguay

123456
728296
6
1234567879
2 5677653 12345677 4
 253
3 96 2 49 6  4
 25375
578
875
5
4

6527 7657595
22 79

75
2 3
5 3757657632
664

76572
32377657926
45
767 23 75
1265452775353732
6265 26 353726545537874
4 353
 79
25773453
9
25
#534
2434256
$29
43275
4
256
3 96 225 6 2
323796 5
7
56

2323736

253 
96 5
72

323
85 66 9 592 6 !534
243425675
"55
2625
79
2 5
25
92 4 5
424 9 23 
5357%
424
 

6924  6 5
Pregunta 77.- R: 5
Existen algunos factores ambientales que estn asociados al desarrollo de la esclerosis sistmica (ES) y de procesos afines a la esclerodermia. La ES parece ms frecuente entre los mineros de carbn y de
oro, especialmente entre los que soportan una exposicin ms intensa, lo que sugiere que el polvo de slice (respuesta 1) quiz sea un
factor predisponente.
Los trabajadores expuestos al cloruro de polivinilo (respuesta 3)
pueden presentar fenmeno de Raynaud, acroostelisis, lesiones cutneas como las de la esclerodermia y alteracin de los capilares.
Existe tambin una relacin entre la exposicin a resinas epoxi e
hidrocarburos aromticos como la bencina, tolueno y tricloroetileno
(respuesta 4) y el desarrollo de esclerodermia.
En 1981 apareci en Espaa una enfermedad generalizada parecida a la esclerodermia debido al consumo de aceite de colza adulterado. Los pacientes presentaron en principio una neumonitis intersticial
con eosinofilia, artritis y miositis, engrosamiento de la piel, fenmeno de
Raynaud, hipertensin pulmonar, sndrome seco y acroostelisis.
Se han descrito casos de esclerodermia y de otras enfermedades
del tejido conectivo en mujeres en las que se han llevado a cabo
implantes de silicona (respuesta 2). Alrededor del implante puede
aparecer fibrosis localizada.
La respuesta correcta es la 5, contacto con asbesto, que produce
una enfermedad pulmonar intersticial fibrosante, pero no un sndrome esclerodermiforme.
Pregunta 78.- R: 5
En esta pregunta debemos tener en cuenta las causas que pueden
producir fenmeno de Raynaud y que se pueden clasificar de la
siguiente forma:

CTO Medicina C/ Nez de Balboa, 115 28006 MADRID (Espaa) Tfno.: (91) 782 43 32 / Fax: (91) 782 43 27
E-mail: secretaria@ctomedicina.com; iberocto@ctomedicina.com WEB: www.ctomedicina.com; www.iberocto.com

RM Pg. 17

Fenmeno de Raynaud primario o idioptico: enfermedad de


Raynaud.
Fenmeno de Raynaud secundario:
- Colagenosis vasculares: esclerosis sistmica, lupus eritematoso
sistmico, artritis reumatoide, dermatomiositis.
- Enfermedades arteriales oclusivas: aterosclerosis de las articulaciones, tromboangetis obliterante, oclusin arterial aguda, sndrome de compresin de la salida del trax.
- Hipertensin pulmonar.
- Alteraciones neurolgicas: trastornos de los discos intervertebrales, siringomielia, tumores de la mdula espinal, poliomielitis, sndrome del tnel del carpo.
- Discrasias sanguneas: crioglobulinemias, macroglobulinemia,
enfermedades mieloproliferativas.
- Traumatismos: lesiones por vibraciones, sndrome de la mano
del martillo, shock elctrico, lesiones por fro, mecanografa,
tocar el piano.
- Frmacos: derivados ergotamnicos, metisergida, bloqueantes
beta-adrenrgicos, bleomicina, vinblastina, cisplatino.
Una vez que hemos recordado las causas que pueden producir
fenmeno de Raynaud, debemos elegir la respuesta correcta.
La crioglobulinemia se incluye dentro de las causas por discrasia
sangunea.
La determinacin de anticuerpos anti-SCL 70 es caracterstica de
la esclerosis sistmica difusa y los anticentrmeros de la esclerosis
sistmica limitada. Ambas enfermedades presentan fenmeno de
Raynaud en la mayora de los casos.
La capilaroscopia es la tcnica que permite el diagnstico diferencial entre el fenmeno de Raynaud primario (donde es normal) del
secundario por colagenosis (donde es patolgica, mostrando dilatacin e irregularidades del lecho vascular ungueal).
La radiologa de trax es til para diagnosticar los signos de compresin torcicos (por ejemplo, la costilla cervical).
La respuesta correcta es la 5, la determinacin de anticuerpos antihistona. Aunque pueden aparecer en el lupus eritematoso
sistmico, no son caractersticos de esta enfermedad y por lo tanto
no orientan para su diagnstico. Se consideran anticuerpos especficos del lupus los anti-DNAds y los anti-Sm.
Pregunta 79.- R: 3
Se considera actualmente que la causa ms frecuente de muerte
en la esclerosis sistmica es la afectacin pulmonar. Adems, dentro
de la afectacin visceral, la enfermedad pulmonar es la segunda en
frecuencia despus de la afectacin del esfago. Dentro de la clnica
pulmonar podemos diferenciar dos formas de presentacin: la fibrosis pulmonar y la hipertensin pulmonar.
En el caso clnico actual, los datos aportados son todos orientativos
al diagnstico de hipertensin pulmonar sin fibrosis pulmonar asociada. Recuerda que esta forma de enfermedad aparece con ms frecuencia en la esclerosis sistmica limitada o sndrome de CREST. Se
caracteriza por la ausencia de fibrosis pulmonar en la radiologa de
trax. De hecho, la hipertensin pulmonar produce un patrn de
oligoemia (falta de vasos visibles) en el parnquima pulmonar. La silueta cardiaca se ve modificada por el incremento de presin pulmonar aumentando el tamao de las cavidades derechas, dilatndose el
cono de la arteria pulmonar.
Aunque no coincidan con los datos de este paciente, debes tener
en cuenta que tambin se pueden observar en los pacientes con
esclerosis sistmicas neumonas por aspiracin secundarias a reflujo
gstrico debido a la atona esofgica inferior. Tambin presentan un
aumento de riesgo de cncer broncognico y alveolar.
Pregunta 80.- R: 4
La mayora de los pacientes con esclerosis sistmica tiene anemia
que puede deberse a varias causas:
Anemia hipoproliferativa propia de los procesos inflamatorios crnicos; es la causa ms frecuente.
Anemia ferropnica debida a un dficit de hierro por hemorragias
digestivas.

Pg. 18 RM

M exico A rgentina
C hile U ruguay

REUMATOLOGA

Preparacin Examen de Seleccin 05/06 1 Vuelta


La proliferacin bacteriana excesiva por atona del intestino delgado puede causar anemia megaloblstica por dficit de vitamina
B12, de cido flico, o de ambos nutrientes.
La anemia hemoltica microangioptica es la que con mayor frecuencia se asocia a enfermedad renal y est causada por el depsito intravascular de fibrina en las arteriolas renales.
La respuesta que debemos elegir es la 4, ya que la anemia refractaria por exceso de blastos no tiene relacin con la esclerosis sistmica.
Esta forma de anemia se incluye dentro de los sndromes mielodisplsicos, encontrndose en la mdula sea un porcentaje de blastos
entre el 5-20% de la celularidad. La supervivencia de este cuadro se
estima entre 12-15 meses.
Pregunta 81.- R: 1
La enfermedad mixta del tejido conectivo (EMTC) rene caractersticas de varias conectivopatas. Poliartritis simtrica similar a la encontrada en la artritis reumatoide. Manifestaciones mucocutneas del
lupus eritematoso sistmico como la erupcin malar, fotosensibilidad,
lesiones discoides, alopecia y lceras dolorosas en la boca. Cambios
esclerodermiformes en la piel similares a los encontrados en la esclerosis sistmica limitada, as como trastornos gastrointestinales caracterizados por la alteracin de la motilidad esofgica y reflujo
gastroesofgico. Algunos pacientes presentan sntomas tpicos de la
polimiositis con debilidad muscular proximal, alteraciones electromiogrficas, elevacin de enzimas musculares, y lesiones inflamatorias en la biopsia muscular. Pocas veces se observa una erupcin eritematosa en nudillos, codos y rodillas, o eritema en heliotropo, datos
caractersticos de una dermatopolimiositis.
Los pacientes con EMTC presentan varias de las manifestaciones
referidas como ocurre en nuestro caso. Para completar el diagnstico
es muy til determinar los anticuerpos anti-RNP que se encuentran a
ttulos muy elevados en esta enfermedad.
Pregunta 82.- R: 2
En esta pregunta se repasan varios conceptos importantes referidos
a las artritis spticas.
En la primera respuesta podemos recordar que la artritis bacteriana
aguda suele manifestarse como una monoartritis aguda que afecta
con ms frecuencia a las articulaciones que soportan carga, siendo
la rodilla la ms afectada seguida de la cadera. La artritis por Gram
negativos tiene un curso ms indolente, con sntomas generales
ms moderados y molestias articulares menos importantes, que la
infeccin por cocos Gram positivos.
La infeccin de prtesis articulares se produce con ms frecuencia
por Staphylococcus epidermidis, seguida del Staphylococcus aureus,
y suele tardar semanas o meses en poder ser diagnosticada, ya que
los sntomas son poco precisos. Por lo tanto, es esta la respuesta que
debemos elegir.
Los ADVP tienen con frecuencia infecciones de la columna vertebral, articulaciones sacroiliacas o esternoclaviculares. Suele presentarse como un proceso agudo con sntomas generales, aumento de temperatura, inflamacin y limitacin de la movilidad. Los
grmenes ms frecuentes son el Staphylococcus aureus y la
Pseudomonas.
Los pacientes en tratamiento con corticoides pueden tener signos
inflamatorios menos llamativos, dificultando el diagnstico.
El Haemophilus influenzae es el germen que produce con ms frecuencia artritis sptica en los dos primeros aos de vida, en aquellas poblaciones no vacunadas. Sin embargo, en la actualidad la
inmunizacin se ha introducido en el programa vacunal en la
mayora de los pases desarrollados, siendo el Staphylococcus aureus el germen ms frecuente en estas poblaciones.
Pregunta 83.- R: 1
La actitud teraputica en los pacientes con artritis sptica se basa en
la administracin de antibiticos sistmicos y el drenaje de la articulacin infectada. Cuando la articulacin tiene un fcil acceso, como
ocurre en la rodilla, el drenaje se puede realizar diariamente mediante
artrocentesis mientras persista el derrame. En articulaciones de difcil
acceso, como puede ser la cadera, se recomienda el drenaje quirrgi-

CTO Medicina C/ Nez de Balboa, 115 28006 MADRID (Espaa) Tfno.: (91) 782 43 32 / Fax: (91) 782 43 27
E-mail: secretaria@ctomedicina.com; iberocto@ctomedicina.com WEB: www.ctomedicina.com; www.iberocto.com

Comentarios TEST

Seguimiento a distancia

REUMATOLOGA

Preparacin Examen de Seleccin 05/06 1 Vuelta

Comentarios TEST

co, al igual que en los pacientes con cultivo persistentemente positivo


del lquido sinovial despus de 7 das de tratamiento.
La eleccin del antibitico se basa inicialmente en el resultado de la
tincin de Gram del lquido articular, y posteriormente en el resultado
del cultivo y antibiograma correspondiente. En la pregunta nos plantean el aislamiento definitivo del germen y nos piden que escojamos la
asociacin de tratamiento incorrecta. La respuesta es la 1, ya que el
tratamiento de la infeccin por Haemophilus influenzae se realiza con
cefalosporina de tercera generacin, como la cefotaxima o la ceftriaxona. En las infecciones menos graves por este germen se pueden
utilizar antimicrobianos por va oral. Hay que tener en cuenta que el
25% de las cepas son productoras de beta-lactamasas y muestran resistencia a la ampicilina, pudiendo ser tratadas con trimetoprim-sulfametoxazol, amoxicilina/cido clavulnico, fluorquinolonas o claritromicina.
El resto de las respuestas corresponden a los antibiticos de eleccin en los distintos grmenes que producen artritis sptica.
Pregunta 84.- R: 5
En esta pregunta se plantean como posibilidades diagnsticas una
serie de infecciones virales que pueden presentar clnica articular, as
como otros diagnsticos diferenciales a tener en cuenta. Veamos a
continuacin cmo debemos analizar las distintas respuestas:
La infeccin por el virus de Epstein-Barr produce un cuadro de
mononucleosis infecciosa caracterizado por la presencia de fiebre, adenopatas y faringitis. En un 5% pueden tener ictericia,
siendo excepcional la aparicin de manifestaciones articulares.
La presentacin clnica no coincide, por lo tanto, con el caso
presentado.
En un paciente con artritis reumatoide de inicio no es esperable la
desaparicin espontnea de los sntomas articulares ni la ictericia.
La aparicin de ictericia es poco habitual al administrar AINEs ( lo
ms frecuente es la hipertransaminasemia), por lo que es poco
probable esta posibilidad.
En los tres das de la erupcin por rubola y en las 2 a 6 semanas
siguientes a la aplicacin de una vacuna con virus atenuados de
rubola, hasta un 50% de las mujeres presentan artralgias y un 10%
artritis persistente. La paciente, sin embargo, no ha presentado erupcin cutnea ni vacunacin reciente, por lo que descartamos esta
posibilidad.
La respuesta correcta es la infeccin por el virus de la hepatitis B.
Hasta un 10% de los pacientes con hepatitis B presentan, en las 2
semanas antes de que aparezca la ictericia, una reaccin mediada
por anticuerpos que se manifiesta por una erupcin maculopapulosa, urticaria, fiebre y artralgias. Tambin puede aparecer una
artritis simtrica de las manos, carpos, codos o tobillos con rigidez
matutina similar a la artritis reumatoide. Los sntomas, como en
nuestro caso, se resuelven cuando aparece la ictericia. Recuerda
que la infeccin por el virus de la hepatitis C se asocia a la aparicin de crioglobulinemia.
Pregunta 85.- R: 2
La infeccin por el virus VIH puede asociar la aparicin de una
serie de manifestaciones musculoesquelticas que resumimos en la
siguiente tabla:

Pregunta 85. Manifestaciones musculoesquelticas en la infeccin VIH.


Artralgias.
Espondiloartropatas:
- Artritis reactiva.
- Artritis psorisica.
- Espondiloartropata indiferenciada.
Artritis asociada a VIH.
Mialgias-Miositis.
Fibromialgia.
Artritis sptica.
Sndrome de Sjgren.
Vasculitis.

M exico A rgentina
C hile U ruguay

Seguimiento a distancia

Como podemos ver en esta tabla, la respuesta es la nmero 2, la


artritis reumatoide.
Debes tener en cuenta que la artritis reactiva en estos pacientes
suele ser grave y, en general, se inicia en la situacin de inmunodepresin severa. La afectacin axial es poco frecuente, predominando la
oligoartritis de miembros inferiores. Las lesiones cutneas pueden predominar en la presentacin clnica. El 70% son HLA-B27. La alta
prevalencia de la artritis reactiva est en relacin con las prcticas
sexuales de riesgo, ms habituales en estos pacientes.
Pregunta 86.- R: 2
La enfermedad de Lyme, debida a la infeccin por la espiroqueta Borrelia burgdorfferi, produce artritis en el 70% de las personas
no tratadas. Das o semanas despus de la inoculacin de la espiroqueta aparecen artralgias y mialgias intermitentes, pero no una autntica artritis. Ms tarde se configuran tres fases de la enfermedad
articular:
El 50% de las personas no tratadas sufren brotes intermitentes de
monoartritis u oligoartritis que afecta a grandes articulaciones como
la rodilla.
El 20% de los no tratados presentan artralgias intermitentes.
El 10% de los pacientes presentan una sinovitis inflamatoria crnica con lesiones erosivas y destruccin articular.
Como podemos ver, el patrn articular ms habitual es el primero,
con afectacin de grandes articulaciones, por lo que debemos elegir
la respuesta 2.
Otras manifestaciones caractersticas de la enfermedad de Lyme no
tratada es la aparicin de:
Afectacin del sistema nervioso (respuestas 3 y 5): meningitis linfocitaria, neuritis craneal, radiculoneuropata, encefalitis.
Afectacin cardiovascular (respuesta 4): bloqueo cardiaco, miocarditis.
En cuanto a la actitud teraputica inicial de la enfermedad, es la
administracin de doxiciclina (respuesta 1). Sin embargo, cuando
aparece clnica articular crnica y/o neurolgica y/o cardiovascular
se recomienda la utilizacin de ceftriaxona parenteral durante 2-4
semanas.
Pregunta 87.- R: 1
Como suele ser habitual para la confirmacin de un diagnstico
de sospecha, el estudio histolgico (respuesta 1) es la prueba ms
especfica para el estudio de la xerostoma en un paciente con sndrome de Sjgren. Dentro de los criterios para la definicin de la enfermedad aparece incluida la biopsia caracterstica tomada de una glndula salivar menor.
El diagnstico subjetivo de la xerostoma se obtiene por la respuesta afirmativa a una de estas preguntas:
Tiene sensacin de sequedad de boca durante los ltimos tres meses?
Tiene que beber cada da ms lquidos para poder deglutir alimentos secos?
Ha notado hinchazn permanente o recurrente en las partidas?
Si el paciente contesta afirmativamente a alguna de las cuestiones
planteadas, la xerostoma se puede objetivar mediante sialografa y
sialometra (medicin de la cantidad de saliva), gammagrafa de partidas, ecografa o resonancia magntica de partida o submaxilares.
Todas estas pruebas tienen escasa especificidad, de ah que sea bsico
realizar la biopsia de glndula salivar menor.
Se debe realizar la biopsia glandular en el labio inferior a mitad de
camino entre la comisura y mitad del labio y se obtendrn 4 o ms
lbulos para valorar la presencia de infiltrado inflamatorio. El diagnstico requiere la presencia de dos o ms focos (presencia de 50 o
ms linfocitos en un rea de 4mm2).
Pregunta 88.- R: 3
La infeccin VIH puede presentar distintas manifestaciones musculoesquelticas, y entre ellas, la aparicin de un sndrome de Sjgren
secundario.

CTO Medicina C/ Nez de Balboa, 115 28006 MADRID (Espaa) Tfno.: (91) 782 43 32 / Fax: (91) 782 43 27
E-mail: secretaria@ctomedicina.com; iberocto@ctomedicina.com WEB: www.ctomedicina.com; www.iberocto.com

RM Pg. 19

REUMATOLOGA

Preparacin Examen de Seleccin 05/06 1 Vuelta

Pregunta 88. Diagnstico diferencial entre el sndrome de Sjgren


primario y el asociado a la infeccin VIH.

4

123455672891
8

 2484842

248 45

66

123456789
6567

963678686252
62

 7652

4884828

64

532
5843
54478 !

532
5843
54478"!

 56567284265

#$

%8&8'$(

46891

)47 2

*6+2 2

2654
345567282

Puedes comprobar que la respuesta 1, infiltrado por linfocitos


CD8+, y la respuesta 2, asociacin con HLA DR5, aparecen como
manifestaciones caractersticas en la infeccin VIH. Las respuestas 4
(test de Schirmer positivo) y 5 (ulceraciones puntiformes en la crnea)
aparecen en el sndrome de Sjgren tanto primario como secundario,
por lo que tambin estarn presentes.
Queda como respuesta correcta la 3, ya que los autoanticuerpos
son caractersticos de la forma primaria.
Pregunta 89.- R: 2
El planteamiento de este caso clnico es caracterstico de una dermatopolimiositis: debilidad muscular de predominio proximal en las
extremidades, acompaado de elevacin de las enzimas musculares
(CPK) y de lesiones cutneas tpicas, las manchas de Gottron (lesiones
eritematosas que predominan en la superficie extensora de pequeas
articulaciones de las manos).
La presentacin clnica de un cuadro de dermatopolimiositis en
un paciente de ms de 55 aos nos obliga a descartar la presencia de
una neoplasia oculta, que ocurre de forma global en el 15% de los
pacientes diagnosticados de esta enfermedad. Dentro de los tumores
que se asocian con ms frecuencia se encuentran el cncer de ovario,
el cncer de mama, el melanoma cutneo y el cncer de colon,
aunque otros tumores como el de pulmn o los hematolgicos tambin hay que tenerlos en cuenta.
Si analizamos las respuestas de la pregunta actual, vemos que parte
de las exploraciones planteadas pueden estar justificadas para la bsqueda de una neoplasia oculta: radiologa de trax-cncer de pulmn, hemograma con frotis-tumor hematolgico, ecografa abdominal-neoplasias abdominales como la de ovario, mamografa-neoplasia de mama.
La respuesta correcta es la 2, ya que es la exploracin menos til
para buscar una neoplasia relacionada con la dematopolimiositis.
Pregunta 90.- R: 1
En la polimiositis o dermatopolimiositis, como ocurre en todas las
enfermedades del tejido conectivo, se han descrito una serie de anticuerpos asociados a la enfermedad, que adems pueden tener relacin con manifestaciones clnicas peculiares. Hasta en un 20% de los
pacientes con miopata inflamatoria de cualquier tipo se observan anticuerpos antinucleares y anticitoplasmticos. Los anticuerpos dirigidos
contra la histidin-ARN de transferencia sintetasa, denominados anti-Jo1, representan el 75% de los anticuerpos anti-sintetasa y son clnicamente tiles, ya que el 80% de los pacientes con anticuerpos anti-Jo1
padece neumonitis intersticial. Algunos pacientes adems presentan
fenmeno de Raynaud y artritis no erosiva. Algunos autores hablan de
sndrome anti-sintetasa cuando el paciente presenta las caractersticas
comentadas: polimiositis, enfermedad pulmonar intersticial y anticuerpos anti-sintetasa.
Otros anticuerpos que aparecen en algunas de las respuestas tambin
se asocian a diferentes manifestaciones clnicas: los anti-SRP se asocian a
un cuadro de polimiositis grave de debut agudo, con frecuente afectacin cardiaca. Los anticuerpos anti-Mi2 se asocian a los pacientes con
dermatopolimiositis con buena respuesta al tratamiento con corticoides.
Pg. 20 RM

M exico A rgentina
C hile U ruguay

En esta enfermedad no se asocia la presencia de anticuerpos antinucleares.


Pregunta 91.- R: 1
La miositis por cuerpos de inclusin supone la forma de miopata
inflamatoria ms frecuente por encima de los 50 aos, con predominio de los varones respecto a las mujeres, por lo que la respuesta 1 es
la correcta.
El comienzo de los sntomas es insidioso (respuesta 2 incorrecta),
desarrollndose a lo largo de meses o aos. A diferencia de la polimiositis o dermatopolimiositis, se afectan grupos musculares proximales y
distales, siendo tpico el desarrollo de debilidad de los flexores del
antebrazo y de la musculatura intrnseca de la mano, y atrofia y adelgazamiento de antebrazos y piernas. En la mitad de los casos aparece
dificultad para la deglucin por compromiso de la musculatura farngea
y esofgica alta. Sin embargo no suele llevar asociada la aparicin de
cardiopata, como algunas formas graves de polimiositis (respuesta 3
incorrecta).
Las enzimas musculares suelen estar elevadas, pero en algn caso
pueden ser normales (respuesta 5 incorrecta). A diferencia de lo que
se observa en algunas formas de polimiositis y dermatopolimiositis, no
se describen anticuerpos antinucleares ni anticitoplasmticos (respuesta
4 incorrecta).
El diagnstico es confirmado mediante la biopsia muscular que
muestra infiltrados perivasculares, necrosis y regeneracin de fibras
musculares y vacuolas intramiocitarias que contienen material basfilo e inclusiones eosinfilas, y tien con rojo congo.
La respuesta a los corticoides e inmunosupresores es deficiente,
habindose empleado en algunos casos inmunoglobulinas intravenosas con resultados variables.
Pregunta 92.- R: 3
En primer lugar debemos resolver, con los datos aportados en el caso
clnico actual, el diagnstico del paciente, destacando que las manifestaciones son recurrentes, y el cuadro probablemente hereditario al
presentar un hermano una clnica similar. Los hallazgos que ms sobresalen son la presencia de fiebre, dolor abdominal con dificultad respiratoria. El origen del paciente, en la zona mediterrnea, facilita todava
ms el diagnstico: fiebre mediterrnea familiar. En esta enfermedad se
producen cuadros recurrentes de fiebre y dolor asociado a la afectacin de serosas (pleuritis, pericarditis, o serositis peritoneal).
Se trata de una enfermedad autosmica recesiva, ligada a un defecto en el brazo corto del cromosoma 16. Junto con los cuadros
recurrentes de fiebre y serositis, los pacientes pueden desarrollar amiloidosis tipo AA, que es la complicacin ms grave de la enfermedad
(que en el caso clnico desarroll el hermano del paciente).
Desde el punto de vista teraputico, el tratamiento para prevenir la
aparicin de las crisis agudas y el desarrollo de amiloidosis es la colchicina,
por lo que la respuesta correcta es la 3. La dosis mnima inicial es de 1 mg/
da y puede incrementarse a 1,5-2 mg/da, si se precisa. Aproximadamente
el 60% responden con completa remisin de los ataques. Un 20-30%
experimentan mejora significativa en el nmero y gravedad de los ataques.
El resto permanece sin cambios, incluso con 2 mg o ms de colchicina al
da. En estos casos se puede probar la colchicina intravenosa.
Pregunta 93.- R: 4
En esta pregunta se realiza un repaso de algunas de las manifestaciones clnicas caractersticas en la amiloidosis. A continuacin repasamos cada una de las respuestas:
Se puede producir infiltracin de material amiloide en las distintas
glndulas endocrinas, aunque no suelen producirse alteraciones
funcionales.
La afectacin del sistema nervioso puede adoptar distintas formas.
La neuropata perifrica es especialmente frecuente en las formas
hereditarias (por lo que es evidente que la respuesta correcta es la
4), aunque tambin aparece en el 15% de las formas sistmicas. La
afectacin del sistema nervioso autnomo es tpica, produciendo
hipotensin ortosttica, incapacidad para sudar, pupila de Adie o
incontinencia esfinteriana. El tnel del carpo aparece especialmente en la amiloidosis primaria (AL) y en la asociada a hemodilisis
crnica (beta-2-microglobulina).

CTO Medicina C/ Nez de Balboa, 115 28006 MADRID (Espaa) Tfno.: (91) 782 43 32 / Fax: (91) 782 43 27
E-mail: secretaria@ctomedicina.com; iberocto@ctomedicina.com WEB: www.ctomedicina.com; www.iberocto.com

Comentarios TEST

Seguimiento a distancia

REUMATOLOGA

Preparacin Examen de Seleccin 05/06 1 Vuelta


El material amiloide puede depositarse en la membrana sinovial,
preferentemente en las articulaciones de hombros (hombro de
jugador de rugby), rodillas, carpos, MCFs e Ifs. La mayora de los
pacientes con artropata amiloide suelen tener mieloma mltiple.
Las formas heredofamiliares comprenden una serie de sndromes
neuropticos (de tipo transtirretina, prealbmina, AF), renales o
cardiovasculares y la amiloidosis asociada a la fiebre mediterrnea
familiar.
Las manifestaciones cardiacas aparecen habitualmente en la amiloidosis primaria en forma de miopata infiltrativa de tipo congestivo.
Pregunta 94.- R: 4
Los datos ms relevantes en este caso clnico son la presencia de
artritis (que afecta a rodillas y tobillos), periostitis a nivel de las espinas
tibiales y dedos en palillo de tambor. Esta trada es caracterstica de la
osteoartropata hipertrfica. Desde el punto de vista etiolgico la mayora de los pacientes suelen ser adultos y presentan una enfermedad
pulmonar. Sin embargo, en esta pregunta debes recordar otras patologas que se asocian a la osteoartropata hipertrfica:
Forma primaria que puede ser idioptica (raro) o familiar (enfermedad de Touraine-Solente-Gol), tambin denominada paquidermoperiostitis por la tendencia a producir hipertrofia cutnea
generalizada. Se hereda con patrn autosmico dominante y las
manifestaciones articulares aparecen aos despus de la afectacin cutnea.
Formas secundarias asociadas a mltiples patologas como:
- Neoplasias pulmonares y pleurales; inflamacin crnica pulmonar. Fibrosis qustica, sarcoidosis y neumonitis intersticial.
- Cardiopata congnita con cortocircuito derecha-izquierda y
endocarditis bacteriana.
- Enfermedades inflamatorias intestinales y neoplasias gastrointestinales.

Comentarios TEST

Como puedes comprobar, cualquiera de las enfermedades, excepto la que debemos elegir, la hemocromatosis, se pueden asociar a
la aparicin de osteoartropata hipertrfica.
Pregunta 95.- R: 2
Las caractersticas clnicas, analticas (incluyendo los datos del lquido sinovial) y radiolgicas son caractersticas de una artrosis de rodilla.
Desde el punto de vista clnico, la paciente presenta dolor que se incrementa con el movimiento (dolor de caractersticas mecnicas). En ocasiones puede haber cierta inflamacin por la aparicin de derrame
articular. El estudio del lquido sinovial muestra caractersticas no inflamatorias: aspecto claro, menos de 3.000 clulas/mm3 con predominio
de mononucleares y ausencia de microcristales. Desde el punto de vista
teraputico es aceptable utilizar en la paciente:
Limitacin de la actividad de la articulacin afecta mientras los
sntomas sean intensos. Sin embargo, es recomendable la realizacin de ejercicios isomtricos y sin carga lo antes posible para
evitar la atrofia muscular.
Los corticoides sistmicos no estn indicados en el tratamiento de
la artrosis (respuesta correcta 2). Sin embargo su utilizacin intra o
periarticular puede producir mejora sintomtica.
Analgsicos o AINEs. Estos ltimos pueden estar ms indicados cuando hay inflamacin articular, como ocurre en este caso.
Recientemente se ha autorizado la inyeccin intraarticular de cido hialurnico para tratar a los pacientes con artrosis de rodilla
que no han mejorado con las medidas analgsicas habituales.
Tambin se ha incorporado recientemente como posibilidad teraputica la administracin de condroitn sulfato oral. Esta molcula
forma parte del cartlago condral y tericamente se incorporara al
mismo, reparndolo.
Pregunta 96.- R: 4
El diagnstico de la artrosis se basa en los hallazgos clnicos y radiolgicos. En las fases iniciales la radiologa puede ser normal, pero el
estrechamiento del espacio articular se vuelve ms evidente a medida
que desaparece el cartlago articular. Otros signos radiolgicos caractersticos son la esclerosis del hueso subcondral, los quistes subcondrales y la presencia de osteofitos marginales. Tambin pueden verse
alteraciones en el contorno articular debidas a la remodelacin sea,
M exico A rgentina
C hile U ruguay

Seguimiento a distancia

y subluxaciones. Aunque se ha considerado el estrechamiento del


espacio de la articulacin femorotibial como un signo indirecto precoz de la artrosis donde no hay cambios evidentes en las radiografas
(por ejemplo esclerosis o quistes subcondrales, osteofitos), la estrechez
del espacio articular por s sola no indica con seguridad el estado del
cartlago articular. De igual modo, la presencia exclusiva de osteofitos,
si no hay otros signos radiolgicos de artrosis, puede deberse ms al
envejecimiento que a la artrosis.
Evidentemente la respuesta a esta pregunta es la osteoporosis yuxtaarticular, hallazgo inverso al que suele aparecer en la artrosis, que es
la esclerosis del hueso subcondral. La osteoporosis es caracterstica
radiolgica de la artritis reumatoide.
Pregunta 97.- R: 5
La artropata neuroptica o artropata de Charcot es una forma
agresiva de artropata degenerativa que se produce en las articulaciones de miembros que por diferentes causas han perdido la sensibilidad algsica, propioceptiva y los reflejos. Estos dficits permiten que
las articulaciones estn sometidas a continuos traumatismos, originando dao progresivo de la articulacin y destruccin sea.
La distribucin de la artropata neuroptica depende del proceso
neurolgico que la desencadena:
En la tabes dorsal se localiza a nivel de cadera, rodilla y tobillo y era
causa habitual de artropata neuroptica hace muchos aos.
En los nios, la causa ms habitual es el mielomeningocele, con
clnica en extremidades inferiores.
Actualmente la causa ms frecuente de artropata neuroptica es la
polineuropata diabtica, que produce una artropata en tarso y
metatarsianos (respuesta correcta).
En la siringomielia se afectarn las articulaciones de hombro (glenohumeral), codo y carpo.
Otras situaciones asociadas son la amiloidosis, la lepra y las inyecciones intraarticulares repetidas de forma excesiva. En estas depender de la localizacin de la enfermedad.
Pregunta 98.- R: 2
En esta pregunta debes recordar cules son las manifestaciones caractersticas en la policondritis recidivante. Esta patologa inflamatoria
crnica, de etiologa desconocida, se caracteriza por la afectacin del
cartlago, especialmente el auricular, que es el ms frecuentemente afectado (por lo que la respuesta es la 2), y en ocasiones se acompaa de
manifestaciones viscerales a nivel renal, cardiacas o vasculitis.
La manifestacin ms habitual y precoz suele ser la condritis auricular (85%). Suele ser bilateral y produce dolor, tumefaccin y eritema en la zona cartilaginosa respetando los lbulos.
La afectacin nasal se produce en el 55% y produce dolor, tumefaccin y eritema en el puente nasal, acompaado de congestin
nasal, epistaxis y rinorrea.
La mitad de los pacientes desarrollan artritis y en el 35% de los
casos es la manifestacin inicial.
Las manifestaciones oculares aparecen en el 50% y pueden ser
mltiples, produciendo conjuntivitis, queratitis, escleritis y epiescleritis.
La afectacin cardiaca es poco frecuente (5%), habitualmente
en forma de insuficiencia artica. Tambin es poco habitual la
aparicin de glomerulonefritis necrotizante, con o sin vasculitis asociada.
Pregunta 99.- R: 3
El proceso descrito en este caso clnico es compatible con el diagnstico de fibromialgia. Esta enfermedad aparece con ms frecuencia
en mujeres en torno a los 50 aos (en nuestro caso tiene 47), que
manifiestan dolor muscular generalizado, con sensacin de rigidez y
fatigabilidad.
La exploracin fsica no demuestra la presencia de signos inflamatorios, observndose dolor selectivo a la palpacin (puntos gatillo).
Se han descrito 19 puntos y debe ser positiva la palpacin en al
menos 11 de ellos.
Desde el punto de vista clnico es habitual que estas pacientes presenten asociadas una serie de manifestaciones psicosomticas como
alteraciones del sueo que es poco reparador, depresin o colon irritable.

CTO Medicina C/ Nez de Balboa, 115 28006 MADRID (Espaa) Tfno.: (91) 782 43 32 / Fax: (91) 782 43 27
E-mail: secretaria@ctomedicina.com; iberocto@ctomedicina.com WEB: www.ctomedicina.com; www.iberocto.com

RM Pg. 21

Seguimiento a distancia

REUMATOLOGA

Preparacin Examen de Seleccin 05/06 1 Vuelta

Al realizar el estudio analtico no encontramos alteraciones. No se


observa elevacin de la VSG, como en la polimialgia reumtica, ni
elevacin de las enzimas musculares, como en la polimiositis. La respuesta es la 3, CPK elevada. El resto de exploraciones que muestran
hallazgos patolgicos en la polimiositis, como el electromiograma o la
biopsia muscular, tambin son normales en la fibromialgia.
Pregunta 100.- R: 2
En esta pregunta se nos plantea el caso de un paciente con datos
analticos y clnicos sugestivos de polimialgia reumtica. Para el diagnstico de esta enfermedad nos basamos en la presencia de los
siguientes criterios:
Edad igual o superior a 50 aos.
Dolor moderado o intenso y rigidez matinal superior a 30 minutos
de ms de un mes de evolucin en al menos dos de las siguientes
tres reas musculares: cuello, cintura escapular y/o cintura pelviana.
Velocidad de sedimentacin superior a 40 mm/1 hora.
Resolucin rpida de los sntomas con dosis bajas de corticoides.

Pg. 22 RM

M exico A rgentina
C hile U ruguay

CTO Medicina C/ Nez de Balboa, 115 28006 MADRID (Espaa) Tfno.: (91) 782 43 32 / Fax: (91) 782 43 27
E-mail: secretaria@ctomedicina.com; iberocto@ctomedicina.com WEB: www.ctomedicina.com; www.iberocto.com

Comentarios TEST

El tratamiento, como aparece dentro de los criterios, se fundamenta en la utilizacin de corticoides. No existe ninguna pauta unnimemente aceptada, pero se considera que se debe administrar una dosis
diaria de 10 mg de prednisona. Posteriormente, la dosis de corticoides
se reduce gradualmente, aunque la mayora de los pacientes requieren mantener el tratamiento al menos dos aos.
Ten en cuenta que en el caso planteado no existen datos de arteritis
de la temporal, que en ocasiones se asocia a la polimialgia reumtica.
Si el paciente hubiera tenido alteracin del pulso de la arteria temporal o alteraciones visuales, deberamos sospechar la presencia de la
vasculitis y utilizar en este caso corticoides, pero en dosis altas (1 mg/
kg/da).

También podría gustarte